Você está na página 1de 1259

POLOS OLÍMPICOS DE

TREINAMENTO INTENSIVO

nível 2

álgebra
geometria
combinatória
teoria dos números
Introdução
Sem vínculo oficial. Janeiro/2022.

O objetivo desta série de pdfs é único: organizar, quase que sistematicamente, os


excelentes materiais disponibilizados gratuitamente no site do POTI (Polos Olímpicos de
Treinamento Intensivo). O POTI tem sua versão presencial e virtual. Aqui, encontram-se
materiais do site https://poti.impa.br (i.e., versão virtual) (avá). Mas, afinal, o que é o
POTI? Mantido e organizado pelo Instituto de Matemática Pura e Aplicada (IMPA),
consiste num programa de treinamentos para estudantes do ensino fundamental II e médio
que desejam participar de olimpíadas de matemática, notadamente a OBM*.

Ao todo, serão três pdfs: um para cada um dos três níveis. Decidi compilar** assim pois,
ao juntar todos os níveis em um único pdf, obtive um pdf com mais de 2300 páginas, o
que não seria prático porque certamente levaria uma eternidade para ser completamente
aberto. Por observação, acredito que os materiais em pdf, principalmente dos níveis 2 e 3,
tenham sido elaborados em 2012 por diversos professores com vasta experiência em
olimpíadas (e em preparação olímpica). Posteriormente, fora elaborado um curso básico
(em 2015) para o nível 2 composto apenas por problemas resolvidos e mais listas foram
adicionadas no site, notadamente do nível 2.

Adendo: é possível encontrar alguns errinhos na numeração das aulas. Por exemplo, o
curso de teoria dos números do nível 3 tem duas aulas de número 6, mas, na verdade, uma
dessas aulas era para ser a aula de número 5 (congruências e bases). Ou também, ter no
título “grafos I” mas não ter pelo menos uma outra aula “grafos II” (curso de
combinatória, nível 2). Sendo assim, elaborei um sumário e acredito que sua ordem seja a
correta, ou que, pelo menos, seja a ordem caso não houvessem os pequenos errinhos na
numeração das aulas.

Aqui, temos os materiais elaborados para o nível 2 (8º e 9º anos), divididos em quatro
partes: álgebra, geometria, combinatória e teoria dos números. Cada uma dessas partes
contêm várias aulas sobre assuntos recorrentes em olimpíadas de matemática e cada uma
dessas aulas contêm parte teórica, problemas resolvidos, problemas propostos, soluções…
O POTI é, de certo, a melhor e mais completa referência em português para se estudar
matemática olímpica gratuitamente.

* A OBM (Olimpíada Brasileira de Matemática) e a OBMEP (Olimpíada Brasileira de


Matemática das Escolas Públicas e Privadas) não são a mesma olimpíada (algumas
pessoas não sabem).
** Material elaborado pelo IMPA. Compilado por Alexander Kahleul. Todo o crédito para
o IMPA.
Sumário
Sem vínculo oficial. Janeiro/2022.

álgebra 7
Aula 0 - curso básico 8
Aula 1 - produtos notáveis 113
Aula 2 - equações e sistemas de equações 122
Aula 3 - sequências 131
Aula 4 - recorrências I 141
Aula 5 - recorrências II 151
Aula 6 - indução I 159
Aula 7 - indução II 165
Aula 8 - desigualdades I 174
Aula 9 - desigualdades II 180
Aula 10 - problemas envolvendo máximos e mínimos 188
Aula 11 - funções definidas implicitamente I 195
Aula 12 - funções definidas implicitamente II 203
Aula 13 - revisão I 209
Aula 14 - revisão II 215
Aula 15 - a ideia de continuidade 222
Aula 16 - números complexos I 229
Aula 17 - números complexos II 236
Aula 18 - relações de girard I 242
Aula 19 - relações de girard II 249
Extra 1 - produtos notáveis 256
Extra 2 - equações e sistemas de equações 259
Extra 3 - sequências 263
Extra 4 - funções quadráticas I 267
Extra 5 - funções quadráticas II 275
Problemas 1 - produtos notáveis 285
Problemas 2 - equações e sistemas de equações 302
Problemas 3 - sequências 314
Problemas 4 - recorrências 328
Problemas 5 - indução 337
Problemas 6 - desigualdades I 350
Problemas 7 - desigualdades II 356

geometria 364
Aula 0 - curso básico 365
Aula 1 - introdução e conceitos iniciais 449
Aula 2 - congruência de triângulos 461
Aula 3 - teorema de tales e aplicações 468
Aula 4 - semelhança de triângulos 474
Aula 5.1 - algumas propriedades importantes de triângulos 477
Aula 5.2 - problemas de primeira fase da OBM 483
Aula 6 - quadriláteros notáveis 496
Aula 7 - ângulos na circunferência 507
Aula 8 - quadriláteros inscritíveis 522
Aula 9 - teorema de ptolomeu 531
Aula 10 - relações métricas no triângulo 539
Aula 11 - potência de ponto e eixo radical 550
Aula 12 - relações entre áreas 561
Aula 13 - revisão 575
Aula 14 - teorema de ceva e teorema de menelaus 579
Aula 15 - pontos notáveis I (baricentro) 589
Aula 16 - pontos notáveis II (incentro) 598
Aula 17 - pontos notáveis III (circuncentro e ortocentro) 607
Aula 18 - circunferências ex-inscritas 615
Extra 1 - geometria básica 623
Extra 2 - geometria básica (soluções) 633
Problemas 1 - conceitos iniciais 671
Problemas 2 - congruência de triângulos 676
Problemas 3 - razões de segmentos 681
Problemas 4 - semelhança de triângulos 687
Problemas 5 - teorema de pitágoras 694
Problemas 6 - algumas propriedades de triângulos 699
Problemas 7 - quadriláteros notáveis 703
Problemas 8 - trabalhando com ângulos 707
Problemas 9 - ângulos na circunferência 713
Problemas 10 - quadriláteros inscritíveis 716
Problemas 11 - relações métricas no triângulo 721
Problemas 12 - áreas 729
Problemas 13 - relações entre áreas I 736
Problemas 14 - relações entre áreas II 741

combinatória 750
Aula 0 - curso básico 751
Aula 1 - lógica I 803
Aula 2 - lógica II 808
Aula 3 - paridade 814
Aula 4 - contagem I 820
Aula 5 - contagem II 826
Aula 6 - jogos 831
Aula 7 - princípio da casa dos pombos I 837
Aula 8 - princípio da casa dos pombos II 844
Aula 9 - tabuleiros 849
Aula 10 - grafos 857
Aula 11 - invariantes 864
Aula 12 - invariantes com restos 868
Aula 13 - revisão 873
Aula 14 - princípio do extremo 877
Aula 15 - combinatória geométrica 881
Aula 16 - combinatória e sequências 885
Aula 17 - combinatória e divisibilidade 890
Aula 18 - miscelânea I 894
Aula 19 - miscelânea II 898
Extra 1 - lógica II 903
Extra 2 - contagem 907
Extra 3 - paridade 912
Extra 4 - mais sobre contagem 917
Problemas 1 - paridade I 921
Problemas 2 - paridade II 926
Problemas 3 - jogos I 921
Problemas 4 - princípio da casa dos pombos I 946
Problemas 5 - princípio da casa dos pombos II 950
Problemas 6 - grafos 956
Problemas 7 - invariantes I 969
Problemas 8 - invariantes II 972

teoria dos números 983


Aula 0 - curso básico 984
Aula 1 - divisibilidade I 1050
Aula 2 - divisibilidade II 1057
Aula 3 - o algoritmo de euclides 1063
Aula 4 - números primos, mdc e mmc 1075
Aula 5 - congruências I 1082
Aula 6 - congruências II 1089
Aula 7 - revisão e aprofundamento I 1097
Aula 8 - equações diofantinas I 1104
Aula 9 - congruências III (o teorema de euler) 1110
Aula 10 - divisores 1116
Aula 11 - o teorema chinês dos restos 1121
Aula 12 - equações diofantinas II 1129
Aula 13 - equações diofantinas III 1135
Aula 14 - a função parte inteira I 1140
Aula 15 - equações diofantinas IV (equações de pell) 1151
Aula 16 - ordem 1161
Aula 17 - revisão e aprofundamento II 1165
Aula 18 - resíduos quadráticos 1169
Aula 19 - a função parte inteira II 1178
Extra 1 - divisibilidade 1182
Extra 2 - o algoritmo de euclides 1185
Extra 3 - números primos, mdc e mmc 1188
Problemas 1 - divisibilidade 1190
Problemas 2 - congruências I 1195
Problemas 3 - congruências II 1202
Problemas 4 - congruências III 1210
Problemas 5 - o teorema chinês dos restos 1220
Problemas 6 - equações diofantinas I 1231
Problemas 7 - equações diofantinas II 1238
Problemas 8 - equações diofantinas III 1247
Problemas 9 - equações diofantinas IV 1253
POLOS OLÍMPICOS DE
TREINAMENTO INTENSIVO

álgebra
nível 2
Polos Olı́mpicos de Treinamento Intensivo
Curso de Álgebra − Nı́vel 2
Professores: Cleber Assis, Samuel Barbosa e Tiago Miranda
Aula 0
POTI 2015
Curso Básico
. . . . . . . . . . . . . . . . . . . . . . . . . . . . . . . . . . . . . . . . . Álgebra . . . . . . . . . . . . . . . . . . . . . . . . . . . . . . . . . . . . . . . . .
Este material compila os arquivos do projeto Portal da Matemática, disponı́vel em
http://matematica.obmep.org.br/

e serve como introdução aos tópicos iniciais de um curso de treinamento olı́mpico. Em geral, os assuntos
são independentes e podem ser estudados em qualquer ordem. Neles, o leitor encontrará muitos exercı́cios
escolares mesclados com problemas elementares de olimpı́adas, todos com respostas e soluções. Além disso,
... endereço do Portal da Matemática, existem vı́deos que podem ser acessados gratuitamente cobrindo todo
no
o conteúdo abaixo. Bons estudos!
Sumário
1 Potenciação . . . . . . . . . . . . . . . . . . . . . . . . . . . . . . . . . . . . . . . . . . . . . . . . . . . . . . . 1
2 Números Racionais . . . . . . . . . . . . . . . . . . . . . . . . . . . . . . . . . . . . . . . . . . . . . . . . . . . 3
3 Números Irracionais . . . . . . . . . . . . . . . . . . . . . . . . . . . . . . . . . . . . . . . . . . . . . . . . . . 5
4 Radiciação e Expressões Algébricas . . . . . . . . . . . . . . . . . . . . . . . . . . . . . . . . . . . . . . . . . 7
5 Introdução aos Polinômios . . . . . . . . . . . . . . . . . . . . . . . . . . . . . . . . . . . . . . . . . . . . . . 9
6 Produtos Notáveis . . . . . . . . . . . . . . . . . . . . . . . . . . . . . . . . . . . . . . . . . . . . . . . . . . . 12
7 Fatoração de Expressões Algébricas . . . . . . . . . . . . . . . . . . . . . . . . . . . . . . . . . . . . . . . . . 15
8 Sentenças Matemáticas e Notação Algébrica . . . . . . . . . . . . . . . . . . . . . . . . . . . . . . . . . . . 18
9 Equações de 1◦ grau . . . . . . . . . . . . . . . . . . . . . . . . . . . . . . . . . . . . . . . . . . . . . . . . . . 20
10 Sistemas de Equações do 1◦ Grau . . . . . . . . . . . . . . . . . . . . . . . . . . . . . . . . . . . . . . . . . 22
11 Equações do 2o grau . . . . . . . . . . . . . . . . . . . . . . . . . . . . . . . . . . . . . . . . . . . . . . . . . . 26
12 Relações entre Coeficientes e Raı́zes . . . . . . . . . . . . . . . . . . . . . . . . . . . . . . . . . . . . . . . . 31
13 Equações Fracionárias . . . . . . . . . . . . . . . . . . . . . . . . . . . . . . . . . . . . . . . . . . . . . . . . . 34
14 Sistema de Equações Fracionárias . . . . . . . . . . . . . . . . . . . . . . . . . . . . . . . . . . . . . . . . . . 36
Potenciação − Soluções . . . . . . . . . . . . . . . . . . . . . . . . . . . . . . . . . . . . . . . . . . . . . . . . 39
Números Racionais − Soluções . . . . . . . . . . . . . . . . . . . . . . . . . . . . . . . . . . . . . . . . . . . . 40
Números Irracionais − Soluções . . . . . . . . . . . . . . . . . . . . . . . . . . . . . . . . . . . . . . . . . . . 43
Radiciação e Expressões Algébricas − Soluções . . . . . . . . . . . . . . . . . . . . . . . . . . . . . . . . . . 48
Introdução aos Polinômios − Soluções . . . . . . . . . . . . . . . . . . . . . . . . . . . . . . . . . . . . . . . 49
Produtos Notáveis − Soluções . . . . . . . . . . . . . . . . . . . . . . . . . . . . . . . . . . . . . . . . . . . . 54
Fatoração de Expressões. Algébricas − Soluções . . . . . . . . . . . . . . . . . . . . . . . . . . . . . . . . . . 58
Sentenças Matemáticas e Notação Algébrica − Soluções . . . . . . . . . . . . . . . . . . . . . . . . . . . . . 65
Equações de 1◦ grau − Soluções . . . . . . . . . . . . . . . . . . . . . . . . . . . . . . . . . . . . . . . . . . . 66
Sistemas de Equações do 1◦ Grau − Soluções . . . . . . . . . . . . . . . . . . . . . . . . . . . . . . . . . . . 69
Equações do 2o grau − Soluções . . . . . . . . . . . . . . . . . . . . . . . . . . . . . . . . . . . . . . . . . . . 76
Relação entre Coeficientes e Raı́zes − Soluções . . . . . . . . . . . . . . . . . . . . . . . . . . . . . . . . . . . 87
Equações Fracionárias − Soluções . . . . . . . . . . . . . . . . . . . . . . . . . . . . . . . . . . . . . . . . . . 94
Sistema de Equações Fracionárias − Soluções . . . . . . . . . . . . . . . . . . . . . . . . . . . . . . . . . . . 100

Versão: 83 (Data: 27 de abril de 2015.)


POTI 2015 − Álgebra − Nı́vel 2 − Aula 0 − Professores Cleber Assis, Samuel Barbosa e Tiago Miranda

1 Potenciação
Problema 1. Calcule o valor das expressões:

a) 35 . b) 22 + 32 . c) 54 . d) 23 + 33 . 1 4
e) · 2 · 3.
2

Problema 2. Calcule o valor das expressões:

1  3 1
a) (0, 01)3 . b) 100 · . 5 d) · (0, 3)2 . e) 200 · (0, 04)4 .
52 c) 80 · . 3
2

Problema 3. Se a = 2 e b = 3, calcule o valor das expressões:

a3 b b) ab . c) a3 b2 . d) ( ab2 )2 . e) (b + a)2 − a2 .
a) .
b2

Problema 4. Escreva como um única potência:

24 · 26 46 · 82 c) (−32)3 .
2
105 · 10−3 · 10 e) 83 : 2−5 .
a) . b) . d) .
37 · 33 163 10−7 · 104

Problema 5. Determine quais das seguintes sentenças são verdadeiras e quais são falsas. Em cada item falso,
indique um contraexemplo para a afirmação.
 a n m
a) an bn = ( a · b)n . b) a−n = − an . c)
n m nm
= ( a − b)n . d) ( a ) = a . e) ( an )m = a(n ) .
b

Problema 6. Determine quais das seguintes sentenças são verdadeiras e quais são falsas. Em cada item falso,
indique um contraexemplo para a afirmação.
 a −n  b n
b) ( a + b)n = an + bn . d) ( an )−n = a0 .
a) = .
b a
c) an+m = an + am . e) Se a 6= 0 então a0 = 1.

Problema 7. Calcule as potências:

a) (0, 3)2 . b) (0, 3)−2 . c) (−0, 02)3 . d) (−3)−2 . e) (1, 2)3

Problema 8. Escreva cada um dos seguintes números como uma potência de 2:

a) (−0, 5)−4 . b) [(−0, 25)2 ]−6 . c) 162 : (0, 25)−4 . d) 32−2 : (0, 25)−4 . e) 0, 16 · 102 .

Problema 9. Determine, em cada item, qual dos números é o maior.


 1/2  1/3
a) 21/2 ou 21/3 . 1 1 c) 31/5 ou 51/3 .
b) ou .
2 2

2
Problema 10. Dividindo-se o número 44 por 44 obtemos o número:

a) 2 b) 43 c) 44 d) 48 e) 412

1
POTI 2015 − Álgebra − Nı́vel 2 − Aula 0 − Professores Cleber Assis, Samuel Barbosa e Tiago Miranda

Problema 11. Definamos a operação a ⊗ b como sendo ab . Por exemplo, 2 ⊗ 3 = 8. Determine o valor de:

2 ⊗ (2 ⊗ (2 ⊗ 2))
.
((2 ⊗ 2) ⊗ 2) ⊗ 2
1 1 c) 1 d) 4 e) 256
a) b)
256 4
Problema 12. Para os inteiros a e b definimos a ∗ b = ab + b a . Se 2 ∗ x = 100, a soma dos algarismos de (4x )4
é igual a:

a) 20 b) 25 c) 30 d) 35 e) 27

Problema 13. Com quantos zeros termina o número 156 · 285 · 557 ?

a) 10 b) 18 c) 26 d) 13 e) 5

Problema 14. As potências 2n e 5n , onde n é um inteiro positivo, começam com o mesmo algarismo d. Qual
é este algarismo?
Problema 15. Se a = 240 , b = 320 e c = 710 , então:

a) c < b < a b) a < c < b c) b < a < c d) b < c < a e) c < a < b

Problema 16. Quanto vale 1212 ?
√ √
a) 66 c) 212 · 36 d) 612

b) 22 3
e) 12
12

Problema 17. Se 2(22x ) = 4x + 64, então x é igual a:

a) −2 b) −1 c) 1 d) 2 e) 3

2
POTI 2015 − Álgebra − Nı́vel 2 − Aula 0 − Professores Cleber Assis, Samuel Barbosa e Tiago Miranda

2 Números Racionais
Problema 18. Escreva os seguintes números na notação cientı́fica:

a) 45673. b) 0, 0012345. c) −555. d) 0, 09

Problema 19. Escreva o perı́odo dos decimais periódicos:

a) 0, 342342342 . . .. b) 58, 6777 . . .. c) 456, 989898 . . ..

Problema 20. Encontre a fração geratriz de:

a) 0, 333 . . .. b) 0, 121212 . . . c) 6, 5. d) −0, 666 . . ..

Problema 21. Obtenha as geratrizes das seguintes dı́zimas periódicas:

a) 4, 7222 . . .. b) 1, 8999 . . .. c) 1, 2010101 . . ..

Problema 22. Sem efetuar a divisão, determine se a fração corresponde a um decimal exato ou a uma dı́zima
periódica.

321 15 41 3
a) . b) . c) . d) .
320 6 15 40

Problema 23. Dizemos que um inteiro positivo x está escrito na notação cientı́fica se é da forma x = m · 10k
onde k é um inteiro e m satisfaz:

a) m é inteiro. b) 1 ≤ |m| < 10. c) m < 1. d) 1 ≤ m < 10. e) 0 < m < 1.

Problema 24. Assinale qual o maior dentre os números seguintes:

a) 1, 01. b) 1, 012. c) 1, 0102. d) 1, 01125. e) 1, 011.

Problema 25. Considere o número


X = 1, 01001000100001 . . . .
(O padrão se mantém, ou seja, a quantidade de zeros entre números uns consecutivos sempre aumenta
exatamente uma unidade).

a) Qual é a sua 25a casa decimal após a vı́rgula?

b) Qual é a sua 500a casa decimal após a vı́rgula?

c) O número X é racional ou irracional?

1
Problema 26. Qual é o primeiro dı́gito não nulo após a vı́rgula na representação decimal da fração ?
512

a) 1 b) 2 c) 4 d) 5 e) 7

3
POTI 2015 − Álgebra − Nı́vel 2 − Aula 0 − Professores Cleber Assis, Samuel Barbosa e Tiago Miranda

Problema 27. O valor da expressão

s s − 1
 3
1
 0
2 1 2
 · (0, 666 . . .) + − 
6 3 (1, 333 . . .)

é igual a:
√ r r √ √
2 2 5 5 2 3 5
a) b) c) d) e)
5 5 2 2 5

Problema 28. Observe as multiplicações:

142857 · 1 = 142857
142857 · 2 = 285714
142857 · 3 = 428571
142857 · 4 = 571428
142857 · 5 = 714285
142857 · 6 = 857142
142857 · 7 = 999999
1 142857
Da última multiplicação, podemos concluir que = = 0, 142857. Veja que as seis primeiras
7 999999
multiplicações produzem números com os mesmos dı́gitos de 142857 e este é exatamente o perı́odo da
1
representação decimal de . Você consegue descobrir um número primo p maior que 7 tal que o perı́odo da
7
1
dı́zima que representa possui p − 1 casas decimais?
p

Problema 29. Considere um primo p que divide 10n + 1 para algum n inteiro positivo. Por exemplo, p = 7
1
divide 103 + 1. Analisando o perı́odo da representação decimal de , verifique que o número de vezes que o
p
dı́gito i aparece é igual ao número de vezes que o dı́gito 9 − i aparece para cada i ∈ {0, 1, 2, . . . , 9}.

Problema 30. Considere um número primo p que não divide 10 e suponha que o perı́odo da representação
1
decimal de seja 2k. É sempre possı́vel decompormos o perı́odo em dois blocos de dı́gitos consecutivos
p
1
que somam 10k − 1? Por exemplo, o perı́odo de tem tamanho 6 = 2k pois é igual à 142857. Veja que
7
142 + 857 = 999 = 103 − 1 = 10k − 1.

4
POTI 2015 − Álgebra − Nı́vel 2 − Aula 0 − Professores Cleber Assis, Samuel Barbosa e Tiago Miranda

3 Números Irracionais
Problema 31. No quadro abaixo, determine quais números são irracionais.


23 5, 345 2

1
2, 313131 . . . 3 0, 01001000100001 . . .

Problema 32. Quais das seguintes afirmações são verdadeiras?

a) N ⊂ Q. b) Z ⊂ Q. c) 1 ∈ Q − Z. d) r ∈ Q ⇒ −r ∈ 35
e) ∈ Q − Z.
Q. 5

Problema 33. Represente em uma reta orientada os seguintes números:

9 14 30
3, 5 − 0 5, 2 −
4 7 7
Problema 34. Utilizando a calculadora podemos obter que

2 = 1, 4142135623730950488016887242097 . . .
Agora, também utilizando uma calculadora, calcule os valores abaixo, faça os registro e observe como o
resultado se aproxima cada vez mais do número 2.

a) 1, 42 = b) 1, 412 = c) 1, 4142 =d) 1, 41422 =



Problema 35. Com base no exercı́cio anterior, utilizando a calculadora, calcule 3. Faça o mesmo procedi-
mento do item anterior, ou seja, calcule o o quadrado do número encontrado apenas com uma casa decimal,
depois com duas casas, depois com três
√ e finalmente com quatro casas. Registre os resultados e observe como
eles se aproximam cada vez mais de 3.
Problema 36. Compare as raı́zes abaixo preenchendo os espaços pontilhadas com os sı́mbolos > ou <.
√ √ √ √ q q √ √
a) 2 . . . . . . 3. b) 81 . . . . . . 121. c) 4
. . . . . . 16
. d) 0, 64 . . . . . . 0, 1.
100 25
√ √
e) n . . . . . . n + 1 com n número real não negativo.

Problema 37. Sem utilizar a calculadora, estime, com uma casa decimal, a melhor aproximação para 11.

Problema 38. Sem utilizar a calculadora, estime, com duas casas decimais, uma boa aproximação para 11.
√ √
Problema 39. Quantos números inteiros positivos existem entre 8 e 80?
√ √
Problema 40. Quantos números inteiros positivos existem entre 37 e 1226?
Problema 41. Quantos dos números abaixo são maiores que 10?
√ √ √ √ √
3 11 , 4 7 , 5 5 , 6 3 , 7 2.

a) 1 b) 2 c) 3 d) 4 e) 5

Problema 42. Explique porque entre dois números racionais sempre podemos encontrar um terceiro número
racional.
√ √
Problema 43. Dados dois reais positivos, 3 3 e 4 4, determine o maior.
p √ √ √ √
Problema 44. O número 1 + 3 4 + 3 16 está situado entre n e n + 2, onde n é inteiro positivo. Deter-
mine n.

5
POTI 2015 − Álgebra − Nı́vel 2 − Aula 0 − Professores Cleber Assis, Samuel Barbosa e Tiago Miranda
√ √
Problema 45. Prove que não é possı́vel escrever 2 como uma fração de inteiros. Ou seja, prove que / Q.
2∈
Problema 46. Prove que não é possı́vel escrever:
√ √
/ Q.
i 3 como uma fração de inteiros. Ou seja, prove que 3 ∈
√ √
/ Q.
ii 5 como uma fração de inteiros. Ou seja, prove que 5 ∈
√ √
/ Q.
iii p como uma fração de inteiros, sendo p um número primo. Ou seja, prove que p ∈

Problema 47. É verdade que existem números irracionais A e B tais que A B é racional?
a
Problema 48. A sequência Fn de Farey é uma sequência de conjuntos formados pelas frações irredutı́veis
b
com 0 ≤ a ≤ b ≤ n arranjados em ordem crescente. Exibimos abaixo os quatro primeiros termos da sequência
de Farey.

F1 = {0/1, 1/1}

F2 = {0/1, 1/2, 1/1}

F3 = {0/1, 1/3, 1/2, 2/3, 1/1}

F4 = {0/1, 1/4, 1/3, 1/2, 2/3, 3/4, 1/1}

Qual deve ser o conjunto F5 ?


a c
Problema 49. É possı́vel mostrar que se duas frações e são vizinhas na sequência de Farey Fn (veja
b d
a
o exercı́cio anterior) então ad − bc = ±1. Sabendo disso, você consegue determinar que fração está
b
5
imediatamente à esquerda de em F7 sem calcular todos os seus elementos?
7
Problema 50. Considere dois tambores de capacidade suficientemente grande.

a) Determine se é possı́vel colocar exatamente um litro do lı́quido de um dos tambores no outro usando dois
baldes, um com capacidade de 5 e o outro com capacidade de 7 litros.

b) Determine se é possı́vel colocar exatamente


√ um litro do lı́quido de um√dos tambores no outro usando dois
baldes, um com capacidade de 2 − 2 e o outro com capacidade de 2 litros.

Problema 51. Achar o menor inteiro positivo n tal que as 73 frações

19 20 21 91
, , ,...,
n + 21 n + 22 n + 23 n + 93
sejam todas irredutı́veis.

6
POTI 2015 − Álgebra − Nı́vel 2 − Aula 0 − Professores Cleber Assis, Samuel Barbosa e Tiago Miranda

4 Radiciação e Expressões Algébricas


Problema 52. Simplifique as expressões envolvendo radicais:

3
√ p √ √
a) x4 . b) ( 3 8)2 . c) 4 81x8 y4 . d) 32 + 162. e) (4a6 b4 )3/2 .

Problema 53. Transforme a expressão dada em outra sem radicais no denominador como indica o exemplo:

5

5
1 1 x3 x3

5
= √
5
· √
5
= .
x2 x2 x3 x
2 1 2 1
r
a) √ . b) √ . 1 d) √ . e) √ .
4 3 c) 9 2 . 3
x 4
a
3 x a

Problema 54. Elimine os expoentes negativos das expressões abaixo:


 −2
x −3 y 4 6st−4 ba−4 a −3 a −5

a) 5 −3 . b) . c) . d) · .
x y 2s−2 t2 ab−3 b −2 b −3

1 2

1
3 − +
2 4
Problema 55. Simplificando a expressão  , obtemos:
1 2

3
3 − −
3 2

6 7 6 7 5
a) − . b) − . c) . d) . e) − .
7 6 7 6 7

Problema 56. Simplifique a expressão:


2
3x2 y


a3 b3
3
3xy2


2a2 b2

Problema 57. Simplifique as expressões:


p
3
√ p qp

a) 64x24 . b) 4 x4 y8 z2 . c) x.

Problema 58. Determine o valor da expressão abaixo quando a = 2014 e n = 1000.

1 1 1 1 1 1 1
+ − n +1 + . . . + −1 + 0 + n + +...+ 1 .
a−n+1 a +1 a +1 a +1 a +1 1+a − n + 1 a +1
a) 10002013 b) 20131000 c) 2013 2001 e) 1000.
d)
2

Problema 59. Ao efetuar a soma 131 + 132 + 133 + . . . + 132006 + 132007 , obtemos um número inteiro. Qual o
algarismo das unidades desse número?

a) 1 b) 3 c) 5 d) 7 e) 9.

7
POTI 2015 − Álgebra − Nı́vel 2 − Aula 0 − Professores Cleber Assis, Samuel Barbosa e Tiago Miranda

Problema 60. Efetuando as operações indicadas na expressão

22007 + 22005
 
· 2006
22006 + 22004

obtemos um número de quatro algarismos. Qual é a soma dos algarismos desse número?

a) 4 b) 5 c) 6 d) 7 e) 8

Problema 61. Sejam a, b e c inteiros e positivos. Entre as opções abaixo, a expressão que não pode representar
o número 24 é:

a) ab3 b) a2 b3 c) ac bc d) ab2 c3 e) ab bc c a .

Problema 62. Calcule o valor de


1001 · 1002 · 1003 · . . . · 2000
A=
1 · 3 · 5 · . . . · 1999
a) 21000 b) 2999 c) 1000 d) 999 e) 2.

8
POTI 2015 − Álgebra − Nı́vel 2 − Aula 0 − Professores Cleber Assis, Samuel Barbosa e Tiago Miranda

5 Introdução aos Polinômios


Problema 63. Seja n um número natural. Indique por meio de expressões algébricas:

a) o dobro de n. c) o sucessor de n. e) o cubo de n.


b) 20% de n. d) a metade da soma entre n e 3.

Problema 64. Determine a área de um retângulo cujas dimensões (comprimento e largura) são:

a) 2x e x. b) 2x e ( x + 1). c) ( x − 1) e ( x + 2).

Problema 65. Escreva os graus de cada monômio:



a) 2x5 y3 . b) − 45 m2 n. c) 5p5 qr5 . d) am bn cd.

Problema 66. Considere os monômios A = 8x3 y2 e B = 4xy. Determine:

a) A · B. A
b) .
B

Problema 67. Sejam os polinômios P = 3x2 + 4x − 8 e Q = x2 + 1, determine:

a) P + Q. b) P − Q. c) P · Q.

Problema 68. Efetue as multiplicações:

a2
 
a) ( a + 1)( a2 − 6a + 4). b) (3a − b)(3ab + 2a − b). 8
c) 1, 2a2 + 1, 6a + .
4 3

Problema 69. Um taxista cobra, por corrida, R$3, 00 como preço fixo inicial e mais R$2, 50 para cada km
rodado.

a) Determine a expressão que representa quanto será cobrado por uma corrida de x km.

b) Quanto custa uma corrida de 9km?

Problema 70. Os produtos algébricos da forma ( x + a)( x + b), onde x é variárel e a e b são números reais
quaisquer, podem ser calculados usando-se a distributividade, obtemos assim ( x + a)( x + b) = x2 + ( a +
b) x + ab. Veja que o coeficiente de x é a soma de a e b e o coeficiente independete de x é ab. Por exemplo,
( x + 2)( x + 5) = x2 + 7x + 10. Utilize este princı́pio e calcule os produtos:
  
a) ( x + 1)( x + 2) d) ( x − 4)( x + 4). 1 3
g) x + x+ .
2 2
b) ( x + 3)( x + 9). e) ( x + 5)( x + 5).
c) ( x − 2)( x + 3). f) ( x − 4)( x − 4).

Problema 71. O Teorema do Resto diz que o resto da divisão de um polinômio P de uma variável x por
outro polinômio da forma ( x + a) é igual ao valor de P quando substituimos x por − a. Use este teorema para
calcular o resto da divisão de x3 − 4x2 + 5x − 1 por:

a) x − 1. b) x + 1. c) x − 3. d) x + 4. 1
e) x − .
2

9
POTI 2015 − Álgebra − Nı́vel 2 − Aula 0 − Professores Cleber Assis, Samuel Barbosa e Tiago Miranda

Problema 72. Use o Teorema do Resto para verificar se x4 + 2x3 − 3x2 − 8x − 4 é divisı́vel por:

a) x − 1. b) x + 1. c) x + 2. d) x − 2. e) x + 3.

Problema 73. Em um jogo de perguntas e respostas, ganham-se 5 pontos por acerto e perdem-se 3 pontos
por erro.

a) Determine a expressão que representa o número de pontos obtidos por alguém que acertou x perguntas e
errou y perguntas.

b) Qual a pontuação de Maycon, se ele acertou 8 e errou 2 perguntas?

Problema 74. Um tanque de combustı́vel possui a capacidade máxima de 50 litros. Se já existem x litros de
combustı́vel neste tanque, determine:

a) A expressão que da quantidade de litros que faltam para completá-lo.

b) A expressão que determina do quanto será gasto para completá-lo, se o litro de combustı́vel custa R$3, 00.

Problema 75. Simplifique as expressões:

a) a9 · a−5 . b) (3y2 )(4y5 ). (2x3 )2 (3x4 ) d) (2a3 b2 )(3ab4 )3 .


 3  2 4
x y x
c) . e) .
( x 3 )4 y z

Problema 76. Elimine os expoentes negativos das expressões abaixo:


 −2
x −3 y 4 6st−4 ba−4 a −3 a −5

a) . b) . c) . d) · .
x 5 y −3 2s−2 t2 ab−3 b −2 b −3

Problema 77. Simplifique a expressão:


2
3x2 y


a3 b3
3
3xy2


2a2 b2
Problema 78. Determine o grau dos monômios abaixo:

a) 5a2 b7 .
7 n +1 n +2
b) 2a b , onde n é um número natural.

c) ab2 c3 d4 . . . z26 (onde é colocado em cada letra do alfabeto um expoente correspondendo à sua posição).

Problema 79. Determine o valor do inteiro positivo n para que o grau do monômio 5x n+1 y2n−1 seja 9.
Problema 80. Determine o valor de k para que o produto (kx − 1)(2x + 1) seja um polinômio cuja soma dos
coeficientes é 3.
Problema 81. Use a propriedade de distributividade da multiplicação e resolva os produtos:

a) ( x + an )( x − an ). b) ( x + a2n )( x + a2n ). c) ( x − 2a)2 .

Problema 82. Determine o quociente e o resto das divisões:

a) ( x2 − a2 ) ÷ ( x − a). b) ( x2 + 2xa + a2 ) ÷ ( x + a). c) ( x3 + a3 ) ÷ ( x + a).

10
POTI 2015 − Álgebra − Nı́vel 2 − Aula 0 − Professores Cleber Assis, Samuel Barbosa e Tiago Miranda

−b + b2 − 4ac
Problema 83. Determine o valor de x = quando:
2a
a) a = 1; b = 4; c = 4. b) a = 1; b = −2; c = −8. c) a = 1; b = 5; c = 0.

Problema 84. Determine k para que o polinômio P( x ) = x3 − 2x2 + 3x + k seja divisı́vel por:

a) x − 1 b) x + 1 c) x − 2.

Problema 85. De uma cartolina quadrada de 50cm de lado, retira-se através de cortes um quadrado de x cm
de lado de cada um dos quatro cantos da cartolina, sendo 0 ≤ x ≤ 25 . Determine:
a) A expressão que determina a área da cartolina após os cortes.
b) A área da cartolina cortada se x = 5cm.
c) Suponha que a cartolina cortada é usada para formar uma caixa sem tampa dobrando-se ao longo das
retas determinadas pelos cortes. Qual a expressão envolvendo x fornece o volume de tal caixa?
Problema 86. Simplifique a expressão:
( x2n+1 + x )( x2n+1 − x ) − ( x4 )(n+1/2)
,
( x n + x )2 − x2n − 2x n+1
definida para x 6= 0.
Problema 87. Sejam A = x3 + 6x2 − 2x + 4 e B = x2 − 1, polinômios. Determine o quociente e o resto de A
na divisão por B.
Problema 88. Leila foi avisada em dezembro de 2012, que a mensalidade escolar de seus filhos para o ano
de 2013 teria um aumento de 80%. Ela não concordou com o aumento e procurou o PROCON que, após
analisar o caso, determinou que a escola reduzisse este último valor em 30%. A escola acatou a decisão do
PROCON. Além disso, como Leila tem 3 filhos matriculados, a escola decidiu lhe dar 10% de desconto nas
mensalidades de cada um de seus filhos. Determine:
a) A expressão que determina o preço da mensalidade de cada filho de Leila em 2013.
b) Quanto Leila gastará com mensalidades em 2013, se a mensalidade, em 2012, era R$ 800,00.
p
Problema 89. A expressão 3 −( x − 1)6 é um número real. Determine:
a) O valor da expressão para x = 2.
b) O maior valor possı́vel para a expressão.
Problema 90. a) Calcule o valor de:
     
1 1 1 1
1+ 1+ 1+ ... 1+
2 3 4 99

b) Calcule o valor de:     


1 1 1
1+ 1+ ... 1+
x+1 x+2 x + 98
Problema 91. a) Calcule o valor de
1001 · 1002 · 1003 · . . . · 2000
A=
1 · 3 · 5 · . . . · 1999
b) Se x é um inteiro positivo, calcule o valor de:
( x + 1)( x + 2)( x + 3) · . . . · (2x )
B=
1 · 3 · 5 · . . . · (2x − 1)

11
POTI 2015 − Álgebra − Nı́vel 2 − Aula 0 − Professores Cleber Assis, Samuel Barbosa e Tiago Miranda

6 Produtos Notáveis
Problema 92. Siga o modelo e calcule os produtos notáveis:

( x + 5)2 = x2 + 2 · x · 5 + 52
= x2 + 10x + 25

a) ( x + 1)2 . b) (4 + x )2 . c) ( x + 3)2 . d) (3x + 1)2 . e) (4x + 2)2 .

Problema 93. Calcule os produtos notáveis:

a) (2x + 3)2 . b) (2x + 3y)2 . c) ( x2 + 3)2 . d) ( a2 + 3b2 )2 . e) ( x4 + 32 )2 .

Problema 94. Veja o seguinte exemplo para calcular o quadrado de um número:

422 = (40 + 2)2


= 402 + 2 · 40 · 2 + 22
= 1600 + 160 + 4
= 1764

Calcule os quadrados de 13, 41 e 19 sem usar a calculadora.


Problema 95. Calcule o valor das expressões:
√ √ √
a) ( a + b)2 − 2 ab. c) ( a + 1)2 + 2( a + 1) a + a2 + 2(2a + 1) + 1.
b) ( x + 1)2 + ( x − 1)2 .

Problema 96. Calcule as expressões:

a) (− a − b)2 . b) (−2a + b)2 . c) (2ab + 3c)2 . d) (2a − 2b)2 .

Problema 97. Calcule os produtos:

a) ( x − 1)( x + 1). c) ( x2 − 3z)( x2 + 3z).


√ √ √ √
b) (4 − a)(4 + a). d) ( x + y)( x − y)( x + y)

Problema 98. Siga o modelo abaixo e calcule o valor das expressões dadas.

27 · 33 = (30 − 3)(30 + 3)
= 302 − 32
= 891

a) 99 · 101. b) 1998 · 2002. c) 5 · 15 + 25

Problema 99. Ao efetuarmos a multiplicação ( a + b)( a + b) usando a distributividade, quantas operações de


multiplicação faremos?
Problema 100. Repita o exercı́cio anterior com a multiplicação ( a + b)( a + b)( a + b). Em seguida, determine
quantas cópias de a2 b aparecem no resultado. Finalmente, conclua com argumentos de contagem que:

( a + b)3 = a3 + 3a2 b + 3ab2 + b3 .

12
POTI 2015 − Álgebra − Nı́vel 2 − Aula 0 − Professores Cleber Assis, Samuel Barbosa e Tiago Miranda

Problema 101. Encontre uma figura que explique geometricamente, através do uso de áreas, a equação:
6·7
1+2+3+4+5+6 = .
2
Problema 102. A figura abaixo explica geometricamente, usando áreas, o desenvolvimento do produto
notável
( a + b)2 = a2 + 2ab + b2 .

Você conseguiria obter uma figura que explicasse geometricamente, também usando áreas, a equação

( a + b )2 + ( a − b )2 = 2( a2 + b2 ) ?

Problema 103. Encontre uma figura que explique geometricamente, através do uso de áreas, a equação:

1 + 3 + 5 + . . . + 17 = 92

Problema 104. O professor Medialdo acaba de explicar a seus alunos que a média aritmética de dois números
a+b √
a e b é e a média geométrica é ab. Antes de entregar as notas de duas provas aplicadas anteriormente,
2
ele decidiu testar o conhecimento dos seus alunos perguntando se eles prefeririam que cada um recebesse a
média geométrica ou a média aritmética das duas notas. Considerando que os alunos desejam a maior nota
possı́vel no boletim, o que eles devem dizer ao professor Medialdo?
Problema 105. Sejam a e b números reais.
a) Verifique que ( a + b)2 ≥ 4ab.
1 1 4
b) Verifique que + ≥ .
a b a+b
1 1 4 16 64
c) Verifique que + + + ≥ .
a b c d a+b+c+d
Problema 106. João deseja construir um retângulo usando um arame com 2 metros de comprimento. Qual é
a maior área possı́vel de tal retângulo?
Problema 107. Sejam:
r s r s r
q √ q √ q √ q √
A= 2+ 3· 2+ 2+ 3eB= 2+ 2+ 2+ 3× 2− 2+ 2+ 3

Quanto vale A · B?
√ √ √ √
a) 2 b) 3 c) 1 d) 2 + 2 e) 2 + 3.

Problema 108. Calcule o valor do número:

201420132 − 2(20142013)(20142012) + 201420122

13
POTI 2015 − Álgebra − Nı́vel 2 − Aula 0 − Professores Cleber Assis, Samuel Barbosa e Tiago Miranda
√ √ √ √
Problema 109. Se x, y, a e b são reais tais que x−y = a e x+ y = b, determine o valor de xy.

b4 − a4 a2 a2 + b2 1 e) a2 .
a) b) c) d)
4b2 b b b

Problema 110. João está ajudando seu pai com as finanças de sua loja. Como a quantidade de produtos
ofertados estava influenciando a quantidade de produtos vendidos, ele decidiu procurar algum padrão que
pudesse ajudá-lo a descobrir qual a quantidade ideal de produtos que deveriam ser ofertadas para maximizar
a quantidade de produtos vendidos. Depois de um bom tempo “quebrando a cabeça”, ele percebeu que se
“a” produtos eram ofertados, então a loja vendia “a(10 − a)” itens. Em seguida, com a ajuda de um produto
notável semelhante a essa expressão, foi possı́vel achar a quantidade ideal de produtos que deveriam ser
vendidos. Como ele fez isso?
Problema 111. O pai de João (veja o problema anterior), percebendo a astúcia do filho, decidiu desafiá-lo a
fazer o mesmo com uma fórmula bem diferente e supondo agora que a é um número real qualquer. Nesse
novo problema, dado “a” real, ele deve tentar achar o valor máximo de 4a − a4 . Novamente usando produtos
notáveis, João conseguiu descobrir que o máximo de tal expressão é 3. Você consegue descobrir como ele fez
isso?

14
POTI 2015 − Álgebra − Nı́vel 2 − Aula 0 − Professores Cleber Assis, Samuel Barbosa e Tiago Miranda

7 Fatoração de Expressões Algébricas


Problema 112. Siga o modelo e fatore as expressões:

3a + ba = a(3 + b)

a) 5a + ba. b) am + an. c) xa + xb + xc. d) ax + a. e) ab + bc + abc.

Problema 113. Simplifique as frações fatorando o denominador e o numerador.


3a + 5b 3x + 3y 3a2 + 5a d) x4 + x3
a) . b) . c) . e) 2 .
6a + 10b 8x + 8y 6a + 10 a( x + y) + b( x + y) x +x
.
( a − b) x + ( a − b)y
Problema 114. Fatore por agrupamento as seguintes expressões:

a) a2 + ab + ac + bc. b) ax − bx + ay − c) 2ab + 2a + b + 1. d) ax − bx + 2a − 2b e) 10ab − 2b +


by. 15a − 3

Problema 115. Fatore o numerador e o denominador e simplifique cada expressão dada:

m4 + m2 x3 + x2 + x + 1 m4 + 3m3 + 2m + 6
a) . b) . c) .
m2 + 1 x3 + x2 ( m + 3)2
Problema 116. Fatore as expressões abaixo usando a diferença de quadrados:

a) a2 − 25b2 . b) 4x2 − 1. c) 7 − x2 . d) a2 x2 − b2 y2 . e) a4 − b4

Problema 117. Para cada um dos itens abaixo, decida se a expressão dada é o quadrado de um binômio,
isto é, se pode ser escrita na forma:
( a + b)2 = a2 + 2ab + b2 ou como ( a − b)2 = a2 − 2ab + b2 .

a) x2 − 4x + 3. 1 c) y2 + 6y + 18. d) 4z2 − 12zy + 9y2 . e) 3z2 + 6z + 3.


b) x2 + x + .
4
Em caso afirmativo, escreva o binômio.
Problema 118. Fatore completamente as expressões abaixo:

a) x4 − 2x2 + 1. b) 5a2 − 10a + 5. c) a2 − b2 − 2bc − c2 .

Problema 119. Efetue as multiplicações e divisões indicadas como no exemplo:


2ab 5xy 2ab 5x y
· = ·
3ax 7by 3a x 7by
2 5
= ·
3 7
10
=
21
4a 5b x3 + x x2 + 1 yx + x xy + y
a) · . b) ÷ . c) · .
b a 3y y2 ( x + 1) ( y + 1)2
2

Problema 120. Se xy = 6 e x + y = 7, quanto vale x2 y + y2 x?


a
Problema 121. Se, ao adicionarmos x ao numerador e subtrairmos x do denominador da fração , com a e
b
c
b reais, obtemos a fração , com c e d reais e c 6= −d, qual o valor de x?
d

15
POTI 2015 − Álgebra − Nı́vel 2 − Aula 0 − Professores Cleber Assis, Samuel Barbosa e Tiago Miranda

Problema 122. Fatore as expressões:

a) a2 b − b3 . b) x2 − 2xy + y2 − 9. c) a4 − 32a2 + 256.

Problema 123. Verifique que:


x3 − y3 = ( x − y)( x2 + xy + y2 ).
Em seguida, fatore x3 − 8.
Problema 124. No exercı́cio anterior, o que acontece se trocarmos y por −z?
Problema 125. A soma de dois números é 4 e seu produto é 1. Encontre a soma dos cubos desses números.
Problema 126. Se xy = x + y = 3, calcule x3 + y3 .
1 1
Problema 127. Seja x um número real tal que x + = 2, calcule x2 + 2 .
x x
Problema 128. Qual a forma mais simplificada da expressão ( a − b)2 + (− a + b)2 + 2( a − b)(b − a)?
Problema 129. Simplifique a expressão
√ √ √ √ √ √ √ √ √ √ √ √
( 5 + 6 + 7)( 5 + 6 − 7)( 5 − 6 + 7)(− 5 + 6 + 7).

Problema 130. Fatore completamente x4 + 4.


Problema 131. Verifique que

n(n + 1)(n + 2)(n + 3) + 1 = (n(n + 3) + 1)2

Problema 132. Calcule o valor de:


q
(2014)(2015)(2016)(2017) + 1

Problema 133. Fatore p4 − 1.


p √ p √
Problema 134. Se x = 3 − 8 − 3 + 8, mostre que x é um inteiro negativo.
Problema 135. Fatore n5 + n4 + 1.
√ √
Problema 136. Qual é o menor inteior positivo n tal que n− n − 1 < 0, 01
Problema 137. Encontre o quociente da divisão de a32 − b32 por

( a16 + b16 )( a8 + b8 )( a4 + b4 )( a2 + b2 )

Problema 138. Verifique que


(23 − 1)(33 − 1) . . . (1003 − 1) 3367
3 3 3
= .
(2 + 1)(3 + 1) . . . (100 + 1) 5050
Problema 139. A sequência de Fibonacci é definida recursivamente por Fn+2 = Fn+1 + Fn para n ∈ Z e
F1 = F2 = 1. Determine o valor de:

F2 F2 F2
    
1 − 22 1 − 32 · . . . · 1 − 2013
2
F3 F4 F2014
F2016 F2014 2
F2015 F2015 F2015
a) 2 b) c) d) e) .
F2013 F2013 F 2 2 2F2013 F2014
2013

Problema 140. Se x + y + z = 0, verifique que:

x3 + y3 + z3 = 3xyz.

16
POTI 2015 − Álgebra − Nı́vel 2 − Aula 0 − Professores Cleber Assis, Samuel Barbosa e Tiago Miranda

Problema 141. Define-se o conjunto de 100 números {1, 1/2, 1/3, ..., 1/100}. Eliminamos dois elementos
quaisquer a e b deste conjunto e se inclui, no conjunto, o número a + b + ab ficando assim um conjunto com
um elemento a menos. Depois de 99 destas operações, ficamos só com um número. Que valores pode ter esse
número?
Problema 142. Verifique que

( x + y + z)3 = x3 + y3 + z3 + 3( x + y)( x + z)(y + z).

Problema 143. Verifique que:

( x + y + z)2 = x2 + y2 + z2 + 2xy + 2xz + 2yz

Problema 144. Fatore a expressão

( b − c )3 + ( c − a )3 + ( a − b )3 .

Problema 145. Sejam a, b, c, x, y, z reais distintos tais que ax + by + cz = 0. Verifique que

ax2 + by2 + cz2


bc(y − z)2 + ca(z − x )2 + ab( x − y)2

não depende de x, nem de y, nem de z.

17
POTI 2015 − Álgebra − Nı́vel 2 − Aula 0 − Professores Cleber Assis, Samuel Barbosa e Tiago Miranda

8 Sentenças Matemáticas e Notação Algébrica


Problema 146. Nos parênteses dos ı́tens abaixo, marque A, caso a sentença seja aberta, ou F, caso a sentença
seja fechada.

a) ( ) 42 = 15 + 1. e) ( ) 7 ∈ N.

b) ( ) 2x − 1 = x + 4. 1
√ f) ( ) = 2x.
c) ( ) 1 < 2. x+1

d) ( ) 2a − 1 = b. g) ( ) x2 = 5.

Problema 147. Quais das sentenças fechadas abaixo são verdadeiras?



a) 52 = 42 + 32 . 1 1 1 g) 25 ∈ Q.
d) · = .
2 3 6 √
b) 7 − 13 = −6. h) 3
/ Z.
−8 ∈
e) −6 ∈ N.
1 1 2 √ 1 1
c) + = . f) 16 > 4. i) − <− .
2 3 5 2 3

Problema 148. Utilize sı́mbolos matemáticos e letras para representar as grandezas e reescrever as sentenças
abaixo.

a) O perı́metro de um quadrado é o quádruplo da medida do seu lado.

b) A área de um quadrado é o quadrado da medida do seu lado.

c) A soma das idades de Luiz e Luı́sa é dezesseis.

d) A metade da raiz quadrada de um número é menor que o triplo desse número.

e) O salário de Rodrigo é setecentos reais mais vinte por cento do valor de suas vendas.

f) A área de um retângulo cuja altura é o dobro da base é o dobro do quadrado da base.

Problema 149. Seja l a medida da aresta de um cubo. Determine as expressões correspondentes

a) a sua área A.

b) ao seu volume V.

c) à soma S das medidas de todas as arestas.

Problema 150. Diz a lenda que no túmulo de Diofanto (matemático grego da antiguidade) havia o seguinte
problema:
Viajante, aqui estão as cinzas de Diofanto. É milagroso que os números possam medir a extensão de sua vida: 1/6 dela foi
uma bela infância; depois de 1/12 de sua vida, sua barba cresceu; 1/7 de sua vida passou em um casamento sem filhos;
cinco anos após isso nasceu seu primeiro filho, que viveu metade da vida de seu pai; e, em profundo pesar, o pobre velho
terminou seus dias na terra quatro anos após perder seu filho. Quantos anos viveu Diofanto?
Construa uma equação, utilizando os dados do túmulo, na qual seja possı́vel calcular a idade de Diofanto e a
resolva.

18
POTI 2015 − Álgebra − Nı́vel 2 − Aula 0 − Professores Cleber Assis, Samuel Barbosa e Tiago Miranda

Problema 151. A figura abaixo é o desenho de um terreno retangular dividido em três retângulos menores.
Determine:

a) uma expressão que representa o perı́metro P do terreno.

b) uma expressão que representa a quantidade Q de cerca gasta, se todos os retângulos serão cercados e lados
comuns recebem cerca apenas uma vez.

c) uma expressão que representa a área A do terreno.

Figura 1

Problema 152. O retângulo ABCD abaixo representa um terreno. Deve-se passar uma cerca que o divida de
maneira que a área do polı́gono CDEF seja o dobro da área do polı́gono ABFE. Sobre o lado AD essa cerca
começa a 5m do vértice A e sobre o lado BC essa cerca termina a x metros do vértice B.

a) Represente algebricamente a área dos dois polı́gonos separados pela cerca.

b) Determine o valor de x.

Figura 3

19
POTI 2015 − Álgebra − Nı́vel 2 − Aula 0 − Professores Cleber Assis, Samuel Barbosa e Tiago Miranda

9 Equações de 1◦ grau
Problema 153. Determine o valor numérico de cada uma das expressões abaixo.

a) 2x + 1, para x = 1. d) xy2 − yx2 , para x = 2 e y = 3.


x − 5y
b) x − 3y, para x = 4 e y = 1. e) , para x = 5 e y = 3.
4
c) x2 − y3 , para x = 3 e y = −1. f) x2 + 2xy + y2 , para x = 4 e y = 2.

Problema 154. Um edifı́cio tem 12 andares, com 4 apartamentos por andar. Cada apartamento possui 6
janelas, que possuem, cada uma, um vidro retangular de dimensões a e b. Dê a expressão algébrica que
representa a área total de vidro utilizado.
Problema 155. A figura abaixo representa um terreno dividido em duas partes retangulares.
Determine:

a) a expressão que representa a área do terreno.

Figura 5 b) a área do terreno para x = 20m e y = 15m.

Problema 156. Para calcular a média bimestral de seus alunos, um professor usa o seguinte critério: multi-
plica a nota da prova por 2, soma o resultado com a nota de um trabalho e divide a soma obtida por 3. Se
você representar por n o número que expressa a média, por p a nota da prova e por t a nota do trabalho, qual
será a fórmula matemática para calcular a média bimestral?
Problema 157. Se numa fração diminuirmos o numerador em 40% e o denominador em 60%, a fração inicial
ficará:

a) diminuı́da em 20%. d) aumentada em 50%.

b) aumentada em 20%. e) aumentada em 30%.

c) diminuı́da em 50%.

Problema 158. Uma piscina, em forma de paralelepı́pedo, tem como dimensões, em metros, x de largura, 2x
de comprimento e y de altura. Determine:

a) a expressão que representa o seu volume. b) a expressão que representa sua área total.
c) a quantidade, em litros, necessária para enchê-la completamente, sendo x = 3m e y = 2m.

Problema 159. A figura abaixo representa uma quadra de tênis, na qual seus dois lados separados pela rede
são simétricos.
Determine:

a) a expressão que representa o perı́metro da quadra.

b) a expressão que representa a soma dos comprimen-


tos das linhas (internas e externas).

Figura 6 c) a área da quadra, sendo x = 2m e y = 2, 5m.

20
POTI 2015 − Álgebra − Nı́vel 2 − Aula 0 − Professores Cleber Assis, Samuel Barbosa e Tiago Miranda

Problema 160. Abaixo, figuras com a mesma forma representam objetos de mesma massa. Quantos quadra-
dos são necessários para que a última balança fique equilibrada?

Figura 7
Problema 161. A figura abaixo é o projeto de um quarto com uma porta de 1m de largura e uma porta
dupla de 2m de largura. As dimensões externas desse quarto são 5m x 3m. Se a espessura das paredes é x,
determine:

a) o perı́metro interno desse quarto.

b) a área interna desse quarto, desconsiderando o vão


deixado pelas portas.

c) se a altura das portas é 2m e a altura das pare-


des é 3m, determine a área interna das paredes,
Figura 9 desconsiderando as portas.
x
Problema 162. Para qualquer número positivo x, dizemos que os números x + 1 e são irmãos e filhos
x+1
5
de x. Encontre um irmão de .
7
Problema 163. Um forro retangular de tecido traz em sua etiqueta a informação de que encolherá após a
primeira lavagem mantendo, entretanto, seu formato. A figura a seguir mostra as medidas originais do forro e
o tamanho do encolhimento x no comprimento e y na largura. A expressão algébrica que representa a área do
forro após ser lavado é (5 − x )(3 − y). Nessas condições, a área perdida do forro, após a primeira lavagem,
será expressa por:

a) 2xy.

b) 15 − 3x.

c) 15 − 5y.

d) −5y − 3x.

Figura 10 e) 5y + 3x − xy.

Problema 164. Um professor ensinou a seus alunos a seguinte identidade:

Para quaisquer inteiros a e b, Conhecendo esta identidade, determine:

a) 1002 − 992 + 982 − 972 + ... + 22 − 12 .


a2 − b2 = ( a + b)( a − b).
b) dois números inteiros maiores que 1 cujo produto
é 999991.

Problema 165. Um feirante tinha uma cesta de ovos para vender e atendeu sucessivamente a 3 fregueses.
Cada freguês levou a metade dos ovos e mais meio ovo do total de ovos existentes na cesta. Se o feirante não
precisou quebrar nenhum ovo e sobraram 10 ovos na cesta, quantos ovos havia inicialmente?

21
POTI 2015 − Álgebra − Nı́vel 2 − Aula 0 − Professores Cleber Assis, Samuel Barbosa e Tiago Miranda

10 Sistemas de Equações do 1◦ Grau


Problema 166. Resolva os sistemas de equações abaixo.
  
 5x − y
  
 x+y = 3
  x+y = 5
  = 34
a) e) i)
  
 x−y = 1
  x−y = 1
  3x − 4y =

0
  
 
 x + y = 10
  x+y = 4
  3x − 2y

 = 11
b) f) j)
  
 x−y =

8  x−y = 3
  − x + 5y =

5
 
 
 x+y =
 10  x + y = 3/2

c) g)
 

 x = y+6  x − y = 1/2

 
 
 x = y + 18
  x+y
 = 5
d) h)
 
 x =

2y  2x − y = 4.

Problema 167. Resolva um sistema de equações no qual a soma de dois números é 70 e a diferença é 28.
Problema 168. Pedro e Mariano têm juntos 195 bolinhas de gude. Se Pedro tem 45 bolinhas de gude a mais
que Mariano, quantas cada um tem?
Problema 169. Guilherme e Santiago juntaram suas economias para comprar um videogame. Guilherme
conseguiu juntar o dobro da quantia de Santiago. Além disso, a diferença entre as economias de ambos é
R$350, 00. Quanto cada um conseguiu guardar?
Problema 170. Rúbia comprou um sapato e uma blusa, pagando o total da compra com uma nota de
R$50, 00 e duas notas de R$20, 00. Sabendo que ela recebeu de troco uma nota de R$10, 00, uma nota de
R$5, 00, três moedas de R$1, 00 e que a blusa é R$10, 00 mais cara que o sapato, quanto custou cada um dos
produtos?
Problema 171. O cientista M. A. Luco tem duas provetas (recipientes para lı́quidos) e cada uma delas está
cheia com uma substância quı́mica (plutônio ou patetônio). Se a capacidade dos dois recipientes somadas é
375ml e sua diferença é 75ml, quanto ele possui de cada substância, sabendo que ele possui mais plutônio que
patetônio?
Problema 172. Resolva o sistema de equações:

 13p − 92q =

 237

 12p − 91q = 237.

Problema 173. Benzildo cria gansos e hipopótamos (que convivem harmoniosamente bem). Se o total de
animais é 50 e o total de patas é 140, qual a quantidade de cada um deles?
Problema 174. Um estacionamento possui 47 veı́culos, entre carros e motos, num total de 164 rodas. Quantos
são os carros e quantas são as motos? (Lembre-se que carros possuem quatro rodas e motos, duas)

22
POTI 2015 − Álgebra − Nı́vel 2 − Aula 0 − Professores Cleber Assis, Samuel Barbosa e Tiago Miranda

Problema 175. L. Santana retirou de um caixa eletrônico R$330, 00 entre cédulas de R$50, 00 e R$10, 00, num
total de 17 cédulas. Qual a quantidade de cada um dos tipos de cédulas?
Problema 176. A sequência (2, x, y, 29) é chamada de progressão aritmética pois a diferença entre cada
termo, com exceção do primeiro, e seu antecessor é constante. Determine x e y.
Problema 177. Encontre todas as soluções dos sistemas abaixo.
 
√ √
 5/x − 3/y = −1
 
  x + 2 y = 13

a) b)
 √ x + √y = 8.
 
 15/x + 7/y = 5.
 

Problema 178. Encontre todos os possı́veis valores dos números reais positivos a e b sabendo que a soma
das raı́zes desses números vale 17 e a raiz de a é o triplo da raiz de b acrescido de três unidades.
Problema 179. Resolva graficamente os sistemas abaixo.
  
 3x − y = 8
  
 x+y=3
  x + 2y = 5
 
a) b) c)
  
 x−y=1
  2x − y = 0
  x − 2y = 1

Problema 180. Encontre as soluções do sistema:



 √ √
 3 r+9 s
 = 21

 10√ √

3
r − s = 28.

Problema 181. Usando uma balança de dois pratos, verificamos que 4 abacates pesam o mesmo que 9
bananas e que 3 bananas pesam o mesmo que 2 laranjas. Se colocarmos 9 laranjas num prato da balança,
quantos abacates deveremos colocar no outro prato, para equilibrar a balança?

a) 1 b) 2 c) 4 d) 5 e) 6.

Problema 182. Quando João vai para a escola a pé e volta de ônibus, ele gasta uma hora e quinze minutos;
quando vai e volta de ônibus, ele gasta meia hora. Para cada meio de transporte, o tempo gasto na ida é igual
ao tempo gasto na volta. Quanto tempo ele gasta quando vai e volta a pé?

a) uma hora e meia. d) duas horas e quinze minutos.


b) uma hora e quarenta e cinco minutos.
e) duas horas e meia.
c) duas horas.

Problema 183. Oito vasos iguais, encaixados, formam uma pilha de 36cm de altura. Dezesseis vasos iguais
aos primeiros, também encaixados, formam outra pilha de 60cm de altura. Qual é a altura de cada vaso?

a) 15cm b) 16cm c) 18cm d) 20cm e) 22cm.

Problema 184. Um dia, curiosamente, Tiago percebeu que havia veı́culos de 1, 2, 3 e 4 rodas na garagem de
seu prédio: carrinhos de mão, bicicletas, triciclos e automóveis. Ele, o irmão e o pai decidiram contar o número
de rodas que estavam na garagem. Tiago contou 26 rodas mas esqueceu-se de contar as dos automóveis. O
irmão dele contou também 26 rodas, mas não contou as dos triciclos e o pai contou 26 rodas mas não contou
as rodas das bicicletas. Determine a quantidade de veı́culos que estavam na garagem.

23
POTI 2015 − Álgebra − Nı́vel 2 − Aula 0 − Professores Cleber Assis, Samuel Barbosa e Tiago Miranda

Problema 185. João e Ana são irmãos. João tem cinco irmãos a mais do que irmãs. Quantos irmãos Ana
tem a mais do que irmãs?

a) 2 b) 3 c) 5 d) 6 e) 7.

Problema 186. As massas de todos os pares possı́veis formados com 5 estudantes são 90 kg, 92 kg, 93 kg, 94
kg, 95 kg, 96 kg, 97 kg, 98 kg, 100 kg e 101 kg. Qual é a massa do estudante de massa intermediária?

a) 52kg b) 51kg c) 49kg d) 48kg e) a2

Problema 187. Rosa resolveu distribuir R$250, 00 para seus sobrinhos, dando a mesma quantia inteira (sem
centavos) para cada um e percebeu que sobrariam R$10, 00. Então, ela pensou em diminuir em R$1, 00 a
quantia de cada um e descobriu que sobrariam R$22, 00. Por fim, ela resolveu distribuir apenas R$240, 00.
Quanto ganhou cada sobrinho?

a) 5 reais. b) 10 reais. c) 12 reais. d) 15 reais. e) 20 reais.

√ √ √
Problema 188. Se x, y, a e b são reais positivos tais que x−y = a e x+ y = b, determine o valor de

xy.

b4 − a4 a2 a2 + b2 1 e) a2
a) b) c) d)
4b2 b b b

Problema 189. Uma balança de dois pratos está equilibrada, onde de um lado estão dois copos cheios e do
outro três copos pela metade. Os copos são idênticos e contêm, ao todo, 1400 gramas de farinha. Qual é o
peso, em gramas de um copo vazio?

a) 50 b) 125 c) 175 d) 200 e) 250.

Problema 190. Certo dia, Isabela e Ana Beatriz saı́ram para vender pastéis na praia. Elas tinham juntas 460
pastéis. No final do dia, verificou-se que Isabela conseguiu vender 3/5 dos pastéis que levara e Ana Beatriz,
5/8 dos pastéis que levara. Ao final do dia, o número de pastéis que restou para Ana Beatriz era a metade do
número de pastéis que restou para Isabela. Se Ana Beatriz, levou x pastéis para vender, então a soma dos
algarismos de x é

a) 6 b) 7 c) 8 d) 9



x  x + 1/y =
 4
Problema 191. Calcule , sabendo que
y 
 y + 1/x = 1/4.

Problema 192. Hoje, dia 29 de julho de 2012, José tem o dobro da idade que Luiz tinha quando José tinha a
idade que Luiz tem. Quando Luiz tiver a idade que José tem, a soma das idades deles será 90 anos. Em 29 de
julho de 2017, a razão entre as idades de José e Luiz, nessa ordem, será

6 9 5 27
a) b) c) d)
5 7 4 20

24
POTI 2015 − Álgebra − Nı́vel 2 − Aula 0 − Professores Cleber Assis, Samuel Barbosa e Tiago Miranda

Problema 193. Pitágoras e Tales possuem hoje, cada um, certa quantia em reais. Se Pitágoras desse para
Tales R$50, 00, eles ficariam com a mesma quantia em reais, cada um. Porém, se Tales desse para Pitágoras
1
R$100, 00, Tales passaria a ter da quantia de Pitágoras. Dessa forma é correto afirmar que
4
a) a quantia que os dois possuem hoje, juntos, é menor que R$600, 00.
2
b) Pitágoras possui hoje do que Tales possui.
3
c) Tales possui hoje mais de R$220, 00.

d) a diferença entre os valores que eles possuem hoje é menor que R$100, 00.

Problema 194. Na fabricação de um produto é utilizado o ingrediente A ou B. Sabe-se que, para cada 100
kg do ingrediente A devem ser utilizados 10 kg do ingrediente B. Se, reunindo x kg do ingrediente A com y
kg do ingrediente B resulta 44000 g do produto, então
√ √ √ √ r

a) y x = 260 b) xy = 5 10 c) 10 y x = 256 d) 4 x y = 20 y
e) = 2 5.
x

Problema 195. Dado que a e b são números reais não nulos, com b diferente de 4a, e tais que

 2
 1+
 = 5
ab
2
 5 − 2b

= 4a + b,

4a − b

qual é o valor de 16a4 b2 − 8a3 b3 + a2 b4 ?

a) 4 1 1 d) 18 1
b) c) e)
18 12 4
Problema 196. Resolva os sistemas de equações abaixo.
 
 
2x1 + x2 + x3 + x4 + x5 = 6







 x1 + x2 + x3 = 6
 


 

 
x1 + 2x2 + x3 + x4 + x5 = 12 x2 + x3 + x4 = 9

 


 


 

 

a) x1 + x2 + 2x3 + x4 + x5 = 24



 x3 + x4 + x5 = 3

 


 


 
x1 + x2 + x3 + 2x4 + x5 = 48 x4 + x5 + x6 = −3
 


 
b)




 
 x + x2 + x3 + x + 2x5 = 96.
 
x5 + x6 + x7 = −9
1 4








x6 + x7 + x8 = −6












 x7 + x8 + x1 = −2




x8 + x1 + x2 = 2.


25
POTI 2015 − Álgebra − Nı́vel 2 − Aula 0 − Professores Cleber Assis, Samuel Barbosa e Tiago Miranda

11 Equações do 2o grau
Problema 197. A equação ax2 + bx + c = 0, com a 6= 0 e a, b e c constantes, é denominada equação do
segundo grau na variável x. Os números a, b e c são os coeficientes da equação. Observe os modelos abaixo e
identifique-os.
i) 2x2 − 9x + 3 = 0 , então a = 2, b = −9 e c = 3.
ii) x2 + 2x − 6 = 0 , então a = 1, b = 2 e c = −6.
iii) − x2 + 5x + 3 = 0 , então a = −1, b = 5 e c = 3.

a) x2 − 2x + 6 = 0 c) − x2 + 4x − 3 = 0 e) x2 + x = 0

b) 2x2 + 3x − 8 = 0 d) −4x2 + 7x − 12 = 0 f) x2 − 25 = 0

Problema 198. Siga o modelo e, após o desenvolvimento dos produtos notáveis, identifique os valores dos
coeficientes a, b e c nas equações do 2◦ grau resultantes das operações.
( x + 1)2 + ( x − 2)2 = ( x + 3)2
x2 + 2x + 1 + x2 − 4x + 4 = x2 + 6x + 9
2x2 − 2x + 5 − x2 − 6x − 9 = 0
x2 − 8x − 4 = 0
Então a = 1, b = −8 e c = −4.
a) ( x − 1)2 + ( x + 2)2 = ( x − 3)2
b) (2x − 5)2 + ( x − 2)( x + 2) = x + ( x + 7)2
c) ( x − 1)2 + x ( x + 1) = 2x − ( x + 3)2
Problema 199. Faça a expansão dos produtos indicados como no exemplo abaixo.
( x + 2)( x + 3) = x2 + 3x + 2x + 2 · 3
= x2 + 5x + 6.

a) x ( x + 7). b) ( x + 2)( x + 5). c) ( x − 2)( x + 3).

Problema 200. O discriminante da equação do segundo grau ax2 + bx + c = 0 é o número ∆ = b2 − 4ac.


Calcule-o em cada um dos itens abaixo.

a) x2 − 5x + 4 = 0 d) 3x2 + 5x + 1 = 0 g) x2 + 3x + 9 = 0

b) 5x2 + 3x − 2 = 0 e) − x2 − 2x − 1 = 0 h) x2 + 9x = 0

c) − x2 + x + 30 = 0 f) 2x2 + 6x − 8 = 0 i) − x2 + 16 = 0

Problema 201. Sejam m e n números tais que


( x − m)( x − n) = x2 − 7x + 10.
a) Determine o valor de m + n.
b) Determine o valor de mn.
c) Encontre m e n que satisfazem a soma e o produto encontrados nos itens anteriores.
d) Encontre as soluções da equação.

26
POTI 2015 − Álgebra − Nı́vel 2 − Aula 0 − Professores Cleber Assis, Samuel Barbosa e Tiago Miranda

Problema 202. Observe os modelos e resolva as equações do 2◦ grau no universo dos números reais.

i) Modelo 1. ii) Modelo 2.


x2 + 9x = 0
x2 − 64 = 0
x ( x + 9) = 0
x2 = 64
Numa multiplicação com resultado nulo, ao me- √
x = ± 64
nos um dos fatores deve ser zero, isto é:
x = ±8
a · b = 0 então a = 0 ou b = 0.
Logo, x = 0 ou x + 9 = 0. Portanto, o conjunto Logo, as raı́zes são x = 8 ou x = −8 e o conjunto
solução é S = {−9, 0}. solução é S = {−8, 8}.

a) x2 − 4x = 0 c) x2 + 9x = 0 e) − x2 − 7x = 0

b) x2 − 4 = 0 d) x2 + 9 = 0 f) − x2 + 121 = 0

Problema 203. Verifique se −1, 2 ou 5 são raı́zes da equação x2 − 7x + 10 = 0.


Problema 204. Qual o valor de m para que −3 seja raiz da equação −mx2 − 4mx + 21 = 0?
Problema 205. As raı́zes da equação do 2◦ grau, ax2 + bx + c = 0, podem ser encontradas através da fórmula

−b ± b2 − 4ac
x=
2a
na qual a expressão b2 − 4ac é normalmente chamada de discriminante e representada pela letra grega ∆.
Verifique em quais equações abaixo o discriminante é positivo, negativo ou nulo.

a) x2 − 5x + 4 = 0 d) x2 + 2x + 10 = 0 g) x2 + 16x + 64 = 0
b) x2 − x − 6 = 0 e) −3x2 + x + 4 = 0
c) −2x2 + 3x + 2 = 0 f) x2 + x + 4 = 0

Problema 206. A partir da fórmula de Bhaskara, observando o modelo abaixo, calcule as raı́zes de cada uma
das equações que seguem.
2x2 + 2x − 4 = 0

Tem-se ∆ = (2)2 − 4 · 2 · (−4) = 4 + 32 = 36 e, portanto, as raı́zes são: Daı́, ∆ = 6 e

−2 + 6
x1 =
4
= 1
−2 − 6
x2 =
4
= −2

Logo, S = {−2, 1}.

a) x2 − 7x + 6 = 0 d) −3x2 + 1x − 10 = 0 g) 3x2 + 5x + 7 = 0
b) x2 − 5x + 4 = 0 e) x2 + 4x + 4 = 0
c) 2x2 + 1x − 10 = 0 f) 5x2 + 2x + 2 = 0

27
POTI 2015 − Álgebra − Nı́vel 2 − Aula 0 − Professores Cleber Assis, Samuel Barbosa e Tiago Miranda

Problema 207. A partir da fórmula geral das soluções de uma equação do segundo grau
√ √
−b + ∆ −b − ∆
x1 = e x2 = ,
2a 2a

−b ± ∆
x= ,
2a
analise a quantidade de raı́zes reais em função do discriminante ∆.
h5 2h6
Problema 208. Sendo h a maior raiz da equação x2 + x − 1 = 0. Então qual o valor de + ?
1 − h (1 − h )2
Problema 209. Um grupo de jovens aluga, por 342 reais, uma van para um passeio, sendo que três deles
saı́ram sem pagar. Por isso, os outros tiveram que completar o total pagando, cada um deles, 19 reais a mais.
Qual o número inicial de jovens no grupo?
Problema 210. Os primeiros dı́gitos da representação decimal do número a são 2, 45. Determine se a2 − 5a + 6
é positivo ou negativo.
Problema 211. Encontre os valores de a para os quais a equação x2 − ax + 1 = 0 não possui raı́zes reais.
Problema 212. Encontre todos os valores de k para os quais a equação x2 + 2(k − 1) x + (k + 5) = 0 possui
pelo menos uma raiz positiva.
Problema 213. Qual a maior raiz da equação −2x2 + 3x + 5 = 0?
Problema 214. Calcule as soluções da equação 2x2 − 4x + 2 = 0.
Problema 215. O número −3 é a raiz da equação x2 − 7x − 2c = 0. Nessas condições, determine o valor do
coeficiente c.
1 1
Problema 216. Seja x um número real não nulo tal que x + = 2. Calcule o valor de x2 + 2 .
x √ x

Problema 217. Qual o conjunto solução da equação no universo dos reais. 3x − 2 = x + 2?
Problema 218. A equação ax4 + bx2 + c, com a 6= 0, é denominada equação biquadrada. É possı́vel encontrar
suas soluções fazendo a mudança de variável x2 = y e assim transformando-a em uma equação do segundo
grau. Por exemplo, para encontrarmos as raı́zes de x4 − 18x2 + 32 = 0, trocamos x2 por y obtendo:
x4 − 18x2 + 32 = 0
y2 − 18y + 32 = 0

18 ± 196
y =
2
y = 9 ± 7.
2
Analisamos agora separadamente cada um dos possı́veis
√ 2 valores para √ x. No primeiro caso, se y = 9 + 7 = 4 ,
então x = ±4. No segundo caso, se y = 9 − 7 = ( 2) , então x = ± 2. Portanto, o conjunto solução é
√ √
S = {−4, 4, − 2, 2}.
Em alguns casos, pode ocorrer que o valor encontrado para y seja negativo e consequentemente não existiriam
valores no conjunto dos números reais correspondentes para x. Seguindo o modelo anterior, encontre as raı́zes
reais das equações abaixo:

a) 9x4 − 13x2 + 4 = 0. b) x4 + 4x2 − 60 = 0. c) x4 + 10x2 + 9 = 0.

Problema 219. Encontre x2 + y2 se x, y ∈ Z e




 xy + x + y = 71


 x2 y + xy2 = 880.

28
POTI 2015 − Álgebra − Nı́vel 2 − Aula 0 − Professores Cleber Assis, Samuel Barbosa e Tiago Miranda

Problema 220. Resolva a equação

( x2 − 3x + 1)2 − 3( x2 − 3x + 1) + 1 = x.

Problema 221. Encontre as soluções de:


p
4 + 2x − x2 = x − 2.

Problema 222. Encontre as soluções de:


p
x2 + 2ax − a = x − 1.

Problema 223. Para quais valores de a a equação

( a2 − 3a + 2) x2 − ( a2 − 5a + 4) x + a − a2 = 0

possui mais que duas raı́zes?


Problema 224. Mostre que se a, b e c são inteiros ı́mpares, a equação ax2 + bx + c = 0 não tem raiz racional.
Problema 225. Encontre todas as soluções reais de:
q √
x = 2+ 2+ x.

Problema 226. A calculadora MK − 97 pode efetuar as seguintes três operações com números em sua
memória:

a) Determinar se dois números escolhidos são iguais;

b) Adicionar dois números escolhidos;

c) Para os números escolhidos a e b, determinar as raı́zes reais da equação x2 + ax + b = 0 ou anunciar que


tal equação não possui raı́zes reais.

Os resultados de cada operação são acumulados em sua memória. Inicialmente a memória contém apenas o
número z que é desconhecido por seus usuários. Como podemos determinar, usando a calculadora MK − 97,
se z é igual a 1?
Problema 227. Encontre todas as soluções reais de
√4

x − 1 + 4 5 − x = 2.
p √
Problema 228. Resolva a equação 5 − 5 − x = x, com 0 < x < 5.
Problema 229. Para quais valores de r vale que:

(r2 + 5r − 24)(r2 − 3r + 2) = (4r − 10)(r2 + 5r − 24)?

Problema 230. Sendo a e b inteiros não nulos, resolva a equação

( ax − b)2 + (bx − a)2 = x

sabendo que uma de suas raı́zes é um inteiro positivo.


Problema 231. Encontre as raı́zes das equações:

a) x2 − | x | − 2 = 0.

b) x2 + 5| x | + 4 = 0.

29
POTI 2015 − Álgebra − Nı́vel 2 − Aula 0 − Professores Cleber Assis, Samuel Barbosa e Tiago Miranda

Problema 232. Determine todos os y tais que

(y2 + y − 6)(y2 − 6y + 9) − 2(y2 − 9) = 0.

Dica: Tente fatorar as expressões dadas.


Problema 233. Na equação x2 + px + q = 0, os coeficientes p e q podem assumir qualquer valor no intervalo
[−1, 1]. Quais são os possı́veis valores das raı́zes de tal equação?
Problema 234. Encontre as soluções de:
p
2( x − 3) = x2 − 2x + 3

Problema 235. Encontre todas as soluções reais de:

4x2
x2 + = 12
( x − 2)2

Problema 236. Deduza a formula para as raı́zes de ax2 + bx + c = 0, com a 6= 0, em função dos coeficientes
da equação.
Problema 237. Suponha que ax2 + bx + c = mx2 + nx + p para três valores reais distintos da variável x. É
verdade que a = m, b = n e c = p?

30
POTI 2015 − Álgebra − Nı́vel 2 − Aula 0 − Professores Cleber Assis, Samuel Barbosa e Tiago Miranda

12 Relações entre Coeficientes e Raı́zes


Problema 238. Fazendo as operações de soma e de produto entre as raı́zes x1 e x2 em uma equação do 2◦
grau, ficamos com:
√ ! √ ! √ ! √ !
−b + ∆ −b − ∆ −b + ∆ −b − ∆
x1 + x2 = + x1 · x2 = ·
2a 2a 2a 2a
√ √
−b + ∆ − b − ∆ b2 − ( ∆ )
= =
2a 4a2
−2b b − (b2 − 4ac)
2
= =
2a 4a2
−b b − b2 + 4ac
2
= =
a 4a2
4ac
=
4a2
c
=
a
−b c
Em resumo, x1 + x2 = e x1 · x2 = . Agora, calcule a soma e o produto das raı́zes das equações abaixo.
a a
a) x2 − 5x + 4 = 0 c) −3x2 + 12x = 0 e) 7x2 − 3x + 1 = 0
b) 2x2 + 7x + 5 = 0 d) − x2 + 8x − 12 = 0

Problema 239. Se x1 e x2 são as raı́zes da equação x2 + 5x + 7 = 0, determine o valor de ( x1 + 3)( x2 + 3).


Problema 240. Sabendo que a e a + 1 são as raı́zes de x2 − ( p − 1) x + p, determine o valor de p.
Problema 241. Qual o valor de b na equação x2 + bx − 25 = 0 para que suas raı́zes sejam simétricas?
Para tal valor, veja que 5 e −5 são raı́zes da equação.
Problema 242. Determine a soma e o produto das raı́zes das equações abaixo:

a) x2 − 4x − 5 = 0 b) − x2 + 7x − 10 = 0 c) 2x2 − 3x + 1 = 0 d) x2 − x − 20 = 0

Problema 243. Determine m para que a equação

2x2 − 8x + m = 0

admita raı́zes iguais.


Problema 244. Determine m para que a equação

3x2 + 6x + m = 0

admita raı́zes reais distintas.


Problema 245. Determine os possı́veis valores de p para que a equação

x2 + p2 x + 3px − 8 = 0

admita raı́zes simétricas1 .


Problema 246. Determine m na equação 2x2 − 12x + 2m = 0 de modo que uma de suas raı́zes seja igual ao
dobro da outra.
Problema 247. Se a média aritmética de dois números reais a e b é 5 e a média geométrica entre eles é 8,
escreva uma equação do segundo grau que admite a e b como raı́zes.
1 Dizemos que x e y são simétricos se x = −y.

31
POTI 2015 − Álgebra − Nı́vel 2 − Aula 0 − Professores Cleber Assis, Samuel Barbosa e Tiago Miranda

Problema 248. Qual o valor de p na equação ( p + 3) x2 + x + p2 − p = 0 para que uma raiz seja inversa da
outra?
Problema 249. Qual o valor de m na equação x2 + (m − 2) x + 2m − 8 = 0 para que os quadrados de suas
raı́zes sejam iguais?
Problema 250. Um aluno resolveu corretamente a equação do 2o grau x2 + Ax + B = 0 e encontrou as raı́zes
1 e −3. Nessas condições, determine as soluções da equação x2 + Bx + A = 0.
Problema 251. Determine uma equação do 2o grau com a soma das suas raı́zes igual a 4 e o produto das
suas raı́zes igual a −12.
 
2 b a
Problema 252. Se a equação quadrática x − 2nx + n + 3 = 0 tem como conjunto solução + 1, + 1 .
a b
Qual o valor de n2 ?
Problema 253. Uma equação do 2o grau possui a soma das suas raı́zes igual a 4 e o produto das suas raı́zes
igual a −12. A partir desses dados, sendo x1 e x2 as raı́zes de tal equação, calcule o que se pede.
1 1 b) x12 + x22 . c) | x1 − x2 |.
a) + .
x1 x2
Problema 254. Suponha que x1 e x2 são as raı́zes de
2
x + x − 7 = 0. Encontre os valores das expressões abaixo sem encontrar explicitamente as raı́zes:

a) x12 + x22 . b) x13 + x23 . c) x14 + x24 .

Problema 255. Se a e b são as raı́zes de 2x2 − x + 3 = 0, encontre o valor de (2a − 1)(2b − 1) + 8.


Problema 256. Sabendo que a equação 4x2 + (3m + 1) x + (n + 3) = 0 possui as mesmas raı́zes de 2x2 + 5x +
8 = 0, determine m e n.
1 1
Problema 257. Os números reais p e q são as raı́zes da equação 15x2 − 11x + 2 = 0. Qual o valor de + ?
p q
Problema 258. Determine o valor de p para que as raı́zes x1 e x2 da equação 2x2 − px − 1 = 0 satisfaçam a
relação x12 + x22 = 1.
Problema 259. Determine a soma das raı́zes da equação
p
x2 + 18x + 30 = 2 · x2 + 18x + 45.

Problema 260. Sejam a e b números reais não nulos tais que a equação x2 + ax + b = 0 possui soluções a e b.
Determine a − b.
Problema 261. (Olimpı́ada Cearense) As raı́zes da equação do segundo grau ax2 + bx + c = 0 são R e S.
Determine a equação do segundo grau cujas raı́zes são aR + b e aS + b.
Problema 262. Mostre que a equação x2 + bx + 17 = 0 não possui raiz inteira positiva, se b é um inteiro não
negativo.
Problema 263. Dentre os trinômios do segundo grau da forma x2 + px + q, onde p e q são inteiros e
1 ≤ p, q ≤ 1997, qual conjunto é maior: o conjunto A dos trinômios que possuem raı́zes inteiras ou o conjunto
B daqueles que não possuem raı́zes reais?
Problema 264. a, b, c e d são números reais distintos tais que a e b são raı́zes da equação x2 − 3cx − 8d = 0 e
c e d são raı́zes da equação x2 − 3ax − 8b = 0. Calcule a soma a + b + c + d.
Problema 265. Se a e b são raı́zes da equação x2 − 2014x − 2004 = 0, determine o valor de

a2 + b2 + a2 b2 + 2ab( a + b + 1).

Problema 266. Determine um valor do número real m para o qual a soma das quartas potências das raı́zes de
x2 − mx − 1 = 0 seja mı́nima.

32
POTI 2015 − Álgebra − Nı́vel 2 − Aula 0 − Professores Cleber Assis, Samuel Barbosa e Tiago Miranda

Problema 267. Seja a a maior raiz de x2 + x − 1. Determine o valor de a5 − 5a.

a) −1 b) −2 c) −3 d) 1 e) 2.

Problema 268. Determine os valores do número real k para que diferença entre as raı́zes da equação em x
dada por x2 − (k + 2) x + k − 1 = 0 seja mı́nima.
Problema 269. Observe:
( x − r )( x − s) = x2 − (r + s) x + rs.
Assim, substituindo x por r e por s, obtemos

 r2 − (r + s)r + rs =

 0

 s2 − (r + s)s + rs = 0.

Multiplicando a primeira equação por a · r n e a segunda por b · sn , temos:



 a(r n+2 − (r + s)r n+1 + rs · r n ) =

 0

 b(sn+2 − (r + s)sn+1 + rs · sn ) = 0.

Somando as duas equações e sendo Sn = a · r n + b · sn , verifica-se que

Sn+2 = (r + s)Sn+1 − rsSn .

Dados que S1 = ar + bs = 1, S2 = ar2 + bs2 = 2, S3 = ar3 + bs3 = 5 e ar4 + bs4 = 6, determine S5 = ar5 + bs5 .

33
POTI 2015 − Álgebra − Nı́vel 2 − Aula 0 − Professores Cleber Assis, Samuel Barbosa e Tiago Miranda

13 Equações Fracionárias
Problema 270. Resolva a equação
3 2
−2 = 7+
x x
Problema 271. Resolva a equação
x 2 2
+ = .
x−1 3 x−1
Problema 272. Resolva a equação
1 2x
+ =2
x x−1
Problema 273. Calcule
x2 x x
2
− +
x −1 x−1 x+1
Problema 274. Resolva as equações fracionárias:
x+6 3
a) = .
x 2
x+1 x−1
b) = .
x−2 x+2
3 1 −3x + 4
c) + = 2 .
x x−1 x −x
Problema 275. Resolva a equação:
2x 3x 5 − x2
− = 2 .
x−1 x+1 x −1
Problema 276. Resolva a equação:
4 1 3
+ = 2 .
x−2 x+2 x −4
Problema 277. Resolva a equação
3 4x
+ = 4.
x−1 x+1
Problema 278. Encontre as soluções de

1 2 6
+ = 2 .
x−3 x+3 x −9
Problema 279. Resolva a equação

3x − 1 3x + 2 1
+ = 3− 2 .
2x − 1 2x + 1 4x − 1

Problema 280. Supondo a2 − b2 6= 0 e b 6= 0, determine o valor de z na equação

5a 5a 2bz
− = 2 .
a−b a+b a − b2
Problema 281. Determine o valor de x, em função a, de modo que

x 1 x
= − 2 .
a−3 a+3 a −9

34
POTI 2015 − Álgebra − Nı́vel 2 − Aula 0 − Professores Cleber Assis, Samuel Barbosa e Tiago Miranda

Problema 282. Determine o valor de x em função dos inteiros m e n de modo que

x x+1 x−3
− = 2 .
m+n m−n m − n2
Problema 283. Encontre o número de valores de x que satisfazem a equação:

2x2 − 10x
= x − 3.
x2 − 5x
Problema 284. Dados os números reais positivos m, n, p, q e r, resolva a equação em x:
x−m x−n−p x−q−r
+ + = 3.
n+ p+q+r q+r+m m+n+ p

Problema 285. Encontre os valores de x que satisfazem a equação


2 2
x−a x 2 − a2
   
x+a a b
+ − + = 0.
x+b x−b b a x 2 − b2

Problema 286. Se ab 6= 0 e | a| 6= |b|, o número de valores distintos de x que satisfazem a equação:

x−a x−b b a
+ = +
b a x−a x−b
é:
a) 0 b) 1 c) 2 d) 3 e) 4.
Problema 287. Os números N1 e N2 são inteiros tais que a equação

35x − 29 N1 N2
= +
x2 − 3x + 2 x−1 x−2
possui infinitas soluções. Qual o valor de N1 N2 ?
Problema 288. Prove que a equação

1 1 1 1 1 1
+ + + + + =1
a b c d e f

não admite soluções com todos os números sendo ı́mpares.


Problema 289. Seja Bn a quantidade de n − uplas ordenadas de inteiros positivos ( a1 , a2 , . . . , an ) tais que

1 1 1
+ +···+ =1
a1 a2 an
Determine se B10 é par ou ı́mpar.
Problema 290. Encontre todas as soluções de

( x + a)( x + b) ( x + b)( x + c) ( x + c)( x + a)


+ + = 3,
( x − a)( x − b) ( x − b)( x − c) ( x − c)( x − a)

onde a, b e c são parâmetros reais positivos.


Problema 291. Encontre todas as soluções reais de

4x2
x2 + = 12.
( x − 2)2

35
POTI 2015 − Álgebra − Nı́vel 2 − Aula 0 − Professores Cleber Assis, Samuel Barbosa e Tiago Miranda

14 Sistema de Equações Fracionárias


Problema 292. Resolva o sistema de equações

 1
 + 3y = 6

x
 2
 − 5y = 1.

x
Problema 293. Resolva o sistema de equações

 x 1
=


x+y 3

 4
= −2.


x−y

Problema 294. Encontre todos os pares ( x, y) tais que



 3 2 7
 − = −


x y 2
 6 4
 + = 9.


x y
Problema 295. Determine as soluções do sistema

 1 1
+ = 1


x y

 12 3


 − = 5
y x
Problema 296. Resolva o sistema de equações

 4 7
 + = −18


x y
 1 2
 + = −7.


x y

Problema 297. Encontre todos os pares ordenados ( x, y) que satisfazem o sistema



 x
 2

 −3 =
y y

 2x − 9y = −8.

Problema 298. Encontre as soluções do sistema



 2 1 1
+ + = 11



 x

 y z

1 2 1

+ + = 12


 x y z


 1 1 2
 + + = 13


x y z

36
POTI 2015 − Álgebra − Nı́vel 2 − Aula 0 − Professores Cleber Assis, Samuel Barbosa e Tiago Miranda

1 1
Problema 299. Se x e y são números não nulos tais que x = 1 + e y = 1 + , então y é igual à:
y x
a) x − 1 b) 1 − x c) 1 + x d) − x e) x.
x
Problema 300. Calcule se
y 
 1
 x+ = 4


y

 1 1
 y+
 =
x 4
Problema 301. Encontre todos os pares ordenados ( x, y) tais que

 3x − 4y

 = −8
xy

 2x + 7y
= 43



xy
Problema 302. Encontre soluções do sistema

 x2 y2
+ = 12


y x

 1 1 1
+ = .



x y 3
Problema 303. Resolva o sistema de equações:

 xyz

 = 2



 x +y

xyz 6
=


 y+z 5

 xyz 3


= .

z+x 2
Problema 304. Dado que todos os números reais a1 , a2 , . . . , an são positivos, encontre as soluções do sistema
de equações abaixo nas incógnitas x1 , x2 , . . . , xn :

 x2 x3 . . . x n

 = a1



 x1
 x1 x3 . . . x n



 = a2
x1

... ...







 x 1 x 2 . . . x n −1


 = an
xn
Problema 305. Supondo que a, b, c 6= 0 e que o sistema abaixo tem solução, determine o valor de x.

 xy

 = a



 x +y

xz
= b


 x+z
yz




 = c.
y+z

37
POTI 2015 − Álgebra − Nı́vel 2 − Aula 0 − Professores Cleber Assis, Samuel Barbosa e Tiago Miranda

Problema 306. Resolva o sistema 


 2a2
= b


1 + a2




2b2

= c
 1 + b2


2c2




 = a
1 + c2
Problema 307. Em sua velocidade usual, um homem desce um rio de 15 quilômetros de comprimento em 5
horas a menos que o tempo que ele gasta nadando no mesmo rio percorrendo o caminho contrário. Se ele
dobrar a sua velocidade usual, ele passa a descer o rio gastando apenas 1 hora a menos que o tempo gasto na
volta. Considerando que a velocidade da correnteza do rio se mantém constante durante os trajetos, qual o
seu valor km/h?
a) 2 b) 5/2 c) 3 d) 7/2 e) 4.

Dica: Quando o homem nada no sentido da correnteza, a sua velocidade relativa deve ser somada com a
do rio e, quando nada no sentido contrário ao da correnteza, a velocidade do rio deve ser subtraı́da de sua
velocidade.

38
POTI 2015 − Álgebra − Nı́vel 2 − Aula 0 − Professores Cleber Assis, Samuel Barbosa e Tiago Miranda

Respostas e Soluções.
1.

a) 243. b) 4 + 9 = 13. c) 625. d) 8 + 27 = 35. e) 24−1 · 3 = 24.


2.

a) 0, 000001. 125 256


c) 80 · = 1250. e) 200 · = 5, 12.
8 10000
1
b) 4. d) · 0, 09 = 0, 03.
3
3.
8 b) 8. c) 72. d) 324. e) 52 − 22 = 21.
a) .
3
4.
 10
2 b) 26 c) −245 . d) 106 . e) 213 .
a) .
3
5.
a) Verdadeiro.
1
b) Falso. Por exemplo, 2−1 = 6= −2.
2
 2
2
c) Falso. Por exemplo, = 4 6= (2 − 1)2 = 1.
1
d) Verdadeiro.
3
e) Falso. Por exemplo, (22 )3 = 64 6= 256 = 2(2 ) .
6.
a) Verdadeiro.
b) Falso. Por exemplo, (1 + 2)3 = 27 6= 9 = 13 + 23 .
c) Falso. Por exemplo, 22+1 = 8 6= 5 = 22 + 21 .
1
d) Falso. Por exemplo, (22 )−2 = 6= 1 = 20 .
16
e) Verdadeiro.
7.

a) 0, 09. 100 c) −0, 000008. 1 e) 1, 728.


b) . d) .
9 9
8.

a) 24 = 16. b) 224 . c) 1. d) 2−18 . e) 24 .


9.
a) Como 23 > 22 , segue que 21/2 = (23 )1/6 > (22 )1/6 = 21/3 .
1 1
b) Pelo item anterior, 21/2 > 21/3 e consequentemente < 1/3 .
21/2 2
c) Como 33 < 55 , segue que 31/5 = (33 )1/15 < (55 )1/15 = 51/3 .

39
POTI 2015 − Álgebra − Nı́vel 2 − Aula 0 − Professores Cleber Assis, Samuel Barbosa e Tiago Miranda
2 2 −4
10. 44 : 44 = 44 = 412 . Resposta E.
11. Fazendo o desenvolvimento segundo a regra definida no enunciado chegaremos a:

2 ⊗ (2 ⊗ (2 ⊗ 2)) 2 ⊗ (2 ⊗ 4)
=
((2 ⊗ 2) ⊗ 2) ⊗ 2 (4 ⊗ 2) ⊗ 2
2 ⊗ 16
=
16 ⊗ 2
216
=
162
= 28 .

Resposta E.

12. Como 2 ∗ x = 2x + x2 e x é inteiro, devemos ter x2 ∈ {12 , 22 , . . . , 102 }. Dentre os elementos listados, o
único possı́vel para o qual 100 − x2 é uma potência de 2 é x2 = 36 pois nesse caso x = 6 e 100 − x2 = 64 = 26 .
Consequentemente (4x )4 = 256x4 = 256 · 1296 = 331776. Resposta E.

13. Utilizando as propriedades de potências teremos que:

156 · 285 · 557 = (36 · 56 ) · (210 · 75 ) · (57 · 117 )


= 36 · 117 · 53 · 1010

Logo, o número termina em 10 zeros. Resposta A.

14. Representemos os dı́gitos desconhecidos de 2n e 5n com asteriscos. Se k e l são as quantidades de


algarismos de cada um deles, temos:

d · 10k < d ∗ ∗ ∗ . . . ∗ = 2n < (d + 1) · 10k


d · 10l < d ∗ ∗ ∗ . . . ∗ = 5n < (d + 1) · 10l

Multiplicando ambas as inequações, obtemos 10k+l · d2 < 10n < 10k+l · (d + 1)2 . Cancelando 10k+l em ambos
os lados, concluı́mos que existe uma potência de 10 entre d2 e (d + 1)2 . Analisando os quadrados dos dı́gitos
de 1 até 9, percebemos que isso ocorre apenas para d = 3( 32 < 10 < 42 ).

15. a = 240 = 1610 , b = 320 = 910 e c = 710 . Como 16 > 9 > 7, temos a > b > c. Resposta A.

16. 1212 = 126 = (22 · 3)6 = 212 · 36 . Resposta C.

17.

64 = 2(22x ) − 4x
= 2 · 22x − 22x
= 22x .

Como 64 = 26 , temos 2x = 6 e x = 3. Resposta E.

18.

a) 4, 5673 · 104 . b) 1, 2345 · 10−3 . c) −5, 55 · 102 . d) 9 · 10−2 .


19.

a) 342. b) 7. c) 98.

40
POTI 2015 − Álgebra − Nı́vel 2 − Aula 0 − Professores Cleber Assis, Samuel Barbosa e Tiago Miranda

20.

a) b) c) d)

x = 0, 333 . . . x = 0, 121212 . . . x = 6, 555 . . . x = −0, 666 . . .


10x = 3, 333 . . . 100x = 12, 121212 . . . 10x = 65, 555 . . . 10x = −6, 666 . . .
9x = 3 99x = 12 9x = 59 9x = −6
3 1 12 4 59 6 2
Logo, x = = . Logo, x = = . Logo, x = . Logo, x = − =− .
9 3 99 33 9 9 3
21.

a) b) c)

x = 4, 7222 . . . x = 1, 8999 . . . x = 1, 2010101 . . .


10x = 47, 222 . . . 10x = 18, 999 . . . 10x = 12, 010101 . . .
100x = 472, 222 . . . 100x = 189, 999 . . . 1000x = 1201, 010101 . . .
90x = 425 90x = 171 990x = 1189
425 85 171 19 1189
Logo, x = = . Logo, x = = . Logo, x = .
90 18 90 10 990
22.

a) Decimal exato. Isso ocorre pois o denominador só possui fatores primos 2 e 5.
15 5
b) Decimal exato. Isso ocorre pois = e o denominador só possui fator 2.
6 2
c) Dı́zima periódica. Trata-se de uma fração irredutı́vel com um fator primo no denominador que não é 2 e
41
nem 5. De fato, = 2, 7333 . . ..
15
d) Decimal exato. Isso ocorre pois o denominador só possui fatores primos 2 e 5.

23. Resposta B.

24. Resposta B.

25.

a) 0.

b) Um grupo de k zeros é separado de um grupo seguinte de k + 1 zeros por exatamente um número 1. Assim,
contando até o dı́gito 1 que sucede um grupo de k zeros, temos:

k ( k + 3)
1| + 2 + 3{z+ . . . + }k + k = .
|{z} 2
algarismos zeros algarismos uns

30(33)
Se k = 30, já teremos = 495. Consequentemente a 500a casa decimal vale zero pois está no grupo
2
com 31 zeros.

c) O número X não é racional porque sua representação decimal não é periódica uma vez que a quantidade
de algarismos zeros entre dois 1’s consecutivos sempre está aumentando.

41
POTI 2015 − Álgebra − Nı́vel 2 − Aula 0 − Professores Cleber Assis, Samuel Barbosa e Tiago Miranda

212
26. Multiplicando a fração inicial por teremos:
212
1 1 212
= ·
512 512 212
212
=
1012
Como 212 = 4096, o primeiro dı́gito não nulo após a vı́rgula é 4. Resposta C.
27. Veja que

s s
 3  3
1 1 6
· (0, 666 . . .) = ·
6 6 9
r
1
=
62·9
1
=
18
Além disso,
s
 0 r
2 1 1
− = 1−
3 1, 333 . . . 12/9
r
9
= 1−
12
r
3
=
12
1
=
2
Assim, o valor da expressão procurada é:
−1/2
10 −1/2
  
1 1
+ =
18 2 18
3
= √
5

3 5
=
5
Resposta E
28. Um valor possı́vel para p é 17 pois:
1
= 0, 05882352994117647.
17
Todos os primos menores que 100 que satisfazem essa propriedade são:

7, 17, 19, 23, 29, 47, 59, 61, 97.


Comentário para professores: Seja p um número primo que não divide 10 e seja n um inteiro com 0 < n < p.
Se d é o menor inteiro positivo tal que 10d − 1 é múltiplo de p, é possı́vel mostrar que o perı́odo da representação
n
decimal de é exatamente d. No exemplo anterior, como 7 não divide 101 − 1, 102 − 1, . . . , 105 − 1 e divide
p
106 − 1, temos d = 6.

42
POTI 2015 − Álgebra − Nı́vel 2 − Aula 0 − Professores Cleber Assis, Samuel Barbosa e Tiago Miranda

29. Podemos escrever 10n + 1 = p · a onde a é um número com não mais que n dı́gitos na base 10, digamos
a = a1 a2 . . . an . Queremos dizer com isso que cada número ai é um dos dı́gitos de a. Mesmo que ele possua
estritamente menos que n dı́gitos, podemos colocar alguns ai ’s da esquerda como sendo 0. Temos

1 a
=
p a·p
a
=
10n + 1
a(10n − 1)
=
102n − 1
[10n ( a − 1) + (10n − 1) − ( a − 1)]
=
102n − 1
O número 10n − 1 é constituı́do por n números iguais a 9 e a diferença (10n − 1) − ( a − 1) reduz cada um
desses dı́gitos 9 por um dı́gito de a. Assim, a representação decimal do numerador é:

a1 a2 . . . an−1 ( an − 1)(9 − a1 )(9 − a2 ) . . . (9 − an−1 )(10 − an ).

1
O numero anterior representa o perı́odo da representação de e cada dı́gito i pode ser pareado com um
p
outro dı́gito da forma 9 − i. Assim, as quantidades de aparições de tais dı́gitos são iguais. No exemplo do
enunciado, o perı́odo de 1/7 é 142857 e temos os seguintes pareamentos:

1 → 8

4 → 5

2 → 7

30. Como 102k − 1 = (10k − 1)(10k + 1) e p é primo, um dentre 10k − 1 e 10k + 1 é múltiplo de p. Não
1 (10k − 1)/p
podemos ter 10k − 1 múltiplo de p pois caso contrário poderı́amos escrever = e obterı́amos
p 10k − 1
uma dı́zima periódica com perı́odo menor do que 2k. Sendo assim, p divide 10k + 1 e podemos usar repetir a
solução anterior para concluir que o perı́odo da representação decimal de 1/p é da forma:

a1 a2 . . . ak−1 ( ak − 1)(9 − a1 )(9 − a2 ) . . . (9 − ak−1 )(10 − ak ).

Somando o número formado pelos k primeiros dı́gitos com o número formado pelos k últimos, obtemos

99 . . . 9} = 10k − 1.
| {z
k vezes

31. Números
√ irracionais são aqueles que possuem representação decimal infinita e não periódica. Sendo
assim, 2 ∈ Q0 e 0, 01001000100001 . . . ∈ Q0 pois possuem representações decimais não periódicas; ao passo
que 23 ∈ N ⊂ Q, 5, 345 ∈ Q, 31 ∈ Q, 2, 313131... ∈ Q possuem representações decimais periódicas.

√ professores: Pode ser difı́cil convencer o aluno em um primeiro contato com os números
Comentário para
irracionais que 2 é irracional e consequentemente nos primeiros exercı́cios o aluno deverá assumir tal fato.
Deixamos a demonstração desta afirmação para o final deste bloco de exercı́cios e sugerimos que o professor
faça o mesmo até seus alunos terem mais familiaridade com as distinções entre os conjuntos numéricos.

43
POTI 2015 − Álgebra − Nı́vel 2 − Aula 0 − Professores Cleber Assis, Samuel Barbosa e Tiago Miranda

32. Já sabemos que valem as inclusões N ⊂ Z ⊂ Q ⊂ R. Assim:

a) N ⊂ Q.Verdadeira! d) r ∈ Q ⇒ −r ∈ Q. Verdadeira!

b) Z ⊂ Q.Verdadeira! 35
e) ∈ Q − Z. Falsa, pois Q − Z é o conjunto das
5
c) 1 ∈ Q − Z.Falsa, pois Q − Z é o conjunto das frações não inteiras e 35
5 = 7.
frações não inteiras.
33. Uma representação seria:

34. Resposta com o uso da calculadora.

a) 1, 42 = 1, 96. b) 1, 412 = 1, 9881. c) 1, 4142 = 1, 999396. d) 1, 41422 = 1, 99996164.


35. Resposta com o uso da calculadora.

3 = 1, 7320508075688772935274463415059 . . .

a) 1, 72 = 2, 89. b) 1, 732 = 2, 9929. c) 1, 7322 = 2, 999824. d) 1, 73202 = 2, 999824.


36.
√ √ q q √ √
a) 3.2< c) 4
< 16 d) 0, 64 > 0, 1.
100 25 .
√ √ √ √
b) 81 < 121. e) n < n + 1 com n.
√ √ √
37. Observe que 9 = 3 < 11 < 16 = 4.
Agora tentemos descobrir a primeira casa decimal após a vı́rgula:

i 3, 12 = 9, 61. ii 3, 22 = 10, 24. iii 3, 32 = 10, 89. iv 3, 42 = 11, 56.

Logo, para apenas a descobrirmos a primeira casa decimal, basta observarmos que:

3, 32 < 11 < 3, 42
10, 89 < 11 < 11, 56,

Então a melhor aproximação com uma casa decimal será o 3, 3.



38. Observe que 11 com uma casa decimal foi aproximado para 3, 3. Agora para a casa do centésimo, basta
considerarmos os quadradados:

(3, 30)2 , (3, 31)2 , (3, 32)2 , . . . , (3, 39)2 , (3, 40)2 .

Repetindo o procedimento do exercı́cio anterior, a melhor aproximação será 3, 31.


√ √ √ √ √ √
39. Como 8 < 9 = 3√ e 64 = 8 < 80 < 81 = 9. O primeiro inteiro positivo maior que 8 é 3 e o
último inteiro menor que 80 é 8. Sendo assim, teremos 6 inteiros positivos, a saber {3, 4, 5, 6, 7, 8}.

44
POTI 2015 − Álgebra − Nı́vel 2 − Aula 0 − Professores Cleber Assis, Samuel Barbosa e Tiago Miranda

40. Temos: √ √
6 = 36 35 = 1225
√ √
< 37 < 1226
√ √
< 49 < 1296
= 7; = 36.

Assim,
√ podemos concluir que o primeiro inteiro positivo maior que 37 é 7 e o último inteiro positivo menor
que 1226 é o 35. Logo, teremos: 35 − 7 + 1 = 29 inteiros positivos compreendidos entre os números do
problema, a saber: {7, 8, 9, . . . , 34, 35}.

41. Os quadrados dos números são respectivamente: 99, 112, 125, 108 e 98. Destes, apenas o primeiro e o
último são menores que o quadrado de 10 que é 100. Assim, os três números do meio são maiores que 10.
Resposta C.

42. Dados dois racionais a e b, somando a aos dois lados da desigualdade, temos:

a < b
a+a < b+a
2a < a + b
a+b
a <
2

Repetindo o procedimento, agora com b, temos:

a < b
a+b < b+b
a + b < 2b
a+b
< b
2

a+b a+b
O que resulta em: a < < b Como também é um racional, isso mostra que existe um racional entre
2 2
a e b.

Comentário para professores: É bom enfatizar que se a construção acima for reiterada com os racionais a e
a+b a+b
(ou com e b) o aluno poderá mostrar que existe uma infinidade de racionais entre a e b. Outros
2 2
comentários comentários que poderiam instigar os alunos sobre a distribuição dos racionais e dos irracionais
na reta seria questioná-los se qualquer intervalo contém números racionais e irracionais.

43. (Extraı́do da UNICAMP)


Uma boa estratégia seria eliminar os radicais elevando ambos números a uma potência múltipla de 3 e 4. Veja
que:

( 3)12 = 34
3

= 81
> 64
= 43
√4
= ( 4)12
√ √ √
Portanto, como ( 3 3)12 > ( 4 4)12 , segue que 3 3 é o maior.

45
POTI 2015 − Álgebra − Nı́vel 2 − Aula 0 − Professores Cleber Assis, Samuel Barbosa e Tiago Miranda

44. (Extraı́do do Colégio Naval)


Façamos uma primeira estimativa: Finalmente, somando as duas últimas desiguldades
1<4<8 obtidas, temos:
13 <√4 < 2√
3 √ √

3 3
3< 3√ 4+ 3 √16 < 5
1 < √4 < 3 8 < + 3
+ 3

1< 34<2
√ 4 p1 √4
3
√16 < 6√
4 < 1 + 4 + 3 16 < 6
Segunda estimativa:
Portanto, n = 4.
8 < 16 < 27
2 3 < 16 < 33
√ √
3

3
8< √ 16 < 3 27
2 < 3 16 < 3

45. Vamos supor que é possı́vel termos uma fração irredutı́vel m
n,m ∈ Z, n ∈ Z∗ tal que 2= m
n. Neste caso,
podemos escrever:
√ m
2 =
n
√ 2  m 2
( 2) =
n
m2
2 =
n2
2n = m2
2

Agora temos a seguinte situação, o membro da esquerda é par, portanto o da direita também o será. Contudo,
não podemos ter m2 par, se m também não for par. Sendo assim, m = 2k, para algum k ∈ Z, e

m = 2k
m2 = 4k2

Agora, voltando à equação 2n2 = m2 e substituindo o m2 pelo 4k2 , e ficamos com:

2n2 = m2
2n2 = 4k2
n2 = 2m2 .

Pelo argumento anterior, n é par, isso contradiz nossa suposição inicial pois tı́nhamos √ assumido que a fração
m
n era irredutı́vel. Essa contradição mostra que a suposição inicial é falsa, ou seja, 2 não é racional.

Comentário para professores: Este é um exemplo clássico de prova por absurdo. Quando mencionado em
sala de aula, sugerimos que o professor comente exemplos cotidianos de afirmações que conduzem a absurdos
para que os alunos se sintam mais confortáveis com tal demonstração.

46. Utilize o mesmo argumento da questão anterior.


√ √ √2
47.Tome A = B = 2. Se o n úmero
√ √2 é racional, o enunciado está satisfeito. Caso contrário, faça
√ √2 √ √ 2
A = 2 e B = 2. Assim, ab = ( 2 ) 2 = 2 servirá como exemplo.

46
POTI 2015 − Álgebra − Nı́vel 2 − Aula 0 − Professores Cleber Assis, Samuel Barbosa e Tiago Miranda
√ √2
Comentário para professores: Já existe uma demonstração de que 2 é de fato irracional.
√ Um exemplo
mais construtivo usando fatos que não são estudados no oitavo ano seria escolher A = 10 e B = log10 4. Daı́,
A B = 2 é um racional.
48. F5 = {0/1, 1/5, 1/4, 1/3, 2/5, 1/2, 3/5, 2/3, 3/4, 4/5, 1/1}.
49. Usando a propriedade dada no enunciado, temos 7a − 5b = ±1. Veja que 7a deve deixar resto 1 ou 6 na
divisão por 5. Dentre os valores possı́veis de a no conjunto {0, 1, 2, . . . , 7}, apenas 2 e 3 satisfazem tal condição.
2 5 3 2
Se a = 2, temos b = 3. Se a = 3, teremos b = 4. Entretanto, como < < , a fração procurada é .
3 7 4 3
50.
a) Basta usar três vezes o balde de 5 litros e, em seguida, retirar duas vezes lı́quido do tambor usando o balde
de 7 litros. Dessa forma, transportamos 3 × 5 − 2 × 7 = 1 litro.
√ √
b) A quantidade a que podemos transportar de um tambor para o outro é da forma k (2 − 2) + l ( 2) litros
onde k e l são inteiros indicando quantas vezes tiramos ou colocamos lı́quidos usando cada um dos baldes.
Se l − k 6= 0, podemos escrever:
√ √
a = k (2 − 2) + l 2

a − 2k = 2( l − k )
a − 2k √
= 2
l−k

Assim, o número 2 seria o quociente√de dois inteiros o que resultaria em um número racional. Sabemos
que isso não pode acontecer porque 2 é irracional. Falta analisarmos o que acontece quando l = k. A
equação se transforma em:
√ √
a = k (2 − 2) + l 2
√ √
= k (2 − 2) + k 2
= 2k.

Veja que 2k é par e assim não podemos levar um valor ı́mpar como a = 1. Em qualquer caso, não é possı́vel
colocar exatamente 1 litro usando os baldes com as capacidades dadas neste item.

51. (Extraı́do da prova da Cone Sul publicada na Revista Eureka número 5)


A fração ba é irredutı́vel se e só se b−a a é irredutı́vel ( se a e b tem um fator comum, então a e b − a têm um
fator comum, e reciprocamente). O problema se transforma em achar o menor valor de n tal que as frações
a
sejam todas irredutı́veis. Observe que as frações anteirores possuem a forma e pelo critério anterior
n+a+2
a
bastaria que fosse irredutı́vel. Tendo isso em mente, se n + 2 é um primo maior que 91, todas as frações
n+2
serão irredutı́veis. Assim, um valor possı́vel de n é 95 pois n + 2 = 97 é um número primo. Verifiquemos que
é o menor possı́vel.
20
(a) Se n + 2 < 97 e n + 2 é par, então n é par e há frações redutı́veis como, por exemplo, n +2 .

(b) Se 19 ≤ n + 2 ≤ 91, obviamente há uma fração redutı́vel.

(c) Se n + 2 < 19, então n + 2 tem um múltiplo entre 19 e 91 e, portanto, há uma fração redutı́vel.
31
(d) Se n + 2 = 93 = 3.31, então n +2 é redutı́vel.
19
(e) Se n + 2 = 95 = 5.19, então n +2 é redutı́vel.
Logo, o valor mı́nimo de n + 2 é 97, que corresponde a n = 95.

47
POTI 2015 − Álgebra − Nı́vel 2 − Aula 0 − Professores Cleber Assis, Samuel Barbosa e Tiago Miranda

52.
√ √
a) x 3 x. c) 3x2 y. e) (4a6 b4 )3/2 = 26 a18 b12 = 8a9 b6 .
√ √ √
b) 4. d) 2 2 + 9 2 = 11 2.
53.
√ √
9

4
2 3 a7 a3
a) . c) . e) .
3 a a
√ √ √
3

3
1 1 4
x 4
x 2 2 x2 2 x2
b) √4
= √
4
· √ = . d) √ =√ ·√ = .
x 3 x 3 4
x x 3
x 3
x 3 x2 x
54.

y7 3s3 a10 b5
a) . b) . c) . d) .
x8 t6 b8 a8
55.

1 2
 
1 3 1
3 − + +
2 4 4 4
=

1 2

3 3 3
3 − − −
3 2 9 2
1
=
−7/6
6
= − .
7
56.
2
3x2 y

9x4 y2
a3 b3 a6 b6
3 =
3xy2 27x3 y6


2a2 b2 8a6 b6
8x
= .
3y4

57.

a) 22 x4 . b) xy2 z1/2 c) x1/8


58. Veja que:

1 1 1 1
− k
+ = +
1+a 1 + ak 1 + 1/a k 1 + ak
ak 1
= +
1 + ak 1 + ak
= 1.

Assim, se agruparmos a primeira fração com a última, a segunda com a penúltima e assim sucessivamente;
sempre obteremos o número 1. A única fração que não fará parte de nenhum par é a do meio que vale
1 1 1 2001
0
= . Como a quantidade de pares é igual a n, a resposta é 1000 + = . Resposta D.
1+a 2 2 2

48
POTI 2015 − Álgebra − Nı́vel 2 − Aula 0 − Professores Cleber Assis, Samuel Barbosa e Tiago Miranda

59. Indiquemos com uma seta o último dı́gito de um número. Assim,

131 → 3 132 → 9 133 → 7 134 → 1

135 → 3 136 → 9 137 → 7 138 → 1

...

Como 134 termina em 1, sempre que multiplicarmos os números de uma linha por esse valor para obtermos
os números da próxima, o último dı́gito se manterá. Podemos então agrupar os número de 4 em 4 e obtermos
uma soma que termina em 3 + 9 + 7 + 1 → 0. Como 2007 = 501 · 4 + 3, teremos 501 grupos e sobrarão
números com os dı́gitos 3, 9 e 7 cuja soma terminará em 9. Resposta E.

60.
22007 + 22005 22005 (22 + 1)
 
· 2006 = · 2006
22006 + 22004 22004 (22 + 1)
= 2 · 2006
= 4012.

A soma dos dı́gitos de 4012 é 7. Resposta D.

61. O número 24 = 23 · 3 tem somente dois divisores cubos perfeitos: 1 e 8. Assim, se é possı́vel representar
24 na forma a2 b3 , então b = 1 ou b = 2 e, portanto, a2 = 24 ou a2 = 3, o que é impossı́vel. Além disso, na
alternativa a podemos tomar a = 3 e b = 2; na alternativa c, podemos tomar a = 24 e b = c = 1; na alternativa
d, podemos tomar a = 3, b = 1 e c = 2; e na alternativa e, podemos tomar a = 2, b = 3 e c = 1. Resposta B.

62. Seja

21000 · 1 · 2 · 3 · . . . · 1000
B =
21000 · 1 · 2 · 3 · . . . · 1000
21000 · 1 · 2 · 3 · . . . · 1000
=
2 · 4 · 6 · . . . · 2000
Assim
21000 · 2000!
A·B =
2000!
= 21000

Como, B = 1, concluı́mos que A = 21000 . Resposta C.

Observação: Estamos escrevendo 2000! no lugar de 1 · 2 · 3 · . . . · 2000.

63.

a) 2n b) n/5 c) n + 1 d) (n + 3)/2 e) n3
64.

a) 2x · x = 2x2 . b) 2x · ( x + 1) = 2x2 + 2x. c) ( x − 1)( x + 2) = x2 + x − 2.

49
POTI 2015 − Álgebra − Nı́vel 2 − Aula 0 − Professores Cleber Assis, Samuel Barbosa e Tiago Miranda

65.

a) 8. b) 3. c) 11. d) m + n + 2.
66.

a) 32x4 y3 . 8x3 y2 8 x3 
y2
b) = = 2x2 y2
4xy x y
4
67.

a) 4x2 + 4x − 7. b) 2x2 + 4x − 9. c) 3x4 + 4x3 − 5x2 + 4x − 8.


68.

a) a3 − 5a2 − 2a + 4. b) 9a2 b + 6a2 − 5ab − 3ab2 + b2 . a2


c) 0, 3a4 + 0, 4a3 + .
3
69.

a) 3 + 2, 5x. b) Para x = 9, teremos:

3 + 2, 5x = 3 + 22, 5
= R$25, 50.

70.

a) x2 + 3x + 2. d) x2 − 16. 3
g) x2 + 2x + .
4
b) x2 + 12x + 27. e) x2 + 10x + 25.
c) x2 + x − 6. f) x2 − 8x + 16.
71.

a) 1 b) −11 c) 5 d) −149 e) 5/8

Comentário para professores: É interessante instigar os alunos a calcularem em alguns exemplos numéricos
simples o resto da divisão de um polinômio em x por x + a e compararem com o valor obtido de tal polinômio
quando avaliado em − a. Isto poderá ajudá-los a intuir e a compreender melhor a demonstração do Teorema
dos Restos que segue:

Se P( X ) possui quociente Q( X ) e resto R( x ) ao ser dividido por x + a, então:

P( x ) = Q( x )( x + a) + R( x )
P(− a) = Q(− a)(− a + a) + R(− a)
P(− a) = R(− a)

Como R( x ) é um polinômio constante pois x + a possui grau 1, podemos concluir que R( x ) = P(− a).

72. As expressões das letras (b), (c) e (d) são divisı́veis pelos polinômios dados enquanto que as expressões
em (a) e (e) não o são.

50
POTI 2015 − Álgebra − Nı́vel 2 − Aula 0 − Professores Cleber Assis, Samuel Barbosa e Tiago Miranda

73.

a) 5x − 3y. b) 5 · 8 − 3 · 2 = 34.
74.

a) 50 − x, para 0 ≤ x ≤ 50. b) 150 − 3x, para 0 ≤ x ≤ 50.


75.

a) a4 . d)

(2a3 b2 )(3ab4 )3 = (2a3 b2 )(27a3 b12 )


b) 12y7 .
= 54a6 b14 .
c)
e)

(2x3 )2 (3x4 ) (4x6 )(3x4 )  3  2 4


x y x
 3  8 4
x y x
= = ·
( x 3 )4 x12 y z y3 z4
12x10 x 7 y5
= =
x12 z4
= 12x −2 .
76.

a) y7 /x8 b) 3s3 /t6 c) a10 /b8 d) b5 /a8


77.
2
3x2 y

9x4 y2
a3 b3 a6 b6
3 =
3xy2 27x3 y6


2a2 b2 8a6 b6
8x
= .
3y4
78.

a) 9. b) 2n + 3. c) 1 + 2 + 3 + 4 + . . . + 26 = 351.
79. Devemos ter 9 = (n + 1) + (2n − 1) = 3n. Portanto, n = 3.
80. Pela propriedade de distributividade:
(kx − 1)(2x + 1) = 2kx2 + kx − 2x − 1
= 2kx2 + x (k − 2) − 1.
A soma dos coeficientes é 2k + (k − 2) − 1 = 3k − 3. Tal soma vale 3 apenas quando k = 2.
81.

a) x2 − a2n . b) x2 + 2xa2n + a4n . c) x2 − 4xa + 4a2 .


82. Pelo Teorema dos Restos, todas as divisões anteriores são exatas. Além disso, como:
( x2 − a2 ) = ( x + a)( x − a)
( x2 + 2xa + a2 ) = ( x + a)( x + a)
( x3 + a3 ) = ( x + a)( x2 − ax + a2 ).
Os quocientes são: ( x + a), ( x + a) e x2 − ax + a2 , respectivamente.

51
POTI 2015 − Álgebra − Nı́vel 2 − Aula 0 − Professores Cleber Assis, Samuel Barbosa e Tiago Miranda

83.

a) x = −2 b) x = 4 c) x = 0.
84. Pelo Teorema dos Restos, basta que P(1), P(−1) e P(2) sejam nulos.

a) Calculando P(1) teremos b) Calculando P(−1) teremos c) Calculando P(2) teremos

P (2) = 8 − 8 + 6 + k
P (1) = 1 − 2 + 3 + k P(−1) = −1 − 2 − 3 + k
= 6+k
= 2+k = −6 + k = 0;
= 0; = 0;

Os valores de k devem ser −2, 6 e −6, respectivamente.

85.

a) (2500 − 4x2 )cm.

b) (2500 − 4 · 25) = 2400cm2 .

c) Será formado uma caixa sem tampa cujo fundo é um quadrado de 50 − 2xcm de lado e cujas alturas
medem xcm. Portanto, o volume de tal caixa é x (50 − 2x )2 .

86. (Adaptado do exame do EPCAR − 2012)


Temos:

( x2n+1 + x )( x2n+1 − x ) − ( x4 )(n+1/2)


=
( x n + x )2 − x2n − 2x n+1
( x4n+2 − x2n+2 + x2n+2 − x2 ) − x4n+2
=
x2n + 2x n+1 + x2 − x2n − 2x n+1
− x2
=
x2
−1
87. Pelo algoritmo da divisão, temos:

x3 + 6x2 − 2x + 4 x2 − 1

− x3 + x

6x2 − x + 4 x+6

−6x2 + 6

− x + 10

Assim, o quociente vale x + 6 e o resto − x + 10.

52
POTI 2015 − Álgebra − Nı́vel 2 − Aula 0 − Professores Cleber Assis, Samuel Barbosa e Tiago Miranda

88. (Adaptado do exame do EPCAR − 2014)

a) Seja x a mensalidade em 2012. Após o aumento de b) Basta substituirmos o valor de x e multiplicarmos


80%, o valor da mensalidade passou para: pela quantidade de meses do ano obtendo:
100%x + 80%x = 1, 8x.
1, 134 · 800 · 12 = R$10.886, 40.
A redução de 30% transformou a mensalidade em
1, 8x · 0, 7. Finalmente, com o desconto de 10%, esse
valor passou para:
x · 1, 8 · 0, 7 · 0, 9 = 1, 134x
89. (Adaptado do exame de acesso do Colégio Naval − 2011)

p √
3
a) 3
−(2 − 1)6 = −1 = −1. b) Como todo quadrado de um número real é não ne-
gativo,
p temos ( x − 1)6 ≥ 0. Assim, −( x − 1)6 ≤ 0
e −( x − 1)6 ≤ 0.
3

p
Como 3 −(1 − 1)6 = 0, em virtude da última desi-
gualdade, podemos concluir que o valor máximo
da expressão é 0.
90.

a) b)
    

1

1

1
 
1
 1 1 1
1+ 1+ 1+ ... 1+ = 1+ 1+ ... 1+ =
2 3 4 99 x+1 x+2 x + 98
3 4 5 100 x+2 x+3 x+4 x + 99
· · ·...· = · · ·...· =
2 3 4 99 x+1 x+2 x+3 x + 98
3 4 5 100 x+
2  x+
3  x+
 4 x + 99
· · ·...· = ·  ·  · . . . ·  =
2 3 4 99
 x+1  x+2  x+3 x+
 98
100 x + 99
= 50. .
2 x+1

91.

a) Seja b) Como no item anterior, considere um número auxi-


liar:
21000 · 1 · 2 · 3 · . . . · 1000
C = 2x · 1 · 2 · 3 · . . . · x
21000 · 1 · 2 · 3 · . . . · 1000 D =
2x · 1 · 2 · 3 · . . . · x
21000 · 1 · 2 · 3 · . . . · 1000 2x · 1 · 2 · 3 · . . . · x
= =
2 · 4 · 6 · . . . · 2000 2 · 4 · 6 · . . . · 2x
Assim, Assim,
21000 · 2000! 2x · (2x )!
A·C = B·D =
2000! (2x )!
= 21000 . x
= 2 .

Como, C = 1, concluı́mos que A = 21000 . Como, D = 1, concluı́mos que D = 2x .

Observação: Estamos escrevendo n! no lugar de 1 · 2 · 3 · . . . · n.

53
POTI 2015 − Álgebra − Nı́vel 2 − Aula 0 − Professores Cleber Assis, Samuel Barbosa e Tiago Miranda

92.

a) x2 + 2x + 1. b) 16 + 8x + x2 . c) x2 + 2 3x + 3. d) 9x2 + 6x + 1. e) 16x2 + 16x + 4.
93.

a) 4x2 + 12x + 9.

b) 4x2 + 12xy + 9y2 .

c) x4 + 6x2 + 9.

d) a4 + 6a2 b2 + 9b4 .

e) x8 + 18x4 + 81.
94.

a) Cálculo do valor de 132 . b) Cálculo do valor de 412 . c) Cálculo do valor de 192 .

192 = (20 − 1)2


132 = (10 + 3)2 412 = (40 + 1)2
= 400 − 40 + 1
= 100 + 60 + 9 = 1600 + 80 + 1
= 361.
= 169; = 1681;
95.

a) c)
√ √ 2

( a + b) − 2 ab =
√ √ ( a + 1)2 + 2( a + 1) a + a2 + 2(2a + 1) + 1 =
a + 2 ab + b − 2 ab = (( a + 1) + a)2 + 2(2a + 1) + 12 =
a + b. (2a + 2)2 .
b)
( x + 1)2 + ( x − 1)2 =
( x2 + 2x + 1) + ( x2 − 2x + 1) =
2x2 + 2.
96.

a) a2 + 2ab + b2 . b) 4a2 − 4ab + b2 . c) 4a2 b2 + 12abc + 9c2 . d) 4a2 − 8ab + 4b2 .


97.

a) x2 − 1. d)
√ √ √ √
( x + y)( x − y)( x + y) =
b) 16 − a2 . ( x − y)( x + y) =
( x 2 − y2 )
c) x4 − 9z2 .
98.

a) 1002 − 12 = 9999. b) 20002 − 4 = 3999996. c) 102 − 52 + 52 = 100.

54
POTI 2015 − Álgebra − Nı́vel 2 − Aula 0 − Professores Cleber Assis, Samuel Barbosa e Tiago Miranda

99. Cada termo obtido após usarmos a distributividade teve um de seus membros vindo de alguma letra entre
os primeiros parênteses e o segundo vindo de alguma entre os segundos parênteses. Assim, como temos duas
possibilidade de escolhas em cada um deles, teremos no total 2 × 2 termos possı́veis na múltiplicação. Isso
pode também pode ser facilmente visualizado se momentaneamente colocarmos um ı́ndice para distinguirmos
de qual parêntese veio cada letra. Por exemplo:

( a1 + b1 )( a2 + b2 ) =
a1 a2 + a1 b2 + b1 a2 + b1 b2

100. Como temos três parênteses e em cada um deles temos duas escolhas, o número de termos é 2 × 2 × 2 = 8.
Para formarmos o termo a2 b, dois parênteses irão fornecer a letra “a“ e o outro a letra b. Uma vez escolhido
aquele que irá fornecer a letra ”b”, os demais estão determinados. Podemos fazer tal escoha de 3 formas e
assim existirão três termos a2 b. O mesmo argumento se aplica ao termo ab2 . A única maneira de formarmos
os termos a3 e b3 é escolhendo a mesma letra em todos os parênteses e isso só pode ser feito de uma forma.
Assim,
( a + b)3 = a3 + 3a2 b + 3ab2 + b3 .

101. O retângulo 6 × 7 desenhando abaixo foi dividido em duas figuras na forma de escada. Em cada
coluna, estamos escrevendo quantos quadrados foram pintados. Como as duas figuras são iguais, a soma dos
quadrados pintados - que corresponde ao termo 1 + 2 + 3 + 4 + 5 + 6 da equação -, deve ser igual à metade
da área do retângulo, ou seja,

Construindo um rentângulo n × (n + 1), é possı́vel


6·7 mostrar que:
1+2+3+4+5+6 = .
2
n ( n + 1)
1+2+3+...+n = .
2

102. Um exemplo seria:

Comentário para professores: O exemplo anterior é de Shirley Wakin e foi retirado do livro “Proofs without
words” escrito por Roger Nelsen. O leitor interessado poderá encontrar mais exemplos interessantes em tal fonte.

55
POTI 2015 − Álgebra − Nı́vel 2 − Aula 0 − Professores Cleber Assis, Samuel Barbosa e Tiago Miranda

103. Um exemplo seria:

Veja que área do quadrado maior de lado 9 é a soma das áreas das regiões destacadas e cada uma delas é da
forma 2n + 1 onde n é o lado do quadrado que a região contorna. É possı́vel construirmos quadrados cada
vez maiores e mostrarmos que a soma dos k primeiros inteiros positivos ı́mpares é igual à k2 .
104. Como todo quandrado perfeito é um número não negativo, se a e b representam as notas de um aluno,
temos:
√ √
( a − b )2 ≥ 0

a − 2 ab + b ≥ 0

a + b ≥ 2 ab
a+b √
≥ ab
2
Assim, é preferı́vel escolher a média aritmética porque ela é sempre maior ou igual à média geométrica.

Comentário: Provamos que se a e b são não negativos, então:


a+b √
≥ ab.
2
Isso é um caso particular do resultado mais geral de que a média aritmética de n números reais não negativos
é sempre maior ou igual à média geométrica de tais números.
105. Vamos usar novamente o fato de que todo quadrado é um número não negativo.

a) Sendo assim, teremos que c) Usaremos o item anterior três vezes:

( a − b )2 ≥ 0 
1 1

4 16
a2 − 2ab + b2 ≥ 0 + + + ≥
a b c d
a2 + 2ab + b2 ≥ 4ab
 
4 4 16
+ + ≥
( a + b )2 ≥ 4ab. a+b c d
 
16 16
b) Dividindo a expressão do item anterior por ab( a + + ≥
a+b+c d
b) obtemos: 64
.
1 1 a+b 4 a+b+c+d
+ = ≥ .
a b ab a+b
106. Sejam a e b as dimensões do retângulo, devemos ter que 2a + 2b = 2, ou seja, a + b = 1. A área obtida
será ab. Pelo exercı́cio anterior,
√ a+b 2 1
ab = ( ab)2 ≤ ( ) = .
2 4
1 1
Assim, a área máxima é . Podemos obtê-la construindo um quadrado de lado .
4 2

56
POTI 2015 − Álgebra − Nı́vel 2 − Aula 0 − Professores Cleber Assis, Samuel Barbosa e Tiago Miranda

107. Pela diferença de quadrados, temos:


s r s r
q √ q √
B = 2+ 2+ 2+ 3× 2− 2+ 2+ 3
r q √
= 2− 2+ 3

Apliquemos novamente a diferença de quadrados para obter o número:


r q √
C = 2+ 2+ 3·B
r r
q √ q √
= 2+ 2+ 3· 2− 2+ 3
q √
= 2− 3

Para terminar, veja que:


q √
A·B = 2+ 3·C
q √ q √
= 2+ 3· 2− 3
= 1
Resposta C
108. Se denotarmos por a = 20142012 o valor da expressão anterior pode ser escrito como:

( a + 1)2 − 2( a + 1) a + a2 = [( a + 1) − a]2
= 12 .
109. (Extraı́do da OBM 2014)
Usando a diferença de quadrados, podemos escrever:
√ √ √ √
( x + y)( x − y) = ( x − y).
Assim, obtemos:
√ √ x−y a2
x− y = √ √ =
x+ y b
√ √
x+ y = b
√ b2 + a2 √ b2 − a2
Resolvendo o sistema anterior, encontramos x= e y= . Assim,
2b 2b
√ b4 − a4
xy = .
4b2
110. Note que:

a(10 − a) = 10a − a2
= 25 − 25 + 10a − a2
= 25 − (5 − a)2

Como (5 − a)2 é sempre um número não negativo, a última expressão é no máximo 25. Tal valor é atingido
apenas quando (5 − a)2 = 0, ou seja, quando a = 5.

57
POTI 2015 − Álgebra − Nı́vel 2 − Aula 0 − Professores Cleber Assis, Samuel Barbosa e Tiago Miranda

111.

4a − a4 = 4a − 2a2 + 2a2 − a4
= 4a + 1 − 2a2 − 1 + 2a2 − a4
= 4a + 1 − 2a2 − ( a2 − 1)2
= 3 − 2 + 4a − 2a2 − ( a2 − 1)2
= 3 − 2( a − 1)2 − ( a2 − 1)2 .

Como 2( a − 1)2 + ( a2 − 1)2 é sempre um número não negativo por se tratar da soma de três quadrados, a
expressão anterior é no máximo 3. Veja que tal valor pode ser atingido quando a = 1.

112.

a) a(5 + b). b) a(m + n). c) x ( a + b + c). d) a( x + 1). e) b( a + c + ac).


113.
1 3 a a+b
a) . b) . c) . d) . e) x2 .
2 8 2 a−b
114.

a) ( a + b)( a + c). b) ( a − b)( x + y). c) (2a + 1)(b + 1). d) ( a − b)( x + 2). e) (5a − 1)(2b + 3).
115.

a) m2 . x2 + 1 m3 + 2
b) . c) .
x2 m+3
116.
√ √
a) ( a − 5b)( a + 5b). b) (2x − 1)(2x + 1). c) ( 7 − x )( 7 + d) ( ax − by)( ax + e) ( a − b)( a +
x ). by). b)( a2 + b2 ).
117.

a) A expressão não representa um bin√ômio perfeito. c) A expressão não representa um binômio


√ perfeito.
Se fosse b2 = 3, deverı́amos ter b = 3. Entretanto, Se fosse b2 = 18, deverı́amos ter b = 3 2. Entre-
−4 6= −2bx. tanto, 6 6= 2by.

d) 4z2 − 12zy + 9y2 = (2z − 3y)2 .


1 √ √
b) x2 + x + = ( x + 1/2)2 . e) 3z2 + 6z + 3 = ( 3z + 3)2 .
4
118.

a) ( x − 1)2 ( x + 1)2 . b) 5( a − 1)2 . c)

a2 − b2 − 2bc − c2 =
a2 − ( b + c )2 =
( a − (b + c))( a + (b + c)) =
( a − b − c)( a + b + c).

119.
xy xy
a) 20. b) . c) .
3 ( x + 1)(y + 1)

58
POTI 2015 − Álgebra − Nı́vel 2 − Aula 0 − Professores Cleber Assis, Samuel Barbosa e Tiago Miranda

120.

x2 y + y2 x = xy( x + y)
= 6·7
= 42.
121. (Extraı́do do vestibular da UNIVASF)
Temos
a+x c
=
b−x d

cb − xc = ad + xd.

Isolando os termos com x de um só lado e fatorando-o, obtemos: cb − ad = xc + xd = x (c + d), ou seja,


bc − ad
x= .
c+d
122.

a) b( a − b)( a + b). b) ( x − y − 3)( x − y + 3). c) ( a − 4)2 ( a + 4)2


123. Pela distributividade, temos:

( x − y)( x2 + xy + y2 ) =
( x3 + 
x2
y + 2
xy ) − (
yx2
+ 2
xy + y3 ) =
x 3 − y3

Usando a fatoração fornecida, temos:

x3 − 8 = ( x − 2)( x2 + 2x + 4).

124. Se y = −z, temos:

x 3 + z3 =
x3 + (−y)3 =
x 3 − y3 =
( x − y)( x2 + xy + y2 ) =
( x + z)( x2 − xz + z2 )

Obtemos assim uma fatoração para a soma dos cubos dada por:

x3 + z3 = ( x + z)( x2 − xz + z2 ).

125. Se x e y são esses números, temos:

x 3 + y3 =
( x + y)( x2 − xy + y2 ) =
( x + y)(( x + y)2 − 3xy) =
4 · ( 42 − 3 ) =
52

59
POTI 2015 − Álgebra − Nı́vel 2 − Aula 0 − Professores Cleber Assis, Samuel Barbosa e Tiago Miranda

126.

x 3 + y3 =
( x + y)( x2 − xy + y2 ) =
( x + y)(( x + y)2 − 3xy) =
3 · (32 − 3) = 18

127.
1
x2 + =
x2
 2
1 1
x+ −2·x· =
x x
22 − 2 = 2
128. (Extraı́do da Olimpı́ada Cearense)

( a − b)2 + (− a + b)2 + 2( a − b)(b − a) =


[( a − b) + (− a + b)]2 = 0.

129. (Extraı́do da AIME) Aplicando a diferença de quadrados nos dois primeiros parênteses e nos dois últimos,
temos:
√ √ √ √ √ √
( 5 + 6 + 7)( 5 + 6 − 7) =
√ √
(( 5 + 6)2 − 7) =

(4 + 2 30)
√ √ √ √ √ √
( 7 + 5 − 6)( 7 − 5 + 6) =
√ √
(7 − ( 5 − 6)2 ) =

(−4 + 2 30)

Assim, o produto é igual à:


√ √
(2 30 + 4)(2 30 − 4) =
4 · 30 − 16 = 104.
130.

x4 + 4 = x4 + 4x2 + 4 − 4x2
= ( x2 + 2)2 − 4x2
= ( x2 − 2x + 2)( x2 + 2x + 2).
131.

( n ( n + 3) + 1)2 = n2 (n + 3)2 + 2n(n + 3) + 1


= n(n + 3)[n(n + 3) + 2] + 1
= n(n + 3)[n2 + 3n + 2] + 1
= n(n + 3)[(n + 1)(n + 2)] + 1
= n(n + 1)(n + 2)(n + 3) + 1

132. Usando o exercı́cio anterior para n = 2014, obtemos (2014)(2017) + 1.

60
POTI 2015 − Álgebra − Nı́vel 2 − Aula 0 − Professores Cleber Assis, Samuel Barbosa e Tiago Miranda

133.

p4 − 1 = ( p2 − 1)( p2 + 1)
= ( p − 1)( p + 1)( p2 + 1)
134. (Extraı́da
p do vestibular da UFRJ)
√ p √
Seja y = 3 − 8 + 3 + 8. Claramente y é um inteiro positivo pois cada um dos radicais o é. Assim, o
produto xy possui o mesmo sinal de x. Calculemos tal produto usando diferença de quadrados:
√ √
xy = (3 − 8) − (3 + 8)

= −2 8.

Portanto, como − 8 é negativo, x também o é.
135.

n5 + n4 + 1 =
n5 + n4 + n3 − n3 − n2 − n + n2 + n + 1 =
n3 ( n2 + n + 1) − n ( n2 + n + 1) + ( n2 + n + 1) =
(n2 + n + 1)(n3 − n + 1).
136. (Extraı́do da Olimpı́ada Cearense)
Usando diferença de quadrados, temos:
√ √ n − ( n − 1)
n− n−1 = √ √
n+ n−1
1
= √ √
n+ n−1
Para que o número anterior seja menor que 0, 01, devemos ter:
√ √
n + n − 1 > 100.

Se n ≤ 502 ,
√ √ √
n+ n − 1 < 50 + 2499
< 100.
Se n = 502 + 1,
√ √ √
n+ n−1 = 2501 + 50
> 100.
Logo, o menor inteiro positivo que satisfaz a desigualdade do enunciado é n = 502 + 1.
137. Aplicando a diferença de quadrados sucessivamente, temos:

a32 − b32 =
( a16 + b16 )( a16 − b16 ) =
( a16 + b16 )( a8 + b8 )( a8 − b8 ) =
( a16 + b16 )( a8 + b8 )( a4 + b4 )( a4 − b4 ) =
( a16 + b16 )( a8 + b8 )( a4 + b4 )( a2 + b2 )( a2 − b2 ) =

Assim, o quociente é a2 − b2 .

61
POTI 2015 − Álgebra − Nı́vel 2 − Aula 0 − Professores Cleber Assis, Samuel Barbosa e Tiago Miranda

138. Note que (( x + 1)2 − ( x + 1) + 1) = ( x2 + x + 1) .

Verifiquemos agora uma fração genérica do produto: Primeiramente vejamos o que acontece quando multi-
plicarmos apenas as frações que constituem a primeira
x3 − 1 parte da expressão:
=
x3 + 1
2
( x − 1)( x + x + 1) 1 2 3 4 5 98
  99

· · · · ·...· · =

= 3 4 5 6 7 100 101
( x + 1)( x2 − x + 1)
2
x − 1 ( x + 1)2 − ( x + 1) + 1 .
· 100 · 101
x+1 x2 − x + 1
A segunda parte produz um cancelamento diferente:
A primeira parte da última expressão é uma fração
onde o numerador e o denominador diferem por 2 e a 32
−3

+1 42
−4+1

1012 − 101 + 1
segunda parte é um quociente de termos envonvendo

2
· 
· . . . · ( =
2 −2+1 
(
32 (2(

−3 + 1

100 −(100
(( +1
a expressão n2 − n + 1 quando n é x + 1 e x. Vamos (
10101
analisar a expressão anterior para cada valor de x no .
3
conjunto {2, 3, . . . , 100}.
Assim, o valor da expressão é:

2 10101 3367
· = .
100 · 101 3 5050
139. (Extraı́do da OBM 2014)
Observe que:
Fk2 Fk2+1 − Fk2
1− =
Fk2+1 Fk2+1
( Fk+1 − Fk )( Fk+1 + Fk )
=
Fk2+1
Fk−1 Fk+2
= .
Fk2+1
Assim,
2
F2 F32
    
F2013
1 − 22 1− 2 ·...· 1− 2 =
F3 F4 F2014
F1 F4 F2 F5 F3 F6 F F
2
· 2 · 2 . . . 20122 2015 =
F3 F4 F5 F2014
F1 F4 F2 F5 F3 F6 F2012
F
2015
2
· 2
· 2
. . . 
2 =
F
3 F
4 F
5
F 
2014
  
F1 · F2 · F2015
=
F3 · F2013 · F2014
F2015
.
2F2013 F2014
Resposta E.
140. Na identidade anterior, podemos trocar a soma de quaisquer dois, pelo simétrico do terceiro obtendo:
( x + y + z )3 =
x3 + y3 + z3 + 3( x + y)( x + z)(y + z) =
x3 + y3 + z3 + 3(−z)(−y)(− x ) =
x3 + y3 + z3 − 3xyz.
Como ( x + y + z)3 = 0, segue o resultado.

62
POTI 2015 − Álgebra − Nı́vel 2 − Aula 0 − Professores Cleber Assis, Samuel Barbosa e Tiago Miranda

141. (Extraı́do da Olimpı́ada do Cone Sul)


Comecemos analisando alguma relação entre a, b e a + b + ab. O último termo lembra a fatoração:

( a + 1)(b + 1) = ab + a + b + 1.

Em cada momento após realizarmos as operações, se analisarmos a quantidade que representa o produto de
todos os números do conjunto acrescidos de uma unidade. A equação anterior nos diz que tal produto nunca
se altera. Consequentemente, no final teremos um único número x tal que:

(1 + 1/2)(1 + 1/3) . . . (1 + 1/100) = (1 + x ).

Ou seja, x = 99/2. Para entender melhor que quantidade estamos analisando, façamos um exemplo pequeno.
Suponha que em um dado momento temos os números 2, 3 e 5, devemos analisar o número

(2 + 1)(3 + 1)(5 + 1).

Se trocarmos a = 2 e b = 3 por ab + a + b = 11 e fizermos o novo produto obteremos:

(11 + 1)(5 + 1).

Perceba que o valor continua sendo o mesmo.

142. Para fazer tal expansão, podemos considerar momentaneamente x + y = w e a expressão que já
conhecemos para o binômio:

( x + y + z )3 =
( w + z )3 =
w3 + 3w2 z + 3wz2 + z3

Além disso,

w3 =
( x + y )3 =
x3 + 3x2 y + 3xy2 + y3 =
x3 + y3 + 3xy( x + y) =
x3 + y3 + 3xyw

Voltando para a expressão original, temos:

( x + y + z )3 =
x3 + y3 + z3 + 3xyw + 3w2 z + 3wz2 .

Resta estudarmos o termo:

3xyw + 3w2 z + 3wz2 =


3w( xy + wz + z2 ) =
3( x + y)( xy + xz + yz + z2 ) =
3( x + y)( x + z)(y + z)

Com isso, podemos concluir que:

( x + y + z)3 = x3 + y3 + z3 + 3( x + y)( x + z)(y + z).

63
POTI 2015 − Álgebra − Nı́vel 2 − Aula 0 − Professores Cleber Assis, Samuel Barbosa e Tiago Miranda

143. Para fazer tal expansão, podemos considerar momentaneamente x + y = w e a expressão que já
conhecemos para o binômio:

( x + y + z )2 =
( w + z )2 =
w2 + 2wz + +z2

Além disso,

w2 =
( x + y )2 =
x2 + 2xy + y2 .

Voltando para a expressão original, temos:

( x + y + z )2 =
x2 + y2 + z2 + 2xy + 2wz =
x2 + y2 + z2 + 2xy + 2( x + y)z =
x2 + y2 + z2 + 2xy + 2x + 2yz

Com isso, podemos concluir que:

( x + y + z)2 = x2 + y2 + z2 + 2xy + 2xz + 2yz.


144. Sejam x = b − c, y = c − a e z = a − b. Pelo exercı́cio anterior, como x + y + z = 0, podemos escrever:

( b − c )3 + ( c − a )3 + ( a − b )3 =
x 3 + y3 + z3 =
3xyz =
3(b − c)(c − a)( a − b).

145. Elevando ao quadrado a igualdade dada, temos

a2 x2 + b2 y2 + c2 z2 + 2( abxy + bcyz + cazx ) = 0

E consequentemente:
−2( abxy + bcyz + cazx ) = a2 x2 + b2 y2 + c2 z2
Daı́, a expressão bc(y − z)2 + ca(z − x )2 + ab( x − y)2 é igual a

x2 ( ab + ac) + y2 (ba + bc) + z2 (ca + cb)


−2( abxy + bcyz + cazx )
= x2 ( a2 + ab + ac) + y2 (ba + b2 + bc) +
+z2 (ca + cb + c2 )
= ax2 ( a + b + c) + by2 ( a + b + c) +
+cz2 ( a + b + c)
= ( ax2 + by2 + cz2 )( a + b + c).

Assim,
ax2 + by2 + cz2 1
= ,
bc(y − z)2 + ca(z − x )2 + ab( x − y)2 a+b+c
que independe de x, y e z.

64
POTI 2015 − Álgebra − Nı́vel 2 − Aula 0 − Professores Cleber Assis, Samuel Barbosa e Tiago Miranda

146.

a) F. d) A. g) A.
b) A. e) F.
c) F. f) A.
147. a, b, d, g, i.
148.
a) P = 4l, sendo P o perı́metro e l a medida do lado.
b) A = l 2 , sendo A a área e l a medida do lado.
c) L + l = 16, sendo L a idade de Luiz e l a idade de Luı́sa.

x
d) < 3x, sendo x o referido número.
2
20
e) S = 700 + · V = 700 + 0, 2V, sendo S o salário e V o valor das vendas.
100
f) A = b · 2b = 2b2 , sendo A a área e b a medida da base.
149.

a) A = 6l 2 . b) V = l 3 . c) S = 12l.
150. (Extraı́do da Vı́deo Aula)
x x x x
Supondo que Diofanto tenha vivido x anos, temos x = + + + 5 + + 4. Para resolver esta equação,
6 12 7 2
primeiramente encontraremos um denominador comum a todas as frações, sendo o menor deles (e mais fácil
de se trabalhar) 84. Escrevendo agora as frações equivalentes, com denominador 84, a cada uma das frações
84x 14x 7x 12x 420 42x 336
da equação, temos = + + + + + . Fazendo as devidas simplificações, chegamos
84 84 84 84 84 84 84
a x = 84, que é a quantidade de anos vividos por Diofanto.
151. (Extraı́do da Vı́deo Aula)
a) P = 2a + b + a + b + b + 2a + b + a = 6a + 4b.
b) Basta somar ao perı́metro encontrado no item anterior, as medidas internas de divisão do terreno. Assim,
ficamos com Q = 6a + 4b + a + b + 2a = 9a + 5b.
c) Vamos calcular cada uma das áreas dos retângulos menores e somá-las. Temos então
A = 2a2 + 2ab + ab + b2 = 2a2 + 3ab + b2 .

Figura 2

65
POTI 2015 − Álgebra − Nı́vel 2 − Aula 0 − Professores Cleber Assis, Samuel Barbosa e Tiago Miranda

152.

a) Representaremos a área de um polı́gono ABCD por [ ABCD ]. Os dois polı́gonos formados são trapézios,
que possui uma fórmula para o cálculo direto de sua área, porém não a utilizaremos. Trace duas
paralelas ao lado AB, uma pelo ponto E e outra pelo ponto F. Pronto! Dividimos cada trapézio em um
30 · ( x − 5)
retângulo e um triângulo. Vamos ao cálculo de suas áreas. [ ABFE] = 30 · 5 + = 75 + 15x.
2
30 · ( x − 5)
[CDEF ] = 30 · (40 − x ) + = 1125 − 15x.
2
b) Como a [CDEF ] é o dobro de [ ABFE], temos:

[CDEF ] = 2[ ABFE]
1125 − 15x = 150 + 30x
45x = 975
65
x= .
3

Figura 4

153.

a) 2 · 1 + 1 = 2 + 1 = 3.

b) 4 − 3 · 1 = 4 − 3 = 1.

c) 32 − (−1)3 = 9 + 1 = 10.

d) 2 · 32 − 3 · 22 = 18 − 12 = 6.
5−5·3 5
e) =− .
4 2
f) 42 + 2 · 4 · 2 + 22 = 16 + 16 + 4 = 36.

154. O total de janelas é 12 · 4 · 6 = 288. Como cada vidro tem dimensões a e b, a área total de vidros é 288ab.

155.

a) a área do terreno é xy + 3y.

b) A = 20 · 15 + 3 · 15 = 300 + 45 = 345m2 .

66
POTI 2015 − Álgebra − Nı́vel 2 − Aula 0 − Professores Cleber Assis, Samuel Barbosa e Tiago Miranda

156. (Extraı́do da Vı́deo Aula)


Multiplicando por dois a nota da prova ficaremos com 2p, somando a nota do teste teremos 2p + t, por fim, a
dividindo por três chegaremos a nota final como

2p + t
n= .
3
157. (Extraı́do da Vı́deo Aula)
x
Seja a fração inicial e procedendo com as operações do enunciado teremos
y

x − 0, 4x 0, 6x 3x x x x
= = = 1, 5 · = + 0, 5 · ,
y − 0, 6y 0, 4y 2y y y y

ou seja, chegamos a fração inicial aumentada em 50%, o que está na letra d.

158.

a) V = x · 2x · y = 2yx2 .

b) A = 2 · xy + 2 · 2xy + 2x2 = 2x2 + 6xy.

c) Se x = 3m e y = 2m, então temos

V = 3 · 6 · 2 = 36m3 = 36.000`.

159.

a) P = 4x + 8y − 4.

b) basta somarmos ao perı́metro, os comprimentos das linhas internas. Temos então

S = (4x + 8y − 4) + (4x + 2y) = 8x + 10y − 4.

c) se x = 2m e y = 2, 5m, temos
A = 2x · (4y − 2) = 4 · 8 = 32m2 .

160. (Extraı́do da Vı́deo Aula)


Inicialmente vamos chamar as balanças, de cima para baixo, de b1 , b2 e b3 . Na balança b2 temos dois triângulos
e quatro cı́rculos equilibrando com oito quadrados. Se tomarmos metade das figuras de cada lado, como na
figura abaixo, a balança continuará em equilı́brio. Vamos chamar esta balança de b4 .

Figura 8

Perceba que, ao juntarmos as figuras do lado esquerdo da balança b1 com as figuras do lado esquerdo
da balança b4 , obtemos exatamente a quantidade de figuras do lado esquerdo da balança b3 , ou seja, para
encontrarmos a quantidade de quadrados do lado direito da balança b3 , basta somarmos as quantidades de
quadrados do lado direito da balança b1 e da balança b4 . Portanto, essa quantidade é 6 + 4 = 10.

67
POTI 2015 − Álgebra − Nı́vel 2 − Aula 0 − Professores Cleber Assis, Samuel Barbosa e Tiago Miranda

161.
a) Como o comprimento interno é 5 − 2x e a largura interna é 3 − 2x, o perı́metro é

10 − 4x + 6 − 4x = 16 − 8x.

b) (5 − 2x )(3 − 2x ) = 15 − 16x + 4x2 .

c) Basta multiplicar o perı́metro pela altura do quarto e subtrair a área das portas. Temos então que a área
interna das paredes é
3(16 − 8x ) − 2 · 3 = 48 − 24x − 6 = (42 − 24x )
metros quadrados.

162. (Extraı́do da Vı́deo Aula)


5 5 2 5
Se = x + 1, terı́amos x = − 1 = − , porém x Assim, podemos concluir que o irmão de é
7 7 7 7
deve ser positivo. Temos então
5 7
5 x x+1 = +1 = .
= 2 2
7 x+1
5x + 5 = 7x
5
x = .
2
163. (Extraı́do do ENEM)
A área perdida ( A p ) é igual à área inicial ( Ai ) menos a área final ( A f ). Temos então:

A p = Ai − A f
= 15 − (5 − x )(3 − y)
= 15 − 15 + 3x + 5y − xy
= 5y + 3x − xy.

O que está na letra e.

164. (Extraı́do da Vı́deo Aula)

a) Observe que temos

1002 − 992 = (100 + 99)(100 − 99) S = (1002 − 992 ) + (982 − 972 ) + ... + (22 − 12 )
= 199 = (100 + 99)(100 − 99) + ... + (2 + 1)(2 − 1)
2 2
98 − 97 = (98 + 97)(98 − 97) = 199 + 195 + 191 + ... + 3
= 195 (3 · 199) · 50
=
2 2 2
96 − 95 = (96 + 95)(96 − 95)
= 5050.
= 191
.. b)
.
42 − 32 = (4 + 3)(4 − 3) 999991 = 1000000 − 9
=7 = 10002 − 32
22 − 12 = (2 + 1)(2 − 1) = (1000 + 3)(1000 − 3)
= 3. = 1003 · 997.

Fazendo S = 1002 − 992 + 982 − 972 + ... + 22 − 12 , Portanto, esses números podem ser 1003 e 997.

68
POTI 2015 − Álgebra − Nı́vel 2 − Aula 0 − Professores Cleber Assis, Samuel Barbosa e Tiago Miranda

165. Chamando a quantidade inicial de ovos na cesta de x, temos:

i) Primeiro cliente: comprou a metade que havia mais freguês. Temos então:
x 1
meio ovo, ou seja, + , deixando para o feirante
2 2 x 1
x 1 −
a metade menos meio ovo, ou seja, + ; 2 2 −1
2 2 2 2 − 1 = 10
ii) Segundo cliente: comprou a metade que havia 2 2
x 1 x 1
− −
2 2 + 1 , deixando 2 2 −1
mais meio ovo, ou seja, 2 2 = 10 + 1
2 2
para o feirante a metade menos meio ovo, ou seja, 2 2
x 1 x 1
− −
2 2 − 1; 2 2 − 1 = 21
2 2 2 2
iii) Segundo cliente: comprou a metade que havia mais x 1

x 1 2 2 = 21 + 1
− 2 2
2 2 −1
2 1
2 + , deixando para x 1
meio ovo, ou seja, − = 43
2 2 2 2
o feirante a metade menos meio ovo, ou seja, x 1
x 1 = 43 +
− 2 2
2 2 −1 x 87
2 2 − 1; =
2 2
2 2 x = 87.
Este último resultado deve ser igual a 10, pois foi o
que restou para o feirante após a passagem do último Portanto, a quantidade inicial de ovos era 87.
166.

a) Pela soma das equações, obtém-se 2x = 4, por- g) Idem item a. S = {(1, 1/2)}.
tanto x = 2. Substituindo o valor de x em uma
das equações, obtém-se y = 1. Assim, o conjunto- h) Idem item a. S = {(3, 2)}.
solução é S = {(2, 1)}. i) Para que seja possı́vel a resolução pelo método da
adição é necessário que a primeira equação seja
b) Idem item a. S = {(9, 1)}.
multiplicada por −4, obtendo-se o sistema equiva-
c) Substituindo o valor de x da segunda equação na lente
primeira, obtém-se (y + 6) + y = 10, ou seja, y = 2. 
Substituindo este valor de y na segunda equação,
 −20x + 4y = −136


obtém-se x = 2 + 6 = 8. Assim, o conjunto-solução
é S = {(8, 2)}. 
3x − 4y = 0.


d) Comparando as duas equações, tem-se 2y = y + 18,
ou seja, y = 18. Como x = 2y, então x = 36. Por- Pela soma das duas equações, obtém-se −17x =
tanto, S = {(36, 18)}. −136 e daı́ segue que x = 8. Substituindo o valor
de x em qualquer uma das equações, chega-se a
e) Idem item a. S = {(3, 2)}. y = 6. Portanto, S = {(8, 6)}.

f) Idem item a. S = {(7/2, 1/2)}. j) Idem item a. S = {(5, 2)}.

69
POTI 2015 − Álgebra − Nı́vel 2 − Aula 0 − Professores Cleber Assis, Samuel Barbosa e Tiago Miranda

167. Sendo os dois números denotados por x e y, onde x > y, chega-se ao sistema de equações


 x + y = 70


 x − y = 28.

Pela soma das equações, obtém-se 2x = 98, ou seja, x = 49. Como a soma dos números é 70, então y = 21.

168. Chamando a quantidade de bolinhas de Pedro de p e a quantidade de bolinhas de Mariano de m,


obtém-se o sistema 

 p+m =
 195

p = m + 45.

Substituindo a segunda equação na primeira, obtém-se (m + 45) + m = 195, ou seja, m = 75. Assim, Mariano
possui 75 bolinhas de gude e, como a soma das quantidades é 195, Pedro possui 120 bolinhas de gude.

169. Suponha que a quantidade obtida por Guilherme seja g e a quantidade obtida por Santiago s. Obtém-se
o sistema 

 g = s + 350


 g = 2s.

Por comparação das duas equações, obtém-se 2s = s + 350 e daı́ segue que s = 350. Substituindo tal valor na
segunda equação, g = 2s = 700. Assim, Guilherme juntou R$700, 00 e Santiago, R$350, 00.

170. Denotando o preço do sapato de s e o da blusa de b, obtemos o sistema




 s+b =
 72

b = s + 10.

Substituindo o valor de b na primeira equação, temos s + s + 10 = 72, segue que s = 31 e, por consequência,
b = 41. Ou seja, a blusa custou R$41, 00 e o sapato, R$31, 00.

171. Sejam x a quantidade de plutônio e y a quantidade de patetônio. Analise o sistema de equações




 x + y = 375


 x − y = 75.

Somando as equações, obtém-se 2x = 450, segue que x = 225 e, por consequência, y = 150, ou seja, a
quantidade de plutônio é 225ml e de patetônio é 150ml.

172. Pela subtração das equações, encontra-se p − q = 0, ou seja, p = q. Utilizando este dado na primeira
equação, temos 13p − 92p = −79p = 273 e daı́ segue que p = q = −273/79.

70
POTI 2015 − Álgebra − Nı́vel 2 − Aula 0 − Professores Cleber Assis, Samuel Barbosa e Tiago Miranda

173. Inicialmente, lembremo-nos que gansos possuem duas patas e, hipopótamos, quatro. Deonotando a
quantidade de gansos por g e a quantidade de hipopótamos por h, obtém-se o sistema de equações



 g+h = 50

 2g + 4h = 140

Multiplicando a primeira equação por (−2), obtém-se o sistema equivalente ao anterior, dado por:

 −2g − 2h = −100


 2g + 4h

= 140.

Pela soma das equações, temos 2h = 40 e daı́ segue que h = 20. Como o total de animais é 50, g = 30, ou seja,
o número de gansos é 30 e o de hipopótamos é 20.

174. Sejam c a quantidade de carros e m a quantidade de motos. Pelas informações dadas, constrói-se o
sistema 

 c + m = 47


 4c + 2m = 164

que é equivalente ao sistema



 −2c − 2m = −94


 4c + 2m

= 164.

Somando as equações, obtém-se 2c = 70 e daı́ segue que c = 35. Como o total de veı́culos é 47, tem-se
m = 47 − 35 = 12, ou seja, a quantidade de carros é 35 e de motos, 12.

175. Temos o seguinte sistema:





 x+y = 17

 50x + 10y = 330

onde x representa a quantidade de cédulas de R$50, 00 e y, a de R$10, 00. Substituindo x = 17 − y na segunda


equação, obtém-se 50(17 − y) + 10y = 330, ou seja, y = 13. Como o total de cédulas é 17, tem-se x = 4.
Portanto, foram retiradas 13 cédulas de R$10, 00 e 4 cédulas de R$50, 00.
176. Pela definição de progressão aritmética, obtém-se o sistema

 x−2 = y−x


 29 − y = y − x

Pela primeira equação, obtém-se y = 2x − 2, donde, substituindo na segunda, chega-se a 2(2x − 2) − x = 29 e


daı́ segue que x = 11. Substituindo tal valor em qualquer uma das equações, obtém-se y = 20.

71
POTI 2015 − Álgebra − Nı́vel 2 − Aula 0 − Professores Cleber Assis, Samuel Barbosa e Tiago Miranda

177.
16
a) Multiplicando a primeira equação por (−3) e somando-a com a segunda, obtemos = 8. Segue que
y
y = 2 e, consequentemente, x = 10. Portanto, S = {(10, 2)}.

b) Subtraindo as equações, obtemos y = 5, donde y = 25 e, consequentemente, x = 9.

178. Vamos construir o sistema:




 √
 a+ b =
 17
 √ √
a = 3 b + 3.


√ √ √ √
Substituindo a na primeira equação, obtemos 3 b + 3 √ + b = 17, segue que b = 72 , ou seja, b = 49/4.
Substituindo este valor na segunda equação, chegamos a a = 3 · 7/2 + 3 = 27/2, ou seja, a = 729/4.

179.

a) S = {(2, 1)} b) S = {(1, 2)} c) S = {(3, 1)}


180. Multiplicando
√ a segunda equação por 9 e somando o resultado com a primeira,
√ obtemos 91 3 r = 273,
segue que 3 r = 3, donde r = 27. Substituindo na segunda equação, obtemos s = 2, donde s = 4. Portanto,
(r, s) = (27, 4).

181. (Extraı́do da OBMEP − 2005)


Chamando o peso de cada abacate de a, o das bananas de b e o das laranjas de l, obtém-se o sistema


 4a = 9b






 3b = 2l





 9l = k,

onde k é o peso em de abacates que equilibra a terceira pesagem. Partindo da terceira equação, tem-se

2k = 18l = 27b = 12a,

ou seja, 2k = 12a e daı́ segue que k = 6a. Portanto, deverão ser 6 abacates. Resposta E.

72
POTI 2015 − Álgebra − Nı́vel 2 − Aula 0 − Professores Cleber Assis, Samuel Barbosa e Tiago Miranda

182. (Extraı́do da OBMEP − 2011)


Suponhamos que o tempo de ida, ou volta, a pé seja x e o tempo de ida de ônibus seja y. Pelas informações,
obtém-se o sistema 

 x + y = 1h15min


2y = 30min

Pela segunda equação, y = 15. Substituindo tal valor na primeira, obtém-se x = 75 − 15 = 60min. Para ir e
voltar a pé, o tempo gasto é 2x = 120min = 2h. Resposta A.

183. (Extraı́do da OBMEP − 2011)


Dividamos cada vaso em duas partes: a parte de baixo de altura x e a parte de cima de altura y, ou seja, a
altura de cada vaso é x + y. No caso de n vasos empilhados, a altura da pilha é x + ny. Assim, obtém-se o
sistema 

 x + 8y = 36


 x + 16y = 60

Subtraindo as duas equações, obtém-se 8y = 24, segue daı́ que y = 3 e, consequentemente, x = 12. Portanto,
cada vaso tem 3 + 12 = 15cm de altura. Resposta A.

184. (Extraı́do do exame de acesso do PROFMAT)


Vamos denotar a quantidade de carrinhos de mão por c, a quantidade de bicicletas por b, a de triciclos por t e
a de automóveis de a. Assim, teremos o sistema


c + 2b + 3t = 26







 c + 2b + 4a = 26





 c + 3t + 4a = 26

A diferença entre as duas primeiras equações nos mostra que 4a = 3t. Fazendo o mesmo processo entre a
primeira e a última equação, obtemos 2b = 4a. Por fim, as duas últimas equações nos mostram que 2b = 3t, ou
seja, 4a = 3t = 2b = k. Como se tratam de valores inteiros positivos, k deve ser múltiplo de 3 e 4. Além disso,

2k = 2b + 3t < c + 2b + 3t = 26,

ou seja, k < 13. Portanto, como existe apenas um múltiplo de 12 positivo e menor que 13, temos k = 12.
Daı́, 4a = 3t = 2b = 12 e segue que a = 3, b = 6, t = 4 e c = 2. Finalmente, temos como total de veı́culos:
2 + 3 + 4 + 6 = 15.
185. (Extraı́do da OBMEP − 2011)
Chamemos de Joana a mãe de João e Ana. Denotemos por h a quantidade de filhos de Joana e de m a
quantidade de filhas. O enunciado nos diz que h = m + 6. ”Tirando”Ana do cálculo, a equação anterior se
transforma em h = (m − 1) + 7, ou seja, Ana tem 7 irmãos a mais que irmãs. Resposta E.

186. (Extraı́do da OBM − 2012)


A soma de todas as massas é 956/4 = 239, já que cada estudante está presente em quatro pares. Sejam suas
massas, da menor para a maior, a, b, c, d, e. Sabe-se que a + b = 90 e d + e = 101, ou seja, a + b + d + e =
90 + 101 = 191. Assim, a massa do estudante de massa intermediária é 239 − 191 = 48kg. Resposta D.

73
POTI 2015 − Álgebra − Nı́vel 2 − Aula 0 − Professores Cleber Assis, Samuel Barbosa e Tiago Miranda

187. (Extraı́do da OBM − 2014)


Supondo x a quantidade de sobrinhos e y a quantidade, em reais, que cada sobrinho deveria receber na
primeira situação. Analisando as duas situações iniciais, chega-se ao sistema

 xy = 250 − 10 = 240


 x(y − 1) = 250 − 22 = 228

Perceba que não se trata de um sistema de equações do primeiro grau, mas sua solução é simples. Pela
segunda equação, temos xy − x = 228. Pela primeira equação, xy = 240 e assim obtemos 240 − x = 228. Daı́
segue que x = 12. Como ela, depois de muitas indecisões, resolveu distribuir igualmente apenas R$240, 00,
cada sobrinho recebeu 240/12 = R$20, 00. Resposta E.
188. (Extraı́do da OBM − 2014)
Temos:
√ √ x−y
x− y = √ √
x+ y
a2
=
√ b

x + y = b.
√ b2 + a2 √ b2 − a2
Somando e subtraindo a primeira e a terceira equação, segue que x= e y= . Consequen-
2b 2b
√ b4 − a4
temente, xy = . Resposta A.
4b2
189. (Adaptado da OBMEP − 2012)
Denotando o peso do copo vazio por x e o peso da quantidade de farinha que cabe em um copo por y,
obtém-se o sistema 
 3y
 2y +
 = 1400
2
 2x + 2y = 3x + 3y


2
Pela primeira equação, 7y = 2800. Segue que y = 400, ou seja, em cada copo cabe 400g de farinha. Substituindo
3y
o valor de y na segunda equação, obtém-se x = 2y − = y/2 = 200. Resposta D.
2
190. (Extraı́do de EA CPCAR − EPCAR − 2011)
Chamando de y a quantidade de pastéis que Isabela levou para vender, temos x + y = 460. Tem-se, então,
que Isabela vendeu 3/5x e Ana Beatriz vendeu 5/8y pastéis. Como o número de pastéis que restou para Ana
3x 1 2 15
Beatriz era a metade do número de pastéis que restou para Isabela, tem-se = · y, ou seja, y = x.
8 2 5 8
15x
Substituindo este valor na equação x + y = 460, obtém-se + x = 460. Daı́ segue que x = 160 e, portanto, a
8
soma dos algarismos de x é 1 + 6 + 0 = 7. Resposta B.
191. Multiplicando a primeira equação por y e a segunda equação por x, obtemos o sistema equivalente


 xy + 1 = 4y


 xy + 1 =
 x
4
x x
Por comparação, = 4y, ou seja, = 16.
4 y

74
POTI 2015 − Álgebra − Nı́vel 2 − Aula 0 − Professores Cleber Assis, Samuel Barbosa e Tiago Miranda

192. (Extraı́do de EA CPCAR − EPCAR − 2012)


Denotemos a idade que Luiz tinha por x. Assim, José possui o dobro, ou seja, 2x. Se José possuı́a y anos, a
idade atual de Luiz será y anos. No futuro, quando Luiz tiver a idade que José possui hoje, a idade de José
será de 90 − 2x anos. A tabela abaixo reune essas conclusões:

Passado Presente Futuro

José y 2x 90 − 2x

Luis x y 2x

Como a diferença entre o presente e o passado é a mesma para ambos, bem como a diferença entre o futuro e
o presente, obtém-se o sistema:

 2x − y = 90 − 2x − 2x


 y−x

= 2x − y.

Reduzindo termos semelhantes em ambas equações, obtém-se o sistema equivalente



= 6x − 90


 y

 2y − 3x =

0.

Substituindo o valor de y da primeira equação, na segunda equação, chega-se a 2(6x − 90) − 3x = 0, donde se
conclui que x = 20. Substituindo em uma das equações anteriores, obtém-se y = 30. Assim, a razão entre as
idades de José e Luiz em 29 de julho de 2017 será

2x + 5 45 9
= = .
y+5 35 7

Resposta B.

193. (Extraı́do de EA CPCAR – EPCAR − 2012)


Denotando as quantias iniciais, em reais, de Tales e Pitágoras, respectivamente, por t e p, obtém-se o sistema

 p − 50

 = t + 50

 p + 100

= t − 100

4
que é equivalente ao sistema

 p−t

 = 100

 p − 4t = −500

Subtraindo as equações, chega-se a 3t = 600 e daı́ segue que t = 200. Consequentemente, p = 300.
Resposta A.

75
POTI 2015 − Álgebra − Nı́vel 2 − Aula 0 − Professores Cleber Assis, Samuel Barbosa e Tiago Miranda

194. (Extraı́do de Colégio Naval − 2012)


Utilizando os pesos em kg, obtém-se o sistema


 x+y =
 44

x = 10y

√ x √ primeira equação, obtém-se 10y + y = 44. Daı́, segue que y = 4.


Fazendo a substituição do valor de x na
10
Como x = 10y, então x = 40 e y = 440 = 44 = 256. Resposta C.
10

195. (Extraı́do de Colégio Naval − 2013)


A segunda equação pode ser reescrita como 16a2 + b2 = 5 apenas multiplicando-a por 4a − b. Pela primeira
equação, podemos concluir que ab é 1/2. Assim,

16a4 b2 + a2 b4 − 8a3 b3 = ( ab)2 (16a2 + b2 ) − 8( ab)3


1 8
= ·5−
4 8
1
= .
4
Resposta E.

196.

a) Somando todas as equações, obtém-se:

6( x1 + x2 + x3 + x4 + x5 ) = 186,

ou seja, x1 + x2 + x3 + x4 + x5 = 31. Nota-se que a primeira equação pode ser escrita como x1 + x1 + x2 +
x3 + x4 + x5 = x1 + 31 = 6 e daı́ segue que x1 = −25. De forma análoga, obtém-se os demais resultados
repetindo o procendimento com as outras equações. Assim, x2 = −19, x3 = −7, x4 = 17, x5 = 65.

b) (Olimpı́ada Russa − 1946)


0
Somando todas as equações, temos x1 + x2 + x3 + x4 + x5 + x6 + x7 + x8 = = 0. Somando agora primeira,
3
quarta e sétima equações, obtemos x1 + x2 + x3 + x4 + x5 + x6 + x7 + x8 + x1 = 0 + x1 = 6 − 3 − 2 = 1,
ou seja x1 = 1. De forma análoga podemos obter as demais incógnitas. x2 = 2, x3 = 3, x4 = 4, x5 = −4,
x6 = −3, x7 = −2, x8 = −1.
197.

a) Temos: a = 1, b = −2 e c = 6.

b) Temos: a = 2, b = 3 e c = −8.

c) Temos: a = −1, b = 4 e c = −3.

d) Temos: a = −4, b = 7 e c = −12.

e) Temos: a = 1, b = 1 e c = 0.

f) Temos: a = 1, b = 0 e c = −25.

76
POTI 2015 − Álgebra − Nı́vel 2 − Aula 0 − Professores Cleber Assis, Samuel Barbosa e Tiago Miranda

198.

a) ( x − 1)2 + ( x + 2)2 = ( x − 3)2

( x − 1)2 + ( x + 2)2 = ( x − 3)2


x2 − 2x + 1 + x2 + 4x + 4 = x2 − 6x + 9
2x2 + 2x + 5 − x2 + 6x − 9 = 0
x2 + 8x − 4 = 0

Então a = 1, b = 8 e c = −4.

b) (2x − 5)2 + ( x − 2)( x + 2) = x + ( x + 7)2

(2x − 5)2 + ( x − 2)( x + 2) = x + ( x + 7)2


4x2 − 20x + 25 + x2 − 4 = x + x2 + 14x + 49
5x2 − 20x + 21 − x − x2 − 14x − 49 = 0
4x2 − 35x − 28 = 0

Então a = 4, b = −35 e c = −28.

c) ( x − 1)2 + x ( x + 1) = 2x − ( x + 3)2

( x − 1)2 + x ( x + 1) = 2x − ( x + 3)2
x2 − 2x + 1 + x2 + x = 2x − ( x2 + 6x + 9)
2x2 − x + 1 = 2x − x2 − 6x − 9
2x2 − x + 1 − 2x + x2 + 6x + 9 = 0
3x2 + 3x + 10 = 0

Então a = 3, b = 3 e c = 10.
199.

a) x2 + 7x. b) x2 + 7x + 10. c) x2 + x − 6.
200.

a) Tem-se a = 1, b = −5 e c = 4. Portanto, ∆ = b2 − 4ac = (−5)2 − 4 · 1 · 4 = 25 − 16 = 9.

b) Tem-se a = 5, b = 3 e c = −2. Portanto, ∆ = b2 − 4ac = (3)2 − 4 · 5 · (−2) = 9 + 40 = 49.

c) Tem-se a = −1, b = 1 e c = 30. Portanto, ∆ = b2 − 4ac = (1)2 − 4 · (−1) · 30 = 1 + 120 = 121.

d) Tem-se a = 3, b = 5 e c = 1. Portanto,∆ = b2 − 4ac = (5)2 − 4 · 3 · 1 = 25 − 12 = 13.

e) Tem-se a = −1, b = −2 e c = −1. Portanto, ∆ = b2 − 4ac = (−2)2 − 4 · (−1) · (−1) = 4 − 4 = 0.

f) Tem-se a = 2, b = 6 e c = −8. Portanto, ∆ = b2 − 4ac = (6)2 − 4 · 2 · (−8) = 36 + 64 = 100.

g) Tem-se a = 1, b = 3 e c = 9. Portanto, ∆ = b2 − 4ac = (3)2 − 4 · 1 · 9 = 9 − 36 = −27.

h) Tem-se a = 1, b = 9 e c = 0. Portanto, ∆ = b2 − 4ac = (9)2 − 4 · 1 · 0 = 81 − 0 = 81.

i) Tem-se a = −1, b = 0 e c = 16. Portanto, ∆ = b2 − 4ac = (0)2 − 4 · (−1) · 16 = 0 + 64 = 64.

77
POTI 2015 − Álgebra − Nı́vel 2 − Aula 0 − Professores Cleber Assis, Samuel Barbosa e Tiago Miranda

201. Desenvolvendo o produto, obtemos:

( x − m)( x − n) = x2 − nx − mx + mn
= x2 − (m + n) x + mn.

Assim, m + n = 7 e mn = 10. Veja que m = 2 e n = 5 satisfazem a soma e o produto encontrados nos dois
primeiros itens. Se x = m ou x = n, o termo esquerdo será nulo e consequentemente podemos afirmar que m
e n são as raı́zes da equação x2 − 7x + 10. Assim, 2 e 5 são as raı́zes procuradas e não existem outros números
cuja soma é 7 e o produto é 10.

202.

a) x2 − 4x = 0 c) x2 + 9x = 0 e) − x2 − 7x = 0

x2 − 4x = 0 x2 + 9x = 0 − x2 − 7x = 0
x ( x − 4) = 0 x ( x + 9) = 0 x (− x − 7) = 0

Logo, S = {0, 4}.


Logo, S = {−9, 0}. Logo, S = {−7, 0}.
b) x2 − 4 = 0
d) x2 + 9 = 0 f) − x2 + 121 = 0
x2 − 4 = 0 − x2 + 121 = 0
x2 + 9 = 0
2
x = 4 x 2 = −9 121 = x2
√ √
x = ± 4 √
x = ± −9 ± 121 = x
x = ±2 √ ±11 = x.
Como −9 ∈ / R, o conjunto
Logo, S = {−2, 2}. solução é vazio, ou seja, S = ∅. Logo, S = {−11, 11}.

Comentário para professores: É importante destacar o significado prático de uma raiz numa equação, ou
seja, o valor que torna verdadeira a igualdade associada à equação. A substituição do(s) valor(es) calculado(s)
de modo a inspecionar a respectiva validade de cada raiz deve ser enfatizada entre os alunos.

203. Substituindo os valores na equação, temos:

(−1)2 − 7 · (−1) + 10 = 1 + 7 + 10 = 18
(2)2 − 7 · 2 + 10 = 4 − 14 + 10 = 0
(5)2 − 7 · 5 + 10 = 25 − 35 + 10 = 0.

Portanto, apenas 2 e 5 são raı́zes da equação.

204. Para que x = −3 seja raı́z, devemos ter:

−m · (−3)2 − 4m · (−3) + 21 = 0
−9m + 12m + 21 = 0
3m = −21
m = −21/3
m = −7

Portanto, m = −7.

78
POTI 2015 − Álgebra − Nı́vel 2 − Aula 0 − Professores Cleber Assis, Samuel Barbosa e Tiago Miranda

205. Temos:

a) ∆ = 9 > 0 d) ∆ = −36 < 0 g) ∆ = 0


b) ∆ = 25 > 0 e) ∆ = 49 > 0
c) ∆ = 25 > 0 f) ∆ = −15 < 0
206.

a) Tem-se
√ ∆ = (−7)2 − 4 · 1 · 6 = 49 − 24 = 25. Daı́, d) Tem-se
√ ∆ = (1) − 4 · (−3) · (−10) = 1 + 120 = 121.
2

∆=5e Daı́, ∆ = 11 e
7+5
x1 = −1 + 11
2 x1 =
= 6 −6
7−5 −5
x2 = =
2 3
= 1 −1 − 11
x2 =
Logo, S = {1, 6}.
−6
= 2
b) Tem-se
√ ∆ = (−5)2 − 4 · 1 · 4 = 25 − 16 = 9. Daı́,
−5
 
∆=3e Logo, S = ,2 .
5+3 3
x1 =
2
= 4 e) Tem-se
√ ∆ = (4)2 − 4 · 1 · 4 = 16 − 16 = 0. Daı́,
5−3 ∆=0e
x2 =
2
= 1 4+0
x1 =
2
Logo, S = {1, 4}.
= 2
c) Tem-se ∆ = (1)2 − 4 · 2 · (−10) = 1 + 80 = 81.Daı́, 4−0
√ x2 =
∆=9e 2
= 2
−1 + 9
x1 =
4
= 2 Logo, S = {2}.
−1 − 9
x2 =
4 f) Tem-se
√ ∆ = (2)2 − 4 · 5 · 2 = 4 − 40 = −36. Daı́,
−5 ∆∈ / R e, portanto, S = ∅.
=
2
−5 g) Tem-se ∆ = (5)2 − 4 · 3 · 7 = 25 − 84 = −59. Daı́,
 
Logo, S = ,2 . √
2 ∆∈ / R e, portanto, S = ∅.
207.

i) Para ∆ < 0, como ∆ ∈


/ R, não há raı́zes reais (conjunto solução vazio).

ii) Para ∆ = 0, há raı́zes reais iguais e ambas são iguais (conjunto solução unitário).

iii) Para ∆ > 0, há raı́zes reais diferentes (conjuntos solução com dois elementos).

Comentário para professores: Após a questão sobre a importância do valor numérico do delta é salutar
destacar o motivo do seu nome ser discriminante. Discriminar é mostrar, expor, exibir. O exercı́cio anterior
nos permite concluir que o ∆ “mostra” a quantidade de raı́zes de uma equação do 2◦ grau.

79
POTI 2015 − Álgebra − Nı́vel 2 − Aula 0 − Professores Cleber Assis, Samuel Barbosa e Tiago Miranda

208. Como h é uma raiz da equação, temos h2 = 1 − h. Isso nos permite trocar o termo 1 − h por h2 . Logo,

h5 2h6 h5 2h6
+ = + 2 2
1 − h (1 − h )2 h 2 (h )
h 5 2h6
= +
h2 h4
= h + 2h2
3

= h(h2 ) + 2h2
= h(1 − h) + 2h2
= h − h2 + 2h2
= h + h2
= h+1−h
= 1
209. Sejam n o número de jovens e p o valor que cada pessoa deveria pagar. Sendo assim, n · p = 342.
Excluindo-se três jovens do pagamento a aumentando-se o valor pago, teremos:

(n − 3)( p + 19) = 342


(n − 3)(342/n + 19) = 342
 + 19n − 3 · 342 − 57
342 = 
342

n
2
19n − 57n − 1126 = 0
n2 − 3n − 54 = 0.

Como ∆ = (−3)2 − 4 · 1 · (−54) = 225, as raı́zes da equação anterior são:

3 + 15
n1 =
2
n1 = 9
3 − 15
n2 =
2
n2 = −6

Contudo, apenas o 9 é admissı́vel, pois como n representa o número de pessoas do grupo, trata-se de um
número não negativo.

210. As raı́zes de x2 − 5x + 6 = 0 são 2 e 3. Assim, podemos escrever x2 − 5x + 6 = ( x − 2)( x − 3). Como


2 < a < 3, segue que a − 2 > 0 e a − 3 < 0. Daı́,

a2 − 5a + 6 = ( a − 2)( a − 3) < 0.

211. Para que a equação não possua raı́zes reais, seu discriminante deve ser negativo, ou seja,

a2 − 4 < 0
a2 < 4
| a| < 2

Assim, os possı́veis valores de a são aqueles compreendidos entre −2 e 2.

80
POTI 2015 − Álgebra − Nı́vel 2 − Aula 0 − Professores Cleber Assis, Samuel Barbosa e Tiago Miranda

212. Para a equação possuir alguma raiz real, seu discriminante deve ser não-negativo, ou seja,

4(k − 1)2 − 4(k + 5) = 4(k2 − 3k − 4) ≥ 0.

Isso ocorre apenas de k ≥ 4 ou se k ≤ −1. Supondo tais restrições, sejam a e b as raı́zes da equação original.
Podemos fatorá-lo como ( x − a)( x − b). Temos:
a) Se 0 está entre as raı́zes se, e somente se, k + 5 = (0 − a)(0 − b) < 0.

b) Ambas raı́zes são positivas se, e somente se, ab = k + 5 > 0 e 2(k − 1) = −( a + b) < 0.

c) Se 0 é uma raiz, k + 5 = 0 e a outra raı́z é x = 12 > 0.


A interseção da restrição inicial com os três conjuntos encontrados anteriormente é o conjunto

S = { x ∈ R| x ≤ −1}.

213. Pela fórmula de Bhaskara, como ∆ = 7, temos:
−3 + 7
x1 =
−4
= −1
−3 − 7
x2 =
−4
5
=
2
Portanto, a maior raiz é 5/2.

214. Pela fórmula de Bhaskara, como ∆ = 0, ambas as raı́zes são iguais à:

−(−4) ± 0
x =
4
= 1.

Portanto, S = {1}.

215. Substituindo −3 como raiz, temos:

(−3)2 − 7(−3) − 2c = 0
9 + 21 − 2c = 0
9 + 21 = 2c
30 = 2c
2c = 30
c = 15.
216.
1
x+ = 2
x
x2 + 1 = 2x
x2 − 2x + 1 = 0
( x − 1)2 = 0
x = 1
1 1
Portanto, x2 + 2
= 12 + 2 = 2.
x 1

81
POTI 2015 − Álgebra − Nı́vel 2 − Aula 0 − Professores Cleber Assis, Samuel Barbosa e Tiago Miranda

217. Inicialmente, devemos ter como condição necessária para a existência dos radicais que 3x − 2 > 0 e
x > 0, ou seja, x > 3/2.
√ √
3x − 2 = x+2
√ √
( 3x − 2) = ( x + 2)2
2

3x − 2 = x + 4 x + 4

2x − 6 = 4 x

(2x − 6)2 = (4 x )2
4x2 − 24x + 36 = 16x
x2 − 10x + 9 = 0

As raı́zes da equação anterior são 1 e 9. Como, x > 3/2, devemos ter x = 9. É fácil ver que tal valor satisfaz a
equação dada e, portanto, S = {9}.

218.

a)

9x4 − 13x2 + 4 = 0
9y2 − 13y + 4 = 0
13 ± 5
y = .
18
No primeiro caso, y = (13 + 5)/18 = 12 , temos as raı́zes x = ±1. No segundo caso, y = (13 − 5)/18 =
(2/3)2 , temos x = ±2/3. Portanto, o conjunto solução é

S = {−1, 1, −2/3, 2/3}.

b)

x4 + 4x2 − 60 = 0
y2 + 4y − 60 = 0
−4 ± 16
y = .
2
√ √
No primeiro caso, y = −2 + 8 = ( 6)2 , temos as raı́zes x = ± 6. O segundo caso, y = −2 − 8 = −10,
não produz raı́zes reais pois não existe um número real x tal que x2 = −10. Portanto, o conjunto solução é
√ √
S = {− 6, 6}.

c)

x4 + 10x2 + 9 = 0
y2 + 10y + 9 = 0
−10 ± 8
y = .
2
Em ambos os casos, y < 0 e consequentemente não existirão raı́zes reais pois x2 ≥ 0 para qualquer x ∈ R.

82
POTI 2015 − Álgebra − Nı́vel 2 − Aula 0 − Professores Cleber Assis, Samuel Barbosa e Tiago Miranda

219. (Extraı́do da AIME)

Sejam m = x + y e n = xy. Como xy2 + x2 y = Substituindo y = 55 − x na segunda equação ob-


xy( x + y), o sistema pode ser reescrito como: temos:
n + m = 71 x (55 − x ) = 16
n · m = 880 2
x − 55x + 16 = 0
Da primeira equação, n = 71 − m. Substituindo tal
valor na segunda, temos: A
√ equação anterior não possui raı́zes inteiras pois
∆ é irracional. Portando, m = 55 não serve.
(71 − m) · m = 880
71m − m2 = 880 ii) Se m = 16, n = 55 e tem-se:
m2 − 71m + 880 = 0
x + y = 16
As raı́zes da equação anterior são 55 e 16. Vejamos xy = 55
cada caso:
i) Se m = 55, n = 16 e tem-se: Daı́,

x + y = 55 x2 + y2 = ( x + y)2 − 2xy = 256 − 110 = 146.


xy = 16
220. Seja y = x2 − 3x + 1. Observe que y + x = ( x − 1)2 . Assim,

( x2 − 3x + 1)2 − 3( x2 − 3x + 1) + 1 = x
y2 − 3y + 1 − x = 0
( y − 1)2 − y − x = 0
( y − 1)2 − ( x − 1)2 = 0
(y − x )(y + x − 2) = 0
( x − 4x + 1)( x2 − 2x − 1)
2
= 0.

Daı́, x2 − 4x + 1 = 0 ou x2 − 2x − 1 = 0 cujas raı́zes são:


√ √
4 ± 12 √ 2± 8 √
x= = 2± 3 e x = = 1 ± 2.
2 2
n √ √ o
Portanto, S = 2 ± 3, 1 ± 2 .

221.
p
4 + 2x − x2 = x − 2
4 + 2x − x2 = x2 − 4x + 4
2x2 − 6x = 0
2x ( x − 3) = 0.

As possı́veis raı́zes são x = 0 ou x = 3. Contudo, se x = 0, tem-se 4 + 2 · 0 − 02 = 0 − 2, um absurdo. É facil
verificar que x = 3 satisfaz a equação. Portanto, o conjunto solução é S = {3}.

Observação: A operação de elevar ambos os membros de uma equação ao quadrado gera uma nova equação
que contém todas as soluções da equação anterior. Entretanto, novas soluções podem ter surgido e por essa
razão é sempre importante verificar se os valores encontrados satisfazem a equação original.

83
POTI 2015 − Álgebra − Nı́vel 2 − Aula 0 − Professores Cleber Assis, Samuel Barbosa e Tiago Miranda

222. Se a = −1, temos: q


p
x2 + 2ax − a = ( x − 1)2 = | x − 1|.
Assim, a igualdade se verificaria para qualquer x ≥ 1. Suponhamos que a 6= −1, então
p
x2 + 2ax − a = x − 1
x2 + 2ax − a = x2 − 2x + 1
2x ( a + 1) = a + 1
x = 1/2
É imediato verificar que x = 1/2 satisfaz a equação nesse caso. Portanto, S = {1/2}.
223. Se algum dos coeficientes da equação anterior não é nulo, ela possuirá no máximo duas raı́zes pois terá
grau no máximo dois. Sendo assim, devemos ter a2 − 3a + 2 = a2 − 5a + 4 = a − a2 = 0. A única raiz comum
às três equações anteriores é a = 1.
224. Suponha que a fração irredutı́vel p/q, isto é p e q não possuem fatores primos em comum, seja raiz da
equação dada. Susbtituindo tal raiz e multiplicando o resultado por q2 , temos:

ax2 + bx + c = 0
 2  
p p
a +b +c = 0
q q
ap2 + bpq + cq2 = 0
Se ou p ou q for par, na soma anterior, teremos dois números pares somados a um ı́mpar cuja soma resultará
em um número ı́mpar e não no zero. Se ambos são ı́mpares, a soma anterior é constituı́da por três números
ı́mpares e naturalmente sua soma será também ı́mpar (e não o zero). Como supomos inicialmente que eles
eram primos entre si, não há caso para ambos pares. Sendo assim, a equação com coeficientes ı́mpares não
possui solução racional.

225. Seja u = x, então
q √
x = 2+ 2+ x
q √
x−2 = 2+ x

u2 − 2 = 2+u
( u2 − 2)2 = 2 + u
(u2 − 2)2 − u − 2 = 0.
A última equação pode ser fatorada como:
(u2 − 2)2 − u − 2 = (u2 − u − 2)(u2 + u − 1).

Como x > 2, segue que u > 2 > 1. Analisando os raı́zes de u2 + u − 1, nenhuma delas se enquandra nessa
condição. Portanto, u é raiz de u2 − u − 2. Tal equação possui raı́zes −1 e 2. Logo, como u ≥ 0, devemo ter
u = 2 e x = u2 = 4. É fácil verificar que 4 é solução da equação original.
226. (Extraı́do da Olimpı́ada Russa)
Podemos usar a segunda operação e gerar o número 2z. Usando a primeira operação, podemos decidir se z e
2z são iguais, ou seja, se z é ou não igual a zero. Suponha que tenhamos descoberto que z não é zero. Usando
a terceira operação, analisemos as raı́zes de x2 + 2zx + z = 0 que são dadas por:
p
x = −z ± z2 − z.
Como já sabemos que z 6= 0, o discriminante é nulo apenas quando z = 1. Assim, se a calculadora disser que
existe apenas uma raiz real, saberemos que z = 1 e, no caso contrário, teremos z 6= 1.

84
POTI 2015 − Álgebra − Nı́vel 2 − Aula 0 − Professores Cleber Assis, Samuel Barbosa e Tiago Miranda

4

4
227. Sejam s = x−1 e t = 5 − x. Então:

s+t = 2
s + t4 = 4.
4

Usando a primeira equação, se s = 1 + z, temos


t = 1 − z. Assim, substituindo ambos valores na segunda equação, obtemos z4 + 6z2 − 1 = 0. Resolvendo a
equação biquadrada, podemos encontrar q√
z=± 10 − 3.
Como x = (1 + z)4 + 1 = 3 + 4z(1 + z2 ), podemos finalmente obter o conjunto solução:
√ q√
S = { x ∈ R| x = 3 ± 4( 10 − 2) 10 − 3}.

228. (Extraı́do da Revista do Professor de Matemática, números 78 e 79)


Seja x uma solução da equação. Então: Como x > 0, ∆ = 2x + 1. Como 5 é uma das raı́zes
q √ da equação, tem-se:
5− 5−x = x
2x2 + 1 ± (2x + 1)
2

q
5− 5−x = x2 5= .
2

5 − 5 − x = x2
√ Temos dois casos a considerar:
5 − x2 = 5−x
2 2
  √ 2
2x2 + 1 + (2x + 1)
5−x = 5−x = 5
2
2
5 − x2 x2 + x − 4 = 0

= (5 − x )

Desenvolvendo os produtos notáveis anteriores, tem- −1 ± 32
x =
se: 2
2 2x2 + 1 − (2x + 1)
5 − x2 = (5 − x ) = 5
2
52 − 2 · 5 · x 2 + x 4 = 5 − x x2 − x − 5 = 0

52 − (2x2 + 1) · 5 + ( x4 + x ) = 0 1 ± 21
x =
Fixado x, o número 5 é raiz da equação do segundo 2
grau:
z2 − (2x2 + 1) · z + ( x4 + x ) = 0. Como 0 < x < 5, precisa-se eliminar as duas raı́zes
negativas. Além disso, é facil verificar que as outras
O discriminante da equação anterior é: duas satisfazem a equação original. Portanto,
∆ = [−(2x2 + 1)]2 − 4 · 1 · ( x4 + x )
( √ √ )
= 4x4 + 4x2 + 1 − 4x4 − 4x −1 + 21 1 + 32
S= , .
= (2x + 1)2 2 2

229. Se r2 + 5r − 24 = 0, ambos os lados são nulos e a igualdade naturalmente ocorre. As raı́zes da equação
anterior são r = −8 e r = 3. Se r2 + 5r − 24 6= 0, podemos cancelar tal termo obtendo:

r2 − 3r + 2 = 4r − 10
r2 − 7r + 12 = 0.

As raı́zes da equação anterior são r = 4 e r = 3. Portanto, os possı́veis valores de r são: −8, 4 e 3.

85
POTI 2015 − Álgebra − Nı́vel 2 − Aula 0 − Professores Cleber Assis, Samuel Barbosa e Tiago Miranda

230. (Extraı́do e Adaptado da Gazeta Matemática, Romênia)

( ax − b)2 + (bx − a)2 = x


a2 x2 − 2abx + b2 + b2 x2 − 2abx + a2 − x = 0
( a2 + b2 ) x2 + (−1 − 4ab) x + ( a2 + b2 ) = 0

O seu discriminante vale:

∆ = (−1 − 4ab)2 − 4( a2 + b2 )( a2 + b2 )
= (1 + 4ab)2 − [2( a2 + b2 )]2
= (1 + 4ab − 2( a2 + b2 ))(1 + 4ab + 2( a2 + b2 ))
= (1 − 2( a − b)2 )(1 + 2( a + b)2 )

Como a equação possui ao menos uma raiz real, tem-se ∆ ≥ 0. Além disso, 1 + 2( a − b)2 > 0 implica que
1 − 2( a − b)2 ≥ 0. Dado que ( a − b) ∈ Z, devemos ter ( a − b)2 = 0, ou seja, a = b. A equação se transforma
em:
2a2 x − (1 + 4a2 ) x + 2a2 = 0.
1 + 4a2 1
A soma das raı́zes será x1 + x2 = = 2 + 2 e o produto x1 · x2 = 1. Seja x1 a raiz inteira. Por inspeção
2a2 2a
direta na equação anterior, x1 não pode ser nem 0 e nem 1. Logo, x1 ≥ 2. Por outro lado, x2 = 1/x1 > 0 e daı́
1
x1 < x1 + x2 < 2 + 1/2a2 < 3. Consequentemente, x1 = 2 e x2 = . Finalmente, usando a soma das raı́zes,
2
pode-se concluir que a ∈ {−1, 1}. As únicas possibilidades são para a = b = ±1, com raı́zes 2 e 1/2.

231.

a) Se x ≥ 0, a equação se transforma em x2 − x − 2 = 0 cujas raı́zes são −1 e 2. Apenas a raiz 2 convém. Se


x < 0, a equação se transforma em x2 + x − 2 = 0 cujas raı́zes são −2 e 1 e apenas −2 convém. Logo, as
raı́zes são 2 e −2.

b) Se x ≥ 0, a equação se transforma em x2 + 5x + 4 = 0 cujas raı́zes são −1 e −4. Nenhuma das duas


convém. Se x < 0, a equação se transforma em x2 − 5x + 4 = 0 cujas raı́zes são 1 e 4. Novamente nenhuma
das duas raı́zes convém. Logo, a equação não possui raı́zes.

232. Veja que as raı́zes y = 3 e y = −3 aparecem tanto em (y2 − 9) quanto em (y2 − 6y + 9)(y2 + y − 6).
Assim, podemos fatorar a expressão:

(y2 + y − 6)(y2 − 6y + 9) − 2(y2 − 9) = 0


(y − 3)(y + 2)(y + 3)(y + 3) − 2(y − 3)(y + 3) = 0
(y − 3)(y + 3)[(y + 2)(y + 3) − 2] = 0
(y − 3)(y + 3)(y2 + 5y + 4) = 0
(y − 3)(y + 3)(y + 1)(y + 4) = 0.

As raı́zes são y = 3, y = −3, y = −1 ou y = −4.


p √ √
−p ± p2 − 4q 1+ 1+4 1+ 5
233. As raı́zes da equação são dadas por r = cujo valor máximo é = =α
2 2 2
e o mı́nimo é o seu simétrico. Se y é um raiz de tal equação e a é tal que | a| ≤ 1, então z = ay é uma raiz de
x2 + pax + qa2 e os coeficientes ainda estão em [−1, 1]. Consequentemente, todos os os números do intervalo
[−α, α] podem ser raı́zes de tais equações e, como vimos no inı́cio, nenhum outro número fora deste intervalo
pode ser.

86
POTI 2015 − Álgebra − Nı́vel 2 − Aula 0 − Professores Cleber Assis, Samuel Barbosa e Tiago Miranda

234. Elevando ambos os lados ao quadrado:


p
2( x − 3) = x2 − 2x + 3
4( x − 3)2 = ( x − 3)( x + 1)
4( x − 3)2 − ( x − 3)( x + 1) = 0
( x − 3)(3x − 13) = 0.
13
Assim, os candidatos a soluções são x = 3 e x = . É fácil verificar que ambos satisfazem a equação original.
3
235. Multiplicando ambos os lados por ( x − 2)2 , temos:

x2 ( x − 2)2 + 4x2 = 12( x − 2)2


( x2 )2 − 4x2 ( x − 2) − 12( x − 2)2 = 0
( x2 + 2( x − 2))( x2 − 6( x − 2)) = 0.

Analisando as raı́zes das equações anteriores, apenas a primeira delas possui raı́zes reais que são: −1 ± 5.

236. Observe a demonstração abaixo, assumindo que a 6= 0.


2
" #
 ax + bx + c = 0 b 2 4ac − b2

b c a x+ + = 0
a x2 + x + = 0 2a 4a2
a a
b 2 b2 − 4ac
   
2 b c x + =
a x + x+ = 0 2a 4a2
a a √

b b2 b2 c
 b b2 − 4ac
a x2 + 2 x + 2 − 2 + = 0 x+ = ±
2a 4a 4a a 2a 2a

−b ± b2 − 4ac
x =
2a
237. Sim, é verdade. Os três números reais que verificam a equação anterior são raı́zes da equação

( a − m ) x 2 + ( b − n ) x + ( c − p ).

Se a − m 6= 0, tal equação possui no máximo duas raı́zes distintas e isso contradiz a hipótese inicial. Se
a − m = 0 e b − n 6= 0, terı́amos uma equação do primeiro grau que possui solução única e novamente temos
um absurdo. Finalmente, supondo então que a − m = b − n = 0, a equação só possui raı́zes caso tenhamos
também c − p = 0. Ou seja, as três igualdades mencionadas no enunciado devem se verificar.

238.
−(−5) 4
a) Tem-se x1 + x2 = = 5 e x1 · x2 = = 4.
1 1
−7 5
b) Tem-se x1 + x2 = e x1 · x2 = .
2 2
−12 0
c) Tem-se x1 + x2 = = 4 e x1 · x2 = = 0.
−3 −3
−8 −12
d) Tem-se x1 + x2 = = 8 e x1 · x2 = = 12.
−1 −1
−(−3) 3 1
e) Tem-se x1 + x2 = = e x1 · x2 = .
7 7 7

87
POTI 2015 − Álgebra − Nı́vel 2 − Aula 0 − Professores Cleber Assis, Samuel Barbosa e Tiago Miranda

239. Como x1 + x2 = −5 e x1 x2 = 7, temos:

( x1 + 3)( x2 + 3) = x1 x2 + 3( x1 + x2 ) + 9
= 7 + 3 · (−5) + 9
= 1.
240. As relações de soma e produto das raı́zes nos fornecem:

2a + 1 = p − 1
a( a + 1) = p.

Portanto,

a( a + 1) − (2a + 1) = p − ( p − 1)
a2 − a − 2 = 0.

As raı́zes da equação anterior são a = 2 e a = −1. Portanto, como p = a( a + 1), temos p = 6 ou p = 0.


241. Se x1 e x2 são as raı́zes e x1 = − x2 , a soma das raı́zes da equação vale:

− b = x1 + x2
= 0
242.
a) x1 + x2 = 4 e x1 · x2 = −5

b) x1 + x2 = 7 e x1 · x2 = 10
3 1
c) x1 + x2 = e x1 · x2 =
2 2
d) x1 + x2 = 1 e x1 · x2 = −20
243. A condição para raı́zes iguais é ∆ = 0. Sendo assim:

0 = ∆
= b2 − 4ac
= (−8)2 − 4 · 2 · m
= 64 − 8m
Portanto, m = 64/8 = 8.
244. As raı́zes são reais e distintas se, e somente se, ∆ > 0. Devemos ter:

0 < ∆
= b2 − 4ac
= 62 − 4 · 3 · m
= 36 − 12m.
Daı́, 12m < 36, ou seja, m < 3.
245. A equação anterior pode ser reescrita como:

x2 + ( p2 + 3p) x − 8 = 0

Sa as raı́zes da equação, x1 e x2 , são simétricas, tem-se x1 + x2 = 0. Além disso, como x1 + x2 = −( p2 + 3p),


obtemos p2 + 3p = 0. Assim, p = 0 ou p = −3.

88
POTI 2015 − Álgebra − Nı́vel 2 − Aula 0 − Professores Cleber Assis, Samuel Barbosa e Tiago Miranda

246. Sejam x1 e x2 as raı́zes da equação com x1 = 2x2 . Pelas relações de Viète-Girard:


−(−12)
= x1 + x2
2
= 3x2
Portanto, x2 = 2 e x1 = 2 · 2 = 4. Consequentemente 2m/2 = x1 · x2 = 8 e m = 8.
a+b √
247. De = 5 e ab = 8, obtemos a + b = 10 e ab = 64. Assim, eles são raı́zes da equação
2
x2 − 10x + 64 = 0.
248. Se x1 e x2 são as raı́zes da equação com x2 = 1/x1 , pelas relações de Viète-Girard, temos:
p2 − p
= x1 · x2
p+3
1
= x1 ·
x1
= 1.
Daı́,
p2 − p = p + 3
p2 − 2p − 3 = 0.
As raı́zes da equação anterior são p1 = −1 e p2 = 3. Como nenhum desses dois valores produz raiz nula na
equação original, os valores procurados são: p = −1 e p = 3.
249. Para que os quadrados das raı́zes sejam iguais deve acontecer:
x12 = x22
x12 − x22 = 0
( x1 − x2 )( x1 + x2 ) = 0

O que implica que x1 − x2 = 0 ou x1 + x2 = 0, analisando cada caso.


i) Se x1 − x2 = 0, temos x1 = x2 , ou seja, raı́zes iguais e a condição para tal é ∆ = 0.
b2 − 4ac = 0
(m − 2)2 − 4 · 1 · (2m − 8) = 0
m2 − 4m + 4 − 8m + 32 = 0
m2 − 12m + 36 = 0
( m − 6)2 = 0
m = 6

ii) Se x1 + x2 = 0, como a soma das raı́zes é m − 2 = 0, devemos ter m = 2.


Portanto, m = 2 ou m = 6.
250. (Extraı́do do vestibular da UNIFOR CE)
Se 1 e −3 são raı́zes de x2 + Ax + B = 0, concluı́mos que:
1 + (−3) = − A
1 · (−3) = B.
Sendo assim, A = 2 e B = −3. Basta calcular as raı́zes da equação x2 − 3x + 2 − 2 = 0 que são 1 e 2.

89
POTI 2015 − Álgebra − Nı́vel 2 − Aula 0 − Professores Cleber Assis, Samuel Barbosa e Tiago Miranda

251. Basta perceber que qualquer equação do 2o grau com coeficente a = 1 pode ser escrita como:

x2 − (Soma das Raı́zes) · x + (Produto das Raı́zes) = 0

Portanto, x2 − 4x − 12 = 0 é uma possı́vel equação para responder o problema. É importante observar que
qualquer outra equação que satisfaça o enunciado será um múltiplo da equação encontrada pois dividindo-se
a equação pelo coeficiente de x2 , os outros termos necessariamente serão dados pela soma e o produto das
raı́zes.
−(−2n) n+3
252. Tem-se x1 + x2 = = 2n e x1 · x2 = = n + 3 Assim,
1 1
n + 3 = x1 · x2
  
b a 
= +1 · +1
a b
b a
= 1+ + +1
a b
= x1 + x2
= 2n

Portanto, n = 3 e n2 = 9.
253.

a) b)

1 1 x2 + x1 x12 + x22 = ( x1 + x2 )2 − 2x1 x2


+ =
x1 x2 x1 · x2 = (4)2 − 2(−12)
x1 + x2
= = 40
x1 · x2
4
= c)
−12
1
= − ( x1 − x2 )2 = x12 + x22 − 2x1 x2
3
= 40 − 2(−12)
= 64
| x1 − x2 | = 8

254. Temos x1 + x2 = −1 e x1 x2 = −7.

a) c)
x12 + x22 = ( x1 + x2 )2 − 2x1 x2
x14 + x24 = ( x12 + x22 )2 − 2x12 x22
= 1 + 14
= (15)2 − 2(−7)2
= 15
= 225 − 98
b) = 127.
x13 + x23 = ( x1 + x2 )( x12 − x1 x2 + x22 )
= ( x1 + x2 )(( x1 + x2 )2 − 3x1 x2 )
= −1 · ((−1)2 + 3 · 7)
= −22

90
POTI 2015 − Álgebra − Nı́vel 2 − Aula 0 − Professores Cleber Assis, Samuel Barbosa e Tiago Miranda

255. Pelas relações de Viète-Girard, temos a + b = 1/2 e ab = 3/2. Assim

(2a − 1)(2b − 1) + 8 = 4ab − 2( a + b) + 1 + 8


3 1
= 4· −2· +9
2 2
= 14.
256. Como as raı́zes são as mesmas, os resultados obtidos pelas relações de Viète-Girard das duas equações
devem ser os mesmos, ou seja,

3m + 1 5
− = −
4 2
n+3 8
=
4 2
Assim, m = 3 e n = 13.
257. (Extraı́do do vestibular da PUC MG)
Temos p + q = 11/15 e pq = 2/15. Assim,

1 1 p+q
+ =
p q pq
11/15
=
2/15
= 11/2.
258. (Extraı́do da Olimpı́ada Cearense de Matemática)
Se x12 + x22 = 1, usando que x1 x2 = −1/2 e que x1 + x2 = p/2, tem-se:

( p/2)2 = ( x1 + x2 )2
= x12 + 2x1 x2 + x22
= 1 + 2x1 x2
= 1−1
= 0.

Portanto, p = 0.

259. Seja x2 + 18x + 30 = y, portanto:


p
y = 2· y + 15
2
= (2 · y + 15)2
p
y
y2 = 4 · (y + 15)
y2 − 4y − 60 = 0

As raı́zes da última equação são y = 2 ± 8, isto é, y1 = 10 e y2 = −6. Assim, x2 + 18x + 30 = 10 ou


x2 + 18x + 30 = −6. É fácil verificar que as quatro raı́zes encontradas nessas duas equações são reais e
satisfazem a equação original. Em ambos os casos, a soma das raı́zes é −18 e consequentemente a soma total
de todas as raı́zes é −36.
260. (Extraı́do da OBM)
Pelas relações de Viète-Girard, segue que a + b = − a e ab = b, ou seja, b = −2a e
b( a − 1) = 0. Como b é não nulo, temos a − 1 = 0. Consequentemente a = 1 e b = −2 · 1 = −2. Fi-
nalmente, a − b = 1 − (−2) = 3.

91
POTI 2015 − Álgebra − Nı́vel 2 − Aula 0 − Professores Cleber Assis, Samuel Barbosa e Tiago Miranda

261. Como −b/a = R + S e c/a = RS, temos:


( aR + b) + ( aS + b) = a( R + S) + 2b
= −b + 2b
= b.
( aR + b)( aS + b) = a2 RS + ab( R + S) + b2
= ac − b2 + b2
= ac
Portanto, a equação x2 − bx + ac possui as raı́zes desejadas.
262. Suponha que exista tal raiz m. Se n é a outra raiz, como mn = 17 > 0, devemos ter n > 0 . Além disso,
de n + m = −b, podemos concluir que n = −m − b é um número inteiro. Se m e n são números inteiros tais
que m · n = 17, como 17 possui apenas dois divisores positivos, tem-se m = 1, n = 17 ou n = 1, m = 17. Em
qualquer um dos casos, 18 = m + n = −b. Isso produz uma contradição pois b ≥ 0.
263. (Extraı́do da Olimpı́ada Russa)
O conjunto dos trinômios que não possuem raı́zes reais é maior. Seja x2 + ax + b um trinômio possuindo as
raı́zes inteiras m e n com m ≤ n. Como m + n = − a, segue que ambos são negativos. Além disso, como mn = b,
segue que m e n são divisores de um número menor ou igual a 1997. Portanto, −1997 ≤ m ≤ n ≤ 0. Daı́,
podemos concluir que trinômio x2 − nx + mn está em B pois seu discriminante é n2 − 4mn = n(n − 4m) < 0.
Veja que para cada elemento de A associamos um único elemento de B e isso nos diz que B possui pelo menos
tantos elementos quanto A tem. Note que a equação x2 − 3x + 5 está em B e não é da forma mencionada
anteriormente. Assim, B possui estritamente mais elementos do que A.
264. (Extraı́do da OBM)
De inı́cio, fazendo a soma das raı́zes, tem-se que a + b = 3c e c + d = 3a. Somando e subtraindo membro a
membro chega-se ao sistema:


 b + d = 2( a + c )


 b − d = 4( a − c )

Agora, como a é raiz de x2 − 3cx − 8d = 0:


a2 − 3ca − 8d = 0 (1)
E como c é raiz de x2 − 3ax − 8b = 0:
c2 − 3ac − 8b = 0 (2)
Subtraindo 2 de 1:
a2 − c2 = 8( d − b )
( a − c)( a + c) = 8(d − b)
( a − c)( a + c) = 8 · 4( a − c)
Como a − c 6= 0, então a + c = 32. Voltando ao primeiro sistema, obtemos b + d = 64. Por fim,
a + b + c + d = 64 + 32 = 96.
265. Temos a + b = 2014 e ab = −2004. Note que:
a2 + b2 + a2 b2 + 2ab( a + b + 1) =
( a + b + ab)2 =
(2014 − 2004)2 =
(10)2 = 100.

92
POTI 2015 − Álgebra − Nı́vel 2 − Aula 0 − Professores Cleber Assis, Samuel Barbosa e Tiago Miranda

266. Sejam a e a as raı́zes da equação. Pelas relações de Viète-Girard, segue que a + b = m e ab = −1. Daı́,

a2 + b2 = ( a + b)2 − 2ab
= m2 + 2
a4 + b4 = ( a2 + b2 )2 − 2a2 b2
= ( m2 + 2)2 − 2

Como m2 ≥ 0, segue que m2 + 2 ≥ 2 e que (m2 + 2)2 ≥ 4. Portanto,

a4 + b4 = ( m2 + 2)2 − 2
= 22 − 2
= 2.

A igualdade ocorre quando m2 = 0, ou seja, quando m = 0.

267. (Extraı́do da OBM)


Como a é raiz da equação, temos a2 = − a + 1. Multipliquemos esta equação sucessivamente por a sempre
trocando o valor de a2 nos resultados por − a + 1:

a2 = − a + 1
a3 = − a2 + a
= 2a − 1
4
a = 2a2 − a
= −3a + 2
5
a = −3a2 + 2a
= 5a − 3

Portanto, a5 − 5a = −3. Resposta C.

268. Sejam a e a as raı́zes da equação com a ≥ b. Pelas relações de Viète-Girard, segue que a + b = k + 2 e
ab = k − 1. Daı́,

( a − b )2 = a2 − 2ab + b2
= a2 + 2ab + b2 − 4ab
= ( a + b)2 − 4ab
= ( k + 2)2 − 4( k − 1).

Veja que (k + 2)2 − 4(k − 1) = k2 + 8 ≥ 8. Portanto,


q
a−b = ( a − b )2
q
= ( k + 2)2 − 4( k − 1)

≥ 8

= 2 2.

Portanto, o valor mı́mino é 2 2 e ocorre quando k2 + 8 = 8, ou seja, k = 0.

93
POTI 2015 − Álgebra − Nı́vel 2 − Aula 0 − Professores Cleber Assis, Samuel Barbosa e Tiago Miranda

269. (Extraı́do da OBM)


Sejam r + s = m e rs = n. Assim, a partir das equações:
S3 = mS2 − nS1
S4 = mS3 − nS2 .
Podemos obter:
5 = 2m − n
6 = 5m − 2n.
Resolvendo o sistema anterior nas incógnitas m e n, obtemos m = −4 e n = −13. Daı́,
S5 = mS4 − nS3
= (−4) · 6 − (−13) · 5
= 41.
270. Uma condição necessária para que as frações anteriores existam é que x 6= 0. Multiplicando a equação
por x, obtemos
3 − 2x = 7x + 2
1 = 9x
1/9 = x.
É imediato verificar que x = 1/9 satisfaz a equação do problema e é diferente da restrição inicialmente
mencionada. Portanto, o conjunto solução é S = {1/9}.
271. Uma condição necessária para que as frações anteriores existam é que os denominadores sejam não nulos,
ou seja, x 6= 1. Multiplicando a equação por 3( x − 1), obtemos
3x + 2( x − 1) = 6
5x = 8
x = 8/5.
É imediato verificar que x = 8/5 satisfaz a equação do problema e é diferente da restrição inicialmente
mencionada. Portanto, o conjunto solução é S = {8/5}.
272. Uma condição necessária para que a soma dada exista é que os dois denominadores sejam diferentes de
zero, ou seja, x 6= 0 e x 6= 1. Multiplicando ambos os membros da equação por x ( x − 1), obtemos
1 2x
· x ( x − 1) + · x ( x − 1) = 2 · x ( x − 1)
x x−1
x − 1 + 2x2 = 2x2 − 2x
3x = 1
x = 1/3
É imediato verificar que 1/3 verifica a equação dada e, portanto, o conjunto solução é S = {1/3}.
273. Reduzindo todas as frações a um mesmo denominador, temos
x2 x x
2
− + =
x −1 x−1 x+1
x2 x ( x + 1) x ( x − 1)
2
− 2 + 2 =
x −1 x −1 x −1
x − x − x + x2 − x
2 2
=
x2 − 1
x2 − 2x
.
x2 − 1

94
POTI 2015 − Álgebra − Nı́vel 2 − Aula 0 − Professores Cleber Assis, Samuel Barbosa e Tiago Miranda

274.
a) Para que o denominador não seja nulo, é necesário que x 6= 0. Multiplicando a equação por 2x, temos
2x + 12 = 3x e, consequentemente, x = 12.
b) Para que os denominadores não sejam nulos, é necessário que x 6= 2 e x 6= −2. Mulltiplicando a equação
por ( x − 2)( x + 2), temos
( x + 1)( x + 2) = ( x − 1)( x − 2)
x2 + 3x + 2 = x2 − 3x + 2
6x = 0
x = 0.

c) Para os denominadores não serem nulos, devemos ter x 6= 0 e x 6= 1. Multiplicando a equação dada por
x ( x − 1) = x2 − x, temos
3( x − 1) + x = −3x + 4
7x = 7
x = 1.
Em virtude das restrições iniciais, tal valor não é admissı́vel para x. Portanto, o conjunto solução é vazio.
275. Uma condição necessária para que a soma dada exista é que os denominadores sejam não nulos, ou seja,
x 6= 1 e x 6= −1. Multiplicando a equação por ( x − 1)( x + 1) = x2 − 1, obtemos
2x ( x2 − 1) 3x ( x2 − 1) (5 − x2 )( x2 − 1)
− =
x−1 x+1 x2 − 1
2x ( x + 1) − 3x ( x − 1) = 5 − x2
− x2 + 5x = 5 − x2
x = 1.
Em virtude das restrições mencionadas no inı́cio, tal valor é inadmissı́vel para x. Portanto, o conjunto solução
é vazio.
276. Uma condição necessária para que a soma dada exista é que os denominadores sejam não nulos, ou seja,
x 6= 2 e x 6= −2. Multiplicando a equação por ( x − 2)( x + 2) = x2 − 4, obtemos
4 1 3
+ = 2
x−2 x+2 x −4
2
4( x − 4) x − 4 2 3( x 2 − 4)
+ =
x−2 x+2 x2 − 4
4( x + 2) + ( x − 2) = 3
5x = −3
x = −3/5.
Como tal valor não coincide com as restrições mencionadas no inı́cio, o conjunto solução é S = {−3/5}.
277. (Extraı́do Videoaula)
Uma condição necessária para que a soma dada exista é que os dois denominadores sejam diferentes de zero,
ou seja, x 6= 1 e x 6= −1. Multiplicando ambos os membros da equação por ( x − 1)( x + 1), obtemos
3( x + 1) + 4x ( x − 1) = 4( x − 1)( x + 1)
3x + 3 + 4x2 − 4x = 4x2 − 4
x = 7.
Portanto, o conjunto solução é dado por
S = { x ∈ R| x = 7}.

95
POTI 2015 − Álgebra − Nı́vel 2 − Aula 0 − Professores Cleber Assis, Samuel Barbosa e Tiago Miranda

278. (Extraı́do da Videoaula)


Uma condição necessária para que a soma dada exista é que os dois denominadores sejam diferentes de zero, ou
seja, x 6= 3 e
2
x 6= −3. Multiplicando ambos os membros da equação por ( x − 3)( x + 3) = x − 9, obtemos
x + 3 + 2( x − 3) = 6
3x = 9
x = 3.
Pelo comentário inicial, tal valor não é admissı́vel e assim o conjunto solução é vazio.
279. Uma condição necessária para que a soma dada exista é que os dois denominadores sejam diferentes
de zero, ou seja, x 6= 1/2 e x 6= −1/2. Multiplicando ambos os membros da equação por (2x − 1)(2x + 1) =
4x2 − 1, obtemos
(4x2 − 1)(3x − 1) (4x2 − 1)(3x + 2)
+ = 12x2 − 3 − 1
2x − 1 2x + 1
(2x + 1)(3x − 1) + (2x − 1)(3x + 2) = 12x2 − 4
6x2 + x − 1 + 6x2 + x − 2 = 12x2 − 4
2x = 3
x = 3/2.
Como 3/2 é diferente das restrições mencionadas incialmente, segue que o conjunto solução é S = {3/2}.
280. Multiplicando ambos os membros da equação por ( a − b)( a + b) = a2 − b2 , obtemos
5a( a2 − b2 ) 5a( a2 − b2 ) 2bz( a2 − b2 )
− =
a−b a+b ( a2 − b2 )
5a( a + b) − 5a( a − b) = 2bz
10ab = 2bz.
z = 5a.
281. (Extraı́do da Videoaula)
Uma condição necessária para que a soma dada exista é que os dois denominadores sejam diferentes de zero, ou
seja, a 6= 3 e
2
a 6= −3. Multiplicando ambos os membros da equação por ( a − 3)( a + 3) = a − 9, obtemos
x ( a + 3) = a − 3 − x
x ( a + 4) = a − 3

a−3
Como a − 3 6= 0, segue que a + 4 6= 0 e daı́ x = . Portando o conjunto solução pode ser descrito como
a+4
a−3
S = { x ∈ R| x = , a 6= −3; 3; −4}.
a+4
282. (Extraı́do de Videoaula)
Uma condição necessária para que a soma dada exista é que os dois denominadores sejam diferentes de zero,
ou seja, m 6= n e m 6= −n. Multiplicando ambos os membros da equação por (m − n)(m + n) = m2 − n2 ,
obtemos
x (m − n) − ( x + 1)(m + n) = x − 3
xm − xn − xm − xn − (m + n) = x − 3
x (2n + 1) = 3 − (m + n)
3−m−n
Como n é inteiro, segue que 2n + 1 6= 0 e, consequentemente, x = .
2n + 1

96
POTI 2015 − Álgebra − Nı́vel 2 − Aula 0 − Professores Cleber Assis, Samuel Barbosa e Tiago Miranda

283. (Extraı́do da AIME)


Para que a fração do lado esquerdo exista, o seu denominador deve ser diferente de zero, ou seja, x 6= 0
e x 6= 5. Para x diferente de tais valores, o membro do lado esquerdo possui valor constante igual a 2 e,
consequentemente, x − 3 = 2. Assim, x = 5 e isso produz um absurdo em virtude das restrições mencionadas
inicialmente. Portanto, não existe nenhum valor de x que satisfaça a equação.

284. Subtraindo o número 3 de ambos os membros da equação, temos


x−m x−n−p
−1+ −1 +
n+ p+q+r q+r+m
x−q−r
+ −1 =
m+n+p
x − (m + n + p + q + r ) x − (m + n + p + q + r )
+ +
n+ p+q+r q+r+m
x − (m + n + p + q + r )
=
m+n+p
( x − (m + n + p + q + r )) · S,
 
1 1 1
com S = + + 6= 0, pois cada fração de numerador 1 é positiva. Segue
n+ p+q+r q+r+m m+n+ p
então que x − (m + n + p + q + r ) = 0 e x = m + n + p + q + r0.

x 2 − a2 x−a x+a
285. Como 2 2
= · , podemos fatorar a expressão dada como:
x −b x−b x+b
x + a x + a a( x − a) x − a x − a b( x + a)
   
− + − .
x + b x + b b( x − b) x − b x − b a( x + b)

Além disso, podemos escrever

x + a x + a a( x − a)
 
− =
x + b x + b b( x − b)
a( x + a) b( x + a) ( x − a)
 
− .
b( x + b) a( x + b) ( x − b)

Portanto, a equação inicial pode ser fatorada como

x + a a( x − a) x + a b( x − a)
  
− − = 0
x + b b( x − b) x + b a( x − b)

Podemos eliminar os denominadores multiplicando a última equação por ( x + b)( x − b) = x2 − b2 , obtendo:

( x2 − ( a + b) x − ab)( x2 + ( a + b) x − ab) = 0

Como consequência, x deve ser uma das raı́zes dessas duas equações do segundo grau, ou seja,
 p

 ( a + b)2 + 4ab
( a + b) ±

x= p2
 −( a + b ) ± ( a + b)2 + 4ab
.


2

97
POTI 2015 − Álgebra − Nı́vel 2 − Aula 0 − Professores Cleber Assis, Samuel Barbosa e Tiago Miranda

286. Somando as duas frações em cada membro da equação, obtemos:

a( x − a) + b( x − b) b( x − b) + a( x − a)
= .
ab ( x − a)( x − b)

Os numeradores são os mesmos e, caso sejam diferentes de zero, podem ser cancelados produzindo:

( x − a)( x − b) = ab
2
x − x ( a + b) + ab = ab
x ( x − ( a + b)) = 0.

Temos neste caso as soluções x = 0 e x = a + b. Caso os numeradores sejam nulos,

a( x − a) + b( x − b) = 0
x ( a + b ) = a2 + b2
a2 + b2
x = .
a+b
Como ab 6= 0 e | a| 6= |b|, segue que esta nova solução não coincide com nenhuma das soluções já encontradas
e, portanto, o conjunto solução possui três elementos. Resposta letra D.

287. (Extraı́do da AIME)


A soma das frações do membro da esquerda é

( x − 2) N1 + ( x − 1) N2 ( x − 2) N1 + ( x − 1) N2
= .
( x − 1)( x − 2) x2 − 3x + 2

Como o denominador é o mesmo do membro do lado esquerdo, podemos concluir que os numeradores são
iguais, ou seja,

35x − 29 = ( x − 2) N1 + ( x − 1) N2
x (35 − N1 − N2 ) = 29 − 2N1 − N2 .

29 − 2N1 − N2
Se 35 − N1 − N2 6= 0, teremos uma única solução dada por x = . Portanto, 35 − N1 − N − 2 = 0
35 − N1 − N2
e, consequentemente, 29 − 2N1 − N2 = 0. Obtemos assim um sistema:


 N1 + N2
 = 35

 2N1 + N2 = 29.

Resolvendo-o, encontramos ( N1, N2 ) = (−6, 41). Portanto, N1 N2 = −246.

288. Suponha, por absurdo, que existam tais números. Assim

1 1 1 1 1 1
1 = + + + + +
a b c d e f
bcde f + acde f + abde f + abce f + abcde
= .
abcde f

Portanto, o numerador e o denominador da última fração são iguais. Isso é um absurdo, pois o numerador é
um número par e o denominador é ı́mpar.

98
POTI 2015 − Álgebra − Nı́vel 2 − Aula 0 − Professores Cleber Assis, Samuel Barbosa e Tiago Miranda

289. (Extraı́do da Putnam)


Uma ideia natural é tentar agrupar as soluções em pares. Qualquer solução com a1 6= a2 pode ser pareada
com a outra solução obtida pela troca de posição entre a1 e a2 . Logo, B10 tem a mesma paridade que o número
de soluções com a1 = a2 . Das soluções com a1 = a2 , podemos parear aquelas que tem a3 6= a4 da mesma
maneira. Repetindo esse argumento com ( a5 , a6 ), ( a7 , a8 ) e ( a9 , a10 ), concluı́mos que a paridade de B10 é a
mesma do número de soluções com a5 = a6 , a7 = a8 e a9 = a10 , ou seja, das soluções de:

2 2 2 2 2
+ + + + = 1.
a1 a3 a5 a7 a9
Como anteriormente, podemos nos restringir à quantidade de soluções com a1 = a3 e a5 = a7 da equação:

4 4 2
+ + = 1.
a1 a5 a9
Mais uma vez, podemos nos restringir à quantidade de soluções com a1 = a5 da equação:

8 2
+ = 1.
a1 a9
Agora ficou fácil! Basta contar explicitamente o número de soluções da equação anterior. Como fazer isso?
Bem, ela pode ser fatorada como:

( a1 − 8)( a9 − 2) = 16

que admite 5 soluções correspondendo as fatorações de 16 como 2i × 24−i para i = 0, 1, 2, 3, 4. Então B10 é
ı́mpar.

290. Uma condição necessária para que exista uma solução do problema anterior é que os denominadores das
três frações não sejam nulos, ou seja, x é diferente de a, b e c. Multiplicando a equação por ( x − a)( x − b)( x − c),
temos

( x + a)( x + b)( x − c) + ( x + b)( x + c)( x − a) +


( x + c)( x + a)( x − b) =
3( x − a)( x − b)( x − c) =
3 2
3x − 3x ( a + b + c) + 3x ( ab + bc + ac) − 3abc.

Desenvolvendo o produto dos termos do membro esquerdo da primeira equação e cancelando os termos
presentes no membro direito, obtemos

( a + b + c) x2 − ( ab + bc + ca) x = 0
x [( a + b + c) x − ( ab + bc + ac)] = 0.

ab + bc + ac
Assim, x = 0 ou x = . É imediato verificar que x = 0 é solução. Para que o segundo valor também
a+b+c
satisfaça a equação dada, é necessário que esse valor não seja igual a nenhum dos parâmetros. Isso ocorre se,
e somente se,
(bc − a2 )(ca − b2 )( ab − c2 ) 6= 0.
Portanto, o conjunto solução contém dois elementos se (bc − a2 )(ca − b2 )( ab − c2 ) 6= 0 e apenas um em caso
contrário.

99
POTI 2015 − Álgebra − Nı́vel 2 − Aula 0 − Professores Cleber Assis, Samuel Barbosa e Tiago Miranda

291. Uma condição necessária para que a fração da equação exista é ( x − 2)2 6= 0, ou seja, x 6= 2. Multiplicando
a equação por ( x − 2)2 , obtemos

x2 ( x − 2)2 + 4x2 = 12( x − 2)2


( x2 )2 − 4x2 ( x − 2) = 12( x − 2)2
( x2 )2 − 4x2 ( x − 2) − 12( x − 2)2 = 0
( x2 + 2( x − 2))( x2 − 6( x − 2)) = 0.

Assim, ou x2 + 2( x − 2) = 0 ou x2 − 6( x − 2) = 0. A primeira equação
√ possui√as raı́zes −1 ± 5 e a segunda
não possui raı́zes reais. Portanto, o conjunto solução é S = {−1 + 5, −1 − 5}.

1
292. Seja a = . Então o sistema é equivalente à
x


 a + 3y
 = 6

 2a − 5y = 1.

Subtraindo a segunda equação do dobro da primeira, obtemos 11y = 11, ou seja, y = 1. Substituindo tal valor
na primeira equação, encontramos a = 3. Consequentemente ( x, y) = (1/3, 1).

293. (Extraı́do Videoaula)


Uma condição necessária para que o sistema dado exista é que os denominadores sejam diferentes de zero, ou
seja, x 6= y e x 6= −y. Multiplicando a primeira equação por 3( x + y) e a segunda por x − y, obtemos

3x = x + y
4 = −2( x − y )

Pela primeira equação, y = 2x. Substituindo este valor na segunda equação, obtemos 4 = 2x, ou seja, x = 2 e,
finalmente, y = 4.

294. Uma condição necessária para que as frações envolvidas existam é que tanto x quanto y sejam não nulos.
12
Somando o dobro da primeira equação com a segunda, obtemos = 2, ou seja, x = 6. Substituindo esse
x
valor na primeira equação, encontramos y = 1/2. Portando, ( x, y) = (6, 1/2).

15
295. Multiplicando a primeira equação por 3 e somando com a segunda obtemos = 8, ou seja, y = 15/8.
y
Substituindo esse valor na primeira equação, encontramos x = 15/7.

296. (Extraı́do Videoaula)


1 1
Se a = e b = , o sistema pode ser reescrito como
x y

 4a + 7b = −18


 a + 2b = −7.

Multiplicando a segunda equação por 4 e subtraindo-a da primeira, obtemos −b = 10. Portanto, b = −10 e
1 1
a = −7 − 2b = 13. Finalmente, podemos concluir que x = ey=− .
13 10

100
POTI 2015 − Álgebra − Nı́vel 2 − Aula 0 − Professores Cleber Assis, Samuel Barbosa e Tiago Miranda

297. Uma condição necessária para que as frações existam é que y 6= 0. Multiplicando a primeira equação por
2y, obtemos 2x − 6y = 4. Subtraindo a equação encontrada da segunda equação do sistema, temos −3y = −12,
ou seja, y = 4. Substituindo esse valor na segunda equação, obtemos x = 14.
298. Somando todas as equações, obtemos
4 4 4
+ + = 36
x y z
1 1 1
+ + = 9
x y z

Subtraindo a última equação de todas as equações do sistema, obtemos:


1 1 1
= 2, = 3 e = 4.
x y z
Consequentemente, ( x, y, z) = (1/2, 1/3, 1/4).
299. (Extraı́do da AIME 1968)
Das equações dadas, temos:
1
x−1 =
y
1
y−1 = .
x
Portanto, y( x − 1) = 1 = x (y − 1) e, consequentemente,

xy − y = xy − x
x = y

A resposta correta está na letra E.


300. (Extraı́da da AMC)
x
Seja p = . Assim x = py e o sistema pode ser reescrito como
y

 1
 py + = 4


y
 1 1
 y+ =


py 4
1 p p
Multiplicando a segunda equação por p, obtemos py + = . Por comparação, podemos concluir que = 4
y 4 4
e, consequentemente, p = 16.
1 1
301. Sejam a = e b = . Assim, o sistema pode ser reescrito como
x y

 −4a + 3b = −8


 7a + 2b

= 43

Subtraindo o dobro da primeira equação do o triplo da segunda, obtemos 29a = 145, ou seja, a = 5.
Substituindo o valor de a na primeira equação, obtemos b = 4. Portanto, ( x, y) = (1/5, 1/4).

101
POTI 2015 − Álgebra − Nı́vel 2 − Aula 0 − Professores Cleber Assis, Samuel Barbosa e Tiago Miranda

302. Multiplicando as equações do sistema anterior por xy, obtemos o sistema equivalente:

 x 3 + y3

 = 12xy

 3( x + y ) =

xy.

Como

x3 + y3 = ( x + y)( x2 − xy + y2 )
= ( x + y)(( x + y)2 − 3xy),

é conveniente introduzir as variáveis auxiliares a = x + y e b = xy. Assim o sistema pode ser reescrito como

 a( a2 − 3b) = 12b


3a = b.

Substituindo o valor de b da segunda na primeira equação, obtemos a( a2 − 9a) = 36a. Devemos ter a 6= 0, pois
caso contrário terı́amos também b = 0 e algum dos denominadores iniciais seria nulo, o que é um absurdo.
Podemos então cancelá-lo obtendo a equação do segundo grau

a2 − 9a − 36 = 0.

As suas raı́zes são a = 12 e a = −3. Para cada um desses valores, temos um novo sistema nas incógnitas
iniciais:  
 x + y = −3

 x + y = 12 


e
 
 xy = 36.  xy = −9.
 

3 √ 3 √
No primeiro, caso as soluções são ( x, y) = (6, 6) e, no segundo caso, ( x, y) = ( ( 5 − 1), (− 5 − 1)) ou
2 2
3 √ 3 √
( x, y) = ( (− 5 − 1), ( 5 − 1)). É imediato verificar que as três soluções encontradas satisfazem o sistema
2 2
dado.
303.

Dividindo a a primeira equação pela segunda e pela Por comparação, 5x + 2y = x + 4y, ou seja, y = 2x.
terceira, obtemos o novo sistema Substituindo na primeira equação, encontramos 3z =
 5x + 2y = 9x, ou seja, z = 3x. Assim
 y+z 5
=



x+y 3 xyz
2 =
 z+x 4 x+y
=


6x3

x+y 3
=
Multiplicando ambas as equações por x + y, temos 3x
2
 = 2x .
 5x + 2y − 3z = 0


Daı́, x = ±1 e ( x, y, z) = (1, 2, 3) ou ( x, y, z) =
 (− 1, −2, −3). É imediato verificar que os valores en-
 x + 4y − 3z = 0.

contrados são soluções do sistema.

102
POTI 2015 − Álgebra − Nı́vel 2 − Aula 0 − Professores Cleber Assis, Samuel Barbosa e Tiago Miranda

304. Multiplicando todas as equações, obtemos

( x 1 x 2 . . . x n ) n −2 = a 1 a 2 . . . a n .

Daı́,

n −2
x1 x2 . . . x n = a1 a2 . . . an = S.
Multiplicando a k-ésima equação por xk2 , temos:
x 1 x 2 . . . x k −1 x k +1 . . . x n
ak =
xk
ak xk2 = x1 x2 . . . x n
s
S
xk = .
ak

Portanto, a única solução do sistema é


s s s !
S S S
( x1 , x2 , . . . , x n ) = , ,..., ,
a1 a2 an

com S = n −2 a1 a2 . . . a n .

305. O sistema pode ser reescrito como



 1 1 x+y 1

 + = =
x y x a





1 1 x+z 1
+ = =


 x z xz b
1 1 y+z 1



 + =
 = .
y z yz c

Somando as três equações, obtemos

2 2 2 1 1 1
+ + = + +
x y z a b c
1 1 1 ab + bc + ac
+ + = .
x y z 2abc

Somando agora as duas primeiras equações do sistema anterior e subtraindo o valor da última equação, temos

1 2 1 1 ab + bc + ac
= + + −
x x y z 2abc
1 1 ab + bc + ac
= + −
a b 2abc
a + b ab + bc + ac
= −
ab 2abc
2ac + 2bc − ab − bc − ac
=
2abc
ab + bc − ac
= .
2abc
2abc
Portando, x = .
ab + bc − ac

103
POTI 2015 − Álgebra − Nı́vel 2 − Aula 0 − Professores Cleber Assis, Samuel Barbosa e Tiago Miranda

306. (Extraı́do da Olimpı́ada Russa)


O sistema pode ser reescrito como 
 a2 + 1 2
− = 0


a2



 b
 2
b +1 2
− = 0


 b2 c

 c 2+1 2



 = 0
c2 a
Somando essas equações, obtemos
 2  2  2
1 1 1
1− + 1− + 1− = 0.
a b c

Como um quadrado de um real sempre é não negativo, a única maneira para que a soma deles seja nula é:

1 1 1
1= = = ,
a b c
ou seja, a = b = c = 1.

307. Sejam c e v as velocidades da correnteza do rio e do homem. Os dados do enunciado podem ser
traduzidos no seguinte sistema:

15 15
−5


 =
v+c v−c
 15 15

= −1

2v + c 2v − c
Multiplicando a primeira equação por (v2 − c2 ) e a segunda por (4v2 − c2 ), obtemos

 15(v − c) 15(v + c) − 5(v2 − c2 )

 =

 15(2v − c) = 15(2v + c) − (4v2 − c2 )

Por comparação, segue que 5(v2 − c2 ) = (4v2 − c2 ), ou seja, v2 = 4c2 . Como as velocidades são positivas, v = 2c.
Substituindo esse valor em qualquer uma das duas equações do sistema, obtemos c = 2 e, consequentemente,
v = 4. Resposta letra A.

104
Polos Olímpicos de Treinamento
Curso de Álgebra - Nível 2 Aula 1
Prof. Marcelo Mendes

Produtos Notáveis

Vários problemas de Álgebra para alunos do Ensino Fundamental utilizam Produtos


Notáveis, que são identidades clássicas envolvendo multiplicação de expressões.

Vejamos alguns exemplos para diversos produtos notáveis que auxiliarão na formação
de ideias para problemas futuros mais difı́ceis.

1 Quadrado da soma ou da diferença de dois números


(a + b)2 = a2 + 2ab + b2
(a − b)2 = a2 − 2ab + b2
b2 +b
Problema 1. (OCM) Prove que não existem inteiros positivos a e b tais que a2 +a
= 4.

Solução. Suponha que


 existam tais inteiros positivos a e b. A equação dada é equivalente
a b + b = 4 a + a = 4a + 4a. Isso lembra o quadrado de 2a + 1, que é 4a2 + 4a + 1.
2 2 2

Assim, seria bom somarmos 1 a cada lado, para obtermos

b2 + b + 1 = 4a2 + 4a + 1.

Por outro lado,


b2 < b2 + b + 1 < b2 + 2b + 1 = (b + 1)2
pois b é um inteiro positivo. Como b2 e (b + 1)2 são quadrados consecutivos, isso mostra
que não seria possı́vel b2 + b + 1 ser o quadrado de um inteiro.

No próximo exemplo, vamos utilizar um fato útil de pensar que um número com todos
os dı́gitos 1s, como 11...1, pode ser escrito na forma 99...99 . Se o número possuir apenas o
dı́gito 4, por exemplo, como 44...4, então o escrevemos na forma 4 × 99...9 9 . A vantagem
dessas alterações é saber que 99...9 = 10n −1 (verifique esse fato para quantidades pequenas
| {z }
n
de 9s).
POT 2012 - Álgebra - Nı́vel 2 - Aula 1 - Prof. Marcelo Mendes

r
Problema 2. Seja n > 1 um número inteiro. Prove que o número 11...1 | {z } não é
| {z } 44...4
n 2n
racional.


Solução. Mostrar que 11...144...4 não é racional é equivalente a provar que 11...144...4
não é um quadrado perfeito. Ou seja, este problema tenta mostrar que não há outros
quadrados perfeitos com o formato do número 144.

Podemos escrever
10n − 1 102n − 1
11...144...4 = 11...1 × 102n + 44...4 = × 102n + 4 ×
9 9
10n − 1  10n − 1
= 102n + 4 (10n + 1) = (10n + 2)2 .
9 9
Agora, é suficiente mostrarmos que 10n − 1 nunca pode ser quadrado perfeito se n > 1.
Isso é verdade pelo fato de 10n − 1 deixar resto 3 na divisão por 4 e não existir quadrado
perfeito nessa situação.

Problema 3. (i) Se n é um inteiro positivo tal que 2n + 1 é um quadrado perfeito, mostre


que n + 1 é a soma de dois quadrados perfeitos sucessivos.
(ii) Se 3n + 1 é um quadrado perfeito, mostre que n + 1 é a soma de três quadrados.

Problema 4. Suponha que um número inteiro n seja a soma de dois números triangulares,
2 2
ou seja, n = a 2+a + b 2+b . Mostre que 4n+1 pode ser escrito como a soma de dois quadrados
em termos de a e b.

1 1
Problema 5. Seja x ∈ R tal que x + = 5. Calcule x2 + 2 .
x x
Problema 6. (EUA) O número 121b , escrito na base inteira b, é o quadrado de um inteiro
para quais valores de b?

Problema
√ 7. Seja D = a2 + b2 + c2 , sendo a e b inteiros consecutivos e c = ab. Mostre que
D é sempre um inteiro ı́mpar.

 2
2
Problema 8. (EUA) Determine a soma dos dı́gitos na base 10 de 104n +8 + 1 , sendo n
um inteiro positivo.

Problema 9. Mostre que a soma dos quadrados de dois números ı́mpares consecutivos é
um número par não múltiplo de 4.

2
POT 2012 - Álgebra - Nı́vel 2 - Aula 1 - Prof. Marcelo Mendes

Problema 10. (IME) Mostre que os números 49, 4489, 444889, 44448889, ..., obtidos colocando-
se 48 no meio do número anterior, são quadrados de números inteiros.

Problema 11. Se x12 + 2x6 (1 − 2y 2 ) + 1 = 0 e x ∈ R− , então mostre que y < 1.

Problema 12. Ache todos os inteiros positivos x, y tais que y 2 − x(x + 1)(x + 2)(x + 3) = 1.

Problema 13. Determine todas as triplas de números reais (x, y, z) que são solução da
equação 4x4 − x2 4y 4 + 4z 4 − 1 − 2xyz + y 8 + 2y 4 z 4 + y 2 z 2 + z 8 = 0.

Problema 14. (OCM) Determine todos os valores reais de x, y e z satisfazendo a igualdade

3x2 + y 2 + z 2 = 2xy + 2xz.

Problema 15. (OCM) Determine todos os pares de inteiros (x, y) que satisfazem a equação
x2 + x + 1995 = y 2 + y.

Problema 16. (EUA) Encontre x2 + y 2 se x, y ∈ Z e xy + x + y = 71, x2 y + xy 2 = 880.

2 Diferença de quadrados
a2 − b2 = (a + b)(a − b)

Problema 17. Quantos pares de números inteiros positivos m e n satisfazem a equação


m2 − n2 = 2011?

Solução. Suponha que existam inteiros positivos m e n tais que m2 − n2 = 2011. Daı́,
(m + n)(m − n) = 2011. Como 2011 é primo e m + n > m − n, pois n > 0, segue que
m + n = 2011 e m − n = 1 e, portanto, m = 1006 e n = 1005.

Problema 18. Prove que existe exatamente um número natural n tal que 28 + 211 + 2n é
um quadrado perfeito.

Solução. Vamos buscar soluções para a equação 28 + 211 + 2n = k2 , k ∈ Z. Ela é


equivalente a 28 1 + 23 + 2n = k2 ou 2n = k2 − 482 = (k + 48)(k − 48). Assim, k + 48 = 2a
e k − 48 = 2b , sendo n = a + b. Subtraindo essas equações, obtemos 96 = 2a − 2b e,
portanto, 25 · 3 = 2b 2a−b − 1 . Em cada membro dessa igualdade, temos a fatoração em
parte par e parte ı́mpar. Igualando, obtemos b = 5 e a = 7. Portanto, a única solução é
n = a + b = 7 + 5 = 12.
   
Problema 19. Determine o valor do produto 1 − 212 1 − 312 ... 1 − 912 1 − 1012 .

3
POT 2012 - Álgebra - Nı́vel 2 - Aula 1 - Prof. Marcelo Mendes

Problema 20. (EUA) Simplifique a expressão


√ √ √ √ √ √ √ √ √ √ √ √
( 5 + 6 + 7)( 5 + 6 − 7)( 5 − 6 + 7)(− 5 + 6 + 7).
√ √
Problema 21. (OCM/ITA) Qual é o menor inteior positivo n tal que n − n − 1 < 0, 01.

Problema 22. Quantos pares de números inteiros m e n satisfazem a equação m2 − n2 =


2014?

Problema 23. Seja a 6= 1 um número real. Simplifique a expressão


     
1 1 1 1
1+ 1+ 2 1 + 4 ... 1 + 2100 .
a a a a
Problema 24. Racionalize a expressão
1

64

32

16
√ √ √ .
(1 + 2)(1 + 2)(1 + 2)(1 + 8 2)(1 + 4 2)(1 + 2 2)

Problema 25. (OCM) Encontre o quociente da divisão de a128 − b128 por

(a64 + b64 )(a32 + b32 )(a16 + b16 )(a8 + b8 )(a4 + b4 )(a2 + b2 )(a + b).

Problema 26. A expressão 2n + 1 é o quadrado de um inteiro para exatamente quantos


números naturais n?

Problema 27. Determine todas as soluções inteiras da equação 32x − 52y = 104.

Problema 28. (EUA) Se x + x2 − 1 + x−√1x2 −1 = 20, então determine o valor de
p 1
x2 + x4 − 1 + √ .
x2 + x4 − 1
Problema 29. Um quadrado é cortado em 49 quadrados menores. Todos esses quadrados
têm as medidas de seus lados, em centı́metros, expressas por números inteiros positivos.
Há exatamente 48 quadrados com área igual a 1cm2 . Determine o número de resultados
possı́veis para expressar, em cm2 , a medida da área do quadrado original.

Problema 30. Seja


p p um número primo ı́mpar dado. Quantos valores de k inteiro positivo
existem tais que k2 − pk é também um inteiro positivo?

Problema 31. (EUA) Existe um único par de inteiros positivos x e y satisfazendo a equação
x2 + 84x + 2008 = y 2 . Determine o valor de x + y.

(104 + 324)(224 + 324)(344 + 324)(464 + 324)(584 + 324)


Problema 32. (EUA) Calcule .
(44 + 324)(164 + 324)(284 + 324)(404 + 324)(524 + 324)

4
POT 2012 - Álgebra - Nı́vel 2 - Aula 1 - Prof. Marcelo Mendes

3 Produtos notáveis envolvendo cubos


Soma de dois cubos: a3 + b3 = (a + b)(a2 − ab + b2 )

Diferença de dois cubos: a3 − b3 = (a − b)(a2 + ab + b2 )

Cubo da soma de dois números: (a + b)3 = a3 + 3a2 b + 3ab2 + b3

= a3 + b3 + 3ab(a + b)

Cubo da diferença de dois números: (a − b)3 = a3 − 3a2 b + 3ab2 − b3

= a3 − b3 − 3ab(a − b)

Problema 33. (Eslovênia) Sejam a, b ∈ R tais que a3 − 3a2 + 5a = 1 e b3 − 3b2 + 5b = 5.


Calcule o valor de a + b.

Solução. As expressões nessas equações lembram os cubos das diferenças de a e 1 e b e 1,


respectivamente. Assim, podemos reescrevê-las como

(a − 1)3 + 2(a − 1) = −2,

(b − 1)3 + 2(b − 1) = 2.
Somando-as, obtemos
 
(a + b − 2) (a − 1)2 − (a − 1)(b − 1) + (b − 1)2 + 2 = 0.

Agora, observe que


(a − 1)2 − (a − 1)(b − 1) + (b − 1)2 + 2
(b − 1)2 3(b − 1)2
= (a − 1)2 − (a − 1)(b − 1) + + +2
4 4
 2
b−1 3(b − 1)2
= a−1− + + 2 > 0.
2 4
Assim, a + b = 2.

Problema 34. Prove que se a + b + c = 0, então a3 + b3 + c3 = 3abc.

Solução. Se a + b = −c, então (a + b)3 = (−c)3 , ou seja,

a3 + b3 + 3ab(a + b) = −c3 ⇔ a3 + b3 + 3ab(−c) = −c3 .

Logo, a3 + b3 + c3 = 3abc.

Problema 35. (Putnam)


√ Sejam x, y, z números reais distintos dois a dois. Prove que
√ √
3
x − y + 3 y − z + 3 z − x 6= 0.
√ √
Problema 36. Determine o número de soluções reais distintas da equação 3 x+ 3 7 − x = 3.

5
POT 2012 - Álgebra - Nı́vel 2 - Aula 1 - Prof. Marcelo Mendes

√ √
Problema 37. (EUA/OCM) Mostre que se x é um número satisfazendo 3 x + 9− 3 x − 9 =
3, então 75 < x2 < 85.
s r s r
3 125 3 125
Problema 38. (IME 1991) Mostre que 3 + 9 + − −3 + 9 + é um número
27 27
racional.

Problema 39. (EUA) Se x e y são números inteiros tais que x3 +y 3 +(x+y)3 +30xy = 2000,
determine o valor de x + y.
(23 − 1)(33 − 1) . . . (1003 − 1) 3367
Problema 40. (Leningrado) Prove que 3 3 3
= .
(2 + 1)(3 + 1) . . . (100 + 1) 5050

4 Outros produtos notáveis


ab − a − b + 1 = (a − 1)(b − 1)
ab + a + b + 1 = (a + 1)(b + 1)

Problema 41. Determine o número de pares ordenados (m, n) de números inteiros positivos
4
que são soluções da equação m + n2 = 1.

4
Solução. A equação m + n2 = 1 é equivalente a mn − 2m − 4n + 8 = 8 ⇔ (m − 4)(n − 2) = 8,
seguindo os modelos propostos nesta seção.
As possibilidades são m − 4 = 1, n − 2 = 8; m − 4 = 2, n − 2 = 4; m − 4 = 4, n − 2 =
2; m − 4 = 8, n − 2 = 1, ou seja, os pares ordenados (m, n) são (5, 10); (6, 6); (8, 4); (12, 3).

Problema 42. Determine todos os números inteiros tais que a soma e o produto são iguais.

Problema 43. (IME) Sejam x1 e x2 as raı́zes da equação x2 + (m − 15)x + m = 0. Sabendo


que x1 e x2 são números inteiros, determine o conjunto dos possı́veis valores de m.

6
POT 2012 - Álgebra - Nı́vel 2 - Aula 1 - Prof. Marcelo Mendes

Problemas da OBM

x2 y2
Problema 44. (OBM 1a fase/2002) Se xy = 2 e x2 + y 2 = 5, então y2
+ x2
+ 2 vale:

5 25 5 1
a) 2 b) 4 c) 4 d) 2 e) 1
Problema 45. (OBM 3a fase/2003) Mostre que x2 + 4y 2 − 4xy + 2x − 4y + 2 > 0 quaisquer
que sejam os reais x e y.
Problema 46. (OBM 2a fase/2005)
a) Fatore a expressão x2 − 9xy + 8y 2 .
b) Determine todos os pares de inteiros (x; y) tais que 9xy − x2 − 8y 2 = 2005.
Problema
q 47.q(OBM 1a fase/2005) Os inteiros positivos x e y satisfazem a equação
√ √
x + 12 y − x − 21 y = 1. Qual das alternativas apresenta um possı́vel valor de y?

a) 5 b) 6 c) 7 d) 8 e) 9
Problema 48. (OBM 3a fase/2006) Encontre todos os pares ordenados (x; y) de inteiros
tais que x3 − y 3 = 3(x2 − y 2 ).
Problema 49. (OBM 2a fase/2006) Sejam a e b números reais distintos tais que a2 = 6b+5ab
e b2 = 6a + 5ab.
a) Determine o valor de a + b.
b) Determine o valor de ab.
Problema 50. (OBM 2a fase/2008) Sejam x e y números reais positivos satisfazendo as
equações x2 + y 2 = 1 e x4 + y 4 = 17 1
18 . Calcule o valor de xy .
Problema 51. (OBM 1a fase/2010) Quantos são os pares (x, y) de inteiros positivos tais
que x2 − y 2 = 22010 ?

a) 1000 b) 1001 c) 1002 d) 1003 e) 1004


Problema 52. (OBM 3a fase/2010) Sejam a, b e c reais tais que a 6= b e a2 (b + c) =
b2 (c + a) = 2010. Calcule c2 (a + b).
Problema 53. (OBM 1a fase/2011) Qual é o valor da expressão 201120112 + 201120032 −
16 × 20112007?
a) 2 × 201120072
b) 2 × 201120032
c) 2 × 20112007
d) 2 × 20112003
e) 2 × 201120112

7
POT 2012 - Álgebra - Nı́vel 2 - Aula 1 - Prof. Marcelo Mendes

Dicas

3. Observe que 2n + 1 é o quadrado de um inteiro ı́mpar e que 3n + 1 é o quadrado de


um número não múltiplo de 3.

6. Números na base b só utilizam dı́gitos 0, 1, ..., b − 1.


10n+1 −1 10n −1
| {z } 88...8
10. Escreva 44...4 | {z } 9 = 4 · 9 +8· 9 + 9.
n+1 n

12. Agrupe x com x + 3 e x + 1 com x + 2.

13. Comece separando o −1 de dentro dos parênteses (escrevendo x4 depois). Em seguida,


agrupe y 8 , z 8 e 2y 4 z 4 .

14. Se uma soma de quadrados de números reais é 0, então todos os números são iguais
a 0.

15. Veja a resolução do problema 1.

16. Fatore e faça substituições de variáveis x + y = s e xy = p.


√ √ √
21. Multiplique
√ a inequação membro a membro por n + n − 1. Você obterá n +
n − 1 > 100, cuja menor solução é 2500.

23. Multiplique e divida tudo por 1 − a1 .

27. Primeiramente, descarte os casos em que os números são negativos. Depois, use que
soma e diferença de dois números inteiros têm a mesma paridade. Por fim, lembre-se
que o produto de dois números negativos é positivo.

30. Escreva k2 −pk = n2 e complete o trinômio quadrado perfeito que começa com k2 −pk,
2
somando e subtraindo p4 .

31. Complete o trinômio quadrado perfeito que começa com x2 + 84x.

32. Fatore a expressão x4 + 324 = x4 + 182 . A dica é somar e subtrair 2 · x2 · 18.

39. Passe 2000 para o lado esquerdo da equação e fatore fazendo aparecer o fator x + y.

8
POT 2012 - Álgebra - Nı́vel 2 - Aula 1 - Prof. Marcelo Mendes

Respostas

5. 23
6. b > 2
8. 4
12. y = x2 + 3x + 1
13. (t2 , t, t) ou (−t2 , t, −t), t ∈ R
14. x = y = z = 0
15. Não existe par (x, y)
16. 146
11
19. 20

20. 104
21. 2501
22. 0
101
1−a−2
23. 1−a−1

64
24. 2−1
25. a64 − b64
26. 1
27. x = 3, y = 2
28. 51, 005
29. 2
30. 1 (para cada primo ı́mpar p)
31. 80
32. 373
36. 2
39. 10
42. (0, 0), (2, 2)
43. 0, 7, 9, 25, 27, 34

9
Polos Olímpicos de Treinamento
Curso de Álgebra - Nível 2 Aula 2
Prof. Marcelo Mendes

Equações e Sistemas de Equações

Neste 2o texto de Álgebra, veremos diversos exemplos de equações e sistemas de equações


em nı́vel de problemas olı́mpicos do ensino fundamental.

Eles, possivelmente, servirão posteriormente de ideia para problemas mais difı́ceis.

1 Equações
Nossos três primeiros exemplos são de equações em que as soluções utilizam produtos
notáveis, como aplicação do último assunto.

Problema 1. (EUA) Determine o número de soluções inteiras da equação 22x − 32y = 55.

Solução. Inicialmente, observe que o lado esquerdo da equação é a diferença dos quadrados
de 2x e 3y e, portanto, (2x + 3y ) (2x − 3y ) = 55. Veja que x e y são positivos (prove isso!),
além de (2x + 3y ) e (2x − 3y ). Assim, as únicas possibilidades são

2x + 3y = 55 2x + 3y = 11
 
e .
2x − 3y = 1 2x − 3y = 5

Apenas o segundo sistema possui solução, que é (x, y) = (3, 1).

Problema 2. Quantas soluções inteiras possui a equação x2 − 4xy + 6y 2 − 2x − 20y = 29?

Solução. Os dois primeiros termos do lado esquerdo dão a pista do começo pois lembram
o quadrado de x − 2y. Assim, vamos reescrever a equação da seguinte forma

x2 − 4xy + 4y 2 − 2x + 4y + 1 + 2y 2 − 24y + 72 = 102

⇔ (x − 2y)2 − 2(x − 2y) + 1 + 2(y 2 − 12y + 36) = 102


⇔ (x − 2y − 1)2 + 2(y − 6)2 = 102.
POT 2012 - Álgebra - Nı́vel 2 - Aula 2 - Prof. Marcelo Mendes

Assim, x − 2y − 1 é par e não maior que 10. Testanto x − 2y − 1 = 0, ±2, ±4, ±6, ±8, ±10,
obtemos (y − 6)2 = 51, 49, 43, 33, 19, 1. Logo, as únicas soluções vêm de x − 2y − 1 = ±2 e
y − 6 = ±7 ou x − 2y − 1 = ±10 e y − 6 = ±1. As soluções, portanto, são

(29, 13); (25, 13); (1, −1); (−3, −1); (25, 7); (5, 7); (21, 5); (1, 5).

Problema 3. (Romênia/2006) Encontre todos os números reais a e b satisfazendo

2(a2 + 1)(b2 + 1) = (a + 1)(b + 1)(ab + 1).

Solução. Utilizando produtos notáveis, a equação dada fica equivalente a

2 a2 b2 + a2 + b2 + 1 = (ab + a + b + 1) (ab + 1)


⇔ 2a2 b2 + 2a2 + 2b2 + 2 = a2 b2 + a2 b + ab2 + +2ab + a + b + 1


⇔ a2 b2 − b + 2 − a b2 + 2b + 1 + 2b2 − b + 1 = 0,
 

que pode ser considerada uma equação do 2o grau em a cujo discriminante (∆) é

∆ = (b + 1)4 − 4 b2 − b + 2 2b2 − b + 1
 

= −7b4 + 16b3 − 18b2 + 16b − 7.


Esse polinômio possui duas caracterı́sticas interessantes. A primeira, que nós não uti-
lizaremos, é que ele é um polinômio recı́proco de 4o grau e de 1a espécie, pois a leitura
de seus coeficientes da esquerda para direita coincide com a leitura feita da direita para a
esquerda. A segunda é que b = 1 é uma raiz já que o valor 1 zera o ∆. Isso nos leva a
escrever
∆ = −7b4 + 7b3 + 9b3 − 9b2 − 9b2 + 9b + 7b − 7
= (b − 1) −7b3 + 9b2 − 9b + 7 .


Novamente, o segundo fator desse último produto é um polinômio recı́proco de 3o grau,


mas de 2a espécie, já que as leituras dos coeficientes nos dois sentidos são simétricas. Além
disso, b = 1 é novamente uma raiz e, escrevendo −7b3 +9b2 −9b+7 = (b−1) −7b2 + 2b − 7 ,


obtemos
∆ = (b − 1)2 −7b2 + 2b − 7 .


O discriminante de −7b2 + 2b − 7 é negativo e, portanto, −7b2 + 2b − 7 < 0, ∀b. Como


(b − 1)2 ≥ 0, ∀b, segue que ∆ ≤ 0, ∀b. Para a ∈ R, devemos ter ∆ = 0 e, portanto, b = 1 e
a = 1, que é a única solução.

Problema 4. (Croácia) Encontre todas as soluções inteiras da equação


√ √ √
4x + y + 4 xy − 28 x − 14 y + 48 = 0.

Problema 5. Mostre que x2 − y 2 = a3 sempre tem solução inteira (x, y), dado que a ∈ Z.

2
POT 2012 - Álgebra - Nı́vel 2 - Aula 2 - Prof. Marcelo Mendes

Problema 6. Prove que se os coeficientes de uma equação quadrática ax2 + bx + c são


inteiros ı́mpares, então as raı́zes da equação não podem ser números racionais.
 √  p 
Problema 7. Se x e y são reais tais que x + x2 + 1 y + y 2 + 1 = 1, prove que
x + y = 0.

Problema 8. Para quais números reais a, b, c (a 6= 0, b 6= 0, c 6= 0, a + b + c 6= 0) vale a


1 1 1 1
igualdade + + = .
a b c a+b+c

Problema 9. Sejam a, b, c, d inteiros distintos tais que a equação


(x − a)(x − b)(x − c)(x − d) − 4 = 0
possui uma raiz inteira r. Mostre que 4r = a + b + c + d.

4 4 2
Problema 10. (EUA) Se 1 − + 2 = 0, determine o valor de .
x x x

Problema 11. (EUA) Se ab 6= 0 e |a| =


6 |b|, quantos valores distintos de x satisfazem a
x−a x−b b a
equação + = + ?
b a x−a x−b

2 Sistemas de equações
Vamos iniciar com um problema da 1a fase do nı́vel 2 da XXI OBM.
Problema 12. (OBM) Rafael tem 23 da idade de Roberto e é 2 anos mais jovem que Rei-
naldo. A idade de Roberto representa 34 da idade de Reinaldo. Determine a soma em anos
das idades dos três.

Solução. Sejam a, o, e as idades de Rafael, Roberto e Reinaldo, respectivamente. Assim,


a = 32 o, a = e − 2 e o = 43 e. Daı́, a = 23 · 34 e = e − 2, o que dá e = 18. Portanto, a = 16,
o = 24 e a + o + e = 58.
Problema 13. (EUA - Adaptado) Determine todas as triplas ordenadas distintas (x, y, z)
de números inteiros satisfazendo o sistema de equações

 x + 2y + 4z = 12
xy + 4yz + 2zx = 22 .
xyz = 6


 x + 2y + 4z = 12
Solução. Podemos reescrever o sistema como x · 2y + 2y · 4z + x · 4z = 44 .
2y · 4z = 48

3
POT 2012 - Álgebra - Nı́vel 2 - Aula 2 - Prof. Marcelo Mendes

Fazendo x = x′ , 2y = y ′ e 4z = z ′ , chegamos a
 ′
x + y′ + z′ = 12
′ ′
xy + yz + xz ′ ′ ′ ′ = 44 .
x′ y ′ z ′ = 48

Assim, x′ , y ′ , z ′ são raı́zes da equação t3 − 12t2 + 44t − 48 = 0 (verifique!), que possui 2


como raiz. Daı́, podemos reescrevê-la como

t3 − 2t2 − 10t2 + 20t + 24t − 48 = 0

⇔ (t − 2) t2 − 10t + 24 = 0


⇔ (t − 2)(t − 4)(t − 6) = 0,
que gera as soluções 2, 4, 6. Assim, 4z = 4 e z = 1. Além disso, x = 2 e y = 3 ou x = 6 e
y = 1.
Problema 14. (URSS) Encontre todas as soluções inteiras (x, y, z, t) do sistema

xz − 2yt = 3
.
xt + yz = 1

Solução. Nas duas equações, aparecem as 4 letras exatamente uma vez. Assim, pode-
mos elevá-las ao quadrado e somar o resultado da primeira com o dobro do da segunda,
eliminando o produto xyzt

(xz)2 + 2(xt)2 + 4(yt)2 + 2(yz)2 = 11

⇔ x2 z 2 + 2t2 + 2y 2 z 2 + 2t2 = 11
 

⇔ x2 + 2y 2 z 2 + 2t2 = 11.
 
 2
x + 2y 2 = 1
 2
x + 2y 2 = 11
Temos as seguintes possibilidade ou .
z 2 + 2t2 = 11 z 2 + 2t2 = 1
No primeiro, temos x2 = 1, y 2 = 0 e z 2 = 9, t2 = 1. No segundo, x2 = 9, y 2 = 1
e z2 = 1, t2 = 0. Substituindo nas equações iniciais, obtemos as soluções (x, y, z, t) =
(1, 0, 3, 1), (−1, 0, −3, −1), (3, 1, 1, 0), (−3, −1, −1, 0).

Problema 15. (Bielorrússia) Determine todas as soluções reais do sistema (n ≥ 2):


1
x1 + x2 + ... + xn−1 =

 xn
1

 x + x + ... + x
 2 3 n = x1
.. .. .


 . = .
1
xn + x1 + ... + xn−2 =

xn−1

Solução. Inicialmente, observe que x1 + x2 + ... + xn = xk + x1k (∗), ∀k ∈ {1, 2, ..., n} e que
todos os xk são não-nulos. Tomando duas equações quaisquer, obtemos xi + x1i = xj + x1j ,

4
POT 2012 - Álgebra - Nı́vel 2 - Aula 2 - Prof. Marcelo Mendes

1
cujas soluções são xi = xj ou xi = xj .

Supondo a segunda possibilidade e substituindo na equação do sistema original em que


o lado direito é x1j , chegamos a x1 + ... + x̂i + ... + x̂j + ... + xn = 0 (a notação x̂i significa
que xi foi suprimido da soma), o que é impossı́vel já que (*) garante que os xk são todos
positivos ou todos negativos.

Assim, só nos resta a opção em que todos os xk são iguais, digamos a α. Substi-
1
tuindo em qualquer uma das equações, obtemos (n − 1)α = α1 , ou seja, xk = √n−1 , ∀k ou
1
xk = − √n−1 , ∀k.

Problema 16. Resolva o sistema de equações



x + y + z = 2
x2 − y 2 − z 2 = 2 .
x − 3y 2 + z = 0

Problema 17. (IMTS) O conjunto S é formado por 5 inteiros. Se os elementos de S são so-
mados aos pares, obtemos 1967, 1972, 1973, 1974, 1975, 1980, 1983, 1984, 1989, 1991. Quais
são os elementos de S?

Problema 18. (EUA) Resolva o sitema de equações




 2x1 + x2 + x3 + x4 + x5 = 6
 x1 + 2x2 + x3 + x4 + x5 = 12


x1 + x2 + 2x3 + x4 + x5 = 24 .
x + x2 + x3 + 2x4 + x5 = 48

 1



x1 + x2 + x3 + x4 + 2x5 = 96
Problema 19. Mostre que o sistema
 1
 x+ x = y
1
y+y = z
z + z1 = x

não possui soluções reais (x, y, z).


Problema 20. Mostre que a única solução do sistema


 x1 + x2 + x3 = 0
x2 + x3 + x4 = 0




 x3 + x4 + x5 = 0


.. .

 . = .. .
x98 + x99 + x100 = 0




x + x100 + x1 = 0

 99



x100 + x1 + x2 = 0

5
POT 2012 - Álgebra - Nı́vel 2 - Aula 2 - Prof. Marcelo Mendes

é x1 = x2 = ... = x99 = x100 = 0.

Problema 21. (EUA) Quatro inteiros positivos a, b, c, d têm produto igual a 8! e satisfazem

ab + a + b = 524
bc + b + c = 146
cd + c + d = 104

Quanto vale a − d?

Problema 22. (EUA) Quantas triplas ordenadas (x, y, z) de inteiros satisfazem o sistema
de equações abaixo?
 2
 x − 3xy + 2y 2 − z 2 = 31
−x 2 + 6yz + 2z 2 = 44
 2
x + xy + 8z 2 = 100

Problema 23. (Iberoamericana) Ache todas a triplas de números reais (x, y, z) tais que

 x + y − z = −1
x 2 − y2 + z2 = 1
 3
−x + y 3 + z 3 = −1
Problema 24. (Romênia) Os números reais não nulos x, y, z, t verificam as seguintes equações

x + y + z = t
1 1 1 1
+ y + z =
 x3 t
x + y 3 + z 3 = 10003

Determine o valor da soma x + y + z + t.

Problema 25. (OCM) Determine a + b + c + d, se



6a + 2b = 3840
6c + 3d = 4410 .
a + 3b + 2d = 3080

Problema 26. (OBM/IME) Sejam a, b, c e k números reais diferentes de zero satisfazendo


a b c
as relações k = b+c = c+a = a+b . Qual é o número de possı́veis valores que k pode assumir?

Problema 27. (OBM) Determine o número de soluções inteiras e positivas do sistema

= c2

a + b
a + b + c = 30

Problema 28. (OBM) As letras O, B, M representam números inteiros. Se O × B × M =


240, O × B + M = 46 e O + B × M = 64, quanto vale O + B + M ?

6
POT 2012 - Álgebra - Nı́vel 2 - Aula 2 - Prof. Marcelo Mendes

Problema 29. (OBM) Sejam a, b, c números reais positivos tais que a(b+c) = 152, b(c+a) =
162 e c(a + b) = 170. Determine o valor de abc.

Problema 30. (OBM) Quantos pares ordenados (x, y) de números reais satisfazem a equação
2
x − y 2 + (x − y − 2)2 = 0.

Problema 31. (OBM) Os inteiros 0 < x < y < z < w < t são tais que w = z(x + y) e
t = w(y + z). Sendo w = 9, determine o valor de t.
1 1
Problema 32. (EUA) Se x e y são números reais não-nulos tais que x = 1 + e y = 1+ ,
y x
então y é igual a:

a) x − 1

b) 1 − x

c) 1 + x

d) −x

e) x

7
POT 2012 - Álgebra - Nı́vel 2 - Aula 2 - Prof. Marcelo Mendes

Dicas
4. Fatore o lado esquerdo da equação. Comece escrevendo a soma dos 3 primeiros termos
como o quadrado da soma de dois termos.

5. Observe que o problema não pede todas as soluções dessa equação. Assim, fatore o
lado esquerdo e faça x + y = a2 e x − y = a.

6. Use a definição: se x ∈ Q, então existem p, q ∈ Z, q 6= 0 tais que x = pq . Se for


necessário, acrescente que x e y são primos entre si. Com essa última observação, as
paridades de p e q só não podem ser ambas pares. Utilize o fato de que 0 é par para
chegar a contradições em todos os casos.

7. Passe o primeiro fator para o lado direito e racionalize (ou então, racionalize mesmo
na equação inicial). Depois, faça o mesmo com o segundo fator.

8. Para ’equilibrar’ a equação, passe 1c para o lado direito. Em seguida, reduza a um


denominador comum em cada lado. Analise, em seguida, as possibilidades de os
números serem ou não iguais a 0.

9. Use a definição: se r é raiz da equação em x, então substituindo x pelo valor r


a equação fica verdadeira. Depois, escreva 4 como produto de 4 números inteiros
distintos.

16. Combine as equações 1 e 3.

18. Some todas as equações, que nos dará a soma de todas os xi s. Depois, subtraia cada
uma desse resultado.

19. Some todas as equações.

20. Subtraia as equações aos pares.

21. Some 1 a cada membro de cada equação e use a fatoração xy+x+y+1 = (x+1)(y+1).

22. Some todas as equações e perceba soma de quadrados.

23. Subtraia as equações aos pares.

24. Veja o problema 8.

26. Escreva a = k(b + c), b = k(c + a), c = k(a + b) e some todas as equações em seguida.

28. Multiplique a segunda equação por M e a terceira por O.

29. Some todas as equações.

30. Se a soma dos quadrados de dois números reais é 0, então os dois números são iguais
a 0.

8
POT 2012 - Álgebra - Nı́vel 2 - Aula 2 - Prof. Marcelo Mendes

Respostas
4. (0, 36), (1, 16), (4, 4), (9, 0), (0, 64), (1, 36), (4, 16), (9, 4), (16, 0)
8. a = −b ou b = −c ou c = −a
9. 1
10. 3
19 2 1

16. (2, −1, 1), 12 , 3 , − 4
17. S = {983, 984, 989, 991, 1000}
18. (x1 , x2 , x3 , x4 , x5 ) = (−25, −19, −7, 17, 65)
21. 10
22. 0
23. (−1, −1, −1), (1, −1, 1)
24. 2000
25. 1985
1

26. 2 2 ; −1
27. 24
28. 20
29. 720
30. 2
31. 45
32. e

9
Polos Olímpicos de Treinamento
Curso de Álgebra - Nível 2 Aula 3
Prof. Marcelo Mendes

Sequências

Uma sequência nada mais é do que um conjunto de números ordenados. Assim, pode-
mos estabelecer um primeiro termo (a1 ), um segundo termo (a2 ), ... e o termo geral de
uma sequência é escrito na forma an . Os problemas costumam informar qual é o valor de
alguns termos e uma lei de formação para os demais termos. Se necessário, faremos uso de
termos, que na sequência, são anteriores aos termos dados ou posteriores (que será mais
raro).

Algumas vezes, essa lei de formação será implı́cita, ou seja, não poderemos calcular os
termos diretamente a partir da posição que eles ocupam na sequência. Por exemplo, se
cada termo é a soma dos dois termos imediatamente anteriores e os primeiro e segundo
termos são iguais a 1. Possivelmente, precisaremos de uma lei explı́cita, que calcula um
termo da sequência apenas a partir da posição que ele ocupa.

No parágrafo anterior, a sequência em questão é a famosa Sequência de Fibonacci. Na


próxima aula, vamos aprender como encontrar seu termo geral.

1 Sequências simples
Problema 1. Mostre que a sequência definida por an = n2 + n + 2 para n ≥ 1, então na
sequência a1 , a2 , a3 , ... contém an quadrado perfeito, mas apenas em quantidade finita.

Solução. Inicialmente, veja que a1 = 4, que é quadrado perfeito. Mas para n > 1, ocorre

n2 < n2 + n + 2 < n2 + 2n + 1,
ou seja, an está situado entre 2 quadrados perfeitos consecutivos e, portanto, não pode ser
um quadrado.

Problema 2. Uma sequência {an } é definida por a1 = 2 e, para n ≥ 2, an é o maior divisor


primo de a1 · a2 · ... · an−1 + 1. Mostre que an nunca é igual a 5.
POT 2012 - Álgebra - Nı́vel 2 - Aula 3 - Prof. Marcelo Mendes

Solução. O máximo divisor primo de a1 +1 = 3 é a2 = 3. Logo, se n > 2, a1 ·a2 ·...·an−1 +1


não possui fatores 2 nem 3, ou seja, se an = 5, então a1 · a2 · ... · an−1 + 1 = 5k ou
a1 · a2 · ... · an−1 = 5k − 1, que é múltiplo de 4, uma contradição pois o único fator par do
membro esquerdo dessa última equação é a1 = 2.

Problema 3. (OBM) Considere a sequência oscilante:

1, 2, 3, 4, 5, 4, 3, 2, 1, 2, 3, 4, 5, 4, 3, 2, 1, 2, 3, 4, ...

Determine o 2003o termo desta sequência.

Solução. Uma parte da sequência, com 8 algarismos, se repete: 1, 2, 3, 4, 5, 4, 3, 2. Di-


vidindo 2003 por 8, obtemos 3 como resto, e deste modo, o 2003o termo corresponde ao
terceiro elemento da parte da sequência que se repete, isto é, 3.

Problema 4. (OBM-Adaptado) A sequência de algarismos 1, 2, 3, 4, 0, 9, 6, 9, 4, 8, 7, ... é


construı́da da seguinte maneira: cada elemento, a partir do quinto, é igual ao último
algarismo da soma dos quatro anteriores. Os algarismos 2, 0, 0, 4, juntos e nesta ordem,
aparecem na sequência?

Problema 5. Calcule a soma 1 − 2 + 3 − 4 + . . . − 98 + 99 − 100.

2 Somas Telescópicas
Vamos entender o que é uma soma telescópica através do nosso primeiro exemplo.

2F (n) + 1
Problema 6. (EUA) Se F (n + 1) = para n = 1, 2, ..., e F (1) = 2, então deter-
2
mine o valor de F (101).

Solução. Podemos reescrever a equação que define os termos dessa sequência recursiva-
mente (isto é, em função de termos anteriores) da seguinte forma:
1
F (n + 1) − F (n) = .
2
Assim, podemos escrever essas equações variando n de 100 a 1:
1
F (101) − F (100) =
2
1
F (100) − F (99) =
2
:
1
F (3) − F (2) =
2

2
POT 2012 - Álgebra - Nı́vel 2 - Aula 3 - Prof. Marcelo Mendes

1
F (2) − F (1) =
2
Somando telescopicamente todas essas equações, obtemos F (101) − F (1) = 50, ou seja,
F (101) = 52 pois F (1) = 2.

Sequências como essa que acabamos de ver em que a diferença entre os valores dos
termos consecutivos é constante são chamadas de Progressão Aritmética (P.A.).
Acho que deu pra entender o que é uma soma telescópica: são somas em que os termos
intermediários são cancelados e, no final, só restam o primeiro e o último.

Pode até mesmo ser interessante escrever coisas do tipo

1 − n = (1 − 2) + (2 − 3) + ... + [(n − 1) − n] .
Vejamos agora mais um exemplo.

Problema 7. Encontre o valor da soma


1 1 1 1
S= + + + ... + .
1×2 2×3 3×4 999 × 1000
Solução. Essa é uma aplicação clássica para somas telescópicas. Observe que os denomi-
1
nadores são produtos de números consecutivos. Com o auxı́lio da identidade =
k × (k + 1)
1 1
− , concluı́mos que
k k+1
1 1 1 1 1 1 1 1 1 999
S= − + − + − + ... + − ⇒S =1− = .
1 2 2 3 3 4 999 1000 1000 1000
1 1 1 1
Problema 8. (EUA) Encontre a soma + + + ... + .
1×3 3×5 5×7 255 × 257

1 1 1 1
Problema 9. (OBM) Encontre a soma + + + ... + .
1 × 4 4 × 7 7 × 10 2998 × 3001

Problema 10. (Hungria) Prove que para todos os inteiros positivos n,


1 1 1 1 1 1
+ + ... + = + + ... + .
1·2 3·4 (2n − 1) · 2n n+1 n+2 2n

Solução. Veja
1 1 1 1 1 1 1 1
+ + ... + = 1 − + − + ... + −
1·2 3·4 (2n − 1) · 2n 2 3 4 2n − 1 2n
 
1 1 1 1 1 1 1 1
= 1 + + + + ... + + −2 + + ... +
2 3 4 2n − 1 2n 2 4 2n

3
POT 2012 - Álgebra - Nı́vel 2 - Aula 3 - Prof. Marcelo Mendes

 
1 1 1 1 1 1 1
=1+ + + + ... + + − 1 + + ... +
2 3 4 2n − 1 2n 2 n
1 1 1
= + + ... + .
n+1 n+2 2n
Observe que, apesar de muito semelhante aos problemas anteriores, este não utiliza
soma telescópica.

Problema 11. O pagamento de um certo pintor aumenta de acordo com o dias em que
ele trabalha. No primeiro dia ele recebeu 1 real. no segundo dia ele recebeu o que ti-
nha ganho no primeiro dia mais 2 reais. No terceiro dia ele recebeu o que tinha recebido
no segundo dia mais 3 reais. Desse modo, quanto o marceneiro irá receber no centésimo dia?

Solução. Seja Ln o valor pago no n-ésimo dia. O problema no diz que Ln+1 = Ln + (n + 1).
Vamos escrever várias equações seguidas:
Ln+1 = Ln + (n + 1)
Ln = Ln−1 + n
Ln−1 = Ln−2 + (n − 1)
...
L2 = L1 + 2

Somando tudo, obtemos um cancelamento de vários termos (soma telescópica), so-


brando:
(n + 1)(n + 2)
Ln+1 = (n + 1) + n + (n − 1) + . . . + 2 + 1 = .
2
1 1 1
Problema 12. Prove que S = √ √ +√ √ +. . .+ √ √ é um número inteiro.
1+ 2 2+ 3 99 + 100
Solução. A dica é racionalização dos denominadores:
√ √ √
1 1 1− 2 √ 
√ √ =√ √ ·√ √ =− 1− 2 .
1+ 2 1+ 2 1− 2
Repetindo o procedimento para as demais parcelas, chegamos a:
√ √ √ √ √ √ √ √
−S = 1 − 2 + 2 − 3 + ... + 99 − 100 = 1 − 100 = −99
⇔ S = 99,
que é um número inteiro.

Problema 13. Determine o valor da expressão


1√
 √ + √ 1 √ + √ 1 √ +...+ √ 1√
2002 2002 2002 2002 1+ 2 2+ 3 3+ 4 99+ 100
E= + + + ... + .
2·6 6 · 10 10 · 14 1998 · 2002

4
POT 2012 - Álgebra - Nı́vel 2 - Aula 3 - Prof. Marcelo Mendes

Problema 14. (EUA) A Sequência de Fibonacci 1, 1, 2, 3, 5, 8, 13, 21, ... começa com dois
1s e cada termo seguinte é a soma de seus dois antecessores. Qual dos dez dı́gitos (do sis-
tema de numeração decimal) é o último a aparecer na posição das unidades na seqüência
de Fibonacci?

Problema 15. (OBM) Determine o máximo divisor comum de todos os termos da sequência
cujos termos são definidos por an = n3 − n.

Problema 16. (EUA) Considere uma sequência un definida por u1 = 5 e a relação

un+1 − un = 3 + 4(n − 1), n = 1, 2, 3, ...

Se un é expresso como um polinômio em n, determine a soma algébrica de seus coeficientes.

Solução. Podemos escrever

un − un−1 = 3 + 4(n − 2)
un−1 − un−2 = 3 + 4(n − 3)
:
u2 − u1 = 3 + 4 · 1
Somando todas essas equações, obtemos

un − u1 = 3(n − 1) + 4 (1 + 2 + ... + (n − 2)) = 3(n − 1) + 2(n − 1)(n − 2)

⇒ un = 2n2 − 3n + 6,
cuja soma dos coeficientes é 5.

Problema 17. (Estônia) Prove a desigualdade

22 + 1 32 + 1 20102 + 1 1
2010 < 2
+ 2
+ ... + 2
< 2010 .
2 −1 3 −1 2010 − 1 2

n
X 1
Problema 18. Calcule a soma √ √ .
k=1
(k + 1) k + k k + 1

5
POT 2012 - Álgebra - Nı́vel 2 - Aula 3 - Prof. Marcelo Mendes

an
Problema 19. Considere a seqüência definida por a1 = 1 e an+1 = 1+n·an . Calcule a2012 .

Solução. Começaremos com um artifı́cio algébrico bastante útil que é observar que, na
fórmula de an+1 , a fração do membro direito pode ser melhor desenvolvida se for invertida,
porque poderemos desmembrar o resultado. De fato, temos
1 1 + n · an 1
= = +n
an+1 an an
1 1
⇔ − = n.
an+1 an
Assim, obtemos uma chamada equação de diferença. Variando o valor de n de forma
decrescente de 2010 a 1, chegaremos a
1 1
a2011 − a2010 = 2010
1 1
a2011 − a2010 = 2009
.. ..
. = .
1 1
a3 − a2 = 2
1 1
a2 − a1 = 1

Somando essas 2010 equações membro a membro, obtemos


1 1 2010 · 2011
− = 1 + 2 + ... + 2009 + 2010 = = 2021055
a2011 a1 2

1
⇔ = 2021056.
a2011
1
Portanto, a2011 = 2021056 .

6
POT 2012 - Álgebra - Nı́vel 2 - Aula 3 - Prof. Marcelo Mendes

3 Produtos Telescópicos
A ideia é semelhante a das somas telescópicas, mas o cancelamento ocorre pelo produto
e não por soma.

Problema 20. No ano 1 Papai Noel viajou sozinho para entregar seus presentes na noite
de Natal. No ano seguinte, ele percebeu que precisava de um ajundante e contratou um
Matesito (tı́pico habitante do Pólo Norte). A cada ano, ele sempre precisava dobrar a quan-
tidade de Matesitos e contratava mais Matesitos para guiar as renas. Quantos Matesitos
Papai Noel vai precisar contratar no ano de 2012?

Solução. Seja Ln o número de Matesitos em cada ano. O problema no diz que Ln+1 =
2Ln + 1. Somando 1 aos dois lados obtemos Ln+1 + 1 = 2(Ln + 1). Vamos escrever várias
equações seguidas:

Ln+1 + 1 = 2(Ln + 1)
Ln + 1 = 2(Ln−1 + 1)
Ln−1 + 1 = 2(Ln−2 + 1)
...
L2 + 1 = 2(L1 + 1)

Multiplicando tudo, obtemos um cancelamento de vários termos (produto telescópico),


sobrando:
2 . . . × 2} = 2n+1 ⇒ Ln+1 = 2n+1 − 1.
Ln+1 + 1 = |2 × 2 ×{z
n+1vezes

Em particular, L2012 = 22012 − 1.

Problema 21. Uma sequência é definida por a1 = 2 e an = 3an−1 + 1. Determine a soma


a1 + a2 + . . . + an .

Problema 22. Considere 2


p a sequência recorrente definida por a1 = 14 e an+1 = an − 2.
2
Prove que o número 3 (an − 4) é divisı́vel por 4, ∀n ∈ Z, n ≥ 1.
q 
Solução. Primeiro, veja que 3 a21 − 4 = 24. Observe que

an+1 − 2 = a2n − 4 = (an + 2)(an − 2).

Reduzindo os ı́ndices, obtemos também

an − 2 = (an−1 + 2)(an−1 − 2)
:
a3 − 2 = (a2 + 2)(a2 − 2)
a2 − 2 = (a1 + 2)(a1 − 2)

7
POT 2012 - Álgebra - Nı́vel 2 - Aula 3 - Prof. Marcelo Mendes

Multiplicando todas essas equações telescopicamente, obtemos

an+1 − 2 = (an + 2)(an−1 + 2)...(a1 + 2)(a1 − 2)


⇔ a2n − 4 = a2n−1 · a2n−2 · ... · 16 · 12

⇔ 3 a2n − 4 = a2n−1 · a2n−2 · ... · 16 · 36.
p
⇔ 3 (a2n − 4) = an−1 · an−2 · ... · 4 · 6,
que é múltiplo de 4.

Problema 23. Sejam r1 = 3 e rn = rn−1 2 − 2, ∀n ≥ 2. Se sn = rn − 2 para n ≥ 1, prove que


sj tem, no mı́nimo, 2 · 3j−2 divisores positivos, j ≥ 2.

Problema 24. (EUA) Defina uma sequência de números reais a1 , a2 , a3 , ... por a1 = 1 e
a3n+1 = 99a3n , ∀n ≥ 1. Determine o valor de a100 .

    
104 + 324 224 + 324 344 + 324 464 + 324 584 + 324
Problema 25. Calcule o valor de .
(44 + 324) (164 + 324) (284 + 324) (404 + 324) (524 + 324)
8 12 16 4n + 4 2008
Problema 26. Qual é o valor do produto · · · ... · · ... · ?
4 8 12 4n 2004

8
POT 2012 - Álgebra - Nı́vel 2 - Aula 3 - Prof. Marcelo Mendes

Dicas

5. Agrupe os números aos pares.


 
1 1 1 1
8. Use = − .
k · (k + 2) 2 k k+2
9. Pense numa ideia semelhante à sugestão do problema 8.

13. Use mais uma vez uma ideia parecida com a do problema 8 e veja o problema 12.

14. Calcule os primeiros termos até chegar à resposta.


n2 + 1 1 1
17. Use =1+ − .
(n − 1)(n + 1) n−1 n+1
√ √
18. Fatore o denominador pondo k k + 1 em evidência.
√ Depois,
√ racionalize o denomi-
nador multiplicando numerador e denominador por k + 1 − k e surgirá uma soma
telescópica.

21. Subtraindo as equações an = 3an−1 + 1 e an−1 = 3an−2 + 1, obtemos an − an−1 =


3 (an−1 − an−2 ). Depois, multiplique várias dessas equações seguidas (produto te-
lescópico).

23. Veja problema 21.

24. Multiplique várias dessas equações seguidas (produto telescópico).


2  
25. Use a4 +182 = a4 +2a2 ·18+182 −36a2 = a2 + 18 −(6a)2 = a2 + 6a + 18 a2 − 6a + 18 .

9
POT 2012 - Álgebra - Nı́vel 2 - Aula 3 - Prof. Marcelo Mendes

Respostas

4. Não

5. −50
128
8. 257
1000
9. 3001

13. 2002 99

14. 6

15. 6

n+1−1
18. √
n+1

5·3n −2n−5
21. 4

24. 9933

25. 373

26. 502

10
Polos Olímpicos de Treinamento
Curso de Álgebra - Nível 2 Aula 4
Prof. Marcelo Mendes

Recorrências - Parte I

Na aula anterior, vimos alguns exemplos de sequências. Em alguns deles, os termos são
dados em função de termos anteriores, ou seja, eles recorrem a valores de termos anteriores.
Por isso, essas sequências são chamadas de recorrências.

Talvez os exemplos mais clássicos de sequências recorrentes sejam as progressões aritmética


e geométrica, que veremos neste texto.

1 Progressões Aritméticas
O problema 6 da aula anterior é um exemplo de P.A. Por definição, uma P.A. é uma
sequência em que a diferença entre os termos consecutivos é constante. Daı́, se (a, b, c) é
uma P.A., então b − a = c − b, ou então, 2b = a + c, isto é, b = a+c
2 , ou seja, cada termo de
uma P.A. é a média aritmética dos termos adjacentes. Essa propriedade, portanto, justifica
o nome desse tipo de sequência.

Sendo d o valor da diferença constante (tradicionalmente chamada de razão), temos a


seguinte lei de formação para os termos de uma P.A. {an }

an = an−1 + d.
Mas veja que essa é uma fórmula implı́cita, recorrente, que necessita de valores anteriores
para se achar o valor de um determinado termo. Somando telescopicamente várias dessas
equações

an = an−1 + d
an−1 = an−2 + d
:
a3 = a2 + d
a2 = a1 + d
POT 2012 - Álgebra - Nı́vel 2 - Aula 4 - Prof. Marcelo Mendes

chegamos a
an = a1 + (n − 1)d,
que é a fórmula clássica para o termo geral de uma P.A. Todavia, pode ser mais interessante
em determinados problemas a fórmula

an = am + (n − m)d ⇔ an − am = (n − m)d,
que, ao invés de depender do valor do termo a1 , calcula an a partir de qualquer outro termo
am , podendo este, inclusive, ser posterior.

Essa fórmula nos permite concluir que a1 + an = a2 + an−1 = a3 + an−2 = .... Daı́,
somando as duas equações a seguir

S = a1 + a2 + ... + an−1 + an
S = an + an−1 + ... + a2 + a1
chegamos a

(a1 + an ) n
S= .
2
Problema 1. (EUA) Os quatro primeiros termos de uma P.A. são a, x, b, 2x. Determine o
a
valor da razão .
b
a+b 2b a 1
Solução. Temos 2x = a + b e 2b = x + 2x. Assim, = e, portanto, = .
2 3 b 3

Problema 2. (IME) Determine a relação que deve existir entre os números m, n, p, q para
que se verifique a seguinte igualdade entre os termos de uma mesma progressão aritmética
não-constante:
am + an = ap + aq .

Problema 3. Encontre o valor de a2 + a4 + a6 + ... + a98 se a1 , a2 , a3 , ... é uma P.A. de razão


1 e a1 + a2 + a3 + ... + a98 = 137.

Solução. Podemos escrever a1 +a2 +...+a97 +a98 = 137 como (a2 − 1)+a2 +...+(a98 − 1)+
a98 = 137. Daı́, 2 (a2 + a4 + a6 + ... + a98 ) − 49 = 137 e, portanto, a2 + a4 + a6 + ... + a98 =
137 + 49
= 93.
2

Problema 4. (EUA) Seja a1 , a2 , ..., ak uma progressão aritmética finita com a4 + a7 + a10 =
17 e a4 + a5 + a6 + ... + a12 + a13 + a14 = 77. Se ak = 13, determine o valor de k.

Problema 5. Calcule a soma dos 1000 primeiros múltiplos positivos de 7.

2
POT 2012 - Álgebra - Nı́vel 2 - Aula 4 - Prof. Marcelo Mendes

Problema 6. Um jardineiro tem que regar 60 roseiras plantadas ao longo de uma vereda
retilı́nea e distando 1m uma da outra. Ele enche seu regador, a 15m da primeira roseira, e,
a cada viagem, rega 3 roseiras. Começando e terminando na fonte, qual é o percurso total
que ele terá que caminhar até regar todas as roseiras?

Problema 7. Observe a disposição, abaixo, da seqüência dos números naturais ı́mpares.

1a linha 1
2a linha 3, 5
3a linha 7, 9, 11
4a linha 13, 15, 17, 19
5a linha 21, 23, 25, 27, 29
: : :

Determine o quarto termo da vigésima linha.

Problema 8. (Espanha) Encontre uma P.A. tal que a soma de seus n primeiros termos seja
igual a n2 para qualquer valor de n.

Solução. Veja que


Sn = a1 + a2 + ... + an = n2 .
Com n = 1, obtemos S1 = a1 = 1 e, com n = 2, S2 = a1 + a2 = 4. Logo, a2 = 3. Assim,
a razão da P.A. é a2 − a1 = 3 − 1 = 2. Portanto, a P.A. procurada é 1, 3, 5, 7, ...

Problema 9. (IME) O quadrado de qualquer número par 2n pode ser expresso como a soma
de n termos, em progressão aritmética. Determine o primeiro termo e a razão da progressão.

Problema 10. (ITA) Provar que se uma P.A. é tal que a soma dos seus n primeiros termos
é igual a n + 1 vezes a metade do n-ésimo termo, então r = a1 .

Solução. Pelo enunciado, temos

an (a1 + an ) n an
Sn = (n + 1) ⇔ = (n + 1) ⇔ a1 · n = an
2 2 2

⇔ a1 · n = a1 + (n − 1)r ⇔ a1 (n − 1) = (n − 1)r, ∀n.


Portanto, a1 = r.
Sm m2
Problema 11. Numa P.A., tem-se = 2 , sendo Sm e Sn as somas dos m primeiros
Sn n
termos e dos primeiros n termos, respectivamente, com m 6= n. Prove que a razão da P.A.
é o dobro do primeiro termo.

3
POT 2012 - Álgebra - Nı́vel 2 - Aula 4 - Prof. Marcelo Mendes

Problema 12. Se numa P.A. a soma dos m primeiros termos é igual à soma dos n primeiros
termos, m 6= n, mostre que a soma dos m + n primeiros termos é igual a zero.

√ √ √
Problema 13. (OCM) Mostre que 2, 3, 5 não podem ser termos de uma mesma pro-
gressão aritmética.

Problema 14. Cada uma das progressões aritméticas a seguir tem 80 termos: (an ) =
(9, 13, ...) e (bn ) = (10, 13, ...). Quantos números são, ao mesmo tempo, termos das duas
progressões?

Problema 15. Numa P.A., temos ap = q e aq = p, com p 6= q. Determine a1 e ap+q .

Problema 16. (EUA) Se a soma dos 10 primeiros termos e a soma dos 100 primeiros ter-
mos de uma progressão aritmética são 100 e 10, respectivamente, determine a soma dos
110 primeiros termos.

Solução. Vamos escrever os dados do problema da seguinte forma

(a1 + ... + a10 ) + (a11 + ... + a20 ) + ... + (a91 + ... + a100 ) = 10
(a1 + ... + a10 ) + (a1 + ... + a10 ) + ... + (a1 + ... + a10 ) = 100 · 10
Subtraindo termo a termo, obtemos

0 · 10 + 10r · 10 + ... + 90r · 10 = −900

1
⇒ 100r(1 + ... + 9) = −900 ⇒ r = − .
5
Portanto

a1 + ... + a110 = (a1 + ... + a100 ) + (a101 + ... + a110 )

= 10 + [(a1 + 100r) + ... + (a10 + 100r)]

= 10 + (a1 + ... + a10 ) + 1000r = 10 + 100 − 200 = −90.

Problema 17. (EUA) Em uma P.A., a soma dos 50 primeiros termos é 200 e a soma dos
50 próximos é 2700. Determine a razão e o primeiro termo dessa seqüência.

4
POT 2012 - Álgebra - Nı́vel 2 - Aula 4 - Prof. Marcelo Mendes

Problema 18. (EUA) A soma dos n primeiros termos de uma P.A. é 153 e a razão é 2. Se
o primeiro termo é um inteiro e n > 1, determine o número de valores possı́veis de n.

(a1 + an ) n
Solução. Como = 153, temos [a1 + (n − 1)] n = 153. Como a1 + (n − 1) e n
2
são inteiros positivos, eles são divisores positivos de 153. Mas 153 = 32 × 17 e, portanto,
153 possui 6 divisores positivos, sendo 5 deles maiores que 1.

Problema 19. (EUA) A soma dos n primeiros termos de uma P.A. é x e a soma dos n
seguintes é y. Calcular a razão.

Problema 20. A sequência 1, 2, 1, 2, 2, 1, 2, 2, 2, 1, 2, 2, 2, 2, 1, 2, ... consiste de 1s separados


por blocos de 2s, com n 2s no n-ésimo bloco. Determine a soma dos 1234 primeiros termos
dessa seqüência.

2006
Problema 21. Mostre que 20082007 é um termo da P.A. infinita (6, 13, 20, 27, ...).

Problema 22. (EUA) Os três primeiros termos de uma progressão aritmética são 2x −
3, 5x − 11 e 3x + 1, respectivamente. O n-ésimo termo da sequência é 2009. Quel é o valor
de n?

Problema 23. (EUA) Os quatro primeiros termos de uma progressão aritmética são p, 9, 3p−
q e 3p + q. Qual é o 2010o termo dessa sequência?

2 Progressão Geométrica
Semelhante ao que escrevemos para P.A., por definição, uma P.G. é uma sequência em
que cada novo termo, a partir do segundo, é o produto do termo anterior por uma cons-
tante. Daı́, se (a, b, c) é uma P.G., então b2 = ac.

Sendo q o valor da razão constante, temos a seguinte lei de formação para os termos de
uma P.G. {an }

an = an−1 · q.
Mas veja que essa também é uma fórmula implı́cita, recorrente, que necessita de valores
anteriores para se achar o valor de um determinado termo. Multiplicando telescopicamente
várias dessas equações

5
POT 2012 - Álgebra - Nı́vel 2 - Aula 4 - Prof. Marcelo Mendes

an = an−1 · q
an−1 = an−2 · q
:
a3 = a2 · q
a2 = a1 · q
chegamos a
an = a1 · q n−1 ,
que é a fórmula clássica para o termo geral de uma P.G.

A fórmula da soma dos n primeiros termos é

qn − 1
S n = a1 · ,
q−1
se q 6= 1 e Sn = a1 · n, se q = 1, e a fórmula do produto dos n primeiros termos pode ser
apresentada de 2 maneiras
n(n−1)
Pn = an1 · q 2

ou

Pn2 = (a1 · an )n .

Problema 24. (EUA) Suponha que x, y, z estejam em P.G. de razão r e x 6= y. Se x, 2y, 3z


estão em P.A., determine o valor de r.

Solução. Temos y = x · r e z = x · r 2 pela P.G. Pela P.A., segue que 4y = x + 3z. Logo,
4xq = x + 3xq 2 . Se x = 0, então y = 0 = x, o que não pode ocorrer. Daı́, 3q 2 − 4q + 1 = 0,
cujas soluções são q = 1 e q = 13 . Como q = 1 implica x = y, concluı́mos que q = 31 .

Problema 25. Se (a, b, c) formam, nesta ordem, uma P.A. e uma P.G. simultaneamente,
mostre que a = b = c.
 2
a+c a+c
Solução. Por ser P.A., temos b = (*) e, por ser P.G., b2 = ac. Logo, = ac,
2 2
ou seja, (a − c)2 = 0. Assim, a = c e, por (*), a = b = c.

Problema 26. (OCM) Determine a soma dos n primeiros termos da sequência:


 
1, (1 + 2), 1 + 2 + 22 , 1 + 2 + 22 + 23 , ..., 1 + 2 + 22 + 23 + ... + 2k−1 .
 

6
POT 2012 - Álgebra - Nı́vel 2 - Aula 4 - Prof. Marcelo Mendes

Problema 27. 6. Mostre que não existe P.G. de três termos distintos tal que, ao somarmos
um mesmo número real não-nulo a todos os seus termos, a nova sequência seja também
uma P.G.

Problema 28. (EUA) Numa P.G. de 2n termos, a soma dos termos de ordem par é P e a
soma dos termos de ordem ı́mpar é I. Calcule o 1o termo e a razão.

Solução. De a2 + ... + a2n = P , segue que q · (a1 + ... + a2n−1 ) = P ou q · I = P . Logo,


n
P q2 − 1 q 2n+1 − q (P − I)I 2n
q = . Além disso, P = a2 · = a1 · . Logo, a1 = 2n .
I q−1 q−1 P − I 2n

Problema 29. Prove que, quando os lados de um triângulo estão em P.G., o mesmo ocorre
para as alturas.

a a 2 + b2
Problema 30. Sejam a, b, c números reais não-nulos, com a 6= c, tais que = 2 .
c c + b2
Prove que a, b e c formam uma P.G.

Problema 31. (EUA) O 5o e o 8o termos de uma progressão geométrica de números reais


são 7! e 8!, respectivamente. Qual é o 1o termo?

2 Recorrências Lineares de Ordem 2 - Parte I


Por fim, vamos estudar apenas as recorrências em que a equação caracterı́stica possui
raiz real dupla. Mas o que é uma equação caracterı́stica? Vejamos.

Considere a recorrência linear de ordem 2 (isto é, só depende dos 2 termos imediata-
mente anteriores)

an = pan−1 + qan−2 .
A equação caracterı́stica dessa recorrência é a equação quadrática formada repetindo
os mesmos coeficientes da recorrência, ou seja,

x2 = px + q ⇔ x2 − px − q = 0.
Mas como surge essa equação? A resposta será dada no texto da aula seguinte. Por
enquanto, acredite.

Como exemplo, considere uma recorrência definida por a1 = 1, a2 = 3 e, para n ≥ 3,


an = 2an−1 − an−2 . A equação caracterı́stica associada é x2 − 2x + 1 = 0, que possui duas
raı́zes iguais a 1. Entrementes, uma olhadinha mais cuidadosa mostra que a recorrência
em questão é de uma P.A. pois

7
POT 2012 - Álgebra - Nı́vel 2 - Aula 4 - Prof. Marcelo Mendes

an = 2an−1 − an−2 ⇔ an − an−1 = an−1 − an−2 .


Portanto, acabamos de ver que uma P.A. está associada a uma equação caracterı́stica
com raiz dupla 1.

Agora, vejamos outro exemplo, uma recorrência em que a1 = 6, a2 = 27 e, para n ≥ 3,


an = 6an−1 − 9an−2 (*). A equação caracterı́stica associada é x2 − 6x + 9 = 0, cujas
raı́zes são iguais a 3. A saı́da agora é criar uma nova sequência {bn } dada por an = 3n bn .
Substituindo em (*), chegamos a bn = 2bn−1 − bn−2 , o que mostra que {bn } é uma P.A.!
Assim, sendo bn = A + Bn (o termo geral de uma P.A. é uma função polinomial do 1o grau
em função de n ou uma função constante no caso em que a P.A. é constante), obtemos

an = 3n (A + Bn).
Para acharmos A e B, fazemos n assumir os valores 1 e 2:

6 = a1 = 3(A + B)
27 = a2 = 9(A + 2B)
cujas soluções são A = B = 1 e, portanto,

an = 3n (n + 1).

Problema 32. Resolva a recorrência a1 = 4, a2 = 20 e, para n ≥ 3, an = 4an−1 − 4an−2 .

Problema 33. Resolva a recorrência a1 = 8, a2 = 96 e, para n ≥ 3, an = 8an−1 − 16an−2 .

Problema 34. Considere a sequência (an ) dada por a1 = 1, a2 = 3 e an = 10an−1 − 25an−2 ,


para n > 2. Determine o valor de k, dado por an = kn bn tal que a sequência (bn ) seja uma
P.A.

Problema 35. (IME) Considere a sequência {vn }, n = 0, 1, 2, ... definida a partir de seus
dois primeiros termos v0 e v1 e pela fórmula geral vn = 6vn−1 −9vn−2 , para n ≥ 2. Define-se
uma nova sequência {un }, n = 0, 1, 2, ... pela fórmula vn = 3n un .
a) Calcule un − un−1 em função de u0 e u1 .
b) Calcule un e vn em função de n, v1 e v0 .
1
c) Identifique a natureza das sequências {vn } e {un } quando v1 = 1 e v0 = .
3

8
POT 2012 - Álgebra - Nı́vel 2 - Aula 4 - Prof. Marcelo Mendes

Dicas

2. Use ai − aj = (i − j)r, sendo r a razão.

9. Veja o problema 8.
√ √ √
13. Suponha, sem perda de generalidade que 2, 3, 5 sejam o primeiro, o m-ésimo e
o n-ésimo termos, respectivamente. Use a fórmula do termo geral √ em √am e an , isole a
razão
√ em cada uma e iguale essas expressões. Depois, utilize que 2, 3 e, em geral,
k, em que k é um número natural não quadrado perfeito, são números irracionais.

14. O primeiro termo em comum é 13 e a razão dos termos em comum é mmc(4, 3) = 12,
já que 3 e 4 são as razões iniciais.

15. Use ai − aj = (i − j)r, sendo r a razão.

17. Veja a solução do problema 16 ou use a fórmula da soma (que dará mais trabalho).

19. Veja a sugestão do problema 17.

21. Os termos da P.A. em questão são da forma 7k + 6 ou 7k − 1. Assim, basta achar o


2006
resto de 20082007 na divisão por 7.

26. Calcule cada uma das somas parciais separadas por vı́rgulas no enunciado e, em
seguida, calcule a soma dos resultados. Nas duas etapas, use a fórmula da soma da
P.G.
bh
29. Use que a área de um triângulo é .
2

9
POT 2012 - Álgebra - Nı́vel 2 - Aula 4 - Prof. Marcelo Mendes

Respostas

2. m + n = p + q

4. 18

5. 3503500

6. 1820

7. 387

9. a1 = 4 e r = 8

14. 20

15. a1 = q + p − 1, ap+q = 0

17. r = 1 e a1 = −20, 5
y−x
19.
n2
20. 2419

22. 502

23. 8041

26. 2n+1 − n − 2

31. 315

32. 2n (3n − 1)

33. 4n (4n − 2)

34. 5

35. a) u1 − u0 ; b) un = nv3 1 + (1 − n)v0 e vn = 3n−1 nv1 + 3n (1 − n)v0 ; c) un = 13 , sequência


constante e vn = 3n−1 , progressão geométrica

10
Polos Olímpicos de Treinamento
Curso de Álgebra - Nível 2 Aula 5
Prof. Marcelo Mendes

Recorrências - Parte II

Na aula 3, falamos de uma sequência famosa, a Sequência de Fibonacci, cuja definição é


a seguinte: F1 = F2 = 1 e, para n ≥ 3, Fn = Fn−1 + Fn−2 . Essa fórmula é uma recorrência
linear de ordem 2. Um de nossos objetivos neste 5o texto é mostrar que a fórmula explı́cita
para seus termos é
√ !n √ !n
1 1+ 5 1 1− 5
Fn = √ −√ .
5 2 5 2

Surpreendente, não é mesmo? Imaginar que, substituindo n por 1, 2, 3, √ 4, 5, 6, ... na


fórmula acima, acharemos exatamente os termos 1, 1, 2, 3, 5, 8, ..., e nenhum 5 sobra, é
realmente muito belo.

Em geral, nesta aula, trataremos equações de recorrência lineares que dependem so-
mente dos dois termos anteriores. Inicialmente, vamos estudar o caso em que as raı́zes da
equação caracterı́stica (que definiremos no texto) são distintas.

1 Um Exemplo para Organizar as Ideias


Vamos resolver a recorrência a1 = 1, a2 = 3 e, para n ≥ 3,

an = 3an−1 − 2an−2 .
Podemos escrever an − an−1 = 2 (an−1 − an−2 ) e, em seguida, multiplicar telescopica-
mente várias delas

an − an−1 = 2 (an−1 − an−2 )


an−1 − an−2 = 2 (an−2 − an−3 )
:
a3 − a2 = 2 (a2 − a1 )
POT 2012 - Álgebra - Nı́vel 2 - Aula 5 - Prof. Marcelo Mendes

obtendo an − an−1 = 2n−2 (a2 − a1 ) = 2n−1 .

Agora, somamos telescopicamente várias dessa última equação

an − an−1 = 2n−1
an−1 − an−2 = 2n−2
:
a2 − a1 = 2
e chegamos a an − a1 = 2 + ... + 2n−2 + 2n−1 , ou seja, an = 2n − 1.

Observe que, na primeira passagem, para transformar an = 3an−1 − 2an−2 em an −


an−1 = 2 (an−1 − an−2 ), ’pedimos emprestado’ an−1 para o membro esquerdo. Essa operação
gerou proporção entre os coeficientes dos termos dos dois membros (antes e depois da igual-
dade), permitiu colocar o fator de proporção 2 em evidência e a diferença que surgiu entre
parênteses no membro direito ficou com o mesmo padrão da diferença no membro esquerdo,
mas com ı́ndices reduzidos. Essa será nossa ideia para encontrar o termo geral da

2 Sequência de Fibonacci
Como já definimos anteriormente, seus termos são dados por F1 = F2 = 1 e, para
n ≥ 3, Fn = Fn−1 + Fn−2 . Na verdade, os cálculos ficam mais interessantes escrevendo
Fn+1 = Fn +Fn−1 . Seria difı́cil ’pedir emprestado’ uma quantidade inteira desta vez pois há
somente Fn no membro direito. Assim, vamos chamar de λ a quantidade que será passada
para o membro esquerdo, ou seja,

Fn+1 − λFn = (1 − λ)Fn + Fn−1 .


Para repetirmos a ideia bem sucedida do primeiro exemplo, o valor de λ deve cumprir
a relação de proporção

1 1−λ
= ,
−λ 1
ou seja,

λ2 − λ − 1 = 0,
a qual chamaremos de equação caracterı́stica da sequência de Fibonacci. Observe desde
já que os coeficientes dessa equação são os mesmos da recorrência que define a sequência.
Sendo λ1 e λ2 as raı́zes, aqui será mais relevante saber que λ1 + λ2 = 1 e λ1 · λ2 = −1 (mas
veja que ambas são reais e distintas) do que escrever seus valores pela fórmula de Baskara.

Agora, substituindo λ por λ1 , obtemos

Fn+1 − λ1 Fn = (1 − λ1 )Fn + Fn−1 ,

2
POT 2012 - Álgebra - Nı́vel 2 - Aula 5 - Prof. Marcelo Mendes

ou seja,
Fn+1 − λ1 Fn = λ2 (Fn − λ1 Fn−1 ) .
Assim, deixamos a equação pronta para escrevê-la várias vezes e fazer o produto te-
lescópico

Fn+1 − λ1 Fn = λ2 (Fn − λ1 Fn−1 )


Fn − λ1 Fn−1 = λ2 (Fn−1 − λ1 Fn−2 )
:
F3 − λ1 F2 = λ2 (F2 − λ1 F1 ) ,
cujo resultado será

Fn+1 − λ1 Fn = λ2n−1 (F2 − λ1 F1 ) = λ2n−1 (1 − λ1 ) = λn2 .


Analogamente, substituindo λ por λ2 , temos

Fn+1 − λ2 Fn = λn1 .
A diferença entre esses 2 últimos resultados gera

(λ1 − λ2 ) Fn = λn1 − λn2


e, portanto,

λn1 − λn2
Fn =
λ1 − λ2
lembrando que λ1 6= λ2 . Substituindo os valores de λ1 e λ2 , chegamos ao resultado desejado
√ !n √ !n
1 1+ 5 1 1− 5
Fn = √ −√ .
5 2 5 2

Mas há um pequeno problema. Esse método é bastante trabalhoso. A boa notı́cia é que
podemos deixá-lo como uma quase demonstração e realizar, na prática, os seguintes passos:

1o passo: Escreva a equação caracterı́stica.

Basta copiar os mesmos coeficientes da equação de recorrência. Em seguida, calcule as


raı́zes dessa equação.

2o passo: Escreva o termo geral da recorrência.

O termo geral é dado por Fn = Aλn1 + Bλn1 (essa fórmula pode ser encontrada refazendo
os cálculos para a recorrência mais geralmente, ou seja, com a equação xn = axn−1 +bxn−2 ).

3
POT 2012 - Álgebra - Nı́vel 2 - Aula 5 - Prof. Marcelo Mendes

As constantes A e B são dadas pelos valores dos termos iniciais. É interessante, para re-
duzir as contas, calcular o termo de ordem ’0’, que, no caso da sequência de Fibonacci, é
F0 = 0.

Vejamos como seria, então, a resolução na prática para encontrar o termo geral da
sequência de Fibonacci.

Passo 1. Equação caracterı́stica.


√ √
1+ 5 1− 5
De Fn −Fn−1 −Fn−2 = 0, obtemos λ2 −λ−1 = 0, cujas raı́zes são λ1 = 2 e λ2 = 2 .

Passo 2. Termos geral.

Fn = Aλn1 + Bλn1 . Com os valores 0 e 1 para n, obtemos

0=A+B
1 = Aλ1 + Bλ2
cuja solução é A = −B = √1 .
5

Portanto,
√ !n √ !n
1 1+ 5 1 1− 5
Fn = √ −√ .
5 2 5 2

Problema 1. Um garoto tem n reais. Todo dia, ele realiza exatamente uma das seguintes
compras: um bolo que custa R$ 1, 00, um sorvete que custa R$ 2, 00 ou um pastel que
também custa R$ 2, 00. De quantas maneiras o menino pode gastar seu dinheiro?

Solução. Seja an o número de maneiras de ele gastar os n reais.

Assim, para gastar os últimos reais, ou ele gasta n − 1 reais primeiramente e compra
um bolo no final, ou ele gasta n − 2 reais inicialmente e, em seguida, compra um sorvete
ou um pastel. Portanto, podemos escrever

an = an−1 + 2an−2 ,
com a1 = 1 (só dá pra comprar 1 bolo) e a2 = 3 (comprando 2 bolos ou 1 sorvete ou 1
pastel).

Agora, vamos resolver.

i) Equação caracterı́stica: λ2 − λ − 2 = 0, cujas raı́zes são 2 e −1.

4
POT 2012 - Álgebra - Nı́vel 2 - Aula 5 - Prof. Marcelo Mendes

ii) Termos geral: an = A · 2n + B · (−1)n . Podemos calcular a0 , que não faz sentido para
o gasto do dinheiro, mas existe na sequência associada: a2 = a1 + 2a0 ⇒ a0 = 1. Agora,
para n = 0 e n = 1

A+B =1
2A − B = 1,
2
cuja solução é A = 3 e B = 13 . Assim

2n+1 + (−1)n
an = .
3
Problema 2. Determine o termo geral da sequência definida pela recorrência a1 = 1, a2 = 4
e an = 4an−1 − 3an−1 para n ≥ 3.

Problema 3. Determine o termo


√ geral da sequência definida recorrentemente por a0 = 0,
a1 = 3 e, para n ≥ 3, an = 5an−1 + an−1 .

Problema 4. Considere um retângulo 1 × n, que deve ser preenchido por dois tipos de
retângulos menores 1 × 1 e 1 × 2. De quantas maneiras se pode fazer isso?

Problema 5. (OPM) Uma escada tem n degraus. Para subi-la, em cada passo, pode-se
subir um ou dois degraus de cada vez. De quantos modos diferentes pode-se subir a escada?

Problema 6. Uma sequência de números ak é definida por a0 = 0 e ak+1 = 3ak + 1, k ≥ 0.


Prove que a155 é divisı́vel por 11.

Solução. Inicialmente, veja que essa recorrência não depende dos dois termos anteriores.
A parcela 1 no membro da direita, na verdade, não é bem-vinda. Assim, de

ak+1 = 3ak + 1
ak = 3ak−1 + 1
3n − 1
obtemos ak+1 − 4ak + 3ak−1 = 0. O termo geral dessa recorrência é an = (a de-
2
monstração deixamos para o leitor).

3155 − 1
Logo, a155 = . Para finalizar, deixo como sugestão que 35 − 1 = 242 = 11 × 22.
2

21 3
Problema 7. Seja {an } uma sequência tal que a1 = e 2an − 3an−1 = n+1 , n ≥ 2.
16 2
Encontre o valor de a2 e a lei de recorrência de cada termo em função dos dois termos
imediatamente anteriores.

5
POT 2012 - Álgebra - Nı́vel 2 - Aula 5 - Prof. Marcelo Mendes

3 Recorrências e Equações do 2o Grau


Como exemplo para organizar as ideias, vamos supor que α seja uma raiz da equação
x2 + x − 1 = 0. Assim

α2 = −α + 1.
Daı́,

α3 = −α2 + α = 2α − 1
α4 = 2α2 − α = −3α + 2
α5 = −3α2 + 2α = 5α − 3.
Será que existe um padrão entre os coeficientes que aparecem no lado direito de cada
potência de α? Sim, existe! Na próxima aula, que será sobre indução finita, estaremos
aptos a provar que

αn = (−1)n−1 Fn α + (−1)n Fn−1 ,


sendo {Fn } a sequência de Fibonacci.

Problema 8. Se α e β são as raı́zes da equação ax2 + bx + c = 0 e Sn = αn + β n , n ∈ N,


então mostre que aSn+1 + bSn + cSn−1 = 0.

Solução. Como α e β são as raı́zes de ax2 + bx + c = 0, então

aα2 + bα + c = 0
aβ 2 + bβ + c = 0.
Daı́, multiplicando por αn−1 e β n−1 , respectivamente, temos

aαn+1 + bαn + cαn−1 = 0


aβ n+1 + bβ n + cβ n−1 = 0.
Somando, obtemos

a αn+1 + β n+1 + b (αn + β n ) + c αn−1 + β n−1 = 0


 

ou seja,

aSn+1 + bSn + cSn−1 = 0.

Problema 9. Seja α a maior raiz de x2 + x − 1 = 0. Determine o valor de α5 − 5α.

6
POT 2012 - Álgebra - Nı́vel 2 - Aula 5 - Prof. Marcelo Mendes

αn − β n
Problema 10. Sejam α e β as raı́zes de x2 + x − 1 = 0. Sendo an = , n = 1, 2, 3, ....
α−β
Determine os dois primeiros termos a1 e a2 dessa sequência e a lei de recorrência de cada
termo em função dos dois termos imediatamente anteriores.

7
POT 2012 - Álgebra - Nı́vel 2 - Aula 5 - Prof. Marcelo Mendes

Dicas

2. Use a equação caracterı́stica e encontre o termo geral seguindo o exemplo e a questão


1.

3. Use a equação caracterı́stica e encontre o termo geral seguindo o exemplo e a questão


1.

4. Para finalizar, ou ele completa com um quadradinho 1 × 1 o retângulo 1 × (n − 1),


que pode ser preenchido de an−1 maneiras, ou ele completa com um retângulo 1 × 2
o retângulo 1 × (n − 2), que pode ser preenchido de an−2 maneiras.

5. Para finalizar, ou ele sobe um degrau a partir do degrau n − 1, que pode ser alcançado
de an−1 maneiras, ou ele sobe dois degraus a partir do degrau n − 2, que pode ser
alcançado de an−2 maneiras.
3
7. Multiplique a equação de recorrência por 2 e subtraia de 2an−1 − 3an−2 = n , que é
2
a equação dada substituindo n por n − 1.

10. Se a equação caracterı́stica é x2 + x − 1 = 0, então a equação de recorrência é


an = −an−1 + an−2 .

Respostas

3n − 1
2. an =
2
√ !n √ !n
5+3 5−3
3. an = −
2 2

4. Sendo an o número de maneiras, a1 = 1, a2 = 2, an = an−1 + an−2

5. Sendo an o número de maneiras, a1 = 1, a2 = 2, an = an−1 + an−2


69
7. a2 = 32 e 4an − 8an−1 + 3an−2

9. −3

10. a1 = 1 e a2 = −1; an = −an−1 + an−2

8
Polos Olímpicos de Treinamento
Curso de Álgebra - Nível 2 Aula 6
Prof. Marcelo Mendes

Indução - Parte I

O verbo induzir significa gerar. Nesta aula, começaremos a ver o assunto Indução Ma-
temática (ou Indução Finita ou Princı́pio da Indução Finita), que é um método de prova
envolvendo números inteiros que aproveita o trabalho feito na demonstração de casos an-
teriores para se provar o fato para um inteiro maior.

Como assim? Vejamos um exemplo.

1 Um Exemplo para Organizar as Ideias


Considere mais uma vez a sequência de Fibonacci definida por F1 = F2 = 1 e, para
n ≥ 3, Fn = Fn−1 +Fn−2 , ou seja, seus primeiros termos são 1, 1, 2, 3, 5, 8, 13, 21, 34, 55, 89, ...

Vamos mostrar a seguinte identidade

F1 + F2 + ... + Fn = Fn+2 − 1.
Começamos verificando para valores pequenos de n:

i. n = 1 : F1 = F3 − 1 ⇔ 1 = 2 − 1

ii. n = 2 : F1 + F2 = F4 − 1 ⇔ 1 + 1 = 3 − 1

iii. n = 3 : F1 + F2 + F3 = F5 − 1 ⇔ 1 + 1 + 2 = 5 − 1

Mas o que o método da indução procura fazer é realizar a prova a partir de casos an-
teriores. Vamos fazer isso com as identidades acima.

Começando com
F1 = F3 − 1,
somamos F2 a cada membro da equação e obtemos

F1 + F2 = F2 + F3 − 1 = F4 − 1.
POT 2012 - Álgebra - Nı́vel 2 - Aula 6 - Prof. Marcelo Mendes

Observe que primeiro obtemos o lado esquerdo desejado e, só depois, organizamos o
lado direito. Prosseguindo, partimos de

F1 + F2 = F4 − 1,

somamos F3 a cada membro e chegamos a

F1 + F2 + F3 = F3 + F4 − 1 = F5 − 1,

ou seja, repetimos a mesma operação anterior. Isso pode ser feito de maneira geral, supondo
que já tenhamos chegado a

F1 + F2 + ... + Fk = Fk+2 − 1.

Em seguida, somamos Fk+1 a cada membro da equação e concluı́mos que

F1 + F2 + ... + Fk + Fk+1 = Fk+1 + Fk+2 − 1 = Fk+3 − 1.

Portanto, não precisamos fazer as operações uma por uma. Podemos descrever um
método que sirva para todas elas, desde que tenhamos um caso inicial para servir de ’ponta
pé inicial’. Esse método é o que chamamos de indução.

Observe que esse processo nos permite, inclusive, supor os resultado para outros mo-
mentos distintos do passo imediatamente anterior. Quando isso é necessário, costuma-se
chamar de indução forte.

Problema 1. Mostre que 2n > n2 , ∀n > 4.

Solução. Vejamos um caso inicial. Para n = 5, de fato, 25 > 52 . Se duplicamos ambos os


lados, obtemos 26 > 2 · 52 > 62 . Agora, vamos mostrar como essa passagem de um número
para o seguinte é geral.

Suponha que seja verdade para k > 4, ou seja, 2k > k2 . Daı́, duplicamos ambos os
lados

2k+1 > 2k2 ,


e ficará faltando apenas mostrar que

2k2 ≥ (k + 1)2
o que é equivalente a (k − 1)2 ≥ 2, o que é certamente verdade.

Problema 2. Prove que 3n ≥ n3 para todo n inteiro positivo.

2
POT 2012 - Álgebra - Nı́vel 2 - Aula 6 - Prof. Marcelo Mendes

Problema 3. Prove a desigualdade de Bernoulli: para todo número real x > −1 e todo
número natural n, ocorre

(1 + x)n ≥ 1 + nx.
Solução. Nesse problema, temos duas letras (x e n). Como indução é um método para
números inteiros, a variação só poderá ser do valor de n.

No caso inicial, quando n = 1, temos (1 + x)1 ≥ 1 + x · 1, que é verdade.

Em seguida, suponha que a desigualdade seja verdadeira para n = k, isto é, (1 + x)k ≥
1 + kx. Depois, multiplicamos cada lado por 1 + x (que é positivo pois x > −1) e chegamos
a

(1 + x)k+1 ≥ (1 + kx)(1 + x).


Agora, veja que (1 + kx)(1 + x) = 1 + (k + 1)x + kx2 ≥ 1 + (k + 1)x pois kx2 ≥ 0.
Portanto,

(1 + x)k+1 ≥ 1 + (k + 1)x.
Problema 4. Se x e y são números reais quaisquer, então |x + y| ≤ |x| + |y| (desigualdade
triangular). Usando esse fato, prove que se x1 , x2 , ..., xn são números reais quaisquer, então

|x1 + x2 + ... + xn | ≤ |x1 | + |x2 | + ... + |xn |.


Problema 5. Prove que uma soma arbitrária de n ≥ 8 centavos pode ser paga com moedas
de 3 e 5 centavos (tendo essas moedas em quantidade suficiente).

Solução. Como 8 = 3+ 5, então a operação é possı́vel para 8. Suponha que m ≥ 8 centavos


possam ser pagos. Então, será necessário provar que m + 1 centavos podem ser pagos dessa
maneira.
Se a soma de m centavos foi paga com o uso de moedas de 5 centavos, então substitua
uma moeda de 5 centavos por duas de 3 centavos e a quantia final será m + 1 centavos.
Caso contrário, a soma de m centavos foi paga somente com moedas de 3 centavos e,
como m ≥ 8, há ao menos três moedas de 3 centavos. Troque então três moedas de 3 centa-
vos por duas de 5 centavos e novamente a quantia final de m+1 centavos desejada foi obtida.

Problema 6. Seja F1 = F2 = 1 e Fn+2 = Fn+1 + Fn , para n ≥ 1, a sequência de Fibonacci.


Prove que quaisquer dois termos consecutivos dessa sequência são sempre primos entre si,
ou seja, mdc(Fk , Fk+1 ) = 1, ∀k ∈ N.
Problema 7. Seja F1 = F2 = 1 e Fn = Fn−1 + Fn−2 , n ≥ 3, a sequência de Fibonacci.
Mostre que F3n é par.
Problema 8. Seja F1 = F2 = 1 e Fn = Fn−1 + Fn−2 , n ≥ 3, a sequência de Fibonacci.
Mostre que F5n é múltiplo de 5.

3
POT 2012 - Álgebra - Nı́vel 2 - Aula 6 - Prof. Marcelo Mendes

Problema 9. SejaF1= F2 = 1 e Fn = Fn−1 + Fn−2 , n ≥ 3, a sequência de Fibonacci.


7 n
Mostre que Fn < .
4
Problema 10. A sequência (ai ) é definida por a1 = 0, a2 = 1, an+2 = 3an+1 −2an . Encontre
uma fórmula explı́cita para o n-ésimo termo dessa sequência.

Solução. Vamos calcular alguns termos iniciais na busca de algum padrão para a fórmula
explı́cita, aquela que depende apenas de n e não mais de outros termos.

a3 = 3a2 − 2a1 = 3 · 1 − 2 · 0 = 3,
a4 = 3a3 − 2a2 = 3 · 3 − 2 · 1 = 7,
a5 = 3a4 − 2a3 = 3 · 7 − 2 · 3 = 15,
a6 = 3a5 − 2a4 = 3 · 15 − 2 · 7 = 31.
O que os números 0, 1, 3, 7, 15, 31 têm de especial? Uma olhadinha cuidadosa nos faz
perceber que todos eles são potências de 2, menos 1. Mais especificamente

a3 = 3 = 22 − 1,
a4 = 7 = 23 − 1,
a5 = 15 = 24 − 1,
a6 = 31 = 25 − 1.
Portanto, nossa conjectura (sinônimo formal para ’chute’) será que an = 2n−1 − 1. So-
mente agora (após a conjectura feita) aplicaremos a ideia de indução, que só conseguirá
provar a fórmula caso ela seja verdadeira (caso fosse falsa, o processo de indução encontra-
ria um obstáculo intranspassável em algum momento).

Os casos iniciais já estão escritos e validam a conjectura. Em seguida, vamos supor
que tenhamos a fórmula válida para todo n ≤ k (indução forte). Em particular, estamos
supondo para k − 1 e k

ak−1 = 2k−2 − 1
ak = 2k−1 − 1.
Daı́,    
ak+1 = 3ak − 2ak−1 = 3 2k−1 − 1 − 2 2k−2 − 1

⇔ ak+1 = 2k − 1.
1
Problema 11. Sabe-se que a + é um inteiro. Prove que todos os números da forma
a
n 1
a + n , n = 2, 3, ..., também são inteiros.
a

4
POT 2012 - Álgebra - Nı́vel 2 - Aula 6 - Prof. Marcelo Mendes

Problema 12. Sejam a e b números reais distintos. Demonstrar por indução a proposição

(a − b)| (an − bn ) , ∀n ∈ N.
Problema 13. A sequência a1 , a2 , ..., an , ... de números é tal que a1 = 3, a2 = 5 e an+1 =
3an − 2an−1 , para n > 2. Prove que an = 2n + 1, ∀n ∈ N.
Problema 14. Mostre, por indução, que 2 + 4 + ... + 2n = 2n+1 − 2, ∀n ∈ N.
Problema 15. Considere a sequência definida recorrentemente por an+1 = 3an + 4, ∀n ∈ N.
Supondo a0 = 0:
a) Calcule a1 , a2 , a3 , a4 .
b) Conjecture uma fórmula para an e prove-a por indução.

Problema 16. Considere a sequência definida por a1 = 0, a2 = 3, an = 5an−1 −4an−2 , n ≥ 3.


Determine a maior potência de 2 que divide a2012 + 1.

Problema 17. Considere a sequência {an } definida por an = 32n − 1, n ∈ N.

a) Para cada n ∈ N, mostre que an+1 = an + 8 · 32n .


b) Demonstre, por indução sobre n, que an é divisı́vel por 8, para todo n ∈ N.
Problema 18. Sendo n um número inteiro positivo qualquer, demonstrar que a expressão
32n+2 − 2n+1 é divisı́vel por 7.
Problema 19. Mostre que
1 1 1 1 1 1 1
1− + − ... + − = + + ... + ,
2 3 2n − 1 2n n+1 n+2 2n
∀n ∈ N.
Problema 20. Prove que

1 · 1 + 1 · 2 · 2 + 1 · 2 · 3 · 3 + ... + 1 · 2 · ... · (n − 1) · n · n = 1 · 2 · ... · n · (n + 1) − 1,

∀n ≥ 2 natural.
Problema 21. a) Verifique que a soma dos inversos de 2, 3 e 6 é 1.
b) Prove que ∀p natural, p ≥ 3, existem p naturais 2 a 2 distintos n1 , n2 , ..., np tais que
1 1 1
+ + ... + = 1.
n1 n2 np

Problema 22. Mostre que o número de diagonais de um polı́gono convexo de n lados é


n(n − 3)
dn = .
2

5
POT 2012 - Álgebra - Nı́vel 2 - Aula 6 - Prof. Marcelo Mendes

Dicas

2. Veja a solução da questão 1.

6. Suponha que dois números de Fibonacci consecutivos possuam um fator primo em


comum e conclua que todos os números de Fibonacci teriam esse fator.
    
n 1 n−1 1 1 n−2 1
11. Use a identidade a + n = a + n−1 a+ − a + n−2 .
a a a a

12. Use a identidade (an − bn ) = an−1 − bn−1 (a + b) − ab an−2 − bn−2 .


 

16. Resolva a equação de recorrência.

18. Use os fatos 32n+2 − 2n+1 = 9n+1 − 2n+1 , 9 = 7 + 2 e (a + b)k = M a + bk , em que


M a representa ’um múltiplo de a’. Ou, então, chame an+1 = 9n+1 − 2n+1 e use que
an+1 − 2an = 7 · 9n , repetindo a ideia proposta no problema 17.

21. Para o item b, suponha que a igualdade seja verdadeira para k números, divida a
1
equação por 2 e some a ambos os lados da nova identidade.
2
22. Observe que as diagonais de um polı́gono A1 A2 ...An+1 são todas as diagonais de
A1 A2 ...An , além do lado A1 An e das diagonais que partem de An+1 .

Respostas

15. a) a1 = 4, a2 = 16, a3 = 52, a4 = 212. b) Conjectura: an = 2 · 3n − 2.

16. 4022.

6
Polos Olímpicos de Treinamento
Curso de Álgebra - Nível 2 Aula 7
Prof. Marcelo Mendes

Indução - Parte II

Vamos iniciar esta aula com a resolução de alguns problemas propostos na aula anterior.

1 Resolução de Problemas da Última Aula

Problema 1. a) Verifique que a soma dos inversos de 2, 3 e 6 é 1.


b) Prove que ∀p natural, p ≥ 3, existem p naturais 2 a 2 distintos n1 , n2 , ..., np tais que
1 1 1
+ + ... + = 1.
n1 n2 np
Solução. O primeiro item é imediato
1 1 1 3+2+1
+ + = = 1.
2 3 6 6
A partir dessa soma, seguindo a dica dada na aula passada, obtemos
1 1 1 1
+ + =
4 6 12 2
1 1 1 1
⇒+ + + =1
2 4 6 12
1
simplesmente dividindo por 2 e somando a cada lado em seguida.
2
Agora vamos repetir a argumentação para o passo indutivo.

Suponhamos que já tenhamos k números naturais 2 ≤ n1 < n2 < ... < nk tais que
1 1 1
+ + ... + = 1.
n1 n2 nk
Daı́
1 1 1 1
+ + ... + =
2n1 2n2 2nk 2
POT 2012 - Álgebra - Nı́vel 2 - Aula 7 - Prof. Marcelo Mendes

1 1 1 1
+⇒ + + ... + = 1,
2 2n1 2n2 2nk
que é a soma dos inversos de k + 1 números naturais distintos pois 2 < 4 ≤ 2n1 < 2n2 <
... < 2nk .
1 1
Problema 2. Sabe-se que a+ é um inteiro. Prove que todos os números da forma an + n ,
a a
n = 2, 3, ..., também são inteiros.
Solução. Vejamos os casos iniciais para perceber como o padrão novamente é mantido para
o passo indutivo. Para n = 2,
  
2 1 1 1
a + 2 = a+ a+ −2
a a a
1 2
    
1 1
que é claramente inteiro. A multiplicação a + a+ no lugar de a + é para
a a a
preparar melhor para o passo indutivo. Para n = 3,
    
3 1 2 1 1 1
a + 3 = a + 2 a+ − a+
a a a a
que também é inteiro. Pronto, agora estamos bem preparados para o passo indutivo, basta
1
repetir o argumento para os casos pequenos. Suponha que an + n seja inteiro para todo
a
k 1 k−1 1
k ≤ n (indução forte). Em particular, a + k e a + k−1 são inteiros. Logo
a a
    
k+1 1 k 1 1 k−1 1
a + k+1 = a + k a+ − a + k−1
a a a a
também é inteiro.

Problema 3. Prove que

1 · 1 + 1 · 2 · 2 + 1 · 2 · 3 · 3 + ... + 1 · 2 · ... · (n − 1) · n · n = 1 · 2 · ... · n · (n + 1) − 1,

∀n ≥ 2 natural.
Solução. Observe que podemos reescrever a soma da seguinte forma

1! · 1 + 2! · 2 + 3! · 3 + ... + n! · n = (n + 1)! − 1,
sendo k! = 1 · 2 · ... · k o fatorial do número inteiro não-negativo k (0! = 1).

i) Para n = 1, 1! · 1 = 2! − 1.

ii) Suponha que a identidade seja verdadeira para n = k, ou seja,

1! · 1 + 2! · 2 + 3! · 3 + ... + k! · k = (k + 1)! − 1.

2
POT 2012 - Álgebra - Nı́vel 2 - Aula 7 - Prof. Marcelo Mendes

iii) Agora vejamos para n = k + 1. Vamos somar (k + 1)! · (k + 1) a ambos os membros da


suposta identidade do item ii):

1! · 1 + 2! · 2 + 3! · 3 + ... + k! · k + (k + 1)! · (k + 1)
= (k + 1)! + (k + 1)! · (k + 1) − 1 = (k + 1)! · (k + 2) − 1 = (k + 2)! − 1.

2 Mais Problemas

Problema 4. (China) Há pelo menos quatro barras de chocolate em n(n ≥ 4) caixas.
Camila pode, por vez, escolher 2 caixas, pegar uma barra de cada uma dessas caixas e
colocá-las em uma terceira caixa. Determine se sempre é possı́vel por todas as barras em
uma mesma caixa.
Solução. Vejamos o caso inicial com 4 barras. As possibilidades iniciais de quantidades
nas caixas são

1, 1, 1, 1, 0, 0, ...
1, 1, 2, 0, 0, 0, ...
1, 3, 0, 0, 0, 0, ...
2, 2, 0, 0, 0, 0, ...
4, 0, 0, 0, 0, 0, ...
Se ocorrer o caso na 1a linha, então pegamos as barras nas caixas 3 e 4 e as passamos
para a 5a caixa. Assim, chegamos no caso da 2a linha (não faz diferença se o 2 aparece na
3a ou na 5a posição).

Se ocorrer o caso da 2a linha, só precisamos deslocar as barras das caixas 1 e 2 para a
3a caixa e o objetivo está feito.

Se ocorrer o caso da 3a linha, tomamos uma barra de cada caixa e as colocamos em


uma outra caixa, gerando a configuração da 4a linha.

Se ocorrer o caso da 4a linha, tomamos uma barra de cada caixa e as colocamos em


uma outra caixa, obtendo o caso da 2a linha, que já mostrado como se finaliza.

Finalmente, se ocorrer o caso da 5a linha, não há nada a fazer pois todas as barras já
estão na mesma caixa.

Agora, vamos supor que seja possı́vel deslocar k barras, dispostas de maneira aleatória
nas n caixas, para uma única caixa.

3
POT 2012 - Álgebra - Nı́vel 2 - Aula 7 - Prof. Marcelo Mendes

Numa configuração com k +1 barras, separamos uma delas e realizamos o procedimento


indutivo com as demais k barras. Ficamos, assim, com

k, 1, 0, 0, 0, 0, ...
Podemos deslocar todas as barras para uma única caixa a partir dos seguintes movi-
mentos:

k, 1, 0, 0, ... → k − 1, 0, 2, 0, ...

k − 2, 2, 1, 0, ... → k − 3, 2, 0, 2, ...

k − 1, 1, 0, 1, ... → k + 1, 0, 0, 0, ...
e a indução foi finalizada.

Problema 5. Determine, com prova, se é possı́vel arranjar os números 1, 2, 3, ..., 1000 em


uma fila de tal forma que a média de qualquer par de números distintos não esteja localizada
entre esses dois números.
Solução. Vamos provar um resultado mais geral para os números 1, 2, 3, ..., 2n .

i) Vejamos alguns casos iniciais.


Se n = 1, os números são 1 e 2 e não há nada a fazer.
Se n = 2, podemos dispor os números na ordem 1, 3; 2, 4.
Para n = 3, dispomos os números na ordem 2, 6, 4, 8; 1, 5, 3, 7. Essa ordenação
foi obtida a partir do caso n = 2 separando os pares dos ı́mpares da seguinte forma:
duplicamos os números do caso n = 2 e obtemos 2, 6, 4, 8. Se o resultado era válido
para n = 2, então duplicando permanece valendo. Em seguida, subtraı́mos 1 de cada
um desses pares, obtendo 1, 5, 3, 7, que também possui a propriedade desejada. Além
disso, tomando um número par e um número ı́mpar, claramente a média não aparece
no conjunto.

ii) Suponha que seja possı́vel arranjar os números 1, 2, ..., 2k na forma

a1 , a2 , ..., a2k

com a propriedade desejada.

iii) Para 2k+1 , a ideia é a mesma mostrada para se passar de 4 para 8. Separamos os pares
e os ı́mpares de 1 a 2k+1 da seguinte forma

2a1 , 2a2 , ..., 2a2k ; 2a1 − 1, 2a2 − 1, ..., 2a2k − 1.

4
POT 2012 - Álgebra - Nı́vel 2 - Aula 7 - Prof. Marcelo Mendes

Mostrado que o resultado é válido para potências de 2, temos que é verdadeiro, em


particular, para 1024. Após chegar à configuração válida para 1024, é só apagar os números
1001 até 1024.

Problema 6. Prove por indução que

n4
13 + 23 + ... + n3 > ,
4
∀n ∈ N.

Problema 7. Se A é um conjunto finito com n elementos, mostre que A possui 2n subcon-


juntos.

Problema 8. Prove que dentre quaisquer 2m + 1 inteiros distintos, cujos valores absolutos
não excedem 2m − 1, é possı́vel encontrar 3 deles cuja soma é igual a 0.
Solução. Provemos o resultado por indução. Para m = 1, nossos números serão {−1, 0, 1}
e sua soma é zero. Suponha que o resultado seja verdadeiro para m = k − 1, k ≥ 2.

Considere um conjunto arbitrário A formado por 2k + 1 números cujos valores absolu-


tos não excedem 2k − 1. Se dentre eles houver 2k − 1 números cujos valores absolutos não
excedem 2k − 3, o resultado é verdadeiro pela hipótese de indução.

Caso contrário, sem perda de generalidade, podemos considerar que A contém os


números 2k − 1, 2k − 2, −2k + 1(−2k + 1 está) ou 2k − 1, 2k − 2, −2k + 2(−2k + 1 não
está e os demais estão). Primeiro, considere os seguintes 2k − 2 pares de números, supondo
2k − 1 e −2k + 1 no conjunto:
(1, 2k − 2)
(2, 2k − 3)
(3, 2k − 4)
:
(k, k − 1)
e
(−1, −2k + 2)
(−2, −2k + 3)
:
(−k + 1, −k)
Se 0 está dentre os números escolhidos, então juntamente com 2k − 1 ou −2k + 1
terı́amos um tripla com soma zero. Senão, terı́amos 2k − 1 inteiros de A distribuı́dos dentre
os números desses 2k − 2 pares e dois deles estariam no mesmo par, que juntamente com
2k − 1 ou −2k + 1 formariam um tripla com soma zero.

5
POT 2012 - Álgebra - Nı́vel 2 - Aula 7 - Prof. Marcelo Mendes

Agora, considere que −2k + 1 e os 2k − 4 pares

(1, 2k − 3)

(2, 2k − 4)
(3, 2k − 5)
:
(k − 2, k)
e
(−2, −2k + 3)
:
(−k + 1, −k)
Se 0 ou 1 estiver é trivial. Senão, já escolhemos 3 números (0, 1, −2k + 1). Assim
terı́amos que escolher ainda 2k − 3 números dentre os 2k − 4 pares, e terı́amos dois deles
no mesmo par.

Problema 9. Prove que a sequência

22 − 3, 23 − 3, ..., 2n − 3, ...
contém um número infinito de inteiros tais que cada dois deles sejam relativamente primos.

Solução. Provemos por indução.

Suponha que já tenhamos k números a1 = 2n1 − 3, a2 = 2n2 − 3, ..., ak = 2nk − 3, que
sejam primos entre si.

Construamos agora ak+1 relativamente primo com os k demais. Seja N = a1 a2 ...ak .


Pelo Princı́pio da Casa dos Pombos, dentre os N +1 números 20 , 21 , ..., 2N haverá dois deles,
digamos 2r e 2s (r > s), com o mesmo resto na divisão por N . Assim, 2r − 2s = 2s (2r−s − 1)
é divisı́vel por N , assim como 2r−s − 1, pois N é ı́mpar (já que é produto de primos). Por-
tanto, ak+1 = 2r−s − 3 é relativamente primo com N e, consequentemente, com cada um
dos a1 , a2 , ..., ak .

Problema 10. Três inteiros foram escritos em um quadro-negro. Então um deles foi apa-
gado e a soma dos outros dois, menos 1, foi escrito em seu lugar. Esse procedimento foi
repetido várias vezes até que os números 17, 1983, 1989 aparecessem eventualmente. É
possı́vel que os números iniciais fossem 2, 2, 2?

6
POT 2012 - Álgebra - Nı́vel 2 - Aula 7 - Prof. Marcelo Mendes

Problema 11. Uma quantidade finita de cartões é colocada em duas torres, com mais
cartões na torre esquerda que na direita. Cada cartão tem um ou mais nomes distintos
escrito nele. Além disso, diferentes cartões podem compartilhar alguns nomes. Para cada
nome, definimos uma ação pelo movimento de todo cartão que tem esse mesmo nome es-
crito nele para a torre oposta. Prove que é sempre possı́vel finalizar com mais cartões na
torre da direita através de várias ações para diferentes nomes.

Problema 12. Sejam F1 = F2 = 1 e, para n ≥ 3, Fn = Fn−1 + Fn−2 os números de Fibo-


nacci. Prove o Teorema de Zeckendorff: todo número natural pode ser escrito de maneira
única como soma de números de Fibonacci com ı́ndices maiores que 1 e não consecutivos.

7
POT 2012 - Álgebra - Nı́vel 2 - Aula 7 - Prof. Marcelo Mendes

Dicas

7. Use que, ao acrescentarmos um novo elemento a um conjunto já existente, os sub-


conjuntos do conjunto inicial permanecem e todos elementos, unidos com o novo
elemento, formam os novos subconjuntos.

10. Tente mostrar que há sempre 2 números pares em cada tripla.

11. Indução sobre a quantidade de nomes.

8
POT 2012 - Álgebra - Nı́vel 2 - Aula 7 - Prof. Marcelo Mendes

Respostas

10. Sempre há a presença de no mı́nimo 2 números pares. Como no inı́cio temos 3
números pares, em seguida teremos 2 pares e um ı́mpar; e dois pares e um ı́mpar e
dois pares e um ı́mpar ... Pode-se provar por indução que em qualquer passo teremos
sempre 2 números pares e um ı́mpar. Mas a tripla (17, 1983, 1989) só tem números
ı́mpares e nunca poderá ser obtida de (2, 2, 2).

11. Vamos utilizar indução sobre a quantidade de nomes.


i. n = 1 é trivial.
ii. Sejam a1 , a2 , ..., an os n primeiros nomes e a, o nome n + 1.
iii. Para n + 1, há 2 casos, sendo aE e aD as quantidades de cartões com apenas o
nome a nas pilhas esquerdas e direita, respectivamente.

1o caso: aE ≤ aD .
Realize as operações com os nomes a1 , a2 , ..., an .

2o caso: aE > aD .
Realize a troca apenas com os cartões em que está escrito apenas o nome a e repita
as operações do 1o caso.

12. F2 = 1. Se Fk é o maior número de Fibonacci menor que ou igual a n, devemos ter


n − Fk < Fk−1 pois, caso contrário, n − Fk ≥ Fk−1 e, portanto, n ≥ Fk + Fk−1 = Fk+1 ,
absurdo. Assim, é só escrever n = Fk + (n − Fk ) e utilizar hipótese de indução para
n − Fk .

9
Polos Olímpicos de Treinamento
Curso de Álgebra - Nível 2 Aula 8
Prof. Marcelo Mendes

Desigualdades - Parte I

1 Fatos Elementares

i) Nenhum quadrado de número real é negativo.

ii) Desigualdade de Cauchy (Médias Aritmética e Geométrica)


Se a1 , a2 , ..., an são números reais positivos, então

a1 + a2 + ... + an √
≥ n a1 a2 ...an ,
n
com igualdade ocorrendo se, e somente se, a1 = a2 = ... = an .

Para mostrar essa última desigualdade, vamos utilizar um tipo diferente de indução
(que não serve para qualquer problema).

a1 + a2 √ √ √ 2
1. Se n = 2, então ≥ a1 a2 pois a1 − a2 ≥ 0.
2
2. Para n = 4, então utilizando o caso já mostrado para 2 números, temos

a1 +a2
+ a3 +a
r
a1 + a2 + a3 + a4 2 2
4
a1 + a2 a3 + a4
= ≥ ·
4 2 2 2
√ √ √
q
≥ a1 a2 · a3 a4 = 4 a1 a2 a3 a4 ,

quaisquer que sejam a1 , a2 , a3 , a4 reais positivos.


a1 + a2 + a3
3. Assim, podemos escolher a4 = , obter
3
a1 + a2 + a3 + a1 +a32 +a3
r
a1 + a2 + a3
≥ 4 a1 a2 a3
4 3
POT 2012 - Álgebra - Nı́vel 2 - Aula 8 - Prof. Marcelo Mendes

 4
a1 + a2 + a3 a1 + a2 + a3
⇔ ≥ a1 a2 a3
3 3

a1 + a2 + a3 √
⇔ ≥ 3 a1 a2 a3 ,
3
e concluir que o resultado também é verdadeiro para n = 3.

A demonstração segue copiando as ideias acima. Já temos os casos iniciais. Em seguida,
supondo o resultado verdadeiro para k, obtemos o resultado para 2k e para k − 1 repetindo
os procedimentos realizados nos itens 2 e 3 acima. Assim, provamos a desigualdade para
qualquer quantidade natural maior que ou igual a 2 de números reais positivos.

2 Problemas

Problema 1. Determine o valor máximo da função f (x) = x(1 − x), sendo x ∈ (0; 1).

Solução. Essa é uma função quadrática. Poderı́amos encontrar o seu valor máximo através
da ordenada do vértice da parábola (desde que a abscissa do vértice esteja em (0; 1), o que,
de fato, é verdade).

Mas se resolvermos utilizando a Desigualdade de Cauchy, poderemos aplicar a ideia


para funções de grau maior que 2:

x + (1 − x) p
≥ x(1 − x)
2
1
⇒ x(1 − x) ≤ ,
4
com igualdade ocorrendo se, e somente se, x = 1−x, ou seja, x = 12 . Assim, o valor máximo
de f é 14 .

Observação. Existe uma diferença entre descobrir que f (x) ≤ 41 e concluir que 41 é seu
valor máximo. Por exemplo, podemos afirmar que sen x ≤ 3, porém o valor máximo de
sen x é 1, pois a igualdade em sen x ≤ 3 não ocorre.

Problema 2. Determine o valor máximo da função f (x) = x3 (1 − x), sendo x ∈ (0; 1).

Solução. Uma ideia possı́vel seria aplicar a Desigualdade de Cauchy com os números reais
positivos x3 e 1 − x:

x3 + (1 − x) p 3
≥ x (1 − x).
2

2
POT 2012 - Álgebra - Nı́vel 2 - Aula 8 - Prof. Marcelo Mendes

Apesar de verdadeiro, esse fato não nos dá um valor (não poder ser variável) máximo
para f .

Outra tentativa seria com x, x, x, 1 − x, todos positivos:

x + x + x + (1 − x) p
≥ 4 x3 (1 − x),
4
ou seja,
 4
3 2x + 1
x (1 − x) ≤
4
e, novamente, não achamos um valor máximo. Todavia, chegamos bem perto. Basta
substituir 1 − x por 3(1 − x):

x + x + x + 3(1 − x) p
≥ 4 x3 3(1 − x),
4
e daı́
 4
3
x3 3(1 − x) ≤
4
27
⇔ x3 (1 − x) ≤ .
81
3 27
Como a igualdade ocorre com x = 3(1 − x) ⇔ x = , o valor máximo de f é .
4 81

Problema 3. Determine o valor máximo da função f (x) = x(1 − x)3 , sendo x ∈ (0; 1).

Problema 4. (Treinamento Cone Sul) Sejam a e b números reais positivos tais que a+b = 1.
4
Prove que ab2 ≤ .
27
Problema 5. Sejam A, B, C os vértices de um triângulo inscrito em um cı́rculo unitário (ou
seja, cujo raio mede 1) e seja P um ponto no perı́metro do triângulo. Mostre que
32
PA · PB · PC ≤ .
27
Problema 6. Dados números positivos arbitrários a, b, c, prove que ao menos uma das
seguintes desigualdades é falsa:
1 1 1
a(1 − b) > , b(1 − c) > , c(1 − a) > .
4 4 4

3
POT 2012 - Álgebra - Nı́vel 2 - Aula 8 - Prof. Marcelo Mendes

Problema 7. (IMO) Sendo K, L, M pontos sobre os lados BC, CA, AB do ∆ABC, mostre
1
que a área de ao menos um dos triângulos AM L, BKM, CLK é menor que ou igual da
4
área do triângulo ABC.

Solução. Sendo k, l, m ∈ [0; 1], podemos escrever

BK = ka, KC = (1 − k)a
CL = lb, LA = (1 − l)b
AM = mc, M B = (1 − m)c.
Assim,
1
[AM L] = mc · (1 − l)b · sen∠A
2
⇒ [AM L] = m(1 − l) · [ABC].
Analogamente,

[BKM ] = k(1 − m) · [ABC],


[CLK] = l(1 − k) · [ABC].
1
Supondo que as três áreas em questão sejam maiores que da área de ABC, o resultado
4
segue pelo problema 6.

Problema 8. (Treinamento Cone Sul) Sejam ha , hb , hc as alturas do ∆ABC. Prove que


∆ABC é equilátero ⇔ ahb + bhc + cha é igual a 6 vezes a área do ∆ABC.

Problema 9. (Treinamento Cone Sul) Seja P um polı́gono convexo com 2012 lados e com
todos os ângulos internos iguais. Sejam l1 , l2 , ..., l2012 os comprimentos dos lados consecu-
tivos. Prove que se
l1 l2 l2011 l2012
+ + ... + + = 2012,
l2 l3 l2012 l1
então P é um polı́gono regular.

Problema 10. Mostre que, se x, y, z são números reais positivos, então


1 1 1
(1 + xy) + (1 + yz) + (1 + zx) ≥ 6.
x y z
Problema 11. Prove a desigualdade entre as médias geométrica e harmônica para 2 números
a e b reais positivos, ou seja,
√ 2
ab ≥ 1 .
a + 1b

4
POT 2012 - Álgebra - Nı́vel 2 - Aula 8 - Prof. Marcelo Mendes

Problema 12. Prove a desigualdade entre as médias quadrática e aritmética para 2 números
reais positivos.

Solução. Devemos mostrar que


r
a 2 + b2 a+b
≥ ,
2 2
que é equivalente a (a − b)2 ≥ 0.

Problema 13. Prove que se a, b, c são as medidas dos lados de um triângulo e a2 + b2 = kc2 ,
1
então k > .
2
Problema 14. a) Prove que se a, b são inteiros positivos com a 6= −b, então
1 1 4
+ ≥ .
a b a+b

b) Em uma lousa, escrevemos n números. É permitido apagar qualquer par deles a e b,


a+b
escrevendo no lugar. Repetindo tal procedimento n − 1 vezes, obtemos o número
4
2012
k. Se os n números iniciais eram 2012, prove que k ≥ .
n
Problema 15. Seja x um número real e m, um natural. Prove que

x(x + 1)(x + 2)...(x + m − 1) 1 1 1


≥ x1+ 2 + 3 +...+ m .
m(m − 1)(m − 2)...1

5
POT 2012 - Álgebra - Nı́vel 2 - Aula 8 - Prof. Marcelo Mendes

Dicas

3. Repita a ideia da solução do problema 2.


4. Repita a ideia da solução do problema 2.
5. Repita a ideia da solução do problema 2. Use também potência do ponto P e que,
supondo P sobre o lado BC, a corda contendo P A tem medida menor que ou igual
à medida 2 do diâmentro.
6. Suponha a possibilidade de ocorrerem as 3 desigualdades e multiplique-as.
8. Use a Desigualdade de Cauchy com ahb , bhc , cha .
l1 l2 l2011 l2012
9. Use a Desigualdade de Cauchy com , , ..., , .
l2 l3 l2012 l1
13. Use o problema 12 e a desigualdade triangular.
14. Compare a soma dos inversos dos números antes e depois de cada substituição de
números.
x+2 x+1+1
15. Escreva, por exemplo, = e aplique a Desigualdade de Cauchy. Faça
3 3
o mesmo para os demais fatores do numerador e do denominador aos pares.

Respostas

13. Pelo enunciado, pelo problema 12 e pela desigualdade triangular, temos


2
kc2 a 2 + b2

a+b  c 2
= ≥ >
2 2 2 2
1
⇒k> .
2
1 1 4
14. a) + ≥ ⇔ (a + b)2 ≥ 4ab ⇔ (a − b)2 ≥ 0.
a b a+b
b) Por a), segue que a soma dos inversos dos números envolvidos nunca aumenta.
Assim, comparando o inı́cio e o final dos procedimentos, temos

1 1 1
+ ... + ≥
2012 2012 k
1 1
⇔n· ≥
2012 k
2012
⇔k≥ .
n

6
Polos Olímpicos de Treinamento
Curso de Álgebra - Nível 2 Aula 9
Prof. Marcelo Mendes

Desigualdades - Parte II

1 A Desigualdade de Cauchy-Schwarz
Sejam a1 , a2 , ..., an , b1 , b2 , ..., bn números reais. Então:

a21 + a22 + ... + a2n b21 + b22 + ... + b2n ≥ (a1 b1 + a2 b2 + ... + an bn )2 .
 

Perceba o padrão que há na aplicação desta desigualdade: multiplicamos os termos de


mesma ordem de cada soma do lado esquerdo e, em seguida, extraı́mos a raiz quadrada;
o resultado é colocado na posição correspondente no somatório do lado direito, que será
elevado ao quadrado.

Vejamos agora alguns exemplos antes de apresentarmos a demonstração da desigual-


dade.

Exemplo 1 Prove a desigualdade entre as médias quadrática e aritmética para n números


reais positivos.
Solução. O caso particular para n = 2 dessa desigualdade apareceu na aula anterior e a
solução apresentada apenas utilizou o fato de um quadrado de número real ser não-negativo.
Vejamos agora a solução para uma quantidade qualquer.

Por Cauchy-Schwarz, temos

a21 + a22 + ... + a2n 12 + 12 + ... + 12 ≥ (a1 + a2 + ... + an )2


 

r
a21 + a22 + ... + a2n a1 + a2 + ... + an
⇔ ≥ ,
n n
como desejávamos (a expressão no lado esquerdo é a média quadrática dos n números
a1 , a2 , ..., an ).
POT 2012 - Álgebra - Nı́vel 2 - Aula 9 - Prof. Marcelo Mendes

Exemplo 2 Sejam a, b, c números reais positivos. Prove que

a2 + b2 + c2 ≥ ab + bc + ca.
Solução. Por Cauchy-Schwarz:

a2 + b2 + c2 b2 + c2 + a2 ≥ (ab + bc + ca)2 .
 

Extraindo a raiz quadrada, temos o resultado.

Demonstração (da Desigualdade de Cauchy-Schwarz)


n
X
Considere a função f (x) = (ak x − bk )2 . Desenvolvendo, teremos uma função do 2o grau
k=1
em x:
n
X n
X n
X
f (x) = x2 a2k − 2x ak bk + b2k ,
k=1 k=1 k=1
cujo descriminante é dado por

n
!2 n
! n
!
X X X
∆=4 a k bk −4 a2k b2k .
k=1 k=1 k=1

Como f é uma soma de quadrados, f (x) ≥ 0, ∀x ∈ R. Daı́, ∆ ≤ 0 e segue o resultado.

Já a igualdade ocorre quando ∆ = 0. Nesse caso, f possui uma raiz (dupla) x0 . Isso
implica que
n
X
(ak x0 − bk )2 = 0.
k=1
Nessa soma, todos os números envolvidos são reais e, portanto, seus quadrados são
não-negativos. Assim, ak x0 − bk = 0, ∀k, o que indica que, para a igualdade, ak e bk são
proporcionais para todo k.

2 Problemas

Problema 1. Seja c o comprimento √da hipotenusa de um triângulo retângulo cujos catetos


medem a e b. Prove que a + b ≤ c 2.
Solução. Do primeiro exemplo, com n = 2, temos
2
a 2 + b2

a+b
≥ .
2 2
Daı́
c2 (a + b)2 √
≥ ⇔ a + b ≤ c 2.
2 4

2
POT 2012 - Álgebra - Nı́vel 2 - Aula 9 - Prof. Marcelo Mendes

Problema 2. Sendo a, b, c números reais positivos, mostre que


 
1 1 1 9
2 + + ≥ .
a+b b+c c+a a+b+c

Problema 3. Sejam x1 , x2 , ..., xn números reais positivos e y1 , y2 , ..., yn uma permutação


dos xi , 1 ≤ i ≤ n. Prove a desigualdade

x21 x22 x2
+ + ... + n ≥ x1 + x2 + ... + xn .
y1 y2 yn
Solução. Pela desigualdade de Cauchy-Schwarz, segue
 2
x1 x22 x2n

+ + ... + (y1 + y2 + ... + yn ) ≥ (x1 + x2 + ... + xn )2 .
y1 y2 yn
O resultado segue do fato de x1 + x2 + ... + xn = y1 + y2 + ... + yn pois os yi são os xi
em alguma ordem.

Problema 4. (Baltic-Way) Prove que para quaisquer reais positivos x1 , x2 , ..., xn , y1 , y2 , ..., yn
ocorre
n
X 1 4n2
≥ n .
xi y i X 2
k=1 (xi + yi )
k=1

Solução. Por Cauchy-Schwarz, garantimos que

n n n
!2
X 1 X X xi + y i
(xi + yi )2 ≥ √ .
xi y i xi y i
k=1 k=1 k=1

Pela desigualdade entre as médias aritmética e geométrica, obtemos


xi + y i
√ ≥ 2.
xi y i
Assim,

n n n
!2
X 1 X X
(xi + yi )2 ≥ 2 = (2n)2 = 4n2 .
xi y i
k=1 k=1 k=1

3
POT 2012 - Álgebra - Nı́vel 2 - Aula 9 - Prof. Marcelo Mendes

Problema 5. Se a, b, c são números positivos, prove que

a2 b + b2 c + c2 a a2 c + b2 a + c2 b ≥ 9a2 b2 c2 .
 

Obs: Resolva esse problema de duas maneiras, utilizando a desigualdade entre as médias
aritmética e geométrica e através da desigualdade de Cauchy-Schwarz.
n
X
Problema 6. Sejam a1 , a2 , ..., an , b1 , b2 , ..., bn números reais positivos tais que ak =
k=1
n n n
X X a2k 1X
bk . Mostre que ≥ ak .
ak + bk 2
k=1 k=1 k=1

Problema 7. Se a1 , a2 , ..., an , b1 , b2 , ..., bn são 2n números reais positivos, mostre que ou


a1 a2 an
+ + ... + ≥n
b1 b2 bn
ou

b1 b2 bn
+ + ... + ≥ n.
a1 a2 an

Problema 8. Sejam a, b, c os lados de um triângulo. Mostre que


a b c 3
+ + ≥ .
b+c c+a a+b 2
Solução. Somando 1 a cada fração do lado esquerdo, obtemos
a+b+c a+b+c a+b+c 9
+ + ≥ .
b+c c+a a+b 2
Para mostrar essa última desigualdade, vamos utilizar a desigualdade de Cauchy-Schwarz

  h
a+b+c a+b+c a+b+c i2
[(b + c) + (c + a) + (a + b)] + + ≥ 3 a+b+c
b+c c+a a+b
 
a+b+c a+b+c a+b+c
[2(a + b + c)] + + ≥ 9(a + b + c),
b+c c+a a+b
de onde segue o resultado.

bc ca ab
Problema 9. Se a > 0, b > 0, c > 0, então prove que + + ≥ a + b + c.
a b c

4
POT 2012 - Álgebra - Nı́vel 2 - Aula 9 - Prof. Marcelo Mendes

a3 b3 c3
Problema 10. (Romênia) Prove que + + ≥ a + b + c, quaisquer que sejam a, b, c
bc ca ab
reais positivos.

Problema 11. Sejam a, b, c, d números reais positivos. Mostre que


1 1 4 16 64
+ + + ≥ .
a b c d a+b+c+d
Solução. Pela desigualdade de Cauchy-Schwarz, obtemos
 
1 1 4 16
(a + b + c + d) + + + ≥ (1 + 1 + 2 + 4)2 = 64,
a b c d
de onde segue o resultado.

5
POT 2012 - Álgebra - Nı́vel 2 - Aula 9 - Prof. Marcelo Mendes

Dicas

2. Passe a + b + c para a esquerda e escreva 2(a + b + c) = (a + b) + (b + c) + (c + a).

5. Faça a primeira solução através da desigualdade entre as médias aritmética e geométrica


aplicada a cada soma em parênteses do lado esquerdo. A segunda pode ser ob-
tida através da desigualdade de Cauchy-Schwarz reescrevendo a2 c + b2 a + c2 b como
c2 b + a2 c + b2 a.
n n
X a2k X
6. Multiplique por (ak + bk ), aplique a desigualdade de Cauchy-Schwarz
ak + bk
k=1 k=1
X n n
X
e lembre-se de que (ak + bk ) = 2 ak .
k=1 k=1

a1 a2 an b1 b2 bn
7. Suponha + + ... + < n ou + + ... + < n e aplique a desigualdade
b1 b2 bn a1 a2 an
de Cauchy-Schwarz com as somas nos lados esquerdos dessas duas desigualdades.
Conclua, assim, um absurdo.
r 2
bc bc
9. Escreva = (repita o mesmo para as demais parcelas do lado esquerdo) e
a a
utilize o exemplo 2.

10. Repita a ideia do problema 9.

6
POT 2012 - Álgebra - Nı́vel 2 - Aula 9 - Prof. Marcelo Mendes

Soluções

2. Por Cauchy-Schwarz
 
1 1 1
((a + b) + (b + c) + (c + a)) + + ≥ (1 + 1 + 1)2 = 9.
a+b b+c c+a

5. 1a Solução. Pela desigualdade entre as médias aritmética e geométrica, temos

a2 b + b2 c + c2 a √
3
≥ a2 b · b2 c · c2 a
3
⇔ a2 b + b2 c + c2 a ≥ 3abc.

Analogamente,
a2 c + b2 a + c2 b ≥ 3abc.

Multiplicando essas duas desigualdades, segue o resultado.

2a Solução. Por Cauchy-Schwarz, obtemos

a2 b + b2 c + c2 a c2 b + a2 c + b2 a ≥ (abc + abc + abc)2 = 9a2 b2 c2 .


 

6. Por Cauchy-Schwarz, obtemos


n n n
!2
X a2k X X
(ak + bk ) ≥ ak
ak + bk
k=1 k=1 k=1

n n n
!2
X a2k X X
⇔ ·2 ak ≥ ak
a k + bk
k=1 k=1 k=1

n n
X a2k 1X
⇔ ≥ ak .
ak + bk 2
k=1 k=1

a1 a2 an b1 b2 bn
7. Supondo + + ... + < n ou + + ... + < n. Pela desigualdade de
b1 b2 bn a1 a2 an
Cauchy-Schwarz e pela última hipótese, temos
  
2 a1 a2 an b1 b2 bn
n ≤ + + ... + + + ... + < n2 ,
b1 b2 bn a1 a2 an

7
POT 2012 - Álgebra - Nı́vel 2 - Aula 9 - Prof. Marcelo Mendes

um absurdo. Logo, ao menos uma das desigualdades


a1 a2 an
+ + ... + ≥ n,
b1 b2 bn
b1 b2 bn
+ + ... + ≥n
a1 a2 an
é verdadeira.

9. Utilizando o resultado do exemplo 2, obtemos


r 2 r 2 r 2 r r r r r r
bc ca ab bc ca ca ab ab bc
+ + ≥ + + = c + a + b.
a b c a b b c c a

8
Polos Olímpicos de Treinamento
Curso de Álgebra - Nível 2 Aula 10
Prof. Marcelo Mendes

Problemas Envolvendo Máximos e Mı́nimos

Vamos iniciar esta aula aplicando desigualdades aprendidas nas últimas duas aulas fo-
cando mais em exemplos envolvendo máximos e mı́nimos de funções.

Problema 1. Determine o valor máximo da função f (x) = x(1 − x)3 , sendo x ∈ (0; 1).
Solução. A ideia da solução desse problema já foi aprendida na aula 8. Vamos rever
como resolvê-lo e, mais uma vez, chamar a atenção para a diferença existente entre obter
f (x) ≤ k e garantir que k é o valor máximo de f .

Através da desigualdade entre as médias aritmética e geométrica, já que x e 1 − x são


positivos, obtemos

3x + (1 − x) + (1 − x) + (1 − x) p
≥ 4 3x(1 − x)3
4
 4
3
⇔ ≥ 3x(1 − x)3
4
27
⇔ x(1 − x)3 ≤ .
256
27
Nesse momento, a expectativa óbvia é de que deva, de fato, ser o valor máximo de
256
f , mas ainda precisamos garantir esse fato.

E como conseguiremos essa garantia? Da mesma forma que procedemos na aula 8.


27
Mostrar que o máximo de f é é equivalente a achar um valor de x ∈ (0; 1) que dê a
256
igualdade na desigualdade, e isso ocorre (graças à condição de igualdade em MA ≥ MG)
1
quando 3x = 1 − x ⇔ x = , que é um valor no intervalo (0; 1).
4
27
Portanto, realmente é o valor máximo de f .
256
POT 2012 - Álgebra - Nı́vel 2 - Aula 10 - Prof. Marcelo Mendes

Problema 2. Determine o valor máximo da função f (x) = x4 (2 − x), sendo x ∈ (0; 2).

Problema 3. Seja a um número real positivo dado. Determine o valor de x ∈ [0; a] que
maximiza o valor de f (x) = x5 (a − x).
2
Problema 4. Seja x > 0, x ∈ R. Determine o valor mı́nimo de x2 + .
x
Problema 5. (EUA) Considere a equação 3x2 − 4x + k = 0 com raı́zes reais. Determine o
valor de k para o qual o produto das raı́zes da equação seja máximo.

Problema 6. Se x, y, z são reais e satisfazem x + y + z = 5 e yz + zx + xy = 3, prove que


13
−1 ≤ z ≤ e determine o valor mı́nimo de z.
3
Solução. De (x + y + z)2 = x2 + y 2 + z 2 + 2(xy + yz + zx), obtemos x2 + y 2 + z 2 = 19.
Assim, por Caychy-Schwarz, chegamos a

(x2 + y 2 )(1 + 1) ≥ (x + y)2 ⇔ 19 − z 2 · 2 ≥ (5 − z)2




⇔ 38 − 2z 2 ≥ 25 − 10z + z 2
⇔ 3z 2 − 10z − 13 ≤ 0,
13
cuja solução é −1 ≤ z ≤ . O valor mı́nimo de z, de fato, é -1, quando x = y = 3.
3
Problema 7. Seja k uma constante real positiva. Dentre todos os triângulos tendo base a
e altura relativa a essa base h, sendo a + h = k, determine aquele(s) cuja área é máxima.

Problema 8. Sejam A, B, C os vértices de um triângulo inscrito em um cı́rculo unitário (ou


seja, cujo raio mede 1) e seja P um ponto no perı́metro do triângulo. Mostre que
32
PA · PB · PC ≤ .
27
Solução. Nesse problema, não foi pedido o valor máximo de P A · P B · P C. Mesmo assim,
32
vamos mostrar que P A·P B ·P C ≤ e, em seguida, examinar se a igualdade pode ocorrer,
27
32
ou seja, se o valor máximo de P A · P B · P C é .
27

2
POT 2012 - Álgebra - Nı́vel 2 - Aula 10 - Prof. Marcelo Mendes

B P C

Pela potência do ponto P em relação a (ABC) (ABC entre parênteses representa o a


circunferência que passa pelos pontos A, B e C), temos

P B · P C = P A · P X.
Daı́, utilizando a desigualdade entre as médias aritmética e geométrica, obtemos
 3
2 P A + P A + 2P X
2P A · P B · P C = P A · 2P X ≤
3

2AX 3
   3
4
= ≤
3 3
32
⇒ PA · PB · PC ≤ ,
27
utilizando que AX é uma corda e, portanto, tem medida menor que ou igual à medida do
diâmetro, que é 2.

Já conseguimos chegar ao resultado pedido no enunciado. Agora, vamos verificar se é


32
possı́vel obtermos o valor máximo .
27
Essa igualdade ocorre se, e somente se, P está sobre o diâmetro que passa por A e
4
P A = 2P X = , que depende do triângulo ABC inicial. Portanto, nem sempre a igual-
3
dade ocorre.

3
POT 2012 - Álgebra - Nı́vel 2 - Aula 10 - Prof. Marcelo Mendes

Problema 9. Sejam a e b números reais positivos. Ache o valor máximo da função real e
de variável real
x
y= 2 .
ax + b
Solução. Podemos escrever a equação acima da seguinte forma

ayx2 − x + by = 0,
cujo discriminante é

∆ = 1 − 4aby.
1
Como x ∈ R, temos ∆ ≥ 0, ou seja, y ≤ .
4ab
1
Assim, é o nosso candidato a valor máximo da função. Para esse valor ser atingido,
4ab
devemos ter ∆ = 0 e, portanto
1
x= = 2b.
2ay
Pense também em uma solução começando com MA ≥ MG entre ax2 e b.

Problema 10. Seja P um ponto no interior de um triângulo A1 A2 A3 e P1 , P2 , P3 , os pés


das perpendiculares de P a A2 A3 , A3 A1 , A1 A2 . Localize o ponto P tal que

A1 A2 A2 A3 A3 A1
+ + .
P P3 P P1 P P2
seja mı́nimo.
Solução. Vamos pensar um pouquinho. As frações envolvidas nessa soma relacionam ba-
ses e alturas (pense sempre que distâncias lembram alturas e que alturas lembram área)
dos triângulos A2 P A3 , A3 P A1 , A1 P A2 e, portanto, nos fazem pensar nas áreas desses
triângulos, e a área do triângulo A1 A2 A3 será a soma dessas áreas.

Pela desigualdade de Cauchy-Schwarz, temos


 
A1 A2 A2 A3 A3 A1
+ + (A1 A2 · P P3 + A2 A3 · P P1 + A3 A1 · P P2 )
P P3 P P1 P P2

≥ (A1 A2 + A2 A3 + A3 A1 )2 .
Como
A1 A2 · P P3 + A2 A3 · P P1 + A3 A1 · P P2 = 2S
e
A1 A2 + A2 A3 + A3 A1 = p,
sendo S a área e p o perı́metro do triângulo A1 A2 A3 , chegamos a

4
POT 2012 - Álgebra - Nı́vel 2 - Aula 10 - Prof. Marcelo Mendes

A1 A2 A2 A3 A3 A1 p2
+ + ≥ .
P P3 P P1 P P2 2S
A1 A2 A2 A3 A3 A1 p2
Portanto, o candidato a valor mı́nimo de + + é .
P P3 P P1 P P2 2S
Esse valor mı́nimo será atingido se a igualdade ocorrer na desigualdade. A igualdade
na desigualdade de Cauchy-Schwarz ocorre com a proporção entre as respectivas parcelas
das somas envolvidas, ou seja,

A1 A2 A2 A3 A3 A1
P P3 P P1 P P2
= =
A1 A2 · P P3 A2 A3 · P P1 A3 A1 · P P2

⇔ P P1 = P P2 = P P3 ,
de onde segue que o valor mı́nimo é atingido e é quando P é o incentro do triângulo A1 A2 A3 .

5
POT 2012 - Álgebra - Nı́vel 2 - Aula 10 - Prof. Marcelo Mendes

Dicas

2. Repita as ideias da solução do problema 1.

3. Repita as ideias da solução do problema 1.

4. Repita as ideias da solução do problema 1.


4 k 4
5. Denote por x1 e x2 as raı́zes. Assim, x1 + x2 = e x1 x2 = . Escreva x2 = − x1
3 3 3
e repita as ideias da solução do problema 1.

7. Use h = k − a e repita as ideias da solução do problema 1.

6
POT 2012 - Álgebra - Nı́vel 2 - Aula 10 - Prof. Marcelo Mendes

Soluções

2. Por MA ≥ MG, temos

x + x + x + x + 4(2 − x) p
≥ 5 4x4 (2 − x)
5
 5
4 1 8 49
⇒ x (2 − x) ≤ · = 5.
4 5 5

3. Por MA ≥ MG, temos

x + x + x + x + x + 5(a − x) p
≥ 6 5x5 (a − x)
6
 6
1 5a  a 6
⇒ x5 (a − x) ≤ · = 55 .
5 6 6

7. Por MA ≥ MG, temos

a + (k − a) p
≥ a(k − a)
2

a(k − a) k2
⇒ ≤ ,
2 8
k k
com igualdade se, e somente se, a = k −a, ou seja, a = . Nesse caso, h = também,
2 2
o que determina os triângulos com área máxima.

7
Polos Olímpicos de Treinamento
Curso de Álgebra - Nível 2 Aula 11
Prof. Marcelo Mendes

Funções Definidas Implicitamente - Parte I

Talvez a experiência de alguns de vocês diga que as soluções de uma equação devam
ser necessariamente números. Mas isso não é verdade. Em matemática, podemos ter, por
exemplo, matrizes ou funções como soluções para equações matriciais ou funcionais, res-
pectivamente.

Nesta aula, vamos aprender resolver algumas equações funcionais, que têm funções como
soluções e, por isso, dizemos que essas funções foram definidas implicitamente (implı́cito
significa escondido).

Problema 1. Determine todas as funções f : Z+ → Z+ tais que f (1) = c e

f (x + y) = f (x) + f (y), ∀x, y ∈ Z+ .

Solução. Nesse problema, a equação funcional é f (x + y) = f (x) + f (y). Precisamos


resolvê-la.

Observe que o problema nos permite utilizarmos quaisquer valores inteiros não-negativos
para x e y. Assim, vamos iniciar com x = y = 0:

f (0 + 0) = f (0) + f (0) ⇒ f (0) = 0.


Agora, façamos a escolha de deixar x variável e y = 1:

f (x + 1) = f (x) + f (1) = f (x) + c.


Acabamos de gerar uma equação de diferença, tipo explorado na aula de somas te-
lescópicas. Vamos escrever várias dessas equações, variando n ≥ 2:

f (n) = f (n − 1) + c
f (n − 1) = f (n − 2) + c
:
POT 2012 - Álgebra - Nı́vel 2 - Aula 11 - Prof. Marcelo Mendes

f (2) = f (1) + c
e somá-las, obtendo

f (n) = cn.
Assim, a (única) solução é f (x) = cx, sendo c = f (1). (Poderı́amos, também, ter aplicado
a fórmula do termo geral da P.A.)

Problema 2. Seja f : R → R uma função tal que f (0) = 1 e, para quaisquer x, y ∈ R,

f (xy + 1) = f (x)f (y) − f (y) − x + 2.

Determine o valor de f(2012).

Problema 3. Seja f : R∗+ → R uma função satisfazendo a equação funcional

f (a) + f (b) = f (ab), ∀a, b ∈ R∗+ .

Mostre que:

a) f (1) = 0.

b) f (an ) = n · f (a), ∀a ∈ R∗+ , ∀n ∈ N.


 
1
c) f = −f (a), ∀a ∈ R∗+ .
a
Solução.

a) Com a = b = 1, obtemos f (1) + f (1) = f (1 · 1), ou seja, f (1) = 0.

b) Observe que a equação funcional dada nos dá permição para operar apenas com 2
números (a e b). Podemos mostrar, utilizando indução, que também será possı́vel operar
com qualquer quantidade finita (no mı́nimo 2) de números. Supondo ser possı́vel para
k números, ou seja, que

f (a1 ) + ... + f (ak ) = f (a1 · ... · ak ) ,

podemos garantir que

f (a1 ) + ... + f (ak ) + f (ak+1 ) = f (a1 · ... · ak ) + f (ak+1 ) = f (a1 · ... · ak · ak+1 ) .

Assim, podemos escrever

f (an ) = f (a ... · a}) = f (a) + ... + f (a) = n · f (a).


| · {z | {z }
n n

2
POT 2012 - Álgebra - Nı́vel 2 - Aula 11 - Prof. Marcelo Mendes

     
1 1 1
c) f (a) + f =f a· = f (1) = 0, ou seja, f = −f (a).
a a a

Problema 4. Seja f : Z → Z uma função satisfazendo

f (n2 ) = f (n + m)f (n − m) + m2 , ∀m, n ∈ Z.

Determine o conjunto de todos os possı́veis valores de f (0).


Problema 5. Seja f uma função com duas propriedades:
i. f (x + y) = x + f (y), ∀x, y ∈ R;

ii. f (0) = 2.
Determine o valor de f (2012).
f (x)
Problema 6. (EUA) Seja f uma função satisfazendo f (xy) = para todos os números
y
reais positivos x e y. Se f (500) = 3, qual é o valor de f (600)?
1 + f (x)
Problema 7. Para todos os inteiros x, a função f satisfaz f (x+1) = . Se f (1) = 2,
1 − f (x)
calcule f (2012).

Solução. Com:
1+3
x = 1, obtemos f (2) = = −2;
1−3
1−2 1
x = 2, obtemos f (3) = =− ;
1+2 3
1
1− 3 1
x = 3, obtemos f (4) = 1 = ;
1+ 3
2
1
1+ 2
x = 4, obtemos f (5) = 1 = 3.
1− 2

Como o valor 3 apareceu novamente e a regra que calcula cada novo valor é sempre a
1 1
mesma, os valores vão sempre se repetir de 4 em 4 na sequência 3, −2, − , . Por 2012 ser
3 2
1
múltiplo de 4, segue que f (2012) = f (4) = .
2
Problema 8. Determine todas as funções f : R → R satisfazendo

f (x)f (y) − f (xy) = x + y, ∀x, y.

Solução. Atribuindo o valor 0 a x e a y, temos (f (0))2 − f (0) = 0. Assim, f (0) = 0 ou


f (0) = 1.

3
POT 2012 - Álgebra - Nı́vel 2 - Aula 11 - Prof. Marcelo Mendes

Suponha f (0) = 0 e atribua 0 apenas à variável y. Daı́, f (x) · 0 − 0 = x, ∀x ∈ R,


absurdo. Portanto, f (0) = 1.

Fazendo y = 0, obtemos f (x)f (0) − f (0) = x + 0, ou seja, f (x) = x + 1, que é a única


solução. (Substitua essa solução na equação funcional para verificar a igualdade!)

Problema 9. Considere uma função f definida no conjunto dos números naturais tal que

f (n + 2) = 3 + f (n), ∀n ∈ N, f (0) = 10, f (1) = 5.


p
Qual o valor de f (81) − f (70)?

Problema 10. Seja f : Q∗+ → Q∗+ uma função tal que

f (x)
f (xf (y)) = , ∀x, y ∈ Q∗+ .
y
1
Mostre que f (f (x)) = , ∀x ∈ Q∗+ .
x
Solução. Temos

f (y) 1
y · f (xf (y)) = f (x) ⇒ f (f (x)) = f (y · f (xf (y))) = = .
xf (y) x

Problema 11. A função f é definida para todos os pares ordenados (x, y) de inteiros posi-
tivos e tem as seguintes propriedades:

i. f (x, x) = x,

ii. f (x, y) = f (y, x),

iii. (x + y)f (x, y) = yf (x, x + y).

Qual é o valor de f (22, 55)?

Solução. De iii, obtemos

55f (22, 33) = 33f (22, 55)


33f (22, 11) = 11f (22, 33)
22f (11, 11) = 11f (11, 22),
utilizando ii no lado direito da última equação. Multiplicando-as, obtemos:

55 · 33 · 22f (11, 11) = 33 · 11 · 11f (22, 55)


⇒ f (22, 55) = 110.

4
POT 2012 - Álgebra - Nı́vel 2 - Aula 11 - Prof. Marcelo Mendes

Problema 12. A função f , definida sobre o conjunto de todos os pares ordenados de inteiros
positivos, satisfaz as seguintes propriedades:
i. f (x, x) = x;
ii. f (x, y) = f (y, x);
iii. (x + y)f (x, y) = yf (x, x + y).
Calcule f (14, 52).
Problema 13. Se
f (n + 1) = (−1)n+1 · n − 2f (n)
para os inteiros n ≥ 1 e f (1) = f (2013), determine valor da soma

f (1) + f (2) + f (3) + ... + f (2012).


Problema 14. Determine todas as funções f : Z+ → Z+ , injetoras, satisfazendo
f (m + f (n)) = f (f (m)) + f (n), ∀m, n ∈ Z+ .
Solução. Atribuindo o valor 0 às variáveis m e n, temos

f (f (0)) = f (f (0)) + f (0) ⇒ f (0) = 0.


Agora, atribuindo o valor 0 apenas à variável n, obtemos

f (m) = f (f (m)).
Como f é injetora, segue que f (m) = m, ∀m. (Se f é uma função injetora, então
f (x) = f (y) implica x = y.)
Problema 15. Considere a equação funcional
f (xy) = f (x) + f (y), ∀x, y ∈ Dom(f ).
Mostre que se 0 ∈ Dom(f ), então existe uma única solução para a equação dada.
Problema 16. Seja f : N → N uma função estritamente crescente (isto é, x < y ⇒ f (x) <
f (y)) tal que f (2) = 2 e f (mn) = f (m)f (n) para todo par de inteiros positivos m e n
primos entre si. Determine o valor de f (3).

Solução. Inicialmente, observe que o problema não permite escrever f (4) = f (2)f (2) = 4,
pois o máximo divisor comum entre 2 e 2 não é 1.
i. 2 e 5 são primos entre si: f (10) = f (2)f (5) = 2f (5).
ii. 2 e 9 são primos entre si: f (18) = f (2)f (9) = 2f (9).
iii. 3 e 5 são primos entre si: f (15) = f (3)f (5).
iv. f é estritamente crescente: f (9) < f (10), f (15) < f (18) e f (3) > f (2) = 2.
Logo, 4f (5) = 2f (10) > 2f (9) = f (18) > f (15) = f (3)f (5) e, portanto, f (3) < 4.
Assim, 2 < f (3) < 4, ou seja, f (3) = 3.

5
POT 2012 - Álgebra - Nı́vel 2 - Aula 11 - Prof. Marcelo Mendes

Dicas

2. Calcule f (1). Depois faça y = 1.

4. Faça m = n = 0. Em seguida, faça m = n = 2.

5. Faça x = 1, y = 0. Depois faça y = 1 e deixe x variável.

6. Faça x = 100 e y = 5.

9. Calcule as imagens de números pares e ı́mpares separadamente.

12. Repita a solução do problema 11.

13. Faça n = 1, 2, ..., 2011.

15. Faça y = 0.

6
POT 2012 - Álgebra - Nı́vel 2 - Aula 11 - Prof. Marcelo Mendes

Respostas e Soluções

2. Com x = y = 0, obtemos f (1) = 2. Com y = 1, segue

f (x + 1) = f (x)f (1) − f (1) − x + 2 = 2f (x) − x.

Com x = 1 e x = 2, obtemos, respectivamente:

f (2) = 2 · 2 − 1 = 3,
f (3) = 2 · 3 − 2 = 4.

Vamos mostrar por indução que f (x) = x + 1, ∀x ∈ Z+ . Os casos iniciais já foram
verificados. Além disso, f (x+1) = 2(x+1)−x = x+2. Em particular, f (2012) = 2013.

4. Inicialmente, com m = n = 0 temos f (0) = (f (0))2 ⇒ f (0) = 0 ou f (0) = 1. Por


outro lado, com m = n = 2 obtemos f (4) = f (4)f (0) + 4, que mostra que f (0) = 1.

5. Com x = 1, y = 0, achamos f (1) = f (1 + 0) = 1 + f (0) = 3. Em seguida, escolhemos


apenas y = 1, obtemos f (x + 1) = x + f (1) = x + 3, que é uma P.A. se x ∈ Z+ .
Assim, f (2012) = f (0) + 2012 · 3 = 6038.
f (100) f (100) 5
6. f (500) = ⇒ f (100) = 15. Logo, f (600) = = .
5 6 2
9. Somando

f (2) = 3 + f (0)
:
f (70) = 3 + f (68)

obtemos f (70) = 3 · 35 + 10 = 115. E pela soma de

f (3) = 3 + f (1)
:
f (81) = 3 + f (79)
p √
obtemos f (81) = 3 · 40 + 5 = 125. Logo, f (81) − f (70) = 10.

12. 364.

7
POT 2012 - Álgebra - Nı́vel 2 - Aula 11 - Prof. Marcelo Mendes

13. Com n = 1, 2, ..., 2012, temos:

f (2) = 1 − 2f (1)
f (3) = −2 − 2f (2)
:
f (2013) = −2012 − 2f (2012).

Sendo f (1) = f (2013) e S = f (1) + ... + f (2012), obtemos:

1006
S = (1 − 2) + ... + (2011 − 2012) − 2S ⇒ S = − .
3
15. Com y = 0, obtemos f (x) = 0, ∀x.

8
Polos Olímpicos de Treinamento
Curso de Álgebra - Nível 2 Aula 12
Prof. Marcelo Mendes

Funções Definidas Implicitamente - Parte II

Nesta segunda aula sobre funções definidas implicitamente, aprofundaremos alguns


exercı́cios vistos na aula anterior e veremos exemplos envolvendo funções compostas, ou
seja, funções de outras funções.

Iniciaremos resolvendo o problema 1 da aula anterior mas, agora, com domı́nio racional.

Problema 1. Determine todas as funções f : Q → Q tais que f (1) = c e

f (x + y) = f (x) + f (y), ∀x, y ∈ Q.

Solução. Já vimos na aula anterior que f (x) = cx quando x é inteiro não-negativo.

Agora, suponha x inteiro negativo. Fazendo y receber o valor contrário de x (assim,


y = −x > 0), obtemos

0 = f (0) = f (x) + f (−x)(∗)


⇒ f (x) = −f (−x) = −(c(−x)) = cx,
e o resultado também é válido para inteiros negativos.

Suponha, agora, x inteiro positivo. Podemos escrever


 
     
1 1 1 1 1
f (1) = f  + ... +  = f + ... + f = xf
x
| {z x} x x x
| {z }
x x
 
1 1
f =c· ,
x x
lembrando que no problema 3 da aula anterior já mostramos por indução para uma equação
funcional semelhante que podemos aplicar a regra, dada no problema para 2 números, para
POT 2012 - Álgebra - Nı́vel 2 - Aula 12 - Prof. Marcelo Mendes

 
1 1
qualquer quantidade finita de números. Por (*), segue que f = c · , ∀x ∈ Z∗ .
x x
p
Seja agora x racional não-nulo, isto é, x = , com p e q inteiros, sendo p > 0. Temos
q
 
  1
p  1
f (x) = f = f  + ... + 
q q q
| {z }
p
     
1 1 1 1
=f + ... + f = pf = p · c · = cx,
q q q q
| {z }
p
finalizando a solução.
Problema 2. Determine todas as funções f : Q → Q tais que f (1) = a e
f (x + y) = f (x) · f (y), ∀x, y ∈ Q.
Problema 3. Determine todas as funções f : R → R satisfazendo
f (x) + f (2x + y) + 5xy = f (3x − y) + 2x2 + 1, ∀x, y ∈ R.
Solução. Fazendo x = 2y, obtemos 2x + y = 3x − y e

5x2 x2
f (x) += 2x2 + 1 ⇒ f (x) = 1 − .
2 2
Problema 4. Seja f : R → R uma função satisfazendo a equação funcional

f (x − y)2 = (f (x))2 − 2x · f (y) + y 2 .
Determine todos os possı́veis valores de f (0).
Problema 5. (ITA) Sejam f, g : R → R funções tais que g(x) = 1 − x e
f (x) + 2f (2 − x) = (x − 1)3 , ∀x ∈ R.
Determine f (g(x)).

Solução. A equação funcional em questão possui uma composição de funções em f (2 − x).


Sendo h(x) = 2 − x, temos f (2 − x) = f (h(x)). Essa função h é a chave do problema.
Reescreva a equação substituindo x por h(x) = 2 − x, ou seja, x ← 2 − x (lê-se: x recebe
2 − x):

f (2 − x) + 2f (x) = (1 − x)3 .
Dessa e da equação dada, obtemos f (x) = (1 − x)3 e, portanto,

f (g(x)) = (1 − (1 − x))3 = x3 .
Obs: h(h(x)) = 2 − h(x) = x. Por isso são suficientes apenas 2 equações para encontrarmos
f (x).

2
POT 2012 - Álgebra - Nı́vel 2 - Aula 12 - Prof. Marcelo Mendes

 
 1 
Problema 6. Se 2f x2 + 3f = x2 − 1, determine f x2 .
x2

Problema 7. Se f (x) + 2f (1 − x) = x2 + 2, ∀x ∈ R, determine f (x).


 
1
Problema 8. Determine f sabendo que f (x) + f = x, x 6= 1.
1−x
1
Solução. Novamente, a chave do problema está em fazer x ← . Nesse caso, temos
1−x
1 1 x−1
← = . Assim:
1−x 1 x
1−
1−x    
1 x 1
f +f = .
1−x x−1 1−x
Mas diferentemente dos problemas mais simples, envolvendo funções mais simples, não
podemos combinar apenas essa última equação com a equação dada porque ainda não ob-
tivemos outro f (x).
1 x−1
Assim, vamos novamente promover a substituição x ← , que nos dá ←x
1−x x
(verifique!). Logo:
 
x x−1
f + f (x) = .
x−1 x
Combinando as equações, obtemos
1 x−1
2f (x) = x − +
1−x x

x3 − x + 1
⇒ f (x) = .
2x(x − 1)
1 x−1
Obs: h(x) = ⇒ h(h(x)) = e h(h(h(x))) = x. Por isso são necessárias 3
1−x x
equações para encontrarmos f (x).

Problema 9. Determine a função f : R → R satisfazendo à equação funcional

x2 f (x) + f (1 − x) = 2x − x4 .
 
1−x
Problema 10. (Ibero) Ache todas as f tais que (f (x))2 ·f = 64x, para todo x
1+x
distinto de 0, 1 e -1.

Problema 11. (EUA) A função  fnão está definida para x = 0, mas para todos os valores de
1
x não-nulos temos f (x) + 2f = 3x. Determine as soluções da equação f (x) = f (−x).
x

3
POT 2012 - Álgebra - Nı́vel 2 - Aula 12 - Prof. Marcelo Mendes

Problema 12. Seja f uma função cujo domı́nio é o conjunto de todos os números reais. Se
 
x + 2011
f (x) + 2f = 4033 − x
x−1

para todo x 6= 1, encontre o valor de f (2013).


 
x−3
Problema 13. Encontre todas as funções f : R − {−1, 1} → R satisfazendo f +
  x+1
x+3
f = x, ∀x 6= ±1.
x−1
Solução. Faça x ← −x:
   
−x − 3 −x + 3
f +f = −x
−x + 1 −x − 1
   
x+3 x−3
⇔f +f = −x, ∀x 6= ±1.
x−1 x+1
Portanto, x = −x, ∀x 6= ±1, absurdo. Logo, não existe função f cumprindo a equação
funcional dada.

Problema 14. Determine todas


 as funções f reais, de variável real positiva, satisfazendo a
2002
condição f (x) + 2f = 3x.
x
Problema 15. Determine todas as funções f : R → R tais que 2f (x) + f (1 − x) = 1 + x.

4
POT 2012 - Álgebra - Nı́vel 2 - Aula 12 - Prof. Marcelo Mendes

Dicas

2. Repita os procedimentos do problema 1.

4. Faça x = y = 0.
1
6. Faça x ← .
x
7. Faça x ← 1 − x.

9. Faça x ← 1 − x.
1−x
10. Faça x ← .
1+x
1
11. Faça x ← .
x
x + 2011
12. Faça x ← .
x−1
2002
14. Faça x ← .
x
15. Faça x ← 1 − x.

5
POT 2012 - Álgebra - Nı́vel 2 - Aula 12 - Prof. Marcelo Mendes

Respostas e Soluções

2. f (x) = ax .

4. f (0) = 0 ou f (0) = 1.
 
1 − x2 3 + 2x2
6. f (x) = .
5x2
(x − 2)2
7. f (x) =
3
9. Fazendo x ← 1 − x e combinando com a equação dada, chegamos a f (x) = 1 − x2 .
r
2
3 x (1 + x)
10. f (x) = 4 .
1−x
1
11. Fazendo x ← e combinando o resultado com a equação inicial, obtemos f (x) =
x
2 √
− x. Assim, as soluções de f (x) = f (−x) são ± 2.
x
12. 2014.
2002 4004
14. Fazendo x ← e combinando com a equação inicial, obtemos f (x) = − x.
x x
15. f (x) = x.

6
Polos Olímpicos de Treinamento
Curso de Álgebra - Nível 2 Aula 13
Prof. Marcelo Mendes

Revisão - Parte I

Como o tı́tulo indica, faremos uma breve revisão de temas já abordados em nosso trei-
namento, a fim de consolidar conceitos e ideias importantes para a fase final da OBM.

Iniciaremos resolvendo os problemas das OBMs passadas, propostos na 1a aula sobre


produtos notáveis, mas não resolvidos.

Ao final, estão propostos mais alguns problemas semelhantes aos tratados até agora.

x2 y2
Problema 1. (OBM 1a fase/2002) Se xy = 2 e x2 + y 2 = 5, então + + 2 vale:
y2 x2
5
a)
2
25
b)
4
5
c)
4
1
d)
2
e) 1

x2 y2 x4 + y 4 + 2x2 y 2 (x2 + y 2 )2 25
Solução. Veja que 2
+ 2
+ 2 = 2 2
= 2 2
= . Letra B.
y x x y x y 4

Problema 2. (OBM 3a fase/2003) Mostre que x2 + 4y 2 − 4xy + 2x − 4y + 2 > 0 quaisquer


que sejam os reais x e y.

Solução. Procurando agrupar os termos para obter uma fatoração, podemos escrever

x2 + 4y 2 − 4xy + 2x − 4y + 2 = x2 − 4xy + 4y 2 + 2(x − 2y) + 2


POT 2012 - Álgebra - Nı́vel 2 - Aula 13 - Prof. Marcelo Mendes

= (x − 2y)2 + 2(x − 2y) + 1 + 1 = [(x − 2y) + 1]2 + 1 ≥ 1 > 0,


pois [(x − 2y) + 1]2 é não-negativo já que é o quadrado de um número real.

Problema 3. (OBM 2a fase/2005)

a) Fatore a expressão x2 − 9xy + 8y 2 .

b) Determine todos os pares de inteiros (x; y) tais que 9xy − x2 − 8y 2 = 2005.

Solução.

a) x2 − 9xy + 8y 2 = x2 − xy − 8xy + 8y 2 = x(x − y) − 8y(x − y) = (x − y)(x − 8y).

b) Pelo item a, podemos escrever

(x − y)(8y − x) = 2005 = 5 · 401,

e 401 é primo. Agora vejamos as possibilidades:



x−y = 5
i. ⇒ x = 63, y = 58.
8y − x = 401

x−y = −5
ii. ⇒ x = −63, y = −58.
8y − x = −401

x−y = 401
iii. ⇒ x = 459, y = 58.
8y − x = 5

x−y = −401
iv. ⇒ x = −459, y = −58,
8y − x = −5

que são todos os casos.

Problema 4. (OBM 1a fase/2005) Os inteiros positivos x e y satisfazem a equação


r r
1√ 1√
x+ y− x− y = 1.
2 2
Qual das alternativas apresenta um possı́vel valor de y?

a) 5

b) 6

c) 7

d) 8

e) 9

2
POT 2012 - Álgebra - Nı́vel 2 - Aula 13 - Prof. Marcelo Mendes

√ √
q q
Solução. Talvez fiquem interessantes as substituições x + 21 y = A e x − 12 y = B.

2 2 √ √ y+1
Assim, temos A − B = 1 e A − B = y, que implica A + B = y. Daı́, A = e,
2
portanto:

1√ y+2 y+1 y + 1 1√
x+ y= = + y
2 4 4 2
⇒ y = 4x − 1.
Dentre as opções, a única que deixa resto ’-1’ na divisão por 4 é 7. Letra C.

Problema 5. (OBM 3a fase/2006) Encontre todos os pares ordenados (x; y) de inteiros tais
que x3 − y 3 = 3(x2 − y 2 ).

Solução. Fatorando, obtemos (x − y)(x2 + xy + y 2 ) = 3(x − y)(x + y).

Assim, temos solução quando x = y, x ∈ R.

Se x 6= y, então x2 +xy +y 2 = 3(x+y) ⇔ x2 +x(y −3)+y 2 −3y = 0, cujo discriminante é


∆ = −3y 2 + 6y + 9. Para que as raı́zes sejam reais, devemos ter ∆ ≥ 0, ou seja, −1 ≤ y ≤ 3.
Testando, apenas y = 1 não gera ∆ quadrado perfeito. Os demais valores dão as seguintes
soluções:

y = −1, ∆ = 0, x = 2;
y = 0, ∆ = 9, x = 0; x = 6;
y = 2, ∆ = 9, x = 2; x = −1;
y = 3, ∆ = 0, x = 0.
S
Resposta: {(2, 0); (0, 0); (6, 0); (2, 2); (2, −1); (3, 0)} {(x, x), x ∈ R}.

Problema 6. (OBM 2a fase/2006) Sejam a e b números reais distintos tais que a2 = 6b+5ab
e b2 = 6a + 5ab.

a) Determine o valor de a + b.

b) Determine o valor de ab.

Solução.

a) Subtraindo as equações membro a membro, obtemos a2 − b2 = 6(b − a). Como a e b são


distintos, chegamos a a + b = 6.

b) Agora, somando as equações membro a membro, temos a2 + b2 = 6(a + b) + 10ab ⇔


(a + b)2 = 6(a + b) + 12ab, o que dá ab = 0.

3
POT 2012 - Álgebra - Nı́vel 2 - Aula 13 - Prof. Marcelo Mendes

Problema 7. (OBM 2a fase/2008) Sejam x e y números reais positivos satisfazendo as


equações x2 + y 2 = 1 e x4 + y 4 = 17 1
18 . Calcule o valor de xy .

Solução. De x2 + y 2 = 1, obtemos
17 1 1
x4 + y 4 + 2x2 y 2 = 1 ⇔ 2x2 y 2 = 1 −
= ⇔ x2 y 2 = .
18 18 36
1 1
Como x e y são positivos, segue que xy = e = 6.
6 xy

Problema 8. (OBM 1a fase/2010) Quantos são os pares (x, y) de inteiros positivos tais que
x2 − y 2 = 22010 ?

a) 1000

b) 1001

c) 1002

d) 1003

e) 1004

Solução. Temos (x + y)(x − y) = 22010 . Como x + y > 0, então x − y > 0. Além disso,
x + y > x − y (pois y > 0) e x + y deve ter a mesma paridade de x − y.

Assim, as possibilidades são

x + y = 22009 , 22008 , ... , 21006



,
x−y = 2 , 22 , ... , 21004
que geram 1004 pares ordenados. Letra E.

Problema 9. (OBM 3a fase/2010) Sejam a, b e c reais tais que a 6= b e a2 (b+c) = b2 (c+a) =


2010. Calcule c2 (a + b).

Solução. De a2 (b + c) = b2 (c + a) = 2010, obtemos

a2 b − b2 a + a2 c − b2 c = 0
⇔ ab(a − b) + c(a + b)(a − b) = 0
⇔ (a − b)(ab + bc + ca) = 0.
Como a 6= b, concluı́mos que ab + bc + ca = 0.
Agora, vamos fazer o mesmo com b2 (c + a) = 2010 e c2 (a + b) = k. Subtraindo obtemos

b2 c − c2 b + b2 a − c2 a = 2010 − k

4
POT 2012 - Álgebra - Nı́vel 2 - Aula 13 - Prof. Marcelo Mendes

⇔ bc(b − c) + a(b + c)(b − c) = 2010 − k


⇔ (b − c)(ab + bc + ca) = 2010 − k.
Mas ab + bc + ca = 0. Assim, k = 2010.

Problema 10. (OBM 1a fase/2011) Qual é o valor da expressão 201120112 + 201120032 −


16 × 20112007?
a) 2 × 201120072
b) 2 × 201120032

c) 2 × 20112007

d) 2 × 20112003
e) 2 × 201120112
Solução. Para simplificar os cálculos, vamos escrever a = 20112007. Assim, a expressão
desejada será

(a + 4)2 + (a − 4)2 − 16a = 2(a2 − 8a + 16) = 2(a − 4)2 = 2 × 201120072 .


Letra A.

Problema 11. (IMO-Adaptado) Sejam k, m, n números naturais. Defina cs = s(s + 1).

a) Fatore cj − ck , sendo j ∈ N.
c1 · c2 · ... · cn
b) Mostre que (cm+1 − ck )(cm+2 − ck )...(cm+n − ck ) é divisı́vel por .
n+1
Problema 12. Determine todos os valores de x para os quais (1999x − 99)3 = (1234x −
56)3 + (765x − 43)3 .

Problema 13. Simplifique a expressão

22 − 1 32 − 1 42 − 1 20122 − 1
S= × × × ... × .
22 32 42 20122

p
3
√ p
3

Problema 14. Mostre que 20 + 14 2 + 20 − 14 2 = 4.

Problema 15. Determine todas as soluções inteiras da equação

2(x + y) = xy + 7.

5
POT 2012 - Álgebra - Nı́vel 2 - Aula 13 - Prof. Marcelo Mendes

Dicas

11. Use o fato de que o produto de n inteiros consecutivos é divisı́vel por n!.

12. Resolva a equação (a + b)3 = a3 + b3 .

13. Fatore os numeradores, que são diferenças de quadrados.

14. Eleve ao cubo a expressão no lado esquerdo.

15. Encontre k tal que xy − 2x − 2y + k possa ser fatorado.

Respostas

99 43 56
12. , , .
1999 65 1234
2013
13. .
4024
15. {(x, y) = (3, −1), (1, 5), (5, 1), (−1, 3)}.

6
Polos Olímpicos de Treinamento
Curso de Álgebra - Nível 2 Aula 14
Prof. Marcelo Mendes

Revisão - Parte II

Continuando nossa breve revisão de temas já abordados, propomos mais problemas de
equações e sistemas de equações.

Problema 1. Se x ∈ R e 4y 2 + 4xy + x + 6 = 0, determine:


a) o conjunto de todos os valores de x para os quais y ∈ R;

b) y em função dos valores de x encontrados no item anterior.


4

4
Problema 2. Encontre todas as soluções reais da equação 13 + x + 4 − x = 3.

r
x2 + 3 3
r
x
Problema 3. Determine o conjunto solução da equação − = , x ∈ R.
x x2 +3 2

Problema 4. Sejam x, y, z números reais tais que


1 y 2
= = .
xy z−x+1 z+1
Prove que um desses números é a média aritmética dos outros dois.
Problema 5. Prove que a equação

an + 2012 · bn = cn+1

tem infinitas soluções naturais a, b, c para todo inteiro positivo n.

Problema 6. (OBM) Mostre que a equação

x3 + 1990y 3 = z 4

possui infinitas soluções inteiras com x > 0, y > 0, z > 0.


POT 2012 - Álgebra - Nı́vel 2 - Aula 14 - Prof. Marcelo Mendes

Problema 7. Determine o conjunto solução da equação x = ⌊1 − x⌋, onde ⌊a⌋ representa a


parte inteira de a.

 p 2
 px − y = z − 1
Problema 8. Quantas soluções reais possui o sistema y2 − z = x − 1 ?
 √ 2
z −x = y−1

Problema 9. (Banco IMO) Encontre todas as triplas de inteiros positivos x, y, z satisfazendo


1 1 1 4
+ + = .
x y z 5
4 1 1 1 3
Solução. Suponha, sem perda de generalidade, que x ≤ y ≤ z. Daı́, = + + ≤ ,
5 x y z x
1 4
ou seja, x ≤ 3. Também < ⇒ x ≥ 2. Assim, precisamos analisar os casos x = 2 e
x 5
x = 3.

i) x = 2:
1 1 4 1 3 10z
+ = − = ⇒y=
y z 5 2 10 3z − 10

30z 30z − 100 + 100 100


⇒ 3y = = = 10 + .
3z − 10 3z − 10 3z − 10
Como 3y ∈ Z, segue que (3z − 10) 100 (essa notação significa 3z − 10 divide 100) e
3z − 10 > 0. Assim, temos as possibilidades

3z − 10 = 1, 2, 4, 5, 10, 20, 25, 50, 100.

Mas z ∈ Z. Logo, restam

3z − 10 = 2, 5, 20, 50
⇒ z = 4, 5, 10, 20
⇒ y = 20, 10, 5, 4,

que geram as soluções (2, 4, 20), (2, 5, 10), (2, 10, 5), (2, 20, 4).

ii) x = 3:
1 1 4 1 7 15z
+ = − = ⇒y=
y z 5 3 15 7z − 15

105z 105z − 225 + 225 225


⇒ 7y = = = 15 + .
7z − 15 7z − 15 7z − 15

2
POT 2012 - Álgebra - Nı́vel 2 - Aula 14 - Prof. Marcelo Mendes

Como 7y ∈ Z, segue que (7z − 15) 225 e 7z − 15 > 0. Assim, temos as possibilidades

7z − 15 = 1, 3, 5, 15, 45, 75, 225,


7z = 16, 18, 20, 30, 60, 90, 240,

que não produzem solução inteira.

Assim, as únicas soluções são (2, 4, 20), (2, 5, 10), (2, 10, 5), (2, 20, 4).

Comentário. Existe um problema muito parecido com esse, proposto no livro das Olimpı́adas
Brasileiras de Matemática - 1a à 8a , que possui o seguinte enunciado:

Mostre que o número de soluções x, y, z de inteiros positivos da equação


1 1 1 1
+ + =
x y z 1983
é finito.

Tente resolvê-lo a partir das ideias do problema 9!

Problema 10. Determine todas as soluções em números reais x, y, z, w do sistema de


equações 
 x + y + z = w
1 1 1 1 .
+ + =
x y z w

Problema 11. (Romênia) Os números reais não-nulos x, y, z, t verificam as seguintes igual-


dades 

 x + y + z = t
 1 1 1 1
+ + = .
 x y z t
 x3 + y 3 + z 3 =

10003
Determine o valor da soma x + y + z + t.

Problema 12. Seja n um dado inteiro positivo. Quantas soluções existem em pares orde-
nados (x, y) de inteiros positivos para a equação
xy
= n?
x+y

Solução. Primeiro reescrevemos a equação como

xy − nx − ny + n2 = n2 ⇔ (x − n)(y − n) = n2 .

3
POT 2012 - Álgebra - Nı́vel 2 - Aula 14 - Prof. Marcelo Mendes

Portanto, a cada divisor positivo de n2 corresponde um valor de x − n e uma solução.


Os divisores negativos sempre geram x ou y não-positivo.

Sendo n = pa11 · pa22 · ... · pakk , temos n2 = p2a 2a2 2ak


1 · p2 · ... · pk , que possui
1

(2a1 + 1)(2a2 + 1)...(2ak + 1)

divisores positivos, que é a quantidade de soluções da equação inicial.

Problema 13. Determine todos os pares ordenados (m, n) de números inteiros positivos
4 2
que são soluções da equação + = 1.
m n

Problema 14. Os três números distintos a, b, c verificam as igualdades


 3
 a + pa + q = 0
b3 + pb + q = 0 .
 3
c + pc + q = 0
Prove que a + b + c = 0.

Problema 15. (Czech and Slovak) Encontre todos os pares de inteiros a, b tais que a soma
a + b seja uma raiz da equação x2 + ax + b = 0.

4
POT 2012 - Álgebra - Nı́vel 2 - Aula 14 - Prof. Marcelo Mendes

Dicas

1. Calcule o ∆ da equação com variável em y e resolva a inequação ∆ ≥ 0.

2. Eleve ao cubo a equação membro a membro.


r
x2 + 3
3. Use a substituição = a.
x
1 y 1 2
4. Combine os resultados das equações = e = .
xy z − x + 1 xy z+1
5. Comece procurando soluções em que a = b.

5. Repita a ideia do problema 5.

7. Observe que, se a equação possui solução, então x ∈ Z.

8. Eleve as equações ao quadrado e lembre que raı́zes quadradas são não-negativas.


1
10. Passe z e para o lado direito da respectiva equação, equilibrando a quantidade de
z
termos em cada membro das equações.

11. Mesma dica do problema 10.

13. Tome como base as ideias da questão 10.

14. Subtraia as equações 2 a 2.

15. Substitua a + b na equação, calcule ∆ e iguale-o a um quadrado perfeito.

5
POT 2012 - Álgebra - Nı́vel 2 - Aula 14 - Prof. Marcelo Mendes

Soluções e Respostas


−x ± x2 − x − 6
1. a) (−∞, −2] ∪ [3, +∞); b) y = .
2
2. 3 e -12.

3. Não há solução real.


1 y 1 2
4. = ⇒ xy 2 = z − x + 1 e = ⇒ z + 1 = 2xy.
xy z−x+1 xy z+1

Daı́, xy 2 + x = 2xy ⇒ x(y − 1)2 = 0. Como x 6= 0, então y = 1. Portanto,


z+y
z + y = z + 1 = 2x ⇔ x = .
2

5. Entenda inicialmente que o problema não exige que se encontre todas as soluções,
mas apenas (pode não parecer a palavra mais adequada, mas pode ser ela sim!) que
existem infinitas soluções.

Comecemos buscando soluções em que a = b. A equação se tornaria 2013an = cn+1 .


Nesse caso, a = c = 2013 é solução, porém ainda não as conseguimos em quantidade
infinita.

Mas isso, agora, é fácil. Escolha a = b = 2013kn+1 e c = 2013kn . Variando k sobre


N, geramos as infinitas soluções pedidas.

7. Suponha que a equação possua solução. O lado direito da equação é um número


1
inteiro. Assim, x ∈ Z e, portanto, x = 1 − x, o que dá x = , que não é um número
2
inteiro, absurdo. Ou seja, não há solução para a equação.

8. Naturalmente, vamos elevar ao quadrado as 3 equações envolvidas. Somando os re-


sultados, obtemos x + y + z = 3.

Por outro lado, a partir da primeira equação, obtemos z ≥ 1. Analogamente, x ≥ 1


e y ≥ 1 das demais equações, que geram x + y + z ≥ 3.

Portanto, as igualdades devem ocorrer e a única solução é x = y = z = 1.

10. As soluções são x = −y, y = −z ou z = −x.

11. 2000.

6
POT 2012 - Álgebra - Nı́vel 2 - Aula 14 - Prof. Marcelo Mendes

4 2
13. + = 1 ⇔ (m−4)(n−2) = 8. As soluções inteiras positivas vêm de m−4 = 1, 2, 4, 8
m n
e n − 2 = 8, 4, 2, 1, respectivamente, ou seja, (m, n) = (5, 10), (6, 6), (8, 4), (12, 3).

14. Subtraindo as 2 primeiras equações membro a membro, temos

(a − b)(a2 + ab + b2 + q) = 0.

Como a 6= b, obtemos a2 + ab + b2 + q = 0. Analogamente, b2 + bc + c2 + q = 0.


Combinando esses 2 resultados (pela subtração), chegamos a (a − c)(a + b + c) = 0.
Mas a 6= c. Portanto, a + b + c = 0.

15. Se a + b é raiz, então

(a + b)2 + a(a + b) + b = 0 ⇒ 2a2 + 3ab + b2 + b = 0,

cujo discriminante é

∆ = b2 − 8b = (b − 4)2 − 16,
que deve ser um quadrado perfeito, digamos k2 , k ∈ Z+ , pois a ∈ Z, ou seja,

(b − 4)2 − k2 = 16 ⇔ (b − 4 + k)(b − 4 − k) = 16.

Lembrando que soma e diferença dos mesmos números inteiros têm a mesma paridade
e que b − 4 + k ≥ b − 4 − k pois assumimos, sem perda de generalidade, que k ≥ 0,
temos as seguintes possibilidades:

b − 4 + k = 4, 8, −2, −4
,
b − 4 − k = 4, 2, −8, −4
que produzem as soluções (a, b) = (−6, 8), (−6, 9), (0, −1), (0, 0).

7
Polos Olímpicos de Treinamento
Curso de Álgebra - Nível 2 Aula 15
Prof. Marcelo Mendes

A Ideia de Continuidade

Quando dizemos que um processo funciona de forma contı́nua, estamos dizendo que ele
ocorre sem interrupção.

Da mesmo forma, em matemática, o conceito de continuidade em funções, por exemplo,


significa que seu gráfico não tem interrupção. Os primeiros exemplos de funções contı́nuas
são os polinômios, que são somas de parcelas do tipo ak xk , sendo k um número inteiro
não-negativo, ou seja, funções do tipo

P (x) = an xn + an−1 xn−1 + ... + a1 x + a0 , an 6= 0,


sendo que, no ensino fundamental, sempre são estudadas as funções polinomiais de 1o e 2o
graus, cujos gráficos são, respectivamente, retas e parábolas, ambos sem interrupção, isto
é, contı́nuos, daı́ essas funções serem ditas contı́nuas.

A formalização do conceito de continuidade envolve noções de Cálculo - limite, espe-


cificamente - mas já é possı́vel fazer muito sem formalidades. Por outro lado, precisamos
assumir detalhes que não provaremos aqui nessas aulas. Por exemplo, a soma (e a diferença,
portanto) e o produto de funções contı́nuas também são funções contı́nuas.

Às vezes, também, podemos considerar uma função contı́nua apenas em um determi-
1
nado intervalo. Por exemplo, a função f (x) = possui uma descontinuidade em x = 0,
x
mas é contı́nua para reais positivos.

Vejamos alguns exemplos.

Problema 1. Mostre que a equação x3 − 5x + 2 = 0 possui uma raiz real positiva.

Solução. O lado esquerdo dessa equação é um polinômio, digamos P (x), ou seja, podemos
escrever P (x) = x3 − 5x + 2.
POT 2012 - Álgebra - Nı́vel 2 - Aula 15 - Prof. Marcelo Mendes

Agora, veja que P (0) = 2 > 0 e P (1) = −2 < 0. Como o gráfico de P é contı́nuo, pois
é um polinômio, ele une os pontos (0,2) e (1, -2) e, necessariamente, intercepta o eixo x
entre 0 e 1, isto é, P possui uma raiz real entre 0 e 1.

Problema 2. Prove que a equação x5 + 8x4 − 3x2 − 4x − 2 = 0 possui ao menos duas raı́zes
reais negativas e uma positiva.

Problema 3. Seja h : R → R uma função polinomial. Sabe-se que

h(−1) = 4, h(0) = 0, h(1) = 8.

Definimos g por g(x) = h(x) − 2. Mostre que a equação g(x) = 0 admite pelo menos 2
soluções distintas.

Problema 4. A equação
n4 − 2n3 + 3n2 + n − 33 = 0
possui solução real positiva?

Problema 5. De uma função g contı́nua em R, sabe-se que:

i. 1 é raiz de g;

ii. g(3) > 0.

g(3)
Prove que a equação g(x) = tem, pelo menos, uma solução no intervalo ]1; 3[.
2

Problema 6. Prove que o gráfico de f (x) = x3 −3x2 +1 intersecta o gráfico de g(x) = 2x−2
em pelo menos um ponto do intervalo ]0, 1[.

Problema 7. (Paulista) Demonstre que, no conjunto dos números reais, a equação

(x − b)(x − c) + (x − a)(x − c) + (x − a)(x − b) = 0

sempre tem solução, quaisquer que sejam os números reais a, b, c dados.

Solução. Seja

P (x) = (x − b)(x − c) + (x − a)(x − c) + (x − a)(x − b).

Inicialmente, observe que se a = b ou b = c ou c = a, então P possui uma raiz (a, b ou c).

2
POT 2012 - Álgebra - Nı́vel 2 - Aula 15 - Prof. Marcelo Mendes

Caso a, b, c sejam distintos 2 a 2, podemos supor, sem perda de generalidade (devido à


simetria da equação), a < b < c.

Assim, temos

P (a) = (a − b)(a − c) > 0,


P (b) = (b − a)(b − c) < 0,
P (c) = (c − a)(c − b) > 0,
que garantem uma raiz entre a e b e outra entre b e c.

Obs: Esta é uma solução envolvendo o conceito de continuidade dos polinômios. Tente
resolvê-la calculando o ∆ da equação do 2o grau.

Problema 8. (IME) Considere a, b, c ∈ R tais que a < b < c. Prove que a equação abaixo
possui exatamente duas raı́zes x1 e x2 que satisfazem a condição a < x1 < b < x2 < c.
1 1 1
+ + = 0.
x−a x−b x−c
Problema 9. Seja f uma função quadrática (função polinomial do 2o grau) tal que a equação
f (x) = x não tem soluções. Prove que a equação f (f (x)) = x também não tem soluções.

Solução. Primeiramente, veja que a condição de f (x) = x, que é uma equação do 2o grau,
não possuir raı́zes poderia nos levar a pensar em usar seu discriminante ∆. Porém, seria
complicado aplicar tal resultado na equação f (f (x)) = x, que é do 4o grau.

Pensemos, então, com nossos argumentos de continuidade, já que f é contı́nua pois seu
gráfico (uma parábola) não possui interrupção. Para simplificar um pouco nossa solução,
vamos criar a função g(x) = f (x) − x, que também é contı́nua utilizando um resultado
(que não iremos provar) que garante que soma de funções contı́nuas também é uma função
contı́nua.

Assim, a condição do problema que garante que f (x) = x não possui solução é equiva-
lente a dizer que g não possui raiz real, ou seja, seu gráfico está todo acima do eixo x ou
todo abaixo, devido à continuidade.

Sem perda de generalidade (o outro caso é análogo), vamos supor que o gráfico de g
esteja todo acima do eixo x, isto é, g(x) > 0, ∀x ∈ R. Portanto, f (x) > x, ∀x ∈ R.

Da mesma forma, também temos f (f (x)) > f (x), ∀x ∈ R.

3
POT 2012 - Álgebra - Nı́vel 2 - Aula 15 - Prof. Marcelo Mendes

Finalmente, combinando esses 2 resultados, concluı́mos que f (f (x)) > x, ∀x ∈ R (ou


f (f (x)) < x, ∀x ∈ R), garantindo que f (f (x)) = x não possui solução real.

Obs. A função em questão poderia ser qualquer função contı́nua.

Problema 10. Seja f uma função contı́nua para todo x real tal que a equação f (x) = x
não possui solução. Prove que f (f...f (x)...) = x também não possui solução, ∀n ∈ N.
| {z }
n
Problema 11. A função f : R → R é contı́nua e
f (x) · f (f (x)) = 1, ∀x ∈ R.
Sendo f (2004) = 2003, determine f (1999).
1
Solução. Vários alunos iniciam esse problema substituindo f (x) por y, obtêm f (y) = e,
y
1
portanto, f (1999) = , que é a resposta correta.
1999
1
Mas como explicar que f (2004) não é e, sim, 2003?
2004
A resposta está no fato de nem todo número (y, no caso) poder ser escrito como f (x).
Isso só é verdade se y está no conjunto imagem da função. Por exemplo, 2003 é imagem de
1 1
2004 e isso é suficiente para garantir que f (2003) = e, portanto, também está
  2003 2003
1
no conjunto imagem e f = 2003.
2003
Sendo assim, nosso objetivo é conseguir mostrar que 1999 está no conjunto
imagem dessa

1
função. No parágrafo anterior, vimos que o gráfico de f passa pelos pontos 2003,
  2003
1
e , 2003 . A continuidade de f garante que seu gráfico percorrerá pontos com or-
2003
1
denadas (coordenadas y) desde até 2003, ou seja, vai passar por um com ordenada
2003
1
1999, o que significa dizer que 1999 está no conjunto imagem de f . Logo, f (1999) = .
1999
Obs: Para uma melhor compreensão, faça um esboço no plano cartesiano dos dados encon-
trados nessa solução.

Problema 12. A função f : R → R é contı́nua e


f (x) · f (f (x)) = 1, ∀x ∈ R.
Se f (2011) = 2010, mostre que 2012 não pertence ao conjunto imagem de f , ou seja, não
existe x real tal que f (x) = 2012.

4
POT 2012 - Álgebra - Nı́vel 2 - Aula 15 - Prof. Marcelo Mendes

Problema 13. Sejam f, g : R → R funções contı́nuas tais que f (a) < g(a) e f (b) > g(b).
Mostre que existe c ∈ (a, b) tal que f (c) = g(c).

Solução. Vamos criar a função h(x) = f (x) − g(x), também contı́nua. Pelo problema,

h(a) = f (a) − g(a) < 0,

h(b) = f (b) − g(b) > 0.


A continuidade de h garante que existe c ∈ (a, b) tal que h(c) = 0, ou seja, f (c) = g(c).

Problema 14. Seja f : [a, b] → [a, b] uma função contı́nua (pois é a diferença entre funções
contı́nuas). Prove que f tem um ponto fixo, isto é, existe c ∈ [a, b] tal que f (c) = c.
(Sugestão: considere a função g(x) = f (x) − x.)

5
POT 2012 - Álgebra - Nı́vel 2 - Aula 15 - Prof. Marcelo Mendes

Dicas

2. Sendo P (x) = x5 + 8x4 − 3x2 − 4x − 2, calcule P (0), P (−1). Mostre que P assume
apenas valores negativos quando x diminui suficientemente. Depois, calcule P (1).

3. Calcule g(−1), g(0), g(1).

4. Sendo P (n) = n4 − 2n3 + 3n2 + n − 33, calcule P (0), P (1), P (2), P (3).

5. Escreva f (x) = 2g(x) − g(3). Calcule f (1) e f (3).

6. Escreva h(x) = f (x) − g(x) e calcule h(0) e h(1).

8. Veja solução do problema 7.

10. Veja solução do problema 9.

12. Veja solução do problema 11.

14. Veja solução do problema 13.

6
POT 2012 - Álgebra - Nı́vel 2 - Aula 15 - Prof. Marcelo Mendes

Soluções

2. Sendo P (x) = x5 + 8x4 − 3x2 − 4x − 2, então P (0) = −2 e P (−1) = 6. Assim, já


existe uma raiz negativa no intervalo (−1, 0). Quem controla o sinal de P quando
x caminha para −∞ é o termo x5 , que é negativo nesse caso. Daı́, partindo de um
valor positivo P (−1) = 6, o gráfico, quando x caminha para −∞, cortará novamente
o eixo x gerando uma nova raiz negativa. Além disso, P (1) = 0, ou seja, 1 é uma raiz
positiva.

3. g(−1) = h(−1) − 2 = 2, g(0) = h(0) − 2 = −2, g(1) = h(1) − 2 = 6, ou seja, existe


uma raiz no intervalo (−1, 0) e outra no intervalo (0, 1).

4. P (2) = −19, P (3) = 24 e continuidade garantem que existe uma raiz no intervalo
(2, 3).

5. Sendo f (x) = 2g(x) − g(3), temos f (1) = 2g(1) − g(3) = −g(3) < 0 e f (3) =
2g(3) − g(3) = g(3) > 0, ou seja, existe uma raiz no intervalo (1, 3).

6. Seja h(x) = f (x) − g(x) = x3 − 3x2 − 2x + 3. Como h(0) = 3 e h(1) = −1 e h


é contı́nua, segue que h possui uma raiz em (0, 1), isto é, os gráficos de f e g se
intersectam em pelo menos um ponto desse intervalo.
 
1 1
12. Pela solução do problema 11, temos que f (2010) = ef = 2010, ou seja,
2010 2010
2010 está no conjunto imagem de f . Se 2012 também estivesse, então a continuidade
1
de f garantiria que 2011 também estaria e, portanto, f (2011) = , absurdo.
2011
14. Se f (a) = a ou f (b) = b, então não há nada a fazer. Caso contrário, temos f (a) > a
e f (b) < b. O resultado segue análogo ao encontrado na solução do problema 13
criando a função h(x) = f (x) − x.

7
Polos Olímpicos de Treinamento
Curso de Álgebra - Nível 2 Aula 16
Prof. Marcelo Mendes

Números Complexos - Parte I

Introdução e Forma Algébrica



São as expressões da forma a + bi, em que a e b são números reais e i = −1 (i é a
primeira letra da palavra imaginário, sinônimo de número complexo) ou i2 = −1. Dizemos
que z = a + bi é forma algébrica do número complexo z.

Os números complexos da forma a + 0i são chamados números reais. Assim, R ⊂ C.

Os números complexos da forma 0 + bi são chamados números imaginários puros. Em


particular, 0 + 1i = i é chamado de unidade imaginária.

Interpretação Geométrica dos Números Complexos

Im

z = a + bi
bi
|z|

R
a

A figura acima mostra um número complexo no plano, que chamaremos de plano com-
plexo. Nele, o eixo horizontal contém números reais e o eixo vertical, números imaginários
puros. A distância de z à origem é o módulo de z (assim como acontece com os números
reais) e representamos da maneira usual, ou seja, |z|. Observe que existe uma associação
POT 2012 - Álgebra - Nı́vel 2 - Aula 16 - Prof. Marcelo Mendes

entre a notação cartesiana de um ponto (x, y) e a notação complexa do número x + yi.

Operações
1. Igualdade: a + bi = c + di ⇔ a = c e b = d. (Observação: Não há comparação dos tipos
>, <, ≥, ≤)

2. Soma: a + bi + c + di = (a + c) + (b + d)i.

3. Produto: (a + bi)(c + di) = (ac − bd) + (ad + bc)i.


a + bi ac + bd bc − ad
4. Quociente: = 2 +i· 2 .
c + di c + d2 c + d2

5. Módulo: |z| = a2 + b2 .

Potências do i
As potências de i são periódicas. De fato, i1 = i i2 = −1, i3 = −i, i4 = 1, i5 = i,
i6 = −1, ... A repetição ocorre a cada 4 potências (o perı́odo da repetição é 4.) Também é
comum precisar calcular potências de 1+i ou 1−i. É só usar que (1±i)2 = 1±2i−1 = ±2i.

O Conjugado e suas Propriedades


Im

z = a + bi
bi
|z|

R
a

|z|
−bi
z = a − bi

O conjugado z número complexo z = a + bi é, por definição, o número complexo


z = a − bi. Vejamos algumas propriedades úteis.

1. O conjugado do número complexo z é z e, por isso, z e z são mutuamente conjugados.

2
POT 2012 - Álgebra - Nı́vel 2 - Aula 16 - Prof. Marcelo Mendes


2. |z| = |z|, pois cada lado da igualdade é a 2 + b2 .

3. z = z ⇔ z ∈ R, já que a + bi = a − bi implica b = 0. Esse fato deve ser usado quando o


objetivo for provar que um determinado número é real.

4. z = −z ⇔ z é um número imaginário puro, pois a + bi = −(a − bi) implica a = 0.


Esse fato deve ser usado quando o objetivo for provar que um determinado número é
imaginário puro.

5. |z|2 = z · z, pois z · z = (a + bi)(a − bi) = a2 + b2 . ESSA É UMA PROPRIEDADES


MAIS ÚTEIS, pois consegue eliminar o módulo dos cálculos, algo bom mesmo que os
números envolvidos sejam reais.

6. A soma ((a + bi) + (a − bi) = 2a) e o produto (visto no item anterior) de números
mutuamente conjugados é um número real.

7. z1 + z2 = z1 + z2 . De fato, z1 + z2 = (a1 + ib1 ) + (a2 + ib2 ) = (a1 + a2 ) + i(b1 + b2 ) =


(a1 + a2 ) − i(b1 + b2 ) = (a1 − ib1 ) + (a2 − ib2 ) = z1 + z2 (Esse fato já foi uma questão
proposta pelo IME).

8. z1 · z2 = z1 · z2 .
 
z1 z1
9. = .
z2 z2

Tente verificar esses dois últimos itens fazendo z1 = a + bi e z2 = c + di. Vejamos agora
alguns exemplos.

Problema 1. Calcule i2011 , i2012 , i2013 .

Problema 2. Calcule o valor de i8n+3 + i4n+1 .

Problema 3. Calcule (1 + i)2011 , (1 − i)2012 , (1 + i)2013 .

Problema 4. Encontre todas as raı́zes da equação z 3 = 1.

Solução. A equação pode ser reescrita como

z 3 − 1 = 0 ⇔ (z − 1)(z 2 + z + 1) = 0.
A primeira raiz z = 1 vem de z − 1 = 0. As demais vêm de√z 2 + z + 1 =√0, cujo
−1 ± −3 −1 ± i 3
discriminante é ∆ = −3. Logo, essas últimas duas raı́zes são = .
2 2

3
POT 2012 - Álgebra - Nı́vel 2 - Aula 16 - Prof. Marcelo Mendes

Problema 5. Encontre as raı́zes das equações

a) z 3 = 8;

b) z 4 = 81.

Problema 6. Encontre números reais x, y, u, v satisfazendo

z = x + i, w = 3 + iy,

z + w = u − i, zw = 14 + iv.

Problema 7. Seja z = a + bi, em que a, b ∈ R. Encontre condições sobre a e b para que:


a) z 3 seja real;

b) z 3 seja imaginário puro.

Solução. Veja:

z = a + bi ⇒ z 3 = (a + bi)3 = a3 + 3a2 bi + 3a(bi)2 + (bi)3

⇒ z 3 = (a3 − 3ab2 ) + i(3a2 b − b3 ).

a) z 3 é real se, e somente se, sua √


parte imaginária é nula, ou seja, 3a2 b − b3 = 0, o que
ocorre quando b = 0 ou b = ±a 3.

b) z 3 é imaginário puro se, e somente √


se, sua parte real é nula, ou seja, a3 − 3ab2 = 0, o
que ocorre quando a = 0 ou a = ±b 3.

Problema 8. Para z ∈ C, prove que


1
|z| = 1 ⇔ z = .
z
Problema 9. Prove que |1 + iz| = |1 − iz| se, e somente se, z é um número real.

Solução. Sabendo que |z|2 = z · z, temos

|1 + iz| = |1 − iz| ⇔ |1 + iz|2 = |1 − iz|2


⇔ (1 + iz)(1 + iz) = (1 − iz)(1 − iz)
⇔ (1 + iz)(1 − iz) = (1 − iz)(1 + iz)
⇔ 1 + iz − iz + |z|2 = 1 − iz + iz + |z|2
⇔ iz = iz ⇔ z = z,
que é a condição necessária e suficiente para z ser real.

4
POT 2012 - Álgebra - Nı́vel 2 - Aula 16 - Prof. Marcelo Mendes

Problema 10. Sejam a e b números reais. Se a + bi 6= 0, determine a forma algébrica do


1
número .
a + bi
Solução. A ideia de tornar o denominador real é sempre utilizar o conjugado, parecido
com o que fazemos quando queremos racionalizar um denominador irracional:

1 a − bi a − bi a −b
= = 2 2
= 2 2
+i· 2 ,
a + bi (a + bi)(a − bi) a +b a +b a + b2
que é a forma algébrica desejada.

1
Problema 11. (ITA) Seja z = a + bi um número complexo. Se z + é um número real,
z
então mostre que b = 0 ou |z| = 1.

Problema 12. (ITA) Se z1 e z2 são números complexos e z1 + z2 e z1 · z2 são ambos reais,


então mostre que z1 e z2 são ambos reais ou z1 = z.

5
POT 2012 - Álgebra - Nı́vel 2 - Aula 16 - Prof. Marcelo Mendes

Dicas

5. Veja a solução da questão 3.

6. Use que a igualdade entre números complexos, ou seja,a + bi = c + di ⇔ a = c e


b = d, se a, b, c, d são reais.

8. Use |z|2 = z · z.

11. Use o problema 10.

12. Escreva z1 e z2 em suas formas algébricas, ou seja, z1 = a + bi e z2 + c + di, sendo


a, b, c, d números reais.

6
POT 2012 - Álgebra - Nı́vel 2 - Aula 16 - Prof. Marcelo Mendes

Respostas e Soluções

1. −i, 1, i.

2. 0.

3. (1 + i)2011 = (1 + i)2010 (1 + i) = (2i)1005 (1 + i) = 21005 i1004 i(1 + i) = 21005 (i − 1).


(1 − i)2012 = (−2i)1006 = (−2)1006 i1004 i2 = −21006 .
(1 + i)2013 = (1 + i)2011 (1 + i)2 = 21005 (i − 1)2i = −21006 (1 + i).

1. a) 2, 1 ± 3.
b) ±3, ±3i.

6. x = 4, y = −2, u = 7 e v = −5.

8. Sabendo que |z|2 = z · z, temos

1
|z| = 1 ⇔ |z|2 = 1 ⇔ zz = 1 ⇔ z = .
z
1 a −b b b(a2 + b2 − 1)
11. z + = a + bi + 2 + i · será real se b − = = 0,
z a + b√2 a 2 + b2 a2 + b2 a 2 + b2
ou seja, b = 0 ou |z| = a2 + b2 = 1.

12. Escrevendo z1 = a + bi e z2 + c + di, temos z1 + z2 = (a + c) + i(b + d) e z1 · z2 =


(ac − bd) + i(ad + bc). Como esses números são reais, devemos ter:

1. b + d = 0 ⇔ b = −d;
2. ad + bc = 0 ⇔ ad = −bc.

Se d = 0, então b = 0. Daı́, z1 e z2 são reais. Se d 6= 0, então podemos fazer


o cancelamento na equação do item 1 e achar a = c. Isso mostra que z1 e z2 são
conjugados complexos.

7
Polos Olímpicos de Treinamento
Curso de Álgebra - Nível 2 Aula 17
Prof. Marcelo Mendes

Números Complexos - Parte II

Vamos finalizar nosso estudo dos números complexos apresentando a forma de escrevê-
los com o auxı́lio da Trigonometria, que dará suporte a mais teoria posterior, e mais
exercı́cios.

Forma Trigonométrica

Im

z = a + bi
bi
|z|

θ
R
a

a b
A figura acima nos permite escrever cosθ = e senθ = . Assim, temos
|z| |z|
z = a + ib = |z|cosθ + i|z|senθ,
ou seja,

z = |z|(cosθ + isenθ).
O ângulo θ é chamado de argumento do número complexo z e o denotamos por arg z.
Completando as propriedades do conjugado, temos arg z = 360o − arg z.

Problema 1. Escreva os seguintes números na forma trigonométrica.


POT 2012 - Álgebra - Nı́vel 2 - Aula 17 - Prof. Marcelo Mendes

a) 2.

b) 3i.

c) 1 + i.

d) 1 + i 3.

Solução.

a) 2 = 2(cos0 + isen0).

b) 3i = 3(cos90o + isen90o ).
√ √ !
√ 2 2 √
c) 1 + i = 2 +i = 2(cos45o + isen45o ).
2 2
√ !
√ 1 3
d) 1 + i 3 = 2 +i = 2(cos60o + isen60o ).
2 2

Para ilustrar o item c) dessa questão, observe a figura a seguir.


Im

z = 1+i
i

| 2|

45o
R
1

Após localizar o número 1 + i no plano complexo, visualizamos um quadrado √ de lado


o
1. Assi, fica mais fácil enxergar que o argumento de z é 45 e que o módulo é 2.

Problema 2. Determine o polinômio de menor grau e com coeficientes reais que possui um

número complexo com módulo 1 e argumento como raiz.
3

Problema 3. Sejam x = a + b, y = aω + bω 2 , z = aω 2 + bω, onde ω 2 + ω + 1 = 0. Calcule


x + y + z e expresse x3 + y 3 + z 3 em termos de a e b.

2
POT 2012 - Álgebra - Nı́vel 2 - Aula 17 - Prof. Marcelo Mendes

Problema 4. (EUA) O número complexo z satisfaz z + |z| = 2 + 8i. Calcule |z|2 .

Solução. Supondo z = a + bi, a equação fica


p
a + bi + a2 + b2 = 2 + 8i.
A igualdade entre números complexos√nos garante que b = 8 (comparando as partes
imaginárias dos lados da equação) e a + a2 + b2 = 2 (comparando as partes reais dos
lados da equação), ou seja,
p
a2 + b2 = 2 − a ≥ 0 ⇒ a = −15.

Portanto, z = −15 + 8i é a única solução e |z|2 = 152 + 82 = 289.

Problema 5. (IME) Dois números complexos z1 e z2 , não-nulos, são tais que |z1 + z2 | =
z2
|z1 − z2 |. Mostre que é imaginário puro.
z1

Problema 6. (IME) Sendo a, b e c números naturais em progressão aritmética e z um


número complexo de módulo unitário, determine um valor para cada um dos números
a, b, c e z de forma que eles satisfaçam a igualdade
1 1 1
a
+ b + c = z9.
z z z
Problema 7. (ITA) Determine todos os números complexos z, que são raı́zes da equação
|z| − z = 1 + 2i, sendo i a unidade imaginária.

Problema 8. (ITA) Considerando z e w números complexos arbitrários e u = z · w + z · w,


mostre que o conjugado de u é igual ao dobro da parte real do número z · w.

Problema 9. (ITA) Determine o valor da expressão |1 − z|2 + |1 + z|2 , sendo z um número


complexo unitário.

Problema 10. (ITA)


√ Determine o produto dos números complexos z = x + yi que têm
módulo igual a 2 e tais que y = 2x − 1.

Solução. |z|2 = 2 ⇒ x2 + (2x − 1)2 = 2 ⇒ 5x2 − 4x − 1 = 0, cujas raı́zes são x = 1 e


1
x=− .
5
1 7 6 8
Assim, os números são 1 + i e − − i, cujo produto é − i.
5 5 5 5

3
POT 2012 - Álgebra - Nı́vel 2 - Aula 17 - Prof. Marcelo Mendes

Problema 11. (ITA) Mostre que, resolvendo a equação z 2 = 2 + z no conjunto dos números
complexos, todas as raı́zes são números inteiros.

Problema 12. (ITA) Sejam x e y números reais, com x 6= 0, satisfazendo (x+iy)2 = (x+y)i.
Mostre que x é uma raiz da equação x3 + 3x2 + 2x − 6 = 0.

Solução. (x + iy)2 = (x + y)i ⇒ x2 + 2xyi − y 2 = (x + y)i. Pela igualdade entre números


complexos, temos x2 − y 2 = 0, o que dá x = y ou x = −y.

Se x = −y, então x + y = 0 e 2xy = 0, absurdo pois x 6= 0.

Logo, x = y e 2xy = x + y equivale a x2 = x. Assim, x = 1, que é uma raiz da equação


dada.

Problema 13. Resolva a equação (z + i)2 + (z − i)2 = 2.


√ !93
2
Problema 14. (ITA) Escreva as formas algébrica e trigonométrica da potência .
1+i

4
POT 2012 - Álgebra - Nı́vel 2 - Aula 17 - Prof. Marcelo Mendes

Dicas


1 3
2. Esse número é z = cos120o + isen120o =− +i .
2 2
3. Mostre que ω 3 = 1 e use esse resultado.

5. Use |z|2 = z · z.

6. Tome z = i, que tem módulo unitário e encontre valores para a, b e c.

7. Escreva z = a + bi, com a e b reais.

8. Use z · w = z · w e escreva zw = a + bi, com a e b reais.

9. Use |w|2 = w · w.

11. Escreva z = a + bi, com a e b reais.

14. Use (1 + i)2 = 2i.

5
POT 2012 - Álgebra - Nı́vel 2 - Aula 17 - Prof. Marcelo Mendes

Respostas e Soluções

2. P (z) = z 2 + z + 1.

3. x + y + z = a(1 + ω + ω 2 ) + b(1 + ω + ω 2 ) = 0. Assim, x3 + y 3 + z 3 = 3xyz = 3(a3 + b3 )


pois
xyz = (a + b)(aω + bω 2 )(aω 2 + bω)
= (a + b)(a2 + b2 + ab(ω 4 + ω 2 )
= (a + b)(a2 + b2 − ab) = a3 + b3 ,
visto que ω 4 = ω pois ω 3 − 1 = (ω − 1)(ω 2 + ω + 1) = 0.
Outra forma de calcular x3 + y 3 + z 3 é elevar ao cubo as expressões de x, y, z e,
depois, somar os resultados.

5. |z1 + z2 | = |z1 − z2 | ⇔ |z1 + z2 |2 = |z1 − z2 |2 ⇔ (z1 + z2 )(z1 + z2 ) = (z1 − z2 )(z1 − z2 )


(z1 + z2 )(z1 + z2 ) = (z1 − z2 )(z1 − z2 ) ⇔ z1 z1 + z1 z2 + z2 z1 + z2 z2 = z1 z1 − z1 z2 −
z2 z1 + z2 z2 ⇔ z1 z2 + z2 z1 = 0 ⇔

6. Tomando z = i, que tem módulo 1, uma possı́vel solução é a = 2, b = 3 e c = 4


(P.A.), como pedido no enunciado (não foi pedido encontrar todas as soluções).

7. Fazendo z = a + bi, temos a2 + b2 − a − bi = 1 + 2i. A igualdade entre números
complexos nos dá b = −2 e
p 3
a2 + 4 = a + 1 ⇒ a = .
2
3
Logo, z = − 2i é a única solução.
2
8. u = u. Se zw = a + bi, então u = 2a.

9. |1 − z|2 + |1 + z|2 = (1 − z)(1 − z) + (1 + z)(1 + z) = 2 + 2|z|2 = 4.

11. Fazendo z = a + bi, temos a2 + b2 = 2 + a − bi. A igualdade entre números complexos


garante que b = 0 e que a2 = 2 + a, cujas raı́zes são 2 e -1.

13. z = ± 2.
√ 93 √ 92 √ √
14. 2 = 2 2 = 246 2.
(1 + i)93 = (1 + i)92 (1 + i) = (2i)46 (1 + i) = −246 (1 + i)
√ !93 √ √ √ √
2 2 2(1 − i) 2 2
Assim, =− =− =− + i = cos315o + isen315o .
1+i 1+i (1 + i)(1 − i) 2 2

6
Polos Olímpicos de Treinamento
Curso de Álgebra - Nível 2 Aula 18
Prof. Marcelo Mendes

Relações de Girard - Parte I

Ao estudar as equações (polinomiais) do 2o grau, você deve ter aprendido que é possı́vel
calcular a soma e o produto das raı́zes, mesmo sem conhecê-las. Vamos recordar essas
fórmulas.

Sejam x1 e x2 as raı́zes da equação ax2 + bx + c = 0. Assim, podemos escrever

ax2 + bx + c = a(x − x1 )(x − x2 )

⇒ ax2 + bx + c = ax2 − a(x1 + x2 )x + ax1 x2 .


Igualando os coeficientes de termos de mesmo grau, obtemos b = −a(x1 + x2 ) e c =
ax1 x2 , ou seja,

b
x1 + x2 = − ,
a
c
x1 x2 = .
a
Agora, não pense mais em soma e produto das raı́zes e, sim, que são somas das raı́zes
separadamente e, depois, aos pares. Para um conjunto de 3 raı́zes, por exemplo, a soma
delas separadamente seria x1 + x2 + x3 e, aos pares, x1 x2 + x2 x3 + x3 x1 . Além disso, ainda
haveria a soma delas de 3 em 3: x1 x2 x3 , que, nesse caso, coincide com o produto.

Com essas expressões e repetindo os cálculos acima para uma equação ax3 +bx2 +cx+d =
0, cujas raı́zes são x1 , x2 e x3 , obtemos

b
x1 + x2 + x3 = − ,
a
c
x1 x2 + x2 x3 + x3 x1 = ,
a
d
x1 x2 x3 = − .
a
E isso pode ser extendido para equações polinomiais de qualquer grau, sempre dividindo
pelo primeiro coeficiente, que é sempre diferente de zero, e fazendo alternância de sinais.
POT 2012 - Álgebra - Nı́vel 2 - Aula 18 - Prof. Marcelo Mendes

Tais resultados são conhecidos como Relações de Girard (Albert Girard (1590 - 1639)).

Por exemplo, se a equação for 2x4 + 3x2 + 8x + 1 = 0, então

x1 + x2 + x3 + x4 = 0,
3
x1 x2 + x1 x3 + x1 x4 + x2 x3 + x2 x4 + x3 x4 = ,
2
8
x1 x2 x3 + x1 x2 x4 + x1 x3 x4 + x2 x3 x4 = − = −4,
2
1
x1 x2 x3 x4 = .
2
o
Problema 1. Determine uma equação do 3 grau cujas raı́zes sejam 1, 2 e 3.

Solução. O resultado é imediato escrevendo

(x − 1)(x − 2)(x − 3) = 0 ⇔ x3 − 6x2 + 11x − 6 = 0.


Mas também podemos criar os coeficientes da equação através das Relações de Girard.
Sendo x3 − ax2 + bx − c = 0 uma equação (pois podemos multiplicá-la por qualquer valor
não-nulo sem que suas raı́zes sejam alteradas) procurada (os sinais nos coeficientes estão
alternados pelo padrão das Relações de Girard). Assim:

a = 1 + 2 + 3 = 6,
b = 1 · 2 + 2 · 3 + 3 · 1 = 11,
c = 1 · 2 · 3 = 6,
obtendo assim a equação x3 − 6x2 + 11x − 6 = 0.
Problema 2. Resolva a equação polinomial x3 + 4x2 + x − 6 = 0 sabendo que uma de suas
raı́zes é 1.

Solução. Sejam 1, r, s as raı́zes dessa equação. Assim, por Girard, temos:

1 + r + s = −4,
1 · r · s = 6,
ou seja,
r + s = −5,
rs = 6,
donde r = −2 e s = −3.
Problema 3. (EUA) Para quantos inteiros positivos n entre 1 e 100 é possı́vel fatorar
x2 + x − n como produto de dois fatores lineares com coeficientes inteiros?

2
POT 2012 - Álgebra - Nı́vel 2 - Aula 18 - Prof. Marcelo Mendes

Problema 4. (ITA) Se a, b, c são as raı́zes da equação x3 − 2x3 + x − 4 = 0, determine o


1 1 1
valor de + + .
a b c

Problema 5. (ITA) As raı́zes da equação x4 + qx3 + rx2 + sx + t = 0, com q, r, s, t ∈ Q∗+ ,


são L, M, N, P . Determine o valor de
L M N P
+ + + .
MNP LN P LM P LM N
Problema 6. (IME) Sejam x1 e x2 as raı́zes da equação x2 + (m − 15)x + m = 0. Sabendo
que x1 e x2 são números inteiros, determine o conjunto de valores possı́veis para m.

Problema 7. Os três números distintos a, b, c verificam as igualdades


 3
 a + pa + q = 0
b3 + pb + q = 0 .
 3
c + pc + q = 0

Prove que a + b + c = 0.

Solução. As relações dadas significam que a, b e c são as raı́zes da equação polinomial do


3o grau x3 + px + q = 0, que, por Girard, tem soma das raı́zes igual a 0, isto é, a + b + c = 0.

Problema 8. Sejam m, n, k ∈ Q as raı́zes de t3 + at + b. Prove que as raı́zes de mt2 + nt + k


também são racionais.

Solução. Observe que não existe o termo do 2o grau em t3 + at + b. Assim, por Girard,
temos que m + n + k = 0. Ora, mas isso mostra que 1 é uma raiz de mt2 + nt + k.
k
Novamente, Girard, através do produto das raı́zes, nos dá que a outra raiz é , que
m
é racional por ser quociente de racionais. Isso mostra que as 2 raı́zes de mt2 + nt + k são
racionais.

Problema 9. (ITA) As raı́zes da equação de coeficientes reais x3 + ax2 + bx + c = 0 são


inteiros positivos consecutivos. A soma dos quadrados dessas raı́zes é igual a 14. Determine
o valor de a2 + b2 + c2 .

Problema 10. Determine o valor da soma a + b para que as raı́zes do polinômio

4x4 − 20x3 + ax2 − 25x + b


1
estejam em progressão aritmética de razão .
2

3
POT 2012 - Álgebra - Nı́vel 2 - Aula 18 - Prof. Marcelo Mendes

1 3
Solução. Sejam r, r + , r + 1, r + as raı́zes da equação. Daı́,
2 2
1 3 −20 1
r+r+ +r+1+r+ =− =5⇒r= ,
2 2 4 2
1 3
ou seja, as raı́zes são , 1, e 2. Por Girard, temos:
2 2
a 1 1 3 1 1 3 35
= ·1+ · + ·2+1· +1·2+ ·2 = ⇒ a = 35,
4 2 2 2 2 2 2 4
b 1 3 6
= · 1 · · 2 = ⇒ b = 6.
4 2 2 4
Portanto, a + b = 41.

4
POT 2012 - Álgebra - Nı́vel 2 - Aula 18 - Prof. Marcelo Mendes

Dicas

3. Chame as raı́zes de a e b. Em seguida, utilize Relações de Girard.

4. Reduza a soma das frações a um denominador comum (o que se chama comumente


de ’tirar o mı́nimo’. Mas nem sempre o mmc é o produto, além de só ser definido
para naturais). Em seguida, use Girard.

5. Reduza a soma das frações a um denominador comum. Em seguida, use Girard.

6. Além das Relações de Girard, é interessante conseguir utilizar a identidade

xy + x + y + 1 = (x + 1)(y + 1).

9. Denote as raı́zes por n − 1, n, n + 1. Em seguida, use a dica do enunciado e Girard.

5
POT 2012 - Álgebra - Nı́vel 2 - Aula 18 - Prof. Marcelo Mendes

Respostas e Soluções

3. Ao expressarmos x2 + x − n como produto de dois fatores lineares com coeficientes


inteiros, os fatores serão (x − a) e (x − b), com a e b inteiros. Por Girard, temos

a + b = −1,
ab = −n.
Assim, precisamos encontrar n tal que a(a + 1) = n, 1 < n < 100. As possibilidades
são a = 1, 2, 3, 4, 5, 6, 7, 8, 9, que dão, respectivamente, n = 2, 6, 12, 20, 30, 42, 56, 72, 90.
Portanto, o resultado é possı́vel para 9 valores de n.

4. Por Girard, temos que ab + bc + ca = 1 e abc = 4. Logo,

1 1 1 ab + bc + ca 1
+ + = = .
a b c abc 4
5. Manipulando a equação e utilizando as Relações de Girard, temos

L M N P L2 + M 2 + N 2 + P 2
+ + + =
MNP LN P LM P LM N LM N P
(L + M + N + P )2 − 2(LM + LN + LP + M N + M P + N P )
=
LM N P
q 2 − 2r
= .
t
6. As Relações de Girard nos dão x1 + x2 = 15 − m e x1 x2 = m. Portanto, soluções
(x1 , x2 ) e (x2 , x1 ) dão o mesmo resultado (valor de m) e

x1 x2 + x1 + x2 + 1 = 16 ⇔ (x1 + 1)(x2 + 1) = 16.

Daı́, temos as possibilidades



x1 + 1 = 1, 2, 4, −4, −2, −1
x2 + 1 = 16, 8, 4, −4, −8, −16


x1 = 0, 1, 3, −5, −3, −2
⇒ ,
x2 = 15, 7, 3, −5, −9, −17

que dão os seguintes possı́veis valores para m = x1 x2 : 0, 7, 9, 25, 27, 34.

6
POT 2012 - Álgebra - Nı́vel 2 - Aula 18 - Prof. Marcelo Mendes

9. Sejam n − 1, n e n + 1 as raı́zes dessa equação. Assim,

(n − 1)2 + n2 + (n + 1)2 = 14 ⇔ 3n2 + 2 = 14 ⇔ n = 2.

Pelas Relações de Girard, temos

1 + 2 + 3 = −a ⇒ a = −6,
1 · 2 + 2 · 3 + 3 · 1 = b ⇒ 11,
1 · 2 · 3 = −c ⇒ c = −6.

Logo,
a2 + b2 + c2 = 62 + 122 + 62 = 36 + 121 + 36 = 193.

7
Polos Olímpicos de Treinamento
Curso de Álgebra - Nível 2 Aula 19
Prof. Marcelo Mendes

Relações de Girard - Parte II

Vamos continuar vendo mais exemplos das Relações de Girard. Veremos também um
resultado novo que relaciona esse assunto com números complexos.

Problema 1. (ITA) Seja k ∈ R tal que a equação 2x3 + 7x2 + 4x + k = 0 possua uma
raiz dupla e inteira x1 e uma raiz x2 (ou seja, as raı́zes são x1 , x1 e x2 ), distinta de x1 .
Determine o valor de (k + x1 )x2 .

Solução. Vamos utilizar as Relações de Girard para soma e soma aos pares:
7
x1 + x1 + x2 = 2x1 + x2 = − ,
2
4
x1 x1 + x1 x2 + x1 x2 = x21 + 2x1 x2 = = 2.
2
Eliminando x2 , obtemos:

3x21 + 7x1 + 2 = 0,
3
cuja raiz inteira é x1 = −1. Assim, x2 = − e, portanto,
2
3 k
x1 x1 x2 = − = − ⇒ k = 3.
2 2

Problema 2. Mostrar que f (x) = x3 +x2 −10x+8 é divisı́vel por (x−1) mas não é divisı́vel
por (x − 1)2 .

Solução. Veja que f (1) = 1+ 1− 10+ 8 = 0, o que mostra que f (x) possui um fator (x− 1).

Agora, suponha que f (x) seja divisı́vel por (x − 1)2 . Isto significaria que f possui 1
como raiz dupla. Suponha, então, que as raı́zes sejam 1, 1 e r. Por Girard, temos
POT 2012 - Álgebra - Nı́vel 2 - Aula 19 - Prof. Marcelo Mendes

1 + 1 + r = −1 ⇒ r = −3,
1 · 1 · r = −8 ⇒ r = −8,
um absurdo. Logo, f não é divisı́vel por (x − 1)2 .

Problema 3. Verifique se a equação x3 − 3x + 8 = 0 tem raı́zes iguais.

Problema 4. Determinar m para que a equação x3 − 7x + m = 0 tenha uma raiz igual ao


dobro de uma outra.

Problema 5. (IME) Seja

p(x) = x5 + bx4 + cx3 + dx2 + ex + f

um polinômio com coeficientes inteiros. Sabe-se que as cinco raı́zes de p(x) são números
inteiros positivos, sendo quatro deles pares e um ı́mpar. Determine o número de coeficientes
pares de p(x).

Problema 6. (OCM) Considere todas as retas que encontram o gráfico da função

f (x) = 2x4 + 7x3 + 3x − 5

em quatro pontos distintos, digamos (x1 , y1 ), (x2 , y2 ), (x3 , y3 ), (x4 , y4 ). Mostre que o valor
x1 + x2 + x3 + x4
de é independente da reta e ache esse valor.
4
Solução. Seja y = ax + b a equação de uma dessas retas que cortam o gráfico de f em 4
pontos distintos. Queremos resolver a equação f (x) = y, ou seja:

2x4 + 7x3 + 3x − 5 = ax + b ⇔ 2x4 + 7x3 + (3 − a)x − b − 5 = 0.


7
Por Girard, x1 + x2 + x3 + x4 = − e, portanto,
2
x1 + x2 + x3 + x4 7
=− ,
4 8
que independe da reta pois não varia com os valores de a ou b. A chave dessa ideia funcio-
nar é que os coeficientes de f que dão a soma (os dois primeiros) não foram afetados por
a ou b.

Problema 7. (IME) Determine o valor da soma das raı́zes da equação


3 1
y 2 + 5y + 2y 2 + 8 = 0.

2
POT 2012 - Álgebra - Nı́vel 2 - Aula 19 - Prof. Marcelo Mendes

Problema 8. São dados a, b, c ∈ R. Sabe-se que

a + b + c > 0,

bc + ca + ab > 0,
abc > 0.
Prove que a > 0, b > 0, c > 0.

Solução. Seja x3 − Ax2 + Bx − C = 0 a equação cuja raı́zes são a, b, c. Por Girard, temos

A = a + b + c ⇒ A > 0,
B = ab + bc + ca ⇒ B > 0,
C = abc ⇒ C > 0.
Suponha que a > 0, b > 0, c > 0 não ocorra, ou seja, existe uma raiz r ≤ 0. Mas

r 3 − |{z}
|{z} Ar 2 + |{z}
Br − |{z}
C
≤0 ≥0 ≤0 >0

seria negativo, contrariando o fato de r ser raiz. Portanto, a > 0, b > 0, c > 0.

Problema 9. Suponha que t3 + pt + q = 0 tenha uma raiz não real a + bi, sendo a, b, p, q
todos reais e q 6= 0. Mostre que aq > 0.

Solução. Vamos iniciar com o seguinte

Teorema (das Raı́zes Complexas). Se uma equação polinomial com coeficientes reais
possui uma raiz complexa z = a + bi (b =
6 0), então z = a − bi também é uma raiz dessa
equação.

Demonstração. Uma equação polinomial é da forma

an xn + an−1 xn−1 + ... + a1 x + a0 = 0.(∗)


Se z é uma raiz dessa equação, significa que

an z n + an−1 z n−1 + ... + a1 z + a0 = 0.


A igualdade acima garante que também vale a igualdade entre os conjugados. Utilizando
as propriedades vistas a respeito dos números complexos, temos

an z n + an−1 z n−1 + ... + a1 z + a0 = 0

⇒ an z n + an−1 z n−1 + ... + a1 z + a0 = 0

3
POT 2012 - Álgebra - Nı́vel 2 - Aula 19 - Prof. Marcelo Mendes

⇒ an z n + an−1 z n−1 + ... + a1 z + a0 = 0

⇒ an z n + an−1 z n−1 + ... + a1 z + a0 = 0,


onde a condição de os coeficientes serem reais foi usada pois o conjugado de um número
real é esse próprio número. A última equação garante que z também é raiz de (*).

Voltando ao problema, temos as condições do teorema acima, uma vez que 1, 0, p, q


são reais. Logo, a − bi também é uma raiz. Seja r a terceira raiz. Por Girard, temos soma
0, ou seja

a + bi + a − bi + r = 0,
que mostra que r = −2a é a terceira raiz. Agora, pelo produto

(a + bi)(a − bi)(−2a) = −q ⇔ 2a(a2 + b2 ) = q,


temos que a e q têm o mesmo sinal. Além disso, q 6= 0 ⇒ a 6= 0. Portanto, aq > 0.

Problema 10. (OCM) Mostre que 1 é a única raiz real da equação x3 + x2 = 2.

Problema 11. (ITA) A equação 4x3 − 3x2 + 4x − 3 = 0 admite i (unidade imaginária) como
raiz. Determine as demais raı́zes.

4
POT 2012 - Álgebra - Nı́vel 2 - Aula 19 - Prof. Marcelo Mendes

Dicas

3. Use Girard para analisar os casos:


i) as 3 raı́zes são iguais.
ii) as raı́zes são r, r, s (r 6= s).

4. Denote as raı́zes por a, 2a, b e utilize Girard.

5. Utilize Girard para analisar a paridade das raı́zes.


1
7. Faça a substituição y 2 = x e use Girard. Tenha cuidado a soma pedida é em relação
à variável y.

10. Denote as raı́zes diferentes de 1 por a + bi e a − bi (o Teorema das Raı́zes Complexas


garante que as últimas 2 raı́zes, de fato, são números complexos conjugados). Depois,
use Girard.

11. Use o Teorema das Raı́zes Complexas para obter que −i também é raiz. Depois, use
Girard.

5
POT 2012 - Álgebra - Nı́vel 2 - Aula 19 - Prof. Marcelo Mendes

Respostas e Soluções

3. Se suposermos que há 3 raı́zes iguais, então, pela fórmula de Girard para a soma,
obterı́amos 0 como raiz, um absurdo.

Assim, só nos resta analisar o caso em que as raı́zes são r, r e s. Por Girard, terı́amos:

2r + s = 0 ⇒ s = −2r,
r s = −8 ⇒ −2r 3 = −8 ⇒ r 3 = 4.
2

Para concluirmos o absurdo desta parte, podemos utilizar que, se r é uma raiz, então
r 3 − 3r + 8 = 0 ⇒ 3r = 8, que contradiz a igualdade r 3 = 4.

4. Sejam a, 2a, b as raı́zes. Por Girard, temos

a + 2a + b = 0 ⇒ 3a + b = 0,
a · 2a + a · b + 2a · b = −3 ⇒ 2a2 + 3ab = −3.
Eliminando b, temos a = ±1. Assim:

i) a = 1 ⇒ b = −3. Logo, m = 6.
ii) a = −1 ⇒ b = 3. Logo, m = −6.

5. Sejam p1 , p2 , p3 , p4 as 4 raı́zes pares e i a raı́z ı́mpar. Por Girard, −b é a soma de


quatro número pares e um ı́mpar, ou seja, b é ı́mpar; os demais coeficientes serão
somas de produtos em que pelo menos um fator é pk (k = 1, 2, 3 ou 4) e, portanto,
são todos pares. Logo, p possui 4 coeficientes pares.

1
7. Vamos começar com a substituição y 2 = x. A equação se torna

x3 + 5x2 + 2x + 8 = 0.
Todavia, devemos ficar atentos que não nos interessa o valor de x1 + x2 + x3 , uma
vez que a letra x não é a incógnita inicial.
Nosso objetivo é calcular
y1 + y2 + y3 = x21 + x22 + x23 ,
ou seja,

(x1 + x2 + x3 )2 − 2(x1 x2 + x2 x3 + x3 x1 ) = (−5)2 − 2(2) = 21.

6
POT 2012 - Álgebra - Nı́vel 2 - Aula 19 - Prof. Marcelo Mendes

10. Inicialmente, veja que 1 é raiz de x3 + x2 = 2, pois 13 + 12 = 2, e que essa equação


pode ser reescrita como x3 + x2 − 2 = 0.

Suponha que, além da raiz 1, essa equação possua uma raiz complexa e não-real a+bi.
Como temos a condição do teorema visto no problema 20 (que é termos coeficientes
reais), a − bi também deve ser uma raiz. Por Girard, temos

1 + a + bi + a − bi = −1 ⇒ a = −1,
1 · (a + bi) · (a − bi) = 2 ⇒ 1 + b2 = 2 ⇒ b = ±1.

Temos, assim, raı́zes 1, −1 ± i, que verificam a relação de Girard restante

1(−1 + i) + 1(−1 − i) + (−1 + i)(−1 − i) = 0.

Assim, 1 é, de fato, a única raiz real.

11. Observe, inicialmente, que todos os coeficientes dessa equação são reais. Pelo teo-
rema visto no problema 20, podemos concluir que −i (que é o conjugado do número
complexo i) também é raiz da equação dada.

Por Girard, sendo r a terceira raiz, então


3 3
i + (−i) + r = ⇒r= .
4 4

Outra maneira, talvez até mais natural de se resolver esse problema, é através de
fatoração:

4x3 − 3x2 + 4x − 3 = 0
⇔ x2 (4x − 3) + (4x − 3) = 0
⇔ (4x − 3)(x2 + 1) = 0,
3
cujas raı́zes são, de fato, , i e −i.
4

7
Algebra 01 - Produtos Notáveis

Problema 1. Encontre as soluções inteiras positivas (a, b, c) da equação

a2 bc − 2ab2 c − 2abc2 + b3 c + bc3 + 2b2 c2 = 11.

Solução. Fatorando obtemos:

a2 bc − 2ab2 c − 2abc2 + b3 c + bc3 + 2b2 c2 = 11 ⇐⇒ bc(a2 − 2ab − 2ac + b2 + c2 + 2bc) = 11


⇐⇒ bc[a2 − 2a(b + c) + (b + c)2 ] = 11
⇐⇒ bc[a − (b + c)]2 = 11.

Como 11 é livre de quadrados, devemos ter [a − (b + c)]2 = 1 e bc = 11. De forma que as


soluções são (a; b; c) = (11; 11; 1), (13; 11; 1), (11; 1; 11), (13; 1; 11).

1 1
Problema 2. Se x + = 3, encontre x5 + 5 .
x x
1 1 1 1
Solução. Sabemos que x2 + 2 = (x+ )2 −2·x· = 7. Analogamente, x4 + 4 = 72 −2 = 47.
x x x x
Além disso, fazendo
1 1 1 1
(x + )(x2 + 2 ) = x3 + 3 + x +
x x x x
1
encontramos x3 + 3 = 18.
x
Agora, usando que
1 1 1 1
(x4 + 4 )(x + ) = x5 + 5 + x3 + 3
x x x x
1
finalmente encontramos x5 + 5 = 123.
x

Problema 3. Encontre todos os reais x, y e z que satisfazem

2x2 + y 2 + z 2 − 2xz + 2y − 2x + 2 = 0

Solução. Note que

2x2 + y 2 + z 2 − 2xz + 2y − 2x + 2 = (x2 − 2xz + z 2 ) + (x2 − 2x + 1) + (y 2 + 2y + 1)


= (x − z)2 + (x − 1)2 + (y + 1)2 = 0.

Dessa forma, devemos ter y + 1 = x − 1 = x − z = 0. Ou seja, (x, y, z) = (1, −1, 1).

Problema 4. Sabendo que x2 + y 2 + z 2 = 6 e que x + y + z = 4, x, y e z reais. Encontre


xy + xz + yz.
Solução. Façamos a seguinte expansão.

(x + y + x)2 = [x + (y + z)]2
= x2 + 2x(y + z) + (y + z)2
= x2 + y 2 + z 2 + 2xy + 2xz + 2yz.

Então, encontramos que

(x + y + x)2 − (x2 + y 2 + z 2 )
xy + xz + yz = .
2
42 − 6
Substituindo os valores, chegamos na resposta, = 5.
2

√ √ √
q
Problema 5. Mostre que 5 + 2 6 = 2 + 3.
√ √ 2 √ 2 √ √ √ √ 2
Solução. Note que 5 + 2 6 = ( 2) + ( 3) + 2 2 · 3 = ( 2 + 3) . Portanto,
√ q√ √ √ √
q
5 + 2 6 = ( 2 + 3)2 = 2 + 3.


q
4
Problema 6. Simplifique 7 + 4 3.

Solução. Perceba que


√ √ √ √
7 + 4 3 = 22 + ( 3)2 + 2 · 2 · 3 = (2 + 3)2 .
√ √
q q
4
Sendo assim, 7 + 4 3 = 2 + 3. Todavia, também temos que
r !2

r r r
1 3 1 3 1 3
2+ 3= + +2 · = + .
2 2 2 2 2 2


q r r
4 1 3
Finalmente, chegamos em 7+4 3= + .
2 2

Problema 7. Encontre todos os inteiros n para os quais 4n4 + 1 é primo.

Solução.

4n4 + 1 = 4n4 + 4n2 + 1 − 4n2


= (2n2 + 1)2 − (2n)2
= (2n2 − 2n + 1)(2n2 + 2n + 1)

2
Para n = 1, 4n4 + 1 = 5 é primo. Para n > 1, ambos os fatores 2n2 − 2n + 1 e 2n2 + 2n + 1
são maiores que 1, de forma que 4n4 + 1 não pode ser primo. Sendo assim, 4n4 + 1 é primo
somente para n = 1.

1 1 1
Problema 8. Determine o número de pares de inteiros a e b que satisfazem + = .
a b 10
1 1 1
Solução. A equação + = é equivalente a 10a + 10b = ab, com a, b 6= 0. Temos
a b 10
ab−10a−10b+100 = 100 ⇐⇒ (a−10)(b−10) = 100. a−10 pode ser qualquer divisor de 100
com exceção de −10. Logo, pode assumir os valores ±100, ±50, ±25, ±20, 10, ±5, ±4, ±2, ±1.
Então, o número de soluções é 2 · 8 + 1 = 17.

√3
2
Problema 9. Expresse F = √ √ com um denominador racional.
1+ 2+ 34
3

√ a
. Sabemos que a3 − 1 = (a − 1)(a2 + a + 1).
3
Solução. Seja a = 2, então F =
1 + a + a2
a a(a − 1)
Dessa forma, F = 3 = 3 . Substituindo os valores obtemos
a −1 a −1
a−1
√ √
3
2( 3 2 − 1) √ 3

3
F = √ 3
= 4 − 2.
3
( 2) − 1

Problema 10. Prove que se (a+b)2 +(b+c)2 +(c+d)2 = 4(ab+bc+cd), então a = b = c = b.

Solução. Desenvolvendo, obtemos

(a + b)2 + (b + c)2 + (c + d)2 = 4(ab + bc + cd) ⇐⇒ a2 + 2b2 + 2c2 + d2 = 2ab + 2bc + 2cd
⇐⇒ (a2 − 2ab + b2 ) + (b2 − 2bc + c2 ) + (c2 − 2cd + d2 ) = 0
⇐⇒ (a − b)2 + (b − c)2 + (c − d)2 = 0.

A única maneira dessa soma ser 0 é com (a − b)2 = (b − c)2 = (c − d)2 = 0. Isto é,
a = b = c = d.
Algebra 02 - Equações e Sistemas de Equações


x− x+1 11
Problema 1. Encontre todos os valores de x satisfazendo a equação √ = .
x+ x+1 5
√ √ √
Solução. Multiplicando obtemos 5x − 5 x + 1 = 11x + 11 x + 1. Então, −3x = 8 x + 1,
9x2 = 64x + 64, 9x2 − 64x − 64 = 0. Como 9x2 − 64x − 64 = (9x + 8)(x − 8), ou 9x + 8 = 0
−8
ou x − 8 = 0. Testando, verificamos que a solução é x = .
9

Problema 2. Encontre o menor valor de x satisfazendo as condições: x3 + 2x2 = a, onde x é


um inteiro positivo ı́mpar, e a é o quadrado de um inteiro.

Solução. x3 + 2x2 = x2 (x + 2) = n2 , onde n é um inteiro. Então, x + 2 deve ser um quadrado.


Testanto x = 1, 3, 5 e 7 descobrimos que o menor valor ı́mpar de x para que x + 2 seja um
quadrado é x = 7, para o qual x + 2 = 32 .

Problema 3. Encontre as soluções inteiras da equação x2 − 2y 2 − xy = 7.

Solução. Note que

x2 − 2y 2 − xy = (x2 − y 2 ) − (y 2 + xy)
= (x − y)(x + y) − y(y + x)
= (x + y)(x − 2y).

Então a equação é equivalente a (x + y)(x − 2y) = 7. Podemos ter x + y = 1, −1, 7 e −7.


As soluções são (x, y) = (3, −2), (5, 2), (−5, −2), (−3, 2).

Problema 4. Resolva a equação


x − ab x − ac x − bc
+ + =a+b+c
a+b a+c b+c
Solução. Nós temos
     
x − ab x − ac x − bc
−c + −b + − a = 0.
a+b a+c b+c
Então,
x − ab − ac − bc x − ab − ac − bc x − ab − ac − bc
+ + =0
a+b a+c b+c
ou,  
1 1 1
(x − ab − ac − bc) + + = 0.
a+b a+c b+c
Assumindo que
1 1 1
+ +
a+b a+c b+c
é diferente de zero, nós obtemos
x = ab + ac + bc.
Se, entretanto,
1 1 1
+ + = 0,
a+b a+c b+c
então a equação dada se torna um identidade que vale para todo x.

Problema 5. Resolva a equação


6x + 2a + 3b + c 2x + 6a + b + 3c
= .
6x + 2a − 3b − c 2x + 6a − b − 3c
Solução. Se colocarmos na nossa equação 6x + 2a = A, 3b + c = B, 2x + 6a = C, b + 3c = D,
então ela fica reescrita da seguinte maneira
A+B C +D
= .
A−B C −D
Adicionando 1 aos dois lados da equação, nós encontramos
2A 2C
= .
A−B C −D
Analogamente, subtraindo 1, nós temos
2B 2D
= .
A−B C −D
Dividindo as duas igualdades, nós temos
A C
= ,
B D
isto é,
6x + 2a 2x + 6a
= .
3b + c b + 3c
Então    
6 2 6 2
− x= − a.
3b + c b + 3c b + 3c 3b + c
Finalmente,
ab
x= .
c

2
Problema 6. Resolva a sistema

x+y+z = a
x+y+w = b
x+z+w = c
y + z + w = d.

Solução. Somando todas as equações, nós temos


a+b+c+d
x+y+z+w = .
3
Consequentemente
a+b+c+d b + c + d − 2a
w = (x + y + z + w) − (x + y + z) = −a= .
3 3
Analogamente, nós obtemos
a + b + c − 2d a + b + d − 2c a + c + d − 2b
x= , y= , z= .
3 3 3

Problema 7. Resolva o sistema


xy = y x
xm = y n .
Solução. Nós temos
x/y
x=y .
Consequentemente, mx
xm = y y .
Usando a segunda equação, nós encontramos
mx
y y = yn.
mx
Agora, ou y = 1 e então x = 1 ou = n, isto é,
y
ny
x= .
m
Substituindo na segunda equação, nós temos:
 ny m  m m
= y n , y m−n = .
m n
Dai, n m
m x
m
m−n m−n
y= , x = yy = .
n n

3
Problema 8. Resolva o sistema
x2 + y 2 = 5
x4 + y 4 = 17
Solução. Temos que
(x2 + y 2 )2 = x4 + y 4 + 2x2 y 2 .
Então,
x2 y 2 = (x2 + y 2 )2 − (x4 + y 4) 2 = 4.
Fazendo a = x2 e b = y 2 temos
a + b = 5, ab = 4
Esse sistema nos leva a equação λ2 − 5λ + 4 = 0 com soluções λ = 1 e λ = 4. Dessa forma,
temos (a, b) = (1, 4), (4, 1). Consequentemente, (x, y) = (1, 2), (−1, −2), (1, −2), (−1, 2),
(2, 1), (−2, 1), (2, −1), (−2, −1).

Problema 9. Dado que x2 = y + a e y 2 = x + a, onde a é um inteiro positivo, encontre


expressões para a tais que existem soluções inteiras para x e y.

Solução. x2 = y +a, y 2 = x+a, então x2 −y 2 = y −x. Portanto, (x−y)(x+y)+(x−y) = 0,


(x − y)(x√+ y + 1) = 0. Daı́, x = y ou x = −1 − y. Portanto,√ ou y 2 − y − a = 0 dando
1 ± 1 + 4a −1 ± 4a − 3
y= , ou y 2 + y + 1 − a = 0 dando y = .
2 √ √ 2
Para y ser inteiro, 1 + 4a = 2n + 1 e 4a − 3 = 2m − 1. Então, a = n2 + n = n(n + 1),
n = 0, 1, 2, . . . ou a = m2 − m + 1 = m(m − 1) + 1, m = 1, 2, 3, . . ..

Problema 10. Resolva o sistema


x(x + y + x) = a2
y(x + y + x) = b2
z(x + y + x) = c2 .
Solução. Somando as três equações nós encontramos
(x + y + z)2 = a2 + b2 + c2 .
Daı́, p
x + y + z = ± a2 + b2 + c2 .
Consequentemente
a2 b2
x= √ , y= √ ,
± a2 + b2 + c2 ± a2 + b2 + c2
c2
z= √ .
± a2 + b2 + c2
Álgebra 03 - Sequências

Problema 1. Sabendo que a2 , b2 , c2 formam uma progressão aritmética, nessa ordem. Prove
1 1 1
que , , também formam uma progressão aritmética na ordem dada.
b+c c+a a+b
x+z
Solução. Os números (x, y, z) formam uma P.A. se, e só se, y = . Sendo assim, temos
2
que 2b2 = a2 + c2 . Utilizando isso, devemos mostrar que
 
1 1 1
2· = + .
c+a b+c a+b

Lembrando que b + c, c + a, a + b são não nulos. Temos que


2 1 1 2 a + 2b + c
= + ⇐⇒ =
c+a b+c a+b c+a (a + b)(b + c)
⇐⇒ 2(a + b)(b + c) = (a + 2b + c)(a + c)
⇐⇒ 2b2 = a2 + c2 .

Isto é,
1 1 1
, ,
b+c c+a a+b
formam uma P.A. se, e somente se,
a2 , b2 , c2
também formam. Isso conclui a demonstração.

Problema 2. Prove que, se a, b, c são respectivamente o p-ésimo, q-ésimo e r-ésimo termos de


uma progressão aritmética, então

(q − r)a + (r − p)b + (p − q)c = 0.

Solução. Se an é o n-ésimo termo e am é o m-ésimo termo da progressão aritmética, então


nós temos
an = a1 + d(n − 1)
am = a1 = d(m − 1),
onde d é a razão da progressão. Daı́,

an − am = (n − m)d.

Por hipótese, nós temos as seguintes igualdades

b − c = (q − r)d,
c − a = (r − p)d,
a − b = (p − q)d.
Multiplocando a primeira equação por a, a segunda por b, e a terceira por c, nós temos

d[(q − r)a + (r − p)b + (p − q)c] = a(b − c) + b(c − a) + c(a − b) = 0,

onde
(q − r)a + (r − p)b + (p − q)c = 0.

Problema 3. Uma sequência de termos positivos A1 , A2 , . . . , An , . . . satisfaz a relação recur-


3(1 + An )
siva An+1 = . Para que valores de A1 a sequência é monótona decrescente (isto é,
3 + An
A1 ≥ A2 ≥ . . . ≥ An ≥ . . .)?

Solução. Devemos ter An ≥ An+1 para todo n natural.

3(1 + An )
An ≥ ⇐⇒ (3 + An )An ≥ 3 + 3An
3 + An
⇐⇒ A2n ≥ 3.
√ √
Logo, devemos ter An ≥ 3 para todo n. De forma que A1 ≥ 3 é condição necessária para
que a sequência seja monôtona
√ decrescente. Vamos verificar se isso é condição suficiente.
Devemos garantir que An ≥ 3 para todo n. Ou seja,
√ 3(1 + An ) √
An+1 ≥ 3 ⇐⇒ 3
3 + An
√ √
⇐⇒ 3 + 3An ≥ 3 3 + An 3

3( 3 − 1)
⇐⇒ An ≥ √
3− 3
√ √
3( 3 − 1) 3 + 3
⇐⇒ An ≥ √ · √
3− 3 3+ 3

⇐⇒ An ≥ 3.
√ √ √
Ou seja, A1 ≥ 3 garante An ≥ 3 para todo n. De forma que A1 ≥ 3 é condição sufici-
ente para que a sequência satisfaça
√ as condições enunciadas. Isto é, a sequência é monôtona
decrescente se, e só se, A1 ≥ 3.

2
 
1 2 3 99
Problema 4. Seja x = 101! · + + + ... + . Encontre 101! − x.
2! 3! 4! 100!
Solução. Escrevamos x = 101! · A.

n (n + 1) − 1 1 1
Note que = = − . Então,
(n + 1)! (n + 1)! n! (n + 1)!
         
1 1 1 1 1 1 1 1 1 1 1
A= − + − + − +. . .+ − + − = 1− .
1! 2! 2! 3! 3! 4! 98! 99! 99! 100! 100!

Dessa forma, agora nós temos


 
1 101!
101! − x = 101! − 101! · 1 − = = 101.
100! 100!

Problema 5. Considere uma sequência definida recursivamente por xn = xn−1 + 2n e x0 = 0.


Encontre uma expressão não recursiva para o termo geral xn .

Solução. Observe que

xn = xn − x0 = (xn − xn−1 ) + (xn−1 + xn−2 ) + . . . + (x2 − x1 ) + (x1 − x0 ).

Sabemos que para todo k inteiro positivo xk+1 − xk = 2(k + 1). Logo,

xn = 2n + 2(n − 1) + 2(n − 2) + . . . + 2 · 2 + 2 · 1.
= 2 · (1 + 2 + 3 + . . . + n)
n(n + 1)
= 2·
2
= n(n + 1).

Problema 6. Prove a identidade


 
1 1 1 1 1 1
+ + ... + = − .
1·2·3 2·3·4 n(n + 1)(n + 2) 2 2 (n + 1)(n + 2)

Solução. Fazendo
1 A B
= +
n(n + 1)(n + 2) n(n + 1) (n + 1)(n + 2)

temos n(A + B) + 2A = 1. Portanto, A + B = 0 e A = 1/2. De forma que


 
1 1 1 1
= − .
n(n + 1)(n + 2) 2 n(n + 1) (n + 1)(n + 2)

3
Daı́, se
1 1 1
S= + + ... +
1·2·3 2·3·4 n(n + 1)(n + 2)
então
     
1 1 1 1 1 1 1
S= − + − + ... + − .
2 1·2 2·3 2·3 3·4 n(n + 1) (n + 1)(n + 2)

Eliminando os termos do meio encontramos


 
1 1 1
S= − .
2 2 (n + 1)(n + 2)

Problema 7. Calcule
1 1 1
A= √ +√ √ + ... + √ √ .
1+ 2 2+ 3 n+ n+1
√ √ √ √ 1 √ √
Solução. ( k + 1 − k)( k + 1 + k) = 1. Logo, √ √ = k+1− k.
k+ k+1
Temos então que
√ √ √ √ √ √ √ √ √
A = ( 2 − 1) + ( 3 − 2) + . . . + ( n − n − 1) + ( n + 1 − n) = n + 1 − 1.
Programa Olı́mpico de Treinamento
Curso de Álgebra - Nı́vel 2 Aula Complementar I
Prof. Hugo Araújo

Funções quadráticas - Parte 1

Você muito provavelmente já ouviu alguém dizer “Mais um dia na vida que eu não
usei Bhaskara pra nada...”. O caro leitor não deve compartilhar o mesmo sentimento e
imagino que compreenda a importância de um método rápido para o cálculo das soluções
de uma equação do segundo grau.
Talvez você ainda não saiba, mas este tipo de equação tem aplicações no estudo do
movimento dos corpos, de situações de equilı́brio quı́mico e também em problemas de
otimização, entre outros. Além disso, é um tópico muito recorrente em problemas de
olimpı́ada, porém de uma maneira um pouco diferente daquela que estamos acostumados
na escola.
O objetivo desta aula é explorar alguns exemplos envolvendo equações e funções
quadráticas. Começamos demonstrando como a fórmula de Bhaskara é deduzida. A
principal ideia é o que chamamos de completar quadrados.

1) Equação do segundo grau


Uma equação quadrática ou do segundo grau é uma equação do tipo
ax2 + bx + c = 0,
onde a 6= 0, b e c são números reais. Iremos deduzir uma fórmula para suas soluções.
Primeiro, como a 6= 0, dividimos a equação por a, obtendo
b c
x2 + x + = 0.
a a
Para completarmos o quadrado, adicionamos um valor (livre do termo x) a ambos os lados
da equação de maneira que o lado esquerdo seja um “trinômio quadrado perfeito”, ou seja,
uma expressão do tipo (x + d)2 , que é igual a x2 + 2dx + d2 . Comparando termo a termo,
b b2 c
devemos escolher d = 2a , donde segue que precisamos adicionar 4a 2 − a a ambos os lados.
Podemos então resolver a equação da seguinte forma:
b c b c b2 c b2 c
x2 + x + = 0 ⇐⇒ x2 + x + C + 2 − C = 2 −
a a a aC 4a aC 4a a
b 2 b2 − 4ac
 
⇐⇒ x + =
2a 4a2
r
b b2 − 4ac
⇐⇒ x + =±
2a √ 4a2
−b ± b2 − 4ac
⇐⇒ x = .
2a

1
POTI 2020 - Álgebra - Nı́vel 2 - Aula Complementar - Prof. Hugo Araújo

Esta fórmula é conhecida como fórmula de Bhaskara, em homenagem a Bhaskara


Acharya, matemático que viveu na Índia no século XII. Apesar disto, ele não foi o autor
da fórmula e métodos para resolução de equações do segundo grau já eram conhecidos
pelos babilônios, milênios antes.
Chamamos de discriminante a quantidade dentro da raiz quadrada e a denotamos pela
letra grega delta, ou seja, escrevemos ∆ = b2 − 4ac. A fórmula de Bhaskara toma sua
forma usual √
−b ± ∆
x= .
2a
Chamamos os dois possı́veis valores do lado direito de raı́zes da equação. Convém
observar que estas soluções x não correspondem sempre a número reais. Existem três
possibilidades, dependendo do valor de ∆:

1. ∆ > 0: Neste caso as duas raı́zes da equação são dois números reais distintos.

2. ∆ = 0: Neste caso as duas raı́zes da equação são idênticas, ambas iguais a um


número real.

3. ∆ < 0: Neste caso nenhuma das duas raı́zes é um número real. Ainda assim, existem
dois números complexos conjugados que são raı́zes da equação.

Segue da fórmula que, se x1 e x2 são as raı́zes da equação,


b c
x1 + x2 = − e x1 x2 = .
a a
Não é muito difı́cil então verificar que o lado esquerdo da equação pode ser fatorado
como
ax2 + bx + c = a(x − x1 )(x − x2 ).
Esta é toda a teoria que iremos precisar a respeito de equações. Vamos resolver agora
um problema que aponta para uma ideia bastante comum: considerar os coeficientes da
equação como variáveis.

Exemplo 1. (OBMEP 2017 - adaptado) Um número inteiro n é chamado de quase-legal


se n é maior do que 1 e (n + 1)2 é igual à soma de n inteiros positivos consecutivos.
Encontre todos os números quase-legais.

Solução. Começamos lembrando que 1 + 2 + · · · + n − 1 = (n−1)n


2 . Segue daı́ que a soma
de n inteiros consecutivos, o menor deles sendo igual a k, é igual a

k + (k + 1) + · · · + (k + n − 1) = (1 + 2 + · · · + k + n − 1) − (1 + 2 + · · · + k − 1)
(k + n − 1)(k + n) (k − 1)k
= −
2 2
2
n + 2kn − n
=
2

2
POTI 2020 - Álgebra - Nı́vel 2 - Aula Complementar - Prof. Hugo Araújo

O número n é quase-legal se e somente se existe k inteiro positivo tal que

n2 + (2k − 1)n
(n + 1)2 = ⇐⇒
2
2n2 + 4n + 2 = n2 + (2k − 1)n ⇐⇒
n2 − (2k − 5)n + 2 = 0.

Considerando a última linha como uma equação do segundo grau em n, a fórmula de


Bhaskara nos diz que suas soluções são dadas por
p
2k − 5 ± (2k − 5)2 − 8
n=
2
Observe que, por hipótese, n é inteiro positivo e k também. Isto implica que se n é quase-
legal, então o inteiro k tem que ser tal que (2k − 5)2 − 8 é um quadrado perfeito. Dessa
maneira, o problema se resume a encontrar os valores de k tais que (2k − 5)2 − 8 é um
quadrado perfeito, ou seja, para quais valores de k inteiro positivo existe algum inteiro
positivo l tal que

(2k − 5)2 − 8 = l2 ⇐⇒
(2k + l − 5)(2k − l − 5) = 8.

Analisando a paridade dos lados da equação, notamos que l é ı́mpar. Além disso,
2k + l − 5 > 2k − l − 5 e, pela paridade de l, ambos são pares. Existem então duas
possibilidades

2k + l − 5 = 4 e 2k − l − 5 = 2 =⇒ k = 4 e l = 1;
2k + l − 5 = −2 e 2k − l − 5 = −4 =⇒ k = 1 e l = 1.

O primeiro caso nos dá n = (3 ± 1)/2, ou seja, n = 1 ou n = 2. O segundo caso resulta


em n = (−3 ± 1)/2, resultando em valores de n negativos. O único número quase-legal é
portanto o número 2 (9 = 4+ 5). 

Mais um exemplo, este agora envolvendo a análise do discriminante ∆ a respeito do


tipo de raı́zes.

Exemplo 2. Seja a um real positivo tal que

a3 = 6(a + 1).

Prove que a equação


x2 + ax + a2 − 6 = 0
não possui solução real.

3
POTI 2020 - Álgebra - Nı́vel 2 - Aula Complementar - Prof. Hugo Araújo

Solução. Suponha por absurdo que x2 + ax + a2 − 6 = 0 tem alguma raiz real. Calculando
o discriminante da equação em questão, obtemos

∆ = a2 − 4(a2 − 6) = −3a2 + 24.

Como há raiz real, o ∆ é não-negativo, logo a2 ≤ 8.


Tá, mas e daı́? Precisamos usar ainda, com bastante cuidado, todas informações que
nos foram dadas. A primeira equação pode ser reescrita como a3 − 6a = 6. Ela parece
muito com o termo livre de x na segunda equação. De fato, multiplicando a segunda
equação por a (que sabemos ser positivo), ficamos com:

ax2 + a2 x + a3 − 6a = 0 ⇐⇒
ax2 + a2 x + 6 = 0.

Como esta equação equivale a original, seu discriminante também tem que ser não-
negativo. Portanto, ∆ = a4 − 24a ≥ 0, e como √ a3 ≥ 24.
a é positivo, isto equivale a √
2 3
Entretanto, a ≤ 8 implica que a ≤ 16 2. Em√contrapartida, 16 2 < 24 pois
16 × 2 = 512 < 576 = 242 . Segue então que a3 ≤ 16 2 < 24 ≤ a3 , uma contradição.
2

Concluı́mos que não há raı́zes reais. 

Em outros problemas, pode ser necessário explorar as relações de soma e produto de


raı́zes de forma direta. Elas também são conhecidas como relações de Girard e tem forma
similar para equações de ordem maior. Contudo, iremos nos restringir a equações de
segundo grau nesta aula.

Exemplo 3. (OBM) Sejam a, b, c e d números reais distintos tais que a e b são as raı́zes
da equação x2 − 3cx − 8d = 0; e c e d são as raı́zes da equação x2 − 3ax − 8b = 0. Calcule
a soma a + b + c + d.

Solução. Existem muitas maneiras de resolver este problema, todas elas envolvem mani-
pular as relações entre a, b, c e d de forma inteligente. Pela relações de soma e produto
de raı́zes observamos que

a + b = 3c c + d = 3a
ab = −8d cd = −8b;

donde segue que b = 3c − a e d = 3a − c. Segue também que a + b + c + d = 3(a + c).


Em contrapartida, substituindo b e d nas segundas equações de cada sistema,

a(3c − a) = −8(3a − c) e c(3a − c) = −8(3c − a)

e subtraindo-as vem que c2 − a2 = (c − a)(c + a) = 32(c − a). Como os reais são distintos,
c − a 6= 0 e a + c = 32. Logo, a + b + c + d = 96. 

4
POTI 2020 - Álgebra - Nı́vel 2 - Aula Complementar - Prof. Hugo Araújo

Encerramos esta sessão lembrando que a fórmula de Bhaskara vale em contextos além
dos números reais. Note que, além da operação de tirar a raiz quadrada, realizamos apenas
as 4 operações fundamentais para obter a fórmula. Em espaços onde conseguimos realizar
estas operações de forma similar, podemos encontrar a fórmula de Bhaskara.
Por exemplo, e aqui é necessário entender um pouco de Teoria dos Números, considere
p um número primo diferente de 2 e Zp o espaço de restos módulo p, Zp = {0, 1, . . . , p − 1}.
Podemos realizar as 4 operações fundamentais neste espaço. Você pode estar um pouco
encucado com a divisão (por 2), mas assim como na reta, se a for diferente de 0 (o elemento
neutro), existe a−1 , um elemento tal que a−1 · a ≡ 1 mod p. Em Zp , dividir por a é o
mesmo que multiplicar por a−1 . Contudo, saber se o discriminante da equação admite ou
não alguma raiz quadrada dentro do espaço Zp pode ser mais complicado. Costumamos
chamar os elementos de Zp que tem raiz quadrada de resı́duos quadráticos, ou seja, aqueles
a ∈ Zp tais que a equação x2 ≡ a mod p tem solução em Zp . Nosso último exemplo está
relacionado com estas observações.
Exemplo 4. (OBM 2003) Determine o menor número primo positivo que divide x2 +
5x + 23 para algum inteiro x.

Solução. Observe que se x = 23 então 23 | x2 +5x+23, e portanto podemos nos restringir


a analisar primos menores que 23.
Seja então p primo menor que 23. Analisemos a equação módulo p. Note que para
qualquer valor de x inteiro, o número x2 + 5x + 23 é ı́mpar, logo p = 2 nunca divide a
expressão. Podemos supor então que 2 6≡ 0 mod p.
Por clareza, procedemos de forma similar a dedução da fórmula de Bhaskara.

x2 + 5x + 23 ≡ 0 mod p ⇐⇒
2
4x + 4 · 5x + 82 ≡ 0 mod p ⇐⇒
2
4x + 4 · 5x + 82 + 25 − 82 ≡ 25 − 82 mod p ⇐⇒
(2x + 5)2 ≡ −67 mod p ⇐⇒
−1

x≡2 (−5 ± −67) mod p.

A existência de algum inteiro x tal que p | x2 + 5x + 23 equivale a −67 ser resı́duo


quadrático módulo p. A análise desta propriedade em geral é bem mais simples se o leitor
tem familiaridade com o sı́mbolo de Legendre e até reciprocidade quadrática. Iremos fazer
aqui a coisa de um jeito mais braçal. Note que x2 ≡ (−x)2 mod p. Assim, para encontrar
quem são os resı́duos quadráticos, basta calcular os quadrados de 0, 1, ..., p−1
2 módulo p.
Os resı́duos quadráticos módulo cada primo p são:
• p = 3: 0, 1;

• p = 5: 0, 1, 4

• p = 7: 0, 1, 4, 2;

• p = 11: 0, 1, 4, 9, 5, 3;

5
POTI 2020 - Álgebra - Nı́vel 2 - Aula Complementar - Prof. Hugo Araújo

• p = 13: 0, 1, 4, 9, 3, 12, 10;

• p = 17: 0, 1, 4, 9, 16, 8, 2, 15, 13.

Calculando −67 módulo p e comparando com os valores acima, notamos que este número
só é resı́duo quadrático módulo 17, pois −67 ≡ 1 mod 17. Segue que este é o menor primo
que satisfaz as condições do enunciado. Mais ainda, segue que

x ≡ 2−1 (−5 ± −67) ≡ 9(−5 ± 1) ≡ 6 ± 9 mod 17

satisfaz 17 | x2 + 5x + 23. Podemos tomar então x = −3, por exemplo. 

Seguem agora alguns exercı́cios para praticar o conteúdo desta aula até aqui.

Problema 1. (OBM 2011) Sejam a e b números reais não nulos tais que a equação
x2 + ax + b = 0 possui soluções a e b. Determine a − b.

Problema 2. Existe algum inteiro positivo n tal que 1 + 2 + · · · + n seja igual ao dobro
de um quadrado perfeito?

Problema 3. (Alemanha 2001) Determine todos os númerors reais q tais que a equação
x4 − 40x2 + q = 0 tem quatro raı́zes reais em progressão aritmética.

Problema 4. (URSS 86) A equação x2 + ax + b + 1 = 0 tem duas raı́zes inteiras. Prove


que (a2 + b2 ) é um número composto.

Problema 5. (África do Sul 2016) Determine todos os pares de números reais a e b, b > 0,
tais que as soluções das duas equações

x2 + ax + a = b e x2 + ax + a = −b

são quatro inteiros consecutivos.

Problema 6. (OBM 2010) Sejam r e s números inteiros. Sabe-se que a equação do


segundo grau
x2 − (r + s)x + rs + 2010 = 0
tem as duas soluções inteiras. Quantos são os possı́veis valores de |r − s|?

Problema 7. (Rússia 2007) Sejam a, b e c três números reais. Prove que ao menos uma
das três equações
x2 + (a − b)x + (b − c) = 0

6
POTI 2020 - Álgebra - Nı́vel 2 - Aula Complementar - Prof. Hugo Araújo

x2 + (b − c)x + (c − a) = 0
x2 + (c − a)x + (a − b) = 0
tem alguma raiz real.

Problema 8. (OBM 2001) Mostre que não existem dois números inteiros a e b tais que
(a + b)(a2 + b2 ) = 2001.

Problema 9. Mostre que se a, b e c são números inteiros ı́mpares, a equação

ax2 + bx + c = 0

não tem raiz racional.

Problema 10. Sejam x, y inteiros positivos tais que x2 + xy + 7y 2 é divisı́vel por 23.
Encontre o menor valor possı́vel para (x + y − 103)2 .

7
POTI 2020 - Álgebra - Nı́vel 2 - Aula Complementar - Prof. Hugo Araújo

Dicas e respostas
1. Usando o produto das raı́zes encontramos a = 1. Pela soma, segue que b = −2. Logo
a − b = 3.
2. Assim como no Exemplo 1, mostre que tal inteiro n satisfaz n2 + n − 4k 2 = 0 para
algum k inteiro positivo. Segue então que o discriminante é um quadrado perfeito, o que
acontece apenas quando k = 0, logo a resposta é não.
√ √
3. Substituindo t = x2 , as raı́zes da equação são ± x1 e ± x2 , onde x1 e x2 são raı́zes
√ √ √
da equação t2 − 40t + q = 0. Como as quatro estão em PA, segue que x2 − x1 = 2 x1 .
Daı́, x1 + x2 = x1 + 9x1 = 10x1 = 40. Obtemos que x1 = 4 e x2 = 36, donde segue que
q = 144.
4. Sejam x1 e x2 as raı́zes da equação. Escreva a2 + b2 = (x1 + x2 )2 + (x1 x2 − 1)2 e fatore.
5. Mostre que as raı́zes da equação tem que ser os números − a+3 a+1 a−1
2 ,− 2 ,− 2 e − 2 .
a−3

Além disso, os dois do meio são raı́zes de uma equação e os dois da ponta de outra. Segue
que (− a+3 a−3 a+1 a−1
2 )(− 2 ) + (− 2 )(− 2 ) = (a − b) + (a + b) = 2a. Resolva esta equação do
segundo grau encontrando os pares (−1, 1) e (5, 1).
6. Como as duas soluções são inteiras, seu discriminante é um quadrado perfeito, ou seja,
(r + s)2 − 4rs − 4 × 2010 = t2 para algum inteiro t. Fatorando e analisando paridade,
segue que 2010 é o produto de dois inteiros r−s+t2 e r−s−t
2 . Dado qualquer par de inteiros
(d, 2010/d) tal que d divide 2010 encontramos uma solução correspondente com r − s =
d + 2010/d. Como 2010 tem 16 divisores positivos, segue que a expressão toma 8 valores
positivos distintos.
7. Analise o discriminante das três equações e mostre que todos não podem ser negativos
ao mesmo tempo.
8. Note que a + b tem que ser positivo, e que a2 + b2 ≥ a + b também. Analisando os
divisores de 2001 e notando que a2 + b2 nunca deixa resto 3 dividido por 4, restringimos
o problema a dois casos: a + b = 1 e a2 + b2 = 2001 ou a + b = 29 e a2 + b2 = 69.
Resolvendo o sistema em cada um dos casos, encontramos, em cada um, uma equação do
segundo grau em uma variável (a ou b, dependendo da sua preferência). Como buscamos
soluções inteiras, os discriminantes tem que ser quadrados perfeitos. Verifique que isto
não acontece.
9. Suponha que pq é uma fração irredutı́vel que é raiz da equação. Substitua na equação
e analise a “paridade” da expressão obtida.
10. De maneira análoga ao Exemplo 4, a condição equivale a (2x+y)2 ≡ −27y 2 ≡ −(2y)2
mod 23. Suponha que 23 não divide y. Como 2 é invertı́vel, segue que −1 é resı́duo
quadrático módulo 23. Contudo, uma simples verificação braçal mostra que isto é falso.
Concluı́mos que y ≡ 0 mod 23 e consequentemente x ≡ 0 mod 23. Em contrapartida,
qualquer par que satisfaz estas condições satisfaz a do enunciado. O valor da expressão é
minimizado pelo múltiplo de 23 mais próximo de 103, ou seja, x + y = 92. O mı́nimo é
portanto 112 = 121.

8
Programa Olı́mpico de Treinamento
Curso de Álgebra - Nı́vel 2 Aula Complementar I
Prof. Hugo Araújo

Funções quadráticas - Parte 2

Dando continuidade a esta aula, iremos agora considerar funções dadas pela fórmula
f (x) = ax2 + bx + c
onde a 6= 0, b e c são números reais. Na primeira parte, estudamos as soluções da equação
f (x) = 0 para a função f escrita acima. Nesta parte, estaremos mais interessado no estudo
dos possı́veis valores de f (x) e como estes dependem de x.

2) Função do segundo grau


Uma função f : R → R é dita quadrática ou do segundo grau quando existem números
reais a 6= 0, b e c tais que
f (x) = ax2 + bx + c
para todo x ∈ R. O termo a costuma ser chamado de coeficiente lı́der. Assim como para
equações, dizemos que ∆ = b2 − 4ac é o discriminante da função.
Começamos escrevendo a função em uma forma mais útil, também conhecida como
forma canônica. Seguindo as manipulações feitas na demonstração da fórmula de Bhaskara
na primeira parte desta aula, encontramos a seguinte relação
" 2 #
b b2 − 4ac
f (x) = ax2 + bx + c = a x + − .
2a 4a2

Podemos então notar que esta função se comporta de maneira semelhante a x2 quando
a > 0 e semelhante a −x2 quando a < 0: seu gráfico é como o gráfico destas funções, a
menos de uma translação por (+b/2a) no eixo x e −∆/4a2 no eixo y, seguida de uma di-
latação por |a| na direção y. Mais abaixo iremos analisar como as diferentes possibilidades
para estes valores influenciam o gráfico de f .
Antes disso, algumas consequências da forma canônica acima. Note que o termo entre
parênteses é o quadrado de um número real e portanto é maior ou igual a 0. Consequen-
temente, quando a > 0, o menor valor de f (x) é atingindo quando este quadrado é igual
a 0.
Ou seja, o mı́nimo de f é atingido quando x = −b/2a e este mı́nimo é igual a −∆/4a.
Analogamente, para a < 0, a função tem um máximo que é atingido também quando
x = −b/2a e este máximo é igual a −∆/4a. O ponto
 
b −∆
V = − ,
2a 4a
é chamado de vértice da parábola que representa o gráfico de f .

1
POTI 2020 - Álgebra - Nı́vel 2 - Aula Complementar - Prof. Hugo Araújo

Nossa análise a respeito do gráfico será dividida em casos, dependendo do sinal de a e


∆.
Caso 1.1) a > 0 e ∆ < 0:
Pela forma canônica, segue que
" 2 #
b ∆
a · f (x) = a2 x+ − 2 >0
2a 4a

para todo x ∈ R. Segue que f (x) é positivo para todo x ∈ R.


Caso 1.2) a > 0 e ∆ = 0:
Procedendo da mesma forma acima, obtemos a · f (x) ≥ 0 para todo x ∈ R, com
igualdade apenas quando x = −b/2a, ou seja, no vértice da parábola.
Caso 1.3) a > 0 e ∆ > 0:
Neste caso, pela aula anterior, sabemos que a equação f (x) = 0 tem duas raı́zes reais
distintas x1 e x2 . Além disso, vimos que

f (x) = ax2 + bx + c = a(x − x1 )(x − x2 ).

Assumindo sem perda de generalidade que x1 < x2 e analisando o sinal de cada um


dos parênteses do lado direito obtemos:
• f (x) > 0 se x < x1 ou x > x2 ;

• f (x) < 0 se x1 < x < x2 .


Os outros casos são análogos, basta trocar o sinal de f na conclusão.
Caso 2.1) a < 0 e ∆ < 0:
Temos f (x) negativo para todo x ∈ R.
Caso 2.2) a < 0 e ∆ = 0:
Temos f (x) ≤ 0 para todo x ∈ R, com igualdade apenas para x = −b/2a.
Caso 2.3) a < 0 e ∆ > 0:
Se x1 < x2 são as raı́zes de f (x) = 0,
• f (x) < 0 se x < x1 ou x > x2 ;

• f (x) > 0 se x1 < x < x2 .


Finalmente, note que f (x) = f (y) se e somente se

b 2 b 2
   
b
x+ = y+ ⇐⇒ x = y ou x+y =− .
2a 2a a
Segue também desta observação que o gráfico de f é simétrico em relação à reta
(−b/2a, y), y ∈ R. Mais ainda, supondo que a > 0, a partir da fórmula canônica segue
que f é crescente no intervalo [−b/2a, ∞) e decrescente no intervalo (−∞, −b/2a]. As
ilustrações na próxima página exemplificam cada um dos casos acima.

2
POTI 2020 - Álgebra - Nı́vel 2 - Aula Complementar - Prof. Hugo Araújo

V + +
x x x
V x1 − x2

V
1.1 : a > 0 e ∆ < 0 1.2 : a > 0 e ∆ = 0 1.3 : a > 0 e ∆ > 0

+ x2
V
x x x
x1
V − −

2.1 : a < 0 e ∆ < 0 2.2 : a < 0 e ∆ = 0 2.3 : a < 0 e ∆ > 0

Figura 1: Os possı́veis gráficos de uma função quadrática. Note que o módulo do valor de
a é maior no exemplo 1.3 e menor no exemplo 2.1. Supondo que o eixo y está no centro
de cada figura, o exemplo 1.1 indica um valor de b menor que 0, já o exemplo 2.3 um valor
de b maior que zero.

Começamos com um exemplo simples porém importante, relacionado ao cálculo do


máximo de uma função quadrática.
Exemplo 1. Fixe um número inteiro positivo n. Sejam x e y dois números inteiros tais
que x + y = n. Qual é o maior valor possı́vel para o produto xy? Quando ele é atingido?

Solução. Podemos escrever y = n − x, donde segue que xy = x(n − x) = −x2 + nx. Como
−1 é negativo, a função f (x) = −x2 + nx atinge um máximo para x = n/2. Logo se n é
par, o máximo é atingido quando x = y = n/2 e ele é igual a n2 /4.
Se n é ı́mpar, n/2 não é inteiro, contudo uma análise do gráfico indica que xy atinge
seus maiores valores para escolhas de x próximas de n/2. Esta intuição pode ser confirmada
ao analisarmos a forma canônica
n 2 n 2
 
f (x) = − x − − .
2 4
Esta expressão atinge seu valor máximo para valores de x que tornam o quadrado en-
tre parênteses mı́nimo. Dentre os inteiros, isto acontece para os dois valores de x mais

3
POTI 2020 - Álgebra - Nı́vel 2 - Aula Complementar - Prof. Hugo Araújo

próximos de n/2. Se escrevermos n = 2n0 + 1, estes valores são n0 e n0 + 1. Em ambos os


2
casos a expressão é igual a n 4−1 . 

Pelo exemplo acima, podemos concluir que o produto xy toma valores maiores quando
escolhemos x e y próximos de sua média n/2 = (x + y)/2. Por outro lado, toma valores
cada vez menores (inclusive valores negativos) quando escolhemos estes números de forma
que eles fiquem cada vez mais distantes de sua média. Este tipo de simetria pode ser
muito útil no estudo de funções quadráticas, como veremos no exemplo abaixo.
Exemplo 2. (OBM 2014) Sejam p e q inteiros. Sabendo que x2 + px + q é positivo para
todo x inteiro, prove que a equação x2 + px + q = 0 não possui solução real.

Solução. Suponha que existam raı́zes reais, ou seja, que ∆ ≥ 0. Note que nenhuma delas
pode ser inteira, porque isso contradiz a hipótese de x2 + px + q ser positivo para todo x
inteiro.
Se ∆ = 0, então p2 = 4q, donde segue que p é par e as duas raı́zes são iguais a p/2,
que é inteiro. Logo este caso não pode acontecer.
Por outro lado, se ∆ > 0, as duas raı́zes x1 < x2 da equação tem que estar entre dois
inteiros consecutivos z e z + 1, pois caso contrário existiria algum inteiro r tal que x1 <
r < x2 e pela análise que fizemos anteriormente f (r) teria que ser negativo, contrariando
a hipótese do enunciado.
Como x1 + x2 = −p, segue que 2z < −p < 2z + 2, e como p é um inteiro, concluı́mos
que −p = 2z + 1. Logo existe r ∈ (0, 1/2) tal que
1 1
x1 = z + −r e x2 = z + + r.
2 2
Por outro lado x1 x2 = q, o que implica
1
z2 + z + − r2 = q ∈ Z.
4
Como z é inteiro, r2 tem que ser maior ou igual a 1/4. Isto contradiz o fato que r ∈ (0, 1/2).
Concluı́-se que a equação não admite raı́zes reais. 

Algumas perguntas interessantes podem ser feitas a respeito das parábolas que repre-
sentam os gráficos de funções quadráticas.
Exemplo 3. (Rússia 1998) O ângulo formado pelos raios y = x e y = 2x (x ≥ 0) delimita
dois arcos na parábola dada por y = x2 + px + q, onde p e q são números reais. Prove
que a diferença entre os comprimentos das projeções de cada um dos arcos sobre o eixo x
é igual a 1.

Solução. Inicialmente, esboçamos a situação descrita pelo problema para que possamos
compreendê-la melhor. A figura a seguir está esticada na direção horizontal para melhorar
o entendimento.

4
POTI 2020 - Álgebra - Nı́vel 2 - Aula Complementar - Prof. Hugo Araújo

y
y = 2x y=x
y = x2 + px + q

x1 x2 x3 x4 x

O problema nos pede para demonstrar que (x4 − x3 ) − (x2 − x1 ) = 1.


Como x1 e x4 correspondem a interseção de y = x2 + px + q com y = 2x, eles são raı́zes
da equação x2 + px + q = 2x ⇐⇒ x2 + (p − 2)x + q = 0 e portanto x1 + x4 = −(p − 2).
Analogamente, x2 e x3 são raı́zes da equação x2 + px + q = x ⇐⇒ x2 + (p − 1)x + q = 0
e portanto x2 + x3 = −(p − 1).
Substituindo estas descobertas na expressão que queremos calcular, obtemos

(x4 − x3 ) − (x2 − x1 ) = x4 + x1 − (x2 + x3 ) = −(p − 2) + (p − 1) = 1,

como querı́amos demonstrar.

Nosso último exemplo envolve a análise da trajetória de pontos após sucessivas aplicações
da associação x 7→ f (x).

Exemplo 4. (OBM 2007) Seja f (x) = x2 + 2007x + 1. Prove que, para todo inteiro
positivo n, a equação
f (f (. . . (f (x)) . . .)) = 0
| {z }
n vezes

possui ao menos uma solução real.

Solução. Para n = 1, basta mostra que a equação f (x) = 0 tem raiz real, o que é claro
pois seu discriminante é ∆ = 20072 − 4 > 0. Sejam x1 < x2 as raı́zes.
Para n = 2, precisamos mostrar que f (x) = x1 ou f (x) = x2 admite alguma raiz real.
A soma x1 + x2 é igual a −2007, logo x2 > −2007/2. Note porém, que o mı́nimo de f
é igual a −∆/4 = −20072 /4 + 1, que é um número bastante negativo, muito menor que
−2007/2. Isto indica que f (x) = x2 admite solução real.
Para resolver o problema, mostremos que para todo d > −∆/4 = −20072 /4 + 1, a
equação f (x) = d admite alguma solução maior que −2007/2. Com efeito, f (x) = d é

5
POTI 2020 - Álgebra - Nı́vel 2 - Aula Complementar - Prof. Hugo Araújo

equivalente a
x2 + 2007x + (1 − d) = 0,
cujo discriminante é 20072 − 4(1 − d) = 20072 + 4d − 4 > 20072 − 20072 + 4 − 4 = 0. Mais
ainda, a soma das raı́zes da equação é igual a −2007, sendo assim, pelo menos uma delas
é maior que −2007/2.
Podemos agora argumentar por indução. Afirmamos que para todo n a equação fun-
cional admite solução e que ao menos uma delas é maior que −2007/2.
O caso n = 1 já está feito.
Suponha que seja verdade para n. Seja y uma solução maior que −2007/2 da equação

f (f (. . . (f (x)) . . .)) = 0.
| {z }
n vezes

Pelo que provamos no terceiro parágrafo, a equação f (x) = y admite alguma solução z
com z maior que −2007/2. Este número real z é claramente solução de

f (f (. . . (f (x)) . . .)) = 0.
| {z }
n+1 vezes

Isto conclui a indução e portanto a demonstração.

A ideia deste exercı́cio pode ser resumida por notar que, se xmin indica a abcissa do
vértice da parábola, a imagem por f do intervalo [xmin , ∞) contém o próprio intervalo
[xmin , ∞). Note também que se f (x) = ax2 + bx + c com a > 0, o argumento do terceiro
parágrafo mostra que f (x) = d admite solução real para todo d > −∆/4a.

Terminamos com os seguintes exercı́cios (12) para você praticar.

Problema 1. Para cada m ∈ R, considere a parábola que representa o gráfico da função


fm : R → R definida por fm (x) = (m2 + m + 1)x2 − 2(m2 + 1)x + m2 − m + 1. Mostre
que todas essas parábolas tem um ponto em comum.

Problema 2. (OBM 2007) Sejam α e β as raı́zes da equação quadrática (x–2)(x–3) +


(x–3)(x + 1) + (x + 1)(x–2) = 0. Determine o valor de
1 1 1
+ + .
(α + 1)(β + 1) (α − 2)(β − 2) (α − 3)(β − 3)

Problema 3. (São Petersburgo 2009) Seja f (x) = ax2 + bx + c uma função quadrática.
Seja também P o conjunto dos valores de f (x) para x par e I o conjunto dos valores de
f (x) para x ı́mpar. Mostre que P = I ou P ∩ I = ∅.

Problema 4. (OBM 2006) Encontre todos os pares ordenados (x, y) de inteiros tais que
x3 − y 3 = 3(x2 − y 2 ).

6
POTI 2020 - Álgebra - Nı́vel 2 - Aula Complementar - Prof. Hugo Araújo

Problema 5. (Coreia) É possı́vel, através de aplicações sucessivas das seguintes trans-


formações:
   
2 1 2 1
f (x) 7→ x · f +1 ou f (x) 7→ (x − 1) · f ,
x x−1

partir da função f (x) = x2 + 5x + 4 e terminar na função g(x) = x2 + 10x + 8 ?

Problema 6. (Aberto do Canadá 2011) Seja f (x) = x2 − ax + b uma função quadrática,


onde a e b são inteiros positivos.

a) Suponha que a = 2 e b = 2. Determine o conjunto de raı́zes de f (x) − x, e o conjunto


de raı́zes f (f (x)) − x.

b) Determine o número de pares de inteiros positivos (a, b) tais que 1 ≤ a, b ≤ 2011


para os quais cada todas as raı́zes de f (f (x)) − x sejam inteiras.

Problema 7. (Seleção Cone Sul) Demonstre que, se para todo n inteiro não nulo, existe
x ∈ Z tal que n | x2 + bx + c (b, c ∈ Z), então x2 + bx + c = 0 tem raı́zes inteiras.

Problema 8. (Coreia 2018) Seja f uma função quadrática que satisfaz a seguinte condição:
para quaisquer números reais a e b distintos, se f (a) = f (b), então f (a2 −6b−1) = f (b2 +8).
f (8) − f (2)
Encontre o valor de .
f (2) − f (1)

Problema 9. (São Petersburgo 2008) Sejam a e b números reais. O gráfico de y =


x2 + ax + b intersecta os eixos coordenados nos pontos A, B e C. O circumcentro do
triângulo ABC é um ponto da reta y = x. Prove que a + b + 1 = 0.

Problema 10. (Rússia 2006) Dados a e b reais, seja f (x) = x2 + ax + b. Suponha que a
equação f (f (x)) = 0 tem quatro raı́zes reais distintas e que a soma de duas dessas raı́zes
seja igual a −1. Prove que b ≤ − 14 .

Problema 11. Seja f (x) = ax2 + bx + c uma função quadrática tal que a > 0 e f (0) = 4.
Encontre todos os valores possı́veis de a, b e c tais que 0 ≤ f (x) ≤ 4 para todo x ∈ [0, 3].

7
POTI 2020 - Álgebra - Nı́vel 2 - Aula Complementar - Prof. Hugo Araújo

Dicas e Soluções
1. Queremos encontrar um ponto (x0 , y0 ) tal que y0 = fm (x0 ) para todo m ∈ R. Basta
encontrar x0 ∈ R tal que o valor de fm (x0 ) seja independente de m. Sendo assim reescreva
fm (x0 ) em termos de m:
fm (x0 ) = (x20 − 2x0 + 1)m2 + (x20 − 1)m + x20 − 2x0 + 1.
Encontre x0 tal que a expressão acima seja uma função constante em termos de m.
2. Expandindo a expressão, verifique que α e β são raı́zes de uma equação do segundo
grau com discriminante positivo, o que implica que α − β 6= 0. Seja I o valor da expressão.
Note que  
1 1 1 1
= − ·
(α + 1)(β + 1) α+1 β+1 α−β
e a expressão análoga vale para −2 e −3 no lugar de 1. Sendo assim (α − β)I é igual a
1 1 1 1 1 1
+ + − − −
α+1 α−2 α−3 β+1 β−2 β−3
e expandindo esta expressão em duas frações, uma em função de α e outra em função β,
observamos que seus numeradores são iguais a expressão do enunciado calculadas em α e
β respectivamente. Como são as raı́zes, isto implica que o resultado dá 0 e portanto I = 0.
3. Lembre da observação ao fim da página 2. Se os conjuntos tem interseção então existem
m e n inteiros tais que f (2n) = f (2m + 1). Isto implica que 2n + 2m + 1 = −b/a.
Segue também que dado 2k par, basta tomar l tal que n − k = l − m e observar que
2k + (2l + 1) = −b/a para concluir que f (2k) = f (2l + 1). O argumento similar pode ser
feito para o caso ı́mpar. Isto prova que os conjuntos são iguais.
4. A equação é claramente satisfeita quando x = y, logo (a, a) com a ∈ Z é solução.
Supondo que x 6= y, podemos fatorar o x − y dos dois lados, obtendo
x2 + (y − 3)x + y 2 − 3y = 0.
Enxergando como uma equação em x, seu discriminante é igual a −3y 2 + 6y + 9, que tem
como raı́zes −1 e 3. Como queremos soluções inteiras, logo pelo menos reais, esta expressão
tem que ser não-negativa e portanto −1 ≤ y ≤ 3. Como y é inteiro, seus valores podem
ser apenas −1, 0, 1, 2 ou 3. Resolvendo a equação em cada um dos casos correspondentes,
encontramos as outras soluções (x, y) = (2, −1), (−1, 2), (3, 0), (0, 3).
5. Observe que o discriminante de uma função quadrática ax2 + bx + c é invariante por
estas transformações. Note que os discriminantes do destino e da chegada são distintos.
6. Para a letra a), f (x) − x = 0 ⇐⇒ x2 − 3x + 2 = 0, e pela fórmula de Bhaskara as
raı́zes são 2 e 1. Para encontrar as soluções de f (f (x)) − x = 0, escreva y = f (x). A
equação vira um sistema de equações
x2 − ax + b = y
y 2 − ay + b = x.

8
POTI 2020 - Álgebra - Nı́vel 2 - Aula Complementar - Prof. Hugo Araújo

Subtraindo, ficamos com (x−y)(x+y) = (x−y)(a−1) e portanto x = y ou x+y = a−1. O


primeiro caso equivale a x2 −(a+1)+b = 0. Já o segundo equivale a x2 −(a−1)x+b+1−a =
0.
Para a letra a), o segundo caso nos dá uma equação de discriminante negativo, logo as
soluções são, novamente, apenas 1 e 2.
Para que todas as soluções de f (f (x)) − x = 0 sejam inteiras, como vimos na aula
anterior, é necessário e suficiente que o discriminante seja um quadrado perfeito. Ou seja,
que (a+1)2 −4b = a2 +2a+1−4b e (a−1)2 −4(b+1−a) = a2 +2a−3−4b sejam quadrados
perfeitos. Note que estes dois quadrados tem diferença igual a 4, logo (a + 1)2 − 4b = 4
e (a − 1)2 − 4(b + 1 − a) = 0. Como 1 ≤ b ≤ 2011, estamos interessados nos valores de a
ı́mpares tais que 8 ≤ (a + 1)2 ≤ 4 ∗ 2011 + 4 = 8048 ≈ 89.72 . Segue então que para todo
a entre 3 e 87 existe uma única solução, sendo assim existem 43 pares de soluções.
7. Suponha que a equação não admita raı́zes inteiras. Então ∆ = b2 − 4c não é quadrado
perfeito. Assim, existem p primo, r um inteiro primo com p, e k ı́mpar tais que ∆ = pk r.
Tome n = pk+1 . Como n | x2 + bx + c para algum valor de x inteiro, existe algum m inteiro
diferente de 0 tal que a equação x2 + bx + c = mn ⇐⇒ x2 + bx + c − mn = 0 admite
raı́zes inteiras. Isto implica que o discriminante desta equação é um quadrado perfeito.
Mas ele é igual a b2 − 4c + 4mn = pk r + 4pk+1 m = pk (r + 4pm), que não pode ser quadrado
perfeito pois contém um número ı́mpar (k) de fatores primos p.
8. Escreva f na forma canônica, f (x) = m(x − r)2 + n. A condição implica que se
a + b = 2r, então a2 − 6b − 1 = b2 + 8 ou a2 − 6b − 1 + b2 + 8 = 2r . Mostre que o segundo
caso nunca acontece, ou seja, que a soma não pode ser sempre igual a 2r. O primeiro caso
implica que r = −3/2. A expressão pode ser calculada apenas com o conhecimento deste
valor. O resultado é 13.
9. Dois vértices estão no eixo x (correspondendo às raı́zes x1 < x2 ) e um no eixo y. Seja
C = (0, b) o que está no eixo Y . Seja O o circumcentro. Prove que O = ( x1 +x 2 ,
2 x1 +x2
2 ).
x1 +x2 x1 +x2
Seja D = ( 2 , 0) e E = (0, 2 ). Considere os triângulos retângulos ODA, ODB e
OEC. Prove que eles são congruentes. Segue que
x1 + x2 x2 − x1
−b = .
2 2
Consequentemente, b = x1 ou b = x2 . A prova é concluı́da com mais algumas mani-
pulações.
10. Sejam x1 e x2 as raı́zes de f (x) = 0. Se as duas raı́zes de f (f (x)) = 0 que somam
−1, z e w, são tais que f (z) = f (w), então a = 1. Como as 4 raı́zes são distintas,
os discriminantes de f (x) = x1 e f (x) = x2 são positivos, donde 1 − 4b + 4x1 ≥ 0 e
1 − 4b + 4x2 ≥ 0. Somando, encontramos a desigualdade pedida.
Caso contrário, f (w) + f (z) = x1 + x2 = −a. Logo,

w2 + z 2 + a(w1 + w2 ) + 2b = −a =⇒ b ≤ −(w2 + z 2 )/2 ≤ −1/4

por causa da desigualdade das médias.

9
POTI 2020 - Álgebra - Nı́vel 2 - Aula Complementar - Prof. Hugo Araújo

11. Observe que c = 4, pois f (0) = 4. Além disso, f tem que ser decrescente perto do 0,
portanto −b/2a tem que ser positivo. Mais ainda, pela simetria em relação ao eixo que
passa por este ponto, −b/2a ≥ 3/2, caso contrário f (3) > 4.
Se −b/2a ≤ 3, temos −6a ≤ b ≤ −3a. Neste caso, f satisfaz as condições do enunciado
se e só se o seu mı́nimo é maior ou igual a 0, ou seja, (−b2 + 16a) ≥ 0. Isto implica

que (3a)2 ≤ 16a, logo 0 < a ≤ 16/9. Como b é negativo, segue também que b ≥ −4 a.

Note que −4 a < −6a se e só se a ∈ (0, 4/9). Logo, dentro deste caso, para valores de
a ∈ (0, 4/9), a função satisfaz as condições se e só se b ∈ [−6a, −3a] e para valores de

a ∈ [4/9, 16/9] se e só se b ∈ [−4 a, −3a] .
Se −b/2a ≥ 3, é necessário e suficiente que f (3) ≥ 0, ou seja, 9a + 3b + 4 ≥ 0. Isto
implica que −3a − 4/3 ≤ b ≤ −6a. Em particular, −3a − 4/3 ≤ −6a ⇐⇒ a ≤ 4/9. Tente
refletir porque obtemos uma cota pra a similar à anterior. Encontramos mais um intervalo
possı́vel de soluções: para a ∈ (0, 4/9), este caso funciona se e só se b ∈ [−3a − 4/3, −6a].
A resposta é c = 4, a ∈ (0, 4/9) e b ∈ [−3a − 4/3, −3a] ou a ∈ [4/9, 16/9] e b ∈

[−4 a, −3a].

10
Problemas Resolvidos

Nı́vel 2

Produtos Notáveis

Material elaborado por Hugo Fonseca Araújo


Problemas

Problema 1. Prove a identidade de Brahmagupta: seja n ∈ N; então

(a2 + nb2 )(c2 + nd2 ) = (ac − nbd)2 + n(ad + bc)2 = (ac + nbd)2 + n(ad − bc)2 .

Problema 2. a) Mostre que (a + b + c)2 = a2 + b2 + c2 + 2ab + 2bc + 2ca.


1 1 1
b) Determine o valor da expressão x2 + y 2 + z 2 quando x + y + z = 13, xyz = 72 e x + y + z = 34 .

Problema 3. (Kiev Festival 2007) Encontre todos os pares de inteiros positivos (a, b) tais que
√ √ √
a − 1 + b − 1 = ab − 1.

Problema 4. Determine x2 + y 2 e x4 + y 4 quando x3 + y 3 = 6 e x + y = 1.

Problema 5. Considere uma sequência {an }n∈N de números reais satisfazendo a0 > 0 e an+1 = an + a1n
para n ≥ 0.
Prove que, para qualquer valor do real positivo a0 , a1996 é maior que 63.

1 1 1
Problema 6. Quantos pares de inteiros positivos (a, b) satisfazem a + b = 2004 ?

Problema 7. (OBM 2003) Mostre que x2 + 4y 2 − 4xy + 2x − 4y + 2 > 0, quaisquer que sejam os reais
x e y.

{z } − 22...22
Problema 8. Prove que |11...11 | {z } é um quadrado perfeito.
2n dı́gitos n dı́gitos

Problema 9. Para quais números naturais n, o número n4 − 11n2 + 49 é primo?

Problema 10. a) (Pan-Africana 2004) O número


√ √
q q
4 4 − 2 3 + 97 − 56 3

é inteiro?

b) (Irlanda 2016) Seja


251 1
a= 1 √
√ + 251 √
√ .

3 − 10 3 63 √ + 10 3 63
252−5 3 2 3
252+5 3 2

Mostre que a3 é inteiro.

Problema 11. (OBM 2007) Ache todos os pares (x, y) de inteiros positivos tais que

2(x + y) + xy = x2 + y 2 .

2
Problema 12. (Lusófona 2018) Determine os pares de inteiros positivos m e n que satisfazem a
equação m2 = n2 + m + n + 2018.

Problema 13. Dados dois inteiros x e y tais que x2 + 2y é um quadrado perfeito, mostre que x2 + y
é a soma de dois quadrados perfeitos.

Problema 14. (Japão 2019) Se d(x) expressa o número de divisores positivos de um inteiro positivo
x, encontre o menor inteiro positivo n tal que d(n2 ) = d(n2 + 72019 ).

Problema 15. Sejam a e b inteiros positivos. Se c = a + b, mostre que a4 + b4 + c4 é igual a duas


vezes um quadrado perfeito.

Problema 16. Encontre todos os pares de inteiros positivos (x, y) tais que
√ √ √ √
xy = x + y + x + y.

Problema 17. (Torneio das Cidades 1985) Um quadrado é dividido em 5 retângulos da maneira
desenhada na figura abaixo. Cada um dos seus 4 vértices pertence a um dentre 4 retângulos cujas
áreas são iguais, e o quinto retângulo não tem nenhum ponto em comum com os lados do quadrado.
Mostre que o quinto retângulo é um quadrado.

√ √ √
Problema 18. Sejam a, b e c números racionais positivos tais que a+ b+ c também é racional.
√ √ √
Mostre que a, b e c são racionais.

Problema 19. Sejam a, b e c números reais tais que a2 + b2 = c2 . Encontre as soluções reais (x, y, z)
do sistema (
z 2 = x2 + y 2
(z + c)2 = (x + a)2 + (y + b)2 .

Problema 20. (Alemanha 2015) Determine todos os pares de números reais (x, y) que são soluções
de (
x3 + 9x2 y = 10
y 3 + xy 2 = 2.

Problema 21. (Cone-Sul 2014) Seja n um inteiro positivo. Mostre que

n2 − 22014 × 2014 n + 42013 (20142 − 1)

não é um número primo.

3
Problema 22. (Holanda 2011) Determine todos os pares de números reais positivos (a, b) com a > b
satisfazendo as seguintes equações:
√ √ √ √
a a + b b = 134 e a b + b a = 126.

Problema 23. (Centro-americana 2018) Sejam x e y números reais tais que x − y, x2 − y 2 e x3 − y 3


são todos números primos. Prove que x − y = 3.

Problema 24. Se a, b e c são reais não nulos que satisfazem a + b + c = 0, calcule

(a3 + b3 + c3 )2 · (a4 + b4 + c4 )
.
(a5 + b5 + c5 )2

Problema 25. Fatore x4 + x2 + 1 e (x + y + z)3 − x3 − y 3 − z 3 .

Problema 26. (Polônia 2018) Existem números reais positivos a, b, c, x tais que a2 + b2 = c2 e
(a + x)2 + (b + x)2 = (c + x)2 ?

Problema 27. Considere o triângulo ABC cujos comprimentos dos lados são a, b e c, onde a é o
maior lado. Prove que ABC é retângulo se, e somente se,
√ √ √ √ √
( a + b + a − b)( a + c + a − c) = (a + b + c) 2.

Problema 28. (Turquia 2006) Encontre todas as triplas de números inteiros (x, y, z) tais que
(
x − yz = 11
xz + y = 13.

Problema 29. a) Mostre que (m2 − n2 )2 + (2mn)2 = (m2 + n2 )2 .


Observe que se m e n são inteiros positivos com m > n, então a = m2 −n2 , b = 2mn e c = m2 +n2
formam uma terna pitagórica, ou seja, são inteiros positivos que satisfazem a2 + b2 = c2 .

b) Mostre que toda terna pitagórica (a, b, c) na qual a, b e c são primos entre si dois a dois se escreve
como a = m2 − n2 , b = 2mn e c = m2 + n2 , onde m e n são inteiros primos entre si.

Problema 30. Este exercı́cio tem vários itens esobre uma relação importante envolvendo cubos de
três números.

a) Sejam a, b e c números reais tais que a + b + c = 0. Mostre que a3 + b3 + c3 − 3abc = 0.

b) Fatore a3 + b3 + c3 − 3abc. Observe que a afirmação da letra a) indica que (a + b + c) é um dos


fatores.

c) Fatore (a + 2b − 3c)3 + (b + 2c − 3a)3 + (c + 2a − 3b)3 .

d) Encontre todos os pares (x, y) de números reais positivos que satisfazem

x3 + y 3 = 3xy − 1.

4
Problema 31. (Ásia-Pacı́fico 2000) Calcule
101
X x3i i
2 , onde xi = .
i=0
1 − 3xi + 3xi 101

Problema 32. a) Usando o exercı́cio 30, mostre que se a, b e c são números reais positivos, então

a+b+c √
3
≥ abc.
3
Quando vale a igualdade?

b) Sejam a, b e c números reais positivos tais que

a3 + b3 + c3 = (a + b − c)3 + (a − b + c)3 + (b + c − a)3 .

Prove que a = b = c.

5
Soluções
1. Expandindo as expressões, encontramos

• (a2 + nb2 )(c2 + nd2 ) = a2 c2 + na2 d2 + nb2 c2 + n2 b2 d2

• (ac − nbd)2 + n(ad + bc)2 = a2 c2 − 2nacbd + n2 b2 d2 + n(a2 d2 + 2adbc + b2 c2 ) = a2 c2 + na2 d2 +


nb2 c2 + n2 b2 d2

• (ac + nbd)2 + n(ad − bc)2 = a2 c2 + 2acnbd + n2 b2 d2 + n(a2 d2 − 2adbc + b2 c2 ) = a2 c2 + na2 d2 +


nb2 c2 + n2 b2 d2 ,

e a igualdade fica evidente.

2. a) (a + b + c)2 = (a + b)2 + 2(a + b)c + c2 = a2 + 2ab + b2 + 2ac + 2bc.


yz+zx+xy
b) A última relação se escreve como xyz = 43 .
Juntando com a segunda, segue que xy + yz + zx = 54. Pela fórmula da letra a) e a primeira
relação, obtemos

132 = (x + y + z)2 = x2 + y 2 + z 2 + 2(xy + yz + zx) = x2 + y 2 + z 2 + 2 · 54 =⇒ x2 + y 2 + z 2 = 61.

3. Elevamos os dois lados ao quadrado:


p p
a − 1 + b − 1 + 2 (a − 1)(b − 1) = ab − 1 ⇐⇒ 2 (a − 1)(b − 1) = ab − a − b + 1 = (a − 1)(b − 1).

Escrevendo (a − 1)(b − 1) = t, vem que 2 t = t, implicando que t = 4. Segue que (a − 1)(b − 1) = 4,
e, como a e b são inteiros, obtemos a − 1 = 1, a − 1 = 2, ou a − 1 = 4. Assim, os pares (a, b) que
resolvem a equação são (2, 5), (3, 3) e (5, 2).

4. Usando o produto notável para o cubo da soma e o quadrado da soma vem que

x3 + y 3 = (x + y)(x2 − xy + y 2 ) = (x + y)[(x + y)2 − 3xy]

Daı́, pela hipótese no enunciado, segue que 6 = 1 · (1 − 3xy), o que implica que xy = −5/3. Como
x2 + y 2 = (x + y)2 − 2xy, segue que x2 + y 2 = 13/3. Note que

(x3 + y 3 )(x + y) = x4 + xy 3 + x3 y + y 4 = x4 + y 4 + xy(x2 + y 2 ).

Daı́, x4 + y 4 = 6 · 1 − (− 35 · 13
3 ) = 119
9 .

5. Elevando ao quadrado, obtemos a2n+1 = a2n + 2an · a1n + a12 = a2n + 2 + a12 . Segue que a2n+1 > a2n + 2
n n
e, consequentemente, a21996 > a0 + 1996 · 2 > 3992, pois a0 > 0. Mas como 632 = 3969 < 3992 < a21996 ,
segue que a1996 > 63.

6. Expandindo a equação, obtemos


a+b 1
= ⇐⇒ 2004(a + b) = ab.
ab 2004

6
Somando 20042 dos dois lados, ficamos com

20042 = 20042 − 2004a − 2004b + ab = (2004 − a)(2004 − b).

Dessa maneira, 2004 − a divide 20042 e 2004 − b também. Observe que ambos inteiros são menores
que 2004, pois a e b são positivos. Daı́, como seu produto tem que ser igual a 20042 , ambos tem que
ser negativos. Note que cada escolha 2004 − a = d, com d | 20042 e d negativo, determina um par de
2
soluções, a = 2004 − d e b = 2004 − 2004 2 2 2
d . Como 2004 fatorado em primos é 2004 = (2 · 3 · 167) ,
2

ele tem 45 divisores negativos, donde há 45 soluções para o problema original.

7. Devemos começar tentando fatorar a expressão:

x2 +4y 2 −4xy +2x−4y +2 = (x−2y)2 +2(x−2y)+2 = (x−2y)2 +2(x−2y)+1+1 = (x−2y +1)2 +1.

Apesar de não termos fatorado a expressão, como o quadrado de um número real é maior ou igual
a 0, segue que a expressão original é maior ou igual a 1, logo é maior que 0.

8. Note que 10n − 1 é um número formado por n dı́gitos, todos eles iguais a 9. Dessa forma, o número
que aparece no enunciado é igual a

102n − 1 10n − 1 102n − 2 · 10n + 1 10 − 1 2


 n 
−2 = = ,
9 9 9 3
e como o número entre parênteses é inteiro, pois 10n − 1 é múltiplo de 9, temos um quadrado perfeito.

9. A expressão pode ser fatorada:

n4 − 11n2 + 49 = (n2 + 7)2 − 25n2 = (n2 − 5n + 7)(n2 + 5n + 7).

Como n é natural, o segundo termo é sempre maior que o primeiro. Para que o resultado seja primo,
é necessário que n2 − 5n + 7 = 1 ou n2 + 5n + 7 = −1. Resolvendo a primeira equação, encontramos
n = 2 ou n = 3. Mas neste caso, n2 + 5n + 7 é igual a 21 ou 31 respectivamente. Já a segunda equação
não tem raı́zes reais. Segue que a expressão resulta em um número primo apenas quando n = 3.

10. a) A ideia deste exercı́cio


√ é tentar
√ usar o produto notável para o quadrado da soma. Note que
1 + 3 = 4. Logo, 4 − 2 3 = (1 − 3)2 .
√ √
Analogamente, observamos inicialmente √ que 56 3 = 2 · 28 3. Se encontramos
p dois números
a e b tais 2 2 2
que a + b = 97 e ab = 28 3, a segunda raiz será igual a (a − b) . Escrevemos

28 3
b = a , donde, fazendo t = a2 ,

2 2 2 2352 2 97 ± 972 − 4 · 2352 97 ± 1
a + b = a + 2 = 97 =⇒ t − 97t + 2352 = 0 =⇒ t = = ,
a 2 2
√ √
e concluı́mos que a = 7 e b = 48 = 4 3 é uma possı́vel solução. √ Lembrando√que a raiz
quadrada é sempre não-negativa, a expressão original é igual a 4( 3 − 1) + 7 − 4 3 = 3, um
número inteiro.
√ √
b) Façamos√x = 3 252 e y = 5 3 2. Atente ao fato que x3 − y 3 = 2 e x3 + y 3 = 2 · 251. Além disso,
xy = 10 3 63. Assim, o denominador na primeira expressão é igual a
x3 − y 3 √
3 x2 + xy + y 2 x2 − xy + y 2
− 10 63 = − xy =
2(x − y) 2 2

7
e, analogamente, o denominador da segunda é igual a
x3 + y 3 √
3 x2 − xy + y 2 x2 + xy + y 2
+ 10 63 = + xy = .
2(x + y) 2 2
Dessa forma,
x3 + y 3 x3 − y 3 √
3
a= 2 2
+ 2 2
= (x + y) + (x − y) = 2x = 2 · 252
x − xy + y x + xy + y
e fica claro que a3 = 8 · 252 é inteiro.

11. Multiplicando por 2 dos dois lados e passando os termos para a direita, a equação fica:
2(x2 + y 2 ) − 4(x + y) − 2xy = 0 ⇐⇒ (x − 2)2 + (y − 2)2 + (x − y)2 = 8.
Como x e y são inteiros, o número 8 está escrito como soma de três quadrados perfeitos (0 sendo
considerado quadrado perfeito). Mas a única forma de fazer isto é 8 = 0 + 4 + 4. Daı́ segue que
x − y = 0, x − y = 2 ou y − x = 2.
No primeiro caso, temos x − 2 = 2 e y − 2 = 2, e daı́ x = y = 4 é uma solução.
No segundo, se y − 2 = 2 obtemos x = 6 e y = 4, mas daı́ x − 2 = 4, logo não fornece solução. Se
y − 2 = 0, obtemos x = 4 e y = 2, e verificamos que esta é uma solução.
O terceiro caso é idêntico ao segundo quando trocamos os papéis de x e y.
Os pares que resolvem a equação são (4, 4), (4, 2) e (2, 4).

12. Escrevemos a equação de uma maneira equivalente mais simples:


m2 = n2 + m + n + 2018 ⇐⇒ m2 − n2 − (m + n) = 2018 ⇐⇒ (m + n)(m − n − 1) = 2018.
Como o número 2018 é igual a 2 × 1009, o produto de dois primos, e m + n é maior que m − n − 1,
pois m e n são inteiros positivos, segue que as possı́veis soluções se dividem em dois casos:
• m + n = 1009 e m − n − 1 = 2 =⇒ m = 506 e n = 503.
• m + n = 2018 e m + n − 1 = 1 =⇒ m = 1010 e n = 1008.

2 2 2 2
13. Suponha que x2 + 2y = Z 2 . Segue que y = Z −x 2 e então x2 + y = Z +x
2 . Analisando a fórmula
para o quadrado da soma, notamos que (a + b) + (a − b) = a + 2ab + b + a − 2ab + b2 = 2(a2 + b2 ).
2 2 2 2 2

Dessa maneira, a relação acima se escreve como


Z 2 + x2 2(Z 2 + x2 ) (Z + x)2 + (Z − x)2 Z +x 2 Z −x 2
   
2
x +y = = = = + .
2 4 4 2 2
Observe que x e Z tem a mesma paridade, pois x2 + 2y = Z 2 . Assim, os termos dentro dos parêntese
na equação acima são ambos inteiros e x2 + y é a soma de dois quadrados perfeitos.

14. Note que d(x) é um número ı́mpar se, e somente, se x é um quadrado perfeito. Isto implica que
n2 + 72019 também tem que ser um quadrado perfeito, pois d(n2 + 72019 ) = d(n2 ).
Escrevendo m2 = n2 + 72019 , obtemos (m − n)(m + n) = 72019 . Quanto menor for n, mais próximos
serão os fatores m − n e m + n, assim, como o segundo é maior que o primeiro, o menor n tal que
n2 +72019 é quadrado perfeito é atingido quando m−n = 71009 e m+n = 71010 . Neste caso, n = 3·71009
e m2 = n2 + 72019 = 16 · 72018 = 24 · 72018 , mas infelizmente 5 · 2019 = d(n2 + 72019 ) > d(n2 ) = 3 · 2019.
Por outro lado, quando m − n = 71008 e m + n = 71011 , encontramos n = 171 · 31008 = 32 · 19 · 71008 e
m = 172 · 31008 = 22 · 43 · 71008 , portanto d(m2 ) = d(n2 + 72019 ) = d(n2 ). Concluı́mos que n = 171 · 31008
é o menor valor.

8
15. Começamos calculando c4 :

c2 = (a2 + b2 + 2ab) =⇒ c4 = (a2 + b2 )2 + 2 · 2ab(a2 + b2 ) + (2ab)2 = a4 + b4 + 6a2 b2 + 4ab(a2 + b2 ).

Daı́,

a4 + b4 + c4 = 2a4 + 2b4 + 6a2 b2 + 4ab(a2 + b2 ) = 2[(a4 + b4 + 2a2 b2 ) + 2ab(a2 + b2 ) + a2 b2 ] =


2[(a2 + b2 )2 + 2ab(a2 + b2 ) + (ab)2 ] = 2(a2 + b2 + ab)2

que é o dobro de um quadrado perfeito.


16. Passando o termo x + y para o lado esquerdo e elevando ao quadrado, obtemos
√ √ √ √ √ √ √
( xy − x + y)2 = ( x + y)2 ⇐⇒ xy − 2 x + y xy + x + y = x + 2 xy + y.

Cancelando os termos em comum dos dois lados e dividindo por xy, que é positivo, a equação
equivale a √

xy = 2( x + y + 1).
Elevando ao quadrado dos dois lados, obtemos

xy = 4(x + y + 2 x + y + 1).

Isto implica que x + y é um inteiro positivo e xy é um quadrado perfeito par. Escrevendo

xy = 4a2 com a ∈ N e manipulando a segunda equação acima, obtemos a − 1 = x + y. Substituindo
na equação original, obtemos √ √
x + y = 2a − (a − 1) = a + 1.
√ √
Dessa forma, x e y são raı́zes da equação do segundo grau

t2 − (a + 1)t + 2a = 0.

Para que as soluções sejam inteiras, seu ∆ tem que ser um quadrado perfeito b2 . Assim,

∆ = (a + 1)2 − 8a = b2 =⇒ (a − 3)2 − 8 = b2 .

Os únicos quadrados perfeitos que distam 8 um do outro1 são 1 e 9. Dessa forma, a − 3 = 3, o que
implica que a = 6. Substituindo na equação em t acima, ficamos com t2 − 7t + 12 = 0, o que implica
que t = 3 ou t = 4. Segue daı́ que (9, 16) e (16, 9) são os únicos pares que resolvem a equação original.

17. Suponha que o quadrado que é dividido em retângulos tenha lados iguais a 1. Sempre podemos
supor isto ampliando ou reduzindo a figura. Sejam a e b os lados de um dos quatro retângulos que
contém os vértices. Como ab é a área deste retângulo, usando o fato que os quatro tem a mesma área,
podemos encontrar os lados dos outros retângulos, como indicado na figura.

1− ab ab
1−a 1−a
1−a
ab
1− 1−b

a
ab
1−b
1−b b
1
Note que se a > 3, (a + 1)2 − a2 = 2a + 1 ≥ 9. Basta checar casos pequenos então.

9
Segue daı́ que   
ab ab
1− 1− = ab.
1−b 1−a
Multiplicando os dois lados por (1 − a)(1 − b), ficamos com

[(1 − b) − ab] · [(1 − a) − ab] = ab(1 − a)(1 − b) ⇐⇒


⇐⇒ (1 − a)(1 − b) − ab(2 − a − b) + (ab)2 = ab(1 − a − b + ab) ⇐⇒
⇐⇒ 1 − a − b + ab − 2ab + a2 b + ab2 + (ab)2 = ab − a2 b − ab2 + (ab2 ) ⇐⇒
⇐⇒ 1 − a − b − 2ab + 2a2 b + 2ab2 = 0 ⇐⇒
⇐⇒ 1 − a − b − (1 − a − b) · 2ab = 0 ⇐⇒ (1 − a − b)(1 − 2ab) = 0.

Note que ab < 1/4, pois a soma das áreas dos quatro retângulos é menor que a área do quadrado
original, que é igual a 1. Segue daı́ que 1 − 2ab 6= 0 =⇒ 1 − a − b = 0 =⇒ 1 − a = b. Substituindo
o valor de b, encontramos que todos os retângulos tem lados iguais a a e 1 − a. Concluı́mos a partir
daı́ que o retângulo central é um quadrado de lados 2a − 1, supondo que a corresponde ao maior dos
lados de qualquer um dos retângulos que contém vértices do quadrado inicial.

√ √ √
18. Seja a+ b+ c = q ∈ Q. Se q = 0, então a = b = c = 0. Se q 6= 0, segue que

√ √ 2 √ 2
√ 2 √ √ q 2 + c − a − 2 ab − b
( a+ b) = (q − c) ⇐⇒ a + 2 ab + b = q − 2q c + c ⇐⇒ c= .
2q
√ √
Daı́, c = p1 · ab + p2 com

1 a + b − q2 − c
p1 = − e p2 = −
q 2q

números racionais. Note que p1 6= 0. Por outro lado, q 2 = a + b + c + 2ab + 2bc + 2ca e então
q 2 + c − a − b > 0, donde p2 6= 0. Elevando ao quadrado novamente, segue que
√ √ c − p21 ab − p22 √
c = p21 ab + 2p1 p2 ab + p22 =⇒ ab = ∈ Q =⇒ c ∈ Q.
2p1 p2
√ √
Poderı́amos progredir de forma completamente análoga para concluir que a e b também são
racionais.

19. A segunda igualdade se escreve

z 2 + 2zc + c2 = x2 + 2xa + a2 + y 2 + 2yb + b2 .

Usando que a2 + b2 = c2 e x2 + y 2 = z 2 , segue que 2zc = 2xa + 2yb =⇒ ax + by = cz.


Multiplicando a condição em a, b e c pela primeira equação, segue que

(x2 + y 2 )(a2 + b2 ) = z 2 c2 = (ax + by)2 =⇒ a2 y 2 + b2 x2 = 2abxy =⇒ (ay − bx)2 = 0.

Encontramos assim ay − bx = 0. Multiplicando x2 + y 2 = z 2 por a2 e substituindo esta relação,


obtemos
a
a2 z 2 = a2 x2 + a2 y 2 = (a2 + b2 )x2 = c2 x2 =⇒ x = ± z.
c
Daı́, como ay − bx = 0 e ax + by = cz, segue que x = ac z e y = cb z. É fácil verificar que para cada
z ∈ R a tripla ac z, cb z, z resolve o problema.


10
20. A primeira equação se escreve como x3 + 3(x2 · 3y) = 10. Já a segunda, quando multiplicada por
27, se escreve como (3y)3 + 3 · x(3y)2 = 54. Somando os dois termos, obtemos

x3 + 3(x2 · 3y) + 3 · x(3y)2 + (3y)3 = (x + 3y)3 = 64 =⇒ x + 3y = 4.

Substituindo 3y = 4 − x na primeira equação ficamos com

x3 + 3[x2 · (4 − x)] = 10 ⇐⇒ x3 − 6x2 + 5 = 0.

É fácil ver que 1 é solução da equação acima. Isto indica que é possı́vel fatorar a expressão acima
usando algum fator (x − 1). Com efeito,

x3 −6x2 +5 = x3 −x2 −5x2 +5 = x2 (x−1)−5(x2 −1) = x2 (x−1)−5(x+1)(x−1) = (x−1)(x2 −5x−5).

Daı́, as soluções da equação acima são


√ √
5+3 5 5−3 5
x = 1, x= , x= .
2 2
Assim, os pares de solução são
√ √ ! √ √ !
5+3 5 1− 5 5−3 5 1+ 5
(1, 1) , , e , .
2 2 2 2

21. Observe que 42013 = (22013 )2 . Daı́,

n2 − 22014 × 2014n + 42013 (20142 − 1) = n2 − 2 · 22013 · 2014 · n + (22013 )2 · 20142 − (22013 )2 =


(n − 22013 · 2014)2 − (22013 )2 = (n − 22013 · 2014 − 22013 )(n − 22013 · 2014 + 22013 ) =
(n − 22013 · 2015)(n − 22013 · 2013).

Assim a expressão se escreve com produto de dois números inteiros. Ela só pode resultar em um
número primo quando um dos fatores tem módulo igual a 1 e o outro tem módulo igual a um número
primo e o mesmo sinal do outro fator. Isto, devido ao fato que o segundo fator é maior que o primeiro,
nos deixa com dois casos:

• n − 22013 · 2015 = 1 =⇒ n − 22013 · 2013 = 1 + 22013 · 2 = 1 + 22014 . Mas, como 24 ≡ 1 mod 5,


22014 ≡ 22 ≡ 4 mod 5. Segue daı́ que 1 + 22014 é múltiplo de 5 e o número original não é primo.

• n − 22013 · 2013 = −1 =⇒ n − 22013 · 2015 = −1 − 22013 · 2 = −(1 + 22014 ). Neste caso seguimos
a análise anterior e concluı́mos que o número também não é primo.

√ √
22. Escreva a=xe b = y. As equações se escrevem como

x3 + y 3 = 134
x2 y + xy 2 = 126.

Somando a primeira com três vezes a segunda obtemos

(x + y)3 = x3 + y 3 + 3x2 y + 3xy 2 = 512 =⇒ x + y = 8.

Por outro lado, subtraindo as duas equações, ficamos com

x3 + y 3 − x2 y + xy 2 = x(x2 − y 2 ) − y(x2 − y 2 ) = x(x + y)(x − y) − y(x + y)(x − y) = (x + y)(x − y)2 = 8.

11
E como x + y = 8 obtemos x − y = 1 ou x − y = −1.
No primeiro caso, resolvendo o sistema linear, encontramos x = 92 e y = 72 , donde a = 81 49
4 e b= 2 .
7 9
No segundo caso, resolvendo o sistema linear, encontramos x = 2 e y = 2 , o que implica b > a.
Mas como estamos interessados apenas em pares (a, b) com a > b, podemos desconsiderar este caso.
81 49
A única solução é a = eb= .
4 2

23. Sejam p, q e r primos, e x − y = p, x2 − y 2 = q e x3 − y 3 = r. Lembramos que x e y não são


necessariamente inteiros. Comparando a segunda equação com a primeira, encontramos
q
q = x2 − y 2 = (x + y)(x − y) = p(x + y) =⇒ x + y = .
p
Resolvendo o sistema linear (
q
x+y = p
x−y =p
q + p2 q − p2 2 −p4
encontramos x = e y= . Isto implica que xy = q 4p 2 . A terceira equação pode ser
2p 2p
combinada com as outras duas e esta última relação de forma a obter
r = x3 − y 3 = (x − y)(x2 + xy + y 2 ) = (x − y)[(x + y)2 − xy] =
"  #
q 2 q 2 − p4 3q 2 − p4
p − = .
p 4p2 4p

E daı́ segue que p(4r − p3 ) = 3q 2 . Como p e q são primos entre si, segue que 3 | p. Como p é primo,
segue que p = 3. Note que existe solução, por exemplo, p = 3, q = 5 e r = 13 são obtidos quando
x = 37 e y = − 32 . Isto conclui a prova.

24. Da relação inicial, sabemos que c = −(a + b). O objetivo é escrever a expressão em termos de
(a + b) e ab. Primeiramente, note que (a + b)3 = a3 + b3 + 3a2 b + 3ab2 = a3 + b3 + 3ab(a + b). Isto
implica que a3 + b3 = (a + b)3 − 3ab(a + b). Por outro lado,
(a2 + b2 )2 = a4 + b4 + 2a2 b2 =⇒ a4 + b4 = [(a + b)2 − 2ab]2 − 2(ab)2 ,
e (a2 + b2 )(a3 + b3 ) = a5 + b5 + a3 b2 + a2 b3 implica que
a5 + b5 = [(a + b)2 − 2ab] · [(a + b)3 − 3ab(a + b)] − (ab)2 (a + b).
Expandindo as expressões acima, ficamos com
a4 + b4 = (a + b)4 − 4ab(a + b)2 + 2(ab)2
a5 + b5 = (a + b)5 − 5ab(a + b)3 + 5(ab)2 (a + b).
Substituindo na expressão original, usando o fato que c = −(a + b) ficamos com
[(a + b)3 − 3ab(a + b) − (a + b)3 ]2 · [(a + b)4 − 4ab(a + b)2 + 2(ab)2 + (a + b)4 ]
=
[(a + b)5 − 5ab(a + b)3 + 5(ab)2 (a + b) − (a + b)5 ]2
[−3ab(a + b)]2 · [2(a + b)4 − 4ab(a + b)2 + 2(ab)2 ]
=
[−5ab(a + b)3 + 5(ab)2 (a + b)]2
9[(ab)2 (a + b)2 ] · 2[(a + b)4 − 2ab(a + b)2 + (ab)2 ]
=
25[(ab)2 (a + b)6 − 2(ab)3 (a + b)4 + (ab)4 (a + b)2 ]
18 (ab)2 (a + b)6 − 2(ab)3 (a + b)4 + (ab)4 (a + b)2 18
· = .
25 (ab)2 (a + b)6 − 2(ab)3 (a + b)4 + (ab)4 (a + b)2 25

12
25. Para fatorar a primeira expressão usamos o truque de somar e subtrair x2 :

x4 + x2 + 1 = x4 + 2x2 + 1 − x2 = (x2 + 1)2 − x2 = (x2 + x + 1)(x2 − x − 1).

Já para a segunda basta usar produtos notáveis de cubos repetidamente

(x + y + z)3 − x3 − y 3 − z 3 = (x + y + z)3 − x3 − (y 3 + z 3 ) =
(x + y + z − x) · [(x + y + z)2 + (x + y + z)x + x2 ] − (y + z) · (y 2 − yz + z 2 ) =
(y + z)(x2 + y 2 + z 2 + 2xy + 2yz + 2zx + x2 + yx + zx + x2 ) − (y + z)(y 2 − yz + z 2 ) =
(y + z)(3x2 + 3xy + 3yz + 3zx) = 3(y + z)[x(x + y) + z(y + x)] =
3(y + z)(x + z)(x + y),

onde na passagem da segunda linha para a terceira usamos a fórmula para (x + y + z)2 encontrado no
primeiro exercı́cio desta lista. A segunda expressão então fatora como

(x + y + z)3 − x3 − y 3 − z 3 = 3(x + y)(y + z)(z + x).

26. Suponha que existam tais números. Expandindo a igualdade do enunciado, obtemos

(a + x)2 + (b + x)2 = (c + x)2 ⇐⇒ a2 + 2ax + x2 + b2 + 2bx + x2 = c2 + 2cx + x2 .

Como a2 + b2 = c2 e x > 0, a equação acima nos dá x = 2(c − b − a). Mas isto implica que c > a + b,
novamente porque x > 0. Contudo segue daı́, como a, b e c também são positivos, que

c2 > (a + b)2 = a2 + 2ab + b2 = c2 + 2ab =⇒ 2ab < 0,

uma contradição com a e b positivos. Logo não existem tais números.

27. Pelo teorema de Pitágoras, o triângulo é retângulo se e somente se a2 = b2 + c2 . Por outro lado,
como os dois lados da igualdade no enunciado são positivos, segue que
√ √ √ √ √
( a + b + a − b)( a + c + a − c) = 2(a + b + c) ⇐⇒
√ √ √ √ √
( a + b + a − b)2 · ( a + c + a − c)2 = [ 2(a + b + c)]2 ⇐⇒
p p
(a + b + 2 (a + b)(a − b) + a − b) · (a + c + 2 (a + c)(a − c) + a − c) = 2(a + b + c)2 ⇐⇒
p p
(2a + 2 a2 − b2 ) · (2a + 2 a2 − c2 ) = 2(a + b + c)2 .

Suponhamos primeiro que o triângulo é retângulo. Daı́, como a2 = b2 + c2 ,


p p
(2a + 2 a2 − b2 ) · (2a + 2 a2 − c2 ) = 2(a + b + c)2 ⇐⇒
2(a + c)(a + b) = a2 + b2 + c2 + 2ab + 2bc + 2ca ⇐⇒
2a2 + 2ab + 2bc + 2ca = a2 + b2 + c2 + 2ab + 2bc + 2ca ⇐⇒
a2 = b2 + c2 ,

que é verdade.
Suponha por outro lado que ABC não é retângulo. Para concluir o problema, basta mostrar que a
igualdade acima não
√ acontece. p
Pelo teorema de Pitágoras,
√ temos dois casos:√a2 > b2 +c2 e a2 < b2 +c2 .
No primeiro caso, a2 − b2 > (b2 + c2 ) − b2 > c2 = c e analogamente a2 − c2 > b. Segue que

p p
(2a + 2 a2 − b2 ) · (2a + 2 a2 − c2 ) > (2a + 2c)(2a + 2b) =
2(2a2 + 2ab + 2bc + 2ca) > 2(a2 + b2 + c2 + 2ab + 2bc + 2ca) = 2(a + b + c)2 ,

13
logo a igualdade não é verdadeira. √ p √
Por outro lado, se a 2 < b2 + c2 , vem que a 2 − b2 < (b2 + c2 ) − b2 < c2 = c e analogamente

2 2
a − c < b. Segue que
p p
(2a + 2 a2 − b2 ) · (2a + 2 a2 − c2 ) < (2a + 2c)(2a + 2b) =
2(2a2 + 2ab + 2bc + 2ca) < 2(a2 + b2 + c2 + 2ab + 2bc + 2ca) = 2(a + b + c)2 ,

donde se conclui a equivalência.

28. Somando as duas equações elevadas ao quadrado ficamos com

(x − yz)2 + (xz + y)2 = 112 + 132 ⇐⇒ x2 − 2xyz + y 2 z 2 + x2 z 2 + 2xyz + y 2 = 290 ⇐⇒


x2 + y 2 + x2 z 2 + y 2 z 2 = (x2 + y 2 )(z 2 + 1) = 290 = 2 × 5 × 29.

Como os números x, y e z são inteiros, eles tem que ser divisores de 290. Como este número tem
8 divisores, analisamos 8 casos. Se z 2 + 1 =

• 1, então z = 0 e isto implica que x = 11 e y = 13 substituindo no sistema original.

• 2, então z = 1 ou z = −1. Substituindo z = 1 no sistema original ele vira x−y = 11 e x+y = 13,
e daı́ x = 12 e y = 1. Substituindo z = −1 e resolvendo o sistema linear que aparece, obtemos
x = −1 e y = 12.

• 5, então z = 2 ou z = −2. Substituindo no sistema z = 2 e resolvendo encontramos x = 37/5 e


y = −9/5, logo não são inteiros. Analogamente, fazendo z = −2, vem que x = −3 e y = 7.

• 29, então z não é inteiro.

• 10, então z = 3 ou z = −3. Quando z = 3, o sistema linear que obtemos nos dá como soluções
x = 5 e y = −2. Quando z = −3 obtemos x = −14/5 e y = 23/5.

• 58, então z não é inteiro.

• 145, então z = 12 ou z = −12. Mas neste caso x2 + y 2 = 2 e daı́ x2 = y 2 = 1. Testando os casos


encontramos x = 1, y = −1 e z = 12; e x = −1, y = 1 e z = −12 como soluções.

• 290, então z não é inteiro.

Dessa forma, as soluções são

(11, 13, 0), (12, 1, 1), (−1, 12, −1), (−3, 7, −2), (5, −2, 3), (1, −1, 12), (−1, 1, −12).

29. a) Este item é simples, basta usar os produtos notáveis:

(m2 − n2 )2 + (2mn)2 = m4 − 2m2 n2 + n4 + 4m2 n2 = m4 + 2m2 n2 + n4 = (m2 + n2 )2 .

b) Se a, b e c são inteiros primos entre si dois a dois que formam uma terna pitagórica, então, como
a2 + b2 = c2 , obtemos que c é ı́mpar e um dos dois números a ou b é par e o outro ı́mpar. Com
efeito, se c fosse par, então a e b teriam que ser ı́mpares, mas neste caso o lado esquerdo deixaria
resto 2 quando dividido por 4 e o direito seria múltiplo de 4, o que é impossı́vel. Suponhamos
sem perda de generalidade que b é par.

14
Daı́, b2 = c2 − a2 = (c + a)(c − a), e se k divide c + a e c − a então k divide (c + a) + (c − a) = 2c
e (c + a) − (c − a) = 2a. Dessa forma, k divide 2, pois a e c são primos entre si. De fato,
mdc(c + a, c − a) = 2, pois ambos são pares. Como b é par, 4 divide b2 . Assim,
 2
b c+a c−a
= ·
2 2 2
e como os dois números do lado direito são primos entre si obtemos c + a = 2m2 e c − a = 2n2
para algum par de inteiros m e n primos entre si. Segue daı́ que a = m2 − n2 , c = m2 + n2 e
b2 = (c + a)(c − a) = 2m2 · 2n2 = 4m2 n2 =⇒ b = 2mn. 

30. a) Note que c = −(a + b). Daı́,


c3 = −(a + b)3 = −(a3 + 3a2 b + 3ab2 + b3 ),
donde segue que
a3 + b3 + c3 − 3abc = a3 + b3 − (a3 + 3a2 b + 3ab2 + b3 ) − 3ab[−(a + b)]) =
a3 + b3 − a3 − b3 − 3a2 b − 3ab2 + 3a2 b + 3ab2 = 0.

b) a3 + b3 + c3 − 3abc = (a + b)(a2 − ab + b2 ) + c3 − 3abc. Como observado no enunciado, a + b + c


é um bom candidato a fator. Isto nos incentiva a somar e subtrair c(a2 − ab + b2 ) na expressão
acima. Daı́, obtemos
a3 + b3 + c3 − 3abc = (a + b)(a2 − ab + b2 ) + c3 − 3abc + c(a2 − ab + b2 ) − c(a2 − ab + b2 ) =
(a + b + c)(a2 − ab + b2 ) + c3 − 3abc − c(a2 − ab + b2 ) =
(a + b + c)(a2 − ab + b2 ) + c3 − 3abc − c[(a + b)2 − 3ab] =
(a + b + c)(a2 − ab + b2 ) + c3 − c(a + b)2 =
(a + b + c)(a2 − ab + b2 ) + c[c2 − (a + b)2 ] =
(a + b + c)(a2 − ab + b2 ) + c(c + a + b)(c − a − b) =
(a + b + c)[a2 − ab + b2 + c(c − a − b)] =
(a + b + c)(a2 + b2 + c2 − ab − bc − ca).

c) Escreva x = a + 2b − 3c, y = b + 2c − 3a e z = c + 2a − 3b. Note que x + y + z = 0. Segue daı́,


pela letra a), que x3 + y 3 + z 3 = 3xyz. Obtemos então,
(a + 2b − 3c)3 + (b + 2c − 3a)3 + (c + 2a − 3b)3 = 3(a + 2b − 3c)(b + 2c − 3a)(c + 2a − 3b).

d) Faça a = x, b = y e c = 1. A equação se escreve como a3 + b3 + c3 − 3abc = 0. Pela letra b), o


lado esquero fatora como
(a + b + c)(a2 + b2 + c2 − ab − bc − ca) = a3 + b3 + c3 − 3abc = 0 ⇐⇒
(x + y + 1)(x2 + y 2 + 1 − x − y − xy) = 0.
Segue que x + y = −1 ou x2 + y 2 + 1 − x − y − xy = 0. No primeiro caso, um dos dois números,
x ou y, tem que ser negativo. No segundo, podemos multiplicar a expressão por 2 e encontrar
2x2 + 2y 2 + 2 − 2x − 2y − 2xy = x2 − 2x + 1 + y 2 − 2y + 1 + x2 − 2xy + y 2 = 0 ⇐⇒
(x − 1)2 + (y − 1)2 + (x − y)2 = 0.
O lado esquerdo só se anula quando x = y = 1. Segue que esta é a única solução da equação.

15
31. Observe que, pelo produto notável para o cubo da soma,
x3i x3i
2 = .
1 − 3xi + 3xi (1 − xi )3 + x3i
i 101 − i
Note porém que 1 − xi = 1 − = = x101−i . Logo,
101 101
101 101 101 101
X x3i X x3i X x3i X x3i
2 = 2 = + =
i=0
1 − 3xi + 3x2i i=0
(1 − xi )3 + x3i i=0
(x101−i )3 + x3i i=0
(x101−i )3 + x3i
101 101 101 3 101
X x3i X x3101−j X xi + (x101−i )3 X
+ = = 1 = 102.
i=0
(x101−i )3 + x3i j=0
(xj )3 + x3101−j i=0
(x101−i )3 + x3i i=0

102
Segue que a soma pedida é igual a 2 = 51.

32. a) Escreva a = x3 , b = y 3 e c = z 3 . Daı́


a+b+c √
3
≥ abc ⇐⇒ x3 + y 3 + z 3 − 3xyz ≥ 0 ⇐⇒ (x + y + z)(x2 + y 2 + z 2 − xy − yz − zx) ≥ 0,
3
por causa da fatoração encontrada na letra b) do exercı́cio 30. Como a, b e c são positivos, x, y
e z também são, e a desigualdade acima equivale a
x2 + y 2 + z 2 − xy − yz − zx ≥ 0.
Multiplicando por 2, obtemos a desigualdade equivalente
2x2 + 2y 2 + 2z 2 − 2xy − 2yz − 2zx ≥ 0 ⇐⇒
⇐⇒ x2 − 2xy + y 2 + y 2 − 2yz + z 2 + z 2 − 2zx + x2 ≥ 0 ⇐⇒
⇐⇒ (x − y)2 + (y − z)2 + (z − x)2 ≥ 0,
o que é sempre verdade, pois um número real elevado ao quadrado sempre resultam em um
número não-negativo. Observe que a igualdade acontece somente quando
x − y = 0, y − z = 0 e z − x = 0 =⇒ a = b = c.

b) Expandimos o primeiro termo do lado direito usando produtos notáveis:


(a + b − c)3 = (a + b)3 − 3(a + b)c(a + b − c) − c3 =
a3 + 3a2 b + 3ab2 + b3 − 3a2 c − 3abc + 3ac2 − 3abc − 3b2 c + 3bc2 − c3 =
a3 + b3 − c3 + 3a2 b + 3ab2 + 3ac2 + 3bc2 − 3a2 c − 3b2 c − 6abc.
Trocando as posições de a, b e c na expressão acima, encontramos
(a − b + c)3 = a3 − b3 + c3 + 3a2 c + 3ac2 + 3ab2 + 3b2 c − 3a2 b − 3bc2 − 6abc
(b + c − a)3 = −a3 + b3 + c3 + 3b2 c + 3bc2 + 3a2 b + 3a2 c − 3ab2 − 3ac2 − 6abc,
donde segue que
(a + b − c)3 + (a − b + c)3 + (b + c − a)3 = a3 + b3 − c3 + a3 − b3 + c3 − a3 + b3 + c3 +
(3a2 b − 3a2 b + 3a2 b) + (3ab2 + 3ab2 − 3ab2 ) + (−3b2 c + 3b2 c + 3b2 c) + (3bc2 − 3bc2 + 3bc2 )+
(3c2 a + 3c2 a − 3c2 a) + (−3ca2 + 3ca2 + 3ca2 ) − 18abc =
a3 + b3 + c3 + 3(a2 b + ab2 + b2 c + bc2 + c2 a + ca2 − 6abc)

16
Dessa maneira, a igualdade equivale a

a2 b + ab2 + b2 c + bc2 + c2 a + ca2 = 6abc.


2 2 2 2 2 2
Mas
√ pela desigualdade
√ √ das médias aplicada termo a termo, a b + ab + b c + bc + c a + ca ≥
3 3 3 3 3 3
2 a b + 2 b c + 2 c a , e por outro lado, pela letra a),
√ √ √ 3 √ √ √ √
q
6
3 3 3 3 3
2 a b +2 b c +2 c a ≥2·3 3 a3 b3 · b3 c3 · c3 a3 = 6 a6 b6 c6 = 6abc.
√ √ √
Só existe igualdade quando a3 b3 = b3 c3 = c3 a3 , o que implica que a = b = c.

Material elaborado por Hugo Fonseca Araújo

17
Problemas Resolvidos

Nı́vel 2

Equações e Sistemas de Equações

Material elaborado por Hugo Fonseca Araújo


Problemas

Problemas que envolvem manipulações inteligentes ou substituições


Problema 1. (OBM 1997) Encontre as soluções reais de
(
x2 − xy − y 2 + 1 = 0
x3 − x2 y − xy 2 + x − y + 2 = 0.

Problema 2. (OBM 1997) Quantos pares (x, y) de inteiros satisfazem a equação x + y + xy = 120?

Problema 3. (OBM 2006) Sejam a e b números reais distintos tais que a2 = 6b + 5ab e b2 = 6a + 5ab.
Determine a + b e ab.

Problema 4. (OBM 2005) Os inteiros positivos (x, y) satisfazem a equação


r r
1√ 1√
x+ y− x− y = 1.
2 2
Qual das alternativas apresenta um possı́vel valor de y?
A) 5 B) 6 C) 7 D) 8 E) 9

Problema 5. Prove que a equação


1 1 2
√ √ +√ √ =√ √
x+ 1006 2012 − x + 1006 x + 2012 − x
tem exatamente 2013 soluções inteiras.

Problema 6. Os números reais a, b e c são distintos dois a dois e tais que:


1 1 1
a+ =b+ =c+ .
b c a
Encontre todos os valores possı́veis de abc.

Problema 7. (OBM 2012) Quantas soluções reais tem o sistema


 1
x + y = z 2

y + z = x12 ?
z + x = y12

Problema 8. Prove que 4x3 − 3x + 1 = 2y 2 tem ao menos 31 soluções com x e y inteiros positivos e
x ≤ 1980.

2
p
3
√ p
3
√ √
Problema 9. (OBM 2008 adaptado) Prove que −27 + 5 33 − 27 + 5 33 = − 3 18.

Problema 10. Encontre as soluções reais de


3 5 17 19
+ + + = x2 − 11x − 4.
x − 3 x − 5 x − 17 x − 19

Problema 11. Encontre as soluções complexas de

(3z + 1)(4z + 1)(6z + 1)(12z + 1) = 2.

Problema 12. (Armênia 1999) Resolva a equação


1 1
2
+ √ = 1.
x (4 − 3x)2

Problema 13. (OBM 2006) Sejam x, y, z números reais não nulos tais que x + y + z = 0. Qual é o
valor de  
2 2 2 1 1 1
(x y z ) + + ?
x3 y 3 x3 z 3 y 3 z 3

(a − b)(b − c)(c − a) 1
Problema 14. (OBM 2005) Dado que = , qual o valor de
(a + b)(b + c)(c + a) 11
a b c
+ + ?
a+b b+c c+a

Use também estimativas e desigualdades para resolver


Problema 15. Encontre as soluções reais de x2 + y 2 + z 2 + xy + yz + xz + 3/2 = 2(x + y + z).

Problema 16. Encontre todos os inteiros estritamente positivos x, y, z, w tais que x! + y! + z! = w!

Problema 17. (Irlanda 2002) Suponha que n seja o produto de quatro números primos a, b, c, d
distintos dois a dois tais que:
(i) a + c = d
(ii) a(a + b + c + d) = c(d − b)
(iii) 1 + bc + d = bd
Determine n.

Problema 18. (Asian-Pacific 2011) Sejam a, b e c inteiros positivos. Prove que é impossı́vel que os
três números a2 + b + c, b2 + c + a e c2 + a + b sejam quadrados perfeitos.

(
x3 + y 3 = 1
Problema 19. (Moscou 55) Encontre todas as soluções reais de .
x4 + y 4 = 1

3
Problema 20. (Arábia Saudita 2015) Encontre as soluções inteiras de x2 y 5 − 2x 5y = 2015 + 4xy.

Problema 21. (Norte da China 2013) Dado um número real x denotamos por bxc o maior inteiro n
tal que n ≤ x. Exemplos: b2c = b2, 5c = 2 e b−2c = b−1, 1c = −2. Quais são as soluções não-inteiras
da equação
13 13
x+ = bxc + ?
x bxc

Problema 22. (Ibero 2018) Para cada inteiro n ≥ 2, encontre todas as soluções inteiras do seguinte
sistema de equações:

x1 = (x2 + x3 + x4 + · · · + xn )2018
x2 = (x1 + x3 + x4 + · · · + xn )2018
..
.
xn = (x1 + x2 + x3 + · · · + xn−1 )2018 .

p √
Problema 23. Sejam x e y números reais tais que (x + 1 + y 2 )(y + 1 + x2 ) = 1. Mostre que
x + y = 0.

Problema 24. (Irlanda 2001) Determine os número reais x não-negativos para os quais
√ √
q q
3 3
13 + x + 13 − x

é um número inteiro.

Problema 25. Encontre todas as soluções reais positivas de

abc = (a + b − c)(b + c − a)(c + a − b).

Problema 26. (Vietnam 2015 (Regional)) Encontre as soluções reais do sistema:


(
y 4 − 2xy 2 + 7y 2 = −x2 + 7x + 8
√ p
3 − x + y 2 + 1 = x3 + x2 − 4y 2 + 3

4
Soluções

1. Multiplicando a primeira equação por x e subtraindo ela da segunda obtemos −y + 2 = 0. Segue


que y = 2 e substituindo na primeira equação obtemos x2 − 2x − 3 = 0 que tem como raı́zes x = 3 e
x = −1. As soluções são (−1, 2), (3, 2).

2. Somando 1 dos dois lados ficamos com

(x + 1)(y + 1) = 121,

logo x+1 (e y+1 também!) divide 121. Segue que (x, y) = (−122, −2), (−12, −12), (−2, −122), (0, 120),
(10, 10) e (120, 0) são todas as soluções.

3. Subtraindo a segunda equação da primeira obtemos a2 − b2 = (a − b)(a + b) = 6(b − a). Como


a 6= b, a − b 6= 0, e cancelando dos dois lados ficamos com a + b = −6. Por outro lado, somando as
duas equações ficamos com a2 + b2 = 6(a + b) + 10ab. Somando 2ab dos dois lados e fatorando obtemos
(a + b)2 = 6(a + b) + 12ab. Substituindo nossa resposta anterior vem que ab = 6.

1√ √
q q
4. Racionalizando, ou seja, multiplicando por x + 2 y + x − 12 y, obtemos
r r    
1√ 1√ 1√ 1√ √
x+ y+ x− y= x+ y − x− y = y.
2 2 2 2

Elevando ao quadrado e somando com o quadrado da equação no enunciado temos:


   
1√ 1√
1+y =2 x+ y +2 x− y = 4x,
2 2

logo y ≡ −1 mod 4. A única opção correspondente é a letra C).

5. Observe que a equação só faz sentido quando 0 ≤ x ≤ 2012, logo os 2013 inteiros entre esses dois
números tem que ser soluções. Isto nos leva a crer que a relação acima é uma identidade. Em verdade,
um número tão grande de soluções já poderia nos levar a crer isso neste caso. Racionalizando os
termos da equação obtemos:
√ √ √ √ √ √
1 1006 − x 1 2012 − x − 1006 2 2( 2012 − x − x)
√ √ = , √ √ = , √ √ = .
x + 1006 1006 − x 2012 − x + 1006 2012 − x − 1006 x + 2012 − x 2012 − x − x

Dessa forma, a equação original equivale a


√ √ √ √ √ √ √ √
1006 − x 2012 − x − 1006 2012 − x − x 2( 2012 − x − x)
+ = = ,
1006 − x 2012 − x − 1006 1006 − x 2012 − x − x
que é claramente uma identidade, e concluı́mos que todo inteiro entre 0 e 2012 é solução.

6. Expandindo, temos

abc + c = b2 c + b, abc + a = c2 a + c, abc + b = a2 b + a ⇐⇒


bc(a − b) = b − c, ca(b − c) = c − a, ab(c − a) = a − b

5
e multiplicando as três relações obtemos a2 b2 c2 (a − b)(b − c)(c − a) = (b − c)(c − a)(a − b). Como os
números são distintos dois a dois temos (abc)2 = 1 =⇒ abc = ±1. Resta verificar que há soluções
tais que abc = +1 e abc = −1. Substituindo abc = 1 obtemos um sistema equivalente a

c(1 − b)(1 + b) = b − 1

a(1 − c)(1 + c) = c − 1

b(1 − a)(1 + a) = a − 1

e fazendo a = 1 é fácil descobrir que b = −1/2 e c = −2 resolvem o sistema e assim 1 é de fato um


valor possı́vel. Para obter −1 como o produto abc basta inverter o sinal da solução anterior.

7. O sistema equivale a:  1
xz + yz =
 z
1
xy + zx = x
 1
zy + xy =

y
xy+zy+zx
e somando as equações obtemos 2(xy + yz + zx) = donde xy + yz + zy = 0 ou xyz = 1/2.
xyz ,
No primeiro caso, o sistema fica
 1
−xy = z

−yz = x1 ⇐⇒ xyz = −1.
 1
−zx = y

Fazendo z = − xy 1
, as equações originais viram xy = x1 + y1 = x+yxy . Note que se x e y satisfizerem
1
essa equação, ao tomarmos z = − xy , teremos uma solução do sistema original, logo basta encontrar
as soluções de xy = x+y 2 2
xy . Expandindo obtemos a equação do segundo grau x y − x − y = 0 e, √
1± 1+4x3
considerando x como um parâmetro, podemos aplicar a fórmula de Bhaskara obtendo y = 2x2
.
p
Basta então tomar x ≥ 3 1/4 e então
√ !
1 + 1 + 4x3 −2x
x, , √
2x2 1 + 1 + 4x3
será solução. Segue que existem infinitas soluções.

8. Multiplicando por 2, temos


8x3 − 6x + 2 = 4y 2 ⇐⇒ (2x3 ) − 3(2x) + 2 = 4y 2 ⇐⇒ (2x)3 − 2x − 2(2x) + 2 = 4y 2 ⇐⇒
(2x)((2x)2 − 1) − 2(2x − 1) = 4y 2 ⇐⇒ (2x − 1)((2x + 1)2x − 2) = 4y 2 ⇐⇒
(2x − 1)(4x2 + 2x − 2) = 4y 2 ⇐⇒ (2x − 1)(4x2 − 1 + 2x − 1) = 4y 2 ⇐⇒
(2x − 1)(2x − 1)(2x + 2) = (2y)2 .
Esta fatoração foi obtida separando os termos da equação de forma esperta. Poderı́amos também ter
fatorado diretamente o polinômio, mas para isso precisarı́amos chutar uma de suas raı́zes, −1 ou 1/2.
A primeira é um chute razoável e daı́ terı́amos 4x3 − 3x + 1 = (x + 1)p(x), na qual p(x) é um polinômio
do segundo grau e poderı́amos usar Bhaskara.1
Voltando ao problema, percebemos que sempre que 2x + 2 for um quadrado perfeito (logo par!)
existirá y inteiro positivo que satisfaz a igualdade. Precisamos então que 2x+2 = n2 e x ≤ 1980, logo o
problema se resume a encontrar o número de quadrados perfeitos pares menores que 2·1980+2 = 3962.
Usando o número sugerido verificamos que 622 < 3962, e assim cada n par entre 1 e 62 nos dá uma
solução (verifique que x será sempre positivo).
11 π 1
⁄2 pode parecer mais difı́cil de chutar, mas usando a fórmula para cos 3x poderı́amos deduzir que cos 3
= 2
é solução.

6
p3
√ p
3

9. Escreva α = −27 + 5 33 − 27 + 5 33. Temos:
√ 3 √ √
q q
3
α3 = −54 − 3α −27 + 5 33 27 + 5 33 = −54 − 3α( 96).
3


3

Observando que 96 = 25 · 3 somos incentivados a tomar 2 · 32 β = 3 18β = α, obtendo:
18β 3 = −54 − 36β ⇐⇒ 18β 3 + 36β + 54 = 0
e verificamos que −1 é uma solução desta equação.
Dividindo por (β + 1) obtemos 18β 3 + 36β + 54 = 18(β + 1)(β 2 − β + 3). Note porém que o
polinômio β 2 − β + 3 não tem raı́zes reais (!), pois seu ∆√é negativo. Como β é real (já que α o é) e
raiz do polinômio acima, segue que β = −1 e que α = − 3 18.

10. Somando 4 dos dois lados ficamos com


x x x x
+ + + = x2 − 11x
x − 3 x − 5 x − 17 x − 19
O ponto médio dos números 3, 5, 17, 19 é 11. Convém então por simetria escrever x = y + 11 e nossa
equação vira
y + 11 y + 11 y + 11 y + 11 y 2 + 11y y 2 + 11y
+ + + = y 2 + 11y ⇐⇒ 2 2 +2 2 = y 2 + 11y
y+8 y+6 y−6 y−8 y − 64 y − 36
donde y = 0, −11 são soluções, pois zeram y 2 + 11y. Assim temos x = 0 e x = 11 como soluções.
Excluindo esses valores, fazendo t = y 2 e algumas manipulações algébricas, a equação acima equivale
a √
t2 − 104t + 36 · 64 + 200 = 0 ⇐⇒ t = 52 ± 10 2,
p √
donde x = 11 ± 52 ± 10 2 são as outras soluções.

11. Observe que os fatores do lado esquerdo tem um padrão razoável então talvez consigamos encontrar
um bom padrão ao expandirmos este lado de forma esperta. Defina v = (4z+1)(6z+1) = 24z 2 +10z+1.
Por outro lado,
3 1
(3z + 1)(12z + 1) = 36z 2 + 15z + 1 = (24z 2 + 10z + 1) − ,
2 2
e assim nossa equação vira v(3v − 1) = 4 ⇐⇒ 3v 2 − v −√ 4 = 0 ⇐⇒ v = −1 ou v = 4/3.
5 ± i 23
Se v = −1 temos 24z 2 + 10z + 1 = −1 =⇒ z = .
24

−5 ± 33
Se v = 4/3 temos 72z 2 + 30z + 3 = 4 =⇒ z = .
24

12. A ideia é fazer uma substituição trigonométrica. Observe que quando a2 + b2 = 1 podemos
supor que a = cos θ e b = sen θ para algum ângulo θ ∈ [0, 2π]. Dessa forma, fazemos x1 = cos θ e
1

4− 3x
= sen θ, e isolando x obtemos:

1 4sen θ − 1 √
= √ ⇐⇒ 3sen θ + cos θ = 4sen θ cos θ,
cos θ 3sen θ
Dividindo por 2 e usando fórmulas de trigonometria podemos simplificar esta equação para:

3 1 π 
sen θ + cos θ = 2sen θ cos θ ⇐⇒ sen + θ = sen 2θ.
2 2 6
π 5π 7π 19π
Segue que θ = , , ou (+2kπ, k ∈ Z) e para cada uma delas x = cos1 θ é solução.
6 18 18 18

7
13. Para resolver este exercı́cio convém introduzir uma notação muito útil no cálculo de expressões.
Sejam x, y, z três variáveis quaisquer. Defina

σ1 = x + y + z, σ2 = xy + yz + zx, σ3 = xyz.

Costuma ser útil também considerar Sn = xn + y n + z n para n natural. Assim,

(x + y + z)2 = x2 + y 2 + z 2 + 2(xy + yz + zx) =⇒ S2 = σ12 − 2σ2 .

Podemos usar esta expressão para calcular S3 . Com efeito,

σ1 S2 = (x + y + z)(x2 + y 2 + z 2 ) = x3 + y 3 + z 3 + x2 y + xy 2 + y 2 z + yz 2 + z 2 x + zx2 ,
mas x2 y + xy 2 + y 2 z + yz 2 + z 2 x + zx2 = xy(x + y) + yz(y + z) + zx(z + x)
= xy(σ1 − z) + yz(σ1 − x) + zx(σ1 − y)
= (xy + yz + zx)σ1 − 3(xyz) = σ1 σ2 − 3σ3

e usando a expressão para S2 obtemos

S3 = σ13 − 3σ1 σ2 + 3σ3 .

Mas por que fizemos tudo isso? Voltando ao problema, nossa hipótese corresponde a σ1 = 0 e a
S3
expressão em seu enunciado, após expansão, é igual a . Usando nossa nova fórmula ficamos com
xyz

σ13 − 3σ1 σ2 + 3σ3


 
2 2 2 1 1 1
(x y z ) + + = .
x3 y 3 x3 z 3 y 3 z 3 σ3

Como σ1 = 0, segue que a expressão desejada é igual a 3.

14. Este é um bom exercı́cio para introduzir somatórios cı́clicos. Se f (x, y, z) é uma expressão em x,
y e z denotamos X
f (x, y, z) := f (x, y, z) + f (y, z, x) + f (z, x, y)
cyc
Xx x y z
Por exemplo, =+ + . As manipulações algébricas deste exercı́cio podem ser feitas todas
cyc
y y z x
diretamente, mas o uso de expressões como acima pode ser muito útil para escrever equações de forma
organizada, facilitando sua compreensão2 .
Antes de começar, seguindo a notação da solução anterior denote

σ1 = a + b + c, σ2 = ab + bc + ca, σ3 = abc.

Expandindo o denominador (essa expressão é um produtório cı́clico!), obtemos:


X
(a + b)(b + c)(c + a) =a2 b + ab2 + b2 c + bc2 + c2 a + a2 c + 2abc = (a2 b + ab2 ) + 2abc
cyc

=σ1 σ2 − 3σ3 + 2σ3 = σ1 σ2 − σ3 .

Veja a resolução do exercı́cio anterior para a dedução da fórmula utilizada acima.


2
de forma similar poderı́amos definir somatórios simétricos considerandoXtodas as permutações de x, y e z, não apenas
x x y z y z x
aquelas que preservam a ordem cı́clica das letras. Dessa forma terı́amos = + + + + +
sym
y y z x x y z

8
Por outro lado a expansão do numerador (outro produtório cı́clico!) nos dá:
X
(a − b)(b − c)(c − a) = abc − ac2 − a2 b + a2 c − b2 c + bc2 + ab2 − abc = (−a2 b + ab2 )
cyc
X X X
2 2 2
= (a b + ab ) − 2 (a b) = σ1 σ2 − 3σ3 − 2 (a2 b).
cyc cyc cyc
X
(a2 b) + σ3
cyc 1 5
Escrevendo K = , obtemos 1 − 2K = 11 =⇒ K = 11 . Por outro lado, a expressão que
σ1 σ2 − σ3
devemos calcular se escreve como
X a X a(b + c)(c + a) X abc + a2 b + ac2 + a2 c
= =
cyc
a+b cyc
(a + b)(b + c)(c + a) cyc
(a + b)(b + c)(c + a)
!
X X X X
abc + (a2 b) + (ac2 + a2 c) 3σ3 + 2
a b + σ1 σ2 − 3σ3
cyc cyc cyc cyc
= = ,
σ1 σ2 − σ3 σ1 σ2 − σ3
X X
pois (ac2 + a2 c) = (a2 b + ab2 ). Mas esta última expressão é igual a 1 + K = 16
11 , resolvendo o
cyc cyc
problema. Confira também a solução de Marcelo Matheus Gauy no site da OBM ou na Eureka nº24,
página 44.

15.
x2 + y 2 + z 2 + xy + yz + xz + 3/2 = 2(x + y + z) ⇐⇒
1 2 1 1
(x + y 2 + 1 + 2xy − 2x − 2y) + (z 2 + y 2 + 1 + 2yz − 2y − 2z) + (x2 + z 2 + 1 + 2zx − 2z − 2x) = 0 ⇐⇒
2 2 2
1 1 1
(x + y − 1)2 + (y + z − 1)2 + (z + x − 1)2 = 0.
2 2 2
Como a2 ≥ 0 com igualdade apenas quando a = 0, obtemos x + y = y + z = z + x = 1 o que implica
em x = y = z = 1/2.

16. Sem perda de generalidade podemos supor x ≤ y ≤ z < w. Suponha que z ≥ 3. Nesse caso,
w! ≥ (z + 1)! = (z + 1)z! > 3z! ≥ x! + y! + z!, uma contradição. Logo x ≤ y ≤ z ≤ 2. Analisando
todas as possibilidades obtemos que x = y = z = 2 e w = 3 é única solução.

17. Uma observação crucial é que existe apenas um primo par, o número 2. Assim, analisando a
paridade da primeira equação temos a = 2 ou c = 2 (note que d = a + c ≥ 3 tem que ser primo ı́mpar).
Por outro lado, note que se c = 2 o lado esquerdo de (iii) é par e o direito ı́mpar. Assim a = 2. Segue
que d = c + 2 e substituindo temos o sistema:
( (
2(b + 2c + 4) = c(c + 2 − b) 2b + 2c + 8 = c2 − bc
⇐⇒
1 + bc + c + 2 = b(c + 2) c + 3 = 2b

e substituindo o valor de b novamente obtemos 6c + 22 = 2c2 − c2 − 3c ⇐⇒ c2 − 9c − 22 = 0 que


tem como raı́zes c = 11 e c = −2. Segue então que a = 2, b = 7, c = 11, d = 13 é a única solução e
n = 2002, o que era previsı́vel, pois a prova foi em 2002.

9
18. A ideia aqui é que se x2 + k é um quadrado perfeito com x e k inteiros positivos então x2 + k ≥
(x+1)2 , pois é o quadrado de um inteiro e é maior que x2 . Mas isto implica em k ≥ 2x+1. Aplicando
a nosso problema, suponha que fosse possı́vel. Segue que:

b + c ≥ 2a + 1, c + a ≥ 2b + 1, c + a ≥ 2c + 1

e somando todas as desigualdades terı́amos 2(a + b + c) ≥ 2(a + b + c) + 3 =⇒ 0 ≥ 3, uma contradição.

19. Os números x4 e y 4 são não-negativos e somam 1, logo 0 ≤ x4 , y 4 ≤ 1, o que implica em


0 ≤ |x|, |y| ≤ 1. Note porém que se x < 0 então x3 < 0 e a primeira equação nos dá y 3 = 1 − x3 =⇒
y 3 > 1 e daı́ y > 1, uma contradição. Segue que x ≥ 0 e y ≥ 0 analogamente. Por outro lado, se
a ∈ (0, 1) é um número real temos que a4 < a3 . Assim, se x 6= 0, 1 (o que implica y 6= 0, 1) terı́amos
1 = x4 + y 4 < x3 + y 3 = 1, outra contradição. Segue então que (0, 1) e (1, 0) são as únicas soluções do
sistema.

20. A primeira observação é que x e y são não-negativos, pois caso contrário o lado esquerdo não seria
inteiro. Além disso, se x = 0 ou y = 0 o lado esquerdo seria negativo e o direito igual a 2015. Logo
todas as soluções são dadas por pares de inteiros positivos.
O importante agora é observar que se a é inteiro positivo, então xa é muito menor que ax quando
x é suficientemente grande e restringir os casos. Um argumento de indução demonstra que se x ≥ 4
então x2 ≤ 2x e se y ≥ 5 então y 5 ≤ 5y . Como o lado direito é maior que 2015, é impossı́vel termos
x ≥ 4 e y ≥ 5 simultaneamente, pois isso faria o lado esquerdo ser negativo. Analisando a paridade
obtemos também que x e y tem que ser ı́mpares. Basta analisar 4 casos.
(i) x = 1 =⇒ y 5 − 2 · 5y = 2015 + 4y, mas nesse caso o lado esquerdo é sempre menor que 2015.
(ii) x = 3 =⇒ 9y 5 − 8 · 5y = 2015 + 12y. Nesse caso, se y ≥ 6 o lado esquerdo é negativo (pode
ser provado por indução). Testando os outros dois casos pequenos, verificamos que não há soluções.
(iii) y = 1 =⇒ x2 − 5 · 2x = 2015 + 4x, mas nesse caso o lado esquerdo é sempre menor que 2015.
(iv) y = 3 =⇒ 243x2 − 125 · 2x = 2015 + 12x. Nesse caso, devemos ter x2 > 2015/243 > 9.
Verificamos que x = 5 é solução. Se x ≥ 7 então 2x > 2x2 (prove por indução), e o lado esquerdo é
novamente negativo. Logo (5, 3) é a única solução.

21. Escreva x = n + a, com n = bxc ∈ Z e a ∈ [0, 1). Como estamos interessados em soluções não
inteiras, podemos supor que a 6= 0. Nossa equação fica:

13 13 13 13 13 − n2
n+a+ =n+ ⇐⇒ a = − ⇐⇒ n2 + an = 13 ⇐⇒ a = .
n+a n n n+a n
13−n2
Queremos encontrar os valores de n ∈ Z tais que 0 < a = n < 1. Contudo, analisando separada-
mente n positivo ou negativo, temos:

13 − n2 √ √
0< =⇒ 0 < n < 13 ou n < − 13
n √ √
13 − n2 −1 + 53 −1 − 53
< 1 =⇒ < n ou < n < 0.
n 2 2

−1+ 53

Como 3 < 2 < 13 < 4, não há solução com n > 0. Porém,

−1 − 53 √
−5 < < −4 < − 13 < −3
2
−13
indica que há exatamente uma solução, exatamente quando n = −4, o que implica a = 3/4 e x = 4 .

10
22. O lado direito é sempre não-negativo, logo xi ≥ 0. Seja então xa o menor dos xi (escolha qualquer
um se houver empate). Segue que xa ≤ x1 + x2 + · · · + xn − xa ≤ (x1 + x2 + · · · + xn − xa )2018 , com
igualdade se e só se xa = x1 + x2 + · · · + xn − xa = 1 ou 0.
No primeiro caso temos xa = x1 + x2 + · · · + x (n − xa ≥ (n − 1)xa implicando que xa = 0 (uma
x1 = x2018
2
contradição) ou n = 2. Segue que nosso sistema é e x2 = x1 = 1.
x2 = x2018
1
Se xa = 0, como x1 + x2 + · · · + xn − xa = 0 e xi ≥ 0, temos x1 = x2 = . . . xn = 0, e esta é
claramente uma solução. As soluções são (1, 1) para n = 2 e xi = 0 para i = 1, . . . , n, n ≥ 2.

√ p p
23. Expandindo obtemos x 1 + x2 +y 1 + y 2 = (1−xy)− (1 + x2 )(1 + y 2 ) e elevando ao quadrado
p
x2 + x4 + 2xy (1 + x2 )(1 + y 2 ) + y 2 + y 4 =
p p
1 − 2xy + x2 y 2 − 2 (1 + x2 )(1 + y 2 ) + 2xy (1 + x2 )(1 + y 2 ) + (1 + x2 )(1 + y 2 ) ⇐⇒
p
x4 + y 4 = 2 − 2xy − 2 (1 + x2 )(1 + y 2 ) + 2x2 y 2 ⇐⇒
p
(x2 − y 2 )2 = 2(1 − xy − (1 + x2 )(1 + y 2 )).

O lado esquerdo é ≥ 0. Iremos usar puma ideia de desigualdade, vamos provar que o lado direito é
menor ou igual a 0, ou seja, 1−xy ≤ (1 + x2 )(1 + y 2 ). Como a raiz quadrada é sempre não negativa,
basta verificar que (1 − xy)2 ≤ (1 + x2 )(1 + y 2 ). Contudo, apenas uma rápida manipulação algébrica
esta última equivale a 0 ≤ x2 + 2xy + y 2 = (x + y)2 , o que é verdade. Segue então que ambos os lados
da equação são zero, donde x2 = y 2 . Se x = −y o problema acaba. Caso contrário x = y e resolvendo
a equação obtida substituindo esta relação no enunciado, obtemos x = y = 0.

p √ p √
24. Seja α = 3 13 + x e β = 3 13 − x. A ideia é observar que, se x for grande, α e −β estão
próximos, logo α + β ≈ 0. Daı́ α + β, como função de x, é limitada.

Primeiramente, como x ≥ 0 por definição, temos |α3 | ≥ |β 3 | e daı́ segue que |α| ≥ |β|. Como
α > 0, temos α + β > 0 (a igualdade entre os módulos só poderia ocorrer quando x = 0, mas nesse
caso α = β > 0). Por outro lado,

(α + β)3 = α3 + β 3 + 3αβ(α + β) = 26 + 3( 3 169 − x)(α + β).

Segue que
√ √ (α + β)3 − 26 26
= (α + β)2 −
3
3 169 ≥ 3( 3 169 − x) =
α+β α+β
Observe que o lado√ direito é uma função crescente de α + β(!). Juntando este fato ao fato que
26
25 − 25 > 21 > 3 3 169, obtemos que α + β < 5 (independentemente de x!). Como queremos encontrar
os valores de x para os quais α + β é inteiro, basta analisar os casos α + β = 1, 2, 3, 4 e isolar x na
equação acima, obtendo
20188 123200 29645
x= , 196, ,
27 720 216
como soluções.

25. O lado esquerdo é real e positivo. Por simetria podemos supor que a ≥ b e a ≥ c, logo ao menos
dois dos termos no produto do lado direito são positivos: a + b − c e a + c − b. Segue que o terceiro
também tem que ser. Podemos assim escrever

x = (b + c − a), y = (c + a − b), z = (a + b − c)

11
com x, y e z positivos. Observe que
y+z x+z x+y
a= , b= , c=
2 2 2
e nossa equação se torna

(x + y)(y + z)(z + x) = 8xyz ⇐⇒ x2 y + xy 2 + y 2 z + yz 2 + z 2 x + zx2 = 6xyx.

Mas a desigualdade das médias aplicada nos seis termos3 (todos eles são positivos!) implica que o
Lado Esquerdo é maior ou igual ao Lado Direito com igualdade se e somente se todos os termos são
iguais, o que implica x = y = z. Por outro lado, isto implica que a = b = c e é fácil verificar que estas
são as soluções da equação.

26. A primeira equação equivale a

y 4 − 2xy 2 + 7y 2 + x2 − 7x − 8 = 0 ⇐⇒ (y 2 − x)2 + 8y 2 − 8x − 8 − (y 2 − x) = 0 ⇐⇒
(y 2 − x)(y 2 − x − 1) + 8y 2 − 8x − 8 = 0 ⇐⇒ (y 2 − x + 8)(y 2 − x − 1) = 0

donde x = y 2 + 8 ou x = y 2 − 1. Entretanto, a segunda equação só está definida quando x ≤ 3, logo


o primeiro caso não acontece. Substituindo y 2 = x + 1 na segunda ficamos com
√ √ √ √
3 − x + x + 2 = x3 + x2 − 4x − 1 ⇐⇒ 3 − x + x + 2 − 3 = x3 + x2 − 4x − 4 ⇐⇒
√ √
3 − x + x + 2 − 3 = (x + 1)(x2 − 4) = (x + 2)(x + 1)(x − 2)

O Lado Esquerdo só faz√sentido √ se −2 ≤ x ≤ 3. Além √ disso, elevando ao quadrado e fazendo


manipulações algébricas 3 − x + x + 2 − 3 ≤ 0 ⇐⇒ 6 + x − x2 ≤ 2 ⇐⇒ x2 − x − 2 ≥ 0 ⇐⇒
x ≥ 2 ou x ≤ −1. Por outro lado, analisando o produto do lado direito termo a termo temos:

(x + 2)(x + 1)(x − 2) ≤ 0 ⇐⇒ x ≤ −2 ou − 1 ≤ x ≤ 2,

que é quase o complementar da região anterior (só estamos interessados em −2 ≤ x ≤ 3). Segue √ que
basta√testar se os valores x = −2, −1, 2 são soluções. Por verificação, obtemos que (−1, 0), (2, − 3)
e (2, 3) são as soluções do sistema.

Material elaborado por Hugo Fonseca Araújo

3
poderia ser aplicada também em cada um dos pares x2 y + xy 2

12
Problemas Resolvidos

Nı́vel 2

Sequências

Material elaborado por Hugo Fonseca Araújo


Problemas

Problema 1. (OBM 2007) Em 1949 o matemático indiano D. R. Kaprekar inventou um processo


conhecido como Operação de Kaprekar. Primeiramente escolha um número de quatro dı́gitos (não
todos iguais), em seguida escreva a diferença entre o maior e o menor número que podem ser formados
a partir de uma permutação dos dı́gitos do número inicial. Repetindo o processo com cada número
assim obtido, obtemos uma sequência.
Por exemplo, se o primeiro número for 2007, o segundo será 7200 − 0027 = 7173. O terceiro será
7731 − 1377 = 6354.
Começando com o número 1998, qual será o 2007-ésimo termo da sequência?

Problema 2. Seja a0 = 0. Para n ≥ 0, defina an+1 recursivamente por an+1 = 2an + 2n . Encontre
uma fórmula fechada para an .

Problema 3. Seja a0 = 2, a1 = 5, e a2 = 8, e para n > 2 defina an recursivamente como o resto da


divisão de 4(an−1 + an−2 + an−3 ) por 11. Calcule a2018 · a2020 · a2022 .

Problema 4. Calcule:
1 1 1 1 1 1 1 1 1 1
1+ 32
+ 52
+ 72
+ ··· + 9972
+ 999 2 − 10022 − 10042 − 10062 − ··· − 19982
− 20002
.
1 + 212 + 312 + 412 + · · · + 999
1 1
2 + 10002

Problema 5. (Albânia 2011) A sequência (an )n∈N é definida por a1 = 1 e an = n(a1 + a2 + · · · + an−1 )
∀n > 1. Prove que:

a) Se n é par, n! divide an .

b) Se n é ı́mpar e maior que 1, n! não divide an .

p
2n + 1 + n(n + 1)
Problema 6. (Helênica Júnior 2005 adaptado) Seja f (n) = √ √ para cada inteiro
n+1+ n
positivo n. Calcule:

a) f (1) e f (1) + f (2).

b) a soma A = f (1) + f (2) + · · · + f (400).

Problema 7. (OBM 2013)


X Y
Na tabela ao lado, a partir da segunda linha, o número escrito na coluna X é
1ª 2 1
igual ao produto dos números da linha anterior e o número escrito na coluna
2ª 2 2
Y é igual ao quociente da divisão do número escrito na coluna X da linha
3ª 4 1
anterior pelo número escrito na coluna Y da linha anterior.
... ... ...

a) Qual o número que aparece na décima linha?

2
b) Qual a soma dos números que aparecem na linha 21?

Problema 8. Sejam a e b inteiros positivos. Arnaldo escreve sucessivamente números reais em um


quadro negro x1 , x2 , x3 , . . . usando a seguinte regra
1
x1 = a, x2 = b e xn+2 = xn − .
xn+1
Ele para de escrever quando escreve o número zero. Quantos números xk , não necessariamente dis-
tintos, Arnaldo escreveu?

Problema 9. Um grupo de n ≥ 3 amigos está sentado em uma mesa circular em um restaurante.


Sabemos que a quantidade de dinheiro que cada um deles possui é igual a média aritmética das
quantidades de dinheiro que seus dois vizinhos possuem. Mostre que todos tem a mesma quantidade
de dinheiro.

Problema
  10. (Grécia 2013) Seja (an )n∈N uma sequência de números reais com a1 = 2 e
n+1
an = n−1 (a1 + a2 + ... + an−1 ) ∀n ≥ 2. Encontre o termo a2013 .

Problema 11. Mostre que não existe uma sequência infinita de números reais x1 , x2 , . . . tais que,
para todo n ≥ 1, r
1
xn+2 = xn+1 − xn .
2

Problema 12. Calcule:


2020
X k+2 3 4 2022
, ou seja, + + ··· + .
k · (k + 1) · 2k 1 · 2 · 2 2 · 3 · 22 2020 · 2021 · 22020
k=1

Problema 13. (Croácia 2005 adaptado) A sequência (an )n∈N é definida por an = a1 a2 ...an−1 + 1
para n ≥ 2 e a1 = 1. Mostre que, para todo m inteiro positivo,
m
X 1
< 2.
an
n=1

Problema 14. Calcule


2020
Y 
1
1 + 2n .
3
n=1

Problema 15. (OMERJ 2018) Seja (an )n∈N uma sequência de números inteiros com a1 = 1 e, para
todo n ≥ 1, (
a2n = an + 1
a2n+1 = 10an .
Quantas vezes o número 111 aparece nesta sequência?

3

Problema 16. (Japão 1994) Para cada inteiro positivo n, seja an o inteiro mais próximo de n, e
seja bn = n + an . Determine a sequência crescente (cn ) de inteiros positivos que não aparecem na
sequência (bn )n∈N .

Problema 17. (Rússia 2000) Arnaldo escreve em um quadro-negro a sequência a1 = 1, a2 , a3 , . . .


respeitando a seguinte regra: se an −2 é um inteiro positivo que ainda não apareceu como a1 , a2 , a3 , . . . ,
ou an , então ele escreve an+1 = an −2; caso contrário, escreve an+1 = an +3. Prove que todo quadrado
perfeito não-nulo aparece na sequência como seu antecessor somado de 3.

Problema 18. Calcule:


2020
X i 1 2 3 2020
, ou seja, + 2 + 3 + · · · + 2020 .
2i 2 2 2 2
i=1

Problema 19. Seja a1 , a2 , . . . , an uma sequência. Calcule


a1 a2 an
+ + ··· + .
1 + a1 (1 + a1 )(1 + a2 ) (1 + a1 )(1 + a2 ) . . . (1 + an )

Problema 20. Seja (an )n∈N uma sequência definida recursivamente por a1 = 1 e an = n(an−1 + 1)
para n ≥ 2. Defina     
1 1 1
Pn = 1 + 1+ ··· 1 + .
a1 a2 an
Pn
Encontre .
an+1

Problema 21. (São Petersburgo 2009) Seja (xn )n∈N uma sequência infinita de números reais tais que

xn+2 = |xn+1 | − xn .

Prove que ela é periódica.

Problema 22. Calcule:


     
1 1 1 1
1− 2 1− 2 1 − 2 ··· 1 − .
2 3 4 20202

Problema 23. Calcule a soma


n
X k
.
4k 4 +1
k=1

Problema 24. (Irã 2013 adaptado) Seja {an }∞


n=1 uma sequência de inteiros positivos tal que
   
2an 2an+1
an+2 = + .
an+1 an
Prove que existe um inteiro positivo m tal que am = 4 ou am = 3.
OBS: bxc denota o maior inteiro menor ou igual a x.

4
Problema 25. (Centro-americana 2016) Dizemos que um número racional é irie quando ele pode ser
1
escrito na forma 1 + para algum inteiro positivo k. Prove que todo inteiro n ≥ 2 pode ser escrito
k
como o produto de r números irie distintos para cada inteiro r ≥ n − 1.

Problema 26. (Norte da China 2019-parcial) Para cada inteiro positivo n, defina f (n) como o menor
inteiro positivo que não divide n. Considere a sequência (an )n∈N definida por
a1 = a2 = 1 e an = af (n) + 1 ∀n ≥ 3.
Por exemplo, a3 = a2 + 1 = 2, a4 = a3 + 1 = 3.
Prove que existe um inteiro positivo C tal que am ≤ C para todo inteiro positivo m.

Problema 27. (Itália 2011) Uma sequência de inteiros positivos a1 , a2 , . . . , an é chamada de escada
de comprimento n quando ela consiste de n inteiros consecutivos em ordem crescente.
a) Prove que para cada inteiro positivo n existem duas escadas de comprimento n sem elementos
em comum, a1 , a2 , . . . , an e b1 , b2 , . . . , bn , tais que, para todo i = 1, . . . , n, o maior divisor
comum entre ai e bi é igual a 1.
b) Prove que para cada inteiro positivo n existem duas escadas de comprimento n sem elementos
em comum, a1 , a2 , . . . , an e b1 , b2 , . . . , bn , tais que, para todo i = 1, . . . , n, o maior divisor
comum entre ai e bi é maior que 1.

Problema 28. Considere os números


1 3 5 595 597
A= · · · ··· ·
4 6 8 598 600
2 4 6 596 598
B = · · · ··· · .
5 7 9 599 601
Prove que:
a) A < B,
1
b) A < .
5990

Problema 29. (Norte da China 2019) Seja n um número inteiro positivo dado e a1 , a2 , . . . , an ,
an+1 , an+2 números reais tais que an+1 = a1 e an+2 = a2 . Para cada i = 1, 2, . . . , n vale que
a2i + ai
ai 6= −1 e ai+2 = .
ai+1 + 1
Prove que a1 = a2 = · · · = an .

Problema 30. (Rússia 2008) As sequências (an )n∈N e (bn )n∈N são definidas por a1 = 1, b1 = 2 e
1 + an + an bn 1 + bn + an bn
an+1 = , bn+1 = .
bn an
Mostre que a2008 < 5.

Problema 31. Os números da sequência a0 , a1 , a2 , . . . são todos reais positivos e tais que, para todo
n ≥ 0,
n
X 1
= an+1 .
a2 + ak ak+1 + a2k+1
k=0 k
Se a8 = 2, qual o valor de a16 ?

5
Soluções
1. A sequência é 1998 → 9981 − 1899 = 8082 → 8820 − 0288 = 8532 → 8532 − 2358 = 6174 →
7641 − 1467 = 6174. Note que, depois de 6174, todos os termos serão iguais a 6174, pois este é um
ponto fixo da operação. Logo a resposta é 6174. Na verdade, começando com qualquer número de
4 dı́gitos, obtemos este número, 6174, após executarmos um número finito de vezes a operação de
Kaprekar.

an+1 an an+1 an
2. Segue que = n−1 + 1 =⇒ − n−1 = 1. Somando todas as relações vem que
2n 2 2n 2
   
an+1 an an an−1 a
1 a0 
− + − + · · · + − = 1 + 1 + · · · + 1 = n + 1,
2n 2n−1 2n−1 2n−2 20 2−1
an+1 a0 an+1
donde n
− −1 = n = n + 1. Segue que an+1 = (n + 1) · 2n e an = n · 2n−1 .
2 2 2

3. Calculando os primeiros termos da sequência obtemos a3 = 5, a4 = 6, a5 = 10, a6 = 7, a7 = 4,


a8 = 7, a9 = 6, a10 = 2, a11 = 5 e a12 = 8, obtendo assim uma repetição dos três primeiros termos.
Segue que a sequência tem um perı́odo 10, ou seja, an = an+10 para todo n natural. Daı́ a2020 = a0 = 2,
a2018 = a8 = 7 e a2022 = a2 = 8 e o produto é igual a 112.

4. Note que
1 1 1 1 1 1 1 1 1 1
1+
2
+ 2 + 2 + ··· + 2
+ 2
− 2
− 2
− 2
− ··· − 2
− 2
=
 3 5 7 997   999 1002 1004 1006  1998 2000
1 1 1 1 1 1 1 1 1 1 1 1
1 + 2 + ··· + + + 2 + ··· + + − + 2 + ··· + + − ··· − =
3 9992 22 4 9982 10002 22 4 9982 10002 10022 20002
 
1 1 1 1 1 1 1 1 1
1 + 2 + 2 + ··· + + − 2 1 + 2 + 2 + ··· + + =
2 3 9992 10002 2 2 3 9992 10002
 
3 1 1 1 1
1 + 2 + 2 + ··· + + .
4 2 3 9992 10002

3
Logo a resposta é .
4

5. Escrevemos sn = a1 + a2 + · · · an . Segue que

sn − sn−1 = nsn−1 =⇒ sn = (n + 1)sn−1 .

Como s1 = 1, obtemos

sn sn−1 s2 (n + 1)!
sn = ··· = (n + 1) · n · · · · · 3 = .
sn−1 sn−2 s1 2

(n + 1)! − n! (n + 1) − 1 n
Logo, para n > 1, an = sn − sn−1 = = n! = n! .
2 2 2
Segue que n! | an ⇐⇒ n é par ou 1. Isto conclui os dois itens.


3+√2
√ √ √ √ √ √ √
6. a) f (1) = 1+ 2
= 2 2 − 1; f (1) + f (2) = 2 2 − 1 + √5+ √6 = 2 2 − 1 + 3 3 − 2 2 = 3 3 − 1.
2+ 3

6
b) Note que
p √ √
2n + 1 + n(n + 1) n+1− n
f (n) = √ √ · √ √ =
n+1+ n n+1− n
√ √ √ √ √ √
(2n + 1)( n + 1 − n) − n n + 1 − (n + 1) n = (n + 1) n + 1 − n n.

A soma de todos os f (n) é telescópica e obtemos A = 401 401 − 1.

7. Seja an o número na coluna X e linha n e bn o número na coluna Y e linha n. Temos a1 = 2 e


b1 = 2. Além disso, an+1 = an bn e bn+1 = abnn . Segue que

an an+1 an bn
an+2 = an+1 bn+1 = an bn = a2n e bn+2 = = an = b2n .
bn bn+1 bn

k k 4
Obtemos assim que an+2k = (an )2 e bn+2k = (bn )2 . Logo, na décima linha, o número é (a2 )2 =
4 10 10 10
(b2 )2 = 216 = 65536. Por outro lado, a21 = a1+2·10 = a21 = 22 e b21 = b1+2·10 = b21 = 1, logo a
10
soma da letra b) é 22 + 1.

8. Para n ≥ 2 tal que xn 6= 0, vale que, pela regra de definição, xn+1 xn = xn xn−1 − 1. Considerando
então, para todo n ≥ 1, yn = xn xn+1 , descobrimos que yn+1 = yn − 1. Assim, a sequência yn é

y1 = x1 x2 = ab, y2 = ab − 1, . . . , yab+1 = 0,

caindo de 1 em 1 até se anular. Como yab+1 = xab+1 xab+2 = 0 mas yn = xn+1 xn 6= 0 para todo n ≤ ab,
temos xn 6= 0 para todo n ≤ ab + 1 e então xab+2 = 0. Ou seja, Arnaldo escreve ab + 2 números no
quadro.

9. Sejam p1 , p2 , . . . , pn os amigos sentados nesta ordem e a1 , a2 , . . . , an a quantidade de dinheiro


que cada um deles possui. Seja a1 = a e a2 = b. Como a2 é a média de a1 e a3 , segue que
a3 = 2a2 − a1 = 2b − a. Analogamente, a4 = 2a3 − a2 = 2(2b − a) − b = 3b − 2a.
Afirmamos que, para 3 ≤ k ≤ n, an = (n − 1)b − (n − 2)a. A prova é por indução forte.
O caso base k = 3 e k = 4 já foi feito. Por outro lado, ak−1 é a média de ak−2 e ak . Logo
ak = 2ak−1 − ak−2 = 2[(k − 2)b − (k − 3)a] − [(k − 3)b − (k − 4)a] = (k − 1)b − (k − 2)a, o que conclui
a prova dessa afirmação.
Contudo, observe que a1 é a média entre an e a2 . Dessa forma,

an + a2 = nb − (n − 2)a = 2a1 = 2a ⇐⇒ nb = na ⇐⇒ a = b.

Pela fórmula para an que obtivemos anteriormente, segue que todos são iguais.

n+1 sn 2n
10. Escrevemos sn = a1 + a2 + · · · + an . Segue que sn − sn−1 = n−1 sn−1 ⇐⇒ sn−1 = n−1 . Como
s1 = 2, obtemos

sn sn sn−1 s2 2n 2(X
nX−X
1) 2(X
nX−X
2) 2 · 2A
= · · ··· · =X · X · X · ··· · = 2n−1 n.
2 sn−1 sn−2 s1 nX−X1 nX−X2 nX−X3 1

Segue que sn = 2n n, donde an = sn − sn−1 = 2n n − 2n−1 (n − 1) = 2n−1 (2n − n + 1) = (n + 1)2n−1 e


a2013 = 2014 · 22012 .

7
11. Para que a raiz quadrada faça sentido, é necessário que 21 xn+1 > xn , ou seja, xn+1 > 2xn > 0
para todo n ≥ 1. Segue por indução que xn+1 > 2n x1 para todo n ≥ 1.
Por outro lado, x2n+2 = 21 xn+1 − xn =⇒ xn+1 > 2x2n+2 para todo n ≥ 1. Trocando o n + 1 por n
nesta relação e juntando com a anterior vem que xn > 2x2n+1 > 2(4x2n ) =⇒ xn < 18 , para todo n ≥ 2.
Contudo, xn+1 > 2n x1 , e independentemente do valor de x1 , existe n tal que 2n x1 > 81 . Logo a
sequência não pode ser infinita.

12. Observe que


1 1 2(k + 1) − k k+2
k−1
− k
= k
= .
k·2 (k + 1)2 k(k + 1)2 k(k + 1)2k
1 1 1
Assim, a soma é igual a 1−1
− 2020
=1− .
1·2 (2021)2 (2021)22020

13. Se m = 1 a afirmação é claramente verdadeira. Observamos que, quando n ≥ 3,

an = a1 · a2 · · · · · an−1 + 1 = (a1 · a2 · · · · · an−2 )an−1 + 1 = (an−1 − 1)an−1 + 1.


1 1 1 1 1 1
Daı́, an − 1 = (an−1 − 1)an−1 =⇒ = − =⇒ = − e
an − 1 an−1 − 1 an−1 an−1 an−1 − 1 an − 1
então, se m ≥ 2,
m m  
X 1 1 X 1 1 1 1 1 1
= + − = + − =2− .
an a1 an − 1 an+1 − 1 1 a2 − 1 am+1 − 1 am+1 − 1
n=1 n=2

Como am > 1 para todo m ≥ 2, a afirmação também segue neste caso.

1 8
14. Multiplicando por 1 − 32
= 9 obtemos
         
1 1 1 1 1
1− 2 · 1+ 2 · 1+ · 1 + 23 · · · · · 1 + 22020 =
3 3 322 3 3
"  2 #      
1 1 1 1
1− · 1+ · 1 + 23 · · · · · 1 + 22020 =
32 322 3 3
       
1 1 1 1
1 − 22 · 1 + · 1 + 23 · · · · · 1 + 22020 =
3 322 3 3
"  # 
1 2
     
1 1 1
1− · 1+ · 1 + 24 · · · · · 1 + 22020 −
322 323 3 3
       
1 1 1 1
1 − 23 · 1 + · 1 + 24 · · · · · 1 + 22020 =
3 323 3 3
..
.
     
1 1 1
1 − 22020 · 1 + 22020 = 1 − 22021 .
3 3 3
 
9 1
Logo o produtório inicial é igual a 1 − 22021 .
8 3

8
15. Considere as duas operações no valor do ı́ndice: O1 (n) = 2n e O2 (n) = 2n + 1. Afirmamos que,
partindo de um mesmo valor n ≥ 1, duas sequências de operações resultam no mesmo ı́ndice somente
se elas forem idênticas. A prova é por indução no tamanho da menor sequência de operações.
(i) Caso base: Se uma das operações é realizada apenas uma vez, o ı́ndice resultante é 2n ou
2n + 1. Se na outra sequência executamos mais de uma operação o resultado final é ao menos 4n, que
é maior que 2n + 1, pois n ≥ 1. Logo para que valha a igualdade é necessário que a outra sequência
de operações envolva apenas uma operação e esta tem que ser idêntica a original.
(ii) Passo indutivo: Se a última operação realizada para cada uma das sequências não for a mesma,
uma das duas resulta num número ı́mpar e a outra num número par, logo o ı́ndice resultante seria dife-
rente. Segue que ambas tem que terminar com a mesma operação. Invertendo-as, obtemos sequências
com exatamente uma operação a menos resultando no mesmo ı́ndice. Pela hipótese de indução, elas
tem que ser idênticas. Isto conclui a prova.
Voltando ao problema, iremos usar estas operações para, partindo de 1, construir ı́ndices n tais que
an = 111. Note que executar a operação O1 aumentá o valor de an em 1 e a operação O2 o multiplica
por 10. Dessa maneira, podemos executar a operação O2 no máximo duas vezes, pois caso contrário
o número an obtido seria ao menos 1 · 103 = 1000. Temos então os seguintes casos:

1. O2 nunca é realizada. Segue que aO1110 (1) = 1 + 110 = 111, onde o ı́ndice indica que aplicamos a
operação O1 110 vezes.

2. O2 é aplicada uma vez. Ela não pode aparecer depois de 11 operações do tipo O1 , pois nesse
caso o número final seria ao menos (1 + 1 + · · · + 1) · 10 = 120. Podemos assim construir ı́ndices
| {z }
12 vezes
n tais que an = 111 aplicando a operação O1 i vezes, com i = 0, 1, . . . , 10, aplicar O2 e depois
aplicar O1 novamente o número de vezes que for necessário. Temos então 11 ı́ndices deste tipo.

3. Por fim, O2 é aplicada duas vezes. Neste caso, é fácil verificar que n1 = O1 (O2 (O1 (O2 (1)))) e
n2 = O1 (· · · (O1 ((O2 (O2 (1)) · · · ) são os únicos ı́ndices tais que an = 111.
| {z }
11vezes

Concluı́mos que 111 aparece 14 vezes na sequência.

16. A observação crucial aqui é que an é uma sequência que fica muito tempo parada, e quando
aumenta, aumenta no máximo em 1. Vamos encontrar esse momento em que ela salta.
Seja n um inteiro positivo e m um inteiro tal que n2 ≤ m ≤ (n + 1)2 − 1 = n2 + 2n. Neste caso
√ √ √
n ≤ m < (n + 1). Porém, am = n quando m < n + 12 e am = n + 1 quando m > n + 12 (note que
nunca pode ser igual a n + 21 !!!).

Observe que m < n + 1/2 ⇐⇒ m < n2 + n + 1/4, e, como m é inteiro, segue que am = n para
todos os valores de m entre n e n2 + n inclusive e am = n + 1 para todos os valores entre n2 + n + 1
e n2 + 2n = (n + 12 ) − 1 inclusive. Como consequência, entre n2 e (n + 1)2 − 1 inclusive, bn toma os
valores1

bn2 = n2 + n, bn2 +1 = n2 + 1 + n, bn2 +2 = n2 + 2 + n, ... , bn2 +n = (n2 + n) + n = n2 + 2n,


bn2 +n+1 = (n2 + n + 1) + n + 1 = n2 + 2n + 2, bn2 +n+2 = (n2 + n + 2) + n + 1 = n2 + 2n + 3,
... , bn2 +2n = (n2 + 2n) + n + 1 = n2 + 3n + 2,

percorrendo todos os valores entre n2 +n+1 e n2 +3n+2 inclusive, excluindo-se n2 +2n+1 = (n+1)2 .
Segue, por indução ou particionando os inteiros positivos em intervalos do tipo [n2 , (n + 1)2 − 1],
que apenas os quadrados perfeitos não aparecem na sequência, ou seja, cn = n2 .
1
o importante aqui é notar que bm+1 − bm é sempre 1, exceto quando m = n2 + n, neste caso sendo igual a 2.

9
17. Escrevemos os primeiros termos da sequência:

a1 = 1, a2 = 4, a3 = 2, a4 = 5, a5 = 3, a6 = 6, a7 = 9, . . .

Afirmamos que, para todo k natural, incluindo o zero,




 5k + 1, quando n = 5k + 1

5k + 4, quando n = 5k + 2



an = 5k + 2, quando n = 5k + 3

5k + 5, quando n = 5k + 4





5k + 3, quando n = 5k + 5.

A prova é por indução forte em k.


(i) Caso base: k = 0. Segue da lista dos primeiros termos acima.
(ii) Passo indutivo: Suponha válido para k 0 ≤ k. Temos que a5k+6 = a5k+5 + 3 = 5k + 6, pois
a5k+5 − 2 = 5k + 1 já apareceu como a5k+1 . Da mesma maneira, a5k+7 = a5k+6 + 3 = 5k + 9, pois
a5k+6 − 2 = 5k + 4 já apareceu como a5k . Em contrapartida, a5k+8 = a5k+7 − 2 = 5k + 7, pois
este número ainda não apareceu na sequência (aqui estamos usando a hipótese de indução forte!).
Analogamente, a5k+9 = a5k+8 + 3 = 5k + 10 e por fim obtemos a5k+10 = a5k+9 − 2 = 5k + 8, mostrando
que a afirmação permanece válida pra k + 1 e concluindo a prova.
Por fim, todo quadrado perfeito é do tipo 5k + 5, 5k + 1 ou 5k + 4. Estes valores correspondem a
ı́ndices do tipo 5k + 4, 5k + 1 e 5k + 2. Checando nossa fórmula, verificamos que são obtidos a partir
do seu antecessor somando 3.

2020
X i
18. Seja S = . Segue que
2i
i=1

2020 2020 2019


X i + 1 2019 2019
X 2i X i X i X 1
2S = = = = + ,
2i 2i−1 2i 2i 2i
i=1 i=1 i=0 i=0 i=0

donde
1
0 2020 1 − 22020 22021 − 2 − 2020 22021 − 2022
2S = S + − + = S + =⇒ S = .
20 22020 1 − 12 22020 22020

19. Observe que, para todo i = 1, 2, . . . , n − 1,


1 1 ai+1
− =
(1 + a1 ) · · · (1 + ai ) (1 + a1 ) · · · (1 + ai )(1 + ai+1 ) (1 + a1 ) · · · (1 + ai )(1 + ai+1 )
Logo a soma é telescópica e igual a
a1 1 1 1
+ − =1− .
1 + a1 1 + a1 (1 + a1 ) · · · (1 + an ) (1 + a1 ) · · · (1 + an )

an+1
20. Da regra, obtemos an + 1 = n+1 para todo n ≥ 1. Usando o fato que a1 = 1, temos
a1 + 1 a2 + 1 an + 1 a2 a3 an an+1 an+1 an+1
Pn = · ··· = · ··· · = = .
a1 a2 an 2a1 3a2 nan−1 (n + 1)an 2 · 3 · · · (n + 1)a1 (n + 1)!
Pn 1
Logo = .
an+1 (n + 1)!

10
21. Começamos mostrando que a sequência não pode ser, a partir de certo ponto, para sempre positiva.
Se xn , xn+1 e xn+2 são todos positivos, segue que xn+2 = xn+1 − xn e daı́ xn+3 = xn+1 − xn − xn+1 =
−xn é negativo.
Mostremos que existem sempre dois termos consecutivos que são não-positivos. Suponha que não
existem tais termos. Pela afirmação anterior existiria n tal que xn−1 é positivo, xn é não-positivo e
xn+1 é positivo. Logo xn+1 = −xn − xn−1 < −xn , donde xn+1 + xn < 0. Segue que xn+2 = xn+1 − xn é
positivo, donde xn+3 = xn+2 − xn+1 = −xn é não-negativo, xn+4 = xn+3 − xn+2 = −xn+1 é negativo,
e por fim xn+5 = −xn+4 − xn−3 = xn+1 + xn < 0 é negativo, contradição.
Sendo então xn = a e xn+1 = b dois termos não-positivos consecutivos da sequência, podemos
verificar que os próximos termos da sequencia tomam os seguintes valores:

a, b, −b − a, −2b − a, −b, a + b, −a, −2a − b, −a − b, a, b.

Conclui-se que a sequência é periódica, como querı́amos demonstrar.

22. Este exercı́cio envolve cancelamentos e produtos notáveis:


         
1 1 1 1 1
1 − 2 · 1 − 2 · 1 − 2 · ··· 1 − · 1− =
2 3 4 20192 20202
               
1 1 1 1 1 1 1 1
1− · 1+ · 1− · 1+ · ··· 1 − · 1+ · 1− · 1− =
2 2 3 3 2019 2019 2020 2020
               
2−1 2+1 3−1 3+1 2019 − 1 2019 + 1 2020 − 1 2020 + 1
· · · · ··· · · · =
2 2 3 3 2019 2019 2020 2020
                   
1 3  2  4  3 5 2018 2020 2019
 2021 2021
· · · · · · ··· · · · = .
  
2  2 3  3 4 4 2019  2019 2020 2020 4040
   

23. Começamos fatorando 4k 4 +1 = 4k 4 +4k 2 +1−4k 2 = (2k 2 +1)2 −4k 2 = (2k 2 −2k+1)(2k 2 +2k+1).
Segue que
1 1 4k 4k
− 2 = = 4 .
2k 2 − 2k + 1 2k + 2k + 1 2 2
(2k − 2k + 1)(2k + 2k + 1) 4k + 1

Dividindo por 4 obtemos


n n  
X k 1X 1 1
= − .
4k 4 + 1 4 2k 2 − 2k + 1 2k 2 + 2k + 1
k=1 k=1

Definindo ak = 2k 2 − 2k + 1, obtemos ak+1 = 2(k + 1)2 − 2(k + 1) + 1 = 2k 2 + 2k + 1, logo


n n      
X k 1X 1 1 1 1 1 1 1 n(n + 2)
= − = − = 1− 2 = .
4k 4 + 1 4 ak ak+1 4 a1 an+1 4 2n + 2n + 1 4n2 + 4n + 2
k=1 k=1

24. Começamos mostrando que, para algum n muito grande, an ≤ 4. Suponha por absurdo que exista
N tal que an > 4 para todo n ≥ N . Observe que an = an+1 =⇒ an+2 = 4. Logo podemos assumir
que an 6= an+1 para todo n ≥ N .
Como a < b =⇒ bac ≤ bbc, segue que
       
2an+1 2an 2an+1 2an 2an+1 2an an + an+1
an+2 = + ≤ + ≤ + ≤
an an+1 4 4 4 4 2

11
e, como an 6= an+1 , an+2 < max(an , an+1 ).
Da mesma forma, an+3 < max(an+1 , an+2 ), an+4 < max(an+2 , an+3 ) e an+5 < max(an+4 , an+3 ),
donde max(an+4 , an+5 ) < max(an , an+1 ). Logo, como os números ai são inteiros, a sequência bk =
max(aN +4k , aN +4k+1 ) sempre diminui ao menos 1, e em algum momento tem que ser menor ou igual
a 4, absurdo.
Por fim, pela desigualdades das médias e pelo fato que bxc + byc ≥ bx + yc − 1,
     
2an+1 2an 2an+1 2an
an+2 = + ≥ + − 1 ≥ 4 − 1 = 3.
an an+1 an an+1
Dessa forma, an ≥ 3 sempre que n ≥ 2. Juntando com o fato anterior, obtemos o resultado.

25. Observe inicialmente que, por um produto telescópico,


       
1 1 1 1
n= 1+ · 1+ · 1+ ··· 1 + ,
1 2 3 n−1
logo todo n ≥ 2 tem uma representação como produto de n − 1 números irie distintos. Por outro lado,
     
1 1 1
1+ = 1+ · 1+
m−1 2m − 2 2m − 1
para qualquer inteiro m ≥ 2. Dessa forma podemos, a partir de uma representação de n como produto
de k ≥ n − 1 números irie distintos podemos obter uma nova representação, dessa
 vez como produto
1
de k + 1 números irie distintos, trocando o menor número irie da original, 1 + m−1 , pelos dois
     
1 1 1
números 1 + 2m−2 e 1 + 2m−1 , pois estes números são menores que 1 + m−1 e logo menores
que, e distintos de, todos os números irie da representação original. O resultado segue por indução
em k.

26. Fixe algum valor de n ≥ 3. Seja f (n) = pα1 1 · pα2 2 · · · · · pαk k a fatoração em primos de f (n). Se
k > 1, algum dos números pαi i , i = 1, 2, . . . , k, não pode dividir n, pois se todos o dividissem, então
f (n) = pα1 1 · pα2 2 · · · · · pαk k também o dividiria, contrariando a definição de f (n). Em contrapartida,
para qualquer i = 1, 2, . . . , k vale que pαi i < pα1 1 · pα2 2 · · · · · pαk k = f (n), e tomando pαi i - n entrarı́amos
em contradição com a minimalidade de f (n). Logo k tem que ser igual a 1 e f (n) é sempre um número
da forma pα , onde p é primo.
Por outro lado, f (pα ) é igual a 2 se p for primo ı́mpar e igual a 3 se p for igual a 2 e α > 1. Segue
que, quando n ≥ 3,

 af (n) + 1 = a2 + 1 = 2, quando
 ( f (n) = 2, ou seja, n é ı́mpar.
an = a2 + 2 = 3, quando f (n) = pα com p primo ı́mpar.
 a f (n) + 1 = a f (f (n)) + 2 =
a3 + 2 = 4, quando f (n) = pα com p = 2 primo e α > 1.

Logo basta tomar C = 4 para obter a limitação.

27. A ideia deste exercı́cio é usar que n! é múltiplo de todos os números de 1 até n
a) Tome ai = i e bi = n! + i + 1 para i = 1, . . . , n. Note que para cada i existe k inteiro tal que
bi = k · ai + 1, logo mdc(ai , bi ) = mdc(ai , bi − k · ai ) = 1.

b) Tome ai = i + 1 e bi = (n + 1)! + i + 1 para i = 1, . . . , n. Segue da observação que (i + 1) | (n + 1)!


para todo i = 1, . . . , n.

12
28. Observe que, para x > 0,
x x+1
< ⇐⇒ x2 + 4x < x2 + 4x + 3.
x+3 x+4

Logo, a primeira fração do produto A, 14 , é menor que a primeira do produto B, 52 ; a segunda de A,


3 4 597 598
6 , é menor que a segunda de B, 7 e assim por diante até 600 < 601 . Portanto, A < B.
Consequentemente,
 2
2 1 2 3 4 597 598 1·2·3 1 1 1
A < AB = · · · · · · · · = = = < ,
4 5 6 7 600 601 599 · 600 · 601 599 · 100 · 601 5990 · 6010 5990

donde segue o resultado.

29. Multiplicando, obtemos ai+1 ai+2 + ai+2 = a2i + ai , para todo i = 1, 2, . . . , n. Somando todas as
equações, obtemos
X n Xn
(ai+1 ai+2 + ai+2 ) = (a2i + ai )
i=1 i=1

Xn Xn
Porém, usando o fato que an+1 = a1 e an+2 = a2 , a soma ai+2 é igual a ai e
i=1 i=1
n n n
!
X 1 X X
a2i = a2i+1 + a2i+2 . Dessa forma, a equação original se torna
2
i=1 i=1 i=1

n n n
1X 1X 2 1X
2ai+1 ai+2 = (ai+1 + a2i+2 ) ⇐⇒ (ai+1 − ai+2 )2 = 0.
2 2 2
i=1 i=1 i=1

Daı́ ai+1 = ai+2 para todo i = 1, 2, . . . , n e então a1 = a2 = · · · = an .

30. O importante aqui é manipular as equações de forma esperta. Somando 1 dos dois lados na
equações, ficamos com

(1 + an )(1 + bn ) (1 + bn )(1 + an )
1 + an+1 = , 1 + bn+1 =
bn an
e consequentemente,
1 1 bn an 1 1
− = − = − .
1 + an+1 1 + bn+1 (1 + an )(1 + bn ) (1 + bn )(1 + an ) 1 + an 1 + bn

Segue então que


1 1 1 1 1 1 1
− = − = ··· = − = ,
an+1 + 1 bn+1 + 1 an + 1 bn + 1 a1 + 1 b1 + 1 6

por causa das condições inciais a1 = 1 e b1 = 2. Mas como bn e an são sempre positivos (por indução),
isto implica que
1 1 1 1
= + > =⇒ a2008 + 1 < 6 =⇒ a2008 < 5.
a2008 + 1 6 b2008 + 1 6

13
31. Observe que a sequência dos an é crescente. Por outro lado,
n n
X 1 X ak+1 − ak
an+1 = = ,
a2k + ak ak+1 + a2k+1 a3k+1 − a3k
k=0 k=0

an+1 − an
donde, para todo n ≥ 1, an+1 − an = . Como a sequência é crescente, an+1 − an 6= 0, e
a3n+1 − a3n
então a3n+1 − a3n = 1 para todo n ≥ 1. Segue que

a316 − a38 = (a316 − a315 ) + (a315 − a314 ) + · · · + (a39 − a38 ) = 1 + 1 + · · · + 1 = 8.


| {z }
8 vezes

Mas isto implica que a316 = 8 + a38 = 8 + 8 = 16 =⇒ a16 = 2 2. 3

Material elaborado por Hugo Fonseca Araújo

14
Problemas Resolvidos

Nı́vel 2

Recorrências

Material elaborado por Hugo Fonseca Araújo


Problemas

Problema 1. (São Petersburgo 2019) Seja (an )n∈N uma progressão aritmética não-constante para a
qual existe um número natural n tal que

an + an+1 = a1 + a2 + · · · + a3n−1 .

Mostre que não existem termos iguais a 0 nesta sequência.

Problema 2. (Espanha 2016) Sejam (an )n∈N uma progressão aritmética e (gn )n∈N uma progressão
geométrica tais que:
a1 = g1 6= 0, a2 = g2 , a10 = g3 .
Prove que para cada inteiro positivo k existe um inteiro positivo l tal que gk = al .

Problema 3. Considere a sequência (an )n∈N dada por

a1 = 1, an = 5an−1 + 3n−1 para n ≥ 2.

Prove que p divide ap sempre que p for um primo maior que 5.

Problema 4. (Moscou 52) Considere uma progressão geométrica não-nula cuja razão q é um inteiro
diferente de 0 e −1. Mostre que a soma de dois ou mais termos desta sequência não pode ser igual a
algum de seus termos.

Problema 5. Prove que toda progressão aritmética infinita cujos termos são todos números inteiros
possui uma subsequência que é uma progressão geométrica.

Problema 6. (Pan-Africana 2019) Seja (an )∞


n=0 uma sequência de números reais definida por:

a0 = 3, a1 = 2 e a2 = 12;
2an+3 − an+2 − 8an+1 + 4an = 0.

Mostre que an é sempre um inteiro positivo.

1
Problema 7. A sequência (xn )n∈N é dada por x1 = 2 e xn+1 = x2n + xn para todo n ≥ 1. Encontre
o número inteiro mais próximo de
1 1 1
+ + ··· + .
x1 + 1 x2 + 1 x100 + 1

Problema 8. Os números reais u1 , u2 , . . . satisfazem u1 = 1 e, para n > 1,


1
un = .
u1 + · · · + un−1
Mostre que existe inteiro positivo m tal que

u1 + u2 + · · · + um > 2020.

2
Problema 9. (Coreia-Júnior 2007). A sequência a1 , a2 , . . . , a2007 satisfaz ai ∈ {2, 3} para i =
1, 2, . . . , 2007. Já a sequência de inteiros x1 , x2 , . . . , x2007 satisfaz ai xi + xi+2 ≡ 0 mod 5 para
i = 1, 2, . . . , 2005, e a2006 x2006 + x1 ≡ 0 mod 5 e a2007 x2007 + x2 ≡ 0 mod 5 (completando o ciclo).
Mostre que os inteiros xi , i = 1, 2, . . . , 2007, são todos múltiplos de 5.

Problema 10. Uma sequência (an )n∈N de números reais satisfaz, para cada n ≥ 1, a igualdade
an+3 − an+2 an+3 + an+2
= .
an − an+1 an + an+1
Sabendo que a11 = 4, a22 = 2 e a33 = 1, mostre que, para todo natural k, a soma,

ak1 + ak2 + · · · + ak100

é um quadrado perfeito.

Problema 11. (Vojtěch Jarnı́k 2019) Seja {an }∞


n=0 uma sequência tal que a0 = 1 e, para todo n ≥ 0,
p
7an + 45a2n − 36
an+1 = .
2
Mostre que:
ˆ Para todo natural n, an é um número inteiro.

ˆ Para todo natural n, an an+1 − 1 é um quadrado perfeito.

Problema 12. (Israel 2015 adaptado) Considere a sequência de Fibonacci Fn definida por F0 =
0, F1 = 1 e a relação de recorrência Fn = Fn−1 + Fn−2 para todos inteiros n ≥ 2. Seja p ≥ 3 um
número primo. Mostre que Fp−1 + Fp+1 − 1 é divisı́vel por p.

Problema 13. (URSS 82) Cada membro, a partir do terceiro, das duas sequências {an } e {bn } é
igual a soma dos dois que o antecedem. Os primeiros membros são: a1 = 1, a2 = 2, b1 = 2 e b2 = 1.
Quantos números naturais, talvez em lugares distintos, são encontrados em ambas sequências?

Problema 14. Esmeralda quer escrever todos os inteiros positivos de n dı́gitos que usam apenas os
dı́gitos 1, 2 e 3 e que não tem nenhum par de 1’s adjacentes (um do lado do outro). Quantos números
ela pode escrever?

Problema 15. (Lusófona 2011 adaptado) Considere a sequência de triângulos equiláteros Tn ladri-
lhados representada abaixo:

T1 T2 T3

3
Um sub-triângulo equilátero de um triângulo ladrilhado Tn qualquer é chamado de Delta quando
ele contém um vértice em seu topo, ou seja, esta na configuração ∆. Por exemplo, T3 possui 10
sub-triângulos do tipo Delta. Encontre o número de sub-triângulos do tipo Delta em T20 .

Problema 16. (AIME 1985) Sejam A, B, C e D os vértices de um tetraedro regular cujas arestas
medem 1 metro. Uma joaninha começa a caminhar a partir do vértice A ao longo das arestas deste
tetraedro obedecendo a seguinte regra: em cada um dos vértices ela escolhe, com igual probabilidade,
uma das arestas que se encontram nele e a percorre até encontrar o vértice que jaz em seu outro
extremo. Encontre a probabilidade da joaninha retornar a A após percorrer 7 metros.

Problema 17. Mostre que o m.d.c entre 2n + 3n e 2n+1 + 3n+1 é igual a 1 para todo n natural.

Problema 18. Dado um número real x denotamos por bxc o maior inteiro n tal que n ≤ x. Para
cada n natural, encontre o resto da divisão de

b(3 + 8)n c

por 6.

Problema 19. Prove que existe uma única função f : (0, ∞) → (0, ∞) tal que

f (f (x)) = 6x − f (x).

Problema 20. Seja (an )n∈N uma sequência tal que a1 = 4 e, para todo n natural
2 an
an+1 = + .
an 2
Determine an em função de n.

4
Soluções

1. Sendo an = a + (n − 1)d, a igualdade equivale a 2a + (2n − 1)d = (3n − 1)a + (3n−1)(3n−2)


2 d. Segue
que
9n2 − 13n + 4 2 6
−(3n − 3)a = d ⇐⇒ d = − 4 a= a.
2 3n − 3 4 − 9n
Então am+1 = 0, m ≥ 0, é equivalente a
6
a+m a = 0 ⇐⇒ 6m = 9n − 4,
4 − 9n
pois a 6= 0 (caso contrário, terı́amos d = 0 e a progressão seria constante). Mas o lado esquerdo é
múltiplo de 3 enquanto o direito não é, logo am+1 nunca é zero.

2. Como a1 = g1 , podemos escrever an = a + (n − 1)d e gn = arn−1 . Segue que a + d = ar e


a + 9d = ar2 , donde d = a(r − 1) e 9d = a(r2 − 1).
Se r = 1, gk = a = a1 para todo inteiro positivo k.
Caso contrário, obtemos 9 = r + 1, e daı́ r = 8 e d = 7a. Observe porém que 8k−1 ≡ 1k−1 ≡ 1
mod 7, logo existe l ∈ N tal que 7(l − 1) + 1 = 8k−1 . Segue que

al = a + (l − 1)d = a + 7(l − 1)a = a(7(l − 1) + 1) = a8k−1 = gk ,

como querı́amos demonstrar.

3. Escrevendo, para j = 0, . . . , n − 2,

5j an−j = 5j+1 an−j−1 + 5j 3n−j−1

e somando todos os termos, obtemos an = 3n−1 + 5 · 3n−2 + · · · + 5n−2 · 3 + 5n−1 a1 . Como a1 = 1,


do lado direito temos uma progressão geométrica cujo primeiro termo é 3n−1 e a razão é 53 . A soma
(( 5 )n − 1)3n−1 5n − 3n 5p − 3p
então é igual a 3 5 = . Para demonstrar que p divide ap = , note que pelo
3 −1
2 2
pequeno Teorema de Fermat p|5p − 3p = 2ap . Como p é primo com 2, p|ap .

4. Suponha por contradição que seja possı́vel. Sendo r a razão da P.G., existem m + 1 ≥ 3 inteiros
não-negativos 1 x0 , x1 , x2 , . . . , xm tais que

c = r x0 = r x1 + r x2 + · · · + r xm .

Se r > 1, c admite duas representações na base r, absurdo. Se r = 1, terı́amos 1 = m ≥ 2, absurdo.


Por fim, se r é negativo, considere o caso x0 ı́mpar. Separando os xi , i = 1, . . . , m, em valores pares
e ı́mpares, obterı́amos
|r|xi1 + · · · |r|xil = |r|x0 + |r|xp1 + · · · |r|xpm−l ,
onde xpj é par e xij é ı́mpar. Novamente, pela unicidade da representação em base |r|, os ı́ndices pares
não podem existir e l = 1, donde 1 = m ≥ 2, absurdo. O caso par é análogo.
1
mesmo se considerássemos uma P.G {an }n∈Z poderı́amos fazer todos os xi positivos mediante multiplicação por rN ,
com N muito grande

5
5. Seja bn = a + (n − 1)d, n ≥ 0, a progressão aritmética. Considere inicialmente a subsequência
cn = ban+1 = a + (an)d = a(1 + nd), n ≥ 0, que é uma P.A. de primeiro termo a e razão ad. A P.G.
determinada por c0 e c1 tem como termos
gn = a(1 + d)n = a (1 + d · pn (d)) = cpn (d) ,
(1+d)n −1
onde pn (d) = d é um inteiro. Segue que gn é uma subsequência da P.A. original bn .

6. Fazendo casos pequenos somos levados a conjecturar que an = 4an−2 , se n ≥ 2. Provaremos por
indução.
(i) Caso base: n = 2. Então a2 = 12 = 4 · 3 = a0 .
(ii) Passo indutivo: Se a hipótese é válida para n, da relação no enunciado temos, 2an+1 =
an + 8an−1 − 4an−2 = 8an−1 =⇒ an+1 = 4an−1 .
Segue, também por indução, que an é sempre inteiro.

1 1 1
7. Note que xk+1 = xk (xk + 1). Usaremos um truque bastante comum: a − a+1 = a(a+1) . Segue que
1 1 1 1
+ = . Assim, somando-se à expressão, obtemos:
xk+1 xk + 1 xk x101
1 1 1 1
+ + ··· + + =
x1 + 1 x2 + 1 x100 + 1 x101
1 1 1 1
+ + ··· + + =
x1 + 1 x2 + 1 x99 + 1 x100
1 1 1 1
+ + ··· + + =
x1 + 1 x2 + 1 x98 + 1 x99
..
.
1 1 1 1
+ + ··· + + , n ∈ {1, 2, . . . , 97}
x1 + 1 x2 + 1 xn + 1 xn+1
..
.
1 1 1
+ = = 2.
x1 + 1 x2 x1
1 1
Por outro lado, não é difı́cil mostrar que x101 > 2, logo x101 < 2 e o inteiro buscado é 2.

8. Suponha por absurdo que u1 + u2 + · · · + um ≤ 2020 para todo m inteiro. Segue que
1 1
um+1 = ≥
u1 + u2 + · · · + um 2020
para todo m ≥ 1 (u1 = 1). Daı́,
 
2 1
u1 + u2 + · · · + u20202 +1 ≥ (2020 + 1) > 2020,
2020
uma contradição.

9. Observe que 22 ≡ 32 ≡ −1 mod 5, logo a2i ≡ −1 mod 5 para qualquer escolha dos ai . To-
mando os ı́ndices módulo 2007 e elevando ao quadrado, temos que x2i ≡ −x2i+2 mod 5. Por indução,
x2i+2n ≡ (−1)n x2i mod 5. Fazendo n = 2007 obtemos x2i ≡ −x2i mod 5, pois i+2·2007 ≡ i mod 2007
(lembre que estamos considerando os ı́ndices módulo 2007), donde obtemos x2i ≡ 0 mod 5 e segue que
xi ≡ 0 mod 5 para todo i = 1, 2, . . . , 2007.

6
10. Da igualdade obtemos:

(an+3 − an+2 )(an + an+1 ) = (an+3 + an+2 )(an − an+1 ) ⇐⇒ an+3 an+1 = an+2 an .

Colocando n+1 no lugar de n na relação acima obtemos an+4 an+2 = an+3 an+1 = an+2 an , pela própria
relação acima. Se an+2 = 0 para algum n, segue ainda desta relação que seu sucessor ou antecessor
também é zero. Sem perda de generalidade suponha que seja an+1 . neste caso a relação
an+4 − an+3 an+4 + an+3
=
an+1 + an+2 an+1 − an+2
não faria sentido. Dessa forma an 6= 0 para todo n ≥ 3 e assim, como an an+2 6= 0 para todo n ≥ 1,
pois é constante, concluı́mos que an 6= 0 para todo n ≥ 1. Segue que an+4 = an para todo n ≥ 1, ou
seja, a sequência tem perı́odo 4, e o valor de an depende apenas do resto de n dividido por 4. Observe
que 11, 22 e 33 deixam restos distintos módulo 4. Assim, podemos calcular an para n múltiplo de 4
por
a1 a3 a33 a11 4·1
a4 = = = = 2.
a2 a22 2
A expressão buscada é então igual a

25(4k + 2 · 2k + 1k ) = [5(2k + 1)]2

e é um quadrado perfeito como querı́amos demonstrar.

11. Pela fórmula de Bhaskara, an+1 é a maior das raı́zes de x2 − 7an x + a2n − 9 = 0, logo
a2n+1 −7an an+1 +a2n +9 = 0. Substituindo n+1 no lugar de n obtemos a2n+2 −7an+1 an+2 +a2n+1 +9 = 0
e subtraindo as duas equações

(an+2 − an )(an+2 + an ) − 7an+1 (an+2 − an ) = 0 ⇐⇒ (an+2 − an )(an+2 − 7an+1 + an ) = 0.

Note porém que a sequência é crescente, logo

an+2 = 7an+1 − an .

Como a0 = 1 e a1 = 7+2 9 = 5, segue por indução que an é sempre inteiro.
Por outro lado, pela relação inicial, temos
 2
an+1 + an
a2n+1 − 7an an+1 + a2n 2
+ 9 = 0 ⇐⇒ (an+1 + an ) = 9(an+1 an − 1) ⇐⇒ an+1 an − 1 = .
3

Contudo an+1 an − 1 é inteiro e an+13+an é racional pela letra a) , consequentemente an+1 +an
3 também
tem que inteiro, completando a prova.

12. Observe que Gn = Fn−1 + Fn+1 satisfaza mesma


√ p
recorrência
 √ p que a sequência de Fibonacci, porém
1+ 5
G1 = 1 e G2 = 3. Logo Fp−1 + Fp+1 − 1 = 2 + 1−2 5 − 1. Segue que
√ !p √ !p
1+ 5 1− 5 √ √
p| + − 1 ⇐⇒ p | (1 + 5)p + (1 − 5)p − 2p .
2 2

Expandindo o Binômio de Newton, notamos que os termos irracionais, ou seja, aqueles do tipo
p
√ j √ j
± j 5 com j ı́mpar cancelam e aqueles que sobram, do tipo ± pj 5 com j par, são múltiplos de
p, exceto quando j = 0. Resta demonstrar que p | 2 − 2p , mas esta é uma consequência direta do
pequeno Teorema de Fermat.

7
13. Note que 4 = b4 < 5 = a4 < 7 = b5 . Afirmamos que bn < an < bn+1 para todo n ≥ 4. Provaremos
por indução forte. Já temos o caso base. Dividimos o passo indutivo em dois casos:
(i) n = 5: Basta verificar diretamente.
(ii) n > 5: Por hipótese, bn−2 < an−2 < bn−1 e bn−1 < an−1 < bn . Somando as duas expressões
concluı́mos que bn < an < bn+1 .
Como bn é crescente, an = bm implica que n ≤ 3 e m ≤ 3. Por verificação, apenas 1, 2 e 3 estão
em ambas sequências.

14. Seja An o conjunto de números de n dı́gitos que terminam com 1 e satisfazem o exigido e Bn o
conjunto de números de n dı́gitos que terminam com 2 ou 3 e satisfazem o exigido. Seja an = #An
e bn = #Bn , ou seja, o número de elementos desses conjuntos. Se k é um dos elementos de An+1 ,
após apagarmos seu último dı́gito 1 obtemos um elemento de Bn . Em contrapartida, adicionando um
dı́gito 1 à direita de um elemento de Bn obtemos um elemento de An+1 . Observe que cada uma dessas
operações é a inversa da outra. Logo an+1 = bn .
Por outro lado, se k é um dos elementos de An , não podemos adicionar um dı́gito 1 à sua direita.
Adicionando um dı́gito 2 ou 3 obtemos dois elementos distintos de Bn+1 . O mesmo pode ser dito se
partirmos de k ∈ Bn . Note que essas construções cobrem cada elemento de Bn+1 exatamente uma
vez. Assim, bn+1 = 2(an + bn ). Substituindo a relação anterior nesta, obtemos a recorrência:

an+2 = 2an+1 + 2an , e a1 = 1, a2 = 2 (fazendo casos pequenos).


√ √
(1 + 3)n − (1 − 3)n
Segue que an = √ e
2 3
√ √ √ √
(1 + 3)n + (1 − 3)n (1 + 3)n − (1 − 3)n
an + bn = an + an+1 = + √ .
2 3

15. Seja an o número de triângulos do tipo Delta em Tn . Observe que cada par de pontos marcados
em uma linha horizontal determina exatamente um dos triângulos do tipo Delta e em contrapartida
cada um destes triângulos determina tal par de pontos. Considerando o sub-triângulo de lado n − 1 e
que contém o vértice no topo de Tn , obtemos a seguinte recorrência
 
n+1
an = an−1 + ,
2

na qual estebinomial vem dos pares de pontos na base de Tn . Por indução, demonstramos que
n+2
an = :
3
(i) Caso base: n = 1, temos a1 = 1 = 33 , ok!


(ii) Passo indutivo: Por hipótese an−1 = n+1 n+1 n+1 n+2
   
3 , logo an = 3 + 2 = 3 pela regra de
Stifel.
Segue que a20 = 22

3 = 1540. Logo existem 1540 desses triângulos.

16. Seja an a probabilidade da Joaninha estar em A após percorrer n metros. Temos a0 = 1. Por
outro lado, ela só estará em A após n + 1 metros se estivesse em algum dos vértices B, C ou D após
percorrer n metros. A chance de estar em um desses neste momento é 1 − an . Além disso, a partir de
cada um deles, exatamente uma das três arestas levá-la de volta a A. Dessa maneira,
1 − an
an+1 = .
3

8
Escrevendo
(−3)−j − (−3)−j an−j
(−3)−j an+1−j =
3
e somando sobre todas as equações para j = 0, 1, . . . , n obtemos
1 − (−3)−(n) 1 − (−3)−n
an+1 = 3−1 − 3−2 + · · · + (−(−3)−j ) + · · · + (−(−3−n )) = 3−1 = .
1 − (−3−1 ) 4
182
E segue que a7 = 729 .

17. A sequência xn = 2n + 3n satisfaz xn+1 = 5xn − 6xn−1 com x0 = 2 e x1 = 5. Suponha que p|xn+1
e p|xn , com n ≥ 1. Então p|6xn−1 . Contudo, se n ≥ 1, 2 - 2n + 3n e 3 - 2n + 3n , logo (p, 2) = (p, 3) = 1.
Segue então que p | xn−1 . Assim, por indução (para trás!), obtemos que p divide x1 e p divide x0 ,
donde p = 1.
√ √
18. Seja α = 3 + 8 e β = 3 − 8. Como α + β = 6 e ab = 1, αn + β n satisfaz a recorrência
xn+1 = 6xn − xn−1 com condições iniciais x0 = 2 e x1 = 6. Então xn+1 ≡ −xn−1 mod 6. Por
indução, segue que xn é múltiplo de 6 quando n é ı́mpar. Ainda por indução, podemos demonstrar
que xn ≡ 2 mod 6 quando n é da forma 4k e xn ≡ −2 ≡ 4 mod 6 quando n é da forma 4k + 2. Por
fim, note que 0 < β n < 1 para todo natural n ≥ 1, logo bαn c = xn − 1. Segue que os restos são
ˆ 1, quando n = 4k;
ˆ 3, quando n = 4k + 2;
ˆ 5, quando n é ı́mpar.

19. Seja f uma função que satisfaz tal equação e x ∈ (0, ∞). Considere a sequência xn = f n (x), onde
f n indica a função f aplicada n vezes. Pela equação funcional, segue que xn satisfaz a recorrência
xn+2 = −xn+1 + 6xn para todo n ≥ 0 e além disso x0 = x. Note porém que a solução geral desta
recorrência é xn = α2n + β(−3)n .
Sabemos que x0 = x, logo α + β = x. Por outro lado, sabemos que xn é sempre positivo,
pois f tem contra-domı́nio (0, ∞). Note que, se β > 0, então para n suficientemente grande e ı́mpar
−(−3)n > αβ 2n , donde xn < 0 uma contradição. Por outro lado, se β < 0, então para n suficientemente
grande e par (−3)n > − αβ 2n , donde xn < 0, nova contradição. Segue que β tem que ser 0 e α = x (!),
donde f (x) = 2x é a única solução.

pn
20. Escreva an = . A recorrência fica
qn
pn+1 4q 2 + p2n
= n
qn+1 2pn qn
e, para calcular an , podemos considerar pn+1 = 4qn2 + p2n e qn+1 = 2pn qn (as frações não precisam ser
irredutı́veis). Segue que
xn+1 = pn+1 + 2qn+1 = (pn + 2qn )2 = x2n e yn+1 = pn+1 − 2qn+1 = (pn − 2qn )2 = yn2 .
n−1 n−1
Logo, xn = x21 e yn = y12 . Mas x1 = p1 + 2q1 = 4 + 2 · 1 = 6 e y1 = p1 − 2q1 = 4 − 2 · 1 = 2,
2n−1 2n−1 x n + yn x n − yn
donde xn = 6 e yn = 2 . Como pn = e qn = , vem que
2 4
n−1 n−1 n−1
2(62 + 22 ) 2(32 + 1) 4
an = 2n−1 2n−1 = 2n−1 = 2 + 2n−1 .
6 −2 3 −1 3 −1

Material elaborado por Hugo Fonseca Araújo

9
Problemas Resolvidos

Nı́vel 2

Indução

Material elaborado por Hugo Fonseca Araújo


Problemas

Problema 1. Prove que, para todo natural n ≥ 6,

(n + 3)3 ≤ 3n .

Problema 2. Prove que, para todo n inteiro positivo,

(n + 1) · (n + 2) · · · (2n) = 2n · 1 · 3 · 5 · · · · (2n − 1).

Problema 3. Prove que, para todo natural n ≥ 1,


 2
3 3 3 n(n + 1)
1 + 2 + ··· + n = .
2

Problema 4. (OBM 2006) Esmeralda posicionou todos os números naturais de 1 a 2006 no seguinte
arranjo em forma de pirâmide:
21
20 13 22
19 12 7 14 23
18 11 6 3 8 15 24
17 10 5 2 1 4 9 16 25

Em qual andar se encontrará o número 2006? (Por exemplo: o número 1 está no primeiro andar, o 6
no segundo andar e o 23 no terceiro).

Problema 5. Seja (Fn )n∈N a sequência de Fibonacci, definida por Fn+2 = Fn+1 +Fn para todo n ≥ 0,
F0 = 0 e F1 = 1. Mostre que se a e b são inteiros positivos, então

Fa+b = Fa−1 Fb + Fa Fb+1 .

Mostre que se a | b, então Fa | Fb .

Problema 6. O número de Euler é uma constante muito importante em estudos mais avançados de
matemática. Uma das maneiras de defini-lo é a partir da identidade
1 1 1 1
e=1+ + + + ··· + + ···
1! 2! 3! n!
Este número é irracional e vale aproximadamente 2, 718281828459045235360287. Usando apenas
indução podemos demonstrar que as aproximações 1 + 1!1 + 2!1 + 3!1 + · · · + n!
1
nunca passam de 3.
Curiosamente, tentar aplicar indução diretamente é mais difı́cil que mostrar por indução outro resul-
tado mais forte: mostre que, para todo n ≥ 1,
1 1 1 1 2
1+ + + + ··· + ≤3− .
1! 2! 3! n! (n + 1)!

2
Problema 7. (Torneio das Cidades 1986) Para cada subconjunto do conjunto {1, 2, 3, . . . , N } cal-
culamos o produto dos inversos dos seus elementos. Qual o valor, em função de N , da soma de todos
os produtos deste tipo?

n(n − 3)
Problema 8. Mostre que o número de diagonais de um polı́gono convexo de n lados é igual a .
2

Problema 9. (OBM 2015) Prove que existe um número que pode ser representado de pelo menos
2015 maneiras diferentes como soma de quadrados de números naturais não nulos, não necessariamente
todos distintos. Considera-se que duas somas que alteram apenas a ordem das parcelas constituem
uma mesma representação.
Por exemplo, 12 + 12 + 32 + 32 + 72 + 102 e 52 + 122 são duas maneiras distintas de escrevermos
169 como soma de quadrados.

Problema 10. Seja f : N∗ × N∗ → N∗ uma função satisfazendo f (1, 1) = 2 e

f (m + 1, n) = f (m, n) + 2(m + n)
f (m, n + 1) = f (m, n) + 2(m + n − 1).

Prove que f (m, n) = (m + n)2 − (m + n) − 2n + 2.

Problema 11. Seja n ≥ 2 e G um grafo com 2n vértices e pelo menos n2 + 1 arestas. Mostre que
existe um triângulo neste grafo, ou seja, três vértices que são ligados dois a dois por arestas do grafo.

Problema 12. (OBM 2003) Há N ≥ 3 cidades em Tumbólia. Cada duas cidades desse paı́s são
ligadas por uma rodovia ou uma ferrovia, não existindo nenhum par de cidades ligadas por ambos os
meios.
Um turista deseja viajar por toda Tumbólia, visitando cada cidade exatamente uma vez, e retornar
a cidade onde ele começou sua jornada.
Prove que é possı́vel escolher a ordem na qual as cidades serão visitadas de modo que o turista
mude o meio de transporte no máximo uma vez.

Problema 13. Existem n carros do mesmo modelo em diferentes pontos de um circuito de corrida.
Juntando o conteúdo dos seus tanques, obtemos a quantidade de combustı́vel necessária para que um
carro complete uma volta no circuito. Prove que existe um carro que pode completar uma volta no
circuito se ele “pegar” todo combustı́vel dos outros carros ao passar por cada um deles no circuito.

Problema 14. (Pan-Africana 2002) Sejam a1 ≥ a2 ≥ · · · ≥ an ≥ 0 números reais tais que


a1 + a2 + · · · + an = 1. Mostre que

a21 + 3a22 + 5a23 + · · · + (2n − 1)a2n ≤ 1.

Problema 15. Mostre que 17 é um resı́duo quadrático módulo 2k para todo k inteiro positivo1 . Em
outras palavras, mostre que para todo k inteiro positivo existem n e b inteiros positivos tais que

b2 = 2k n + 17.
1
ou seja, a equação x2 ≡ 17 mod 2k admite solução.

3
Problema 16. (Torneio das Cidades 2000) Cada quadradinho 1 × 1 de um tabuleiro n × n contém
um número diferente. Inicialmente, o menor número em cada linha é marcado, e verifica-se que estes
números marcados estão todos em colunas diferentes. Em seguida, o menor número em cada coluna
é marcado, e verifica-se que estes números marcados estão todos em linhas diferentes. Prove que os
conjuntos de números marcados em cada um dos procedimentos são idênticos.

Problema 17. (EUA 2003) Prove que, para todo inteiro positivo n, existe um número de n algarismos,
todos eles ı́mpares, divisı́vel por 5n .

Problema 18. (Coreia 2013) Seja (an )n∈N uma sequência de inteiros positivos satisfazendo an+2 =
an+1 + an para todo n ≥ 1. Defina, para cada n inteiro positivo,
4n−2
1 X
bn = ai .
a2n+1
i=1

Mostre que bn é inteiro para todo n ≥ 1.

Problema 19. (México 2002) Se m = 4k + 1 com k ∈ Z dizemos que m é do tipo 4k − 1; se for


m = 4k − 1 com k ∈ Z, dizemos que é do tipo 4k − 1. Seja n um inteiro positivo. O número n2 tem
mais divisores positivos do tipo 4k + 1 ou do tipo 4k − 1?

Problema 20. (Irã 2011) Arco-ı́ris é o nome de um passarinho que muda de cor. Ele pode escolher a
cada dia uma dentre n cores, mas suas cores em dois dias consecutivos nunca são iguais. Além disso,
não existem 4 dias em sua vida i < j < k < l tais que ele tenha a mesma cor nos dias i e k e a mesma
cor nos dias j e l, sendo esta cor diferente da cor que ele tinha nos dias i e k.
Qual é o maior número de dias que o pássaro Arco-ı́ris pode viver em função de n?

Problema 21. Considere a sequência (xn )n∈N definida por x0 = 1 e xn = ( n−2008


n )xn−1 ∀n ≥ 1.
Calcule
2007
X
S= 2i xi .
i=0

Problema 22. Sejam x1 , . . . , xm e y1 , . . . , yn inteiros positivos tais que as somas x1 + · · · + xm e


y1 + · · · + yn são iguais e, além disso, menores que mn. Prove que podemos cortar alguns termos da
igualdade
x1 + · · · + xm = y1 + · · · + xn
e ainda assim obter outra igualdade.

Problema 23. (EUA 1991) Mostre que, para qualquer inteiro fixado m ≥ 1, a sequência
2 22
2, 22 , 22 , 22 , . . .
é eventualmente constante módulo m. Ou seja, a partir de certo ponto, os números da sequência acima
tem o mesmo resto quando divididos por m.

Problema 24. (Torneio das Cidades 83) A sequência (an )n∈N é estritamente crescente e tal que
aak = 3k para todo inteiro k ≥ 1. Encontre a100 .

4
Soluções
1. Como esta lista é sobre indução, provaremos por indução.
Caso base: n = 6 =⇒ (n + 3)6 = (6 + 3)3 = 93 = 36 = 3n , ok!
Passo indutivo: Suponha válido para n, ou seja, (n + 3)3 ≤ 3n . Note que, para todo n ≥ 3,
3
1 3
 3
(n + 1 + 3)3
  
1 7 343
= 1+ ≤ 1+ = = < 3.
(n + 3)3 n+3 6 6 216

Dessa maneira, pela hipótese de indução,

(n + 1 + 3)3
(n + 1 + 3)3 = (n + 3)3 < 3 · 3n = 3n+1 ,
(n + 3)3

o que conclui a prova.

2. Caso base: Se n = 1, o lado esquerdo é igual a 2 e o direito 21 · 1 = 2, ok.


Passo indutivo: Suponha que vale para n, ou seja,

An = (n + 1) · (n + 2) · · · (2n) = 2n · 1 · 3 · 5 · · · · (2n − 1).

Para n + 1, o lado esquerdo da igualdade que queremos provar é

(2n + 1)(2n + 2)
(n + 2) · (n + 3) · · · (2n) · (2n + 1) · (2n + 2) = An = An · (2n + 1) · 2.
n+1
Mas usando a expressão acima, esta última expressão é igual a

2n · 1 · 3 · 5 · · · · (2n − 1) · (2n + 1) · 2 = 2n+1 · 1 · 3 · 5 · · · · (2n − 1) · (2n + 1),

e isto conclui a prova.

1·2
3. Caso base: Quando n = 1, os dois lados da identidade são iguais a 1, pois 2 = 1.
Passo indutivo: Suponha válido para n. Segue que

n(n + 1) 2
   2 
n
13 + 23 + · · · + n3 + (n + 1)3 = + (n + 1)3 = (n + 1)2 +n+1 =
2 4
(n + 1)(n + 2) 2
 2
(n + 1)2 (n + 2)2
  
2 n + 4n + 4
(n + 1) = = ,
4 4 2

sendo assim a identidade também válida para n + 1, o que conclui a prova.

4. Observamos inicialmente que o arranjo de Esmeralda segue as camadas ∧ descritas abaixo


21
20 13 22
19 12 7 14 23
18 11 6 3 8 15 24
17 10 5 2 1 4 9 16 25

5
Afirmamos que a n-ésima camada tem 2n − 1 números e eles vão de (n − 1)2 + 1 até n2 . A prova é
por indução.
Caso base: Para a 1ª camada é óbvio, só tem o 1. Para a segunda, os números são 2, 3, 4, e a
afirmação é válida para n = 2.
Passo indutivo: Suponha válido para a n-ésima camada. A próxima necessariamente começa com
n2 + 1. Além disso, ela tem exatamente dois números a mais que a anterior (você pode imaginar que
no total adicionamos o número inicial e o final e levantamos a figura ∧ um andar para cima). Assim,
ela tem 2n − 1 + 2 = 2(n + 1) − 1 números e vai até n2 + 1 + 2n = (n + 1)2 .
O menor quadrado perfeito mais próximo de 2006 é 442 = 1936. O número 2006 tem que estar
então na 45ª camada. Além disso, ele é o 70º número nela, pois 2006 − 1937 = 69. Isto significa que
nesta camada ainda aparecem 2 · 45 − 1 − 70 = 19 números após 2006. Dessa maneira, 2006 está no
20º andar.

5. Provaremos a igualdade para a, b ≥ 1 por indução em a.


Caso base: Quando a = 1, a igualdade fica Fb+1 = F0 Fb + F1 Fb+1 = Fb+1 , ok.
Passo indutivo: Fa+1+b = Fa+b + Fa+b−1 = F(a)+b + F(a−1)+b . Pela hipótese de indução,

Fa+1+b = Fa−1 Fb +Fa Fb+1 +Fa−2 Fb +Fa−1 Fb+1 = Fb (Fa−1 +Fa−2 )+Fb+1 (Fa +Fa−1 ) = Fa Fb +Fa+1 Fb+1 ,

o que conclui esta indução.


Agora consideraremos a fixo e b = ka. Provaremos por indução em k que Fa | Fka para todo k ≥ 1.
Caso base: Se k = 1, então a = ka =⇒ Fa = Fka .
Passo indutivo: Seja k ≥ 2. Pela fórmula que provamos acima,

Fka = F(k−1)a+a = Fa−1 F(k−1)a + Fa F(k−1)a+1 ,

e pela hipótese de indução, Fa | Fka .

6. Caso base: n = 1 =⇒ 1 + 1!1 = 2 = 3 − (1+1)!2


, ok!
Passo indutivo: Pela hipótese de indução
1 1 1 1 2 1 1 2
1+ + + ··· + + ≤3− + =3− ≤3− ,
1! 2! n! (n + 1)! (n + 1)! (n + 1)! (n + 1)! (n + 2)!
1 2
pois (n+1)! ≥ (n+2)! ⇐⇒ (n + 2) ≥ 2, o que é sempre verdade para n ≥ 1.

7. Seja An a soma desses produtos. Fazendo casos pequenos N = 1, 2, 3 ,obtemos AN igual a 1, 2, 3


respectivamente. Somos levados a conjecturar que o resultado é sempre igual a N . Provaremos por
indução. O caso base N = 1 já foi feito.
Passo indutivo: Suponha válido para N . Para N +1, dividimos os produtos em dois grupos, aqueles
para os quais o fator N 1+1 aparece e aqueles em que não. Excluindo-se o produto constituı́do apenas de
um fator N 1+1 , todos aqueles do primeiro grupo são obtidos a partir de um do segundo adicionando-se
o fator N 1+1 e cada um do segundo é obtido a partir de um do primeiro ignorando-se o fator N 1+1 .
Eles estão assim em bijeção. Além disso, a soma dos termos do segundo grupo é igual a AN , pois
correspondem aos produtos relativos ao conjunto {1, 2, 3, . . . , N }. Logo AN +1 = N 1+1 AN + N 1+1 +AN ,
que pela hipótese é igual a N 1+1 N + N 1+1 + N = N + 1. 

8. Caso base: Quando n = 3 verificamos que um triângulo tem 0 diagonais, ok!


Passo indutivo: Sejam A1 , A2 , . . . , An+1 os vértices consecutivos de um polı́gono convexo P com
n + 1 lados. Considere o polı́gono convexo P 0 definido pelos n vértices A1 , A3 , . . . , An+1 . Observe

6
que toda diagonal de P 0 é diagonal de P . Pela hipótese, elas são n(n−3)2 . Além dessas, os segmentos
A1 A3 , A2 A4 , A2 A5 , . . . , A2 An também são diagonais de P . Assim, o número de diagonais de P é

n(n − 3) n2 − n − 2 (n + 1)(n − 2)
+1+n−2= = .
2 2 2

9. Afirmamos que, para cada n natural, existe um inteiro positivo m que pode ser escrito de, ao
menos, n formas diferentes como soma de quadrados perfeitos.
Caso base: Para n = 1 tome m um quadrado perfeito.
Passo indutivo: Seja m que pode ser escrito de n maneiras diferentes como soma de quadrados
perfeitos. Note que 4m pode ser escrito de n maneiras diferentes como soma de quadrados perfeitos
sendo todos eles pares, basta multiplicar por 4 cada termo de cada uma das maneiras que conhecemos
para m. Assim, o número m0 = 4m + 52 também pode ser escrito de n maneiras distintas como soma
de quadrado perfeitos, tendo apenas o 52 como fator ı́mpar. Podemos tomar uma destas maneiras e
construir uma nova trocando 52 por 32 + 42 e ela será distinta das n outras porque nenhuma delas
continha o fator 32 . Isto conclui a prova.

10. Faremos indução dupla. Começamos mostrando que f (m, 1) = (m + 1)2 − (m + 1) para todo
m ∈ N∗ .
Caso base: Quando m = 1, f (1, 1) = 2 = (1 + 1)2 − (1 + 1).
Passo indutivo: Suponha válido para m. Segue que

f (m + 1, 1) = f (m, 1) + 2(m + 1) =(m + 1)2 − (m + 1) + 2(m + 1) =


(m + 1)2 + 2(m + 1) + 1 − (m + 2) = (m + 2)2 − (m + 2).

Provado este fato, mostremos por indução em n que, para todo m ∈ N∗ e todo n ∈ N∗ ,
f (m, n) = (m + n)2 − (m + n) − 2n + 2.
Caso base: Quando n = 1, 2n = 2 e caı́mos exatamente na afirmação que acabamos de provar.
Passo indutivo: Suponha válido para algum n e todo m ∈ N∗ . Segue que, para todo m ∈ N∗ ,

f (m, n + 1) = f (m, n) + 2(m + n − 1) = (m + n)2 − (m + n) − 2n + 2 + 2(m + n − 1) =


(m + n)2 + 2(m + n) + 1 − 1 − (m + n) − 2n + 2 − 2 = (m + n + 1)2 − (m + n + 1) − 2(n + 1) + 2. 

11. Este é um bom exemplo no qual a indução é melhor descrita por uma redução ao caso anterior.
Em geral, é conveniente enxergar a indução como um procedimento deste tipo.
Caso base: Se n = 2, o grafo tem 4 vértices. Como ele tem 5 arestas, pelo princı́pio das casas dos
pombos, existe algum vértice v com grau 3, conectando-se a todos os outros vértices. Existe um par
destes outros vértices ligado por alguma aresta, já que 5 > 3. Juntando este par a v formamos um
triângulo.
Passo indutivo: Considere um grafo com 2n + 2 vértices, (n + 1)2 + 1 arestas, e um par de vértices,
(v1 , v2 ), ligados por uma aresta. Seja E o conjunto dos 2n vértices restantes. Caso o número de
arestas ligando v1 e v2 a E seja maior que 2n, existe algum vértice de E ligado ao mesmo tempo a v1
e a v2 , completando um triângulo. Caso contrário, podemos retirar v1 e v2 do grafo, removendo todas
as arestas que estão conectadas a eles, num total de no máximo 2n + 1 arestas, obtendo um grafo
com 2n vértices e ao menos (n + 1)2 + 1 − 2n − 1 = n2 + 1 arestas. Pela hipótese de indução, há um
triângulo neste grafo, logo também há um no original. 

12. Caso base: Quando N = 3, as cidades de Tumbólia formam um triângulo, com três estradas,
ferrovias ou rodovias. Ou são todas do mesmo tipo, ou duas de um tipo e uma do outro. No primeiro

7
caso não é necessário mudar de transporte. No segundo, começando o caminho percorrendo o tipo de
estrada que só aparece uma vez, o turista muda de transporte só uma vez.
Passo indutivo: Suponha que Tumbólia tem N + 1 cidades. Ignorando uma cidade A e descon-
siderando todas estradas a ela conectadas, obtemos Nova Tumbólia, com N cidades. Por hipótese
de indução, o turista consegue fazer seu trajeto passando por todas cidades de Nova Tumbólia
B1 , B2 , B3 , . . . , Bn e regressando a B1 nesta ordem. Analisaremos vários casos:

1. Se todas estradas neste ciclo são rodovias, temos dois casos:

a) se A conecta-se às cidades de Nova Tumbólia apenas por ferrovias, o caminho


B1 , A, B2 , B3 , . . . , Bn , B1 começa por duas ferrovias e segue por rodovias, logo funciona
para Tumbólia original.
b) se existe j tal que uma rodovia conecta A a Bj , no caminho A, Bj , Bj+1 , . . . , Bj−1 , A,
onde os ı́ndices são tomados módulo n, é necessário trocar de transporte no máximo uma
vez, ao ir de Bj−1 para A.

2. Se o turista necessita trocar de transporte ao passar por Bj , sem perda de generalidade podemos
supor que de B1 até Bj ele percorre rodovias e daı́ por diante ferrovias. Temos três casos:

a) Se A é conectada a Bj−1 por uma rodovia, o caminho B1 , . . . , Bj−1 , A, Bj , Bj+1 , . . . , Bn , B1


funciona para Tumbólia original, trocando de meio só em A ou Bj+1 .
b) Se A é conectada a Bj+1 por uma ferrovia, o caminho B1 , . . . , Bj , A, Bj+1 , . . . , Bn , B1 ,
analogamente, funciona para Tumbólia original, trocando de meio só em Bj ou A.
c) Se A é conectada a Bj−1 por uma ferrovia e a Bj+1 por uma rodovia, temos dois casos.
Se A é conectada a Bj por rodovia, o turista segue por B1 , . . . , Bj , A, Bj+1 , . . . , Bn , B1 ,
trocando de meio de transporte apenas em Bj+1 . Se for conectada por uma ferrovia, o
turista segue por B1 , . . . , Bj−1 , A, Bj , Bj+1 , . . . , Bn , B1 , trocando apenas em Bj−1 . 

13. Caso base: Se n = 1 a afirmação é óbvia.


Passo indutivo: Sejam n + 1 carros posicionados no circuito como no enunciado do problema. Para
cada um deles, considere o trajeto que pode percorrer usando apenas o combustı́vel no seu tanque.
Algum destes trajetos tem que alcançar a posição original de algum outro carro, pois se isso não
acontece, todos os trajetos seriam disjuntos, implicando que o combustı́vel total não é suficiente para
uma volta. Seja então C o primeiro carro cuja posição original é alcançada pelo trajeto do carro B.
Considere outro circuito no qual há n carros, idêntico ao original, exceto que C não existe mais e
B tem a soma do combustı́vel de C com o de B na situação original. Pela hipótese, há algum carro A
capaz de dar a volta no novo circuito.
Este mesmo carro é capaz de dar a volta no circuito original. O procedimento que ele realiza é
o mesmo até chegar ao carro B (o que pode acontecer no primeiro instante, caso A = B). Neste
instante, tomando todo o combustı́vel de B, ele passa a ter uma quantidade suficiente para ir até C,
pelo que foi discutido no primeiro parágrafo. Chegando em C, A toma todo seu combustı́vel, e daı́
em diante tudo transcorre como no sistema original, pois a soma da quantidade de combustı́vel de C
com o de B é igual a quantidade de combustı́vel de B no sistema modificado. 

14. Mostraremos por indução que a desigualdade vale para cada n natural.
Caso base: Quando n = 1, a1 = 1 e a desigualdade é óbvia.
Passo indutivo: Sejam a1 ≥ a2 ≥ · · · ≥ an ≥ an+1 ≥ 0 com a1 + · · · + an + an+1 = 1. Isto implica
1
que (n + 1)an+1 ≤ 1 =⇒ an+1 ≤ n+1 .

8
Definindo, para i = 1, . . . , n, bi = 1−aan+1
i
, obtemos b1 ≥ b2 ≥ · · · ≥ bn ≥ 0 e b1 + . . . bn = 1. Pela
hipótese de indução,
b21 + 3b22 + 5b23 + · · · + (2n − 1)b2n ≤ 1.
Substituindo o valor de bi , obtemos

3a22 + 5a23 + · · · + (2n − 1)a2n + (2n + 1)a2n+1 ≤ (1 − an+1 )2 + (2n + 1)a2n+1 ,


1
que é menor ou igual a 1 se e somente se 2(n + 1)a2n+1 ≤ 2an+1 ⇐⇒ an+1 ≤ n+1 . Mas isto já foi
provado. 

15. Caso base: Seja b = 5. Quando k = 1, 2, 3 tome n = 4, 2, 1 respectivamente.


Passo indutivo: Sejam b e n tais que b2 = 2k n + 17 para k ≥ 3. Iremos construir b0 e n0 a partir
de b e n de forma que b0 2 = 2k+1 n0 + 17. Se n é par, basta tomar b0 = b e n0 = n/2. Caso contrário, é
intuitivo somar algo a b para obter b0 e analisar o que acontece com b0 2 . Se o que somamos é múltiplo
de 2k , obtemos (b0 )2 ≡ b2 mod 2k+1 , o que não funciona. Somos levados então a testar b0 = b + 2k−1 .
Neste caso,
2
b0 = b2 + 2 · 2k−1 b + 22k−2 = 2k (n + b) + 2(k+1)+k−3 + 17.
Porém, como b e n são ı́mpares, b+n é inteiro, donde b0 2 = 2k+1 b+n k−3 + 17 e tomando

2 2 +2
b0 = b2 + 2k−1 e n0 = b+n 2 +2
k−3 resolvemos o problema. 

16. Caso base: É óbvio quando n = 1.


Passo indutivo: Em um tabuleiro (n + 1) × (n + 1) considere a casinha que contém o menor inteiro.
Observe que como os números são todos distintos, existe um número x que é estritamente menor do
que todos outros. Suponha que esta casa está na linha a e coluna b. Note que, pela condição do
enunciado, para qualquer linha i 6= a o menor número não esta na coluna b . Analogamente, o menor
número de qualquer coluna j 6= b não está na linha a. Considere então o tabuleiro n × n obtido
eliminado a linha a e a coluna b. Ele ainda satisfaz a condição do enunciado, pois os menores números
de cada linha ou coluna são os mesmos das respectivas linha e coluna do tabuleiro original. Assim,
pela hipótese de indução, as n casas marcadas em cada um dos procedimentos são as mesmas. Observe
porém que no tabuleiro original marcamos exatamente estas casas, e além disto, na linha a marcamos
a casa que tem o x, já que é o menor de todos, e na coluna b também marcamos este número. Assim,
as casa marcadas são as mesmas nos dois procedimentos.

17. Caso base: Para n = 1, tome o número 5.


Passo indutivo: Seja A um número de n algarismos, todos eles ı́mpares, tal que 5n | A. Seja
a = 1, 3, 5, 7 ou 9. O número B = 10n a + A tem todos seus algarismos ı́mpares. Além disso, se
A = 5n a0 , então B = 5n (2n a + a0 ). Basta encontrar a tal que 2n a + a0 ≡ 0 mod 5. Note que
a = 1, 3, 5, 7, 9 cobre todos os resı́duos módulo 5. Assim, basta tomar aquele tal que

a ≡ −(2n )−1 a0 mod 5,

pois 2n é invertı́vel módulo 5. 

18. Neste exercı́cio, a dificuldade é encontrar afirmações secundárias que devemos provar por indução
para chegar ao resultado. Observe inicialmente que esta sequência é definida pela mesma regra da
sequência de Fibonacci, o que podem mudar são seus termos iniciais (que na sequência de Fibonacci são
F1 = F2 = 1). Fazendo alguns casos pequenos, encontramos a3 = a1 +a2 , a4 = a1 +2a2 , a5 = 2a1 +3a2 ,
a6 = 3a1 + 5a2 . Somos levados a conjecturar que, para todo n ≥ 3, an = Fn−2 a1 + Fn−1 a2 . Provemos
por indução forte em n.

9
Caso base: Para n = 3 já foi verificado.
Passo indutivo: an+1 = an + an−1 , mas pela hipótese de indução, an + an−1 = Fn−2 a1 + Fn−1 a2 +
Fn−3 a1 + Fn−2 a2 = (Fn−2 + Fn−3 )a1 + (Fn−1 + Fn−2 )a2 = Fn−1 a1 + Fn a2 .
Precisamos encontrar uma maneira de calcular uma soma do tipo ki=1 ai . Pela expressão acima,
P

ela é igual a (1 + k−2


P Pk−1
i=1 Fi )a1 + (1 + i=2 Fi )a2 . Fazendo alguns casos pequenos, podemos encontrar
boas expressões para os somatórios na expressão anterior: F1 +F2 +F3 = 4 = F5 −1,
P F1 +F2 +F3 +F4 =
7 = F6 − 1, F1 + F2 + F3 + F4 + F5 = 12 = F7 − 1. Provemos por indução que ki=1 Fi = Fk+2 − 1:
Caso base: Para k = 1, temos F1 = 1 = 2 − 1 = F3 − 1.
Passo indutivo: Pela hipótese, F1 + F2 + · · · + Fn + Fn+1 = Fn+2 − 1 + Fn+1 = Fn+3 − 1.
Voltemos agora ao problema original. Quebramos a soma 4n−2
P P2n P4n−2
i=1 ai em i=1 ai + i=2n+1 ai e
definimos ci = ai+2n . Note que ci+2 = ci+1 + ci . Dessa maneira, aplicando a fórmula que acabamos
de encontrar, obtemos
2n 2n−2 2n−1
! !
X X X
ai = 1 + Fi a1 + 1 + Fi a2 = F2n a1 + (F2n+1 − 1)a2
i=1 i=1 i=2
4n−2 2n−2 2n−4 2n−3
! !
X X X X
ai = ci = 1+ Fi c1 + 1+ Fi c2 = F2n−2 c1 + (F2n−1 − 1)c2 .
i=2n+1 i=1 i=1 i=2

Contudo, c1 = a2n+1 = F2n−1 a1 + F2n a2 e c2 = a2n+2 = F2n a1 + F2n+1 a2 . Juntando tudo, obtemos
4n−2
X
ai = (F2n F2n−1 + F2n−2 F2n−1 )a1 + [F2n+1 − 1 + F2n−2 F2n + (F2n−1 − 1)F2n+1 ]a2 =
i=1
(F2n + F2n−2 )F2n−1 a1 + (F2n+1 F2n−1 − 1 + F2n−2 F2n )a2 .

Como a2n+1 = F2n−1 a1 + F2n a2 , se mostrarmos que F2n+1 F2n−1 − 1 = F2n 2 , concluı́mos que b =
n
F2n + F2n−2 , logo um inteiro. Mais uma vez, demonstraremos este último fato por indução2 .
Caso base: Para n = 1, a equação fica F3 F1 − 1 = 2 · 1 − 1 = 1 = F22 , ok!
Passo indutivo: Note que
2 2
F2n+2 − F2n = (F2n+2 + F2n )(F2n+2 − F2n ) = (F2n+2 + F2n )F2n+1 .
2 =F
daı́, se F2n 2n−1 F2n+1 − 1, então

2
F2n+2 = F2n−1 F2n+1 − 1 + F2n+1 (F2n+2 + F2n ) = F2n+1 (F2n+2 + F2n + F2n−1 ) − 1 =
F2n+1 (F2n+2 + F2n+1 ) − 1 = F2n+1 F2n+3 − 1

Isto conclui a prova.

19. Seja n2 = pα1 1 · pα2 2 · · · pαk k , onde cada αi é par, e A(n) o número de seus divisores do tipo 4k + 1 e
B(n) o número dos seus divisores do tipo 4k + 3. Mostraremos, por indução em k, ou seja, no número
de fatores primos distintos de n, que n2 tem mais divisores do tipo 4k + 1 do que do tipo 4k + 3, isto
é, A(n) > B(n).
Caso base: Quando n = 1, existe apenas um divisor e ele é do tipo 4k+1, logo A(1) = 1 > 0 = B(1).
αk+1 αk+1
Passo indutivo: Seja n2 = pα1 1 · pα2 2 · · · pαk k · pk+1 = n0 2 · pk+1 . Pela hipótese, A(n0 ) > B(n0 ).
Analisamos três casos:

1. Se pk+1 = 2, A(n) = A(n0 ) e B(n) = B(n0 ). Segue que A(n) > B(n).
2
Na verdade, pode-se mostrar um resultado mais geral, que Fn−1 Fn+1 = Fn2 + (−1)n ∀n ∈ N.

10
2. Se pk+1 é do tipo 4k + 1, todos números pik+1 , i = 0, 1, . . . , αk+1 , são do tipo 4k + 1, logo
A(n) = (αk+1 + 1) A(n0 ) e B(n) = (αk+1 + 1) B(n0 ). Segue que A(n) > B(n).
3. Se pk+1 é do tipo 4k + 3, pik+1 é um número do tipo 4k + 1 para i = 0, 2, 4, . . . , αk+1 e do tipo
4k + 3 para i = 1, 3, . . . , αk+1 − 1. Segue que
a  ak+1 ak+1 a 
k+1 k+1
A(n) = + 1 A(n0 ) + B(n0 ) e B(n) = A(n0 ) + + 1 B(n0 ),
2 2 2 2
e, pela hipótese, A(n) − B(n) = A(n0 ) − B(n0 ) > 0, obtendo A(n) > B(n) como querı́amos
demonstrar.

20. Arco-ı́ris pode viver no máximo 2n − 1 dias. Provaremos por indução forte em n.
Caso base: Quando n = 1, ele não pode viver dois dias, pois no segundo teria que ter a mesma cor
do primeiro. O número máximo de dias é 1 = 2 · 1 − 1.
Passo indutivo: Suponha válido para todo m < n. Considere uma vida do passarinho na qual ele
assumiu n cores ao longo dos seus dias. Seja C sua cor no primeiro dia e 1 = c1 , c2 , . . . , ck os dias
nos quais ele teve esta cor. Afirmamos que se ele teve uma cor A 6= C em algum dia entre os dias ci e
ci+1 para algum i = 1, . . . , k − 1, ou após o dia ck , ele não teve esta cor em dias que não estão entre ci
e ci+1 ou após o dia ck respectivamente. Com efeito, se ele tivesse esta cor em algum outro intervalo,
existiria uma sequência crescente de dias nos quais ele tem as cores A, C, A, C ou C, A, C, A, mas isto
não é possı́vel, por causa da segunda condição do enunciado.
Para cada i = 1, . . . , k − 1 seja ai o números de cores que ele assumiu entre os dias ci e ci+1 e ak
o número de cores que ele toma após o dia ck . Note que, como ele não toma a mesma cor dois dias
consecutivos, ai ≥ 1 para todo i = 1, . . . , k − 1. Assim, pela hipótese de indução, o intervalo entre ci
e ci+1 tem no máximo 2ai − 1 dias. Se ak ≥ 1, ele viveu no máximo 2ak − 1 dias após ck ; se ak = 0
ele não viveu após ck . No primeiro caso, como a cor C apareceu k vezes e 1 + a1 + a2 + · · · + ak = n,
pois totalizam todas as n cores que Arco-ı́ris teve, o número máximo de dias que Arco-ı́ris viveu é
k + (2a1 − 1) + (2a2 − 1) + · · · + (2ak − 1) = 2n − 2. No segundo caso temos 1 + a1 + a2 + · · · + ak−1 = n,
e ele viveu no máximo k + (2a1 − 1) + (2a2 − 1) + · · · + (2ak−1 − 1) = 2n − 1 dias.
Não é difı́cil construir um exemplo no qual ele vive 2n − 1 dias. Se C1 , . . . , Cn são suas cores,
basta escolher sucessivamente as cores C1 , C2 , C1 , C3 , C1 , . . . , C1 , Cn , C1 . Isto conclui a prova.

21. Calculando os primeiros termos encontramos


−2006 2007 · 2006 −2005 2007 · 2006 · 2005
x1 = −2007, x2 = x1 = , x3 = x2 = −
2 2 3 6
 
2007
e somos levados a conjecturar que xn = (−1)n . Provaremos por indução.
n  
n 2007
O caso base já está feito, fizemos até mais que o necessário. Supondo que xn = (−1) vem
n
 
n + 1 − 2008 n + 1 − 2008 n 2007 n + 1 − 2008 2007!
xn+1 = xn = (−1) = (−1)n =
n+1 n+1 n n+1 n!(2007 − n)!
 
2007!(−1)(2007 − n) 2007! 2007
(−1)n = (−1)n+1 = (−1)n+1
(n + 1)n!(2007 − n)! (n + 1)!(2007 − n − 1)! n+1
e a indução está provada. Pelo binômio de Newton e o fato que i e 2007 − i tem paridades distintas,
a soma pedida é
2007 2007
X  2007
X 2007
X
i 2007 i i
S= 2 xi = · 2 · (−1) = − · 2i · (−1)2007−i = −(2 − 1)2007 = −1.
i i
i=0 i=0 i=0

11
22. Faremos indução forte no valor da soma x1 + · · · + xm = y1 + · · · + yn = k. Quando k = 1, os
lados da equação tem que ser x1 = y1 = 1, donde m = n = mn = 1. Mas o enunciado pede que a
soma seja menor que o produto mn, assim este caso não acontece.
Caso base: Se k = 2, como os xi e yj são inteiros positivos, temos m, n ≤ 2. O único valor possı́vel
de mn maior que 2 é 4 e ele acontece somente na igualdade 1 + 1 = 1 + 1. Assim podemos cortar um
número 1 de cada lado, resolvendo o problema neste caso.
Passo indutivo: Suponha válido para todas as igualdades possı́veis x1 +· · ·+xm0 = y1 +· · ·+yn0 = k 0
satisfazendo 2 ≤ k 0 < k e k 0 ≤ m0 n0 . Considere uma igualdade x1 + · · · + xm = y1 + · · · + yn = k
k
satisfazendo k ≤ mn. Algum valor xa do lado esquerdo é maior ou igual a m e algum valor yb do
lado direito é maior ou igual a nk . Sem perda de generalidade considere xa ≤ yb . Se há igualdade,
cancelamos estes valores e o problema terminou. Caso contrário, apague xa e troque yb por yb0 = yb −xa .
O lado esquerdo tem agora m − 1 termos e o direito n. Além disso, a soma agora é menor ou igual
k 1 1

a k−m = k 1− m ≤ mn 1 − m = (m − 1)n. Pela hipótese, existe alguma maneira de cancelar
termos dessa nova equação, de forma a obter uma igualdade. Se este cancelamento não envolve yb0 ,
ele também vale para a igualdade original. Se envolver, como yb = xa + yb0 e xa não faz parte deste
cancelamento, pois xa foi apagado, cancelamos na igualdade original os mesmos termos, com yb no
lugar de yb0 e cancelamos também xa , obtendo nova igualdade. Isto conclui a prova.

23. Lembre do
Teorema de Euler. Se a e m são primos entre si, então

aφ(m) ≡ 1 mod m,

onde φ(m) é igual ao número de inteiros entre 1 e m relativamente primos com m.


2 22
O que importa para nós é que φ(m) < m para todo m ≥ 2. A sequência 2, 22 , 22 , 22 , . . . é dada
por a1 = 2 e an+1 = 2an ∀n ≥ 1. Provaremos por indução forte em m ≥ 2 que ela é eventualmente
constante módulo m.
Caso base: Quando m = 2 a sequência é congruente a 0 módulo 2.
Passo indutivo: Suponha que a sequência é eventualmente constante módulo k para todo 2 ≤ k <
m. Em particular, ela é eventualmente constante módulo φ(m).
Se m é par3 , então m = 2k b com k ≥ 1 e b < m ı́mpar. Note que an é eventualmente congruente
a 0 módulo 2k , pois a partir de certo ponto é formada por potências de 2 maiores. Basta então, pelo
Teorema Chinês dos Restos, mostrar que ela é eventualmente constante módulo b. Mas isto vem da
hipótese de indução.
Se m é ı́mpar, pelo Teorema de Euler,

an1 ≡ an2 mod φ(m) =⇒ 2an1 ≡ 2an2 mod m ⇐⇒ an1 +1 ≡ an2 +1 mod m.

Como, pela hipótese, ela é eventualmente constante módulo φ(m), ela é eventualmente constante
módulo m. Isto conclui a prova.

24. Começamos calculando valores pequenos de ak . Note que aa1 = 3. Como an é estritamente
crescente, devemos ter a1 < a2 < a3 e logo a3 > 3. Mas isto significa então que a1 < 3, pois caso
contrário aa1 > a3 > 3. Assim a1 = 1 ou 2. Contudo, se a1 = 1 terı́amos aa1 = a1 = 1, contradição.
Segue que
a1 = 2 e daı́ a2 = 3, a3 = aa2 = 3 · 2 = 6 e a6 = aa3 = 9.
Porém vale que a3 < a4 < a5 < a6 , e isto força a4 = 7 e a5 = 8. A regra aak = 3k nos permite calcular
a7 = 12, a8 = 15, a9 = 18. Podemos começar a notar um padrão, não muito óbvio na sequência,
3
Observe que neste caso não podemos usar o Teorema de Euler para a = 2.

12
baseado na sua aproximação por potências de 3. Fazendo mais uns casos se necessário, podemos
conjecturar que, fixado k inteiro,

an = 2 · 3k , 2 · 3k + 1, 2 · 3k + 2, . . . , 2 · 3k + 3k = 3k+1 para n = 3k , 3k + 1, 3k + 2, . . . , 2 · 3k

respectivamente e

an = 3k+1 , 3k+1 +3, 3k+1 +2·3, . . . , 3k+1 +3k ·3 = 2·3k+1 para n = 2·3k , 2·3k +1, 2·3k +2, . . . , 2·3k +3k = 3k+1

respectivamente. A grande dificuldade desta questão é chegar nesta conjectura. Provemos por indução
em k
Caso base: Os casos k = 0 e k = 1 se encontram dentre os casos pequenos que já fizemos.
Passo indutivo: Note que a3k+1 = 2 · 3k+1 pela hipótese de indução. Daı́ a2·3k+1 = aa3k+1 =
3(3k+1 ) = 3k+2 . Como

2 · 3k+1 = a3k+1 < a3k+1 +1 < a3k+1 +2 < · · · < a3k+1 +3k+1 = a2·3k+1 = 3k+2
| {z }
3k+1 +1 termos

e 3k+2 − 2 · 3k+1 = 3k + 1, os valores de an para n = 3k+1 , 3k+1 + 1, 3k+1 + 2, . . . , 2 · 3k+1 tem que
ser iguais a 2 · 3k+1 , 2 · 3k+1 + 1, 2 · 3k+1 + 2, . . . , 3k+2 respectivamente. A partir daı́, os valores de an
para n = 2 · 3k+1 , 2 · 3k+1 + 1, 2 · 3k+1 + 2, . . . , 3k+2 , podem ser calculados pela regra aak = 3k. Se i é
inteiro entre 0 e 3k+1 , então a2·3k +i = aa3k+1 +i = 3(3k+1 + i) = 3k+2 + 3i por causa do que acabamos
de demonstrar, concluindo a indução.
Como 100 = 34 + 19, da afirmação acima segue que a100 = 2 · 34 + 19 = 181.

Material elaborado por Hugo Fonseca Araújo

13
Problemas Resolvidos

Nı́vel 2

Desigualdades I

Material elaborado por Valentino Amadeus Sichinel


Problemas

1 Desigualdades Elementares 2 Desigualdade das Médias

Problema 1. Sejam a, b e c números reais. Problema 8. Sejam a e b reais positivos. Prove


Mostre que que
a b
+ ≥ 2.
(a + b + c)2 ≥ 3(ab + bc + ca). b a
Quando a igualdade se verifica?

Problema 2. Seja a um número real. Mostre Problema 9. Sejam a e b reais positivos. Prove
que que
a2 + b2  a + b 2
a4 − 4a2 ≥ −4. ≥ .
2 2

Problema 3. Sejam a, b, c e d reais positivos. Problema 10. Sejam a, b e c reais positivos.


Suponha que ab ≥ dc . Prove que Mostre que

(a + b)(b + c)(c + a) ≥ 8abc.


a+b c+d
≥ .
b d
Problema 11. Prove que, dentre todos os
retângulos com o mesmo perı́metro p, o qua-
Problema 4. Mostre que drado é o que possui a maior área.

a2 + b2 + 1 ≥ ab + b + a,
Problema 12. Sejam a1 , a2 , . . . , an números re-
quaisquer que sejam a, b ∈ R. ais positivos. Prove que
1 1 1
(a1 + a2 + · · · + an ) + +···+ ≥ n2 .
a1 a2 an
Problema 5. Seja a um número real. Mostre
que
Problema 13 (OBM2001). Prove que
4a − a4 ≤ 3. p
(a + b)(a + c) ≥ 2 abc(a + b + c)

para quaisquer números reais positivos a, b e c.


Problema 6. Determine se existe uma função
injetiva f : R → R tal que f (x2 ) − f (x)2 ≥ 41
para todo x ∈ R. Problema 14. Prove que, dentre todos os
triângulos com o mesmo perı́metro p, o triângulo
equilátero é o que possui a maior área.
Problema 7. Encontre todos os pares (a, b) de
números reais que satisfazem
Problema 15. Sejam a e b reais positivos tais
que a + b = 1. Prove que
(4a2 + 4a + 3)(b2 − 6b + 13) = 8.
a2 b2 1
+ ≥ .
a+1 b+1 3

2
Soluções

1. Abramos o lado esquerdo da desigualdade: temos

(a + b + c)2 = (a2 + b2 + c2 ) + 2(ab + bc + ca).

Então, mostrar que (a+b+c)2 ≥ 3(ab+bc+ca) é o mesmo que mostrar que a2 +b2 +c2 ≥ ab+bc+ca.
Mas isso é um dos problemas que vimos em aula1 !

2. Temos
a4 − 4a2 ≥ −4 ⇐⇒ a4 − 4a2 + 4 ≥ 0 ⇐⇒ (a2 − 2)2 ≥ 0.
Como o quadrado de um número real é sempre maior que ou igual a zero, a última desigualdade
é válida. Assim (as desigualdades são todas equivalentes), a primeira também é.

b
3. O objetivo deste problema era exercitar a manipulação de desigualdades, apenas. Como b = 1 = dd ,

a+b c+d a+b b c+d d


≥ ⇐⇒ − ≥ −
b d b b d d
a+b−b c+d−d
⇐⇒ ≥
b d
a c
⇐⇒ ≥ .
b d

4. É uma mera questão de enxergar os quadrados:

2(a2 + b2 + 1) − 2(ab + a + b) = (a2 + b2 − 2ab) + (a2 + 1 − 2a) + (b2 + 1 − 2b)


= (a − b)2 + (a − 1)2 + (b − 1)2 .

Sendo soma de quadrados, 2(a2 + b2 + 1) − 2(ab + a + b) é maior que ou igual a zero. Assim,
2(a2 + b2 + 1) ≥ 2(ab + a + b), isto é, a2 + b2 + 1 ≥ ab + a + b.

5. Mais uma vez, o que precisamos fazer é encontrar os quadrados. Temos

4a − a4 ≤ 3 ⇐⇒ a4 − 4a + 3 ≥ 0. (1)

Se a4 é parte de um quadrado, provavelmente esse quadrado envolve a2 . Uma parcela com a2 pode
ser parte também de um quadrado que envolve (−4a). Após algumas tentativas, vemos que tudo de
que precisamos é somar e subtrair 2a2 . De fato,

a4 − 4a + 3 ≥ 0 ⇐⇒ a4 − 2a2 + 2a2 − 4a + 3 ≥ 0
⇐⇒ a4 − 2a2 + 1 + 2a2 − 4a + 2 ≥ 0
⇐⇒ (a2 − 1)2 + 2(a − 1)2 ≥ 0.

Uma soma de quadrados sempre é maior que ou igual a 0. Assim, a última desigualdade é válida
e, como as desigualdades são todas equivalentes, a primeira desigualdade se verifica também. Por (1),
o problema está resolvido.
1
Você consegue demonstrá-lo?

3
6. A melhor maneira de começar a busca por uma função que satisfaz certas condições - ou a demons-
tração de que tal função não existe - é supor que já encontramos uma função assim e descobrir que
propriedades essa função deve possuir. Suponhamos, então, que temos uma função f : R → R que é
injetiva e tal que f (x2 ) − f (x)2 ≥ 14 para todo x ∈ R.
A princı́pio, não temos nenhuma informação sobre a relação entre f (x) e f (x2 ) (a não ser a
desigualdade acima): se x e x2 são números distintos, o fato de um ser quadrado do outro não é
suficiente para tirarmos conclusões sobre as suas imagens por f . Tudo que sabemos é que f (x2 ) −
f (x)2 ≥ 14 ... Se torna tentador, então, considerarmos os casos em que x = x2 .
A igualdade x = x2 ocorre exatamente quando x = 0 ou x = 1. Consideremos o caso x = 1, por
exemplo. Devemos ter f (1) − f (1)2 ≥ 14 . Por que não manipular essa desigualdade? Temos

1
f (1) − f (1)2 ≥ ⇐⇒ 4f (1) − 4f (1)2 ≥ 1
4
⇐⇒ 0 ≥ 4f (1)2 − 4f (1) + 1
⇐⇒ 0 ≥ (2f (1) − 1)2 .

Ora, o quadrado de um número não pode ser menor que zero, e só é igual a zero se o próprio
número é igual a zero. Assim, devemos ter
1
2f (1) − 1 = 0 ⇐⇒ f (1) = .
2
Se fizermos o mesmo processo com 0 ao invés de 1, vamos encontrar o mesmo resultado2 : f (0) = 12 .
Portanto, devemos ter
1
f (0) = = f (1).
2
Mas isso é um absurdo, já que f deve ser injetiva! Dessa forma, não existe função satisfazendo as
condições descritas no enunciado.

7. Observe que

4a2 + 4a + 3 = (2a + 1)2 + 2, e b2 − 6b + 13 = (b − 3)2 + 4.

Assim,
4x2 + 4x + 3 ≥ 2, e y 2 − 6y + 13 ≥ 4.
Para que o produto das duas expressões seja exatamente 8, precisamos ter igualdade nas duas
desigualdades. Assim, precisamos ter
1
2x + 1 = 0 ⇐⇒ x = − , e y − 3 = 0 ⇐⇒ y = 3.
2
Logo, o único par de números reais que satisfaz a condição dada é (− 12 , 3).

8. Pela desigualdade das médias, r


a b a b
+ ≥2· · = 2.
b a b a
A desigualdade se verifica quando ab = ab , isto é, quando a2 = b2 ⇐⇒ a = b (veja que, como os
números são positivos, a igualdade entre os seus quadrados implica a igualdade entre eles).
2
Que tal você tentar fazer essa parte? Vamos lá, é só seguir os passos que já fizemos, só que desta vez com f (0) no
lugar de f (1)!

4
9. Basta manipular:

a2 + b2  a + b 2 a2 + b2 (a + b)2
≥ ⇐⇒ ≥
2 2 2 4
a2 + b2 a2 + b2 + 2ab
⇐⇒ ≥
2 4
⇐⇒ 2(a2 + b2 ) ≥ a2 + b2 + 2ab
⇐⇒ a2 + b2 ≥ 2ab.

Essa aı́ é exatamente a desigualdade das médias aplicada a a2 e b2 .

√ √ √
10. Pela desigualdade das médias, a + b ≥ 2 ab, b + c ≥ 2 bc e c + a ≥ 2 ca. Multiplicando as
desigualdades, obtemos (a + b)(b + c)(c + a) ≥ 8abc.

11. Se um retângulo de perı́metro p possui lados de medidas a e b, então 2a + 2b = p, e a área do


retângulo é igual a ab. Pela desigualdade das médias,
a+b √  a + b 2  p/2 2  p 2
≥ ab, isto é, ab ≤ = = ,
2 2 2 4
p 2
ou seja, a área do retângulo é menor que ou igual a 16 . Também pelo teorema da desigualdade das
médias, a igualdade pode ocorrer, e ocorre exatamente quando a = b. Portanto, dentre todos os
retângulos de perı́metro p, o quadrado (aquele em que os lados são iguais) é o de maior área - e esta
p2
área é exatamente igual a 16 .

12. Aplicando a desigualdade das médias a a1 , a2 , . . . , an , vemos que


a1 + a2 + · · · + an √ √
≥ n a1 a2 · · · an ⇐⇒ (a1 + a2 + · · · + an ) ≥ n n a1 a2 · · · an .
n
1 1 1
Fazendo o mesmo para a1 , a2 , . . . , an , temos
1 1
+ ··· +
r
a1 an n 1 1 1 1 1 n
≥ ··· ⇐⇒ + + ··· + ≥ √ .
n a1 an a1 a2 an n a1 a2 · · · an

Multiplicando essas duas desigualdades, obtemos o desejado.

13. O lado esquerdo da desigualdade pode ser reescrito como

(a + b)(a + c) = bc + a2 + ab + ac = bc + a(a + b + c).

Aplicando a desigualdade das médias a bc e a(a + b + c), vemos que


p
bc + a(a + b + c) ≥ 2 abc(a + b + c).

14. Consideremos um triângulo de perı́metro p, e chamemos as medidas dos seus lados de a, b e c e


a sua área de A. Pela definição do perı́metro, a + b + c = p. Utilizaremos a Fórmula de Heron para a
área de um triângulo: r 
p p  p  p 
A= −a −b −c .
2 2 2 2

5
Pela desigualdade das médias,
p  p  p  p −a+ p
−b+ p
− c 3  p/2 3  p 3
−a −b −c ≤ 2 2 2
= = .
2 2 2 3 3 6
Assim, r
p p3 p2
A≤ · 3 = √ .
2 6 12 3
A igualdade ocorre quando temos igualdade na desigualdade das médias. Pelo teorema da desi-
gualdade das médias, isso acontece exatamente quando p2 − a = p2 − b = p2 − c, ou seja, exatamente
quando a = b = c.
Portanto, dentre todos os triângulos de perı́metro p, o equilátero (aquele em que os lados são todos
2
iguais) é o que possui a maior área - e esta área é igual a 12p√3 .

a+1 b+1 1
15. Somamos 9 + 9 = 3 para observar que

a2 b2 1 a + 1 a2   b + 1 b2  2a 2b 2
+ + = + + + ≥ + = .
a+1 b+1 3 9 a+1 9 b+1 3 3 3
Daı́,
a2 b2 1
+ ≥ .
a+1 b+1 3

P.S.: A passagem
a + 1 a2   b + 1 b2  2a 2b
+ + + ≥ +
9 a+1 9 b+1 3 3
é consequência de duas aplicações de MA-MG, uma em cada parêntesis.

Material elaborado por Valentino Amadeus Sichinel

6
Problemas Resolvidos

Nı́vel 2

Desigualdades II

Material elaborado por Valentino Amadeus Sichinel


Problemas

1 Cauchy-Schwarz 2 Problemas legais


Problema 1. Sejam a, b e c reais positivos. Problema 8. Sejam a e b reais positivos. Mos-
Prove que a2 + b2 + c2 ≥ ab + bc + ca. tre que
3 3 4
+ ≥ .
Problema 2. Sejam a1 , . . . , an reais positivos. 2a + b 2b + a a+b
Prove que

1 1
 Problema 9. Sejam a, b e c reais positivos tais
(a1 + · · · + an ) + ··· + ≥ n2 . que a + b + c = 1. Prove que
a1 an
 1  1  1
1+ 1+ 1+ ≥ 64.
a b c
Problema 3. Dados os reais positivos a, b, c,
mostre que
Problema 10. Encontre todos os pares (x, y)
de números reais que satisfazem
p p p
3a2 + ab + 3b2 + bc + 3c2 + ca ≤
2(a + b + c). x2 + y 2 + 2 = (x − 1)(y − 1).

Problema 4. Sejam a, b, c e d reais positivos. Problema 11. Sejam a, b e c reais positivos.


Prove que Prove que
1 1 4 16 64 a b c 3
+ + + ≥ . + + ≤ .
a b c d a+b+c+d 2a + b + c a + 2b + c a + b + 2c 4

Problema 5. Sejam a, b e c reais positivos. Problema 12 (IMO). Sejam n ≥ 3


Prove que e a2 , a3 , . . . , an reais positivos tais que
a2 a3 · · · an = 1. Prove que
a b c
+ + ≤ 1.
2a + b 2b + c 2c + a (1 + a2 )2 (1 + a3 )3 · · · (1 + an )n > nn .

Problema 6. Sejam a1 , . . . , an e b1 , . . . , bn reais Problema 13. Sejam a, b e c reais positivos


positivos. Prove que tais que a + b + c = 3. Mostre que

a3 + b3 + c3 ≥ a2 + b2 + c2 .
p
(a1 + b1 )2 + · · · + (an + bn )2 ≤
q q
a21 + · · · + a2n + b21 + · · · + b2n .
Problema 14. Sejam n > 3 um in-
teiro e a1 , a2 , . . . , an reais positivos tais que
Problema 7 (Irã). Sejam a, b e c reais maiores a1 a2 · · · an = 1. Prove que
do que 1 tais que a1 + 1b + 1c = 2. Prove que
1 1
√ √ √ √ + + ···+
a + b + c ≥ a − 1 + b − 1 + c − 1. 1 + a1 + a1 a2 1 + a2 + a2 a3
1 1
+ + >1
1 + an−1 + an−1 an 1 + an + an a1

2
Soluções

1. A desigualdade é equivalente a

2(a2 + b2 + c2 ) ≥ 2(ab + bc + ca),

que por sua vez é equivalente a

3(a2 + b2 + c2 ) ≥ (a + b + c)2 .

Essa comparação entre a soma dos quadrados e o quadrado da soma é exatamente a desigualdade
de Cauchy-Schwarz aplicada às sequências (a, b, c) e (1, 1, 1).

2. Como os números são todos positivos, podemos trabalhar com as raı́zes deles. Aplicando a desi-
√ √
gualdade de Cauchy-Schwarz às sequências a1 , . . . , an e √1a1 , . . . , √1an , obtemos
  √
1 1 1 √ 1 2
(a1 + · · · + an ) + ··· + ≥ a1 · √ + · · · + an · √ = n2 .
a1 an a1 an

√ √ √ √ √ √
3. Aplicando Cauchy-Schwarz às sequências ( a, b, c) e ( 3a + b, 3b + c, 3c + a), vemos que
p p p p
3a2 + ab + 3b2 + bc + 3c2 + ca ≤ (a + b + c)(3a + b + 3b + c + 3c + a) =
p
4(a + b + c)2 =
2(a + b + c).

4. Aplicando o Lema de Titu, vemos que

1 1 4 16 12 12 22 42 (1 + 1 + 2 + 4)2 64
+ + + = + + + ≥ = ,
a b c c a b c d a+b+c+d a+b+c+d
tal como querı́amos.

5. Observe que
a 1 1 b a 1 1 b
− =− · , que é o mesmo que = − · .
2a + b 2 2 2a + b 2a + b 2 2 2a + b
Da mesma forma,
b 1 1 c c 1 1 a
= − · , e = − · .
2b + c 2 2 2b + c 2c + a 2 2 2c + a
Assim,
a b c 3 1 b c a 
+ + = − + + .
2a + b 2b + c 2c + a 2 2 2a + b 2b + c 2c + a
Logo, para mostrarmos que
a b c
+ + ≤ 1,
2a + b 2b + c 2c + a

3
é suficiente que mostremos que
1 1 b c a 
− + + ≤ 0,
2 2 2a + b 2b + c 2c + a
ou seja, que
b c a
+ + ≥ 1.
2a + b 2b + c 2c + a
Finalmente, uma desigualdade que se parece com Titu! Bom, quase, na verdade, mas podemos
resolver a falta de quadrados facilmente:

b c a b2 c2 a2
+ + = + + ≥
2a + b 2b + c 2c + a 2ab + b2 2bc + c2 2ca + a2
(a + b + c)2
=
2(ab + bc + ca) + (a2 + b2 + c2 )
(a + b + c)2
= 1.
(a + b + c)2

6. Como estamos lidando com números reais positivos, podemos elevar ambos os lados da desigualdade
ao quadrado. Vemos, com isso, que ela é equivalente a
q q
a21 + 2a1 b1 + b21 + · · · + a2n + 2an bn + b2n ≤ a21 + · · · + a2n + b21 + · · · + b2n + 2 a21 + · · · + a2n b21 + · · · + b2n .

Cancelando os termos repetidos, ficamos com


q q
2a1 b1 + · · · + 2an bn ≤ 2 a21 + · · · + a2n b21 + · · · + b2n ,

que é equivalente a q q
a1 b1 + · · · + an bn ≤ a1 + · · · + an b21 + · · · + b2n .
2 2

Esta última desigualdade se verifica porque é Cauchy-Schwarz aplicada às sequências a1 , . . . , an e


b1 , . . . , bn . Portanto, a desigualdade do enunciado da questão também se verifica.

7. É uma única aplicação de Cauchy-Schwarz, mas q


encontrarqa aplicação
q perfeita não é tão simples
√ √ √ 1 1 1

assim. Utilizaremos as sequências ( a, b, c) e 1 − a , 1 − b , 1 − c . Aplicando Cauchy-
Schwarz, ficamos com
r
√ 1 1 1 √ √ √
a + b + c · 1 − + 1 − + 1 − ≥ a − 1 + b − 1 + c − 1.
a b c
1 1 1
Como a + b + c = 2,
r
√ 1 1 1 √
a+b+c· 1− + 1 − + 1 − = a + b + c,
a b c
e o problema está resolvido.

4
8. Para começar, multipliquemos tudo por a + b. Como a + b é positivo, podemos fazer isso. Temos

3 3 4 3(a + b) 3(a + b)
+ ≥ ⇐⇒ + ≥ 4.
2a + b 2b + a a+b 2a + b 2b + a
a + b é positivo. Então, em particular, não é nulo. Assim, podemos dividir coisas por a + b. Isso
quer dizer que podemos reescrever a desigualdade acima da seguinte forma:
3 3
a b
+ b a
≥ 4.
2 a+b + a+b 2 a+b + a+b

a b
Considere agora os números a+b e a+b . Chamemo-los de x e y, respectivamente. Temos x + y =
a b a+b
a+b + a+b = a+b = 1 e queremos provar que

3 3
+ ≥ 4.
2x + y 2y + x
Observe que esta é exatamente a desigualdade do enunciado escrita com x e y no lugar de a e b -
4
quando botamos x e y, x+y vira 4, já que x + y = 1.
Como x + y = 1, nossa desigualdade é equivalente a
3 3 9 9
+ ≥ 4 ⇐⇒ + ≥ 12,
x+1 y+1 x+1 y+1
que é equivalente a
9 9
+ ≥ 16 − 4(x + y). (1)
x+1 y+1
Mas
9
≥ 8 − 4x ⇐⇒ 4x2 − 4x + 1 ≥ 0 ⇐⇒ (2x − 1)2 ≥ 0
x+1
e
9
≥ 8 − 4y ⇐⇒ 4y 2 − 4y + 1 ≥ 0 ⇐⇒ (2y − 1)2 ≥ 0.
y+1
Logo, (1), que é a soma das duas desigualdades acima, é válida.

9. Queremos mostrar que


 1  1  1
1+ 1+ 1+ ≥ 64 ⇐⇒ (a + 1)(b + 1)(c + 1) ≥ 64abc.
a b c
Como a + b + c = 1, isso é o mesmo que

(2a + b + c)(a + 2b + c)(a + b + 2c) ≥ 64abc. (2)

Pela desigualdade das médias,



4

4
2a + b + c = a + a + b + c ≥ 4 a2 bc, a + 2b + c = a + b + b + c ≥ 4 ab2 c,
√4
e a + b + 2c = a + b + c + c ≥ 4 abc2 .
Multiplicando as três desigualdades, ficamos com (2).

5
10. Temos

x2 + y 2 + 2 = (x − 1)(y − 1) ⇐⇒ x2 − xy + y 2 + 1 + x + y = 0

x2 y 2 x2 1 y2 1
⇐⇒ − xy + + +x+ + +y+ =0
2 2 2 2 2 2
1 1 1
⇐⇒ (x − y)2 + (x − 1)2 + (y − 1)2 = 0
2 2 2
⇐⇒ (x − y)2 + (x − 1)2 + (y − 1)2 = 0.

Ora, sabemos que todo quadrado de um número real é maior que ou igual a zero, e só é igual a zero
se o próprio número é igual a zero. Dessa forma, a igualdade acima ocorre se, e somente se, x = y,
x = 1 e y = 1, isto é, x = y = 1.
Portanto, o único par de números reais (x, y) que satisfazem x2 + y 2 + 2 = (x − 1)(y − 1) é (1, 1).

11. Observe que


a a+b+c b a+b+c c a+b+c
=1− , =1− , e =1− .
2a + b + c 2a + b + c a + 2b + c a + 2b + c a + b + 2c a + b + 2c
Assim, para mostrar que
a b c 3
+ + ≤ ,
2a + b + c a + 2b + c a + b + 2c 4
devemos mostrar que
 a+b+c a+b+c a+b+c  3
3− + + ≤ ,
2a + b + c a + 2b + c a + b + 2c 4
ou que
9 a+b+c a+b+c a+b+c
≤ + + .
4 2a + b + c a + 2b + c a + b + 2c
Veja agora que
a+b+c a+b+c a+b+c 1 1 1
+ + = a + b
+ c .
2a + b + c a + 2b + c a + b + 2c a+b+c +1 a+b+c +1 a+b+c +1

Dessa forma, podemos reescrever a desigualdade que queremos provar em função das variáveis
a b c
x = a+b+c , y = a+b+c e z = a+b+c : queremos mostrar que

9 1 1 1
≤ + + .
4 1+x 1+y 1+z
Essas novas variáveis têm uma vantagem sobre as antigas: temos
a b c a+b+c
x+y+z = + + = = 1.
a+b+c a+b+c a+b+c a+b+c
Isso vai nos ajudar a resolver o problema: pelo Lema de Titu,

1 1 1 12 12 12 (1 + 1 + 1)2
+ + = + + ≥ =
1+x 1+y 1+z 1+x 1+y 1+z (1 + x) + (1 + y) + (1 + z)
9 9
= .
3+x+y+z 4

6
12. Faremos uma aplicação da desigualdade das médias a cada um dos parêntesis. Temos

1 + a2 ≥ 2 a2 ⇐⇒ (1 + a2 )2 ≥ 4 · a2 ,

33
r
1 1 a3
1 + a3 = + + a3 ≥ 3 3 ⇐⇒ (1 + a3 )3 ≥ · a3 ,
2 2 4 4
44
r
1 1 1 a4
1 + a4 = + + + a4 ≥ 4 4 3 ⇐⇒ (1 + a4 )4 ≥ 3 · a4
3 3 3 3 3
.
.
.
nn
r
1 1 an n
1 + an = + ··· + + an ≥ n n ⇐⇒ (1 + a n ) ≥ · an .
n−1 n−1 (n − 1)n−1 (n − 1)n−1

Multiplicando as (n − 1) desigualdades, obtemos

(1 + a2 )2 (1 + a3 )3 · · · (1 + an )n ≥ nn · a2 a3 · · · an = nn .

Finalmente, precisamos mostrar que a desigualdade é estrita. Para tanto, observemos que, para
que tenhamos uma igualdade, precisamos ter igualdade em cada umas das (n − 1) desigualdades das
médias que consideramos acima. Como a igualdade na desigualdade das médias só ocorre quando os
elementos são todos iguais, precisarı́amos ter, então,
1 1 1
a2 = 1, a3 = , a4 = ,..., an = .
2 3 n−1
1
Se fosse esse o caso, no entanto, o produto a2 a3 · · · an seria igual a (n−1)! , e não a 1. Logo, não há
como termos igualdade em todas as desigualdades das médias e, portanto, a desigualdade que obtemos
quando multiplicamos elas é, de fato, estrita.

13. Queremos mostrar que


(a3 − a2 ) + (b3 − b2 ) + (c3 − c2 ) ≥ 0.
Afirmo que x3 − x2 ≥ x − 1 para todo número real positivo x. De fato, essa desigualdade é
equivalente a

x2 (x − 1) ≥ x − 1 ⇐⇒ (x − 1)(x2 − 1) ≥ 0 ⇐⇒ (x − 1)2 (x + 1) ≥ 0.

Sendo o quadrado de um número real, (x − 1)2 é maior que ou igual a zero. Sendo x positivo,
(x + 1) também é maior que ou igual a zero. Logo, a desigualdade é válida. Aplicando-a a a, b e c e
somando, obtemos

(a3 − a2 ) + (b3 − b2 ) + (c3 − c2 ) ≥ (a − 1) + (b − 1) + (c − 1) = a + b + c − 3 = 0.

Exatamente o que querı́amos.

14. Como a1 a2 · · · an = 1, existem reais positivos x1 , x2 , . . . , xn tais que


x2 x3 xn x1
a1 = , a2 = , ... , an−1 = , e an = .
x1 x2 xn−1 xn
De fato, podemos considerar, por exemplo,

x1 = 1, x 2 = a1 , x3 = a2 a1 , ... , xn = a1 a2 · · · an−1 .

7
Assim, podemos reescrever a expressão do lado esquerdo da desigualdade como

1 1 1 1
+ + ··· + + =
1 + a1 + a1 a2 1 + a2 + a2 a3 1 + an−1 + an−1 an 1 + an + an a1
1 1 1 1
x2 x3 + x3 x4 + ··· + xn x1 + x1 x2 =
1+ x1 + x1 1+ x2 + x2 1+ xn−1 + xn−1 1+ xn + xn
x1 x2 xn−1 xn
+ + ··· + + .
x1 + x2 + x3 x2 + x3 + x4 xn−1 + xn + x1 xn + x1 + x2

Como n > 3 e estamos lidando com reais positivos, cada um dos denominadores da última expressão
é estritamente menor que x1 + x2 + · · · + xn . Assim,
x1 x2 xn−1 xn
+ + ··· + + >
x1 + x2 + x3 x2 + x3 + x4 xn−1 + xn + x1 xn + x1 + x2
x1 x2 xn
+ + · · · + xn−1 x1 + · · · + xn + = 1.
x1 + · · · + xn x1 + · · · + xn x1 + · · · + xn

Material elaborado por Valentino Amadeus Sichinel

8
POLOS OLÍMPICOS DE
TREINAMENTO INTENSIVO

geometria
nível 2
Polos Olı́mpicos de Treinamento Intensivo
Curso de Geometria Plana − Nı́vel 2

Professores: Cleber Assis, Samuel Barbosa e Tiago Miranda


Aula 0
POTI 2015
Curso Básico
. . . . . . . . . . . . . . . . . . . . . . . . . . . . . . . . . . . . . Geometria Plana . . . . . . . . . . . . . . . . . . . . . . . . . . . . . . . . . . . . .

Este material compila os arquivos do projeto Portal da Matemática, disponı́vel em

http://matematica.obmep.org.br/

e serve como introdução aos tópicos iniciais de um curso de treinamento olı́mpico. Em geral, os assuntos
são independentes e podem ser estudados em qualquer ordem. Neles, o leitor encontrará muitos exercı́cios
escolares mesclados com problemas elementares de olimpı́adas, todos com respostas e soluções. Além disso,
no endereço do Portal da Matemática, existem vı́deos que podem ser acessados gratuitamente cobrindo todo
o conteúdo abaixo. Bons estudos! ...

Sumário
1 Conceitos Geométricos Básicos . . . . . . . . . . . . . . . . . . . . . . . . . . . . . . . . . . . . . 1
2 Ângulos . . . . . . . . . . . . . . . . . . . . . . . . . . . . . . . . . . . . . . . . . . . . . . . . . . . . 3
3 Condição de Alinhamentos de Três Pontos e a Desigualdade Triangular . . . . . . . . . . . . . 10
4 Segmentos Comensuráveis e Incomensuráveis. . . . . . . . . . . . . . . . . . . . . . . . . . . . . 12
5 Teorema de Tales . . . . . . . . . . . . . . . . . . . . . . . . . . . . . . . . . . . . . . . . . . . . . . 13
6 Área de Figuras Planas: Resultados Básicos . . . . . . . . . . . . . . . . . . . . . . . . . . . . . . 16
7 Áreas de Figuras Planas: Mais Alguns Resultados . . . . . . . . . . . . . . . . . . . . . . . . . . 22
8 Triângulos . . . . . . . . . . . . . . . . . . . . . . . . . . . . . . . . . . . . . . . . . . . . . . . . . . 27
9 Relações Métricas no Triângulo Retângulo . . . . . . . . . . . . . . . . . . . . . . . . . . . . . . . 36
10 Semelhanças entre Figuras e Polı́gonos . . . . . . . . . . . . . . . . . . . . . . . . . . . . . . . . . 40
Conceitos Básicos − Soluções . . . . . . . . . . . . . . . . . . . . . . . . . . . . . . . . . . . . . . . 44
Ângulos − Soluções . . . . . . . . . . . . . . . . . . . . . . . . . . . . . . . . . . . . . . . . . . . . 49
Condição de Alinhamentos de Três Pontos e a Desigualdade Triangular − Soluções . . . . . . . 52
Segmentos Comensuráveis e Incomensuráveis − Soluções . . . . . . . . . . . . . . . . . . . . . . 55
Teorema de Tales − Soluções . . . . . . . . . . . . . . . . . . . . . . . . . . . . . . . . . . . . . . . 57
Área de Figuras Planas: Resultados Básicos − Soluções . . . . . . . . . . . . . . . . . . . . . . . . 59
Áreas de Figuras Planas: Mais Alguns Resultados − Soluções . . . . . . . . . . . . . . . . . . . . 65
Triângulos − Soluções . . . . . . . . . . . . . . . . . . . . . . . . . . . . . . . . . . . . . . . . . . . 70
Relações Métricas no Triângulo Retângulo − Soluções . . . . . . . . . . . . . . . . . . . . . . . . 75
Semelhanças entre Figuras e Polı́gonos − Soluções . . . . . . . . . . . . . . . . . . . . . . . . . . 79

Versão: 798 (Data: 27 de abril de 2015.)


POTI 2015 − Geo Plana − Nı́vel 2 − Aula 0 − Professores Cleber Assis, Samuel Barbosa e Tiago Miranda

1 Conceitos Geométricos Básicos


Problema 1. Dados quatro pontos distintos A, B, C e D, todos sobre uma mesma reta como indica a figura
abaixo, determine o número de segmentos distintos que podem ser formados com vértices em tais pontos.

Problema 2. Usando o compasso, determine na figura abaixo quais segmentos são congruentes.

Problema 3. Determine o único item verdadeiro.

a) Se dois segmentos são consecutivos, então eles são d) Se dois segmentos são colineares, então eles são
colineares. consecutivos.
b) Se dois segmentos são adjacentes, então eles são e) Dois segmentos consecutivos e congruentes sempre
consecutivos. são colineares.
c) Se dois segmentos são congruentes, então eles são
colineares.

Problema 4. Sabendo que o segmento AB mede 20cm, determine o comprimento do segmento AC nos
seguintes casos:

a) Quando CB = 8cm. b) Quando AC − CB = 1cm. c) Se AC = 2x e CB = x − 1.

Problema 5. Abaixo estão representados cinco pontos distintos sobre uma mesma reta. Quantas semirretas
possuem origem em algum desses cinco pontos e não contêm o vértice B?

Problema 6. Seja M o ponto médio de AB. Se AM = 2x − 5 e MB = x + 7, encontre o valor de x.

Problema 7. Os pontos A, B e P são distintos e estão sobre uma mesma reta com A situado à esquerda de B.
Se PA > AB e PB < AB, o que podemos dizer sobre a ordem dos três pontos na reta?
Problema 8. Existem quatro pontos consecutivos A, B, C e D sobre uma reta. Se AD = 2BC e AB + CD = 20,
determine o valor de AD.
Problema 9. Seja M o ponto médio de AB. Se AM = 7x − 1 e MB = x + 11, encontre o valor de x.

1
POTI 2015 − Geo Plana − Nı́vel 2 − Aula 0 − Professores Cleber Assis, Samuel Barbosa e Tiago Miranda

Problema 10. No desenho abaixo, M é o ponto médio de AB. Se AM = x, BC = x − 1 e AC = 4x − 9,


determine o comprimento de AB

Problema 11. Os pontos A, B e C são colineares com AB = 30cm e BC = 10cm. Determine os possı́veis
valores de AC.
Problema 12. Dados quatro pontos consecutivos A, B, C e D sobre uma mesma reta tais que AB · BD =
AC · CD. Se AB = 9cm, encontre o valor de CD.
Problema 13. No desenho abaixo, M é o ponto médio do segmento AB. Se DB − DA = 10cm, determine o
comprimento de DM.

Problema 14. No desenho abaixo, C é o ponto médio de AB e E é o ponto médio de CD. Sabendo que
AB + ED − AC = 30cm, determine o comprimento de AE.

Problema 15. Em uma reta se encontram os quatro pontos consecutivos A, B, C e D com AB = AC − 3,


AB + CD = 4 e que satisfazem a seguinte relação 3AB − BD − 2CD = 3. Determine o valor de AD.
Problema 16. Os pontos A, B, C e D estão sobre uma mesma reta e são consecutivos. Sabendo que BC = CD
e que AC · BC = 40, determine o valor de AD2 − AB2 .
Problema 17. Sejam M e N os pontos médios, respectivamente, dos segmentos AB e BC, contidos numa
mesma reta de modo que AB = BC, com A 6= C. É sempre verdade que MN é congruente a AB? Justifique.
Problema 18. João deseja construir um circuito para o seu trem de brinquedo usando trilhos no formato de
segmentos de reta de comprimento fixo. Na interseção de dois trilhos, ele precisa colocar uma peça para que
o trem mude sua direção. É possı́vel João construir um circuito fechado com exatamente 10 peças de mudança
e de forma que cada trilho possua exatamente 4 tais peças?
Problema 19. a) São dados 3 pontos escolhidos sobre a reta surpote de AB, todos fora do segmento de reta
AB. É possı́vel que a soma das distâncias desses pontos ao vértice A seja igual à soma das distâncias desses
pontos ao vértice B?

b) Se fossem 1001 pontos ao invés de três, seria possı́vel que a soma das distâncias desses pontos ao vértice A
fosse igual à soma das distâncias desses pontos ao vértice B?
Problema 20. Em um tabuleiro 5 × 5, João deve desenhar segmentos de reta ligando vértices opostos dos
quadrados 1 × 1 de modo que quaisquer dois segmentos desenhados não possuam pontos em comum
(incluindo seus vértices). Qual o número máximo de tais segmentos que podem ser desenhados por João?
Problema 21. a) Em quantas partes distintas três retas dividem um plano se não existem duas delas paralelas
e também não existem três coincidentes?

b) Em quantas partes distintas cinco retas dividem um plano se não existem duas delas paralelas e também
não existem três coincidentes?
Você conseguiria estipular uma fórmula geral para o mesmo problema envolvendo n retas?

2
POTI 2015 − Geo Plana − Nı́vel 2 − Aula 0 − Professores Cleber Assis, Samuel Barbosa e Tiago Miranda

2 Ângulos
Problema 22. No desenho abaixo, OC é bissetriz do ângulo ∠ AOB. Se ∠ AOC = 2x − 5◦ e ∠COB = x + 3◦ ,
quanto vale x?

Problema 23. No desenho abaixo, A, O e B são colinares e OD é bissetriz do ângulo ∠ BOC. Além disso,
∠ BOD = x + 10◦ , ∠ DOC = y + 5◦ , ∠COA = 3y. Determine os valores de x e y.

Problema 24. No desenho abaixo, OE e OD são bissetrizes dos ângulos ∠ BOC e ∠COA, respectivamente.
Se o ângulo ∠ AOB mede 70◦ , determine a medida do ângulo ∠ DOE

Problema 25. Classifique como verdadeiro (V) ou falso (F):

a) Dois ângulos consecutivos são adjacentes. d) Dois ângulos adjacentes são sempre consecutivos.
b) Dois ângulos opostos pelo vértice são adjacentes. e) Dois ângulos opostos pelo vértice não são consecuti-
c) Dois ângulos suplementares são adjacentes. vos.

3
POTI 2015 − Geo Plana − Nı́vel 2 − Aula 0 − Professores Cleber Assis, Samuel Barbosa e Tiago Miranda

Problema 26. Na figura abaixo, temos ∠ BOC = 3x + 5◦ e ∠ AOC = 2x − 5◦ . Sabendo que A, O e B são
colineares, determine o valor do ângulo x.

Problema 27. Na figura abaixo, ∠ AOC = 2∠ BOC. Se ∠ AOB = 60◦ , determine o valor do ângulo formado
entre a bissetriz OD de ∠ BOC e a semirreta OA.

Problema 28. A soma de dois ângulos é 140◦ . Um deles é o quádruplo do outro subtraı́do de 40◦ . Determine
os dois ângulos.
Problema 29. Duas retas se encontram em O como indica a figura abaixo. Se ∠ AOD = 2x + 10◦ e ∠COB =
50◦ , determine o valor de x.

Problema 30. No desenho abaixo, ∠ AOD = 55◦ . Determine o valor do ângulo ∠EOF.

Problema 31. Um ângulo reto foi dividido em três ângulos adjacentes cujas medidas são proporcionais aos
números 2, 3 e 4. Determine os valores desses ângulos.
Problema 32. Os ângulos x e y são complementares e x − y = 10◦ . Qual o valor de x?

4
POTI 2015 − Geo Plana − Nı́vel 2 − Aula 0 − Professores Cleber Assis, Samuel Barbosa e Tiago Miranda

Problema 33. Na figura abaixo, ∠ AOD = 3x + 10◦ e ∠COB = 2x + 20◦ . Determine o ângulo ∠ AOC,

Problema 34. Determine a medida do ângulo formado pelas bissetrizes de dois ângulos adjacentes que
somam 150◦ .
Problema 35. No desenho abaixo, ∠ AOB = ∠COD = ∠EOF = x. Determine o valor de x.

Problema 36. Duas retas são concorrentes em um ponto O. Quantos ângulos distintos ficam determinados
por elas no plano que as contém?
Problema 37. No desenho abaixo, os segmentos AB e CD determinam quatro ângulos. Determine os valores
de x, y e z em cada um dos casos abaixo:

a) ∠COB = 80◦ , ∠ DOB = x + y, ∠CAO = y + z e


∠ DAO = x + z.
b) ∠COB = x + 40◦ , ∠ DOB = 3x + 20◦ e ∠ AOC = z.

Problema 38. Simplifique as seguintes medidas como no modelo:


1◦ 580 23700 = 1◦ 580 5700 + 18000
= 1◦ 610 5700
= 1◦ 010 5700 + 600
= 2◦ 010 5700 .
a) 35◦ 1500 .
b) 50◦ 1300 .
c) 75◦ 200 13700 .
d) 58◦ 580 26000 .

5
POTI 2015 − Geo Plana − Nı́vel 2 − Aula 0 − Professores Cleber Assis, Samuel Barbosa e Tiago Miranda

Problema 39. Nos desenhos abaixo, as retas a e b são paralelas. Determine os valores de x e y.

a) . b) .

Problema 40. No desenho abaixo, os segmentos QR e ST são paralelos. Determine os valores dos ângulos x,
y e z.

Problema 41. No desenho abaixo, os segmentos AB e CD são paralelos. Determine a medida do ângulo x.

Problema 42. No desenho abaixo, CD e AB são segmentos paralelos. Se ∠ AEB = 105◦ , determine a medida
do ângulo x.

6
POTI 2015 − Geo Plana − Nı́vel 2 − Aula 0 − Professores Cleber Assis, Samuel Barbosa e Tiago Miranda

Problema 43. Na figura abaixo, JK, CD e AB são segmentos paralelos. Se x + y = 150◦ , determine o valor
do ângulo z.

Problema 44. No desenho abaixo, AB e CD são paralelos.


a) Determine o valor do ângulo x. b) Determine o valor do ângulo ∠EFB.

Problema 45. Efetue as operações indicadas:

a) 90◦ − 55◦ 370 . b) 3 × (7◦ 130 2300 ). c) (46◦ 380 2800 ) ÷ 2. d) 87◦ 270 1200 + 5◦ 340 4800 .

Problema 46. Qual o ângulo formado entre as bissetrizes de dois ângulos adjacentes e suplementares?
Problema 47. A diferença entre dois ângulos adjacentes mas não consecutivos é 100◦ . Determine o ângulo
formado por suas bissetrizes.
Problema 48. No desenho abaixo, DA é bissetriz do ângulo ∠CAB. Determine o valor do ângulo ∠ DAE
sabendo que ∠CAB + ∠EAB = 120◦ e ∠CAB − ∠EAB = 80◦ .

7
POTI 2015 − Geo Plana − Nı́vel 2 − Aula 0 − Professores Cleber Assis, Samuel Barbosa e Tiago Miranda

Problema 49. Os ângulos x e y são tais que sua diferença é 20◦ . Encontre x sabendo que seu complementar
somado com o suplementar de 2x é o dobro do complemento de y.
Problema 50. Encontre algum ângulo x tal que o seu quadrado excede em 50◦ o quı́ntuplo do seu comple-
mento.
1
Problema 51. A soma dos complementos de x e y é igual da soma de seus suplementares. Se um deles é
10
o quádruplo do outro, determine o menor deles.
Problema 52. A que horas pela primeira vez após o meio-dia, os ponteiros de um relógio formam 110◦ ?
a) 12h18’ b)12h20’ c)13h22’ d)13h23’ e) 15h
Problema 53. Dois ângulos suplementares medem 3x − 40◦ e 2x + 60◦ . Qual o valor do maior desses
ângulos?
a) 56◦ b) 108◦ c) 124◦ d) 132◦ e) 137◦
Problema 54. Efetuando 55◦ 150 3700 − 20◦ 420 3000 , temos:
a) 34◦ 280 700 b )34◦ 330 700 c) 33◦ 280 700 d) 33◦ 330 700 e) 35◦ 280 700
Problema 55. Nas figuras abaixo temos que AB é paralelo a CD.
a) Determine a medida do ângulo x. b) Sendo GH k EF, determine a medida do ângulo β.

Problema 56. Sabendo que CD e AK são paralelos, determine o valor de x.

8
POTI 2015 − Geo Plana − Nı́vel 2 − Aula 0 − Professores Cleber Assis, Samuel Barbosa e Tiago Miranda

Problema 57. Sabendo que CD é paralelo a AB, determine o ângulo x.

Problema 58. Três quadrados são colados pelos seus vértices entre si e a dois bastões verticais, como mostra
a figura. Determine a medida do ângulo x.

Problema 59. No desenho abaixo, mostre que a soma dos ângulos ângulos brancos é igual à soma dos
ângulos cinzas. Tal resultado vale para qualquer quantidade de “bicos” no desenho e o chamamos popular-
mente como Teorema dos Bicos.

9
POTI 2015 − Geo Plana − Nı́vel 2 − Aula 0 − Professores Cleber Assis, Samuel Barbosa e Tiago Miranda

3 Condição de Alinhamentos de Três Pontos e a Desigualdade Triangular


Problema 60. Em cada um dos itens abaixo, determine o número de pontos de interseção dos cı́rculos de
raios r A e r B centrados nos pontos A e B, respectivamente.

a) AB = 5cm, r A = 3cm e r B = 2cm. c) AB = 5cm, r A = 3cm e r B = 4cm.


b) AB = 5cm, r A = 2cm e r B = 2cm.

Problema 61. A desigualdade triangular afirma que qualquer lado de um triângulo é sempre menor que a
soma dos outros dois. É possı́vel demonstrar a partir desta propriedade que se o maior dentre três segmentos
é menor que a soma dos outros dois então existe um triângulo formado por tais segmentos. Nos itens abaixo,
decida se existe um triângulo com as medidas dadas. Justifique sua resposta.

a) 4cm, 5cm e 6cm. c) 4cm, 4cm e 8cm. e) 6cm, 6cm e 6cm.


b) 7cm, 3cm e 3cm. d) 3cm, 3cm e 4cm.

Comentário: Decorre da desigualdade triangular que um lado de um triângulo sempre deve ser maior que
o valor absoluto da diferença dos outros dois lados. Para ver isso, considere as seguintes desigualdades
envolvendo os lados de comprimentos a, b e c de um triângulo qualquer:

a+b > c ⇒ a > c−b


a + c > b ⇒ a > b − c.

Como a deve ser maior que qualquer uma das diferenças possı́veis dos outros dois lados, temos a > |b − c|.
Problema 62. Dois lados de um triângulo medem 3cm e 4cm. Quais as possı́veis medidas do terceiro lado?
Problema 63. O maior lado de um triângulo mede 5cm e o menor 2cm. Quais as possı́veis medidas do
terceiro lado?
Problema 64. Um triângulo isósceles possui base de comprimento 4cm. Quais as possı́veis medidas dos
lados iguais?
Problema 65. Usando uma régua milimetrada e compasso, construa um triângulo de lados 4, 6 e 7 .
Problema 66. Nos itens abaixo, decida se existe um triângulo com as medidas dadas. Justifique sua resposta.

a) 10cm, 15cm e 25cm.

b) 31cm, 33cm e 30cm.

c) 40cm, 40cm e 45cm.

Problema 67. Um triângulo possui dois lados de medidas 10cm e 17cm. Determine os possı́veis valores do
terceiro lado sabendo que ele é o quadrado de um inteiro.
Problema 68. Dois lados de um triângulo medem 7cm e 13cm. Determine os possı́veis valores do outro lado
sabendo que ele é divisı́vel por 5.
Problema 69. O lado AC do triângulo ABC tem comprimento 3, 8cm e o lado AB tem comprimento 0, 6cm.
Se o comprimento do lado BC é um inteiro, qual é o seu comprimento?
Problema 70. Prove que se é possı́vel construirmos um triângulo com lados a, b e c, também é possı́vel
1 1 1
construirmos um triângulo com lados de comprimentos , e .
a+b a+c b+c

10
POTI 2015 − Geo Plana − Nı́vel 2 − Aula 0 − Professores Cleber Assis, Samuel Barbosa e Tiago Miranda

Problema 71. Seja P um ponto interno ao triângulo 4 ABC, verifique que AB + AC > BP + PC.

Problema 72. Mostre que o perı́metro do triângulo 4 DEF da figura abaixo é menor que o perı́metro do
triângulo 4 ABC.

Problema 73. Na figura ao lado, verifique que:


AB + BC + CD + DA
a) < BD + AC.
2
b) BD + AC < AB + BC + CD + DA.

Problema 74. (Desafio) No quadrado ABCD, sejam P e Q pontos pertencentes aos lados BC e CD respecti-
vamente, distintos dos extremos, tais que BP = CQ. Consideram-se pontos X e Y, X 6= Y, pertencentes aos
segmentos AP e AQ respectivamente. Demonstre que, quaisquer que sejam X e Y, existe um triângulo cujos
lados têm os comprimentos dos segmentos BX, XY e DY.

Problema 75. Prove que a distância entre quaisquer dois pontos dentro de um triângulo não é maior que
que metade do perı́metro do triangulo.

11
POTI 2015 − Geo Plana − Nı́vel 2 − Aula 0 − Professores Cleber Assis, Samuel Barbosa e Tiago Miranda

4 Segmentos Comensuráveis e Incomensuráveis.


Problema 76. Determine a razão entre os segmentos AB e CD, sendo

a) AB = 4 e CD = 2.

b) AB = 7 e CD = 3.

c) AB = 1/2 e CD = 1/3.
√ √
d) AB = 3 2 e CD = 2.

e) AB = 5 e CD = 2.

f) AB = 2 e CD = 2.

g) ABCD um paralelogramo.

Problema 77. No exercı́cio anterior, determine em quais ı́tens os segmentos AB e CD são comensurávies.
Problema 78. A razão entre as medidas dos segmentos AB e CD é 7/4. Se AB = 28cm, determine CD.
Problema 79. Sejam AB e CD dois segmentos, onde a razão entre suas medidas é 1/2. Se AB = 8cm é o
segmento de maior medida, determine CD.
Problema 80.√ A razão entre as medidas da diagonal e do lado de um quadrado, qualquer que seja o
quadrado, é 2, ou seja, são segmentos incomensuráveis. Detemine o lado de um quadrado cuja diagonal
mede 8cm.
Problema 81.√ Sabe-se que razão entre a medida da altura e a medida do lado de um triângulo equilátero
3
qualquer é . Determine o comprimento do lado de um triângulo equilátero de altura medindo 6cm.
2
Problema 82. A razão entre o comprimento de uma circunferência e a medida do seu diâmetro é um
número irracional, representado pela letra grega π. Determine a medida do raio de uma circunferência, cujo
comprimento é 8πcm.
Problema 83. Se os três lados de um triângulo ABC são comensuráveis dois a dois, mostre que um segmento
EF, cuja medida é igual à medida do perı́metro do triângulo ABC, e qualquer um dos lados deste triângulo
são comensuráveis.
Problema 84. Mostre que a diagonal de um quadrado e seu lado são segmentos incomensuráveis.

12
POTI 2015 − Geo Plana − Nı́vel 2 − Aula 0 − Professores Cleber Assis, Samuel Barbosa e Tiago Miranda

5 Teorema de Tales
Problema 85. Determine x nas figuras abaixo, com r//s//t, sabendo que:

a) c)

Figura 1 Figura 3

b) d)

Figura 2
Figura 4

Problema 86. Determine o valor de x na figura abaixo, sabendo que DE é paralelo à base BC do 4 ABC.

Figura 5

13
POTI 2015 − Geo Plana − Nı́vel 2 − Aula 0 − Professores Cleber Assis, Samuel Barbosa e Tiago Miranda

Problema 87. Determine o valor de x na figura abaixo, sabendo que AD é bissetriz do 4 ABC.

Figura 6

Problema 88. Determine o valor de x na figura abaixo, sabendo que AD é bissetriz externa do 4 ABC.

Figura 7

Problema 89. No 4 ABC abaixo, determine x, sabendo que seu perı́metro mede 75cm e que AS é bissetriz.

Figura 8

Problema 90. Na figura abaixo, determine as medidas de x e y, sabendo que AR é bissetriz do 4 ABC e
BC = 15.

Figura 9

14
POTI 2015 − Geo Plana − Nı́vel 2 − Aula 0 − Professores Cleber Assis, Samuel Barbosa e Tiago Miranda

Problema 91. Sabendo que BC//DE na figura abaixo, determine a medida do perı́metro do 4 ABC.

Figura 10

Problema 92. Seja um triângulo 4 ABC, no qual AB = 10, AC = 12 e BC = 14. A bissetriz interna que
passa por B, intercepta AC em K. A bissetriz interna que passa por C, intercepta BK em J. Determine se os
segmentos BJ e JK são comensuráveis.
Problema 93. O 4 ABC é retângulo em A. Se sua hipotenusa mede 15cm e um dos catetos é 3cm maior que
outro, sendo que uma das bissetrizes internas intercepta o maior cateto (AC) no ponto D, determine a medida
do segmento BD.

15
POTI 2015 − Geo Plana − Nı́vel 2 − Aula 0 − Professores Cleber Assis, Samuel Barbosa e Tiago Miranda

6 Área de Figuras Planas: Resultados Básicos


Problema 94. Determine a área dos retângulos abaixo:

a) b)

Problema 95. Determine a área de um quadrado

a) cujo lado mede 8cm.

b) cujo lado mede 7, 1cm.



c) cujo lado mede 3cm.

d) cuja diagonal mede 6cm.

Problema 96. Determine a medida do lado de um quadrado cuja área é

a) 25cm2 .

b) 12cm2 .

Problema 97. Determine a área de um losango

a) cujas diagonais medem 5cm e 8cm.

b) cujo lado mede 5cm e a diagonal menor mede 6cm.

c) cujo lado mede 8cm e um dos ângulos internos mede 120 o .

Problema 98. Determine a área de um trapézio de bases medindo 5cm e 7cm e altura medindo 4cm.
Problema 99. Determine a área de um quadrado cujo perı́metro é 72cm.
Problema 100. Determine a área de um trapézio isósceles cujos bases têm 6cm e 12cm de medida e os outros
lados, 5cm.
Problema 101. Calcule a área dos paralelogramos abaixo

a) b)

16
POTI 2015 − Geo Plana − Nı́vel 2 − Aula 0 − Professores Cleber Assis, Samuel Barbosa e Tiago Miranda

Problema 102. Calcule a área dos triângulos abaixo.

Problema 103. A altura de um retângulo é a metade de sua base. Se sua área é 450m2 , determine suas
dimensões.
Problema 104. Aumentando em 10% o comprimento de um retângulo e diminuindo em 10% sua largura,
determine sua nova área, sabendo que a área inicial era 100cm2 .
Problema 105. Determine a área hachurada nas figuras abaixo.

a) b) c)

Problema 106. A cerâmica constitui-se em um artefato bastante presente na história da humanidade. Uma
de suas várias propriedades é a retração (contração), que consiste na evaporação da água existente em um
conjunto ou bloco cerâmico quando submetido a uma determinada temperatura elevada. Essa elevação de
temperatura, que ocorre durante o processo de cozimento, causa uma redução de até 20% nas dimensões
lineares de uma peça. (Disponı́vel em: www.arq.ufsc.br. Acesso em: 3 mar 2012).
Suponha que uma peça, quando moldada em argila, possuı́a uma base retangular cujos lados mediam 30cm e
15cm. Após o cozimento, esses lados foram reduzidos em 20%. Em relação à área original, a área da base
dessa peça, após o cozimento, ficou reduzida em
a) 4% b) 20% c) 36% d) 64% e) 96%.
Problema 107. Determine a área hachurada nas figuras abaixo.

a) b) . c)

17
POTI 2015 − Geo Plana − Nı́vel 2 − Aula 0 − Professores Cleber Assis, Samuel Barbosa e Tiago Miranda

Problema 108. Um forro retangular de tecido traz em sua etiqueta a informação de que encolherá após a
primeira lavagem mantendo, entretanto, seu formato. A figura a seguir mostra as medidas originais do forro e
o tamanho do encolhimento x no comprimento e y na largura. A expressão algébrica que representa a área do
forro após ser lavado é (5–x )(3–y).

Nestas condições, a área perdida do forro, após a primeira lavagem, será expressa por
a) 2x b) 15 − 3x c) 15 − 5x d) −5y − 3x e) 5y + 3x − xy.
Problema 109. Para decorar a fachada de um edifı́cio, um arquiteto projetou a colocação de vitrais compostos
de quadrados de lado medindo 1m, conforme a figura a seguir.

Nesta figura, os pontos A, B, C e D são pontos médios dos lados do quadrado de área 1m e os segmentos AP
e QC medem 1/4. Para confeccionar um vitral, são usados dois tipos de materiais: um para a parte sombreada
da figura, que custa R$30, 00 o m2 e outro para a parte mais clara (regiões ABPDA e BCDQB), que custa
R$50, 00 o m2 . De acordo com esses dados, qual é o custo dos materiais usados na fabricação de um vitral?
a) R$22, 50 b) R$35, 00 c) R$40, 00 d) R$42, 50 e) R$45, 00.
Problema 110. Considere um quadrado ABCD de lado 1. Externamente ao quadrado, são formados os
triângulos equiláteros
√ ABE,√BCF, CDG e DAH. Qual a área do quadrilátero EFGH?
a) 2 b) 2 3 c) 2 + 3 d) 3 e) 6.
Problema 111. O quadrado ABCD da figura abaixo está dividido em 16 quadradinhos iguais. O quadrado
sombreado tem os vértices sobre os pontos médios do quadrado EFGH.

a) A área do quadrado EFGH corresponde a que


fração da área do quadrado ABCD?

b) Se o quadrado ABCD tem 80cm2 de área, qual é o


lado do quadrado sombreado?

18
POTI 2015 − Geo Plana − Nı́vel 2 − Aula 0 − Professores Cleber Assis, Samuel Barbosa e Tiago Miranda

Problema 112. Um prefeito quer construir uma praça quadrada de 10m de lado, que terá canteiros triangu-
lares iguais de pedra e um canteiro quadrado de grama, como na figura. O prefeito ainda não decidiu qual
será a área do canteiro de grama, por isso o comprimento deste segmento AB está indicado por x na figura.

a) Calcule a área do canteiro de grama para x = 2.

b) Escreva a expressão da área do canteiro de grama


em função de x.

c) Sabe-se que o canteiro de grama custa R$4, 00 por


metro quadrado e os canteiros de pedra custam
R$3, 00 por metro quadrado. Qual a menor quan-
tia que o prefeito deve ter para construir os cinco
canteiros?

Problema 113. O retângulo da figura foi repartido por meio de três segmentos em várias regiões, algumas
retangulares e outras triangulares. A linha não paralela aos lados é uma diagonal e os números indicam as
áreas em m2 das regiões brancas em que se encontram. Qual é a do retângulo original?

a) 60cm2
b) 80cm2
c) 90cm2
d) 100cm2
e) Impossı́vel saber.

Problema 114.

a) Temos abaixo um trapézio e suas diagonais. Mos- b) Na figura a seguir, BCFE é um retângulo, o
tre que a área do triângulo ABC é igual à área do triângulo ABC tem área 5cm2 e o triângulo DEF
triângulo ADE. tem átea 4cm2 . Calcule a área do quadrilátero
AGDH.

19
POTI 2015 − Geo Plana − Nı́vel 2 − Aula 0 − Professores Cleber Assis, Samuel Barbosa e Tiago Miranda

Problema 115. João e Maria herdaram um terreno, representado pelo polı́gono ABCDEF. Havia uma cerca
reta separando o terreno em duas partes, mas como as áreas eram diferentes, João e Maria resolveram
deslocá-la, mantendo-a reta, de forma que a extremidade em F fosse para o ponto P. Com isso, as duas áreas
tornaram-se iguais. Supondo que os ângulos em A, B, D, E e F são retos, dequantos metros foi o deslocamento
FP?

a) 5
b) 8
c) 10
d) 12
e) 20.

Problema 116. Seja ABCD um retângulo tal que AD = 6 e DC = 8. Construa um triângulo equilátero CED
tal que E, A e B estão no mesmo semi-plano determinado pela reta CD. Determine a área do triângulo AEC.
Problema 117. Considere o triângulo ABC inscrito em uma circunferência em que os menores arcos AB, BC
e AC são congruentes.

Se a circunferência menor, inscrita ao triângulo ABC,


tem raio igual a 1cm, então o número que representa
a área hachurada, em cm2 , é igual ao número que
representa

a) o comprimento do cı́rculo menor, em cm.

b) a área do cı́rculo maior em cm2 .

c) o comprimento do cı́rculo maior, em cm.

d) o dobro da área do triângulo ABC, em cm2 .

Problema 118. Na figura abaixo, ABCDE é um pentágono regular de lado a e os arcos AB, BC, CD, DE e
EA são congruentes e arcos de circunferência cujo raio mede a.

Assim, determine a área hachurada nessa figura, em função de ”a”.

20
POTI 2015 − Geo Plana − Nı́vel 2 − Aula 0 − Professores Cleber Assis, Samuel Barbosa e Tiago Miranda

Problema 119. Na figura abaixo, ABCD é um quadrado de lado 12 e BE é um segmento de comprimento 9.


Determine o comprimento do segmento AF.

Problema 120. Dado o quadrado ABCD de lado 2. Sejam O o centro do quadrado e E e F os pontos
médios dos lados AB e CD. Se os segmentos FH e GE são iguais e os arcos FE, EH, GO, OG, FG são
semicircunferências, encontre a área sombreada.

Problema 121. Na figura a seguir, ABCD é um quadrado de lado 4, K pertence ao lado AD, L pertence ao
lado AB, M pertence ao lado BC e KLM é um triângulo retângulo isósceles, sendo L o ângulo reto. Então a
área do quadrilátero CDKM é igual a

a) 6 b) 8 c) 10 d) 12 e) 14

21
POTI 2015 − Geo Plana − Nı́vel 2 − Aula 0 − Professores Cleber Assis, Samuel Barbosa e Tiago Miranda

7 Áreas de Figuras Planas: Mais Alguns Resultados


Problema 122. No desenho abaixo, as retas r e s são paralelas. Se o segmento I J é o dobro do segmento EG,
determine a razão entre as áreas dos triângulos 4 FEG e 4 H I J.

Problema 123. A fórmula de Heron afirma que a área de um triângulo de lados a, b e c é dada por
p a+b+c
p( p − a)( p − b)( p − c), onde p = . Calcule a área dos triângulos abaixo.
2
a) c)

d)
b)

Problema 124. Calcule a área de um triângulo cujos lados medem 13cm, 14cm e 15cm.
Problema 125. No triângulo ABC, AC = 5 e AB = 6. Seja P um ponto sobre a bissetriz interna do ângulo
∠ BAC. Se a área de APB é 3/2, a área de APC é: √ √
a) 5/4 b) 9/5 c) 3/4 d) 5/4 e) 4/5

22
POTI 2015 − Geo Plana − Nı́vel 2 − Aula 0 − Professores Cleber Assis, Samuel Barbosa e Tiago Miranda

Problema 126. No desenho abaixo, a área do triângulo 4 ABD é 30m2 e a área do triângulo 4 ADC é 10m2 .
Determine a razão entre os segmentos BD e DC.

Problema 127. No desenho abaixo, E e D são os pontos médios dos lados BC e AC do triângulo 4 ABC.

a) Encontre a razão entre as áreas dos triângulos 4 ABD e 4 BED.

b) Encontre a razão entre os segmentos AG e GE.

Observação: O ponto G é chamado de Baricentro do Triângulo ABC. Como consequência deste exercı́cio,
podemos concluir que o Baricentro divide cada mediana em dois segmentos na razão 2 : 1.
Problema 128. No desenho abaixo, ABCD e AEFG são paralelogramos. Se a área de ABCD é 20cm2 ,
determine a área do paralelogramo EFGA.

23
POTI 2015 − Geo Plana − Nı́vel 2 − Aula 0 − Professores Cleber Assis, Samuel Barbosa e Tiago Miranda

Problema 129. Em cada um dos itens abaixo, a área do triângulo ABC vale 36m2 . Determine a área de cada
região sombreada sabendo que os pontos marcados nos lados o dividem em partes iguais.

a) c)

b) d)

Problema 130. Nos desenhos abaixo, o paralelogramo ABCD possui área 24cm2 e os pontos marcados nos
lados o dividem em partes iguais. Determine a área das regiões sombreadas.

a) c) e)

b) d)

Problema 131. Seja ABCD um trapézio de bases AB = 10 e CD = 6. A altura mede 4. Sejam P o ponto
médio do lado AD e Q o ponto médio do lado PB. Encontre a área do triângulo PQC.

24
POTI 2015 − Geo Plana − Nı́vel 2 − Aula 0 − Professores Cleber Assis, Samuel Barbosa e Tiago Miranda

Problema 132. A área de um quadrilátero inscritı́vel em um cı́rculo e que possui lados a, b, c e d é


p a+b+c+d
( p − a)( p − b)( p − c)( p − d) onde p = . No quadrilátero do desenho abaixo, determine a sua
2
área.

Problema 133. No desenho abaixo, E é o ponto médio do lado BC. Se as áreas dos triângulos 4 ABD e
4 ACD são 20 e 30, determine a área do triângulo 4 AED.

Problema 134. Na figura abaixo, 4 DEF é um triângulo retângulo com ∠ DEF = 90◦ e DF = 1. Se ∠ FDE = β
e ∠ ADE = α:

a) Encontre as medidas dos segmentos AE, EB e DC;

b) Mostre que sen (α + β) = sen α cos β + cos α sin β

Problema 135. Seja 4 ABC um triângulo com lados de medidas a, b e c. Se h a é o comprimento da altura
a+b+c
relativa ao vértice A e p = , verifique que:
2
p
p 2 ( p − b)( p − c)
a) h a = p( p − a) · .
a
p
b) h a ≤ p( p − a)

25
POTI 2015 − Geo Plana − Nı́vel 2 − Aula 0 − Professores Cleber Assis, Samuel Barbosa e Tiago Miranda

Problema 136. Os lados AC e BD do paralelogramo ABCD foram divididos em 4 segmentos iguais. Os


lados AB e CD foram divididos em 3 segmentos iguais. Os pontos de divisão foram conectados como indica a
figura abaixo. Se a área de ABCD é 84, determine a área sombreada.
a) 1
b) 3
c) 4
d) 7
e) 12

Problema 137. Um peso de papel tem a forma de um triângulo de lados BC = 6 cm e AB = AC = 5 cm e


está parcialmente preenchido com água. Quando o peso de papel se apoia sobre o lado BC, a água tem uma
altura de 3 cm. Qual é a altura da água, em cm, quando o peso de papel se apoia sobre o lado AB?
4
a)
3
3
b)
2
8
c)
5
18
d)
5
24
e)
5

Problema 138. Na figura ao lado, E é o ponto médio de AB, G é o ponto médio de AC e BD = DF = FC.
Se a área do triângulo ABC é 252, qual é a área do pentágono AEDFG?

a) 168
b) 189
c) 200
d) 210
e) 220

AB EG
Problema 139. No desenho abaixo, o 4 ABC é equilátero e BD = CE = AF = . Determine a razão .
3 GD

26
POTI 2015 − Geo Plana − Nı́vel 2 − Aula 0 − Professores Cleber Assis, Samuel Barbosa e Tiago Miranda

8 Triângulos
Problema 140. Classifique cada sentença como verdadeira (V) ou falsa (F):

a) Todo triângulo retângulo é isósceles.

b) Os três ângulos de um triângulo equilátero são congruentes.

c) Um triângulo é isósceles se possui os três lados congruentes.

d) Não existe triângulo que seja simultaneamente retângulo e equilátero.

e) Não existe triângulo que seja simultaneamente retângulo e isósceles.

Problema 141. No desenho abaixo, o triângulo 4 ABC é isósceles com base BC. Determine os valores de x e
y.

Problema 142. O teorema do ângulo externo afirma que a medida de um ângulo externo de triângulo é a
soma das medidas dos outros dois ângulos internos não adjacentes a ele. Podemos verificar tal afirmação
lembrando que a soma dos ângulos de um triângulo sempre é 180◦ . Assim, no desenho abaixo, o ângulo
∠ BCA mede 180◦ − α − β e o ângulo externo correspondente é o seu suplementar, ou seja, vale α + β.

Em cada um dos itens abaixo, determine o valor do ângulo x.

27
POTI 2015 − Geo Plana − Nı́vel 2 − Aula 0 − Professores Cleber Assis, Samuel Barbosa e Tiago Miranda

Problema 143. Determine os valores de x e y nos itens abaixo:

Problema 144. Dizemos que um triângulo é acutângulo quando todos os seus ângulos são menores que um
ângulo reto. Dizemos que um triângulo é retângulo quando um de seus ângulos for igual a um ângulo reto.
Por fim, dizemos que um triângulo é obtusângulo quando um dos seus ângulos for maior que um ângulo reto.
Sabendo que a soma dos ângulos de um triângulo é sempre 180◦ , determine em cada um dos itens abaixo se o
triângulo é acutângulo, obtusângulo ou retângulo conhecendo apenas dois de seus ângulos.
a) ∠ ABC = 70◦ e ∠ BCA = 10◦ .

b) ∠ ABC = 80◦ e ∠ BCA = 15◦ .

c) ∠ ABC = 30◦ e ∠ BCA = 60◦ .

d) ∠ ABC = 60◦ e ∠ BCA = 60◦ .


Problema 145. Determine o valor de x no desenho abaixo.

Problema 146. Determine o valor de x no desenho abaixo.

28
POTI 2015 − Geo Plana − Nı́vel 2 − Aula 0 − Professores Cleber Assis, Samuel Barbosa e Tiago Miranda

Problema 147. Na figura abaixo, o triângulo 4 ABC é equilátero. Determine os valores de x e y.

Problema 148. No desenho abaixo, existem dois pares de triângulos congruentes. A partir do desenho,
determine que pares são esses e justifique em quais casos de congruências eles se enquadram.

Problema 149. No desenho abaixo, os triângulos 4 AED e 4CEB são congruentes com CE = ED e ∠ECB =
∠EDA. Determine os valores de x e y.

Problema 150. Diga o nome de cada um dos seguintes pontos notáveis de um triângulo:

a) Ponto de encontro das bissetrizes.

b) Ponto de encontro das retas suportes das alturas.

c) Ponto de encontro das medianas.

d) Ponto de encontro das mediatrizes dos lados.

29
POTI 2015 − Geo Plana − Nı́vel 2 − Aula 0 − Professores Cleber Assis, Samuel Barbosa e Tiago Miranda

Problema 151. No desenho abaixo, O triângulo 4 ABD é congruente ao triângulo 4 BDC com AD = DC.
Se AC = x + y, DC = 4x, AB = 5x e BC = 3x + 2, determine os valores de x e y.

Problema 152. Determine no desenho abaixo os seguintes objetos geométricos: uma mediana, uma altura,
uma bissetriz e uma mediatriz.

Problema 153. No desenho abaixo, BD é uma altura relativa ao lado AC e BC = AC. Se ∠ ACB = 40◦ ,
determine o valor do ângulo ∠ ABD.

Problema 154. No desenho abaixo, sabendo que ∠ BCA = 30◦ , encontre o valor de ∠ ABD.

Problema 155. No desenho abaixo, BE é bissetriz de ∠ ABC. Se ∠ ACB = 30◦ , determine o ângulo ∠ DBE.

30
POTI 2015 − Geo Plana − Nı́vel 2 − Aula 0 − Professores Cleber Assis, Samuel Barbosa e Tiago Miranda

Problema 156. No desenho abaixo, o ponto D é o incentro do triângulo 4 ABC. Sabendo que ∠ ADB = 110◦ ,
∠ ADC = 130◦ e ∠ BDC = 120◦ , determine os ângulos do triângulo.

Problema 157. O triângulo abaixo é isósceles de base AB. Determine o valor do ângulo x.

Problema 158. No desenho abaixo, os triângulos 4 ABC e 4 BED são equiláteros de mesmo lado. Determine
o ângulo ∠ AEB.

Problema 159. No desenho abaixo, os triângulos 4 ABD e 4 BDC são congruentes. Determine as medidas
de α e β.

Problema 160. Na figura abaixo, ∠ODC = ∠OBE e ∠OEB = ∠ DCO. Se DC = EB, ∠ DOB = 60◦ e
OB = 5cm, determine o comprimento de BD.

31
POTI 2015 − Geo Plana − Nı́vel 2 − Aula 0 − Professores Cleber Assis, Samuel Barbosa e Tiago Miranda

Problema 161. No desenho abaixo, GC e BE são as bissetrizes dos ângulos ∠ ACB e ∠ ABC, respectivamente.
Se ∠ BAC = 60◦ e ∠ ABC = 80◦ , determine:

a) o ângulo ∠ ACB;

b) os ângulos ∠ DCB e ∠ DBC;

c) o ângulo ∠ BDC.

Problema 162. No desenho abaixo, BD é uma mediana do triângulo 4 ABC e AE uma mediana do triângulo
4 BAD. Além disso, AD = DE. Se DC = 8cm, determine a medida do segmento BE.

Problema 163. Indique para cada par de triângulos dos itens abaixo qual caso de congruência pode ser
aplicado.

32
POTI 2015 − Geo Plana − Nı́vel 2 − Aula 0 − Professores Cleber Assis, Samuel Barbosa e Tiago Miranda

Problema 164. Na figura, temos AE = BE = CE = CD. Além disso, α e β são medidas de ângulos.
Determine o valor da razão αβ .

Problema 165. Na figura, os dois triângulos são equiláteros e os ângulos dados em graus. Determine o valor
de x.

Problema 166. No triângulo ABC, os pontos D e E pertencem ao lado BC e são tais que BD = BA e
CE = CA. Dado que ∠ DAE = 40◦ , determine a medida do ângulo ∠ BAC.

Problema 167. Em um triângulo ABC, ∠ BAC = 20◦ e ∠ ABC = 110◦ . Se I é o incentro (centro da circun-
ferência inscrita) e O o circuncentro (centro da circunferência circunscrita) do triângulo 4 ABC, determine a
medida do ângulo ∠ I AO.
Problema 168. Na figura, AB = AC, AE = AD e o ângulo ∠ BAD mede 30◦ . Determine a medida do ângulo
x.

33
POTI 2015 − Geo Plana − Nı́vel 2 − Aula 0 − Professores Cleber Assis, Samuel Barbosa e Tiago Miranda

Problema 169. No triângulo ABC, AC = 5 e AB = 6. Seja P um ponto sobre a bissetriz interna do ângulo
∠ BAC. Se a área do 4 APB é 32 , determine a área do 4 APC.
Problema 170. Na figura, o triângulo 4 ABC é isósceles de base BC e o ângulo ∠ BAC mede 30◦ . O triângulo
4 BCD é isósceles de base BD. Determine a medida do ângulo ∠ DCA.

Problema 171. Na figura abaixo, o ângulo ∠ ADC mede 48◦ e os triângulos 4 ACD, 4 DBE e 4 EAF são
isósceles de bases AD, DE e EF, respectivamente. Determine a medida do ângulo ∠ DEF.

Problema 172. Em um triângulo 4 ABC, com ∠ ABC − ∠ BAC = 50◦ , a bissetriz do ângulo ∠ ACB intersecta
o lado AB em D. Seja E o ponto do lado AC tal que ∠CDE = 90◦ . Determine a medida do ângulo ∠ ADE.
Problema 173. Os pontos M e N são escolhidos na hipotenusa AB do triângulo retângulo 4 ABC de modo
que BC = BM e AC = AN. Prove que o ângulo ∠ MCN mede 45◦ .
Problema 174. Se a soma das medidas em graus dos ângulos A, B, C, D, E e F da figura abaixo é 90n, qual
o valor de n?

34
POTI 2015 − Geo Plana − Nı́vel 2 − Aula 0 − Professores Cleber Assis, Samuel Barbosa e Tiago Miranda

Problema 175. A altura CH e a mediana BK são desenhadas em um triângulo acutângulo 4 ABC. Sabemos
que BK = CH e que ∠KBC = ∠ HCB. Prove que o triângulo 4 ABC é equilátero.
Problema 176. Demonstre que a soma dos ângulos internos de um triângulo é 180◦ .
Problema 177. A mediana BM, a altura AH e a bissetriz CK de um triângulo 4 ABC são desenhadas.
Sabemos que AH intersecta BM em L, AH intersecta CK em N e BM intersecta CK em P. Os pontos L, N e P
são distintos. Prove que o triângulo LNP não pode ser equilátero.
Problema 178. No triângulo 4 ABC com ∠ BAC = 30◦ , BB1 e CC1 são alturas. Sejam B2 e C2 os pontos
médios de AC e AB, respectivamente. Prove que os segmentos B1 C2 e B2 C1 são perpendiculares.
Problema 179. Os pontos D, E e F são os pontos médios dos lados BC, CA e AB, respectivamente. Se um
dentre os segmentos DE, EF e FD é maior que um dentre os segmentos AD, BE e CF, prove que o 4 ABC é
obtusângulo.
Problema 180. Uma estrela de “n pontas” é formada como segue. Começamos numerando os lados de
um polı́gono convexo de n lados com 1, 2, . . . , n de forma consecutiva. As pontas das estrelas são formadas
pelas interseções das retas que passam por lados que diferem por duas unidades. Nessa ordem, estamos
considerando que os lados n e n − 1 vão formar pontas com os lados 2 e 1, respectivamente. A figura abaixo
mostra um exemplo com n = 5. Determine o valor da soma dos ângulos interiores das n pontas da estrela.

Problema 181. Pinóquio afirma que é possı́vel formar um retângulo usando alguns triângulos, sem sobreposição,
todos com os mesmos ângulos e nenhum dos quais possuindo um ângulo reto. Isso é realmente possı́vel ou
Pinóquio está mentindo?
Problema 182. No triângulo 4 ABC abaixo, BP é bissetriz do ângulo B, M é o ponto médio do lado AC e
AP é perpendicular a BP. Se AB = 6 e BC = 10, determine PM.

Problema 183. Em um triângulo 4 ABC, AB = AC e ∠ BAC = 30◦ , marca-se um ponto Q sobre o lado AB e
um ponto P na mediana AD, de modo que PC = PQ( Q 6= B). Determine ∠ PQC.

35
POTI 2015 − Geo Plana − Nı́vel 2 − Aula 0 − Professores Cleber Assis, Samuel Barbosa e Tiago Miranda

9 Relações Métricas no Triângulo Retângulo


Problema 184. Determine a medida da hipotenusa de um triângulo retângulo se seus catetos medem:
√ √
a) 3cm e 4cm. b) 5cm e 12cm. c) 1cm e 1cm. d) 1/2cm e 3/2cm. e) 3cm e 5cm.

Problema 185. Determine x e y no triângulo da figura abaixo, sendo x + y = 5.

Figura 13

Problema 186. Determine o valor de k na figura abaixo.

Figura 14

Problema 187. Determine os valores de x, y, z, no triângulo abaixo.

Figura 15

Problema 188. Determine a altura de um triângulo equilátero de lado medindo l.


Problema 189. Determine o comprimento da diagonal d de um quadrado de lado l.
Problema 190. Na figura, temos duas circunferências tangentes externamente de raios 3cm e 2cm, além de
uma reta tangente às circunferências nos pontos A e B. Determine AB.

Figura 16

36
POTI 2015 − Geo Plana − Nı́vel 2 − Aula 0 − Professores Cleber Assis, Samuel Barbosa e Tiago Miranda

Problema 191. Determine o perı́metro do 4 ABC abaixo, sabendo que AB = 7 2.

Figura 18
Problema 192. Na figura abaixo, temos duas semicircunferências de centros O e O0 . Um segmento
√ perpendi-
cular a AB intercepta as semicircunferências em D e E. Determine AE, sabendo que AD = 7 2.

Figura 20
Problema 193. Na figura abaixo temos três semicircunferências de centros em B, C e D, além de uma
circunferência de centro O e tangente às semicircunferências. Sabendo que AB = 4cm, determine o raio x da
circunferência.

Figura 22

Problema 194. Na figura abaixo, os pontos A, C e F estão alinhados, FC = CA = (1 + 2 3), EDCF é um
quadrado e ABC é um triângulo equilátero. Determine CG.

Figura 23

37
POTI 2015 − Geo Plana − Nı́vel 2 − Aula 0 − Professores Cleber Assis, Samuel Barbosa e Tiago Miranda

Problema 195. No triângulo retângulo abaixo, BC = 30cm, AC − AB = 6cm e ∠ ABD ≡ ∠CBD. Determine
BD.

Figura 25

Problema 196. Duas circunferências são tangentes internamente, como na figura. Os segmentos AB e
CD são perpendiculares e o ponto O é o centro da circunferência maior. Os segmentos AP e CQ medem,
respectivamente, 4 e 3 centı́metros. Qual é a medida do raio do cı́rculo menor?

Figura 26

a) 2, 25cm. b) 2, 5cm. c) 2, 75cm. d) 3cm. e) 3, 5cm.

Problema 197. A figura mostra quatro cı́rculos de raio 1cm dentro de um triângulo. Os pontos marcados
são os pontos de tangência. Qual é o comprimento do menor lado desse triângulo?

a) 4cm.


b) 3 + 3cm.

c) 5cm.


d) 3 3cm.


Figura 28 e) 2 + 2 3cm.

38
POTI 2015 − Geo Plana − Nı́vel 2 − Aula 0 − Professores Cleber Assis, Samuel Barbosa e Tiago Miranda

Problema 198. Seja ABC um triângulo retângulo em A. Seja D o ponto médio de AC. Sabendo que
BD = 3DC e que AC = 2, a hipotenusa do triângulo é:
√ √ √ √
a) 7. b) 2 2. c) 3. d) 10. e) 2 3.

Problema 199. No triângulo ABC, o comprimento dos lados AB, BC e CA, nessa ordem, são números
inteiros e consecutivos. A altura relativa a BC divide este lado em dois segmentos de comprimentos m e n,
como indicado. Quanto vale m − n?

a) 1.

b) 2.

c) 3.

d) 4.

Figura 31 e) 6.

Problema 200. O grande artilheiro Tornado está prestes a fazer o gol mais bonito de sua carreira. Ele está
de frente para o gol e apenas o goleiro está entre ele e a trave. Ele está a x metros do goleiro que, por sua vez,
se encontra a 2 metros da linha do gol, onde Tornado deseja que a bola caia após passar por cima do goleiro.
Em um gol dessa magnitude, a trajetória da bola deve ser uma semicircunferência. Tornado sabe que a bola
deve passar a exatamente 3 metros de altura do solo quando ela estiver acima do goleiro. Qual a distância de
Tornado até o goleiro, ou seja, x, em metros?

a) 3. b) 3, 5. c) 4. d) 4, 5. e) 5.

Problema 201. Na figura abaixo temos um semicı́rculo de raio 1 inscrito em um quadrado de modo que seu
centro passe por uma das diagonais do quadrado. Qual é a área do quadrado?

Figura 34
√ √ √ √
a) 3/2 + 2. b) 1 + 2 2. c) 5 + 2/2. d) 4. e) 2/3 + 2.

39
POTI 2015 − Geo Plana − Nı́vel 2 − Aula 0 − Professores Cleber Assis, Samuel Barbosa e Tiago Miranda

10 Semelhanças entre Figuras e Polı́gonos


Problema 202. Observe as figuras abaixo e responda:

Figura 36

a) os triângulos 4 ABC e 4 ADE são semelhantes?


Figura 37
b) caso sejam semelhantes, quais são os lados homólogos?

c) os triângulos 4KLM e 4 MPQ são semelhantes?

Problema 203. Qual a razão de semelhança dos triângulos abaixo?

Figura 38

Problema 204. Como João pode medir a altura de um poste, conhecendo sua altura, 1, 60m, o comprimento
de sua sombra, 2m, o comprimento da sombra do poste no mesmo instante que mediu sua sombra, 7m?
Problema 205. Na figura abaixo, BC = 12cm e AH = 8cm, sendo AH altura do 4 ABC. Determine o lado
do quadrado MNPQ.

Figura 39

40
POTI 2015 − Geo Plana − Nı́vel 2 − Aula 0 − Professores Cleber Assis, Samuel Barbosa e Tiago Miranda

Problema 206. Na figura abaixo, temos uma reta que passa pelos pontos A, B e C e outra que passa por A e
é tangente às circunferências de centros B e C e raios 3cm e 5cm. Se AB = 7cm, determine BC.

Figura 40

Problema 207. Sabendo que AB = 15, BC = 20, AD = 10 e DC = 15, determine a medida de DE na figura
abaixo.

Figura 42

Problema 208. Na figura abaixo, temos AC = 4 e AB = 6. Determine o perı́metro do quadrado AEDF.

Figura 43

Problema 209. No retângulo da figura abaixo temos que AB = 20, BC = 12 e AM = MB. Determine a
medida de EF.

Figura 44

41
POTI 2015 − Geo Plana − Nı́vel 2 − Aula 0 − Professores Cleber Assis, Samuel Barbosa e Tiago Miranda

Problema 210. Determine x na figura abaixo, na qual existem três quadrados de lados 9, x e 4.

Figura 45
Problema 211. Na figura abaixo, temos um triângulo inscrito. Se AB = 10, AC = 12 e AH = 4, determine o
raio da circunferência.

Figura 47
Problema 212. Na figura abaixo, temos AC = CB = 10cm, AB = 6cm e AM = MB. Além disso, o segmento
BH tangencia a semicircunferência com centro em M. Determine o raio dessa semicircunferência.

Figura 49
Problema 213. Na figura abaixo, temos duas semicircunferências. Se AD = 36 e BC = CD, determine CD.

Figura 50

42
POTI 2015 − Geo Plana − Nı́vel 2 − Aula 0 − Professores Cleber Assis, Samuel Barbosa e Tiago Miranda

Problema 214. Na figura abaixo, DE//AC, ∠ ACD ≡ ∠ BCD, BC = m e AC = n. Determine a medida de


DE em função de m e n.

Figura 52

Problema 215. No desenho abaixo, o triângulo ABC é equilátero e BD = CE = AF = AB/3. Determine a


razão EG/GD.

Figura 53

Problema 216. O quadrado ABCD está inscrito em um cı́rculo cujo raio mede 30. A corda AM intercepta a
diagonal BD no ponto P. Se o segmento AM mede 50, determine a medida do segmento AP.

Figura 54

43
POTI 2015 − Geo Plana − Nı́vel 2 − Aula 0 − Professores Cleber Assis, Samuel Barbosa e Tiago Miranda

Respostas e Soluções.
1. Existem 6 segmentos de reta com vértices nesses 4 pontos: AB, AC, AD, BC, BD e CD. Veja que a resposta
não seria diferente se os pontos não fossem colineares.
2. Temos I J = KL, GH = FE e AB = CD.
3. Resposta B.
4.
a) Como AC + CB = 20cm, se CB = 8cm temos AC = 12cm.
21
b) Somando AC + CB = 20cm com AC − CB = 1cm, temos 2AC = 21cm. Portanto, AC = .
2
c) Temos 20 = AC + CB = 2x + ( x − 1). Portanto, x = 7 e AC = 2x = 14.
5. Com a exceção do ponto B, por qualquer um dos outros pontos, existe exatamente uma semirreta que
satisfaz a condição do enunciado. Portanto, existem 4 semirretas.
6. Como AM = MB, temos 2x − 5 = x + 7, ou seja, x = 2x − x = 7 + 5 = 12.
7. Se o ponto P se encontra à esquerda de A, o segmento PB é a soma de PA e AB e consequentemente maior
que AB. Se o ponto P se encontra entre A e B, o comprimento de PA é estritamente menor que o comprimento
de AB. Consequentemente, a única possibilidade é P estar situado à direita de B. O exemplo abaixo mostra
que tal configuração é admissı́vel.

8. Sejam AB = x e CD = y. Como
2BC = AD = AB + BC + CD = 20 + BC,
temos BC = 20 e consequentemente AD = 40.
9. Como AM = MB, temos 7x − 1 = x + 11, ou seja, 6x = 12 e consequentemente x = 2.
10. Como AM = MB, temos AB = 2x. Consequentemente:
4x − 9 = AC = AB + BC = 2x + x − 1.
Isso produz: x = 9 − 1 = 8. Portanto, AB = 2x = 16.
11. Não podemos ter o vértice A entre B e C pois BC < AB. Assim, A está situado à esquerda ou à direita do
segmento BC. Quando A está mais próximo de C, o segmento AC mede AB − BC = 20cm. Quando A está
mais próximo de B, o segmento AC mede AB + BC = 40cm.
12. Considere o desenho abaixo:

Sejam BC = a e CD = x. Assim,
AB · BD = AC · CD
9( a + x ) = (9 + a ) x
9a + 9x = 9x + ax
9a = ax
9 = x.
Portanto, o comprimento de CD é 9cm.

44
POTI 2015 − Geo Plana − Nı́vel 2 − Aula 0 − Professores Cleber Assis, Samuel Barbosa e Tiago Miranda

13. Sejam AM = MB = x e DM = y. Temos:

10 = DB − DA
= ( x + y) − ( x − y)
= 2y.

Portanto, y = 5cm.

14. Sejam AC = CB = x e AE = y. Então CE = ED = y − x e

30 = AB + ED − AC
= (2x ) + (y − x ) − ( x )
= y

Portanto, AE = 30cm.
15. Considere o desenho abaixo:

Sejam AB = x, BC = y e CD = z. Temos y = AC − AB = 3. Além disso,

3 = 3AB − BD − 2CD
= 3x − (3 + z) − 2z
= 3x − 3z − 3 ⇒
6 = 3x − 3z.

Também temos 12 = 3AB + 3CD = 3x + 3z. Somando com a última equação, obtemos 18 = 6x. Portanto,
x = 3 e z = 4 − 3 = 1. Finalmente, AD = AB + BC + CD = 3 + 3 + 1 = 7.
16. Sejam AB = x e BC = CD = y. Assim,

AD2 − AB2 = ( AD − AB)( AD + AB)


= (2y)(2x + 2y)
= 4y( x + y)
= 4BC · AC
= 160.
17. Sim. Veja que:

MN = MB + NB
AB BC
= +
2 2
AB AB
= +
2 2
= AB.

45
POTI 2015 − Geo Plana − Nı́vel 2 − Aula 0 − Professores Cleber Assis, Samuel Barbosa e Tiago Miranda

18. Sim, é possı́vel. No exemplo abaixo, os pontos pretos simbolizam as estações e os segmentos, os trilhos.

19.

a) Se todos os três pontos estão ambos à esquerda de b) Sejam S A e SB as somas das distâncias de todos
A ou ambos à direita de B, a soma das distâncias os pontos ao vértice A e ao vértice B, respectiva-
dos três pontos a um desses vértices é estritamente mente. Analisemos a contribuição de um ponto X
maior do que a soma das distâncias ao outro vértice. na diferença S A − SB . Quando X está à esquerda de
Precisamos realmente estudar o caso em que exis- A, a contribuição é XA − XB = − AB e quando X
tem dois deles de um lado e um do outro como está à direita de B a contribuição é XA − XB = AB.
indica a figura. Calculemos a diferença entre a Ou seja, alguns pontos vão contribuir com o va-
soma das distâncias ao vértice A e a soma das lor + AB e outros com o valor − AB. Para que a
distâncias ao vértice B. diferença seja zero, a quantidade de parcelas com o
sinal “+” deve ser igual à quantidade de parcelas
com o sinal “−”. Como 1001 é um número ı́mpar,
( XA + YA + ZA) − ( XB + YB + ZB) = tal igualdade não pode ocorrer.
( XA − XB) + (YA − YB) + ( ZA − ZB) =
(− AB) + (− AB) + ( AB) =
− AB 6= 0.
20. (Extraı́do do Torneio das Cidades)

Os 25 quadradinhos determinam 6 × 6 = 36 vértices. Note que não é possı́vel desenhar segmentos usando
Como cada segmento deve usar dois deles, podemos os vértices do lado inferior sem deixar de usar pelo
concluir inicialmente que João não pode desenhar menos mais um vértice de tal lado. Logo, não podere-
36 mos usar pelo menos 3 vértices. Como o número de
mais que = 18 segmentos. Analisando um lado
2 vértices usados deve ser um número par, no máximo
qualquer do quadrado maior, não é possı́vel que os utilizaremos 32 deles e assim teremos não mais que
6 vértices sejam usados. Assim, eliminando-se um 32
vértice do lado superior e um vértice do lado inferior, = 16 segmentos desenhados. O exemplo abaixo
2
teremos apenas 34 vértices utilizáveis e consequente- mostra que tal número é realizável.
34
mente não mais que = 17 segmentos. Essa ainda
2
não é a melhor estimativa. Para que apenas dois
vértices dos lados mencionados anteriormente não se-
jam usados, deve ocorrer a configuração exibida na
próxima figura:

46
POTI 2015 − Geo Plana − Nı́vel 2 − Aula 0 − Professores Cleber Assis, Samuel Barbosa e Tiago Miranda

21.

a) Para três retas, temos 7 regiões como indica a próxima figura.

b) Para cinco retas, temos 16 regiões. Analisando a configuração com três retas, podemos notar que a quarta
reta cria 4 novas regiões ao intersectar as retas que já estavam traçadas. A quinta reta gera mais 5 regiões
ao intersectar as outras quatro.

Como não existem duas retas paralelas e nem três concorrentes, se já estão traçadas k retas, uma nova reta
acrescentaria mais k + 1 regiões porque ela dividirá em duas k + 1 regiões que já existiam. Assim, n + 1 retas
obedecendo as condições do enunciado dividem o plano em:

n ( n + 1)
1 + (1 + 2 + . . . + n ) = 1 + .
2
22. Como OC é bissetriz, 2x − 5 = x + 3 e daı́ x = 8◦ .
23. Temos:

180◦ = ∠ AOC + ∠COD + ∠ DOB


= ∠ AOC + 2∠COD
= 3y + (2y + 10)
= 5y + 10.

Portanto, y = 34◦ . Como


39◦ = ∠COD = ∠ DOB = x + 10◦ ,
temos x = 29◦ .

47
POTI 2015 − Geo Plana − Nı́vel 2 − Aula 0 − Professores Cleber Assis, Samuel Barbosa e Tiago Miranda

24. Sejam ∠ AOC = 2x e ∠COB = 2y. Temos:


2x + 2y ∠ AOB
∠ DOE = x + y = = = 35◦ .
2 2
25. Apenas D e E são verdadeiras.
26. Como 180◦ = ∠ BOC + ∠ AOC = 5x, segue que x = 36◦ .
27. Seja ∠ BOC = 2x, então ∠ AOC = 2∠ BOC = 4x. Como 60◦ = ∠ AOC + ∠ BOC = 6x, segue que x = 10◦ .
Portanto, ∠ DOA = ∠ DOC + ∠COA = x + 4x = 50◦ .
28. Os ângulos são x e 4x − 40◦ . Assim, 140◦ = 5x − 40◦ e x = 36◦ . Os ângulos são 36◦ e 104◦ .
29. Temos 2x + 10◦ = ∠ AOD = ∠COB = 50◦ pois eles são opostos pelo vértice. Consequentemente, x = 20◦ .
30. Temos ∠ AOD + ∠ DOC = 90◦ e consequentemente ∠ DOC = 35◦ . Como ∠ FOE e ∠ DOC são opostos
pelo vértice, temos ∠ FOE = 35◦ .
31. A divisão determina os ângulos 2x, 3x e 4x. Somando-os, temos 90◦ = 9x. Portanto, x = 10◦ e os ângulos
são 20◦ , 30◦ e 40◦ .
90◦ + 10◦
32. Como x + y = 90◦ e x − y = 10◦ , somando e subtraindo as duas equações, temos x = = 50◦ e
2
90◦ − 10◦
y= = 40◦ .
2
33. Como os ângulos ∠ AOD e ∠COB são opostos pelo vértice, temos 3x + 10◦ = 2x + 20◦ , ou seja, x = 10◦ .
Como ∠ AOC e ∠COB são suplementares, obtemos ∠ AOC = 180◦ − (2x + 20◦ ) = 140◦ .
34. Sejam 2x e 2y as medidas dos ângulos adjacentes. O ângulo entre as bissetrizes é
2x + 2y 150◦
x+y = = = 75◦ .
2 2
35. Como os ângulo ∠ BOC, ∠ DOE e ∠ AOF são opostos pelo vértice aos ângulos ∠EOF, ∠ AOB, ∠COD,
respectivamente, temos 360◦ = 6x e consequentemente x = 60◦ .
36. As duas retas determinam quatro semirretas: OA, OB, OC e OD. Todos os ângulos são determinados
pelas combinações de duas delas. Como existem 6 maneiras de escolhermos duas delas - (OA, OB), (OA, OC ),
(OA, OD ), (OB, OC ), (OB, OD ) e (OC, OD ) - ficam determinados 6 ângulos.

37.
a) Como ∠COB = 80◦ , temos x + y = ∠ DOB = 180◦ − 80◦ = 100◦ . Além disso, como ângulos opostos
pelo vértice possuem mesma medida, temos x + z = 80◦ e y + z = ∠ DOB = 100◦ . Resolvendo o sistema
produzido por essas três equações, encontramos x = 40◦ , y = 60◦ e z = 40◦ .
b) Temos (3x + 20◦ ) + ( x + 40◦ ) = ∠ DOB + ∠ BOC = 180◦ . Assim, x = 30◦ . Além disso, z = ∠ AOC =
∠ DOB = 3x + 20◦ = 110◦ .

48
POTI 2015 − Geo Plana − Nı́vel 2 − Aula 0 − Professores Cleber Assis, Samuel Barbosa e Tiago Miranda

38.

a) 37◦ 300 . b) 52◦ 100 . c) 75◦ 220 1700 . d) 59◦ 020 2000 .
39.

a) Temos x = 110◦ pois ângulos opostos pelo vértice possuem igual medida. Como os ângulos de medidas
110◦ e y são colaterais internos, temos y = 180◦ − 110◦ = 70◦ .

b) Temos x = 95◦ pois os ângulos com tais medidas são alternos externos. Além disso, y = 180◦ − 95◦ = 85◦ .

40. Como ângulos correspondentes são iguais, temos y = 180◦ − 120◦ = 60◦ e z = 180◦ − 80◦ = 100◦ .
Analisando agora os três ângulos marcados no vértice O que formam um ângulo raso, temos x + y + 80◦ = 180◦ ,
ou seja, x = 40◦ .

41. Trace pelo ponto F uma reta paralela ao segmento AB. Os pares de ângulos marcados com os mesmos
sı́mbolos são iguais pois são correspondentes. Portanto, 80◦ = x + 40◦ e consequentemente x = 40◦ .

42. Segue do paralelismo que ∠ BED = ∠EBA = x. Somando agora os ângulos marcados no vértice E que
formam um ângulo raso, temos: 2x + 105◦ + x = 180◦ . Assim, x = 25◦ .

43. Do paralelismo segue que ∠ JFH = ∠CHG = x e ∠KFE = ∠ FEA = y. Portanto, 180◦ = x + y + z =
150◦ + z. Daı́, z = 30◦ .
44.

a) Pelo ponto E, trace uma paralela a AB. O ângulo b) Repitamos o procedimento do exercı́cio anterior
x será então formado por dois ângulos que são traçando retas paralelas a AB pelos pontos E e F
◦ ◦
alternos internos aos ângulos que medem 45 e 70 . como indica a figura abaixo.
Portanto, x = 115◦ .

Teremos inicialmente 70◦ = x + 50◦ , ou seja, x =


20◦ . Além disso, ∠EFB = x + 50◦ = 100◦ .

49
POTI 2015 − Geo Plana − Nı́vel 2 − Aula 0 − Professores Cleber Assis, Samuel Barbosa e Tiago Miranda

45.

a) 34◦ 230 . b) 21◦ 400 0900 . c) 23◦ 190 1400 . d) 93◦ 020
46. O ângulo entre as bissetrizes corresponde a soma da medade de cada um dos ângulos originais, ou seja,
180◦
= 90◦ .
2
47.

Sejam ∠ BAD = 2x e ∠ BAC = 2y os ângulos adjacentes. O ângulo entre as bissetrizes é

∠ BAC − ∠ BAD 100◦


y−x = = = 50◦ .
2 2
48. Sejam x = ∠CAD = ∠ DAB e y = ∠EAB. Então 2x + y = 120◦ e 2x − y = 80◦ . Portanto,

∠ DAE = x − y = 50◦ − 20◦ = 30◦ .

49. Temos x − y = 20◦ . Além disso, (90◦ − x ) + (180◦ − 2x ) = 2(90◦ − y), ou seja, 3x − 2y = 90◦ . Resolvendo
o sistema produzido pelas duas últimas equações, obtemos x = 50◦ e y = 30◦ .

50. Devemos encontrar x tal que:

x2 − 50◦ = 5(90◦ − x )
x2 + 5x = 500
x ( x + 5◦ ) = 20 · 25◦ .

Uma solução seria x = 20◦ .

51. Suponhamos que y = 4x. Assim,

(180◦ − x ) + (180◦ − 4x )
(90◦ − x ) + (90◦ − 4x ) =
10
1800◦ − 50x = 360◦ − 5x
1440◦ = 45x
32◦ = x.
52. Letra B.
53. Como 180◦ = (3x − 40◦ ) + (2x + 60◦ ) = 5x + 20◦ , segue que x = 32◦ e o maior dos ângulos vale 124◦ .

54. Resposta B.

50
POTI 2015 − Geo Plana − Nı́vel 2 − Aula 0 − Professores Cleber Assis, Samuel Barbosa e Tiago Miranda

55.
(a) Pelo Teorema dos Bicos(veja o último exercı́cio) aplicado à linha poligonal que passa por E, temos
x = α + θ. Aplicando-o novamente, agora à linha poligonal que passa por J, temos 180◦ − 3x = 2α + 2θ.
Assim, 180◦ − 3x = 2x, ou seja, x = 36◦ .
(b) Prolongue a reta JE. Do paralelismo obtemos um outro ângulo β como indica a figura abaixo.

Pelo Teorema do ângulo externo, temos que


2β = 100◦ , ou seja, β = 50◦ .

56. Prolongue HG e HF até encontrarem CD e AB. Pelo Teorema dos Bicos aplicado à poligonal que passa
pelos vértices F e G, podemos concluir que tais prolongamentos formam ângulos de 90◦ − 2x com esses
segmentos. Aplicado agora o Teorema dos Bicos à linha poligonal que passa por H, podemos concluir que
x = (90◦ − 2x ) + (90◦ − 2x ). Assim, x = 36◦ .

57. Apliquemos o Teorema dos Bicos à linha poligonal que passa pelo vértice C. Os ângulos incidentes em F
e E valem x + α e 90◦ − 2x. Portanto,
80◦ + α = ( x + α) + (90◦ − 2x ).
Consequentemente x = 10◦ .
58. (Extraı́do da OBM − 2006)
Como os dois bastões verticais são paralelos, podemos aplicar o Teorema dos Bicos(veja último exercı́cio) no
caminho poligonal formado pelos lados dos quadrados que contém os ângulos marcados obtendo:
30◦ + 126◦ + 75◦ + x = 90◦ + 90◦ + 90◦ .
Assim, x = 39◦ .

51
POTI 2015 − Geo Plana − Nı́vel 2 − Aula 0 − Professores Cleber Assis, Samuel Barbosa e Tiago Miranda

59. Por cada um dos vértices dos “bicos”, trace uma paralela ao segmento AB. Vários pares de ângulos
alternos internos serão formados como indica a figura abaixo:

Cada um dos ângulos marcados possui exatamente um representante entre os ângulos brancos e pretos. Assim,
cada uma dessas somas de ângulos vale x1 + x2 + . . . + x6 .
60. Temos 1, 0 e 2 para os itens a), b) e c), respectivamente.
61.

a) Sim, pois 6 < 4 + 5. c) Não, pois 8 = 4 + 4. e) Sim, pois 6 < 6 + 6.


b) Não, pois 7 > 3 + 3. d) Sim, pois 4 < 3 + 3.
62. O comprimento do terceiro lado deve estar entre 4 − 3 = 1 e 3 + 4 = 7.
63. O terceiro lado deve ser maior que 5 − 2 = 3 e menor ou igual a 5.
64. Se x é a medida dos lados iguais, devemos ter x + x > 4, ou seja, x > 2.
65. Construa um segmento AB de comprimento 7cm e outros dois de comprimentos 4cm e 6cm. Em seguida,
usando o compasso com centro em A e abertura igual ao segmento de comprimento 4cm, desenhe um cı́rculo.
Faça o mesmo com o compasso centrado em B e usando como abertura o segmento de comprimento 6cm. Seja
G um dos pontos de interseção desses dois cı́rculos. O triângulo 4 ABG possui lados de comprimentos 4cm,
6cm e 7cm.

66.
a) Não, pois o maior deles não é menor que a soma dos outros dois.
b) Sim, pois o maior deles é menor que a soma dos outros dois.
c) Sim, pois o maior deles é menor que a soma dos outros dois.

52
POTI 2015 − Geo Plana − Nı́vel 2 − Aula 0 − Professores Cleber Assis, Samuel Barbosa e Tiago Miranda

67. A princı́pio, o terceiro lado poderia assumir qualquer valor maior que 17 − 10 = 7cm e menor que
10 + 17 = 27cm. Nesse intervalo, apenas 9, 16 e 25 são quadrados de inteiros. Logo, o terceiro lado só pode
assumir um desses três valores.
68. A princı́pio, o terceiro lado poderia assumir qualquer valor menor que 13 + 7 = 20cm e maior que
13 − 7 = 6cm. Como entre tais números, apenas 10 e 15 são múltiplos de 5, esses dois números constituem os
possı́veis valores do terceiro lado.
69. O comprimento do lado BC deve ser menor que 3, 8 + 0, 6 = 4, 4cm e maior que
3, 8 − 0, 6 = 3, 2cm. O lado BC corresponde ao único inteiro entre tais números, ou seja, BC = 4cm.
70. (Extraı́do da Olimpı́ada Russa)
Como todos as frações são positivas, basta mostrarmos que a maior delas é menor que a soma das outras
1
duas. Suponha sem perda de generalidade que é a maior das três frações. Pela desigualdade triangular
a+b
aplicada ao triângulo de lados a, b e c, temos: c < a + b. Consequentemente:

a + c < 2a + b < 2( a + b);


b + c < a + 2b < 2( a + b).

Assim,
1 1 1 1 1
+ > + = .
a+c b+c 2( a + b ) 2( a + b ) a+b
71. Prolongue o segmento BP até ele intersectar o lado AC em D.

Aplicando a desigualdade triangular nos triângulos 4 ABD e 4 PDC, obtemos:

AB + x = AB + AD > BD = BP + z
z + y = PD + DC > PC

Somando as duas desigualdades resultantes, temos:

AB + ( x + y) + z > z + BP + PC
AB + AC > BP + PC.
72. Pela desigualdade triangular aplicada aos triângulos 4 BDE, 4 DAF e 4 EFC, temos:

DE < BD + BE
EF < EC + CF
DF < DA + AF.

Somando as três desigualdades, obtemos:

DE + EF + DF < ( BD + DA) + ( AF + FC )
+( BE + EC )
= AB + AC + BC.

53
POTI 2015 − Geo Plana − Nı́vel 2 − Aula 0 − Professores Cleber Assis, Samuel Barbosa e Tiago Miranda

73. Aplicando a desigualdade triangular nos triângulos 4 ABO, 4 BOC, 4COD e 4 AOD, temos:

AB < AO + OB;
BC < BO + OC;
CD < OC + OD;
AD < AO + OD.

Somando as quatro desigualdades, temos

AB + BC + CD + DA < 2( AO + BO + CO + DO)
= 2( BD + AC ).

Para verificar o segundo item, basta aplicar a desigualdade triangular nos triângulos 4 BCD, 4 ACD, 4 ABD
e 4 ABC para obtermos:

BD < BC + CD;
AC < AD + DC;
BD < AD + AB;
AC < AB + BC.

Somando as quatro desigualdades, podemos concluir que:

2( AC + BD ) < 2( AB + BC + CD + DA).

Basta agora dividirmos as duas desigualdades resultantes para obtermos o enunciado.

74. (Extraı́do da Olimpı́ada Iberoamericana)

Recorte o triângulo 4 PQC e coloque-o virado for- Como BP = PE, QE = DQ e AD = AB, podemos
mando o triângulo 4 PEQ de modo que PE = QC agora dobrar os triângulos ao longo desses segmentos
e QE = PC. Formalmente estamos construindo um e formar um tetraedro como indica a figura abaixo.
triângulo congruente ao inicial.

Imagine agora que os segmentos AP, PQ e AQ são Como X, Y e E são três vértices em arestas distintas
marcas de dobraduras no papel. do tetraedro, eles formam um triângulo.

Comentário para Professores: O apelo fı́sico do uso dobraduras tem como propósito tornar a solução mais
acessı́vel, natural e divertida para alunos jovens. Tal operação pode ser formalizada com o uso de isometrias
no espaço.

54
POTI 2015 − Geo Plana − Nı́vel 2 − Aula 0 − Professores Cleber Assis, Samuel Barbosa e Tiago Miranda

75. Sejam X e Y pontos no interior do triângulo 4 ABC. Trace a reta que os une. Se os dois pontos não estão
em mesmo lado do triângulo, tal reta intersecta os lados em dois pontos. A figura abaixo representa os dois
caso possı́veis.

No primeiro desenho, XY ≤ LM. Além disso, pela desigualdade triangular:


LM < BL + BM;
LM < AL + AM
< AL + AC + CM.
Então,
2LM < ( BL + AL) + ( BM + CM) + AC
= 2p
No segundo desenho, novamente pela desigualdade triangular, temos:
LM < BL + BC
LM < LA + AC.
Somando as desigualdades,
2LM < ( BL + LA) + AC + BC
= 2p
Em ambos os casos, LM < p, como desejado. Caso os dois pontos estejam em um mesmo lado do triângulo, o
comprimento do segmento que os une é menor ou igual que o lado que os contém. Este por sua vez é menor
que o semiperı́metro em virtude da desigualdade triangular.
76.
AB 4
a) = = 2.
CD 2
AB 7
b) = .
CD 3
AB 1/2 3
c) = = .
CD 1/3 2

AB 3 2
d) = √ = 3.
CD 2

AB 5
e) = .
CD 2
AB 2 √
f) = √ = 2.
CD 2
AB AB
g) Em um paralelogramo, lados opostos são congruentes, ou seja, AB = CD. Assim, temos = = 1.
CD AB

55
POTI 2015 − Geo Plana − Nı́vel 2 − Aula 0 − Professores Cleber Assis, Samuel Barbosa e Tiago Miranda

77. Segmentos comensuráveis são aqueles que possuem um número racional como resultado da razão de
suas medidas. Assim, no exercı́cio anterior, os segmentos comensuráveis são os dos ı́tens a, b, c, d, g. E, é
claro, os segmentos incomensuráveis, que são aqueles que obtemos um número irracional na razão de suas
medidas, são os das letras e, f.
28 7
78. Temos = , ou seja, CD = 16cm.
CD 4
CD 1
79. Temos = , ou seja, CD = 4cm. Perceba que a escolha da medida de CD no numerador ocorreu
AB 2
porque a fração é menor que 1 e o segmento de maior medida é AB.

8 √ √
80. Seja l a medida do lado do quadrado. Temos então = 2, ou seja, l = 4 2.
l

6 3 √
81. Seja l a medida do lado, temos = , ou seja, l = 4 3cm.
l 2

82. Como a medida do diâmetro é o dobro da medida do raio de uma circunferência, temos = π, ou seja,
2R
a medida R do raio é 4cm.
83. Sendo a, b, c, as medidas dos lados do 4 ABC, temos como razões entre as medidas dos lados a/b (ou
b/a), a/c (ou c/a) e c/b (ou b/c), números racionais, pois são, dois a dois, segmentos comensuráveis. Fazendo
EF a+b+c
a razão dos segmentos EF e AB, sendo indiferente se tomássemos AC ou BC, temos = =
AB a
a b c b c
+ + = 1 + + , que é um número racional e, portanto, os segmentos são comensuráveis.
a a a a a
84. (Extraı́do da Vı́deo Aula) Chamando a medida do lado de l e a medida da diagonal de d, pelo Teorema
d d p
de Pitágoras, temos d2 = l 2 + l 2 , segue que ( )2 = 2. Supondo que ( ) seja um número racional, digamos
l l q
onde mdc( p, q) = 1 (caso não seja, basta simplificar), temos
p
( )2 = 2
q
p2
= 2
q2
p2 = 2q2 .

Vamos usar o fato de que, se n2 é par, então n também é par (não é difı́cil essa demonstração). Como d2 = 2l 2 ,
então d2 é par e, por consequência, d também é par e faremos d = 2k, k ∈ N. Temos agora

p2 = 2q2
(2k)2 = 2q2
4k2 = 2q2
2k2 = q2 .

Concluı́mos então que q2 e, por consequência, q são pares. Mas se p e q são pares, mdc( p, q) 6= 1, o que é
d
absurdo. Portanto a fração não pode ser um número racional e os segmentos (lado e diagonal do quadrado)
l
não podem ser comensuráveis e os chamamos de incomensuráveis.

56
POTI 2015 − Geo Plana − Nı́vel 2 − Aula 0 − Professores Cleber Assis, Samuel Barbosa e Tiago Miranda

85. Utilizando o Teorema de Tales, temos

a) (Extraı́do da Vı́deo Aula) d)


x 4
= x+3 x+5
6 8 =
4 x
x = 3.
x2 + 3x = 4x + 20
b) (Extraı́do da Vı́deo Aula) x2 − x − 20 = 0
2x + 3 4 x1 = −4
=
5x − 1 7 x2 = 5.
20x − 4 = 14x + 21
x = 25/6. Porém, como se trata de comprimento de segmen-
tos, apenas x = 5 é solução.
c)
5 4
=
x 6
x = 15/2.
86. (Extraı́do da Vı́deo Aula) Aplicando o Teorema de Tales, temos

3 x
=
6 x+6
6x = 3x + 18
x = 6.
87. (Extraı́do da Vı́deo Aula) Usando o Teorema da Bissetriz Interna, temos

4 3
=
x 5
x = 20/3.
88. (Extraı́do da Vı́deo Aula) Aplicando o Teorema da Bissetriz Externa, temos

8 6
=
x + 12 12
8 1
=
x + 12 2
12 + x = 16
x = 4.
89. (Extraı́do da Vı́deo Aula) Se o perı́metro mede 75cm, temos SC = 35 − x. Aplicando o Teorema da
Bissetriz Interna,
10 35 − x
=
x 30
2
35x − x = 300
x2 − 35x + 300 = 0
x1 = 15
x2 = 20.

Perceba que pode ser qualquer um dos dois valores.

x y x+y 15 3
90. Pelo Teorema da Bissetriz Interna, temos = = = = , segue que x = 6 e y = 9.
8 12 8 + 12 20 4

57
POTI 2015 − Geo Plana − Nı́vel 2 − Aula 0 − Professores Cleber Assis, Samuel Barbosa e Tiago Miranda

91. Aplicando o Teorema da Bissetriz Interna, temos


x+1 x+3
=
x−2 4
x2 + x − 6 = 4x + 4
x2 − 3x − 10 = 0
x1 = −2
x2 = 5.
Como se trata de comprimento de segmentos, apenas x = 5 é solução.
92. Incialmente, construiremos o triângulo e seus elementos.

Figura 11
Como BK é bissetriz, vamos aplicar o Teorema da Vamos repetir o processo, porém, agora, CJ como
Bissetriz Interna. bissetriz:
12 − x x BJ JK
= =
10 14 14 7
10x = 168 − 14x BJ
= 2.
x = 7. JK

BJ
Como ∈ Q, então são segmentos comensuráveis.
JK
93.

Figura 12
Temos, inicialmente, BC = 15, AB = x e AC = x + 3, sendo AC o maior dos catetos. Aplicando o Teorema
de Pitágoras, temos x2 + ( x + 3)2 = 152 , segue que x = 9. Como BD é bissetriz, vamos aplicar o Teorema da
Bissetriz Interna:
AD 12 − AD
=
9 15
15AD = 108 − 9AD
9
AD = .
2

2 2 2 9 5
Aplicando, por fim, o Teorema de Pitágoras ao 4 ABD, temos BD = 9 + (9/2) , segue que BD = .
2

58
POTI 2015 − Geo Plana − Nı́vel 2 − Aula 0 − Professores Cleber Assis, Samuel Barbosa e Tiago Miranda

94.
a) A = 8 · 4 = 32cm2 .

b) A altura h mede 12 · sen 30◦ = 6cm e a base b mede 12 · cos 30◦ = 6 3cm. Assim
√ √
A = 6 · 6 3 = 36 3cm2 .
95.
a) A = 82 = 64cm2 .
b) A = 7, 12 = 50, 41cm2 .

c) A = ( 3)2 = 3cm2 .
√ √
d) Se a diagonal mede 6cm, o lado mede 3 2cm, então a área é A = (3 2)2 = 18cm2 .
96.

a) l = 25 = 5cm.
√ √
b) l = 12 = 2 3cm.
97.
a) A = (5 · 8)/2 = 20cm2 .
b) Se 2b é o comprimento da outra diagonal, como as diagonais de um losango são perpendiculares, usando o
Teorema de Pitágoras obtemos:
b 2 + 32 = 52
p
b = 52 − 32
b = 4.
Portanto, a outra diagonal mede 8cm e a área do losango vale A = 24cm2 .
c) Dois ângulos internos consecutivos de um losango são suplementares. Assim, um de seus ângulos internos √
◦ 82 3
será 60 . Temos, então, dois triângulos equiláteros de lados medindo 8cm. A área de cada um deles é .
√ 4
82 3 √
Portanto, a área do losango é 2 · = 32 3cm2 .
4
4(5 + 7)
98. A = = 24cm2 .
2
100. Podemos usar o Teorema de Pitágoras para encontrarmos a altura h.

h2 + 32 = 52
p
h = 52 − 32
h = 4.
4(6 + 12)
Daı́, segue que A = = 36cm2 .
2

101.
a) A = 6 · 4 = 24cm2 .
√ √ √
b) Temos que a altura do paralelogramo mede 6 · sen 60◦ = 3 3. Daı́, segue que A = 8 · 3 3 = 24 3cm2 .

59
POTI 2015 − Geo Plana − Nı́vel 2 − Aula 0 − Professores Cleber Assis, Samuel Barbosa e Tiago Miranda

102.

a) A = (8 · 5)/2 = 20.

b) Pelo Teorema de Pitágoras, a medida do outro cateto é 12cm. Daı́, segue que A = (12 · 5)/2 = 30.

62 3 √
c) A = = 9 3cm2 .
4
6 · 8 · sen 45◦ √
d) A = = 12 2cm2 .
2
102. Se o perı́metro é 72cm2 , então o lado é 72/4 = 18cm, segue que A = 182 = 324cm2 .

103. Chamando a altura de x, a base é 2x. Temos, então, que 2x2 = 450. Daı́ segue que x = 15. Portanto, as
dimensões do retângulo são 15cm e 30cm.

104. Sendo x e y as dimensões iniciais, temos xy = 100. Após as modificações nas dimensões, sua área será

A = 0, 9x · 1, 1y = 0, 99 · 100 = 99cm2 .

105.
√ √
a) A altura de um triângulo de lado 8 é (8 3)/2 = 4 3. Como √ o raio do cı́rculo inscrito é a terça parte da
4 3
altura do triângulo, o raio do cı́rculo do desenho mede . Assim, a área da região hachurada pode ser
3
calculada pela diferença entre as áreas do triângulo equilátero e do cı́rculo.

Ahachurada = Atriângulo equilátero − Acı́rculo


√ √ !2
82 3 4 3
A = −π
4 3
√ 16π
= 16 3 −
√ 3
48 3 − 16π 2
= cm .
3

b) A área hachurada é a diferença entre as áreas do quadrado e do setor circular (quarta parte do cı́rculo).

Ahachurada = Aquadrado − Asetor


102 π
= 102 −
4
= 100 − 25π
= 25(4 − π )cm2 .

c) A medida do ângulo do setor circular é 180◦ − 60◦ = 120◦ , que equivale a 1/3 do cı́rculo. Temos, então

62 π
A= = 12πcm2 .
3

106. (Extraı́do do ENEM 2013) Como a área inicial era 30 · 15 = 450cm2 e a área final ficou (30 − 6)(15 − 3) =
450 − 288
288cm2 , sua redução foi de = 0, 36 = 36%. Resposta C.
450

60
POTI 2015 − Geo Plana − Nı́vel 2 − Aula 0 − Professores Cleber Assis, Samuel Barbosa e Tiago Miranda

107.

a) A área hachurada é a diferença entre as áreas do setor circular e do triângulo. Temos então

3 8 · 8 · sen 135◦
A = π · 82 −
8 √ 2
= 24π − 16 2

= 8(π − 2)cm2 .

b) Para o cálculo do raio r, utilizaremos o Teorema de Pitágoras no triângulo formado pela reta que passa
pelos centros das duas circunferências, pelo centro da circunferência menor e é perpendicular ao lado do
quadrado e pelo lado do quadrado como indica a figura abaixo.

Os lados deste triângulo são (6 + r ), (6 − r ) e (12 − r ). Temos, então

(6 + r )2 = (6 − r )2 + (12 − r )2
12r = −12r + 144 − 24r + r2
r2 − 48r + 144 = 0

r = 12(2 − 3)cm.
√ √
Assim, a área do cı́rculo menor é π [12(2 − 3)]2 = 144(7 − 4 3)cm2

c) Traçando um segmento pelos pontos de intersecção das circunferências, teremos dois segmentos circulares,
cujas áreas são a diferença entre as áreas dos setores circulares e dos triângulos gerados. A medida
do ângulo destes setores é 120◦ , pois pode-se formar dois triângulos equiláteros ligando os√centros das
circunferências e seus pontos de intersecção. A distância entre estes pontos de intersecção é 4 3cm, pois é
o dobro da altura de um dos triângulos. Temos, então

42 π 4 · 4 · sen 120◦
 
A = 2 −
3 2

16π √ 
= 2 −4 3
3

8π √  2
= 4 − 2 3 cm .
3

61
POTI 2015 − Geo Plana − Nı́vel 2 − Aula 0 − Professores Cleber Assis, Samuel Barbosa e Tiago Miranda

108. (Extraı́do do ENEM − 2012)


A área perdida é a diferença entre as áreas inicial e final. Temos, então

A = 15 − (5 − x )(3 − y)
= 15 − 15 + 3x + 5y − xy
= 5y + 3x − xy.
Resposta E.
109. (Extraı́do do ENEM − 2012)
Calculando a área ABPD, temos

[ ABPD ] = [ ABD ] − [ PBD ]


1 · 12 1 · 14
= −
2 2
1 1
= −
4 8
1 2
= m .
8
1 1 1 3
Consequentemente, a soma das áreas não sombreadas é 2 · = e a área restante é 1 − = . Temos, então,
8 4 4 4
1 3
que o custo é · 50 + · 30 = 12, 50 + 22, 50 = R$35, 00. Resposta B.
4 4
110. (Extraı́do da OBM − 2014) √ √
Como cada triângulo equilátero tem altura 23 , as diagonais do quadrado EFGH medem 1 + 3, e sua área

(1 + 3)2 √
pode ser calculada como = 2 + 3. Resposta C.
2

111. (Extraı́do da OBMEP − 2005)

a) Sendo a o lado de cada quadradinho, temos b) Sendo A a área do quadrado sombreado, temos

[ EFGH ] = [ ABCD ] − 4[ AEH ] A = [ EFGH ]/2


= 25a2 − 8a2 =
17
[ ABCD ]
17 50
= 17a2 = [ ABCD ] 136 2
25 = cm
5

Portanto,
√ o lado do quadrado sombreado é
2 170
cm.
5

62
POTI 2015 − Geo Plana − Nı́vel 2 − Aula 0 − Professores Cleber Assis, Samuel Barbosa e Tiago Miranda

112. (Extraı́do da OBMEP − 2005)

2·8
a) A área de cada canteiro de pedra para x = 2 vale = 8. Assim, a área do canteiro de grama é
2
100 − 4 · 8 = 68m2 .
x (10 − x )
b) A área de cada canteiro triangular é dada pela expressão . Assim, a área do canteiro de grama é
2
dada por:
100 − 4 · x (10 − x )2 = 2x2 − 20x + 100m2

c) Como a diferença entre os preços das coberturas de pedra e grama é de 1, o custo total é o mesmo que
gastar 3 por metro quadrado em todo o quadrado e 1 extra pela área dos canteiros de grama, ou seja, o
custo total é:
3 · 100 + 1 · (2x2 − 20x + 100) = 2x2 − 20x + 400.
Fatorando a expressão anterior, obtemos:

2x2 − 20x + 200 = 2( x − 5)2 + 350


≥ 02 + 350
= 350.

A igualdade ocorre apenas quando x = 5. Assim, o prefeito precisa de pelo menos R$150, 00 reais.

113. (Extraı́do da OBM − 2014)


Como a diagonal de um retângulo o divide em dois triângulos de mesma área, as áreas dos triângulos
sombreados são 8m2 e 18m2 . Observando, agora, o retângulo original, sua diagonal o dividiu em dois
triângulos, sendo um deles com área 24 + 8 + 18 = 50m2 , ou seja, a área do retângulo original é 100m2 .
Resposta D.

114. (Extraı́do da OBM − 2013)

a) Como ACBE = A EDC , pois possuem a mesma base e mesma altura, então, decompondo ambas as áreas,
A ABC + A ACE = A ADE + A ACE , segue que A ABC = A ADE .

b) Traçando o segmento GH, temos, pelo item anterior, que A AGH = A ABC = 5cm2 e A DGH = A DEF = 4cm2 .
Temos então que A AGDH = A AGH + A DGH = 5 + 4 = 9cm2 .

115. (Extraı́do da OBM − 2012)


Como a área total do terreno é 160 · 120 − 60 · 50 = 16200m2 , cada parte deverá ter 8100m2 . Calculando a área
do trapézio ABCP, temos

[ ABCP] = 8100
(120 − x + 50)100
= 8100
2
170 − x = 162
x = 8m.

Resposta B.

63
POTI 2015 − Geo Plana − Nı́vel 2 − Aula 0 − Professores Cleber Assis, Samuel Barbosa e Tiago Miranda

116. (Extraı́do da OBM − 2012) √


Se P o pé da altura do 4 DEC no lado DC, então EP = 4 3. Temos que

[ AEC ] = [ AECD ] − [ ACD ]


= [ AEPD ] + [ ECP] − [ ACD ]
√ √
4(6 + 4 3) 4 · 4 3
= + − 24
2√ √ 2
= 12 + 8 3 + 8 3 − 24

= 16 3 − 12

= 4(4 3 − 3).
117. (Extraı́do da EPCAR − 2014)
Como os arcos são congruentes, o 4 ABC é equilátero, sendo 120◦ a medida do seu ângulo central e a medida
do raio da circunferência maior o dobro do raio da menor, ou seja, 2cm. Dividiremos a área hachurada em três
partes:

i) área de 2/3 do cı́rculo menor, que é 2π/3;


4π √
ii) área do segmento do cı́rculo maior, que é − 3;
3
iii) área do quadrilátero formado por
√ dois segmentos perpendiculares aos lados do triângulo partindo do
centro do cı́rculo menor, que é 3.
4π √ √
Portanto, a área hachurada é 2π/3 + − 3 + 3 = 2π. Resposta A.
3
118. (Adaptado da EPCAR − 2013)
Sendo cada uma das cinco partes hachuradas a área de√um segmento circular de uma circunferência de raio
a2 π a2 3
que mede a, temos que a área hachurada é 5( − )
6 4
AB · GE
119. A área do triângulo ABE é = 72. Assim, aplicando a mesma fórmula de área para a base BE e
2
a altura AF, temos:
AF · BE AF · 9
72 = = = 4, 5AF.
2 2
72
Portanto, o comprimento de AF é = 16.
4, 5

64
POTI 2015 − Geo Plana − Nı́vel 2 − Aula 0 − Professores Cleber Assis, Samuel Barbosa e Tiago Miranda

120. Como FH = GE, temos HO = FO − FH = OE − GE = OG. Consequentemente o semicı́rculo de


diâmetro HO possui a mesma área do semicı́rculo de diâmetro OG. Além disso, a área entre os arcos FG e
HO é igual à área entre os arcos GO e HE. Consequentemente, a área procurada corresponde a área de um
semicı́rculo de diâmetro FE. Como o raio do semicı́rculo de diâmetro FE mede 1, a área sombreada mede
22 π
= 2π.
2

121. (Extraı́do da OBM − 2009)


Temos ∠ ALK = 180◦ − ∠KLM − ∠ BLM = 180◦ − 90◦ − ∠ BLM = 90◦ − ∠ BLM = ∠ BLM, ambos os ângulos
∠KAL e ∠ LBM são retos. Como KL = LM, segue que os triângulos KAL e LBM são congruentes pelo caso
LAAo . Portanto, sendo x = AK, AL = 4 − x, LB = x e BM = AL = 4 − x. Logo a área do trapézio AKMB é
AK + BM x + (4 − x )
igual a · AB = · 4 = 8 e, consequentemente, a área de CDKM é 42 − 8 = 8. Resposta B
2 2
Observação: De fato, os trapézios AKMB e KMCD são iguais.

122. Seja h a distância entre as duas retas. Este também é o valor da altura dos triângulos 4 EFG e 4 H I J.
Assim
[ EFG ] h · EG/2
=
[H I J] h · I J/2
EG
=
IJ
1
= .
2
123.
p √
a) Como o semiperı́metro mede 10cm, temos A = 10(10 − 5)(10 − 7)(10 − 8) = 10 3cm2 .
p √
b) Como o semiperı́metro mede 9cm, temos A = 9(9 − 6)(9 − 5)(9 − 7) = 6 6cm2 .
p √
c) Como o semiperı́metro mede 18cm, temos A = 18(18 − 12)(18 − 10)(18 − 14) = 24 6cm2 .
p √
d) Como o semiperı́metro mede 9cm, temos A = 14(14 − 10)(14 − 10)(14 − 8) = 8 21cm2 .
p
124. Como o semiperı́metro mede 21cm, portanto, A = 21(21 − 13)(21 − 14)(21 − 15) = 84cm2 .

65
POTI 2015 − Geo Plana − Nı́vel 2 − Aula 0 − Professores Cleber Assis, Samuel Barbosa e Tiago Miranda

125. (Extraı́do da OBM 2014) Sendo Q o ponto de intersecção da bissetriz de ∠ BAC com o lado BC, temos
5 6 BQ 6
que, pelo Teorema da Bissetriz Interna, = , ou seja, = . Temos também que
CQ BQ CQ 5
BQ [ ABQ]
=
CQ [ ACQ]
[ BPQ]
=
[CPQ]
[ ABQ] − [ BPQ]
=
[ ACQ] − [CPQ]
[ ABP]
=
[ ACP]
3
2 6
= =
[ APC ] 5
Assim, [ APC ] = 5/4. Resposta A.
BD · h DC · h
126. Se h é a altura relativa ao lado BC, temos 30 = e 10 = . Portanto,
2 2
30 BD · h BD
3= = = .
10 DC · h DC
127.
a) Como D é ponto médio de AC, segue que [ ABD ] = [ BDC ] = [ ABC ]/2. Além disso, como E é ponto médio
de BC, seque que [ BED ] = [ BDC ]/2 = [ ABC ]/4.
AG [ AGD ] [ ABG ]
b) Considere os triângulos da figura de bases AG e GE, assim = = .
GE [ GDE] [ BGE]
Consequentemente, usando propriedades de proporções, temos:
AG [ AGD ] + [ ABG ]
=
GE [ GDE] + [ BGE]
[ ABD ]
=
[ BDE]
[ ABC ]/2
=
[ ABC ]/4
= 2.
128. Trace o segmento ED. O triângulo EAD possui metade da área do paralelogramo ABCD pois possui a
mesma base e a mesma altura. Pelo mesmo argumento, também possui metade da área do paralelogramo
EFGA. Assim, as áreas de ambos paralelogramos são iguais a 20cm2 .

66
POTI 2015 − Geo Plana − Nı́vel 2 − Aula 0 − Professores Cleber Assis, Samuel Barbosa e Tiago Miranda

129.
1
a) Os triângulos 4 ADE e 4 ABC possuem mesma altura mas a razão entre suas bases é . Portanto,
4
1 2
[ ADE] = · [ ABC ] = 9cm .
4
2 2
b) Como DC = · BC, temos [ ADC ] = · [ ABC ] = 24cm2 .Como M é o ponto médio de AD, as áreas dos
3 3
triângulos 4 AMC e 4 MDC são iguais e valem metade da área do 4 ADC. Portanto, [ MDC ] = 12cm2 .
2 2 3
c) Como DC = · BC, temos [ ADC ] = · [ ABC ] = 24cm2 . Além disso, como GC = · AC, segue que
3 3 4
3 3 2
[ GDC ] = · [ ADC ] = · 24 = 18cm .
4 4
d) Em virtude do item anterior, [ GDC ] = 18cm2 . Como E é ponto médio de DC, segue que [ GDE] = [ GEC ] =
1 1 1
9. Também temos GF = · GC e daı́ [ GEF ] = · [ GEC ] = · 9 = 3cm2 . Portanto,
3 3 3

[ GDEC ] = [ GDE] + [ GEF ] = 9 + 3 = 12cm2 .

130.

a) O triângulo sombreado possui a mesma base e al- d) O paralelogramo ABCD pode ser dividido em
tura que o paralelogramo dado. Portanto, sua área três paralelogramos congruentes à BPSA. Como
24 sua área vale 24, a área do paralelogramo PCDS
vale = 12cm2 .
2 [ PDC ] 8
vale 16cm2 . Além disso, [ PQC ] = = e
AD CD 3 3
b) Como AQ = e PC = , segue que [SQD ]
3 2 [ RQD ] = = 3. Portanto, teremos que
[ ABD ] [ BDC ] 2
[ ABQ] = e [ BCP] = . Portanto,
3 2
como a diagonal BD divide o paralelogramo em [ PQRS] = [ PCDS] − [ PCQ] − [ QDR]
dois triângulos de mesma área, temos 8
= 16 − − 3
[ ABD ] [ BDC ] 3
[ BQDP] = [ ABCD ] − − 31
3 2 = .
[ ABCD ] [ ABCD ] 13
= [ ABCD ] − −
6 4
2
= 14cm [ ABD ]
e) Temos [ ABQ] = = 3cm2 e
2[ ABD ] 4
c) Temos [ BAR] = = 8cm2 e 3[ BCD ]
3 [ BCP] = = 9cm2 . Além disso,
[ BDC ] 4
[ BPC ] = = 3cm2 . Além disso,
4
2[ APD ]
[ AQD ] [ ACD ] [ PQD ] = = 2cm2 .
[ RDQ] = = = 1cm . 2 3
3 12
Consequentemente, Portanto,

[ BPQR] = [ ABCD ] − [ ABR] − [ BPC ] − [ RQD ]


[ BPQ] = [ ABCD ] − [ BCP] − [ PDQ] − [ ABQ]
= 24 − 8 − 3 − 1
= 24 − 9 − 2 − 3
= 12cm2 .
= 10cm2 .

67
POTI 2015 − Geo Plana − Nı́vel 2 − Aula 0 − Professores Cleber Assis, Samuel Barbosa e Tiago Miranda

131. Pelo exercı́cio anterior,


[ DCB] [ ACB]
[ PCB] = +
2 2
6 · 4 10 · 4
= +
4 4
= 6 + 10
= 16cm2 .
[ PCB]
Como Q é o ponto médio de PB, segue que [ PQC ] = = 8cm2 .
2
132. Pela fórmula de Brahmagupta, sua área é dada por:
q
A = ( p − a)( p − b)( p − c)( p − d)
q
= (88 − 39)(88 − 25)(88 − 60)(88 − 52)

= 49 · 63 · 28 · 36
= 1764.
133. Sejam h1 , h2 e h3 as distâncias dos vértices B, E e C ao lado AD, respectivamente.

h1 + h2
Como E é ponto médio de BC, temos h3 = . Assim
2
h3 · AD
[ EAD ] =
2
(h1 + h2 ) AD
=
4
h1 · AD h2 · AD
= +
4 4
[ ABD ] [ ACD ]
= +
2 2
= 10 + 15
= 25.
134.
a) Como DF = 1, segue que DE = DF cos β = cos β e EF = DF sen β = sen β. Além disso, dado que
∠ ADE = α, obtemos ∠ BEF = 90◦ − α. Assim, AE = DE sen α = sen α cos β e EB = EF cos α = sen β cos α.
Como ∠ FDC = 90◦ − α − β, segue que DC = DF cos(90◦ − α − β) = sin(α + β).

b) Como AE + EB = AB = DC, substituindo os valores encontrados no item anterior, obtemos o resultado


desejado.

68
POTI 2015 − Geo Plana − Nı́vel 2 − Aula 0 − Professores Cleber Assis, Samuel Barbosa e Tiago Miranda

135.

a) Pela fórmula de Heron, temos: b) Sejam x = p − b e y = p − c. Daı́, como ( x − y)2 ≥


0, segue que
a · ha
q
p( p − a)( p − b)( p − c) = [ ABC ] =
2 x2 − 2xy + y2 ≥ 0
2
p
p( p − a)( p − b)( p − c) x2 + 2xy + x2 ≥ 4xy
Consequentemente, h a = . √
a x + y ≥ 2 xy.

Como x + y = 2p − b − c = a, temos
p
2 ( p − b)( p − c)
q
ha = p( p − a) ·
a
a
q
≤ p( p − a) ·
q a
= p ( p − a ).

Observação: Neste item, demonstramos também a desigualdade entre as Médias Aritmética e Geométrica
para dois termos:
x+y √
≥ xy se x, y ≥ 0.
2
136. (Extraı́do da OBM − 2012)

Considerando o paralelogramo ASDT, como


2AB
AT = , temos que a área de ASDT é igual a
3
2
· 84 = 56. Este paralelogramo está dividido em oito
3
paralelogramos iguais, sendo que a área sombreada é
um destes paralelogramos e, portanto, a área desejada
1
é · 56 = 7.
8

137. (Extraı́do da OBM − 2011)


Seja M o ponto médio de BC. Então, como ABC é isósceles com AB = AC o segmento AM é também altura
do triângulo. Logo, s
 2
p
2 2 2
6
AM = AB − BM = 5 − = 4.
2
3
Como a altura da água é 3, o nı́vel da água é igual a da altura do triângulo. Como os triângulos pequenos
4
brancos formados pelos espaços são semelhantes ao triângulo original com a mesma razão de semelhança
3
(raiz quadrada da razão entre as áreas, que é a mesma), a altura h é igual a da altura relativa H a B. Sendo
4
a área de ABC igual a 62·4 = 12cm2 , temos
AC · H
= 12
2
5H = 24
24
H = .
5
3 3 24 18
Portanto, h = H= · = cm. Resposta D.
4 4 5 5

69
POTI 2015 − Geo Plana − Nı́vel 2 − Aula 0 − Professores Cleber Assis, Samuel Barbosa e Tiago Miranda

138. (Extraı́do da OBM − 2009)


Trace os segmentos AD e AF. Como BD = DF = FC, temos
252
[ ABD ] = [ ADF ] = [ AFC ] = .
3
Além disso, como E é ponto médio de AB, obtemos:
[ ABD ] 252
[ BDE] = =
2 6

252
Analogamente, como G é ponto médio de AC, [ GFC ] = . Portanto,
6
2 · 252
[ AEDFG ] = [ ABC ] − [ BDE] − [ GFC ] = = 168.
3
Resposta A.

139. (Extraı́do da OBM − 2014)


A razão EG/GD pode ser calculada através das razões de áreas:
EG [ EGB] [ EFG ] [ EGB] + [ EFG ] [ EFB]
= = = = .
GD [ GDB] [ FDG ] [ GDB] + [ FDG ] [ FDB]
Além disso, temos:
[ EFB] [ EFB] [CFB] 2 2 4
= · = · = .
ABC CFB [ ABC ] 3 3 9
Analogamente,
[ FBD ] 1
=
[ ABC ] 9
Portanto
EG [ EFB] [ ABC ] 4
= · = · 9 = 4.
GD [ ABC ] [ FDB] 9

140. a ) F b )V c ) F d )V e ) F

141. Como 2x + 3 = x + 11, temos x = 8. Além disso, y + 40◦ = 3y + 20◦ implica que y = 10◦ .

142. a) x = 80◦ . b) x = 45◦ . c) x = 50◦ . d) x = 60◦ .

143.
a) Como y + 70◦ + 30◦ = 180◦ , temos y = 80◦ . Além disso, x + 80◦ + 60◦ = 180◦ implica que x = 40◦ .
b) Como x + x + 30◦ = 180◦ , temos x = 75◦ . Além disso, pelo teorema do ângulo externo, x = 75◦ = 20◦ + y.
Daı́, y = 55◦ .

70
POTI 2015 − Geo Plana − Nı́vel 2 − Aula 0 − Professores Cleber Assis, Samuel Barbosa e Tiago Miranda

144.

a) obtusângulo, pois ∠ BAC = 100◦ .

b) acutângulo, pois ∠ BAC = 85◦ .

c) retângulo, pois ∠ BAC = 90◦ .

d) acutângulo (e também equilátero), pois ∠ BAC = 60◦ .

145. Pelo teorema do ângulo externo, ∠ BOA = 130◦ − 30◦ = 100◦ . Além disso, ∠COD = ∠ BOA = 100◦ , pois
são ângulos opostos pelo vértice. Assim, x + 100◦ + 40◦ = 180◦ , ou seja, x = 40◦ .

146. Analisando a soma dos ângulos do triângulo 4 ABD, temos ∠ ADB + 20◦ + 90◦ = 180◦ , ou seja,
∠ ADB = 70◦ . Como AD é paralelo a BC, segue que ∠EBC = 70◦ . Finalmente, analisando a soma dos ângulos
do triângulo 4 BEC, temos 70◦ + 60◦ + x = 180◦ . Assim, x = 50◦ .

147. Temos:

i) 2x − 1 = 7, então x = 4.

ii) x + y = 7, então y = 3.

148.

i) Os triângulos 4 ABC e 4 EFD, pelo caso LLL;

ii) Os triângulos 4 GH I e 4 JKL, pelo caso LAL.

149. Como os triângulos 4 AED e 4CEB são congruentes com CE = ED e ∠ECB = ∠EDA, então EB = AE
e CB = AD, ou seja, y = 7 e 2x + 5 = 13. A última igualdade produz x = 4.

150. Temos: a) Incentro; b) Ortocentro; c) Baricentro e d) Circuncentro.

151. Como AB = BC, temos 5x = 3x + 2, ou seja, x = 1. De AC = 2AD, segue que x + y = 8x e


consequentemente y = 7.

152. Temos: BM é mediana, AE é altura, CD é bissetriz e HM é mediatriz.


153. Se ∠ ACB = 40◦ e AC = BC, então ∠CAB = ∠CBA = 70◦ . Assim, ∠ ABD = 180◦ − 70◦ − 90◦ = 20◦ .
154. Se ∠ BCA = 30◦ , então ∠ BAC = 180◦ − 90◦ − 30◦ = 60◦ . Assim, ∠ ABD = 180◦ − 90◦ − 60◦ = 30◦ .
155. Como ∠ ACB = 30◦ , então ∠CAB = 60◦ e, por consequência, ∠ ABD = 30◦ . Assim, ∠ DBE =
45◦ − ∠ ABD = 15◦ .
156. Dividindo os ângulos internos do triângulo em dois ângulos congruentes a α no vértice A, dois
congruentes a β no vértice B e dois congruentes a γ no vértice C, tem-se α + β = 70◦ , α + γ = 50◦ , e
β + γ = 60◦ . Daı́, resolvendo o sistema formado por essas equações, α = 30◦ , β = 40◦ e γ = 20◦ . Assim, temos
∠ BAC = 60◦ , ∠CBA = 80◦ e ∠ ACB = 40◦ .
157. Como 2x + 6◦ = 36◦ + x, temos x = 30◦ .
158. O triângulo 4 ABE é isósceles pois AB = BE. Seja α o seu ângulo da base. Pelo teorema do ângulo
externo aplicado ao triângulo 4 ABE com respeito ao ângulo externo ∠EBD, temos α + α = ∠EBD = 60◦ .
Portanto, α = 30◦ .
159. Temos 7β = 70◦ e 2α = 50◦ , ou seja, β = 10◦ e α = 25◦ .

71
POTI 2015 − Geo Plana − Nı́vel 2 − Aula 0 − Professores Cleber Assis, Samuel Barbosa e Tiago Miranda

160. Pelo caso ALA, 4ODC ≡ 4OBE e daı́ OD = OB = 5cm. Como o triângulo 4 DOB é isósceles com
ângulo do vértice medindo 60◦ , temos ∠ODB = ∠ DBO = 60◦ e consequentemente 4ODB é equilátero.
Assim, DB = OB = 5cm.
161.

a) ∠ ACB = 180◦ − 60◦ − 80◦ = 40◦ ;

b) Como BE e GD são bissetrizes, ∠ DCB = 20◦ e ∠ DBC = 40◦ ;

c) Analisando a soma dos ângulos do triângulo 4 DBC, temos ∠ BDC = 180◦ − 40◦ − 20◦ = 120◦ .

162. Como BD e AE são medianas e AD = DE, tem-se BE = ED = AD = DC = 8cm.


163.

a) LAL;

b) LAAo ;

c) LLL;

d) ALA.

164. Como 4CDE é isósceles, ∠ DEC = 80◦ = ∠ AEB, pois são ângulos da base. Assim, como ∠ ABE é
isósceles, α = 50◦ . De forma análoga, β = 40◦ . Portanto, αβ = 54 .

165. Como 4CEF é equilátero, ∠ FCD = 180◦ = 75◦ − 60◦ = 45◦ . De forma análoga, ∠ MDC = 55◦ . Assim, o
ângulo determinado pela intersecção das retas MD e CF é 80◦ . Portanto, x = 180◦ − 60◦ − 80◦ = 40◦ .

166. (Extraı́do da OBM 2011)


Como 4 BAD e 4 AEC são isósceles, segue que ∠ BAD = ∠ BDA e ∠CAE = ∠CEA. Tem-se ainda que
∠ BAC = ∠ BAD + ∠CAE − 40◦ = ∠ BDA + ∠CEA − 40◦ = (180◦ − 40◦ ) − 40◦ = 100◦ .

167. (Extraı́do da OBM 2008)


Como ∠ ABC = 110◦ , então ∠ AOC = 140◦ e com isso ∠OAC = 20◦ . Por outro lado, ∠ I AC = 10◦ . Portanto,
∠ I AO = 30◦ .
168. (Extraı́do da OBM 2006)
Pelo teorema do ângulo externo, ∠ ADE + x = 30◦ + ∠ ABD, portanto ∠ ADE = ∠ AED = 30◦ + ∠ ABD − x.
Além disso, ∠ AED = x + ∠ ACD. Igualando as duas equações e usando que ∠ ABC = ∠ ACB, temos
30◦ + ∠ ABD − x = x + ∠ ACD, ou seja, x = 15◦ .

169. (Extraı́do da OBM - 2014)


Como P pertence à bissetriz, sua distância do lado AB é a mesma do lado AC. Se a área do 4 APB é 32 , então
6· h 3 1 5 1 5
2 = 2 , obtendo h = 2 . Assim a área do 4 APC = 2 · 2 = 4 .

170. (Extraı́do da OBMEP - 2005)


Como 4 ABC é isósceles, temos ∠ ABC = ∠ ACB = α. Como 4 BCD é isósceles, também temos ∠ BDC =
∠ DBC = α. Se ∠ DCA = β, tem-se, pelo teorema do ângulo externo, que α = 30◦ + β. Analisando a soma
dos ângulos do 4 BCD, α + α + α − β = 180◦ . Assim, pelas duas equações tem-se 2β + 90◦ = 180◦ , ou seja,
β = 45◦ .

72
POTI 2015 − Geo Plana − Nı́vel 2 − Aula 0 − Professores Cleber Assis, Samuel Barbosa e Tiago Miranda

171. (Extraı́do da OBMEP − 2008)


Pelo teorema do ângulo externo e usando que o triângulo 4CAD é isósceles, ∠ ACB = ∠CAD + ∠CDA =
2 · ∠CDA = 96◦ . Do mesmo modo obtém-se ∠CBA = 2 · ∠ DEA e ∠ BAC = 2 · FEA. Somando as três
igualdades,

180◦ = ∠ ABC + ∠ BCA + ∠CAB


= 96◦ + 2 · ∠ DEA + 2 · ∠ FEA
= 96◦ + 2∠ DEF.

Ou seja, ∠ DEF = 42◦ .

172. (Extraı́do da OBM - 2011)


Sejam ∠ BAC = α e ∠ BCA = 2β, tem-se ∠ ABC = α + 50◦ . Pelo teorema do ângulo externo no triângulo
4 DEC, ∠ AED = 90◦ + β. Analisando a soma dos ângulos dos triângulos 4 ABC, temos

α + (α + 50◦ ) + 2β = 180◦ ,

ou seja, α + β = 65◦ . Finalmente, analisando a soma dos ângulos do triângulo 4 EAD, temos

x + α + (90 + β) = 180◦

e consequentemente x = 90◦ − (α + β) = 25◦ .

173. (Extraı́do do Torneio das Cidades)


Seja ∠CAB = 2α. Assim, ∠CBA = 90◦ − 2α. Os ângulos da base dos triângulos isósceles 4 ACN e 4CMB
valem 90◦ − α e 45◦ + α, respectivamente. Assim, ∠ MCN = 180◦ − ∠CMN − ∠CN M = α + (45◦ − α) = 45◦ .

174. (Extraı́do da AIME)


Chamando a intersecção entre AF e ED de G e a intersecção de AF e BC de H, temos pelo teorema do ângulo
externo que ∠ DGH = E + F e ∠CHG = A + B. Analisando a soma dos ângulos do quadrilátero DCHG,
temos A + B + C + D + E + F = 360, ou seja, n = 4.

175. (Extraı́do da Olimpı́ada de Leningrado)


As igualdades fornecidas implicam que os triângulos 4 BKC e 4CHB são congruentes pelo caso LAL. Assim,
temos CK = HB e BK ⊥ KC. Consequentemente BK é uma altura e 4 ABC é isósceles com AB = BC. Além
disso, ∠ HBC = ∠KCB implicando que AB = AC. Como os três lados são iguais a AB, o triângulo é equilátero.

176. Pelo vértice B, trace uma reta paralela ao lado AC. Tal reta forma dois pares de ângulos alternos internos
de valores α e β. Como a soma dos três ângulos incidentes no vértice B é um ângulo raso, temos:

∠ BAC + ∠ ACB + ∠CBA = α + β + γ = 180◦ .

73
POTI 2015 − Geo Plana − Nı́vel 2 − Aula 0 − Professores Cleber Assis, Samuel Barbosa e Tiago Miranda

177. Suponha, por absurdo, que o triângulo 4 LNP é equilátero. Teremos:


∠ BCA
= 90◦ − ∠ HNC = 30◦ .
2
Além disso,
∠ BCA
∠ BMC = ∠ LPN − = 90◦ .
2
Como BM é altura e mediana, concluı́mos que 4 ABC é isósceles. Sendo ∠ BCA = 60◦ , podemos concluir
que 4 ABC é de fato equilátero. Nesse caso, BM, CK e AH seriam concorrentes em um mesmo ponto e isso
produziria um absurdo pois estamos supondo que L, M e P são distintos.

178. Como B1 C2 é mediana do triângulo retângulo 4 BB1 A, então B1 C2 ≡ C2 A e, por consequência,


∠ AB1 C2 = ∠ B1 AC2 = 30◦ e ∠ B1 C2 A = 120◦ . De forma análoga, tem-se ∠ AB2 C1 = 120◦ e ∠ B2 C1 B = 120◦ .
Seja P a intersecção entre C1 B2 e B1 C2 . Analisando a soma dos ângulos do quadrilátero AB2 PC2 , temos
∠C2 PB2 = 360◦ − 30◦ − 120◦ − 120◦ = 90◦ , ou seja, B1 C2 é perpendicular a C1 B2 .
179. (Extraı́do do Torneio das Cidades)
Supondo FE > AD, sem perda de generalidade. Como FD é base média de AC e ED é base média de AB,
AEDF é paralelogramo, de diagonais AD e FE, cuja soma de dois vértices consecutivos quaisquer é 180◦ .
Como, por hipótese, FE > AD, ∠ FAE > ∠ AFD e se são suplementares, ∠ FAE > 180◦ , ou seja, 4 ABC é
obtusângulo.

180. (Extraı́do da AIME)


Sejam S a soma dos ângulos nas n ”pontas”e R a soma dos outros 2n ângulos internos dos triângulos
que contêm estas ”pontas”. O polı́gono desenhado tem soma dos ângulos externos dado por 360◦ . Como
os ângulos que compõem R são os ângulos externos do polı́gono contados exatamente duas vezes, temos
R = 2 · 360◦ . A soma dos ângulos internos dos n triângulos que contém as pontas vale 180◦ n. Assim,
180◦ n = S + R = S + 720◦ implica que R = 180◦ (n − 4).

181. (Extraı́do do Torneio das Cidades)


Dessa vez Pinóquio não está mentindo. Abaixo é exibido um exemplo com 8 triângulos possuindo os ângulos
30◦ e 120◦ .

182. Seja Z o ponto de interseção do prolongamento de AP e BC. O triângulo 4 ABZ é isósceles, pois o
segmento AP é altura e bissetriz. Logo, BZ = AB = 6 e consequentemente ZC = BC − BZ = 10 − 6 = 4.
Como o triângulo 4 ABZ é isósceles, BP é altura, bissetriz e mediana. Logo P é o ponto médio de AZ. Como
M já é o ponto médio de AC, PM é a base média no triângulo 4 AZC, ou seja, PM = 2.
183.
Como 4 ABC é isósceles e ∠ BAC = 30◦ , ∠ ABC = ∠ ACB = 75◦ . Sejam ∠ PBQ = β, ∠ PBC = θ e E a
interseção de BP com QC. Como PD é mediana e altura do 4 BPC, temos PC = PB. Assim, os triângulos
4 PQB e 4 PBC são isósceles com ângulos das bases β e θ. Pelo teorema do ângulo externo, segue que
∠EPQ = 2β e ∠EPC = 2θ. Assim, ∠QPC = 2β + 2θ = 2∠QBC = 150◦ . Finalmente, como 4 QPC também é
isósceles de base PQ, segue que ∠ PQC = 15◦ .

74
POTI 2015 − Geo Plana − Nı́vel 2 − Aula 0 − Professores Cleber Assis, Samuel Barbosa e Tiago Miranda

184. Chamando a medida da hipotenusa de a, temos

a) c) e)
2 2 2 2 2 2 √ √
a = 3 +4 a = 1 +1 2 2
2 2 a2 = 3 + 5
a = 25 a = 2
√ a2 = 8
a = 5. a = 2. √
a = 2 2.
b) d)
a2 = 52 + 122 a2 = (1/2)2 + (3/2)2
a2 = 169 a2 = 10/4

a = 13. a = 10/2.
185. Usando as relações métricas no triângulo retângulo, temos que 42 = 5 · y, segue que y = 16/5 e, por
consequência, x = 5 − 16/5 = 9/5.

186. Aplicando as relações métricas no triângulo retângulo, temos 3(k − 3) = 42 . Segue que k = 25/3.

187. Usando as relações métricas no triângulo retângulo, temos, inicialmente, y2 = 9 · 16, segue que y = 12.
Aplicando agora o Teorema de Pitágoras, temos x2 = 162 + 122 e z2 = 92 + 122 . Segue que x = 20 e z = 15.

188. (Extraı́do da Vı́deo Aula)


Inicialmente, traçamos a altura h deste triângulo e obtemos dois triângulos retângulos de hipotenusa medindo
l e catetos medindo h e l/2. Basta agora aplicar o Teorema de Pitágoras.

h2 + (l/2)2 = l 2
h2 = 3l 2 /4

3
h = .
2
189. (Extraı́do da Vı́deo Aula)
Traçando uma diagonal no quadrado, dois triângulos retângulos se formam, nos quais seus catetos medem l e
sua hipotenusa mede d. Basta agora aplicar o Teorema de Pitágoras.

d2 = l 2 + l 2

d = l 2.
190. (Extraı́do da Vı́deo Aula)

Chamando os centros das circunferências de E e H Aplicando o Teorema de Pitágoras, obtemos


e traçando os segmentos AE, BH, HE e HC, sendo
C o pé do segmento perpendicular a AE, obtemos a EC2 + HC2 = EH 2
seguinte figura. ( AE − AC )2 + AB2 = EH 2
1 + AB2 = 52

AB = 24

AB = 2 6.

Figura 17

75
POTI 2015 − Geo Plana − Nı́vel 2 − Aula 0 − Professores Cleber Assis, Samuel Barbosa e Tiago Miranda

191. (Extraı́do da Vı́deo Aula)


Traçando a altura BD relativa ao lado AC, podemos observar que ∠ ABD = 45o , segue que AD = BD.
Aplicando o Teorema de Pitágoras no triângulo 4 ABD, obtemos AD = BD = 7. O triângulo 4 BCD
possui os ângulos internos medindo 30◦ , 60◦ e 90◦ , ou seja, BC = 2BD = 14, pois o cateto menor é
a metade da hipotenusa em triângulos com estas medidas de ângulos,√ conforme visto na Vı́deo Aula.
Aplicando agora √ o Teorema√ de Pitágoras
√ no 4
√ BCD, obtemos DC = 7 3. Por fim, o perı́metro do triângulo
4 ABC = 7 + 7 2 + 14 + 7 3 = 7(3 + 2 + 3).

Figura 19

192. (Extraı́do da Vı́deo Aula)


Vamos fazer inicialmente AE = x, O0 C = y e AO0 = 2r. Agora, construiremos dois triângulos retângulos
utilizando os pontos A, B, D, E e O0 , conforme a figura abaixo.

Figura 21

Usando as relações métricas no 4 AO0 D, temos 2r (2r − y) = (7 2)2 = 98 (1). Fazendo o mesmo no 4 ABE,
temos x2 = 4r (2r − y) (2). Dividindo a equação (2) pela equação (1), obtemos x2 = 196, segue que x = 14.

193. (Extraı́do da Vı́deo Aula)


Traçando BO e DO, temos o triângulo isósceles 4 BDO, pois BO = DO = 4 + x. Traçando agora CO, obtemos
dois triângulos retângulos, 4 BCO e 4 DCO. Aplicando o Teorema de Pitágoras em um desses triângulos,
chegamos a

(4 + x )2 = 42 + (8 − x )2
16 + 8x + x2 = 16 + 64 − 16x + x2
24x = 64
x = 8/3cm.

76
POTI 2015 − Geo Plana − Nı́vel 2 − Aula 0 − Professores Cleber Assis, Samuel Barbosa e Tiago Miranda

194. (Extraı́do da Vı́deo Aula)



Marcando o ponto H sobre AC, tal que GH seja√ perpendicular a AC. Se ∠GCH = 60 e, por consequência,
CG x x 3
∠CGH = 30◦ , então CH = = e GH = . Temos ainda que 4 AGH e 4 AEF são semelhantes, o que
2 2 √ 2
x 3 √
GH EF 2 1 2+4 3
implica que
AH
=
AF
, ou seja, √ x = 2 , segue que CG = x = 1 + 2√3 = 2.
1+2 3−
2

Figura 24
195. (Extraı́do da Vı́deo Aula)
Se AB = y, então AC = y + 6. Aplicando o Teorema de Pitágoras no 4 ABC, temos
( y + 6)2 + y2 = 302
y2 + 12y + 36 + y2 = 900
y2 + 6y − 432 = 0
y = 18cm.
Tomamos apenas o valor positivo de y. Chamando AD de z, então CD = 24 − z. Aplicando o Teorema da
z 24 − z
Bissetriz Interna, temos = , segue que z = 9. Usando agora o Teorema de Pitágoras no triângulo
18√ 30
ABD, chegamos a BD = 9 5cm.
196. (Extraı́do da OBMEP − 2013)
Sejam r e R, respectivamente, os raios das circunferências menor e maior, e S o centro da circunferência menor.
Notamos primeiro que 2r = PB = AB − 4 = 2R − 4, donde tiramos R = r + 2. No triângulo retângulo SOQ
temos SQ = r, OQ = OC − 3 = R − 3 = r − 1 e OS = OB − SB = R − r = 2. O Teorema de Pitágoras nos dá
r2 = (r − 1)2 + 22 = r2 − 2r + 5 e segue que 2r = 5, ou seja, r = 5/2 = 2, 5. Resposta B.

Figura 27

77
POTI 2015 − Geo Plana − Nı́vel 2 − Aula 0 − Professores Cleber Assis, Samuel Barbosa e Tiago Miranda

197. (Extraı́do da OBMEP)


Notamos primeiro que o triângulo PQR é equilátero de lado 2cm. Como o segmento RS também mede 2cm, o
triângulo PRS é isósceles de base PS. ∠ PRS mede 120◦ , pois ele é externo ao triângulo PRQ, igual à soma dos
dois ângulos internos não adjacentes, cada um medindo 60◦ . Logo cada um dos ângulos ∠ RSP e ∠ RPS mede
30◦ , e concluı́mos que o triângulo PQS é retângulo em P, com ∠ PQS = 60◦ e ∠ PSQ = 30◦ . Logo o triângulo
ABC é retângulo em A com ∠ ABC = 60◦ e ∠ ACB = 30◦ , pois seus lados são paralelos aos do triângulo PQS.
Além disso, seu menor lado é AB, oposto ao menor ângulo ∠ ACB = 30◦ . Para calcular o comprimento do
lado AB, basta calcular BT, pois claramente AT = 3cm. Notamos que o triângulo QBT é retângulo em T.
Como BQ é bissetriz de ∠ ABC,p segue que ∠√ TBQ = 30◦ . Como QT √ = 1cm, segue que BQ = 2cm, e o Teorema
2 2
de Pitágoras nos dá BT = QB − QT = 3, donde AB = 3 + 3. Resposta B.

Figura 29

198. (Extraı́do da OBM − 2013)

Como AC = 2, temos que AD = DC = 1. Pelo Teo-


rema de Pitágoras no triângulo DAB, temos AB2 =
32 − 12 = 8. Novamente pelo Teorema de Pitágora,
agora no triângulo ABC, temos BC2 = 22 + AB2 = 12.
Resposta E.

Figura 30
199. (Extraı́do da OBMEP)

Colocando AB = x, temos BC = x + 1 e AC = x + 2.
Seja AH = h a altura relativa a BC. Aplicando o
Teorema de Pitágoras aos triângulos AHB e AHC
obtemos n2 + h2 = x2 e ( x + 2)2 = m2 + h2 . Segue
que h2 = x2 − n2 e h2 = ( x + 2)2 − m2 , donde ( x +
2)2 − m2 = x2 − n2 , ou seja, ( x + 2)2 − x2 = m2 − n2 .
Usando a identidade a2 − b2 = ( a + b)( a − b), obtemos
então ( x + 2 − x )( x + 2 + x ) = (m + n)(m − n). Como
m + n = x + 1, segue que 2(2x + 2) = (m + n)(m − n),
donde 4( x + 1) = (m − n)( x + 1). Como x + 1 6= 0
podemos dividir ambos os membros desta última ex-
Figura 32 pressão por x + 1 e obtemos finalmente m − n = 4.
Resposta D.

78
POTI 2015 − Geo Plana − Nı́vel 2 − Aula 0 − Professores Cleber Assis, Samuel Barbosa e Tiago Miranda

200. (Extraı́do da OBM − 2012)


Podemos desenhar uma figura que representa a situação do problema:

Figura 33

Sabemos que, em um triângulo retângulo, o quadrado da altura relativa ao ângulo reto é igual ao produto
das projeções dos catetos sobre a hipotenusa (Relações Métricas no Triângulo Retângulo). Portanto, 9 = 2x,
9
segue que x = = 4, 5. Reposta D.
2
201. (Extraı́do da OBM − 2012)

Seja O o centro da semicircunferência descrita no


enunciado, P e Q os pontos como na figura e R o
ponto de tangência da semicircunferência com o lado
AB. Temos que OR = 1 e OR⊥ AB. Como O está

√ AC,√temos que ∠OAB = 45 . Assim,
na diagonal
OA = OR 2 = 2. Além disso, OC é altura e me-
diana relativa à hipotenusa no triângulo retângulo
PQC, cuja hipotenusa é 2. Assim,√OC = 1. Portanto,
a diagonal do quadrado√ vale 1 = 2 e daı́ sua área é
1 √ 2 3+2 2 3 √
· (1 + 2) = = + 2. Resposta A.
2 2 2

Figura 35
202.

a) Sim, pois ∠ BAC ≡ ∠ DAE e ∠CBA = ∠EDA = 90o (caso Ângulo-Ângulo).

b) Os lados homólogos são: AB e AD; AC e AE; e BC e DE.

c) Como ∠KLM = ∠QPM = 53o e ∠KML ≡ ∠QMP (opostos pelo vértice), então 4KLM ' 4 MPQ, pelo
caso AA.
20 14 12 10 7 6
203. Como = = = 2, a razão de semelhança é 2 ou = = = 1/2.
10 7 6 20 14 12

79
POTI 2015 − Geo Plana − Nı́vel 2 − Aula 0 − Professores Cleber Assis, Samuel Barbosa e Tiago Miranda

204. O triângulo formado por João e sua sombra e o triângulo formado pelo poste e sombra do mesmo são
semelhantes. Usando a razão de semelhança, temos

1, 6 x
=
2 7
2x = 11, 2
x = 5, 6.

Assim, a altura do poste é 5, 6m.

205. Como MNPQ é um quadrado, então PQ//MN//BC, o que implica que 4 ABC 4 APQ. Chamando o
lado do quadrado de x e aplicando a razão de semelhança, temos

12 x
=
8 8−x
8x = 96 − 12x
24
x = .
5
206. Traçando raios ligando os centros das circunferências aos pontos de tangências, obtemos a figura abaixo.

Figura 41

Perceba que 4 ABF 4 ACG. Chamando a distância entre os centros de x e aplicando a razão de semelhança,
temos

7 7+x
=
3 5
21 + 3x = 35
14
x = .
3
207. (Extraı́do da Vı́deo Aula)
Como ∠ECD ≡ ∠ ACB e ∠ ABC = ∠EDC = 90o , os triângulos 4 ABC e 4 EDC são semelhantes. Aplicando a
razão de semelhança, temos:

20 15
=
15 DE
20DE = 225
45
DE = .
4

80
POTI 2015 − Geo Plana − Nı́vel 2 − Aula 0 − Professores Cleber Assis, Samuel Barbosa e Tiago Miranda

208. (Extraı́do da Vı́deo Aula)


Como os triângulos 4CED e 4 DFB são semelhantes, pois ∠CED = ∠ DFB = 90o e ∠CDE ≡ ∠ DBF, vamos
aplicar a razão de semelhança, chamando o lado do quadrado de x. Temos então

4−x x
=
x 6−x
x2 = 24 − 10x + x2
12
x = .
5
48
Assim, temos que o perı́metro do quadrado AEDF é .
5
209. (Extraı́do da Vı́deo Aula)
Fazendo EF = x, FB = y, temos FM = 10 − y. Podemos observar a semelhança dos triângulos 4 ADB e
4 FEB, além dos triângulos 4CBM e 4 EFM. Aplicando a razão de semelhança em ambos os casos, obtemos

 y x

 =
o sistema 20 12
 10 − y = x


10 12 
 5x − 3y = 0


Simplificando, chegamos ao sistema equivalente

 5x + 6y = 60

Segue que EF = x = 4.

210. (Extraı́do da Vı́deo Aula) Nomeando alguns pontos importantes, obtemos a figura abaixo.

Figura 46

Como os triângulos 4 ABC e 4CDE são semelhantes, vamos aplicar a razão de semelhança.

AB BC
=
CD DE
9−x x
=
x−4 4
2
x − 4x = 36 − 4x
x2 = 36
x1 = −6
x2 = 6.

Como trata-se de comprimento de segmento, temos, como solução, apenas x = 6.

81
POTI 2015 − Geo Plana − Nı́vel 2 − Aula 0 − Professores Cleber Assis, Samuel Barbosa e Tiago Miranda

211. (Extraı́do da Vı́deo Aula)

Traçando o diâmetro AD e, em seguida, DC, obtemos Como ∠ ABH ≡ ∠ ABH e ∠ ADC são ângulos inscritos
a figura abaixo. que ”olham”para o mesmo arco, então eles são congru-
entes. Além disso, ∠ ACD = ∠ AHB = 90o e, portanto,
4 ACD 4 AHB. Aplicando a razão de semelhança e
chamando a medida do raio de r, temos

AD AC
=
AB AH
2r 12
=
10 4
8r = 120
r = 15.

Figura 48
212. (Extraı́do da Vı́deo Aula)
Traçando um raio de M até o ponto de tangência entre HB e a semicircunferência e chamando-o de D, temos
4 ABH 4 MBD, segue que AH = 2r, sendo r a medida do raio, pois M é ponto médio e MD é base média.
Aplicando o Teorema de Pitágoras aos triângulos 4 BH A e 4 ABH, chegamos ao sistema:

 HB2 + (10 − 2r )2 = 102


 HB2 + (2r )2 = 62

Resolvendo-o, obtemos r = 9/10cm.


213. (Extraı́do da Vı́deo Aula)
Traçando os segmentos BN e PK, sendo este perpendicular a AD, temos os triângulos retângulos 4 ABN e
4 AKP, que são semelhantes. Observe a figura.

Figura 51
Fazendo BC = CD = 2x, temos CK = KD = x, pois K é ponto médio. Como 4 ABN 4 AKP, temos a seguinte
razão de semelhança.

AP AK
=
AN AB
3AM 36 − x
=
2AM 36 − 4x
72 − 2x = 108 − 12x
x = 18/5.
Concluı́mos que CD = 36/5.

82
POTI 2015 − Geo Plana − Nı́vel 2 − Aula 0 − Professores Cleber Assis, Samuel Barbosa e Tiago Miranda

214. (Extraı́do da Vı́deo Aula)


Como DE//AC, então ∠ DEB = ∠ ACB = 2α. Pelo Teorema do Ângulo Externo, ∠CDE = α e, por
consequência, CE ≡ DE, pois 4CDE é isósceles. Como DE//AC, temos, pelo Teorema de Tales, que
AD CE AD BD
= (1). Além disso, pelo Teorema da Bissetriz Interna, temos = (2). Por (1) e (2), temos
DB EB AC BC
mn
DE = .
m+n
215. (Extraı́do da OBM − 2014)
Pelos pontos E e D, respectivamente, trace paralelas ao lado AC, determinando os pontos H e I sobre
HE HE EB 2x
o segmento FB. Seja AB = 3x. Temos 4 EHB 4CFB e 4 IDB 4 FAB, daı́: = = = e
2x FC BC 3x
ID ID DB x EG HE
= = = . Portanto, HE = 4x/3 e ID = x/3. Como 4 GID 4 HGE, segue que: = =
x FA AB 3x GD ID
4x/3
= 4.
x/3
216. (Extraı́do da OBM − 2013)
Trace a diagonal AC que intersecta DB no ponto O. Sendo ABCD um quadrado, O é o centro da circunferência.
Observe que ∠CMA = 90o e ∠ POA = ∠ DOA = 90o . Logo, pelo caso AA, os triângulos AOP e AMC são
AP AO AP 30
semelhantes e, portanto, = , é equivalente a = , ou seja, AP = 36.
AC AM 60 50

Figura 55

83
Programa Olímpico de Treinamento
Curso de Geometria - Nível 2 Aula 1
Prof. Rodrigo Pinheiro

Introdução

Nesta aula, aprenderemos conceitos iniciais de geometria e alguns teoremas básicos que
utilizaremos em todas as aulas seguintes. É importante o aluno perceber que os exercı́cios
olı́mpicos de geometria exigem muita criatividade, mas sem o conhecimento do colegial,
não há criatividade que resolva. Vamos assumir alguns conhecimentos básicos, que podem
ser encontrados em livros de geometria do colegial. Alguns teoremas enunciados abaixo
serão demonstrados posteriormente, em aulas futuras.

Teorema 1. A soma dos ângulos internos de um triângulo é 180◦ .


Demonstração.
E A D
b b b

b b

B C

Dado um triângulo ABC, tomamos a partir de A uma reta paralela a BC. Pelas
propriedades de paralelismo, temos que ∠EAB = ∠ABC e ∠DAC = ∠ACB. Como
∠EAD é um ângulo raso, temos que ∠EAD = 180◦ , podemos concluir que:

∠ABC + ∠BCA + ∠CAB = 180◦ .

Teorema 2. A medida de um ângulo externo de um triângulo é igual a soma das medidas


dos ângulos internos nâo adjacentes a ele.
Demonstração.
A
b

b b b

B
C D
POT 2012 - Geometria - Nı́vel 2 - Aula 1 - Prof. Rodrigo Pinheiro

Como a soma dos ângulos internos é 180◦ , então ∠ABC + ∠BCA + ∠CAB = 180◦ .
Mas na reta BD, temos que ∠BCA + ∠DCA = 180◦ . Assim,

∠ABC + ∠BCA + ∠CAB = ∠BCA + ∠DCA,


∠ABC + ∠CAB = ∠DCA.

Teorema 3. A soma de todos os ângulos internos de um polı́gono convexo de n lados é


180◦ · (n − 2)
Demonstração. A partir de um vértice do polı́gono, traçaremos todas as suas diagonais,
ou seja dividimos o polı́gono em n − 2 triângulos, portanto, a soma de todos os ângulos
internos do polı́gono é igual a soma de todos os ângulos internos de todos os triângulos que
é 180◦ · (n − 2).

Teorema 4. Dois lados de um triângulo são congruentes se, e somente se os ângulos opostos
a estes lados são congruentes.

Teorema 5. Em todo triângulo isósceles, a altura, mediana e bissetrizes relativas à base


são coincidentes.

Teorema 6. Dados dois lados distintos de um triângulo, o maior ângulo é oposto ao maior
lado.
Demonstração.

A
b

b b b

B C
D

Suponhamos BC > AC. Seja D o ponto sobre o lado BC tal que AC = CD. Por-
tanto, o triângulo ADC é isósceles. Pelo teorema anterior temos que, ∠CAD = ∠CDA.
Pelo teorema do ângulo externo temos que ∠CDA = ∠ABD + ∠DBA > ∠ABC. Como
∠BAC > ∠CAD = ∠CDA > ∠ABC, temos que ∠BAC > ∠ABC.

Teorema 7. A soma de dois lados quaisquer de um triângulo é maior que o terceiro lado.
Demonstração.

2
POT 2012 - Geometria - Nı́vel 2 - Aula 1 - Prof. Rodrigo Pinheiro

D
b

b
B

b b

A C

Seja D o ponto sobre o prolongamento BC, tal que BD = BA. Sendo assim, o triângulo
DBA é isósceles, portanto, ∠BAD = ∠BDA. Pela figura, percebemos que ∠CAD >
∠BAD = ∠BDA. Pelo teorema anterior, temos que CD > CA. Como BD = BA e
CB + BD = CD, podemos concluir que CB + BA > AC. Analogamente provamos para
os outros lados.

Problema 1. Paladino, num belo domingo à tarde decidiu se divertir com a bela geometria.
Ele pegou um triângulo, com três pontos distintos em seu interior, e traçou alguns segmento
entre esses pontos e os vértices do triânguo. Ele notou que dividiu a figura toda em
triângulos como mostrada abaixo.
B
b

b
E

D
b b

A C

Em todos os desenhos onde os segmentos não se cortavam e a figura foi dividida em


triângulos, sempre existiam 5 triângulos pequenos! Ele provou que em um triângulo, se
tomarmos n pontos em seu interior e triagularizarmos a figura unindo os pontos internos
sem cruzamento dos segmentos, sempre dividiremos a figura em 2n+1 triângulos pequenos.
Demonstre esta afirmação.
Solução. Você já escutou falar em contagem dupla? Pois é! Você escutará muito isso em
combinatória! Utilizaremos isso também em geometria.

3
POT 2012 - Geometria - Nı́vel 2 - Aula 1 - Prof. Rodrigo Pinheiro

Vamos calcular a soma de todos os ângulos internos de todos os triângulos pequenos de


duas formas, essas duas somas tem que ser a mesma. Na primeira forma, digamos que
existem T triângulos pequenos, portanto a soma que queremos será 180◦ · T . Na segunda
forma, basta perceber que cada ângulo vértice no interior do triângulo contribui para a
soma com 360◦ , enquanto todos os vértices do triângulo contribui com 180◦ . Temos então
que 180◦ · T = 360◦ · n + 180◦ , simplificando, temos que T = 2n + 1.

Problema 2. Paladino já estava na madrugada de segunda-feira, quando pensou na seguinte


hipótese: “Será que dado um polı́gono convexo, se dividirmos o polı́gono em triângulos
traçando suas diagonais sem se interceptarem, o número de triângulos é sempre o mesmo?”
E aı́, o que você acha?
Solução. Se dividirmos o polı́gono em T triângulos ligando suas diagonais sem se intercep-
tarem, a soma de todos os ângulos internos do polı́gono será 180◦ · T , como a soma sempre
é 180◦ · (n − 2), teremos que T = n − 2.

Problema 3. Demonstre que se em um polı́gono convexo de n lados, 4 desses ângulos forem


retos, então esse polı́gono é um retângulo.
Solução. Obviamente n ≥ 4. Suponhamos n > 4. Seja Sn−4 a soma dos outros n − 4
ângulos. Por ser um polı́gono convexo, cada ângulo é menor que 180◦ . Portanto, Sn−4 <
180◦ (n − 4). Sabendo que a soma de todos os ângulos internos é

180◦ · (n − 2) = 90◦ + 90◦ + 90◦ + 90◦ + Sn−4 < 360◦ + 180◦ · (n − 4)

chegamos que 180◦ (n − 2) < 180◦ · (n − 2), que é um absurdo.

Problema 4. No triângulo ABC abaixo, BP é bissetriz do ângulo B e M é o ponto médio


do lado AC. Se AB = 6 e BC = 10, calcule P M .

B
b

b P
b b b

A C
M

Solução. Veja a nova figura, onde prolongamos AP até encontrar o lado BC em Z.

4
POT 2012 - Geometria - Nı́vel 2 - Aula 1 - Prof. Rodrigo Pinheiro

B
b

6 10
b

P Z
b

b b b

A M C

Note que no triângulo ABZ, o segmento AP é altura e bissetriz. Isso faz com que o
triângulo ABZ seja isósceles! Logo BZ = AB = 6 e portanto:

ZC = BC − BZ = 10 − 6 = 4.

Perceba ainda que como o triângulo ABZ é isósceles, BP é altura, bissetriz e mediana.
Logo P é o ponto médio de AZ. Como M já é o ponto médio de AC, vemos que P M é a
base média no triângulo AZC. Conclusão:
ZC
PM = = 2.
2
Problema 5. Em um triângulo ABC (AB = AC, ∠BAC = 30◦ ) marcamos um ponto Q no
lado AB e um ponto P na mediana AD, de modo que P C = P Q(Q 6= B). Ache ∠P QC.
Solução.
A
b

Q b

E
b

P
b

β
b
θ b b

B D C

Como o △ABC é isósceles e ∠BAC = 30◦ , temos que ∠ABC = ∠ACB = 75◦ . Chame
∠BP Q = β e ∠P BC = θ. Como P D é mediana e altura do △BP C, então ∠BCP = θ
e P C = P B, pela propriedade de ângulo externo, concluı́mos que ∠CP E = 2θ. Como
P C = P B = P Q, temos que △P BQ é isósceles, portanto ∠P QB = ∠QBP = β. Pela
propriedade de ângulos externos ∠QP E = 2β. Daı́ temos que QP C = 2.(β + θ) = 150◦ .
Como P Q = P C, temos que △P QC é isósceles, então ∠P QC = ∠P CQ, concluindo que
∠P QC = 15◦ .

5
POT 2012 - Geometria - Nı́vel 2 - Aula 1 - Prof. Rodrigo Pinheiro

Problema 6. (OBM - 99) Na figura, os triângulos ABC e EGF são equiláteros. O perı́metro
do triângulo ABC é 132cm e, além disso, AE = EC, BD = DC, EF = F C e DG = GE.
B
b

b
D

b
G

b b b b

A E F C

Qual o perı́metro da área sombreada?


Solução. Como o △ABC é equilátero, então todos os seus lados são iguais, assim como
seus ângulos são todos iguais a 60◦ . Portanto, AB = BC = CA = 132 3 = 44cm. Como
BD = DC, temos que BD = DC = 44 2 = 22, analogamente EC = 22. Dado que o △DEC
é isósceles com um ângulo de 60 , então ele é equilátero, consequentemente DE = 22.

Sabendo que DG = GE, obtemos que DG = GE = 11. Analogamente obtemos que


EF = F C = 11, assim o perı́metro da área sombreada é AB +BD+DG+GF +F E +EA =
44 + 22 + 11 + 11 + 11 + 22 = 121.

Problema 7. Na figura abaixo, ABGH, BCF G e CDEF são quadrados iguais. Determine
a soma ∠ABH + ∠ACH + ∠ADH.

H G F E
b b b b

b b b b

A B C D

Solução. Observemos a figura abaixo.

6
POT 2012 - Geometria - Nı́vel 2 - Aula 1 - Prof. Rodrigo Pinheiro

H G F E
b b b b

β
B α C
A b b
90◦
b b
D
90◦ −α
α

J K
I b b b b
L

b b b b

M N O P

Pela propriedade de ângulos alternos internos, temos que ∠ACH = ∠CHE = α.


Pela simetria da figura, vemos que ∠N HM = ∠HDA = β, da mesma forma vemos que
∠OCN = ∠HCA = α e CN = HC, como ∠ACO = 90◦ , concluı́mos que ∠HCN = 90◦ .
Como o △HCN é isósceles com um ângulo de 90◦ , temos que CHN = 45◦ . Sabendo
que ∠M HE = 90◦ , vemos que α + β + 45◦ = 90◦ , como ∠ABH = 45◦ , temos que:
∠ADH + ∠ACH + ∠ABH = 90◦

Problema 8. Dados n pontos A1 , A2 , . . . , An e um cı́rculo unitário, prove que é possı́vel


encontrar um ponto M sobre o cı́rculo tal que M A1 + M A2 + · · · + M An ≥ n.
Solução. Sejam M1 e M2 pontos diametralmente opostos no cı́rculo. Então M1 Ak +
M2 Ak ≥ M1 M2 = 2. Adcionando essas desigualdades para k = 1, 2, 3, . . . , n temos

(M1 A1 + · · · + M1 An ) + (M2 A1 + · · · + M2 An ) ≥ 2n.

Portanto, M1 A1 + · · · + M1 An ≥ n ou M2 A1 + · · · + M2 An ≥ n, então basta tomar M = M1


ou M = M2 .

Problema 9. Os ângulos ∠BAD e ∠CBA do quadrilátero convexo ABCD são iguais e


BC = 1, AD = 3. Prove que o comprimento de CD é maior que 2.
Solução. Tomemos um ponto E sobre AD tal que AE = BC. Seja F o ponto de encontro
das retas AD e BC, como ∠BAD = ∠CBA ⇒ ∠F AB = ∠F BA. Portanto o triângulo
F AB é isósceles, com F A = F B. Assim, F E = F C, pois AE = BC, podemos concluir
então que o △F EC também é isósceles, consequentemente, ∠F EC = ∠F CA. A partir
disto, temos que ∠DEC = ∠GCE. Pela figura, percebemos que ∠GCE > ∠DCE, ou
seja, ∠DEC > ∠DCE, pelo Teorema 6, concluı́mos que CD > ED = 3 − 1 = 2.

7
POT 2012 - Geometria - Nı́vel 2 - Aula 1 - Prof. Rodrigo Pinheiro

Problemas Propostos
Problema 10. Dados os pontos colineares e consecutivos A, B, C, D e E, tal que AB +
CD = 3 × BC e DE = AB. Sendo M o ponto médio de BE, onde M D = 2 e AE = 16,
calcule M C.
Problema 11. Em um reta, temos quatro pontos A, B, C e D que satisfazem as seguintes
relações 4 × AB − BD − 2 × CD = 4, AB = 3 e AC = 5, calcule AD.
Problema 12. (OBM-2011) Dois triângulos equiláteros de perı́metro 36cm cada um sâo
sobrepostos de modo que sua interseção forme um hexágono com pares de lados paralelos,
conforme ilustrado no desenho. Qual é o perı́metro desse hexágono?

H
b

J b b L

b b
G I
b

Problema 13. Um trapézio ABCD de bases BC e AD com BC < AD é tal que 2.AB = CD
e ∠BAD + ∠CDA = 120◦ . Determine os ângulos do trapézio ABCD.
Problema 14. No △ABC , E e D são pontos interiores aos lados AC e BC, respectiva-
mente. Se AF bissecta ∠CAD e BF bissecta CBE. Prove que ∠AEB +∠ADB = 2∠AF B.
C

F D
E

A B
Problema 15. No △ABC, um ponto D está sobre AC tal que AB = AD. Se ∠ABC −
∠ACB = 30, encontre ∠CBD.
Problema 16. A bissetriz interior de B, e a bissetriz exterior de C do triângulo ABC
encontram-se em D. Através de D, uma reta paralela a CB encontra AC em L e AB
em M . Se as medidas dos comprimentos de LC e M B do trapézio CLM B são 5 e 7,
respectivamente, encontre a medida de LM . Prove seu resultado.
Problema 17. No △ABC, CF é mediana relativa á hipotenusa AB, CE é bissetriz de
∠ACB, e CD é uma altura realativa à AB. Prove que ∠DCE = ∠ECF .

8
POT 2012 - Geometria - Nı́vel 2 - Aula 1 - Prof. Rodrigo Pinheiro

Problema 18. A medida do segmento de reta P C, perpendicular á hipotenusa AC do


triângulo retãngulo ABC, é igual à medida do comprimento BC. Mostre que BP deve ser
perpendicular ou paralelo à bissetriz de A.

Problema 19. Prove que para quaisquer três pontos A, B, C nós temos AC ≥ |AB − BC|.

Problema 20. O lado AC do triângulo ABC tem comprimento 3.8, e o lado AB tem
comprimento 0.6. Se o comprimento do lado BC é um inteiro, qual é o seu comprimento?

Problema 21. Prove que o comprimento de qualquer lado de um triângulo não é maior que
metade do perı́metro.

Problema 22. A distância de Leningrado para Moscou é 660 quilômetros. De Leningrado


para Likovo são 310 quilômetros, de Likovo para Klin são 200 quilômetros e de Klin para
Moscou são 150 quilômetros. Qual é a distância entre Likovo e Moscou?

Problema 23. Encontre um ponto dentro de um quadrilátero convexo tal que a soma das
distâncias do ponto aos vértices é mı́nima.

Problema 24. O ponto O é dado no plano do quadrado ABCD. Prove que a distância
de O até um dos vértices do quadrado não é maior que a soma das distâncias de O até os
outros três vértices.

Problema 25. Prove que a soma das diagonais de um quadrilátero convexo é menor que o
perı́metro mas é maior que o semiperı́metro.

Problema 26. Prove que a soma das diagonais de um pentágono convexo maior que o
perı́metro mas é menor que o dobro do perı́metro.

Problema 27. Um ponto A, dentro de um ângulo acutângulo, é refletido em cada lado do


ângulo para obtermos os pontos B e C. O segmento de reta BC intersecta os lados do
ângulo em D e E. Mostre que BC/2 > DE

Problema 28. Prove que a distância entre quaisquer dois pontos dentro de um triângulo
não é maior que que metade do perı́metro do triangulo.

Problema 29. Se o ponto O está dentro do triângulo ABC, prove que AO+OC < AB+BC.

9
POT 2012 - Geometria - Nı́vel 2 - Aula 1 - Prof. Rodrigo Pinheiro

Problema 30. Prove que a soma das distâncias de O aos vértices de um dado triângulo é
menor que o perı́metro, se o ponto O está dentro do triângulo. O que acontece se o ponto
O estiver fora do triângulo?

Problema 31. O ponto C está dentro de um ângulo reto, e os pontos A e B estão sobre seus
lados. Prove que o perı́metro do triângulo ABC não é menor que duas vezes a distância
OC, onde O é o vértice do ângulo reto.

O
B
Problema 32. Prove que o comprimento da mediana AM em um triângulo ABC não é
maior que a metade da soma dos lados AB e AC. Prove que a soma dos comprimentos das
três medianas não é maior que o perı́metro do triângulo.

Problema 33. Prove que um polı́gono convexo não pode ter três lados , cada um maior
que a maior diagonal.

Problema 34. Prove que o perı́metro de um triângulo não é maior que 4/3 da soma das
medianas.

Problema 35. Prove que um pentágono convexo tem três diagonais que são lados de um
triângulo.

Problema 36. Qual é o ângulo formado pelas agulhas do relógio as 12:35?

Problema 37. Na figura, os vértices do retângulo P QRS pertencem aos lados do retângulo
ABCD. Sendo AP = 3cm, AS = 4cm, SC = 6cm e CR = 8cm, qual a área do retângulo
P QRS, em cm2 ?
A P B
b b b

S b

b
Q

b b

C b
D
R

10
POT 2012 - Geometria - Nı́vel 2 - Aula 1 - Prof. Rodrigo Pinheiro

Problema 38. A piscina do clube que Samuel frequenta tem a forma de um hexágono
(polı́gono com seis lados), com um ângulo intero de 270◦ , os demais ângulos de 90◦ e os
quatro lados menores com 12m cada. Samuel costuma nadar pelo meio da piscina, a partir
do ponto A, descrevendo o trajeto representado, na figura, pelo ângulo reto ABC, em que
AB = BC. Certo dia, ele nadou por esse trajeto 4 vezes, isto é, foi e voltou 2 vezes.
Quantos metros ele percorreu?
A
b b b

b b

b b
C
B

b b

Problema 39. Uma folha de papel tem 20cm de comprimento por 15cm de largura. Do-
bramos essa folha ao meio, paralelamente à sua largura. Em seguida, dobramos a folha
retangular dupla, de modo que dois vértices opostos coincidam. Ao desdobrar a folha, as
marcas da segunda dobra dividem a folha em duas partes, conforme mostrado na figura ao
lado. Qual é a área da parte escura, em cm2 ?

b b b

b b

dobra 1

dobra 2 dobra 2

b b

11
POT 2012 - Geometria - Nı́vel 2 - Aula 1 - Prof. Rodrigo Pinheiro

Problema 40. Prove que é impossı́vel desenhar uma estrela (veja a figura abaixo) de modo
que AB < BC, CD < DE, EF < F G, GH < HI e IK < KA.

C E
B F
A G

I
K H

Problema 41. Seja ABCD um paralelogramo. O ponto E está sobre AD de modo que
AE = CD. Se ∠ABE = 30◦ encontre o valor do ângulo ∠EBC.

Problema 42. Seja △ABC um triângulo com ∠A = 50◦ . O lado BC é prolongado em


ambas as direções e sobre os prolongamentos são marcados os pontos P e Q de modo que
P B = BA, CQ = CA e P B + BC + CQ = P Q. Calcule a medida do ângulo ∠P AQ.

Problema 43. Seja △ABC um triângulo retângulo em C. Sejam M e N pontos sobre a


hipotenusa tais que BN = BC e AM = AC. Ache o valor do ângulo ∠N CM .

12
Programa Olímpico de Treinamento
Curso de Geometria - Nível 2 Aula 2
Prof. Rodrigo Pinheiro

Congruência de triângulos

Definição: Um triângulo é congruente a outro se, e somente se, é possı́vel estabelecer


uma correspondência entre seus vértices de modo que:

1. Seus lados são ordenadamente congruentes aos lados do outro e

2. Seus ângulos são ordenadamente congruentes aos ângulos do outro.

b
B b
F

c a n
m

b b b b

b C D p E
A

Notação:
 
∠A = ∠D a = n
△ABC ≡ △DF E ⇐⇒  ∠B = ∠F e b = p 
∠C = ∠E c = m
Teorema 1. Dois lados de um triângulo são congruentes se,e somente se os ângulos opostos
a estes lados são congruentes.
Demonstração.

A
b

b b b

B M C
POT 2012 - Geometria - Nı́vel 2 - Aula 1 - Prof. Rodrigo Pinheiro

(⇒) Seja ABC um triângulo com AB = AC e M o ponto médio do lado BC. Observe
que △ABM ≡ △AM C, pelo caso LLL, portanto ∠ABC = ∠ACB.
(⇐) Seja ABC um triângulo com ∠ABC = ∠ACB e M o pé da altura relativa a BC.
Como ∠AM B = ∠AM C = 90◦ e ∠ABC = ∠ACB, temos então que ∠M AB = ∠M AC
pois a soma dos ângulos internos de um triângulo interno é sempre 180◦ . Concluı́mos assim
que △AM B ≡ △AM C pelo caso ALA, consequentemente, AB = AC.

Teorema 2. Em todo triângulo isósceles, a altura, mediana e bissetrizes relativas à base


são coincidentes.
Demonstração.

A
b

b b b

B M C

Seja ABC um triângulo com AB = AC e M o ponto médio do lado BC. Observe que
△ABM ≡ △AM C, pelo caso LLL, portanto ∠AM B = ∠AM C e ∠BAM = ∠CAM , por
definição temos então que AM também é bissetria. Como ∠AM B + ∠AM C = 180◦ e
∠AM B = ∠AM C, então ∠AM B = ∠AM C = 90◦ , concluindo assim que AM também é
altura.

Teorema 3. Seja ABCD um polı́gono convexo, demonstre que se dois lados opostos são
iguais e paralelos, então ABCD é um paralelogramo.
Demonstração. Suponhamos que AB = CD e AB\\CD. Pela propriedade de paralelismo,
temos que: ∠ABD = ∠BDC, concluindo assim que △ABD ≡ △CDB. Daı́, tiramos que
∠DBC = ∠BDA, pela propriedade de paralelismo podemos concluir que BC \ \AD, isto
é, ABCD é um paralelogramo.
B C
b b

b b

A D

Problema 1. Dado um triângulo ABC, onde AB = BC. Tomam-se dois pontos M e N


em AB e BC, respectivamente. Demonstre que se ∠M CA = ∠N AC, então AM = CN .
Solução.

2
POT 2012 - Geometria - Nı́vel 2 - Aula 1 - Prof. Rodrigo Pinheiro

B
b

M b b
N

b b

A C

Dado que o triângulo é isósceles, temos que: ∠M AC = ∠N CA, portanto concluı́mos


que △M AC ≡ △N CA, daı́ tiramos que M A = N C.

Problema 2. Mostre que se um triângulo possui duas altura iguais, então o triângulo é
isósceles.
Solução. Seja ABC um triângulo onde as alturas AD e CE têm o mesmo comprimento.
Pelo caso especial de congruência temos que △AEC ≡ △ADC, portanto ∠EAC = ∠DCA,
isto é, △ABC isósceles.

B
b

E b b
D

b b

A C

Problema 3. Demonstre que se duas retas AB e AC são duas retas tangentes a um


circunferência de centro O /nos pontos B e C, respectivamente, então AB = AC e
∠OAB = ∠OAC.
Solução.

3
POT 2012 - Geometria - Nı́vel 2 - Aula 1 - Prof. Rodrigo Pinheiro

B
b

A b b
O

Observe que BO = OC e AO pertence ao dois triângulos ABO e ACO, então pelo caso
especial de congruência △ABO ≡ △ACO, assim AB = AC e ∠OAB = ∠OAC.

Problema 4. (Teorema de pitot) Mostre que um quadrilátero pode ser circunscrito a uma
circunferência se, e somente se, a soma de dois lados opostos for igual à soma dos outros
dois lados.
Solução. (⇒) Suponha que o quadrilátero ABCD seja circunscrito a uma circunferência,
e os pontos de tangência da circunferência com os lados sejam E, F , G, H, como mostra a
figura abaixo.

b
C
F
B b

G
E b
b

b
O

b b b

A H D

Pelo problema anterior, vemos que: AH = AE; BE = BF ; CF = CG; GD = HD.


Portanto, AE + BE + CG + GD = BF + CF + +HD + AH, isto implica dizer que:
AB + CD = BC + AD.
(⇐) Suponha que ABCD seja um quadrilátero tal que AB + CD = BC + AD e não
seja circunscritı́vel.

4
POT 2012 - Geometria - Nı́vel 2 - Aula 1 - Prof. Rodrigo Pinheiro

z C
y F
b

B b

b z
β
y
β
G
E b
b
z+w
b
O

x
k
α
b
α
b
k b
w-k b
D
A x H
wI

Sejam AO e BO as bissetrizes internas dos ângulos ∠DAB e ∠ABC. Tomamos E, F e


H como sendo os pés das alturas de O aos lados AB, BC e AD, respectivamente. Pelo caso
especial de congruência temos que △AOH ≡ △AOE e △BOE ≡ △BOF , assim sendo,
AE = AH = x e BE = BF = y. Defina CF = z e HD = w. Pela hipótese, temos que:
(x + y) + CD = (y + z) + (x + w) ⇒ CD = z + w
Como CD não é tangente à circunferência pois, caso contrário, o quadrilátero seria circuns-
critı́vel. Tomamos G tal que CG seja tangente a circunferência e defina CG ∩ AD = I,
perceba que pelo problema anterior, temos: CG = CF = z e GI = HI = k. Dessa ma-
neira, ID = w − k, mas analisando o triângulo CID isso é um absurdo pois CI + ID = CD
e CID é um triângulo. Então se a soma dos lados opostos de um quadrilátero forem iguais,
então ele será circunscritı́vel.
Problema 5. São construı́dos exteriormente ao △ABC, os triângulos equiláteros ABM ,
BCN , ACP . Prove que N A = BP = CM .
Solução.

b
N

M
b B
b
60◦
60◦
α

b b

A C

Primeiramente, construı́mos somente os triângulos equiláteros ABM e BCN . Dessa


forma, temos que: AB = M B e BC = BN e ∠M BA = ∠CBN = 60◦ . Assim sendo,

5
POT 2012 - Geometria - Nı́vel 2 - Aula 1 - Prof. Rodrigo Pinheiro

temos que △M BC ≡ △ABN pelo caso LAL, pois AB = M B; BC = BN e


∠M BA = ∠CBN , portanto AN = CM . Analogamente, provamos que AN = CM = BP .

Problemas Propostos
Problema 6. ABCD é um paralelogramo e ABF e ADE são triângulos equiláteros cons-
truı́dos exteriormente ao paralelogramo. Prove que F CE também é equilátero.

Problema 7. Quatro quadrados são construı́dos exteriormente nos lados de um paralelo-


gramo. Mostre que os centros destes quadrados também formam um quadrado

Problema 8. Um hexágono convexo ABCDEF está circunscrito a uma circunferência.


Mostre que AB + CD + EF = BC + DE + F A.

Problema 9. Na figura, ABCD e AEF G são quadrados. Mostre que BE = DG.

b
C b
B

b
F

b
E
G
b

b b

D A

Problema 10. (Rússia 1946) Dados três pontos A, B, C sobre uma reta l, são construı́dos
triângulos equiláteros ABC1 e BCA1 em um mesmo semi-plano com respeito a l. Se M ,
N são os pontos médios de AA1 , CC1 , respectivamente, mostre que o triângulo BM N é
equilátero.

Problema 11. (Inglaterra/95) Seja ABC um triângulo retângulo em C. As bissetrizes


internas de BAC e ABC encontram BC e CA em P e Q, respectivamente. Sejam M e N
os pés das perpendiculares a partir de P e Q até AB, respectivamente. Encontre a medida
do ângulo M CN

Problema 12. (Polônia/92) Os segmentos AC e BD intersectam-se no ponto P de modo


que P A = P D, P B = P C. Seja O o circucentro do triângulo P AB. Prove que as retas
OP e CD são perpendiculares.

Problema 13. Prove que se em um triângulo ABC, a mediana AM é tal que ∠BAC é
dividido na razão 1 : 2, e D está sobre AM , com M entre A e D, tal que ∠DBA = 90◦ ,
então AC = AD2 . Dica: Escolha P sobre AD tal que AM = M P .

6
POT 2012 - Geometria - Nı́vel 2 - Aula 1 - Prof. Rodrigo Pinheiro

Problema 14. Em um quadrado ABCD, M é o ponto médio de AB. Uma reta perpendi-
cular a M C em M toca AD em K. Prove que ∠BCM = ∠KCM .

Problema 15. Dado um quadrado ABCD com ∠EDC = ∠ECD = 15◦ , prove que △ABE
é equilátero.

Problema 16. Dado um triângulo qualquer ABC, D, E e F são pontos médios dos lados
AC, AB e BC, respectivamente. Sendo BG a altura do triângulo ABC. Prove que
∠EGF = ∠EDF .

Problema 17. (congruencia) No losango ABCD com ∠BAD = 60◦ , tomamos pontos F ,
H e G nos lados AD, DC e na diagonal AC, respectivamente, de modo que DF GH seja
um paralelogramo. Prove que o triângulo BF H é equilátero.

Problema 18. (congruência) Seja ABCD um paralelogramo. A bissetriz de ∠BAD corta


BC em M e o prolongamento de CD em N . Se O é o circuncentro do triângulo M CN ,
mostre que B, O, C, D são concı́clicos.

Problema 19. (congruencia) Sejam ABC um triângulo, D um ponto sobre o prolonga-


mento da semi-reta BC a partir de B tal que BD = BA e M o ponto médio de AC. A
bissetriz do ângulo ∠ABC corta DM em P . Mostre que ∠BAP = ∠ACB.

Problema 20. (congruencia) Seja ABCDE um pentágono com AE = ED, AB+CD = BC


e ∠BAE + ∠CDE = 180◦ . Prove que ∠AED = 2∠BEC.

Problema 21. (Congruencia) Sejam ABC um triângulo de circuncı́rculo ω1 , O o circuncen-


tro de ABC e ω2 o ex-incı́rculo relativo ao lado BC. Se M, N, L são os pontos de tangência
de ω2 com as retas BC, AC, AB e os raios de ω1 e ω2 são iguais, mostre que O é o ortocentro
do triângulo M N L.

7
Programa Olímpico de Treinamento
Curso de Geometria - Nível 2 Aula 3
Prof. Rodrigo Pinheiro

Teorema de Tales e Aplicações

Divisão Harmônica
MA
Dizemos que os pontos M e N dividem harmonicamente o segmento AB quando MB =
NA
NB .

b
N b
A b
M b
B

MA NA
Como M B = k = N B , os pontos M e N dividem o segmento AB na mesma razão. Estes
pontos são chamados conjugados harmônicos de AB na razão k.
Problema 1. Prove que em uma divisão harmônica com k > 1, temos que:
2 1 1
= +
AB AM AN
Solução.

b
A b
M b
B b
N

MA NA AM AN
= ⇒ =
MB NB AB − AM AN − AB
⇒ AM (AN − AB) = AN (AB − AM ) ⇒ AM.AN − AM.AB = AN.AB − AM.AN
2 1 1
⇒ 2.AM.AN = AN.AB + AM.AB ⇒ = +
AB AM AN
Problema 2. Prove que em uma divisão harmônica com k < 1, temos que:
2 1 1
= −
AB AM AN
Problema 3. Sendo O o ponto médio de AB em uma divisão harmônica, prove que:

OA2 = OM.ON

Solução.
POT 2012 - Geometria - Nı́vel 2 - Aula 3 - Prof. Rodrigo Pinheiro

b
A b
O b
M b
B b
N

MA NA OM + OA ON + OA
= ⇒ =
MB NB OB − OM ON − OB
Como OB = OA, temos que:

(OM + OA)(ON − OA) = (ON + OA)(OA − OM ) ⇒


OM.ON − OM.OA + ON.OA − OA2 = ON.OA − OM.ON + OA2 − OM.OA ⇒
OA2 = OM.ON

Problema 4. Sejam M e N conjugados harmônicos na razão k > 1 do segmento AB = l.


Qual é a distância entre os divisores harmônicos de AB?
Solução.

b
A b
M b
B MN = x b
N
AB = l M B = a BN = b

MA 1−a 1
=k⇒ = k ⇒ 1 − a = a.k ⇒ a =
MB a k+1
NA 1+b 1
=k⇒ = k ⇒ 1 + b = b.k ⇒ a =
NB b k−1
Portanto,

2k.l
x=a+b⇒x=
k2 − 1
Problema 5. Sejam M e N conjugados harmônicos na razão k < 1 do segmento AB = l.
Qual é a distância entre os divisores harmônicos de AB?

Teorema de Tales

Teorema 1. Se um feixe de retas paralelas é cortado por duas retas transversais, r e s,


então a razão entre quaisquer dois segmentos determinados em r é igual a à razão entre os
segmentos correspondentes em s.

2
POT 2012 - Geometria - Nı́vel 2 - Aula 3 - Prof. Rodrigo Pinheiro

b A E
b b

B b b
F

C G
b b

e D H
b b

Se b, c, d e e são retas paralelas cortadas pelas transversais r e s, então:


AB BC CD AC BD AD
= = = = =
EF FG GH EG FH EH
Teorema da bissetriz interna

Teorema 2. A bissetriz interna de um ângulo interno de um triângulo determina sobre o


lado oposto ao ângulo dois segmentos proporcionais aos lados adjacentes.
Assim, por exemplo, a bissetriz interna do ângulo A do triângulo ABC divide o lado
BC em dois segmentos x e y tais que:

b
A

c
b

B D C
b
x b
y b

x y
=
c b
Demonstração. Traçamos por C um reta paralela a bissetriz interna AD, e seja E a in-
terseção dessa paralela com o prolongamento da reta AB. Pela propriedade de paralelismo,
temos que ∠BAD = ∠BEC e ∠DAC = ∠ACE, como AD é bissetriz, concluı́mos que
∠ACE = ∠AEC, portanto △ACE é isósceles, com AE = AC = b. Sendo assim, pelo
teorema de tales, temos que:
x y
=
c b

3
POT 2012 - Geometria - Nı́vel 2 - Aula 3 - Prof. Rodrigo Pinheiro

b
E

A b

c
b

B D C
b
x b
y b

Teorema da bissetriz externa

Teorema 3. A bissetriz externa de um ângulo de um triângulo determina sobre o lado


oposto ao ângulo dois segmentos proporcionais aos lados adjacentes.
Assim, por exemplo, a bissetriz externa do ângulo A do triângulo ABC determina sobre
o lado BC dois segmentos x e y tais que:
x y
=
c b
A
b

b
c

D x B C
b b b

Demonstração. Analogo ao teorema da bissetriz interna.

Problema 6. Seja ABC um triângulo tal que AB = 6, AC = 7 e BC = 8. Tome S ∈ AC


onde BS é bissetriz do ângulo B e tome I ∈ BS tal que CI é bissetriz do ângulo C,
determine a razão BI
IS .
Solução.

4
POT 2012 - Geometria - Nı́vel 2 - Aula 3 - Prof. Rodrigo Pinheiro

b
A

S
b

I
b

B b b
C

Seja SC = x. Temos então que AS = 7 − x. Pelo teorema da bissetriz interna no


triângulo ABC temos que:

6 AS 7−x
= = ⇒ 6x = 56 − 8x ⇒ x = 4
8 SC x
Pelo teorema da bissetriz interna no triângulo BSC, temos que:

BI 8
= =2
IS x
Problema 7. Seja ABC um triângulo retângulo em A, com hipotenusa BC = 30 e AC −
AB = 6. Calcule o comprimento da bissetriz BS.
Solução. Seja AC = x e AB = y, então temos que: x − y = 6 e x2 + y 2 = 900 pelo teorema
de pitágoras. Isolando x na primeira equação e substituindo na segunda, teremos que:

(y + 6)2 + y 2 = 900 ⇒ y 2 + 6y − 432 = 0


onde teremos as raı́zes 18 e −24, portanto, y = 18, assim x = 24, como BS é bissetriz, pelo
teorema da bissetriz interna, teremos que:
18 AS
= ⇒ AS = 9
30 24 − AS

Pelo teorema de pitágoras, teremos que: BS 2 = 182 + 92 ⇒ BS = 9. 5.

b
A

S
b

b b

B C

Problema 8. Sendo AS e AP bissetrizes dos ângulos internos e externos em A, determine


o valor de CP , sabendo que BS = 8 e CS = 6.

5
POT 2012 - Geometria - Nı́vel 2 - Aula 3 - Prof. Rodrigo Pinheiro

Problema 9. Seja ABC um triângulo de lados a, b, c opostos aos vértices A, B, C, respec-


ac ab
tivamente. Se D ∈ BC tal que AD é bissetriz interna, mostre que BD = b+c e CD = b+c .

Problema 10. O incentro do triângulo ABC divide a bissetriz interna do ângulo A na razão
AI : ID = 2 : 1. Mostre que os lados do triângulo estão em progressão aritmética.

Problema 11. (Cı́rculo de Apolonius) Seja k um número real positivo, k 6= 1. Mostre que
o lugar geométrico dos pontos P do plano tais que P A : P B = k é uma circunferência cujo
centro pertence à reta AB.
AB
Problema 12. Em um triângulo ABC, BC = 7, BC = 3. Calcule o valor da altura relativa
ao lado a sabendo que ela é máxima.

Problema 13. Em um triângulo ABC, BC = 16 e a altura relativa ao lado BC é 8. Calcule


a razão AB
AC sabendo que ela é máxima.

Problema 14. Os comprimentos dos lados de um triângulo são os inteiros x − 1, x e x + 1


e seu maior ângulo é o dobro do menor. Determine o valor de x.

Problema 15. Em um triângulo ABC, de lados AB = 12, AC = 8 e BC = 10, encontre o


maior segmento que a bissetriz interna de A determina sobre BC.

6
Programa Olímpico de Treinamento
Curso de Geometria - Nível 2 Aula 4
Prof. Rodrigo Pinheiro

Semelhança de Triângulos

Dois triângulos são semelhantes se, e somente se, possuem ângulos ordenadamente con-
gruentes e os lados homólogos proporcionais. Sendo k a razão entre os lados homólogos,
k é chamado de razâo de semelhança. Observe que se k = 1, então os triângulos são
congruentes. Igualmente a congruência de triângulos, temos os casos de semelhança.
1◦ Caso: Se dois triângulos têm congruentes dois a dois os três ângulos internos, então
esses dois triângulos são semelhantes.
2◦ Caso: Se dois triângulos têm dois pares de lados proporcionais e os ângulos com-
preendidos entre eles congruentes, então esses dois triângulos são semelhantes.
3◦ Caso: Se dois triângulos têm os três lados correspondentes proporcionais, então
esses triângulos são semelhantes.

Teorema 1. Se uma reta é paralela a um dos lados de um triângulo, então o triângulo que
ele determina é semelhante ao primeiro.
Demonstração. Basta ver que eles têm os mesmo ângulos por paralelismo.
Observação 1: Se dois triângulos são semelhantes na razão k, então também é igual
a k:

• a razão entre as alturas

• a razão entre as medianas

• a razão entre as bissetrizes, etc.

Observação 2: A razão entre as áreas de dois triângulos semelhantes (na razão k) é


igual a k 2 .

Problema 1. As bases de um trapézio medem 12m e 18m e os lados oblı́quos às bases
medem 5m e 7m. Determine o perı́metro do triângulo menor que obtemos ao prolongar os
lados oblı́quos às bases.
Solução.
POT 2012 - Geometria - Nı́vel 2 - Aula 4 - Prof. Rodrigo Pinheiro

x y

b b

12
5 7

b b

18

Como as bases do trapézio são paralelas, teremos que os dois triângulos são semelhantes,
portanto:
x 12 y
= = ⇒
x+5 18 7+y
18x = 12x + 90 e 18y = 12y + 84, então: x = 15 e y = 14, assim, o perı́metro será
15 + 12 + 14 = 41
Problema 2. Num triângulo ABC, os lados medem AB = 4cm, BC = 5cm e AC = 6cm.
Calcule os lados de um triângulo semelhante a ABC cujo perı́metro mede 20cm.
Solução. Sejam x, y e z os lados do triângulo. Como os dois triângulos são semelhantes,
então:
x y z x+y+z 20
= = = = ⇒
4 5 6 4+5+6 15
x = 16/3, y = 20/3 e z = 8.
Problema 3. Seja ABC um triângulo eqüilátero de lado 20. Uma reta passando pelo ponto
médio M do lado AB corta o lado AC no ponto N e o prolongamento do lado BC no ponto
P , de tal modo que CP = 12. Determine o comprimento de CN e N A.
Solução.

b
A

M
b

N
10 b

x
B C P
b b b b

O 10 12

Tomemos O como sendo o ponto médio de BC. Como M O é base média, temos que
M O = 10 e M O é paralelo a AC, assim o triângulo N CP é semelhante a M OP , então:
x 12 60
= ⇒x=
10 22 11

2
POT 2012 - Geometria - Nı́vel 2 - Aula 4 - Prof. Rodrigo Pinheiro

Problema 4. Sejam D e E pontos sobre os lados AB e AC do triângulo ABC. Sendo BC =


22cm, AD = 8cm, DB = 3cm, AE = 5cm e ∠ABE = ∠ACD, calcule o comprimento de
DE.

Problema 5. Considere a circunferência circunscrita ao triângulo ABC. Seja AE um


diâmetro dessa circunferência e AD a altura do triângulo. Sendo AB = 6cm, AC = 10cm
e AE = 30cm, calcule AD.

Problema 6. Calcule o raio da circunferência circunscrita ao triângulo ABC sabendo que


AB = 4, AC = 6 e a altura AH relativa ao lado BC é igual a 3.

Problema 7. (Base média de um triângulo) Sejam M e N os pontos médios, respectiva-


mente, dos lados AB e AC do triângulo ABC. O segmento M N é chamado de base média,
relativa ao lado BC. Mostre que M N é paralela a BC e que M N = BC
2 .

Problema 8. Sejam ABCD um trapézio com AB paralelo a CD, M e N os pontos médios


dos lados oblı́quos AD e BC. Use o exercı́cio anterior para concluir que M N = AB+CD
2 .

Problema 9. No triângulo ABC, a bissetriz interna do ângulo ∠A encontra BC em D. A


reta por B, perpendicular a AD, encontra AD em E. Seja M o ponto médio do lado BC.
Se AB = 26, BC = 28 e AC = 30, ache os comprimentos de DM e M E.

Problema 10. No triângulo ABC, Z é um ponto sobre o lado AB. Uma reta por A e
paralela a CZ, encontra BC em X; uma reta por B e paralela a CZ encontra AC em Y .
1 1 1
Mostre que AX + BY = CZ .

Problema 11. Seja P um ponto no interior do triângulo eqüilátero ABC. Por P traçamos
três retas paralelas aos lados de ABC, determinando três triângulos menores, de áreas 4,
9 e 49. Determine a área do triângulo ABC.

Problema 12. Duas circunferências c1 e c2 interceptam-se em dois pontos A e B. Construa


um segmento P Q pelo ponto B com uma extremidade sobre c1 e a outra sobre c2 de modo
que P Q seja o maior possı́vel.

Problema 13. Os lados de um triângulo ABC medem AB = 6, AC = 9 e BC = 11. Se


J é o ponto de tangência do cı́rculo ex-inscrito relativo ao lado AB. Sabendo que JL é
paralelo a BC (com L sobre o lado AC), determine o comprimento do segmento AL.

Problema 14. Seja C1 a circunferência inscrita num triângulo ABC cujo perı́metro mede
18cm. Uma tangente a C1 é paralela a um dos lados do triângulo e mede 2cm. Quais os
possı́veis valores do lado ao qual esta tangente é paralela?

3
Polos Olímpicos de Treinamento
Curso de Geometria - Nível 2 Aula 5
Prof. Cícero Thiago

Algumas propriedades importantes de triângulos

Propriedade 1. Num triângulo retângulo ABC, a mediana BM relativa à hipotenusa


mede metade da hipotenusa AC.
A D

B C
Demonstração. Seja D o ponto sobre o prolongamento da mediana BM tal que BM =
M D. Os triângulos AM B e CM D são congruentes, pelo caso LAL. Daı́, AB = CD e
∠BAM = ∠DCM , ou seja, AB e CD são segmentos iguais e paralelos e portanto
∠ABC = ∠DCB = 90◦ .
Assim, os triângulos ABC e DCB são congruentes, pelo caso LAL, e portanto
AC
BD = AC =⇒ 2 · BM = AC =⇒ BM = ·
2
Afirmação. Uma base média de um triângulo é um segmento que une os pontos médios de
dois de seus lados.

Assim, todo triângulo possui exatamente três bases médias.

Propriedade 2. Sejam ABC um triângulo e M , N os pontos médios dos lados AB, AC,
respectivamente. Então
BC
M N k BC e M N = ·
2
A

M N P

B C
POT 2012 - Geometria - Nı́vel 2 - Aula 5 - Prof. Cı́cero Thiago

Demonstração. Inicialmente, prolonguemos a base média M N até um ponto P tal que


M N = N P . Em seguida, construı́mos o triângulo CN P . Note que os triângulos AN M e
CN P são congruentes, pelo caso LAL. Daı́, CP = AM e ∠M AN = ∠P CN e portanto

CP k AM =⇒ CP k BM.

Assim, M BCP é um paralelogramo, pois CP e BM são segmentos paralelos e iguais. Mas


então M P k BC e
BC
M P = BC =⇒ 2M N = BC =⇒ MN = ·
2
Afirmação. A base média de um trapézio é o segmento que une os pontos médios de seus
lados não paralelos.

Propriedade 3. Seja ABCD um trapézio de bases AB e CD, e sejam M e N os pontos


médios dos lados BC e AD, respectivamente. Então,
AB + CD
M N k AB, M N k CD e MN = .
2

b
A b
B

N
b b
M

D b b
C b
E

Demonstração. Inicialmente, prolonguemos AM até encontrar DC no ponto E. É fácil


ver que
∆ABM ≡ ∆CM E (ALA) ⇒ AB = CE.
Portanto, M N é base média do triângulo ADE. Assim,
DE
M N k BE ⇒ M N k DC ⇒ M N = .
2
DC + CE DC + AB
Finalmente, M N = = .
2 2
Problema 1. (OBM) Considere um triângulo acutângulo ABC com ∠BAC = 30◦ . Sejam
B1 , C1 os pés das alturas relativas aos lados AC, AB, respectivamente, e B2 , C2 os pontos
médios dos lados AC, AB, respectivamente. Mostre que os segmentos B1 C2 e B2 C1 são
perpendiculares.

2
POT 2012 - Geometria - Nı́vel 2 - Aula 5 - Prof. Cı́cero Thiago

C2 B2
O

B1
C1

B C

Solução.
Seja O a interseção entre B1 C2 e B2 C1 . O segmento B1 C2 é uma mediana do triângulo
retângulo AB1 B e portanto

AC2 = B1 C2 e ∠C2 B1 A = ∠BAB1 = 30◦ .

Analogamente, AC1 B2 = 30◦ . Daı́,

∠BC2 B1 = ∠C2 B1 A + ∠BAB1 = 60◦

e portanto
∠C1 OC2 = 180◦ − ∠BC2 B1 − ∠AC1 B2 = 90◦ .

Problema 2. Sejam ABC um triângulo e M o ponto médio do lado BC. Se D, E são os


pés das alturas relativas aos lados AC, AB, respectivamente, prove que M E = M D.

Solução.
A

D
E

B M C

Note que M E é mediana relativa à hipotenusa do triângulo BEC. Daı́,

M E = BM = CM

e, analogamente,
M D = BM = CM.
Assim, M E = M D.

3
POT 2012 - Geometria - Nı́vel 2 - Aula 5 - Prof. Cı́cero Thiago

Problema 3. Dado um quadrilátero ABCD, prove que os pontos médios M, N, P, Q dos


lados AB, BC, CD, DA formam um paralelogramo.

Solução.
A

M Q

B D

N P
C

Temos

• Triângulo ABC: M N k AC e M N = AC/2.

• Triângulo DAC: P Q k AC e P Q = AC/2.

Assim, M N k P Q e M N = P Q, isto é, M N P Q é paralelogramo.

Problema 4. Sejam ABC um triângulo e M o ponto médio de BC. Se AM = BM = CM ,


prove que ∠BAC = 90◦ .

Problema 5. (Torneio das Cidades) Sejam ABCD um paralelogramo, M o ponto médio


de CD e H o pé da perpendicular baixada de B a AM . Prove que BCH é um triângulo
isósceles.

Problema 6. Em um triângulo ABC, retângulo em A e isósceles, sejam D um ponto no


lado AC (A 6= D 6= C) e E o ponto no prolongamento de BA tal que o triângulo ADE é
isósceles. Se P é o ponto médio de BD, R o ponto médio de CE e Q a interseção entre
ED e BC, prove que o quadrilátero ARQP é um quadrado.

Problema 7. Seja ABC um triângulo acutângulo tal que ∠B = 2∠C, AD é perpendicular


a BC, com D sobre BC, e E o ponto médio de BC. Prove que AB = 2DE.

Problema 8. (China) Seja ABCD um trapézio, AD//BC, ∠B = 30o , ∠C = 60o , E, M, F, N


os pontos médios de AB, BC, CD, DA respectivamente. Se BC = 7, M N = 3, determine
a medida de EF .

4
POT 2012 - Geometria - Nı́vel 2 - Aula 5 - Prof. Cı́cero Thiago

Problema 9. (China) Seja ABCD um trapézio, AB//CD, ∠DAB = ∠ADC = 90o , e o


3
triângulo ABC é equilátero. Se a base média do trapézio EF = a, determine o compri-
4
mento da menor base AB, em função de a.

Problema 10. (Moscou) Seja ABCD um quadrilátero convexo e O um ponto em seu inte-
rior tal que ∠AOB = ∠COD = 120o , AO = OB, CO = OD. Sejam K, L, M os pontos
médios de AB, BC, CD respectivamente, prove que ∆KLM é equilátero.

Problema 11. (OBM) Num quadrilátero convexo, a reta que passa pelos pontos médios
de dois lados opostos forma ângulos iguais com ambas as diagonais. Mostre que as duas
diagonais têm o mesmo comprimento.

Problema 12. Se um segmento paralelo a um lado de um triângulo tem uma extremidade


no ponto médio de um lado e a outra extremidade no terceiro lado, prove que esta extre-
midade é ponto médio do terceiro lado.

Problema 13. (OBM) No triângulo ABC, D é ponto médio de AB e E ponto sobre o lado
BC tal que BE = 2 · EC. Sabendo que ∠ADC = ∠BAE, calcule o valor de ∠BAC.

Problema 14. (Austrália) Sejam ABC um triângulo e P um ponto em seu interior de modo
que ∠P AC = ∠P BC. Se L, M são os pés das perpendiculares por P aos lados BC, AC,
respectivamente, e D é o ponto médio de AB, prove que DL = DM .

Problema 15. (Romênia) Sejam ABC um triângulo isósceles com AB = AC, D o ponto
médio de BC, M o ponto médio de AD e N a projeção de D sobre BM . Prove que
∠AN C = 90◦ .

Problema 16. (Eslovênia) Seja ABCD um trapézio, com AB paralelo a CD. Sabendo que
a distância entre os pontos médios das bases é igual à distância entre os pontos médios das
diagonais, prove que ∠DAC e ∠DBC são ângulos obtusos.

Problema 17. Em um triângulo isósceles ABC, com AB = BC, sejam K, L pontos sobre
AB, BC, respectivamente, tais que AK + LC = KL. A reta paralela a BC passando pelo
ponto médio M de KL intersecta AC em N . Ache a medida de ∠KN L.

Problema 18. Sejam ABC um triângulo e D, E, F os pontos médios de BC, CA, AB,
respectivamente. Prove que

∠DAC = ∠ABE ⇐⇒ ∠AF C = ∠ADB.

5
POT 2012 - Geometria - Nı́vel 2 - Aula 5 - Prof. Cı́cero Thiago

Problema 19. Seja ABCD um trapézio com bases AB = a e CD = b. Sejam também M , N


os pontos médios dos lados AB, CD, respectivamente. Sabendo que ∠DAB+∠ABC = 90◦ ,
determine o comprimento de M N .

Problema 20. (Cone Sul) Seja ABC um triângulo acutângulo e sejam AN , BM e CP as


alturas relativas aos lados BC, CA e AB, respectivamente. Sejam R, S as projeções de N
sobre os lados AB, CA, respectivamente, e Q, W as projeções de N sobre as alturas BM ,
CP , respectivamente.

(a) Mostre que R, Q, W , S são colineares.

(b) Mostre que M P = RS − QW .

Problema 21. (TST Brasil) Sejam Q o ponto médio do lado AB de um quadrilátero ins-
critı́vel ABCD e S a interseção das diagonais AC e BD. Sejam P , R as projeções ortogonais
de S sobre AD, BC, respectivamente. Prove que P Q = QR.

Bibliografia

Lecture Notes on Mathematical Olympiad Courses


For Junior Section, vol. 1
Xu Jiagu

6
Programa Olímpico de Treinamento
Curso de Geometria - Nível 2 Aula 5
Prof. Rodrigo Pinheiro

Problemas OBM - 1◦ Fase

Problema 1. Dois espelhos formam um ângulo de 30◦ no ponto V . Um raio de luz, vindo
de uma fonte S, é emitido paralelamente a um dos espelhos e pe refletido pelo outro espelho
no ponto A, como mostra a figura. Depois de uma certa quantidade de reflexões, o raio
retorna a S. Se AS e AV têm 1m de comprimento, a distância percorrida pelo raio de luz,
em metros, é:

a) 2

b) 2 + 3
√ p
c) 2(1 + 3)
√ √
d) 1 + 2 + 3

e) 5 3

b
S A
b

30◦ V
b

Problema 2. Na figura, quanto vale x?

a) 6◦

b) 12◦

c) 18◦

d) 20◦

e) 24◦
POT 2012 - Geometria - Nı́vel 2 - Aula 5 - Prof. Rodrigo Pinheiro

5x

3x 2x

4x 6x

Problema 3. No triângulo P QR, a altura P F divide o lado QR em dois segmentos de


medidas QF = 9 e RF = 5. Se P R = 13, qual é a media de P Q?

a) 5

b) 10

c) 15

d) 20

e) 25

Problema 4. No desenho abaixo, o quadrilátero ABCD é um quadrado de lado 3cm e os


triângulos ABF e AED são ambos equiláteros. Qual a área do triangulo AEF?

a) 2cm2

b) 1, 5cm2

c) 3cm2

d) 4, 5cm2

e) 2, 5cm2

2
POT 2012 - Geometria - Nı́vel 2 - Aula 5 - Prof. Rodrigo Pinheiro

b
F

A
b

b
D

B b

b
E

b
C

Problema 5. Eu planejava fazer um curral quadrado, com uma certa área, usando uma
certa quantidade de cerca de arame farpado. Descobri, porém, que tenho 10% a menos de
cerca do que esperava. Por esta razão, a área cercada será:

a) 5% menor

b) 10% menor

c) 19% menor

d) 20% menor

e) 25% menor

Problema 6. O retângulo da figura a seguir está dividido em 7 quadrados. Se a área do


menor quadrado é igual a 1, a área do retângulo é igual a:

a) 42

b) 44

c) 45

d) 48

e) 49

3
POT 2012 - Geometria - Nı́vel 2 - Aula 5 - Prof. Rodrigo Pinheiro

Problema 7. Na organização retangular de pontos da figura abaixo, a distância entre pontos


vizinhos em uma mesma linha ou coluna é igual a 1cm. A área do pentágono ABCDE,
em cm2 , é igual a:

a) 9
19
b) 2

c) 10
21
d) 2

e) 11

b b b b
C b

b
D b b b b

b b b b b

b b
E b b b
B

b b b
A b b

Problema 8. A figura a seguir mostra um quadrado ABCD e um triângulo equilátero


BEF , ambos com lado de medida 1cm. Os pontos A, B e E são colineares, assim como os
pontos A, G e F . A área do triângulo BF G é, em cm2 :
1
a) 4
1
b) 3

3
c) 4

3
d) 12
3
e) 10

4
POT 2012 - Geometria - Nı́vel 2 - Aula 5 - Prof. Rodrigo Pinheiro

b
D b
C
b
F

b
G

b
A b
B b
E

Problema 9. No triângulo ABC, AB = 20, AC = 21 e BC = 29. Os pontos D e E sobre


o lado BC são tais que BD = 8 e EC = 9. A medida do ângulo ∠DAE, em graus, é igual
a:
a) 30

b) 40

c) 45

d) 60

e) 75
Problema 10. Seis retângulos idênticos são reunidos para formar um retângulo maior con-
forme indicado na figura com AB = 21cm. Qual é a área deste retângulo maior?

a) 210cm2

b) 280cm2

c) 430cm2

d) 504cm2

e) 588cm2

b A

b B

Problema 11. O desenho ao lado mostra um pedaço de papelão que será dobrado e colado
nas bordas para formar uma caixa retangular. Os ângulos nos cantos do papelão são todos
retos. Qual será o volume da caixa em cm2 ?
a) 1500

5
POT 2012 - Geometria - Nı́vel 2 - Aula 5 - Prof. Rodrigo Pinheiro

b) 3000

c) 4500

d) 6000

e) 12000

b
15cm b

b b

40cm 20cm

b b

b b

Problema 12. Na figura, os dois triângulos são equiláteros. Qual é o valor do ângulo x?

a) 30◦

b) 40◦

c) 50◦

d) 60◦

e) 70◦

75◦
65◦

Problema 13. Na figura, todas as circunferências menores têm o mesmo raio r e os centros
das circunferências que tocam circunferência maior são vértices de um quadrado. Sejam a
e b as áreas cinzas indicadas na figura. Então a razão ab é igual a:
1
a) 2
2
b) 3

6
POT 2012 - Geometria - Nı́vel 2 - Aula 5 - Prof. Rodrigo Pinheiro

c) 1
3
d) 2

e) 2

Problema 14. Na figura, a reta P Q toca em N o cı́rculo que passa por L, M e N . A reta
LM corta a reta P Q em R. Se LM = LN e a medida do ângulo P N L é α, α < 60◦ , quanto
mede o ângulo LRP ?

a) 3α − 180◦

b) 180◦ − 2α

c) 180◦ − α
α
d) 90◦ − 2

e) α

L
b

b
M

α Q
b
P b b
R b

Problema 15. São dadas duas tiras retangulares de papel com 20cm de comprimento, uma
com 5cm de largura e outra com 11cm de largura. Uma delas foi colocada sobre a outra,
perpendicularmente, de modo a formar a figura ilustrada ao lado. O perı́metro dessa figura,
em centı́metros é:

a) 50

b) 60

7
POT 2012 - Geometria - Nı́vel 2 - Aula 5 - Prof. Rodrigo Pinheiro

c) 80

d) 100

e) 120

Problema 16. Na figura, AB = AC, AE = AD e o ângulo BAD mede 30◦ . Então o ângulo
x, mede:

a) 10◦

b) 20◦

c) 15◦

d) 30◦

e) 5◦

b
A

30◦ E
b

b b
x b

B D C

Problema 17. Três quadrados são colocados pelos seus vértices entre si e a dois bastões
verticais, como mostra a figura.

A medida do ângulo x é:

a) 39◦

8
POT 2012 - Geometria - Nı́vel 2 - Aula 5 - Prof. Rodrigo Pinheiro

b) 41◦

c) 43◦

d) 44◦

e) 46◦

Problema 18. Um triângulo equilátero e um hexágono regular tem o mesmo perı́metro. A


razão entre a área do triângulo e a área do hexágono é:
1
a) 2

b) 1
2
c) 3
3
d) 2
1
e) 3

Problema 19. A figura a seguir representa um Tangram, quebra-cabeças chinês formado


por 5 triângulos, 1 paralelogramo e 1 quadrado. Sabendo que a área do Tangram a seguir
é 64cm2 , qual é a área, em cm2 , da regiâo sombreada?

a) 7.6

b) 8

c) 10.6

d) 12

e) 21.3

Problema 20. Na figura a seguir, ABC é um triângulo qualquer e ACD e AEB são
triângulos equiláteros. Se F e G são os pontos médios de EA e AC, respectivamente, a
razão BD
F G é:

9
POT 2012 - Geometria - Nı́vel 2 - Aula 5 - Prof. Rodrigo Pinheiro

b
D

E F b
A
b
b

b
G

b b
C
B

1
a) 2

b) 1
3
c) 2

d) 2

e) Depende das medidas dos lados de ABC.

Problema 21. Na figura, o lado AB do triângulo equilátero ABC é paralelo ao lado DG


do quadrado DEF G. Qual é o valor do ângulo x?

a) 80◦

b) 90◦

c) 100◦

d) 110◦

e) 120◦

Problema 22. O jardim da casa de Maria é formado por cinco quadrados de igual área e
tem a forma da figura abaixo. Se AB = 10m, então a área do jardim em metros quadrados
é:

a) 200

10
POT 2012 - Geometria - Nı́vel 2 - Aula 5 - Prof. Rodrigo Pinheiro


b) 10 5

c) 100
500
d) 3
100
e) 3

Problema 23. A figura abaixo é formada por três quadrados de lado 1cm e um retângulo
que os contorna. A área do retângulo é:


a) 3 2

b) 4 2

c) 6

d) 6 2

e) 8

Problema 24. Qual o menor√perı́metro inteiro possı́vel de um triângulo que possui um dos
lados com medida igual a 5. 23 ?

a) 8

b) 9

c) 10

d) 11

e) 12

11
POT 2012 - Geometria - Nı́vel 2 - Aula 5 - Prof. Rodrigo Pinheiro

Problema 25. Uma mesa de bilhar tem dimensões de 3m por 6m e tem caçapas nos seus
quatro cantos P , Q, R e S. Quando uma bola atinge na borda da mesa, sua trajetória
forma um ângulo igual ao que o ângulo anterior formava. Uma bola inicialmente a 1 metro
da caçapa P , é batida do lado SP em direção ao lado P Q, como mostra a figura. A quantos
metros de P a bola acerta o lado P Q se a bola cai na caçapa S após duas batidas na borda
da mesa?

a) 1
6
b) 7
3
c) 4
2
d) 3
3
e) 5

Gabarito
1) B
2) C
3) C
4) D
5) D
6) C
7) B
8) D
9) C
10) E
11) B
12) B
13) C
14) A
15) C
16) C
17) A
18) C
19) D

12
POT 2012 - Geometria - Nı́vel 2 - Aula 5 - Prof. Rodrigo Pinheiro

20) D
21) E
22) C
23) C
24) B
25) B

13
Polos Olímpicos de Treinamento
Curso de Geometria - Nível 2 Aula 6
Prof. Cícero Thiago

Quadriláteros Notáveis

1. Paralelogramo: Um quadrilátero convexo é dito um paralelogramo quando possuir


lados opostos paralelos.

Teorema 1. Um quadrilátero convexo é paralelogramo se, e somente se:


a) Ângulos opostos são iguais;
b) Lados opostos são iguais;
c) Diagonais cortam - se em seus pontos médios;

Demonstração.
(a)
A B E

D C

Suponhamos inicialmente que ABCD é um paralelogramo e seja E um ponto no prolon-


gamento do lado AB. É fácil perceber que ∠DAB = ∠CBE, pois são ângulos corres-
pondentes de retas paralelas. Por outro lado ∠CBE = ∠DCA, pois são ângulos alternos
internos. Portanto, ∠DAB = ∠DCA. Com o mesmo raciocı́nio podemos provar que
∠ADC = ∠ABC.

Reciprocamente, seja ABCD um quadrilátero convexo tal que ∠DAB = ∠DCB e ∠ADC =
∠ABC. Sabemos que ∠DAB + ∠DCB + ∠ADC + ∠ABC = 360◦ e com isso ∠DAB +
∠ABC = 180◦ e ∠DCB + ∠ADC = 180◦ . Por outro lado, ∠ABC + ∠CBE = 180◦ Con-
cluı́mos então, que ∠DAB = ∠CBE e, com isso, AD k BC. Com o mesmo raciocı́nio
podemos provar que AB k CD. E com isso ABCD é um paralelogramo.

(b)
POT 2012 - Geometria - Nı́vel 2 - Aula 6 - Prof. Cı́cero Thiago
A B

D C

Seja ABCD um paralelogramo. É fácil perceber, que ∠DCA = ∠BAC, pois são ângulos
alternos internos. Da mesma forma, ∠DAC = ∠BCA. Com isso, ∆DAC ≡ ∆ABC, pelo
caso A.L.A. Portanto, AD = BC e AB = CD.

Reciprocamente, seja ABCD um quadrilátero convexo tal que AD = BC e AB = CD. É


fácil perceber que, ∆DAC ≡ ∆ABC, pelo caso L.L.L. Portanto, ∠ADC = ∠ABC. De
maneira similar, podemos provar que ∠DAB = ∠DCB. Usando o fato provado no item
(a), podemos concluir que ABCD é um paralelogramo.

(c)
A B

D C

Seja ABCD um paralelogramo e seja M o ponto de encontro de suas diagonais. Já sa-
bemos, pelos itens anteriores, que os ângulos e lados opostos são iguais. Por outro lado,
∠DAC = ∠BCA, pois são ângulos alternos internos. Pelo mesmo motivo ∠ADB = ∠CBD
e com isso ∆ADM ≡ ∆CBM , pelo caso A.L.A. Portanto, AM = M C e DM = M B.

Reciprocamente, seja ABCD um quadrilátero convexo tal que suas diagonais se intersec-
tam em seus pontos médios, ou seja, AM = M C e DM = M B. É fácil perceber, que
∠DM A = ∠CM B, pois são ângulos opostos pelo vértice. Então, ∆ADM ≡ ∆CBM , pelo
caso L.A.L. Portanto, AD = BC. De maneira similar, podemos provar, que AB = CD.
Usando agora, o que foi provado no item (b), concluı́mos que ABCD é um paralelogramo.

2. Trapézio: Um quadrilátero convexo é trapézio se, e somente se, possui dois lados
paralelos. Um trapézio será dito isósceles se os lados não paralelos forem iguais e será dito
retângulo se um dos ângulos da base for reto.

Teorema 2. Os ângulos de cada base de um trapézio isósceles são congruentes e as diago-


nais também são congruentes.

Demonstração. Sejam AE e BF alturas do trapézio. Como AB e CD são parale-


los então AE = BF . Se AD = BC então ∆ADE ≡ ∆BCF pelo caso especial para

2
POT 2012 - Geometria - Nı́vel 2 - Aula 6 - Prof. Cı́cero Thiago

triângulos retângulos cateto - hipotenusa. Com isso, ∠ADC = ∠BCD. Temos também
que ∆ADC ≡ ∆BCD pelo caso L.A.L, portanto AC = BD.

b
A b
B

b b b b

D E F C

3. Losango: Paralelogramo com todos os lados iguais.

Teorema 3. As diagonais do losango são perpendiculares.

Demonstração. Como o losango é um paralelogramo então as diagonais cortam - se em


seus ponyos médios, ou seja, AM = M C e BM = M D. Com isso, ∆AM B ≡ ∆AM D,
pelo caso L.L.L, portanto ∠AM B = ∠AM D. Como ∠AM B + ∠AM D = 180◦ , então
∠AM B = ∠AM D = 90◦ .

b
A

B b b
M b
D

4. Retângulo: Paralelogramo com quatro ângulos retos.

Teorema 4. A diagonais de um retângulo são iguais.

3
POT 2012 - Geometria - Nı́vel 2 - Aula 6 - Prof. Cı́cero Thiago

Demonstração. É fácil ver que ∆ADC ≡ ∆BCD pelo caso L.A.L. Portanto, AC = BD.

A b b
B

b b

D C

5. Quadrado: Retângulo com os quatro lados iguais.

Exercı́cios Resolvidos

1. Se dois segmentos são iguais e paralelos, então suas extremidades são os vértices de
um paralelogramo.
Solução.
A B
M

D C

Sejam AD e BC os segmentos iguais e paralelos. Vamos então construir os segmen-


tos DB e AC, que se intersectam em M . É fácil perceber que ∠DAC = ∠BCA,
pois são ângulos alternos internos. Pelo mesmo motivo, ∠ADC = ∠CBD. Portanto,
∆ADM ≡ ∆BCM , pelo caso A.L.A. Usando o resultado provado no item (c) do
teorema (1), provamos que ABCD é um paralelogramo.

2. Mostre que se por um ponto na base de um triângulo isósceles traçamos retas para-
lelas aos lados congruentes, então se forma um paralelogramo cujo perı́metro é igual
a soma dos comprimentos dos lados congruentes.

Solução. Seja D um ponto da base do triângulo isósceles ABC e sejam DE e DF


os segmentos paralelos aos lados iguais. É fácil ver que AF DE é um paralelogramo
pois DE k AC e DF k AB. Portanto, AF = DE, AE = DF e os triângulos BDE
e CDF são isósceles assim BE = DE e DF = CF . É fácil perceber que o triângulo

4
POT 2012 - Geometria - Nı́vel 2 - Aula 6 - Prof. Cı́cero Thiago

ABC e o paralelogramo AF DE possuem o mesmo perı́metro.

b
A

b F

E b

α α α α
b b b

B D C

3. (OCM) Sejam AB e CD as bases de um trapézio tal que a base menor CD é igual à


soma dos lados não paralelos do trapézio. Se E é um ponto de CD e EA é a bissetriz
do ângulo ∠A, mostre que EB é também bissetriz do ângulo ∠B.

Solução. Como AB k CD então ∠BAE = ∠DEA e, com isso, AD = BE. Como


CD = AD + BC então EC = CB. Assim, ∠CEB = ∠CBE. Mas AB k CD então
∠CEB = ∠EBA.

b
D E b
C
b

α β

α β
b
α β b

A B

4. (Cone Sul) Sejam A, B e C três pontos (não colineares) e E(6= B) um ponto qual-
quer que não pertence à reta AC. Construa paralelogramos ABCD (nesta ordem) e
AECF (também nesta ordem). Demonstre que BE k DF .

Solução. ABCD e AECF são paralelogramos de diagonais AC, BD e AC, F E


respectivamente. Como as diagonais de um paralelogramo se cortam em seus pontos

5
POT 2012 - Geometria - Nı́vel 2 - Aula 6 - Prof. Cı́cero Thiago

médios e AC é uma diagonal comum, o ponto médio de AC é o ponto médio de BD


e de F E. Logo BEDF é um quadrilátero cujas diagonais BD e F E cortam - se em
seus pontos médios. Portanto BEDF é um paralelogramo e BE k DF .

b
B

b
E

A
b b

F b

5. (Torneio das Cidades) Em um quadrado ABCD, K é um ponto do lado BC e a


bissetriz do ∠KAD intersecta o lado CD no ponto M . Prove que o comprimento do
segmento AK é igual à soma dos comprimentos dos segmentos DM e BK.

Solução. Seja L o ponto no prolongamento de BC tal que BL = DM . Como AB =


AD e ∠ABL = 90◦ = ∠ADM então ∆ABL ≡ ∆ADM . Assim, ∠BAL = ∠DAM e
∠ALK = ∠AM D. Por outro lado

∠KAL = ∠BAL + ∠KAB

= ∠M AD + ∠KAB
= ∠M AK + ∠KAB
= ∠M AB
= ∠AM D.

a última igualdade acontece porque AB e DC são paralelos. Segue que ∠KAL =


∠ALK e, portanto, AK = KL = KB + BL = KB + DM.

6
POT 2012 - Geometria - Nı́vel 2 - Aula 6 - Prof. Cı́cero Thiago

D A
b b

M b

b b b b
L
C K B

6. (Torneio das Cidades) ABCD é um paralelogramo. Um ponto M é escolhido sobre


o lado AB tal que ∠M AD = ∠AM O, onde O é o ponto de interseção das diagonais
do paralelogramo. Prove que M D = M C.

Solução. Seja N o interseção de M O e CD. Temos que ∠M AD = ∠AM N então


AM N D é um trapézio isósceles. Por simetria, AM = N C então AM CN é um para-
lelogramo. Com isso, ∠M DC = AN D = ∠M CD e, portanto, M C = M D.

b
A b
M b
B

b
O

b b
N b
D C

Exercı́cios Propostos

7
POT 2012 - Geometria - Nı́vel 2 - Aula 6 - Prof. Cı́cero Thiago

1. No quadrado ABCD consideram - se as diagonais AC e BD. Seja P um ponto qual-


quer pertencente a um dos lados. Demonstrar que a soma das distâncias de P às
duas diagonais é constante.

2. (Maio) Num paralelogramo ABCD, BD é a diagonal maior. Ao fazer coincidir B


com D mediante uma dobra se forma um pentágono regular. Calcular as medidas
dos ângulos que forma a diagonal BD com cada um dos lados do paralelogramo.

3. (Maio) No retângulo ABCD de lados AB, BC, CD e DA, seja P um ponto do lado
AD tal que ∠BP C = 90◦ . A perpendicular a BP traçada por A corta BP em M e
a perpendicular a CP traçada por D corta CP em N . Demonstre que o centro do
retângulo está no segmento M N .

4. Sejam ABC e ABD triângulo com o lado AB comum. O triângulo ABC tem
∠BAC = 90◦ e AB = 2AC. O triângulo ABD tem ∠ADB = 90◦ e AD = BD.
O segmento CD corta o segmento AB em O. Calcule a medida de BO sabendo que
AC = 4.

5. (OBM) O trapézio ABCD tem bases AB eCD. O lado DA mede x e o lado BC


mede 2x. A soma dos ângulos ∠DAB e ∠ABC é 120◦ . Determine o ângulo ∠DAB.

6. No quadrilátero convexo ABCD, sejam E e F os pontos médios dos lados AD e BC,


respectivamente. Os segmentos CE e DF se cortam em O. Demonstre que se as retas
AO e BO dividem o lado CD em três partes iguais então ABCD é um paralelogramo.

7. Seja ABCDEF um hexágono tal que seus lados opostos são respectivamente para-
lelos, ou seja, AB k DE, BC k EF e CD k F A. Se AB = DE, demonstre que
BC = EF e CD = F A.

8. Seja ABCD um paralelogramo tal que M é o ponto médio de BC. Seja T a projeção
de D sobre M A. Prove que CT = CD.

B A b T
b b

M b

b b
C D

8
POT 2012 - Geometria - Nı́vel 2 - Aula 6 - Prof. Cı́cero Thiago

9. Prove que o segmento que liga os pontos médios dos lados opostos de um quadrilátero
convexo passa pelo ponto médio do segmento que liga os pontos médios das diagonais.

10. Seja ABCD um paralelogramo. Pelo vértice A é traçada uma reta r e sejam E, F
e G as projeções de B, C e D sobre r, respectivamente. Prove que se r estiver no
exterior do paralelogramo, então CF = BE + DG e, se r estiver no interior, então
CF = |BE − DG|.

11. Sobre os lados AB e AC do triângulo ABC são construı́dos no exterior triângulos


isósceles semelhantes ABC ′ e CAB ′ . Prove que AB ′ A′ C ′ é um paralelogramo.

12. Os lados AB, BC, CD e DA de um quadrilátero ABCD são divididos pelos pelos
E, F , G e H da seguinte forma:
AE CF CG DH
= = = .
EB FB GD HA

Prove que EF GH é um paralelogramo.

13. Seja P1 P2 P3 P4 P5 um pentágono convexo. Seja Qi o ponto de interseção dos segmen-


tos que unem os pontos médios dos lados opostos do quadrilátero Pi+1 Pi+2 Pi+3 Pi+4
onde Pk+5 = Pk , k ∈ N e i ∈ {1, 2, 3, 4, 5}. Prove que os pentágonos P1 P2 P3 P4 P5 e
Q1 Q2 Q3 Q4 Q5 são semelhantes.

Sugestões
2. Seja O o ponto de interseção das diagonais de um paralelogramo ABCD. Seja EF
um segmento que passa por O com extremidades E e F sobre os lados AB e CD,
respectivamente. Então, EO = EF .

3. Use o fato que as diagonais de um paralelogramo cortam - se em seus pontos médios.

8. Trace CP ⊥ DT , com P em DT .

9. Use base média.

11. Use semelhança de triângulos.

12. Use Teorema de Tales.

9
POT 2012 - Geometria - Nı́vel 2 - Aula 6 - Prof. Cı́cero Thiago

13. Os pontos médios dos lados de um quadrilátero são vértices de um paralelogramo.

Bibliografia

1. Problemas 18 - Olimpı́ada Matemática Argentina


Patrı́cia Fauring e Flora Gutierrez
Red Olı́mpica

2. Problemas 19 - Olimpı́ada Matemática Argentina


Patrı́cia Fauring e Flora Gutierrez
Red Olı́mpica

3. Problemas 20 - Olimpı́ada Matemática Argentina


Patrı́cia Fauring e Flora Gutierrez
Red Olı́mpica

4. Olimpı́adas de Mayo - I a VIII


Patrı́cia Fauring, Flora Gutierrez, Carlos Bosch e Marı́a Gaspar
Red Olı́mpica

5. Olimpı́adas de Mayo - IX a XVI


Patrı́cia Fauring, Flora Gutierrez, Carlos Bosch e Marı́a Gaspar
Red Olı́mpica

6. International Mathematics Tournament of Towns - 1997 - 2002


AM Storozhev
AMT

7. Coleção Elementos da Matemática, vol. 2 - Geometria Plana


Marcelo Rufino de Oliveira e Márcio Rodrigo da Rocha Pinheiro

8. Challenging Problems in Geometry


Alfred S. Posamentier e Charles T. Salkind

9. Problems and Solutions in Euclidean Geometry


M. N. Aref e William Wernick

10. Geometrı́a
Radmila Bulajich Manfrino e José Antonio Gómez Ortega
Cuadernos de Olimpiadas de Matemáticas

10
POT 2012 - Geometria - Nı́vel 2 - Aula 6 - Prof. Cı́cero Thiago

11. Tópicos de Matemática Elementar, vol. 2


Geometria Euclidiana Plana
Antonio Caminha Muniz Neto
SBM

12. Episodes in Nineteenth and Twentieth Euclidean Geometry


Ross Honsberger
MAA

13. Problems in Plane and Solid Geometry, vol. 1 - Plane Geometry


Viktor Prasolov

14. Advanced Euclidean Geometry


Alfred Posamentier

15. Lessons in Geometry


I. Plane Geometry
Jacques Hadamard
AMS

16. Hadamard’s Plane Geometry


A Reader’s Companion
Mark Saul
AMS

17. Olimpı́adas Cearenses de Matemática, Ensino Fundamental, 1981 - 2005


Emanuel Carneiro, Francisco Antônio M. de Paiva e Onofre Campos

18. Problemas de las Olimpiadas Matematicas del Cono Sur (I a IV)


Fauring - Wagner - Wykowski - Gutierrez - Pedraza - Moreira
Red Olı́mpica

19. Explorations in Geometry


Bruce Shawyer
World Scientific

20. Treinamento Cone Sul, vol.2.


Bruno Holanda, Cı́cero Magalhães, Samuel Barbosa e Yuri Lima.

11
Polos Olímpicos de Treinamento
Curso de Geometria - Nível 2 Aula 7
Prof. Cícero Thiago

Ângulos na circunferência

Definição 1: O ângulo inscrito relativo a uma circunferência é um ângulo que tem o vértice
na circunferência e os lados são secantes a ela.

Ab b
P

b
O

Assim, ∠AP B é o ângulo inscrito e ∠AOB é o ângulo central que é igual à medida do arco,
que não contém P , determinado na circunferência pelos pontos A e B.

Teorema 1. Um ângulo inscrito é metade do ângulo central correspondente.

Demonstração. A prova será dividida em três casos.


1◦ caso:
O triângulo OBC é isósceles e, com isso, ∠OBC = ∠OCB . Então, ∠AOC = ∠OBC +
∠OCB = 2∠OBC (propriedade do ângulo externo).
POT 2012 - Geometria - Nı́vel 2 - Aula 7 - Prof. Cı́cero Thiago

b
B
C b α
α

2α b

2◦ caso:
Pelo 1◦ caso temos que ∠AOC = 2∠ABC e ∠AOD = 2∠ABD. Portanto, ∠COD =
2∠CBD.

b
B
C α
b
α
β
2α b
O


β
b

A
b

3◦ caso:

Pelo 1◦ caso temos que ∠EOD = 2∠ECD, então 2α + 2θ = 2 · (α + β) ⇔ θ = β. Portanto,


∠AOD = 2∠ACD.

2
POT 2012 - Geometria - Nı́vel 2 - Aula 7 - Prof. Cı́cero Thiago

b
C

α β
O b

2θ D
2α α
b
E b

Definição 2: Dizemos que uma reta é tangente a uma circunferência se essa reta intersecta
a circunferência em um único ponto.

Teorema 2. Toda reta perpendicular a um raio na sua extremidade da circunferência é


tangente à circunferência.
Demonstração.

b
A

B
b

b
O

Suponha que OA ⊥ r mas r não é tangente à circunferência, e seja B 6= A o segundo


ponto de interseção. Isso é um absurdo pois o triâgulo OAB seria isósceles, pois OA = OB
(raio da circunferência), com os ângulos da base iguais a 90◦ . Portanto, r é tangente à
circunferência.

Teorema 3. Toda tangente a uma circunferência é perpendicular ao raio no ponto de


tangência.

Demonstração.

3
POT 2012 - Geometria - Nı́vel 2 - Aula 7 - Prof. Cı́cero Thiago

r
b
B b
A

b
O

Seja A o ponto de tangencia. Qualquer ponto de r está a uma distância maior do que A
do centro. Com isso, OA é a menor distância de O para a reta r. Portanto, OA ⊥ r.

Definição 3: Um ângulo de segmento relativo a uma circunferência é um ângulo que tem


o vértice na circunferência, um lado secante e outro lado tangente à circunferência.

b
O

b b

A B
θ

O ângulo θ da figura é um ângulo de segmento.

Teorema 4. Um ângulo de segmento é a metade do ângulo central correspondente.

Demonstração.
Calculando a soma dos ângulos internos no triângulo AOB temos 2α+2β = 180◦ ⇔ α+β =
90◦ . Mas, θ + β = 90◦ . Portanto, θ = α.

4
POT 2012 - Geometria - Nı́vel 2 - Aula 7 - Prof. Cı́cero Thiago

b
O

β 2α β b
b

A B
θ

Definição 4: Chamaremos de ângulo excêntrico interior qualquer ângulo formado por duas
cordas de uma circunferência. Na figura abaixo, temos que ∠BED é um ângulo excêntrico
∠AOC + ∠BOD
interior que satisfaz ∠BED = , pois ∠BED = ∠ABC + ∠DCB =
2
∠AOC ∠BOD ∠AOC + ∠BOD
∠EBC + ∠ECB = + = .
2 2 2
A
b b
D

E
b

C b b O

Definição 5: Chamaremos de ângulo excêntrico exterior o ângulo formado por duas


secantes a uma circunferência traçadas por um ponto no exterior. Na figura abaixo,
∠BOD − ∠AOC
∠BP D é um ângulo excêntrico exterior que satisfaz ∠BP D = , pois
2
∠BOD ∠AOC ∠BOD − ∠AOC
∠BP D = ∠BAD − ∠ADP = − = .
2 2 2

5
POT 2012 - Geometria - Nı́vel 2 - Aula 7 - Prof. Cı́cero Thiago

B
b

b
A

O P
b b

C
b

Exercı́cios Resolvidos

1. (OBM) O triângulo ABC está inscrito na circunferência S e AB < AC. A reta que
contém A e é perpendicular a BC encontra S em P (P 6= A). O ponto X situa-se so-
bre o segmento AC e a reta BX intersecta S em Q (Q 6= B). Mostre que BX = CX
se, e somente se, P Q é um diâmetro de S.

Solução. Vamos dividir o problema em duas partes:


(a) BX = CX ⇒ P Q é um diâmetro de S. Seja ∠ACB = α. Assim, temos que
∠QBC = α (já que BX = CX) e ∠P AC = 180◦ − 90◦ − α = 90◦ − α. Observe
que os ângulos ∠P AC = ∠P BC. Assim, vemos que ∠P BC = ∠P AC = 90◦ − α ⇒
∠P BQ = 90◦ − α + α = 90◦ ⇒ P Q é diâmetro.

A Q
b b

X
90◦ − α b

b
α α b

B C
90◦ −α

6
POT 2012 - Geometria - Nı́vel 2 - Aula 7 - Prof. Cı́cero Thiago

(b) P Q é um diâmetro de S ⇒ BX = CX. Se ∠ACB = α, ∠P AC = ∠P BC =


90◦ − α. Mas P Q é diâmetro, ou seja, ∠P BQ = 90◦ ⇒ 90◦ − α + ∠QBC = 90◦ ⇒
∠QBC = α ⇒ ∆BXC é isósceles ⇒ BX = XC.

2. Sobre um cı́rculo de diâmetro AB são escolhidos os pontos C, D e E em um semiplano


determinado por AB e F no outro semiplano, tais que ¯ AC = CD¯ = BE ¯ = 20◦ e
¯ = 60◦ . Seja M a intersecção de BD e CE. Prove que F M = F E.
BF
Solução.

D
b

C
b b
M b
E

A b b b
B
O

Seja O o centro da circunferência. Vamos provar que os triângulos ∆OM F e ∆BEF


são congruentes. Como ∠BOF = 60◦ então o triângulo ∆BOF é equilátero e
OF = BF . Além disso, BE ¯ = CD ¯ ⇒ BE = CD e ∠DCE = ∠EBD = 60◦ ,
ou seja, ∆CDM ≡ ∆EBM ⇒ CM = BM ⇒ ∆OCM = ∆OBM ⇒ ∠M OB = 80◦ .
Como ∠OBE = ∠OBM + ∠EBD = ∠ABD + 60◦ = 20◦ + 60◦ = 80◦ então o trapézio
M OBE é isósceles e, com isso, M O = EB. Finalmente, ∠M OF = 80◦ +60◦ = 140◦ =
∠EBF . Isto prova que os triângulos ∆OM F e ∆BEF são congruentes. Portanto,
F M = F E.

3. Sejam dois cı́rculos C1 e C2 , com C2 tangente interno a C1 no ponto P . Seja s uma


reta tangente a C2 em um ponto B, e que corta C1 em A e C. Mostre que P B é
bissetriz do ângulo ∠AP C.

Solução.

7
POT 2012 - Geometria - Nı́vel 2 - Aula 7 - Prof. Cı́cero Thiago

b
A

P b

Seja r a reta tangente às duas circunferências em P . Seja O o ponto de intersecção


de r com o prolongamento AC. Temos que

¯
CP
∠CP O = ∠P AC =
2
OP = OB ⇒ ∠P BO = ∠BP C + CP O.

Mas ∠P BO é ângulo externo do triângulo ∆P BA ⇒ ∠P BO = ∠P AC + ∠BP A.


Portanto, ∠BP C = ∠BP A.

4. Seja O o centro da circunferência circunscrita ao triângulo acutângulo ABC e seja D


a projeção de A sobre BC. Prove que ∠DAB = ∠OAC.
Solução.

8
POT 2012 - Geometria - Nı́vel 2 - Aula 7 - Prof. Cı́cero Thiago

A
b

b
O

β
b b b

B D C
β

Seja AE um diâmetro. Além disso, ∠ABC = ∠AEC. Portanto, ∠BAD = ∠EAC.

5. (Itália) Um triângulo ABC acutângulo está inscrito em um cı́rculo de centro O . Seja


D a intersecção da bissetriz de A com BC e suponha que a perpendicular a AO por
D, corta a reta AC em um ponto P interior a AC. Mostre que AB = AP .
Solução.

9
POT 2012 - Geometria - Nı́vel 2 - Aula 7 - Prof. Cı́cero Thiago

A
b

b
O

b
P

b
E

b b b b

B F D C

Usando o resultado obtido no problema 4 temos que ∠BAF = ∠EAP . Como AD é


bissetriz do ângulo ∠A, então ∠F AD = ∠DAE. Com isso, ∆F AD ≡ ∆DAE, pelo
caso A.L.A., assim AF = AE. Desse modo, ∆AEP ≡ ∆ABF , pelo caso A.L.A.
Finalmente, AB = AP .

6. (Irã) Em um triângulo ABC a bissetriz do ângulo ∠BAC intersecta o lado BC no


ponto D. Seja Γ um cı́rculo tangente a BC no ponto D e que passa pelo ponto A. Se
M é o segundo ponto de intersecção de AC com Γ e se BM intersecta o cı́rculo em
P , mostre que AP é uma mediana do triângulo ABD.
Solução.

10
POT 2012 - Geometria - Nı́vel 2 - Aula 7 - Prof. Cı́cero Thiago

A
b

α α
β

b
M

P b
α

b α−β b
β b b

B N D C

Como BC é tangente à circunferência Γ, temos que ∠N DP = ∠DM P = ∠P AD = β.


Além disso, ∠M P D = ∠M AD = ∠BAD = α. Sendo assim, ∆N P D ∼ ∆N AD,
NP ND
então = ⇔ N D 2 = N P · N A. Temos também que ∆N BP ∼ ∆N AB,
ND NA
PN NB
então = ⇔ N B 2 = N P · N A. Portanto, N B 2 = N D 2 ⇔ N B = N D.
NB AN

Exercı́cios propostos

1. Na figura, a reta t é tangente ao cı́rculo e paralela ao segmento DE. Se AD = 6,


AE = 5 e CE = 7, o valor da medida do segmento BD é:

11
POT 2012 - Geometria - Nı́vel 2 - Aula 7 - Prof. Cı́cero Thiago

t
A
b

D b b E

(a) 3, 5 (b) 4 (c) 4, 5 (d) 5 (e) 5, 5

2. São dadas duas circunferências secantes, com pontos de intersecção C e D. Traça - se


por C uma secante à duas circunferências, que intersecta uma delas em E e a outra
em F . Mostre que o ângulo ∠EDF é constante.

3. As extremidades de uma corda ST , com comprimento constante, são movidos ao


longo de um semicı́rculo com diâmetro AB. Seja M o ponto médio de ST e P o pé
da perpendicular de S sobre AB. Prove que a medida do ângulo ∠SP M independe
da posição de ST .

4. É dado um triângulo ABC. Sejam O o centro da circunferência circunscrita ao


triângulo, I o centro da circunferência inscrita no triângulo, D 6= A a intersecção da
reta AI com a circunferência circunscrita. Prove que CD = BD = ID.

5. Se os lados AB e AC de um triângulo são diâmetros de duas circunferências, prove


que o outro ponto comum às circunferências está em BC.

6. Sejam C1 e C2 duas circunferência tangentes exteriores em T . Sejam A e B pontos


de C1 tais que a reta AB é tangente a C2 em P . Prove que T P é bissetriz externa

12
POT 2012 - Geometria - Nı́vel 2 - Aula 7 - Prof. Cı́cero Thiago

do triângulo ∆AT B.

7. Na figura abaixo seja T o ponto de tangencia das circunferências. Prove que ∠M T P =


∠QT N .

b
T

M
b

P
b

b Q
b N

Sugestões/Soluções

1. BD = 4. Use ângulo de segmento para concluir que ∆ADE ∼ ∆ABC.


2. Use que em uma circunferência, a medida do ângulo inscrito é metade da medida do
ângulo central que subtende o mesmo arco.

3.

13
POT 2012 - Geometria - Nı́vel 2 - Aula 7 - Prof. Cı́cero Thiago

b
T
b
M
S b

b b b
B
A P

b
C
S1

Seja C a circunferência de diâmetro AB. Seja S1 o simétrico de S com relação a AB. É fácil
ver que P é o ponto médio de SS1 e seja M o ponto médio de ST . Temos que P M k S1 T .
1 ˜ ˜ só depende do comprimento de ST .
Então ∠SP M = ∠SS1 T = · ST . Por outro lado, ST
2
Portanto, segue o resultado.

4. Use ângulo externo e ângulos inscritos.

5. Use que todo ângulo inscrito em uma semicircunferência mede 90◦ .

6. Use ângulos de segmento.

7. Use ângulos inscritos.

Bibliografia

1. Fundamentos de Matemática Elementar, vol.9.


Osvaldo Dolce e José Nicolau Pompeo.

2. Geometrı́a - Una visión de la planimetrı́a.


Lumbreras.

14
POT 2012 - Geometria - Nı́vel 2 - Aula 7 - Prof. Cı́cero Thiago

3. Challenging Problems in Geometry


Alfred S. Posamentier e Charles T. Salkind

4. Problems and Solutions in Euclidean Geometry


M. N. Aref e William Wernick

5. Geometrı́a
Radmila Bulajich Manfrino e José Antonio Gómez Ortega
Cuadernos de Olimpiadas de Matemáticas

6. Tópicos de Matemática Elementar, vol. 2


Geometria Euclidiana Plana
Antonio Caminha Muniz Neto
SBM

15
Polos Olímpicos de Treinamento
Curso de Geometria - Nível 2 Aula 8
Prof. Cícero Thiago

Quadriláteros inscritı́veis

Teorema 1. Um quadrilátero é inscritı́vel se, e somente se, a soma dos ângulos opostos é
180◦ .

˙
BAD ˙
BCD
Demonstração. ⇒ Seja ABCD um quadrilátero inscritı́vel. Temos que + =
2 2
360 , ou seja, 2∠A + 2∠C = 360 ⇔ ∠A + ∠C = 180 . Como a soma dos ângulos internos
◦ ◦ ◦

de um quadrilátero convexo é 360◦ , então ∠B + ∠D = 180◦ .

Ab

b
B

b
O

b b

D C

⇐ Seja ABCD um quadrilátero tal que ∠A + ∠C = ∠B + ∠D = 180◦ . Vamos admitir,


de maneira falsa, que ABCD não é inscritı́vel. Seja E a intersecção de BC com a circun-
ferência circunscrita ao triângulo ABD. Sendo assim, ∠A + ∠E = 180◦ ⇒ ∠C = ∠E, o
que é um absurdo pela propriedade do ângulo externo. Portanto, ABCD é inscritı́vel.
POT 2012 - Geometria - Nı́vel 2 - Aula 8 - Prof. Cı́cero Thiago

A
b

b
B

b
O

b
C

b b

D E

Teorema 2. Um quadrilátero é inscritı́vel se, e somente se, o ângulo entre um lado e uma
diagonal é igual ao ângulo entre o lado oposto e a outra diagonal.

Demonstração. ⇒

A
b

B
b

O
b

b b

D C

¯
DC
Seja ABCD um quadrilátero inscritı́vel. É fácil ver que ∠DAC = ∠DBC = .
2

2
POT 2012 - Geometria - Nı́vel 2 - Aula 8 - Prof. Cı́cero Thiago

A
b

B
b

O
b

C
b

b b

D E

⇐ Seja ABCD um quadrilátero tal que ∠ADB = ∠ACB. Vamos admitir, de maneira
falsa, que ABCD não é inscritı́vel. Seja E a intersecção de BC com a circunferência cir-
cunscrita ao triângulo ABD. Sendo assim, ∠ADB = ∠ACB = ∠AEB, o que é um absurdo
pela propriedade do ângulo externo. Portanto, ABCD é inscritı́vel.

Exercı́cios Resolvidos

1. Em um triângulo ABC, ∠BAC = 100◦ e AB = AC. Seja BD a bissetriz de ∠ABC,


com D sobre o lado AC. Prove que AD + BD = BC.

Solução.
É fácil ver que ∠ABD = ∠DBC = 20◦ . Seja E um ponto sobre BC tal que
BD = BE. Basta provar que EC = AD. Veja que ∠BDE = ∠BED = 80◦ .
Como ∠BED = 80◦ e ∠BCD = 40◦ , então ∠EDC = 40◦ , ou seja, ED = EC. Por
outro lado, ABED é um quadrilátero inscritı́vel pois ∠BAD + ∠BED = 180◦ , assim
∠EAD = ∠EBD = 20◦ e ∠AED = ∠ABD = 20◦ . Portanto, AD = ED = EC e,
dessa forma, BC = AD + BD.

3
POT 2012 - Geometria - Nı́vel 2 - Aula 8 - Prof. Cı́cero Thiago

A
b

b
D

b b b

B E C

2. (Inglaterra) No triângulo acutângulo ABC, CF é altura, com F em AB e BM é


mediana, com M em CA. Se BM = CF e ∠M BC = ∠F CA, prove que o triângulo
ABC é equilátero.

Solução.
A
b

F b b
M

b b

B C

Temos que F M = AM = M C e, com isso, ∠M F C = ∠F CM , ou seja, o quadrilátero


F BCM é inscritı́vel. Dessa forma, ∠F CM = ∠F BM e ∠BM C = ∠BF C = 90◦ . É
fácil ver que ∆BM C ≡ ∆BM A, pelo caso A.L.A, então AB = BC. Veja também
que ∆BM C ≡ ∆BF C, pelo caso cateto - hipotenusa, então ∠BCM = ∠CBF e,
portanto, AC = AB. Finalmente, AB = AC = BC.

4
POT 2012 - Geometria - Nı́vel 2 - Aula 8 - Prof. Cı́cero Thiago

3. Seja M um ponto no interior de um quadrilátero convexo ABCD tal que ABM D é


um paralelogramo. Prove que se ∠CBM = ∠CDM , então ∠ACD = ∠BCM .

Solução.

A B
b b

b
N

b b

D M

Seja N um ponto tal que BN k M C e N C k BM . Então N A k CD, ∠N CB =


∠CBM = ∠CDM = ∠N AB, ou seja, os pontos A, B, N e C são concı́clicos. Então,
∠ACD = ∠N BC = ∠BCM .

4. (Cone Sul) Seja ABCD um quadrilátero convexo tal que suas diagonais AC e BD
são perpendiculares. Seja P a intersecção de AC e BD e seja M o ponto médio de
AB. Mostre que o quadrilátero ABCD é inscritı́vel se, e somente se, as retas P M e
CD são perpendiculares.

Solução. Primeiramente vejamos quando P M e CD são perpendiculares. Seja K a


intersecção de P M e CD. Como no triângulo ABP , retângulo em P , M é o ponto
médio da hipotenusa AB ⇒ P M = M A = M B. Assim, seja

∠ABD = θ ⇒ ∠M P B = θ ⇒ ∠AM P = 2θ ⇒ ∠M P A = 90◦ − θ ⇒

∠CP K = ∠AP M = 90◦ − θ.

5
POT 2012 - Geometria - Nı́vel 2 - Aula 8 - Prof. Cı́cero Thiago

Como ∠P KC = 90◦ ⇒ ∠P CD = θ. Logo, ∠ABD = ∠ACD = θ ⇒ o quadrilátero


ABCD é inscritı́vel.

b
B
E
b
θ
A b

90◦ − θ
90◦ − θ θ

b P

90◦ − θ

D
b
θ
K b

Vejamos agora o caso em que o quadrilátero ABCD é inscritı́vel. Do mesmo modo


como M é o ponto médio da hipotenusa AB do triângulo retângulo AP B então
P M = M A = M B. Logo, se ∠ABD = θ ⇒ ∠BAP = ∠M P A = 90◦ − θ ⇒
∠CP K = 90◦ − θ e como ABCD é inscritı́vel ⇒ ∠ACD = ∠ABD = θ ⇒ ∠P KC =
180◦ − (90◦ − θ + θ) = 90◦ ⇒ M P ⊥ CD. Portanto, ABCD é inscritı́vel se, e somente
se, P M ⊥ CD.

6
POT 2012 - Geometria - Nı́vel 2 - Aula 8 - Prof. Cı́cero Thiago

b
B

b
E
θ
A b 2θ
90◦ − θ
90◦ − θ

b P

90◦ − θ

D
b
θ
K b

5. Prove que as bissetrizes internas dos quatro ângulos de um quadrilátero convexo de-
terminam um quadrilátero inscritı́vel.

Solução. É fácil ver que ξ = 180◦ − α − β e que ψ = 180◦ − γ − θ. Dessa forma,

ξ + ψ = 360◦ − (α + β + γ + θ)

= 360◦ − 180◦ = 180◦ .

7
POT 2012 - Geometria - Nı́vel 2 - Aula 8 - Prof. Cı́cero Thiago

B
b

γ
γ
A E
b
b

α b
H
α F
b
ξ ψ
β θ
b β θ b

D C

Exercı́cios Propostos

1. (BAMO) Seja k um cı́rculo no plano xy com centro sobre o eixo y e passando pelos
pontos A(0, a) e B(0, b) com 0 < a < b. Seja P um ponto qualquer do cı́rculo, dife-
rente de A e B. Seja Q a intersecção da reta que passa por P e A com o eixo x, e
seja O(0, 0). Prove que ∠BQP = ∠BOP .

2. (OBM) As diagonais de um quadrilátero inscritı́vel ABCD se intersectam em O. Os


cı́rculos circunscritos aos triângulos AOB e COD intersectam as retas BC e AD,
pela segunda vez, nos pontos M , N , O e Q. Prove que o quadrilátero M N P Q está
inscrito em um cı́rculo de centro O.

3. Um quadrilátero convexo está inscrito em um cı́rculo de centro O. As diagonais AC


e BD intersectam - se em P . Os cı́rculos circunscritos aos triângulos ABP e CDP
intersectam - se novamente em Q. Se O, P e Q são três pontos distintos, prove que
OQ é perpendicular a P Q.

4. (Ibero) Num triângulo escaleno ABC traça-se a bissetriz interna BD, com D sobre
AC. Sejam E e F , respectivamente, os pés das perpendiculares traçadas desde A e
C até à reta BD, e seja M o ponto sobre o lado BC tal que DM é perpendicular a
BC. Prove que ∠EM D = ∠DM F.

8
POT 2012 - Geometria - Nı́vel 2 - Aula 8 - Prof. Cı́cero Thiago

5. Seja M o ponto de interseção das diagonais de um quadrilátero inscritı́vel ABCD,


em que ∠AM B é agudo. O triângulo isósceles BCK é construı́do exteriormente ao
quadrilátero, com base a base sendo BC, tal que ∠KBC + ∠AM B = 90◦ . Prove que
KM é perpendicular a AD.

6. (Romênia) Seja ABC um triângulo acutângulo, e seja T um ponto no interior tal que
∠AT B = ∠BT C = ∠CT A. Sejam M , N e P as projeções de T sobre BC, CA, e
AB, respectivamente. O cı́rculo circunscrito ao triângulo M N P intersecta os lados
′ ′ ′
BC, CA e AB, pela segunda vez, em M , N e P , respectivamente. Prove que o
′ ′ ′
triângulo M N P é equilátero.

7. (Cone Sul) Seja ABCD um quadrado (os vértices estão nomeados no sentido horário)
e P um ponto qualquer pertencente ao interior do segmento BC. Constrói - se o qua-
drado AP RS (os vértices novamente nomeados no sentido horário). Demonstrar que
a reta CR é tangente à circunferência circuscrita ao triângulo ABC.

8. (IMO) Duas circunferências Γ1 e Γ2 intersectam - se em M e N . Seja l a tangente


comum a Γ1 e Γ2 que está mais próxima de M do que de N . A reta l é tangente a
Γ1 em A e a Γ2 em B. A reta paralela a l que passa por M intersecta novamente a
circunferência Γ1 em C e novamente a circunferência Γ2 em D. As retas CA e DB
intersectam - se em E; as retas AN e CD intersectam - se em P ; as retas BN e CD
intersectam - se em Q. Mostre que EP = EQ.

9. Seja Q o ponto médio do lado AB de um quadrilátero inscritı́vel ABCD e S a in-


terseção de suas diagonais. Sejam P e R as projeções ortogonais de S sobre AD e
BC, respectivamente. Prove que P Q = QR.

10. (Itália) Um triângulo ABC acutângulo está inscrito em um cı́rculo de centro O. Seja
D a interseção da bissetriz de A com BC e suponha que a perpendicular a AO por
D, corta a reta AC em um ponto P , interior a AC. Mostre que AB = AP .

Bibliografia
1. A Decade of Berkeley Math Circle
Zvesdelina Stankova

2. Problems in plane and solid geometry


Viktor Prasolov

3. Episodes in nineteenth and twentieth century euclidean geometry


Ross Honsberger

9
Polos Olímpicos de Treinamento
Curso de Geometria - Nível 2 Aula 9
Prof. Cícero Thiago

Teorema de Ptolomeu

Teorema 1. (Ptolomeu) O produto dos comprimentos das diagonais de um quadrilátero


inscritı́vel é igual a soma dos produtos dos comprimentos dos pares de lados opostos.

Demonstração.

b
A

B b

b b
P
b

C D

Seja P o ponto sobre o prolongamento do lado CD tal que ∠BAC = ∠DAP . Como o
quadrilátero ABCD é inscritı́vel então ∠ABC = ∠ADP , assim ∆ABC ∼ ∆DAP . Com
isso,
AB BC AC AD · BC
= = ⇒ DP = .
AD DP AP AB
AB AC
Como ∠BAD = ∠CAP e = . Portanto, ∆ABD ∼ ∆ACP . Assim,
AD AP
BD AB AC · BD
= ⇒ CP = .
CP AC AB
Mas CP = CD + DP , dessa forma
AC · BD AD · BC
= CD + ⇔
AB AB
POT 2012 - Geometria - Nı́vel 2 - Aula 9 - Prof. Cı́cero Thiago

AC · BD = AB · CD + AD · BC.
Exercı́cios Resolvidos

¯ que não contém


1. Seja ABC um triângulo equilátero e seja P um ponto sobre o arco BC,
A, da circunferência circunscrita ao triângulo ABC. Prove que P A = P B + P C.

Solução.
A
b

b b

B C

Como o triângulo ABC é equilátero então AB = BC = CA = a. Aplicando o


teorema de Ptolomeu ao quadrilátero ABP C temos que

AB · P C + AC · P B = BC · P A ⇔

a · PC + a · PB = a · PA ⇔
P C + P B = P A.

2. (IME) Dado o quadrilátero ABCD, inscrito num cı́rculo de raio r, conforme a figura
abaixo, prove que:
AC AB · AD + BC · CD
= .
BD AB · BC + CD · AD

2
POT 2012 - Geometria - Nı́vel 2 - Aula 9 - Prof. Cı́cero Thiago

b
C

B b

b b

A D

Solução.

b
C

B b

b b

A D

b
N
M

Sejam M e N pontos sobre a circunferência circunscrita ao quadrilátero ABCD tais


¯ = CD
que AM ¯ e DN¯ = AB.¯ Dessa forma AM = CD, DN = AB, BM = CN e
M C = BN = AD. Aplicando o teorema de Ptolomeu nos quadriláteros M ABC e
N BCD temos
BM · AC = AB · M C + BC · AM (I)
CN · BD = DN · BC + CD · BN (II)

Fazendo (I) ÷ (II), temos

AC AB · AD + BC · CD
= .
BD AB · BC + CD · AD

3
POT 2012 - Geometria - Nı́vel 2 - Aula 9 - Prof. Cı́cero Thiago

Este resultado é conhecido como Teorema de Hiparco.

3. (Seletiva do Brasil para a Cone Sul) Prove que as distâncias entre um ponto sobre
uma circunferência e os quatro vértices de um quadrado nesta inscrita não podem ser
todas números racionais.

Solução.

A b b
D

b
P

b b

B C

Como ABCD é um quadrado então AB = BC = CD = DA = a. Pelo toerema de


Pitágoras no triângulo
√ ABC temos que AC 2 = AB 2 + BC 2 ⇔ AC 2 = a2 + a2 =
2
2 · a ⇔ AC = 2 · a. Aplicando o teorema de Ptolomeu no quadrilátero ABCP ,
temos

AC · BP = AP · BC + CP · AB ⇔ 2 · a · BP = AP · a + CP · a ⇔
√ AP + CP
2= .
BP
Se todas as medidas fossem números racionais estarı́amos afirmando, de maneira falsa,
√ BP √
que 2 ∈ Q. Se P coincidir com um dos vértices, ou seja, P ≡ D, então = 2.
CP
Assim, as medidas não podem ser todas racionais.

4. (Irã) Seja ABC um triângulo com BC > CA > AB. Seja D um ponto sobre o
lado BC e seja E o ponto no prolongamento de BA, com A entre E e B, tal que
BD = BE = CA. Seja P o ponto sobre AC tal que E, B, D e P são concı́clicos e
seja Q o segundo ponto de interseção de BP com o cı́rculo circunscrito ao triângulo
ABC. Prove que AQ + CQ = BP .

4
POT 2012 - Geometria - Nı́vel 2 - Aula 9 - Prof. Cı́cero Thiago

Solução.
b
E

A
b

Q
b P b

b
B
b b
C
D

Veja que ∆AQC ∼ ∆EP D, pois ∠CAQ = ∠CBQ = ∠DEP e ∠AQC = 180◦ −
∠ABD = ∠EP D. Por outro lado, pelo teorema de Ptolomeu, temos

BP · DE = BE · DP + BD · EP.

Então,

DP EP CQ AQ
BP = BE · + BD · = CA · + CA · = AQ + CQ.
DE DE CA CA
5. Seja A1 A2 A3 . . . An um polı́gono regular de n lados tal que
1 1 1
= + .
A1 A2 A1 A3 A1 A4

Determine n, ou seja, o número de lados do polı́gono regular.

Solução. Temos que


1 1 1
= + ⇔
A1 A2 A1 A3 A1 A4
A1 A2 · A1 A3 + A1 A2 · A1 A4 = A1 A3 · A1 A4 . (I)

5
POT 2012 - Geometria - Nı́vel 2 - Aula 9 - Prof. Cı́cero Thiago

A5
b A4
b

b
A3

b
An
b
A2

A1

Como o quadrilátero A1 A3 A4 A5 é inscritı́vel podemos aplicar o teorema de Ptolomeu,


assim
A4 A5 · A1 A3 + A3 A4 · A1 A5 = A3 A5 · A1 A4 . (II)
Além disso, como o polı́gono é regular, temos
A1 A2 = A4 A5 , A1 A2 = A3 A4 , A1 A3 = A3 A5 .

Comparando (I) e (II), encontramos

A1 A4 = A1 A5 .
Como as diagonais A1 A4 e A1 A5 são iguais, segue que existe o mesmo número de
vértices entre A1 e A4 e entre A1 e A5 . Dessa forma concluı́mos que n = 7.

Exercı́cios Propostos

1. (AHSME) Seja ABCD um quadrilátero e seja O o ponto de interseção das diagonais


AC e BD. Se BO = 4, OD = 6, AO = 8, OC = 3 e AB = 6, determine a medida de
AD.

2. Seja ABCD um quadrilátero convexo tal que


AC · BD = AB · CD + AD · BC.

Prove que ABCD é inscritı́vel.

6
POT 2012 - Geometria - Nı́vel 2 - Aula 9 - Prof. Cı́cero Thiago

3. Seja ABCD um quadrado. Determine o lugar geométrico dos pontos P , no mesmo


plano do quadrado ABCD, tais que
1
máx {P A, P C} = √ (P B + P D).
2

4. Uma circunferência passa pelo vértice A de um paralelogramo ABCD intersectando


os lados AB e AD nos pontos P e R, respectivamente. Além disso intersecta a dia-
gonal AC no ponto Q. Prove que AQ · AC = AP · AB + AR · AD.

5. Um ponto P é escolhido o interior do paralelogramo ABCD de tal forma que ∠AP B+


∠CP D = 180◦ . Prove que AB · AD = BP · DP + AP · CP .

6. Seja ABC um triângulo isósceles, com AB = AC, inscrito em uma circunferência Γ.


Seja P um ponto sobre o arco BC,¯ que não contém A, da circunferência Γ. Prove
PA AC
que = .
PB + PC BC

7. Seja ABCD um quadrado inscrito em uma circunferência Γ. Seja P um ponto sobre


¯ que não contém A e D, da circunferência Γ. Prove que
o arco BC,
PA + PC PD
= .
PB + PD PA

8. A bissetriz do ângulo ∠A do triângulo ABC intersecta o cı́rculo circunscrito no ponto


D. Prove que AB + AC ≤ 2AD.

9. (IMO) Seja ABCDEF um hexágono convexo tal que AB = BC = CD, DE = EF =


F A e ∠BCD = ∠EF A = 60◦ . Sejam G e H pontos no interior do hexágono tais que
∠AGB = ∠DHE = 120◦ . Prove que AG + GB + GH + DH + HE ≥ CF .

√ √
10. √
(Mandelbrot) Um quadrilátero inscritı́vel é tal que seus lados medem 1, 7, 3 e
21, nesta ordem. Determine a distância entre os pontos médios das diagonais.

Bibliografia
1. Mandelbrot Morsels
Sam Vandervelde

2. Advanced Euclidean Geometry


Alfred Posamentier

7
POT 2012 - Geometria - Nı́vel 2 - Aula 9 - Prof. Cı́cero Thiago

3. Problem primer for the olympiad


C R Pranesachar, B J Venkatachala e C S Yogananda

4. 360 Problems for Mathematical Contests


Titu Andreescu e Dorin Andrica

5. Olimpı́adas de Matemática 97
Antonio Caminha, Onofre Campos e Paulo Rodrigues

6. Geometrı́a - Una Visión de la planimetrı́a


Lumbreras

7. the Art of Problem Solving, vol. 2: and Beyond


Richard Rusczyk e Sandor Lehoczky

8. Problems in plane and solid geometry, vol. 1 - Plane Geometry


Viktor Prasolov

8
Polos Olímpicos de Treinamento
Curso de Geometria - Nível 2 Aula 10
Prof. Cícero Thiago

Relações métricas no triângulo.

1. Seno, cosseno e tangente

C
b

α
b b

A B

cateto oposto AC AC
seno de α = = ⇒ sin α = .
hipotenusa BC BC
cateto adjacente AB AB
cosseno de α = = ⇒ cos α = .
hipotenusa BC BC
AC
seno de α AC AC
tangente de α = = BC = ⇒ tan α = .
cosseno de α AB AB AB
BC
Como α + β = 90 é fácil provar que sin α = cos β e cos α = sin β.

Teorema 1. (Pitágoras) Em um triângulo retângulo a soma dos quadrados dos catetos


é igual ao quadrado da hipotenusa.

Demonstração.
POT 2012 - Geometria - Nı́vel 2 - Aula 10 - Prof. Cı́cero Thiago

A
b

c b

b b b

B m D a−m C

1. ∆ABD ∼ ∆ABC
c m
= ⇔ c2 = a · m
a c
2. ∆ACD ∼ ∆ABC

b a−m
= ⇔ b2 = a · (a − m) = a2 − a · m
a b
Então,
c2 + b2 = a · m + a2 − a · m ⇔
c2 + b2 = a2 .
2. Ângulos notáveis

2.1. 45◦

45◦

a 2
a

45◦

2
POT 2012 - Geometria - Nı́vel 2 - Aula 10 - Prof. Cı́cero Thiago


a 2
sin 45 = √ =

a 2 2

a 2
cos 45 = √ =

a 2 2

2
tan 45◦ = √2 = 1.
2
2
2.2 30◦ e 60◦

30◦

a √
a 3
2

60◦

a
2

a
1
sin 30◦ = cos 60◦ = 2 =
a 2

a 3 √
◦ ◦ 2 3
cos 30 = sin 60 = =
a 2
1 √
2 3
tan 30 = √ =

3 3
2

3
◦ 2 √
tan 60 = = 3.
1
2

3
POT 2012 - Geometria - Nı́vel 2 - Aula 10 - Prof. Cı́cero Thiago

Usaremos, sem demonstração, no restante deste material que sin x = sin(180◦ − x) e


cos x = − cos(180◦ − x).

Teorema 2. (Lei dos Senos) Seja ABC um triângulo tal que BC = a, CA = b e AB = c.


Seja R o raio da circunferência circunscrita. Então
a b c
= = = 2R.
sin ∠A sin ∠B sin ∠C
Demonstração.
A
b

c b

b
O


b b

B a C

Seja AD um diâmetro. É fácil ver que ∠ABC = ∠ADC. Assim, no triângulo ∆ADC,
b b
sin ∠B = ⇔ = 2R. Analogamente,
2R sin ∠B
a c
= = 2R.
sin ∠A sin ∠C
Finalmente,

a b c
= = = 2R.
sin ∠A sin ∠B sin ∠C
Teorema 3. (Lei dos Cossenos) Seja ABC um triângulo tal que BC = a, CA = b e
AB = c. Então,
a2 = b2 + c2 − 2bc cos ∠A,
b2 = a2 + c2 − 2ac cos ∠B,
c2 = a2 + b2 − 2ab cos ∠C.

4
POT 2012 - Geometria - Nı́vel 2 - Aula 10 - Prof. Cı́cero Thiago

Demonstração.

b
A

c b
H


b b b

B m D a−m C

Vamos fazer o caso em que o triângulo é acutângulo. O caso em que o triângulo é ob-
tusângulo fica como exercı́cio. Aplicando o teorema de Pitágoras nos triângulos ∆ABD e
∆ACD, temos:
c2 = m2 + H 2 e
b2 = (a − m)2 + H 2 ⇔
b2 = a2 − 2am + m2 + H 2 .
m
Assim, b2 = a2 + c2 − 2am. Por outro lado, cos ∠B = ⇔ m = c · cos ∠B. Finalmente,
c
2 2 2
b = a + c − 2ac cos ∠B. Analogamente,

a2 = b2 + c2 − 2bc cos ∠A e

c2 = a2 + b2 − 2ab cos ∠C.

Teorema 4. (Stewart) Seja ABC um triângulo tal que BC = a, CA = b e AB = c. Seja


D um ponto sobre o lado BC tal que BD = x, CD = y e AD = z. Então,

c2 y + b2 x − z 2 a = axy.

Demonstração.

5
POT 2012 - Geometria - Nı́vel 2 - Aula 10 - Prof. Cı́cero Thiago

A
b

c b
z

180◦ − α α
b b b

B x D y C

Aplicando a lei dos Cossenos nos triângulos ∆ABD e ∆ACD, temos

c2 = x2 + z 2 − 2xz cos(180◦ − α) ⇔

c2 z2
=x+ − 2z cos(180◦ − α). (1)
x x
E

b2 = y 2 + z 2 − 2yz cos α ⇔
b2 z2
=y+ − 2z cos α. (2)
y y
Adicionando (1) e (2), encontramos

b2 c2 z2 z2
+ =x+y+ + ⇔
y x y x

b2 c2 z2 z2
+ =a+ + ⇔
y x y x
c2 y + b2 x − z 2 a = axy.
Exercı́cios resolvidos

1. Num triângulo ABC são dados ∠A = 60◦ , ∠B = 45◦ e BC = 4 cm. Determine a


medida de AC.

Solução.

6
POT 2012 - Geometria - Nı́vel 2 - Aula 10 - Prof. Cı́cero Thiago

b
C

60◦ 45◦
b b

A B

Aplicando a lei dos senos temos

BC AC
= ⇔
sin ∠A sin ∠B
4 AC
= ⇔
sin 60 ◦ sin 45◦
4 AC
√ = √ ⇔
3 2
2 2

4 6
AC = .
3
2. (OCM) Um observador estando a 25 m de um prédio o visualiza sob um certo ângulo.
Afastando - se, na direção perpendicular ao prédio mais 50 m o ângulo de visualização
é a metade do anterior. Qual a altura do prédio?

b
C

α
α
b 2 b b

A 50 m D 25 m B

Solução.

7
POT 2012 - Geometria - Nı́vel 2 - Aula 10 - Prof. Cı́cero Thiago

É fácil ver que ∆ADC é isósceles, ou seja, AD = CD = 50 m. Aplicando o teorema


de Pitágoras no triângulo ∆BDC temos

CD 2 = BD 2 + BC 2 ⇔ 502 = 252 + BC 2 ⇔ BC = 25 3.

3. (China Western) Em um trapézio ABCD, AD//BC . Sejam E um ponto variando


sobre o lado AB, O1 e O2 os circuncentros dos triângulos AED e BEC, respectiva-
mente. Prove que o comprimento de O1 O2 é fixo.

Solução.

Ab b
D

E b

b
O1

b O2
b b

B C

É fácil ver que ∠AEO1 = 90◦ − ∠ADE e ∠BEO2 = 90◦ − ∠BCE. Então,

∠O1 EO2 = ∠ADE + ∠ECB.

Como AD k BC, construa uma paralela a AD, por E. Dessa forma ∠DEC =
∠ADE + ∠BCE, ou seja, ∠O1 EO2 = ∠DEC. Usando lei dos senos, temos
DE 2O1 E sin ∠A O1 E
= = .
EC 2O2 E sin ∠B O2 E

Assim, ∆DEC ∼ ∆O1 EO2 . Portanto,


O1 O2 O1 E O1 E 1
= = = .
DC DE 2O1 E sin ∠A 2 sin ∠A

DC
Portanto, O1 O2 = , que é um valor fixo.
2 sin ∠A

4. Seja ABCD um quadrilátero inscrito em uma circunferência de diâmetro AD. Se


AB = BC = 1 e AD = 3, ache o comprimento da corda CD.

Solução.

8
POT 2012 - Geometria - Nı́vel 2 - Aula 10 - Prof. Cı́cero Thiago

C
b

B b

α
A b
α b
D

Temos que AD = 3, AB = BC = 1. Aplicando o teorema de Pitágoras no triângulo


ABD, temos

AD 2 = AB 2 + BD 2 ⇔ 32 = 11 + BD 2 ⇔ BD = 2 2.

BD 2 2
Além disso, cos α = = . Aplicando a lei dos cossenos no triângulo BCD,
AD 3
temos
BC 2 = BD 2 + CD 2 − 2 · BD · CD cos α ⇔

2 2
√ 2 2
1 = 8 + CD − 2 · 2 2 · CD ⇔
3
7
CD = 3 ou .
3
7
Como o diâmetro mede 3, então CD = .
3

5. (Teste de seleção do Brasil para a Cone Sul) Em um triângulo acutângulo ABC,


∠A = 30◦ , H é seu ortocentro e M é o ponto médio de BC. Sobre a reta HM
tomemos um ponto T 6= H tal que HM = M T . Mostre que AT = 2BC.

Solução.

9
POT 2012 - Geometria - Nı́vel 2 - Aula 10 - Prof. Cı́cero Thiago

b
C

H b

b
M

b
T

b b

A B

HBT C é um paralelogramo pois M é o ponto médio de BC e HM = M T . Além


disso, BC ⊥ AC e BH k AC, assim CT ⊥ AC, ou seja, ∠T CA = ∠90◦ . Com
isso, T pertence à circunferência circunferência circunscrita a ABC e AT é diâmetro.
Portanto,
BC BC
AT = 2R = = = 2BC.
sin ∠A sin 30◦
Exercı́cios propostos

1. (OCM) Se as diagonais de um quadrilátero (convexo) são perpendiculares, prove que


as somas dos quadrados dos lados opostos são iguais.

2. (OCM) Dobra - se um pedaço de arame de 32 cm de comprimento formando um


triângulo isósceles de 12 cm de base. Calcule a medida do comprimento da bissetriz
do ângulo oposto à base.

3. (OBM) P é um ponto interior a um quadrado ABCD. As distâncias de P aos vértices


A e D e ao lado BC são iguais a 10 cm. O lado do quadrado mede:
(a) 10 cm (b) 12 cm (c) 14 (d)16 cm (e) 18 cm

4. Um ponto P , interno de um ângulo de 60◦ , dista 6 m e 9 m dos lados desse ângulo.


Qual a distância entre P e a bissetriz do ângulo?

5. Seja ABC um triângulo tal que ∠ABC = 45o . Seja D o ponto sobre o segmento BC
tal que 2BD = CD e ∠DAB = 15o . Determine o ângulo ∠ACB.

6. (AIME) Seja ABC um triângulo tal que AB = 13, BC = 15 e CA = 14. Seja D o


ponto do segmento BC tal que CD = 6. Seja E o ponto de BC tal que CE > CD e
∠BAE = ∠CAD. Determine BE.

10
POT 2012 - Geometria - Nı́vel 2 - Aula 10 - Prof. Cı́cero Thiago

7. No triângulo ABC, ∠BAC = 20◦ e AB = AC. Os pontos M e N estão sobre os lados


AB e AC, respectivamente, e são tais que ∠BCM = 60◦ e ∠CBN = 50◦ . Calcule a
medida do ângulo ∠CM N .

8. Em um triângulo ABC, ∠BAC = 100◦ e AB = AC. Seja BD a bissetriz de ∠ABC,


com D sobre o lado AC. Prove que AD + BD = BC.

9. (Teste de seleção do Brasil para a IMO) Seja Γ uma circunferência de centro O tan-
gente aos lados AB e AC do triângulo ABC nos pontos E e F . A reta perpendicular
ao lado BC por O intersecta EF no ponto D. Mostre que A, D e M (ponto médio
de BC) são colineares.

Bibliografia

1. 103 Trigonometry Problems - From the training of the USA IMO team
Titu Andreescu

2. Precalculus
Richard Rusczyk

3. Olimpı́adas de Matemática 97
Antonio Caminha, Onofre Campos e Paulo Rodrigues

4. Olimpı́adas Cearenses de Matemática, Ensino Fundamental, 1981 - 1985


Emanuel Carneiro, Francisco Antônio M. de Paiva e Onofre Campos.

5. Olimpı́adas Cearenses de Matemática, Ensino Médio, 1981 - 1985


Emanuel Carneiro, Francisco Antônio M. de Paiva e Onofre Campos.

11
Polos Olímpicos de Treinamento
Curso de Geometria - Nível 2 Aula 11
Prof. Cícero Thiago

Potência de ponto e eixo radical

1. Definição
Seja Γ uma circunferência de centro O e raio R. Seja P um ponto que está a uma distância
d de O, vamos definir a potência do ponto P em relação à circunferência Γ por
PotPΓ = d2 − R2 .
É fácil ver que se P é um ponto no exterior de Γ então a potência será positiva, se P é um
ponto sobre a circunferência então sua potência será zero e se P é um ponto no interior da
circunferência então sua potência será negativa.

Teorema 1. Seja P um ponto e Γ uma circunferência. Se uma reta que passa por P
intersecta a circunferência nos pontos A e B, então o produto P A · P B é constante.

Demonstração.
1◦ caso: P é um ponto no exterior.

b
B
m
M
m b

Ab R

b b

P O
POT 2012 - Geometria - Nı́vel 2 - Aula 11 - Prof. Cı́cero Thiago

Seja OM a mediatriz de AB. Então

P A · P B = (P M − m) · (P M + m) = P M 2 − m2 = P M 2 + OM 2 − (OM 2 + m2 )

= P O 2 − R2 = PotPΓ .
Vamos analisar também o caso em que pelo ponto P é traçada uma tangente a Γ.

T
b

O
b

Dessa forma pelo teorema de Pitágoras temos que

P O 2 = P T 2 + R2 ⇔ P T 2 = P O 2 − R2 = PotPΓ .

2◦ caso: P é um ponto no interior.

2
POT 2012 - Geometria - Nı́vel 2 - Aula 11 - Prof. Cı́cero Thiago

B
m b

M
m b

R
b

A b
P

Seja OM a mediatriz de AB. Então

P A · P B = (m − P M ) · (m + P M ) = m2 − P M 2 = m2 + OM 2 − (OM 2 + P M 2 )

= R2 − P O 2 = −PotPΓ .
2. Eixo radical

Chamaremos de Eixo radical o lugar geométrico dos pontos que possuem a mesma
potência com relação a duas circunferências dadas.

Teorema 2. O conjunto dos pontos que possuem a mesma potência com relação a duas
circunferências dadas é uma reta perpendicular à reta que contém os centros.

Demonstração.

3
POT 2012 - Geometria - Nı́vel 2 - Aula 11 - Prof. Cı́cero Thiago

b
P

b b b b

O1 M Q O2

Sejam Γ1 e Γ2 circunferências com centros O1 e O2 e raios R1 e R2 , respectivamente. Além


disso, seja P um ponto que possui a mesma potência com relação as duas circunferências.
Assim,
PotPΓ1 = PotPΓ2 ⇔
P O12 − R12 = P O22 − R22 ⇔
P O12 − P O22 = R12 − R22 .
Seja M o ponto médio de O1 O2 , Q a projeção de P sobre O1 O2 e ∠P M Q = α. Aplicando
a lei dos cossenos nos triângulos ∆P O1 M e ∆P O2 M temos

P O12 = O1 M 2 + P M 2 − 2 · O1 M · P M · cos(180◦ − α) =

P O12 = O1 M 2 + P M 2 + 2 · O1 M · P M · cos α
P O22 = O2 M 2 + P M 2 − 2 · O2 M · P M · cos α.
Então,

P O12 − P O22 = 2 · O1 O2 · P M · cos α.


MQ
Por outro lado, cos α = ⇔ M Q = P M · cos α, com isso
PM
R2 − R22
MQ = 1 = Fixo.
2 · O1 O2
Portanto, o lugar geométrico dos pontos P é a reta perpendicular a O1 O2 que passa por Q.

4
POT 2012 - Geometria - Nı́vel 2 - Aula 11 - Prof. Cı́cero Thiago

Por outro lado, seja P1 um ponto de P Q. Vamos provar que P1 possui a mesma potência
com relação às duas circunferências. Assim, pelo toerema de Pitágoras

P1 O12 = O1 Q2 + P1 Q2 ,

P1 O22 = O2 Q2 + P1 Q2 .
Então,
P1 O12 − P1 O22 = O1 Q2 − O2 Q2 .

b
P

P1
b

b b b

O1 Q O2

Além disso,

P O12 = O1 Q2 + P Q2 ,
P O22 = O2 Q2 + P Q2 .
Então,

P O12 − P O22 = R12 − R22 = O1 Q2 − O2 Q2 = P1 O12 − P1 O22 ⇔


P1 O12 − R12 = P1 O22 − R22 ⇔
PotPΓ11 = PotPΓ21 .

Problema 1. Dois cı́rculos Γ1 e Γ2 intersectam - se em P e Q. Uma reta passando por P


intersecta Γ1 e Γ2 novamente em A e B, respectivamente, se X é o ponto médio de AB e a
reta que passa por Q e X intersecta Γ1 e Γ2 novamente em Y e Z, respectivamente. Prove
que X é o ponto médio de Y Z.

5
POT 2012 - Geometria - Nı́vel 2 - Aula 11 - Prof. Cı́cero Thiago

Solução.

PotX
Γ2 = XP · XB = XZ · XQ,

−PotX
Γ1 = XP · XA = XY · XQ.

Então,

XP · XB XZ · XQ
= ⇔
XP · XA XY · XQ
XY = XZ.

Γ1
Y
b
Γ2

X P b
B
b
A b

Z
O1 b O2
b b

Problema 2. (OCM) Duas tangentes OA e OB são traçadas a um cı́rculo de um ponto


externo O. Uma corda AC é construı́da paralela a OB e uma secante OC é desenhada
intersectando o cı́rculo em E. Se K é o ponto de interseção de OB com o prolongamento
de AE, prove que OK = KB.

6
POT 2012 - Geometria - Nı́vel 2 - Aula 11 - Prof. Cı́cero Thiago

Solução.
Temos que ∠KOC = ∠ECA pois OB k AC e ∠ECA = ∠EAO pois OA é tangente ao
cı́rculo. Então ∆OKE ∼ ∆AKO assim
OK KE
= ⇔ OK 2 = KE · KA.
KA OK
Usando a potência de K com relação à circunferência temos
KB 2 = KE · KA.
Portanto, OK = KB.

b
A

C
α b

E
b

O
b
α

b
B

Problema 3. Seja C uma semicircunferência de centro O e diâmetro AB e D é o ponto


médio do arco AB. Sobre a reta OD toma - se o ponto E, do mesmo lado de D com relação
a AB, tal que OE = BD. Se BE corta a semicircunferência em F e P é o ponto de AB
AB
tal que F P é perpendicular a AB. Prove que BP = .
3
Solução. √
Sem perda
√ de generalidade faça OA = OB = 1. Logo, OD = 1, OE = BD = 2 e
EB = 3. Utilizando a potência de E com relação à circunferência de diâmetro AB temos
EF · EB = EO 2 − R2 = EO2 − 1.

7
POT 2012 - Geometria - Nı́vel 2 - Aula 11 - Prof. Cı́cero Thiago

Assim, √ √
√ √ 3 2 3
EF · 3 = ( 2)2 − 1 ⇔ EF = e FB = .
3 3
Além disso, ∆BP F ∼ ∆BOE então

BP BF 2
= ⇔ BP = .
BO BE 3
Portanto,
2
BP 1 AB
= 3 = ⇔ BP = .
AB 2 3 3

b
E

D
b

b
F

b b b b

A O P B

Problema 4. Considere três cı́rculos Γ1 , Γ2 e Γ3 tais que seus centros O1 , O2 e O3 , respecti-


vamente, não estão alinhados. Sejam r, s e t os eixos radicais de Γ1 e Γ2 , Γ1 e Γ3 e Γ2 e Γ3 ,
respectivamente. Prove que r, s e t são concorrentes em um ponto chamado centro radical.

Solução.

8
POT 2012 - Geometria - Nı́vel 2 - Aula 11 - Prof. Cı́cero Thiago

Seja P um ponto sobre r ∩ s, ou seja, P possui a mesma potência com relação Γ1 , Γ2 e Γ3 .


Portanto, P está sobre a reta t.

O1
b

O2
b

P
b

O3

Exercı́cios propostos

1. Em um triângulo ABC, a bissetriz do ângulo A e a mediana relativa a BC intersec-


tam este lado em pontos distintos O e M , respectivamente. O cı́rculo circunscrito ao
triângulo AOM intersecta os lados AB e AC em E e F , respectivamente. Prove que
BE = CF .

2. Seja BD a bissetriz do ângulo B do triângulo ABC. Se o cı́rculo circunscrito ao


triângulo BDC intersecta AB em E e o cı́rculo circunscrito ao triângulo ABD inter-
secta BC em F , prove que AE = CF .

3. Um triângulo acutângulo ABC está inscrito numa circunferência de centro O. As


alturas do triângulo são AD, BE e CF . A reta EF intersecta a circunferência em P
e Q.

9
POT 2012 - Geometria - Nı́vel 2 - Aula 11 - Prof. Cı́cero Thiago

(a) Prove que OA é perpendicular a P Q.


(b) Se M é o ponto médio de BC, prove que AP 2 = 2AD.OM .

4. Seja C um ponto sobre o semicı́rculo de diâmetro AB e seja D o ponto médio do arco


AC. Se E é a projeção de D sobre BC e F é a interseção de AE com o semicı́rculo,
prove que BF bissecta o segmento DE.

5. Seja P um ponto no interior de um cı́rculo tal que existem três cordas que passam
por P e tem o mesmo comprimento. Prove que P é o centro do cı́rculo.

6. Sejam Γ1 e Γ2 cı́rculos concêntricos, com Γ2 no interior de Γ1 . Partindo de um ponto


A pertencente a Γ1 , é desenhada uma tangente AB à Γ2 (B ∈ Γ2 ). Seja C o segundo
ponto de interseção de AB com Γ1 , e D o ponto médio de AB. Um reta passando
por A intersecta Γ2 em E e F de tal maneira que as mediatrizes de DE e CF se
AM
intersectam em um ponto M sobre AC. Determine a razão .
MC

7. (IMO) Seja ABC um triângulo com circuncentro O. Sejam P e Q pontos no interior


dos lados CA e AB, respectivamente. Sejam K, L e M os pontos médios dos seg-
mentos BP , CQ e P Q, respectivamente, e seja Γ o cı́rculo que passa por K, L e M .
Se P Q é tangente a Γ, prove que OP = OQ.

8. (IMO) Um cı́rculo de centro O passa pelos vértices A e C de um triângulo ABC e


intersecta os segmentos AB e BC novamente em pontos distintos K e N , respecti-
vamente. Os cı́rculos circunscritos aos triângulos ABC e KBN se intersectam em
exatamente 2 pontos distintos B e M . Prove que ∠OM B = 90◦ .

Bibliografia

1. Problemas de las olimpiadas matematicas del Cono Sur (I a a IV a )


Fauring - Wagner - Wykowski - Gutierrez - Pedraza - Moreira

2. Olimpı́adas Cearenses de Matemática - Ensino Fundamental - 1981 - 2005


Emanuel Carneiro, Francisco Antônio M. de Paiva e Onofre Campos

3. Potência de um ponto em relação a uma circunferência


Eduardo Wagner
Revista do professor de matemática - Número 45

10
POT 2012 - Geometria - Nı́vel 2 - Aula 11 - Prof. Cı́cero Thiago

4. Mathematical Olympiad Challenges


Titu Andreescu e Razvan Gelca

5. Lecture Notes on Mathematical Olympiad Courses - For senior section - Vol. 1


Xu Jiagu

6. Tópicos de Matemática Elementar - Volume 2


Antonio Caminha Muniz Neto

11
Polos Olímpicos de Treinamento
Curso de Geometria - Nível 2 Aula 12
Prof. Cícero Thiago

Relações entre áreas I

Teorema 1. (Fórmula tradicional.)

A
b

b b b

B D C

BC · AD
A área do triângulo ∆ABC pode ser calculada por [∆ABC] = .
2

Teorema 2. (Área de um triângulo em função do raio da circunferência inscrita.)

Sejam a, b e c as medidas dos lados BC, CA e AB do triângulo ∆ABC, respectivamente, e


seja r a medida do raio da circunferência inscrita. Então, a área do triângulo ∆ABC pode
ser calculada por
[∆ABC] = p · r,
a+b+c
em que p = .
2
Demonstração.
POT 2012 - Geometria - Nı́vel 2 - Aula 12 - Prof. Cı́cero Thiago

A
b

b E
F b

r
r
b

b b b

B D C

[∆ABC] = [∆BIC] + [∆CIA] + [∆AIB] ⇔


a·r b·r c·r
[∆ABC] = + + ⇔
2 2 2
a+b+c
Å ã
[∆ABC] = ·r ⇔
2
[∆ABC] = p · r.

Teorema 3. (Fórmula trigonométrica da área de um triângulo.)

Sejam a, b e c as medidas dos lados BC, CA e AB do triângulo ∆ABC, respectivamente.


A área do triângulo ∆ABC pode ser calculada por
b · c · sin∠A a · c · sin∠B a · b · sin∠C
[∆ABC] = = = .
2 2 2
Demonstração. Vamos demonstrar uma das igualdades. As outras são análogas.

2
POT 2012 - Geometria - Nı́vel 2 - Aula 12 - Prof. Cı́cero Thiago

b
B

α
b b b

A D C

Seja ∠A = α. Temos que


AC · BD a·H
[∆ABC] = = .
2 2
H
Por outro lado, no triângulo ∆ABD, temos sin α = ⇔ H = c · sin α, então
c
a · c · sin α
[∆ABC] = .
2

Teorema 4. (Área de um triângulo em função do raio da circunferência circuns-


crita.)

Sejam a, b e c as medidas dos lados BC, CA e AB do triângulo ∆ABC, respectivamente,


e seja R o raio da circunferência circunscrita. Então, a área do triângulo [∆ABC] pode ser
calculada por
abc
[∆ABC] = .
4R
Demonstração.

3
POT 2012 - Geometria - Nı́vel 2 - Aula 12 - Prof. Cı́cero Thiago

A
b

O
b

β
b b

B β C

Sejam a, b e c as medidas dos lados BC, CA e AB do triângulo ∆ABC, respectivamente.


Temos que
a · c · sin β
[∆ABC] = .
2
Por outro lado, seja AD um diâmetro então, no ∆ACD, temos que
b
sin β = .
2R
Portanto,
abc
[∆ABC] = .
4R
Teorema 5. (Fórmula de Heron.)

Sejam a, b e c as medidas dos lados BC, CA e AB do triângulo ∆ABC, respectivamente.


Então, a área do triângulo ∆ABC pode ser calculada por
»
[∆ABC] = p · (p − a) · (p − b) · (p − c),

a+b+c
em que p = .
2
Demonstração.

4
POT 2012 - Geometria - Nı́vel 2 - Aula 12 - Prof. Cı́cero Thiago

A
b

c h b

b b b

B m D a−m C

Aplicando o teorema de Pitágoras nos triângulos ∆ABD e ∆ACD, temos:


1. c2 = m2 + h2 .
2. b2 = (a − m)2 + h2 .
De (2), temos:
b2 = (a − m)2 + h2 ⇔
b2 = a2 − 2am + m2 + h2 ⇔
b2 = a2 − 2am + c2 ⇔
a2 + c2 − b2
m= .
2a
Substituindo em (1), temos:
å2
a2 + c2 − b2
Ç
2
c = + h2 ⇔
2a
å2
a2 + c2 − b2
Ç
2 2
h =c − ⇔
2a
a2 + c2 − b2 a2 + c2 − b2
Ç å Ç å
2
h = c+ · c− ⇔
2a 2a
2ac + a2 + c2 − b2 2ac − a2 − c2 + b2
Ç å Ç å
2
h = · ⇔
2a 2a
4a2 h2 = [(a + c)2 − b2 ] · [(b2 − (a − c)2 ] ⇔
4a2 h2 = (a + c + b) · (a + c − b) · (b + a − c) · (b + c − a) ⇔
4a2 h2 = (a + b + c) · (b + c − a) · (a + c − b) · (a + b − c) ⇔
4a2 h2 = 2p · (2p − 2a) · (2p − 2b) · (2p − 2c) ⇔
a2 h2
= p · (p − a) · (p − b) · (p − c) ⇔
2

5
POT 2012 - Geometria - Nı́vel 2 - Aula 12 - Prof. Cı́cero Thiago

[∆ABC]2 = p · (p − a) · (p − b) · (p − c) ⇔
»
[∆ABC] = p · (p − a) · (p − b) · (p − c).

Teorema 6. (Relação entre as áreas de triângulos semelhantes.)

AB AC BC
Sejam ∆ABC e ∆DEF dois triângulos semelhantes tais que = = = k, então
DE DF EF
[∆ABC]
= k2 .
[∆DEF ]
Demonstração.
AB AC BC AG
Se ∆ABC ∼ ∆DEF com = = = = k, então
DE DF EF DH
BC · AG
[∆ABC] 2 BC AG
= = · = k · k = k2 .
[∆DEF ] EF · DH EF DH
2
A
b

D
b

b b b b b b

B G C E H F

Teorema 7. Sejam r e s retas paralelas. Sejam A e B pontos distintos sobre a reta s e C1


e C2 pontos distintos sobre a reta r. Então, [∆ABC1 ] = [∆ABC2 ].

AB · H
Demonstração. O resultado é imediato pois [∆ABC1 ] = [∆ABC2 ] = .
2

6
POT 2012 - Geometria - Nı́vel 2 - Aula 12 - Prof. Cı́cero Thiago

r C1 C2
b b

s
b b

A B

Teorema 8. (Usando áreas para calcular razão de segmentos.)


Seja ABC um triângulo e D, E e F pontos sobre os lados BC, CA e AB tais que AD,
BE e CF são concorrentes no ponto P . Defina K = [ABC], KA = [P BC], KB = [P CA]
e KC = [P AB]. Como K = KA + KB + KC , então

(a)

BD KC CE KA AF KB
= , = e = .
DC KB EA KC F B KA
(b)

AP KB + KC BP KA + KC CP KA + KB
= , = e =
PD KA PE KB PF KC
Demonstração.
A
b

F b

b
E
H2 b
P

H1
b b b b b

B S R D C

(a) Temos que


BD [∆ABD] [∆BP D] [∆ABD] − [∆BP D] [∆AP B] KC
= = = = = .
CD [∆ACD] [∆CP D] [∆ACD] − [∆CP D] [∆ACP ] KB

7
POT 2012 - Geometria - Nı́vel 2 - Aula 12 - Prof. Cı́cero Thiago

CE KA AF KB
Da mesma maneira demonstra - se que = e = .
EA KC F B KA
(b) Temos que
∆ADS ∼ ∆P DR ⇒
AD H2 [∆ABC] KA + KB + KC
= = = ⇔
PD H1 [∆BP C] KA
AP KB + KC
= .
PD KA
BP KA + KC CP KA + KB
Da mesma maneira demonstra - se que = e = .
PE KB PF KC

Teorema 9. (Área de quadrilátero convexo qualquer.)

Seja ABCD um quadrilátero convexo qualquer tal que θ é o menor ângulo entre as diago-
AC · BD · sin θ
nais. Então, [∆ABCD] = .
2
Demonstração.
Temos que
[ABCD] = [∆AP D] + [∆BP C] + [∆CP D] + [∆DP A] ⇒
P A · P D · sin θ P A · P B · sin θ P B · P C · sin θ P C · P D · sin θ
[ABCD] = + + + ⇒
2 2 2 2
(P A · P D + P A · P B + P B · P C + P C · P D) sin θ
[ABCD] = ⇒
2
(P A + P C)(P B + P D) sin θ AC · BD · sin θ
[ABCD] = ⇒ [ABCD] = .
2 2
Exercı́cios Resolvidos

1. (Olimpı́ada de Maio) ABC é um triângulo equilátero. N é o ponto do lado AC tal


que AC = 7AN , M é o ponto do lado AB tal que M N é paralelo a BC e P o ponto
área(M N P )
do lado BC tal que M P é paralelo a AC. Determine o valor de .
área(ABC)

Solução. É fácil ver que CP M N é um paralelogramo e, com isso, área(M N P ) =


Å ã2
1 área(AM N ) 1 área(BM P ) 6 36
· área(CP M N ). Além disso, = e = = .
2 área(ABC) 49 área(ABC) 7 49
Portanto,
1 36
área(P M N ) 1− −
= 49 49 = 6 .
área(ABC) 2 49

8
POT 2012 - Geometria - Nı́vel 2 - Aula 12 - Prof. Cı́cero Thiago

2. São dados 1000 pontos no plano não colineares tais que se três deles determinam um
triângulo então sua área é menor ou igual a 1. Prove que todos os pontos estão em
um triângulo de área menor ou igual a quatro.

Solução.
b
C

Z b b
Y

b
D

b b b

A X B

Como existe um número finito de triângulos que podem ser construı́dos usando os
1000 pontos então, escolhemos aquele de área máxima que chamaremos de ∆XY Z.
Seja ∆ABC o triângulo tal que X, Y e Z são os pontos médios de BC, CA e AB,
respectivamente, então [∆ABC] = 4[∆XY Z] ≤ 4. Seja D, um ponto no conjunto
dos 1000 pontos dados, no exterior do triângulo ∆ABC então [∆XY Z] < [∆XZD],
o que contradiz a escolha de ∆ABC. Portanto, todos os pontos estão no interior do
triângulo ∆ABC.

3. (Coréia) Seja ABCD um quadrilátero convexo e seja P o ponto de interseção das


diagonais. Prove que
[∆P AB] + [∆P CD] = [∆P BC] + [∆P DA]
se, e somente se, P é o ponto médio de AC ou BD.

1
Solução. Observe que [∆P AB] · [∆P CD] = [∆P BC] · [∆P DA] = · PA · PB ·
4
P C · P D · sin P . Os números [∆P AB], [∆P CD] e [∆P BC], [∆P DA] tem a mesma
soma e o mesmo produto, então [∆P AB] = [∆P BC] e [∆P CD] = [∆P DA] ou
[∆P AB] = [∆P DA] e [∆P BC] = [∆P CD], ou seja, P é o ponto médio de AC ou
BD.

4. (OCM) Os lados de um triângulo são expressos, em cm, por três inteiros consecutivos
e sua área, em cm2 , é dada por um inteiro. Prove que o menor lado do triângulo é
ı́mpar.

9
POT 2012 - Geometria - Nı́vel 2 - Aula 12 - Prof. Cı́cero Thiago

Solução.
Sejam x−1, x, x+1 os lados do triângulo. Pela fórmula de Heron, a área do triângulo
é €
3x (x + 2) x (x − 2)
[∆ABC] = · · ·
2 2 2 2
€
3x2 (x2 − 4) 1» 2 2
= = 3x (x − 4).
16 4
Como [∆ABC] ∈ Z, devemos ter 3x2 (x2 − 4) par, o que nos diz que x deve ser par.
Portanto, o menor lado do triângulo, que é x − 1, deve ser ı́mpar.

5. (Hong Kong) Seja ABC um triângulo e sejam X, Y e Z pontos sobre os lados AB,
AX 4 BY 6 CZ 8
BC e CA, respectivamente, tais que = , = e = . Se a área do
XB 5 YC 7 ZA 9
triângulo ∆ABC é 1989, determine a área do triângulo ∆XY Z.

Solução.
Ç å
[∆XY Z] [∆AXZ] [∆BXY ] [∆CY Z]
=1− + +
1989 1989 1989 1989
4 9 5 6 7 8
Å ã
=1− · + · + ·
9 17 9 13 13 17
1482
1− ,
1989
Portanto, a área do triângulo ∆XY Z é 1989 − 1482 = 507.

Exercı́cios Propostos

1. No triângulo ABC, os pontos L, M e N estão sobre BC, CA e AB respectivamente,


e AL, BM e CN são concorrentes no ponto P .
(a) Encontre o valor numérico de

PL PM PN
+ +
AL BM CN
(b) Encontre o valor numérico de

AP BP CP
+ +
AL BM CN

10
POT 2012 - Geometria - Nı́vel 2 - Aula 12 - Prof. Cı́cero Thiago

2. (Ibero) Se AD, BE e CF são três cevianas concorrentes no circuncentro O do


triângulo ABC, demonstre que
1 1 1 2
+ + = .
AD BE CF R
3. (AIME) Num triângulo ABC, A1 , B1 e C1 estão sobre os lados BC, CA e AB,
respectivamente. Dado que AA1 , BB1 e CC1 são concorrentes no ponto O, e que
AO BO CO AO BO CO
+ + = 92. Encontre o valor de · · .
OA1 OB1 OC1 OA1 OB1 OC1

4. Em um ∆ABC, AD, BE e CF são concorrentes no ponto P tal que AP = P D = 6,


EP = 3, P B = 9 e CF = 20. Qual é a área do ∆ABC?

5. Em um triângulo ABC, sejam S o ponto médio da mediana correspondente ao vértice


A e Q o ponto de interseção de BS com o lado AC. Demonstrar que BS = 3QS.

6. Três segmentos C1 A2 , C2 B1 e A1 B2 com extremos sobre os lados do triângulo ABC


são paralelos aos lados e passam pelo ponto P . Prove que as áreas dos triângulos
A1 B1 C1 e A2 B2 C2 são iguais.

7. (OBM) É dado um quadrilátero convexo ABCD. Sejam E, F, G e H os pontos


médios dos lados AB, BC, CD e DA, respectivamente. Determine a posição de um
ponto P de forma que os quadriláteros P HAE, P EBF , P F CG e P GDH tenham a
mesma área.

8. Seja ABCDE um pentágono convexo (não necessariamente regular) tal que os triângulos
ABC, BCD, CDE, DEA e EAB tem área 1. Qual a área do pentágono?

9. Seja ABCD um quadrilátero convexo e EH, EI, EF e EG são segmentos paralelos


e iguais a AB, BC, CD e DA, como mostra a figura abaixo. Determine a razão entre
as áreas dos triângulos HIF G e ABCD.
G

C
D
F
H E

A B
I

11
POT 2012 - Geometria - Nı́vel 2 - Aula 12 - Prof. Cı́cero Thiago

10. (AIME) Quadrados S1 e S2 são inscritos em um triângulo retângulo ABC, como


mostrado na figura abaixo. Determine AC + CB se área(S1 ) = 441 e área(S2 ) = 440.

A A
b b

S2
S1

b b b b

C B C B

11. Seja P um ponto no interior de um triângulo equilátero ABC, e sejam D, E e F os


simétricos de P em relação aos lados BC, CA e AB, respectivamente. Qual é maior,
a área do triângulo ABC ou a área do triângulo DEF ?

A B

12. (Portugal) Seja [∆ABC] um triângulo retângulo em A. Considere um ponto E sobre


a hipotenusa e traça - se a partir desse ponto uma paralela ao cateto AC. Seja a
interseção desta paralela com o cateto AB. Prove que
BD DE BC 2
+ = ,
DE BD 2S

sendo S a área do triângulo ∆ABC.

13. (Portugal) Os lados AB, BC e AC do triângulo representado na figura medem, res-


pectivamente, 7, 11 e 8. Traçam - se W R, U P e V Q, perpendiculares aos lados.

12
POT 2012 - Geometria - Nı́vel 2 - Aula 12 - Prof. Cı́cero Thiago

Sabendo que U W mede 2, determine a razão entre a área do triângulo ∆U V W e a


área do triângulo ∆ABC.

A
b

R b Q
b
U b

V b

b b b

B P C

14. (OBM) ABCD é um quadrilátero convexo e inscritı́vel e M é um ponto sobre o lado


CD, tal que o triângulo ADM e o quadrilátero ABCM têm a mesma área e o mesmo
perı́metro. Prove que ABCD tem dois lados de comprimentos iguais.

15. Os pontos médios das diagonais AC, CE, EA, BD, DF e F B do hexágono convexo
ABCDEF são vértices de um novo hexágono. Calcular a relação entre as áreas do
dois hexágonos.

16. (Mandelbrot) Seja ABCD um quadrilátero convexo tal que AB = 12, BC = 6 e


CD = 20. Suponha que ABCD possui uma circunferência inscrita que é tangente ao
lado BC em seu ponto médio. Qual é a área do quadrilátero ABCD?

Bibliografia

1. Coleção Elementos da Matemática, vol.2 - Geometria Plana


Marcelo Rufino de Oliveira e Márcio Rodrigo da Rocha Pinheiro.

2. Olimpı́adas Cearenses de Matemática, Ensino Médio, 1981 - 1985


Emanuel Carneiro, Francisco Antônio M. de Paiva e Onofre Campos.

3. Olimpı́adas de Matemática, Categoria B, 10◦ , 11◦ e 12◦ anos, vol.1


Jorge Picado e Paulo Eduardo Oliveira.

13
POT 2012 - Geometria - Nı́vel 2 - Aula 12 - Prof. Cı́cero Thiago

4. Tópicos de Matemática Elementar, vol.2, Geometria Euclidiana Plana


Antonio Caminha Muniz Neto.

5. Area y Volumen, en la geometria elemental.


José Araujo, Guilermo Keilhauer, Norma Pietrocola e Valeri Vavilov.

6. Which Way did the Bicycle Go? And other intriguing mathematical mysteries
Joseph D. E. Konhauser, Dan Velleman e Stan Wagon.

7. 360 Problems for Mathematical Contests


Titu Andreescu e Dorin Andrica.

8. Áreas para achar razões de segmentos


Cı́cero Thiago e Marcelo Mendes.
Revista Eureka 25

9. Mathematical Olympiad Treasures


Titu Andreescu e Bogdan Enescu˙
10. Mandelbrot Morsels
Sam Vandervelde.

14
Polos Olímpicos de Treinamento
Curso de Geometria - Nível 2 Aula 13
Prof. Cícero Thiago

Revisão I

Problema 1. Na figura abaixo tem - se AD = DE, ∠A = ∠DEC e ∠ADE = ∠BDC.


Mostre que os triângulos ABD e EDC são congruentes.
D C

A E B

Solução. Como ∠ADE = ∠BDC então ∠ADB = ∠EDC. Portanto, ∆ABD ≡ ∆EDC
pelo caso ALA.

Problema 2. (OCM) Um triângulo ABC é tal que ∠C = 2∠A e AC = 2BC. Prove que
este triângulo é retângulo.

Solução. Seja CD a bissetriz interna do ângulo ∠C. Então, ∠BCD = ∠DCA = ∠DAC e
AC
o triângulo CDA é isósceles. Trace a altura DE deste triângulo. Observe que CE = =
2
BC. Daı́, os triângulos BCD e ECD são congruentes, pelo caso LAL, de onde concluı́mos
que ∠CBD = ∠CED = 90◦ , e o triângulo ABC é retângulo em B.
C
b

b
D

b b b

B E A
POT 2012 - Geometria - Nı́vel 2 - Aula 13 - Prof. Cı́cero Thiago

Problema 3. Seja ABC um triângulo isósceles, com AB = BC e ∠ABC = 82◦ . Seja M


um ponto no interior do triângulo tal que AM = AB e ∠M AC = 11◦ . Ache a medida do
ângulo ∠M CB.

Solução.

b
B

b
M

b b b

A D C

É fácil ver que ∠M AB = 38◦ . Como AM = AB, então ∠ABM = ∠AM B = 71◦ . Dessa
forma, ∠M BC = 11◦ . Seja D o ponto sobre o segmento AC tal que AD = BM . Assim,
∆M BC ≡ ∆DAM pelo caso LAL. Portanto, M D = M C e ∠BCM = ∠AM D = α. Pela
propriedade do ângulo externo temos que ∠M DC = 11◦ + α e, como M D = M C então
11◦ + α = 49◦ − α ⇔ α = ∠M CB = 19◦ .

Problema 4. Seja ABC um triângulo isósceles de base AC tal que ∠B = 20◦ . Prove que
AB < 3AC.

Solução. Inicialmente construa os triângulos BCD e DBE congruentes ao triângulo ABC


como feito na figura abaixo. Dessa forma é fácil ver que o triângulo ABE é equilátero.
Então, AE = AB. É fácil verificar, usando a desigualdade triangular, que AC +CD+DE >
AE. Como os triângulos BCD, DBE e ABC são congruentes então AC = CD = DE.
Portanto, AB < 3AC.

2
POT 2012 - Geometria - Nı́vel 2 - Aula 13 - Prof. Cı́cero Thiago

b
B


20◦
20◦ 20

E
80◦ b

80◦
80◦
b

80◦ D
80◦ 80◦
b b

A C

Problemas propostos

1. Sobre os lados de um triângulo ABC constroem - se externamente os triângulos


equiláteros BCD, CAE e ABF . Prove que os segmentos AD, BE e CF são congru-
entes.

2. Mostre que a hipotenusa de um triângulo retângulo é maior que a semi - soma dos
catetos.

3. (Torneio das Cidades) Se a, b e c são os comprimentos dos lados de um triângulo,


prove que a3 + b3 + 3abc > c3 .

4. Seja ABC um triângulo tal que ∠ABC = 2∠BCA, ademais, seja D o ponto do lado
BC tal que AD é bissetriz do ângulo ∠CAB e CD = AB. Calcule as medidas dos
ângulos do triângulo ABC.

5. Seja ABC um triângulo isósceles de base AC tal que ∠B = 20◦ . Prove que AB >

3
POT 2012 - Geometria - Nı́vel 2 - Aula 13 - Prof. Cı́cero Thiago

2AC.

6. Seja ABC um triângulo tal que ∠A = 20◦ . Sejam D e E pontos sobre os lados AC e
AB, respectivamente, tais que ∠AED = 40◦ e ED = DC = BC, determine a medida
do ângulo ∠B.

7. (OBM) O canto de um quadrado de cartolina foi cortado com uma tesoura. A soma
dos comprimentos dos catetos do triângulo recortado é igual ao comprimento do lado
do quadrado. Qual o valor da soma dos ângulos α e β marcados na figura abaixo?

27◦
α

8. Seja ABC um triângulo retângulo em C com AC < BC. Sejam D o ponto do lado AC
BK
e seja K o ponto do segmento BD tais que ∠KAD = ∠AKD = ∠ABC. Calcule .
CD

4
Polos Olímpicos de Treinamento
Curso de Geometria - Nível 2 Aula 14
Prof. Cícero Thiago

Teorema de Ceva e Teorema de Menelaus.

Teorema 1. (Ceva) Sejam D, E e F pontos sobre os lados BC, AC e AB, respectiva-


mente, do triângulo ∆ABC. Os segmentos AD, BE e CF intersectam - se em um ponto
BD CE AF
P se, e somente se, · · = 1.
CD EA F B
Demonstração.

A
b

F b

b
E
b
P

b b b

B D C

Defina K = [ABC], KA = [P BC], KB = [P CA] e KC = [P AB].

Temos que

BD [∆ABD] [∆BP D] [∆ABD] − [∆BP D] [∆AP B] KC


= = = = = .
CD [∆ACD] [∆CP D] [∆ACD] − [∆CP D] [∆ACP ] KB

CE KA AF KB BD CE AF KC KA KB
De maneira análoga, = e = . Assim, · · = · · = 1.
EA KC F B KA CD EA F B KB KC KA
BD CE AF
⇐ Sejam D, E e F pontos sobre os lados BC, CA e AB tais que · · =1
CD EA F B
mas AD, BE e CF não são concorrentes. Seja F1 sobre AB tal que AD, BE e CF1 são
POT 2012 - Geometria - Nı́vel 2 - Aula 14 - Prof. Cı́cero Thiago

BD CE AF1 AF AF1
concorrentes em P . Assim, · · = 1. Dessa forma, = ⇔ F = F1 .
CD EA F1 B FB F1 B
A
b

F1
b

b
E
F
b b
P

b b b

B D C

Exercı́cios resolvidos

1. Prove que as medianas de um triângulo são concorrentes em um ponto que se chama


baricentro.

Solução.
Sejam M , N e R os pontos médios de AC, BC e BA, respectivamente. Então
AM CN BR
· · = 1,
M C N B RA

ou seja, AN , BM e CR são concorrentes.

2. Prove que as bissetrizes internas de um triângulo são concorrentes em um ponto que


se chama incentro.

Solução.
Sejam X, Y e Z os pés das bissetrizes relativas aos lados BC, AC e AB, respectiva-
mente. Pelo teorema das bissetrizes internas temos que
AY CX BZ AB CA BC
· · = · · = 1,
Y C XB ZA BC AB CA

ou seja, AX, BY e CZ são concorrentes.

3. Prove que as alturas de um triângulo são concorrentes em um ponto que se chama


ortocentro.

2
POT 2012 - Geometria - Nı́vel 2 - Aula 14 - Prof. Cı́cero Thiago

Solução.

A A
b b

M b
M
b

N
b

b b b b
C
L
H B

b b b b N
B L C

Sejam AL, BM e CN as alturas do triângulo ∆ABC. É fácil ver que


AN AC
∆AN C ∼ ∆AM B ⇒ = (I)
MA AB
BL AB
∆BLA ∼ ∆BN C ⇒ = (II)
NB BC
CM BC
∆CM B ∼ ∆CLA ⇒ = (III).
LC AC
Multiplicando (I), (II) e (III) temos que

AN BL CM AC AB BC
· · = · · = 1,
M A N B LC AB BC AC

ou seja, as alturas são concorrentes.

4. Seja ABCDEF um hexágono convexo tal que cada uma das diagonais AD, BE e
CF dividem o hexágono em duas regiões de áreas iguais. Prove que AD, BE e CF
são concorrentes.

Solução.

3
POT 2012 - Geometria - Nı́vel 2 - Aula 14 - Prof. Cı́cero Thiago

F
b

b
A
Y
b

b
E

Z b
b

X
b

B b D

Sejam X a intersecção de AD e CE, Y a intersecção de AE e CF e Z é a intersecção


de AC e BE. Denotaremos por [M N P ] a área do triângulo ∆M N P , e seja K a área
do hexágono ABCDEF . É fácil ver que
K
CX [ACX] [CDX] [ACX] + [CDX] [ACD] − [ABC]
= = = = = 2 .
XE [AXE] [DEX] [AXE] + [DEX] [ADE] K
− [AEF ]
2

De maneira análoga,
K
EY − [CDE]
= 2
YA K
− [ABC]
2
e

K
AZ − [AF E]
= 2 .
ZC K
− [CDE]
2
Portanto,

K K K
CX EY AZ − [ABC] − [CDE] − [AF E]
· · = 2 · 2 · 2 = 1.
XE Y A ZC K K K
− [AEF ] − [ABC] − [CDE]
2 2 2

4
POT 2012 - Geometria - Nı́vel 2 - Aula 14 - Prof. Cı́cero Thiago

Pela recı́proca do teorema de Ceva no triângulo ∆ACE temos que AX, CY e EZ


são concorrentes e, com isso, AD, BE e CF são concorrentes.

5. Seja ∆ABC um triângulo e sejam P e Q pontos sobre os lados AB e AC, respecti-


vamente, tais que P Q k BC. Prove que P C, QB e a mediana AM , com M em BC,
são concorrentes.

Solução.
Como P Q k BC, então

AP AQ AP QC
= ⇔ · = 1 (I).
PB QC P B AQ

Como AM é um mediana então BM = M C, assim


BM
= 1 (II).
MC

Multiplicando (I) e (II), temos

AP QC BM
· · = 1.
P B AQ M C

Pela recı́proca do teorema de Ceva temos que AM , QB e P C são concorrentes.

b
A

P Q
b b

b b b

B M C

Exercı́cios propostos

5
POT 2012 - Geometria - Nı́vel 2 - Aula 14 - Prof. Cı́cero Thiago

1. Sejam D, E e F os pontos de contato da circunferência inscrita com os lados BC,


CA e AB, respectivamente, do triângulo ABC. Prove que AD, BE e CF são con-
correntes em um ponto que se chama Ponto de Gergonne.

2. Sejam l e l1 duas retas paralelas dadas no plano. Usando apenas régua encontre o
ponto médio do segmento AB que está na reta l.

3. (Coréia) Seja ABC um triângulo com AB 6= AC, seja V a intersecção da bissetriz


do ângulo ∠A com BC e seja D pé da altura relativa ao vértice A. Se E e F são as
intersecções dos cı́rculos circunscritos aos triângulos ∆AV D com CA e AB, respec-
tivamente, mostre que AD, BE e CF são concorrentes.

4. Seja P um ponto no interior de um triângulo. As bissetrizes de ∠BP C, ∠CP A e


∠AP B intersectam BC, CA e AB em X, Y e Z, respectivamente. Prove que AX,
BY e CZ são concorrentes.

Teorema 2. Se uma reta intersecta as retas BC, CA e AB de um triângulo ABC nos


pontos L, M e N , respectivamente, então
CL BN AM
· · = 1.
BL N A M C
Inversamente, se L, M e N são pontos sobre os lados BC, CA e AB do triângulo ABC
CL BN AM
tais que · · = 1, então L, M e N são colineares.
BL N A M C
Demonstração. ⇒
A
b

Q
b
N
b

P M
b
b
R

b b b
L
B C

Sejam AP , BQ e CR as perpendiculares traçadas a partir de A, B e C, respectivamente,


à reta em que se encontram L, M e N . É fácil ver que os triângulos retângulos AP N e

6
POT 2012 - Geometria - Nı́vel 2 - Aula 14 - Prof. Cı́cero Thiago

BQN são semelhantes, assim como os triângulos retângulos QBL e RCL. Então
BN BQ CL RC
= e = .
AN AP BL QB
Por outro lado, os triângulos retângulos AP M e CRM também são semelhantes. De modo
que
AM AP
= .
CM CR
Portanto,
BN CL AM BQ RC AP
· · = · · = 1.
AN BL CM AP QB CR

A
b

N
b

N1
b

b
M

b b b
L
B C

CL BN AM
Suponha, de maneira falsa, que · · = 1 e os pontos L, M e N não são
BL N A M C
colineares. Prolongue LM até intersectar AB em N1 . Pelo que foi provado acima temos
CL BN1 AM
que · · = 1, assim
BL N1 A M C
BN1 BN
= ⇔ N = N1 .
N1 A NA
Dessa forma, L, M e N são colineares.

Exercı́cios resolvidos

1. Dadas três circunferências C1 , C2 e C3 de centros O1 , O2 e O3 e raios r1 , r2 e r3 ,


respectivamente. Seja X a intersecção das tangentes comuns externas de C1 e C2 ,
Y a intersecção das tangentes comuns externas de C1 e C3 e, finalmente, Z a inter-
secção das tangentes comuns externas de C2 e C3 . Prove que X, Y e Z são colineares.

Solução. É fácil verificar que X, O1 e O2 são colineares. Assim, ∆XO1 P1 ∼ ∆XO2 P2


O1 X O1 P1 r1 O3 Y r3 O2 Z r2
e, com isso, = = . Analogamente, = e = . Portanto,
O2 X O2 P2 r2 O1 Y r1 O3 Z r3

7
POT 2012 - Geometria - Nı́vel 2 - Aula 14 - Prof. Cı́cero Thiago

O1 X O3 Y O2 Z
· · = 1.
O2 X O1 Y O3 Z
Pela recı́proca do teorema de Menelaus concluı́mos que X, Y e Z são colineares. Este
resultado é conhecido como teorema de Monge.

Z
b

Y
b

O2
b
b
O1 X
b

b
P1
P2

O3
b

2. Prove que as bissetrizes internas de dois ângulos de um triângulo isósceles e a bissetriz


externa do terceiro ângulo do triângulo intersectam os lados opostos em três pontos
colineares.

Solução.

8
POT 2012 - Geometria - Nı́vel 2 - Aula 14 - Prof. Cı́cero Thiago

No triângulo ABC, BM e CN são bissetrizes internas dos ângulos ∠B e ∠C, res-


pectivamente, e AL é a bissetriz externa do ângulo ∠A. Pelo teorema da bissetriz
interna temos que
AM AB BN BC
= e = .
MC BC N A AC
Além disso, pelo teorema da bissetriz externa temos que
CL AC
= .
BL AB

Assim,

AM BN CL AB BC AC
· · = · · = 1.
M C N A BL BC AC AB
Pela recı́proca do teorema de Menelaus temos que N , M e L são colineares.

A
b

Nb
b
M

b b b
L
B C

Exercı́cios propostos

1. Prove que as bissetrizes externas dos ângulos de um triângulo, não isósceles, inter-
sectam os lados opostos em três pontos colineares.

2. O ortocentro de um triângulo ABC é o ponto médio da altura relativa ao vértice


C. Prove que cos ∠C = cos ∠A · cos ∠B, em que ∠A, ∠B e ∠C são os ângulos do
triângulo ABC.

3. A bissetriz AD de um triângulo ABC divide o lado BC na razão 2 : 1. Determine a


razão em que a mediana CE divide a bissetriz.

4. (OBM) No triângulo ABC, D é ponto médio de AB e E ponto sobre o lado BC tal


que BE = 2 · EC. Sabendo que ∠ADC = ∠BAE, calcule o valor de ∠BAC.

9
POT 2012 - Geometria - Nı́vel 2 - Aula 14 - Prof. Cı́cero Thiago

5. (IMO) As diagonais AC e CE de um hexágono regular ABCDEF são divididas inter-


AM CN
namente pelos pontos M e N , respectivamente, na razão = = r. Determine
AC CE
r se B, M e N são colineares.

6. Seja ABC um triângulo e sejam E e D pontos sobre o lado BC tal que CE = ED =


DB. Seja F o ponto médio de AC e G o ponto médio de AB. Seja H a intersecção
EH
de EG e F D. Determine o valor de .
HG

7. (Cone Sul) Seja C uma circunferência de centro O, AB um diâmetro dela e R um


ponto qualquer em C distinto de A e de B. Seja P a intersecção da perpendicular
traçada por O a AR. Sobre a reta OP se marca o ponto Q, de maneira que QP é a
metade de P O e Q não pertence ao segmento OP . Por Q traçamos a paralela a AB
que corta a reta AR em T . Chamamos de H o ponto de intersecção das retas AQ e
OT . Provar que H, R e B são colineares.

Bibliografia
1. Leccture Notes on Mathematical Olympiad Courses
For senior Section, vol. 1
Xu Jiagu

2. Advanced Euclidean Geometry


Alfred Posamentier

3. III Olimpiada Nacional Escolar de Matemática 2006


Jorge Tipe, John Cuya, Claudio Espinoza e Sergio Vera.

4. Explorations in Geometry
Bruce Shawyer

5. Coleção Elementos de Matemática, vol.2


Marcelo Rufino de Oliveira

6. The theorem of Menelaus


B. Orach
Quantum - May/Jun 2001

7. Problemas de Geometrı́a - Planimetria


I. Shariguin

10
Polos Olímpicos de Treinamento
Curso de Geometria - Nível 2 Aula 15
Prof. Cícero Thiago

Pontos Notáveis 1: Baricentro

Propriedade 1. As três medianas de um triângulo intersectam - se num mesmo ponto,


chamado baricentro, que divide cada uma das medianas em duas partes tais que a parte
que contém o vértice é o dobro da outra.

P G N

B M C

Demonstração.
A

P N

D G1 E

B C

Sejam N e P os pontos médios dos lados AC e AB, respectivamente, D e E os pontos


médios de BG1 e CG1 , respectivamente. Então,
BC
N P k BC e N P =
2
POT 2012 - Geometria - Nı́vel 3 - Aula 15 - Prof. Cı́cero Thiago

e
BC
DE k BC e DE =
2
portanto, P DEN é uma paralelogramo. Com isso, BD = DG1 = G1 N , CE = EG1 =
G1 P , então BG1 = 2G1 N e CG1 = 2G1 P . De maneira análoga, as medianas AM e BN
intersectam - se em um ponto G2 tal que AG2 = 2G2 M e BG2 = 2G2 N . Encontramos,
então, dois pontos distintos G1 e G2 , no interior do segmento BN que o dividem na mesma
razão, o que é uma contradição logo, G1 = G2 = G. Portanto, as três medianas intersectam
- se em um mesmo ponto G que chamaremos de baricentro.

Exercı́cios Resolvidos

1. (OBM) Seja N o ponto do lado AC do triângulo ABC tal que AN = 2N C e M o


ponto do lado AB tal que M N é perpendicular a AB . Sabendo que AC = 12 cm e que o
baricentro G do triângulo ABC pertence ao segmento M N , determine o comprimento do
segmento BG.
OBS: Baricentro é o ponto de interseção das medianas do triângulo.

Solução.

Se BP é uma mediana do triângulo então AP = CP = 6 e P N = 2. Como G é o baricentro


PG 1 PN 1
do triângulo então = e = , assim, pela recı́proca do teorema de Tales, GN é
GB 2 NC 2
paralelo a BC e ∠B = 90◦ . Como o triângulo ABC é retângulo então AP = CP = BP = 6.
Com isso, BG = 4 e GP = 2.

b
B

M b

G b

b b b b
C
A P N

2. (Bulgária) Seja ∆ABC um triângulo isósceles (AC = BC) tal que A1 , B1 e C1 são os
pontos médios de BC, AC e AB, respectivamente. Os pontos A2 e B2 são os simétricos de
A1 e B1 com relação ao lado AB. Seja M a interseção de CA2 e A1 C1 e seja N a interseção
de CB2 e B1 C1 . Seja P a interseção de AN e BM , prove que AP = BP .

Solução. Como CC1 k A1 A2 e CC1 = A1 A2 , temos que CC1 A2 A1 é um paralelogramo.


Então, A1 M = C1 M . Mas A1 B1 C1 B é também um paralelogramo e, portanto, a in-

2
POT 2012 - Geometria - Nı́vel 3 - Aula 15 - Prof. Cı́cero Thiago

terseção BM e AC é B1 . Então, P está sobre a mediana BB1 . Analogamente, P está


sobre a mediana AA1 . No triângulo isósceles ABC as medianas AA1 e BB1 possuem o
2 2
mesmo comprimento. Portanto, AP = AA1 = BB1 = BP .
3 3
Exercı́cios Propostos

Problema 1. Uma reta r passa pelo baricentro de um triângulo ABC deixando o vértice A
em um semiplano e os vértices B e C no outro semiplano determinado por r. As projeções
de A, B e C sobre a reta r são M , N e P , respectivamente. Prove que AM = BN + CP .

Problema 2. (OBM) Seja ABCD um quadrilátero convexo, onde N é o ponto médio de


DC, M é o ponto médio de BC, e O é a interseção entre as diagonais AC e BD. Mostre
que O é o baricentro do triângulo AM N se, e somente se, ABCD é um paralelogramo.

Problema 3. (Portugal) No triângulo ABC as medianas dos lados AB e AC são perpen-


diculares. Sabendo que AB = 6 e AC = 8, determine BC.

Problema 4. (Estônia) As medianas relativas aos vértices A e B do triângulo ABC são


perpendiculares. Prove que AB é o menor lado do triângulo ABC.

Problema 5. (OCM) Seja ABC um triângulo tal que as medianas BM e CN , que se cor-
tam em G, são iguais. Prove que o triângulo ABC é isósceles.

Problema 6. Prove que a soma dos quadrados das distâncias de um ponto P aos vértices
de um triângulo ABC é mı́nima quando P é o baricentro do triângulo.

Soluções

1. Seja AD um mediana e Q o ponto médio de N P . Então, DQ é a base média do trapézio


BN + CP
N BCP assim DQ k BN e DQ = . Como G é o baricentro do triângulo ABC
2
AM
então AG = 2GD. É fácil ver que ∆AM G ∼ ∆GQD, então = DQ. Portanto,
2
AM = BN + CP .

3
POT 2012 - Geometria - Nı́vel 3 - Aula 15 - Prof. Cı́cero Thiago

A
b

b
P

G Q
b b
b
M
b
N

b b b

B D C

2. (⇒) Suponha que ABCD é um paralelogramo, então AO = OC e BO = BD. Se M


BD
e N são os pontos médios de BC e CD então M N k BD e M N = . É fácil concluir
2
BO DO
que P é o ponto de médio de OC então M P k BO, M P = , N P k DO e N P = .
2 2
Portanto, N P = P M e AO = 2OP , ou seja, O é o baricentro de AM N .

(⇐) Suponha que O é o baricentro do triângulo AM N então N P = P M e AO = 2OP . Se


BD
M e N são os pontos médios de BC e CD então M N k BD e M N = . É fácil concluir
2
BO
que P é o ponto de médio de OC então OP = P C, M P k BO, M P = , N P k DO e
2
DO
NP = . Daı́, AO = OC e DO = OB, ou seja, ABCD é um paralelogramo.
2

4
POT 2012 - Geometria - Nı́vel 3 - Aula 15 - Prof. Cı́cero Thiago

b
A b
B

b b M
O

b b b

D N C

3. Sejam M e N os pontos médios de AB e AC, respectivamente, e G o ponto de encontro


das medianas M C e N B. Aplicando o teorema de Pitágoras BGM e CN G, temos:

GM 2 + 4GN 2 = GM 2 + GB 2 = BM 2 = 32 = 9

4GM 2 + GN 2 = GC 2 + GN 2 = CN 2 = 42 = 16.
√ √
Deste modo, 5GM 2 + 5GN 2 = 9 + 16 = 25, logo N M = 5. Portanto, BC = 2 5.

b
C

N b

b
G

b b b

A B
M

4. Seja M o baricentro do triângulo A1 B1 C1 . Seja A2 um ponto sobre a reta M A tal que


B1 A1 C1 A2 é um paralelogramo. Os pontos B2 e C2 são construı́dos analogamente. Como
A1 C1 k A1 B1 k C1 B2 então os pontos A2 , C1 e B2 são colineares e C1 é o ponto médio de

5
POT 2012 - Geometria - Nı́vel 3 - Aula 15 - Prof. Cı́cero Thiago

A2 B2 . O mesmo é verdade para os pontos A2 , B1 e C2 e C2 , A1 e B2 . Vamos mostrar que


A2 = A, B2 = B e C2 = C, o que resolve o problema. Assuma que A2 6= A e A está entre
A2 e M . Então C2 está entre C e M , B está entre B2 e M e consequentemente A2 está
entre A e M , que é uma contradição.

5. As medianas intersectam - se no ponto M e a mediana que parte do vértice C intersecta


AB no ponto F . Então, F é o ponto médio da hipotenusa do triângulo retângulo ABM ,
ou seja, AB = 2F M . Como M divide a mediana CF na razão 2 : 1, então AB = CM .
O maior ângulo do triângulo AM C é o ângulo obtuso AM C, portanto AC é o maior lado
deste triângulo. Assim, AC > M C = AB. De maneira análoga BC > AB.
m
6. Seja BM = CN = m. Como G é o baricentro de ABC, temos GM = = GN e
3
2m
BG = = CG. Daı́, segue que os triângulos BGN e CGM são congruentes (pelo caso
3
LAL), de modo que BN = CM . Logo, AB = 2 · BN = 2 · CM = AC, e o triângulo ABC
é isósceles.

A
b

N b b
M

G
b

b b

B C

7. Seja ABC um triângulo com BC = a, AC = b e AB = c. Seja M o ponto médio de


BC, G o baricentro do triângulo ABC e P um ponto qualquer. Usando que, a soma dos
quadrados de dois dos lados de um triângulo é igual a duas vezes o quadrado da mediana
relativa ao terceiro lado mais a metade do quadrado do terceiro lado (a demonstração desse
resultado usa lei dos Cossenos e será provado na aula de relações métricas), no triângulo

6
POT 2012 - Geometria - Nı́vel 3 - Aula 15 - Prof. Cı́cero Thiago

P BC com mediana P M temos:


a2
P B 2 + P C 2 = 2P M 2 +
. (I)
2
O baricentro G é tal que GA = 2GM . Faça GM = m; GA = 2m e tome H em AG tal que
GH = AH = m. Assim, o triângulo HP M , com mediana P G satisfaz
1
P H 2 + P M 2 = 2P G2 + (2m)2 = 2P G2 + 2m2 (II)
2
e o triângulo AP G com mediana P H satisfaz
1
P A2 + P G2 = 2P H 2 + (2m)2 = 2P H 2 + 2m2 . (III)
2
Somando (I) e (III)
a2
P A2 + P B 2 + P C 2 + P G2 = 2P M 2 + + 2P H 2 + 2m2 =
2
a2
= 2(P M 2 + P H 2 ) + + 2m2 = por (II)
2
a2
2(2P G2 + 2m2 ) + + 2m2 =
2
a2
4P G2 + 6m2 + .
2
a2
Portanto, P A2 + P B 2 + P C 2 = 3P G2 + 6m2 + . (IV)
2
Como o triângulo a e m são constantes, P A2 + P B 2 + P C 2 é mı́nimo quando P G = 0, ou
seja, P = G é o baricentro do triângulo ABC.

A
b

b
H
b
P

b
G

b b b

B M C

7
POT 2012 - Geometria - Nı́vel 3 - Aula 15 - Prof. Cı́cero Thiago

Bibliografia

1. Lecture Notes on Mathematical Olympiad Courses


For Junior Section, vol. 1
Xu Jiagu

2. Puntos Notables - Teorı́a - Demostraciones - Trazos Auxiliares


440 problemas resueltos e propuestos
Julio Orihuela Bastidas
Editorial Cuzcan

3. Geometrı́a
Radmila Bulajich Manfrino e José Antonio Gómez Ortega
Cuadernos de Olimpiadas de Matemáticas

4. Tópicos de Matemática Elementar, vol. 2


Geometria Euclidiana Plana
Antonio Caminha Muniz Neto
SBM

5. Episodes in Nineteenth and Twentieth Euclidean Geometry


Ross Honsberger
MAA

6. Problems in Plane and Solid Geometry, vol. 1 - Plane Geometry


Viktor Prasolov

7. Advanced Euclidean Geometry


Alfred Posamentier

8. Lessons in Geometry
I. Plane Geometry
Jacques Hadamard
AMS

9. Hadamard’s Plane Geometry


A Reader’s Companion
Mark Saul
AMS

10. Coleção Elementos da Matemática


Geometria Plana, vol. 2
Marcelo Rufino de Oliveira e Márcio Rodrigo da Rocha Pinheiro

8
POT 2012 - Geometria - Nı́vel 3 - Aula 15 - Prof. Cı́cero Thiago

11. Olimpı́adas Cearenses de Matemática, Ensino Médio, 1981 - 2005


Emanuel Carneiro, Francisco Antônio M. de Paiva e Onofre Campos

12. Problemas de las Olimpiadas Matematicas del Cono Sur (I a IV)


Fauring - Wagner - Wykowski - Gutierrez - Pedraza - Moreira
Red Olı́mpica

13. Fundamentos de Matemática Elementar, vol. 9 - Geometria Plana


Osvaldo Dolce e José Nicolau Pompeo

14. Olimpiada Matemática Española


15000 problemas de diferentes Olimpiadas de Matemática en el mundo

9
Polos Olímpicos de Treinamento
Curso de Geometria - Nível 2 Aula 16
Prof. Cícero Thiago

Pontos Notáveis 2: Incentro

Teorema 1. Seja ∠XOY um ângulo dado e P um ponto em seu interior. Então, a distância
de P a XO é igual à distância de P a Y O se, e somente se, o ponto P pertence à bissetriz.
Demonstração.
b
X

M
b

O b b P

N
b
Y

Suponhamos inicialmente que o ponto P pertence à bissetriz. Então ∠XOP = ∠Y OP .


Sejam M e N os pés das perpendiculares baixadas desde P sobre OX e OY , respecti-
vamente. Podemos concluir, que ∆M OP ≡ ∆N OP , pelo caso L.A.A., pois OP é lado
comum, ∠M OP = ∠N OP e ∠OM P = ∠ON P = 90◦ . Portanto, P M = P N .
Reciprocamente, suponhamos agora que P M = P N . Pelo caso especial de congruência
de triângulos, cateto-hipotenusa, os triângulos M OP e N OP são congruentes. Portanto,
∠M OP = ∠N OP , e assim, P pertence à bissetriz.

Provemos agora que as três bissetrizes de um triângulo ABC se intersectam num ponto
chamado incentro, que é equidistante dos lados do triângulo.
POT 2012 - Geometria - Nı́vel 3 - Aula 16 - Prof. Cı́cero Thiago

A
b

b
N1
P1 b
N
b

P b
I
b

b b b

B M1 C

Sejam BN e CP as bissetrizes relativas aos vértices B e C, respectivamente, e I o seu


ponto de interseção. Como o ponto I pertence às bissetrizes BN e CP , então IM1 = IP1 e
IM1 = IN1 , em que M1 , N1 , P1 são os pés das perpendiculares baixadas desde I sobre os
lados BC, CA e AB, respectivamente. Como IP1 = IN1 , então, pela proposição anterior,
I pertence à bissetriz do ângulo ∠A. Portanto, as três bissetrizes passam por um mesmo
ponto chamado incentro que será o centro da circunferência inscrita no triângulo pois I
equidista dos lados do triângulo. Além disso, M1 , N1 e P1 são os pontos de tangência do
cı́rculo com os lados BC, CA e AB, respectivamente.

A
b

b
N1
P1 b
N
b

P b
I
b

b b b b

B M1 M C

Teorema 2. Seja ABC um triângulo tal que BC = a, CA = b e AB = c. Sejam M1 ,


N1 e P1 os pontos de tangência com os lados BC, CA e AB, respectivamente. Então,
a+b+c
AN1 = AP1 = p − a, BM1 = BP1 = p − b e CM1 = CN1 = p − c, em que p = .
2

2
POT 2012 - Geometria - Nı́vel 3 - Aula 16 - Prof. Cı́cero Thiago

Demonstração.

A
b
x
x
b
N1
P1
b

I
y b z

b b b

B y M1 z C

Temos que y + z = a, x + z = b e x + y = c. Resolvendo o sistema encontramos x = p − a,


y = p − b e z = p − c.

Teorema 3. (Bissetriz interna) A bissetriz interna AL do ângulo ∠A de um triângulo


AB
ABC divide internamente o lado oposto BC na razão , ou seja,
CA
BL AB
=
LC CA
em que L é o ponto de intersecção da bissetriz interna com o lado BC.

Demonstração.

b
R

A
b

α α

b b b

B L C

3
POT 2012 - Geometria - Nı́vel 3 - Aula 16 - Prof. Cı́cero Thiago

Seja R a intersecção da paralela à bissetriz AL traçada pelo ponto C. É fácil ver que
∠BAL = ∠CAL = ∠ACR = ∠ARC, com isso, AR = AC. Pelo teorema de Tales temos
que
AB BL
= .
AR LC
Como AR = AC, então
AB BL
= .
AC LC
Teorema 4. Seja ABC um triângulo tal que BC = a, CA = b, AB = c e seja AM a
a·c
bissetriz relativa ao ângulo ∠A, com M em BC. Então, BM = .
b+c
Demonstração.

A
b

α α
c b

b b b

B m M a−m C

Usando o teorema da bissetriz interna temos que


AB AC c b a·c
= ⇔ = ⇔m= .
BM CM m a−m b+c
Teorema 5. Seja ABC um triângulo tal que BC = a, CA = b, AB = c, AM a bissetriz
AI b+c
relativa ao ângulo ∠A, com M em BC, e seja I o incentro. Então, = .
IM a
Demonstração.
Aplicando o teorema da bissetriz interna no triângulo BAM temos que
AI AB AI b+c
= ⇔ = .
IM BM IM a

4
POT 2012 - Geometria - Nı́vel 3 - Aula 16 - Prof. Cı́cero Thiago

A
b

α α
c b

b
I

b b b

B a·c M C

b+c

Teorema 6. Seja ABC um triângulo e I seu incentro. Seja E o ponto de interseção de


AI com a circunferência circunscrita ao triângulo ABC. Então IE = IB = IC.

Demonstração.
A
b

α α

b
I

α+β
β
β
b b b

B α D α C

É fácil ver que ∠BAE = ∠CAE = ∠CBE = ∠BCE e, portanto, BE = CE. Além disso,
pela propriedade do ângulo externo, ∠BIE = α+β. Portanto, ∠BIE = ∠IBE e BE = IE.

5
POT 2012 - Geometria - Nı́vel 3 - Aula 16 - Prof. Cı́cero Thiago

Observe agora uma parte da figura acima.


a
B 2 M C
b b b

Temos que
BM a
cos α = ⇔ BE = = CE = IE.
BE 2 cos α
Teorema 7. Seja ABC um triângulo tal que BC = a, CA = b, AB = c e seja AM a
bissetriz relativa ao ângulo ∠A, com M em BC. Além disso, ∠BAM = CAM = α. Então
2 · b · c · cos α
AM = .
b+c
Demonstração.

A
b

α α
c b

b b b

B M C

É fácil ver que [ABC] = [BAM ] + [CAM ]. Então,


b · c · sin 2α c · AM · sin α b · AM · sin α
= + ⇔
2 2 2
2 · b · c · cos α
AM = .
b+c
Teorema 8. (Área de um triângulo em função do raio da circunferência inscrita.)

Sejam a, b e c as medidas dos lados BC, CA e AB do triângulo ∆ABC, respectivamente, e


seja r a medida do raio da circunferência inscrita. Então, a área do triângulo ∆ABC pode

6
POT 2012 - Geometria - Nı́vel 3 - Aula 16 - Prof. Cı́cero Thiago

ser calculada por


[∆ABC] = p · r,
a+b+c
em que p = .
2
Demonstração.

A
b

b E
F b

r
r
b

b b b

B D C

[∆ABC] = [∆BIC] + [∆CIA] + [∆AIB] ⇔


a·r b·r c·r
[∆ABC] = + + ⇔
2 2 2
 
a+b+c
[∆ABC] = ·r ⇔
2
[∆ABC] = p · r.

Problema 1. (OBM) O triângulo ABC é retângulo em B. Sejam I o centro da circun-


ferência inscrita em ABC e O o ponto médio do lado AC. Se ∠AOI = 45◦ , quanto mede,
em graus, o ângulo ∠ACB?

Solução.
Como ABC é um triângulo retângulo, então AO = BO = CO. Se ∠ABI = ∠AOI = 45◦
e ∠BAI = ∠OAI, então ∆ABI ≡ ∆AOI (ALA). Com isso, AB = AO = BO e, portanto,
triângulo ABO é equilátero. Assim, ∠ACB = 30◦ .

7
POT 2012 - Geometria - Nı́vel 3 - Aula 16 - Prof. Cı́cero Thiago

b
B

b
I

b b b

A O C

Problema 2. Em um triângulo não equilátero, a reta que passa pelo baricentro e pelo in-
centro é paralela a um dos lados do triângulo. Demonstre que os lados do triângulo estão
em progressão aritmética.

Solução.
Como IG é paralelo a BC então podemos aplicar o teorema de Tales. Assim,
AI AG b+c 2
= ⇔ = ⇔ b + c = 2a.
IE GD a 1
b
A

b
I b
G

b b
E b
D
B

Exercı́cios propostos

8
POT 2012 - Geometria - Nı́vel 3 - Aula 16 - Prof. Cı́cero Thiago

1.
2. (IMO Shortlist) Seja ABC um triângulo tal que AB + BC = 3AC. Sejam I o seu
incentro e D e E os pontos de tangência da circunferência inscrita com os lados AB
e BC, respectivamente. Além disso, sejam K e L os simétricos de D e E com relação
ao incentro I. Prove que o quadrilátero ACKL é inscritı́vel.

3. (Teste de seleção do Brasil para IMO) Seja I o incentro do triângulo ABC e D o


ponto de interseção de AI com o cı́rculo circunscrito de ABC. Sejam E e F os pés
das perpendiculares baixadas a partir de I sobre BD e CD, respectivamente. SE
AD
IE + IF = , determine o ângulo BAC.
2

4. (IMO) O prolongamento da bissetriz AL do triângulo acutângulo ABC encontra o


cı́rculo circunscrito em N . Por L traçam - se perpendiculares LK e LM aos lados
AB e AC, respectivamente. Prove que a área do triângulo ABC é igual à área do
quadrilátero AKN M .

5. Num triângulo ABC tem - se AB = BC, e D é um ponto sobre a base AC tal que
o raio do cı́rculo inscrito no triângulo ABD é igual ao raio do cı́rculo tangente ao
segmento DC e aos prolongamentos das retas BD e BC. Prove que o raio deste
1
cı́rculo é igual a da medida h de uma das alturas iguais do triângulo ABC.
4

6. Seja um quadrilátero ABCD inscrito num cı́rculo de tal forma que os prolongamentos
dos lados AD e BC se encontram em Q e os prolongamentos de AB e CD, em P .
Prove que as bissetrizes dos ângulos ∠DQC = ∠AP D são perpendiculares.

7. Do incentro de um triângulo retângulo, avista - se a metade da hipotenusa, isto é,


o segmento que une um vértice ao ponto médio da hipotenusa, segundo um ângulo
m
reto. Se é a fração irredutı́vel que expressa a razão entre as medidas dos catetos
n
deste triângulo, então m + n é igual a:
(a) 7 (b) 17 (c) 23 (d) 31 (e) 41

8. O cı́rculo, de centro O, inscrito no triângulo ABC é cortado pela mediana AD nos


pontos X e Y . Sabendo que AC = AB + AD, determine a medida do ângulo ∠XOY .

9. (OCM) Seja ABC um triângulo cuja medida dos lados são números inteiros e con-
secutivos. Além disso, o maior ângulo ∠A é o dobro do menor ângulo. Determine a
medida dos lados deste triângulo.

9
Polos Olímpicos de Treinamento
Curso de Geometria - Nível 2 Aula 17
Prof. Cícero Thiago

Pontos Notáveis 3: Circuncentro e Ortocentro

Teorema 1. Sejam A, B e P três pontos distintos no plano. Temos que P A = P B se, e


somente se, o ponto P pertence à mediatriz do segmento AB.
Demonstração.
r

b
P

b b b

A M B

Sejam M o ponto médio de AB e r a sua mediatriz. Suponha inicialmente que P pertence


à mediatriz. Com isso AM = M B e r é perpendicular à AB. É fácil ver que os triângulos
∆AM P e ∆BM P são congruentes pelo caso L.A.L. e, com isso, P A = P B.
Reciprocamente, suponha agora, que P A = P B, com isso ∆ABP é isósceles de base AB.
Tracemos a mediana relativa ao lado AB. É fácil ver que os triângulos ∆AM P e ∆BM P
são congruentes pelo caso L.L.L. e, com isso, ∠AM P ≡ ∠BM P = 90◦ , ou seja, P está
sobre a mediatriz.

Teorema 2. As três mediatrizes de um triângulo ABC se intersectam num ponto chamado


circuncentro que é o centro da circunferência circunscrita.

Demonstração.
POT 2012 - Geometria - Nı́vel 2 - Aula 17 - Prof. Cı́cero Thiago

A
b r

b
O

b b

B s C

Sejam r e s as mediatrizes relativas aos lados BC e AB, respectivamente, e seja O o ponto


de interseção das duas mediatrizes. Pelo teorema 1, temos que BO = CO e BO = AO.
Então, CO = AO e, também pelo teorema 1, O deve estar sobre a mediatriz relativa ao
lado AC.

Além disso o circuncentro é o centro da circunferência circunscrita ao triângulo ABC pois


é equidistante dos três vértices do triângulo.

Teorema 3. As três alturas de um triângulo ABC se intersectam num ponto chamado


ortocentro.

Demonstração.

2
POT 2012 - Geometria - Nı́vel 2 - Aula 17 - Prof. Cı́cero Thiago

P A N
b b b

b
E

F b

b H

b b
D b

B C

Inicialmente tracemos pelos vértices A, B e C, retas paralelas aos lados BC, CA e AB,
respectivamente, que determinam o triângulo M N P . Já sabemos que as três mediatrizes
de um triângulo se intersectam em seu circuncentro. É fácil perceber que A, B e C são
os pontos médios dos segmentos N P , M P e M N , respectivamente, pois P ACB, N ABC e
ABM C são paralelogramos e, portanto, os lados opostos de um paralelogramo são iguais.
Tracemos as mediatrizes dos segmentos M P , M N e P N que irão se intersectar no ponto
H. Mas as mediatrizes do triângulo M N P são as alturas do triângulo ABC. Portanto,
provamos que as três alturas de um triângulo ABC se intersectam em um ponto que será
chamado de ortocentro.

Teorema 4. Seja O o centro da circunferência circunscrita ao triângulo acutângulo ABC


e seja D a projeção de A sobre BC então ∠DAB = ∠OAC.
Demonstração.

3
POT 2012 - Geometria - Nı́vel 2 - Aula 17 - Prof. Cı́cero Thiago

A
b

b
O

β
b b b

B D C
β

Seja AE um diâmetro. Além disso, ∠ABC = ∠AEC. Portanto, ∠BAD = ∠EAC.

Teorema 5. O ortocentro, o baricentro e o circuncentro de um triângulo, não equilátero,


são colineares. A reta determinada por esses pontos é chamada de Reta de Euler.

Demonstração.
Sejam M e N os pontos médios de BC e AC, respectivamente. Então, M N k AB e
AB
MN = . O teorema 4 garante que ∠BAD = ∠OAC. Como O é o circuncentro
2
então OA = OC e, com isso, ∠OAC = ∠OCA. O quadrilátero M CN O é inscritı́vel
então ∠OCA = ∠N CO = ∠OM N e ∠M ON = 180◦ − ∠ACB. Além disso, o qua-
drilátero DCEH também é inscritı́vel e, com isso, ∠DHE = 180◦ − ∠ACB. Como
∠DHE = ∠AHB concluı́mos que o triângulo AHB é semelhante ao triângulo M N O e, com
AB AH
isso, = = 2. Temos que ∠HAG = ∠GM O pois AH é paralelo a OM e, como
MN OM
AG
G é o baricentro, = 2. Portanto, o triângulo AHG é semelhante ao triângulo GM O
GM
e, com isso, ∠HGA = ∠M GO provando então que H, G e O estão alinhados e HG = 2GO.

4
POT 2012 - Geometria - Nı́vel 2 - Aula 17 - Prof. Cı́cero Thiago

A b

b
E

b
b
N
H

G b
O

b b b b

B D M C

Teorema 6. Os pés das alturas de um triângulo, os pontos médios do três lados e os pontos
médios dos segmentos que ligam os vértices ao ortocentro estão sobre uma circunferência
chamada Circunferência dos 9 pontos.

Demonstração. Queremos provar que M , L, P , D, E, F , R, S e T são concı́clicos. É


suficiente provar que R e D estão sobre a circunferência circunscrita ao triângulo M LP ,
pois o restante é análogo. Considere a circunferência Γ de diâmetro RM . É fácil ver que
D pertence a Γ. Por outro lado, RL k HC, LM k AB e HC ⊥ AB, o que implica que
∠RLM = 90◦ . Portanto, L (e por simetria P ) pertence a Γ.

b
A

R E
b b

F b

b H
P b b
L

b
N

b b
T
S
b
O

b b b b
C
B D M

Teorema 7. O centro da circunferência dos 9 pontos é o ponto médio do segmento formado


pelo ortocentro e pelo circuncentro.

Demonstração.

Seja RM um diâmetro da circunferência dos 9 pontos e seja N a interseção de RM e OH.

5
POT 2012 - Geometria - Nı́vel 2 - Aula 17 - Prof. Cı́cero Thiago

Como R é ponto médio de AH então RH = OM . Além disso, AH k OM . Portanto,


∆RHN ≡ ∆N OM , RN = N M e HN = ON .
A
b

E
R b
b

F b

P b b
L
H
b

N b G
b b
T
S
b
O

b b b b
C
B D M

Problema 1. Seja ABC um triângulo e sejam H o ortocentro e o O o circuncentro do


triângulo. Se ∠ABH = ∠HBO = ∠OBC e BH = BO determine a medida do ângulo ∠A.

Solução.
Como O é o circuncentro então OC = OB = BH. Além disso, BH = OC = 2OM . Como
o triângulo M OC é retângulo então ∠M OC = 60◦ . Assim, ∠AOC = 120◦ e ∠ABC = 60◦ .

6
POT 2012 - Geometria - Nı́vel 2 - Aula 17 - Prof. Cı́cero Thiago

B
b

b
O
b
H

b b b b

A D M C

Problema 2. Seja H o ortocentro de um triângulo ABC, tal que AC 6= BC. O segmento


que une os pontos médios de HC e AB intersecta a bissetriz de ∠ACB no ponto N . Sa-
bendo que o circuncentro do triângulo ABC pertence à reta que passa pelos pontos H e
N , determine a medida do ∠ACB.

Solução.
Seja M o ponto médio de AB, L o ponto médio de HC, O o circuncentro do triângulo ABC
1
e R o raio do cı́rculo circunscrito ao triângulo ABC. É bem sabido que OM = HC = LC.
2
Como OM é paralelo a LC, então OM LC é um paralelogramo. Por outro lado, a bissetriz
do ângulo ∠ACB é bissetriz também do ângulo ∠OCH, daı́ ∠LN C = ∠N CO = ∠N CL e
N L = CL = LH, o que implica ∠HN C = 90◦ , logo CN é altura e bissetriz do triângulo
HOC, assim HC = CO e, portanto, ∠ACB = 60◦ .

Problemas propostos

1. Seja ABC um triângulo tal que ∠ABC = 50◦ . Seja F um ponto qualquer sobre o
lado AC. Se M e N são os ortocentros dos triângulos ABF e BF C, respectivamente,
determine a medida do ângulo ∠M F N .

7
POT 2012 - Geometria - Nı́vel 2 - Aula 17 - Prof. Cı́cero Thiago

2. Seja ABC um triângulo tal que ∠BAC = 40◦ e seja P um ponto sobre o lado AB tal
que o ortocentro de ABC coincide com o circuncentro de P BC. Determine a medida
do ângulo ∠P CB.

3. (ITA) Em um triângulo de vértices A, B e C, a altura, a bissetriz e a mediana, rela-


tivamente ao vértice C, dividem o ângulo ∠BCA em quatro ângulos iguais. Se l é a
medida do lado oposto ao vértice C, calcule:
(a) A medida da mediana em função de l.
(b) Os ângulos ∠CAB, ∠ABC e ∠BCA.

4. Seja ABC um triângulo que não é isósceles. Os pontos O e H são, respectivamente,


o circuncentro e o ortocentro e M o ponto médio de OH.
(a) Se ABC é um triângulo acutângulo e a bissetriz interna de ∠BAC passa por M ,
determine a medida do ângulo ∠BAC.
(b) Se ABC é um triângulo obtusângulo e a bissetriz externa do ângulo ∠BAC passa
por M , determine a medida do ângulo ∠BAC.

5. (Torneio das cidades) AD, BE e CF são alturas de um triângulo ABC. K, M e N


são os ortocentros dos triângulos AEF , BF D e CDE. Prove que KM N e DEF são
triângulos congruentes.

6. Seja ABC um triângulo. Sobre os lados AB e AC são construı́dos no exterior do


triângulo os quadrados ABDE e ACF G. Prove que CD, BF e a altura relativa ao
vértice A são concorrentes.

7. (OBM) Sejam H, I e O o ortocentro, o incentro e o circuncentro do triângulo ABC,


respectivamente. A reta CI corta o circuncı́rculo de ABC no ponto L, distinto de C.
Sabe-se que AB = IL e AH = OH. Determine os ângulos do triângulo ABC.

8. (Irã) Em um triângulo ABC temos que ∠A = 60◦ . Seja D um ponto que varia sobre
o lado BC. Sejam O1 o circuncentro de ABD e O2 o circuncentro de ACD. Seja M
a interseção de BO1 e CO2 e N o circuncentro de DO1 O2 . Prove que M N passa por
um ponto fixo.

8
Polos Olímpicos de Treinamento
Curso de Geometria - Nível 2 Aula 18
Prof. Cícero Thiago

Circunferências ex - inscritas

Teorema 1. Seja ∠XOY um ângulo dado e P um ponto em seu interior. Então, a distância
de P a XO é igual a distância de P a Y O se, e somente se, o ponto P pertence a bissetriz.
Demonstração.
b
X
M
b

b
P

b b b
Y
O N

Suponhamos inicialmente que o ponto P pertence à bissetriz. Então, ∠XOP = ∠Y OP . Se-


jam M e N os pés das perpendiculares baixadas desde P sobre OX e OY , respectivamente.
Podemos concluir, que ∆M OP ≡ ∆N OP , pelo caso L.A.A.. Portanto, P M = P N .
Reciprocamente, suponhamos agora que P M = P N . Pelo caso especial de congruência de
triângulos, cateto - hipotenusa, os triângulos M OP e N OP são congruentes. Portanto,
∠M OP = ∠N OP e, assim, P pertence à bissetriz.

Teorema 2. As bissetrizes externas de quaisquer dois ângulos de um triângulo são con-


correntes com a bissetriz interna do terceiro ângulo.

Demonstração.
POT 2012 - Geometria - Nı́vel 2 - Aula 18 - Prof. Cı́cero Thiago

b
E

α A
b

P b α
F
β β
b

b b b

D B C

No triângulo ABC traçamos as bissetrizes externas dos ângulos ∠A e ∠B os quais se


intersectam em P . Do teorema 1, como P pertence à bissetriz externa do ângulo ∠A, então
P E = P F . Além disso, P pertence à bissetriz externa do ângulo ∠B, então P F = P D.
Como P D = P E, pelo teorema 1, concluı́mos que P pertence à bissetriz do ângulo ∠C.
Dessa forma, se P equidista dos três lados do triângulo ABC e é um ponto no exterior do
triânglo então P é o centro de uma das três circunferências ex - inscritas do trângulo ABC.
A circunferência com centro Ia e raio ra é uma das três circunferências ex - inscritas que
representaremos apenas por (Ia , ra ). Analogamente são definidas as circunferências (Ib , rb )
e (Ic , rc ). Os pontos Ia , Ib e Ic são os ex - incentros. Cada circunferência ex - inscrita
toca um dos lados do triâgulo internamente e os outros dois externamente, ou seja, toca no
prolongamento. Na figura a seguir, observe que pela propriedade de segmentos tangentes a
uma circunferência, vulgarmente conhecido com Teorema do bico, temos que BL = BG,
além disso
BL + BG = (BC + CL) + (AG + AB)
= BC + CE + AE + AB = a + b + c = 2p.
Portanto, as tangentes traçadas por B à circunferência (Ib , rb ) tem medida p. Dessa forma
é fácil ver que
AJ = AK = BG = BL = CH = CM = p.
Além disso, CL = BL − BC = p − a. Então,

BM = BF = CL = CE = p − a,
CK = CD = AH = AF = p − b,
AG = AE = BJ = BD = p − c.

2
POT 2012 - Geometria - Nı́vel 2 - Aula 18 - Prof. Cı́cero Thiago

G b

Ib
H b
b

A
b

Ic
b

I
F b b
E
b

L
b b b b b

B
M D K
C b

J b

Ia

Teorema 3. (Bissetriz externa) A bissetriz externa AL do ângulo ∠A de um triângulo


AB
ABC divide externamente o lado oposto BC na razão , ou seja,
CA
BL AB
=
LC CA
em que L é o ponto de intersecção da bissetriz externa com o lado BC.

Demonstração.

3
POT 2012 - Geometria - Nı́vel 2 - Aula 18 - Prof. Cı́cero Thiago

A
b
β

β
R
b β
β
b b b

B C L

Seja R a intersecção da paralela à bissetriz AL traçada pelo ponto C. É fácil ver que
∠EAL = ∠CAL = ∠ACR = ∠ARC, com isso, AR = AC. Pelo teorema de Tales temos
que
AB BL
= .
AR LC
Como AR = AC, então
AB BL
= .
AC LC
Teorema 4. (Área de um triângulo em função do raio de uma circunferência ex
- inscrita.)

Sejam a, b e c as medidas dos lados BC, CA e AB do triângulo ∆ABC, respectivamente,


e sejam ra , rb e rc os raios das circunferências ex - inscritas relativas aos lados a, b e c,
respectivamente. Então, a área do triângulo ∆ABC pode ser calculada por

[∆ABC] = ra (p − a) = rb (p − b) = rc (p − c),

a+b+c
em que p = .
2
Demonstração.

4
POT 2012 - Geometria - Nı́vel 2 - Aula 18 - Prof. Cı́cero Thiago

b
F
a−x
C
ra
b

Ia
b

b a−x ra
ra
b

D
x E
b b b

A c B x

Pela propriedade dos segmentos tangentes, temos que DB = BE = x e DC = CF = a − x.


Então,

[∆ABC] = [∆AIa E] + [∆AIa F ] − 2[∆BCIa ] ⇔


(c + x) · ra (b + a − x) · ra a · ra
[∆ABC] = + −2· ⇔
2 2 2
ra ra
[∆ABC] = · (a + b + c − 2a) = · (2p − 2a) = ra (p − a).
2 2
Analogamente,

[∆ABC] = rb (p − b) = rc (p − c),

Problema 1. Sejam ABC um triângulo, M o pé da bissetriz interna do ângulo A e N o


pé da bissetriz interna do ângulo ∠B. Suponha que M N seja bissetriz do ângulo ∠AM C.
Calcule a medida do ângulo ∠A.

Solução.
É fácil ver que N é um dos ex - incentros do triângulo ABC pois é a interseção da bisse-
triz externa do ângulo ∠AM B e da bissetriz interna do ângulo ∠B. Logo, AN é bissetriz
externa do ângulo A. Portanto, ∠A = 120◦ .

5
POT 2012 - Geometria - Nı́vel 2 - Aula 18 - Prof. Cı́cero Thiago

A α
b

α α b
N

θ
β θ
b
β b b

B M C

Problema 2. (OBM) Um triângulo ABC, de lados AB = c, AC = b e BC = a, tem


perı́metro 2p. Uma circunferência tangencia o lado BC e os prolongamentos dos lados AB
e AC nos pontos P , Q e R, respectivamente. O comprimento AR é igual a:
(a) p − a (b) p − b (c) p − c (d) p (e) 2p

Solução.
A
b

b b b
B C
P R
b

Q
b

IA

6
POT 2012 - Geometria - Nı́vel 2 - Aula 18 - Prof. Cı́cero Thiago

Pelo teorema 2 é fácil ver que AR = AQ = p. Portanto, a resposta é o item (b).

Problema 3. No quadrilátero ABCD determine a medida do ângulo ∠AED.

b
C

D
b

60◦

60◦
E
b

70◦
40◦
b b

A B

Solução.
b
C

F b

60◦
D
b

60◦

60◦
E
b

40◦ 70◦ 70◦


40◦ 40◦
b b b

A B G

Na figura, ∠F DC = 60◦ e ∠GBC = 70◦ . Então, BC e DC são bissetrizes externas dos


ângulos ∠ABD e ∠ADB. Dessa forma, AC é bissetriz interna do ângulo ∠BAD. Portanto,
∠DAE = ∠BAE = 40◦ . Finalmente, ∠AED = 80◦ .

Exercı́cios propostos

1. Num triângulo ABC tem - se AB = BC, e D é um ponto sobre a base AC tal que
o raio do cı́rculo inscrito no triângulo ABD é igual ao raio do cı́rculo tangente ao
segmento DC e aos prolongamentos das retas BD e BC. Prove que o raio deste
1
cı́rculo é igual a da medida h de uma das alturas iguais do triângulo ABC.
4

2. Prove que os três segmentos determinados por um vértice e pelo ponto de tangência

7
POT 2012 - Geometria - Nı́vel 2 - Aula 18 - Prof. Cı́cero Thiago

da circunferência ex - inscrita com o lado oposto a esse vértice são concorrentes em


um ponto chamado ponto de Nagel.

3. (OBM) A medida do ângulo ∠B de um triângulo ABC é 120◦ . Sejam M um ponto so-


bre o lado AC e K um ponto sobre o prolongamento do lado AB, tais que BM é a bis-
setriz interna do ângulo ∠ABC e CK é a bissetriz externa correspondente ao ângulo
∠ACB. O segmento M K intersecta BC no ponto P . Prove que ∠AP M = 30◦ .

4. (Leningrado) Sejam AF , BG e CH as bissetrizes de um triângulo ABC que tem


ângulo ∠A medindo 120◦ . Prove que o ângulo ∠GF H mede 90◦ .

5. (Belarus) Seja O o centro do cı́rculo ex - inscrito do triângulo ABC oposto ao vértice


A. Seja M o ponto médio de AC e seja P a interseção das retas M O e BC. Prove
que se ∠BAC = 2∠ACB, então AB = BP .

6. (IMO) Dado um triângulo ABC, o ponto J é o centro da circunferência ex-inscrita


oposta ao vértice A. Esta circunferência ex-inscrita é tangente ao lado BC em M ,
e às retas AB e AC em K e L, respectivamente. As retas LM e BJ intersectam-se
em F , e as retas KM e CJ intersectam-se em G. Seja S o ponto de interseção das
retas AF e BC, e seja T o ponto de interseção das retas AG e BC. Prove que M é
o ponto médio de ST .
(A circunferência ex-inscrita de ABC oposta ao vértice A é a circunferência tangente
ao segmento BC, ao prolongamento do segmento AB no sentido de A para B e ao
prolongamento do segmento AC no sentido de A para C.)

Bibliografia
1. Tópicos de Matemática Elementar - Vol. 2
Antonio Caminha Muniz Neto

2. Geometria
Radmila Bulajich Manfrino e José Antonio Gómez Ortega

8
Geometria Básica
Bruno Holanda∗

12 de novembro de 2011

Resumo
Este trabalho representa um conjunto de notas de aulas de um curso inicial em Geometria
Euclidiana Plana para alunos do ensino fundamental. A principal tafera dos exercı́cios aqui
apresentados é a formação do rigor matemático necessário em problemas de geometria, porém
sem grandes aprofundamentos teóricos. Portanto, nos focaremos em três pontos principais:
Teorema de Pitágoras, áreas e ângulos.

1 Teorema de Pitágoras
O Teorema de Pitágoras é um dos mais antigos e usados teoremas da geometria plana. Também
é ele que forma a base da geometria analı́tica de Descartes. Apesar de toda a sua fama, muitos
estudiosos da História da Matemática afirmam que Pitágoras não foi o verdadeiro autor desse
teorema. E que, muito possı́velmente, os alunos da escola pitagórica sejam os reais autores.

Existem muitas provas, a maioria delas usa algum argumento de área. A solução a seguir é
uma das mais simples.

Figura 1: Teorema de Pitágoras

Prova. Na figura 1, temos um quadrado de lado (a + b) particionado em um quadrado de


lado c e quatro triângulos retângulos de área a·b
2 . Daı́, por uma equivalência de áreas, temos que

Outros materiais como este podem ser encontrados em http://brunolholanda.wordpress.com/

1
(a + b)2 = c2 + 2ab, ou seja:
a 2 + b2 = c 2 .
Problema 1. Prove o Teorema de Pitágoras de duas novas maneiras:
(a) (George Airy) Mostre como cortar dois quadrados em triângulos e quadriláteros e usar os
pedaços para formar um único quadrado maior.
(b) (Henry Perigal) Dados dois quadrados, mostre como cortar um deles em quatro partes iguais
que, juntas com o outro quadrado, formem um único quadrado maior.
Problema 2. Determine x nas seguintes figuras:

x x
6

6 2 5
4
8
3
4
9

x 10 10

2 13
17 x x

Problema 3. Na figura abaixo ABCD é um quadrado de área 64cm2 e EF GH um quadrado de


área 36cm2 . Determine a área do quadrado CM HN .
N

D C

H G

A B M E F
Problema 4. Na figura abaixo os dois cı́rculos são têm o mesmo centro e os cinco quadriláteros
são quadrados. Se o cı́rculo menor tem raio igual a 1cm determine o raio do cı́rculo maior.

2
Problema 5. Seja ABCD um quadrado de lado 28. Seja P um ponto no seu interior e E um
ponto no lado CD de modo que CD⊥P E e AP = BP = P E. Ache AP .
Problema 6. Suponha que ABC seja um triângulo retâgulo escaleno, e P seja o ponto na hi-
potenusa AC tal que ∠ABP = 45◦ . Dado que AP = 1 e CP = 2, calcule a área do triângulo
ABC.
Problema 7. Na figura abaixo temos dois cı́rculos de raios 3 e 2 e cuja distância centro a centro
é 10. Ache o comprimento da tangente comum P Q.

Q
P

Problema 8. (Cone Sul 1989 - adaptado) Na figura abaixo temos dois quadrados, um de lado
dois e outro de lado um. Determine o raio do cı́rculo que é tangente aos lados do maior e passa
pelo vértice do menor.

Problema 9. Nos lados AB e DC do retângulo ABCD, os pontos F e E são escolhidos de modo


que AF CE seja um losango. Se AB = 16 e BC = 12, ache EF .
F
A B

D C
E
Problema 10. P é um ponto no interior do retângulo ABCD. Se P A = 2; P B = 3, e P C = 10.
Ache P D.
Problema 11. (Maio 2006) Um retângulo de papel 3cm × 9cm é dobrado ao longo de uma reta,
fazendo coincidir dois vértices opostos. Deste modo se forma um pentágono. Calcular sua área.

Solução. Seja ABCD o retângulo e ABEF D′ o pentágono formado ao dobrar o paple como é
mostrado na figura a seguir:

3
b
D′

A E D
b b b

b b b

B F C

2 Trabalhando com Ângulos


Ao lado da “distância”, o “ângulo” é uma unidade de medida fundamental para o estudo da geo-
metria plana. Explicando de uma maneira formal, o ângulo mede a diferença entre as inclinações
de duas retas.

Problema 12. (Torneio das Cidades 1994) No triângulo ABC, retângulo em C, os pontos M e
N são escolhidos sobre a hipotenusa de modo que BN = BC e AM = AC. Ache a medida do
ângulo ∠N CM .

Problema 13. Seja ABCDEF um hexágono com todos os ângulos internos iguais a 120◦ . Mostre
que
AB − DE = CD − F A = EF − BC
Solução.

Sejam AF ∩ BC = P, ED ∩ AF = Q, BC ∩ ED = R. Como todos os ângulos internos são


120◦ obtemos que todos os triângulos △P AB, △QEF, △RCD, △P QR são equiláteros. Como
P Q = P R temos: AB + AF + F E = P A + AF + F Q = P B + BC + CR = AB + BC + CD,
então CD − AF = F E − BC. A outra igualdade é análoga.

Problema 14. O octógono ABCDEF GH é equiangular. Sabendo que AB = 1, BC = 2, CD = 3,


DE = 4, e EF = F G = 2, calcule o perimetro do octógono.

Problema 15. Seja ABCDEF um hexágono com todos os ângulos internos iguais a 120◦ . Mostre
que
AB − DE = CD − F A = EF − BC

4
Problema 16. Seja RST U V pentagono regular. Construa um triangulo eqüilátero P RS com P
no interior do pentagono. Ache a medida do ângulo ∠P T V .

Problema 17. Ache a soma dos ângulos internos de uma estrela

Problema 18. No triângulo isósceles ABC com AB = AC, P é o ponto médio do lado AB tal
que AP = P C. Se a bissetriz do ângulo ∠ABC corta P C em O de modo que P O = BO, ache
os ângulo do triângulo.

Problema 19. No trapésio ABCD, de bases AB e CD, temos AD = 39, CD = 14, ∠ABC = 69◦
e ∠CDA = 138◦ . Ache a medida de AB.

Problema 20. (OBM) No retângulo ABCD, E é o ponto médio do lado BC e F é o ponto médio
b mede 20◦ . Quanto vale o ângulo
do lado CD. A interseção de DE com F B é G. O ângulo E AF
b
E GB?
F
D C
G
E

A B
Problema 21. DEF G é um quadrado no exterior do pentágono regular ABCDE. Quanto mede
b ?
o ângulo E AF

Problema 22. No triângulo ABC, D e E são pontos sobre os lados BC e AC respectivamente.


Determine ∠CDE sabendo que AB = AC, AE = AD e ∠BAD = 48◦ .

Problema 23. Determine B ÂC na figura abaixo sabendo que AB = AC e BC = CD = DE =


EF = F A.
A

F
E
D
C B

5
Problema 24. No triângulo ABC com AB = BC, ∠ABC = 144◦ . Seja K um ponto em AB, L
um ponto em BC e M em AC de modo que KL k AC, KM k BC e KL = KM . A reta LM
corta o prolongamento de AB em P . Ache a medida do ângulo ∠BP L.
Problema 25. No triângulo ABC com AB = BC, P , Q e R são pontos nos lados AC, BC e AB,
respectivamente tais que P Q k AB, RP k BC e RB = AP . Se ∠AQB = 105◦ , ache as medidas
dos ângulos do △ABC.
Problema 26. BE e AD são as alturas do triângulo ABC, H é o ortocentro e F , G, K são os
pontos médios dos segmentos AH, AB, BC, respectivamente. Prove que ∠F GK é reto.
Problema 27. Em um triângulo ABC temos que B̂ = 37◦ e Ĉ = 38◦ . Sejam P e Q pontos sobre
o lado BC tais que ∠BAP = ∠P AQ = ∠QAC. Se traça por B uma paralela à AP e por C uma
paralela à AQ. O ponto de encontro destas duas retas é D. Calcule ∠DBC.
Problema 28. Em um romboide ABCD (AB = BC e CD = DA) as diagonais se cortam em
um ponto F . Sobre o prolongamento do lado BC se marca um ponto E de modo que CF = CE
e F CED também seja um romboide. Se ∠ABC = 122◦ , quanto mede ∠ADE?
Problema 29. (Maio 1996) Seja ABCD um quadrado e F um ponto qualquer do lado BC.
Traça-se por B a perpendicular à reta DF que corta a reta DC em Q. Quanto mede o ângulo
∠F QC?

3 Áreas
Problema 30. (OBM 2006) ABC é um triângulo retângulo e M e N são pontos que trisectam
a hipotenuza BC. Sejam X e Y os simétricos de N e M em relação ao ponto A. Determine a
área do quadrilátro XY CB, sabendo que o triângulo ABC tem área 1 cm2 .

Solução.
C

A B

Y
Observe que △AXY ≡ △AN M e ∠Y XA = ∠AM N . Assim, XY k M N e como XY = M N =
M C = N B, segue que os quadriláteros XY CM e XY N B são paralelogramos, como A é ponto
médio de XM e N Y temos que [AY C] = [BAX] = 32 . Logo, [XY CB] = 83 .

6
Problema 31. Na figuras abaixo ABC é um triângulo de área 72cm2 e M, N, P são pontos
médios. Determine a área da região sombreada.
A

N
P

B C
M
Problema 32. Na figura abaixo D, E, F, G são pontos médios. Determine a área que está faltando.

D
210 E
250

G 240
F

Problema 33. Na próxima figura ABCD é um quadrilátero de área 200cm2 e D, E, F, G são


pontos médios. Determine a área sombreada.
B
E
H
A
C
G
F
D
Problema 34. Na figura abaixo ABCD é um quadrado de lado 6cm e EF é um segmento paralelo
ao lado AD. Sabendo que a área sombreada é um terço da área do quadrado determine a medida
do segmento EF .
B C

F
E
A D
Problema 35. No trapezio ABCD, AD k BC. ∠A = ∠D = 45◦ , enquanto ∠B = ∠C = 135◦ .
Se AB = 6 e a área de ABCD é 30, ache BC.
Problema 36. Na figura abaixo ABCD é um quadrado de lado 4cm e O é o seu centro. Determine
a área marcada sabendo que o ângulo EOF é reto.

7
B C
E
O

A D
F
Problema 37. Na figura abaixo ABCD é um retângulo de área 11cm2 . Sabemos também que
A′ A = AD, BB ′ = BA, CC ′ = CB e DD′ = DC. Determine a área do quadrilátero A′ B ′ C ′ D′
B’

B C C’

A D
A’

D’
Problema 38. Na figura abaixo DEF G é um quadrado de lado 4cm e ABCD um retângulo cujos
lados têm medidas 1cm e 4cm. O encontro da reta AC com a reta F G é o ponto H. Determine
a área marcada.
E F

H
B C
G
A D
Problema 39. O quadrado ABCD abaixo tem lado 10cm. Sabe-se que P C = QD e que a área
do triângulo ABP é 37 da área do triângulo P CQ. Calcule o perı́metro do quadrilátero AP QD.
Q
D C
P

A B
Problema 40. Um quadrado de lado 5 é dividido em cinco partes de áreas iguais usando cortes
paralelos às suas diagonais. Ache o perı́metro do pentágono BEF GH.
I
D C
J F
G
E
A B
H
Problema 41. (Teste Rioplatense 2005) Paladino dividiu uma folha de papel quadrada, com 20
cm de lado, em 5 pedaços de mesma área. O primeiro corte teve inı́cio no centro do quadrado e
prolongou-se até a fronteira do papel a 7 cm de um canto, como indicado na figura seguinte.

8
D C

A B
Sabendo que o João fez todos os cortes em linha recta a partir do centro do quadrado, de que
forma cortou o papel?

Problema 42. Na figuras abaixo ABC é um triângulo de área 72cm2 e M, N, P, Q são pontos
médios. Determine a área da região sombreada.
A

N
P
Q
B C
M
Problema 43. Sejam ABCD um quadrado de lado 12cm, E o ponto médio de DA e F o ponto
médio de BC. Traçamos os segmentos EF , AC e BE, que dividem o quadrado em seis regiões.
Calcular a área de cada uma dessas regiões.

Problema 44. Seja ABCD um retângulo com área 1, e E um ponto sobre CD. Qual é a área
do triângulo formado pelos baricentros dos triângulos ABE, BCE, e ADE?

Problema 45. Em um parapelogramo ABCD de área igual a 1, seja E o ponto médio do lado
DC, K o ponto de encontro das diagonais BD e AC e L o ponto de encontro de BD com AE.
Ache a área do quadrilátero ELKC.

Problema 46. No triângulo ABC sabe-se que Ĉ = 90◦ , AC = 20 e AB = 101. Seja D o ponto
médio de BC. Ache a área do triângulo ADB.

Problema 47. Seja ABCDEF um hexágono regular de área 1cm2 . Determine a área do triângulo
ABC.

Problema 48. Suponha que ABCDE seja um pentágono convexo (não necessariamente regular)
tal que as áreas dos triângulos ABC, BCD, CDE, DEA e EAB saõ iguais a 1. Qual a área do
pentágono?

Problema 49. No triângulo ABC, D é o ponto médio de BC, E o ponto médio de AD, F o
ponto médio de BE e G o ponto médio de F C. Calcule a relação entre as áreas dos triângulo
ABC e EF G.

Problema 50. (Maio 1996) Um terreno (ABCD) tem forma de trapézio retangular. O ângulo
em  mede 90◦ e o ângulo em D̂ mede 90◦ . AB mede 30m; AD mede 20m e DC mede 45m.
Este terreno tem que ser dividido em dois terrenos de área iguais traçando uma paralela ao lado
AD. A que distância de D deve-se traçar a paralela?

9
Problema 51. Na figura abaixo ABCD e DEF G são paralelogramos. Além disso, F , C e G são
colineares. Prove que ambos têm a mesma área.
G
A B

F
D C

E
Problema 52. (Maio 2006) Seja ABCD um trapézio de bases AB e CD. Seja O o ponto de
interseção de suas diagonais AC e BD. Se a área do triângulo ABC é 150 e a área do triângulo
ACD é 120, calcular a área do triângulo BOC.

Problema 53. (Maio 2006) Um retângulo de papel 3cm × 9cm é dobrado ao longo de uma reta,
fazendo coincidir dois vértices opostos. Deste modo se forma um pentágono. Calcular sua área.

Problema 54. (Torneio das Cidades 1981) O quadrilátero convexo ABCD está inscrito em um
cı́rculo de centro O e possui suas diagonais perpendiculares. Prove que a linha quebrada AOC
divide o quadrilátero em duas regiões de mesma área.

Problema 55. (Banco IMO) Sejam ABCD um quadrilátero convexo e M e N os pontos médios
dos lados BC e DA, respectivamente. Prove que [DF A] + [CN B] = [ABCD].

Este material faz parte de um conjunto de notas de aulas voltadas para o treinamento de alunos
para competições de matemática. É permitida a cópia apenas no caso de uso pessoal.
Pode conter falhas.

10
Polos Olímpicos de Treinamento
Curso de Geometria - Nível 2 Aula 0
Prof. Armando Barbosa

Respostas e Soluções

1 Teorema de Pitágoras
Problema 1. (a) Consideremos dois quadrados ABCD (de lado a) e DEF G (de lado b),
de forma que o ponto D fica entre os pontos A e G. Seja H o ponto pertencente ao
segmento AG tal que GH tem comprimento a. Conectemos os segmentos BH e GH.
Façamos os recortes e colagens a seguir, sem rotacionar as figuras:

• Recortemos o triângulo AHB e colemos ele no lado EF de forma que os lados


AH e EF coincidam;
• Recortemos o triângulo HGF e colemos ele no lado BC de forma que os lados
HG e BC coincidam.

Notemos que a figura obtida é um novo quadrado cujo lado é a hipotenusa de um


triângulo retângulo cujos lados são a e b. Daı́, como a área inicial dos dois quadrados
(a2 + b2 ) é igual à área final, fica demonstrado o teorema de Pitágoras.

(b) Em primeiro lugar, lembremos que o ponto de encontro das diagonais de um quadrado
é o centro do mesmo.
Voltando à demonstração, consideremos um triângulo retângulo com catetos a e b e
hipotenusa c. Considere, sem perda de generalidade, que a > b.
Desenhemos um quadrado de lado a, fazendo um dos lados desse quadrado ser o cateto
a, externamente ao triângulo retângulo.
Façamos, também, um quadrado de lado b externamente ao cateto b de forma análoga.
Em cada um desses quadrados, tracemos, pelo centro (ponto de encontro das diagonais),
uma reta paralela e uma reta perpendicular a hipotenusa.
Para facilitar o entendimento, demos nomes aos pontos. Consideremos:

• o triângulo retângulo ABC, retângulo em C, de forma que os catetos a e b são


BC e AC, respectivamente;
• o quadrado externo BCDE de centro O;
• as retas F G e HI, sendo as retas paralela e perpendicular a AB passando por O,
respectivamente.
1 TEOREMA DE PITÁGORAS

Seja x tal que


x = BI = CG = DH = EF
Como ABF G é paralelogramo, então temos que:

a−x=b+x

Além disso, podemos concluir que AB = F G = HI.


Desenhemos o quadrado ABKJ externo à hipotenusa.
Recortemos os quadriláteros BF OI, CIOG, DGOH e EHOF e colemos eles dentro
do quadrado ABKJ fazendo O coincidir com cada um dos vértices A, B, K e J. Ob-
servemos que essa colagem interna é possı́vel sem interseção entre esses quadriláteros,
já que OF = OG = OH = OI = c/2.
Notemos que a figura que sobra no quadrado ABKJ após essa colagem, possui 4 ângulos
retos e cada lado tem tamanho igual a:

(a − x) − x = a − 2x = b

Portanto, a figura que sobra é um quadrado de lado b.


Analisando o quadrado ABKJ, segue a demonstração do teorema de Pitágoras, pois tal
quadrado de área c2 foi dividido em um quadrado de lado b2 mais quatro quadriláteros
cuja soma das áreas é igual a a2 .

Problema 2. (a) Analisando o lado em comum dos dois triângulos, temos que:

x2 − 62 = 82 + 42

x = 2 29

(b) Indo da esquerda para a direita, temos que:


  √ 2
6 2 + 4 2 + 3 2 = x2 − 2 5
61 = x2 − 20
x = 9

(c) Traçando a altura a partir da extremidade da base menor, temos que:

x2 + (17 − 9)2 = 102


x = 10

(d) Olhando o cateto em comum dos dois triângulos, temos que:


 √ 2
102 − (2x)2 = 2 13 − x2
48 = 3x2
x = 4

POT 2014 - Geometria - Nı́vel 2 - Aula 0 - Soluções 2


1 TEOREMA DE PITÁGORAS

Problema 3. Marcando ângulos, podemos concluir que:

∠BCM = 90◦ − ∠BM C


= 90◦ − (180◦ − 90◦ − ∠HM E)
∠BCM = ∠HM E

Como ∠CBM = ∠M EH = 90◦ e CM = M H = lado do quadrado CM HN , então, por


ângulo-lado-ângulo oposto, temos que: 4CBM ≡ 4M EH.
Sejam x, y, z os lados dos quadrados ABCD, EF GH, CM HN , respectivamente. Daı́,
podemos concluir que: x2 = 64cm2 e y 2 = 36cm2 .
Aplicando o teorema de Pitágoras no 4CBM , temos que:
2
z 2 = x2 + BM
= x2 + y 2
= 64cm2 + 36cm2
z 2 = 100cm2

Portanto, a área do quadrado CM HN é igual a 100cm2 .

Problema 4. Notemos que, se o cı́rculo do centro tem raio igual a 1cm, então o lado de
cada um dos quadrados tem lado igual a 2cm.
Liguemos o centro dos cı́rculos:

• para o vértice de um quadrado que pertença também a circunferência maior (pode


ser, por exemplo, o quadrado mais à esquerda);

• ao ponto médio do lado que possui o vértice do passo anterior, de forma que esse
segmento passe pelo ponto que pertence a circunferência menor e ao quadrado, ao
mesmo tempo.

Daı́, notemos que foi gerado um triângulo retângulo cujos:

• os catetos são:

? 1cm (metade do lado do quadrado maior);


? 1cm + 2cm = 3cm (raio da circunferência menor + lado do quadrado).

• a hipotenusa é R (raio da circunferência maior).

Portanto, pelo teorema de Pitágoras, temos que:

R2 = (1cm)2 + (3cm)2
= 10cm2

R = 10cm

POT 2014 - Geometria - Nı́vel 2 - Aula 0 - Soluções 3


1 TEOREMA DE PITÁGORAS

Problema 5. Seja F um ponto em AB, tal que P F ⊥ AB. Traçando o segmento P F ,


notemos que AP = BP implica em 4AP F ≡ 4BP F , pois ambos são retângulos e possuem
hipotenusa e um dos catetos iguais, o que implica, pelo teorema de Pitágoras, que o outro
cateto também é igual.
Como consequência, podemos concluir que F é ponto médio de AB. Portanto:
AF = BF = 14
Percebamos, então, que como CD ⊥ P E, então EF k BC. Com isso, podemos concluir
que E é ponto médio de CD.
Seja x tal que AP = BP = P E = x. Daı́, temos que
P F = 28 − x
.
Aplicando o teorema de Pitágoras no 4AP F , podemos conclur que:
2 2 2
AP = P F + AF
x2 = (28 − x)2 + 142
x2 = 282 − 2 · 28 · x + x2 + 142
56 · x = 784 + 196
35
x =
2
35
Portanto, AP = x = .
2
Problema 6. Notemos que BP é bissetriz, ou seja, que ∠ABP = ∠CBP = 45◦ .
Considere os catetos AB = x e BC = y. Façamos duas soluções:

Solução usando teorema da bissetriz interna


Pelo teorema da bissetriz interna, temos que:
AB CB
=
AP CP
x y
=
1 2
y = 2x
Pelo teorema de Pitágoras no triângulo ABC, podemos concluir que:
x2 + y 2 = (1 + 2)2
x2 + 4x2 = 9
9
x2 =
5
Portanto, temos que área do triângulo ABC é igual a:
x·y x·2·x 9
= = x2 =
2 2 5

POT 2014 - Geometria - Nı́vel 2 - Aula 0 - Soluções 4


1 TEOREMA DE PITÁGORAS

Solução usando apenas teorema de Tales


Seja Q o ponto pertencente a BC tal que P Q k AB. Seja P Q = z. Note que o triângulo
BP Q é isósceles, pois ∠P BQ = ∠BP Q = 45◦ .
Pelo teorema de Tales, podemos concluir que:
x 3
=
z 2
2x
z =
3
Pelo teorema de Tales, também temos que:
y 3
=
y−z 2
2y = 3y − 3z
y = 3z
Dos dois últimos resultados encontrados, podemos concluir que: y = 2x.
Pelo teorema de Pitágoras no triângulo ABC, podemos concluir que:
x2 + y 2 = (1 + 2)2
x2 + 4x2 = 9
9
x2 =
5
Portanto, temos que área do triângulo ABC é igual a:
x·y x·2·x 9
= = x2 =
2 2 5
.

Problema 7. Consideremos os pontos:


• O1 e O2 os centros da maior e menor circunferências, respectivamente;
• R o ponto do segmento O1 P , tal que RO2 k P Q.
Liguemos os segmentos O1 O2 , O1 P e O2 Q.
Notemos que P QO2 R é retângulo. Com isso podemos concluir que: RO2 = P Q e P R = 2.
Daı́, temos que:
RO1 = 3 − 2 = 1
Aplicanto teorema de Pitágoras no triângulo RO1 O2 , podemos concluir que:
2 2 2
RO2 + RO1 = O1 O2
2
P Q + 12 = 102

PQ = 99

POT 2014 - Geometria - Nı́vel 2 - Aula 0 - Soluções 5


1 TEOREMA DE PITÁGORAS

Problema 8. Sejam:
• r o raio e O o centro do cı́rculo da figura;
• ABCD o quadrado de lado 1, mais ao alto e à direita, com C sendo também o centro
do quadrado de lado 2;
• M e N os pontos de tangência da circunferência com os lados do quadrado.
Como AN = AM e ∠N AM = 90◦ , segue que AM ON é um quadrado e, pelo teorema de
Pitágoras no triângulo OM A temos que:
2 2 2
AO = OM + M A
p
AO = r2 + r2

AO = r 2

Aplicando novamente o teorema de Pitágoras, podemos concluir que:



AC = 2

Além disso, como ∠OAB = ∠CAB , então temos que C, O e A são colineares. Com isso,
podemos concluir que:

AC = AO + OC
√ √
2 = r· 2+r
√ √
2 2−1
r = √ ·√
2+1 2−1

r = 2− 2

Problema 9. Seja x tal que AF = F C = CE = EA = x. Daı́, temos que: BF = 16 − x.


Aplicando teorema de Pitágoras no triângulo F BC, podemos concluir que:

(16 − x)2 + 122 = x2


162 − 2 · 16 · x + x2 + 122 = x2
256 + 144 = 32 · x
25
x =
2
Seja G o pé da altura do ponto F no triângulo EF C. Então, temos que:

F G = BC = 12
25 7
Além disso, podemos concluir que: GC = BF = 16 − = . Daı́, temos que:
2 2
EG = EC − GC
25 7
= −
2 2
EG = 9

POT 2014 - Geometria - Nı́vel 2 - Aula 0 - Soluções 6


1 TEOREMA DE PITÁGORAS

Aplicando teorema de Pitágoras no triângulo EF G, podemos concluir que:


2 2 2
EF = F G + EG
2
EF = 122 + 92
EF = 15

Problema 10. Tracemos perpendiculares do ponto P para cada um dos lados do retângulo.
Sejam P E = x, P F = y, P G = z e P H = w essas perpendiculares de forma que E, F , G
e H estão nos segmentos AB, BC, CD e DA, respectivamente.
Aplicando teorema de Pitágoras em alguns triângulos, temos que:
1. 4AEP : x2 + w2 = 22 = 4;

2. 4BF P : x2 + y 2 = 32 = 9;

3. 4CGP : y 2 + z 2 = 102 = 100;


2
4. 4DHP : z 2 + w2 = P D .
Dos itens 1 e 3, concluı́mos que:

x2 + y 2 + z 2 + w2 = 104

Dos itens 2 e 4, concluı́mos que:


2
9 + PD = 104

P D = 95

Problema 11. Seja x tal que CF = x. Daı́, temos que: F A = x e BF = 9 − x. Lembrando


que AB = 3, podemos aplicar teorema de Pitágoras no 4ABF para concluir que:

x2 = (9 − x)2 + 32
x2 = 92 − 2 · 9 · x + x2 + 32
18 · x = 90
x = 5

Seja y tal que ED = y. Daı́, temos que: D0 E = y e AE = 9 − y. Lembrando que


D0 A = DC = 3, podemos aplicar teorema de Pitágoras no 4AD0 E para concluir que:

(9 − y)2 = 32 + y 2
92 − 2 · 9 · y + y 2 = 9 + y 2
72 = 18 · y
y = 4

Podemos calcular a área do pentágono ABEF D0 somando a área do trapézio BF EA mais


a área do triângulo EAD0 . Calculemos cada uma dessas áreas:

POT 2014 - Geometria - Nı́vel 2 - Aula 0 - Soluções 7


2 TRABALHANDO COM ÂNGULOS

• área do trapézio BF EA:

(4 + 5) · 3 27
[BF EA] = =
2 2

• área do triângulo EAD0 :


3·4
[EAD0 ] = =6
2
Portanto, a área do pentágono ABEF D0 é igual a:
27 39
+6=
2 2

2 Trabalhando com ângulos


Problema 12. Seja x tal que ∠CAM = x. Como o triângulo CAM é isósceles em A,
portanto podemos concluir que:

∠CM A = ∠CM N = x

Seja y tal que ∠CBN = y. Como o triângulo CBN é isósceles em B, portanto podemos
concluir que:
∠CN B = ∠CN M = y
Analisando o triângulo CN M , temos que:

∠N CM = 180◦ − ∠CM N − ∠CN M = 180◦ − (x + y)

Como o triângulo ACB é retângulo em C, temos que:

x + y = 180◦ − 90◦ = 90◦

Usando os dois últimos resultados encontrados, podemos concluir que:

∠N CM = 180◦ − 90◦
∠N CM = 90◦

Problema 13. Resolvido no próprio material.

Problema 14. Sejam x, y tais que GH = x e HA = y. Considere os seguintes pontos:

• M o ponto de interseção dos prolongamentos de AB e CD;

• N o ponto de interseção dos prolongamentos de CD e EF ;

• O o ponto de interseção dos prolongamentos de EF e GH;

• P o ponto de interseção dos prolongamentos de GH e AB.

POT 2014 - Geometria - Nı́vel 2 - Aula 0 - Soluções 8


2 TRABALHANDO COM ÂNGULOS

Como o octógono ABCDEF GH é equiangular, podemos concluir que cada ângulo interno
é igual a:
180◦ · (8 − 2)
= 135◦
8
Note que o 4BCM é retângulo e isósceles, pois:

∠CBM = ∠BCM = 180◦ − 135◦ = 45◦


∠BM C = 90◦

Lembrando que o lado BC = 2, então podemos aplicar o teorema de Pitágoras no 4BCM


para concluir que: √
BM = CM = 2
De forma análoga, temos que:

• 4DEN é retângulo e isósceles implica em: DN = EN = 2 · 2;

• 4F GO é retângulo e isósceles implica em: F O = GO = 2;

y· 2
• 4AHP é retângulo e isósceles implica em: HP = AP = .
2
Note que o quadrilátero M N OP é retângulo, pois possui quatro ângulos retos. Daı́, podemos
calcular y e x:

• valor de y:

√ MP = NO
y· 2 √ √ √
+1+ 2 = 2+2+2· 2
2 √
y = 4+ 2

• valor de x:

OP
√ = MN
√ y· 2 √ √
2+x+ = 2+3+2· 2
2
x = 2

Daı́, temos que o perı́metro p do octógono ABCDEF GH é igual a:

p = 1+2+3+4+2+2+x+y
 √ 
= 14 + 2 + 4 + 2

p = 20 + 2

Problema 15. Solucionado na questão 13 do próprio material.

POT 2014 - Geometria - Nı́vel 2 - Aula 0 - Soluções 9


2 TRABALHANDO COM ÂNGULOS

Problema 16. Além do triângulo P RS, tracemos os segmentos P V e P T .


Como o pentágono RST U V é regular, então temos que cada ângulo interno desse pentágono
é igual a:
180◦ · (5 − 2)
= 108◦
8
Como o triângulo P RS é equilátero, então temos que cada ângulo interno desse triângulo
é igual a:
180◦
= 60◦
3
Notemos que

∠V RP = ∠V RS − ∠P RS = 108◦ − 60◦
∠V RP = 48◦

Sabendo que o triângulo P RV é isósceles em R, pois RS = RP = RV , então, analisando


o 4P RV , temos que:
180◦ − 48◦
∠V P R = ∠P V R =
2

∠V P R = 66

Analogamente, podemos concluir que: ∠SP T = 66◦ .


Analisando o ponto P , temos que:

∠V P T = 360◦ − ∠V P R − ∠RP S − ∠SP T


= 360◦ − 192◦
∠V P T = 168◦

Note que 4V RP ≡ 4T SP pelo caso lado-ângulo-lado, pois:

• RV = RP = SP = ST ;

• ∠V RP = ∠T SP = 48◦

Com isso, podemos concluir que V P = T P e, consequentemente, que 4V P T é isósceles


em P .
Das nossas últimas duas conclusões, podemos concluir que:
180◦ − 168◦
∠P V T = ∠P T V =
2
∠P T V = 6◦

Problema 17. Considere os pontos da estrela A, B, C, D e E e os respectivos ângulos


internos: a, b, c, d e e.
Considere os pontos do pentágono interno P , Q, R, S e T de tal forma que:

• A, P , Q e C são colineares;

POT 2014 - Geometria - Nı́vel 2 - Aula 0 - Soluções 10


2 TRABALHANDO COM ÂNGULOS

• B, Q, R e D são colineares;

• C, R, S e E são colineares;

• D, S, T e A são colineares;

• E, T , P e B são colineares.

Analisando o triângulo ASC, temos que:

∠ASC = ∠RST = 180 − a − c

De forma análoga, podemos analisar alguns outros triângulos e encontrar valores de ângulos:

• 4BT D: ∠BT D = ∠ST P = 180◦ − b − d;

• 4CP E: ∠CP E = ∠T P Q = 180◦ − c − e;

• 4DQA: ∠DQA = ∠P QR = 180◦ − d − a;

• 4ERB: ∠ERB = ∠QRS = 180◦ − e − b.

Analisando a soma dos ângulos internos do pentágono P QRST , temos que:

180◦ · (5 − 2) = ∠RST + ∠ST P + ∠T P Q + ∠P QR + ∠QRS


540◦ = 900◦ − 2 · (a + b + c + d + e)
a + b + c + d + e = 180◦

Problema 18. Obs.: O enunciado está errado, pois, com as informações dele, podemos
concluir que os ângulos C e B são iguais a 90◦ .
Retiremos, então, a informação de que o triângulo ABC é isósceles e mantemos o restante
do enunciado.
Seja x e y tais que ∠BAC = x e ∠ABC = y. Analisando alguns triângulos, temos que:

• 4AP C é isósceles em P implica em ∠P AC = ∠P CA = x;

• 4BP C é isósceles em P implica em ∠P BC = ∠P CB = y;

• 4ABC: soma dos ângulos internos:

x + x + y + y = 180◦
x + y = ∠ACB = 90◦

Como a bissetriz do ∠ABC corta P C em O de forma que P O = BO, podemos concluir


que:

• BO é bissetriz de ∠P BC implica em:


y
∠P BO = ∠OBC =
2

POT 2014 - Geometria - Nı́vel 2 - Aula 0 - Soluções 11


2 TRABALHANDO COM ÂNGULOS

• 4P BO é isósceles em O implica em:


y
∠BP O = ∠P BO =
2

• 4P BC: soma dos ângulos internos:


y
+ y + y = 180◦
2
y = ∠ABC = 72◦

Por último, temos que:

x + y = 90◦
x = ∠BAC = 18◦

Problema 19. Seja E o ponto de encontro da bissetriz do ângulo ∠ADC com o segmento
AB.
Daı́, temos que 4DAE é isósceles em A, pois:
138◦
• ∠ADE = ∠EDC = = 69◦ ;
2
• ∠AED = 69◦ , uma vez que AE k DC.

Consequentemente, podemos concluir que:

AD = AE = 39

.
Além disso, temos que o quadrilátero CDEB é paralelogramo, pois EB k DC e ∠ABC =
∠AED = 69◦ implica em BC k ED.
Com isso, podemos concluir que:

DC = EB = 14

.
Usando as duas últimas conclusões, temos que:

AB = AE + EB
AB = 53

Problema 20. Seja x tal que ∠BAE = x.


Como E é ponto médio de BC, então podemos concluir que 4ABE ≡ 4DCE pelo caso
lado-ângulo-lado, pois:

• AB = DC;

• ∠ABE = ∠DCE = 90◦ ;

POT 2014 - Geometria - Nı́vel 2 - Aula 0 - Soluções 12


2 TRABALHANDO COM ÂNGULOS

• BE = EC

Daı́, temos que: ∠BAE = ∠CDE = x.


Lembrando que F é ponto médio do lado CD, podemos aplicar teorema de Pitágoras nos
triângulos ADF e BCF para concluir que: AF = F B. Com isso, temos que o triângulo
AF B é isósceles em F e, consequentemente, como ∠EAF = 20◦ , então temos que:

∠F AB = ∠F BA = 20◦ + x

Como AB k CF , então podemos concluir que: ∠F BA = ∠BF C = 20◦ + x. Dai, temos


que:
∠GF D = 180◦ − (20◦ + x) = 160◦ − x
Analisando a soma dos ângulos internos do triângulo DF G, podemos concluir que:

∠DGF + (160◦ − x) + x = 180◦


∠DGF = 20◦

Observando dois ângulos opostos pelo vértice no ponto G, temos que:

∠DGF = ∠EGB = 20◦

Problema 21. Como DEF G é quadrado, então podemos concluir que: ∠DEF = 90◦ .
Como o pentágono ABCDE é regular, então temos que:

180◦ · (5 − 2)
∠DEA = = 108◦
5
Analisando a soma dos ângulos ao redor do ponto E, temos que:

∠AEF = 360◦ − 108◦ − 90◦ = 162◦

Além disso, notemos que o triângulo AEF é isósceles em E, pois AE = ED = EF .


Com isso, podemos concluir que:
180◦ − 162◦
∠EAF = ∠EF A =
2
∠EAF = 9◦

Problema 22. Sejam x, y tais que ∠CDE = x e ∠DAE = y.


Como AE = AD, então temos que 4ADE é isósceles em A. Portanto, como ∠DAE = y,
podemos concluir que:
180◦ − y y
∠AED = ∠ADE = = 90◦ −
2 2

POT 2014 - Geometria - Nı́vel 2 - Aula 0 - Soluções 13


2 TRABALHANDO COM ÂNGULOS

Como AB = AC, então temos que 4ABC é isósceles em A. Portanto, como ∠CAB =
y + 48◦ , podemos concluir que:

180◦ − (y + 48◦ ) y
∠ABC = ∠ACB = = 66◦ −
2 2
Somando os ângulos internos do 4CDA temos que:
y y
y + 66◦ − + x + 90◦ − = 180◦
2 2
x = ∠CDE = 24◦

Problema 23. Seja x tal que ∠BAC = x.


Analisando alguns triângulos isósceles e alguns ângulos suplementares (“ângulos que somam
180◦ ), podemos calcular alguns ângulos:

• 4F AE é isósceles em F implica em:

? ∠F AE = ∠F EA = x;
? ∠AF E = 180◦ − 2x e ∠EF D = 2x.

• 4EF D é isósceles em E implica em:

? ∠EF D = ∠EDF = 2x;


? ∠F ED = 180◦ − 4x e ∠DEC = 3x.

• 4DEC é isósceles em D implica em:

? ∠DEC = ∠DCE = 3x;


? ∠EDC = 180◦ − 6x e ∠CDB = 4x.

• 4CDB é isósceles em C implica em:

? ∠CDB = ∠CBD = 4x.

• 4ABC é isósceles em A implica em:

? ∠ABC = ∠ACB = 4x.

Somando os ângulos internos do 4ABC temos que:

x + 4x + 4x = 180◦
9x = 180◦
x = ∠BAC = 20◦

Problema 24. Como o triângulo ABC é isósceles em B, então temos que:


180◦ − 144◦
∠BAC = ∠BCA = = 18◦
2

POT 2014 - Geometria - Nı́vel 2 - Aula 0 - Soluções 14


2 TRABALHANDO COM ÂNGULOS

Como KL k AC, então podemos concluir que: ∠BKL = ∠BAC = 18◦ e ∠BLK =
∠BCA = 18◦ .
Sabendo que KM k BC, ou seja, KM k BL e ∠BLK = 18◦ , então temos que: ∠LKM =
∠BLK = 18◦ .
Como o triângulo KLM é isósceles em K, então podemos concluir que:
180◦ − 18◦
∠KLM = ∠KM L = = 81◦
2
Somando os ângulos internos do 4KP M , temos que:

∠KP M + ∠P M K + ∠M KP = 180◦
∠BP L = 180◦ − 81◦ − 36◦
∠BP L = 63◦

Problema 25. Seja x tal que ∠BAC = x. Como o triângulo ABC é isósceles em B, então
temos que: ∠BCA = x e ∠ABC = 180◦ − 2x.
Como P Q k AB, então podemos concluir que: ∠P QC = ∠ABC = 180◦ − 2x e, consequen-
temente, ∠P QB = 2x. Sabendo que ∠AQB = 105◦ , então temos que:

∠AQP = 2x − 105◦

Também, a relação P Q k AB implica em: ∠BAQ = ∠AQP = 2x − 105◦ . Como ∠BAC =


x, então podemos concluir que:

∠QAP = x − (2x − 105◦ ) = 105◦ − x

Notemos que 4ABC ∼ 4P QC pois P Q k AB. Daı́, seja m tal que P Q = QC = m.


Percebamos que o quadrilátero BQP R é paralelogramo. Portanto, temos que: RB = P Q =
m. Pelo enunciado, temos que RB = AP = m.
Com isso, podemos concluir que o triângulo AP Q é isósceles em P , pois AP = P Q = m.
Portanto, temos que:

∠P QA = ∠P AQ
2x − 105◦ = 105◦ − x
x = 70◦

Logo, os ângulos do triângulo ABC são: x = ∠BAC = ∠BCA = 70◦ e 180◦ − 2x =


∠ABC = 40◦ .

Problema 26. Seja A o valor do ângulo ∠BAC.


Pelo triângulo retângulo ABE, temos que: ∠ABE = 90◦ − A. Como G e F são pontos
médios dos segmentos AB e AH, respectivamente, então podemos concluir que GF é base
média no triângulo ABH. Portanto, temos que GF k BH e, consequentemente,

∠AGF = ∠ABH = 90◦ − A

POT 2014 - Geometria - Nı́vel 2 - Aula 0 - Soluções 15


2 TRABALHANDO COM ÂNGULOS

(Obs.: outra forma de achar que as retas GF e BH são paralelas é a aplicação direta do
teorema de Tales levando em consideração os segmentos AB e AH.)
De forma análoga à conclusão anterior, podemos concluir que GK é base média do triângulo
BAC e, consequentemente,
∠BGK = ∠BAC = A
Considerando o ponto G no segmento AB, temos que:

∠AGF + ∠F GK + ∠KGB = 180◦


∠F GK = 180◦ − A − (90◦ − A)
∠F GK = 90◦

Problema 27. Somando os ângulos internos do 4ABC, temos que:

∠BAC + ∠ACB + ∠CBA = 180◦


∠BAC = 180◦ − 37◦ − 38◦
∠BAC = 105◦

Daı́, podemos concluir que:


105◦
∠BAP = ∠P AQ = ∠QAC = = 35◦
3
Note que BD k AP implica em ∠DBA = ∠BAP = 35◦ . Portanto, temos que:

∠DBC = ∠DBA + ∠ABC


= 35◦ + 37◦
∠DBC = 72◦

Obs.: Com as informações do enunciado, podemos achar os demais ângulos do triângulo


DBC:

• Analogamente ao cálculo do ângulo ∠DBC, podemos concluir que: CD k AQ implica


em ∠ACP = 35◦ e, consequentemente, temos que:

∠DCB = 35◦ + 38◦


∠DCB = 73◦

• Somando os ângulos internos do 4DBC, temos que:

∠BDC + ∠DBC + ∠DCB = 180◦


∠BDC = 35◦

Problema 28. Percebamos que 4BAD ≡ 4BCD pelo caso lado-lado-lado (LLL), pois os
três lados dos dois triângulos são iguais. Logo, temos que: ∠ABF = ∠CBF .

POT 2014 - Geometria - Nı́vel 2 - Aula 0 - Soluções 16


2 TRABALHANDO COM ÂNGULOS

Com essa igualdade de ângulos, podemos concluir que: 4ABF ≡ 4CBF pelo caso lado-
ângulo-lado. Daı́, temos que: ∠AF B = ∠CF B.
Como a soma desses ângulos é igual a 180◦ , então temos que:

∠AF B = ∠CF B = 90◦

e, consequentemente, podemos concluir que:

∠AF D = ∠CF D = 90◦

Notemos que o triângulo ABC é isósceles em B. Portanto, temos que:


180◦ − 122◦
∠BAC = ∠BCA = = 29◦
2
Por consequência, podemos concluir que: ∠F CE = 180◦ − 29◦ = 151◦ .
Percebamos que 4CF D ≡ 4CED pelo caso lado-lado-lado (LLL), pois os três lados dos
dois triângulos são iguais. Daı́, temos que:

∠CF D = ∠CED = 90◦

Somando os ângulos internos do quadrilátero F CED, podemos concluir que:

∠F DE + ∠DEC + ∠ECF + ∠CF D = 360◦


∠F DE = ∠BDE = 29◦

Como 4CF D ≡ 4CED, então temos que:


29◦
∠CDE = ∠CDF =
2
Como 4BAD ≡ 4BCD, então podemos concluir que:
29◦
∠CDB = ∠ADB =
2
Finalmente, podemos concluir que:

∠ADE = ∠ADB + ∠BDE


29◦
= + 29◦
2
87◦
∠ADE =
2
Problema 29. Seja α tal que ∠CDF = α. Seja P o ponto de encontro dos segmentos DF
e BQ.
Analisando o triângulo retângulo DP Q, temos que: ∠P QD = 90◦ − α.
Analisando o triângulo retângulo BCQ, podemos concluir que: ∠CBQ = α.
Então, temos que: 4DCF ≡ 4BCQ pelo caso ângulo-lado-ângulo oposto, pois

POT 2014 - Geometria - Nı́vel 2 - Aula 0 - Soluções 17


3 ÁREAS

• ∠CDF = ∠CBQ = α;

• DC = BC = lado do quadrado;

• ∠DCF = ∠BCQ = 90◦

Com isso, podemos concluir que CF = CQ.


Dessa forma, temos que o triângulo F CQ é isósceles em C, além de ser retângulo. Daı́,
podemos concluir que:
180◦ − 90◦
∠QF C = ∠F QC =
2
∠F QC = 45◦

3 Áreas
Problema 30. A solução do material apresenta erros. Segue a versão corrigida da solução.
Observe que 4AXY ≡ 4AM N e ∠Y XA = ∠AM N . Assim, XY k M N e como:

XY = M N = M B = N C

segue que os quadriláteros XY CN e XY N B são paralelogramos.


Como A é ponto médio do segmento XM , temos que:
2
[AXC] = [AM C] =
3
Analogamente, como A é ponto médio do segmento N Y , podemos concluir que:
2
[ABY ] = [ABN ] =
3
1
Lembrando que [AXY ] = [AM N ] = , podemos concluir que:
3
[XY BC] = [AY X] + [AXC] + [ACB] + [ABY ]
1 2 2
[XY BC] = + +1+
3 3 3
8
[XY BC] =
3
Problema 31. Notemos que os dois triângulos gerados, quando traçamos uma mediana,
possuem a mesma área, pois possuem bases de comprimentos iguais (pela própria definição
de ponto médio) e a mesma altura. Daı́, temos que:

• M é ponto médio de BC implica em:

72cm2
[AM B] = [AM C] = = 36cm2
2

POT 2014 - Geometria - Nı́vel 2 - Aula 0 - Soluções 18


3 ÁREAS

• N é ponto médio de AM implica em:

36cm2
[AN B] = [M N B] = = 18cm2
2

• P é ponto médio de BN implica em:

18cm2
[BP M ] = [N P M ] = = 9cm2
2

Problema 32. Sejam M , N , O e P os pontos do quadrilátero maior de forma que D, E,


F e G são os pontos médios dos segmentos M N , N O, OP e P M , respectivamente.
Seja Q o ponto de encontro dos segmentos DF e EG. Conectemos os segmentos M Q, N Q,
OQ e P Q.
Lembrando que os dois triângulos gerados a partir da construção de uma mediana possuem
a mesma área, podemos fazer as seguintes considerações:

• Seja x tal que [M DQ] = [N DQ] = x;

• Seja y tal que [N EQ] = [OEQ] = y;

• Seja z tal que [OF Q] = [P F Q] = z;

• Seja w tal que [P GQ] = [M GQ] = w.

Daı́, temos que:

[DN EQ] + [F P GQ] = x + y + z + w = 210 + 240 = 450

Usando o resultado anterior, podemos concluir que a área que está faltando ([EOF Q]) pode
ser calculada pelo desenvolvimento a seguir:

[EOF Q] + [DM GQ] = x + w + y + z


[EOF Q] = 450 − 250
[EOF Q] = 200

Problema 33. Pelo fato conhecido de mediana e altura, traçando o segmento AH, podemos
concluir que:
[ABC]
• [ABH] = [ACH] = ;
2
[ABH]
• [BEH] = [AEH] = implica em:
2
[ABC]
[BEH] =
4

De forma análoga, temos que:

POT 2014 - Geometria - Nı́vel 2 - Aula 0 - Soluções 19


3 ÁREAS

[BCD]
• [CHF ] = ;
4
[CDA]
• [DF G] = ;
4
[DAB]
• [AGE] = .
4
Daı́, podemos concluir que a área sombreada ([EF GH]) pode ser calculada pelo desenvol-
vimento seguinte:

[EF GH] = [ABCD] − [BEH] − [CHF ] − [DF G] − [AGE]


 
[ABC] [BCD] [CDA] [DAB]
= [ABCD] − + + +
4 4 4 4
[ABC] + [CDA] + [BCD] + [DAB]
= [ABCD] −
4
2 · [ABCD]
= [ABCD] −
4
[ABCD]
=
2
[EF GH] = 100cm2

Problema 34. Seja h a distância entre os segmentos paralelos EF e AD.


Como o lado do quadrado é igual a 6, então temos que a distância entre os segmentos EF
e BC é igual a (6 − h).
Com isso, podemos concluir que:

[BEDF ] = [BEF ] + [DEF ]


EF · h EF · (6 − h)
= +
2 2
EF · 6
=
2
[BEDF ] = 3 · EF

Sabendo que a área sombreada ([BEDF ]) é um terço da área do quadrado, então temos
que:
1 2
3 · EF = ·6
3
EF = 4

Problema 35. Consideremos

• h a altura do trapézio;

• E o ponto de encontro da altura do trapézio que passa pelo ponto B e o lado AD.

POT 2014 - Geometria - Nı́vel 2 - Aula 0 - Soluções 20


3 ÁREAS

Daı́, temos que o triângulo ABE é:

• retângulo, pela própria definição de altura;

• isósceles, pois ∠A = 45◦ implica em ∠ABE = 180◦ − 90◦ − 45◦ = 45◦

Portanto, podemos concluir que: AE = BE = h.


Aplicando teorema de Pitágoras no 4ABE, temos que:

h2 + h2 = 62

h = 3 2

Sejam x e y tais que AD = x e BC = y.


Como a área do trapézio é igual a 30, então podemos concluir que:

(x + y) · h
= 30
2 √
x + y = 10 2

Seja F o ponto de encontro da altura do trapézio que passa pelo ponto C e o lado AD.
Notemos que o quadrilátero BCF E é um retângulo, pois possui os quatro ângulos retos.
Portanto, temos que: BC = EF = y. √
De forma análoga ao que foi feito no 4ABE, podemos concluir que: F D = h = 3 2. Com
isso, temos que: √
AD = x = y + 6 2
Aplicando esse resultado ao valor de (x + y) encontrado anteriormente, podemos concluir
que:

x + y = 10 2
 √  √
y + 6 2 + y = 10 2

BC = y = 2 2

Problema 36. Prolonguemos EO até o lado AB. Seja G o ponto de encontro desses dois
segmentos.
Vamos mostrar que EO = OG. Para isso, tracemos uma reta paralela ao lado AD passando
pelo ponto O. Sejam M e N os pontos de encontro dessa reta paralela com os lados AB e
CD, respectivamente.
Notemos que os triângulos EON e GOM são congruentes pelo caso ângulo-lado-ângulo
oposto, pois:

• ∠EN O = ∠GM O = 90◦ ;

• ON = OM ;

• ∠OEN = ∠OGM , pois AB k CD e EG corta essas retas paralelas.

POT 2014 - Geometria - Nı́vel 2 - Aula 0 - Soluções 21


3 ÁREAS

Daı́, podemos concluir que: EO = OG.


Observemos que os triângulos retângulos EOF e GOF são congruentes pelo caso lado-
ângulo-lado, pois:
• OF é lado comum;

• ∠EOF = ∠GOF = 90◦ ;

• EO = OG.
Portanto, temos que: EF = F G e [EOF ] = [F OG].
Analogamente, sendo G o ponto de encontro do prolongamento do segmento F O com o lado
BC, podemos chegar às seguintes conclusões:

F G = GH = HE = EF
[EF GH]
[F OG] = [GOH] = [HOE] = [EOF ] =
4
Sejam x e y tais que EN = x e F D = y.
Notemos que ED = 2 + x e EN = x implica em GM = x e AG = 2 − x.
Percebamos, também, que F D = y implica em AF = 4 − y.
Aplicando teorema de Pitágoras nos triângulos EDF e F AG, temos que:
2 2
EF = FG
(2 + x)2 + y 2 = (4 − y)2 + (2 − x)2
4 + 4 · x = 16 − 8 · y + 4 − 4 · x
y = 2−x

Portanto, os triângulos EDF e F AG são congruentes, pois ambos são retângulos e pos-
suem um cateto e hipotenusa iguais. Logo, temos que: [EDF ] = [F AG].
De forma análoga, poderemos concluir que: [F AG] = [GBH] = [HCE] = [EDF ]. Como a
soma das 4 áreas citadas é igual a ([ABCD] − [EF GH]), então temos que:

[ABCD] − [EF GH]


[EDF ] =
4
Portanto, a área marcada é igual a:
[EF GH] [ABCD] − [EF GH]
[EOF ] + [EDF ] = +
4 4
[ABCD]
=
4
[EOF ] + [EDF ] = 4cm2

Obs.: Alguns resultados dessa solução podem ser mais rapidamente obtidos usando um
assunto que será tratado em aulas futuras: Quadriláteros inscrı́tiveis. Por exemplo, sa-
bendo que o quadrilátero EOF D é inscrı́tivel, poderı́amos facilmente perceber que ∠OEF =
∠OF E = 45◦ .

POT 2014 - Geometria - Nı́vel 2 - Aula 0 - Soluções 22


3 ÁREAS

Problema 37. Sejam x e y tais que AB = CD = x e BC = DA = y.


Notemos que [ABCD] = 11cm2 implica em x · y = 11.
Percebamos, também, que:

A0 A · AB 0
[A0 AB 0 ] =
2
y · 2x
=
2
[A0 AB 0 ] = x · y = 11cm2

Analogamente, podemos concluir que: [B 0 BC 0 ] = [C 0 CD0 ] = [D0 DA0 ] = 11cm2 .


Portanto, temos que:

[A0 B 0 C 0 D0 ] = [ABCD] + [A0 AB 0 ] + [B 0 BC 0 ] + [C 0 CD0 ] + [D0 DA0 ]


[A0 B 0 C 0 D0 ] = 55cm2

Problema 38. Aplicando teorema de Tales, temos que:

HG GA
=
CD DA
8
=
4
HG = 2cm

Calculando a área marcada, podemos concluir que:

[ABC] + [CEF H] = [ABCD] + [DEF G] − [AHG]


2·8
= 1·4+4·4−
2
[ABC] + [CEF H] = 12cm2

Problema 39. Seja x tal que DQ = CP = x. Daı́, temos que: QC = P B = 10 − x.


Pela relação de áreas dada no enunciado, podemos concluir que:
7
[ABP ] = · [P CQ]
3
10 · (10 − x) 7 x · (10 − x)
= ·
2 3 2
30
x = cm
7
Aplicando teorema de Pitágoras em dois triângulos, temos que:
 
2 30 2
• 4AP B: AP = 10 + 10 −2 implica em:
7

10 65
AP = cm
7

POT 2014 - Geometria - Nı́vel 2 - Aula 0 - Soluções 23


3 ÁREAS

 2  2
2 30 30
• 4P CQ: P Q = + 10 − implica em:
7 7
50
PQ = cm
7

Portanto, podemos concluir que o perı́metro do quadrilátero AP QD é igual a:



10 65 50 30
AP + P Q + QD + DA = + + + 10
√7 7 7
10 65 + 150
AP + P Q + QD + DA = cm
7
Problema 40. Seja x tal que AJ = x. Como JH k DB, então temos que:

• ∠AJH = ∠ADB = 45◦ ;

• ∠AHJ = ∠ABD = 45◦ ;

• 4AJH é isósceles implica em: AJ = AH = x.


2
Sabendo que [AJH] = · [ABCD] , então podemos concluir que: x2 = 10 , ou seja:
5

x = 10
1
Analogamente, sendo y tal que IC = y e sabendo que [ICE] = · [ABCD], então temos
5
que: √
y= 5
Lembremos que já sabemos o comprimento de dois segmentos:

• HB = 5 − x = 5 − 10

• BE = 5 − y = 5 − 5

Faltam os demais comprimentos.


Percebamos que AG é um segmento de reta pertencente à diagonal AC. Dessa forma, temos
que ∠GAH = 45◦ . Como ∠AHG = 45◦ , então podemos concluir que: 4AGH é isósceles
e retângulo. Daı́, temos que:
GH = AG
Tracando o segmento F C, então, de forma totalmente análoga ao passo anterior, podemos
concluir que:
EF = F C
Notemos, também, que

AG + GF + F C = AC
F G = AC − AG − F C

POT 2014 - Geometria - Nı́vel 2 - Aula 0 - Soluções 24


3 ÁREAS

2
Pelo teorema de Pitágoras no 4ABC, temos que AC = 52 + 52 . Daı́, podemos concluir
que: √
AC = 5 2
Juntando todos os resultados obtidos, podemos, finalmente, calcular o perı́metro do pentágono
BEF GH:
 √   √   √ 
BE + EF + F G + GH + HB = 5 − 5 + F C + 5 2 − AG − F C + AG + 5 − 10
√ √ √
= 10 + 5 2 − 5 − 10
√ 
Obs.: Uma forma simplificada de expressar a mesma resposta é a expressão: 5 + 1 + 2 ·
√ 
5− 5 .

Problema 41. Sejam os seguintes pontos:


• O o centro do quadrado;
• E, F , G, H e I os pontos que reprensentam os primeiro, segundo, terceiro, quarto e
quinto cortes, respectivamente.
Consideremos que OE é o primeiro corte de forma que DE = 7cm e EA = 13cm .
Lembremos que a área de cada pedaço é igual a:
202
= 80cm2
5
Note que:
13 · 10
[AEO] = = 65cm2 < 80cm2
2
Portanto, F não está no segmento DA. Como só faltam 15cm2 para [AEO] atingir 80cm2 ,
então podemos concluir que F pertence ao segmento AB de forma que:
AF · 10
[AF O] = = 15cm2
2
AF = 3cm
Analogamente, podemos encontrar os demais pontos:
• o ponto G pertence ao segmento AB de forma que F G = 16cm e GB = 1cm;
• o ponto H pertence ao segmento BC de forma que BH = 15cm e HC = 5cm;
• o ponto I pertence ao segmento CD de forma que CI = 11cm e ID = 9cm.
Problema 42. Como M é ponto médio, então temos que BM e M C. Como a altura dos
triângulos ABM e ACM também é igual, logo podemos concluir que:
72
[ABM ] = [ACM ] = = 36cm2
2
Analogamente, temos que:

POT 2014 - Geometria - Nı́vel 2 - Aula 0 - Soluções 25


3 ÁREAS

• N é ponto médio de AM implica em:


36
? [ABN ] = [M BN ] = = 18cm2 ;
2
36
? [ACN ] = [M CN ] = = 18cm2 ;
2
• [BN C] = [M BN ] + [M CN ] = 36cm2 ;
36
• P é ponto médio de BN implica em: [BP C] = [N P C] = = 18cm2 ;
2
• Q é ponto médio de P C implica em:
18
[CQN ] = [P QN ] =
2
[P QN ] = 9cm2

Problema 43. Seja O o centro do quadrado. Notemos que O é o ponto de encontro de EF


com AC, pois pertence a ambas as retas. Percebamos, também, que EO = OF = 6cm.
Seja P o ponto de encontro entre AC e BE.
Calculemos, então, as duas primeiras áreas:
• BEOC é trapézio. Daı́, temos que:
(12 + 6) · 6
[BEOC] =
2
[BEOC] = 54cm2

• COF é triângulo retângulo em F . Com isso, podemos concluir que:


6·6
[COF ] =
2
[COF ] = 18cm2

Sejam S1 , S2 tais que S1 = [BP A] e S2 = [AP E]. Como o triângulo EAB é retângulo em
A, então temos que:
12 · 6
[EAB] =
2
S1 + S2 = 36cm2 (1)
Observando que EO k BA, podemos notar que 4BP A ∼ 4EP O, pois todos os ângulos
dos dois triângulos são iguais. Daı́, podemos concluir que:
BP BA
=
EP EO
12
=
6
BP
= 2
EP

POT 2014 - Geometria - Nı́vel 2 - Aula 0 - Soluções 26


3 ÁREAS

Analisando os triângulos BP A e AP E, temos que, em relação as bases BP e P E, os dois


triângulos possuem a mesma altura. Seja h essa altura. Daı́, podemos concluir que:

BP · h
S1 [BP A] 2
= =
S2 [AP E] EP · h
2
BP
=
EP
S1 = 2 · S2

Aplicando esse resultado ao resultado (1), temos que:

• S2 = [AP E] = 12cm2 ;

• S1 = [BP A] = 24cm2 .

Seja S3 tal que S3 = [EP O]. Como o triângulo AEO é retângulo em O, então podemos
concluir que:
[AEO] = S2 + S3 = 18cm2
Lembrando que já sabemos o valor de S2 , então temos que:

S3 = [EP O] = 6cm2

Como o triângulo EF B é retângulo em F , então podemos concluir que:


12 · 6
[EP O] + [OP BF ] =
2
[EP O] + [OP BF ] = 36cm2

Aplicando o resultado anterior, temos que [EP O] = 6cm2 implica em:

[OP BF ] = 30cm2

Obs.: A ideia de encontrar proporção entre áreas de dois triângulos, com mesma altura, a
partir da razão entre suas bases é conhecida como metódo k.

Problema 44. Vamos usar um fato bastante conhecido: Sendo G o baricentro de um


triângulo XY Z e M ponto médio do lado Y Z então temos que:

XG = 2 · GM

Obs.: A demonstração desse fato está no fim desse material (seção fatos que
ajudam).
Voltando à solução do problema, sejam k e j tais que:
1
• AB = k e, consequentemente, AD = ;
k

POT 2014 - Geometria - Nı́vel 2 - Aula 0 - Soluções 27


3 ÁREAS

• DE = j e, consequentemente, EC = k − j.

Consideremos os seguintes pontos:

• N , P e Q os baricentros dos triângulos ADE, EBC e ABE, respectivamente;

• R e S as projeções de N e P sobre o lado DC, respectivamente. Em outras palavras,


R é o ponto no lado DC tal que ∠N RC = 90◦ . Definição análoga para o ponto S;

• T a projeção de N sobre o lado AD;

• U a projeção de P sobre o lado BC.

Sendo L o ponto médio do lado DE, aplicando o fato conhecido citado no começo da solução
e o teorema de Tales no 4LDA , podemos calcular alguns segmentos:

• segmento N R:

NR NL
=
AD AL
NR NL
=
1
k NL + 2 · NL
1
NR =
3k

• segmento N T :

NT NA
=
LD LA
NT 2 · NL
=
j
2
2 · NL + NL
j
NT =
3

Analogamente, considerando o ponto médio do lado EC, o fato conhecido citado e o teorema
de Tales, podemos calcular outros segmentos:

• segmento P S:
1
PS =
3k
• segmento P U :
k−j
PU =
3
Por último, sejam:

• V e W as projeções de Q e E sobre o lado AB, respectivamente;

POT 2014 - Geometria - Nı́vel 2 - Aula 0 - Soluções 28


3 ÁREAS

• I o ponto médio do lado AB.

Aplicando o fato conhecido e o teorema de Tales no 4IEW de forma análoga ao já feito
duas vezes nessa solução, temos que:
1
QV =
3k
Com isso, temos todas as informações para calcular [N P Q]:

• base:

NP = TU −
 TN −P U 
j k−j
= k− −
3 3
2k
NP =
3

• altura h:

h = BC − QV − N R
   
1 1 1
= − −
k 3k 3k
1
h =
3k

Portanto, a área procurada é igual a:


2k
3 · 3k
1
[N P Q] =
2
1
[N P Q] =
9
Problema 45. Primeiramente, como AB = CD e AB k CD, então temos que:
1
[ABC] = [ADC] =
2
Como K é o encontro das diagonais do paralelogramo ABCD, então podemos concluir que
K é ponto médio do lado AC. Portanto, L é baricentro do 4ADC, isto é, L é o encontro
das três medianas de tal triângulo.
Seja M o ponto médio do lado AD. Aplicando o lema conhecido, cuja demonstração
encontra-se no fim desse material (seção fatos que ajudam), no 4ADC , temos que:
2
[ELKC] = · [ADC]
6
1
[ELKC] =
6

POT 2014 - Geometria - Nı́vel 2 - Aula 0 - Soluções 29


3 ÁREAS

Problema 46. Aplicando teorema de Pitágoras no 4ABC, temos que:


p
BC = (1012 − 202 )
p
= (101 + 20) · (101 − 20)
BC = 99

Sabendo que o triângulo ADB tem base DB e altura AC, então podemos concluir que:
99
· 20
[ADB] = 2 = 495
2
Problema 47. Seja O o centro do hexágono. Liguemos os segmentos AC, CE, EA, OA,
OC e OE.
Notemos que os triângulos ABC, CDE e EF A possuem a mesma área, pois são congru-
entes pelo caso lado-ângulo-lado. Como o triângulo ABC é isósceles em B, temos que:
180◦ − 120◦
∠BAC = ∠BCA = = 30◦
2
Percebamos, também, que os triângulos OAC, OCE e OEA possuem a mesma área, pois
são congruentes pelo caso lado-lado-lado. Como
360◦
∠AOC = ∠COE = ∠EOA = = 120◦
3
e o triângulo AOC é isósceles em O, então podemos concluir que
180◦ − 120◦
∠OAC = ∠OCA = = 30◦
2
Com isso, temos que 4ABC ≡ 4AOC pelo caso ângulo-lado-ângulo oposto, pois ambos
possuem o lado AC e os três ângulos iguais. Portanto, tais triângulos têm a mesma área.
Dessa forma, podemos notar que os 6 segmentos desenhados dividiram o hexágono em
6 triângulos congruentes e, consequentemente, com mesma área. Daı́, como a área do
hexágono é igual a 1cm2 , podemos concluir que:
1
[ABC] = cm2
6
Problema 48. Sejam P , Q, R, S e T os pontos de interseção das diagonais do pentágono
ABCDE de forma que:

• P é o encontro das diagonais BE e AC;

• Q é o encontro das diagonais AC e BD;

• R é o encontro das diagonais BD e CE;

• S é o encontro das diagonais CE e AD;

POT 2014 - Geometria - Nı́vel 2 - Aula 0 - Soluções 30


3 ÁREAS

• T é o encontro das diagonais AD e BE.

Sejam h1 e h2 as alturas relacionadas aos pontos C e E em relação ao lado AB nos


triângulos ABC e EAB, respectivamente. Daı́, temos que:

[ABC] = [EAB]
AB · h1 AB · h2
=
2 2
h1 = h2

Com isso, podemos perceber que os pontos C e E estão “igualmente distantes” da reta que
contém o segmento AB. Portanto, podemos concluir que:

CE k AB

Analogamente, temos que:

• BC k DA;

• CD k EB;

• DE k AC;

• EA k BD.

Sejam x, y, z, w e k tais que: [AP B] = x, [BQC] = y, [CRD] = z, [DSE] = w e


[AT E] = k. Como [ABC] = [BCD] = [CDE] = [DEA] = [EAB] = 1, então podemos
concluir que:

• [BP Q] = 1 − x − y;

• [CQR] = 1 − y − z;

• [DRS] = 1 − z − w;

• [EST ] = 1 − w − k;

• [AT P ] = 1 − k − x.

Pelas relações de paralelismo encontradas acima, temos que: ABRE é paralelogramo.


Como BE é diagonal desse paralelogramo, temos que:

• 1 = [ABE] = [RBE]; (analogamente, [EP C] = 1)

• [ABRE] = 2;

• [ABCDRE] = [ABRE] + [BCD] = 3;

• [ABCDE] = [ABCDRE] + [DRE] = 3 + (1 − z) = 4 − z.

POT 2014 - Geometria - Nı́vel 2 - Aula 0 - Soluções 31


3 ÁREAS

De forma análoga, podemos concluir que:

[ABCDE] = 4 − x = 4 − y = 4 − z = 4 − w = 4 − k

Com isso, temos que:


x=y=z=w=k
Falta apenas encontrar o valor de x.
Analisando triângulos com mesma altura para comparar áreas através da razão entre bases,
observando a razão entre os segmentos AP e P C, podemos concluir que:
• olhando os triângulos AP B e BP C:

AP [AP B]
=
PC [BP C]
AP x
=
PC 1−x

• olhando os triângulos AP E e EP C:

AP [AP E]
=
PC [EP C]
AP 1−x
=
PC 1

Daı́, temos que:


x 1−x
=
1−x 1
x = (1 − x)2
x2 − 3x + 1 = 0

Como x < 1, então temos que: √


3− 5
x=
2
Portanto, podemos concluir que:

[ABCDE] = 4 − x
√ !
3− 5
= 4−
2

5+ 5
[ABCDE] =
2
Problema 49. Essa questão é totalmente análoga à questão 42. Resposta:
[ABC]
=8
[EF G]

POT 2014 - Geometria - Nı́vel 2 - Aula 0 - Soluções 32


3 ÁREAS

Problema 50. Seja EF a tal reta paralela que vai dividir o trapézio em duas retas sendo
que E e F pertencem oas segmentos AB e CD, respectivamente.
Seja x tal que AE = x.
Pela igualdade de áreas, temos que:
[AEF D] = [EBCF ]
(75 − 2x) · 20
20 · x =
2
4 · x = 75
75
x = m
4
Problema 51. Obs.: A questão apresenta um pequeno erro de enunciado: Os pontos coli-
neares são F , C e E (e não G).
Seja P o ponto de interseção das retas BC e F G.
Consideremos as seguintes áreas:
• [ADG] = a;
• [GCP ] = b;
• [GP B] = c;
• [P F C] = d;
• [DCE] = e;
• [GCD] = f .
Sejam x e h o comprimento dos lados paralelos AB e DC e a distância entre eles, respec-
tivamente. Considerando y o comprimento de GB, podemos encontrar algumas relações
entre áreas:
• a + b + c:
(x − y) · h y · h
a + (b + c) = +
2 2
x·h
a + (b + c) =
2
• f:
x·h
f=
2
Portanto, podemos concluir que: a + (b + c) = f . Daı́, temos que:
[ABCD] = 2 · f
De forma totalmente análoga, a partir do paralelogramo DEF G, pdemos concluir que:
(b + d) + e = f . Com isso, temos que:
[DEF G] = 2 · f

POT 2014 - Geometria - Nı́vel 2 - Aula 0 - Soluções 33


3 ÁREAS

Pelas últimas duas equações encontradas, podemos concluir que:

2 · f = [ABCD] = [DEF G]

Problema 52. Consideremos as seguintes áreas:

• [AOB] = x;

• [BOC] = y;

• [COD] = z;

• [DOA] = w.

Nossa estratégia será encontrar equações que relacionam as outras áreas (x, z e w) com y.
Em primeiro lugar, notemos que: [ABC] = 150 implica em:

x = 150 − y

Em segundo lugar, percebamos que os triângulos ACD e BCD possuem a mesma base CD
e altura igual. Dessa forma, temos que: [ACD] = [BCD] implica em:

w=y

Além disso, podemos concluir que: [ACD] = 120 implica em:

z = 120 − y

Em terceiro lugar, podemos analisar a relação entre os segmentos DO e OB para encontrar


relação entre áreas de alguns triângulos (em outras palavras, método k). Daı́, temos que:

• triângulos DOC e COB:


DO z
=
OB y
pois esses triângulos possuem mesma altura e, com isso, a relação entre as áreas deles
é igual a relação entre suas respectivas bases;

• triângulos DOA e AOB:


DO w y
= =
OB x x
pois esses triângulos possuem mesma altura e, com isso, a relação entre as áreas deles
é igual a relação entre suas respectivas bases.

Com as últimas equações encontradas, temos que:

DO z y
= =
OB y x
y2 = x · z

POT 2014 - Geometria - Nı́vel 2 - Aula 0 - Soluções 34


3 ÁREAS

Usando as relações que encontramos no começo dessa solução, temos que:

y 2 = (150 − y) · (120 − y)
(150 + 120) · y = 150 · 120
200
y = [BOC] =
3
39
Problema 53. Essa questão é igual à questão 11. Resposta: .
2
Problema 54. Inicialmente, façamos um bom desenho, começando pela circunferência e,
em seguida, colocando os quatro pontos A, B, C e D de forma que AC ⊥ BD. Considere-
mos:

• h a altura do triângulo AOC em relação à base AC;

• P o ponto de interseção entre as retas AC e BD.

Como as diagonais do quadrilátero ABCD são perpendiculares, então temos que:

[ABCD] = [ABC] + [BCD]


AC · BP AC · P D
= +
2 2
AC · BP + P D
[ABCD] =
2
Além disso, podemos concluir que:

[ABCO] = [ABC] + [AOC]


AC · BP AC · h
[ABCO] = +
2 2
Portanto, podemos dizer que a linha quebrada AOC dividir ABCD em duas regiões de
mesma área é o mesmo que
[ABCD]
[ABCO] =
2
Pelas equações encontradas anteriormente, isso ocorrerá se

P D − BP
h=
2
Tentemos então encontrar essa relação.
Sejam os pontos:

• E o ponto de encontro resultante da interseção entre o prolongamento do segmento


AO por O e a circunferência;

• X a projeção ortogonal de E no segmento BD, ou seja, seja X o ponto do segmento


BD tal que ∠EXD = 90◦ .

POT 2014 - Geometria - Nı́vel 2 - Aula 0 - Soluções 35


3 ÁREAS

Notemos que AE é diâmetro da circunferência. Portanto, temos que ∠ACE = 90◦ . Perce-
bamos que o triângulo AOC é isósceles em O. Logo, a altura h é também mediana. Como
h k CE, então podemos aplicar teorema de Tales para concluir que:

CE = 2 · h

Como ∠CP D = 90◦ , então temos que P CEX é retângulo, pois possui os quatro ângulos
retos. Daı́, temos que:
CE = P X = 2h
Com esse resultado, notemos que só falta provar que: BP = XD.
Para provar o que falta, basta perceber o seguinte fato: BD k CD implica em ∠CBP =
∠XDE, pois ∠CBP = ∠XDE implica em 4BP C ≡ 4DXE.
Essa última implicação é consequência dos dois triângulos, BP C e DXE possuirem os três
ângulos iguais, inclusive um ângulo reto, e um cateto igual P C = XD , pois P CXD é
retângulo. Provemos tal fato numa observação após o término da solução.
Com todos os resultados encontrados, fica provado a solução, pois “invertendo” a ordem
dos resultados que encontramos, temos que:

• BP = XD, uma vez que:

BD k CD ⇒ ∠CBP = ∠XDE ⇒ 4BP C ≡ 4DXE ⇒ BP = XD

• P D = 2h + BP , pois:

CE = P X = 2h ⇒ P D = P X + XD = 2h + BP

[ABCD]
• [ABCO] = :
2
P D − BP [ABCD]
P D = 2h + BP ⇒ h = ⇒ [ABCO] =
2 2

Obs.: Provemos que BD k CD implica em: ∠CBP = ∠XDE.


Para isso, sejam α e β tais que ∠BDC = α e ∠CDE = β. Com isso, temos que:

∠XDE = α + β

Liguemos o segmento CE.


Notemos que
∠CDE = ∠CBE = β
pois ambos olham para o mesmo arco (CE). Analogamente, podemos concluir que: ∠BDC =
∠BEC = α.
Como CE k BD, então temos que:

∠EBD = ∠BEC = α

POT 2014 - Geometria - Nı́vel 2 - Aula 0 - Soluções 36


3 ÁREAS

Portanto, podemos concluir que:

∠CBP = ∠CBD = ∠CBE + ∠EBD


= β+α
∠CBP = ∠XDE

Problema 55. Obs.: A questão apresenta um pequeno erro de enunciado: A relação a ser
provada é: [DM A] + [CN B] = [ABCD].
Consideremos:

• comprimentos x e y tais que AN = N D = x e BM = M C = y;

• alturas hB , hC e hM as distâncias dos pontos B, C e M ao segmento AD, respecti-


vamente;

• ponto P a interseção dos segmentos AD e BC;

• comprimento z tal que BP = z.

Aplicando o teorema de Tales, temos que:

PB PM PC
= =
hB hM hC
z z + y z + 2y
= =
hB hM hC
Aplicando a propriedade bem conhecida de “soma de numerador com numerador e deno-
minador com denominador não altera a razão na proporção”, podemos concluir que:

z+y z + (z + 2y)
=
hM hB + hC
2 · hM = hB + hC

Daı́, temos que:

[ABCD] = [DCN ] + [CN B] + [N BA]


x · hC x · hB
= + [CN B] +
2 2
x · (2 · hM )
= + [CN B]
2
[ABCD] = [DM A] + [CN B]

POT 2014 - Geometria - Nı́vel 2 - Aula 0 - Soluções 37


4 FATOS QUE AJUDAM

4 Fatos que ajudam


Lema: Os 6 triângulos resultantes das construções das 3 medianas de um triângulo possuem
a mesma área.
Prova: Consideremos o triângulo ABC e os pontos médios M , N e P dos lados AB, BC
e CA, respectivamente. Seja G o baricentro do 4ABC.
Consideremos as seguintes áreas:

• X1 = [AGM ];

• X2 = [M GB];

• X3 = [BGN ];

• X4 = [N GC];

• X5 = [CGP ];

• X6 = [P GA].

Comparando bases iguais e mesma altura, temos que:

• [ABN ] = [ACN ] implica em:

X1 + X2 + X3 = X4 + X5 + X6

• [ABP ] = [CBP ] implica em:

X1 + X2 + X6 = X4 + X5 + X3

• subtraindo as duas equações encontradas, podemos concluir que: X3 −X6 = X6 −X3 .


Daı́, temos que:
X3 = X6

Analogamente, usando que [AM C] = [BM C], podemos concluir que X1 = X4 e X2 = X5 .


Para concluir o lema, basta perceber que X1 = X2 , X3 = X4 e X5 = X6 , pois tais triângulos
possuem bases iguais e mesma altura.

POT 2014 - Geometria - Nı́vel 2 - Aula 0 - Soluções 38


Problemas Resolvidos

Nı́vel 2

Geometria - Conceitos iniciais

Material elaborado por Susana Frómeta Fernández


Problemas

Problema 1. Dados os pontos colineares e consecutivos A, B, C, D e E tal que AB + CD = 3 × BC


e DE = AB. Sendo M o ponto médio de BE, onde M D = 2 e AE = 16, calcule M C.

Problema 2. Em uma reta temos 4 pontos consecutivos A, B, C e D que satisfazem as seguintes


relações 4 × AB − BD − 2 × CD = 4, AB = 3 e AC = 5, calcule AD.

Problema 3. (OBM-2011) Dois triângulos equiláteros de perı́metro 36cm cada um são sobrepostos
de modo que sua interseção forme um hexágono com pares de lados paralelos, conforme ilustrado no
desenho. Qual é o perı́metro desse hexágono?

Problema 4. Um trapézio ABCD de bases BC e AD com BC < AD é tal que 2 · AB = CD e


∠BAD + ∠CDA = 120◦ . Determine os ângulos do trapézio ABCD.

Problema 5. No 4ABC, E e D são pontos interiores aos lados AC e BC, respectivamente. Se AF


bissecta ∠CAD e BF bissecta ∠CBE. Prove que ∠AEB + ∠ADB = 2∠AF B.

F D

A B

2
Problema 6. No 4ABC, um ponto D está sobre AC tal que AB = AD. Se ∠ABC − ∠ACB = 30◦ ,
encontre ∠CBD.

Problema 7. A bissetriz interior de B e a bissetriz exterior de C do triângulo ABC encontram-se


em D. Através de D, uma reta paralela a CB encontra AC em L e AB em M . Se as medidas dos
comprimentos de LC e M B do trapézio CLM B são 5 e 7, respectivamente, encontre a medida de
LM .

Problema 8. No 4ABC, CF é a mediana relativa à hipotenusa AB, CE é bissetriz de ∠ACB, e


CD é uma altura relativa a AB. Prove que ∠DCE = ∠ECF .

Problema 9. A medida do segmento de reta P C, perpendicular à hipotenusa AC do triângulo


retângulo ABC, é igual à medida do segmento BC. Mostre que BP deve ser perpendicular ou
paralelo à bissetriz de A.

3
Soluções

1. Suponha que AB > BC. Dizemos que AB = DE = x + y e BC = x. Como AB + CD = 3 × BC


temos que CD = 2x − y. Veja que BE = BC + CD + DE = x + (2x − y) + (x + y) = 4x, logo
BM = 2x, M D = x − y e AE = 5x + y. Como M D = 2 e AE = 16 temos que x − y = 2 e 5x + y = 16.
Somando ambas equações obtemos 6x = 18, logo M C = x = 3.
O caso AB < BC resolve de maneira análoga. O caso AB = CD corresponde a AB = BC =
CM = M D = DE e da mesma forma concluı́mos que M C = 3.

2. Veja que BC = AC − AB = 5 − 3 = 2. Chamemos CD = x, então BD = BC + CD + 2 + x.


Escrevemos 4 × AB − BD − 2 × CD = 4 como 4 × 3 − (2 + x) − 2x = 4, donde temos x = 2. Logo
AD = AC + CD = 5 + 2 = 7.

3. Como os dois triângulos são equiláteros com perı́metro igual a 36cm, o comprimento de seus lados
é 12cm. Queremos encontrar o perı́metro do hexágono GHIJKL, conforme mostra a figura:

Como AB k DF , BC k ED e AC k EF , temos que os triângulos 4BLG, 4F GH e 4AJK são


também equiláteros. Veja então que LG + GH + HI = BG + GH + HC = BC = 12cm e que
IJ + JK + KL = DJ + JK + KE = DE = 12cm, logo o perı́metro do hexágono será igual a
12cm + 12cm = 24cm.

4. Seja E um ponto no segmento AD tal que AB k CE. Temos que ∠CED = ∠BAD por serem
correspondentes, logo teremos ∠CED + ∠CDA = 120◦ . Como a soma dos ângulo internos de um
triângulo é igual a 180◦ , temos que ∠ECD = 60◦ .
Por outro lado, veja que o quadrilátero ABCE tem os lados opostos paralelos, logo ele é um
paralelogramo, segue disso que AB = CE, logo temos que 2 · CE = CD. Considere o ponto médio M
do segmento CD, então CE = CM e o triângulo 4CEM é isósceles, como ∠ECM = 60◦ temos que
os outros ângulos internos do triângulo 4CEM são iguais a 60◦ e, portanto, o triângulo é equilátero.
Como consequência teremos EM = M D, logo 4EM D é isósceles. Também temos ∠EM D = 180◦ −
∠EM C = 120◦ , logo ∠M ED = ∠M DE = 30◦ . Com isso achamos um dos ângulos do trapézio:
∠CDA = 30◦ .
No triângulo 4CED temos ∠ECD = 60◦ e ∠CDE = 30◦ , segue então que ∠CED = 90◦ e, como
AB k EC, temos também que ∠BAD = 90◦ . Os ângulos ∠BAD e ∠ABC são colaterais internos,
logo ∠ABC = 180◦ − ∠BAD = 90◦ . Por último, ∠BCD = 360◦ − 30◦ − 90◦ − 90◦ = 120◦ .

4
5. Como AF é bissetriz de ∠CAD e BF é bissetriz de ∠CBE, chamamos x = ∠CAF = ∠F AD
e y = ∠CBF = ∠F BE. Também chamamos a = ∠DAB e b = ∠EBA. Olhando para os ângulos
internos dos triângulos 4EAB e 4DAB, temos ∠AEB = 180◦ − ∠EAB − ∠EBA = 180◦ − 2x − a − b
e ∠ADB = 180◦ − ∠DBA − ∠DAB = 180◦ − 2y − a − b. Logo ∠AEB + ∠ADB = 360◦ − 2x −
2y − 2a − 2b = 2(180◦ − x − y − a − b). Finalmente, olhando para o triângulo 4AF B temos que
∠AF B = 180◦ − ∠F AB − ∠F BA = 180◦ − x − a − y − b, o que conclui a prova.

6. Vamos chamar de α o ângulo ∠ACB, então ∠ABC = α + 30◦ . Queremos encontrar o ângulo
x = ∠CBD. Veja que ∠ADB é externo ao triângulo 4DCB, logo ∠ADB = ∠DCB +∠CBD = α+x.
Veja também que ∠ABD = ∠ABC − ∠CDB = α + 30◦ − x. Como AB = AD, o triângulo ABD é
isósceles e ∠ADB = ∠ABD, logo α + x = α + 30◦ − x. Concluı́mos que x = 15◦ .

7. ∠CBD = ∠BDM , por serem ângulos alternos internos. Como BD é bissetriz de ∠ABC, temos
que ∠M BD = ∠BDM , logo 4BM D é isósceles e M D = M B = 7.
Seja E um ponto qualquer na reta BC tal que C se encontre entre B e E. Veja que ∠ECD =
∠CDM , por serem ângulos alternos internos. Como CD é bissetriz de ACE, temos que ∠DCL =
∠LDC, logo 4CDL também é isósceles e LD = LC = 5.
Finalmente, M L = M D − LD = 7 − 5 = 2.

8. Os triângulos 4ABC e 4ACD são retângulos e tem em comum também o ângulo agudo ∠BAC,
logo o outro ângulo agudo de ambos triângulos também tem que ter a mesma medida, isto é: ∠ACD =
∠ABC. Como o triângulo ABC é retângulo, temos que a mediana relativa a hipotenusa é igual à
metade da hipotenusa, ou seja, AF = F B = CF . Temos então que 4BCF é isósceles e ∠F BC =
∠F CB. Isso mostra que ∠ACD = ∠F CB, como AE é bissetriz de ∠ACB, concluı́mos que ∠DCE =
∠ECF , como querı́amos mostrar.

9. Pelo ponto C, traçamos uma perpendicular a AC. Nessa perpendicular consideramos os pontos P
e P 0 tais que BC = CP = CP 0 e o ponto B está mais próximo de P do que de P 0 . Vamos mostrar
que a bissetriz do ângulo BAC é paralela a BP e perpendicular a BP 0 .
Observe que o triângulo BP P 0 tem a mediana, BC, igual à metade do lado P P 0 , logo o ângulo
∠P BP 0 deve ser reto. Para ver isso, usando que 4BCP e 4BCP 0 são isósceles, dizemos que α =
∠CBP = ∠CP B e β = ∠CP 0 B = ∠CBP 0 , logo a soma dos ângulos internos de 4BP P 0 é 180◦ =
2α + 2β, donde ∠P BP 0 = α + β = 90◦ .
Agora é suficiente mostrar que a bissetriz de A é perpendicular a BP 0 . Como os ângulos ∠ABC
e ∠P BP 0 são ambos retos, temos que ∠ABP 0 = 90◦ − P 0 BC = ∠CBP = α.
Denotemos por D o ponto de interseção de AC com BP 0 . Os triângulos 4BP P 0 e 4CDP 0 são
ambos retângulos (pois ∠P BP 0 = ∠DCP 0 = 90◦ ) e têm o ângulo agudo ∠BP 0 P em comum, logo o
outro ângulo agudo de ambos triângulos também tem que ter a mesma medida, isto é: ∠CDP 0 =
∠BP P 0 = α. Veja também que ∠CDP 0 e ∠ADB são opostos pelo vértice, logo são iguais. Segue
que ∠ADB = α. Já tı́nhamos mostrado que ∠ABD = α, então concluı́mos que 4ABD é isósceles e,
portanto, a bissetriz do ângulo ∠BAD coincide com a altura relativa ao segmento oposto BD. Isso
conclui a prova.

Material elaborado por Susana Frómeta Fernández

5
Problemas Resolvidos

Nı́vel 2

Congruência de triângulos

Material elaborado por Susana Frómeta Fernández


Problemas

Problema 1. ABCD é uma paralelogramo e ABF e ADE são triângulos equiláteros construı́dos
exteriormente ao paralelogramo. Prove que F CE também é equilátero.

Problema 2. (Rússia 1946) Dados três pontos A, B, C sobre uma reta l, são construı́dos triângulos
equiláteros ABC1 e BCA1 em um mesmo semi-plano com respeito a l. Se M e N são os pontos médios
de AA1 e CC1 , respectivamente, mostre que o triângulo BM N é equilátero.

Problema 3. (Inglaterra 1995) Seja ABC um triângulo retângulo em C. As bissetrizes internas de


BAC e ABC encontram BC e CA em P e Q, respectivamente. Sejam M e N os pés das perpendicu-
lares a partir de P e Q até AB, respectivamente. Encontre a medida do ângulo M CN .

Problema 4. (Polônia 1992) Os segmentos AC e BD intersectam-se no ponto P de modo que P A =


P D, P B = P C. Seja O o circuncentro do triângulo P AB. Prove que as retas OP e CD são
perpendiculares.

Problema 5. Em um quadrado ABCD, M é o ponto médio de AB. Uma reta perpendicular a M C


em M toca AD em K. Prove que ∠BCM = ∠KCM .

Problema 6. Dado um triângulo qualquer ABC, D, E e F são pontos médios dos lados AC, AB e
BC, respectivamente. Sendo BG uma altura do triângulo ABC, prove que ∠EGF = ∠EDF .

Problema 7. No losango ABCD com BAD = 60◦ , tomamos pontos F , H e G nos lados AD, DC e
na diagonal AC, respectivamente, de modo que DF GH seja um paralelogramo. Prove que o triângulo
BF H é equilátero.

Problema 8. Dado um quadrado ABCD e E um ponto no interior dele tal que ∠EDC = ∠ECD =
15◦ , prove que 4ABE é equilátero.

Problema 9. Sejam ABC um triângulo, D um ponto sobre o prolongamento da semi-reta BC a


partir de B tal que BD = BA e M o ponto médio de AC. A bissetriz do ângulo ∠ABC corta DM
em P . Mostre que ∠BAP = ∠ACB.

Problema 10. Prove que se em um triângulo ABC, a mediana AM é tal que ∠BAC é dividido na
razão 1 : 2, e D está sobre AM , com M entre A e D, tal que ∠DBA = 90◦ , então AC = AD
2 . Dica:
Escolha P sobre AD tal que AM = M P .

2
Soluções

1. Denotemos por α e β os ângulos α = ∠BAD = ∠BCD e β = ∠ABC = ∠ADC. Note que


α + β = 180◦ .
Como ABCD é um paralelogramo e os triângulos ABF e ADE são equiláteros, temos que ED =
AD = BC e F B = AB = DC, e além disso ∠CBF = ∠CDE = 360◦ − 60◦ − β. Concluı́mos que, pelo
critério l.a.l., os triângulos 4CBF e 4CDE são congruentes. Consequentemente, CE = CF .
Por outro lado ∠ECF = α + ∠DCE + ∠BCF . Como 4CBF ≡ 4CDE, temos que ∠DEC =
∠BCF , logo ∠DCE+∠BCF = ∠DCE+∠DEC = 180◦ −∠CDE = 180◦ −(360◦ −60◦ −β) = β−120◦ .
Isso mostra que ∠ECF = α + β − 120◦ = 180◦ − 120◦ = 60◦ .
Mostramos então que o triângulo F CE é isósceles (com CE = CF ) e ∠ECF = 60◦ . Os outros
dois ângulos do triângulo terão que ser iguais entre si e somar 120◦ , logo cada um deles será igual a
60◦ , donde concluı́mos que 4F CE é equilátero.

2. Começaremos observando que 4BAA1 ≡ 4BCC1 , pois BA = BC1 , BA1 = BC e ∠ABA1 =


∠CBC1 = 120◦ . Consequentemente teremos que BM = BN por serem as medianas correspondentes
aos lados AA1 e CC1 dos respectivos triângulos congruentes 4BAA1 e 4BCC1 .
Basta mostrar que ∠M BN = 60◦ . Para isso, usaremos novamente que 4BAA1 ≡ 4BCC1 e
que BM e BN são medianas de esses triângulos, então teremos que 4BM A1 ≡ 4BN C e, portanto,
∠M BA1 = ∠N BC. Finalmente, ∠M BN = ∠CBA1 − ∠N BC + ∠M BA1 = ∠CBA1 = 60◦ .

3. Denotemos por α e β os ângulos α = ∠CAP = ∠P AB e β = ∠ABQ = ∠QBC. Olhando para os


ângulos internos do 4ABC temos que 90◦ + 2α + 2β = 180◦ , donde concluı́mos que α + β = 45◦ .
Olhando os ângulos internos do 4ACP , vemos que ∠AP C = 90◦ − α. Como AP é bissetriz
de ∠CAB e ∠ACP = ∠AM P = 90◦ , temos que CP = P M e as retas AP e CM se intersectam
perpendicularmente. Com isso, temos que ∠P CM = 90◦ − ∠AP C = 90◦ − (90◦ − α) = α.
De maneira análoga, usando a bissetriz BQ, podemos mostrar que ∠QCN = β.
Assim, ∠M CN = 90◦ − α − β = 90◦ − 45◦ = 45◦ .

4. Chamemos de Q o ponto de intersecção de OP e CD. Devemos mostrar que ∠DQO = 90◦ .


Seja R o pé da perpendicular a partir de O até P B. Como O é o circuncentro de 4P AB temos
que OR também é bissetriz de ∠P OB. Veja que ∠P OB é o ângulo central correspondente ao arco
P AB da circunferência circunscrita ao triângulo 4P AB, então temos que ∠P2OB + ∠P AB = 180◦ , ou
seja, ∠P OR + ∠P AB = 180◦ .
Como P A = P D e P B = P C, temos que 4CP D ≡ 4BP A, e, portanto, ∠P DC = ∠P AB.
Também, ∠QDR = ∠P DC, por serem opostos pelo vértice. Logo teremos ∠QOR + ∠QDR = 180◦ .
A soma dos ângulos interno do quadrilátero DQOR será ∠QOR+∠QDR+∠DRO+∠DQO = 360◦ ,
ou seja, 180◦ + 90◦ + ∠DQO = 360◦ , donde concluı́mos que ∠DQO = 90◦ .

5. Seja L o ponto de interseção das retas CB e KM . Veja que 4AKM ≡ 4M BL pelo critério a.l.a.,
pois AM = M B, ∠KAM = ∠M BL e ∠AKM = ∠M LB (os ângulos são iguales poe serem alternos
internos). Consequentemente teremos que KM = M L. Isto implica que a altura CM do 4KCL
também será mediana, donde concluı́mos que 4KCL será isósceles e, portanto, CM será bissetriz de
∠KCL, o que conclui a prova.

3
6. EG é a mediana do triângulo retângulo ABG, logo ela será igual à metade da hipotenusa, ou seja,
AE = EB = EG. Por outro lado, veja que DF é a base média do 4ABC correspondente ao lado
AB, logo DF = AB 2 . Em particular temos que EG = DF o que implica que EF DG é uma trapézio
isósceles. Consequentemente ∠F EG = ∠EF D e, pelo critério l.a.l., temos 4EF G = 4EF D, donde
concluı́mos que ∠EGF = ∠EDF .

7. Chamemos de a o comprimento de cada lado do losango ABCD. Denotemos também x e y os


seguintes comprimentos: x = DH e y = HC. Note que x + y = a.
Como ABCD é um losango, temos que AC é bissetriz de ∠BCD, logo ∠ACD = 30◦ . Também,
como GH k F D, temos ∠CGH = ∠CAD = 30◦ , donde concluı́mos que 4CHG é isósceles com
GH = HC = y. Teremos também DF = GH = y, e, como x + y = a, também vale F A = x
Considere em AB o ponto L tal que 4AF L é equilátero, logo F L = AL = F A = x e ∠ALF = 60◦ .
Note que, pelo critério l.a.l. 4DF H ≡ 4LBF e então F H = F B.
Para concluir basta mostrar que ∠BF H = 60◦ . Para isso, observe que ∠GF H = ∠F HD (alternos
internos) e que ∠GF B = ∠F BL = ∠DF H (na primeira igualdade os ângulos são alternos internos
e na segunda são ângulos dos triângulos congruentes 4DF H e 4LBF ). Temos então que ∠BF H =
∠GF H + ∠GF B = ∠F HD + ∠DF H = 180◦ − ∠F DH = 180◦ − 120◦ = 60◦ .

8. Seja F o pé da altura correspondente ao lado CD do triângulo isóscele 4CDE. Seja G o ponto de
interseção de EF com o lado AB. Como ABCD é um quadrado, F e G são pontos médios de CD e
AB, respectivamente.
Considere o ponto P em EF tal que 4CP D é equilátero. Teremos que 4ADP é isósceles, pois
DP = DC = AD, e também ∠ADP = 90◦ − 60◦ = 30◦ . Logo ∠AP D = ∠P AD = 75◦ .
Veja agora que ∠GAP = 90◦ − 75◦ = 15◦ e AG = DF , logo pelo critério a.l.a. temos que
4AP G ≡ 4DEF . Consequentemente teremos que AP = DE e ADEP será um trapézio isósceles.
As diagonais do trapézio devem então ser iguais, ou seja, DP = AE.
Como também DP = AB, acabamos de mostrar que AE = AB. Um argumento simétrico ao feito
anteriormente mostra que BE = AB, e isto conclui a prova.

9. Chamemos de α os ângulos α = ∠BDA = ∠BAD. Como ∠ABC é ângulo externo do 4ABD


temos que ∠ABC = 2α e, como BP é bissetriz, temos que ∠ABP = ∠P BC = α. Como consequência
teremos que BP k AD.
Considere N o ponto médio de AD. Note que M N k CD. Sejam E e F os pontos de interseção
da reta BP com as retas M N e AC, respectivamente, e seja G o ponto de interseção de AE com BC.
Como E está na reta M N , temos que E é o ponto médio de AG, logo BE será mediana e bissetriz no
4ABG. Isso implica que 4ABG é isósceles com AB = BG e que ∠AEB = 90◦ .
No 4AM D temos P F k AD e, como N é ponto médio de AD, também teremos que E é ponto
médio de P F . Observamos então que as diagonais do quadrilátero AP GF se cortam no ponto médio
e são perpendiculares. Isso mostra que AP GF é um losango, ou seja, todos os lados são iguais e os
opostos são paralelos.
Em particular, mostramos que P G k AC, donde temos ∠ACB = ∠P GB. Veja também que
4ABP ≡ 4GBP , pois BP é lado comum, ∠ABP = ∠P BG e AB = BG (usamos o critério l.a.l.).
Consequentemente temos que ∠BAP = ∠P GB, o que conclui a prova.

10. Seguindo a dica dada no enunciado do exercı́cio, consideramos em AD o ponto P tal que AM =
M P . Observamos então que as diagonais do quadrilátero ABP C se intersetam no ponto médio, o que
implica que ABCD é um paralelogramo. Chamemos α = ∠BAM e 2α = ∠CAM . Como AB k CP e

4
AC k BP , também teremos que ∠AP C = α e ∠AP B = 2α. Chamemos também de x o comprimento
de AC, temos x = AC = BP . Queremos mostrar que AD = 2x.
Vamos escolher os pontos E e F sobre AP tal que ∠ECP = α e AF = AC = x. Veja então que
4ECP é isósceles com CE = EP . O ângulo ∠AEC é externo ao 4ECP , logo ∠AEC = 2α. Vemos
então que 4ACE tem dois ângulos iguais a 2α, logo ele será isósceles com AC = CE = x.
Juntando as informações anteriores temos AC = AF = CE = EP = BP = x. Como M é ponto
médio de AP e AF = EP , temos que M também é ponto médio de F E. Temos então que as diagonais
do quadrilátero CEBF se intersetam no ponto médio, e portanto CEBF será um paralelogramo. Isso
mostra que F B = CE = x e 4AF B é isósceles com ∠BAF = ∠ABF = α.
No 4P EB, chamamos de β os ângulos β = ∠P EB = ∠P BE. Note que α + β = 90◦ . Olhando
para a soma dos ângulos internos do triângulo retângulo 4ABD, temos que ∠ADB = β.
Os triângulos 4DF B e 4EBP têm ambos um ângulo igual a 2α e outro igual a β, logo o terceiro
ângulo de ambos triângulos será a β (lembre que α+β = 90◦ ). Como F B = BP = x, concluı́mos, pelo
critério a.l.a., que 4DF B ≡ 4EBP . Como consequência, temos F D = EP = x, donde concluı́mos
que AD = AF + F D = x + x = 2x, como querı́amos mostrar.

Material elaborado por Susana Frómeta Fernández

5
Problemas Resolvidos

Nı́vel 2

Razões de segmentos

Material elaborado por Susana Frómeta Fernández

1
Problemas

Problema 1. Sendo AS e AP bissetrizes dos ângulos internos e externos em A, determine o valor de


CP , sabendo que BS = 8 e CS = 6.

Problema 2. Seja ABC um triângulo de lados a, b, c opostos aos vértices A, B, C, respectivamente.


ac ab
Se D ∈ BC tal que AD é bissetriz interna, mostre que BD = b+c e CD = b+c .

Problema 3. O incentro do triângulo ABC divide a bissetriz interna do ângulo A na razão AI :


ID = 2 : 1. Mostre que os lados do triângulo estão em progressão aritmética

Problema 4. Num triângulo ABC, temos os comprimentos dos lados são: AB = x − 1, BC = x


e AC = x + 1, para algum inteiro x. Sabemos também que seu maior ângulo é o dobro do menor.
Determine o valor de x.

Problema 5. Em um triângulo ABC, de lados AB = 12, AC = 8 e BC = 10, encontre o maior


segmento que a bissetriz interna de A determina sobre BC.

Problema 6. (Cı́rculo de Apolônio) Seja k um número real positivo, k 6= 1. Mostre que o lugar
geométrico dos pontos P do plano tais que PP B
A
= k é uma circunferência cujo centro pertence à reta
AB.

Problema 7. Seja ` a distância entre A e B. Para k real positivo, k 6= 1 considere o cı́rculo de


k`
Apolônio de razão k, descrito no problema anterior. Mostre que o raio desse cı́rculo é 2 .
|k − 1|

AB
Problema 8. Em um triângulo ABC, BC = 7 e AC = 3. Calcule o valor da altura relativa ao lado
oposto ao vértice A sabendo que ela é máxima.

Problema 9. Em um triângulo ABC, BC = 16 e a altura relativa ao lado BC é 8. Calcule a razão


AB
AC sabendo que ela é máxima.

2
Soluções

1. .

8 6
B S C P

Pelo teorema de bissetriz interna, temos


AC SC 6 3
= = = .
AB SB 8 4
Pelo teorema da bissetriz externa, temos
AC CP CP
= = .
AB BP CP + 14
Isto implica que
CP 3
= ,
CP + 14 4
donde obtemos que 4CP = 3CP + 42, e, portanto, CP = 42.

2. Pelo teorema da bissetriz interna, temos


c AB BD BD
= = = .
b AC CD a − BD
ac ac ab
Então b·BD = ac−c·BD, o que implica BD = b+c . Também obtemos CD = a−BD = a− b+c = b+c

AB AI
3. Como BI é bissetriz do ∠ABD, pelo teorema da bissetriz interna, temos que BD = ID = 2. Da
AC
mesma forma, como CI é bissetriz do ∠ACD, temos CD = 2.
Assim, se denotamos x = BD e y = CD, teremos que AB = 2x e AC = 2y. Finalmente, note que

AC = 2y = x + y + y − x = BC + (y − x)

e que
BC = x + y = 2x + y − x = AB + (y − x).

3
4. Tracemos a bissetriz BD do ∠ABC. Usando o Problema 2, temos que
AB · AC (x − 1)(x + 1) x2 − 1
AD = = = .
AB + BC x−1+x 2x − 1
Vejamos agora que os triângulos ABD e ABC são semelhantes, pois ∠BAD = ∠BAC (ângulo
comum a ambos triângulos) e, além disso, como ∠ABC = 2∠ACB, temos que ∠ABD = ∠CBD =
∠ACB. Então os lados de ambos triângulos se encontram na mesma proporção:
AB AD
= .
AC AB
Substituindo em função de x, obtemos
x−1 (x2 − 1)/(2x − 1) x+1
= = ,
x+1 x−1 2x − 1
donde concluı́mos que x = 0 ou x = 5. Como x deve ser estritamente positivo, temos que x = 5.

5. Usando o Problema 2, encontramos


AB · BC 12 · 10
BD = = =6
AB + AC 20
e
AC · BC 8 · 10
CD == = = 4.
AB + AC 20
O maior desses segmentos é BD = 6.
6. Sejam M e N os conjugados harmônicos do segmento AB na razão k, isto é, os pontos pertencentes
MA NA
à reta AB tais que M B = N B = k. Vamos mostrar que o lugar geométrico dos pontos P tais que
PA
P B = k é a circunferência de diâmetro M N .

A M B N

Na figura acima estamos considerando k > 1. Quando k < 1, os papéis de A e B se invertem e a


análise é análoga.

PA
Afirmação 1. Se P é tal que PB = k então P pertence à circunferência de diâmetro M N .

Demonstração. Seja P um ponto tal que PP B A


= k. Então PP B
A
= M MA
B e portanto, pela recı́proca
do teorema da bissetriz interna, temos que P M é bissetriz interna no triângulo P AB. Da mesma
forma, da relação PP B
A NA
=N B e da recı́proca do teorema da bissetriz externa segue que P N é bissetriz
externa no triângulo P AB. Como as bissetrizes interna e externa no ponto P do triângulo P AB
são perpendiculares entre si concluı́mos que P M é perpendicular a P N e portanto P pertence à
circunferência de diâmetro M N .

4
PA
Afirmação 2. Se P pertence à circunferência de diâmetro M N então PB = k.

Demonstração. Seja P um ponto pertencente à circunferência de diâmetro M N . Tracemos por B


uma reta paralela à reta P M e chamemos de X a interseção dessa reta com a reta AP . Da mesma
forma, tracemos por B uma reta paralela à reta P N e chamemos de Y a interseção dessa reta com a
reta AP .
X

A M B N

Pelo Teorema de Tales temos que


PA MA PA NA
= =k e = = k,
PX MB PY NB
donde segue que P X = P Y . Sendo assim, no triângulo XBY , o qual é retângulo em B, temos que
P B é mediana relativa à hipotenusa e portanto P B = P X = P Y . Portanto, PP B
A
= PP X
A
= k.

7. Com a mesma notação da solução do exercı́cio anterior, o raio do cı́rculo de Apolônio é metade do
MA NA
segmento M N , onde M e N são tais que M B = N B = k.

A M B N

A figura considera co caso em que k > 1. Se k < 1, basta inverter os papéis de A e B e considerar
k 0 = k1 . Vamos considerar apenas o caso k > 1.
Note que
M N = M B + N B.
Vamos calcular cada parte separadamente.
Vamos usar que se ab = dc então ab = dc = a+c
b+d = a−c
b−d sempre que os denominadores são não nulos.

MA MA MB MA + MB ` `
= k =⇒ = = = =⇒ M B = .
MB k 1 k+1 k+1 k+1
NA NA NB NA − NB ` `
= k =⇒ = = = =⇒ N B = .
NB k 1 k−1 k−1 k−1
Com isso,
` ` 2k`
MN = + = 2 .
k+1 k−1 k −1
kl
E portanto, o raio do cı́rculo de Apolônio é 2 .
k −1

5
8. O ponto A pertence ao cı́rculo de Apolônio de razão k = 3 relativo aos pontos B e C, os quais
estão a uma distância ` = 7. A altura relativa ao vértice A é máxima quando ela coincide com o raio
do cı́rculo de Apolônio.
A

altura
máxima
B C

Portanto, pelo exercı́cio anterior, no triângulo ABC, a altura máxima relativa ao vértice A é
k` 3×7 21
= 2 = .
|k 2− 1| |3 − 1| 8
9. Como no triângulo ABC a altura relativa ao vértice A é 8, o ponto A pertence à reta r que é
paralela a BC e dista 8 da reta BC.
Seja k = ABAC , então A pertence ao cı́rculo de Apolônio de razão k relativo aos pontos B e C, os
quais estão a uma distância ` = 16. Vamos chamar esse cı́rculo de Ck . Queremos determinar o maior
valor possı́vel para a constante k.

k`
Afirmação: Quando k > 1, o raio do cı́rculo de Apolônio Ck , que é dado por k2 −1
, é uma função
decrescente de k.
k k−1+1 1 1
Demonstração. Basta ver que = = + 2 .
k2 −1 2
k −1 k+1 k −1
Como A deve pertencer à interseção entre a reta r e o cı́rculo Ck , temos que o maior k possı́vel é
o maior k para o qual essa interseção é não vazia. E isso ocorre quando a circunferência Ck tangencia
a reta r. Nesse caso, A é esse ponto de interseção e o raio de Ck é igual a 8.

r A

B C

Portanto, o maior k possı́vel satisfaz


16k √
= 8 =⇒ k = 1 + 2.
k2 − 1

Material elaborado por Susana Frómeta Fernández

6
Problemas Resolvidos

Nı́vel 2

Semelhança de triângulos

Material elaborado por Susana Frómeta Fernández

1
Problemas

Problema 1. Sejam D e E pontos sobre os lados AB e AC do triângulo ABC. Sendo BC = 22,


AD = 8, DB = 3, AE = 5 e ∠ABE = ∠ACD, calcule o comprimento de DE.

Problema 2. Considere a circunferência circunscrita ao triângulo ABC. Seja AE um diâmetro dessa


circunferência e AD a altura do triângulo. Sendo AB = 6, AC = 10 e AE = 30, calcule AD.

Problema 3. Calcule o raio da circunferência circunscrita ao triângulo ABC sabendo que AB = 4,


AC = 6 e a altura AH relativa ao lado BC é igual a 3.

Problema 4. (Base média de um triângulo) Sejam M e N os pontos médios, respectivamente, dos


lados AB e AC do triângulo ABC. O segmento M N é chamado de base média, relativa ao lado BC.
Mostre que M N é paralela a BC e que M N = BC
2 .

Problema 5. Seja ABCD um trapézio com AB paralelo a CD, M e N os pontos médios dos lados
oblı́quos AD e BC. Use o exercı́cio anterior para concluir que M N = AB+CD
2 .

Problema 6. (Teorema da bissetriz interna) No 4ABC, seja AD a bissetriz do ∠A, com D no


segmento BC. Então AB BD
AC = DC .

Problema 7. No triângulo ABC, a bissetriz interna do ângulo ∠A encontra BC em D. A reta por B,


perpendicular a AD, encontra AD em E. Seja M o ponto médio do lado BC. Se AB = 26, BC = 28
e AC = 30, ache os comprimentos de DM e M E.

Problema 8. No triângulo ABC, Z é um ponto sobre o lado AB. Uma reta por A e paralela a CZ,
1 1 1
encontra BC em X; uma reta por B e paralela a CZ encontra AC em Y . Mostre que AX + BY = CZ .

Problema 9. Seja P um ponto no interior do triângulo equilátero ABC. Por P traçamos três retas
paralelas aos lados de ABC, determinando três triângulos menores, de áreas 4, 9 e 49. Determine a
área do triângulo ABC.

Problema 10. Duas circunferências c1 e c2 interceptam-se em dois pontos A e B. Construa um


segmento P Q pelo ponto B com uma extremidade sobre c1 e a outra sobre c2 de modo que P Q seja
o maior possı́vel.

2
Soluções
1. Os triângulos ABE e ACD são semelhantes, pois ∠BAC é comum a ambos triângulos e ∠ABE =
∠ACD. Então temos
AE AB
= ,
AD AC
ou, equivalentemente,
AE AD
= .
AB AC
Esta última igualdade implica que dois lados dos triângulos ADE e ABC se encontram na mesma
razão. Como o ângulo compreendido entre esses dois lados é comum em ambos triângulos (é o ∠BAC),
temos que os triângulos ADE e ABC são semelhantes. Logo
AE DE
= .
AB BC
O que implica que
AE 5
DE = BC · = 22 · = 10.
AB 11

2. Seja O o centro do cı́rculo circunscrito. Como ∠AOC é um ângulo central e ∠ABC é um ângulo
inscrito na circunferência, relativos ao mesmo arco, temos
180◦ − ∠AOC ∠AOC
∠OAC = ∠OCA = = 90◦ − = 90◦ − ∠ABC = ∠BAD.
2 2
Também, como ∠ADB = ∠ACE = 90◦ , temos que os triângulos ABD e ACE são semelhantes. Então
AD AB
= .
AC AE
Portanto
AB 6
AD = AC · = 10 · = 2.
AE 30

3. Traçamos o diâmetro AE da circunferência circunscrita ao triângulo ABC. Exatamente como no


problema anterior, temos que os triângulos AEC e ABH são semelhantes. Logo
AE AC
= .
AB AH
Donde obtemos
AC 6
AE = AB · = 4 · = 8,
AH 3
o que mostra que o diâmetro da circunferência circunscrita é igual a 8, logo o raio é igual a 4.

4. Os triângulos AM N e ABC são semelhantes, pois ∠BAC é comum a ambos triângulos e, além
disso, os lados nos quais esse ângulo está compreendidos estão na mesma proporção, já que, como M
e N são pontos médios de AB e AC, respectivamente, temos
AM AN 1
= = .
AB AC 2
Consequentemente, os lados M N e BC se encontram na mesma proporção:
MN 1
= .
BC 2
Também, como consequência da semelhança dos triângulos, os outros dois ângulos são congruentes,
em particular ∠AM N = ∠ABC, o que mostra que M N k BC.

3
5. Denotemos por P o ponto de interseção das retas AN e CD.

A B

M N

D C P

Como ∠AN B = ∠CN P , por serem opostos pelo vértice, BN = N C por N ser ponto médio de
BC, e ∠ABN = ∠N CP , por serem alternos internos, temos que os triângulos 4ABN e 4CN P são
congruentes. Consequentemente, AN = N P e AB = CP .
Temos então que M N é base média do 4ADP . Usando o exercı́cio anterior, concluı́mos que
DP DC + CP DC + AB
MN = = = .
2 2 2

6. A perpendicular a AD passando por B corta AD em E e AC em F . Seja também G o pé da


perpendicular traçada desde C até a reta AD.

B D C

Note que como AE é altura e bissetriz no 4ABF , este triângulo é isósceles, com AB = AF .
Também, uma vez que EF k GC, temos que os triângulos 4AEF e 4AGC são semelhantes. Assim,
AB AF EF BE
= = = .
AC AC GC GC
Por outro lado, os triângulos 4BDE e 4CDG são semelhantes. Logo
BE BD
= .
GC DC
E isto conclui a prova.

4
7. Seja F o ponto de interseção de AC com o prolongamento de BE.

B D M C

Como AE é, ao mesmo tempo, altura relativa à base BF e bissetriz do ∠BAF , temos que o 4ABF
é isósceles com base BF . Consequentemente, E também será ponto médio de BF .
Temos então que M E é base média do 4BCF , logo M E k CF e

CF AC − AF AC − AB 30 − 26
ME = = = = = 2.
2 2 2 2
Usando novamente que M E k AC, temos que os triângulos 4DEM e 4DAC são semelhantes, logo

DM EM 2
= = .
DC AC 30
Como DC = DM + M C = DM + 14, temos
DM 1
= ,
DM + 14 15
donde concluı́mos que DM = 1.

8. Note que os triângulos 4BCZ e 4BAX são semelhantes, logo


CZ BZ
= .
AX AB
Também, os triângulos 4ACZ e 4ABY são semelhantes, logo
CZ AZ
= .
BY AB
Logo
CZ CZ BZ + AZ AB
+ = = = 1.
AX BY AB AB
Dividindo ambos lados por CZ obtemos a igualdade desejada.

5
9. Sejam x, y e z, respectivamente, os comprimentos dos lados dos triângulos que têm área 4, 9 e 49.
Seja ` o comprimento dos lados do 4ABC.

y y
x
P

z z

A z B

Como estes quatro triângulos são equiláteros, eles são, em particular, semelhantes entre si.
Temos que ( xy )2 = 49 , donde concluı́mos que y = 3x x 2
2 . Por outro lado, temos que ( z ) =
4
49 , donde
concluı́mos que z = 7x
2 .
Como ` = x + y + z, temos
3x 7x
`=x+ + = 6x.
2 2
Agora, usando que o triângulo equilátero de lado 4 é semelhante ao 4ABC, temos
 2
[ABC] `
= ,
4 x
portanto
 2
6x
[ABC] = 4 = 144.
x

6
10. Sejam P e P1 pontos na circunferência c1 , Q e Q1 pontos na circunferência c2 , de modo que P Q
e P1 Q1 passem pelo ponto B.

c2

c1 A

P1
Q

B
P

Q1

Note que ∠AP B = ∠AP1 B, por estarem inscritos no mesmo arco da circunferência c1 . Também,
∠AQB = ∠AQ1 B, por estarem inscritos no mesmo arco da circunferência c2 .
Isto implica que os triângulos 4AP Q e 4AP1 Q1 são semelhantes. Como os lados de ambos
triângulos se encontram então na mesma proporção, temos que a escolha dos pontos P e Q que nos
garante que o segmento P Q é o maior possı́vel, deve ser feita de modo que os segmentos AP e AQ
sejam também o maior possı́vel. Ou seja, escolheremos P e Q de modo que AP e AQ sejam diâmetros
das circunferências c1 e c2 , respectivamente.
Esta escolha é possı́vel porque ∠ABP = 90◦ e ∠ABQ = 90◦ (por AP e AQ serem diâmetros de
c1 e c2 , respectivamente). Logo os pontos P , Q e B estão alinhados.

Material elaborado por Susana Frómeta Fernández

7
Problemas Resolvidos

Nı́vel 2

Teorema de Pitágoras

Material elaborado por Susana Frómeta Fernández

1
Problemas

Problema 1. Na figura abaixo ABCD é um quadrado de área 64 cm2 e EF GH um quadrado de


área 36 cm2 . Determine a área do quadrado CM HN .

D C

H G

A B M E F

Problema 2. Seja ABCD um quadrado de lado 28. Seja P um ponto no seu interior e E um ponto
no lado CD de modo que CD ⊥ P E e AP = BP = P E. Ache AP .

Problema 3. Suponha que ABC seja um triângulo retângulo escaleno, e P seja o ponto na hipotenusa
AC tal que ∠ABP = 45◦ . Dado que AP = 1 e CP = 2, calcule a área do triângulo ABC.

Problema 4. (Cone Sul 1989 - adaptado) Na figura abaixo temos dois quadrados. O quadrado
ABCD, que tem lado 2; e o quadrado AEF G, que tem lado 1. Determine o raio do cı́rculo que é
tangente aos lados BC e CD do quadrado ABCD e passa pelo vértice F do quadrado AEF G.

D C

G F

A E B

Problema 5. P é um ponto no interior do retângulo ABCD. Se P A = 2; P B = 3, e P C = 10. Ache


P D.

2
Problema 6. Nos lados AB e DC do retângulo ABCD, os pontos F e E são escolhidos de modo que
AF CE seja um losango. Se AB = 16 e BC = 12, ache EF .

A F B

D E C

3
Soluções

1. Note que

∠BM C = 180◦ − ∠CM E = 180◦ − 90◦ − ∠HM E = 90◦ − ∠HM E = ∠EHM.

Analogamente, ∠BCM = ∠EM H. Como CM = M H (são lados de um quadrado), temos que, pelo
critério a.l.a., 4CBM ≡ 4EM H. Consequentemente, BM = EH = 6.
Vamos aplicar o teorema de Pitágoras no 4CBM :

CM 2 = BC 2 + BM 2 = 82 + 62 = 100.

A área do quadrado CM HN é igual a

CM · M H = CM 2 = 100.

2. Chamemos AP de x. Seja F o ponto de interseção de EP e AB. Como F E ⊥ CD, temos também


F E ⊥ AB. Note que o 4ABP é isósceles (com AP = P B) e, como P F é altura, será também
mediana. Ou seja, F é ponto médio de AB.
Note que ADEF é um retângulo, logo ED = AD = 28. Vejamos os lados do 4ADP : AF = 14,
AP = x e P F = 28 − x. Aplicando o teorema de Pitágoras nesse triângulo, temos

x2 = 142 + (28 − x)2 = 142 + 282 − 56x + x2 .

Logo
142 + 282
x= = 17, 5.
56

3. Note que BP é bissetriz do ∠ABC. Pelo teorema da bissetriz, temos


AB AP 1
= = ,
BC PC 2
logo BC = 2 · AB.
Pelo teorema de Pitágoras, temos

32 = AC 2 = AB 2 + BC 2 = AB 2 + (2 · AB)2 = 5 · AB 2 ,

o que implica que AB 2 = 59 .


A área do 4ABC é igual a
AB · BC AB · 2AB 9
= = AB 2 = .
2 2 5

4. Chamemos de O o centro do cı́rculo tangente aos lados BC e CD e que passa pelo ponto F . Seja
H o pé da perpendicular traçada desde O até o lado BC e seja I 6= F ponto de interseção do cı́rculo
com a diagonal AC.
Usando
√ o teorema de Pitágoras no 4AEF , temos que AF 2 = AE 2 + EF 2 = 2 e, portanto,
AF = 2.
Agora chamemos de r o raio do cı́rculo e de x o comprimento do segmento IC. Note que OF =
OH = OI = r.

4
Note que √
2 = F C = 2r + x.
Usando o teorema de Pitágoras no 4OHC, temos

2r2 = OH 2 + HC 2 = OC 2 = (r + x)2 = r2 + 2rx + x2 .

O que implica que


r2 = 2rx + x2 .

Substituindo x = 2 − 2r, temos √
r2 + 2 2r − 2 = 0,

cuja única raiz positiva é r = 2 − 2.

5. A reta que passa por P e é perpendicular aos lados AB e CD intersecta ditos segmentos nos pontos
E e F , respectivamente.
Usando o teorema de Pitágoras nos triângulos retângulos P BE e P AE, temos

BE 2 + EP 2 = 32 e AE 2 + EP 2 = 22 .

Subtraindo as equações acima, obtemos

BE 2 − AE 2 = 5.

Da mesma forma, usaremos o teorema de Pitágoras agora nos triângulos retângulos P CF e P DF ,


obtendo
CF 2 + F P 2 = 102 e DF 2 + F P 2 = P D2 .
Subtraindo as equações acima obtemos

CF 2 − DF 2 = 100 − P D2 .

Como AEF D e BEF C são retângulos, temos que AE = DF e BE = CF . Logo BE 2 − AE 2 =


CF 2 − DF 2 , o que implica que
100 − P D2 = 5,
e, portanto, √
PD = 95.

6. Chamemos de x o comprimento do lado do losango AF CE. No triângulo retângulo BCF temos


BC = 12, BF = AB − AF = 16 − x e CF = x, logo, pelo teorema de Pitágoras, temos

(16 − x)2 + 122 = x2 ,

donde concluı́mos
x = 12, 5.
Também temos F B = DE = 16 − x = 3, 5. Denotemos por G o pé da perpendicular traçada desde E
até o lado AB. Como ADEG é um retângulo, temos AG = DE = 3, 5 e EG = ED = 12.
Agora, no triângulo retângulo EF G conhecemos o comprimento dos catetos EG = 12 e F G =
AF − AG = 12, 5 − 3, 5 = 9. Usando o teorema de Pitágoras, encontramos
p
EF = 122 + 92 = 15.

Material elaborado por Susana Frómeta Fernández

5
Problemas Resolvidos

Nı́vel 2

Algumas propriedades de triângulos

Material elaborado por Susana Frómeta Fernández


Problemas

Problema 1. Sejam ABC um triângulo e M o ponto médio de BC. Se AM = BM = CM , prove


que ∠BAC = 90◦ .

Problema 2. (Torneio das cidades) Sejam ABCD um paralelogramo, M o ponto médio de CD e H


o pé da perpendicular baixada de B a AM . Prove que BCH é um triângulo isósceles.

Problema 3. Em um triângulo ABC, retângulo em A é isósceles, sejam D um ponto no lado AC


(A 6= C 6= D) e E o ponto no prolongamento de BA tal que o triângulo ADE é isósceles. Se P
é o ponto médio de BD, R o ponto médio de CE e Q a interseção entre ED e BC, prove que o
quadrilátero ARP Q é um quadrado.

Problema 4. Seja ABC um triângulo acutângulo tal que ∠B = 2∠C, AD é perpendicular a BC,
com D sobre BC, e E o ponto médio de BC. Prove que AB = 2DE.

Problema 5. (China) Seja ABCD um trapézio, AD k BC, ∠B = 30◦ , ∠C = 60◦ , E, M , F , N os


pontos médios de AB, BC, CD, DA, respectivamente. Se BC = 7, M N = 3, determine a medida de
EF .

Problema 6. (China) Seja ABCD um trapézio, AB k CD, ∠DAB = ∠ADC = 90◦ , e o triângulo
ABC é equilátero. Se a base média do trapézio EF = 43 a, determine o comprimento das duas bases
do trapézio, em função de a.

Problema 7. (Moscou) Seja ABCD um quadrilátero convexo e O um ponto em seu interior tal que
∠AOB = ∠COD = 120◦ , AO = OB, CO = OD. Sejam K, L, M os pontos médios de AB, BC, CD,
respectivamente. Prove que 4KLM é equilátero.

Problema 8. (OBM) No triângulo ABC, D é o ponto médio de AB e E ponto sobre o lado BC tal
que BE = 2 · CE. Sabendo que ∠ADC = ∠BAE, calculo o valor de ∠BAC.

Problema 9. Em um triângulo isósceles ABC, com AB = BC, sejam K, L pontos sobre AB, BC,
respectivamente, tais que AK + LC = KL. A reta paralela a BC passando pelo ponto médio M de
KL intersecta AC em N . Ache a medida de ∠KN L.

2
Soluções

1. Note que os triângulos 4ABM e 4ACM são isósceles com bases AB e AC, respectivamente. Logo
teremos que ∠ABM = ∠BAM e ∠ACM = ∠CAM . A soma dos ângulos interiores do 4ABC será
180◦ = ∠ABM + ∠BAM + ∠ACM + ∠CAM = 2(∠BAM + ∠CAM ) = 2∠BAC. Concluı́mos que
∠BAC = 90◦ .

2. Seja E o ponto de encontro das retas AM e BC. Como CM k AB e CM = CD AB


2 = 2 , temos
que CM é base média do 4ABE. Consequentemente teremos BC = CE. Note então que HC é
mediana relativa à hipotenusa do triângulo retângulo 4BHE, logo BC = HC = EC. Isso mostra
que o 4BCH é isósceles.

3. Pelo critério l.a.l. podemos ver que 4ABD ≡ 4ACE (AE = AD, AC = AB e ∠CAE = ∠BAD =
90◦ ). Veja então que AP e AR são medianas relativas à hipotenusa dos triângulos retângulos equi-
valentes ABD e ACE. Chamando de x o comprimento de AR e usando que a mediana relativa
à hipotenusa de um triângulo retângulo é igual à metade da hipotenusa, podemos concluir que
x = AR = ER = CR = AP = BP = DP .
Note também que QR e QP são as medianas relativas à hipotenusa dos triângulos retângulos
4EQC e 4BQD, respectivamente. Logo QR = CR = ER = x e QP = BP = DP = x.
Acabamos de mostrar que o quadrilátero ARP Q têm todos os lados iguais a x, ou seja, é um
losango. Mostraremos então que um dos seus ângulo internos, por exemplo ∠P AR, é reto. Para
ver isso, note que 4AER ≡ 4ADP (critério l.l.l.), então, em particular temos ∠EAR = ∠P AD.
Finalmente teremos ∠P AR = ∠DAR + ∠P AD = ∠DAR + ∠EAR = ∠DAE = 90◦ .

4. Chamemos ∠C de α, logo teremos que ∠B = 2α. Seja F o ponto médio de AB, então EF é base
média do 4ABC relativa ao lado AC, logo ∠BEF = ∠BCA = α.
Como DF é mediana relativa à hipotenusa do 4ABD retângulo, temos que AF = BF = DF .
Basta mostrar então que DF = DE.
Como 4F BD é isósceles (com BF = F D), temos que ∠F DB = ∠F BD = 2α. Como ∠F DB é
externo ao 4F DE, temos ∠F DE = ∠DEF + ∠DF E, ou seja, 2α = α + ∠DF E, donde concluı́mos
que ∠DF E = α. Logo 4DF E é isósceles com DE = DF como querı́amos mostrar.

5. Seja P o ponto de interseção de BA e CD. Olhando para a soma dos ângulos internos do 4BP C,
temos que ∠BP C = 90◦ .
Como AD k BC, a mediana P M do 4BP C corta o segmento AD também no seu ponto médio,
logo a interseção de P M com AD acontece justamente no ponto N .
Usando 4BP C é retângulo e que P M é mediana relativa à hipotenusa, temos que P M = BC2 =
3, 5. Logo P N = P M − M N = 3, 5 − 3 = 0, 5. Como P N também é mediana relativa à hipotenusa
do 4AP D retângulo, temos AD = 2P N = 2 × 0, 5 = 1.
Por último, a como EF é a base média do trapézio ABCD, temos que EF = BC+AD2 = 7+1
2 = 4.

6. Antes de resolver o problema, mostraremos o seguinte resultado: Num triângulo retângulo ABC
com ∠A = 90◦ , ∠B = 60◦ e ∠C = 30◦ . O cateto oposto ao ângulo de 30◦ é igual à metade da
hipotenusa, ou seja, AB = BC 2 . Para ver isso, considere o ponto médio M de BC. Logo teremos
AM = BM = CM . Como ∠B = 60◦ , o triângulo ABM , que já tı́nhamos que era isósceles, na
verdade será equilátero. Logo AB = BM = BC 2 , como querı́amos mostrar.

3
Vamos resolver agora o exercı́cio. Chamaremos de x o comprimento dos lados do 4ABC equilátero,
ou seja x = AB = BC = CA. Por serem ângulos alternos internos, temos que ∠ACD = ∠BAC = 60◦ .
Logo 4ACD é um triângulo retângulo onde os ângulos agudos medem 60◦ e 30◦ . Pelo resultado citado
no inı́cio, temos que CD = AC2 = 2.
x

Temos então  que a maior base do trapézio mede x e a menor mede x2 . Logo a base média será
3 1 x 3
4 a = 2 x + 2 = 4 x, donde concluı́mos que a = x.
Mostramos então que AB = a (base maior) e CD = a2 (base menor).

7. Veja que 4AOC ≡ 4BOD (critério l.a.l., pois AO = OB, ∠AOC = ∠BOD = 120◦ + ∠BOC e
CO = OD). Consequentemente teremos AC = BD. Como KL e LM são bases médias dos triângulos
ABC e BCD, respectivamente, temos que KL = AC BD
2 = 2 = LM . Para concluir, basta mostrar que
∠KLM = 60 . ◦

Chamemos de P o ponto de interseção das diagonais AC e BD. Note que, como KL k AC


e LM k BD temos que ∠KLM = ∠AP D. Logo será suficiente mostrar que ∠AP D = 60◦ , ou
equivalentemente, ∠CP D = 120◦ . Usando que 4AOC ≡ 4BOD, temos que, em particular, ∠ACO =
∠BDO, logo, olhando para a soma dos ângulos internos dos triângulos 4OCD e 4P CD, teremos
que ∠CP D = 120◦ , como querı́amos mostrar.

8. Começaremos mostrando uma propriedade clássica de triângulos:

Lema 1. As medianas de um triângulo se intersectam em um ponto, chamado baricentro, que divide


cada mediana em dois segmentos cujos comprimentos estão na razão de 2 para 1.

Demonstração. Considere um 4ABC. Sejam K, L, M os pontos médios dos lados BC, AC, AB.
AG
Seja G o ponto de interseção de AK e BL. Mostraremos que GK = BG
GL = 2. Para ver isso, note
AB
que KL é base média do 4ABC, logo KL k AB e KL = 2 . Os triângulos 4ABG e 4KLG são
semelhantes (veja que ∠BAG = ∠GKL e ∠ABG = ∠GLK por serem alternos internos), logo os seus
AG
lados se encontram na mesma proporção, ou seja, GK = BG AB
GL = KL = 2.
0
Analogamente, se chamarmos de G o ponto de interseção das medianas AK e CM , teremos que
AG0 0 0
G0 K = 2. Ou seja, G e G dividem o segmento AK na mesma proporção. Isso implica que G = G , o
que conclui a prova.

Vamos agora resolver o exercı́cio. Seja F o ponto no prolongamento de AC tal que C é o ponto
médio de AF . Veja então que BC é mediana do 4ABF relativa ao lado AF , e, como E é o ponto na
mediana que divide ela na proporção 2 : 1, temos que E é o baricentro do 4ABF . Chamando de G
o ponto de interseção de AE com BF , temos então que G é ponto médio de BF .
Veja que CG e GD são bases médias do 4ABF , logo ADGC é um paralelogramo. Chamemos
de H o ponto de interseção das diagonais AG e CD do paralelogramo ADGC, logo H é ponto médio
de AG e CD. Como ∠ADC = ∠BAE, temos que AH = HD, o que mostra que as diagonais do
paralelogramo ADGC são iguais. Isso implica que ADGC é um retângulo e, portanto, ∠BAC = 90◦ .

9. Seja P o ponto de interseção de AC com a reta que passa por K e é paralela a BC. Como
∠AP K = ∠ACB (ângulos correspondentes), temos que 4AKP é isósceles com AK = KP . Temos
que M N é a base média do trapézio KP CL, logo M N = KP +LC
2 = AK+LC
2 = KL
2 . Logo, no 4KN L
a mediana M N relativa ao lado KL é igual à metade de KL, o que implica que ∠KN L = 90◦ .

Material elaborado por Susana Frómeta Fernández

4
Problemas Resolvidos

Nı́vel 2

Quadriláteros Notáveis

Material elaborado por Susana Frómeta Fernández

1
Problemas

Problema 1. No quadrado ABCD consideram-se as diagonais AC e BD. Seja P um ponto qualquer


pertencente a um dos lados. Demonstrar que a soma das distâncias de P às duas diagonais é constante.

Problema 2. (Maio) No retângulo ABCD de lados AB, BC, CD e DA, seja P um ponto do lado
AD tal que ∠BP C = 90◦ . A perpendicular a BP traçada por A corta BP em M e a perpendicular
a CP traçada por D corta CP em N . Demonstre que o centro do retângulo está no segmento M N .

Problema 3. Sejam ABC e ABD triângulos com o lado AB comum. O triângulo ABC tem ∠BAC =
90◦ e AB = 2AC. O triângulo ABD tem ∠ADB = 90◦ e AD = BD. O segmento CD corta o segmento
AB em O. Calcule a medida de BO sabendo que AC = 4.

Problema 4. (OBM) O trapézio ABCD tem bases AB e CD. O lado DA mede x e o lado BC mede
2x. A soma dos ângulos ∠DAB e ∠ABC é 120◦ . Determine o ângulo ∠DAB.

Problema 5. No quadrilátero convexo ABCD, sejam E e F os pontos médios dos lados AD e BC,
respectivamente. Os segmentos CE e DF se cortam em O. Demonstre que se as retas AO e BO
dividem o lado CD em três partes iguais então ABCD é um paralelogramo.

Problema 6. Seja ABCD um paralelogramo tal que M é o ponto médio de BC. Seja T a projeção
de D sobre M A. Prove que CT = CD.

T
B A

C D

Problema 7. Prove que o segmento que liga os pontos médios dos lados opostos de um quadrilátero
convexo passa pelo ponto médio do segmento que liga os pontos médios das diagonais.

2
Soluções

1. Denotemos por M o ponto de interseção das diagonais AC e BD. Como ABCD é um quadrado,
temos que suas diagonais são iguais e se intersetam perpendicularmente no seu ponto médio. Denote-
mos por E e F os pés das perpendiculares desde P até AC e BD, respectivamente. Note que P EM F
é um retângulo e que 4P CE é isósceles e retângulo (pois, por ABCD ser um quadrado, temos que
∠P CE = 45◦ ). Finalmente, P F + P E = M E + EC = M C = AC 2 , que é independente da escolha do
ponto P .

2. Denotemos por O o ponto de interseção da diagonal BD com o segmento M N . Basta mostrar que
O é o ponto médio da diagonal BD.
Note que AM k CP e DN k BP , pois ∠AM B = ∠BP C = ∠CN D = 90◦ .
Note que ∠BAM = 90◦ − ∠ABM = ∠P BC = 90◦ − ∠P CB = ∠N CD. Um argumento similar
mostra também que ∠ABM = ∠CDN . Como, além disso, AB = CD, temos que 4ABM ≡ CDN ,
pelo caso a.l.a. Isso mostra, em particular que BM = N D. Como temos também que BM k N D,
temos que ∠M BO = ∠N DO e ∠BM O = ∠DN O, por serem alternos internos.
Logo, pelo caso a.l.a. temos que 4BM O ≡ 4DN O. Em particular, BO = OD, como querı́amos
mostrar.

3. Denotamos por x o comprimento de BO e por y o comprimento de AO. Temos que x+y = AB = 8.


Seja M no segmento AB tal que DM ⊥ AB. Como 4ABD é retângulo isósceles, temos que DM é
também mediana e, como a mediana relativa à hipotenusa de um triângulo retângulo é igual à metade
de hipotenusa, temos que DM = AM = BM = 4.
Como AC e DM são segmentos iguais e paralelos temos que 4ACO ≡ 4DM O. Consequen-
temente, AO = OM . Temos então 8 = AB = 2AM = 4AO = 4y, logo y = 2 e, portanto,
x = 8 − y = 8 − 2 = 6.

4. Seja E em AB tal que CE k AD. Note que AECD é um paralelogramo, logo CE = AD = x.


Também temos ∠CEB = ∠DAB por serem correspondentes. Encontraremos ∠CEB.
Veja que ∠BCE = 180◦ − (∠CEB + ∠EBC) = 180◦ − 120◦ = 60◦ . Seja F o ponto médio de BC,
temos BF = F C = x. Note que 4CEF é isósceles (CE = CF = x) e possui um ângulo igual a 60◦ ,
considerando a soma dos ângulo internos, concluı́mos que todos os ângulo do triângulo são iguais e
ele é, portanto, equilátero.
Finalmente temos EF = CF = BF = x o que implica que ∠CEB = 90◦ , pois se um triângulo
possui uma mediana igual à metade do lado correspondente, então o ângulo oposto a esse lado é reto.

5. Denotemos por G e H os pontos de interseção de DC com as retas BO e AO, respectivamente.


É suficiente mostrar que DEF C é um paralelogramo. Para isso, mostraremos que suas diagonais
se cortam no ponto médio, ou seja, que O é ponto médio de DF e CE.
Como H e F são pontos médios de CG e BC, respectivamente, temos que HF é base média do
4CGB, ou seja, HF k GB.
Usando que GO k HF e que G é ponto médio de DH, temos que GO é base média do 4DF H.
Isso mostra que O é ponto médio de DF .
Um argumento análogo mostra que O também é ponto médio de CE.

3
6. Seja E o ponto de interseção das retas AM e CD. Como AB k CE temos que os triângulos 4ABM
e 4CEM são semelhantes. A razão de semelhança é 1, pois BM = M C. Ou seja, 4ABM ≡ 4CEM .
Consequentemente temos AB = CE. Como ABCD é paralelogramo, temos AB = CD. Logo T C
é mediana do 4T ED. Como 4T ED é retângulo, temos que a mediana relativa à hipotenusa é igual
à metade da hipotenusa. Logo CT = CE = CD.

7. Considere o quadrilátero ABCD representado na figura. Sejam E e F os pontos médios dos lados
AB e CD, respectivamente, e sejam M e N os pontos médios das diagonais AC e BD, respectivamente.

A
M
C

Note que EN é base média do 4ABD relativa ao lado AD e que F M é base média do 4ACD
relativa ao lado AD. Logo EN e M F são iguais e paralelos, o que implica que EM F N é um
paralelogramo e, portanto, suas diagonais (EF e M N ) se cortam em seu ponto médio.

Material elaborado por Susana Frómeta Fernández

4
Problemas Resolvidos

Nı́vel 2

Trabalhando com ângulos

Material elaborado por Susana Frómeta Fernández

1
Problemas

Problema 1. (Torneio das Cidades 1994) No triângulo ABC, retângulo em C, os pontos M e N


são escolhidos sobre a hipotenusa de modo que BN = BC e AM = AC. Ache a medida do ângulo
∠N CM .

Problema 2. Seja ABCDEF um hexágono com todos os ângulos internos iguais a 120◦ . Mostre que

AB − DE = CD − AF = EF − BC.

Problema 3. Seja RST U V pentágono regular. Construa um triangulo equilátero P RS com P no


interior do pentágono. Ache a medida do ângulo ∠P T V .

Problema 4. No trapézio ABCD, de bases AB e CD, temos AD = 39, CD = 14, ∠ABC = 69◦ e
∠CDA = 138◦ . Ache a medida de AB.

Problema 5. (OBM) No retângulo ABCD, E é o ponto médio do lado BC e F é o ponto médio


do lado CD. A interseção de DE com F B é G. O ângulo ∠EAF mede 20◦ . Quanto vale o ângulo
∠EGB?

D F C

G
E

A B

Problema 6. DEF G é um quadrado no exterior do pentágono regular ABCDE. Quanto mede o


ângulo ∠EAF ?

Problema 7. No triângulo ABC, D e E são pontos sobre os lados BC e AC respectivamente.


Determine ∠CDE sabendo que AB = AC, AE = AD e ∠BAD = 48◦ .

2
Problema 8. Determine ∠BAC na figura abaixo sabendo que AB = AC e BC = CD = DE =
EF = F A.

C B

Problema 9. No triângulo ABC com AB = BC, ∠ABC = 144◦ . Seja K um ponto em AB, L um
ponto em BC e M em AC de modo que KL k AC, KM k BC e KL = KM . A reta LM corta o
prolongamento de AB em P . Ache a medida do ângulo ∠BP L.

Problema 10. No triângulo ABC com AB = BC, P , Q e R são pontos nos lados AC, BC e AB,
respectivamente tais que P Q k AB, RP k BC e RB = AP . Se ∠AQB = 105◦ , ache as medidas dos
ângulos do 4ABC.

Problema 11. BE e AD são as alturas do triângulo ABC, H é o ortocentro e F , G, K são os pontos


médios dos segmentos AH, AB, BC, respectivamente. Prove que ∠F GK é reto.

Problema 12. Em um triângulo ABC temos que ∠ABC = 37◦ e ∠ACB = 38◦ . Sejam P e Q pontos
sobre o lado BC tais que ∠BAP = ∠P AQ = ∠QAC. Se traça por B uma paralela a AP e por C
uma paralela a AQ. O ponto de encontro destas duas retas é D. Calcule ∠BDC.

Problema 13. (Maio 1996) Seja ABCD um quadrado e F um ponto qualquer do lado BC. Traça-se
por B a perpendicular à reta DF que corta a reta DC em Q. Quanto mede o ângulo ∠F QC?

3
Soluções

1. Digamos que α = ∠BAC e β = ∠ABC. Como o 4ABC é retângulo, temos que α + β = 90◦ .
Como o 4ACM é isósceles com base CM , temos que ∠ACM = ∠AM C = 90◦ − α2 . Da mesma
forma, como o 4BCN é isósceles com base CN , temos que ∠BCN = ∠BN C = 90◦ − β2 .
Considerando a soma dos ângulos internos do 4CM N , temos

90◦
 
α β α+β
∠N CM = 180◦ − (∠CM N + ∠CN M ) = 180◦ − 90◦ − + 90◦ − = = = 45◦ .
2 2 2 2

2. Prolongamos os lados AF e BC do hexágono até se encontrarem num ponto P , da mesma maneira


as retas BC e DE se encontram no ponto Q e as retas ED e AF se encontram no ponto R.

A B

R E D Q

Note que ∠P AB = 180◦ − ∠BAF = 180◦ − 120◦ = 60◦ . Analogamente, ∠P BA = ∠QCD =


∠QDC = ∠REF = ∠RF E = 60◦ .
Assim, como os ângulos internos dos triângulos 4P AB, 4QCD e 4REF são todos iguais a 60◦ ,
estes triângulos são equiláteros. Além disso, o 4P QR também é equilátero.
Chamemos de a, b, c e `, o comprimento dos lados dos triângulos 4P AB, 4QCD, 4REF e
4P QR, respectivamente.
Veja que DE = RQ − RE − DQ = ` − b − c, logo

AB − DE = a − (` − b − c) = a + b + c − `.

Usando exatamente o mesmo argumento, teremos que

CD − AF = b − (` − a − c) = a + b + c − `

e que
EF − BC = c − (` − a − b) = a + b + c − `.

3. Os ângulos internos de um pentágono regular são todos iguais a 108◦ . Logo ∠P ST = 108◦ −
∠RSP = 108◦ − 60◦ = 48◦ . Veja agora que o 4P ST é isósceles com P S = ST , logo considerando
a soma dos ângulos internos desse triângulo, concluı́mos que ∠SP T = ∠ST P = 66◦ . Analogamente,
teremos que o 4RV P também é isósceles, e os seus ângulos internos medem ∠V RP = 48◦ , ∠RV P =
∠RP V = 66◦ .

4
Observe que os triângulos 4RV P e 4SP T são congruentes (podemos usar o critério l.a.l., com
∠V RP = ∠P ST = 48◦ e RV = RP = SP = ST ). Consequentemente, teremos que V P = P T , o
que implica que 4V P T é isósceles. Agora veja que ∠V P T = 360◦ − ∠V P R − ∠RP S − ∠SP T =
360◦ − 66◦ − 60◦ − 66◦ = 168◦ . Logo, considerando a soma dos ângulos internos do 4V P T temos que
180◦ − 168◦
∠P T V = ∠P V T = = 6◦ .
2

4. Seja E o ponto de interseção das retas AD e BC, conforme mostra a figura. Note que ∠EDC =
180◦ − ∠ADC = 180◦ − 138◦ = 42◦ , e que ∠DCE = ∠ABE = 69◦ (por serem correspondentes)
No 4CDE, temos
∠DEC = 180◦ − ∠EDC − ∠ECD = 180◦ − 42◦ − 69◦ = 69◦ ,
o que implica que 4CDE é isósceles com DE = DC = 14.
Como o 4ABE também tem dois ângulos iguais (a 69◦ ), temos que ele também é isósceles com
base AB = AE = AD + DE = 39 + 14 = 53.

5. Pelo critério de l.a.l. temos que os triângulos 4ABE e 4DCE são congruentes (AB = CD,
BE = EC e ∠ABE = ∠DCE = 90◦ ), logo, consequentemente teremos que ∠BAE = ∠CDE.
Analogamente teremos que os triângulos 4ADF e 4BCF são congruentes (AD = BC, DF = F C e
∠ADF = ∠BCF = 90◦ ), o que implica que ∠DAF = ∠CBF .
Note que
∠CDE + ∠CBF = ∠ABE + ∠DAF = ∠BAD − ∠EAF = 90◦ − 20◦ = 70◦ .
Considerando a soma dos ângulos internos do 4BCD, temos
∠GBD + ∠GDB = 180◦ − ∠BCD − (∠CDE + ∠CBF ) = 180◦ − 90◦ − 70◦ = 20◦ .
Como ∠BGE é ângulo externo do 4BDG, temos
∠BGE = ∠GBD + ∠GDB = 20◦ .

6. Os ângulos internos de um pentágono regular são todos iguais a 108◦ . Veja que
∠AEF = 360◦ − ∠AED − ∠DEF = 360◦ − 108◦ − 90◦ = 162◦ .
Como ABCDE é um pentágono regular e DEF G um quadrado, temos que AE = ED = EF .
Portanto 4AEF é isósceles com base AF . Então
180◦ − ∠AEF 180◦ − 162◦
∠EAF = ∠EF A = = = 9◦ .
2 2

7. Digamos que o ∠CAD mede α. Como 4ABC é isósceles com base BC, temos que
180◦ − ∠BAC 180◦ − (48◦ + α) α
∠ACB = ∠ABC = = = 66◦ − .
2 2 2
Como 4ADE é isósceles com base DE, temos
180◦ − ∠DAE 180◦ − α α
∠AED = ∠ADE = = = 90◦ − .
2 2 2
Veja agora que ∠CED = 180◦ − ∠AED = 180◦ − (90◦ − α2 ) = 90◦ + α2 . Considerando a soma dos
ângulo interno do 4CDE, temos
α α
∠CDE = 180◦ − (∠CED + ∠ECD) = 180◦ − (90◦ + + 66◦ − ) = 180◦ − (90◦ + 66◦ ) = 24◦ .
2 2

5
8. Chamemos de α a medida do ∠BAC. Como 4AEF é isósceles com base AE, temos que ∠AEF =
∠EAF = α.
Como ∠DF E é ângulo exterior ao 4AEF , temos ∠DF E = ∠AEF + ∠EAF = 2α. Veja que
4DEF é isósceles com base DF , logo ∠F DE = ∠DF E = 2α.
Como ∠CED é ângulo exterior ao 4ADE, temos ∠CED = ∠EAD + ∠EDA = α + 2α = 3α.
Também, como o 4CDE é isósceles com base CE, temos ∠ECD = ∠CED = 3α.
Como ∠BDC é ângulo exterior ao 4ACD, temos ∠BDC = ∠CAD + ∠ACD = α + 3α = 4α.
Além disso, como o 4BCD é isósceles com base BD, temos que ∠CBD = ∠CDB = 4α.
Por outro lado, como o 4ABC é isósceles com base BC, temos que ∠ABC = ∠ACB = 4α.
Finalmente, considerando a soma dos ângulos internos do 4ABC, temos que α + 4α + 4α = 180◦ ,
ou seja 9α = 180◦ . Donde concluı́mos que α = 20◦ .

9. Como 4ABC é isósceles e ∠ABC = 144◦ , temos que ∠BAC = ∠BCA = 18◦ . Como KL k AC,
∠BKL = ∠BAC = 18◦ .
Veja também que CLKM é um paralelogramo (os lados opostos são paralelos), logo ∠LKM =
∠LCM = 18◦ . Como 4KLM é isósceles, temos que ∠KM L = ∠KLM = 81◦ . Logo
∠KP M = 180◦ − ∠P KM − ∠P M K = 180◦ − 36◦ − 81◦ = 63◦ .

10. Denotemos por α = ∠ABC e β = ∠BAQ. Veja que


α + β = 180◦ − ∠AQB = 180◦ − 105◦ = 75◦ . (1)
Como P Q k AB e RP k BC, temos que ∠AQP = ∠BAQ = β e que BQP R é um paralelogramo.
Logo RB = P Q. Como é dado que RB = AP , temos que AP = P Q e, portanto, 4AP Q é isósceles
com base AQ. Consequentemente, ∠P AQ = ∠AQP = β.
Assim, teremos que ∠BAC = ∠BAQ + ∠P AQ = 2β e, como 4ABC é isósceles, temos que
∠BCA = 2β. Considerando a soma dos ângulos internos do 4ABC, temos
α + 4β = 180◦ . (2)
Resolvendo o sistema de equações composto pelas equações (1) e (2), obtemos que α = 40◦ e
β = 35◦ . Donde obtemos os ângulos do 4ABC: ∠ABC = α = 40◦ e ∠BAC = ∠BCA = 2β = 70◦ .

11. Note que GK e F G são bases médias dos triângulos 4ABC e 4ABH, respectivamente. Logo
GK k AC e F G k BH.
Seja I a interseção de AC com o prolongamento de GF e seja J a interseção de GK com BE.
Note que no quadrilátero GIEJ os lados opostos são paralelos e que ∠IEJ = 90◦ , o que implica que
GIEJ é um retângulo. Assim ∠F GK = IGJ = 90◦ .

12. Note que ∠BAC = 180◦ − 37◦ − 38◦ = 105◦ , logo ∠BAP = ∠P AQ = ∠QAC = 105 ◦
3 = 35 .
Como BD k AP e CD k AQ, temos que ∠CBD = ∠QP A e ∠BCD = ∠P QA (por serem
correspondentes). Logo

∠BDC = 180◦ − (∠CBD + ∠BCD) = 180◦ − (∠QP A + ∠P QA) = ∠P AQ = 35◦ .

13. Note que F é o ortocentro do 4BDQ, logo o prolongamento de QF corta perpendicularmente


BD, chamemos de E esse ponto de interseção. Como BD é diagonal do quadrado ABCD, temos que
∠BDC = 45◦ . Veja que
∠F QC = ∠DQE = 180◦ − ∠DEQ − ∠EDQ = 180◦ − 90◦ − 45◦ = 45◦ .

Material elaborado por Susana Frómeta Fernández

6
Problemas Resolvidos

Nı́vel 2

Ângulos na circunferência

Material elaborado por Susana Frómeta Fernández


Problemas

Problema 1. São dadas duas circunferências secantes, com pontos de intersecção C e D. Traça-se
por C uma secante às duas circunferências, que intersecta uma delas em E e a outra em F . Mostre
que o ângulo ∠EDF é constante.

Problema 2. As extremidades de uma corda ST , com comprimento constante, são movidos ao longo
de um semicı́rculo com diâmetro AB. Seja M o ponto médio de ST e P o pé da perpendicular de S
sobre AB. Prove que a medida do ângulo ∠SP M independe da posição de ST .

Problema 3. É dado um triângulo ABC. Sejam O o centro da circunferência circunscrita ao


triângulo, I o centro da circunferência inscrita ao triângulo, D 6= A a intersecção da reta AI com
a circunferência circunscrita. Prove que CD = BD = ID.

Problema 4. Se os lados AB e AC de um triângulo são diâmetros de duas circunferências, prove que


o ponto comum às circunferências está em BC.

Problema 5. Sejam C1 e C2 duas circunferências tangentes exteriores em T . Sejam A e B pontos de


C1 tais que a reta AB é tangente a C2 em P . Prove que T P é bissetriz externa do triângulo 4AT B.

Soluções

1. Chame de S1 a circunferência que contém o ponto E e de S2 a circunferência que contém o ponto


F . Considere o triângulo 4EDF . Temos que ∠EDF = 180◦ − (∠CED + ∠CF D). Basta mostrar que
∠CED + ∠CF D é constante. Veja que ∠CED é ângulo inscrito na circunferência S1 e que ∠CF D é
ângulo inscrito na circunferência S2 , logo ambos ângulos dependem apenas dos pontos C e D.

2. Seja O o ponto médio de AB. Veja que O é também o centro da circunferência de diâmetro AB.
O 4SOT é isósceles com OS = OT , logo OM é bissetriz do ∠SOT . Também, como ∠SOT é ângulo
_ _
central temos que ∠SOT = ST e ∠SOM = ST 2 (note que esse valor depende do comprimento fixo da
corda ST e não muda quando S e T são movidos).
Como ∠SP O = ∠SM O = 90◦ , a circunferência Γ circunscrita ao 4SP O também contém o ponto
M e SO é diâmetro dessa circunferência. Note que os ângulos ∠SP M e ∠SOM são inscritos no
_
mesmo arco SM da circunferência Γ, logo são ângulos iguais.
Veja outra solução aqui.

3. Lembre que o centro da circunferência inscrita a um triângulo é o ponto de interseção das bissetrizes.
Como AD é bissetriz do ∠BAC e os ângulos ∠BAD e ∠CAD são inscritos na circunferência
_ _
circunscrita ao 4ABC, temos que CD = DB , donde concluı́mos que CD = DB. Provaremos que
4CDI é isósceles com CD = ID.
Seja E 6= C a intersecção da reta CI com a circunferência circunscrita. Como CE também é
_ _
bissetriz do ∠ACB temos que AE = EB . Como ∠DCE é ângulo inscrito e ∠CID é ângulo excêntrico
_ _ _ _
interior, temos que ∠DCE = DB + BE = CD + AE = ∠CID.
2 2 2 2

2
4. Seja D o ponto de intersecção de BC com a circunferência de diâmetro AB, e seja E o ponto de
intersecção de BC com a circunferência de diâmetro AC. Usando que AB e AC são diâmetros das
respectivas circunferências, temos ∠ADB = 90◦ e também ∠AEC = 90◦ . Note AD e AE são alturas
do 4ABC relativas à base BC, logo os pontos D e E coincidem.

5. Sejam M e N os pontos de intersecção de AT com a circunferência C2 e de AP com a reta tangente


a C1 e C2 em T , respectivamente.
_
Note que ∠N P T e ∠N T P são ângulos de segmento relativos ao arco P T da circunferência C2 e
_
que ∠P M T é inscrito no arco P T . Temos então que ∠N P T = ∠N T P = ∠P M T .
Por outro lado, temos que ∠N T B e ∠BAT são, respectivamente, ângulo de segmento e ângulo
_
inscrito relativo ao arco BT da circunferência C1 . Temos então ∠N T B = ∠BAT .
Como ∠P T M é ângulo externo do 4AP T , temos ∠P T M = ∠AP T +∠P AT = ∠N T P +∠N T B =
∠BT P , como querı́amos provar.

Material elaborado por Susana Frómeta Fernández

3
Problemas Resolvidos

Nı́vel 2

Quadriláteros Inscritı́veis

Material elaborado por Susana Frómeta Fernández

1
Problemas

Problema 1. Seja ABCD um paralelogramo. A bissetriz de ∠BAD corta BC em M e o prolonga-


mento de CD em N . Se O é o circuncentro do triângulo 4M CN , mostre que o quadrilátero BOCD
é inscritı́vel.

Problema 2. (BAMO) Seja k um cı́rculo no plano xy com centro sobre o eixo y e passando pelos
pontos A = (0, a) e B = (0, b) com 0 < a < b. Seja P um ponto qualquer do cı́rculo, diferente de A
e B. Seja Q a intersecção da reta que passa por P e A com o eixo x, e seja O = (0, 0). Prove que
∠BQP = ∠BOP .

Problema 3. (OBM) As diagonais de um quadrilátero inscritı́vel ABCD se intersectam em O. Os


cı́rculos circunscritos aos triângulos AOB e COD intersectam as retas BC e AD, pela segunda vez,
nos pontos M , N , P e Q. Prove que o quadrilátero M N P Q está inscrito em um cı́rculo de centro O.

Problema 4. (Ibero) Num triângulo escaleno ABC traça-se a bissetriz interna BD, com D sobre
AC. Sejam E e F , respectivamente, os pés das perpendiculares traçadas desde A e C até a reta BD,
e seja M o ponto sobre o lado BC tal que DM é perpendicular a BC. Prove que ∠EM D = ∠DM F .

Problema 5. Seja M o ponto de interseção das diagonais de um quadrilátero inscritı́vel ABCD, em


que ∠AM B é agudo. O triângulo isósceles BCK é construı́do exteriormente ao quadrilátero, com a
base sendo BC, tal que ∠KBC + ∠AM B = 90◦ . Prove que KM é perpendicular a AD.

Problema 6. (Cone Sul) Seja ABCD um quadrado (os vértices estão nomeados no sentido horário)
e P um ponto qualquer pertencente ao interior do segmento BC. Constrói-se o quadrado AP RS (os
vértices novamente nomeados no sentido horário). Demonstrar que a reta CR é tangente à circun-
ferência circunscrita ao triângulo ABC.

Problema 7. (IMO) Duas circunferências Γ1 e Γ2 intersectam-se em M e N . Seja ` a tangente


comum a Γ1 e Γ2 que está mais próxima de M do que de N . A reta ` é tangente a Γ1 em A e a Γ2 em
B. A reta paralela a ` que passa por M intersecta novamente a circunferência Γ1 em C e novamente
a circunferência Γ2 em D. As retas CA e DB intersectam-se em E; as retas AN e CD intersectam-se
em P ; as retas BN e CD intersectam-se em Q. Mostre que EP = EQ.

2
Soluções

1. Denotemos por α os ângulos α = ∠BAN = ∠N AD. Como ABCD é paralelogramos temos que
∠M N C = ∠BAN = α e ∠CM N = ∠N AD = α. Isso implica que 4M CN é isósceles e, portanto CO
é mediatriz, bissetriz e altura do triângulo M CN relativos ao lado M N . Denotemos por β os ângulos
β = ∠M CO = ∠OCN .
Usando que O é o circuncentro de 4M CN temos que OC = OM e que ∠M OC = 2∠M N C = 2α
pois ∠M OC é o ângulo central relativo ao mesmo arco do circuncı́rculo de 4M CN onde ∠M N C
está inscrito. Veja também que ∠DCO = ∠DCB + ∠BCO = 2α + β e que ∠BM O = 2α + β por
ser ângulo externo do 4M OC. Concluı́mos que ∠DCO = ∠BM O. Por último, veja que 4ABM
também é isósceles por ter dois ângulos iguais a α, logo M B = AB = DC. Então podemos concluir,
pelo critério l.a.l., que 4DCO ≡ 4BM O.
Em particular, como 4DCO ≡ 4BM O, temos que ∠CBO = ∠CDO, o que conclui a prova de
que BOCD é inscritı́vel.

2. Seja R 6= B o ponto de intersecção de BQ com o cı́rculo k. Como AB é diâmetro do cı́rculo k,


temos que ∠BRA = ∠BP A = 90◦ . Seja S o ponto de interseção das retas BP e QO. Note que A é
o ortocentro do 4BQS, pois é o ponto de intersecção das alturas BO e QP . Por tanto SA e BQ se
intersetam formando um ângulo reto. Como AR e BQ são perpendiculares, teremos que R é o pé da
altura do 4BQS correspondente ao vértice S (ou seja, S, A e R estão alinhados).
Considerando a soma do ângulos internos dos triângulos retângulos 4BP Q e 4BRS temos que
∠BQP = 90◦ − ∠P BQ = 90◦ − ∠RBS = ∠BSR.
O AOSP é inscritı́vel pois possui um par de ângulos opostos iguais a 90◦ (ou seja, a soma dos
ângulos opostos é 180◦ ). Consequentemente, temos ∠ASP = ∠AOP = ∠BOP , o que conclui a prova.

3. Usando que os quadriláteros ABM N , ABCD e CDP Q são inscritı́veis (conforme mostra a figura),
temos que ∠P N M = ∠AN M = 180◦ − ABM = ∠ABC = 180◦ − ADC = ∠CDP = 180◦ − CQP =
180◦ − M QP . Isso mostra que os ângulos opostos do quadrilátero M N P Q somam 180◦ , logo é um
quadrilátero inscritı́vel.

D P
N A

M
B
C
Q

3
Usando que os quadriláteros AN BO, ABCD e DP CO são inscritı́veis, temos que ∠P N O =
∠AN O = ∠ABO = ∠ABD = ∠ACD = ∠OCD = ∠OP D = ∠OP N . Logo 4ON P é isósceles com
ON = OP . Analogamente, também vale OM = OQ.
Para finalizar, basta mostrar que ON = OM . Para isso, veja que ∠N M O = 180◦ − ∠N AO =
∠CAD = ∠CBD e também que ∠N M O = ∠N BO. Isso mostra que BO é bissetriz do ∠N BC.
Analogamente teremos também que AO é bissetriz do ∠M AD, ou seja, ∠M AO = ∠OAD. Como
∠M AO = ∠M N O, obtemos finalmente que ∠M N O = ∠N M O, o que conclui a prova.

4. Como ∠CF D = ∠CM D = 90◦ temos que o quadrilátero CF DM é inscritı́vel, logo temos ∠DM F =
∠DCF . Note que ∠DCF = ∠DAE por serem alternos internos, uma vez que CF k AE.
Agora considere o ponto N na reta AB tal que AB ⊥ N D. Note que, como ∠AED = ∠AN D, o
quadrilátero AN ED é inscritı́vel, logo teremos ∠DAE = ∠DN E.
Mostramos então que ∠DM F = ∠DN E. Agora, como BD é bissetriz de ∠ABC e M e N são,
respectivamente, os pés das perpendiculares traçadas desde D até as retas BC e BA, temos que
DM = DN e ∠M DE = ∠N DE. Logo teremos 4M DE ≡ 4N DE (critério l.a.l., considerando que
DE é um lado comum a ambos triângulos). Consequentemente teremos ∠EM D = ∠EN D, o que
conclui a prova.

5. ∠BKC = 180◦ − 2∠KBC = 2(90◦ − ∠KBC) = 2∠AM B. Isso, junto com o BK = KC, implica
_
que ∠BKC é ângulo central correspondente ao arco BM C da circunferência circunscrita ao 4BCM .
Consequentemente teremos KB = KM = KC e, em particular, 4BKM é isósceles, com ∠KBM =
∠KM B.
Seja N o ponto de interseção de KM com AD. Queremos mostrar que ∠M N D = 90◦ .
Como ABCD é um quadrilátero inscritı́vel, temos que ∠ADB = ∠ACB. Logo teremos ∠N DM +
∠N M D = ∠ACB + ∠BM K = ∠ACB + ∠M BK = ∠M CB + (∠M BC + ∠KBC) = (∠M CB +
∠M BC) + ∠KBC = ∠AM B + ∠KBC = 90◦ , o que conclui a prova.

6. A circunferência circunscrita ao 4ABC é também a circunferência circunscrita ao quadrado ABCD.


O centro dessa circunferência é o ponto de interseção das diagonais do quadrado. Portanto, basta
mostrar que ∠ACR = 90◦ .
Como AC e AR são diagonais dos quadrados ABCD e AP RS, respectivamente, temos que
∠ACB = ∠ARP = 45◦ . Logo o quadrilátero AP CR é inscritı́vel (pois ∠ACP = ∠ARP ) e, por-
tanto ∠ACR = ∠AP R = 90◦ , como querı́amos mostrar.

No próximo exercı́cio vamos usar o seguinte resultado:

Lema 1. Seja Γ uma circunferência e A um ponto externo a Γ. Traçamos por A uma reta tangente a
Γ que intersecta ela no ponto B e uma outra reta que intersecta Γ em dois pontos (distintos) C e D,
conforme mostra a figura. Então AB 2 = AC · AD.

4
B

A
C

Demonstração. Note que ∠ABC = ∠BDC por serem, respectivamente, ângulo de segmento relativo
_
à circunferência Γ e ângulo inscrito, ambos correspondentes ao arco BC . Logo os triângulos 4ABC
e 4ABD são semelhantes (note que ∠BAC é comum a ambos triângulos).
AB AC
Vale então a razão de semelhança: AD = AB , donde obtemos AB 2 = AC · AD.

7. Veja que ∠ABM = ∠BDM , pois são, respectivamente, ângulo de segmento e ângulo inscrito rela-
_
tivos ao arco M B na circunferência Γ2 . Como ∠EBA = ∠BDM por serem ângulos correspondentes,
teremos que BA é bissetriz do ∠EBM . Analogamente temos também que AB é bissetriz do ∠EAM .
Pelo critério a.l.a., temos que 4AEB ≡ 4AM B. Consequentemente, os triângulos 4AEM e 4BEM
são isósceles, ambos com base EM . Então a reta AB, além de ser bissetriz dos ângulos ∠EAM e
∠EBM , ela também corta o segmento EM no seu ponto médio e formando um ângulo de 90◦ . Como
AB k CD, temos que ∠EM D = 90◦ . Basta mostrar que P M = M Q.
Seja R o ponto de interseção de M N com AB. Como AB é tangente às circunferências Γ1 e Γ2 .
Usando o Lemma 1 em ambas circunferências, temos que AR2 = RM · RN e BR2 = RM · RN , donde
concluı́mos que AR = RB.
Como AB k P Q, vamos ter P M = M Q. E isso conclui a nossa prova.

Material elaborado por Susana Frómeta Fernández

5
Problemas Resolvidos

Nı́vel 2

Relações métricas no triângulo

Material elaborado por Susana Frómeta Fernández

1
Problemas

Problema 1. (OCM) Se as diagonais de um quadrilátero (convexo) são perpendiculares, prove que


as somas dos quadrados dos lados opostos são iguais.

Problema 2. (OCM) Dobra-se um pedaço de arame de 32 cm de comprimento formando um triângulo


isósceles de 12 cm de base. Calcule a medida do comprimento da bissetriz do ângulo oposto à base.

Problema 3. (OBM) P é um ponto interior a um quadrado ABCD. As distâncias de P aos vértices


A e D e ao lado BC são iguais a 10 cm. Encontre o comprimento do lado do quadrado.

Problema 4. Num triângulo com ângulos iguais a 90◦ , 60◦ e 30◦ , o cateto oposto ao ângulo de 30◦
mede a metade da medida de hipotenusa.

Problema 5. Um ponto P , interno de um ângulo de 60◦ , está a uma distância de 6 e 9 dos lados
desse ângulo. Qual a distância entre P e a bissetriz do ângulo?

Problema 6. Seja ABCD um quadrado de lado 1. Sejam E e F pontos nos lados BC e CD,
respectivamente, tais que ∠DAF = ∠BAE = 15◦ .

a) Mostre 4AEF é equilátero.

b) Encontre o comprimento de BE.

c) Deduza tan 15◦ e tan 75◦ .

Problema 7. Seja ABC um triângulo tal que ∠ABC = 45◦ . Seja D o ponto sobre o segmento BC
tal que 2BD = CD e ∠DAB = 15◦ . Determine o ângulo ∠ACB.

Problema 8. (AIME) Seja ABC um triângulo tal que AB = 13, BC = 15 e CA = 14. Seja D o
ponto do segmento BC tal que CD = 6. Seja E o ponto de BC tal que CE > CD e ∠BAE = ∠CAD.
Determine BE.

Problema 9. No triângulo ABC, ∠ABC = 20◦ e AB = AC. Os pontos M e N estão sobre os lados
AB e AC, respectivamente, e são tais que ∠BCM = 50◦ e ∠CBN = 60◦ . Calcule a medida do ângulo
∠M N B.

2
Soluções

1. Chamemos de A, B, C e D, os vértices do quadrilátero, e chamemos de P o ponto de interseção


das suas diagonais (AC e BD).
Aplicando o teorema de Pitágoras nos triângulos 4ABP , 4DCP , 4ADP e 4BCP , temos

AB 2 + DC 2 = (AP 2 + BP 2 ) + (CP 2 + DP 2 ) = (AP 2 + DP 2 ) + (CP 2 + BP 2 ) = AD2 + BC 2 .

2. Chamemos de A, B e C os vértices do triângulo isósceles formado pelo arame dobrado. Seja BC


a base do triângulo. Temos então BC = 12 cm e AB = AC = 10 cm. Seja D em BC tal que AD é
bissetriz do ∠BAC. Queremos calcular o comprimento de AD.
Como 4ABC é isósceles, AD também será altura e mediana do triângulo. Aplicando o teorema
de Pitágoras no 4ADC, obtemos

AD2 = AC 2 − CD2 = 102 − 62 = 64.

Logo AD = 8 cm.

3. Seja E o pé da perpendicular traçada desde P até o lado BC. Seja F a interseção da reta EP com
o segmento AD.
Chamemos de x o comprimento do lado do quadrado. Veja que P E = 10 cm e P F = x − 10 cm.
Aplicando o teorema de Pitágoras no 4DF P , obtemos
 x 2
(x − 10)2 + = 102 .
2
O que implica que
5x2 − 80x = 0.
Logo x = 0 ou x = 16. Como x é estritamente positivo, obtemos que o comprimento do lado do
quadrado é 16 cm.

4. Sejam A, B e C os vértices correspondentes aos ângulos de 90◦ , 60◦ e 30◦ , respectivamente.


Prolongamos o cateto AB e marcamos o ponto B 0 tal que A é ponto médio de BB 0 , como mostra a
figura.

30◦

60◦

B0 A B

3
Os triângulos ABC e AB 0 C são congruentes e, como ∠CBB 0 = ∠CB 0 B = 60◦ , temos que 4BB 0 C
é equilátero. Assim, temos que CA é altura, bissetriz e mediana do 4BB 0 C. Logo
BC = BB 0 = 2 · AB.
Perceba que essa construção demonstra que sen 30◦ = 12 . Com essa mesma construção e o teorema

de Pitágoras também se mostra que cos 30◦ = 3
2 .

5. Chamemos de A o vértice do ângulo de 60◦ , e sejam B e C, respectivamente, os pés das perpendi-


culares traçadas a partir de P até os lados do ângulo, tal que P B = 6 e P C = 9. Chamemos de Q o
pé da perpendicular traçada desde P até a bissetriz do ∠BAC. Queremos encontrar o comprimento
de P Q. Vamos denotar P Q = x.
Sejam E e F , respectivamente, os pontos de encontro da bissetriz AQ com as retas BP e CP . Note
que ∠AEB = 180◦ − (∠ABE + ∠BAE) = 180◦ − (90◦ + 30◦ ) = 60◦ . E, analogamente, ∠AF C = 60◦ .
Então temos que 4EF P é equilátero. Sua altura, P Q é igual a x. Chamando de a o comprimento
do lado desde triângulo, e usando Pitágoras em 4F P Q, obtemos
 a 2
x2 + = a2 .
2
Donde concluı́mos que a = √23 x. Agora, note que os triângulos 4ABE e 4ACF são retângulos com
um ângulo igual a 30◦ e o outro igual a 60◦ . Então, usando o problema anterior, temos que
 
2 4
AE = 2 · BE = 2 · 6 + √ x = 12 + √ x
3 3
e  
2 4
AF = 2 · F C = 2 · (P C − P F ) = 2 · 9 − √ x = 18 − √ x.
3 3
Por último, como AF = AE − F E, temos
4 4 2 2
18 − √ x = 12 + √ x − √ x = 12 + √ x,
3 3 3 3

donde concluı́mos que x = 3.

6. ,
(a) Como ∠BAE = ∠DAF e AB = AD, pelo critério a.l.a. temos que 4ABE ≡ 4ADF . Assim,
4AF E é isósceles e como ∠EAF = 60◦ , concluı́mos que 4AEF é equilátero.
(b) Chamemos de a o comprimento dos segmentos BE = DF , logo o comprimento dos segmentos
CE = CF é igual a 1 − a.

D a F 1−a C

1−a

15◦

E
60◦ 75◦ a
15◦
A B

4
Usando o teorema de Pitágoras no 4ABE temos

AE 2 = a2 + 12 .

Usando o teorema de Pitágoras no 4CEF , temos

EF 2 = (1 − a)2 + (1 − a)2 .

Como AE = EF , temos
a2 + 1 = 2(1 − a)2 .

Resolvendo esta
√ equação, encontramos as soluções 2 ± 3. Como devemos ter a < 1, concluı́mos
que a = 2 − 3.
c) Para responder este item, vamos olhar para o 4ABE:
BE √
tan(15◦ ) = =2− 3
AB √
◦ AB 1 1 2+ 3 √
tan(75 ) = = √ = √ · √ = 2 + 3.
BE 2− 3 2− 3 2+ 3

7. Usaremos a fórmula do seno da soma:

sen(α ± β) = sen α cos β ± sen β cos α.

No 4ACD, temos
∠ADC = ∠BAD + ∠ABD = 15◦ + 45◦ = 60◦
e que
∠CAD = 180◦ − ∠ADC − ∠ACD = 120◦ − ∠ACB.
Logo

sen ∠CAD = sen(120◦ − ∠ACB)


= sen 120◦ cos ∠ACB − sen ∠ACB cos 120◦

3 1
= cos ∠ACB − sen ∠ACB.
2 2
Aplicando a Lei dos senos no 4ABD, temos
BD sen 15◦
= .
AD sen 45◦
Aplicando a Lei dos senos no 4ACD, e usando que CD = 2BD, temos
2BD CD sen ∠CAD
= = .
AD AD sen ∠ACB
Concluı́mos então que
√ √
sen 15◦ 3 cos ∠ACB + sen ∠ACB 3 1
= = + .
sen 45◦ 4 sen ∠ACB 4 tan ∠ACB 4
√ √ 
◦ ◦ ◦ ◦ ◦ ◦ ◦
Como sen(15 ) = sen(45 − 30 ) = sen 45 cos 30 − sen 30 cos 45 = 2 2 3 1
2 − 2 , na equação acima,
temos √ √
3 1 3 1
− = + .
2 2 4 tan ∠ACB 4

Donde obtemos que tan ∠ACB = 2 + 3. Usando o exercı́cio anterior, concluı́mos que ∠ACB = 75◦ .

5
8. Aplicando a Lei dos senos nos triângulos BAE e BAD, usando que ∠BAE = ∠CAD, e posterior-
mente aplicando Lei dos senos nos triângulos d e d, temos
BE · BD sen ∠BAE sen ∠BAD
2
= ·
AB sen ∠AEB sen ∠ADB
sen ∠CAD sen ∠CAE
= ·
sen ∠AEC sen ∠ADC
sen ∠CAD sen ∠CAE
= ·
sen ∠ADC sen ∠AEC
CD · CE
= .
AC 2
Então
CE AC 2 BD
= · .
BE AB 2 CD
Substituindo os valores conhecidos, encontramos a equação para BE:

15 − BE 142 9
= 2· ,
BE 13 6
2535
cuja solução é BE = 463 .

9. Apresentaremos duas soluções:

Problema 9 - Solução 1 - Usando Trigonometria

Usaremos algumas identidades úteis. Das clássicas:

sen(a + b) = sen a cos b + sen b cos a (1)


sen(a − b) = sen a cos b − sen b cos a

se deduz que

sen(a + b) + sen(a − b) = 2 sen a cos b (2)


sen(a + b) − sen(a − b) = 2 sen b cos a

E fazendo u = a + b e v = a − b obtemos
   
u+v u−v
sen u + sen v = 2 sen cos
2 2
   
u−v u+v
sen u − sen v = 2 sen cos (3)
2 2

Vamos à solução do problema.

6
A
Como o triângulo ABC é isósceles, os ângulos da base medem 80◦ .
Portanto, ∠M BN = 20◦ e ∠M CN = 30◦ . Usando que a soma dos
ângulos internos de um triângulo é 180◦ deduzimos que ∠BN C = 40◦ ,
∠BM C = 50◦ e ∠BM N = 180◦ − (x + 20◦ ) onde x = ∠BN M é o que
queremos descobrir. Vamos supor que BC = 1. Então como M BC é
isósceles temos que BM = 1. Pela lei dos senos no triângulo BM N
temos que
sen(∠BM N ) sen(x + 20◦ )
sen x = = . (4)
N BN BN
Pela lei dos senos no triângulo BN C temos que
M x 40◦ BN 1 sen 80◦
= =⇒ BN = .
sen 80◦ sen 40◦ sen 40◦
50◦
Usando que 80 = 40 + 40 e a identidade (1), temos então que BN =
20◦ 2 cos 40◦ e portanto, por (4), temos que
30◦
60◦ 50◦ 2 sen x cos 40◦ = sen(x + 20◦ ) (5)

B C
Usando a identidade (2) no lado esquerdo (5), obtemos

sen(x + 40◦ ) + sen(x − 40◦ ) = sen(x + 20◦ )

e então
sen(x + 40◦ ) = sen(x + 20◦ ) − sen(x − 40◦ ). (6)
Usando a identidade (3) no lado direito de (6) obtemos

sen(x + 40◦ ) = 2 sen(30◦ ) cos(x − 10◦ ) = cos(x − 10◦ )

Como cos(x − 10◦ ) = sen(90◦ − (x − 10◦ )) = sen(100◦ − x) temos então que

sen(x + 40◦ ) = sen(100◦ − x)

Isso acontece quando x + 40◦ = 100◦ − x ou x + 40◦ = 180◦ − (100◦ − x). O segundo caso é impossı́vel
e o primeiro caso nos fornece x = 30◦ .

7
Problema 9 - Solução 2 - Usando construção auxiliar

Como anteriormente, comece deduzindo os ângulos explicitados na fi-


gura da Solução 1. Seja ` o comprimento de BC.
Marque o ponto P sobre o AC tal que ∠N BP = 40◦ . Dessa forma
∠CBP = 60◦ − 40◦ = 20◦ .
Consequentemente, ∠CP B = 180◦ − 80◦ − 20◦ = 80◦ . Assim, CBP é
isósceles (por ter dois ângulos de 80◦ ) e então BP = BC = `.
Como M BC é isósceles (por ter dois ângulos de 50◦ ) temos também que
BM = BC = `.
Como ∠M BP = 60◦ e BM = BP = ` concluı́mos que M BP é N
equilátero e portanto M P = `.
Como o triangulo BP N é isósceles (por ter dois ângulos de 40◦ ) temos M x 40◦
que também P N = BP = `.
Como ∠CP M = 80◦ + 60◦ = 140◦ , concluı́mos que ∠M P N = 180◦ −
140◦ = 40◦ . 50◦
Como M P N é isósceles (por ter dois lados iguais a `) com ∠M P N =
40◦ , os ângulos da base ∠P M N e ∠P N M medem 70◦ . 20◦ P
Portanto x = 70◦ − 40◦ = 30◦ .
40◦ 30◦
20◦ 50◦

B C

Material elaborado por Susana Frómeta Fernández

8
Problemas Resolvidos

Nı́vel 2

Áreas

Material elaborado por Susana Frómeta Fernández

1
Problemas

Problema 1. Na figuras abaixo ABC é um triângulo de área 72cm2 e M , N , P são pontos médios.
Determine a área da região sombreada.
A

B M C

Problema 2. Na figura, ABCD é um quadrilátero de área 200cm2 e E, F , G, H são pontos médios.


Determine a área sombreada.
B

E
H
A

G
F

Problema 3. Na figura abaixo ABCD é um quadrilátero e E, F , G, H são pontos médios. Determine


a área que está faltando.

B
E
A 210 H
250
C

G 240 F

2
Problema 4. Na figura abaixo ABCD é um quadrado de lado 6cm e EF é um segmento paralelo
ao lado AD. Sabendo que a área sombreada é um terço da área do quadrado determine a medida do
segmento EF .
B C

E F

A D

Problema 5. No trapézio ABCD, AD k BC. ∠A = ∠D = 45◦ , enquanto ∠B = ∠C = 135◦ . Se


AB = 6 e a área de ABCD é 30, ache BC.

Problema 6. Na figura abaixo ABCD é um quadrado de lado 4cm e O é o seu centro. Determine a
área marcada sabendo que o ângulo ∠EOF é reto.

B C

A F D

Problema 7. Na figura abaixo ABCD é um retângulo de área 11cm2 . Sabemos também que A0 A =
AD, BB 0 = BA, CC 0 = CB e DD0 = DC. Determine a área do quadrilátero A0 B 0 C 0 D0 .
B0

B C C0

A0 A D

D0

3
Problema 8. Na figura abaixo DEF G é um quadrado de lado 4cm e ABCD um retângulo cujos
lados têm medidas 1cm e 4cm. O encontro da reta AC com a reta F G é o ponto H. Determine a
área marcada.

E F

B C

A D G

Problema 9. Na figuras abaixo ABC é um triângulo de área 72cm2 . Seja M um ponto qualquer no
lado BC e sejam N , P e Q pontos médios. Determine a área da região sombreada.

P
Q

B M C

4
Soluções

1. Usando que P é ponto médio de BN , temos que

[BM N ]
[M N P ] = .
2
Usando que N é ponto médio de AM , temos que

[ABM ]
[BM N ] = .
2
Usando que M é ponto médio de BC, temos que

[ABC]
[ABM ] = .
2
Assim, temos
[BM N ] [ABM ] [ABC]
[M N P ] = = = = 9cm2 .
2 4 8

2. Note que EH é base média do 4ABC e que F G é base média do 4ADC. Logo temos as seguintes
relações entre áreas:
[ABC] [ADC]
[BEH] = e [DF G] = .
4 4
E isto implica que
[ABCD]
[BEH] + [DF G] = .
4
Analogamente, como EG e F H são bases médias dos triângulos 4ABD e 4CBD, temos

[ABCD]
[AEG] + [CF H] = .
4
Consequentemente,
[ABCD]
[EGF H] = = 100cm2 .
2
Note que também foi mostrado que EGF H é um paralelogramo.

3. Como mostrado no exercı́cio anterior, EGF H é um paralelogramo, logo ele é dividido pelas suas
diagonais, EF e GH, em quatro triângulos de área igual. Chamemos a área desses triângulos de x.
Denotemos também y = [CF H]. Veja que [ABCD] = 210 + 250 + 240 + x + y = 700 + x + y.
No exercı́cio anterior foi mostrado também que [BEH] + [DF G] = [ABCD] 4 e que [EGF H] =
[ABCD]
2 . Isto implica que
700 + x + y
210 − x + 240 − x =
4
e
700 + x + y
4x = .
2
Resolvendo este sistema, encontramos x = 112, 5 e y = 87, 5. Assim, a área sombreada é

x + y = 200.

5
4. Chamemos h a distância entre as retas EF e BC, portanto a distância entre as retas EF e AD
será igual a 6 − h.
Veja que
h · EF (6 − h) · EF
[BEDF ] = [BEF ] + [DEF ] = + = 3 · EF.
2 2
[ABCD] 36
Como [BEDF ] = 3 = 3 = 12, encontramos que EF = 4.

5. Seja E o ponto de encontro das retas AB e CD. Note que 4EBC e 4EAD são retângulos e
isósceles. Desta forma, temos

AE 2 BE 2
= [EAD] = [EBC] + [ABCD] = + 30.
2 2
Como AE = 6 + BE, temos
(6 + BE)2 = BE 2 + 60,
donde concluı́mos que BE = 2. Usando Pitágoras no 4EBC, obtemos
p √ √ √
BC = BE 2 + EC 2 = 2 · BE 2 = 8 = 2 2.

6. Tracemos OC e OD. Como O é o centro do quadrado (ponto de interseção das diagonais), temos
que
∠OCE = ∠ODF = 45◦ .
Por outro lado, como ∠COD = ∠EOF = 90◦ , temos

∠COE = ∠COD − ∠DOE = ∠EOF − ∠DOE = ∠DOF.

Assim, como também temos que OC = OD, concluı́mos, pelo critério a.l.a., que os triângulos 4OCE
e 4ODF são congruentes. Consequentemente, eles tem a mesma área. Logo

[ABCD]
[OEDF ] = [OCD] = = 4cm2 .
4

7. Veja que
AB 0 · AA0 2 · AB · AD
[AA0 B 0 ] = = = [ABCD].
2 2
Analogamente, também teremos que

[BB 0 C 0 ] = [CC 0 D0 ] = [DD0 A0 ] = [ABCD].

Logo
[A0 B 0 C 0 D0 ] = 4 · [ABCD] = 44cm2 .

8. Note que CD é base média do 4AGH, logo GH = 2 · CD = 2cm.


Seja I o ponto de encontro de BC com F G. Como os retângulos ABCD e CDGI são iguais.
Temos que GI = IH = 1cm.
Desta forma vemos que os triângulos ABC e CHI são congruentes e, portanto, têm a mesma área.
A área sombrada será igual então a

CEF I = EF · EC = 4cm · 3cm = 12cm2 .

6
9. Usando que Q é o ponto médio de P C, temos

[N P C]
[N P Q] = .
2
Usando que P é o ponto médio de N B, temos

[N BC]
[N P C] = .
2
Usando que N é o ponto médio de AM , temos

[ABC]
[N BC] = .
2
Assim, obtemos
[N P C] [N BC] [ABC]
[N P Q] = = = = 9cm2 .
2 4 8

Material elaborado por Susana Frómeta Fernández

7
Problemas Resolvidos

Nı́vel 2

Relações entre áreas

Material elaborado por Susana Frómeta Fernández

1
Problemas
Problema 1. No triângulo ABC, os pontos L, M e N estão sobre BC, CA e AB respectivamente,
e AL, BM e CN são concorrentes no ponto P .
(a) Encontre o valor numérico de
PL PM PN
+ + .
AL BM CN
(b) Encontre o valor numérico de
AP BP CP
+ + .
AL BM CN

Problema 2. (Ibero) Se AD, BE e CF são três cevianas concorrentes no circuncentro O do triângulo


ABC, demonstre que
1 1 1 2
+ + = ,
AD BE CF R
onde R é o raio da circunferência circunscrita.

Problema 3. Num triângulo ABC, A1 , B1 e C1 estão sobre os lados BC, CA e AB, respectivamente
tais que as cevianas AA1 , BB1 e CC1 são concorrentes no ponto O. Defina K = [ABC], KA = [OBC],
KB = [AOC] e KC = [AOB]. Mostre que
AO KB + KC BO KA + KC CO KA + KB
= , = e = .
OA1 KA OB1 KB OC1 KC

Problema 4. (AIME) Num triângulo ABC, A1 , B1 e C1 estão sobre os lados BC, CA e AB,
AO BO CO
respectivamente. Dado que AA1 , BB1 e CC1 são concorrentes no ponto O, e que OA1
+ OB1
+ OC1
= 92.
AO BO CO
Encontre o valor de OA1 · OB1 · OC1 .

Problema 5. Em um 4ABC, AD, BE e CF são concorrentes no ponto P tal que AP = P D = 6,


EP = 3, P B = 9 e CF = 20. Qual é a área do 4ABC?

Problema 6. Em um triângulo 4ABC, sejam S o ponto médio da mediana AP correspondente ao


vértice A e Q o ponto de interseção de BS com o lado AC. Demonstrar que BS = 3QS.

Problema 7. No triângulo 4ABC, os pontos A1 e A2 se encontram no segmento BC, B1 e B2 se


encontram no segmento AC e C1 e C2 se encontram no segmento AB, de forma tal que os segmentos
C1 A2 , C2 B1 e A1 B2 são paralelos aos lados do 4ABC e se intersectam no ponto P . Prove que as
áreas dos triângulos A1 B1 C1 e A2 B2 C2 são iguais.
A

C1 B2

C2 P B1

B C
A1 A2

2
Soluções

1. Os triângulos 4P BC e 4ABC compartilham a base BC, logo as respectivas cevianas P L e AL


se encontram na mesma proporção que as áreas, ou seja
PL [P BC]
= .
AL [ABC]
Analogamente, temos
PM [AP C] PN [ABP ]
= e = .
BM [ABC] CN [ABC]
Isto implica
PL PM PN [P BC] + [AP C] + [ABP ] [ABC]
+ + = = = 1.
AL BM CN [ABC] [ABC]
Para o item (b), escrevemos

AP = AL − P L, BP = BM − P M e CP = CN − P N.

Usando o item (a), obtemos


 
AP BP CP PL PM PN
+ + =3− + + = 3 − 1 = 2.
AL BM CN AL BM CN

2. Como O é o circuncentro do 4ABC, temos que AO = BO = CO = R. Usando o item (b) do


problema anterior, temos
R R R AO BO CO
+ + = + + = 2.
AD BE CF AD BE CF
Dividindo por R em ambos lados, obtemos o resultado desejado.

AO
3. Provaremos OA 1
= KBK+K
A
C
. As outras igualdades são análogas.
Sejam R e S, respectivamente, os pés das perpendiculares traçadas desde O e A até BC. Como
4AA1 S ∼ 4OA1 R temos que
AA1 AS K
= = .
OA1 OR KA
Como AA1 = AO + OA1 e K = KA + KB + KC , a igualdade acima implica
AO KB + KC
1+ =1+ .
OA1 KA
Subtraindo 1 em ambos lados, obtemos a igualdade desejada.

4. Denotemos por KA = [OBC], KB = [AOC] e KC = [AOB]. No problema anterior mostramos que


AO KB + KC BO KA + KC CO KA + KB
= , = e = .
OA1 KA OB1 KB OC1 KC
AO BO CO
Logo OA1 + OB1 + OC1 = 92 implica que
2
KB KC + KB KC2 + KA
2
KC + KA KC2 + KA
2 2
KB + KA KB = 92 KA KB KC .

3
Então
AO BO CO (KB + KC )(KA + KC )(KA + KB )
· · =
OA1 OB1 OC1 KA KB KC
2
K KC + KB KC2 + KA 2 K + K K2 + K2 K + K K2 + 2 K K K
C A C A B A B A B C
= B
KA KB KC
92 KA KB KC + 2 KA KB KC
=
KA KB KC
= 94.

5. Chamemos de S = [AP E]. Como as bases dos triângulos 4AP E e 4ABP se encontram numa
mesma reta, temos
[AP E] 3
= =⇒ [ABP ] = 3S.
[ABP ] 9
Como P é ponto médio de AD, temos
[BP D] = [ABP ] = 3S.
Chamemos de Q = [CP E]. Temos
[CP D] = [ACP ] = [AP E] + [CP E] = S + Q.
A ceviana CP do 4BCE, o divide em dois triângulos cujas áreas se encontram na proporção
[BP C] 9
= = 3.
[CP E] 3
Logo
[BP D] + [CP D] 3S + S + Q
3= = ,
[CP E] Q
donde concluı́mos Q = 2S.
Agora chamemos de T = [AF P ], logo [BF P ] = 3S − T . Olhando para o 4AF C e a ceviana AP ,
temos
FP [AF P ] T
= = .
20 − F P [ACP ] 3S
E, olhando para o 4BF C e a ceviana BP , temos
FP [BF P ] 3S − T
= = .
20 − F P [BCP ] 6S
T
Isso implica que 3S = 3S−T FP 1
6S , donde obtemos T = S. Concluı́mos também que 20−F P = 3 , donde
obtemos F P = 5 e, consequentemente, CP = 15.
Note agora que [ABD] = [ADC] = 6S, o que implica que D é ponto médio de BC.
Chamemos de x o comprimento dos segmentos BD e DC. Usaremos a fórmula de Heron nos
triângulos (de área igual) 4P BD e 4P DC:
s     s    
15 + x 15 − x x+3 x−3 21 + x 21 − x x+9 x−9
= ,
2 2 2 2 2 2 2 2

=⇒ (152 − x2 )(x2 − 32 ) = (212 − x2 )(x2 − 92 ),


donde obtemos x2 = 117. Como [P BD] = 3S, temos, novamente usando a fórmula de Heron, que
r r
(152 − x2 )(x2 − 32 ) (152 − 117)(117 − 32 )
3S = = = 27,
16 16
donde obtemos S = 9 e, finalmente, [ABC] = 12S = 108.

4
6. Usando o Problema 3, temos que

BS [ABS] + [BSC]
= .
QS [ACS]

Como BS, CS e AM são medianas dos triângulos ABM , ACM e ABC, respectivamente, temos que
[ABS] = [BSM ] = 12 [BSC] e [ACS] = [M SC] = 12 [BSC].
1
BS [BSC] + [BSC]
= 2 1 = 3.
QS 2 [BSC]

7. Os triângulos 4A1 B1 P e 4A2 B1 P compartilham a base B1 P e, como B1 C2 k BC, também


possuem a mesma altura relativa a essa base, logo [A1 B1 P ] = [A2 B1 P ]. Agora, os triângulos 4A2 B1 P
e 4A2 B2 P compartilham a base A2 P e, como A2 C1 k AC, eles também possuem a mesma altura,
logo [A2 B1 P ] = [A2 B2 P ].
Acabamos de mostrar que
[A1 B1 P ] = [A2 B1 P ] = [A2 B2 P ].
Um argumento análogo mostra que

[A1 C1 P ] = [A1 C2 P ] = [A2 C2 P ]

e
[B1 C1 P ] = [B2 C1 P ] = [B2 C2 P ].
Como [A1 B1 C1 ] = [A1 B1 P ] + [A1 C1 P ] + [B1 C1 P ] e [A2 B2 C2 ] = [A2 B2 P ] + [A2 C2 P ] + [B2 C2 P ],
concluı́mos a prova.

Material elaborado por Susana Frómeta Fernández

5
Problemas Resolvidos

Nı́vel 2

Relações entre áreas - Parte 2

Material elaborado por Susana Frómeta Fernández

1
Problemas

Problema 1. (Portugal) Seja ABC um triângulo retângulo em A. Considere um ponto E sobre a


hipotenusa e traça-se a partir desse ponto uma paralela ao cateto AC. Seja D a interseção desta
paralela com o cateto AB. Prove que

BD DE BC 2
+ = ,
DE BD 2S
sendo S a área do triângulo ABC.

Problema 2. (OBM) É dado um quadrilátero convexo ABCD. Sejam E, F , G e H os pontos médios


dos lados AB, BC, CD e DA, respectivamente. Determine a posição de um ponto P de forma que os
quadriláteros P HAE, P EBF , P F CG e P GDH tenham a mesma área.

Problema 3. Seja ABCDE um pentágono convexo (não necessariamente regular) tal que os triângulos
ABC, BCD, CDE, DEA e EAB tem área 1. Qual a área do pentágono?

Problema 4. Seja ABCD um quadrilátero convexo e sejam EH, EI, EF e EG segmentos paralelos
e iguais a AB, BC, CD e DA, como mostra a figura abaixo.

C
D F
H E

A B

Determine a razão entre as áreas dos quadriláteros HIF G e ABCD.

Problema 5. (OBM) ABCD é um quadrilátero convexo e inscritı́vel e M é um ponto sobre o lado


CD, tal que o triângulo ADM e o quadrilátero ABCM têm a mesma área e o mesmo perı́metro.
Prove que ABCD tem dois lados de comprimentos iguais.

2
Problema 6. (AIME) Quadrados S1 e S2 são inscritos em um triângulo retângulo ABC, como
mostrado na figura abaixo.

A A

S2
S1

C B C B

Determine AC + CB se a área de S1 é igual a 441 e a área de S2 é igual a 440.

Problema 7. Os pontos médios das diagonais AC, CE, EA, BD, DF e F B do hexágono convexo
ABCDEF são vértices de um novo hexágono. Calcular a relação entre as áreas do dois hexágonos.

Problema 8. Seja P um ponto no interior de um triângulo equilátero ABC, e sejam D, E e F os


simétricos de P em relação aos lados BC, CA e AB, respectivamente.

A B

Qual é maior, a área do triângulo ABC ou a área do triângulo DEF ?

3
Soluções

1. Usando que os triângulos 4BDE e 4ABC são semelhantes, que 2[ABC] = AB · AC e o Teorema
de Pitágoras, temos
BD DE AB AC AB 2 + AC 2 BC 2
+ = + = = .
DE BD AC AB AB · AC 2S

2. Queremos construir um ponto P tal que cada um dos quadriláteros P HAE, P EBF , P F CG e
P GDH tenha área igual a [ABCD]
4 .
Como E e F são pontos médios de AB e BC, respectivamente, temos que EF é a base média do
4ABC relativa à AC, isto é EF k AC e EF = AC 1
2 . Isto implica que [BEF ] = 4 [ABC]. Analogamente
1
GH é base média do 4ADC e, portanto, [DGH] = 4 [ACD].
Temos então que
[ABC] + [ACD] [ABCD]
[BEF ] + [DGH] = = .
4 4
Logo o ponto P deve ser tal que

[P GH] = [BEF ] e [P EF ] = [DGH].

Denotemos por Q o ponto de interseção das diagonais AC e BD, e chamemos de L e M os pontos


de interseção de BD com EF e GH, respectivamente. Note que BL = LQ e QM = M D.
Traçamos uma reta r paralela a AC que intersecta BD num ponto P1 tal que M P1 = BL e
LP1 = DM . Qualquer ponto R que pertença à reta r satisfaz [RHDG] = [REBF ].
Analogamente temos que F G e EH são bases médias dos triângulos 4CBD e 4ABD, e que

[CBD] + [ABD] [ABCD]


[CF G] + [AEH] = = .
4 4
Chamemos de N e O os pontos de interseção de AC com F G e EH, respectivamente.
Traçamos uma reta ` paralela a BD que intersecta AC num ponto P2 tal que N P2 = OA e
OP2 = CN . Qualquer ponto R que pertença à reta ` satisfaz [RF CG] = [REAH].
O ponto P procurado será o ponto de interseção das retas r e `.

C
G r
D
` N
M
Q
F
P2
H P1
O L
P

A E B

4
3. Como [ABC] = [ABE] e ambos triângulos compartilham a base AB, temos que AB k CE.
Analogamente também teremos BC k AD, CD k BE, DE k AC e AE k BD.
Seja N o ponto de interseção de AC com BE. Como CDEN é um paralelogramo, temos que
[CEN ] = [CDE] = 1. Denotemos a = [ABN ] e b = [AN E]. Observe que [BCN ] = [ABC]−[ABN ] =
[ABE] − [ABN ] = b.
Veja que
a [ABN ] BN [BCN ] b
= = = = = b.
b [AN E] NE [CEN ] 1
Logo a = b2 . Como a + b = 1, temos

2 2 −1 + 5
1 − b = b =⇒ b + b − 1 = 0 =⇒ b = .
2

B D

A E

A área do pentágono ABCDE é igual a


√ √
−1 + 5 5+ 5
[ABC] + [CDE] + [CEN ] + [AN E] = 1 + 1 + 1 + b = 3 + = .
2 2

4. Como AB k EH e AD k EG, temos que ∠GEH = 180◦ − ∠BAD.


Isto implica que sen∠GEH = sen∠BAD, e analogamente, também vale sen∠GEF = sen∠ADC,
sen∠F EI = sen∠DCB e sen∠IEH = sen∠ABC.
Veja que

[ABD] + [BCD] + [ABC] + [ADC]


[ABCD] =
2
AB · AD · sen∠BAD + AD · CD · sen∠ADC + CD · CB · sen∠DCB + AB · BC · sen∠ABC
=
4
EH · EG · sen∠GEH + EG · EF · sen∠GEF + EF · EI · sen∠F EI + EH · EI · sen∠IEH
=
4
[EGH] + [EF G] + [EF I] = [EIH]
=
2
[HIF G]
= .
2

5. Como ABCD é inscritı́vel, vale que ∠ABC + ∠ADC = 180◦ . Digamos que ∠ABC = α. Temos
então que sen∠ABC = sen∠ADC = senα.

5
Podemos escrever
AD · DM · senx
[ADM ] = ,
2
e
senx
[ABCM ] = [ABC] + [ACD] − [ADM ] = (AB · BC + AD · CD − AD · DM ).
2
Como [ADM ] = [ABCM ], igualamos as duas equação acima e obtemos

2 · AD · DM = AB · BC + AD · CD. (1)

Como ADM e ABCM têm também o mesmo perı́metro, temos que

AD + DM = AB + BC + CM = AB + BC + CD − DM,

o que implica que


2DM = AB + BC + CD − AD.
Substituı́mos em (1) e obtemos

AD(AB + BC + CD − AD) = AB · BC + AD · CD,


o que implica que
AD · AB − AD2 + AD · BC − AB · BC = 0.
Fatorando, obtemos
(AD − BC)(AB − AD) = 0,
e isto mostra que AD = BC ou AD = AB.

6. Seja x = AC + BC e y = AB.
Sejam E em AC, F em BC e G em AB vértices do quadrado S1 , conforme mostra a figura.
Veja que

AC · EG + BC · F G 441 21x
[ABC] = [ACG] + [CBG] = = (AC + BC) = . (2)
2 2 2
Por outro lado, também temos que

AC · BC (AC + BC)2 − (AC 2 + BC 2 ) (AC + BC)2 − AB 2 x2 − y 2


[ABC] = = = = .
2 4 4 4
Igualando ambas equações, obtemos

x2 − y 2 = 42x. (3)

Agora, sejam H em AC e I em BC, vértices do quadrado S2 . Seja M o pé da perpendicular


traçada desde C até o lado AB, e L o ponto de interseção de HI com CM .
Como os triângulos CHI e ABC são semelhantes, temos

CL CL HI 440
√ = = = .
CL + 440 CM AB y

Logo √
440 √
CL = (CL + 440).
y

6
Isolando CL nessa equação, obtemos
440
CL = √ .
y − 440
Podemos escrever a área do 4CHI como

CL · HI 440 440
[CHI] = = √ . (4)
2 2(y − 440)
Por outro lado, temos
[CHI] HI 2 440
= 2
= 2 .
[ABC] AB y
Usando (2), temos
440 · 21x
[CHI] = . (5)
2y 2
Igualando as equação (4) e (5), obtemos
√ √
440y 2 = 21xy − 21 440x.
Usando (2), obtemos
√ p √
440(x2 − 42x) = 21x x2 − 42x − 21 440x
√ p √
440(x − 42) = 21 x2 − 42x − 21 440
√ p
440(x − 21) = 21 x2 − 42x
440(x − 21)2 = 441(x2 − 42x).

E isto implica que x satisfaz a equação


x2 − 42x − 440 · 441 = 0,
cuja única solução positiva é

42 + 422 + 4 · 440 · 441 √
x= = 21 + 21 1 + 440 = 21 + 212 = 462.
2

7. Neste problema usaremos que a área de um quadrilátero ABCD pode ser calculada mediante a
fórmula
AC · BD · sen θ
[ABCD] = ,
2
onde AC e BD são as diagonais do quadrilátero e θ é menor ângulo entre elas.
Sejam A1 , B1 , C1 , D1 , E1 e F1 os pontos médios de CE, DF, AE, BF, AC e BD, respectivamente.
Note que A1 C1 é base média do 4AEC relativa à base AC; e que B1 D1 é base média do 4BDF
relativa à base BD. Logo A1 C1 = AC BD
2 e A1 C1 k AC; e também B1 D1 = 2 e B1 D1 k BD. Como
consequência do paralelismo, o menor ângulo formado entre os segmentos AC e BD é igual ao menor
ângulo formado entre os segmentos A1 C1 e B1 D1 ; chamemos esse ângulo de θ. Temos então
A1 C1 · B1 D1 · sen θ AC · BD · sen θ [ABCD]
[A1 B1 C1 D1 ] = = = .
2 8 4
De maneira análoga, teremos também
[ADEF ]
[A1 D1 E1 F1 ] = .
4
E, finalmente,
[ABCD] + [ADEF ] [ABCDEF ]
[A1 B1 C1 E1 D1 F1 ] = [A1 B1 C1 D1 ] + [A1 D1 E1 F1 ] = = .
4 4

7
8. Mostraremos que [DEF ] ≤ [ABC].
Denotemos por A1 o ponto de interseção de BC com P D; B1 o ponto de interseção de AC com
P E; e C1 o ponto de interseção de AB com P F .
Veja que, como A1 e B1 são pontos médios de P D e P E, respectivamente, teremos que A1 B1 é
base média do 4P DE e, consequentemente [P A1 B1 ] = [P DE] 4 . Analogamente, [P A1 C1 ] =
[P DF ]
4 e
[P EF ]
[P B1 C1 ] = 4 . Logo
[DEF ]
[A1 B1 C1 ] = .
4
Seja agora Q o baricentro (e também incentro, circuncentro e ortocentro) do triângulo equilátero
4ABC. Sejam A2 , B2 e C2 respectivamente os pés das perpendiculares traçadas desde Q até os lados
BC, AC e AB, respectivamente (note que são também os pontos médios dos lados do 4ABC). Como
[A2 B2 C2 ] = [ABC]
4 , a razão entre as área dos triângulos ABC e DEF é a mesma razão entre as áreas
dos triângulos A2 B2 C2 e A1 B1 C1 .
Logo [DEF ] ≤ [ABC] se e somente se [A1 B1 C1 ] ≤ [A2 B2 C2 ]. Esqueceremos os pontos D, E e F e
nos concentraremos no que acontece no interior do 4ABC.
C

B1

A1
P

A C1 B

Mostraremos que [A1 B1 C1 ] ≤ [A2 B2 C2 ].


Olhando para a soma dos ângulos internos do quadrilátero AB1 P C1 , temos que ∠B1 P C1 = 120◦ .
Analogamente ∠A1 P B1 = ∠A1 P C1 = 120◦ .
Dessa forma teremos
sen 120◦
[A1 B1 C1 ] = [A1 P B1 ] + [A1 P C1 ] + [B1 P C1 ] = (P A1 · P B1 + P A1 · P C1 + P B1 · P C1 ).
2
Vamos supor, sem perda de generalidade, que as medianas do 4ABC medem 1. Logo QA2 =
QB2 = QC2 = 13 .
Assim,

sen 120◦ 1 1 1 1 1 1 sen 120◦ 1


 
[A2 B2 C2 ] = [A2 QB2 ] + [A2 QC2 ] + [B2 QC2 ] = · + · + · = · .
2 3 3 3 3 3 3 2 3

É suficiente mostrar então que


1
P A1 · P B1 + P A1 · P C1 + P B1 · P C1 ≤ .
3
Seja D = C2 o pé da altura do 4ABC relativa ao lado AB. Sejam E e F os pontos de interseção
da reta P B1 com os segmentos CD e BC, respectivamente. Seja G o pé da perpendicular traçada
desde P até CD.
Denotemos por x e y os comprimentos de B1 E e EG, respectivamente.

8
C

B1
x
E A1
y P
G
F

A D C1 B

Como 4B1 CE é retângulo tendo ângulos medindo 60◦ e 30◦ , temos que CE = 2 · B1 E = 2x. Pelo
mesmo motivo, no 4EGP , temos EP = 2 · EG = 2y. Chamemos de H o pé da perpendicular traçada
desde E até BC. Note que, como CD é bissetriz do ∠ACB, temos que EH = EB1 = x. Também, no
4EHF (que também é retângulo com os ângulos agudos medindo 30◦ e 60◦ ), temos EF = 2·EH = 2x.
Logo P F = EF −EP = 2x−2y. Agora, no 4A1 P F retângulo, temos P A1 = P2F = x−y. Lembrando
que estamos assumindo CD = 1, temos GD = 1 − CE − EG = 1 − 2x − y. Como GDC1 P é um
retângulo, temos que P C1 = 1 − 2x − y.
Resumindo, acabamos de mostrar que

P A1 = x − y, P B1 = x + 2y e P C1 = 1 − 2x − y.

Temos então

P A1 · P B1 + P A1 · P C1 + P B1 · P C1 = (x − y)(x + 2y) + (x − y)(1 − 2x − y) + (x + 2y)(1 − 2x − y)


= −3x2 − 3xy − 3y 2 + 2x + y
3
≤ −3x2 − 3xy − y 2 + 2x + y
4
3
= − (4x + 4xy + y 2 ) + 2x + y
2
4
3
= − (2x + y)2 + (2x + y).
4
onde na linha onde tem a desigualdade foi usado que 43 < 3.
Seja z = 2x + y. Para concluir, vamos mostrar que, para todo z, vale a desigualdade
3 1
− z2 + z ≤ .
4 3
A prova segue da seguinte fatoração
   2
3 1 1 9 2 1 3
− z2 + z − = − z − 3z + 1 =− z − 1 ≤ 0.
4 3 3 4 3 2

Material elaborado por Susana Frómeta Fernández

9
POLOS OLÍMPICOS DE
TREINAMENTO INTENSIVO

combinatória
nível 2
Polos Olı́mpicos de Treinamento Intensivo
Curso de Combinatória − Nı́vel 2

Professores: Cleber Assis, Samuel Barbosa e Tiago Miranda


Aula 0
POTI 2015
Curso Básico
. . . . . . . . . . . . . . . . . . . . . . . . . . . . . . . . . . . . . . Combinatória . . . . . . . . . . . . . . . . . . . . . . . . . . . . . . . . . . . . . .

Este material compila os arquivos do projeto Portal da Matemática, disponı́vel em

http://matematica.obmep.org.br/

e serve como introdução aos tópicos iniciais de um curso de treinamento olı́mpico. Em geral, os assuntos
são independentes e podem ser estudados em qualquer ordem. Neles, o leitor encontrará muitos exercı́cios
escolares mesclados com problemas elementares de olimpı́adas, todos com respostas e soluções. Além disso,
no endereço do Portal da Matemática, existem vı́deos que podem ser acessados gratuitamente cobrindo todo
o conteúdo abaixo. Bons estudos!
...

Sumário
1 Números Naturais e Problemas de Contagem . . . . . . . . . . . . . . . . . . . . . . . . . . . . . 1
2 Princı́pio Fundamental da Contagem . . . . . . . . . . . . . . . . . . . . . . . . . . . . . . . . . . 4
3 Permutação Simples . . . . . . . . . . . . . . . . . . . . . . . . . . . . . . . . . . . . . . . . . . . . 6
4 Combinações . . . . . . . . . . . . . . . . . . . . . . . . . . . . . . . . . . . . . . . . . . . . . . . . 8
5 Permutação com Repetição (com Elementos nem Todos Distintos) . . . . . . . . . . . . . . . 10
6 Permutações Circulares . . . . . . . . . . . . . . . . . . . . . . . . . . . . . . . . . . . . . . . . . . 11
7 Combinações Completas . . . . . . . . . . . . . . . . . . . . . . . . . . . . . . . . . . . . . . . . . . 13
8 Princı́pio da Casa dos Pombos (PCP) . . . . . . . . . . . . . . . . . . . . . . . . . . . . . . . . . 16
Números Naturais e Problemas de Contagem − Soluções . . . . . . . . . . . . . . . . . . . . . . 19
Princı́pio Fundamental da Contagem − Soluções . . . . . . . . . . . . . . . . . . . . . . . . . . . 24
Permutação Simples − Soluções . . . . . . . . . . . . . . . . . . . . . . . . . . . . . . . . . . . . . 26
Combinações − Soluções . . . . . . . . . . . . . . . . . . . . . . . . . . . . . . . . . . . . . . . . . 28
Permutação com Repetição − Soluções . . . . . . . . . . . . . . . . . . . . . . . . . . . . . . . . . 30
Permutações Circulares − Soluções . . . . . . . . . . . . . . . . . . . . . . . . . . . . . . . . . . . 32
Combinações Completas − Soluções . . . . . . . . . . . . . . . . . . . . . . . . . . . . . . . . . . . 38
Princı́pio da Casa dos Pombos − Soluções . . . . . . . . . . . . . . . . . . . . . . . . . . . . . . . 45

Versão: 227 (Data: 27 de abril de 2015.)


POTI 2015 − Combinatória − Nı́vel 2 − Aula 0 − Professores Cleber Assis, Samuel Barbosa e Tiago Miranda

1 Números Naturais e Problemas de Contagem


Problema 1. Qual a quantidade de elementos do conjunto que possui todos os números naturais de 8 até
908?
Problema 2. Quantos elementos há no conjunto {7, 14, 21, . . . , 679, 686}?
Problema 3. Quantos elementos há no conjunto {14, 19, 24, . . . , 1004, 1009}?
Problema 4. Entre n pessoas existem duas com o mesmo signo. Qual o menor valor de n que garante esse
fato?
Problema 5. Quantos números escrevemos ao numerarmos as páginas de um livro de 10 até 20? E quantos
algarismos são usados para isso?
Problema 6. Uma pessoa entrou num quarto escuro, sem enxergar absolutamente nada, e abriu uma gaveta
na qual havia exatamente 20 meias pretas, 15 meias brancas e 10 meias marrons. Todas estavam misturados e
eram indistinguı́veis ao tato. Qual a quantidade mı́nima de meias que essa pessoa deve retirar para que tenha
certeza de ter retirado:
a) um par de meias de mesma cor? b) um par de meias brancas?
Problema 7. Prove que:
a) a soma de dois números pares é igual a um número d) o produto de dois números ı́mpares é igual a um
par. número ı́mpar.

b) a soma de dois números ı́mpares resulta em um e) o produto de dois números pares é um número
número par. par.

c) a soma de um número par com um número ı́mpar f) o produto de um número par com um número
resulta em um número ı́mpar. ı́mpar resulta em um número par.

Problema 8. Ao escrevermos todos os números naturais de 40 até 1200, quantos algarismos utilizamos?
Problema 9. Qual é a soma de todos os números de três algarismos?
Problema 10. Qual o número mı́nimo necessário de pessoas num grupo para que tenhamos certeza de que:

a) três delas façam aniversário no mesmo mês? b) quatro tenham nascido no mesmo dia da semana?

Problema 11. Numa gaveta há 10 blusas amarelas, 12 blusas beges e 8 blusas cinzas. Suponha que sejam
retiradas “n” blusas, no escuro, dessa gaveta (não há como perceber as cores). Qual o valor mı́nimo de “n”
para que tenhamos certeza de que saiam 3 de cores distintas?
Problema 12. Discos dentados geram um tipo de sistema associado que funciona pela propulsão em um
dos discos e esse proporciona o funcionamento dos demais. A figura 1 ilustra um desses sistemas e o disco
“número 1” gira no sentido horário. Analise as proposições e responda o que se pede.

i) O disco 2 gira no sentido anti-horário.

ii) O disco 4 gira no sentido horário.

iii) O disco 7 gira no mesmo sentido do disco 5.

iv) O disco 10 gira no mesmo sentido do disco 3.

v) Seria possı́vel colocar um disco 11 em contato


Figura 1
simultâneo com os discos 1 e 10.

Quantas das proposições acima são verdadeiras?


Problema 13. Se uma urna contém 7 bolas vermelhas, 9 pretas, 10 azuis e 8 verdes. Qual é o número mı́nimo
de bolas que devemos retirar para que possamos ter certeza da retirada de pelo menos 4 da mesma cor?

1
POTI 2015 − Combinatória − Nı́vel 2 − Aula 0 − Professores Cleber Assis, Samuel Barbosa e Tiago Miranda

Problema 14. Considere o número

S = 1 + 2 + 3 + +4 + · · · + 2011 + 2012 + 2013 + 2014.

Esse número é par ou ı́mpar?

Problema 15. Escrevendo os números naturais de 1 até 10 em fila e mantendo um espaço vazio entre eles
( ~ ) obtemos
1 ~ 2 ~ 3 ~ 4 ~ 5 ~ 6 ~ 7 ~ 8 ~ 9 ~ 10.
É possı́vel ocupar os ~ com sinais de “+” ou “−” de modo que o resultado da expressão que aparecerá após
a colocação dos sinais seja zero?

Problema 16. Em uma urna há 32 bolas brancas, 16 bolas verdes, 7 bolas vermelhas, 3 bolas pretas e 11
bolas cinzas. Qual é o número mı́nimo de bolas que devemos sacar dessa urna para termos certeza de que
obteremos pelo menos 13 bolas da mesma cor?

Problema 17. Se n é um número inteiro qualquer, qual das expressões abaixo resulta num número ı́mpar?

a) n2 − n + 2 c) n2 + n + 5 e) n3 + 5
b) n2 + n + 2 d) n2 + 5

Problema 18. Qual a paridade do algarismo das unidades do número

20102010 + 20112011 + 20122012 + · · · + 20152015 + 20162016 ?

Problema 19. Qual o menor número de pessoas num grupo para garantir que pelo menos 4 nasceram no
mesmo mês?

Problema 20. Uma máquina contém pequenas bolas de borracha de 10 cores distintas, sendo 10 bolas de
cada cor. Ao inserir uma moeda, uma bola é expelida ao acaso. Para garantir a retirada de 4 bolas da mesma
cor, qual o menor número de moedas inseridas na máquina?

Problema 21. Observe a sequência de algarismos

12345678910121314151617 . . . .

Qual será o 1002◦ algarismo usado nela?


Problema 22. Depois de d lançamentos de um dado de 6 faces temos certeza que uma das faces saiu mais
de 5 vezes. Qual o valor de d?

Problema 23. Qual a soma dos múltiplos de 3 entre 1 e 301?

xy
Problema 24. Quais são os pares de números inteiros ( x, y) tais que = 144?
x+y
Problema 25. Quantos números inteiros e positivos satisfazem a dupla inequação
q
2000 < n · (n − 1) < 2005?
a) 1. b) 2. c) 3. d) 4. e) 5.

2
POTI 2015 − Combinatória − Nı́vel 2 − Aula 0 − Professores Cleber Assis, Samuel Barbosa e Tiago Miranda

Problema 26. Observe que


( a + b)3 = a3 + 3a2 b + 3ab2 + b3 ,
daı́ poderı́amos calcular

23 = (1 + 1)3 = 13 + 3 · 12 · 1 + 3 · 1 · 12 + 13
33 = (2 + 1)3 = 23 + 3 · 22 · 1 + 3 · 2 · 12 + 13
43 = (3 + 1)3 = 33 + 3 · 32 · 1 + 3 · 3 · 12 + 13
53 = (4 + 1)3 = 43 + 3 · 42 · 1 + 3 · 4 · 12 + 13

A partir da análise dos exemplos acima, desenvolva uma fórmula para o cálculo de

12 + 22 + 32 + · · · + n 2 .

Problema 27. A figura 2 é o composta por 100 quadrados colocados lado a lado, na qual tem-se indicadas as
medidas dos lados de cada quadrado.

···

1 2 3 100
Figura 2

Qual o valor da área total dessa figura?


Problema 28. Uma rede de computadores é formada por seis computadores. Cada computador é conectado
diretamente a pelo menos um dos outros computadores. Mostre que há pelo menos dois computadores na
rede que estão diretamente conectados ao mesmo número de outros computadores.

3
POTI 2015 − Combinatória − Nı́vel 2 − Aula 0 − Professores Cleber Assis, Samuel Barbosa e Tiago Miranda

2 Princı́pio Fundamental da Contagem


Problema 29. Considere três cidades A, B e C, de forma tal que existem três estradas ligando A à B e dois
caminhos ligando B à C.

a) De quantas formas diferentes podemos ir de A até C, passando por B?

b) De quantas formas diferentes podemos ir de A até C, passando por B, e voltar para A novamente, passando
por B?

c) De quantas formas diferentes podemos ir de A até C, passando por B, e depois voltar para A sem repetir
estradas e novamente passando por B?

Problema 30. Um construtor dispõe de quatro cores (verde, amarelo, cinza e bege) para pintar cinco casas
dispostas lado a lado. Ele deseja que cada casa seja pintada com apenas uma cor e que duas casas consecutivas
não possuam a mesma cor. Por exemplo, duas possibilidades diferentes de pinturas estão indicadas abaixo:
Primeira: verde, amarelo, bege, verde, cinza;
Segunda: verde, cinza, verde, bege, cinza.
Quantas são as possibilidades?
Problema 31. Em um computador digital, um bit é um dos algarismos 0 ou 1 e uma palavra é uma sucessão
de bits. Por exemplo, todas as possı́veis palavras de dois bits são: 00, 01, 10, 11. Qual é o número de palavras
distintas de 32 bits?
Problema 32. De quantas formas se pode dispor quatro pessoas em fila indiana?
Problema 33. Quantos são os números de 3 algarismos distintos?
Problema 34. Quantos são os números de 4 algarismos formados apenas por algarismos pares?
Problema 35. Em uma competição de atletismo, participam 8 corredores. De quantas maneiras diferentes
pode ser composto o pódium com os três primeiros colocados?
Problema 36. De quantos modos 3 pessoas podem se sentar em 6 cadeiras alinhadas?
Problema 37. Uma prova possui dez questões do tipo múltipla escolha, com cinco alternativas cada. De
quantas maneiras diferentes é possı́vel responder esta prova, marcando todas as dez respostas?
Problema 38. As placas de veı́culos são compostas por 3 letras e 4 algarismos. Qual é o total de placas
diferentes que podem existir?
Problema 39. Dispondo dos algarismos 0, 1, 2, 3, 4, 5, 6, 7, pode-se formar quantos números
a) de quatro algarismos?

b) de quatro algarismos distintos?

c) ı́mpares de três algarismos distintos?


Problema 40. Dispondo dos algarismos 0, 1, 2, 3, 4, 5, 6, 7, quantos números pares de quatro algarismos
distintos podem ser formados?
Problema 41. Um automóvel comporta dois passageiros nos bancos da frente e três no banco traseiro.
Qualquer uma das 7 pessoas, dentre elas Pedro que tem 5 anos de idade e portanto não pode sentar na parte
da frente do carro, pode ser escolhida para entrar no automóvel. Calcule o número de maneiras distintas de
lotar este automóvel.

4
POTI 2015 − Combinatória − Nı́vel 2 − Aula 0 − Professores Cleber Assis, Samuel Barbosa e Tiago Miranda

Problema 42. As letras em código Morse são formadas por sequências de traços (−) e pontos (·), sendo
permitida repetições. Por exemplo (−)(−)(−)(·). Quantas letras podem ser representadas usando:

a) exatamente 3 sı́mbolos?

b) usando no máximo 8 sı́mbolos?

Problema 43. Vai ser formada uma fila com 6 pessoas, dentre as quais Pedro e Ana. De quantas maneiras
esta fila poderá ser formada se:

a) Ana deve ser a primeira da fila?

b) Ana ou Pedro devem ser o primeiro da fila?

c) Ana e Pedro não devem ficar juntos na fila?

Problema 44. João escreveu todos os números de 4 dı́gitos contendo cada um dos algarismos de 1 até 4
exatamente uma vez. Em quantos desses números a soma dos dois últimos dı́gitos é maior que a soma dos
dois primeiros?

a) 8. b) 12. c) 4. d) 16. e) 2.

Problema 45. Quantos são os números ı́mpares, de cinco algarismos, nos quais a soma dos algarismos das
unidades e das dezenas é 16 e a soma de todos os algarismos é um múltiplo de 5?

a) 90. b) 180. c) 216. d) 360. e) 532.

Problema 46. Ana quer fazer duas aulas de natação por semana, uma de manhã e a outra à tarde. A escola
de natação tem aulas de segunda a sábado às 9h, 10h e 11h e de segunda a sexta às 17h e 18h. De quantas
maneiras distintas Ana pode escolher o seu horário semanal, de modo que ela não tenha suas aulas no mesmo
dia nem em dias consecutivos?

a) 96. b) 102. c) 126. d) 144. e) 180.

Problema 47. Bitonho está jogando em seu computador o Super Paciência, cujo objetivo é preencher um
tabuleiro 2 × 2014 com algarismos 0’s e 1’s de modo que dois números vizinhos iguais em uma mesma linha
impedem que se preencha também com números iguais as casas correspondentes da outra linha. Por exemplo,
no desenho abaixo, os valores de A e B não podem ser iguais.

0 1 0 ... 1 1 ...

1 1 0 ... A B ...

Determine o número de possı́veis preenchimentos distintos de tal tabuleiro seguindo as regras do Super
Paciência.

5
POTI 2015 − Combinatória − Nı́vel 2 − Aula 0 − Professores Cleber Assis, Samuel Barbosa e Tiago Miranda

3 Permutação Simples
Problema 48. De quantas formas se pode dispor quatro pessoas em fila indiana?
Problema 49. Quantos são os anagramas da palavra MATRIZ?
Problema 50. Luiz precisa trocar a lâmpada da sala, lavar a louça, estudar para a prova de matemática e
arrumar seu quarto. De quantas maneiras diferentes ele pode executar essa sequência de atividades?
Problema 51. Com os algarismos 1, 2, 3, 4, 5, quantos números de 5 algarismos podemos formar, sem repeti-
los?
Problema 52. Escreva todos os anagramas com as letras da palavra BOLA que começam com a letra L.
Problema 53. Considerando a palavra MATRIZ, determine o número de anagramas que:

a) começam por MA. c) tenhas as letras M e A juntas.


b) tenham as letras M e A juntas, nessa ordem.

Problema 54. Considere a palavra CONTAGEM. Determine o número de anagramas que

a) começam com A e terminam com E. c) começam e terminam com vogal.

d) têm a letra T antes da letra M (por exemplo, a


b) começam com A ou terminam com E. própria palavra CONTAGEM).

Problema 55. Em quantos anagramas da palavra QUEIJO as vogais não aparecem todas juntas?
Problema 56. De quantas maneiras três homens e três mulheres podem ficar em fila, de modo que os homens
fiquem intercalados pelas mulheres?
Problema 57. Três ingleses, quatro americanos e cinco franceses serão dispostos em fila (dispostos em linha
reta) de modo que pessoas de mesma nacionalidade estejam sempre juntas. De quantas maneiras distintas a
fila poderá ser formada de modo que o primeiro da fila seja um francês?
Problema 58. O setor de recursos humanos de uma empresa vai realizar uma entrevista com 120 candidatos
a uma vaga de contador. Por sorteio, eles pretendem atribuir a cada candidato um número, colocar a lista de
números em ordem numérica crescente e usá-la para convocar os interessados. Acontece que, por um defeito
no computador, foram gerados números com 5 algarismos distintos e em nenhum deles aparecem dı́gitos
pares. Em razão disso, a ordem de chamada do candidato que tiver o número 75913 é

a) 24. b) 31. c) 32. d) 88. e) 89.

Problema 59. Com os algarismos 2, 3, 4, 5, 6 formam-se todos os números de 5 algarismos distintos. Deter-
mine a soma de todos eles.
Problema 60. Uma lotação possui três bancos para passageiros, cada um com três lugares, e deve transportar
os três membros da famı́lia SOUZA, o casal LÚCIA e MAURO e mais quatro pessoas. Além disso, a famı́lia
SOUZA quer ocupar um mesmo banco e LÚCIA e MAURO querem sentar-se lado a lado. Nessas condições, o
número de maneiras de se dispor as nove pessoas na lotação é igual a

a) 928. b) 1152. c) 1828. d) 2412. e) 3456.

Problema 61. Quantos números de seis algarismos distintos podemos formar usando os dı́gitos 1, 2, 3, 4, 5, 6,
nos quais o 1 e o 2 nunca ocupam posições adjacentes, mas o 3 e o 4 sempre ocupam posições adjacentes?

a) 144. b) 180. c) 240. d) 288. e) 360.

6
POTI 2015 − Combinatória − Nı́vel 2 − Aula 0 − Professores Cleber Assis, Samuel Barbosa e Tiago Miranda

Problema 62. Dos anagramas da palavra CASTELO, quantos têm as vogais em ordem alfabética e juntas?

a) 180. b) 144. c) 120. d) 720. e) 360.

Problema 63.

a) Mostre uma maneira de separar todos os números de 1 a 16 em quatro conjuntos com quatro números
cada, de modo que cada conjunto tenha mesma soma.

b) Mostre que existem pelo menos 1024 maneiras de escrever os números de 1 até 16 em cada uma das
casinhas de um tabuleiro 4 × 4 de modo que a soma dos números de cada linha seja igual.

7
POTI 2015 − Combinatória − Nı́vel 2 − Aula 0 − Professores Cleber Assis, Samuel Barbosa e Tiago Miranda

4 Combinações
Problema 64. Numa sala há 6 pessoas e cada uma cumprimenta todas as outras pessoas com um único
aperto de mão. Quantos foram os apertos de mão?
Problema 65. De quantas formas podemos escolher 2 pessoas, de um grupo de 5, para uma viagem?
Problema 66. Dispondo de 6 frutas, quantas vitaminas podemos fazer utilizando exatamente três destas
frutas?
Problema 67. Quantos drinks podem ser feitos com três bebidas, se dispomos de cinco tipos de bebidas?
Problema 68. São dados 10 pontos no plano, de maneira que não existe reta que contenha mais de dois
destes pontos.

a) Qual o número de retas que contém dois destes pontos?

b) Quantos triângulos podem ser desenhados, cujos vértices são três destes pontos?

c) Quantos heptágonos podem ser desenhados, cujos vértices são sete destes pontos?

Problema 69. Dado o conjunto {1, 2, 3, 4, 5, 6}. Quantos são os subconjuntos com apenas

a) 2 elementos? b) 4 elementos?

Problema 70. Quantos são os subconjuntos de {1, 2, 3, 4, 5} que possuem apenas 3 elementos?
Problema 71. Num torneio com 6 times, cada time joga com cada um dos outros uma única vez. Quantos
são os jogos?
Problema 72. O volante da Mega-Sena contém 60 números (cada um chamado de dezena), que são
01, 02, 03, ..., 60. O resultado de um sorteio é composto de 6 dezenas, sorteadas entre as 60 dezenas.

a) Quantos são os resultados possı́veis?

b) Quantos resultados são formados por 4 números pares e 2 números ı́mpares?

c) Quantos são os resultados contendo o número 13?

Problema 73. Em grupo de 14 pessoas, existem 5 médicos, 6 advogados e 3 engenheiros. Quantas comissões
de 7 pessoas podem ser formadas, cada qual constituı́da de 3 médicos, 2 advogados e 2 engenheiros?
Problema 74. Em um grupo de 10 pessoas, das quais figuram Ana, Beatriz, Carla e Daniela. Quantas
comissões com cinco pessoas podemos formar

a) ao todo?

b) nas quais figura Ana, mas não Beatriz?

c) nas quais figuram Ana ou Beatriz ou Carla ou Daniela, mas nunca as quatro juntas?

Problema 75. Sejam os vértices de um octógono regular.

a) Quantos triângulos podemos obter unindo três destes vértices?

b) Destes triângulos, quantos são retângulos?

Problema 76. Quantas são as diagonais de um decágono convexo?


Problema 77. Em tabuleiro de xadrez (8 x 8), quantos retângulos podemos desenhar cujos lados estão sobre
as linhas deste tabuleiro?
Problema 78. Dadas duas retas paralelas r e s. Sobre r tomam-se 5 pontos e sobre s tomam-se 4 pontos.
Quantos triângulos podemos formar com vértices em 3 desses 9 pontos?

8
POTI 2015 − Combinatória − Nı́vel 2 − Aula 0 − Professores Cleber Assis, Samuel Barbosa e Tiago Miranda

Problema 79. De um pelotão de 10 soldados, quantas equipes de cinco soldados podem ser formadas se em
cada equipe um soldado é destacado como lı́der?
Problema 80. Uma estante de biblioteca tem 16 livros: 11 exemplares do livro COMBINATÓRIA É FÁCIL
e 5 exemplares do livro COMBINATÓRIA NÃO É DIFÍCIL. Considere que os livros de mesmo tı́tulo sejam
indistinguı́veis. Determine de quantas maneiras diferentes podemos dispor os 16 livros na estante de modo
que dois exemplares de COMBINATÓRIA NÃO É DIFÍCIL nunca estejam juntos.
Problema 81. Em todos os 53 finais de semana do ano 2000, Júlia irá convidar duas de suas amigas para sua
casa em Teresópolis, sendo que nunca o mesmo par de amigas se repetirá durante o ano.

a) Determine o maior número possı́vel de amigas que Júlia poderá convidar.

b) Determine o menor número possı́vel de amigas que Júlia poderá convidar.

Problema 82. De quantas maneiras podem ser escolhidos três números naturais distintos, de 1 a 30, de
modo que sua soma seja par?
Problema 83. O número 2568 possui dı́gitos em ordem crescente. Os números 5667 e 3769 não possuem
dı́gitos em ordem crescente. Quantos são os números naturais entre 1000 e 9999 que possuem seus dı́gitos em
ordem crescente?
Problema 84. Led, um famoso herói de jogos, tem um novo desafio: abrir o portal do dragão. O portal
possui 10 cadeados distintos. Para o portal ser aberto, o herói deve possuir pelo menos uma chave para cada
cadeado. Para conseguir as chaves dos cadeados, Led deve abrir caixas espalhadas pelo jogo. Existem 45
caixas em tal jogo e cada uma delas contém duas chaves distintas. Além disso, cada chave abre exatamente
um dos 10 cadeados, duas chaves de uma mesma caixa abrem cadeados diferentes e não existem duas caixas
tais que suas chaves abrem exatamente os mesmos dois cadeados. Qual o número mı́nimo de caixas que Led
deve abrir para garantir a posse de 10 chaves distintas e assim abrir o portal?

9
POTI 2015 − Combinatória − Nı́vel 2 − Aula 0 − Professores Cleber Assis, Samuel Barbosa e Tiago Miranda

5 Permutação com Repetição (com Elementos nem Todos Distintos)


Problema 85. Quais são os anagramas que se pode formar com as letras da palavra CASA?
Problema 86. Quantos são os anagramas que se pode formar com as letras da palavra CASA?
Problema 87. Quantos são os anagramas que se pode formar com as letras da palavra ARARA?
Problema 88. Com dois algarismos 1, dois algarismos 2 e três algarismos 3, quantos números de sete
algarismos podem ser formados?
Problema 89. Quantos anagramas podem ser formados com as letras da palavra ARQUIMEDES que

a) começam e terminam com a letra E?

b) não possuem vogais nem consoantes consecutivas?

Problema 90. Quantos são os anagramas da palavra BANANADA que começam com consoante?
Problema 91. Quantos são os anagramas que se pode formar com as letras da palavra BATATA nos quais

a) as vogais estejam sempre juntas?

b) vogais e consoantes estejam intercaladas?

c) a letra B esteja sempre entre as letras T? (não necessariamente consecutivas)

Problema 92. De quantas maneiras podemos alinhar 8 moedas sobre uma mesa, sendo 4 de R$0, 25 e 4 de
R$0, 50?
Problema 93. Quinze pessoas, sendo 5 homens de alturas diferentes e 10 mulheres também de alturas
diferentes, devem ser dispostas em fila, obedecendo ao critério: homens em ordem crescente de altura e
mulheres em ordem decrescente de altura. De quantos modos diferentes essas 15 pessoas podem ser dispostas
na fila?
Problema 94. Uma partı́cula desloca-se sobre uma reta, percorrendo 1cm para a esquerda ou para a direita
a cada movimento. Calcule de quantas maneiras diferentes a partı́cula pode realizar uma sequência de 10
movimentos terminados na posição de partida.
Problema 95. De quantas maneiras diferentes um professor pode premiar cinco alunos com três bombons
exatamente iguais? (um aluno pode receber mais de um bombom)
Problema 96. Quantas soluções compostas apenas por números naturais possui a equação x + y + z = 7?
Problema 97. Quantas soluções compostas apenas por números inteiros positivos possui a equação x + y +
z = 7?
Problema 98. Uma aranha tem uma meia e um sapato para cada um de seus oito pés. De quantas maneiras
diferentes a aranha pode se calçar admitindo que a meia tem que ser colocada antes do sapato?

10
POTI 2015 − Combinatória − Nı́vel 2 − Aula 0 − Professores Cleber Assis, Samuel Barbosa e Tiago Miranda

6 Permutações Circulares
Problema 99. Dois colares de pérolas serão considerados iguais se um deles puder ser obtido através de
uma rotação do outro, como ilustra a figura 3.

Figura 3: Colares Iguais.

De quantas formas 4 pérolas distintas (A, B, C e D) podem ser usadas para formar um colar circular?
Problema 100. Um grupo de 6 pessoas, incluindo Nilton e Lucimar, decide jogar cartas com rodadas
circulares. Após a jogada de um jogador, o próximo a jogar é aquele que está à sua direita.

a) Por questões estratégicas, Nilton decide se posicionar sempre imediatamente à direita de Lucimar. De
quantas formas esses 6 jogadores podem sentar ao redor da mesa?

b) Suponha que agora Nilton deseja ficar em qualquer um dos dois lado de Lucimar. A resposta anterior
muda?

Problema 101. De quantas maneiras 6 pessoas podem se sentar em torno de uma mesa circular?
Problema 102. Um grupo 6 crianças decide brincar de ciranda dando as mãos e fazendo uma roda. Dentre
elas estão Aline, Bianca e Carla que são muito amigas e querem sempre ficar juntas. Nessa condição, qual o
número de rodas distintas que podem ser formadas?
Problema 103. De quantos modos podemos formar uma roda com 7 crianças de modo que duas determina-
das dessas crianças não fiquem juntas?
Problema 104. De quantos modos 7 crianças, entre elas João e Maria, podem brincar de roda, ficando João
sempre ao lado de Maria?
Problema 105. Em uma brincadeira em um programa de TV, 6 casais devem se sentar em bancos arrumados
de modo circular com a seguinte restrição: homens e mulheres devem se sentar de modo alternado, cada
homem ao lado apenas de mulheres e vice-versa. De quantas maneiras esses casais podem se arrumar para a
brincadeira?
Problema 106. Fábio, Denise e Ledo vão brincar de roda, juntamente com outras 5 pessoas. De quantas
formas essa roda poderá ser formada, de modo que os três fiquem juntos, mas com Denise entre Fábio e Ledo?
Problema 107. Um grupo constituı́do por 4 casais se sentará entorno de uma mesa com 8 cadeiras. As
pessoas se sentarão de modo alternado pelo sexo e, além disso, João e Maria estão brigados e não querem
sentar-se lado a lado. Quantas arrumações diferentes poderão ser feitas com essas pessoas sentando-se nos
lugares disponı́veis?
Problema 108. De quantos modos 5 meninos e 5 meninas podem formar um ciranda com as crianças dos
mesmo sexo todas juntas?
Problema 109. De quantas maneiras podemos dispor 5 casais e uma ciranda de modo que:

a) cada mulher esteja ao lado do seu marido?

b) cada mulher esteja ao lado do seu marido e pessoas do mesmo sexo não possam ficar juntas?

11
POTI 2015 − Combinatória − Nı́vel 2 − Aula 0 − Professores Cleber Assis, Samuel Barbosa e Tiago Miranda

Problema 110. Uma pirâmide pentagonal regular deve ser colorida, cada face com uma única cor, usando 6
cores distintas. De quantos modos isso pode ser feito?
Problema 111. De quantos modos 12 crianças podem ocupar seis bancos com dois lugares cada em uma
roda gigante?
Problema 112. São dados n pontos em cı́rculo. Quantos n−ágonos (não necessariamente convexos) existem
com vértices nesses pontos?
Problema 113. Quantos dados diferentes existem com a soma das faces opostas igual a 7?
Problema 114. Uma pulseira deve ser cravejada com um rubi, uma esmeralda, um topázio, uma água-
marinha, uma turmalina e uma ametista. De quantos modos isso pode ser feito supondo:

a) que a pulseira tem fecho e um relógio engastado no fecho?

b) que a pulseira tem fecho?

c) que a pulseira não tem fecho e o braço só pode entrar na pulseira em um sentido?

d) que a pulseira não tem fecho e o braço pode entrar na pulseira nos dois sentidos?

Problema 115. Dos 12 estudantes da uma turma, seis serão escolhidos para participar de um debate em
uma mesa circular. José, Cléber, Márcia e Luı́za só irão se forem juntos; de tal forma que Márcia e Luı́za vão
sentar lado a lado e o Samuel e o Cléber nunca irão sentar lado a lado à mesa. De quantas maneiras distintas
podem se sentar?
Problema 116. As 8 faces de um prisma hexagonal regular devem ser pintadas, usando oito cores distintas,
sem que haja repetição. De quantos modos isso pode ser feito?

12
POTI 2015 − Combinatória − Nı́vel 2 − Aula 0 − Professores Cleber Assis, Samuel Barbosa e Tiago Miranda

7 Combinações Completas
Problema 117. Observe o modelo e depois faça o que se pede.

A equação x + y + z = 4 pode ser resolvida, em U = N, pelas triplas (4, 0, 0) ou (1, 2, 1)


ou (0, 1, 3) ou várias outras. Cada tripla pode ser associada à distribuição de bolas em
caixas, a saber:

i) (4, 0, 0) distribui-se em

••• • + ••• • + ••• • ;

ii) (1, 2, 1) distribui-se em

•• • • + ••• • + •• • • ; e

iii) (0, 1, 3) distribui-se em

••• • + •• • • + ••• • .

Os três casos destacados ilustram uma mudança de posição entre 6 elementos, dos quais,
quatro são • e dois são +. Esse método é chamado de Combinações Completas (ou com
elementos Repetidos), com o sı́mbolo “CR”, nesse caso, sendo CR3,4 , e serve para calcular
o número de soluções inteiras (não negativas) em equações lineares. Agora, analisando
o disposto, isso também pode ser calculado, de modo análogo pelas permutações com
elementos repetidos, usando a fórmula P64,2 = 15, ou seja, 15 soluções inteiras não
negativas.
Calcule, nos universos destacados, a quantidade de soluções das equações abaixo.
a) x + y = 3, com U = N.

b) a + b + c = 7, com U = N.

c) A + B + C + D = 9, com U = N∗ .
Problema 118. Uma fábrica possui 3 cores diferentes para pintar 6 carros iguais, cada um com uma cor. De
quantos modos isso pode ser feito?
Problema 119. Um dominó comum possui peças que vão do zero/zero até o seis/seis. Fazendo todas as
duplas possı́veis entre os números {0, 1, 2, 3, 4, 5, 6}, podendo haver repetições (duplas com o mesmo número).
Quantas peças existem nesse jogo?
Problema 120. Podendo escolher entre 5 tipos de doces e 4 marcas de refrigerante, de quantos modos é
possı́vel fazer um pedido com dois doces e três garrafas de refrigerante?
Problema 121. Uma pessoa quer comprar 6 empadas numa lanchonete. Há empadas de camarão, frango,
legumes e palmito.
a) Sabendo que podem ser compradas de zero a seis empadas de cada tipo, de quantas maneiras distintas
essa compra pode ser feita?

b) Sabendo que ele quer provar todos os sabores das empadas, de quantas maneiras distintas essa compra
pode ser feita?

13
POTI 2015 − Combinatória − Nı́vel 2 − Aula 0 − Professores Cleber Assis, Samuel Barbosa e Tiago Miranda

Problema 122. Quantos são os anagramas da palavra

PARAMETRIZADA

que não possuem duas letras “A” juntas?


Problema 123. Qual o número de soluções inteiras e não negativas de x + y + z = 5?
Problema 124. Quantas são as soluções inteiras não negativas da equação x + y + z = 4 que possuem apenas
uma incógnita nula ?
Problema 125. De quantas formas podemos colocar 6 anéis iguais em 4 dedos?
Problema 126. Há seis modos distintos de guardar dois cadernos iguais em três gavetas, são eles:

- guardar os dois na primeira gaveta;

- guardar os dois na segunda gaveta;

- guardar os dois na terceira gaveta;

- guardar um na primeira gaveta e o outro, na segunda;

- guardar um na primeira gaveta e o outro, na terceira; e

- guardar um na segunda gaveta e o outro, na terceira.

Qual o número de modos distintos de guardar três cadernos iguais em três gavetas?
Problema 127. Calcule o número de maneiras diferentes pelas quais podemos repartir uma dúzia de balas
iguais entre três crianças, de modo que cada uma receba pelo menos uma bala.
Problema 128. Uma loja vende barras de chocolate de diversos sabores. Em uma promoção era possı́vel
comprar três barras de chocolate com descontos, desde que essas fossem dos sabores: ao leite, amargo, branco
ou com amêndoas, repetidos ou não. Assim, um cliente para comprar as três barras na promoção poderá
escolher os sabores de n modos distintos. Qual o valor de n?
Problema 129. Uma pessoa dispõe de balas de hortelã, caramelo e coco, cada uma com apenas um sabor.
Ele pretende “montar” saquinhos com 13 balas cada, de modo que em cada saquinho haja no mı́nimo 3 balas
de cada sabor. Um saquinho se diferencia do outro pela quantidade de balas de cada sabor. Sendo assim,
quantos saquinhos diferentes podem ser “montados”?
Problema 130. Qual o número de soluções inteiras e não negativas de x + y + z ≤ 6?
Problema 131. Quantos números inteiros entre 1 e 10000 têm soma dos seus algarismos igual a 6?
Problema 132. De quantos modos podemos formar um subconjunto com 4 elementos do conjunto {1, 2, 3, 4, 5, 6, 7, 8, 9}
de modo que não haja números consecutivos?
Problema 133. Os números de 1 até 10 foram arrumados em volta de um cı́rculo em ordem crescente até
chegar ao número 10. De quantos modos podemos escolher 4 deles sem que haja dois vizinhos no cı́rculo?
Problema 134. Uma pessoa deseja escolher 3 dias da semana para ir à academia. De quantas formas ela
pode montar o seu horário se ela não pode ir em 2 dias consecutivos?
Problema 135. Sejam Im = {1, 2, . . . , m} e In = {1, 2, . . . , n}, com m ≤ n. Quantas são as possı́veis funções
f : Im −→ In ...

a) ... ? c) ... estritamente crescentes?

b) ... que são injetoras? d) ... não decrescentes?

14
POTI 2015 − Combinatória − Nı́vel 2 − Aula 0 − Professores Cleber Assis, Samuel Barbosa e Tiago Miranda

Problema 136. De quantas formas podemos colocar 6 anéis diferentes em 4 dedos?


Problema 137. Doze cavaleiros estão sentados em torno de uma mesa redonda. Cada um dos doze cavaleiros
considera seus dois vizinhos como rivais. Deseja-se formar um grupo de cinco cavaleiros para libertar uma
princesa, nesse grupo não poderá haver cavaleiros rivais. Determine de quantas maneiras é possı́vel escolher
esse grupo.

15
POTI 2015 − Combinatória − Nı́vel 2 − Aula 0 − Professores Cleber Assis, Samuel Barbosa e Tiago Miranda

8 Princı́pio da Casa dos Pombos (PCP)


Problema 138. Entre n pessoas existem duas com o mesmo signo. Qual o menor valor de n que garante esse
fato?
Problema 139. Numa floresta há 1000 jaqueiras. É conhecido que uma jaqueira não tem mais do que 600
frutos. Prove que existem 2 jaqueiras que têm a mesma quantidade de frutos.
Problema 140. Uma pessoa entrou num quarto escuro, sem enxergar absolutamente nada, e abriu uma
gaveta na qual havia exatamente: 20 meias pretas; 15 meias brancas; e 10 meias marrons.
Todas estavam misturados e eram indistinguı́veis ao tato. Qual a quantidade mı́nima de meias que essa pessoa
deve retirar para que tenha certeza de ter retirado:

a) um par de meias de mesma cor?

b) um par de meias brancas?

Problema 141. A “Média aritmética” de n termos é a razão entre a soma dos n termos pela respectiva
quantidade n. Em um grupo de 54 pessoas, qual a média aritmética de aniversariantes por mês?
Problema 142. Uma relação entre a média aritmética e o PCP está enunciada abaixo.

Proposição 1 Seja x a média aritmética de uma lista de números. Então ao menos um desses termos é maior
do que ou igual a x. E quando algum desses números for maior do que x, então haverá outro que será menor
do que essa média (e vice-versa).

Seguem exemplos numéricos para complementar a teoria.

a) Para a lista 1, 2, 3, 6 e 8. A média será

1+2+3+6+8
x= =4
5
com 1, 2 e 3 menores do que x = 4 e 6 e 8 maior.

b) Para a lista 6, 6, 6, 6 e 6. A média será


6+6+6+6+6
x= =6
5
com todos os elementos iguais a média.

Responda os itens abaixo aplicando o que enuncia a proposição 1.

a) Demonstre que num grupo de 54 pessoas, ao menos 5 fazem aniversário no mês.

b) 40100 candidatos estão fazendo uma prova de 20 questões, com 5 alternativas por questão. Suponha que
nenhum candidato deixe de responder alguma questão. Considere a afirmação: “Pelo menos k candidatos
responderam de modo idêntico as 4 primeiras questões da prova.” Determine o maior valor de n para o
qual a afirmação é certamente verdadeira.

c) 98305 candidatos estão fazendo uma prova de 20 questões de certo/errado. Suponha que nenhum candidato
deixe de responder alguma questão. Considere a afirmação: “Pelo menos 4 candidatos responderam de
modo idêntico as n primeiras questões da prova.” Determine o maior valor de n para o qual a afirmação é
certamente verdadeira.

16
POTI 2015 − Combinatória − Nı́vel 2 − Aula 0 − Professores Cleber Assis, Samuel Barbosa e Tiago Miranda

Problema 143. Demonstre a proposição 1 da questão anterior.


Problema 144. Considere um grupo com 90 torcedores, cada um torcendo ou pelo Flamengo, ou pelo
Botafogo, ou pelo Fluminense ou pelo Vasco. Mostre que nesse grupo há pelo menos 23 torcedores de um
mesmo time.
Problema 145. Uma prova de concurso terá 10 questões de múltipla escolha, com cinco alternativas cada.
Qual é o menor número de candidatos para o qual poderemos garantir que pelo menos dois deles darão
exatamente as mesmas respostas para todas as questões?
Problema 146. Depois de d lançamentos de um dado de 6 faces temos certeza que uma das faces saiu mais
de 5 vezes. Qual o valor de d?
Problema 147. Prove que se 5 pontos forem tomados no interior √ de um quadrado de lado 2, ao menos dois
desses pontos ficarão numa distância menor do que ou igual a 2.
Problema 148. A soma das idades de 5 estudantes é igual a 86 anos. Prove que podem ser escolhidos 3 cuja
soma das idades é maior que 51 anos.
Problema 149. Dados 9 inteiros quaisquer. Prove que a diferença entre dois deles é divisı́vel por 8.
Problema 150. Uma prova de concurso é formada por questões de múltipla escolha, com 4 alternativas por
questão. Admita que nenhum candidato deixe questões sem responder.

a) Qual é o número mı́nimo de candidatos para que seja possı́vel garantir que pelo menos 3 deles darão
exatamente as mesmas respostas nas 5 primeiras questões?

b) Qual é o valor máximo de n para o qual é possı́vel garantir que, em um concurso com 1000 candidatos,
pelo menos 2 darão as mesmas respostas nas n primeiras questões?

Problema 151. Qual o menor número de pessoas num grupo para garantir que pelo menos 4 nasceram no
mesmo mês?
Problema 152. Uma máquina contém pequenas bolas de borracha de 10 cores distintas, sendo 10 bolas de
cada cor. Ao inserir uma moeda, uma bola é expelida ao acaso. Para garantir a retirar de 4 bolas da mesma
cor, o menor número de moedas inseridas na máquina corresponde a:
Problema 153. Escolhendo-se aleatoriamente 41 elementos do conjunto {1, 2, 3, · · · , 80}, mostre que, entre
os escolhidos, há dois números tais que um divide o outro.
Problema 154. Em uma reunião com n pessoas. Supondo que se A conhece B, então B conhece A. Mostre
que existem duas pessoas que conhecem a mesma quantidade de pessoas.
Problema 155. Mostre que entre sete inteiros positivos distintos menores do que 127 sempre haverá um par,
y
digamos x e y tal que 1 < ≤ 2.
x
Problema 156. Mostre que em todo subconjunto de n + 1 elementos do conjunto {1, 2, 3, · · · , 2n − 1, 2n} há
dois números primos entre si.
Problema 157. Dado um inteiro positivo n, mostre que existe algum múltiplo dele que se escreve apenas
com os algarismos 0 e 1.
Problema 158. Mostre que existe algum múltiplo de 7 que se escreve apenas com uma sequência de
algarismos iguais a 1.
Problema 159. Mostre que existe um número da forma 199 . . . 91 (com pelo menos três noves) que é múltiplo
de 1991.
Problema 160. Prove que todo conjunto de 10 números de dois dı́gitos possui dois subconjuntos disjuntos
com a mesma soma dos elementos
Problema 161. Dezessete pessoas se correspondem por e-mail, cada uma com todas as outras. Nessas
correspondências são debatidos os temas I, I I e I I I. Prove que ao menos 3 pessoas debatem o mesmo tema
entre si.

17
POTI 2015 − Combinatória − Nı́vel 2 − Aula 0 − Professores Cleber Assis, Samuel Barbosa e Tiago Miranda

Problema 162. Um enxadrista, durante 11 semanas, joga pelos menos uma partida por dia, mas não joga
mais de 12 partidas por semana. Mostre que é possı́vel achar um conjunto de dias consecutivos durante os
quais ele jogou exatamente 20 partidas.

18
POTI 2015 − Combinatória − Nı́vel 2 − Aula 0 − Professores Cleber Assis, Samuel Barbosa e Tiago Miranda

Respostas e Soluções.
1. (Extraı́do da Vı́deo Aula)
Observe que se tivéssemos começado a contar pelo número 1, não haveria dúvidas quanto a quantidade de
elementos do conjunto {1, 2, 3, . . . , 908}. Como começamos sete unidades a mais que o 1, a resposta automática
seria 908 − 8 = 900. Este é um excelente ponto para lembrar que subtração não indica quantidade e sim
“distância” entre dois números. Ao calcularmos a distância do 908 (ou de m) até o 8 (ou de n) estamos contando
apenas o espaço entre eles, sendo assim, após a subtração devemos adicionar uma unidade para calcular a
exata quantidade. Por fim, a quantidade será
908 − 8 + 1 = 901 números.
De modo geral, a quantidade de números inteiros de m até n, sendo m > n, é m − n + 1.

Outra solução: Uma outra estratégia é fazermos um ajuste na contagem deslocando cada valor até o ponto
inicial, o 1, e depois simplesmente olhar onde terminou. Como 8 − 7 = 1 e 908 − 7 = 901, a quantidade de
elementos do conjunto {8, 9, 10, . . . , 908} é mesma que a do conjunto {1, 2, 3, . . . , 901}, isto é, 901 elementos.
2. Perceba que poderı́amos dividir todos os elementos do conjunto por 7 para começarmos a contar do 1
ficando com {1, 2, 3, . . . , 97, 98}. Portanto, há 98 elementos no conjunto inicial.
3. Perceba que podemos subtrair 9 de cada elemento do conjunto inicial e ficaremos com o conjunto
{5, 10, 15, . . . , 995, 1000}. Agora, dividindo todos os elementos do novo conjunto por 5 ficamos com
{1, 2, 3, . . . , 199, 200}.
Portanto, há 200 elementos no conjunto inicial.
4. Como há 12 signos do zodı́aco, basta n = 13 para que duas pessoas tenham o mesmo signo. A ideia é
pensar nos Signos como as casas e nas pessoas como os pombos.
• ◦
Pombo sem casa, o 13 elemento.

• • • ··· •
Casa 1 Casa 2 Casa 3 Casa 12

Logo, há 12 casas, e para garantir que alguma das casas tenha dois pombos, basta ter n = 12 + 1 pombos.
5. (Extraı́do da Vı́deo Aula)
Observe que os números usados são
{10, 11, 12, 13, 14, 15, 16, 17, 18, 19, 20}.
São 20 − 10 + 1 = 11 números, cada um com dois algarismos, logo foram usados 11 × 2 = 22 algarismos.
6. Considere as três cores como sendo as casas e as meias retiradas como os pombos.

a) Pelo Princı́pio da Casa dos Pombos, se retirarmos b) Observe que o cenário mais difı́cil para o objetivo é
4 meias, pelo menos duas delas terão a mesma cor. retirar todas as meias de cor preta, todas as meias
Para ver que esse é o número mı́nimo, note que é de cor marrom e depois o par de cor branca. As-
possı́vel pegarmos uma meia de cada cor nas três sim, deveremos retirar 20 + 10 + 2 = 32 meias para
primeiras retiradas e não formarmos um par. garantir o par de cor branca.

• •
4a meia. 32a meia

• • • 20 meias • 10 meias
Meias Pretas Meias Brancas Meias Marrons Meias Pretas Meias Brancas Meias Marrons

Resposta: 4 meias. Resposta: 32 meias.

19
POTI 2015 − Combinatória − Nı́vel 2 − Aula 0 − Professores Cleber Assis, Samuel Barbosa e Tiago Miranda

7. Sejam x e y números inteiros pares, então podemos escrevê-los como x = 2a e y = 2b, para a e b inteiros.
Analogamente, se w e z são números inteiros ı́mpares, podemos escrever w = 2c + 1 e z = 2d + 1, com c e d
inteiros.

a) d)

x + y = 2a + 2b w · z = (2c + 1)(2d + 1)
= 2( a + b ), = 2(2cd + c + d) + 1,

é par. é ı́mpar.

b) e)

w + z = 2c + 1 + 2d + 1 x · y = 2a · 2b
= 2( c + d + 1), = 2 · 2ab,

é par. é par.

c) f)

x + w = 2a + 2c + 1 x · w = 2a · (2c + 1)
= 2( a + c) + 1, = 2(2ac + a),

é ı́mpar. é par.
8. (Extraı́do da Vı́deo Aula)
Observe que de 40 até 99 há 99 − 40 + 1 = 60 números de dois algarismos cada, logo foram utilizados
60 × 2 = 120 algarismos. Agora, de 100 até 999 há
999 − 100 + 1 = 900 números de três algarismos, o que totaliza 900 × 3 = 2700 algarismos. Seguindo
de 1000 até 1200 são 1200 − 1000 + 1 = 201 números com quatro algarismos, ou seja, 201 × 4 = 804. Por fim,
teremos
120 + 2700 + 804 = 3624 algarismos utilizados.
9. A soma pedida é
S = 100 + 101 + 102 + · · · + 999
900 · (100 + 999)
=
2
= 494550.
10. (Extraı́do da Vı́deo Aula)

a) Como são 12 meses, com 24 pessoas no grupo não é possı́vel garantir que três delas façam aniversário no
mesmo mês, afinal poderı́amos ter exatamente 2 em cada mês. Agora, com 25 pessoas teremos certeza pois,
se cada mês receber no máximo dois aniversariantes, a 25a pessoa ficará sem data de aniversário possı́vel.
Logo, é preciso, no mı́nimo, 25 pessoas.
b) Como são 7 dias na semana, não basta termos 21 pessoas, pois poderı́amos ter 3 pessoas nascidas em cada
dia. Com a 22a pessoa, com certeza, haverá um dia no qual 4 pessoas nasceram. Portanto, no mı́nimo,
deveremos ter 22 pessoas.
11. (Extraı́do da Vı́deo Aula)
Se tirarmos 8 blusas, podem ser todas cinzas; tirando 10 blusas, podem ser todas amarelas; e sendo 12,
podemos ser todas beges. No caso de 18 poderiam ser as cinzas e as amarelos; para 20, as beges e as cinzas; e
para 22 as amarelas e as beges. Mas, com certeza, se forem 23 teremos uma de cada cor.

20
POTI 2015 − Combinatória − Nı́vel 2 − Aula 0 − Professores Cleber Assis, Samuel Barbosa e Tiago Miranda

12. (Adaptado do livro Cı́rculos Matemáticos)


Observe que cada disco dentado gira no sentido inverso que o dos seus vizinhos. Como o disco 1 gira no
sentido horário, o 2 ficará no anti-horário, o 3 no horário, e assim por diante. O que conclui que os ı́mpares
ficaram no sentido horário e os pares no anti-horário. Portanto, as proposições verdadeiras são as i e iii. Serão
apenas 2 proposições corretas.
13. Como são 4 cores, poderemos dar o “azar” de em várias retiradas sempre chegarmos em 3 bolas de cada
cor, sem antes obtermos na 4a bola de cor repetida. Tirar 3 bolas de cada cor pode ser obtido após 4 × 3 = 12
retiradas. Daı́, com certeza, a 13a bola repetirá pela quarta vez alguma cor. Portanto, temos que retirar, no
mı́nimo, 13 bolas.
14. (Extraı́do da Vı́deo Aula)
Observe que se escrevermos a soma pedida no sentido inverso obteremos
S = 2014 + 2013 + 2012 + 2011 + · · · + 4 + 3 + 2 + 1
e, somando a forma original com sua escrita invertida, também obteremos
S = 1 + 2 +···+ 2013 + 2014
S = 2014 + 2013 +···+ 2 + 1
2S = 2015 + 2015 +···+ 2015 + 2015
2S = 2014 × 2015
S = 1007 × 2015,
que é o produto de números ı́mpares. Logo a soma dada é ı́mpar.
Observação: Veja que
2S = 2015
| + 2015 + ·{z
· · + 2015 + 2015}
2014 parcelas iguas a 2015.
pode ser facilmente transformada em uma multiplicação em função da igualdade das parcelas, resultando em
2S = 2014 × 2015.
Essa ideia pode ser aplicada na soma
S = 1 + 2 + 3 + · · · + (n − 2) + (n − 1) + n.
Repetindo o método chegaremos a
S = 1 + 2 +···+ ( n − 1) + n
S = n + ( n − 1) +···+ 2 + 1
2S = ( n + 1) + ( n + 1) +···+ ( n + 1) + ( n + 1)
2S = n × ( n + 1)
que produz a fórmula para a soma S dos naturais de 1 até n:
n · ( n + 1)
S= .
2
15. (Adaptado da Vı́deo Aula)
Observe que se isso for possı́vel, poderemos separar os números de 1 até 10 em dois conjuntos de modo que a
soma S dos elementos do primeiro seja igual a soma dos elementos do segundo. Como esses conjuntos têm
todos os números citados, então
S + S = 1 + 2 + 3 + 4 + · · · + 10
10 · (1 + 10)
2S =
2
2S = 55.
Mas 2S é um número par e 55 é um número ı́mpar, então essa equação não tem solução inteira, daı́, não tem
como cumprir o que o problema perguntou.

21
POTI 2015 − Combinatória − Nı́vel 2 − Aula 0 − Professores Cleber Assis, Samuel Barbosa e Tiago Miranda

16. Primeiro observe que não poderemos ter qualquer cor com 13 bolas, apenas conseguiremos isso com
as brancas e as verdes. Sendo assim, por “azar”, poderı́amos ter tirado todas as cores que não resolvem o
problema, totalizando 7 + 3 + 11 = 21 bolas. Agora restam apenas duas cores e como queremos treze bolas de
cor repetida devemos tirar ao menos mais 12 + 12 + 1 = 25. O que resulta em

21 + 25 = 46.
17. (Adaptado da OBMEP)
Observe que se n é ı́mpar, então n2 é ı́mpar, como está provado no exercı́cio 7. Assim, n2 ± n será par. Como
deseja-se um número ı́mpar, basta somarmos um ı́mpar. A resposta está na letra c.

18. Observe que 20102010 possui unidade par, pois é o produto de números pares, já 20112011 ficará com
unidade ı́mpar, 20122012 terá unidade par e essa alternância continuará. Por fim, a paridade resultante será

par + ı́mpar + par + ı́mpar + par + ı́mpar + par = ı́mpar.

19. (Extraı́do do Vestibular da PUC/RJ)


Tome os 12 meses como as casas e as n pessoas como os pombos. Se houver uma distribuição de 3 pessoas em
cada mês, não se chegará ao objetivo do problema e já terı́amos 12 × 3 = 36 pessoas no grupo. Agora basta
que mais uma pessoa seja colocada em qualquer uma das casas para concluir o problema. Portanto, 37 pessoas
num grupo garantem que ao menos 4 nasceram no mesmo mês.

20. (Extraı́do do Vestibular da UERJ/RJ - 2011)


Se retirarmos 30 bolas, é possı́vel que existam 3 bolas de cada cor e o objetivo não será cumprido. Com 31
bolas, pelo menos uma cor terá 4 representantes.

21. (Adaptado da Vı́deo Aula)

i) de 1 até 9 são 9 − 1 + 1 = 9 dı́gitos.

ii) de 10 até 99 são (99 − 10 + 1) × 2 = 180 dı́gitos.

iii) de 100 até 999 são (999 − 100 + 1) × 3 = 2700 dı́gitos.

COmo 9 e 180 são divisı́veis por 3 e 9 + 180 < 1002 < 9 + 180 + 2700, o 1002◦ será o último dı́gito de um
número de três dı́gitos. Observe que de 100 até um número de três algarismos n, temos 100 − n + 1 números
de 3 algarismos, logo, são (n − 100 + 1) × 3 dı́gitos nessa sequência. Queremos encontrar n tal que:

9 + 180 + 3 · (n − 99) = 1002


3 · (n − 99) = 1002 − 189
3 · (n − 99) = 813
3n − 297 = 813
3n = 813 + 297
3n = 1110
1110
n= .
3
n = 370.

Então, ao escrevermos o número 370, teremos 1002 termos na sequência, logo o 1002◦ termo será o 0.

22
POTI 2015 − Combinatória − Nı́vel 2 − Aula 0 − Professores Cleber Assis, Samuel Barbosa e Tiago Miranda

22. Como há 6 faces, para ter certeza que ao menos um delas saiu:

i) 2 vezes, deveremos ter ao menos 7 = 1 · 6 + 1 lançamentos;

ii) 3 vezes, deveremos ter ao menos 13 = 2 · 6 + 1 lançamentos;

iii) 4 vezes, deveremos ter ao menos 19 = 3 · 6 + 1 lançamentos;

iv) 5 vezes, deveremos ter ao menos 26 = 4 · 6 + 1 lançamentos; e

v) 6 vezes, deveremos ter ao menos 31 = 5 · 6 + 1 lançamentos.

A resposta é d = 31 lançamentos. A ideia é pensar que o número em cada face representa uma casa
(6 números = 6 casas). Queremos alguma casa com mais do que d pombos (lançamentos) então deve-se
distribuir os resultados dos lançamentos nas respectivas casas. Se tivermos 6d + 1 lançamentos, não é possı́vel
que cada número tenha saı́do no máximo d vezes e assim teremos uma casa com pelo menos d + 1 pombos.

23. (Extraı́do da Vı́deo Aula)


Os múltiplos de 3 entre 1 e 301 são
{3, 6, 9, . . . , 297, 300}.
A sua soma S pode ser escrita como

S = 3 + 6 + 9 + · · · + 297 + 300
S = 3(1 + 2 + 3 + · · · + 99 + 100)
100 · (1 + 100)
 
S = 3·
2
S = 15150.

24. (Extraı́do da OBMEP)


Observe que podemos desenvolver a equação pedida da seguinte forma:
xy
= 144
x+y
xy = 144x + 144y
xy − 144x − 144y + 1442 = 1442
x (y − 144) − 144(y − 144) = 124
( x − 144)(y − 144) = (22 · 3)4
( x − 144)(y − 144) = 28 · 34 .

Como estamos trabalhando com os números inteiros, ( x − 144) e (y − 144) dividem 1442 , ou seja, basta
calcularmos o número de divisores de 1442 = 28 · 34 . Esse número possui

(8 + 1) · (4 + 1) = 45

divisores inteiros positivos. Como não há restrição para os valores positivos, teremos

90 pares ordenados

que resolvem o problema.

23
POTI 2015 − Combinatória − Nı́vel 2 − Aula 0 − Professores Cleber Assis, Samuel Barbosa e Tiago Miranda

25. (Extraı́do da OBMEP)


Observe que podemos desenvolver a inequação dupla (ou simultânea) da seguinte maneira:
q
2000 < n · (n − 1) < 2005
q 2
2
2000 < n · (n − 1) < 20052

2000 · 2000 < n · (n − 1) < 2005 · 2005.

Então, podemos concluir que


n ∈ {2001, 2002, 2003, 2004, 2005},
totalizando 5 números inteiros e positivos. O que está na letra e.

26. Chame a soma pedida de S2 e siga o que foi iniciado nos exemplos do enunciado até o (n + 1)3 .

23 = 13 + 3 · 12 + 3 · 1 + 1
33 = 23 + 3 · 22 + 3 · 2 + 1
43 = 33 + 3 · 32 + 3 · 3 + 1
53 = 43 + 3 · 42 + 3 · 4 + 1
.. ..
.=.
( n − 1 + 1)3 = ( n − 1)3 + 3 · ( n − 1)2 + 3 · ( n − 1) + 1
( n + 1)3 = n3 + 3 · n2 + 3 · n + 1

Agora, some todos os membros dessas equações observando que todos os termos ao cubo do lado esquerdo se
anulam com os do lado direito, exceto o (n + 1)3 e o 13 . Obtemos assim
 
3 3 n ( n + 1)
( n + 1 ) = 1 + 3 · S2 + 3 · + n.
2

Por fim, chegaremos a


n(n + 1)(2n + 1)
12 + 22 + 32 + · · · + n2 = .
6
27. (Adaptado da Vı́deo Aula)
Utilizaremos a fórmula desenvolvida no exercı́cio 26, pois a área total é equivalente a soma 12 + 22 + 32 +
· · · + 1002 , ou seja, é uma soma de quadrados de números inteiros. Sendo assim, obteremos
100 · 101 · 201
12 + 22 + 32 + · · · + 1002 =
6
= 338350 u.a..

28. Cada computador pode estar conectado a 1, 2, 3, 4 ou 5 outras máquinas. Como há 6 computadores e
cinco opções de conexão, então ao menos dois computadores terão o mesmo número de conexões.

29. (Extraı́do da Vı́deo Aula)

a) Pelo Princı́pio Fundamental de Contagem (PFC), são 3 · 2 = 6 possibilidades.

b) Como, para ir são 6 possibilidades, para voltar também são 6. Pelo PFC, 6 · 6 = 36possibilidades.

c) Como, para ir são 6 possibilidades, mas apenas uma delas foi escolhida, para não repetir estradas na volta,
resta 1 possibilidade de C para B e 2 de B para A. Temos então 6 · 1 · 2 = 12 possibilidades.

24
POTI 2015 − Combinatória − Nı́vel 2 − Aula 0 − Professores Cleber Assis, Samuel Barbosa e Tiago Miranda

30. (Extraı́do da Vı́deo Aula) Iniciando a pintura pela primeira casa, que pode ser pintada com qualquer
uma das quatro cores, seguindo para sua vizinha, que não poderá ser pintada apenas com a cor utilizada na
primeira, e seguindo o mesmo raciocı́nio até a última casa, temos 4 · 3 · 3 · 3 · 3 = 324 possibilidades.

· ... · 2} = 232 palavras.


31. (Extraı́do da Vı́deo Aula) Como cada bit equivale a uma letra, temos 2| · 2{z
32 vezes

32. (Extraı́do da Vı́deo Aula) 4 · 3 · 2 · 1 = 24.

33. Como o algarismo da centena não pode ser 0, o total de possibilidades é 9 · 9 · 8 = 648.

34. Como existem 5 algarismos pares e o algarismo da unidade do milhar não pode ser 0, o total de
possibilidades é 4 · 5 · 5 · 5 = 500.

35. Para ocupar o primeiro lugar do pódium, são 8 possibilidades; para o segundo lugar, sobram apenas 7; e,
para o terceiro, apenas 6. Assim, temos o total de possibilidades expresso por 8 · 7 · 6 = 336.

36. A primeira pessoa tem 6 possibilidades; a segunda, 5; e a terceira, 4. Assim, pelo PFC, são 6 · 5 · 4 = 120
possibilidades.

37. Como são 5 possibilidades para cada questão, o total de maneiras é 5 · 5 · ... · 5 = 510 .

38. Como são 26 letras e 10 algarismos, o total de placas é 26 · 26 · 26 · 10 · 10 · 10 · 10 = 175.760.000.

39. (Extraı́do da Vı́deo Aula)

a) Como não podemos contar com o 0 no inı́cio (unidade do milhar), pelo PFC, temos 7 · 8 · 8 · 8 = 3584
números.

b) 7 · 7 · 6 · 5 = 1470 números.

c) Para enfrentarmos as adversidades logo no inı́cio, seguiremos a sequência (unidade, centena, dezena):
4 · 6 · 6 = 144 números.
40. (Extraı́do da Vı́deo Aula) Vamos dividir o problema em dois casos para que não haja conflito:

i) Números terminados com 0: começando pela unidade, temos 1 · 7 · 6 · 5 = 210 números;

ii) Números terminados em 2, 4 ou 6: (unidade, unidade do milhar, centena, dezena) 3 · 6 · 6 · 5 = 540


números.

Por i e ii, temos que o total de números é 210 + 540 = 750.

41. (Extraı́do de Exercı́cios Resolvidos) Dividindo em dois casos, temos:

i) Com Pedro: como Pedro pode ocupar qualquer um dos três locais do banco traseiro, temos 3 · 6 · 5 · 4 · 3 =
1080;

ii) Sem Pedro: 6 · 5 · 4 · 3 · 2 = 720.


Portanto o total de maneiras diferentes é 1080 + 720 = 1800.
42. (Extraı́do de Exercı́cios Resolvidos)

a) 2 · 2 · 2 = 8.

b) Como existem 2 · 2 · . . . · 2 = 2k sequências possı́veis com exatamente k sı́mbolos, o total de sequências pode
2(1 − 28 )
ser calculado somando-se as quantidades para cada tamanho possı́vel: 2 + 4 + 8 + ... + 28 = = 510.
1−2

25
POTI 2015 − Combinatória − Nı́vel 2 − Aula 0 − Professores Cleber Assis, Samuel Barbosa e Tiago Miranda

43.

a) Se Ana deve ser a primeira, sobram cinco pessoas para cinco lugares, ou seja, 5 · 4 · 3 · 2 · 1 = 120
possibilidades.

b) Como em primeiro deve ficar Ana ou Pedro, temos


2 · 5 · 4 · 3 · 2 · 1 = 240 possibilidades.

c) O total de possibilidades, sem restrições, é 6! = 720. Mas, deste total, subtrairemos as possibilidades nas
quais Ana e Pedro ficam juntos. Assim, temos
720 − 2 · 5! = 480 possibilidades.

44. (OBM − 2013)


Qualquer que seja o número, a soma dos algarismos é 10. A soma dos dois últimos é maior que a soma dos
dois primeiros quando for maior que 5. Isso ocorre para 24, 34, 42, 43. Resposta C.

45. (OBMEP − 2014)


Como os números devem ser ı́mpares e como a soma dos algarismos das unidades e das dezenas deve ser
igual a 16, os números devem terminar em 79 ou 97 (duas possibilidades). Na casa das dezenas de milhar
temos 9 possibilidades, pois os números, tendo cinco algarismos, não podem ter 0 nesta casa. Para a casa
das unidades de milhar temos 10 possibilidades (todos os algarismos de 0 a 9) e, para cada uma das escolhas
anteriores, podemos escolher o algarismo das centenas de duas maneiras distintas, a fim de que a soma de
todos os algarismos do número seja um múltiplo de 5. Logo, há 2 · 9 · 10 · 2 = 360 possibilidades. Resposta D.

46. (OBMEP 2013) Dividindo os possı́veis horários em dois casos, temos:

i) Com aula aos sábados: escolhendo aula sábado, são 3 possibilidades; sua aula à tarde, são 2 possibilidades
de horário e 4 possibilidades de dias. Temos então 3 · 2 · 4 = 24 possibilidades;

ii) Sem aula aos sábados: são 6 possibilidades de dias não consecutivos, sendo um pela manhã outro
pela tarde (2 possibilidades). O horário pela manhã tem 3 possibilidades e pela tarde, 2 possibilidades,
chegando a um total de 6 · 2 · 3 · 2· = 72 possibilidades para este caso.

Assim, o total de possibilidades é 24 + 72 = 96. Resposta A.

47. (Extraı́do da OBM) Existem 4 tipos possı́veis de colunas e as regras do Super Paciência se resumem a não
preenchermos uma certa coluna com a mesma configuração da coluna imediatamente anterior. Assim, uma
vez que Bitonho escolheu os números de uma determinada coluna, ele possui 3 opções de preenchimento para
a próxima. No inı́cio, podemos escolher livremente como preencher a primeira coluna mais à esquerda e isso
pode ser feito de 4 formas. Em seguida, ao preenchermos as próximas colunas à direita, teremos 3 opções.
Portanto, o total de preenchimentos é:

4 · 3 · 3 · 3 · . . . · 3 = 4 · 32013
48. (Extraı́do da Vı́deo Aula)
4 · 3 · 2 · 1 = 24.
49. (Extraı́do da Vı́deo Aula)
P6 = 6! = 6 · 5 · 4 · 3 · 2 · 1· = 720.
50. Como são quatro atividades e ele deverá executar todas elas, temos uma permutação de quatro elementos,
ou seja, P4 = 4! = 24 maneiras diferentes.

51. P5 = 5! = 120.
52. LABO, LAOB, LBAO, LBOA, LOAB, LOBA.

26
POTI 2015 − Combinatória − Nı́vel 2 − Aula 0 − Professores Cleber Assis, Samuel Barbosa e Tiago Miranda

53. (Extraı́do da Vı́deo Aula)


a) Se começam por MA, resta apenas permutar as outras 4 letras, ou seja, P4 = 4! = 24 anagramas.

b) Se duas letras devem estar juntas e em uma determinada ordem, consideramo-nas como um bloco, ou seja,
P5 = 5! = 120 anagramas.

c) Parecido com o item anterior, porém como não existe uma ordem especı́fica para as letras que ficam juntas,
elas devem ser permutadas dentro do bloco. Sendo assim, o número de anagramas é P5 · P2 = 5! · 2! = 240.
54. (Extraı́do da Vı́deo Aula)
a) Resta permutar as outras 6 letras. Segue que o número de anagramas é P6 = 6! = 720

b) Vamos contar a quantidade de anagramas que começam com A, somado à quantidade de anagramas que
terminam com E. Como os anagramas que começam com A e terminam com E foram contados duas vezes,
subtraı́mo-no do resultado. Temos então P7 + P7 − P6 = 7! + 7! − 6! = 10800.

c) Como deve terminar e começar com vogal e são 3 vogais para 2 espaços, segue que é 3 · 2 = 6 o número de
maneiras de organizá-las. Agora, basta permutar as demais. Temos então 6 · P6 = 6 · 720 = 4320 anagramas.
P8
d) Basta pensar que a letra T fica antes da letra M em metade dos anagramas, ou seja, = 20.160 anagramas.
2
Comentário para professores: Esse é um bom momento para sugestionar em sala que identificar as seguintes
estruturas pode simplificar os problemas:
i) Bloco Rı́gido: agrupamento de sı́mbolos sem permutação entre as respectivas posições.

ii) Bloco: agrupamento de sı́mbolos com permutação entre as respectivas posições.


55. (Extraı́do da Vı́deo Aula) Basta subtrair, do total, a quantidade de anagramas nos quais as vogais aparecem
todas juntas, ou seja, P6 − P3 · P4 = 6! − 3!4! = 576 anagramas.
56. (Extraı́do da Vı́deo Aula) Como a fila pode começar com homem ou mulher e para ambos os casos a
quantidade de filas é a mesma, teremos 2 · P3 · P3 = 2 · 3! · 3! = 72 filas diferentes.
57. (Extraı́do da UFF - Extraı́do da Vı́deo Aula) Seja F o grupo formado por franceses, A, o de americanos e I,
o de ingleses, teremos dois tipos de filas: FAI e FIA, ou seja, 2 · P3 · P4 · P5 = 34.560 possibilidades.
58. (ENEM - Extraı́do da Vı́deo Aula) Todas as pessoas cujas senhas iniciam por 1, serão chamadas antes,
ou seja, 4! = 24 pessoas. O mesmo ocorre com as pessoas cujas senhas começam por 3 (24 pessoas) e 5 (24
pessoas). Das senhas que começam com 7, apenas as que tem, na sequência, 1 (6 senhas), 3 (6 senhas), 51 (2
senhas), 53 (2 senhas), são chamadas antes. Portanto, são 24 + 24 + 24 + 6 + 6 + 2 + 2 = 88 pessoas chamadas
antes de quem possuir a senha 75913, ou seja, será o 89o a ser chamado. Resposta E.
59. (Extraı́do da Vı́deo Aula) O total de parcelas desta soma é P5 = 120. Cada um dos 5 algarismos aparece
120
= 24 vezes em cada uma das cinco posições. Somando apenas as unidades, teremos 24(2 + 3 + 4 + 5 + 6) =
5
480. Assim, a soma de todas as parcelas é 480 + 480 · 10 + 480 · 102 + 480 · 103 + 480 · 104 = 5.333.280.
60. (FUVEST - Extraı́do da Vı́deo Aula) Acomodando inicialmente a famı́lia SOUZA, temos 3 · 3! = 18
possibilidades. Agora, o casal poderá escolher entre os dois bancos restantes e, para cada banco são 4
possibilidades, ou seja, 2 · 4 = 8 possibilidades. Por fim, as quatro pessoas restantes em quatro lugares: 4! = 24.
Portanto, o total de possibilidades é 18 · 8 · 24 = 3.456. Resposta E.
61. (ITA) O total de números nos quais 3 e 4 ocupam posições adjacentes é 2P5 = 2 · 5! = 240. Basta agora
subtrair os números em que 1 e 2 ocupam posições adjacentes, que são 2 · 2 · P4 = 4 · 4! = 96. Assim temos
240 − 96 = 144 números. Resposta A.

27
POTI 2015 − Combinatória − Nı́vel 2 − Aula 0 − Professores Cleber Assis, Samuel Barbosa e Tiago Miranda

62. (PUC - PR) Como são três vogais, consideramo-nas como um bloco rı́gido, passando a ter 5 letras. Assim
o total de anagramas é P5 = 120. Resposta C.

63. (Extraı́do do Banco de Problemas da OBMEP)

a) Primeiramente formemos oito pares de números escolhendo números opostos ao “meio” da sequência, ou
seja, (1, 16), (2, 15), . . ., (7, 10) e (8, 9). Veja que cada par possui soma 17. Agora junte os pares em quatro
grupos, cada um com soma 34, por exemplo: (1, 16, 2, 15), (3, 14, 4, 13), (5, 12, 6, 11) e (7, 10, 8, 9).

b) Veja que os números obtidos no item anterior fornecem um exemplo de como colocar os números em
cada linha. Vamos mostrar que temos pelo menos 1024 variações distintas desse exemplo. Em cada linha
podemos “girar”os números quatro vezes para a esquerda obtendo as sequências: (1, 16, 2, 15), (16, 2, 15, 1),
(2, 15, 1, 16) e (15, 1, 16, 2). Além disso, podemos “girar” as linhas quatro vezes de cima para baixo. Então,
apenas rodando o “exemplo” contruı́do, temos pelo menos 4 variações dentro de cada linha e mais outras
4 para rotações entre as linhas. Assim, no total teremos

(4 × 4 × 4 × 4) × 4
|{z} = 1024
| {z }
giros dentro das linhas giros entre as linhas

maneiras de realizar esta tarefa. A figura abaixo mostra alguns exemplos de tabuleiros que podem ser
obtidos pelas operações de rotações descritas:

1 16 2 15 16 2 15 1 10 8 9 7

3 14 4 13 3 14 4 13 16 2 15 1

5 12 6 11 12 6 11 5 3 14 4 13

7 10 8 9 10 8 9 7 12 6 11 5

6!
64. (Extraı́do da Vı́deo Aula) C6,2 = = 15.
4! · 2!
5!
65. (Extraı́do da Vı́deo Aula) C5,2 = = 10.
3! · 2!
6!
66. C6,3 = = 20.
3! · 3!
5!
67. C5,3 = = 10.
2! · 3!
68.

10! 10! 10!


a) C10,2 = = 45. b) C10,3 = = 120. c) C10,7 = = 120.
8! · 2! 7! · 3! 3! · 7!
Comentário para professores: A quantidade de maneiras das quais podemos tormar p elementos de um
conjunto de n elementos (n ≥ p) é a mesma que tomar (n − p) elementos, ou seja, é indiferente se contamos
de quantas maneiras podemos tomar p ou deixar p. Assim Cn,p = Cn,n− p .

28
POTI 2015 − Combinatória − Nı́vel 2 − Aula 0 − Professores Cleber Assis, Samuel Barbosa e Tiago Miranda

69.

6! 6!
a) C6,2 = = 15. b) C6,4 = = 15.
4! · 2! 2! · 4!
5!
70. (Extraı́do da Vı́deo Aula) C5,3 = = 10.
3! · 2!
6!
71. (Extraı́do da Vı́deo Aula) C6,2 = = 15.
4! · 2!
72. (Extraı́do da Vı́deo Aula)

a) C60,6 = 50.063.860. b) C30,4 · C30,2 = 27.405 · 435 = c) C59,5 = 5.006.386.


= 11.921.175.
73. (Extraı́do da Vı́deo Aula) C5,3 · C6,2 · C3,2 = 10 · 15 · 3 = 450 comissões.

74.
10!
a) C10,5 = = 252.
5! · 5!
8!
b) Como Ana participará e Beatriz não, restam oito pessoas para quatro vagas, ou seja, C8,4 = = 70.
4! · 4!
c) Devemos subtrair do total de comissões, as comissões nas quais nenhuma delas participa e as comissões
nas quais todas participam. Temos então C10,5 − C6,5 − C6,1 = 252 − 6 − 6 = 240 comissões.

75.
8!
a) C8,3 = = 56.
5! · 3!
b) Traçando a circunferência circunscrita à este octógono, basta tomar dois vértices pertencentes às extre-
midades de um diâmetro desta circunferência que, juntamente com qualquer outro vértice, formam um
triângulo retângulo. Como são 4 possibilidades de diâmetros sobre os vértices e 6 pontos que sobrarão,
teremos um total de 4 · 6 = 24 triângulos retângulos.

76. Basta contar de quantas maneiras podemos tomar dois dos dez vértices do decágono e descontarmos os
segmentos que formam lados. Temos então C10,2 − 10 = 45 − 10 = 35.

77. Para desenhar o tabuleiro foram necessários nove segmentos de retas horizontais e nove verticais. Para
construir o retângulo, basta tomarmos duas retas horizontais e duas verticais, ou seja, C9,2 · C9,2 = 36 · 36 = 1296
retângulos.

78. (Extraı́do da Vı́deo Aula) Basta tomar dois pontos de uma reta e um da outra, ou seja, 4 · C5,2 + 5 · C4,2 =
4 · 10 + 5 · 6 = 70 triângulos.

79. (PUC - RJ - Extraı́do da Vı́deo Aula) Escolhe-se primeiro o lı́der e, com o restante, escolhem-se os outros
quatro da equipe, ou seja, 10 · C9,4 = 10 · 126 = 1.260 equipes.

80. (UFRJ - Extraı́do da Vı́deo Aula) No inı́cio, final ou entre dois livros de COMBINATÓRIA É FÁCIL, deve
haver no máximo um livro de COMBINATÓRIA NÃO É DIFÍCIL. Assim, organizando espaçadamente os
onze livros daquele tı́tulo, obtemos doze espaços (inı́cio, fim e entre eles) que deverão ser preenchidos com
12!
livros deste tı́tulo, ou seja, C12,5 = = 792 maneiras diferentes de dispor todos os livros.
7! · 5!

29
POTI 2015 − Combinatória − Nı́vel 2 − Aula 0 − Professores Cleber Assis, Samuel Barbosa e Tiago Miranda

81. (UFRJ - Extraı́do da Vı́deo Aula)

a) Escolhendo duas amigas diferentes a cada final de semana, ela conseguirá convidar no máximo 2 · 53 = 106
amigas.

b) Júlia deverá escolher uma quantidade mı́nima de amigas e combiná-las de maneira que consiga preencher
todos os finais de semana, ou seja, ela deverá escolher uma quantidade n de amigas de tal forma que
n!
Cn,2 ≥ 53. Daı́ temos ≥ 53. Segue que n(n − 1) ≥ 106. Como n deve ser inteiro e positivo, seu
(n − 2)! · 2!
menor valor é 10.
82. (UNICAMP - Extraı́do da Vı́deo Aula)
Para que tenhamos soma par, devemos dividir em dois casos:

i) três números pares: C15,3 = 455;

ii) dois números ı́mpares e um número par: 15 · C15,2 = 15 · 105 = 1.575.

Assim, o total de maneiras é 455 + 1575 = 2030.


83. (Extraı́do da Vı́deo Aula)
O primeiro fato é que o 0 não pode fazer parte do número pois, se fizesse, não poderia estar à esquerda e
se estivesse na casa da unidade, dezena ou centena, seria menor que alguém à esquerda. O segundo fato
é que todos os algarismos devem ser diferentes. Tomemos agora um destes que atende às caracterı́stica do
problema, por exemplo, 1234. É fácil perceber que, de todas as permutações com os algarismos 1, 2, 3 e 4,
apenas em uma delas eles estão em ordem crescente, ou seja, basta escolher quatro algarismos de nove que
teremos apenas uma sequência possı́vel. Sendo assim, o total de números é C9,4 = 126.

84. (Extraı́do da OBM − 2013)


Suponha que, após abrir C caixas, Led ainda não consiga abrir o portal. Isso significa que há pelo menos
uma chave que ele não possui. Então, as caixas que ele abriu possuı́am pares de chaves capazes de abrir não
mais que 9 cadeados. Logo, teremos C ≤ C9,2 = 36. De fato, se ele abrir caixas que possuem todos os pares
das chaves de um conjunto de 9 cadeados, ele não conseguirá abrir o portal. Por outro lado, nota-se também
que se Led abrir 37 caixas distintas, saberemos que suas chaves não poderão ser um subconjunto de chaves
capazes de abrir não mais que 9 cadeados, pois é maior que 36. Então, o número mı́nimo de caixas que Led
deve abrir é 37.
85. AACS, AASC, ACAS, ACSA, ASAC, ASCA, CAAS, CASA, CSAA, SAAC, SACA, SCAA.
Comentário para professores: Muitos dos problemas de contagem, em especial os de permutação, podem ser
facilmente resolvidos através da listagem de todas as possibilidades. Seja através do diagrama da árvore ou
através de uma listagem simples, como na solução do exercı́cio anterior, é interessante que essa construção
ocorra de forma organizada. No caso de anagramas, sugerimos que as palavras sejam listadas em ordem
alfabética; no caso de números, em ordem crescente ou decrescente.
86. Se as duas letras A fossem distintas, digamos A1 e A2 , terı́amos P4 = 4! anagramas. Esses 24 anagramas
podem ser agrupados em 12 pares que representam palavras iguais caso A1 e A2 sejam trocados por A. Por
24
exemplo, A1 CA2 S e A2 CA2 S geram a mesma palavra ACAS. Sendo assim, Existem P42 = = 12 anagramas.
2
5!
87. P53,2 = = 10.
3! · 2!
7!
88. P72,2,3 = = 210.
2! · 2! · 3!

30
POTI 2015 − Combinatória − Nı́vel 2 − Aula 0 − Professores Cleber Assis, Samuel Barbosa e Tiago Miranda

89.
a) P8 = 8! = 40.320.

b) Como as vogais e consoantes não podem ser consecutivas, temos dois casos gerais: começando com vogal
(VCVCVCVCVC) ou começando com consoante (CVCVCVCVCV). É fácil perceber que o número de
anagramas é o mesmo para ambos os casos. Como todas as consoantes são diferentes e apenas a letra E
está repetindo entre as vogais, temos que 2 · P52 · P5 = 2 · 60 · 120 = 14.400.

90. (Extraı́do da Vı́deo Aula)


Vamos dividir em dois casos:
7!
i) começando com N: P74 = = 210;
4!
7!
ii) começando com B ou D: 2P74,2 = 2 · = 210.
4! · 2!
Assim, o total de anagramas é 210 + 210 = 420.
91.
a) Se deve haver um bloco formado por três A’s, temos então P42 = 12 anagramas.

b) Podemos iniciar com vogal (VCVCVC) ou consoante (CVCVCV). Basta permutar, em ambos, apenas as
consoantes, ou seja, P32 = 3. Temos então 2 · 3 = 6 anagramas.

P62,3 6!
c) = = 20.
3 2! · 3! · 3
92. É o mesmo que contar a quantidade de anagramas de uma palavra com oito letras, sendo quatro iguais e
outras quatro iguais também, como por exemplo a palavra VCCVVVCC. Temos então uma permutação com
8!
repetição. Sendo assim, são P84,4 = = 70 maneiras.
4! · 4!
93. (Extraı́do da Vı́deo Aula)
Se são quinze pessoas, teremos quinze lugares na fila. Como existe uma sequência fixa de posicionamente
entre os homens, ou seja, primeiro deve estar o menor, depois o segundo menor e assim por diante, precisamos
apenas escolher as cinco posições, dentre as quinze, para os homens. O mesmo acontece para as mulheres.
Sendo assim, resolver esse problema é o mesmo que contar a quantidade de anagramas de uma palavra com
10,5 15!
cinco letras iguais e outras dez letras iguais (permutação com repetição). Temos então P15 = = 3.003.
10! · 5!
94. (UFRJ - Extraı́do da Vı́deo Aula)
Para que a partı́cula volte à posição inicial, o número de movimentos para a direita e para a esquerda devem ser
iguais a 5, independentemente da ordem, ou seja, resolver esse problema é o mesmo que contar a quantidade
de anagramas com as letras da palavra DEDDEEEDDE. Assim temos que o total de maneiras diferentes é
5,5
P10 = 252.

95. Vamos pensar em uma sequência de quatro letras iguais com espaços antes, depois e entre as letras
( A A A A ). Esses cinco espaços representam as cinco crianças, ou seja, o primeiro espaço representa Ana, o
segundo, Bruna, o terceiro, Carla, o quarto, Daniela e o quinto, Esmeralda. Agora, podemos preencher esses
espaços com os bombons (podendo ser mais de um bombom por espaço), por exemplo AABABAB, significa
que Ana e Bruna não receberam bombons e as demais receberam um bombom cada; outro exemplo seria
BAABBAA, onde Ana recebeu um bombom, Carla, dois e as demais, nenhum. Assim, para resolvermos o
7!
problema, basta calcularmos o total de anagramas da palavra AAAABBB, ou seja, P74,3 = = 35 maneiras
4! · 3!
de premiar os alunos.

31
POTI 2015 − Combinatória − Nı́vel 2 − Aula 0 − Professores Cleber Assis, Samuel Barbosa e Tiago Miranda

96. Vamos analisar a sequência (•• + • + ••••). Basta pensar que o número de pontos antes do primeiro sinal
de mais é o valor de x; entre os sinais de mais, o valor de y; e, depois do segundo sinal, de z. Perceba que,
para este exemplo, temos x = 2, y = 1 e z = 4, o que nos dá soma 7 e, portanto, é uma solução da equação
x + y + z = 7. Assim, para encontrarmos o número de soluções naturais, basta permutarmos 7 pontos e 2
9!
sinais de mais entre si, ou seja, P92,7 = = 36.
2! · 7!
97. Como não existe zero na solução, usaremos o artifı́cio de substituição de incógnitas, ou seja, faremos
x = a + 1, y = b + 1 e z = c + 1, transformando a equação x + y + z = 7 em a + b + c = 4, sendo a, b, c
números naturais. Seguindo a ideia da solução do exercı́cio anterior, temos que permutar 2 sinais de mais e 4
6!
pontos, ou seja, P62,4 = = 15, que é o total de soluções inteiras e positivas para x, y e z.
2! · 4!
98. (Extraı́do da American Mathematics Competitions)
Representemos os sapatos pelos sı́mbolos si , com 1 ≤ i ≤ 8, e as meias com mi , também com 1 ≤ i ≤ 8. Uma
sequência desses sı́mbolos em linha produz uma ordem na forma como a aranha deve se calçar. Queremos
então determinar todos os anagramas de uma palavra formada por todos esses sı́mbolos em que mi , com
1 ≤ i ≤ 8, sempre esteja à esquerda de si . Das 16! permutações desses sı́mbolos, em exatamente metade delas
m1 está à esquerda de s1 e na outra metade ele está à direita. Analisando então os 16!/2 anagramas em que
m1 está à esquerda de s1 , temos que em metade deles m2 está à esquerda de s2 e na outra metade à direita.
Assim, em 16!/4 anagramas, as meias m1 e m2 são calçadas antes dos sapatos s1 e s2 . Repetindo o argumento,
16! 16!
podemos concluir que em = 8 anagramas, a aranha calça as meias antes dos sapatos
2 · 2 · 2 · 2 · 2 · 2 · 2 · 2· 2
correspondentes.
99. Cada distribuição das pérolas pode ser associada a uma palavra envolvendo as quatro letras A, B, C e D
como indica a figura 4.

Figura 4: Associando colares e palavras.

Algumas palavras devem ser consideradas iguais pois podemos rodar o colar. Por exemplo:
ABCD = BCDA = CDAB = DABC.
Veja que cada palavra é igual à exatamente outras três palavras pois podemos rodar o colar três vezes antes
de voltarmos à posição inicial e que as rotações correspondem a transposições da letra inicial para o final da
palavra. Assim, uma maneira para contarmos as distribuições distintas é contarmos a quantidade total de
palavras e depois as agruparmos em grupos de 4 palavras correspondendo a colares iguais entre si. Como
existem 4! = 24 palavras obtidas pelas permutações das letras A, B, C e D, o total de colares é
4!
= 3! = 6.
4
Observação: Em geral, se tivéssemos n pérolas distintas, cada palavra teria n letras e seria igual à exatamente
n!
n outras palavras. Consequentemente, o total de colares distintos seria = (n − 1)!. Usaremos o a notação
n
PCn = (n − 1)! para designar o número de permutações circulares distintas de n objetos distintos em um
cı́rculo.

32
POTI 2015 − Combinatória − Nı́vel 2 − Aula 0 − Professores Cleber Assis, Samuel Barbosa e Tiago Miranda

100. O grupo inicial de 6 pessoas pode ser representado pelas letras: A, B, C, D, L( de Lucimar) e N (de
Nilton).

a) Por conta da preferência de Nilton, podemos pen-


sar em NL como se fosse uma única letra (bloco) e
em cada distribuição como a permutação circular
dos sı́mbolos: A, B, C, D e NL, como ilustra a fi-
gura 5. Portanto, a resposta para este problema seria
PC5 = 4! = 24 disposições.

Figura 5: Nilton à direita de Lucimar.

b) Sim, a reposta muda. Assim como no item ante-


rior, podemos usar NL para representar apenas o
local onde Lucimar e Nilton estarão posicionados,
conforme a figura 6.
Usaremos a contagem obtida no item anterior, pois
uma vez escolhido o lugar em que eles ficarão, tere-
mos duas opções de posicioná-los: Nilton à esquerda
ou à direita de Lucimar. Portanto, pelo princı́pio mul- Figura 6: Nilton e Lucimar permutando...
tiplicativo, o total buscado é 2 · PC5 = 48 disposições.
Comentário para professores: Esse é um bom momento para sugestionar em sala que identificar as seguintes
estruturas pode simplificar os problemas:
i) Bloco Rı́gido: agrupamento de sı́mbolos sem permutação entre as respectivas posições.
ii) Bloco: agrupamento de sı́mbolos com permutação entre as respectivas posições.
101. Serão PC6 = 5! = 120 maneiras.
102. O grupo inicial de 6 pessoas pode ser representado pelas seguintes letras: A (de Aline), B (de Bianca), C
(de Carla), D, E e F. Como as três letras A, B e C devem aparecer juntas em alguma ordem, podemos consi-
derar o Bloco ABC, isto é, teremos apenas as 4 “pessoas”: D, E, F e ABC; distribuı́das no cı́rculo (figura 7).

Figura 7: ABC juntas.

Portanto, inicialmente calculamos a permutação circular de 4 objetos: PC4 = 3! = 6 e, em seguida, multiplica-


mos tal quantidade pelo número de maneiras de permutarmos as amigas que devem ficar juntas. Como as
três letras A, B e C podem formar 3! permutações distintas. Portanto, a distribuição dessas crianças pode ser
feita de
3! · PC4 = 6 · 6 = 36 modos.

33
POTI 2015 − Combinatória − Nı́vel 2 − Aula 0 − Professores Cleber Assis, Samuel Barbosa e Tiago Miranda

103. Sejam A e B as crianças que não podem ficar juntas.

Uma solução:
Pode-se formar PC5 = 4! = 24 rodas com as outras
cinco crianças, representadas na figura 8 pelo •. Agora
há 5 espaços (representados pelo  e pela “?”) para
colocar a criança A e existem 4 modos para a criança
B, o que resulta em

24 × 5 × 4 = 480 rodas

Figura 8: “?” são espaços que podem ser ocupados.

Outra solução:
Observando os 7 lugares na roda, podemos fixar a
posição da criança A, ficando com 6 lugares livres,
dos quais apenas 4 podem ser ocupados pela criança
B (marcados com um “×” na figura 9).
Agora são 5 espaços para cinco crianças serem posici-
onais, logo 5! maneiras. Logo, obtemos

4 × 5! = 480 rodas.
Figura 9: “×” para B.

Mais uma solução:


Se não houvesse condição terı́amos PC7 = 6! = 720 maneiras de criar essa roda. Agora, se as duas crianças
devessem ficar juntas terı́amos um bloco AB que pode ser permutado de 2 maneiras e os 6 integrantes (os
5 originais e o bloco AB) podem ser permutados circularmente de PC6 = 5! = 120 maneiras, o resulta em
2 × 120 = 240 formas. Por fim, o total é 720 e em 240 rodas temos formações que não cumpre a condição,
então, temos
720 − 240 = 480 rodas.
104. Uma solução:
Como João e Maria devem ficar juntos, J M deve ser um bloco, e agora devemos pensar apenas em “seis”
crianças. Dessa forma, pode-se formar PC6 = 5! = 120 rodas. Mas o bloco pode ser J M ou MJ, o que resulta
em
2 × 120 = 240 rodas.
Outra solução:
Observando os 7 lugares na roda, podemos fixar a posição de João, ficando com 2 lugares livres para Maria
(esquerda ou direita de João), as demais crianças pode ser permutadas de 5! = 120 maneiras. Logo, temos

2 × 120 = 240 maneiras.


105. Apenas como referência, primeiro organizam-se os homens na roda deixando sempre um banco vazio
entre cada um deles, isso pode ser feito de PC6 = 5! = 120 maneiras. Depois, em cada espaço entre os homens
devem-se arrumar as mulheres de 6! = 720 modos. Resultando num total de 120 × 720 = 8640.

Observação: Não importa se primeiro arrumam-se os homens ou as mulheres, a permutação circular


independerá desse inı́cio, pois observando apenas um giro no cı́rculo poderia ser colocada a outra arrumação
em foco.

34
POTI 2015 − Combinatória − Nı́vel 2 − Aula 0 − Professores Cleber Assis, Samuel Barbosa e Tiago Miranda

106. Fábio (F), Denise (D) e Ledo (L) devem formar um bloco de modo que temos as opções de “FDL” ou
“LDF”. Agora devemos permutar circularmente o bloco e as outras 5 crianças, isto é, PC6 = 5! = 120, por fim,
temos 2 × 120 = 240 rodas.
107. Uma solução:
Sem perda de generalidade, podemos começar arrumando os 4 homens deixando sempre uma cadeira vaga
entre eles. Isso pode ser feito de PC4 = 3! = 6 modos. Uma vez fixada a forma como os homens estarão
dispostos, bastará arrumarmos as mulheres nas 4 cadeiras restantes. O que pode ser feito de 4! = 24 maneiras.
Assim, o total de formações possı́veis é
6 × 24 = 144.
Agora, há uma condição no problema, João e Maria não podem ficar juntos. Vamos pensar ao contrário, com
João e Maria juntos, criando o bloco J M. Fixando João, temos dois lugares possı́veis para Maria (à direita ou à
esquerda). Depois disso, há 3! = 6 de dispor os homens e 3! = 6 modos para posicionar as mulheres. Sendo
assim,
2 × 6 × 6 = 72
formações que não desejamos. O que gera
144 − 72 = 72 arrumações com João e Maria separados.

Outra solução:
Primeiro fixemos João como referência, logo, dos 4
espaços reservados para as mulheres, apenas 2 po-
derão ser ocupados por Maria (os dois mais distantes
de João, conforme a figura 10). Os homens poderão
se sentar nos 3 lugares restantes de 3! = 6 modos e as
mulheres também. O que resulta em

2 × 6 × 6 = 72 arrumações possı́veis.
Figura 10: João e Maria.

108. Só há dois blocos na questão, o dos meninos e o das meninas, o que gera apenas um tipo de ciranda
(ou para os acostumados com fórmulas: PC2 = 1! = 1). Em cada bloco há 5! = 120 maneiras de permutar as
crianças do mesmo gênero. Portanto, obtemos
PC2 × 120 × 120 = 14400 cirandas.
109.
a) Se cada mulher deve ficar ao lado do seu marido, serão 5 blocos diferentes para serem arrumados em
cı́rculo e isso pode ser feito de PC5 = 4! maneiras para os blocos e podemos ter mulher à esquerda e
homem à direita ou vice-vera, 2 arrumações por bloco, resultando em
24 × 25 = 24 × 32 = 768 arrumações possı́veis.

b) Com cada mulher ao lado do seu marido e pessoas do mesmo sexo separadas temos as mesmas PC5 = 4!
formações dos blocos, só que não haverá permutação individual em cada bloco e sim apenas as arrumações
gerais de homens e mulheres alternadas e isso só pode ser feito de duas maneiras, resultando em
24 × 2 = 48 arrumações possı́veis.

110. Primeiro, observemos que, com a base pentagonal sobre uma mesa, o giro da pirâmide apoiada não cria
novas disposições, apenas novas visualizações do mesmo objeto. Sendo assim, devemos fixar uma das faces
laterais (“travar o giro”) para perceber as diferentes colorações possı́veis. Há 6 cores para a base e com as 5
cores restante haverá uma permutação circular (PC5 = 4!). O que resulta em
6 × 24 = 144 colorações possı́veis.

35
POTI 2015 − Combinatória − Nı́vel 2 − Aula 0 − Professores Cleber Assis, Samuel Barbosa e Tiago Miranda

111. Uma solução:


As possı́veis duplas para ocupar os bancos podem ser dispostas de:

C12,2 · C10,2 · C8,2 · C6,2 · C4,2 · C2,2 =


12 × 11 10 × 9 8 × 7 6 × 5 4 × 3 2 × 1 12!
· · · · · =
2! 2! 2! 2! 2! 2! 26
Como não importa a ordem de escolha dos casais houve contagens em excesso, 6! vezes a mais. Para corrigir
isso, basta
12!
26 12!
= 6 .
6! 2 · 6!
Com as 6 duplas formadas podemos permutá-las circularmente de

PC6 = 5! maneiras.

Em um banco, cada pessoa pode se sentar de 2 modos (na direita ou na esquerda), como são 6 bancos, serão
26 formações. Em resumo, chega-se a

12! · 5! · 26 12 · 11!
6
= = 2 · 11! modos.
2 · 6! 6
Uma solução mais simples:
Vamos fixar, sem perda de generalidade, uma das crianças em um dos bancos. Ela tem 2 opções para sentar em
um dos bancos. Agora, observando essa criança como referencial, as outras 11 poderão fazer uma permutação
simples dos locais restantes. O que resulta em

2 · 11! = 79833600 modos.


112. Tomando os casos de:

i) n = 3, denominemos os pontos como A, B e C. Assim, só há um triângulo, o ABC, pois todas as outras
permutações se referem ao mesmo polı́gono citado, contando inclusive, giros, rotações e translações.
Poderı́amos pensar em fixar o ponto A, portanto há 2! permutações circulares possı́veis, e como o ABC é
igual ao triângulo ACB basta dividir por dois o que gera apenas um polı́gono.

ii) No caso de n = 4, denominemos os pontos como A, B, C e D. Fixemos o ponto A, portanto há 3!


6
permutações circulares possı́veis, e como as translações não criam novos polı́gonos, ficaremos com = 3
2
polı́gonos.

iii) No caso de n qualquer, denominemos os pontos como A, B, · · · e N. Fixemos o ponto A, portanto há
(n − 1)! permutações circulares possı́veis, e como as translações não criam novos polı́gonos, ficaremos
( n − 1) !
com polı́gonos.
2
113. Os dados podem ser girados em duas direções sem criar formações novas, sendo assim, é necessário
fixar dois valores para compor as rotações. Com essas referências postas, automaticamente se definem duas
outras faces. Portanto, apenas duas ficam para serem escolhidas, o que gera 2 × 1 = 2 dados.

Observação: De modo concreto, coloque o dado sobre uma mesa e fixe a face superior com o 6, portanto
a face oposta (a que está em contato com a superfı́cie) será o 1. Agora escolha alguma das faces laterais e
coloque o 5, girando para ficar esse número de frente para você, então a respectiva face oposta (a que não
está sendo vista) será o 2. Daı́, restam apenas duas faces laterais (esquerda e direita) para encaixar o 3 e o
4, poderemos colocar o 3 na esquerda (automaticamente o 4 vai para a direitas) ou colocar o 4 na esquerda
(automaticamente o 3 vai para a direitas). Por fim, dois dados diferentes.

36
POTI 2015 − Combinatória − Nı́vel 2 − Aula 0 − Professores Cleber Assis, Samuel Barbosa e Tiago Miranda

114. Sendo um rubi, uma esmeralda, um topázio, uma água-marinha, uma turmalina e uma ametista, ou seja,
6 pedras diferentes.

a) Como há um relógio, só há um sentido do braço ser colocado da pulseira e usando-o como referência
fica-se com a permutação das 6 pedras, ou seja, 6! = 720 arrumações.

b) Usando o fecho como referência fica-se com a permutação das 6 pedras ou 6! = 720 arrumações. Mas
720
podendo haver a rotação na pulseira foram feitas contagens duplas, portanto, há = 360 arrumações.
2
c) Agora teremos que criar uma das pedras como referência e utilizar a a permutação circular e ficaremos
com PC6 = 5! = 120 arrumações.

d) Mais uma vez, teremos que criar uma das pedras como referência e utilizar a permutação circular e
ficaremos com PC6 = 5! = 120. Mas podendo haver a rotação na pulseira foram feitas contagens duplas,
120
portanto, há = 60 arrumações.
2
115. Primeiramente devemos observar quantos são as formas de escolher esses estudantes, para isso, vamos
separar em casos:

i) No caso de não haver a participação de José, Cléber, Márcia e Luı́za, restarão 8 estudantes dos quais
escolheremos 6, sendo assim, a quantidade será igual a C8,6 , que para efeito de cálculo é o mesmo
8·7
número que C8,2 = = 28 modos. E esses 6 estudantes podem ser permutados circularmente do
2!
PC6 = 5! = 120 maneiras. No total de

28 × 120 = 3360 formas.

ii) No caso da participação de José, Cléber, Márcia


e Luı́za, restarão 8 estudantes dos quais escolhe-
8·7
remos 2, sendo assim, C8,2 = = 28 modos,
2!
porém após escolhidos esses estudantes tem-se as
condições de Márcia e Luı́za lado a lado (formarão
um bloco ML) e o José e o Cléber separados, ob-
serve a figura 11.
Portanto, após posicionar ML (duas maneiras para
tal) e os outros 2 estudantes E1 e E2 (fora José e o
Cléber) teremos uma permutação circular, PC3 =
2! = 2, agora podemos colocar cadeiras vazias en-
tre o bloco ML e a outra dupla de estudantes de
modo que José e Cléber ficarão separados, José tem Figura 11: Márcia e Luı́za, E1 e E2 .
3 opções e Cléber tem 2, o que resulta em
28 × 2 × 2 × 3 × 2 = 672 formas.

Somando os dois casos temos 3360 + 672 = 4032 arrumações.

Comentário para professores: No item 115 houve o uso da fórmula das combinações complementares
(C8,6 = C8,2 ). Existe um argumento combinatório interessante para justificar (demonstrar) a validade dessa
relação. Afinal, em um grupo de 8 pessoas, ao escolher 6 delas para participar de algo, concomitantemente
excluem-se 2 do evento. Cada sexteto escolhido gera uma dupla excluı́da, se for construı́da uma função com
esse fato, ela será uma bijeção entre o conjunto dos sextetos formados e as duplas dispensadas. De modo
geral, entre n pessoas escolher p (fórmula: Cn,p ) resulta no mesmo número que entre n pessoas excluir n − p
(fórmula: Cn,n− p ).

37
POTI 2015 − Combinatória − Nı́vel 2 − Aula 0 − Professores Cleber Assis, Samuel Barbosa e Tiago Miranda

116. Uma solução:


Inicialmente deve escolher as cores das faces hexagonais e isso pode ser feito de C8,2 = 28 modos. Agora,
apoie uma das faces sobre uma mesa e perceba que girando as faces laterais não criamos novas formas, mas
sim outras observações do mesmo objeto. Devemos fixar uma das faces laterais e executar a permutação
circular com as 6 cores restantes, isto é, PC6 = 5! = 120 formas. O que finaliza com

28 × 120 = 3360 colorações distintas.

Outra solução:
Observe que se fosse uma bandeira com 8 listras terı́amos 8! maneiras de pintá-la com cores distintas. Agora,
o giro entre as bases do prisma não cria uma nova disposição, logo devemos dividir esse número por 2.
Ademais, as rotações do prisma não criam novas pinturas, logo há 6 vezes mais contagens dessa forma. Por
8!
fim, obtemos = 3360 maneiras.
2·6
Observação: Nesse bloco de conteúdos, quando for citado o conjunto dos naturais estamos falando do
conjunto
N = {0, 1, 2, 3, . . . }.

117. Vamos utilizar o método desenvolvido no modelo.

a) Para calcular a quantidade de soluções naturais Os dois casos destacados ilustram permutações
de x + y = 3, devemos distribuir três • em duas de posição entre 9 elementos, dos quais, sete
• • • • . Dois exemplos seriam: são • e os dois +. Isso pode ser calculado de
CR3,7 = P97,2 = 36 modos, ou seja, 36 soluções
i) (3, 0) distribui-se como
inteiras não negativas.

••• + ••• ; c) A quantidade de soluções inteiras positivas da


equação
ii) (1, 2) distribui-se como A+B+C+D = 9

pode ser calculada através de uma substituição de


••• + ••• . variáveis. Fazendo A = a + 1, B = b + 1, C = c + 1
e D = d + 1, com a, b, c e d números naturais, após
Ou seja, permutar três objetos, sendo dois • e um substituição chegamos a
+, o que resulta em CR2,3 = P43,1 = 4 pares ordena-
dos de números naturais que resolvem a equação. a + b + c + d = 5,
b) Para x + y + z = 7, no U = N, devemos distribuir
que, através do método do modelo fica com 5 • em
sete • em três • • • • . Dois exemplos seriam:
4 caixas • • • • , que é o mesmo que 3 sinais de +.
i) (4, 3, 0) distribui-se em
•••••
••• • + ••• • + ••• • ; e
• • •• + • • •• + • • •• + • • ••
ii) (1, 3, 3) distribui-se em
Essa última equação possui CR4,5 = P85,3 = 56
•• • • + ••• • + ••• • . soluções inteiras não negativas (ou inteiras positi-
vas para a primeira).
Comentário para professores: Apesar do tópico de contagem ser de combinações completas, uma das
estratégias mais simples para resolver os problemas, como visto nos vı́deos, será utilizando um método que
se assemelha ao das permutações com elementos repetidos. Sendo assim, em muitas questões preferiremos
utilizar o desenvolvimento final pelas permutações, pela simplicidade e beleza das soluções que surgem.

38
POTI 2015 − Combinatória − Nı́vel 2 − Aula 0 − Professores Cleber Assis, Samuel Barbosa e Tiago Miranda

118. Seja Ci a cor que vai ser pintada no carro i ∈ {1, 2, 3}. Devemos calcular o número de soluções inteiras
não negativas da equação
C1 + C2 + C3 = 6.
Seguindo:
i) pelas combinações completas com

CR3,6 = C3+6−1,6 = C8,6 = 28 maneiras; ou

ii) pelo método da permutação de P86,2 = 28 modos.

119. Uma solução:


A ideia é escolher dois números entre {0, 1, 2, 3, 4, 5, 6}, sendo permitida a escolha com repetição. Para
calcularmos todas as opções, teremos que quantificar o número de soluções inteiras não negativas da equação:

f0 + f1 + f2 + f3 + f4 + f5 + f6 = 2

na qual f i , i ∈ {0, 1, · · · , 6}, é a quantidade em cada metade do dominó. Isso resulta em:
i) pelas combinações completas,

CR7,2 = C7+2−1,6 = C8,6 = 28 soluções naturais; ou

ii) pelo método das permutações, em P86,2 = 28 soluções naturais (peças).


Outra solução:
Podemos separar as peças pelos tipos, peças com lados:
i) iguais (buchas): são 7; e

ii) diferentes: são C7,2 = 21.


O que totaliza 7 + 21 = 28 peças.

120. Sejam di a quantidade de doces e o ı́ndice i o tipo do doce escolhido, i ∈ {1, 2, 3, 4, 5} e rh a quantidade
de garrafas do refrigerante e o ı́ndice h o tipo de refrigerante escolhido, h ∈ {1, 2, 3, 4}. Vamos calcular a
quantidade das soluções naturais de duas equações, a saber:

d1 + d2 + d3 + d4 + d5 = 2 e r1 + r2 + r3 + r4 = 3

A primeira possui P62,4 = 15 soluções naturais e a segunda de P63,3 = 20 soluções naturais. Por fim, o número
de total de pedidos pode ser feito de
15 × 20 = 300 modos.
121. Para saber de quantos modos podemos comprar 6 empadas Ei , i ∈ {c, f , l, p}, deveremos
a) calcular o número de soluções naturais da equação

Ec + E f + El + E p = 6,

que se resolve de P96,3 = 84 soluções inteiras não negativas.

b) Já que ele quer provar todos os sabores das empadas, teremos todos os Ei 6= 0. Apenas haverá escolha nos
sabores para as últimas duas empadas, na quantidades de ei , i ∈ {c, f , l, p}, isto é

ec + e f + el + e p = 2

que possui P52,3 = 10 soluções naturais.

39
POTI 2015 − Combinatória − Nı́vel 2 − Aula 0 − Professores Cleber Assis, Samuel Barbosa e Tiago Miranda

122. Uma solução:


Observe que não ter duas letras “A” juntas, automaticamente impede de termos três ou os quatro “A’s” juntos.
Seja Ei , i ∈ {1, 2, 3, 4, 5}, a quantidade de letras antes/depois de cada um dos “A’s”.

E1 A E2 A E3 A E4 A E5

Pelo enunciado, podemos construir a equação

E1 + E2 + E3 + E4 + E5 = 9.

E como temos que separar os “A’s”, então E2 , E3 e E4 são positivos. Logo,

i) E2 > 0 ⇒ E2 = e2 + 1

ii) E3 > 0 ⇒ E3 = e3 + 1

iii) E4 > 0 ⇒ E4 = e4 + 1

Que após substituição fica


E1 + e2 + e3 + e4 + E5 = 6.
Para calcularmos o número de soluções inteiras não negativas, teremos:

i) pelas combinações completas,

CR5,6 = C5+6−1,6 = C10,6 = 210 maneiras; ou

6,4
ii) pelo método das permutações em P10 = 210 ocupações dos espaços.
Agora, com os espaços escolhidos, vamos permutar as letras nessas escolhas de P92 = 181440 modos. Por fim,

210 × 181440 = 38102400 anagramas.

Outra solução:
Podemos começar posicionando as letras diferentes de “A”.

Agora, a permutação das letras diferentes de “A” pode ser feita de

P92 = 181440 modos.

Para escolher quais espaços entre as letras serão ocupados pelos “A’s” (para separá-los) teremos C10,4 = 210
formas.
Por fim, serão 210 × 181440 = 38102400 anagramas.

123. Para resolver x + y + z = 5 no universo dos naturais deveremos fazer P72,5 = 21.
Por fim, 21 soluções.

124. Primeiro devemos observar as possibilidades de incógnita nula (3 formas: ou x ou y ou z) e depois


resolver a nova equação (por exemplo, com x = 0) que será y + z = 4 que possui P54 = 5 soluções naturais.
Por fim, temos
3 × 5 = 15 soluções inteiras não negativas.

40
POTI 2015 − Combinatória − Nı́vel 2 − Aula 0 − Professores Cleber Assis, Samuel Barbosa e Tiago Miranda

125. Sejam di a quantidade de anéis em cada dedo i ∈ {1, 2, 3, 4}. Vamos calcular o número de soluções
inteiras não negativas da equação
d1 + d2 + d3 + d4 = 6.
Ela possui P96,3 = 84 soluções naturais não nulas (maneiras de distribuir os anéis).
126. (Extraı́do do concurso do IBGE)
Podemos tentar seguir listando os casos, como o enunciado ilustra ou buscar um método direto. Sendo assim,
seja Ci a quantidade de cadernos na gaveta i ∈ {1, 2, 3}, então a solução do problema será através do número
de soluções naturais da equação
C1 + C2 + C3 = 3
que pode ser resolvida de CR3,3 = C5,3 = P52,3 = 10 modos distintos (soluções inteiras não negativas).
127. (Extraı́do do vestibular da UNIRIO)
Seja Ci a quantidade de balas recebidas pela criança
i ∈ {1, 2, 3} e ci a quantidade acima de 1 que cada uma recebe, ou seja Ci = ci + 1. Donde seguem as
equações
C1 + C2 + C3 = 12 e c1 + c2 + c3 = 9.
9,2
A última possui P11 = 55 soluções inteiras não negativas. Então há 55 maneiras diferentes de distribuir as
balas.
128. (Adaptado do concurso para o Banco do Brasil)
Seja Si a quantidade de chocolates de cada sabor
i ∈ {1, 2, 3, 4}. O problema equivale a calcular o número de soluções naturais da equação
S1 + S2 + S3 + S4 = 3
que possui P63,3 = 20 soluções inteiras não negativas distintas. Logo, n = 20 modos.
129. (Adaptado do concurso para o Banco do Brasil - 2012)
Seja Si a quantidade de balas de cada sabor i ∈ {1, 2, 3} e si a quantidade acima de 3 que cada uma recebe, ou
seja Si = si + 3. O que segue as equações
S1 + S2 + S3 = 13 e s1 + s2 + s3 = 4.
Essa última possui P64,2 = 15 soluções inteiras não negativas.
Logo, ficamos com 15 montagens distintas.
130. Uma solução:
Para calcular o número de soluções naturais de
x+y+z ≤ 6
deveremos calcular o número de soluções naturais das equações:
i) x + y + z = 0 =⇒ P22 = 1;
ii) x + y + z = 1 =⇒ P32 = 3;

iii) x + y + z = 2 =⇒ P42,2 = 6;

iv) x + y + z = 3 =⇒ P52,3 = 10;

v) x + y + z = 4 =⇒ P62,4 = 15;

vi) x + y + z = 5 =⇒ P72,5 = 21; e

vii) x + y + z = 6 =⇒ P82,6 = 28.


Por fim, obtemos
1 + 3 + 6 + 10 + 15 + 21 + 28 = 84 soluções naturais.

41
POTI 2015 − Combinatória − Nı́vel 2 − Aula 0 − Professores Cleber Assis, Samuel Barbosa e Tiago Miranda

Comentário para professores: Na questão 130 ficamos com a soma

S = P22 + P32 + P42,2 + P52,3 + P62,4 + P72,5 + P82,6 .

Uma expressão equivalente a seria

S = C2,2 + C3,2 + C4,2 + P5,2 + C6,2 + C7,2 + C8,2

que pode ser resolvido pelo “Teorema das Colunas”

C2,2 + C3,2 + C4,2 + P5,2 + C6,2 + C7,2 + C8,2 = C9,3 .

Daı́, esse tipo de problema pode ser resolvido com menos contas.

130. Outra solução:


Defina para cada solução, no universo N, de x + y + z ≤ 6 o conceito “folga” da solução como o valor

f = 6 − x − y − z.

Observe o quadro abaixo para entender a utilidade da folga em alguns exemplos de soluções.

x y z x+y+z f

3 2 1 6 0

2 0 1 3 3

1 1 1 3 3

0 1 0 1 4

Utilizando a folga poderemos transformar a inequação inicial na equação

x+y+z+ f = 6

que possui, pelo método,


P96,3 = 84 soluções naturais.

131. Separando os casos pela quantidade de algarismos, com {W,X,Y,Z,...} os algarismos utilizados, sendo
W o de maior ordem, W > 0, W = w + 1, obtemos as equações e a quantidade de soluções inteiros em cada
indicarão o valor procurado.

i) W = 6 ⇒ w = 5 ⇒ P55 = 1

ii) W + X = 6 ⇒ w + Y = 5 ⇒ P65,1 = 6

iii) W + X + Y = 6 ⇒ w + X + Y = 5 ⇒ P75,2 = 21

iv) W + X + Y + Z = 6 ⇒ w + X + Y + Z = 5 ⇒ P85,3 = 56

Por fim, temos 1 + 6 + 21 + 56 = 84 números naturais.

42
POTI 2015 − Combinatória − Nı́vel 2 − Aula 0 − Professores Cleber Assis, Samuel Barbosa e Tiago Miranda

132. Para escolher os 4 elementos do conjunto {1, 2, 3, 4, 5, 6, 7, 8, 9} podemos pensar em relacionar cada
elemento escolhido com um mais (+) e cada um dos excluı́dos com um menos (−). Seguem alguns exemplos:

i) {1, 3, 5, 7} =⇒ {+, −, +, −, +, −, +, −, −}

ii) {1, 4, 6, 8} =⇒ {+, −, −, +, −, +, −, +, −}

iii) {2, 4, 7, 9} =⇒ {−, +, −, +, −, −, +, −, +}

Portanto, estamos permutando 4 sinais de + e 5 de − e o solicitado é que não haja dois sinais de mais (+)
seguidos.

? − ? − ? − ? − ? − ?

Por fim, dentro os 6 espaços livres ( ? ) devemos escolher 4 para os sinais de + e isso pode ser feito de
C6,4 = 15 maneiras.

Observação: Esse resultado ilustra (com exemplo) o Primeiro Lema de Kaplansky.

133. Vamos tentar reduzir o problema atual para o método do Primeiro Lema de Kaplansky.

i) Caso o elemento 1 seja escolhido, faltarão 3 elementos no conjunto que sairão de {3, 4, 5, 6, 7, 8, 9}, não
perca de vista que os vizinhos não poderão ficar juntos. Algumas soluções seriam:

i) {3, 5, 7} =⇒ {+, −, +, −, +, −, −}
ii) {3, 6, 8} =⇒ {+, −, −, +, −, +, −}
iii) {4, 7, 9} =⇒ {−, +, −, −, +, −, +}

Portanto, estamos permutando 3 sinais de + e 4 de − e o solicitado é que não haja dois sinais de +
seguidos.

? − ? − ? − ? − ?

Por fim, dentro os 5 espaços livres ( ? ) devemos escolher 3 para os sinais de + e isso pode ser feito de
C5,3 = 20 maneiras.

ii) Caso o elemento 1 não seja escolhido teremos a escolha de 4 elementos no conjunto {2, 3, 4, 5, 6, 7, 8, 9, 10},
não perca de vista que os vizinhos não poderão ficar juntos. Algumas soluções seriam:

i) {2, 4, 7, 10} =⇒ {+, −, +, −, −, +, −, −, +}


ii) {3, 5, 7, 9} =⇒ {−, +, −, +, −, +, −, +, −}

Portanto, estamos permutando 4 sinais de + e 5 de − e o solicitado é que não haja dois sinais de +
seguidos.

? − ? − ? − ? − ? − ?

Daı́, dentro os 6 espaços livres ( ? ) devemos escolher 4 para os sinais de + e isso pode ser feito de
C6,4 = 15 maneiras.

Por fim, 20 + 15 = 35 maneiras.

Observação: Esse resultado ilustra (com exemplo) o Segundo Lema de Kaplansky.

43
POTI 2015 − Combinatória − Nı́vel 2 − Aula 0 − Professores Cleber Assis, Samuel Barbosa e Tiago Miranda

134. Fica claro que a semana poderia ser disposta num cı́rculo a partir de Segunda (por exemplo) e
“finalizando” no Domingo (no caso especı́fico) e recomeçando ciclicamente.
Vamos tentar, outra vez, reduzir o problema ao raciocı́nio do Primeiro Lema de Kaplansky.
i) Caso ele vá Segunda para a academia, portanto faltam 2 dias no conjunto {Quarta, Quinta, Sexta, Sábado},
precisamos escolher dois deles sem ser sequenciados. Ou seja,

? − ? − ?

Daı́, dentro os 3 espaços livres devemos escolher 2 para os sinais de + e isso pode ser feito de C3,2 = 3
maneiras.
ii) Caso ele não vá Segunda para a academia, portanto escolherá 3 dias no conjunto {Terça, Quarta, Quinta, Sexta, Sába
precisamos escolher três deles sem ser sequenciados. Ou seja,

? − ? − ? − ?

Daı́, dentre os 4 espaços livres devemos escolher 3 para os sinais de + e isso pode ser feito de C4,3 = 4
maneiras.
Por fim, 3 + 4 = 7 maneiras.
Observação: Outra ilustração do Segundo Lema de Kaplansky.
Comentário para professores: Os Lemas de Kaplansky possuem fórmulas que reduzem substancialmente os
cálculos anteriores, contudo, ficou claro que com atenção e uso dos recursos (procedimentos de contagem)
é possı́vel resolver os problemas. Esses Lemas preparam o terreno para resolver o Problema de Lucas que
enuncia-se com:
De quantas maneiras n casais podem sentar em 2n cadeiras diferentes em torno de um cı́rculo de modo que
pessoas do mesmo sexo não se sentem juntas e que nenhum homem fique ao lado de sua mulher?
Cuja resolução não será feita aqui!
135.
a) Para cada elemento im ∈ Im temos n opções de imagem em In , portanto serão
m
| ×n×
n {z· · · × n} = n funções.
m fatores iguas a n

b) Para o elemento i1 ∈ Im temos n opções de imagem em In , para i2 são n − 1 opções, para 3 são n − 2, . . . ,
para o elemento m são n − m + 1 , portanto serão
n!
n · (n − 1) · (n − 2) · ... · (n − m + 1) = funções.
(n − m)!

c) Devemos escolher os elementos que farão parte da imagem e isso pode ser feito de Cn,m , a ordenação deles
será de modo crescente associando o menor yi ∈ In destacado como a imagem de 1 e segundo menor como
imagem de 2 e assim por diante.
d) Seja Yi a quantidade de domı́nios associados ao elemento i ∈ In , fica claro que o somatório desses Y 0 s deve
ser igual a quantidade de elementos do conjunto Im , isto é, m.
Y1 + Y2 + · · · + Yn = m
−1
e o número de soluções dessa equação é calculado como CRn,m = Cn+m−1,m = Pmm,n
+n−1 , a ordenação deles
será de modo não decrescente associando o menor yi ∈ In destacado como a imagem de 1 e segundo menor
ou igual, como imagem de 2 e assim por diante.

44
POTI 2015 − Combinatória − Nı́vel 2 − Aula 0 − Professores Cleber Assis, Samuel Barbosa e Tiago Miranda

136. Sejam di a quantidade de anéis em cada dedo i ∈ {1, 2, 3, 4}. Primeiro, vamos escolher os dedos e a
quantidade de anéis em cada dedo e isso é feito observando as soluções naturais da equação d1 + d2 + d3 + d4 =
6, que são P96,3 = 84.
Segundo, os anéis são diferentes, sendo assim, podem ser permutados de 6! = 720 maneiras. Portanto,
obtemos
84 × 720 = 60480 colocações.
137. (Extraı́do do vestibular do IME)
Vamos separar em casos a partir de um determinado cavaleiro como referência (seja o 1), agora no sentido
horário chame os demais de 2, 3, · · · , 12 (observe que 1 e 12 são vizinhos).
i) se 1 for escolhido restam 4 vagas entre os cavaleiros {3, 4, 5, 6, 7, 8, 9, 10, 11}, usando o raciocı́nio de itens
anteriores, serão cinco − e quatro +, de modo que dois sinais de + sempre serão separados.

? − ? − ? − ? − ? − ?

Ou seja, vamos escolher 4 espaços vazios ( ? ) dentro os seis disponı́veis de C6,4 = 15 modos.
ii) se 1 não for escolhido ficam as 5 vagas entre os cavaleiros {2, 3, 4, 5, 6, 7, 8, 9, 10, 11, 12}, usando o raciocı́nio
de itens anteriores, serão seis − e cinco +, de modo que dois sinais de + sempre serão separados.

? − ? − ? − ? − ? − ? − ?

Ou seja, vamos escolher 5 espaços vazios ( ? ) dentre os sete disponı́veis de C7,5 = 21 modos. Por fim, serão
15 + 21 = 36 grupos.
138. Como há 12 signos do zodı́aco, basta n = 13 para que duas pessoas tenham o mesmo signo.
A ideia é pensar nos Signos como as casas e nas pessoas como os pombos.


Pombo sem casa, o 13◦ elemento.

• • • ··· •
Casa 1 Casa 2 Casa 3 Casa 12
Logo, há 12 casas, e para que alguma das casas tenha dois pombos, basta ter n = 12 + 1 pombos.
139. Vamos pensar na quantidade de frutos como as casas e nas jaqueiras como os pombos (•)
0, 1, 2, · · · , 600

• • • ··· •
0 fruto 1 fruto 2 frutos 600 frutos
Agora coloquemos as jaqueiras (que serão os pombos) nas respectivas casas que representam suas quantidades
de frutos.


&
Caso ocupemos todas as casas, ainda haverá
399 jaqueiras a serem distribuı́das.

• • • ··· •
0 fruto 1 fruto 2 frutos 600 frutos
Como 1000 > 601, o PCP garante que alguma casa terá dois pombos. O que prova o que foi pedido! 

45
POTI 2015 − Combinatória − Nı́vel 2 − Aula 0 − Professores Cleber Assis, Samuel Barbosa e Tiago Miranda

140. Considere as três cores como sendo as casas e as meias retiradas como os pombos.
a) Pelo Princı́pio da Casa dos Pombos, se retirarmos 4 meias, pelo menos duas delas terão a mesma cor. Para
ver que esse é o número mı́nimo, note que é possı́vel pegarmos uma meia de cada cor nas três primeiras
retiradas e não formarmos um par.


4a meia.

• • •
Meias Pretas Meias Brancas Meias Marrons

Resposta: 4 meias.
b) Observe que o cenário mais difı́cil para o objetivo é retirar todas as meias de cor preta, todas as meias de
cor marrom e depois o par de cor branca. Para isso, deveremos retirar 20 + 10 + 2 = 32 meias para garantir
o par de cor branca.


32a meia

20 meias • 10 meias
Meias Pretas Meias Brancas Meias Marrons
Resposta: 32 meias.
141. Seja xi a quantidade de aniversariantes no mês i ∈ 1, 2, . . . , 12. Daı́
x1 + x2 + · · · + x12 = 54.
Fazendo a média aritmética temos
x1 + x2 + · · · + x12 54
= = 4, 5.
12 12
O número médio de aniversariantes por mês é 4, 5.
142.
a) Seja xi ∈ N a quantidade de aniversariantes no mês i ∈ 1, 2, . . . , 12. Daı́
x1 + x2 + · · · + x12 = 54.
Fazendo a média aritmética temos
x1 + x2 + · · · + x12 54
= = 4, 5.
12 12
O número médio de aniversariantes por mês é 4, 5, e pela proposição 1, ao menos um dos xi será maior do
que ou igual a média. Como xi é natural, seu menor valor será, no mı́nimo, 5. 
b) As 4 primeiras questões podem ser respondidas de 5 × 5 × 5 × 5 = 625 maneiras. O número médio de
40100
candidatos para cada possı́vel resposta é = 64, 16. Pelo proposição 1, garante-se que ao menos 65
625
candidatos terão mesma resposta para as primeiras 4 questões.
c) As n primeiras questões podem ser respondidas de 2 × 2 × · · · × 2 = 2n maneiras. Para garantir que pelo
menos 4 candidatos respondam a estas questões do mesmo modo, precisa-se ter pelo menos 3 · 2n + 1
candidatos. Portanto, deve-se ter no mı́nimo
3 · 2n + 1 ≤ 98305.
O que ocorre para n ≤ 15. Por fim, o valor máximo é para n = 15.

46
POTI 2015 − Combinatória − Nı́vel 2 − Aula 0 − Professores Cleber Assis, Samuel Barbosa e Tiago Miranda

143. Demonstração:
Dada a lista x1 , x2 , x3 , · · · , xn com todos esses números menores que a média, isto é:
x1 < x
x2 < x
x3 < x
.. .. ..
. . .
xn < x
Somando todas as inequações obtemos:
x1 + x2 + x3 + · · · + xn < |x + x + x{z+ · · · + x}
n parcelas iguais a x.
x1 + x2 + x3 + · · · + x n < n·x
x1 + x2 + x3 + · · · + x n
< x
n
x1 + x2 + x3 + · · · + x n
< x
n
x < x
Concluı́mos que a média aritmética é menor do que a média aritmética, absurdo. Portanto, não podemos ter
todos os xi , com i ∈ {1, 2..., n}, menores do que x. Ou seja, ao menos um dos xi é maior do que ou igual a x.
(Análogo para o caso de ter valores menores do que ou iguais a média aritmética.) 
144. Seja xi , com i ∈ {Flamengo, Botafogo, Fluminense, Vasco} a quantidade de torcedores do time i.
x Flamengo + x Bota f ogo + x Fluminense + xVasco = 90
e a média de torcedores por time fica:
90
x= = 22, 5.
4
Portanto, utilizando a proposição 1, algum xi > 22, 5 e como xi ∈ N, há alguma torcida com, no mı́nimo, 23
integrantes. 
145. Considere os candidatos como os pombos e as sequências de respostas como as casas. Como cada
questão possui de 5 alternativas, a prova poderá ser respondida de
5 × 5 × 5 × · · · × 5 = 510 = 9765625 modos.
Logo, pelo PCP, para que dois candidatos forneçam exatamente as mesmas respostas, deverão participar pelo
menos 9765626 pessoas.
146. Como há 6 faces, para ter certeza que ao menos um delas saiu:
i) 2 vezes, deveremos ter ao menos 7 lançamentos;
ii) 3 vezes, deveremos ter ao menos 13 lançamentos;
iii) 4 vezes, deveremos ter ao menos 19 lançamentos;
iv) 5 vezes, deveremos ter ao menos 26 lançamentos; e
v) 6 vezes, deveremos ter ao menos 31 lançamentos.
No mı́nimo, d = 31 lançamentos.
A ideia é pensar que o número em cada face representa uma casa (6 números = 6 casas). Queremos alguma
casa com mais do que 5 pombos (lançamentos) então deve-se distribuir os resultados dos lançamentos nas
respectivas casas, nosso objetivo é alcançar uma casa com mais de 5 repetições. Isso pode acontecer “no pior
dos cenários” se todas as casas forem sendo preenchidas até chegar ao número 5, esse quadro será alcançado
após 30 lançamentos. Portanto, no 31◦ lançamento teremos o objetivo alcançado!

47
POTI 2015 − Combinatória − Nı́vel 2 − Aula 0 − Professores Cleber Assis, Samuel Barbosa e Tiago Miranda

147. Em um problema geométrico nem sempre é simples perceber como construir as casas dos pombos, mas
é imediato concluir que os pontos representam os pombos.√As casas deverão ser pensadas de modo a que em
algum momento surja algum segmento de comprimento 2. Sendo assim, observe a construção abaixo:

e√perceba que há quatro quadrados internos (surgiram as casas) de lado medindo 1 cm e diagonal medindo
2. Por fim,√
como são cinco pontos, ao menos uma das casa terá dois pontos e a distância entre eles será de,
no máximo, 2. 
148. Sejam xi , com i ∈ 1, 2, 3, 4, 5 as idades dos cinco estudantes, de modo que
x1 + x2 + x3 + x4 + x5 = 86 anos.
Com 5 estudantes podemos formar C5,3 = 10 trios, de modo que cada estudante participe de C4,2 = 6 trios
(uma das três vagas seria de x1 , por exemplo, e os outros 4 estudantes disputam as outras duas vagas).
Podemos destacar todos os trios possı́veis sendo somados em 10 expressões:
x1 + x2 + x3 =
x1 + x2 + x4 =
x1 + x2 + x5 =
x1 + x3 + x4 =
x1 + x3 + x5 =
x1 + x4 + x5 =
x2 + x3 + x4 =
x2 + x3 + x5 =
x2 + x4 + x5 =
x3 + x4 + x5 = , somando-as, obtemos
6 · ( x1 + x2 + x3 + x4 + x5 ) = 6 · 86 = 516.
516
Portanto, a média dos trios será = 51, 6 anos. Usando a proposição 1, ao menos um dos trios terá soma
10
maior do que ou igual a 51. 
149. Numa divisão por oito existem os restos {0, 1, 2, 3, 4, 5, 6, 7}, portanto 8 opções (casas). Tomando os 9
inteiros (pombos), disporemos cada um na respectiva casa que representa o seu resto na divisão por 8. Sendo
9 pombos, pelo PCP, alguma casa terá dois deles e a diferença entre esses será divisı́vel por 8. 
150.
a) Para as 5 primeiras questões há 45 = 1024 gabaritos distintos. É possı́vel termos 2048 candidatos de modo
a cada gabarito se repetir duas vezes. Daı́, se houver 2049 pessoas está garantido que alguma sequência de
respostas foi repetida ao menos 3 vezes.
b) Para as n primeiras questões há 4n gabaritos distintos. Entre 1000 candidatos garante-se que ao menos 2
responderam da mesma forma se
1000 ≥ 4n + 1,
ou seja 4n ≤ 999, que, observando o item anterior, se resolve com no máximo n = 4.

48
POTI 2015 − Combinatória − Nı́vel 2 − Aula 0 − Professores Cleber Assis, Samuel Barbosa e Tiago Miranda

151. (Extraı́do do Vestibular da PUC/RJ)


Pensando nos 12 meses como as casas e as n pessoas como os pombos. Se houver uma distribuição de 3
pessoas em cada mês não se chega ao objetivo do problema e já teriam sido observadas 12 × 3 = 36 pessoas
no grupo. Agora basta que mais uma pessoa seja colocada em qualquer das casas para concluir o problema.
Portanto, 37 pessoas num grupo garantem que ao menos 4 nasceram no mesmo mês.

152. (Extraı́do do Vestibular da UERJ/RJ - 2011)


10 cores = 10 casas. Busca-se uma casa com 4 pombos (bolas de mesma cor). Isso acontecerá, “no pior dos
cenários”, se todas as casas forem sendo preenchidas até chegar ao número 4, esse quadro será alcançado após
31 retiradas. Na 30a retirada cada casa terá três pombos e no lançamento seguinte está garantido o objetivo
alcançado!

153. Todo número inteiro pode ser escrito da forma 2n · b, com b ı́mpar e n natural. Observando o conjunto
percebemos que b ∈ {1, 3, 5, 7, 9, ·, 79}, subconjunto com 40 elementos (casas). Como serão escolhidos 41
números (pombos), ao menos dois deles , pelo PCP, terão o mesmo b (x = 2n1 · b e y = 2n2 · b, com x < y). Daı́
concluı́mos que x divide y. 

154. Vamos pensar na quantidade de conhecidos como as casas

0, 1, 2, · · · , n − 1

Agora coloquemos as pessoas (que serão os pombos) nas respectivas casas que representam suas quantidades
de conhecidos. Há n pessoas e n gavetas, mas as gavetas 0 e n − 1 não poderão ser preenchidas ao mesmo
tempo, afinal, se alguém conhece n − 1 dos presentes, não há como outra pessoa ficar sem conhecidos no
grupo (e vice-versa). O que reduz a “quantidade prática” das gavetas para n − 1. O PCP garante que alguma
casa terá dois pombos. O que prova o que foi pedido! 
y
155. A inequação simultânea 1 < ≤ 2 pode ser reescrita como x < y ≤ 2x. Vamos criar 6 conjuntos (casas)
x
com elementos que verificam essa inequação. Para tal, construiremos conjuntos com os números de x até 2x
como segue:

{1, 2}
{3, 4, 5, 6}
{7, 8, 9, 10, 11, 12, 13, 14}
{15, 16, 17, · · · , 28, 29, 30}
{31, 32, 33, · · · , 60, 61, 62}
{63, 64, 65, · · · , 122, 123, 126}

Qualquer dupla pertencente a um mesmo conjunto verifica a inequação. Se fossem 6 números (pombos),
poderı́amos escolher um de cada conjunto e a afirmação não seria verdadeira. Como são 7 números (pombos),
pelo PCP, haverá dois deles pertencentes ao mesmo conjunto e isso demonstra o que foi pedido. 

156. Construa os n subconjuntos de números consecutivos

{1, 2}, {3, 4}, {5, 6}, · · · , {2n − 1, 2n}

essas serão as casas. Como serão destacados n + 1 elementos (pombos), pelo PCP, alguma dupla será escolhida
no mesmo conjunto e será composta por números primos entre si. 

49
POTI 2015 − Combinatória − Nı́vel 2 − Aula 0 − Professores Cleber Assis, Samuel Barbosa e Tiago Miranda

157. Tome a sequência


1, 11, 111, 1111, 11111, · · · , 111
| .{z
. . 111}
“n + 1” uns.
Esses n + 1 números (pombos), serão distribuı́dos nas n casas que representam os possı́veis restos numa
divisão por n. Pelo PCP, teremos dois pombos na mesma casa. A subtração entre esses números será:
111 . . 111} − 111
| .{z . . 111} = 111
| .{z | .{z
. . 111} 000
| .{z
. . 000}
“a” uns. “b” uns. “a − b” uns “b” zeros.
sendo “a” maior do que “b”. O que finaliza a demonstração. 
158. Tome a sequência
1, 11, 111, 1111, · · · , 11111111
Esses 8 números (pombos), serão distribuı́dos nas 7 casas que representam os possı́veis restos numa divisão
por 7. Se algum deles estiver na casa do “zero”, então a demonstração acabou (isso ocorre com o 111111).
Caso contrário, pelo PCP, teremos dois pombos na mesma casa. A subtração entre esses números será:
111 . . 111} − 111
| .{z . . 111} = 111
| .{z | .{z
. . 111} 000
| .{z
. . 000}
“a” uns. “b” uns. “a − b” uns “b” zeros.
sendo “a” maior do que “b”. Como
111
| .{z
. . 111} 000 . . 000} = 111
| .{z . . 111} ·10b .
| .{z
“a − b” uns “b” zeros. “a − b” uns

que será múltiplo de 7, que, por sua vez, não divide 10b , portando dividirá 111
| .{z
. . 111}. 
“a − b” uns

Comentário para professores: O método desenvolvido no problema 158 poderá ser aplicado para qualquer
número que seja relativamente primo com 10. Ou seja, qualquer número que não possua em sua fatoração o 2
ou o 5, possui um múltiplo apenas com algarismos iguais a 1.
159. (Extraı́do da OBM)
Considere todos os números da forma 199 . . . 91 (pombos) e observe os 1991 possı́veis restos numa divisão
por 1991 (casas). Como há mais pombos do que casas, teremos dois deles na mesma casa e a subtração entre
eles será um múltiplo de 1991 da forma 199 . . . 9980 . . . 0. Sendo mdc(10, 1991) = 1 podemos eliminar zeros à
direita sem perder a divisibilidade por 1991. Façamos isso até chegarmos ao número 199 . . . 998000 , agora,
basta somarmos 1991 e chegamos a um número do tipo 199 . . . 91 
160. (Extraı́do da IMO)
Seja C um conjunto com 10 elementos. Ele possui
210 − 2 = 1022 subconjuntos
diferentes de C e não vazios, serão os pombos. Como os elementos possuem dois dı́gitos, então:
i) a menor soma entre nove deles1 será
10 + 11 + 12 + 13 + 14 + 15 + 16 + 17 + 18 = 145

ii) e e a maior será


91 + 92 + 93 + 94 + 95 + 96 + 97 + 98 + 99 = 855.
Os números de 145 até 855 representam as casas, isso resulta em
855 − 145 + 1 = 711 casas.
Como há mais pombos do que casas teremos ao menos dois na mesma casa. Agora, tome os subconjuntos que
geraram mesma soma (estão na mesma casa) e descarte os elementos comuns (criando assim os subconjuntos
disjuntos). Pronto, chegamos aos dois conjuntos disjuntos com elementos somando o mesmo valor. 
1 Não iremos usar os 10 elementos pois queremos dois subconjuntos disjuntos, sem interseção.

50
POTI 2015 − Combinatória − Nı́vel 2 − Aula 0 − Professores Cleber Assis, Samuel Barbosa e Tiago Miranda

161. (Extraı́do da IMO)


Escolha uma pessoa do grupo, por exemplo, Maria. Ela se corresponde com 16 outras pessoas. Pelo PCP,
Maria debaterá algum tópico com ao menos 6 pessoas, por exemplo o tema I. Se duas dessas 6 debaterem
esse tema, esta demonstrado. Caso contrário, essas seis só debatem entre si os temas I I e I I I. Seja uma delas o
Paulo. Pelo PCP, das 5 pessoas restantes, ao menos 3 vão debater um dos tópicos restantes, suponha que seja
o I I. Se entre essas três tivermos um par que debate do tópico I I entre si, então somamos Paulo e finalizamos
a demonstração. Caso negativo, esses três elementos só debaterão entre si no tópico I I I e a demonstração
acabou. 
162. (Extraı́do do material do PROFMAT)
Em 11 semanas temos 77 dias. Defina Si , i ∈ 1, 2, · · · , 77 como sendo o número de partidas jogadas a partir do
dia 1 até o dia i. Como ele joga ao menos uma partida por dia, temos

1 ≤ S1 < S2 < S3 < · · · < S77 .

Além disso, S77 ≤ 132, pois ele não joga mais de 12 partidas por semana, ou seja, o total de jogos não atinge
11 · 12 = 132. Agora, chamando S0 = 0, a quantidade de jogos entre os dias a e b, inclusive, é igual a Sb − Sa−1 .
Queremos mostrar que é possı́vel determinar a e b de momo que Sb − Sa−1 = 20. Considere os 154 números
(que serão os pombos)
S1 , S2 , . . . , S77 , S1 + 20, S2 + 20, . . . , S77 + 20.
|{z} | {z }
≤132 ≤152

Eles pertencem a {1, 2, . . . , 152}, que serão as casas. O PCP assegura que alguma casa terá dois pombos. Como

S1 < S2 < S3 < · · · < S77

e, por consequência,
S1 + 20 < S2 + 20 < S3 + 20 < · · · < S77 + 20,
os números iguais deverão estar em metades diferentes da lista2 . Então existem a0 e b0 tais que Sb0 = Sa0 + 20.
O enxadrista joga 20 partidas entre os dias b0 + 1 e a0 , inclusive. 

2 Nãopodemos concluir que para todo j e i, j, i ∈ {1, 2, . . . , 77}, é verdade que Si < S j + 20. Se, por exemplo, S1 = 1 e S1 4 = 22,
terı́amos S1 4 > S1 + 20. Por isso não podemos fazer afirmações desse tipo sobre Si e S j + 20.

51
Polos Olímpicos de Treinamento
Curso de Combinatória - Nível 2 Aula 1
Prof. Bruno Holanda

Lógica

Nos últimos anos, a participação brasileira em competições internacionais de matemática


vem melhorado significamente. E uma das consequências do sucesso de nossos alunos é o
crescimento da demanda de interessados em aprender mais sobre o que é a olimpı́ada e que
tipo de problemas são abordados em suas competições.

O grande diferencial de problemas de olimpı́ada de matemática para os problemas usu-


ais, são seu alto nı́vel de exigência do uso raciocı́nio lógico. Portanto, em muitos casos,
a matemática aparece como uma ferramenta para desenvolver a argumentação de ideias
abstratas.

Este é o primeiro de dois artigos escritos com o objetivo de apresentar tais problemas,
mesmo sem desenvolver uma teoria matemática propriamente dita. Vamos nos focar dire-
tamente nas ideias.

Problema 1. Quatro garotos jogam tiro ao alvo. Cada um deles atirou três vezes. No alvo
abaixo, pode-se ver os lugares atingidos. A pontuação é 6 para o centro e diminui um ponto
para cada nı́vel mais distante.

⊗ ⊗


⊗ ⊗ ⊗
⊗ ⊗
⊗ ⊗

Se os quatro garotos empataram, determine:

(a) a pontuação total de cada jogador.


POT 2012 - Combinatória - Nı́vel 2 - Aula 1 - Prof. Bruno Holanda

(b) a pontuação dos três tiros de cada jogador.

Solução. A soma de todos os pontos obtidos foi 6+5+4×3+3×3+2×4 = 40. Como todos
empataram, cada um deve ter feito exatamente 10 pontos (isso responde o item a). Além
disso é importante perceber que ninguém errou nenhum dos tiros, já que há exatamente 12
dardos no alvo.

Note que um dos jogadores (digamos A) acertou um dos dardos no centro do alvo,
fazendo 6 pontos. Para completar os 10 pontos ele deve ter feito mais 4 pontos. Como é
impossı́vel fazer apenas 1 ponto, ou dele ter errado, só nos resta a possibilidade dele ter
feito 2 pontos nos dois outros tiros. (Continue a solução)

O objetivo de outro tipo de problema é achar um exemplo que cumpra alguma propri-
edade.

Problema 2. (OBM 1998) Encotre uma maneira de se escrever os algarismos de 1 a 9 em


seqüência, de forma que os números determinados por quaisquer dois algarismos consecu-
tivos sejam divisı́veis por 7 ou por 13.

Solução. Primeiramente vamos listar todos os números de dois algarismos que são múltiplos
de 7 ou 13. São eles:
Múltiplos de 7: 14, 21, 28, 35, 42, 49, 56, 63, 70, 77, 84, 91, 98
Múltiplos de 13: 13, 26, 39, 52, 65, 78, 91

Como não podemos repetir nenhum algarismo, devemos descartar o 77. Por outro lado,
nenhum dos números acima (excluindo o 77) termina em 7. Daı́, pode-se ter certeza que o
primeiro número da lista deve ser 7. Para saber as possı́veis listas, usamos um diagrama
de árvore:
== 5⊗ {
{{
{{{
{
{{
3 // 9 // 1⊗
~ ~>>
~~
~~~
~
@@ 6
// 5⊗ == 5 // 6⊗
 {{{
 {
 {{
 {{{
7 // 8 // 4
<<
// 2 // 1 // 3 // 9⊗
<<
<<
<<
 √
9 // 1 // 3 // 5
BB
// 2 // 6
BB
BB
BB
B!!
6⊗

2
POT 2012 - Combinatória - Nı́vel 2 - Aula 1 - Prof. Bruno Holanda

Representamos com um ⊗ quando não foi possı́vel continuar a lista sem repetir nenhum
dı́gito. Assim, o modo correto de se escrever os algarismo é: 784913526.

Em alguns casos é necessário o uso de variáveis para resolver um problema. Isto acon-
tece pois existem informações não especificadas no enunciado, e o uso de letras se mostra
uma forma inteligente e fácil de trabalhar com valores desconhecidos. A seguir vamos re-
solver um problema que apareceu em uma olimpı́ada russa de 1995.

Problema 3. (Rússia 1995) Um trêm deixa Moscou às x horas e y minutos, chegando em
Saratov às y horas e z minutos. O tempo da viagem foi de z horas e x minutos. Ache todos
os possı́veis valores para x.

Solução. Das condições do problema, temos que:

(60y + z) − (60x + y) = 60z + x

⇒ 60(y − x − z) = x + y − z.
Com isso, podemos garantir que x + y − z é um múltiplo de 60. Por outro lado, como
0 ≤ x, y, z ≤ 23, o único valor possı́vel para x + y − z é 0. Ou seja, x + y = z. Além disso,
na equação inicial temos que 60(y − x − z) = 0. Daı́, y = x + z. Logo, o único valor de x
que garante essas igualdades é x = 0.

É importante perceber que no exemplo anterior que apenas o uso de letras não seria
o suficiente para resolver o problema. O fundamental para resolver as equações acima era
o significado das letras: números inteiros entre 0 e 60. Sem esta restrição o problema
apresentaria infinitas soluções. Então fica a dica: nunca se esqueça do signigficado das
variáveis que estiver usando, se são dı́gitos, números inteiros, racionais ou seja qual
for a propriedade. Lembre-se que esta propriedade terá papel importante na solução do
problema.

Organizar as informações também é útil na maioria dos problemas, como veremos no


exemplo a seguir.

Problema 4. Paulo possui 13 caixas vermelhas e cada uma delas está vazia ou contém 7
caixas azuis. Cada caixa azul está vazia ou contém 7 caixas verdes. Se ele possui 145 caixas
vazias, quantas caixas ele possui no total?

Solução. Vamos montar uma tabela que ajudará na solução do problema

Vermelhas Azuis Verdes


Cheias x y 0
Vazias 13 − x 7x − y 7y
Total 13 7x 7y

3
POT 2012 - Combinatória - Nı́vel 2 - Aula 1 - Prof. Bruno Holanda

Suponha que o número de caixas vermelhas cheias seja x e que o número de caixas azuis
cheias seja y. Portanto, temos 7x caixas azuis e 7y caixas verdes. Note também que todas as
caixas verdes estão vazias. Dessa forma, o total de caixas vazias é (13 − x) + (7x − y) + 7y =
145. Assim, podemos concluir que x+y = 22. Como o número total de caixas é 13+7(x+y),
a resposta correta será 13 + 7 × 22 = 167.

Problemas Propostos

Problema 5. Samuel possui três irmãos a mais do que irmãs. Samila, a irmã de Samuel,
possui o número de irmãos igual ao dobro do número de irmãs. Quantos filhos (homens e
mulheres) possui o pai de Samuel e Samila?
Problema 6. Em um hotel para cães e gatos, 10% dos cães acham que são gatos e 10% dos
gatos acham que são cães. Verificou-se também que 20% dos animais acham que são gatos.
Se no hotel existem 10 gatos, quantos são os cães?
Problema 7. É possı́vel cortar um tabuleiro 39 × 55 em vários retângulos 5 × 11?
Problema 8. No fim de 1994, Neto tinha metade da idade de seu avô. A soma dos anos de
nascimento dos dois é 3844. Quantos anos Neto completou em 2006?
Problema 9. Um professor propõe 80 problemas a um aluno, informando que ele ganha 5
pontos ao acertar cada problema corretamente e perde 3 pontos caso não resolva o problema.
No final, o aluno tinha 8 pontos. Quantos problemas ele resolveu corretamente?
Problema 10. (Leningrado 1987) Na ilha de Anchúria existem quatro tipos de notas: 1$,
10$, 100$ e 1000$. Podemos obter 1$ milhão com exatamente 500.000 notas?
Problema 11. Você tem uma lista de números reais, cuja soma é 40. Se você trocar todo
número x da lista por 1 − x, a soma dos novos números será 20. Agora, se você trocar todo
número x por 1 + x, qual será o valor da soma?
Problema 12. (Eslovênia 1992) Complete a tabela abaixo de modo que:
i. A soma de quaisquer três vizinhos seja a mesma.
ii. A soma total dos números seja 171.

15 13

Problema 13. Trabalhando juntos Alvo e Ivo, pintam uma casa em três dias; Ivo e Eva
pintam a mesma casa em quatro dias; Alvo e Eva em seis dias. Se os três trabalharem
juntos, quantos em quantos dias pintarão a casa?
Problema 14. (Rioplatense 1997) Em cada casa de um tabuleiro 4 × 4 é colocado um
número secreto. Sabe-se que a soma dos números em cada linha, coluna e diagonal é 1.
Com essa informação é possı́vel determinar a soma dos números escritos nos quatro cantos?
E a soma dos quatro números escritos no centro? Se for, quais são essas somas?

4
POT 2012 - Combinatória - Nı́vel 2 - Aula 1 - Prof. Bruno Holanda

Dicas e Soluções

6. Construa uma tabela, tente usar apenas uma variável!

7. Não. Demonstre que não é possı́vel cobrir um dos lados do tabuleiro.

10. Sejam x, y, z e w as quantidades de notas. Monte um sistema com duas equações e


use o fato de 500.000 não ser múltiplo de 9.

13. Use o fato de Alvo e Ivo pintarem um terço da casa em um dia.

14. Separe o tabuleiro em três regiões. Não se preocupe com os números, mas com a
soma dos números nestas regiões espertas.

5
Polos Olímpicos de Treinamento
Curso de Combinatória - Nível 2 Aula 2
Prof. Bruno Holanda

Lógica II

Quando lemos um problema de matemática imediatamente podemos ver que ele está
dividido em duas partes: as informações e as perguntas. Você vai aprender, durante sua
jornada como olı́mpico, que para resolver um problema de matemática você deve conhecer
várias técnicas. Uma das mais básicas é saber organizar as informações que são oferecidas
pelos problemas.

Problema 1. (OCM 1990) A pesquisa realizada com as crianças de um conjunto habitacio-


nal, que apurou as preferências em relação aos três programas de televisão: Alegre Amanhã
(designado por A), Brincolândia (designado por B) e Criança Feliz (designado por C)
indicou os seguintes resultados:

Prog A B C AeB AeC BeC A,B e C Nenhum


Pref 100 150 200 20 30 40 10 130

Pergunta-se:

(a) Quantas crianças foram consultadas?

(b) Quantas crianças apreciam apenas um programa?

(c) Quantas crianças apreciam mais de um programa?

Solução. Você deve ter percebido que existe um grande número de informações dadas.
De certa forma, essas informações já estão organizadas em uma tabela. Mas para resolver
o problema vamos mudar nossa representação, nosso ponto de vista. Vamos construir um
diagrama de Venn, o popular diagrama de conjuntos:
POT 2012 - Combinatória - Nı́vel 2 - Aula 2 - Prof. Bruno Holanda

C
140 130

20 30
10

60 10 100

A B

Podemos agora responder às perguntas facilmente:

a) Foram consultadas 10 + 10 + 20 + 30 + 60 + 100 + 140 + 130 = 500 crianças.

b) 60 + 100 + 140 = 300 crianças gostam de apenas um programa.

c) 10 + 10 + 20 + 30 = 70 crianças apreciam mais de um programa.

O próximo exemplo usa apenas o raciocı́nio lógico.

Problema 2. (Torneio das Cidades) Carlixtos possui seis moedas, sendo uma delas falsa.
Nós não sabemos o peso de uma moeda falsa e nem o peso de uma moeda verdadeira,
sabemos apenas que as moedas verdadeiras possuem todas o mesmo peso e que o peso da
moeda falsa é diferente. Dispomos de uma balança de dois pratos. Mostre como é possı́vel
descobrir a moeda falsa usando apenas três pesagens.

Solução. Sejam A, B, C, D, E e F as moedas. Primeiramente fazemos a pesagem


(AB) <> (CD) (que significa A e B em um prato e C e D em outro). Se (AB) = (CD)
(ou seja, se equilibrar), então ou E ou F é falsa. Neste caso fazemos a pesagem (A) <> (E).
Se equilibrar, F é falsa. Caso contrário, E é falsa.

Agora, se não houve equilı́brio em (AB) <> (CD), então E e F são verdadeiras. Fa-
zemos então a pesagem (AB) <> (EF ). Se equilibrar, ou C ou D é falsa. Neste caso,
fazemos a pesagem (A) <> (C). Se equilibrar, D é falsa. Caso contrário, C é falsa.

Para finalizar, se (AB) 6= (EF ), então ou A ou B é falsa. Neste caso, fazemos a pesa-
gem (A) <> (C). Se equilibrar B é falsa. Caso contrário, A é falsa.

Continuando o processo de desenvolvimento do raciocı́nio, vamos resolver a seguir duas


questões relacionadas com a seguinte pergunta: Será possı́vel?. Ao longo do ano você verá

2
POT 2012 - Combinatória - Nı́vel 2 - Aula 2 - Prof. Bruno Holanda

como essa pergunta é frequente na olimpı́ada. Na verdade, ela é recorrente em toda a ma-
temática. Aqui também vamos desenvolver uma das técnicas mais poderosas usadas para
resolver problemas de matemática. Que é a idéia de prova por absurdo.

Problema 3. (Ivan Borsenco) É possı́vel cortar um retângulo 5 × 6 em oito retângulos


distintos com dimensões inteiras e lados paralelos aos lados do retângulo maior?

Solução. Vamos assumir que todos os retângulos são distintos. Os retângulos de menor
área possı́vel são:

Área 1: 1 × 1 Área 4: 2 × 2 e 1 × 4
Área 2: 1 × 2 Área 5: 1 × 5
Área 3: 1 × 3 Área 6: 2 × 3 e 1 × 6

Note que a menor área coberta por oito retângulos distintos deve ser pelo menos
1 + 2 + 3 + 4 + 4 + 5 + 6 + 6 = 31 > 30. Logo é impossı́vel obter 8 retângulos distin-
tos.

É importante tomar cuidado com esse tipo de enunciado pois, em alguns casos, é
possı́vel.

Problema 4. (Torneio das Cidades 2001) Podemos trocar um inteiro positivo n pelo pro-
duto a × b onde a e b são inteiros positivos tais que a + b = n. Podemos obter 2001 a partir
de 22, por uma seqüência de trocas?

Solução. Note que 2001 = 3× 667 pode ser obtido de 3+ 667 = 670, que pode ser obtido de
67 + 10 = 77 que pode ser obtido de 7 + 11 = 18. Por outro, todo número n − 1 = (n − 1) × 1
pode ser obtido de (n − 1) + 1 = n. Assim, basta seguir a seqüência abaixo:

22 → 21 → 20 → 19 → 18 → 77 → 670 → 2001.

Problemas Propostos

Problema 5. São dadas 4 moedas aparentemente iguais. Sabe-se que uma delas é falsa
(tem peso diferente das demais e não se sabe se ela é mais leve ou mais pesada). Mostre
como descobrir a moeda falsa com 2 pesagens em uma balançaa de dois pratos.

Problema 6. Mostre que é possı́vel dispor os números de 1 a 16 em sequância de modo que


a soma de dois números vizinho seja sempre um número quadrado perfeito.

3
POT 2012 - Combinatória - Nı́vel 2 - Aula 2 - Prof. Bruno Holanda

Problema 7. Victor e Maria começam a trabalhar no mesmo dia. Victor trabalha 3 dias
seguidos e depois tem um dia de descanso. Maria trabalha 7 dias seguidos e descansa os
outros 3. Quantos dias de descanso em comum tiveram os dois durante os 1000 primeiros
dias.

Problema 8. Como recortar um retângulo 3 × 13 em treze retângulos menores de lados


inteiros distintos?

Problema 9. (Olimpiada de Maio) Num ano que tem 53 sábados, que dia da semana é 12
de maio? Diga todas as possibilidades.

Problema 10. Um número é dito lindo se é divisı́vel por cada um dos seus dı́gitos não
nulos. Qual é a maior quantidade de números lindos consecutivos que pode existir?

Problema 11. (Búlgaria 2005) Ivo escreve todos os inteiros de 1 a 100 (inclusive) em cartas
e dá algumas delas para Iana. Sabe-se que quaisquer duas destas, uma de Ivo e outra de
Iana a soma dos números não está com Ivo e o produto não está com Iana. Determine o
número de cartas de Iana sabendo que a carta 13 está com Ivo.

Problema 12. (Rússia 1999) Mostre que os números de 1 a 15 não podem ser divididos
em um grupo A de dois elementos e um grupo B de 13 elementos tais que a soma dos
elementos de B seja igual ao produto dos elementos de A.

Problema 13. (Seletiva Rioplatense 2004) Em cada casa de um tabuleiro 8 × 8 escrevemos


um número inteiro. Sabe-se que para cada casa, a soma dos seus vizinhos é 1. Encontre a
soma de todos os números do tabuleiro.
Obs: Consideramos vizinhas casas com um lado em comum.

Problema 14. Etevaldo pensou em cinco números distintos e escreveu no quadro todos dez
números que são somas de dois destes cinco números. Será que Ovozildo pode descobrir os
números que Etevaldo pensou observando apenas os números escritos no quadro?

4
POT 2012 - Combinatória - Nı́vel 2 - Aula 2 - Prof. Bruno Holanda

Dicas e Soluções

6. Liste todas as possı́veis somas cujo resultado é um quadrado perfeito. Observe que a
sequência deve ser iniciada por 8 ou 16.

7. Use perı́odo 20.

11. (Inı́cio da solução) Iana possui pelo menos uma carta, digamos a carta com o número
k. Se 1 está com Ivo, o produto 1.k = k não está com Iana, que é uma contradição.
Logo, 1 está com Iana.
Se 12 está com Ivo, a soma 1 + 12 = 13 não está com Ivo, que também é uma
contradição. Logo, 12 está com Iana.
Agora, se as cartas 3 e 4 estiverem com pessoas diferentes o produto 3.4 = 12 não
estará com Iana. Porém, acabamos de ver que 12 está com Iana. Daı́, 3 e 4 estão
com a mesma pessoa. Se ambas estiverem com Ivo, a soma 1 + 3 = 4 não está com
Ivo, contadição. Logo, 3 e 4 estão com Iana. Conseqüêntimente, 10 e 9 também estão
com Iana, pois a soma 10 + 3 = 9 + 4 = 13 estão com Ivo.

12. Sejam a e b os dois elementos de de A. Pela condições do problema podemos montar


a seguinte equação:
(1 + 2 + · · · + 15) − a − b = ab
⇒ 120 = ab + a + b ⇒ 121 = (a + 1)(b + 1).
Como a e b são inteiros menores que 16, a única solução possı́vel para a equação é
a = b = 10. Que é um absurdo, já que a e b são elementos distintos.

13. Observe as casas marcadas no tabuleiro abaixo:

⋆⋆ ⋆⋆

⋆ ⋆⋆ ⋆
⋆ ⋆
⋆ ⋆
⋆ ⋆
⋆ ⋆
⋆ ⋆⋆ ⋆

Se olharmos para os vizinhos das casas marcadas acima, vemos que eles cobrem todo
o tabuleiro e de maneira disjunta! Como a soma dos vizinhos de cada casa é 1, a
soma total dos números do tabuleiro será igual ao número de casas marcadas, que é
20.

14. Sim, é possı́vel. Sejam a < b < c < d < e os números escolhidos por Etevaldo. A soma
dos números escritos no quadro é igual ao quadruplo da soma S = a + b + c + d + e.
Podemos escolher o maior e o menor valor escrito no quadro. Somando estes valores

5
POT 2012 - Combinatória - Nı́vel 2 - Aula 2 - Prof. Bruno Holanda

e multiplicando-o por quatro obtemos 4S − 4c. Assim, é possı́vel achar o valor de c.


Note que os três maiores valores escritos por Etevaldo são e + d > e + c > d + c. Daı́,
fazendo (e + d) + (e + c) − (d + c) = 2e é possı́vel achar o valor de e. Mais ainda,
como conhecemos e + d, consequentemente, também achamos d. De modo análogo,
observando os três menores valores (a + b < a + c < b + c) é possı́vel determinar a e b.

6
Polos Olímpicos de Treinamento
Curso de Combinatória - Nível 2 Aula 3
Prof. Bruno Holanda

Paridade

Todo número é par ou ı́mpar. Óbvio, não? Pois é com essa simples afirmação que
vamos resolver os problemas deste capı́tulo.

Problema 1. No reino da Frutilândia existe uma árvore mágica que possui 2005 maçãs e
2006 tomates. Todo dia um garoto sobe na árvore e come duas frutas. Quando ele come
duas frutas iguais, nasce um tomate na árvore; quando ele come duas frutas diferentes,
nasce uma maçã. Após alguns dias restará apenas uma fruta na árvore. Que fruta será?

Solução. Sempre que o garoto pega duas frutas da árvore, o número de maçãs diminuirá
de 2 ou permanecerá constante. Dessa forma a paridade do número de maçãs será sempre
o mesmo. Como inicialmente tı́nhamos um número ı́mpar de maças, a quantidade delas
continuará ı́mpar até o final. Logo, a última fruta deve ser uma maçã.

Problema 2. Um jogo consiste de 9 botões luminosos (de cor verde ou amarelo) dispostos
da seguinte forma:
1 2 3

4 5 6

7 8 9
Apertando um botão do bordo do retângulo, trocam de cor ele e os seus vizinhos (do lado
ou em diagonal). Apertando o botão do centro, trocam de cor todos os seus oito vizinhos
POT 2012 - Combinatória - Nı́vel 2 - Aula 3 - Prof. Bruno Holanda

porém ele não. Inicialmente todos os botões estão verdes. É possı́vel, apertando sucessiva-
mente alguns botões, torná-los todos amarelos?

Solução. Note que ao apertar um dos botões 1, 3, 7 ou 9 trocamos de cor 4 botões. Aper-
tando um dos botões 2, 4, 6 ou 8 trocamos a cor de 6 botões. Apertando o botão do centro
trocamos a cor de 8 botões. Como 4, 6 e 8 são números pares a quantidade total de botões
verdes é sempre um número par e para ter os 9 botões amarelos, deveriamos ter zero botões
verdes. Absurdo, já que 0 é um número par.

Para mostrar a relevância do tema que estamos estudando em competições de ma-


temática, vamos resolver dois problemas que apareceram na olimpı́ada do Leningrado (com
o final na União Soviética, passou a ser conhecida como São Petersburgo).

Problema 3. (Leningrado 1990) Paula comprou um caderno com 96 folhas, com páginas
enumeradas de 1 a 192. Nicolas arrancou 25 folhas aleatórias e somou todos os 50 números
escritos nestas folhas. É possı́vel que esta soma seja 1990?

Solução. Observe que a soma dos números escritos em uma mesma folha sempre é ı́mpar.
Dessa forma, se Nicolas arrancou 25 folhas, a soma de todos os números será ı́mpar. Pois é
a soma de uma quantidade ı́mpar de números ı́mpares. Logo, esta soma não pode ser 1990.

Problema 4. (Leningrado 1989) Um grupo de K fı́sicos e K quı́micos está sentado ao redor


de uma mesa. Alguns deles sempre falam a verdade e outros sempre mentem. Sabe-se que
o número de mentirosos entre os fı́sicos e quı́micos é o mesmo. Quando foi perguntado:
“Qual é a profissão de seu vizinho da direita?”, todos responderam “Quı́mico.” Mostre que
K é par.

Solução. Pela resposta das pessoas do grupo, podemos concluir que do lado esquerdo de
um fı́sico sempre está sentado um mentiroso e que do lado direito de um mentiroso sempre
existe um fı́sico. Então, o número de fı́sicos é igual ao número de mentirosos, que é clara-
mente par. Então K é par.

Problema 5. Um gafanhoto vive na reta coordenada. Inicialmente, ele se encontra no ponto


1. Ele pode pular 1 ou 5 unidades, tanto para direita quanto para esquerda. Porém, a reta
coordenada possui buracos em todos os pontos que são múltiplos de 4 (i.e. existem buracos
nos pontos −4, 0, 4, 8 etc), então ele não pode pular para estes pontos. Pode o gafanhoto
chegar ao ponto 3 após 2003 saltos?

Solução. Note que a cada salto, muda a paridade do ponto em que o gafanhoto se encon-
tra. Logo, após 2003 saltos, ele estará em uma coordenada par. Portanto, não pode ser 3.

2
POT 2012 - Combinatória - Nı́vel 2 - Aula 3 - Prof. Bruno Holanda

Para finalizar vamos resolver um problema interessante onde o uso da paridade não
é tão fácil de perceber. Convidamos o leitor a tentar achar uma solução, antes de ler a
resposta em sequência.

O PROBLEMA DOS CHAPÉUS

Imagine que 10 prisioneiros estejam trancados em uma cela quando chega um carcereiro
com o seguinte comunicado:

— Amanhã todos vocês passarão por um teste. Todos vocês ficarão em fila indiana e
serão colocados chapéus nas cabeças de um de vocês. Cada um poderá ver os chapéus dos
que estarão a sua frente. Porém, não poderão ver os chapéus dos que estão atrás, nem
o seu próprio chapéu. Os chapéus serão pretos ou brancos. Feito isso, será perguntado a
cada um de vocês, do último para o primeiro, em ordem, qual a cor do seu chapéu. Se a
pessoa errar a cor do seu chapéu, será morta.

Será que os prisioneiros podem montar uma estratégia para salvar pelo menos 9 deles?

Pensando no problema:

Bem, vamos começar a discutir o problema da seguinte maneira: será que se eles com-
binarem de cada um deles falar a cor do chapéu que está imediatamente a sua frente, eles
podem salvar a maior parte do bando?

Esta é a ideia que todos têm inicialmente, mas logo verifica-se que essa estratégia não
funciona, pois basta que as cores dos chapéus estejam alternadas para a estratégia não fun-
cionar. (Lembre-se: estamos procurando uma estratégia que seja independente da escolha
dos chapéus).

Então devemos pensar de maneira mais profunda. Veja que durante o teste, cada um
dos prisioneiros pode falar apenas uma entre duas palavras que são; preto ou branco. Isto
corresponde a um sistema de linguagem binário. Outras formas de linguagem binária são:
sim e não, zero ou um, par ou ı́mpar. E é exatamente esta analogia que vamos utilizar para
montar nossa estratégia. Que será a seguinte:

O último da fila deve olhar para a frente e contar o número de chapéus pretos. Se este
número for ı́mpar, ele deve gritar preto. Caso contrário, ele deve gritar branco. Com isso,

3
POT 2012 - Combinatória - Nı́vel 2 - Aula 3 - Prof. Bruno Holanda

todos ficam sabendo a paridade da quantidade de chapéus pretos que existem entre os nove
da fila.

Agora, o penúltimo vai olhar para frente e ver a quantidade de chapéis pretos. Se a
paridade continuar a mesma informada pelo último, então seu chapéu é branco. Se mudar,
ele pode concluir que seu chapéu é preto. E isto pode ser feito para todos os membros
da fila, pois todos saberão a cor dos chapéus dos anteriores (tirando a cor do chapéu do
último) e a paridade dos chapéus pretos que existem entre os nove primeiros.

Portanto, é possı́vel salvar os nove primeiros, enquanto o último pode ser salvo, se ele
tiver sorte!

Vale ressaltar que as ideias presentes nesta aula serão de certa forma generalizadas em
aulas futuras como nas aulas de tabuleiros e invariantes.

Problemas Propostos

Problema 6. Existe alguma solução inteira para a equação a · b · (a − b) = 45045.

Problema 7. Os número 1, 2, ..., n estão escritos em sequência. É permitido permutar


quaisquer dois elementos. É possı́vel retornar à posição inicial após 2001 permutações?

Problema 8. Um cı́rculo está dividiso em seis setores que estão marcados com os números
1, 0, 1, 0, 0, 0 no sentido horário. É permitido somar 1 a dois setores vizinhos. É possı́vel,
repetindo esta operação várias vezes, fazer com que todos os números se tornem iguais?

Problema 9. É possı́vel que as seis diferenças entre dois elementos de um conjunto de


quatro números inteiros serem iguais a 2, 2, 3, 4, 4 e 6?

Problema 10. Raul falou que tinha dois anos a mais que Kátia. Kátia falou que tinha
o dobro da idade de Pedro. Pedro falou que Raul tinha 17 anos. Mostre que um deles
mentiu.

Problema 11. (Torneio das Cidades 1987) Uma máquina dá cinco fichas vermelhas quando
alguém insere uma ficha azul e dá cinco fichas azuis quando alguém insere uma ficha ver-
melha. Pedro possui apenas uma ficha azul e deseja obter a mesma quantidade de fichas
azuis e vermelhas usando essa máquina. É possı́vel fazer isto?

Problema 12. (China 1986) Considere uma permutação dos números 1, 1, 2, 2, ..., 1998, 1998
tal que entre dois números k existem k números. É ou não possı́vel fazer isto?

Problema 13. (Rússia 2004) É possı́vel colocarmos números inteiros positivos nas casas
de um tabuleiro 9 × 2004 de modo que a soma dos números de cada linha e a soma dos
números de cada coluna sejam primos? Justifique sua resposta.

4
POT 2012 - Combinatória - Nı́vel 2 - Aula 3 - Prof. Bruno Holanda

Problema 14. O número A possui 17 dı́gitos. O número B possui os mesmos dı́gitos de A,


porém em ordem inversa. É possı́vel que todos os dı́gitos de A + B sejam ı́mpares?

Problema 15. *Considere um tabuleiro 1998 × 2002 pintado alternadamente de preto e


branco da maneira usual. Em cada casa do tabuleiro, escrevemos 0 ou 1, de modo que
a quantidade de 1’s em cada linha e em cada coluna do tabuleiro é ı́mpar. Prove que a
quantidade de 1’s escritos nas casas brancas é par.

Problema 16. *(Ucrânia 1997) Considere um tabuleiro pintado de preto e branco da ma-
neira usual e, em cada casa do tabuleiro, escreva um número inteiro, de modo que a soma
dos números em cada coluna e em cada linha é par. Mostre que a soma dos números nas
casas pretas é par.

5
POT 2012 - Combinatória - Nı́vel 2 - Aula 3 - Prof. Bruno Holanda

Dicas e Soluções

6. Analise as quatro possibilidades de paridade do par (a, b).

9. Se x e y são números inteiros, x + y e x − y possuem a mesma paridade.

13. Suponha que seja possı́vel fazer tal construção. Sejam L1 , L2 , ..., L9 as somas dos
números de cada uma das 9 linhas, e C1 , C2 , ..., C2004 as somas dos números de cada
uma das 2004 colunas. Como cada Li e Cj são primos, estes devem ser números
ı́mpares (já que são soma de pelo menos nove inteiros positivos). Seja S a soma de
todos os números do tabuleiro. Por um lado terı́amos:

S = L1 + L2 + · · · + L9

donde concluı́mos que S é ı́mpar, pois é soma de 9 ı́mpares. Por outro lado:

S = C1 + C2 + · · · + C2004

e daqui concluirı́amos que S é par, o que é um absurdo. Logo tal construção não é
possı́vel.

15. Seja ai,j o número escrito na casa da i-ésima linha e da j-ésima coluna, 1 ≤ i ≤ 1998 e
1 ≤ j ≤ 2002. A casa (i, j) é branca se e somente se i e j possuem a mesma paridade.

X X
999 2002
L= a2i−1,j
i=1 j=1

é a soma dos números nas 999 linhas de ordem ı́mpar. Como a soma dos números de
cada linha é ı́mpar, L é ı́mpar. De maneira análoga, a soma dos números nas 1001
colunas de ordem par
X 1998
1001 X
C= a2j,i
j=1 i=1

também é ı́mpar. Seja P o conjunto de todas as casas pretas que estão em colunas
de ordem par, e S(P ) a soma de todos os números escritos nas casas de P .

Cada número escrito em uma casa de P aparece exatamente uma vez na soma L e
exatamente uma vez na soma C. Ademais, cada número escrito em uma casa branca
aparece exatamente uma vez na soma L + C. Assim, a soma dos números escritos
nas casas brancas é igual a L + C − 2S(P ), que é par.

6
Polos Olímpicos de Treinamento
Curso de Combinatória - Nível 2 Aula 4
Prof. Bruno Holanda

Contagem I

De quantos modos podemos nos vestir? Quantos números menores que 1000 possuem
todos os algarismos pares? Contar coisas é algo tão antigo quanto a própria humanidade.
Porém, ao longo do tempo as idéias evoluiram e novas técnicas surgiram.

Existem várias formas de contar coisas, a mais simples delas é a contagem caso a caso.
Este é o processo que mais usamos em nosso cotidiado. Mas, é uma forma primitiva de
resolver os problemas. Vamos aprender uma técnica mais prática pensando no seguinte
exemplo:

Problema 1. Uma porta só é aberta quando usamos simultaneamente a chave e o cartão
corretos. Se você possui duas chaves e três cartões, quantos testes devemos fazer para
garantir que a porta irá abrir?

Solução. Podemos montar um diagrama (figura 1) para auxilar na solução do problema.

Figura 1: Abrindo uma Porta.

No diagrama acima podemos ver todas as combinações possı́veis de uma chave com um
cartão. Assim, a solução é visual e igual a 6. Por outro lado, poderı́amos ter resolvido o
problema da seguinte forma:

Note que para cada escolha de chave existem três maneiras para escolher o cartão. Como
temos duas chaves, o total de combinações é 2 × 3 = 6. Nesse caso, seriam necessários 6
POT 2012 - Combinatória - Nı́vel 2 - Aula 4 - Prof. Bruno Holanda

testes para achar a combinação correta.

Assim, se houvesse 30 chaves e 5 cartões não seria necessário fazer um diagrama para
contar as combinações uma por uma, o resultado seria simplesmente 30 × 5 = 150. O
método que acabamos de usar é conhecido como princı́pio multiplicativo. Nos próximos
problemas vamos usá-lo de uma forma mais geral.

Problema 2. Teddy possui 5 blusas, 3 calções e 2 pares sapatos. De quantas maneiras


diferentes ele pode se vestir?

Solução. Vamos primeiro contar o número de maneiras que Teddy pode escolher a blusa
e a calça. Bem, para cada calça que Teddy escolhe, ele tem ainda cinco maneiras de es-
colher a blusa. Como ele possui três calças, o número total de modo de escolher o par
(calça e blusa) é 5 × 3 = 15. Agora, para cada maneira de escolher esse par, ele ainda tem
duas maneiras de escolher os sapatos. Daı́, é fácil concluir que Teddy pode se vestir de
5 × 3 × 2 = 30 maneiras diferentes.

Problema 3. De quantos modos podemos pintar um tabuleiro 1 × 4 usando apenas três


cores, sem pintar casas vizinhas da mesma cor?

Solução. Podemos pintar a primeira casa de três maneiras diferentes, a segunda de duas
maneiras (não podemos usar a cor da primeira casa), a terceira casa pode ser pintada de
duas maneiras (não podemos usar a cor da segunda casa), o mesmo ocorre com a quarta
casa. Assim, o total de maneiras de pintar o tabuleiro é 3 × 2 × 2 × 2 = 24.

Suponha que Carlos, Felipe, Marina e Ana estejam em uma fila. Se trocarmos a posição
de alguns deles dizemos que fizemos uma permutação. A pergunta é: Quantas permutações
podemos ter usando quatro pessoas? Antes de resolver o problema vamos introduzir uma
notação muito usada em problemas de contagem por simplificar algumas contas.

Notação. Dado um número natural n, seja n! (leia n fatorial) o produto 1· 2· 3 · · · (n − 1) · n.

Observe que o conseito de fatorial está fortemente ligado à noção de permutação. Para
fixar essa notação, vamos resolver alguns exercı́cios simples:

1. Calcule 4!, 5! e 6!
100! 47!
2. Calcule e
98! 44!3!
3. Resolva a equação (m + 2)! = 72 · m!

2
POT 2012 - Combinatória - Nı́vel 2 - Aula 4 - Prof. Bruno Holanda

4. Prove que
1 1 n
(a) − =
n! (n + 1)! (n + 1)!
(b) 2 · 4 · 6 · 8 · · · (2n) = 2n · n!
Problema 4. De quantas maneiras podemos formar uma fila com Carlos, Felipe, Marina e
Ana?

Solução. Podemos escolher o primeiro da fila de quatro maneiras, a segunda de três, a


terceira de duas e a última de apenas uma maneira (a pessoa que sobrar). Desse modo
temos 4 · 3 · 2 · 1 = 4! permutações.

Problema 5. (OBM 2005) Num relógio digital, as horas são exibidas por meio de quatro
algarismos. O relógio varia das 00:00 às 23:00 horas. Quantas vezes por dia os quatro
algarismos mostrados são todos pares?

Solução. Note que neste problema existe uma restrição nos dı́gitos que marcam as horas
e no primeiro dı́tido que marca os minutos. Dessa forma, em vez de pensar em cada dı́gito
separadamente, vamos pensar em três blocos de algarismos. O primeiro, que é formado
pelos dois primeiros algarismos, pode assumir 7 valores diferentes (00, 02, 04, 06, 08, 20 ou
22); o segundo é formado apenas pelo terceiro dı́gito e pode assumir 3 valores (0,2 ou 4); e
o último dı́gito pode assumir 5 valores (0,2,4,6 ou 8). Logo, o total de vezes em que todos
aparecem pares é 7 × 3 × 5 = 105.

Agora vamos nos preocupar com alguns problemas mais “clássicos”. Apesar de serem
problemas bem conhecidos por todos, vamos abordá-los aqui, pois empregam idéias que
são constantemente usadas em vários problemas.

Problema 6. (Quantidade de Subconjuntos) Quantos subconjuntos possui o conjunto M =


{1, 2, 3, ..., 10}?

Solução. A cada subconjunto de M vamos associar uma seqüência de 10 dı́gitos que podem
ser 0 ou 1. Essa associação será dada através da seguinte regra: O primeiro termo dessa
seqüência será 1 se o elemento 1 estiver no subconjunto e 0 caso contrário; O segundo termo
dessa seqüência será 1 se o elemento 2 estiver no subconjunto e 0 caso contrário; O terceiro
termo dessa seqüência será 1 se o elemento 3 estiver no subconjunto e 0 caso contrário; e
assim por diante.
Por exemplo, o subconjunto {1, 2, 5, 8, 10} está associado à seqüência 1100100101, o sub-
conjunto {2, 3, 5, 8} está associado à seqüência 0110100100, enquanto o subconjuto vazio
∅ é representado por 0000000000. Note que a quantidade de subconjuntos de M é igual à
quantidade destas seqüências. Por outro lado, podemos escolher cada dı́gito de duas formas
e, conseqüêntimente, temos 210 seqüências (que é a mesma quantidade de subconjuntos).

3
POT 2012 - Combinatória - Nı́vel 2 - Aula 4 - Prof. Bruno Holanda

Problema 7. (Quantidade de Divisores) Seja n = pα1 1 · pα2 2 · · · pαk k um número natural na


sua forma fatorada. Então, n possui exatamente

(α1 + 1)(α2 + 1) · · · (αk + 1)

divisores inteiros positivos. Incluindo 1 e n.

Solução. Note que cada divisor positivo de n é da forma n = pβ1 1 · pβ2 2 · · · pβk k , onde cada
expoente βi é um número entre 0 e αi (inclusive). Dessa forma, temos (α1 + 1) maneiras
de escolher o expoente de p1 ; (α2 + 1) maneiras de escolher o expoente de p2 ; assim por
diante. Logo, segue o resultado do princı́pio multiplicativo.

Problemas Propostos

Problema 8. Numa sala existem 3 homens e 4 mulheres. De quantos modos é possı́vel


selecionar um casal?

Problema 9. Cada casa de um tabuleiro 2 × 2 pode ser pintado de verde ou amarelo. De


quantas maneiras podemos pintar o tabuleiro todo?

Problema 10. (OBM 2004) De quantos modos diferentes podemos pintar (usando apenas
uma cor) as casas de um tabuleiro 4 × 4 de modo que cada linha e cada coluna possua
exatamente uma casa pintada?

Problema 11. Quantos números naturais de três algarismos distintos existem?

Problema 12. De quantos modos podemos por três torres de três cores diferentes em um
tabuleiro 8 × 8 de modo que nenhuma delas ataque outra?

Problema 13. Uma embarcação deve ser tripulada por oito homens, dois dos quais só
remam do lado direito e um apenas do lado esquerdo. Determine de quantos modos esta
tripulação pode ser formada, se de cada lado deve haver quatro homens.
Obs : A ordem dos homens deve ser considerada.

Problema 14. De quantas maneiras podemos ir de A até B sobre a seguinte grade sem
passar duas vezes pelo mesmo local e sem mover-se para esquerda? A figura abaixo mostra
um caminho possı́vel.

4
POT 2012 - Combinatória - Nı́vel 2 - Aula 4 - Prof. Bruno Holanda

Problema 15. Ache a quantidade númemos de quatro dı́gitos tais que toda seqüência de
três algarismos consecutivos é formada por elementos distintos.

Problema 16. (OBM 2005) Num tabuleiro quadrado 5 × 5, serão colocados três botões
idênticos, cada um no centro de uma casa, determinando um triângulo. De quantas ma-
neiras podemos colocar os botões formando um triângulo retângulo com catetos paralelos
às bordas do tabuleiro?

Problema 17. Dizemos que a palavra algoritmo é um anagrama da palavra logaritmo pois é
uma permutação da letras de logaritmo. Sabendo disso, calcule a quantidade de anagramas
da palavra vetor.

Problema 18. Quantos anagramas da palavra vetor termina em uma vogal?

Problema 19. De quantas maneiras é possı́vel colocar em uma prateleira 5 livros de ma-
temática, 3 de fı́sica e 2 de biologia, de modo que livros de um mesmo assunto permaneçam
juntos?

Problema 20. Quantos anagramas da palavra vetor possuem as vogais separadas?

Problema 21. De quantas formas podemos colocar 4 bolas verdes e 4 bolas amarelas em
um tabuleiro 4 × 4 de modo que cada coluna e cada linha possua exatamente uma bola de
cada cor.

Problema 22. Responda os itens a seguir:


a) Ache a quantidade de divisores positivos de 3600.
b) Quantos desses divisores são pares?
c) Quantos são quadrados perfeitos?

Problema 23. (Maio 2006) Um calendário digital exibe a data: dia, mês e ano, com 2
dı́gitos para o dia, 2 dı́gitos para o mês e 2 dı́gitos para o ano. Por exemplo, 01-01-01
corresponde a primeiro de janairo de 2001 e 25-05-23 corresponde a 25 de maio de 2023.
Em frente ao calendário há um espelho. Os dı́gitos do calendário são como os da figura
abaixo.

Se 0, 1, 2, 5 e 8 se reflentem, respectivamente, em 0, 1, 5, 2 e 8, e os outros dı́gitos perdem


sentido ao se refletirem, determine quantos dias do século, ao se refletirem no espelho,
correspondem também a uma data.

Problema 24. (Rússia) Um número natural n é dito elegante se pode ser escrito como soma
de cubo com um quadrado (n = a3 + b2 , onde a, b ∈ N). Entre 1 e 1000000 existem mais
números que são elegantes ou que não são?

Problema 25. Quantos são os números de cinco dı́gitos que são múltiplos de 3 e possuem
6 como um de seus dı́gitos?

5
POT 2012 - Combinatória - Nı́vel 2 - Aula 4 - Prof. Bruno Holanda

Dicas e Soluções

10. Em cada coluna devemos escolher exatamente uma casa para pintar. Temos 4 pos-
sibilidades de escolher a da primeira coluna, 3 para a segunda, 2 para a terceira, e 1
para última. Dessa forma, temos 4 · 3 · 2 · 1 = 24 maneiras de pintar o tabuleiro.

11. O primeiro algarismo pode ser escolhido de 9 modos (não podemos escolher o zero),
para o segundo temos 9 possibilidades (pois deve ser diferente do primeiro) e o terceiro
de 8 modos (deve ser diferente dos outros dois). Desse modo, a quantidade de números
é 9 · 9 · 8 = 648.

12. Temos 64 maneiras de escolher a posição da primeira torre, 49 para a segundo e 36


para a terceira. Total de maneiras é 64 · 49 · 36 = 112896.

13. 4 × 3 × 4 × 5! = 5760

14. A formiga deve ir para direita extamente 5 vezes. Ao escolhermos esses movimentos,
o resto do caminho estará bem definido. Como podemos escolher cada um destes
cinco movimentos de seis maneiras, o total de caminhos será 6 · 6 · 6 · 6 · 6 = 65 .

17. Considere os três blocos formados por livros da mesma matéria. Podemos organizar
esses blocos de 3! maneiras. Agora, em cada bloco ainda podemos permutar seus
livros. Assim, o número correto de maneiras é 5! · 3! · 2! · 3!.

18. A palavra vetor possui 5! = 120 anagramas. Usando a mesma idéia do problema 17
(separar em blocos), podemos achar que a quantidade destes anagramas com vogais
juntas é 2 × 4! = 48. Logo, temos 120 − 48 = 72 anagramas com as vogais separadas.

19. Existem 4! maneiras de colocar as bolas verdes. Depois disso, escolha uma das bolas
verdes. Ponha uma bola amerela na sua linha e uma na sua coluna. Note que, ao
fazermos isto, as posições das outras duas bolas amarelas estará bem definida. Dessa
maneira, temos um total de 4! · 3 · 3 = 216 configurações.

21. Como não podemos usar os dı́gitos 3, 4, 6, 7, 9 para formar uma data, os únicos valores
possı́veis para os dois primeiros dı́gitos (os que marcam o dia) são: 01, 02, 05, 08, 10,
11, 12, 15, 18, 20, 21, 22, 25, 28. Para os dois próximos dı́gitos temos as seguintes
possibilidades: 01, 02, 05, 08, 10, 11, 12. Por outro lado, apenas os pares 01, 10 e
11 também correspondem a um mês quando são refletidos. Para os dois últimos as
possibilidades são: 10, 20, 50, 80, 01, 11, 21, 51, 81, 02, 12, 22, 52, 82. Pois seus
reflexos devem corresponder a um dia. Logo, o total de datas pedidas é 14 × 3 × 14 =
588.

6
Polos Olímpicos de Treinamento
Curso de Combinatória - Nível 2 Aula 5
Prof. Bruno Holanda

Contagem II

Neste material vamos aprender novas técnicas relacionadas a problemas de contagem.

1. Separando em casos

Quando encontramos dificuldades em resolver um problema, uma estratégia útil é se-


pará-lo em casos menores em que essas dificuldades diminuam. Essa ideia é tão significativa
que os especialistas da ciência da computação nomearam-na de divide and conquer algo-
rithm, em analogia às estratégias polı́tico-militares.

Problema 1. O alfabeto da Tanzunlândia é formado por apenas três letras: A, B e C.


Uma palavra na Tanzunlândia é uma seqüência com no máximo 4 letras. Quantas palavras
existem neste paı́s?

Solução. Existem 3 palavras com uma letra, 32 com duas letras, 33 com três letras, e 34
com quatro letras. Logo, o total de palavras é 3 + 32 + 33 + 34 = 120.

Problema 2. De quantos modos podemos pintar (usando uma de quatro cores) as casas da
figura a baixo de modo que as casas vizinhas tenham cores diferentes?

1 2

4 3

Solução. Vamos separar o problema em dois casos:

i. Se as casas 1 e 3 tiverem a mesma cor, temos quatro maneiras de escolher essa cor.
Podemos escolher a cor da casa 2 de três maneiras (basta não ser a cor usadas nas
casas 1 e 3), o mesmo vale para casa 4. Logo, temos 4 × 3 × 3 = 36 maneiras de pintar
dessa forma.
POT 2012 - Combinatória - Nı́vel 2 - Aula 5 - Prof. Bruno Holanda

ii. Agora se 1 e 3 têm cores diferentes, podemos escolher a cor da casa 1 de quatro
maneiras, da casa 3 de três maneiras e, das casas 2 e 4, podemos escolher de duas
maneiras cada. Assim, temos 4 × 3 × 2 × 2 = 48 maneiras de pintar desta outra forma.
Desse modo, podemos concluir que existem 36 + 48 = 84 maneiras de pintar a rosquinha.

Problema 3. Quantos são os números de quatro dı́gitos que não possuem dois algarismos
consecutivos com a mesma paridade?

Solução. Vamos separar o problema em dois casos:


i. Quando o primeiro algarismo for par, temos 4 possibilidades para o primeiro dı́gito, 5
para o segundo, 5 para o terceiro e 5 para o último. Totalizando 4 × 5 × 5 × 5 = 500
números.
ii. Quando o primeiro algarismo for ı́mpar, temos 5 possibilidades para cada um dos
dı́gitos. Logo, a quantidade de números dessa forma é 5 × 5 × 5 × 5 = 625.
Portanto, temos um total de 625 + 500 = 1125 números de quatro dı́gitos que não possuem
dois algarismos consecutivos com a mesma paridade.

2. Contagens Múltiplas
Os problemas que abordamos até agora tinham algo em comum: o papel da ordenação
na diferenciação das possibilidades. Porém, há casos em que a ordem dos elementos não é
relevante para a contagem. Isso fica claro quando analisamos as seguintes situações:

Situação 1. De um grupo de 7 pessoas, devemos escolher 3 delas para formar um


pódio (primeiro, segundo e terceiro lugares). De quantas formas podemos fazer isso?

Situação 2. De um grupo de 7 pessoas, devemos escolher 3 delas para formar um


comitê (sem hierarquias). De quantas formas podemos fazer isso?

Perceba que, apesar de serem semelhantes, são problemas diferentes, com respostas
também diferentes. O primeiro sabemos resolver. A resposta é 7 × 6 × 5 = 210. Agora,
sabendo a essa resposta podemos dar uma solução para o segundo problema.

Note que, para cada comitê formado, podemos montar 3 × 2 × 1 = 6 pódios distintos.
Logo, o número de pódios é seis vezes o número de comitês. Portanto, a resposta para o
210
segundo problema é = 35.
6
Podemos usar essa estratégia para resolver problemas de anagramas em que as palavras
possuem letras repetidas.

2
POT 2012 - Combinatória - Nı́vel 2 - Aula 5 - Prof. Bruno Holanda

Problema 4. Quantos anagramas possui a palavra matematica (desconsidere o acento)?

Solução. Se imaginarmos por um momento uma palavra de 10 letras diferentes:

m1 a1 t1 em2 a2 t2 ica3 ,

o número total de anagramas será 10!. Porém, ao trocarmos letras que na realidade são
iguais (como a1 e a3 ) o anagrama continua o mesmo. Dessa forma, cada anagrama foi
10!
contado 2 × 2 × 3! vezes. Portanto, a resposta é .
2 × 2 × 3!

Problema 5. De quantas formas podemos por oito pessoas em uma fila se Alice e Bob
devem estar juntos, e Carol deve estar em algum lugar atrás de Daniel?

Solução. Vamos imaginar Alice e Bob como uma única pessoa. Existirão 7! = 5040 pos-
sibilidades. Alice pode estar na frente de Bob ou vice versa. Então devemos multiplicar o
número de possibilidades por 2. Por outro lado, Carol está atrás de Daniel em exatamente
metade dessas permutações, então a resposta é apenas 5040.

Problemas Propostos

Problema 6. Escrevem-se todos os inteiros de 1 a 9999. Quantos números têm pelo menos
um zero?

Problema 7. Quantos números de três dı́gitos possuem todos os seus algarismos com a
mesma paridade?

Problema 8. Quantos são os números de quatro algarismos que possui pelo menos um
dı́gito repetido?

Problema 9. Quantos são os números de quatro dı́gitos distintos que não possuem dois
algarismos consecutivos com a mesma paridade?

Problema 10. De quantas maneiras podemos colocar um rei preto e um rei branco em um
tabuleiro de xadrez (8 × 8) sem que nenhum deles ataque o outro?

Problema 11. Quantos são os naturais pares que se escrevem com três algarismos distintos?

Problema 12. Na cidade Gótica as placas das motos consistem de três letras. A primeira
letra deve estar no conjunto {C, H, L, P, R}, a segunda letra no conjunto {A, I, O}, e a
terceira letra no conjunto {D, M, N, T }. Certo dia, decidiu-se aumentar o número de
placas usando duas novas letras J e K. O intendente dos transportes ordenou que as novas
letras fossem postas em conjuntos diferentes. Determine com qual opção podemos obter o
maior número de placas.

3
POT 2012 - Combinatória - Nı́vel 2 - Aula 5 - Prof. Bruno Holanda

Problema 13. (Maio 1998) Cada um dos seis segmentos da figura abaixo deve ser pintado
de uma de quatro cores de modo que segmentos vizinhos não tenham a mesma cor. De
quantas maneiras podemos fazer isso?

Problema 14. Em uma festa havia 6 homens e 4 mulheres. De quantos modos podemos
formar 3 pares como essas pessoas?

Problema 15. De quantas maneiras podemos por três torres de mesma cor em um tabuleiro
8 × 8 de modo que nenhuma delas ataque a outra?

Problema 16. (AIME 1996) Duas casas de um tabuleiro 7 × 7 são pintadas de amarelo e as
outras são pintadas de verde. Duas pinturas são ditas equivalentes se uma é obtida a partir
de uma rotação aplicada no plano do tabuleiro. Quantas pinturas inequivalentes existem?

Problema 17. Em uma sala de aula existem a meninas e b meninos. De quantas formas
eles podem ficar em uma fila, se as meninas devem ficar em ordem crescente de peso, e os
meninos também? (Suponha que 2 pessoas quaisquer não tenham o mesmo peso.)

Problema 18. Considere um torneio de xadrez com 10 participantes. Na primeira rodada


cada participante joga somente uma vez, de modo que há 5 jogos realizados simultanea-
mente. De quantas maneiras esta primeira rodada pode ser realizada?

Problema 19. Doze cavaleiros estão sentados em torno de uma mesa redonda. Cada um
dos 12 cavaleiros considera seus dois vizinhos como rivais. Deseja-se formar um grupo de
5 cavaleiros para salvar uma princesa. Nesse grupo não poderá haver cavaleiros rivais.
Determine de quantas maneiras é possı́vel escolher esse grupo.

4
POT 2012 - Combinatória - Nı́vel 2 - Aula 5 - Prof. Bruno Holanda

Dicas e Soluções

6. Ache a quantidade de números de 0 a 9999 sem nenhum dı́gito zero. Faça essa
contagem separando em quatro casos (de acordo com a quantidade de algarismos).

7. Separe em dois casos: 1) quando todos os dı́gitos são pares; 2) quando todos os dı́gitos
são ı́mpares. Não se esqueça que zero não pode ser o primeiro dı́gito!

10. Podemos dividir o tabuleiro em três regiões: A primeira é formada pelas quatro casas
nos cantos do tabuleiro; a segunda pelas 24 casas da borda (que não estão nos cantos);
e a terceira pelo tabuleiro 6 × 6 no interior do tabuleiro. Se o primeiro rei for posto
na primeira região, temos 60 maneiras de colocar o segundo rei; se ele for posto na
segunda, temos 58 maneiras; e se for posto na terceira, temos 55 maneiras. Logo,
temos um total de 4 × 60 + 24 × 58 + 36 × 55 = 3612 modos diferentes de colocar os
dois reis.

12. Inicialmente temos 5·3·4 = 60 placas. De acordo com o problema, temos as seguintes
opções para o novo número de placas: 6 · 4 · 4 = 96, 5 · 4 · 5 = 100 e 6 · 3 · 5 = 90.
Logo, o número máximo é 100.
(6 × 5 × 4) × (4 × 3 × 2)
14. .
3!
64 × 49 × 36
15. .
3!
16. Separe o problema em dois casos. Quando as casas amarelas são simetricas em relação
ao centro do tabuleiro e quando não são. Conte o número de pinturas equivalentes
em casa caso.

17. Temos (a + b)! maneiras de permutar todas as crianças. Porém apenas uma das a!
permutações das meninas está na ordem correta e apenas b! das permutações dos
(a + b)!
meninos está correta. Logo, a resposta é .
a!b!

5
Polos Olímpicos de Treinamento
Curso de Combinatória - Nível 2 Aula 6
Prof. Bruno Holanda

Jogos

Quando falamos em jogos, pensamos em vários conhecidos como: xadrez, as damas e os


jogos com baralho. Porém, não são desses jogos que iremos falar neste material. Imagine
que exista algum tipo de jogo em que você pudesse ganhar sempre, independente de como
seu adversário jogasse? Seria uma boa, não?! Pois esses jogos existem e são um dos
assuntos mais abordados em provas de olimpı́ada. Nesta aula vamos mostrar vários destes
jogos e as principais estratégias vencedoras: a simetria e o uso das posições vencedoras.

1. Simetria

Uma das estratégias mais simples é o uso de alguma simetria que pode ocorrer durante
o jogo em vantagem de um dos jogadores, forçando sempre uma nova rodada para o jogador
“destinado à derrota”. Para entender melhor veja o seguinte exemplo:

Problema 1. Pedro e Mônica jogam em um tabuleiro 1 × 11. Cada um, em sua vez, pode
pintar um dos quadrados (que não foram pintados anteriormente), ou dois quadrados con-
secutivos (se ambos estiverem brancos). Quem não puder mais jogar perde. Sabe-se que
Pedro será o primeiro a jogar. Quem pode sempre garantir a vitória?

Solução. Pedro sempre poderá ganhar se seguir a seguinte estratégia:

(i) Inicialmente, Pedro deve pintar o quadrado do meio.

(ii) Agora, depois que Mônica fizer sua jogada, Pedro deve jogar sempre simetricamente
em relação ao centro do tabuleiro (i.e. sempre deixando o tabuleiro simétrico). Por
exemplo, se Mônica jogar nas casas 9 e 10, Pedro deve jogar nas casas 2 e 3.

× × × z z
POT 2012 - Combinatória - Nı́vel 2 - Aula 6 - Prof. Bruno Holanda

(iii) Assim, Mônica nunca poderá ganhar, pois na sua jogada ela “quebra a simetria” e a
configuração final do jogo todas as casas estarão pintadas, ou seja, a configuração é
simétrica.

O próximo exemplo é um dos problemas que apareceu na prova da OBM de 2004. Vamos
apresentar uma solução diferente da solução proposta na Eureka! 22, usando simetria:

Problema 2. Arnaldo e Bernardo disputam um jogo em um tabuleiro 2 × n :

As peças do jogo são dominós 2 × 1. Inicialmente Arnaldo coloca um dominó cobrindo


exatamente duas casas do tabuleiro, na horizontal ou na vertical. Os jogadores se revezam
colocando uma peça no tabuleiro, na horizontal ou na vertical, sempre cobrindo exata-
mente duas casas do tabuleiro. Não é permitido colocar uma peça sobre outra já colocada
anteriormente. Quem não conseguir colocar uma peça no tabuleiro perde.

Qual dos dois jogadores tem uma estratégia vencedora, ou seja, uma estratégia que o
leva à vitória quaisquer que sejam as jogadas de seu adversário, para:
a) n = 2004?
b) n = 2005?

Solução. Quando n = 2005 o primeiro jogador garante a vitória. Ele pode fazer isto colo-
cando um dominó na vertical no meio do tabuleiro e, em seguida, jogar simetricamente ao
segundo jogador. Quando n = 2004 o tabuleiro possui um número par de colunas. Desse
modo, o segundo ganha jogando simetricamente ao primeiro jogador.

Como você deve ter visto, usar a simetria é realmente uma técnica muito eficiente.
Porém, às vezes, usar apenas a simetria não é suficiente para resolver o problema. Observe
o próximo exemplo retirado da olı́mpiada da Bielorússia de 2000.

Problema 3. Tom e Jerry jogam o seguinte jogo. Eles colocam alternadamente pinos
idênticos em casas vazias de um tabuleiro 20 × 20 (um pino de cada vez). Tom é o primeiro
a jogar. Vence quem, em sua jogada, formar um bloco de quatro pinos vizinhos. Dois pinos
são vizinhos se estiverem em casas com um lado em comum. Determine quem possui a
estratégia vencedora.

Solução. Jerry deve jogar simetricamente em relação ao centro do tabuleiro. Assim


que Tom formar três um bloco de três pinos vizinhos, Jerry deve abandonar a estratégia
simétrica e completar o bloco de quatro pinos vizinhos.

2
POT 2012 - Combinatória - Nı́vel 2 - Aula 6 - Prof. Bruno Holanda

2. Posições Vencedoras

Alguns tipos de jogos possuem certas configurações que sempre levam um jogador à
vitória. Essas configurações são chamadas de posições vencedoras. O próximo exemplo é
um jogo bastante simples em que essa estratégia aparece facilmente.

Problema 4. Na primeira casa de um tabuleiro 1 × 13 está uma moeda. Tiago e Maria


movem a moeda alternadamente. Em cada turno é permitido avançar 1, 2, 3, 4 ou 5 casas.
Quem colocar a moeda na última casa é o vencedor. Se Maria começar jogando, ela pode
ter certeza da vitória?

Solução. Como em muitos problemas de olimpı́ada, vamos analisar alguns casos pequenos.
Vamos supor que em vez de 13 casas o tabuleiro tivesse apenas quatro. Neste caso, fica
fácil ver que quem começa ganha basta avançar três casas.
O mesmo iria ocorrer se o tabuleiro tivesse 2, 3, 4, 5 ou 6 casas. Porém, em um tabuleiro
7 × 1 o primeiro jogador perde. Veja que após a primeira jogada a moeda estará em uma
das casas 2, 3, 4, 5 ou 6. E já sabemos que essas casas levam o jogador à vitória.

Desse modo, vamos dizer que 7 é uma posição perdedora e 6, 5, 4, 3 e 2 são posições
vencedoras. Assim, se um o jogador estiver em uma das casas 8, 9, 10, 11 ou 12, ele pode
garantir a vitória movendo a moeda para a casa 7, deixando o seu adversário em uma
posição perdedora. Com isso, podemos afirmar que as posições 8, 9, 10, 11 e 12 também são
posições vencedoras.
Resta analisar a 13a casa. Observe que a partir desta casa podemos mover a moeda
apenas para uma das casas 8, 9, 10, 11 ou 12 que são vencedoras. Daı́, quem começar perde
pelo simples fato de iniciar em uma posição perdedora.

A grande dificuldade para a maioria dos alunos é descobrir quais são as posições vence-
doras de um jogo. Para evitar esse tipo de problema, tenha sempre em mente as seguintes
definições:

(a) Posição vencedora: A partir dela, podemos escolher um movimento


e repassar uma posição perdedora para o adversário.

(b) Posição perdedora: A partir dela, é impossı́vel escolher um mo-


vimento e repassar uma posição perdedora para o adversário. Ou
seja, não importa o movimento escolhido, o adversário irá receber uma
posição vencedora.

3
POT 2012 - Combinatória - Nı́vel 2 - Aula 6 - Prof. Bruno Holanda

E como fazer para descobrir quais são as posições vencedoras e perdedoras? A melhor
maneira de se fazer isto é analisando o final do jogo e aplicar as definições acima. Vamos
praticar um pouco resolvendo o próximo problema.

Problema 5. Em um tabuleiro 8 × 8, uma torre está na casa a1. Dois jogadores movem a
torre com objetivo de colocar a torre na casa h8. Sabendo que a torre pode mover-se apenas
para cima ou para direita (quantas casas o jogador desejar) e que não pode-se passar a vez,
determine qual jogador tem a estratégia vencedora.

Solução. Primeiramente note que todas as casas da última linha e da última coluna (exceto
a h8) são vencedoras pois, a partir delas podemos escolher um movimento que nos leve à
vitória. Com, isso a casa g7 se torna perdedora pois, a partir dela qualquer movimento leva
o outro jogador a uma posição vencedora (veja a figura 1).

Figura 1 Figura 2 Figura 3

Agora, como g7 é perdedora, as demais casas da sétima linha e da sétima coluna são
vencedoras. Mais ainda, a casa f 6 também deve ser perdedora (figura 2). Continuando
de maneira análoga, obtemos que a casa a1 é perdedora (figura 3). Logo, quem começar,
perde.

Problemas Propostos

Problema 6. Sobre uma mesa existem duas pilhas (uma com 15 e outra com 16 pedras). Em
um jogo cada jogador pode, em sua vez, retirar qualquer quantidade de pedras de apenas
uma pilha. Quem não puder mais jogar perde. Quem possui a estratégia vencedora?

Problema 7. Dois jogadores colocam alternadamente bispos (da mesma cor) em um ta-
buleiro 8 × 8, de forma que nenhum bispo ataque outro. Quem não puder mais jogar
perde.

Problema 8. Dois jogadores colocam alternadamente reis (da mesma cor) em um tabuleiro
9 × 9, de forma que nenhum rei ataque outro. Quem não puder mais jogar perde.

Problema 9. São dados um tabuleiro de xadrez (8 × 8) e palitinhos do tamanho dos lados


das casas do tabuleiro. Dois jogadores jogam alternadamente e, em cada rodada, um dos

4
POT 2012 - Combinatória - Nı́vel 2 - Aula 6 - Prof. Bruno Holanda

jogadores coloca um palitinho sobre um lado de uma das casas do tabuleiro, sendo proibido
sobrepor os palitinhos.
Vence o jogador que conseguir completar primeiro um quadrado 1 × 1 de palitinhos. Su-
pondo que nenhum dos jogadores cometa erros, qual dos dois tem a estratégia vencedora?

Problema 10. São dados vinte pontos ao redor de um cı́rculo. Cada jogador em sua vez
pode ligar dois desses pontos se essa novo segmento não cortar os feitos anteriormente.
Quem não puder mais traçar nenhum segmento perde.

Problema 11. Dois jogadores colocam alternadamente x’s e o’s em um tabuleiro 9 × 9. O


primeiro escreve x’s e o segundo o’s. Quando o tabuleiro for completamente preenchido o
jogo termina e os pontos são contados. Um ponto é dado ao jogador para cada linha ou
coluna em que ele possuir mais casas dos que o adversário. O jogador que possuir mais
pontos vence. Quem pode sempre ganhar?

Problema 12. Um pino está no centro de um tabuleiro 11 × 11. Dois jogadores movem
alternadamente o pino para qualquer outra casa do tabuleiro, mas a cada movimento (a
partir do segundo) deve ser maior que o anterior. O jogador que não puder mais jogar
perde. Ache a estratégia vencedora.

Problema 13. Um jogo consiste em quebrar um tabuleiro 5 × 10 ao longo de suas linhas.


Ganha o primeiro jogador que obter um quadrado 1 × 1. Quem tem a estratégia vencedora?

Problema 14. (Rússia 1997) Os números 1, 2, 3, ..., 1000 são escritos no quadro. Dois joga-
dores apagam alternadamente um dos números da lista até que só restem dois números. Se
a soma desses números for divisı́vel por 3, o primeiro jogador vence, caso contrário vence o
segundo. Quem tem a estratégia vencedora?

Problema 15. Sobre uma mesa existem duas pilhas de moedas com 11 moedas cada. Em
cada turno, um jogador pode retirar duas moedas de uma das pilhas ou retirar uma moeda
de cada pilha. O jogador que não puder mais fazer movimentos perde.

Problema 16. Tom e Jerry jogam um jogo e Tom faz a primeiro passo. Em cada turno o
jogador pode diminuir de um dado natural N um dos seus dı́gitos não-nulos. Inicialmente
o número N é 1234. O jogador que obter zero ganha. Quem pode garantir a vitória?

Problema 17. Uma pilha de 500 pedras é dada. Dois jogadores jogam o seguinte jogo: Em
cada turno, o jogador pode retirar 1, 2, 4, 8, ... (qualquer potência de 2) pedras da pilha. O
jogador que não puder mais jogar perde.

Problema 18. Em uma caixa existem 300 bolinhas. Cada jogador pode retirar não mais do
que a metade das bolinhas que estão na caixa. O jogador que não puder mais jogar perde.

Problema 19. Sobre uma mesa existem duas pilhas (uma com 7 e outra com 15 pedras).
Em um jogo cada jogador pode, em sua vez, retirar qualquer quantidade de pedras de
apenas uma pilha ou a mesma quantidade de ambas as pinhas. Quem não puder mais
jogar perde. Quem possui a estratégia vencedora?

5
POT 2012 - Combinatória - Nı́vel 2 - Aula 6 - Prof. Bruno Holanda

Dicas e Soluções

6. O jogador 1 deve retirar uma pedra da pilha com 16. Em seguida, deve jogar sime-
tricamente em relação ao jogador 2.

7. Divida o tabuleiro em duas partes, cada uma formada por 4 linhas. O jogador 1 deve
jogar então simetricamente.

8. O primeiro jogador deve colocar um rei no centro, e depois jogar simetricamente em


relação ao centro do tabuleiro.

14. Observe que a soma de dois elementos opostos sempre é 1002, que é um múltiplo de
3.

15. Construa um tabuleiro 11 × 11, onde a casa (i, j) represente quantidade de pedras em
cada pilha. Observe que o movimento do jogo original é equivalente ao movimento
do cavalo no tabuleiro. Termine o problema descobrindo as posições vencedoras e
perdedoras através de indução retroativa.

17. Pense nos múltiplos de 3. Nenhuma potência de 2 é múltiplo de 3.

18. Pense nas potências de 2.

19. Novamente, use a idéia do tabuleiro que foi usada para resolver o problema 15.

6
Programa Olímpico de Treinamento
Curso de Combinatória - Nível 2 Aula 7
Prof. Bruno Holanda

Princı́pio da Casa dos Pombos I

O princı́pio da casa dos pombos também é conhecido em alguns paı́ses (na Rússia, por
exemplo) como Princı́pio de Dirichlet pois, foi o matemático Lejeune Dirichlet o primeiro
matemático a usa este método para resolver problemas não triviais. Outros matemáticos
que se destacaram por usarem essa idéia para resolver diversos problemas foram os húngaros
Erdős e Szekeres. Vamos abordar este princı́pio da seguinte maneira:

“Se em n caixas são postos n + 1 pombos, então pelo


menos uma caixa terá mais de um pombo.”

Alguns Exemplos:

i. Em um grupo de três pessoas, pelo menos duas delas são do mesmo sexo.

ii. Em um grupo de 13 pessoas, pelo menos duas delas têm o mesmo signo.

iii. Em um grupo de 5 cartas de baralho, pelo menos duas são do mesmo naipe.

iv. Na cidade de Fortaleza, existem pelo menos duas pessoas com o mesmo número de fios
de cabelo.

Agora vamos ver como algo tão simples pode resolver problemas aparentemente difı́ceis:

Problema 1. Escolhem-se 5 pontos ao acaso sobre a superfı́cie de um quadrado de lado √


2. Mostre que pelo menos dois deste pontos estão em um distância menor que ou igual a 2.

Solução. Divida o quadrado em quatro quadrados menores como na figura ao lado. Como
temos cinco pontos e quatro quadrados, teremos pelo menos dois pontos no mesmo qua-
dradinho. Como a maior distância entre dois pontos do mesmo quadradinho não supera a
medida de sua diagonal, o resultado segue de imediato.
POT 2012 - Combinatória - Nı́vel 2 - Aula 7 - Prof. Bruno Holanda


2
Passo de Mágica?

Para o aluno iniciante a solução do problema anterior pode ter parecida um pouco mágica.
Vamos mostrar que não é bem assim, que existe um método na solução de alguns problemas
simples que usam a idéia da casa dos pombos.

A primeira coisa que devemos aprender a reconhecer é quando um problema se trata


de um problema sobre casa dos pombos. Isso pode ser ganho com experiencia, mas vamos
dar um empurãozinho para você. Um problema de PCP tem quase sempre a seguinte cara:

• Dado um conjunto de n objetos, prove que podemos escolher k deles satisfazendo


uma propriedade.

Bem, depois de identificar que o enunciado do problema no traz a idéia de usar PCP,
devemos nos concentrar em responder as seguintes perguntas:

(i) Quem são os pombos?

(ii) Quantas são as casas?

(iii) Quem são as casas?

Quase sempre as duas primeiras perguntas são as mais fáceis de serem respondidas.
Para responder a terceira pergunta devemos pensar no conceito dual de espaço amostral.
Por um lado, o espaço amostral é o conjunto das possı́veis posições dos pombos. Por outro,
é a união de todas as casas.

Para finalizar, devemos separar o espaço amostral no número de casas já descoberto.
Nessa hora é importante lembrar que as casas devem fletir a propriedade desejada.
Como acabamos de ver, usar o princı́pio da casas dos pombos não é difı́cil. O difı́cil
está em achar o que serão nossos “pombos” e “caixas”. O próximo problema é, a priori,
um problema de teoria dos números. Porém, vamos usar o princı́pio da casa dos pombos
para resolvê-lo.

2
POT 2012 - Combinatória - Nı́vel 2 - Aula 7 - Prof. Bruno Holanda

Problema 2. Prove que dados sete inteiros positivos, existem dois cuja soma ou a diferença
é um múltiplo de 10.

Solução. Vamos montar seis caixas C0 , C2 , ..., C5 onde um inteiro está na caixa Ci se é
congruente a i ou a −i módulo 10. Sabemos que existirão dois inteiros na mesma caixa.
Dessa forma, se eles forem incongruentes módulo 10, basta somá-los. Caso contrário, faça
a sua diferença.

Problema 3. Dados 5 pontos no plano com coordenadas inteiras, prove que pelo menos um
dos dez pontos médio gerados por eles também possui coordenadas inteiras.

Solução. Podemos separar os pontos de coordenadas inteiras (que é representado por Z×Z)
em quatro grupos G1 , G2 , G3 , G4 como a seguir.

i) G1 = {(x, y) ∈ Z × Z|x, y são ambos pares}.

ii) G2 = {(x, y) ∈ Z × Z|x, y são ambos ı́mpares}.

iii) G3 = {(x, y) ∈ Z × Z|x é par e y é ı́mpar}.

iv) G4 = {(x, y) ∈ Z × Z|x é ı́mpar e y é par}.

Observe que pontos que pertencem ao mesmo grupo, possuem pontos médios com coorde-
nadas inteiras. Como temos 5 pontos, o princı́pio da casa dos pombos nos garante que há
pelo menos dois pontos no mesmo grupo.

Problema 4. Nove pontos são postos sobre a superfı́cie de um tetraedro regular com 1cm
de aresta. Prove que detre esses pontos é possı́vel acahr dois com distância (espacial) não
maior que 0.5cm.

Solução. Vamos particionar a superfı́cie do tetraedro em 16 triângulos eqüiláteros con-


gruentes, dividindo cada face em quatro partes usando suas bases médias. Agora vamos
criar 8 regiões pitando esses triângulos de acordo com a seguinte regra: os triângulos que
possuem um mesmo vértice do tetraedro serão pintados da mesma cor; dessa forma já usa-
mos quatro cores diferentes para 12 triângulos e os outros quatro vamos pintar usando as
demais cores. De acordo com o Princı́pio da Casa dos Pombos, pelo menos dois dos nove
pontos estarão na mesma região. Fica apenas faltando que a distância máxima entre dois
pontos da mesma região é no máximo 0.5cm.

3
POT 2012 - Combinatória - Nı́vel 2 - Aula 7 - Prof. Bruno Holanda

Problemas Propostos

Problema 5. Cinquenta e um pontos são postos no interior de um quadrado de lado 1


metro. Prove que existe um conjunto de três desses pontos podem ser cobertos por um
quadrado de lado 20 centı́metros.

Problema 6. Em cada casa de um tabuleiro 3 × 3 é colocado um dos números −1, 0, 1.


Prove que, dentre as oito somas ao longo de uma mesma linha, coluna ou diagonal, existem
duas iguais.

Problema 7. Prove que de qualquer conjunto de dez inteiros podemos escolher um subcon-
junto cuja soma é um múltiplo de 10.

Problema 8. Prove que existe uma potência de 3 terminada nos dı́gitos 001 (na base
decimal).

Problema 9. Mostre que um triângulo eqüilátero não pode ser totalmente coberto por
outros dois triângulos eqüiláteros menores.

Problema 10. (Longlist IMO 1977 - Romênia) Dados 37 pontos no espaço com coordenadas
inteiras, prove que pelo menos um dos triângulos formado por três destes pontos possui o
baricentro com coordenadas inteiras.

Problema 11. (Bielorussia 1996) Em um grupo de 29 hobbits existem alguns deles que
falam a verdade e os outros que sempre mentem. Em um certo dia de primavera, todos
eles se sentaram ao redor de uma mesa, e cada um deles falou que seus dois vizinhos eram
mentirosos.
a) Prove que pelo menos 10 hobbtis falavam a verdade.
b) É possı́vel que exatamente 10 deles falem a verdade?

Problema 12. Em cada casa de um tabuleiro 10 × 10 é posto um inteiro de modo que a


diferença positiva entre os inteiros de duas casas vizinhas (lado em comum) é no máximo
5. Prove que dois destes inteiros devem ser iguais.

4
POT 2012 - Combinatória - Nı́vel 2 - Aula 7 - Prof. Bruno Holanda

Problema 13. Trinta e três torres são postas em um tabuleiro 8 × 8. Prove que podemos
escolher cinco delas sem que nenhuma ataque a outra.

Problema 14. (Longlist IMO 1979 - Bulgária) Colocamos 4n + 1 reis em um tabuleiro


infinito. Prove que podemos escolher n + 1 deles de modo que não existam dois que se
ataquem.

Problema 15. Prove que de qualquer subconjunto de n+1 elementos do conjunto {1, 2, ..., 2n}
é possı́vel escolher dois que sejam primos entre si.

Problema 16. (IMO 1972) Prove que, de qualquer conjunto de dez números distintos de dois
dı́gitos, podemos escolher dois subconjuntos A e B (disjuntos) cuja a soma dos elementos
é a mesma em ambos.

Problema 17. Quarenta estudantes participaram de uma olimpı́ada de matemática. A


prova consistia de cinco problemas ao todo. Sabe-se que cada problema foi resolvido cor-
retamente por pelo menos 23 participantes. Prove que deve existir dois participantes tais
que todo problema foi resolvido por pelo menos um deles dois.

Problema 18. Prove que em qualquer grupo de 17 números escolhidos do conjunto

M = {1, 2, 3, ..., 24, 25}

é possı́vel escolher dois cujo produto é um quadrado perfeito.

5
POT 2012 - Combinatória - Nı́vel 2 - Aula 7 - Prof. Bruno Holanda

Dicas e Soluções

5. Imite aa solução do problema 1.

6. A soma de três números varia no conjunto {−3, −2, −1, 0, 1, 2, 3} como são 8 somas,
pelo menos uma será usada mais de uma vez.

7. Se a1 , a2 , ..., a10 são os números, considere as somas

S1 = a1

S2 = a1 + a2
S3 = a1 + a2 + a3
..
.
S10 = a1 + a2 + · · · + a10 .
Se uma delas for um múltiplo de 10, teremos encontrado a solução do problema. Caso
contrário, como há 9 restos possı́veis (dintintos de zero) na divisão por 10, pelo PCP,
existirão duas destas somas que serão congruentes módulo 10. Se Si ≡ Sj (mod 10),
então
Si − Sj ≡ ai + ai+1 + · · · + aj ≡ 0 (mod 10).
Isso conclui a solução.

8. Use PCP para demonstrar que existem duas potências de 3 com o mesmo resto na
divisão por 1000.

9. Observe o que acontece nos vértices do triângulo maior.

10. Adapte a solução do problema 3

13. Pinte o tabuleiro usando 8 cores como no diagrama a seguir

1 2 3 4 5 6 7 8
2 3 4 5 6 7 8 1
3 4 5 6 7 8 1 2
4 5 6 7 8 1 2 3
5 6 7 8 1 2 3 4
6 7 8 1 2 3 4 5
7 8 1 2 3 4 5 6
8 1 2 3 4 5 6 7

Pelo PCP existirão pelo menos 5 torres em casas de mesma cor. Observe que torres
em casas de mesma cor não se atacam.

6
POT 2012 - Combinatória - Nı́vel 2 - Aula 7 - Prof. Bruno Holanda

14. Pinte o tabuleiro usando 4 cores como no diagrama a seguir

1 2 3 4 1 2 3 4
2 3 4 1 2 3 4 1
3 4 1 2 3 4 1 2
4 1 2 3 4 1 2 3

Repita o argumento anterior.

15. Separe o conjunto em n pares de elementos consecutivos.

7
Programa Olímpico de Treinamento
Curso de Combinatória - Nível 2 Aula 8
Prof. Bruno Holanda

Princı́pio da Casa dos Pombos II

Nesta aula vamos continuar praticando as ideias da aula anterior, aplicando o princı́pio
da casa dos pontos em problemas mais sofisticados e em alguns tipos de problemas que
chamaremos de problemas de coloração.

Problema 1. Cada casa de um tabuleiro 3 × 7 é pintado de preto ou branco. Mostre que


é possı́vel achar um retângulo (com lados paralelos aos do tabuleiro) cujas quatro pontos
são da mesma cor.

Solução. Cada coluna deste tabuleiro pode ser pintado de uma das seguintes formas:

1 2 3 4 5 6 7 8

Observe que se a pintura 1 for escolhida, bastaria uma coluna do tipo 2, 3 ou 4 para
formar um retângulo. Com isso, nos restariam apenas mais quatro outras pinturas porém,
temos sete colunas. Daı́, pelo principio da casa dos pombos terı́amos duas colunas iguais.
O mesmo ocorre com a coluna do tipo 8.
Agora suponha que nenhuma das colunas for do tipo 1 ou 8. Dessa forma, restaria
apenas 6 tipos de pinturas. Assim, pelo princı́pio da casas dos pombos, duas delas seriam
iguais.

Problema 2. (Belarus 2007 - adaptado) Os pontos de um plano são pointados usando três
cores. Prove que existe um triângulo isósceles monocromático.

Solução. Suponha que exista uma forma de pintar o plano de forma que não exista um
triângulo isóceles monocromático. Assuma que as cores sejam verde, azul e vermelho.
Construa um suponha sem perda de generalidade que o seu centro O seja verde. Dessa
forma, pode haver no máximo um único ponto verde dentre os pontos dos cı́rculo. Assim
é possı́vel construir um pentágono regular A1 A2 A3 A4 A5 cujos vértices são todos azuis ou
POT 2012 - Combinatória - Nı́vel 2 - Aula 8 - Prof. Bruno Holanda

vermelhos.

Daı́, pelo princı́pio da casa dos pombos, existirão três vértices do pentágono que serão da
mesma cor. E como quaisquer três vértices de um pentágono regular formam um triângulo
isósceles, existirá um triângulo isósceles monocromático.

Problema 3. (Leningrado) Considere 70 inteiros positivos distintos menores ou iguais a


200. Prove que existem dois deles cuja diferença é 4, 5 ou 9.

Solução. Sejam a1 , a2 , ..., a70 esses inteiros positivos. Considere as seguintes listas:

{a1 , a2 , ..., a70 };

{a1 + 4, a2 + 4, ..., a70 + 4};


{a1 + 9, a2 + 9, ..., a70 + 9}.
Temos um total de 210 números que estão compreendidos entre 1 e 209 (inclusive). Por-
tanto, pelo princı́pio da casa dos pombos, existirão dois iguais. Como números na mesma
lista são sempre diferentes, será possı́vel encontrar dois números em listas diferentes que
são iguais. Estes dois números irão satisfazer à condição do problema.

Problema 4. (Torneio das Cidades 1998) Em um tabuleiro 8 × 8, 17 casas são marcadas.


Prove que é possı́vel escolher duas dessas casas marcadas de modo que um cavalo de xadrez
leve pelo menos três movimentos para ir de uma a outra.

Solução. Pinte as casas do tabuleiro usando 16 cores conforme a figura a seguir.

10 12 14 16 2 4 6 8
10 12 14 16 2 4 6 8
9 11 13 15 1 3 5 7
9 11 13 15 1 3 5 7
2 4 6 8 10 12 14 16
2 4 6 8 10 12 14 16
1 3 5 7 9 11 13 15
1 3 5 7 9 11 13 15

Observe que para se deslocar entre duas casas de mesma cor o cavalo necessita de pelo
menos três movimentos. Portanto, pelo princı́pio da casa dos pontos, dentre 17 casas mar-
cadas, sempre haverá pelo menos duas da mesma cor.

Problema 5. (Teste Cone Sul) Os inteiros 1, 2, . . . , 200 são divididos em 50 conjuntos. Mos-
tre que pelo menos um desses 50 conjuntos contém três números distintos que podem ser

2
POT 2012 - Combinatória - Nı́vel 2 - Aula 8 - Prof. Bruno Holanda

medidas dos lados de um mesmo triângulo.

Pelo Princı́pio da Casa dos Pombos, dentre os 101 inteiros 100, 101, . . . , 200, pelo menos
três deles estão em um mesmo conjunto. Sendo a < b < c tais inteiros, temos

a + b ≥ 100 + 101 = 201 > 200 ≥ c ⇒ a + b > c,

e portanto a, b, c podem ser medidas dos lados de um mesmo triângulo.

Problemas Propostos

Problema 6. Mostre que para todo n > 1 de qualquer subconjunto de n + 2 elementos do


conjunto 1, 2, ..., 3n podemos escolher dois cuja a diferença é maior que n e menor que 2n.
Problema 7. Em uma sapataria existem 200 botas de tamanho 41, 200 botas de tamanho
42, e 200 botas de tamanho 43. Dessas 600 botas, 300 são para o pé esquerdo e 300 para
o direito. Prove que existem pelo menos 100 pares de botas usáveis.
Problema 8. Onze estudantes formaram cinco grupos de estudo. Prove que existem dois
alunos A e B, tais que em todo grupo que inclui A também inclui B.
Problema 9. Prove que se escolhermos mais do que n números do conjunto {1, 2, . . . , 2n},
então um deles será múltiplo de outro. Isso pode ser evitado com n números?
Problema 10. (Torneio das Cidades 1994) Existem 20 alunos em uma escola. Quaisquer
dois deles possui um avó em comum. Prove que pelo menos 14 deles possui um avó em
comum.
Problema 11. (Rússia 1997) Uma sala de aula possui 33 alunos. Cada aluno tem uma
música e um cantor favorito. Certo dia, cada um deles perguntou aos demais suas músicas
e catores favoritos. Em seguida, cada um falou dois números, o primeiro era a quantidades
de alunos que gostavam da mesma música e o segundo, a quantidade de alunos que tinham
o mesmo cantor favorito. Sabe-se que cada um dos números de 0 a 10 apareceu entre
as respostas. Mostre que existem dois alunos que gostam do mesmo cantor e da mesma
música.
Problema 12. Suponha que para algum inteiro k ≥ 1 a soma de 2k + 1 inteiros positivos
distintos é menor que (k + 1)(3k + 1). Mostre que existem dois deles cuja soma é 2k + 1.
Problema 13. Existe algum conjunto A formado por sete inteiros positivos, nenhum dos
quais maior que 24, tal que as somas dos elementos de cada um dos seus 127 subconjuntos
não-vazios sejam distintas duas a duas?
Problema 14. (USAMO 1985) Em uma festa há n pessoas. Prove que existem duas pessoas
tais que, das n−2 pessoas restantes é possı́vel achar ⌊n/2⌋−1 onde cada uma delas conhece
ou não conhecem ambas.

3
POT 2012 - Combinatória - Nı́vel 2 - Aula 8 - Prof. Bruno Holanda

Problema 15. O plano é pintado usando duas cores. Prove que existem dois pontos de
mesma cor distando exatamente um metro.

Problema 16. (Putnam) O plano é pintado usando três cores. Prove que existem dois
pontos de mesma cor distando exatamente um metro.

Problema 17. O plano é totalmente pintado usando duas cores. Prove que existe um
retângulo cujos vértices são todos da mesma cor.

Problema 18. (IMO 1983) Cada ponto do perı́metro de um triângulo eqüilátero é pintado
de uma de duas cores. Mostre que é possı́vel escolher três pontos da mesma cor formando
um triângulo retângulo.

Problema 19. Nove pontos de um icoságono regular são pintados de vermelho. Prove que
podemos encontrar três deles formando um triângulo isósceles.

Problema 20. (Rússia 2004) Cada ponto de coordenadas inteiras é pintado de uma de
três cores, sendo cada cor usada pelo menos uma vez. Prove que podemos encontrar um
triângulo retângulo cujos vértices são de cores distintas.

Problema 21. O plano é pintado usando três cores. Prove que podemos encontrar um
triângulo retângulo isósceles com os três vértices da mesma cor.

4
POT 2012 - Combinatória - Nı́vel 2 - Aula 8 - Prof. Bruno Holanda

Dicas e Soluções

9. Dado um inteiro positivo m, podemos escrevê-lo de modo único na forma m = 2a b,


em que a ≥ 0 e b é ı́mpar. Chamaremos b de parte ı́mpar do número m.
No conjunto {1, 2, . . . , 2n} só podem existir n possı́veis partes ı́mpares, a saber:
1, 3, . . . , 2n − 1. Se escolhermos mais do que n números, pelo princı́pio da casa dos
pombos, existem dois números m e n que têm a mesma parte ı́mpar, ou seja, a = 2r b
e c = 2s b. Mas então, supondo sem perda de generalidade que r ≤ s, concluı́mos que
a|c.
O resultado pode ser evitado com exatamente n números. Um exemplo é escolhermos
os números n + 1, n + 2, . . . , 2n.

13. Não. Por absurdo, suponha A = {x1 < x2 < · · · < x7 } satisfazendo a condição do
enunciado. Note que

S = x1 + x2 + · · · + x7 < 24 + 23 + 22 + 20 + 19 + 18 + x1 = 126 + x1 .

De fato, 24, 23, 22, 21 não podem estar simultaneamente em A (pois 24+21 = 23+22),
bem como 24, 23, 19, 18 também não (pois 24 + 18 = 19 + 23). Como a soma mı́nima
dos elementos de um subconjunto é x1 e a soma máxima é menor que 126+x1 , existem
no máximo 126 valores para a soma dos elementos de cada subconjunto. O Princı́pio
da Casa dos Pombos garante portanto que existem dois subconjuntos não-vazios de
A com a mesma soma, absurdo.

5
Programa Olímpico de Treinamento
Curso de Combinatória - Nível 2 Aula 9
Prof. Bruno Holanda

Tabuleiros

Quem nunca brincou de quebra-cabeça? Temos várias “pecinhas” e temos que encontrar
uma maneira de unir todas elas para formar uma figura maior. O que costumava ser apenas
um passa-tempo, ganhou uma irmã que estudada por muitos matemáticos sérios pelo mundo
a “Tiling Theory” (traduzindo: Teoria da Cobertura). E por se tratar de um tema muito
atrativo, logo ganhou força nas principais competições de matemática.
Problema 1. Determine se é possı́vel cobrir ou não o tabuleiro abaixo (sem sobreposições)
usando apenas dominós?

Solução. Pinte as casas do tabuleiro acima alternadamente de branco e preto (como no


tabuleiro de xadrez). Note que, não importa como colocamos o dominó no tabuleiro, ele
sempre cobre uma casa branca e ou outra preta. Desse modo se fosse possı́vel cobrir o
tabuleiro usando apenas dominós, deverı́amos ter o tabuleiro com a quantidade de casas
pretas igual a quantidade de casas brancas. Mas no tabuleiro “quebrado” existem 18 casas
brancas e 16 pretas. Logo, não é possı́vel fazer tal cobertura.

Problema 2. Podemos cobrir um tabuleiro 10 × 10 usando apenas T-tetraminós como


abaixo?

Solução. Pinte o tabuleiro de branco e preto da maneira usual (como no xadrez). Note
que ao colocarmos um T-tetraminó no tabuleiro ele pode assumir colorações do tipo 1 ou
2.
POT 2012 - Combinatória - Nı́vel 2 - Aula 9 - Prof. Bruno Holanda

Suponha que ao cobrir o tabuleiro usamos A peças do tipo 1 e B do tipo 2. Sabemos


que devemos usar 25 peças no total ou seja A + B = 25. Cada peça do tipo 1 possui uma
casa branca e cada peça do tipo 2 possui 3 casas brancas, e como temos ao todo 50 casas
brancas no tabuleiro; A + 3B = 50. De modo análogo, obtemos B + 3A = 50. Porém o
sistema acima não possui solução inteira. Logo, não é possı́vel cobrir o tabuleiro.

Tipo 1 Tipo 2

E não é apenas a pintura do xadrez que é útil para resolver problemas. Vejamos o
próximo exemplo.

Problema 3. Para que valores de n, m podemos cobrir um tabuleiro n × m usando apenas


L-tetraminós como abaixo?

Solução. Claramente n · m deve ser múltiplo de 4. Nesse caso, n ou m (possivelmente


ambos) deve ser múltiplo de dois. Suponha sem perca de generalidade que m (i.e., o
número de colunas) é par. Pinte altenadamente as colunas de duas cores como mostrado
na figura a seguir. Para finalizar, adapte a solução do problema anterior.

Figura 1: Pintura por Colunas

Se duas cores ajudam muita gente, quatro cores ajudam muito mais! É isso mesmo!
Não vá pensando que é só pintar o tabuleiro de preto e branco que você vai resolver todos
os problemas de tabuleiro do mundo! O próximo exemplo mostra que às vezes apenas duas
cores não bastam.

Problema 4. É possı́vel que um cavalo do xadrez passe por todas as casas de um tabuleiro
4 × 10 exatamente uma vez e, em seguida retorne para o quadrado original?

1 2 1 2 1 2
3 4 3 4 3 4
4 3 4 3 4 3
2 1 2 1 2 1

2
POT 2012 - Combinatória - Nı́vel 2 - Aula 9 - Prof. Bruno Holanda

Solução. Pinte o tabuleiro 4×n como mostra a figura acima. Assuma que seja possı́vel fazer
que o cavalo passe por todas as casas. Note que, se o cavalo está na casa 1 só poderá ir para
casa 3 desse modo para o cavalo ir para uma casa de cor 1 ele passa por duas casas de cor 3,
e como cada cor possui o mesmo número de casas, fica impossı́vel o cavalo fazer o passeio.

Vimos que pintar tabuleiros usando cores é uma exelente idéia. Uma idéia melhor ainda
é pintar usando números! Você deve estar se perguntando por que? Bem, os números possui
propriedades aritméticas (i.e, podem ser somados e multiplicados), coisa que não podemos
fazer com cores. A não ser que você ache que preto+branco=cinza.

Problema 5. (Estônia 1993) Para quais naturais n é posı́vel cobrir um retângulo de tamanho
3 × n com peças mostradas na figura abaixo sem sobreposição?

Solução. Pinte o tabuleiro da seguinte forma:

1 1 1 1 1 1 1
−1 −1 −1 −1 −1 −1 −1
1 1 1 1 1 1 1

Veja que a soma dos números cobertos por um L-trinimó é sempre 1 ou −1. Enquanto
a soma dos números cobertos por um Z-tetraminó é sempre zero. Além disso, a soma de
todos os números do tabuleiro é n. Observe que para cobrir um tabuleiro 3 × n podemos
usar no máximo n peças. Assim, todas as peças devem ser L-triminós. Além disso, não
podemos dispor nenhum L-triminó de modo que a soma dos números escritos em suas casas
seja −1. Dessa forma, se pintarmos o tabuleiro como no xadrez, cada L-triminó terá que
ocupar duas casas pretas. Portanto, n deve ser um número par.

Problemas Propostos

Problema 6. Ache o menor ladode um tabuleiro quadrado que pode ser montado usando
um mesmo número de peças de cada um dos tipos abaixo.

Problema 7. Sobre uma das casas de um tabuleiro infinito, existe um cubo que cobre a
casa perfeitamente. A face no topo do cubo é branca, enquanto as demais faces são pretas.
A cada passo, podemos tombar o cubo para um dos lados. É possı́vel que:

3
POT 2012 - Combinatória - Nı́vel 2 - Aula 9 - Prof. Bruno Holanda

(a) Após 2004 passos o cubo volte ao mesmo quadrado com a face branca para baixo?

(b) Após 2005 passos?

Problema 8. É possı́vel cobrir o tabuleiro a seguir usando apenas dominós?

Problema 9. É possı́vel cobrir um tabuleiro 5 × 10 usando apenas peças como abaixo?

Problema 10. Queremos cobrir um tabuleiro 7 × 7 usando várias peças de dois tipos:

Tipo 1 Tipo 2

Diga como cobrir o tabuleiro usando o menor número possı́vel de peças do tipo 1.

Problema 11. (Rússia 1997) Podemos cobrir um tabuleiro 75 × 75 usando dominós e cruzes
(como na figura a seguir)?

cruz

Problema 12. (Rioplatense 1999) É possı́vel cobrir um tabuleiro 1999×1999 com quadrados
de lados inteiros maiores que 35 e menores que 1999?
PS: Os quadrados podem ser de tamanhos distintos.

Problema 13. (Rússia 2007) As faces de um cubo 9×9×9 são particionas em quadradinhos
da forma usual. Sua superfı́cie é coberta por 243 tiras de papel 2 × 1 sem sobreposição.
Uma tira é dita dobrada se não está apenas sobre uma face.
Prove que o número de tiras dobradas é ı́mpar.

Problema 14. Podemos cobrir uma caixa 10 × 10 × 10 com 250 caixas 1 × 1 × 4?

Problema 15. Um tabuleiro n × m foi totalmente coberto usando peças 4 × 1 e 2 × 2. Em


seguida, todas as peças foram retiradas do tabuleiro e uma peça 2 × 2 foi substituı́da por
uma peça 4 × 1. Prove que o tabuleiro não poderá ser mais coberto com essa troca.

4
POT 2012 - Combinatória - Nı́vel 2 - Aula 9 - Prof. Bruno Holanda

Problema 16. De um tabuleiro n × n são retiradas suas quatro casas do quanto. Quais são
os valores de n para os quais esse tabuleiro quebrado é coberto por L-tetraminós?

Problema 17. Sejam m e n inteiros maiores que 1. Se um tabuleiro m × n pode ser coberto
com L-tetraminós então é mn é múltiplo de 8.

Problema 18. (Teorema de Klarner) Um tabuleiro a × b pode ser coberto usando apenas
peças 1 × n se e somente se n | a ou n | b.

Problema 19. (Romênia 2000) Determine todos os tabuleiros m×n que podem ser cobertos
usando L-triminós como abaixo:

Problema 20. Um tabuleiro 7 × 7 é coberto usando 16 peças 3 × 1 e um monominó.


Determine todas as posições possı́veis do monominó.

Problema 21. (Estônia 2004) Um tabuleiro 5 × 5 é coberto por oito t-triminós e um


monominó. Determine todas as possı́veis possições que o monominó pode ocupar.

Problema 22. Qual o número máximo de S-tetraminós como o abaixo podem ser colocados,
sem sobreposições em um tabuleiro 10 × 10?

Um tabuleiro 7 × 7 é coberto usando peças do seguinte tipo:

(1) (2) (3)


Prove que uma e apenas uma peça com quatro casas é usada.

Problema 23. (Bielorussia 1999) Temos um tabuleiro 7 × 7 e peças dos três tipos a seguir:

(1) (2) (3)


Samuel possui infinitas peças do tipo 2 e uma peça do tipo 3, enquanto Marcelo possui
apenas uma peça do tipo 1.

a) Prove que Marcelo pode colocar sua peça em algum lugar do tabuleiro de modo que
Samuel não consiga completar o resto do tabuleiro usando suas peças.

b) Suponha que Samuel adquiriu outra peça do tipo 3. Prove que não importa o lugar no
qual Marcelo coloque sua peça, Samuel sempre poderá completar o tabuleiro.

5
POT 2012 - Combinatória - Nı́vel 2 - Aula 9 - Prof. Bruno Holanda

Problema 24. (IMO 2004) Um gancho é uma figura de seis casas como na figura acima ou
qualquer uma das figuras obtidas desta aplicando rotações ou reflexões. Determine todos
os tabuleiros m × n que podem ser cobertos usando esses ganchos.

gancho

Problema 25. (Putnam 1991) Existe algum natural L, tal que se m e n são inteiros maiores
que L, então todo tabuleiro m × n pode ser coberto usando peças 4 × 6 e 5 × 7?

Problema 26. (Bielorussia 2000) Ache o maior número de cruzes que podem cobir um
tabuleiro 8 × 8.

Problema 27. (Bielorussia 2000) Ache o maior número T-hexaminós (como na figura
abaixo) que podem cobir um tabuleiro 9 × 9.

Problema 28. (Estônia 2004) Ache a medida do lado do menor cubo que pode ser coberto
por crymbles (figura 2).

Figura 2: Crymble

Problema 29. (Rússia 1996) Podemos cobrir um tabuleiro 5 × 7 com L-triminós que tal
forma que cada casa do tabuleiro seja coberta por um mesmo número de peças? (pp ??)

Problema 30. Suponha que 99 peças do tipo 2 × 2 são colocadas em um tabuleiro 29 × 29.
Mostre que uma outra peça ainda pode ser colocada.

Problema 31. Determine se a última peça do resta um pode terminar na casa indicada
(figura 3)

6
POT 2012 - Combinatória - Nı́vel 2 - Aula 9 - Prof. Bruno Holanda

Figura 3: Resta Um.

Dicas e Soluções

7. (a) Sim. Vire o cubo duas vezes para a direita, uma para baixo, duas para a esquerda
e uma para cima (figura 4). Após estes seis passos, a face branca estará virada
para baixo. Depois basta repetidamente o cudo para direira e para esquerda 996
vezes.
(b) Não. Pinte o tabuleiro na maneira usual. Note que, a cada movimento, o cubo
muda de uma casa preta para uma casa branca e vice-versa. Logo, após um
número ı́mpar de movimentos não poderá estar na casa inicial.

Figura 4: Virando um Cubo.

9. Sim veja a figura 5.

Figura 5: Cobrindo com Y-pentaminós.

Nota. Com um pouco mais de trabalho podemos provar que o “menor” tabuleiro
(em número de casas) que podemos cobrir usando apenas Y-pentaminós é o 5 × 10.
Note que para cobrı́-lo usamos 10 peças. Dessa forma, dizemos que o Y-pentaminó
tem ordem 10. Veja que alguns polinimós já são um tabuleiro, como acontece com
o monominó e o dominó. Esse tipo de peça tem ordem 1 ou trivial. Algumas peças
(como o Z-tetraminó) possuem ordem infinita, já que não existe nenhum tabuleiro
n × m que possa ser inteiramente coberto usando somente elas.

7
POT 2012 - Combinatória - Nı́vel 2 - Aula 9 - Prof. Bruno Holanda

15. Pinte tabuleiro da seguinte forma:


1) As linhas pares devem ser pintadas como no xadrez, alternando preto e branco.
2) As linhas ı́mpares devem ser pintadas totalmente de branco.

18. Pinte como no xadrez, porém use n cores!

28. Dica: Use a pintura como mostra a figura 6 para mostrar que o cubo de lado 5 não
pode ser coberto.

Figura 6:

29. Pinte o tabuleiro usando −2’s e 1’s como mostrado na figura a seguir. Cada T-triminó
ocupa três casas cuja a soma é 3 ou 0. Por outro lado a soma de todas casas do ta-
buleiro é −1. Logo, é impossı́vel cobrir já que a soma não é um múltiplo positivo de 3.

Figura 7:

31. Use a pintura alternada do xadrez usando três cores.

8
Programa Olímpico de Treinamento
Curso de Combinatória - Nível 2 Aula 10
Prof. Bruno Holanda

Grafos I

O que é um grafo? Se você nunca ouviu falar nisso antes, esta é certamente uma per-
gunta que você deve estar se fazendo. Vamos tentar matar sua curiosidade contando como
foi que a teoria dos grafos surgiu.

Figura 1: Mapa de Königsberg

A Literatura afirma que a teoria dos grafos começou na cidade de Königsberg em 1736
pelo grande matemático suı́ço Leonhard Euler (1707-1783). A cidade era cortada pelo rio
Pregel, que possuı́a duas ilhas (figura 1). Como era muito complicado fazer o transporte de
cargas e pessoas através de barcos, algumas pontes foram construı́das para auxiliar neste
deslocamento entre as ilhas e as duas margens. Após algum tempo as pessoas começaram
a se perguntar se era possı́vel sair de sua casa, passar por cada ponte exatamente uma vez
e voltar para a segurança de seu lar.

Figura 2: Diagrama de Euler


POT 2012 - Combinatória - Nı́vel 2 - Aula 8 - Prof. Bruno Holanda

Para resolver o problema, Euler montou um diagrama que representasse o mapa da


cidade. Ele o fez da seguinte maneira: A cada ilha e margem ele associou a um ponto que
chamaremos de vértice e a cada ponte uma ligação que chamaremos de aresta. Com isso,
ele obteve a figura 2:

Essa figura com vários pontos (vértices) e algumas ligações (arestas) que denominamos
um grafo. Para finalizar seu raciocı́nio, Euler percebeu que existiam vértices com exata-
mente três arestas incidentes. Por outro lado, como os moradores queriam atravessar cada
ponte apenas uma vez, cada vértice deveria ter um número par arestas. Logo, se tornaria
impossı́vel fazer um percurso seguindo as regras impostas pelos moradores.

Como em toda teoria matemática, a teoria dos grafos está repleta de nomeclaturas e
termos técnicos. Nesta seção vamos aprender algumas definições importantes para o enten-
dimento completo deste capı́tulo. A seguir damos um exemplo de um grafo que representa
um mapa de estradas e cidades.

F
D

A G
C

B E

Vamos aproveitar o grafo acima para abordar algumas definições. Por exemplo, o grafo
acima é conexo, pois é possı́vel ir de um vértice a qualquer outro passando usando algu-
mas de suas arestas. Por exemplo, para ir de A até G basta fazer a seguinte seqüência
A → C → E → F → G. Dizemos então, que esta seqüência é um caminho de A até G.
Agora, um caminho fechado é chamado de ciclo. Por exemplo, o caminho A → B → E → A
é um ciclo de tamanho 3 (ou seja um C3 ). Já o ciclo B → E → G → F → B é um C4 .

Outra notação muito importante é o grau. Vamos definir o grau de um vértice v como a
quantidade de arestas que incidem nele. E vamos denotar essa quantidade como d(v). Por
exemplo, d(A) = 4, d(B) = 3 e d(C) = 2. Os próximos exercı́cios servirão para fixarmos as
definições que acabamos de aprender.

Exercı́cios:

1. Sabemos que o grafo anterior era conexo. Porém, existe uma aresta que, se retirada,
o grafo passará a ser desconexo. Que aresta é essa? Explique porque não pode ser
outra.

2. Qual é o menor caminho de D até C? E o maior? (não se pode repetir arestas)

3. Quantos ciclos de tamanho três existem? E de tamanho quatro?

2
POT 2012 - Combinatória - Nı́vel 2 - Aula 8 - Prof. Bruno Holanda

4. Determine o ciclo que possui o maior tamanho.


5. Qual o vértice que tem o maior grau?
6. Calcule a soma dos graus de todos os vértices do grafo.

Você deve ter notado que o grafo de Euler possui uma particularidade: entre o mesmo
par de vértices existem duas arestas que os liga. Porém, a maioria dos grafos que estuda-
mos são grafos simples. Ou seja, grafos que não admitem laços (arestas que começam e
terminam no mesmo vértice) e arestas múltiplas (como no grafo de Euler).

O próximo problema é um dos mais famosos problemas de toda a olimpı́ada de ma-


temática. Pode ter certeza que você ainda vai ouvir falar desse problema muitas vezes.
Problema 1. É possı́vel que os cavalos da figura 1 fiquem na posição da figura 2?

Figura 1 Figura 2

Solução. Vamos enumerar as casas do tabuleiro da seguinte forma:

1 2 3

4 5 6

7 8 9

Agora vamos construir um grafo com vértices 1, 2, ..., 9 onde vamos ligar dois vértice i
e j se é possı́vel o cavalo ir da casa i até a casa j usando apenas um movimento. Dessa
forma, obtemos o seguinte grafo:

1
6 8

7 5 3

2 4
9

3
POT 2012 - Combinatória - Nı́vel 2 - Aula 8 - Prof. Bruno Holanda

Agora colocamos os cavalos de acordo com os tabuleiros mostrados anteriormente.


1 1
6 8 6 8

7 5 3 7 5 3

2 4 2 4
9 9

Dessa forma fica fácil ver que é impossı́vel ir de uma configuração a outra, pois a ordem
cı́clica dos cavalos não pode mudar.

Teorema. Em um grafo simples G = (V, A), a soma dos graus de todos os seus vértices é
igual ao dobro do número de arestas. Ou seja;
X
d(v) = 2 |A|
v∈V

Prova. De cada vértice v partem d(v) arestas. Porém, cada aresta possui dois vértices.
Desse modo, se somarmos os graus de todos os vértices obteremos o dobro do número de
arestas.

Problema 2. Considere um grupo de 1997 pessoas. É possı́vel que cada uma delas conheça
exatamente:
a) 3 pessoas?
b) 4 pessoas?

Solução. Primeiramente, considere o grupo de 1997 pessoas como um grafo de 1997


vértices, em que cada vértice representa uma pessoa. E uma aresta liga dois vértices
se e somente se as duas pessoas assosciadas são amigas. Para o item (a) estamos supondo
a exitência de um grafo cuja soma de todos os graus é 1997 × 3, ou seja, um número ı́mpar.
Isso é uma contradição, já que a soma de todo os graus é igual ao dobro do número de
arestas e, portanto, um número par.

Para o item (b), considere um grafo cujos vértices são v1 , ..., v1998 . Cada vértice vi
está ligado aos vértices vi−2 , vi−1 , vi+1 , vi+2 para todo i = 1, ..., 1998. Em que v1999 = v1 e
v2000 = v2 .

De todas os assuntos abordados pela matemática, a teoria dos grafos é dos que pos-
suem o maior número de idéias diferentes. Nesta seção vamos resolver vários problemas

4
POT 2012 - Combinatória - Nı́vel 2 - Aula 8 - Prof. Bruno Holanda

de grafos usando estratégias que aprendemos anteriormente. Vamos começar provando um


fato conhecido com teorema de Ramsey.

Problema 3. (Teorema de Ramsey) Em um grupo de seis pessoas sempre existem três que
se conhecem mutuamente ou três que não se conhecem mutuamente.

Prova. Para resolver este problema vamos usar a linguagem dos grafos. Dessa forma, pense
em um grafo com seis vértices A, B, C, D, E, F . Uma aresta contı́nua irá representar uma
“amizade” e uma aresta pontilhada, uma “inimizade”. Fixado o vértice A, sabemos que ele
possui cinco arestas. Como só há dois tipos de aresta, um dos tipos foi usado pelo menos
três vezes. Sem perca de generalidade, suponha que o tipo “continua” foi escolhido três
vezes.

F
B
E

C D

Agora, se uma das arestas BC,CD ou DB for contı́nua, teremos três pessoas se conhe-
cendo mutuamente. Caso contrário, as três são pontilhadas. Neste caso, B,C e D não se
conhecem mutuamente.

Veja que no exemplo anterior usamos esencialmente o princı́pio da casa dos pombos. O
próximo problema é da olimpı́ada do Leningrado de 1990. Neste exemplo vamos usar uma
idéia um pouco mais sofisticada, o princı́pio do extremo.

Problema 4. A Brunzundanga e a Zuzunzilandia são paı́ses vizinhos. Sabe-se que cada


cidade está ligada a no máximo dez outras cidades e que cidades do mesmo paı́s não são
ligadas. Prove que podemos pintar essas estradas usando dez cores de modo que estradas
adjacentes possuam cores distintas.
PS: As estradas são adjacentes se possuem uma cidade em comum.

Solução. Suponha que inicialmente todas as estradas estavam incolores. É claro que pode-
mos escolher uma delas e pintar com uma das cores. A partir daı́ vamos pintar as demais
estradas respeitando a seguinte regra:

Sejam X e Y duas cidades (uma de cada paı́s) tais que a estrada XY está incolor.
Desse modo, existe uma cor (digamos a cor 1) que não foi usada em nenhuma das estradas
partindo de X e uma cor (digamos a cor 2) que não foi usada em nenhuma das estradas
partindo de Y . Agora escolha o maior caminho da forma 2 − 1 − 2 − 1 − · · · partindo de X.

5
POT 2012 - Combinatória - Nı́vel 2 - Aula 8 - Prof. Bruno Holanda

F F
1 2
2 1
=⇒
1 2

X 2 Y X 1 Y
2

Suponha, sem perda de generalidade, que esse caminho termine em uma aresta de cor
1 na cidade F . Desse modo, não existe uma estrada de cor 2 partindo de F . Com isso,
podemos trocar as cores das estradas deste caminho (onde for 2 pintamos de 1 e virce-versa)
sem nenhum problema. Para finalizar, basta pintar a estrada XY da cor 2.

Problemas Propostos

Problema 5. Considere um grupo de 1998 pessoas. É possı́vel que cada uma delas conheça
exatamente 101 pessoas do grupo?
Problema 6. Cada um dos 102 estudantes é amigo de pelo menos 68 outros alunos. Prove
que existem quatro estudantes com o mesmo número de amigos.
Problema 7. Todos os vértices de um grafo têm grau 3. Prove que o grafo possui um ciclo.
Problema 8. Em um conjunto de n pessoas, em qualquer grupo de quatro delas existe uma
que conhece as outras três. Prove que existe uma pessoa que conhece todas as outras.
Problema 9. A figura abaixo representa as ligações rodoviárias entre 14 cidades. Existe
um caminho passando por cada cidade exatamente uma vez?

Problema 10. Em um conjunto de 2n pessoas, cada uma delas possui um número par
de amigos. Prove que existem duas pessoas que possuem um número par de amigos em
comum.
Problema 11. (Rússsia 2000) Em um grafo G cada vértice possui grau pelo menos 3. Prove
que nesse grafo há um ciclo com o número de arestas não divisı́vel por 3.
Problema 12. Na Bruzundanga, quaisquer duas cidades são ligadas por uma estrada. Um
imperador tirano decidiu transformar todas essas estradas em estradas de mão única de tal
forma que se uma pessoa sair de sua cidade não poderá mais voltar. É possı́vel fazer tal
crueldade?

6
POT 2012 - Combinatória - Nı́vel 2 - Aula 8 - Prof. Bruno Holanda

Problema 13. (Rússia 1970) Em um torneio completo de tênis haviam 12 jogadores. Prove
que podemos encontrar três jogadores A, B e C tais que A ganhou de B, B ganhou de C
e C ganhou de A.

Problema 14. (Torneio das Cidades 1982) Em certo paı́s existem mais do que 101 cidades.
A capital deste paı́s é conectada por linhas aéreas a outras 100 cidades, e cada cidade,
exceto pela capital, é conectada a outras 10 cidades (se A está conectado a B, B está
conectado a A). Além disso, todas as linhas aéreas são de uma única direção. Sabe-se
que de qualquer cidade é possı́vel chegar a qualquer outra usando essas rotas. Prove que é
possı́vel fechar metade das linhas aéreas conectadas à capital, e preservar a capacidade de
viajar de uma cidade a qualquer outra.

Problema 15. (Rússia 2004) Um grafo orientado tem 1001 vértices. Cada vértice possui
500 entradas e 500 saı́das. Mostre que qualquer subgrafo de 668 vértices é conexo.

Problema 16. Em um grupo de 50 cientistas sabe-se que cada um deles conhece pelo menos
25 outros cientistas. Prove que podemos colocar quatro deles ao redor de uma mesa de
forma que cada cientista esteja sentado ao lado de dois amigos.

Problema 17. (Jr. Balkan) Em um paı́s com seis cidades quaisquer duas são conectadas
por uma linha aérea (ida-volta). Cada linha aérea é operada por exatamente uma das duas
empresas aéreas existentes. Mostre que existem quatro cidades A, B, C, D tais que as linhas
AB, BC, CD, DA são controladas por uma única empresa.

Problema 18. (IMO 1964) Em um grafo de 17 vértices todas as arestas são traçadas e
pintadas de uma de três cores. Prove que existe um triângulo com as três arestas da
mesma cor.

Problema 19. (Proposto IMO 1977) Em uma sala estão nove homens. Sabe-se que em
qualquer grupo de três deles existem dois que se conhecem. Prove que podemos escolher
quatro deles que se conhecem mutuamente.

Problema 20. (Rússia 1974) Em um grupo de n pessoas sabe-se que se duas possuem
mesmo número de amigos, então elas não possuem amigos em comum. Prove que existe
uma pessoa com exatamente um amigo.

7
Programa Olímpico de Treinamento
Curso de Combinatória - Nível 2 Aula 11
Prof. Bruno Holanda

Invariantes

Nesta aula vamos estudar o princı́pio da invariância. Ou seja, vamos resolver proble-
mas que, dada uma transformação, existe uma propriedade associada que nunca muda. Por
exemplo, se somarmos dois a um certo natural, sua paridade é invariante.

Problema 1. Sete moedas estão sobre uma mesa mostrando a cara. Podemos escolher
quaisquer quatro delas e virá-las ao mesmo tempo. Podemos obter todas as moedas mos-
trando a coroa?

Solução. Quando escolhemos quatro moedas par virar, sempre iremos nos deparar com
uma das seguintes possibilidades:

• Todas as moedas são cara;

• Temos três moedas cara e uma coroa;

• Temos duas caras e duas coroas;

• Temos uma cara e três coroas;

• Todas as moedas são coroas.

Na primeira possibilidade, ao virar as quatro moedas, passamos a ter quatro coroas a


mais na configuração. Na segunda possibilidade, passamos a ter duas coroas a mais. Na
terceira, a quantidade de coroas não se altera. Na quarta, perdemos duas coroas. E na
quinta, perdemos quatro.

Ou seja, se temos em um dado momento, K coroas na configuração, após aplicarmos a


transformação permitida, termos K + 2q coroas. Onde q ∈ {−2, −1, 0, 1, 2}. Portanto, a
quantidade de coroas (que é inicialmente zero) sempre será par. Logo, é impossı́vel obter
todas as moedas coroas através de um número finito de operações. Neste caso, a paridade
da quantidade de moedas coroas é invariante.
POT 2012 - Combinatória - Nı́vel 2 - Aula 11 - Prof. Bruno Holanda

Problema 2. Em cada um dos dez degraus de uma escada existe uma rã. Cada rã pode,
dando um pulo, ir para outro degrau. Porém, quando uma rã faz isso, ao mesmo tempo,
uma outra rã deve pular a mesma quantidade de degraus em sentido contrário: uma sobe
e outra desce. Conseguirão as rãs colocar-se todas juntas no mesmo degrau? Justifique.

Solução. Vamos dizer que uma rã tem energia i se ela estiver no i-ésimo degrau. Por
exemplo, uma rã que está no terceiro degrau tem energia 3. Se ela pular para o sétimo
degrau passará a ter energia 7. Dessa forma, observe que a soma das energias de todas as
rãs é invariante. Ou seja, é sempre 1 + 2 + · · · 10 = 55. Portanto, se em algum momento
todas estiverem no mesmo degrau x, todas também terão energia x, ou seja 10x = 55. E
como x ∈ N, concluı́mos que é impossı́vel todas ficarem no mesmo degrau.

Problema 3. Cada um dos números a1 , a2 , ..., an é 1 ou −1, e temos que:

S = a1 a2 a3 a4 + a2 a3 a4 a5 + · · · + an a1 a2 a3 = 0.

Prove que 4 | n.

Esse problema parece muito mais com um problema de teoria dos números do que um
problema de invariância. Na realidade, como isso pode ser um problema de invariância
se, não temos nenhuma transformação? Não seja por isso! Podemos criar nossas próprias
transformações!

Solução. Nosso movimento será o seguinte: “trocar ai por −ai ”. Fazendo essa operação, a
congruência de S módulo 4 é invariante pois, trocam de sinal exatamente quatro parcelas de
S. Assim, basta trocar todos os ai ’s que forem −1 por 1. Portanto 0 ≡ S ≡ 1+1+· · ·+1 ≡ n
(mod 4) ⇒ 4 | n.

Problema 4. Dado um polinômio quadrático ax2 + bx + c pode mos fazer as seguintes


operações:
a. Trocar a com c.
b. Tocar x por x + t onde t é um real.
Usando essas operações é possı́vel transformar x2 − x − 2 em x2 − x − 1?

Solução. Vamos demonstrar que o “delta” é invariante. Observe que os polinômios ax2 +
bx+c e cx2 +bx+a possuem o mesmo delta ∆ = b2 −4ac. Além disso, dado t real, podemos
simplificar:

a(x + t)2 + b(x + t) + c = a(x2 + 2tx + t2 ) + b(x + t) + c = ax2 + (2ta + b) + (at2 + bt + c).

O delta desse último polinômio é:

∆′ = (2ta + b)2 − 4a(at2 + bt + c) = b2 − 4ac.

2
POT 2012 - Combinatória - Nı́vel 2 - Aula 11 - Prof. Bruno Holanda

Note que a maioria dos problemas de invariância têm o enunciado muito parecido. To-
dos eles de alguma forma perguntam se, dado uma configuração é possı́vel chegar em outra.
E como você também deve ter visto, a maioria das respostas é sempre não. Cuidado! Exis-
tem problemas com o enunciado muito parecido mas, a resposta é afirmativa. Nestes casos,
devemos mostrar como chegar na tão desejada configuração.

O próximo exemplo é da olimpı́ada do Leningrado de 1990. Esse exercı́cio irá esclarecer


a idéia de “falsa invariante”.

Problema 5. O número 123 está na tela do computador de Teddy. A cada minuto o número
escrito na tela é somado com 102. Teddy pode trocar a ordem dos dı́gitos do número escrito
na tela quando ele quiser. Ele pode fazer com que o número escrito na tela seja sempre um
número de três dı́gitos?

Solução. É possı́vel, basta ele seguir a seqüência: 123 → 225 → 327 → 429 → 531 ⇒
135 → 237 ⇒ 327 → 429 · · · , onde → denota a operação de computador e ⇒ uma operação
feita por Teddy.

Problemas Propostos

Problema 6. Os números 1, 2, 3, ..., 1989 são escritos em um quadro negro. Podemos apagar
dois números e escrever sua diferença no local. Após muitas operações ficamos apenas com
um número. Esse número pode ser o zero?

Problema 7. Os números 1, 2, ..., 20 são escritos em um quadro negro. Podemos apagar


dois deles a e b e escrever no lugar o número a + b + ab. Após muitas operações ficamos
apenas com um número. Qual deve ser esse número?

Problema 8. Começando com a tripla {3, 4, 12} podemos a cada passo escolher dois número
a e b e trocá-los por 0.6a−0.8b e 0.8a+0.6b. Usando essa operação podemos obter {4, 6, 12}

Problema 9. (Torneio das Cidades) Existem dez moedas em linha reta. É possı́vel virar
quatro consecutivas ou escolher cinco consecutivas e virar quatro que estão na extremidade
(× × × ×).

Problema 10. Em um tabuleiro 8 × 8 uma das casas está pintada de preto e as outras
casas de branco. Podemos escolher qualquer linha ou coluna e trocar a cor de todas as suas
casas. Usando essas operações, podemos obter um tabuleiro inteiramente preto?

Problema 11. Em um tabuleiro 3 × 3 uma das casas do canto está pintada de preto e as
outras casas de branco. Podemos escolher qualquer linha ou coluna e trocar a cor de todas
as suas casas. Usando essas operações, podemos obter um tabuleiro inteiramente preto?

3
POT 2012 - Combinatória - Nı́vel 2 - Aula 11 - Prof. Bruno Holanda

Problema 12. Em um tabuleiro 8 × 8 as quatro casas do canto estão pintadas de preto e as


outras casas de branco. Podemos escolher qualquer linha ou coluna e trocar a cor de todas
as suas casas. Usando essas operações, podemos obter um tabuleiro inteiramente preto?

Problema 13. (Bulgária 2004) Considere todas as “palavras” formadas por a’s e b’s. Nestas
palavras podemos fazer as seguintes operações: Trocar um bloco aba por um bloco b, trocar
um bloco bba por um bloco a. Podemos fazer também as operações ao contrário. É possı́vel
{z } a partir de aa...a
obter a seqüência b |aa...a | {z } b?
2003 2003

Problema 14. (Fortaleza 2003) Sobre uma circunferência tomamos m + n pontos, que a
divide em m + n pequenos arcos. Nós pintamos m pontos de branco e os n restantes de
preto. Em seguida, associamos a cada um dos m + n arcos um dos números 2, 1/2 ou
1, dependendo se as extremidades do arco sejam, respectivamente, ambas brancas, ambas
pretas ou uma preta e uma branca.
Calcule o produto dos números associados a cada um dos m + n arcos.

Problema 15. (Cone Sul 2000) No plano cartesiano, considere os pontos de coordenadas
inteiras. Uma operação consiste em escolher um destes pontos e realizar uma rotação de
90◦ no sentido anti-horário, com centro neste ponto. É possı́vel, através de uma seqüência
dessas operações, levar o triângulo de vértices (0,0);(1,0);(0,1) no triângulo de vértices
(0,0);(1,0);(1,1)?

Problema 16. (Leningrado 1988) Uma pilha com 1001 pedras está sobre uma mesa. Um
jogo consiste em escolher uma pilha sobre a mesa contendo mais de uma pedra, retirar
uma pedra, e separar a pilha em duas pilhas não vazias (não necessariamente iguais). Após
vários movimentos, é possı́vel que todas as pilhas restantes contenham exatamente três
pedras?

Problema 17. (Rússia 1995) Três pilhas de pedras estão sobre uma mesa. Sisyphus pode
escolher duas pilhas e transferir uma pedra de uma pilha para a outra. Para cada trans-
ferência ele recebe de Zeus o número de moedas igual a diferença entre a quantidade de
pedras da pilha de onde foi retirada a pedra e a quantidade de pedras da pilha que rece-
berá a pedra (a pedra na mão de Sisyphus não é levada em conta). Se essa diferença for
negativa, Sisyphus deve pagar a Zeus o número correspondente (o generoso Zeus permite
que ele pague depois se entrar em falência). Após algum tempo todas as pilhas voltaram a
ter a mesma quantidade inicial de pedras. Qual o número máximo de moedas que Sisyphus
pode ter neste momento?

4
Programa Olímpico de Treinamento
Curso de Combinatória - Nível 2 Aula 12
Prof. Bruno Holanda

Invariantes com Restos

Problema 1. (Leningrado 1987) As moedas dos paı́ses Dillia e Dallia são o diller e o daller,
respectivamente. Podemos trocas um diller por dez dallers e um daller por dez dillers.
Zequinha possui um diller e deseja obter a mesma quantidade de dillers e dallers usando
essas operações. É possı́vel que isso ocorra?

Solução. Seja S a diferença entre a quantidade de dillers e dallers. Note que a congruência
de S módulo 11 é invariante. Como inicialmente S ≡ 1 (mod 11), não se pode obter a
mesma quantia de dillers e dallers.

Problema 2. (Rússia 1998) Um inteiro positivo é escrito no quadro. Nós repetimos o pro-
cesso: Apagar o dı́gito das unidades e soma 5 vezes este dı́gito com o número restante.
Começando com 71998 pordemos terminar em 19987 ?

Solução. Seja an o n-ésimo número da lista. Escrevemos esse número da seguinte forma
an = 10tn + un , em que un é um dı́gito e tn representa os primeiros algarismos de an .

Pelas condições dadas no problema, devemos ter an+1 = tn + 5un . Agora, observe que

tn + 5un ≡ 50tn + 5un ≡ 5(10tn + un ) ≡ 5an (mod 7).

Como a1 = 71998 ≡ 0 (mod 7) e 19987 6≡ 0 (mod 7), concluı́mos que é impossı́vel que
19987 apareca na lista.

Problema 3. (Rússia 2008) Um número natural é escrito no quadro-negro. Sempre que o


x
número x está escrito, podemos trocá-lo por 2x + 1 ou por . Em algum momento o
x+2
número 2008 aparece na lista. Prove que 2008 deve ser o primeiro.
a
Solução. Seja x = um número racional escrito na sua forma reduzida. Defina a função
b
f (x) = a + b. Observe que
POT 2012 - Combinatória - Nı́vel 2 - Aula 12 - Prof. Bruno Holanda

2a + b
1. Se x′ = 2x + 1, então x′ = . Como mdc(2a + b, b) = mdc(2a, b) pelo lema de
b
Euclides, então f (x ) = 2f (x) ou f (x′ ) = f (x).

x a
2. Se x′ = = . Como mdc(2b + a, b) = mdc(2b, a) pelo lema de Euclides,
x+2 a + 2b
então f (x ) ≥ 2f (x) ou f (x′ ) = f (x).

Como f (2008) = 2009, ele deve ser o primeiro.

Semi-Invariantes

A idéia de semi-invariante é um pequena generalização da idéia de invariante. Diremos


que uma propriedade é semi-invariante quando ela muda de forma previsı́vel (periodica-
mente, sempre crescendo ou decrescendo). Um exemplo bastante comum de semi-invariante
é a idade de uma pessoa, que sempre cresce de forma periódica (a cada 365 anos).

Problema 4. Nove casas 1 × 1 de um tabuleiro 10 × 10 estão infectadas. A cada segundo,


uma casa que possui duas casas vizinhas (com um lado em comum) infectadas também se
torna infectada. É possı́vel todas as casas se tornarem infectadas?

Solução. Veja que uma casa pode ser infectada de várias formas. Primeiramente vamos
analisar a seguinte “infecção”:

Figura 1: Infecção do Tipo 1.

Olhando para figura fica fácil observar que o perı́metro total da área infectada não muda
após a infecção do tipo 1. Desse modo, poderı́amos pensar que esse perı́metro é invariante
e igual a 4 × 9 = 36. Daı́, como o perı́metro do tabuleiro todo é 4 × 10 = 40 seria impossı́vel
tornar o tabuleiro totalmente infectado. Mas neste caso, estarı́amos cometendo um erro
gravı́ssimo: esquecer de analisar todos os casos. Vejamos o que acontece nos demais casos:
Note que neste tipo de infecção o perı́metro não permanece constante, e sim diminui
em duas unidades! A princı́pio isso pode parecer um problema, mas não é. Se o perı́metro

2
POT 2012 - Combinatória - Nı́vel 2 - Aula 12 - Prof. Bruno Holanda

Figura 2: Infecção do Tipo 2.

Figura 3: Infecção do Tipo 3.

não aumenta, nunca poderá chegar a 40 (já que inicialmente ele é no máximo 36). Porém,
para ter certeza que essa hipótese é verdadeira, ainda temos que analisar o último caso:
Aqui podemos notar que o perı́metro fica menor ainda, diminuindo em quatro unidades.
Com isso, podemos concluir o problema. Ou seja, já que o perı́metro inicial é no máximo 36
(caso em que não há duas casas infectadas vizinhas) e ele nunca cresce, jamais poderemos
infectar completamente o tabuleiro.

Problema 5. Um total de 2000 pessoas estão divididas entre os 115 quartos de uma mansão.
A cada minuto, uma pessoa anda para um quarto com número igual ou maior de pessoas
do qual ela estava. Prove que eventualmente todas as pessoas vão estar em um mesmo
quarto.

Solução. Sejam a1 , a2 , ..., a115 a quantidade de pessoas nos quartos 1, 2, ..., 115 respectiva-
mente em um dado momento. Defina I = a21 + a22 + · · · + a2115 .
Digamos que uma pessoa sai de um quarto com n pessoas e vai para um quarto com m
pessoas (m ≥ n). A variação de I é dada por:

∆I = ((m + 1)2 + (n − 1)2 ) − (m2 + n2 ) = 2(m − n + 1) > 0

Assim, toda vez que uma pessoa muda de quarto o valor de I cresce. Porém, sabemos
que o valor de I não pode crescer indefinidamente pois, o número de pessoas é finito. Ou
seja, em um dado momento I não poderá mais crescer, isso só acontecerá quando nenhuma
pessoa puder mudar de quarto. Logo, todas elas deverão estar no mesmo quarto.

3
POT 2012 - Combinatória - Nı́vel 2 - Aula 12 - Prof. Bruno Holanda

Problemas Propostos

Problema 6. (Rússia 1998) Um número de quatro dı́gitos é escrito no quadro-negro. As


operações permitidas são: adicionar 1 a dois dı́gitos vizinhos (caso nenhum deles seja 9),
ou subtrair 1 de dois dı́gitos vizinhos (caso nenhum deles seja 0). É possı́vel obtermos 2002
a partir de 1234 realizando algumas operações?

Problema 7. Seja d(x) a soma dos dı́gitos de x ∈ N. Determine todas as soluções de


d(d(n)) + d(n) + n = 1997.

Problema 8. (Torneio das Cidades) Todo membro de uma seqüência, iniciando do segundo,
é igual a soma do termo anterior com a soma de seus dı́gitos. O primeiro número é 1. É
possı́vel que 123456 pertença à seqüência?

Problema 9. (Hong Kong 1997) Cinco números 1, 2, 3, 4, 5 estão escritos em um quadro


negro. Um estudante pode apagar dois dos números a e b e escrever nos seus lugares a + b
e ab. Após algumas operações podemos obter a quı́ntupla 21, 27, 64, 180, 540?

Problema 10. (Torneio das Cidades 1985) Na ilha de Camelot vivem 13 camaleões roxos,
15 verdes e 17 amarelos. Quando dois de cores distintas se encontram, mudam simulta-
neamente para a terceira cor. Poderia dar-se a situação na qual todos tenham a mesma
cor?

Problema 11. Em uma fábrica de cartões existem três máquinas. A primeira recebe um
cartão (a, b) e retorna um cartão (a + 1, b + 1). A segunda recebe um cartão (2a, 2b) e
retorna um cartão (a, b). A terceira recebe dois cartões (a, b) e (b, c) e retorna o cartão
(a, c). Todas as máquinas também retornam o(s) cartão(ões) dados. É possı́vel fabricar
um cartão (1, 1988) se temos inicialmente apenas um cartão (5, 19)?

Problema 12. Com a calculadora KPK-1991 podemos efetuar duas operações: (a) elevar
um número ao quadrado; e (b) e obter de um número X de n dı́gitos (n > 3) o número
A + B, onde A é o número formado pelos três últimos de X e B o número formado pelos
(n − 3) dı́gitos de X. Podemos obter o número 703 a partir de 604 usando essa calculadora?

Problema 13. (Rússia 1998) Os número 19 e 98 são escritos no quadro. A cada minuto,
um deles é acrescentado 1 e o outro é elevado ao quadrado. É possı́vel que os dois números
se tornem iguais após diversas operações?

Problema 14. (Rússia 1998) Temos um tabuleiro n × n (n > 100) com n − 1 casas iguais
a 1 e o restante iguais a 0. Podemos escolher uma casa, subtrair 1 dela, e adicionar 1 nas
demais casas que estão na mesma liha e coluna desta. Com essa operação, podemos fazer
com que todas as casas do tabuleiro se tornem iguais?

Problema 15. (Leningrado) Existem n ≥ 2 números não-nulos escritos em um quadro.


Podemos escolher dois números a e b e trocá-los por a + b/2 e b − a/2. Prove que após feito
um movimento não podemos obter os números iniciais novamente.

4
POT 2012 - Combinatória - Nı́vel 2 - Aula 12 - Prof. Bruno Holanda

Problema 16. (Ucrânia 2000) Existem inicialmente n números √ 1 escritos em um quadro.


ab 2
Em cada passo podemos apagar a e b e escrever o número no seu lugar. Após repetir
a+b
1
essa operação n − 1 vezes, prove que o último número escrito não pode ser menor que √
n
Problema 17. (São Petersburgo 1998) Um total de 119 anões vivem em uma aldeia com
120 pequenas casas. Uma casa é dita super-habitada se 15 anões ou mais vivem nela. Todo
dia, os anões de uma casa super-habitada têm uma briga e se mudam para outras casas da
aldeia. Algum dia, necessariamente se encerrará?

Problema 18. (Rússia 1997) Temos uma fileira longa de copos e n pedras no copo central
(copo 0). Os seguintes movimentos são permitidos:

Movimento tipo A:


i−1 i i+1i+2 i−1 i i+1i+2
Se há pelo menos uma pedra no copo i e pelo menos uma no copo i + 1 podemos fa-
zer uma pedra que está no copo i+1 pular para o copo i−1 eliminando uma pedra do copo i.

Movimento tipo B:


i−1 i i+1i+2 i−1 i i+1i+2
Se há pelo menos duas pedras no copo i podemos pular uma pedra para o copo i + 2 e
outra para o copo i − 1.

Demonstre o seguinte fato: fazendo os movimentos tipo A ou B durante um tempo


suficientemente longo sempre chegamos a uma configuração a partir da qual não é possı́vel
fazer nenhum desses dois tipos de movimento. Além disso, essa configuração final não
depende da escolha de movimentos durante o processo.
Dica: Lembre-se de usar energia!

5
Programa Olímpico de Treinamento
Curso de Combinatória - Nível 2 Aula 13
Prof. Bruno Holanda

Aula de Revisão

O objetivo desta aula é relembrar os principais conceitos e as ideias mais importantes


que aprendemos durante a primeira parte do curso. Além disso, o aluno poderá utilizar o
conjunto de problemas a seguir como um treinamento para provas de olimpı́ada. Pois, não
ter um assunto central que serve de guia na solução dos problemas, é uma simulação da
situação real que ocorre durante uma prova.

Problema 1. (AIME 1988) Determine a quantidade de divisores de 1099 que são múltiplos
de 1088 .

Solução. Os divisores de 1099 são da forma 2a · 5b com 0 ≤ a, b ≤ 99 e para um divisor de


1099 ser um múltiplo de 1088 , devemos ter 88 ≤ a, b ≤ 99. Portanto, são 12 × 12 = 144 tais
números.

Problema 2. (Hungria 1989) Em cada um dos quatro vértices de um quadrado existe uma
pedra. É permitido mudar a quantidade de pedras de acordo começando a seguinte re-
gra: Podemos escolher um vértice, retirar qualquer quantidade positiva de pedras deste e
adicionar o dobro destaa quantidade em cada um dos vértices adjacentes. É possı́vel que
após muitos movimentos obter 1989, 1988, 1990 e 1989 pedras em vértices consecutivos do
quadrado.

Solução. Sejam (x, y, z, w) a quantidade de pedras em vértices consecutivos do quadrado


em um dado momento. Considere a expressão

E = x + z − y − w.

Observe que E é invariante módulo 3, i.e. a cada movimento, o resı́duo de E módulo 3 se


mantém o mesmo. Inicialmente temos E ≡ 0 (mod 3) para a quádrupla (1, 1, 1, 1). Porém,
para a quádrupla (1989, 1988, 1990, 1989) o valor da expressão é tal quem E ≡ 2 (mod 3).
Portanto, é impossı́vel obter tal configuração.
POT 2012 - Combinatória - Nı́vel 2 - Aula 12 - Prof. Bruno Holanda

Problema 3. (Romênia 2010) Cada ponto do plano é pinta de uma de duas cores. Dado
um número inteiro ı́mpar n ≥ 3, prove que existem (pelo menos) dois triângulos mono-
cromáticos e semelhantes cuja razão de semelhança é n.

Solução. Suponha que as cores sejam A (azul) e B (branco) e considere todas as semi-retas
que partem da origem O. Dado um real positivo α, dizemos que uma semi-reta ℓ é do tipo
OX
AA se existirem dois pontos azuis X e Y sobre ℓ tais que = α. Defina os tipos AB,
OY
BA e BB de forma análoga.

Como existem infinitas semi-retas, pelo menos um tipo irá se repetir três vezes. Sem
perda de generalidade, assuma que ℓ1 , ℓ2 , ℓ3 são três semi-retas do tipo AB. Dessa forma,
é possı́vel encontrar pontos azuis X1 , X2 , X3 e pontos brancos Y1 , Y2 , Y3 que formam
triângulos semelhantes com razão de semelhança igual a α. Tomando α = n obtemos o
resultado procurado.

Problema 4. (URSS 1990) Em um Senado existem 30 senadores. Cada senador tem exata-
mente 6 inimigos. Quaisquer três deles formam uma comissão. Ache o número de comissões
em que o membros são todos amigos ou todos inimigos.

Solução. Diremos que uma comissão é azul se satisfaz às condições do problema e diremos
que uma comissão é vermelha caso não satisfaça. Seja x o número de comissões azuis e y
o número de comissões vermelhas. De imediato, temos:
 
30
x+y = = 4060.
3

Por outro lado, se cada senador listar todas as comissões em que os outros dois membros são
ambos seus amigos, ou ambos seus inimigos, cada senador fará uma lista com 62 + 23

2 =
268 comissões. Obtendo um total de 30 · 268 = 8040 comissões listadas. Observe que as
comissões azuis são listadas três vezes e as vermelhas apenas uma. Assim,

3x + y = 8040.

Resolvendo o sistema encontramos x = 1990.

Problema 5. (Torneio das Cidades 2000) Em um conjunto de 32 moedas, todas com mesma
aparência, 30 são reais e 2 são falsas. Quaisquer duas moedas reais possuem o mesmo peso
e as duas moedas falsas também possuem o mesmo peso, que é diferente do peso de uma
moeda real. Mostre como dividir as moedas em dois grupos de mesmo peso usando uma
balança de dois pratos no máximo quatro vezes.

Solução. Enumere as moedas de 0 a 31 porém usando a base binária. Ou seja, as moedas


serão rotuladas de 0000 até 1111. Na primeira pesagem coloque todas as moedas cujo
primeiro dı́gito é zero no prato esquerdo da balança e todas as moedas cujo primeiro dı́gito

2
POT 2012 - Combinatória - Nı́vel 2 - Aula 12 - Prof. Bruno Holanda

é 1 no prato direito. Na segunda pesagem coloque todas as moedas cujo segundo dı́gito
é zero no prato esquerdo da balança e todas as moedas cujo segundo dı́gito é 1 no prato
direito. Faça a terceira e a quarta pesagens de forma análoga.

Como os rótulos das moedas são todos diferentes em pelo menos um dı́gito, em alguma das
quatro pesagens as duas moedas falsas estarão em pratos diferentes, e isso equilibrará a
balança e ao mesmo tempo resolverá o problema já que em cada prato haverá 16 moedas.

3
POT 2012 - Combinatória - Nı́vel 2 - Aula 12 - Prof. Bruno Holanda

Problemas Propostos

Problema 6. De quantas formas podemos organizar 10 casais ao redor de um cı́culo de


modo que conjuges estejam sempre juntos?

Problema 7. (Torneio das Cidades 1997) Qual o número máximo de cavalos que podemos
colocar em um tabuleiro 5 × 5 de modo que nenhum deles ataque um outro.

Problema 8. (Torneio das Cidades 1990) Um quadrado unitário preto é desenhado em um


plano. Mostre que é possı́vel cobrir este quadrado com sete outros quadrados unitários
brancos sem sobrepor os quadrados brancos e de modo que cada quadrado branco cubra
pelo menos um ponto preto.

Problema 9. (Torneio das Cidades 1995) Sônia possui moedas de 10, 15 e 20 dinheirus
cujo valor total é 500 dinheirus. Ela possui 30 moedas. Mostre que ela possui mais moedas
de 20 do que moedas de 10.

Problema 10. (Torneio das Cidades 1997) Dois jogadores se enfrentam no seguinte jogo: O
primeiro pinta um ponto branco do plano de vermelho e, em seguida, o segundo pinta dez
pontos brancos do plano de verde. O processo é repetido quantas vezes for necessário. O
primeiro ganha se dezenhar um triângulo equilátero com todos vértices vermelhos. Mostre
que o primeiro pode sempre garantir a vitória.

Problema 11. (Torneio das Cidades 1985) Existem 68 moedas, todas de pesos distintos.
Mostre como achar a mais pesa e a mais leve usando uma balança de dois pratos não mais
do que 100 vezes.

Problema 12. Arnaldo e Bernaldo estão brincando no quadro da sala de aula da seguinte
maneira: eles escrevem inicialmente no quadro um número inteiro positivo n. Então,
alternadamente, começando com Arnaldo, apagam o número que está no quadro e escrevem
um novo número que pode ser:

• o que acabou de ser apagado menos a maior potência de 2 (com expoente inteiro
não-negativo) menor do que ou igual ao número apagado;

• o que acabou de ser apagado dividido por 2, caso o número apagado seja par.

Vence a brincadeira quem obtiver primeiro o número zero.

a) Determine qual dos jogadores possui uma estratégia vencedora para n = 40 e descreva-a.

b) Determine qual dos jogadores possui uma estratégia vencedora para n = 2012 e descreva-
a.

Problema 13. (Índia 2006) Quarenta e seis quadrados de um tabuleiro 9 × 9 são pintados
de vermelho. Prove que existe um subtabueiro 2 × 2 com pelo menos três casas vermelhas.

4
Programa Olímpico de Treinamento
Curso de Combinatória - Nível 2 Aula 14
Prof. Bruno Holanda

Princı́pio do Extremo

A idéia chave na solução de muitos problemas de combinatória, ou até mesmo em te-


oria dos números e álgebra é a simples consideração de um elemento extremo (máximo
ou mı́nimo). O próximo problema mostrará como essa idéia pode ser simples e ao mesmo
tempo poderosa.

Problema 1. (Leningrado 1988) Alguns pinos estão em um tabuleiro de xadrez. A cada


segundo, um dos pinos move para uma casa vizinha (lado em comum). Após muito tempo
verificou-se que cada pino havia passado todos todas as casas do tabuleiro exatamente uma
vez e tinha voltado para a sua casa inicial. Prove que existiu um momento em que todos
os pinos estavam fora de sua casa inicial.

Solução. Seja P o primeiro pino que voltou para a sua posição inicial. Um movimento antes
dele voltar para sua casa, cada um dos outros pinos deve ter feito um movimento. De fato,
se isso não fosse verdade, P não poderia ter passado por todas as casas do tabuleiro. Desse
modo, este será o momento em que todos os pinos estarão em casas diferentes das iniciais.

Problema 2. (Teorema de Sylverste) Um conjunto finito S de pontos no plano possui a


propriedade que qualquer reta que passa por dois destes pontos também passa por um
terceiro. Prove que todos os pontos estão sobre uma reta.

P0

l0 Q N M

Solução. Seja L o conjunto de todas as retas que passam por pelo menos dois pontos de S.
Agora sejam P0 ∈ S e l0 ∈ L tais que a distância entre P0 e l0 é a menor possı́vel porém,
POT 2012 - Combinatória - Nı́vel 2 - Aula 14 - Prof. Bruno Holanda

diferente de zero. Seja Q a projeção de P0 sobre l0 . Como a reta l0 passa por três deles,
pelo menos dois deles N e M estão na mesma semi-reta (em relação a Q). Suponha que
N é o mais próximo de Q desse modo, a distância entre N e a reta P0 M é menor que a
mı́nima. Contradição.

Problema 3. (Leningrado 1989) Dado um número natural k maior que 1, prove que é im-
possı́vel colocar os números 1, 2, ..., k 2 em um tabuleiro k × k de forma que todas as somas
dos números escritos em cada linha e coluna sejam potências de 2.

Solução. Suponha que seja possı́vel fazer tal distribuição para algum inteiro positivo k.
Além disso, seja 2n a menor dentre as somas. Devemos ter

k(k + 1)
2n ≥ 1 + 2 + · · · + k = .
2
Como 2n é menor potência, 2n divide a soma dos elementos em qualquer linha, portanto
divide a soma de todos os elementos do tabuleiro. Assim,

k2 (k2 + 1)
2n | .
2
Como k2 e k2 + 1 tem paridades opostas, 2n+1 deve dividir apenas um deles. Em qualquer
caso temos 2n+1 ≤ k2 + 1. Isso contradiz a primeira desigualdade encontrada.

Problema 4. (São Petersburgo 1998) Em cada uma de dez folhas de papel são escritas
diversas potências de 2. A soma dos números em cada uma das folhas é a mesma. Mostre
que algum número aparece pelo menos 6 vezes.

Solução. Seja N a soma comum, e n o maior inteiro tal que 2n ≤ N . Suponha que cada
potência só ocorra no máximo 5 vezes. Dai,

5(1 + 2 + · · · + 2n ) = 5(2n+1 − 1) < 10N.

E isso gera uma contradição.

2
POT 2012 - Combinatória - Nı́vel 2 - Aula 14 - Prof. Bruno Holanda

Problemas Propostos

Problema 5. Dado um conjunto de n pontos no plano, nem todos numa mesma reta, existe
uma reta que passa por exatamente dois desses pontos.
Problema 6. São dados n ≥ 3 pontos no plano de forma que quaisquer três estão em um
triângulo de área menor que 1. Mostre que todos eles estão em um triângulo de área menor
que 4.
Problema 7. São dados n pontos no plano. Marcamos então, os pontos médios de todos
os segmentos com extremidades nesses n pontos. Prove que há pelo menos 2n − 3 pontos
marcados distintos.
Problema 8. Há 20 paises em um planeta. Sabe-se que dentre quaisquer três desses paı́ses,
existe sempre dois sem relações diplomáticas. Prove que existem, no máximo, 200 embai-
xadas neste platena.
Problema 9. Todo participante de um torneio joga com cada um dos outros participantes
exatamente uma vez. Após o torneio cada jogador faz uma lista com os nomes de todos os
jogadores vencidos por ele e de todos os que foram vencidos pelos jogadores que ele venceu.
Sabendo que neste torneio não há empates, prove que existe um jogador cuja a lista possui
o nome de todos os outros jogadores.
Problema 10. Em um pátio estão localizadas 2n + 1 pessoas tais que as distância entre
quaisquer duas delas são todas distintas. Em um dado momento cada uma delas atira na
pessoa mais próxima de si. Prove que:
(a) Pelo menos uma pessoa irá sobreviver.

(b) Ninguém levará mais de cinco tiros.

(c) Os caminhos das balas não se encotram.

(d) Os segmentos formados pelas tragetórias das balas não formam um polı́gono convexo
fechado.
Problema 11. Considere três escolas, cada uma com n alunos. Cada estudante tem ao
todo n + 1 amigos nas outras duas escolas em que ele não estuda. Prove que é possı́vel
selecionar um estudante de cada escola de tal forma que os três se conheçam mutuamente.
Problema 12. Em cada lattice point do plano é colocado um inteiro positivo. Cada um
desses números é a média aritmética de seus quatro vizinhos. Mostre que todos os números
são iguais.
Problema 13. Cada casa de um tabuleiro 8 × 8 existe um número que pode ser 0 ou 1.
Para cada casa que contém um 0, a soma dos números escritos nas casas que estão ou na
mesma linha ou na mesma coluna desta casa é maior que ou igual a 8. Prove que a soma
de todos os números no tabuleiro é maior que ou igual a 32.

3
POT 2012 - Combinatória - Nı́vel 2 - Aula 14 - Prof. Bruno Holanda

Problema 14. O parlamento da Bruzundanga consiste de uma casa. Todo membro tem no
máximo três inimigos dentre os restantes. Mostre que é possı́vel separar a casa em duas
casas de tal forma que cada membro tenha no máximo um inimigo em sua casa.

Problema 15. (Torneio das Cidades 1987)


(a) 3n estrelas são colocadas em um tabuleiro 2n × 2n. Prove que podemos eliminar n
linhas e n colunas de modo que todas as estrelas sejam eliminadas.
(b) Prove que, com 3n + 1 estrelas, isso não é mais possı́vel.

Problema 16. (Torneio das Cidades 1983) Os números de 1 a 1000 são escritos ao redor
de um cı́rculo. Prove que é possı́vel formar 500 segmentos que não se cruzam, cada um
ligando dois destes números, e de tal modo que a diferença (em valor absoluto) entre dois
números ligados não seja maior que 749.

Problema 17. (Torneio das Cidades 1985) Oito times de futibol participaram de um torneio
com apenas uma rodada onde cada time jogou contra todos os outros exatamente uma vez).
Não houve empates. Prove que após o termino do torneio é possı́vel escolher quatro times,
digamos A, B, C, D tais que A derrotou B, C e D; B derrotou C e D; e C derrotou D.

4
Programa Olímpico de Treinamento
Curso de Combinatória - Nível 2 Aula 15
Prof. Bruno Holanda

Combinatória Geométrica

Vamos começar com um problema relativamente simples que apareceu na olimpı́ada do


Cone Sul de 2000. Este problema ilustra como podemos usar ideias já vistas antes para
resolver problemas que, em sua essência, são problemas de geometria. Neste exemplo par-
ticular, usaremos o princı́pio do extremo.

Problema 1. Um polı́gono S está contido no interior de um quadrado de lado a. Demonstre


que há pelo menos dois pontos do polı́gono que estão separads por uma distância maior
que ou igual a S/a.

Solução. Suponha quem quaisquer dois pontos do polı́gono estejam separados por uma
distância menor que S/a. Considere o ponto mais a esquerda e o ponto mais a direita.

a X bc

S
bc
Y

Se b é a diferença entre suas abscissas, temos


S
S ≤a·b⇒ ≤ b.
a
Por outro lado, XY ≥ b. Portanto,
S
XY ≥ .
a
POT 2012 - Combinatória - Nı́vel 2 - Aula 15 - Prof. Bruno Holanda

Problema 2. (Ibero 1997) Seja P = {P1 , P2 , ..., P1997 } um conjunto de 1997 pontos no
interior de um cı́rculo de raio 1, com P1 sendo o centro do cı́rculo. Para k = 1, 2, ..., 1997
seja xk a distância de Pk ao ponto de P mais próximo de Pk . Mostre que
x21 + x22 + · · · + x21997 ≤ 9.
Solução. Note que xk ≤ 1, para todo k. Para cada k = 1, ..., 1997 trace uma circunferência
de raio xk /2 e centro em Pk . Estas 1997 circuferências não se intersectam (no máximo se
tocam) e estão todas no interior de uma circunferência Γ de centro em P1 e raio 3/2. Desta
forma, a soma de suas áreas é menor que a área de Γ. Escrevendo isto em uma equação,
temos:
1997
X  xk 2 9π
≤ .
2 4
k=1
Dividindo tudo por π/4, obtemos o resultado procurado.
Problema 3. Seja C um cı́rculo de raio 16 e A um anel com raio interior 2 e raio exterior 3.
Agora suponha que um conjunto S de 650 pontos são selecionados no interior de C. Prove
que podemos colocar o anel A no plano de modo que ele cubra pelo menos 10 pontos de S.

bc
Solução. Suponha que uma cópia de A seja centrada em cada um dos 650 pontos de S.
Considere um cı́rculo D, concêntrico de C e de raio 19.

b b

b
b b

Note que a área de A é 32 π − 22 π = 5π. Dessa forma, as 650 cópias de A irão fazer
uma supercobertura de área de 650 × 5π = 3250π.

Agora, se cada ponto de D for coberto por não mais do que 9 anéis, a área coberta não
pode ser maior que 9(192 π) = 3249π. Portanto, existe um ponto X de D que é coberto
por pelo menos 10 aneis.

Se Yi é o centro de um anel que cobre X, o anel de centro X também irá cobrir Yi . Pois
2 ≤ XYi ≤ 3. Logo, esse anel também cobre 10 pontos de S.

2
POT 2012 - Combinatória - Nı́vel 2 - Aula 15 - Prof. Bruno Holanda

Problema 4. (Proposto para IMO 1989) Temos um conjunto finito de segmentos no plano,
de medida total 1. Prove que existe uma reta ℓ tal que a soma das medidas das projeções
destes segmentos a reta ℓ é menor que 2/π.

Solução. Vamos transladar os segmentos de modo que seus pontos médios coincidam em
um ponto V . Vamos designar as 2n extremidades por A1 , A2 , ..., An , A′1 , A′2 , ..., A′n . A
partir de um ponto Pn′ desenhamos o segmento Pn′ P1 igual e paralelo a V A1 , a partir
de P1 desenhamos o segmento P1 P2 igual e paralelo a V A2 , e assim por diante obtendo
P3 , ..., Pn , P1′ , ..., Pn′ . Que é um polı́gono convexo P de 2n vértices, com um centro de
simetria O. Pois os pares de lados opostos são iguais e paralelos.
A3
An A2

A′1 V A1

A′2
A′3

Escolha um par de lado opostos cuja distância D é mı́nima. Considere o cı́rculo Γ de


centro O e diâmetro D; este é tangente aos dois lados opostos e interior ao polı́gono.

T ′ P1 Pn
b b

P2′
b b

P3′ b

b b
O b

b b
P3
P2
P1 T
b b

Pn′
Então,
n
X
πD < perimetro(P) = si = 1.
i=1

Dessa forma, D < 1/π. Por outro lado, as projeções ortogonais de todos os 2n lados de P
sobre T T ′ possui medida total igual a 2D. Logo, como 2D < 2/π, segue o resultado.

3
POT 2012 - Combinatória - Nı́vel 2 - Aula 15 - Prof. Bruno Holanda

Problemas Propostos

Problema 5. Um n-ágono convexo M é particionado em triângulos por várias diagonais


que não se cortam. Mais ainda, cada vértice de M pertence a um número ı́mpar de tais
triângulos. Mostre que n é divisı́vel por 3.

Problema 6. No centro de um terreno cercado quadrado se encontra um lobo, e em cada


vértice do quadrado há um cachorro. O lobo pode correr por todo o terreno, enquanto os
cachorros podem correr apenas pelas bordas. Sabe-se que os cachorros (que possuem todos
a mesma velocidade) são 1, 5 vezes mais rápidos do que o lobo. Demonstre que os cachorros
podem coordenar seus movimentos de modo que o lobo não possa escapar do terreno.

Problema 7. São colocados 100 pontos no plano. Mostre que podemos usar alguns discos
para cobrir estes pontos de modo que a soma dos diâmetros seja menor do que 100 e a
distância entre quaisquer dois disco seja maior do que 1.

Problema 8. (Torneio das Cidades 1980) Um conjunto finito de segmentos, de comprimento


total 18, estão no interior de um quadrado unitário (assuma que o interior também contém
as bordas e os vértices). Os segmentos são paralelos aos lados do quadrado e podem se
cruzar. Prove que dentre as regiôes em que o quadrado é dividido, há pelo menos uma de
área não menor que 0, 01.

Problema 9. (Banco IMO 2003) Seja n ≥ 5 um inteiro positivo. Determine o maior inteiro k
para o qual existe um polı́gono com n vértices (convexo ou não, porém sem auto-interseções)
que possui k ângulos internos de 90◦ .

4
Programa Olímpico de Treinamento
Curso de Combinatória - Nível 2 Aula 16
Prof. Bruno Holanda

Combinatória e Sequências

Nesta aula iremos aplicar muitas das ideias que aprendemos durante esse curso para
resolver problemas sobre sequências. Como aconteceu na aula de Combinatória Geométrica,
tenha sempre em mente os princı́pios da casa dos pombos, do extremo e da invariância.
Mas também não se esqueça de usar indução sempre que isto parecer útil. Porém, iremos
iniciar a aula com um problema de contagem.

Problema 1. Determine a quantidade de diferentes permutações a1 , a2 , ..., a10 dos inteiros


1, 2, ..., 10 tais que ai > a2i (1 ≤ i ≤ 5) e ai > a2i (1 ≤ i ≤ 4).

Solução. Primeiramente, substitua as desigualdades do problema pelo seguinte diagrama:

a1
b

a2 a3
b b

a4 a5 a6 a7
b b b b

b b b

a8 a9 a10

O diagrama é construı́do de forma que a relação ai → aj indica que ai > aj . Pela figura,
fica claro que devemos ter a1 = 10. Dos 9 inteiros restantes, devemos escolher três deles
para formar o conjunto {a3 , a6 , a7 } feito isso,a3 deve ser o maior deles, e a ordem entre a6
e a7 não deve importar. Portanto, temos 93 · 2 possibilidades para os números a3 , a6 , a7
do diagrama.

Até agora escolhemos a posição de quatro números. Os seis restantes farão parte do
conjunto {a2 , a4 , a5 , a8 , a9 , a10 }. Observe que nesta situação, a2 deve ser o maior dentre os
seis inteiros restantes. Além disso, dos cinco que irão sobrar, após fixar a2 , devemos esolher
três para formar o conjunto {a4 , a8 , a9 }. Destes três, a4 deve ser o maior e a ordem entre
POT 2012 - Combinatória - Nı́vel 2 - Aula 16 - Prof. Bruno Holanda

5
a8 e a9 não deverá importar. Logo, teremos 3 ·2 possibilidades para o conjunto {a4 , a8 , a9 }.

Por fim, os dois números restantes serão a5 e a10 , sendo a5 o maior dentre esses dois.
Portanto, temos    
9 5
·2· · 2 = 3360
3 3
permutações com as propriedades requeridas.

Problema 2. Ache o maior valor possı́vel da expressão

x1 x2 + x2 x3 + · · · + xn x1

para n ≥ 3, onde x1 , x2 , ..., xn é uma permutação arbitrária dos inteiros 1, 2, ..., n.

Solução. Seja Sn (x1 , ..., xn ) = x1 x2 + x2 x3 + · · · + xn x1 a função acima e Mn seu valor


máximo. Como Sn é invariante por permutações cı́clicas, podemos escolher aquela em que
x1 = n. Observe que

Sn (n, x2 , ..., xn ) = Sn−1 (x2 , ..., xn ) − x2 xn + nx2 + nxn


= Sn−1 (x2 , ..., xn ) + n2 − (n − x2 )(n − xn )
≤ Mn−1 + n2 − 1 · 2

em que na primeira igualdade usamos a definição de Sn , na segunda apenas reescremos


x2 xn + nx2 + nxn e na terceira maxizamos o termo −(n − x2 )(n − xn ) e usamos a definição
de Mn−1 .

Fazendo uma soma telescópica, temos

Mn ≤ M3 + (42 − 2) + (52 − 2) + · · · + (n2 − 2)


1
= 11 + (42 + 52 + · · · + n2 ) − 2(n − 3) = (2n3 + 3n2 − 11n + 18).
6
Deixaremos a cargo do aluno a comprovação de que a cota obtida é realmente ótima.

Problema 3. (Cone Sul 2012) Ao redor de uma circunferência estão escritos 2012 números,
cada um deles é igual a 1 ou a −1. Se não há 10 números consecutivos cuja soma seja 0,
ache todos os valores possı́veis da soma dos 2012 números.

Solução. Sejam a1 , a2 , ..., a2012 os números escritos na circunferência. Seja Si a soma de


dez números consecutivos começando a parit de ai . Observe que cada Si deve ser um
número par, já que é soma de dez número ı́mpares. Observe que |Si − Si+1 | ∈ {0, 2}. Agora
suponha que existem i e j tais que Si e Sj possuem sinais diferentes. Por uma propriedade
do tipo valor intermediário, deverá existir algum l entre i e j tal que Sl = 0.

2
POT 2012 - Combinatória - Nı́vel 2 - Aula 16 - Prof. Bruno Holanda

Portanto, iremos assumir sem perda de generalidade que todos os Si são positivos.
Neste caso,
S1 + S2 + · · · + S2012 ≥ 2012 × 2 = 4024
Na soma acima, cada ai é contado exatamente 10 vezes. Logo, a soma de todos os números
na circunferência deve ser pelo menos 402, 4. O primeiro número par que satisfaz essa cota
é 404. Observe que é possı́vel obter esse valor repetindo a sequência de blocos

1, 1, 1, 1, 1, 1, −1, −1, −1, −1

a partir de ai . E cada par entre 404 e 2012 (incluindo 2012) é obtido trocando um (−1)
por um (1) na configuração. Enquanto que os números negativos são obtidos trocando os
sinais de todos os números de uma configuração.

Problema 4. (Rússia) Sejam a1 , a2 , ..., am , b1 , b2 , ..., bn reais positivos tais que

a 1 + a 2 + · · · + a m = b1 + b2 + · · · + bn .

Em um tabuleiro vazio de m linhas e n colunas deve escrever números não-negativos


de modo que a soma dos números na i-ésima linha seja igual a ai e a soma dos números
na j-ésima coluna seja igual a bj (1 ≤ i ≤ m, 1 ≤ j ≤ n). Mostre que é possı́vel obter tal
configuração usando no máximo m + n − 1 números positivos.

Solução. Considere um segmento de tamanho a1 + a2 + · · · + am dividido em m segmentos


de tamanhos a1 , a2 , ..., am . Chamaremos de Ai o segmento de comprimento ai .

A1 A2 A3 ··· Am
b b b b b b

Considere também um segmento de tamanho b1 + b2 + · · · + bn dividido em n segmentos


de tamanhos b1 , b2 , ..., bn . Chamaremos de Bj o segmento de comprimento bj .

B1 B2 B3 ··· Bn
b b b b b b

Os dois segmentos acima possuem o mesmo comprimento, já que a1 + a2 + · · · + am =


b1 + b2 + · · · + bn . Dessa forma, ao sobrepor esses segmentos iremos obter um terceiro que
terá o mesmo tamanho e estará dividido em no máximo m + n − 1 subdivisões. Por fim,
escrevemos na casa que está na linha i e coluna j do tabuleiro o tamanho do segmento
Ai ∩ Bj . E caso essa interseção seja vazia, escrevemos 0.

3
POT 2012 - Combinatória - Nı́vel 2 - Aula 16 - Prof. Bruno Holanda

Problema 5. Sejam a1 , a2 , ..., an , b1 , b2 , ..., bn duas permutações do conjunto de números


1, 21 , 13 , ..., n1 tais que
a1 + b1 ≥ a2 + b2 ≥ · · · an + bn .
4
Mostre que a desigualdade ak + bk ≤ k ocorre para todo k = 1, 2, ..., n.

Solução. Fixe um ı́ndice k qualquer. Suponha que no conjunto de ı́ndices {1, 2, ..., k} a
desigualdade aj ≤ bj ocorra x vezes enquanto a desigualdade aj ≥ bj ocorra y vezes. Como
x + y ≥ k, assuma sem perda de generalidade que x ≥ k/2. Defina bs como o menor dentre
os bj ’s tais que aj ≤ bj . Observe que devemos ter bs ≤ x1 , já que bj ≥ bs ocorre pelo menos
x vezes. Dessa forma,
2 2 4
ak + bk ≤ as + bs ≤ 2bs ≤ ≤ ≤ .
x k/2 k

4
POT 2012 - Combinatória - Nı́vel 2 - Aula 16 - Prof. Bruno Holanda

Problemas Propostos

Problema 6. Complete os detalhes nas demonstrações dos problemas da parte teórica que
não foram completamente detalhados.

Problema 7. Diagramas como o elaborado no primeiro problema são conhecidos como


Diagramas de Hasse para conjuntos parcialmente ordenados. Neste exercı́cio, você deve
responder a seguinte pergunta: Dado um diagrama de Hasse com n vértices, quantas das
permutações de 1, 2, ..., n satisfazem a ordem parcial dada por esse diagrama? Faça a análise
dos seguintes casos:

b b b b

b b b b
b
b b

b b b b b b b b
b b b

b b b b b b b b

b b b b

n=8 n = 11 n = 16

Problema 8. Suponha que a soma dos inteiros não negativos a1 , ..., an é igual a 1. Mostre
que existe uma permutação b1 , b2 , ..., bn desses números tal que
1
b1 b2 + b2 b3 + · · · + bn−1 bn + bn b1 ≤ .
n
Problema 9. Existem reais
√ não-negativos a1 , a2 , ..., a7 tais que a1 = a7 = 0 e ao mesmo
tempo ai+1 + ai−1 > ai 3 (2 ≤ i ≤ 6)?

Problema 10. Ache o maior valor possı́vel da soma

S = |x1 − 1| + |x2 − 2| + · · · + |xn − n|,

onde x1 , x2 , ..., xn é uma permutação de 1, 2, ..., n.

Problema 11. Suponha que exista uma sequência infinita de números reais x1 , x2 , ... tais
que para quaisquer ı́ndices m, n temos
1
|xm+n − xn − xm | < .
m+n
Mostre que esta sequência é uma progressão aritmética.

5
Programa Olímpico de Treinamento
Curso de Combinatória - Nível 2 Aula 17
Prof. Bruno Holanda

Combinatória e Divisibilidade

Continuaremos aplicando as principais ideias que aprendemos durante o curso de Com-


binatória em outras áreas da Matemática. Desta vez, abordaremos problemas que envolvam
algum conhecimento sobre Teoria dos Números.

Problema 1. (Rússia 1999) Um conjunto de números naturais é escolhido tal que entre
quaisquer 1999 números naturais consecutivos, existe um número escolhido. Mostre que
existem dois números escolhidos tais que um deles divide o outro.

Solução. Construa uma tabela com 1999 colunas e 2000 linhas. Na primeira linha escreva
1, 2, . . . , 1999. Defina as entradas das futuras linhas recursivamente como segue: suponha
que as entradas na linha i são k + 1, k + 2, . . . , k + 1999 e que seu produto é M . Preencha
a linha i + 1 com M + k + 1, M + k + 2, . . . , M + k + 1999. Todas as entradas na linha
i + 1 são maiores do que as da linha i. Além disso, toda entrada divide a entrada imedia-
tamente abaixo (e consequentemente toda entrada abaixo desta). Em cada linha existem
1999 números consecutivos, e assim cada linha contém um número escolhido. Como temos
2000 linhas, pelo princı́pio da casa dos pombos existem dois números escolhidos na mesma
coluna. Mas daı́ um deles divide o outro, como desejado. 

Problema 2. (Índia 1998) Seja M um inteiro positivo e considere o conjunto S = {n ∈


N | M 2 ≤ n < (M + 1)2 }. Prove que os produtos da forma ab, com a, b ∈ S, são todos
distintos.

Solução. Provaremos a afirmação por contradição. Suponha o contrário, isto é, que existem
a, b, c, d ∈ S tais que ab = cd. Assuma, sem perda de generalidade, que a < c, d.
Sejam p = mdc(a, c), q = a/p e r = c/p. Então mdc(q, r) = 1. Daı́, como

ab cd
q = = rd,
p p

segue que q|d. Seja agora s = d/q. Então b = cd/a = rs, de modo que a = pq, b = rs,
c = pr e d = qs, com p, q, r, s inteiros positivos.
POT 2012 - Combinatória - Nı́vel 2 - Aula 17 - Prof. Bruno Holanda

Como c > a, temos que r > q ⇒ r ≥ q + 1. Analogamente, d > a ⇒ s ≥ p + 1. Assim,

b = rs
≥ (p + 1)(q + 1)
= pq + p + q + 1

≥ pq + 2 pq + 1

= a+2 a+1
≥ M 2 + 2M + 1
= (M + 1)2 ,

uma contradição, já que b pertence a S. 

Problema 3. Mostre que existe um bloco de 2002 inteiros positivos consecutivos contendo
exatamente 150 primos. (Você pode usar o fato de que existem 168 primos menores do que
1000.)

Solução. Defina a função f : N → N por f (a) = quantidade de primos entre os números


a, a + 1, . . . , a + 2001. Como existem 168 primos de 1 até 1000, temos f (1000) > 168.
Observe que:

(i) f (a + 1) = f (a) + 1 se a é composto e a + 2002 é primo;

(ii) f (a + 1) = f (a) se ambos a e a + 2002 são compostos ou primos;

(iii) f (a + 1) = f (a) − 1 se a é primo e a + 2002 é composto.

Esses três caso são mutuamente exclusivos. Também, temos f (2003! + 2) = 0 (verifique).
Como f decresce em cada passo por no máximo 1 e parte de 168 até chegar em 0, f (n)
deve ser igual a 150 para algum n entre 1 e 2003! + 2, como querı́amos. 

Problema 4. (Rioplatense 1999) Sejam p1 , p2 , . . . , pk primos distintos. Considere todos os


inteiros positivos que utilizam apenas esses primos (não necessariamente todos) em sua
fatoração em números primos, formando assim uma seqüência infinita

a1 < a2 < · · · < an < · · · .

Demonstre que, para cada natural c, existe um natural n tal que

an+1 − an > c.

Solução. Suponha, por absurdo, que exista c > 0 tal que an+1 − an ≤ c, ∀ n ∈ N. Isso
significa que as diferenças entre os termos consecutivos de (an )n≥1 pertencem ao conjunto
{1, 2, . . . , c}, logo são finitas. Sejam d1 , d2 , . . . , dr essas diferenças. Seja αi o maior expoente
de pi que aparece na fatoração de todos os dj .

2
POT 2012 - Combinatória - Nı́vel 2 - Aula 17 - Prof. Bruno Holanda

Considere então o número M = p1α1 +1 p2α2 +1 · · · pαk k +1 . É claro que M pertence à


seqüência, ou seja, M = an , para algum n. Vejamos quem será an+1 . Por hipótese,
existe i tal que an+1 − an = di . Como an+1 > an , existe um primo pj que divide an+1 com
expoente maior ou igual a αj + 1. Caso contrário,

an < an+1 < pα1 1 +1 pα2 2 +1 · · · pαk k +1 = an ,


α +1 α +1
absurdo. Daı́, pj j |an ⇒ pj j |di , novamente um absurdo, pela maximalidade de αj .
Logo, o conjunto de todas as diferenças não pode ser finito e, portanto, dado qualquer
c > 0, existe um natural n tal que an+1 − an > c. 

Problema 5. (EUA 1998) Prove que, para cada inteiro n ≥ 2, existe um conjunto S de n
inteiros positivos tal que (a − b)2 | ab para quaisquer a e b distintos pertencentes a S.

Solução. Na hora de montar qualquer exemplo de conjunto, faça sempre casos pequenos.
Considere n = 2, n = 3, n = 4, n = 5 e veja a cara do exemplo. Lembre-se de sempre
seguir um padrão nessa hora, pois se o exemplo que você achar para 3 tiver alguma coisa
em comum com o exemplo para 2, o exemplo para 4 for parecido com o para 3 e assim por
diante. . . Ótimo! O resto sairá por indução.
Depois de fazer alguns casos pequenos, encontramos a cara do conjunto S: dado n,
construiremos S com n elementos tal que (a − b)|a e (a − b)|b para todos a, b pertencentes
a S. Um conjunto com essas propriedades claramente satisfaz o enunciado. Comece com o
conjunto {2, 3}.

Passo indutivo: suponha que encontramos um conjunto S, |S| = n, satisfazendo as condições


do enunciado. Seja m o mı́nimo múltiplo comum dos elementos de S. Tome o conjunto
S ′ = {m + S} ∪ {m} (se X é um conjunto e a é um número qualquer, o conjunto X + a ou
a+X é dado por {a+x | x ∈ X}). Logo, |S ′ | = n+1. Pelos casos particulares, desconfiamos
que S ′ satisfaz as propriedades requeridas. Vejamos. . . Sejam a′ , b′ elementos quaisquer de
S ′ . Podemos considerar dois casos:

I. a′ = m+a, b′ = m+b: então a′ −b′ = (m+a)−(m+b) = a−b. Como (a−b)|a, (a−b)|b


(pela hipótese indutiva) e m é múltiplo comum de a e b, segue que (a′ −b′ )|(m+a) = a′
e (a′ − b′ )|(m + b) = b′ .

II. a′ = m + a, b′ = m: então a′ − b′ = (m + a) − m = a ⇒ (a′ − b′ )|a′ e (a′ − b′ )|b′ , já


que m é múltiplo de a.

Pronto! 

3
POT 2012 - Combinatória - Nı́vel 2 - Aula 17 - Prof. Bruno Holanda

Problemas Propostos

Problema 6. (Ibero 1998) Encontre o menor natural n com a propriedade de que en-
tre quaisquer n números distintos do conjunto {1, 2, 3, . . . , 999} podemos encontrar quatro
números distintos a, b, c, d tais que a + 2b + 3c = d.

Problema 7. (Romênia 1999) Seja p(x) = 2x3 − 3x2 + 2, e sejam

S = {P (n) | n ∈ N, n ≤ 1999},
T = {n2 + 1 | n ∈ N},
U = {n2 + 2 | n ∈ N}.

Prove que S ∩ T e S ∩ U têm o mesmo número de elementos.

Problema 8. (Polônia 2000) A seqüência p1 , p2 , . . . de números primos satisfaz a seguinte


condição: para cada n maior ou igual a 3, pn é o maior divisor primo de pn−1 + pn−2 + 2000.
Prove que a seqüência é limitada.

Problema 9. (Rússia 2000) Prove que o conjunto de todos os inteiros positivos pode ser
particionado em 100 subconjuntos não-vazios de modo que se três inteiros positivos satis-
fazem a + 99b = c, então dois deles pertencem ao mesmo subconjunto.

Problema 10. (Lista Cone Sul 2007) Um subconjunto M de {1, 2, 3, . . . , 15} não contém
três elementos cujo produto é um quadrado perfeito. Determine o número máximo de ele-
mentos de M .

Problema 11. (Reino Unido 1999) Para cada inteiro positivo n, seja Sn = {1, 2, . . . , n}.

(a) Para quais valores de n é possı́vel expressar Sn como união de dois subconjuntos não-
vazios disjuntos tais que a soma dos elementos de cada subconjunto é a mesma?

(b) Para quais valores de n é possı́vel expressar Sn como união de três subconjuntos não-
vazios disjuntos tais que a soma dos elementos de cada subconjunto é a mesma?

Problema 12. (Irã 1999) Seja S = {1, 2, . . . , n} e sejam A1 , A2 , . . . , Ak subconjuntos de S


tais que, para quaisquer 1 ≤ i1 , i2 , i3 , i4 ≤ k, temos

|Ai1 ∪ Ai2 ∪ Ai3 ∪ Ai4 | ≤ n − 2.

Prove que k ≤ 2n−2 .

4
Programa Olímpico de Treinamento
Curso de Combinatória - Nível 2 Aula 18
Prof. Bruno Holanda

Miscelânea I

Nas duas próximas aulas faremos um conjunto de problemas que irão fixar as idéas
aprendidas durante o curso. Além disso, não saber previamente o assunto no qual cada
problema se encaixa serve como treinamento para os diversos exames que existem atual-
mente.
Problema 1. Dados 7 inteiros positivos distintos cuja soma é 100, prove que podemos es-
colher três deles cuja soma é pelo menos 50.

Este primeiro problema parece ser um problema cuja solução é dada através do princı́pio
da casa dos pombos. Porém, vamos usar uma idéia um pouco menos conhecida, mas que é
de fundamental importância conhecer: o Arranjo em Ordem.

Solução. Sejam a < b < c < d < e < f < g estes números. Vamos mostrar que
e + f + g ≥ 50. Se e > 15, então e + f + g ≥ 16 + 17 + 18 = 51. Caso e ≤ 15, então
a + b + c + d ≤ 14 + 13 + 12 + 11 = 50; portanto e + f + g = 100 − a − b − c − d ≥ 50. 

Problema 2. Dados 2n + 3 pontos no plano, não três colineares e não quatro concı́clicos,
prove que existe um cı́rculo passando por três destes pontos de modo que existam n pontos
no seu interior e n pontos no seu exterior.

Solução. Sejam A e B dois pontos consecutivos do fecho convexo. Denote os demais


pontos por P1 , P2 , ..., P2n+1 de modo que ∠APi B > ∠APi+1 B. Isso pode ser feito sem
problemas, já que não há quatro pontos concı́clicos. Note que o cı́rculo que passa pelos
pontos A, B, Pn+1 satisfaz às condições do problema. 

Problema 3. Prove que existe um conjunto S de 31000 pontos no plano tal que, para cada
ponto P de S, existem pelo menos 2000 pontos em S cuja distância para P é exatamente
uma unidade.

Solução. Vamos mostrar, por indução sobre k, que existe um conjunto Sk de 3k pontos
no plano tal que cada ponto de Sk dista uma unidade de pelo menos 2k pontos de Sk .
POT 2012 - Combinatória - Nı́vel 2 - Aula 18 - Prof. Bruno Holanda

Tomando k = 1000, provamos o problema.

Caso inicial: tome S1 como os vértices de um triângulo eqüilátero de lado 1.

Hipótese de indução: suponha a existência de Sk como acima, para algum k ≥ 1.

Passo indutivo: seja v um vetor. A partir de cada ponto P de Sk , construa um triângulo


eqüilátero ∆P de lado 1 na direção de v. Se v puder ser escolhido de modo que os vértices
de ∆P e ∆Q não coincidam sempre que P 6= Q, então o conjunto Sk+1 formado pelos
vértices de todos os triângulos ∆P , P ∈ Sk , satisfaz o problema. Para mostrar isso, note
inicialmente que Sk+1 nada mais é do que a união de Sk e de duas translações de Sk de uma
unidade cujas direções formam um ângulo de 60o . Assim, Sk+1 tem 3 · 3k = 3k+1 pontos, e
cada ponto de Sk+1 dista uma unidade de pelo menos 2(k + 1) pontos de Sk+1 : 2k devido
à propriedade de Sk e 2 devido à construção dos triângulos ∆P .
Para escolher v, basta tomá-lo em uma direção distinta de todas as direções de pares
de pontos de Sk . Isso pode ser feito porque a quantidade de direções de pares de pontos
de Sk é finita. 

Problema 4. (Rioplatense 2003) Sobre uma mesa tem-se n ≥ 2 bolsas de plástico, todas de
cores distintas. Cada uma está em contato com a mesa ou está dentro de outra bolsa. A
operação permitida é escolher uma bolsa que está em contato com a mesa, retirar todas as
bolsas do seu interior e coloca-las sobre a mesa e colocar todas as outras bolsas que estavam
fora e colocar no seu interior (sem modificar o conteúdo das outras bolsas). Determine o
total de configurações diferentes que podem ser obtidas utilizando a operação quantas vezes
o necessário.

Solução. Construa um grafo com n vértices, onde cada vértice representa uma bolsa.
Vamos ligar dois vértices vi , vj se as bolsas bi , bj estão imediatamente uma dentro da outra.
O grafo será algo semelhante ao grafo abaixo.

Agora construa um novo vértice F e ligue-o a todos os vértices que representam as


bolsas que estão sobre a mesa. Note que aplicar a operação, no grafo representa trocar a
posição do vértice F pela posição do vértice que foi operado. E como o grafo possui um
total de n + 1 vértices, existem ao todo n + 1 configurações.

2
POT 2012 - Combinatória - Nı́vel 2 - Aula 18 - Prof. Bruno Holanda

=⇒
v4 v4
v1 v2 v3 F v2 v3

F v1

A figura acima mostra a troca das posições dos vértices F e v1 . Os vértices v1 , v2 , v3


representam as três bolsas que estavam inicialmente sobre a mesa. Note que após aplicar a
operação, os vértices que ficam ligados a F são v1 e v4 , e as bolsas que ficam sobre a mesa
são exatamente b1 e b4 .

Problema 5. (URSS 1990) Suponha que existam 1990 pilhas, consistindo de 1, 2, ..., 1990
pedras, respectivamente. Em um movimento pode-se escolher algumas pilhas (possivel-
mente apenas uma) e retirar de cada pilha escolhida uma quantidade fixa de pedras. Qual
o menor número de movimentos necessários para se retirar todas as pedras de todas as
pilhas?

Solução. Separe as pilhas em grupos com a mesma quantidade pedras, onde pilhas vazias
formam seu próprio grupo. Dessa forma, inicialmente temos 1990 grupos distintos. Supo-
nha que em um dado momento existam n grupos de pilhas e realizamos um movimento que
consiste em retirar uma mesma quantidade de pedras de pilhas que estão em k diferentes
grupos. Este grupo de pilhas que realizamos a operação continua representando k diferen-
tes grupos. Além disso, as demais pilhas dos demais n − k grupos que não foram alterados
continuam representando n − k grupos distintos. Dessa forma, após realizar um movimento
que altera pilhas de k grupos, temos pelo menos max(n−k, k) grupos no próximo momento.
Portanto, o número de diferentes grupos não decresce mais rápido do que a sequência:

995, 498, 249, 125, 63, 32, 16, 8, 4, 2, 1.

Assim, temos que utilizar pelo menos 11 movimentos.

Agora iremos demonstrar que é possı́vel retirar todas as pedras utilizando exatamente
11 movimentos. Seja mn o movimento que retira n pedras de cada pilha que possui n
pedras ou mais. Agora considere a sequência de movimentos:

m995 , m498 , m249 , m125 , m63 , m32 , m16 , m8 , m4 , m2 , m1 .

É fácil verificar que esta sequência é capaz de retirar toas as pedras. 

3
POT 2012 - Combinatória - Nı́vel 2 - Aula 18 - Prof. Bruno Holanda

Problemas Propostos

Problema 6. Considere um conjunto de 2n + 2 pontos no plano, não três colineares. Prove


que podemos escolher dois destes pontos de modo que a reta que os une divive o restante
do conjunto em duas partes com n pontos cada.

Problema 7. Dados 69 inteiros positivos distintos menores do que 101, prove que podemos
escolher quatro deles a, b, c, d tais que a < b < c e a + b + c = d. Este fato continua
verdadeiro para 68 números?

Problema 8. (USAMO 1996) Dados n inteiros positivos, considere todas as possı́veis somas
formadas por um ou mais deles. Prove que todas estas somas podem ser divididas em n
grupos tais que em cada grupo a razão entre o maior elemento e o menor não excede 2.

Problema 9. Prove que qualquer triângulo pode ser particionado em n ≥ 4 triângulos


isósceles.

Problema 10. Prove que um quadrado pode ser particionado em n ≥ 6 quadrados menores.
Prove que isso não pode ser feito para n = 5.

Problema 11. (Romênia 1978) Mostre que um cubo pode ser particionao em n ≥ 55 cubos
menores.

Problema 12. Um retângulo R é coberto por retângulos menores (com lados paralelos ao
do retângulo R) cada um deles com pelo menos um lado inteiro. Prove que R também
possui um lado inteiro.

Problema 13. (São Petersburgo 2000) Em um tabuleiro infinito são colocados 111 L−triminós
sem sobreposiçao de modo que qualquer quadrado 2 × 2 que cubra um deles seja coberto
totalmente por esses L−triminós. Prove que podemos retirar alguns deles (mas não todos)
de modo que a propriedade continue válida.

Problema 14. (São Petersburgo 2000) Em cada casa de um tabuleiro 8×8 é escrito um real
positivo tal que a soma dos números em cada linha é 1. Sabe-se que para quaisquer oito
quadrados (não dois na mesma linha ou coluna) o produto dos números nestes quadrados
não é maior que o produto dos números da diagonal principal. Prove que a soma dos
números na diagonal principal é pelo menos 1.

4
Programa Olímpico de Treinamento
Curso de Combinatória - Nível 2 Aula 19
Prof. Bruno Holanda

Miscelânea II

Como prometido, em nesta última aula do treinamento em combinatória para alunos


do nı́vel 2, iremos continuar resolvendo alguns problemas diversos. Muitos dos exemplos
a seguir foram retirados de treinamentos anteriores para a olimpı́ada do Cone Sul. Dessa
forma, acreditamos que um aluno que utilizou os materiais deste curso como referência
para um treinamento inicial em combinatória está apto a participar da seletiva da Cone
Sul de forma competitiva.

Problema 1. (Komal 2007) Na floresta onde vivem os Smurfs, Gargamel plantou 1280 pi-
nheiros, cada um com 1 metro de diâmetro. A floresta é um campo retangular de dimensões
1001 × 945 metros. Vovô Smurf gostaria de construir nela sete campos de tênis, cada um
deles de dimensões 20 × 34 metros. Vovô Smurf conseguirá fazer a construção sem derrubar
nenhuma árvore, independente de como Gargamel plante as árvores?

Solução. Vovô Smurf, seja esperto e utilize a estratégia abaixo.

Divida a floresta em k retângulos de dimensões 21 × 35. Para cada pinheiro plantado


por Gargamel, associe o retângulo que contém o centro do pinheiro (se o pinheiro pertencer
a mais de um retângulo, associe qualquer um deles). Dessa maneira, em cada retângulo
que não está associado a nenhum pinheiro, é possı́vel construir um campo de tênis no
subretângulo central de dimensões 20 × 34.

Portanto, se k ≥ 1287, Vovô Smurf conseguirá, pelo Princı́pio da Casa dos Pombos, realizar
seu objetivo. E de fato isso é possı́vel, através da seguinte divisão:
POT 2012 - Combinatória - Nı́vel 2 - Aula 19 - Prof. Bruno Holanda

• particione o tabuleiro 980 × 945 em


980 945
· = 28 · 45 = 1260
35 21
subretângulos de dimensões 35 × 21.
• particione o tabuleiro 21 × 945 remanescente em
945
= 27
35
subretângulos de dimensões 21 × 35.

···

···

Isso conclui o afirmado.

Problema 2. Em um torneio de tênis com 14 jogadores, cada um joga com todos os outros
exatamente uma vez e não há empates. Prove que é possı́vel escolher três jogadores para
os quais qualquer um dos outros onze times perdeu para pelo menos um desses três.

Sejam P1 , . . . , P14 os jogadores e suponha que P1 foi quem mais venceu. O total de
partidas é  
14
= 91
2
e portanto, pelo Princı́pio da Casa dos Pombos, P1 venceu pelo menos 7 delas. Vamos
analisar os casos.

(i) P1 venceu 7 partidas: podemos supor que P1 venceu de P2 , . . . , P8 e perdeu de


P9 , . . . , P14 . Entre esses seis últimos, ocorreram
 
6
= 15
2

2
POT 2012 - Combinatória - Nı́vel 2 - Aula 19 - Prof. Bruno Holanda

partidas e novamente, pelo Princı́pio da Casa dos Pombos, algum deles, digamos P9 ,
venceu pelo menos três das quatro partidas disputadas com P10 , . . . , P14 . Suponha
que P9 venceu de P10 , P11 e P12 . Daı́, se P13 venceu de P14 , os jogadores P1 , P9 e P13
satisfazem as condições do problema.
(ii) P1 venceu 8 partidas: suponha que P1 venceu de P2 , . . . , P9 e perdeu de P10 , . . . , P14 .
Entre P10 , . . . , P14 , ocorreram  
5
= 10
2
partidas e daı́ algum deles venceu duas das quatro partidas disputadas. Sem perda
de generalidade, se P10 ganhou de P11 , P12 e P13 ganhou de P14 , a tripla P1 , P10 , P13
satisfaz o requerido.
(iii) P1 venceu 9 partidas: se P1 venceu de P2 , . . . , P10 , perdeu de P11 , . . . , P14 e, além
disso, P11 venceu de P12 e P13 venceu de P14 , a tripla P1 , P11 , P13 tem as propriedades
procuradas. 

Problema 3. Sejam P1 , P2 , . . . , Pn pontos sobre uma circunferência. Entre cada par de


pontos, existe um segmento de reta pintado de vermelho ou azul de modo que Pi Pj é
vermelho se e somente se Pi+1 Pj+1 é azul, para quaisquer ı́ndices 1 ≤ i < j ≤ n (aqui,
Pn+1 = P1 ).
(a) Para quais valores de n tal coloração é possı́vel?
(b) Mostre que tais colorações têm a seguinte propriedade:

”Dados quaisquer dois pontos, existe uma linha poligonal de no máximo três
segmentos, todos vermelhos, unindo esses dois pontos.”

Solução. (a) Vamos mostrar que a coloração é possı́vel se e somente se n é múltiplo de 4.


Se P1 P2 é azul, então P2 P3 é vermelha e, por indução,

Pi Pi+1 é azul ⇐⇒ i é ı́mpar.

Como Pn+1 Pn+2 = P1 P2 , n + 1. Daı́, n = 2m e, aplicando o mesmo raciocı́nio ao par de


ı́ndices (1, 1 + m), obtemos que

Pi Pi+m tem a mesma cor de P1 P1+m ⇐⇒ i é ı́mpar

de modo que m é par, pois P1+m P1+2m = P1 P1+m .

Reciprocamente, se n = 4k, pinte as arestas

Pi Pi+1 , Pi Pi+2 , . . . , Pi Pi+2k

de azul se i é par e de vermelho caso contrário. Note que a coloração está bem-definida,
pois i e i + 2k têm a mesma paridade e portanto a cor da aresta

Pi+m Pi+2m

3
POT 2012 - Combinatória - Nı́vel 2 - Aula 19 - Prof. Bruno Holanda

coincide com a de Pi Pi+m . É claro que a pintura acima satisfaz as condições do problema.

(b) Seja Pi Pj a aresta considerada. Se ela for vermelha, não há nada a fazer. Suponha que
ela seja azul. Então Pi−1 Pj−1 e Pi+1 Pj+1 são vermelhas. Alguma das arestas Pi−1 Pi ou
Pi Pi+1 é vermelha. Sem perda de generalidade, suponha que Pi−1 Pi seja vermelha. Vamos
analisar dois casos.

(i) j − i é par: a aresta Pj−1 Pj é vermelha e portanto o caminho

Pi −→ Pi−1 −→ Pj−1 −→ Pj

é monocromático vermelho.

(ii) j − i é ı́mpar: a aresta Pj Pj+1 é vermelha. Alguma das arestas Pi−1 Pj ou Pi Pj+1 é
vermelha. Se for a primeira,

Pi −→ Pi−1 −→ Pj

é um caminho monocromático vermelho de tamanho 2. Senão, Pi Pj+1 é vermelha e


o caminho monocromático requerido é

Pi −→ Pj+1 −→ Pj .

Problema 4. Seja n ≥ 2 um inteiro. Cada ponto de uma circunferência é colorido com uma
dentre n cores. Prove que existe um trapézio inscrito na circunferência com todos os seus
vértices pintados da mesma cor.

Solução. Vamos tomar um polı́gono regular inscrito na circunferência com uma quantidade
k suficientemente grande de vértices de modo que existam dois pares de vértices que definem
arcos de comprimentos iguais e pintados da mesma cor.
Tome k = (n + 1)(n2 + 1) e divida o polı́gono em n2 + 1 blocos de vértices consecu-
tivos. Pelo Princı́pio da Casa dos Pombos, cada bloco contém dois vértices pintados da
mesma cor c. Se esses vértices definem um arco de tamanho l, associe o par (c, l) ao bloco.
Cada coordenada desse par pode assumir n valores, visto que c representa uma cor e l
representa uma distância entre dois vértices de um bloco. Assim, novamente pelo Princı́pio
da Casa dos Pombos, existem dois blocos associados ao mesmo par. Isso conclui a prova. 

4
POT 2012 - Combinatória - Nı́vel 2 - Aula 19 - Prof. Bruno Holanda

Problemas Propostos

Problema 5. (Rússia) É possı́vel colocar 1965 pontos em um quadrado de lado 1 de ma-


neira que qualquer retângulo de área 1/200 contido no quadrado e com lados paralelos aos
lados dele contenha pelo menos um desses pontos?

Problema 6. (USAMO 1989) Um torneio de xadrez com 20 jogadores teve 14 partidas.


Sabendo que cada competidor jogou pelo menos uma vez, prove que 6 dessas partidas con-
taram com 12 jogadores diferentes.

Problema 7. (IMO 1985) Sejam n, k inteiros positivos primos entre si, com k < n. Pin-
tamos cada número em M = {1, 2, 3, . . . , n − 1} de azul ou branco, de modo que i e n − i
têm a mesma cor. Sabendo também que, se i 6= k, então i e |i − k| têm a mesma cor, prove
que todos os números em M têm a mesma cor.

Problema 8. Quantos são as triplas ordenadas (a, b, c) formadas por inteiros positivos tais
94!
que a + b + c = 94 e seja divisı́vel por 3?
a!b!c!

Problema 9. (Baltic Way 2005) Uma tabela possui n linhas e 6 colunas, onde n ≥ 2.
Em cada casa está escrito 0 ou 1. Todas as linhas são diferentes uma das outras. Além
disso, para cada para de linhas (x1 , x2 , ..., x6 ) e (y1 , y2 , ..., yn ), a linha (x1 y1 , x2 y2 , ..., x6 y6 )
também pode ser encontrada na tabela. Prove que existe uma coluna em que pelo menos
metade das casas são zeros.

Problema 10. (Macedônia 2003) Um tabuleiro 2003 × 2003 pode ser coberto por dominós
1 × 2 horizontais e peças 3 × 1 verticais?

1 1 1
Problema 11. Os números 1, , , ..., estão escritos no quadro negro. Gugu escolhe
2 3 2012
quaisquer dois destes números x e y, apága-os e em seguida escreve x + y + xy no quadro.
Ele faz isso até que sobre apenas um número no quadro. Quais são os possı́veis valores do
último número escrito?

5
Combinatória 02 - Lógica 2

Problema 5. São dadas 4 moedas aparentemente iguais. Sabe-se que uma delas é falsa (tem
peso diferente das demais e não se sabe se ela é mais leve ou mais pesada). Mostre como
descobrir a moeda falsa com 2 pesagens em uma balança de dois pratos.

Solução. Sejam A, B, C e D as moedas. Primeiramente pesamos A e B. Se a balança


equilibrar, então ou C ou D são falsas. Neste caso pesamos A e C: se equilibrar D é falsa,
caso contrário C é falsa. Agora, se a pesagem de A e B não equilibrar a balança então ou
A ou B são falsas. Fazemos então a pesagem de A e C. Se equilibrar então B é falsa, caso
contrário A é falsa.

Problema 6. Mostre que é possı́vel dispor os números de 1 a 16 em sequência de modo que a


soma de dois números vizinho seja sempre um número quadrado perfeito.

Solução. Basta tomar a sequência:

8 − 1 − 15 − 10 − 6 − 13 − 12 − 4 − 5 − 11 − 14 − 2 − 7 − 9 − 16

Problema 7. Victor e Maria começam a trabalhar no mesmo dia. Victor trabalha 3 dias segui-
dos e depois tem um dia de descanso. Maria trabalha 7 dias seguidos e descansa os outros 3.
Quantos dias de descanso em comum tiveram os dois durante os 1000 primeiros dias.

Solução. O primeiro dia de trabalho de Victor é da forma (3 + 1)k + 1 = 4k + 1 e o primeiro


dia de trabalho de Maria é da forma (7 + 3)k 0 + 1 = 10k 0 + 1.

Fazendo 10k 0 + 1 = 4k + 1 temos 5k 0 = 2k e então k 0 = 2t, k = 5t para algum t natural.


Sendo assim 10k 0 + 1 = 4k + 1 = 20t + 1.
Isto é, os dias em que os dois começam a trabalhar juntos são da forma 20t + 1.
Temos então um ciclo de 20 dias.

Nos primeiros 20 dias Victor descansa nos dias 4, 8, 12, 16, 20 e Maria descansa nos dias 8, 9,
10, 18, 19 e 20.
Então há 2 dias de descanso em comum a cada ciclo de 20 dias.
1000
Portanto, nos primeiros 1000 dias há 2 × = 100 dias de descanso em comum.
20
Problema 8. Como recortar um retângulo 3 × 13 em treze retângulos menores distintos?

Solução. A área total é 3 × 13 = 39. Vamos pensar nos retângulos distintos de menor área.

Área 1: 1×1 Área 5: 1×5


Área 2: 1×2 Área 6: 2×3 e 1×6
Área 3: 1×3 Área 7: 1×7
Área 4: 2×2 e 1×4 Área 8: 2×4 e 1×8
Somando as áreas dos 11 retângulos de menor área temos uma área de 1 + 2 + 3 + 4 + 4 +
5 + 6 + 6 + 7 + 8 + 8 = 54 > 39. Dessa forma, 13 retângulos distintos sempre terão área total
maior que 39 e não é possivel fazer a divisão desejada.

Problema 9. (Olimpiada de Maio) Num ano que tem 53 sábados, que dia da semana é 12 de
maio? Diga todas as possibilidades.

Solução. Veja que 365 = 52 × 7 + 1. Logo, o ano tem 52 semanas e 1 dia. Para que ocorram
53 sábados, o dia que sobra - o último dia do ano - deve ser um sábado. Como o último dia
do ano é da forma 7k+1 (k = 52), concluimos que todo dia da forma 7k+1 também será sábado.

Agora note que Janeiro tem 31 dias, Fevereiro tem 28, Março tem 31 e Abril 30. Dessa forma,
como 31 + 28 + 31 + 30 + 12 = 132, 12 de maio é o 132º dia do ano. Como 134 = 7 × 19 + 1
o dia 14 de maio é um sábado e portanto 12 de maio é uma quinta-feira.

Problema 10. Um número é dito lindo se é divisı́vel por cada um dos seus dı́gitos não nulos.
Qual é a maior quantidade de números lindos que pode existir?

Solução. Existem infinitos números lindos. Basta considerar os números da forma 222 . . . 22
em que o número de dı́gitos varia entre os naturais. Esse número será sempre divisı́vel por seu
dı́gito 2.

Problema 11. (Búlgaria 2005) Ivo escreve todos os inteiros de 1 a 100 (inclusive) em cartas
e dá algumas delas para Iana. Sabe-se que para quaisquer duas destas, uma de Ivo e outra
de Iana, a soma dos números não está com Ivo e o produto não está com Iana. Determine o
número de cartas de Iana sabendo que a carta 13 está com Ivo.

Solução. Iana possui pelo menos uma carta, digamos a carta com o número k. Se 1 está com
Ivo, o produto 1 · k = k não está com Iana, que é uma contradição.
Logo, 1 está com Iana.
Se 12 está com Ivo, a soma 1 + 12 = 13 não está com Ivo, que também é uma contradição.
Logo, 12 está com Iana.

2
Agora, se as cartas 3 e 4 estiverem com pessoas diferentes o produto 3 · 4 = 12 não estará com
Iana. Porém, acabamos de ver que 12 está com Iana. Daı́, 3 e 4 estão com a mesma pessoa.
Se ambas estiverem com Ivo, a soma 1 + 3 = 4 não está com Ivo, contadição. Logo, 3 e 4 estão
com Iana. Consequêntemente, 10 e 9 também estão com Iana, pois a soma 10 + 3 = 9 + 4 = 13
estão com Ivo.

Agora, veja que se r está com Iana então 13 + r não está com Ivo e portanto está com Iana.
Do mesmo modo, (13 + r) + 13 = 13 · 2 + r está com Iana e o mesmo vale para 13 · 3 + r. De
modo geral, se r está com Iana então todos os números da forma 13 · k + r estão com Iana.
Sendo assim, já vimos que 13k + 1, 13k + 3, 13k + 4, 13k + 12 estão com Iana.

Se 2 estiver com Ivo então 2 + 3 = 5 está com Iana. Dai 5 · 2 = 10 está com Ivo, do mesmo
modo 10 · 3 = 30 e consequentemente 30 · 3 = 90 está com Ivo. Mas 90 = 13 · 6 + 12 e como
vimos esse número está com Iana, contradição. Logo, 2 está com Iana e consequentemente
13k + 2 está com Iana.

Veja que 5 · 6 = 30 = 13 · 2 + 4 está com Iana. Logo, 5 e 6 devem estar com a mesma pessoa.
Mas se 5 está com Ivo então 1 + 5 = 6 não pode estar com Ivo, contradição. Logo, 5 e 6 estão
com Iana e portanto 13k + 5 e 13k + 6 estão com Iana.
Analogamente 7 · 8 = 56 = 13 · 4 + 4 está com Iana e portanto 13k + 7 e 13k + 8 estão com
Iana.
O mesmo vale para 9 e 10 já que 9 · 10 = 90 = 13 · 6 + 12.
E, por último, como 6 · 11 = 66 = 13 · 5 + 1 está com Iana, 11 e 6 devem estar com a mesma
pessoa e portanto estão com Iana.
Concluimos que Iana está com todos os números que não multiplos de 13.
Como 13 · 2, . . . 13 · 7 não estão com Iana, Ivo tem todos os múltiplos de 13.
Dessa forma, Iana tem 100 − 7 = 93 cartas.

Problema 12. (Rússia 1999) Mostre que os números de 1 a 15 não podem ser divididos em
um grupo A de dois elementos e um grupo B de 13 elementos tais que a soma dos elementos
de B seja igual ao produto dos elementos de A.

Solução. Suponha que o grupo A tenha os dois números distintos x e y. Então a soma dos
elementos de B é 1 + 2 + . . . + 15 − x − y = 120 − x − y.
Queremos que 120 − x − y = xy ⇐⇒ 112 = 121 = x + y + xy + 1 = (x + 1)(y + 1).
As únicas soluções nos naturais são (x, y) = (10, 10), (120, 0).
Nenhum dos dois pares são possı́veis pois x 6= y e x, y ≤ 15.
Logo, não é possı́vel fazer a divisão desejada.

3
Problema 13. (Seletiva Rioplatense 2004) Em cada casa de um tabuleiro 8 × 8 escrevemos um
número inteiro. Sabe-se que para cada casa, a soma dos seus vizinhos é 1. Encontre a soma
de todos os números do tabuleiro.
Obs: Consideramos vizinhas casas com um lado em comum.

Solução. Observe as casas marcadas no tabuleiro abaixo:


? ? ? ?

? ? ? ?
? ?
? ?
? ?
? ?
? ? ? ?

Se olharmos para os vizinhos das casas marcadas acima, vemos que eles cobrem todo o tabuleiro
e de maneira disjunta! Como a soma dos vizinhos de cada casa é 1, a soma total dos números
do tabuleiro será igual ao número de casas marcadas, que é 20.

Problema 14. Etevaldo pensou em cinco números distintos e escreveu no quadro todos dez
números que são somas de dois destes cinco números. Será que Ovozildo pode descobrir os
números que Etevaldo pensou observando apenas os números escritos no quadro?

Solução. Sim. Suponha que os números sejam a, b, c, d, e com a ≤ b ≤ c ≤ d ≤ e.


As somas serão:
x1 =a+b x6 = b + d
x2 =a+c x7 = b + e
x3 =a+d x8 = c + d
x4 =a+e x9 = c + e
x5 =b+c x10 = d + e
Veja que as menores somas são x1 ≤ x2 e as maiores são x10 ≥ x9 .
Portanto, ordenando as 10 somas em ordem crescente já descobrimos quem são x1 , x2 , x9 , x10 ,
basta tomar as duas primeiras e as duas últimas somas.
Além disso, temos que

x1 + . . . + x10 = 4(a + b + c + d + e) = 4(x1 + x10 + c).

Assim encontramos c.
Usando c e x9 encontramos e = x9 − c e com x10 também podemos encontrar d = x10 − e.
Usando x2 e c temos a = x2 − c e analogamente encontramos b = x1 − a.
Isso nos fornece todos os cinco números.
Combinatória 04 - Contagem I

Problema 8. Numa sala existem 3 homens e 4 mulheres. De quantos modos é possı́vel seleci-
onar um casal?

Solução. São 3 × 4 = 12 maneiras de selecionar um casal.

Problema 9. Cada casa de um tabuleiro 2 × 2 pode ser pintado de verde ou amarelo. De


quantas maneiras podemos pintar o tabuleiro todo?

Solução. O tabuleiro tem 4 casas ao todo e cada uma pode ser pintada de duas maneiras. O
número de maneiras de pintar é 24 = 16.
Obs.: Se considerarmos as rotações do tabuleiro a resposta é 4.

Problema 10. (OBM 2004) De quantos modos diferentes podemos pintar (usando apenas uma
cor) as casas de um tabuleiro 4 × 4 de modo que cada linha e cada coluna possua exatamente
uma casa pintada?

Solução. Para a primeira linha temos 4 casas disponı́veis, no segunda linha só temos 3 já que
não podemos ocupar a mesma coluna da casa pintada anteriormente. Para a 3ª linha temos 2
possibilidades e para a 4ª linha só há 1 possibilidade. Logo, a resposta é 4! = 24.

Problema 11. Quantos números naturais de três algarismos distintos existem?

Solução. Seja abc esse número. Então a pode ser 1, 2, . . . , 9 e b, c podem ser 0, 1, . . . , 9.
Inicalmente escolhendo a temos 9 opções. Para b também temos 9 já que ele nao pode ser
igual a a mas pode ser 0. Para c temos 8 possibilidades.
A resposta é 9 · 9 · 8 = 648.

Problema 12. De quantos modos podemos por três torres de três cores diferentes em um
tabuleiro 8 × 8 de modo que nenhuma delas ataque outra?

Solução. Temos 64 maneiras de escolher a posição da primeira torre, 49 para a segunda e 36


para a terceira. Total de maneiras é 64 · 49 · 36 = 112896.

Problema 13. Uma embarcação deve ser tripulada por oito homens, dois dos quais só remam
do lado direito e um apenas do lado esquerdo. Determine de quantos modos esta tripulação
pode ser formada, se de cada lado deve haver quatro homens.
Obs : A ordem dos homens deve ser considerada.

Solução. Do lado direito já estão definidos 2 homens e do lado esquerdo já está definido 1
homem.
Sobraram 5 homens. Desses, devemos escolher 2 para o lado direito e o resto vai para o
esquerdo.
5·4 5!
Temos = maneiras de escolher esses homens sem se preocupar, por enquanto, com
2! 3! · 2!
a ordem (dividimos por 2! para retirar a ordenação).
Uma vez definido quem vai ficar do lado direito e esquerdo, temos 4! maneiras de permuta-los
5!
em cada lado. Portanto a resposta é · 4! · 4! = 5! · 4 · 4 · 3 = 5760.
3! · 2!

Problema 14. (IMG) De quantas maneiras podemos ir de A até B sobre a seguinte grade sem
passar duas vezes pelo mesmo local e sem mover-se para esquerda? A figura abaixo mostra um
caminho possı́vel.

Solução. A formiga deve ir para direita extamente 5 vezes. Ao escolhermos esses movimen-
tos, o resto do caminho estará bem definido. Como podemos escolher cada um destes cinco
movimentos de seis maneiras, o total de caminhos será 6 · 6 · 6 · 6 · 6 = 65 .

Problema 15. Ache a quantidade números de quatro dı́gitos tais que toda sequência de três
algarismos consecutivos é formada por elementos distintos.

Solução. Considere o número com representação decimal abcd. As única sequêcias de algarismo
consecutivos são (a, b, c) e (b, c, d).
Como a não pode ser 0 temos para ele 9 possı́veis valores.
Para b temos também 9 possı́veis valores, já que pode ser igual a 0 mas não a a.
Para c temos 8 possiveis valores, pois nao pode ser igual a b nem a.
Agora, d não pode ser igual nem a c nem a b. Portanto tem 8 possı́veis valores.
A quantidade de números é 9 · 9 · 8 · 8 = 5184.

Problema 16. (OBM 2005) Num tabuleiro quadrado 5 × 5, serão colocados três botões
idênticos, cada um no centro de uma casa, determinando um triângulo. De quantas ma-
neiras podemos colocar os botões formando um triângulo retângulo com catetos paralelos às
bordas do tabuleiro?

Solução. Vamos primeiramente escolher o vértice oposto a hipotenusa do triângulo. Temos


25 maneiras de fazer isso. Escolhido o primeiro vértice devemos escolher uma casa na mesma
coluna e outra na mesma linha, determinando o triângulo. Podemos fazer isso de 4 × 4 = 16
maneiras.
Logo, o número de triângulos é 25 × 16 = 400.

2
Problema 17. Dizemos que a palavra algoritmo é um anagrama da palavra logaritmo pois é
uma permutação da letras de logaritmo. Sabendo disso, calcule a quantidade de anagramas
da palavra vetor.

Solução. Como não há letras repetidas, o número de anagramas é o número de permutações
das letras. Logo, é 5! = 120.

Problema 18. Quantos anagramas da palavra vetor termina em uma vogal?

Solução. Imagine que o anagrama seja da forma ABCDE, então só podemos ter E igual a e
ou o. Além disso, todas as letras são diferentes.
Temos 2 escolhas para E, sobram 4 escolhas para D, 3 para C, 2 para B e A fica determinado.
A resposta é 2 × 4! = 48.

Problema 19. De quantas maneiras é possı́vel colocar em uma prateleira 5 livros de matemática,
3 de fı́sica e 2 de biologia, de modo que livros de um mesmo assunto permaneçam juntos?

Solução. Considere os três blocos formados por livros da mesma matéria. Podemos organizar
esses blocos de 3! maneiras. Agora, em cada bloco ainda podemos permutar seus livros. Assim,
o número correto de maneiras é 5! · 3! · 2! · 3!.

Problema 20. Quantos anagramas da palavra vetor possuem as vogais separadas?

Solução. A palavra vetor possui 5! = 120 anagramas. Usando a mesma idéia do problema 19
(separar em blocos), podemos achar que a quantidade destes anagramas com vogais juntas é
2 × 4! = 48. Logo, temos 120?48 = 72 anagramas com as vogais separadas.

Problema 21. De quantas formas podemos colocar 4 bolas verdes e 4 bolas amarelas em um
tabuleiro 4 × 4 de modo que cada coluna e cada linha possua exatamente uma bola de cada cor.

Solução. Existem 4! maneiras de colocar as bolas verdes. Depois disso, escolha uma das bolas
verdes. Ponha uma bola amerela na sua linha e uma na sua coluna. Note que, ao fazermos
isto, as posições das outras duas bolas amarelas estará bem definida. Dessa maneira, temos
um total de 4! · 3 · 3 = 216 configurações.

Problema 22. Responda os itens a seguir:


a) Ache a quantidade de divisores positivos de 3600.

b) Quantos desses divisores são pares?

c) Quantos são quadrados perfeitos?

3
Solução.
a) Veja que 3600 = 24 · 32 · 52 . Seus divisores são da forma 2a · 3b · 5c , onde a = 0, 1, 2, 3, 4,
b = 0, 1, 2 e c = 0, 1, 2. Logo, temos 5 valores para a e 3 para b e c. Portanto, o número
de divisores deve ser 5 · 3 · 3 = 45.
b) Para que um divisor seja par não pode ocorrer a = 0. O número de possibilidades para a se
reduz a 4. O número de divisores pares é 4 · 3 · 3 = 36.
c) Para que um divisor seja quadrado perfeito, a, b e c devem ser pares. Logo, só poderão
assumir os valores {0, 2, 4} para a e {0, 2} para b e c. O número de divisores satisfazendo
isso é 3 · 2 · 2 = 12.

Problema 23. (IMG) (Maio 2006) Um calendário digital exibe a data: dia, mês e ano, com 2
dı́gitos para o dia, 2 dı́gitos para o mês e 2 dı́gitos para o ano. Por exemplo, 01-01-01 corres-
ponde a primeiro de janeiro de 2001 e 25-05-23 corresponde a 25 de maio de 2023. Em frente
ao calendário há um espelho. Os dı́gitos do calendário são como os da figura abaixo.

Se 0, 1, 2, 5 e 8 se reflentem, respectivamente, em 0, 1, 5, 2 e 8, e os outros dı́gitos perdem


sentido ao se refletirem, determine quantos dias do século, ao se refletirem no espelho, corres-
pondem também a uma data.

Solução. Como não podemos usar os dı́gitos 3, 4, 6, 7, 9 para formar uma data, os únicos
valores possı́veis para os dois primeiros dı́gitos (os que marcam o dia) são: 01, 02, 05, 08, 10, 11,
12, 15, 18, 20, 21, 22, 25, 28. Para os dois próximos dı́gitos temos as seguintes possibilidades:
01, 02, 05, 08, 10, 11, 12. Por outro lado, apenas os pares 01, 10 e 11 também correspondem
a um mês quando são refletidos. Para os dois últimos as possibilidades são: 10, 20, 50, 80, 01,
11, 21, 51, 81, 02, 12, 22, 52, 82. Pois seus reflexos devem corresponder a um dia. Logo, o
total de datas pedidas é 14 × 3 × 14 = 588.

Problema 24. (Rússia) Um número natural n é dito elegante se pode ser escrito como soma
de cubo com um quadrado (n = a3 + b2 , onde a, b ∈ N). Entre 1 e 1000000 existem mais
números que são elegantes ou que não são?

Solução. A quantidade de números elegantes deve ser menor ou igual ao número de soluções
da inequação a3 + b2 ≤ 106 . Note que 0 ≤ a ≤ 102 e 0 ≤ b ≤ 103 .
O número de soluções é menor do que (102 + 1)(103 + 1) = 101101 < 5 · 105 .
Logo, a quantidade de números elegantes é menor do que a metade da quantidade de números
entre 1 e 1000000. Isto é, existem mais números que não são elegantes.

Problema 25. Quantos são os números de cinco dı́gitos que são múltiplos de 3 e possuem 6
como um de seus dı́gitos?

4
Solução.
Combinatória 03 - Paridade

Problema 6. Existe alguma solução inteira para a equação a · b · (a − b) = 45045?

Solução. Não. Se a e b tiverem paridades diferentes então um dos dois é par, de forma que
a · b é par. Mas isso é uma contradição já que 45045 é ı́mpar.
Agora, se a e b tiverem a mesma paridade então a − b deve ser par e do mesmo modo chegamos
a uma contradição.
Logo, não há solução inteira.

Problema 7. Os números 1, 2, . . . , n estão escritos em sequência. É permitido permutar quais-


quer dois elementos. É possı́vel retornar à posição inicial após 2001 permutações?

Solução.
Dizemos que uma sequência tem uma inversão quando um número maior vem antes de um
número menor.
O número de inversões de uma sequência é o número de pares (a, b) com a > b que podemos
encontrar na sequência tais que a aparece antes de b.
Por exemplo, o número de inversões da sequência (1, 3, 2, 5, 4) é 2.
Verifique que ao permutarmos 2 números, a paridade do número de inversões muda.

No problema, a sequência inicial tem 0 inversões. Como são feitas 2001 permutações, temos
2001 mudanças de paridade do número de inversões. Dessa forma, o número de inversões final
deve ser ı́mpar.
Então não podemos ter, ao fim, a sequência inicial.

Problema 8. Um cı́rculo está dividido em seis setores que estão marcados com os números
1, 0, 1, 0, 0, 0 no sentido horário. É permitido somar 1 a dois setores vizinhos. É possı́vel,
repetindo esta operação várias vezes, fazer com que todos os números se tornem iguais?

Solução. Suponha que os números nos setores sejam a1 , a2 , a3 , a4 , a5 e a6 no sentido horário.


Vamos chamar de S o módulo do número a1 − a2 + a3 − a4 + a5 − a6 .
Note que ao somar 1 a dois setores vizinhos o valor de S não se altera.
Então S = 1 − 0 + 1 − 0 + 0 − 0 = 2.
Desse modo, é impossı́vel que todos os números sejam iguais pois terı́amos S = 0.
Problema 9. É possı́vel que as seis diferenças entre dois elementos de um conjunto de quatro
números inteiros serem iguais a 2, 2, 3, 4, 4 e 6?

Solução. Não. Imagine que o conjunto seja {a, b, c, d}. Então podemos supor 3 = a − b.
Mas a − b = (a − c) + (c − b) e a − c e c − b são diferenças de dois elementos do conjunto.
Porém, todas as diferenças, com exceção de 3, são pares. Logo, (a − c) + (c − b) é par.
Isso é uma contradição já que esse valor é igual a 3 que é ı́mpar.
Concluimos que não é possı́vel que as diferenças sejam essas.

Problema 10. Raul falou que tinha dois anos a mais que Kátia. Kátia falou que tinha o dobro
da idade de Pedro. Pedro falou que Raul tinha 17 anos. Mostre que um deles mentiu.

Solução. Suponha que ninguém mentiu. Então Raul tem 17 anos e portanto Kátia tem 15
anos. Mas Kátia tem o dobro da idade de Pedro e, portanto, sua idade deve ser par, contradição.
Logo, alguém deve ter mentido.

Problema 11. (Torneio das Cidades 1987) Uma máquina dá cinco fichas vermelhas quando
alguém insere uma ficha azul e dá cinco fichas azuis quando alguém insere uma ficha verme-
lha. Pedro possui apenas uma ficha azul e deseja obter a mesma quantidade de fichas azuis e
vermelhas usando essa máquina. É possı́vel fazer isto?

Solução. Não. Observe que quando Pedro insere uma ficha e recebe cinco seu número de
fichas aumenta 4 unidades. Logo, a paridade do número de fichas não muda.
Para ter a mesma quantidade de fichas azuis e vermelhas Pedro deve ter um número par de
fichas, mas isso não é possı́vel já que ele inicialmente só possui 1 ficha e 1 é ı́mpar.

Problema 12. (China 1986) Considere uma permutação dos números 1, 1, 2, 2, . . ., 1998,
1998 tal que entre dois números k existem k números. É ou não possı́vel fazer isto?

Solução. Contados da esquerda para a direita, denotemos por ak e bk as posições do primeiro


e segundo número k, respectivamente. Note que 1 ≤ ak < bk ≤ 2 × 1998
Como existem k números entre dois números k’s, devemos ter bk − ak = k + 1. Se é possı́vel
escrever os números 1, 1, 2, 2, . . ., n, n em linha como no enunciado, obtemos:

(a1 + a2 + . . . + an ) + (b1 + b2 + . . . + bn ) = 1 + 2 + . . . + 2n = n(2n + 1)

(n + 1)(n + 2)
(b1 − a1 ) + (b2 − a2 ) + . . . + (bn − an ) = 2 + 3 + . . . + (n + 1) = −1
2
Somando as duas linhas,
5n(n + 1)
2(b1 + b2 + . . . + bn ) =
2

2
5n(n + 1)
Logo, a fração deve ser um inteiro par.
2
Para n = 1998,
5n(n + 1)
= 9985005
2
é ı́mpar e consequentemente não é possı́vel dispormos esses números em linha.

Problema 13. (Rússia 2004) É possı́vel colocarmos números inteiros positivos nas casas de um
tabuleiro 9 × 2004 de modo que a soma dos números de cada linha e a soma dos números de
cada coluna sejam primos? Justifique sua resposta.

Solução. Suponha que seja possı́vel fazer tal construção. Sejam L1 , L2 , . . . , L9 as somas dos
números de cada uma das 9 linhas, e C1 , C2 , . . . , C2004 as somas dos números de cada uma
das 2004 colunas. Como cada Li e Cj são primos, estes devem ser números ı́mpares (já que
são soma de pelo menos nove inteiros positivos e portanto são maiores que 2). Seja S a soma
de todos os números do tabuleiro. Por um lado terı́amos:

S = L1 + L2 + . . . + L9

donde concluı́mos que S é ı́mpar, pois é soma de 9 ı́mpares. Por outro lado:

S = C1 + C2 + . . . + C2004

e daqui concluı́mos que S é par, pois é uma soma de uma quantidade par de ı́mpares, o que é
um absurdo. Logo, tal construção não é possı́vel.

Problema 14. O número A possui 17 dı́gitos. O número B possui os mesmos dı́gitos de A,


porém em ordem inversa. É possı́vel que todos os dı́gitos de A + B sejam ı́mpares?

Solução. Não. Vamos mostrar que algum dos dı́gitos deve ser par. Considere a seguinte soma:
a16 a15 a14 a13 a12 a11 a10 a9 a8 a7 a6 a5 a4 a3 a2 a1 a0
+ a0 a1 a2 a3 a4 a5 a6 a7 a8 a9 a10 a11 a12 a13 a14 a15 a16
r17 r16 r15 r14 r13 r12 r11 r10 r9 r8 r7 r6 r5 r4 r3 r2 r1 r0

Se r8 for par (terı́amos r8 = 2a8 − 10k) então o problema acaba. Suponha então que isso não
ocorre. A única possibilidade é a de que a soma anterior ficou maior do que ou igual a 10 e 1
foi adicionado a soma dos a8 .
Temos dois casos:

• a7 + a9 = 9 e a soma deles (acima de r7 ) recebeu um 1 da soma anterior, isso implicaria


que r7 = 0 e o problema acabaria aqui;

• o segundo caso é a7 + a9 ≥ 10.

3
Vamos então supor que a7 + a9 ≥ 10.
Repare que se a7 + a9 ≥ 10 então r10 = a10 + a6 + 1 − 10k.
Se r10 e r6 tiverem paridades diferentes, um dos dois será par e então o problema acaba.
Vamos supor que isso não ocorre. Para que isso não ocorra, a soma acima de r6 também deve
receber um 1 da soma anterior.
Dessa forma, analogamente como fizermos com a7 + a9 , podemos supor que a5 + a11 ≥ 10.
Usando o mesmo argumenta de paridades diferentes entre r12 e r4 chegamos a suposição de
que a3 + a13 ≥ 10.
Repetindo mais uma vez esse processo nós chegamos em a1 + a15 ≥ 10.
Com isso, nós concluı́mos que a soma acima de r16 receberá um 1 da soma anterior que é a
de a15 + a1 . Isso quer dizer que r16 = a16 + a0 + 1 − 10k. Porém, como não há soma antes
de r0 , devemos ter r0 = a0 + a16 − 10k 0 . Note que r0 e r16 têm paridades diferentes e então
algum dos dois é par. Isso conclui a demonstração.

Repare que esses argumentos valem para qualquer natural com um número ı́mpar de dı́gitos,
basta que exista o dı́gito do meio - nesse caso é o a8 .

Problema 15. Considere um tabuleiro 1998 × 2002 pintado alternadamente de preto e branco
da maneira usual. Em cada casa do tabuleiro, escrevemos 0 ou 1, de modo que a quantidade
de 1s em cada linha e em cada coluna do tabuleiro é ı́mpar. Prove que a quantidade de 1s
escritos nas casas brancas é par.

Solução. Seja ai,j o número escrito na casa da i-ésima linha e da j-ésima coluna, 1 ≤ i ≤ 1998
e 1 ≤ j ≤ 2002. A casa (i, j) é branca se e somente se i e j possuem a mesma paridade.
999 2002
X X
L= a2i−1,j
i=1 j=1

é a soma dos número nas 999 linhas de ordem ı́mpar. Como a soma dos números de cada linha
é ı́mpar, L é ı́mpar. De maneira análoga, a soma dos números nas 1001 colunas de ordem par
1001
X 1998
X
C= ai,2j
j=1 i=1

também é ı́mpar. Seja P o conjunto de todas as casas pretas que estão em colunas de ordem
par, e S(P ) a soma de todos os números escritos nas casas de P .

Cada número escrito em uma casa de P aparece exatamente uma vez na soma L e exatamente
uma vez na soma C. Ademais, cada número escrito em uma casa branca aparece exatamente
uma vez na soma L + C. Assim, a soma dos números escritos nas casas brancas é igual a
L + C − 2S(P ), que é par.

4
Problema 16. (Ucrânia 1997) Considere um tabuleiro pintado de preto e branco da maneira
usual e, em cada casa do tabuleiro, escreva um número inteiro, de modo que a soma dos
números em cada coluna e em cada linha é par. Mostre que a soma dos números nas casas
pretas é par.

Solução. A sólução é análoga à do problema anterior.


A casa (i, j) é a casa da i-ésima linha e j-ésima coluna. A casa (i, j) é preta se e somente se
i e j têm paridades diferentes.
Seja Lk e Ck a soma dos números nas k-ésima linha e coluna respectivamente. Então,

L = L1 + L3 + L5 + L7 + . . .

é a soma dos linhas de ordem ı́mpar e

C = C1 + C3 + C5 + C7 + . . .

é a soma das colunas também de ordem ı́mpar. Como a soma dos números em cada coluna e
em cada linha é par, L e C devem ser pares.

Seja B o conjunto de todas as casas brancas em colunas de ordem ı́mpar, e S(B) a somas dos
números escritos nas casa de B.

Cada casa de B é contada uma vez em C e uma vez em L. Além disso, cada casa preta é
contada exatamente uma vez na soma L + C. Logo, a soma dos números nas casas pretas é
L + C − 2S(B) que é par.
Combinatória 04 - Contagem I

Problema 6. Escrevem-se todos os inteiros de 1 a 9999. Quantos números têm pelo menos
um zero?

Solução. Vamos contar quantos não tem nenhum dı́gito zero. De 1 a 9 temos 9 algarismo
sem nenhum dı́gito 0. De 10 a 99, temos 9 × 9 = 81 números sem nenhum zero.
De 100 a 999 temos 9 × 9 × 9 = 729 números sem nenhum 0.
De 1000 a 9999 temos 94 = 6561 números sem nenhum zero.
Logo, entre 1 e 9999 temos 9 + 81 + 729 + 6561 = 7380 números sem nenhum zero.
Portanto, há 9999 − 7380 = 2619 números com pelo menos um zero.

Problema 7. Quantos números de três dı́gitos possuem todos os seus algarismos com a mesma
paridade?

Solução. Se todos os dı́gitos forem pares temos 4 · 5 · 5 = 100 números, lembre-se que o
primeiro dı́gito não pode ser zero.
Se todos os dı́gitos forem ı́mpares temos 53 = 125 números.
Logo, o total é 100 + 125 = 225.

Problema 8. Quantos são os números de quatro algarismos que possuem pelo menos um dı́gito
repetido?

Solução. Existem 9 · 9 · 8 · 7 = 4536 números de quatro dı́gitos com todos os algarismos


distintos. Como são ao todo 9000 números de quatro dı́gitos, temos 9000 − 4536 = 4464
números com pelo menos um dı́gito repetido.

Problema 9. Quantos são os números de quatro dı́gitos distintos que não possuem dois alga-
rismos consecutivos com a mesma paridade?

Solução. Considere o número de representação decimal abcd. Então, inicialmente temos 9


opções para a. Escolhido a, sabemos que b deve ter paridade diferente e então tem 5 opções.
O dı́gito c deve ter a mesma paridade que a e nao pode ser igual a ele, portanto só temos
4 opções para ele. Já o digito d deve ter a mesma paridade que c e deve ser diferente dele,
portanto temos 4 opções para ele.
A quantidade total de números satisfazendo as condições do enunciado é 9 · 5 · 4 · 4 = 720.

Problema 10. De quantas maneiras podemos colocar um rei preto e um rei branco em um
tabuleiro de xadrez (8 × 8) sem que nenhum deles ataque o outro?
Solução. Podemos dividir o tabuleiro em três regiões: A primeira é formada pelas quatro
casas nos cantos do tabuleiro; a segunda pelas 24 casas da borda (que não estão nos cantos);
e a terceira pelo tabuleiro 6 × 6 no interior do tabuleiro. Se o primeiro rei for posto na
primeira região, temos 60 maneiras de colocar o segundo rei; se ele for posto na segunda,
temos 58 maneiras; e se for posto na terceira, temos 55 maneiras. Logo, temos um total de
4 × 60 + 24 × 58 + 36 × 55 = 3612 modos diferentes de colocar os dois reis.

Problema 11. Quantos são os naturais pares que se escrevem com três algarismos distintos?

Solução. Considere o número abc. Vamos separar o problema em 3 casos.


- c = 0: a pode assumir 9 valores e b somente 8, pois não pode ser igual a nenhum outro;
- b = 0: c pode assumir 4 valores, pois é par, e assim a poderá assumir 8 valores;
- b, c 6= 0: c pode assumir 4 valores, consequentemente a poderá ter 8 valores e assim b poderá
assumir 7 valores.
Logo, ao todo temos 9 · 8 + 4 · 8 + 4 · 8 · 7 = 328 números.

Problema 12. Na cidade Gótica as placas das motos consistem de três letras. A primeira letra
deve estar no conjunto {C, H, L, P, R}, a segunda letra no conjunto {A, I, O}, e a terceira
letra no conjunto {D, M, N, T }. Certo dia, decidiu-se aumentar o número de placas usando
duas novas letras J e K. O intendente dos transportes ordenou que as novas letras fossem
postas em conjuntos diferentes.
Determine com qual opção podemos obter o maior número de placas.

Solução. Inicialmente temos 5 · 3 · 4 = 60 placas. De acordo com o problema, temos as


seguintes opções para o novo número de placas: 6 · 4 · 4 = 96, 5 · 4 · 5 = 100 e 6 · 3 · 5 = 90.
Logo, o número máximo é 100 e ocorre quando as novas opções são adicionadas à segunda e
terceira letras.

Problema 13. (Maio 1998) Cada um dos seis segmentos da figura abaixo deve ser pintado de
uma de quatro cores de modo que segmentos vizinhos não tenham a mesma cor. De quantas
maneiras podemos fazer isso?

Solução. Temos 4 · 3 · 2 = 24 maneiras de pintar o triângulo externo. Note que a quarta


cor (a que não foi usada no triângulo) deve aparecer nenhuma ou uma vez nos três segmentos
internos. Se ela aparecer mais de uma vez teremos dois segmentos vizinhos com a mesma cor.
Temos, portanto, dois casos:

2
Se o quarta cor não aparece nenhuma vez, então só há uma maneira de se pintar os segmentos
internos. Com cada segmento tendo a mesma cor que o lado oposto do triângulo maior.

Se a quarta cor aparece uma vez, ela pode aparecer em qualquer um dos três segmentos in-
ternos. Veja que, uma vez escolhido o segmento que terá a quarta cor, as outras cores ficam
definidas. Temos assim, 3 pinturas diferentes.

O número de maneiras de pintar a figura é 24 · (1 + 3) = 96.


96
Se considerarmos as rotações da figura temos apenas = 32 pinturas diferentes.
3

Problema 14. Em uma festa havia 6 homens e 4 mulheres. De quantos modos podemos formar
3 pares com essas pessoas?

Solução. Temos 6 · 4 maneiras de escolher o primeiro par. Para o segundo temos 5 · 3 e


para o terceiro par 4 · 4. Porém, a ordem em que escolhemos os casais não importa. Devemos
portanto, dividir o resultado por 3!.
(6 · 4) · (5 · 3) · (4 · 2)
Dessa forma, a resposta é = 480.
3!

Problema 15. De quantas maneiras podemos por três torres de mesma cor em um tabuleiro
8 × 8 de modo que nenhuma delas ataque a outra?

Solução. Para a primeira torre temos 64 casas disponı́veis, para a segunda temos 49 casas e
para a segunda temos 36 casas. Porém, como a ordem de colocação das torres não importa,
devemos dividir por 3!.
64 · 49 · 36
O resultado é = 18816.
3!

Problema 16. (AIME 1996) Duas casas de um tabuleiro 7 × 7 são pintadas de amarelo e as
outras são pintadas de verde. Duas pinturas são ditas equivalentes se uma é obtida a partir de
uma rotação aplicada no plano do tabuleiro. Quantas pinturas inequivalentes existem?

Solução. Vamos dividir o problema em dois casos.


No primeiro vamos supor que as duas casas amarelas são simétricas em relação ao centro. Esco-
lhendo uma casa a outra fica definida. Temos 48 maneiras de escolher a primeira casa (não pode
ser a casa central) e 1 de escolher a segunda. Como a ordem de escolha das casas não importa,
devemos dividir o resultado por 2!. Porém, como as casas são simétricas em relação ao centro,
rotacionando o tabuleiro obteremos 2 configurações não idênticas. Dessa forma, devemos no-
48
vamente dividir o resultado por 2. O número de pinturas inequivalentes nesse caso é = 12.
2! · 2
Agora, vamos supor que as duas casas não são simétricas em relação ao centro e nenhuma das

3
duas é a casa central. Temos 48 maneiras de escolher a primeira casa e 46 maneiras de escolher
a segunda casa (não pode ser igual à primeira ou sua simétrica e nem ao centro). Como a
ordem de escolha das casas não importa e a rotação gera 4 configuração diferentes, devemos
dividir o resultado por 2! · 4.
48 · 46
Logo, o número de pinturas inequivalentes aqui é = 276.
2! · 4
Aqui supomos que uma das casas está no centro. O número de maneiras de se obter isso é 48.
Rotacionando o tabuleiro obtemos 4 configurações distintas. Portanto o número de pinturas
48
inequivalentes nesse caso é = 12.
4
O número total de pinturas inequivalentes é 12 + 276 + 12 = 300.

Problema 17. Em uma sala de aula existem a meninas e b meninos. De quantas formas eles
podem ficar em uma fila, se as meninas devem ficar em ordem crescente de peso, e os meninos
também? (Suponha que 2 pessoas quaisquer não tenham o mesmo peso.)

Solução. Temos (a + b)! maneiras de permutar todas as crianças. Porém apenas uma das a!
permutações das meninas está na ordem correta e apenas b! das permutações dos meninos está
(a + b)!
correta. Logo, a resposta é .
a!b!

Problema 18. Considere um torneio de xadrez com 10 participantes. Na primeira rodada cada
participante joga somente uma vez, de modo que há 5 jogos realizados simultaneamente.
De quantas maneiras esta primeira rodada pode ser realizada?

Solução.

Problema 19. Doze cavaleiros estão sentados em torno de uma mesa redonda. Cada um dos
12 cavaleiros considera seus dois vizinhos como rivais. Deseja-se formar um grupo de 5 cava-
leiros para salvar uma princesa. Nesse grupo não poderá haver cavaleiros rivais. Determine de
quantas maneiras é possı́vel escolher esse grupo.

Solução.
Problemas Resolvidos

Nı́vel 2

Paridade

Material elaborado por Valentino Amadeus Sichinel


Problemas

Problema 1. É possı́vel que, em um tabuleiro de xadrez 8x8, um cavalo se desloque do canto inferior
esquerdo ao canto superior direito passando por cada casa exatamente uma vez?

Problema 2. Um destacamento do exército conta com 100 soldados. Em cada noite, três soldados
ficam de guarda. É possı́vel que, após uma certa quantidade de tempo, cada soldado tenha partilhado
a guarda com cada um dos outros soldados exatamente uma vez?

Problema 3. Dados dois pontos distintos A e B no plano, escolhemos 101 pontos na reta determinada
por eles, todos fora do segmento AB. Mostre que a soma das distâncias desses 101 pontos ao ponto
A não pode ser igual à soma das distâncias desses 101 pontos ao ponto B.

Problema 4. Quatro números 1’s e cinco números 0’s estão posicionados ao redor de um cı́rculo.
Entre cada par de números vizinhos, é escrito um 0 se os números são iguais e um 1 se os números
são diferentes. Depois disso, os números “antigos” são apagados. É possı́vel que, após realizar essa
operação sucessivas vezes, os números remanescentes sejam todos iguais?

Problema 5. Sejam a1 , a2 , . . . , an números reais, cada um dos quais igual a 1 ou a −1. Mostre que,
se
a1 a2 a3 a4 + a2 a3 a4 a5 + · · · + an a1 a2 a3 = 0,
então n é divisı́vel por 4.

Problema 6 (RMO1 ). Encontre todos os inteiros k para os quais todas as raı́zes do polinômio

f (x) = x3 − (k − 3)x2 − 11x + (4k − 8)

são inteiras.

1
Regional Mathematical Olympiad, uma das fases da olimpı́ada nacional da Índia.

2
Soluções

1. Não. Observe que, toda vez que se movimenta, o cavalo passa a ocupar uma casa de cor oposta
à cor da casa que ocupava até então. De fato, cada movimento do cavalo pode ser visto como uma
combinação de três passos sucessivos, sendo um ”passo”a passagem para uma casa vizinha (duas casas
são vizinhas se têm um lado em comum), e casas vizinhas sempre têm cores opostas.
Dessa forma, se o cavalo inicia, digamos, em uma casa de cor preta, e passa por todas as casas do
tabuleiro, exatamente uma vez por cada uma delas, ele há de finalizar a movimentação em uma casa
de cor branca. Com efeito, há 8 × 8 = 64 casas no tabuleiro, o que faz com que um deslocamento de
tal natureza seja composto por exatamente 63 movimentos e, ao executar uma quantidade ı́mpar de
movimentos, o cavalo finaliza em uma casa de cor oposta à cor da casa em que começou, já que a cor
da casa que ele ocupa muda a cada movimento.
No entanto, independentemente de como está posicionado o tabuleiro, a casa do canto inferior
esquerdo e a casa do canto superior direito têm a mesma cor. Portanto, é impossı́vel, que o cavalo se
desloque do canto inferior esquerdo ao canto superior direito passando por cada casa exatamente uma
vez.

2. Não. Cada vez que fica de guarda, um soldado partilha a guarda com outros dois colegas. Assim,
a quantidade de colegas com os quais um determinado soldado já partilhou a guarda é sempre par
(contando repetições, claro). Para que todos os soldados tenham partilhado a guarda com cada um
dos outros soldados exatamente uma vez, cada soldado teria que ter compartilhado a guarda com
exatamente outros 99 soldados, mesmo contando repetições. Isso jamais acontecerá, pois 99 não é par.

3. Denotemos por d(X, Y ) a distância entre dois pontos X e Y do plano.


Sejam P1 , P2 , . . . , P101 os pontos escolhidos.
Se um desses pontos, digamos Pi , está mais próximo de A que de B, então

d(Pi , B) = d(Pi , A) + d(A, B).

Por outro lado, se Pi está mais próximo de B que de A, então

d(Pi , B) = d(Pi , A) − d(A, B).

Dessa forma, temos, para cada i,

d(Pi , B) = d(Pi , A) + f (i)d(A, B),

onde (
1, se Pi está mais próximo de A que de B
f (i) = .
−1, se Pi está mais próximo de B que de A
Logo,
101
X 101
X 101
X
d(Pi , B) = d(Pi , A) + f (i)d(A, B).
i=1 i=1 i=1
Assim, se a soma das distâncias dos pontos Pi ao ponto A fosse igual à soma das distâncias desses
pontos ao ponto B, terı́amos
X101
f (i)d(A, B) = 0,
i=1

3
isto é, terı́amos
101
X
f (i) = 0.
i=1

No entanto, 101
P
i=1 f (i) é um somatório de 1’s e de −1’s, em que a quantidade total de parcelas é
igual a 101. Como cada uma das parcelas é ı́mpar, e a quantidade de parcelas também, o valor do
somatório é ı́mpar e, portanto, diferente de 0.
Dessa forma, não é possı́vel que a soma das distâncias dos 101 pontos escolhidos ao ponto A seja
igual à soma das distâncias dos mesmos 101 pontos ao ponto B.

4. Não. Suponhamos que, após sucessivas execuções da operação, obtemos um conjunto homogêneo
de números, isto é, formado apenas por 0’s ou apenas por 1’s. Se em algum momento os números são
todos 0’s, no momento anterior os vizinhos eram todos iguais e, portanto, o conjunto já era homogêneo
(isto é, os números já eram todos iguais). Assim, quando atingimos um estado homogêneo (isto é, um
estado em que os números são todos iguais) pela primeira vez, os números são todos iguais a 1.
No entanto, para que os números sejam todos iguais a 1 em um determinado momento, os vizinhos
deveriam ser todos distintos no momento anterior. Isso não é possı́vel, já que a quantidade de números
escritos ao redor do cı́rculo é sempre ı́mpar (sempre igual a 9).
De fato, se enumerarmos as posições dos números de 1 a 9 no sentido horário, observamos que,
para que os vizinhos sejam sempre distintos, os números das posições ı́mpares devem ser sempre
distintos dos números das posições pares. Consequentemente, os números das posições ı́mpares serão
iguais entre si (assim como os números das posições pares). Mas os números das posições 1 e 9 são
vizinhos; não podem ser iguais. Absurdo! Dessa forma, não é possı́vel que números vizinhos sejam
todos distintos.
Portanto, independentemente de como estiverem posicionados os primeiros quatro números 1’s e
cinco números 0’s, não é possı́vel que, após algumas aplicações da operação descrita no enunciado, os
números escritos ao redor do cı́rculo sejam todos iguais.

5. Como cada ai é igual a 1 ou a −1, cada uma das parcelas da soma

a1 a2 a3 a4 + a2 a3 a4 a5 + · · · + an a1 a2 a3

também é igual a 1 ou a −1.


Sejam a a quantidade de parcelas iguais a 1 e b a quantidade de parcelas iguais a −1. Por um
lado, a + b = n, pois temos ao todo n parcelas. Por outro lado, a − b = 0 - é o que indica a igualdade
do enunciado. Assim, a = b, e temos n = 2b.
Contemos, agora, a quantidade de −1’s que aparecem em

a1 a2 a3 a4 + a2 a3 a4 a5 + · · · + an a1 a2 a3 . (1)

Veja bem: não estamos olhando para a quantidade de parcelas iguais a −1 (que sabemos ser igual a
b), mas para a quantidade de fatores −1 dentro dessas parcelas. Assim, por exemplo, se a1 = −1, a1
contribui com 4 fatores, já que a1 aparece em 4 parcelas.
Por um lado, cada ai que é igual a −1 contribui com 4 fatores, pois aparece em 4 parcelas. Assim,
a quantidade total de −1’s que aparecem em (1) é divisı́vel por 4 e, em particular, par.
Por outro lado, se p0 , p1 , p2 , p3 e p4 são as quantidades de parcelas com 0, 1, 2, 3 e 4 fatores −1,
respectivamente, então a quantidade total de fatores −1 é igual a

p1 + 2p2 + 3p3 + 4p4 .

4
Logo, p1 + 2p2 + 3p3 + 4p4 deve ser par. Como 2p2 + 2p3 + 4p4 sempre é par,

p1 + 2p2 + 3p3 + 4p4 − (2p2 + 2p3 + 4p4 ) = p1 + p3

também deve ser par.


Observe, no entanto, que p1 + p3 = b. Com efeito, as parcelas cujo valor é −1 são exatamente as
parcelas que possuem uma quantidade ı́mpar de fatores −1.
Logo, b é par. Dessa forma, existe um inteiro k para o qual b = 2k.
Como n = 2b, segue daı́ que n = 4k. Portanto, n é divisı́vel por 4.

6. Suponhamos que k é inteiro e tal que todas as raı́zes do polinômio f , definido no enunciado, são
inteiras.
Sejam a, b e c as raı́zes de f .
Pelas relações de Girard, abc = 8 − 4k, que é par. Assim, ao menos uma das raı́zes é par. Por outro
lado, também pelas relações de Girard, ab + bc + ca = −11, que é ı́mpar. Dessa forma, no máximo
uma das raı́zes é par - se duas fossem par, ab + bc + ca seria par também. Logo, exatamente uma das
raı́zes a, b e c é par. Suponhamos, sem perda de generalidade, que a é par.
Observe que
f (2) = 23 − (k − 3)22 − 11 · 2 + (4k − 8) = −10.
Como a, b e c são as raı́zes de f , e f é mônico (isto é, o coeficiente lı́der de f é 1), temos que

f (x) = (x − a)(x − b)(x − c)

para todo x. Em particular,


−10 = f (2) = (2 − a)(2 − b)(2 − c).
Como a é par, (2 − a) também é par. Pela igualdade acima, (2 − a) é divisor de −10. Como os
únicos divisores pares de −10 são 2 e −2, temos ou 2 − a = 2, ou 2 − a = −2. No primeiro caso, a = 0;
no segundo, a = 4. Analisemos cada caso separadamente.
Caso I: a = 0.
Neste caso, temos abc = 0. Mas, pelas equações de Girard, abc = 8 − 4k. Assim, k = 2.
As demais equações de Girard se reduzem a

bc = −11 e b + c = k − 3 = −1.

A segunda nos dá b = −1 − c. Substituindo na primeira, ficamos com

(−1 − c)c = −11 ⇒ c2 + c − 11 = 0,

que não tem solução inteira. Absurdo! Dessa forma, não é possı́vel que tenhamos a = 0.
Caso II: a = 4.
Neste caso, devemos ter f (4) = 0 (com efeito, a é uma raiz de f ), isto é,

43 − (k − 3)42 − 11 · 4 + (4k − 8) = 0 ⇐⇒ 60 − 12k = 0 ⇐⇒ k = 5.

Aplicando as equações de Girard (e utilizando os já conhecidos valores de a e de k), encontramos


que os valores de b e de c hão de ser, em alguma ordem, 1 e −3. Todos inteiros.
Como as equações de Girard estão satisfeitas, concluı́mos que, quando k = 5, as raı́zes de f são,
de fato, 0, 1 e −3.
Portanto, k = 5 é o único valor de k para o qual todas as raı́zes de f são inteiras.

Material elaborado por Valentino Amadeus Sichinel

5
Problemas Resolvidos

Nı́vel 2

Paridade II

Material elaborado por Valentino Amadeus Sichinel


Problemas

Problema 1. 25 meninas e 25 meninos estão sentados ao redor de uma mesa redonda. Mostre que
ao menos uma das crianças está sentada entre dois meninos.

Problema 2. Um caracol se desloca pelo plano em velocidade constante, fazendo um ângulo de 90◦
a cada 15 minutos. Mostre que ele só pode retornar ao seu ponto de origem após uma quantidade
inteira de horas.

Problema 3. Três gafanhotos estão alinhados, brincando de pular. Em cada turno, um dos gafanhotos
pula sobre outro, mas nunca sobre os outros dois ao mesmo tempo. É possı́vel que, após 2021 turnos,
os gafanhotos tenham voltado às suas posições originais?

Problema 4. Em uma coleção de 2021 moedas, 1010 são falsas, e têm um peso que difere do peso
das moedas autênticas por 1 grama. Miriam Maneira tem uma balança que mostra a diferença de
peso entre os objetos posicionados nos dois lados. Ela escolhe uma moeda qualquer, e quer saber se
essa moeda é falsa ou autêntica através de uma única pesagem. É possı́vel fazer isso?

Problema 5. É possı́vel arranjar os números de 1 a 9 em sequência de tal forma que a quantidade


de números entre 1 e 2 é ı́mpar, a quantidade de números entre 2 e 3 é ı́mpar, ..., e a quantidade de
números entre 8 e 9 é ı́mpar?

Problema 6. Cada casa de um tabuleiro de xadrez 2018 × 2022 contém ou um 0 ou um 1, de tal


forma que a quantidade de casas contendo 1 é ı́mpar em cada linha e em cada coluna. Prove que o
número de casas brancas que contém 1 é par.
Observação: uma casa (i, j) (linha i e coluna j) é branca se i e j têm a mesma paridade.

Problema 7 (MOSP1 ). Seja X um conjunto finito de inteiros positivos, e seja A um subconjunto de


X. Mostre que existe um subconjunto B de X tal que A é exatamente o conjunto de elementos de X
que dividem uma quantidade ı́mpar de elementos de B.

1
Mathematical Olympiad Summer Program, programa de treinamento dos alunos estadunidenses para a IMO.

2
Soluções

1. Suponhamos, por absurdo, que nenhuma das crianças está sentada entre dois meninos.
Enumeremos as crianças, de 1 a 50, no sentido horário. Crianças de números consecutivos são
vizinhas e, além disso, a criança de número 50 é vizinha da criança de número 1.
Se uma criança está em uma posição ı́mpar, então suas duas vizinhas estão em posições pares
consecutivas (considerando que 50 e 2 são posições pares consecutivas). Respectivamente, se uma
criança está em uma posição par, então suas duas vizinhas estão em posições ı́mpares consecutivas
(considerando que 49 e 1 são posições ı́mpares consecutivas). Dessa forma, a suposição que estamos
fazendo equivale a dizer que não há dois meninos em posições pares consecutivas, nem dois meninos
em posições ı́mpares consecutivas.
Se não há dois meninos em posições pares consecutivas (e 50 e 2 são posições pares consecutivas),
então no máximo ⌊ 252 ⌋ = 12 das posições (2, 4, . . . , 50) estão ocupadas por meninos.
Da mesma forma, se não há dois meninos em posições ı́mpares consecutivas (e 49 e 1 são posições
ı́mpares consecutivas), então no máximo ⌊ 25 2 ⌋ = 12 das posições (1, 3, . . . , 49) estão ocupadas por
meninos.
Portanto, há no máximo 12 + 12 = 24 meninos. Absurdo!
Como nossa hipótese implica um absurdo, concluı́mos que ela é falsa, isto é, que ao menos uma
das crianças está sentada entre dois meninos.

2. Digamos que, após uma certa quantidade de tempo, o caracol tenha voltado ao seu ponto de origem.
Chamemos de v a quantidade de vezes que ele andou na vertical e de h a quantidade de vezes que ele
andou na horizontal.
Como o caracol voltou ao ponto de partida, ele deve ter andado para baixo a mesma quantidade
de vezes que andou para cima. Assim, a quantidade de vezes que o caracol andou na vertical, v, é par.
Da mesma forma, para ter retornado à origem, o caracol deve ter andado para a esquerda a mesma
quantidade de vezes que andou para a direita. Logo, a quantidade de vezes que o caracol andou na
horizontal, h, também é par.
Veja agora que, como o animal sempre faz um ângulo de 90◦ , as direções em que ele anda ficam
alternando: se agora ele anda na vertical, daqui a 15 minutos andará na horizontal, depois na vertical
novamente, e assim por diante. Dessa forma, v e h se relacionam de uma dentre três possı́veis maneiras:
• se o caracol começa andando na vertical e termina andando na vertical, v = h + 1;
• se o caracol começa andando na vertical e termina andando na horizontal ou vice-versa, v = h;
• se o caracol começa andando na horizontal e termina andando na horizontal, v = h − 1.
Como v e h são ambos pares, só é possı́vel que tenhamos v = h.
Logo, o caracol andou, ao todo, durante 15v + 15h = 15v + 15v = 30v minutos. Como v é par,
30v = 60k para algum inteiro k.
Portanto, o caracol andou durante uma quantidade inteira de horas.

3. Não.
Para facilitar a notação, nomeemos os gafanhotos: chamemo-los a, b e c. Além disso, classifiquemos
os turnos: um turno será do tipo ab se o gafanhoto a tiver pulado sobre o gafanhoto b, um turno será
do tipo bc se o gafanhoto b tiver pulado sobre o gafanhoto c, um turno será do tipo ac se o gafanhoto
a tiver pulado sobre o gafanhoto c, etc. Por fim, denotemos por #(xy) a quantidade de turnos do tipo
xy (isto é, #(ab) é a quantidade de turnos do tipo ab, etc).
Suponhamos que, após n turnos, os gafanhotos tenham voltado às suas posições originais. Isso
implica que, se o gafanhoto a estava, originalmente, à frente do gafanhoto b, então após os n turnos

3
o gafanhoto a está à frente do gafanhoto b. Para que isso tenha acontecido, o gafanhoto a deve ter
pulado sobre o gafanhoto b tantas vezes quanto o gafanhoto b pulou sobre o gafanhoto a. Em outras
palavras, #(ab) = #(ba). De modo totalmente análogo, #(ac) = #(ca) e #(bc) = #(cb). Assim,

n = #(ab) + #(bc) + #(ca) + #(ac) + #(cb) + #(ba) = 2(#(ab) + #(bc) + #(ca)).

Vê-se daı́ que n há de ser par.


Portanto, não é possı́vel que os gafanhotos voltem às suas posições originais após 2021 turnos.

4. Sim; basta que Miriam distribua as outras 2020 moedas na balança, metade em cada lado. Vejamos
por que isso funciona.
Seja p o peso em gramas de uma moeda autêntica. O peso, em gramas, de uma moeda falsa é
p ± 1. Digamos que, ao posicionar as 2020 moedas na balança, 1010 moedas em cada lado, Miriam
tenha colocado a moedas falsas na esquerda e b moedas falsas na direita. Nesse cenário, a diferença
de peso entre os dois conjuntos de moedas é

(1010p ± a) − (1010p ± b) = ±(a − b).

Se a moeda escolhida é falsa, a + b = 1009, donde b = 1009 − a e, portanto,

±(a − b) = ±(a − 1009 + a) = ±(2a − 1009)

é ı́mpar. Por outro lado, se a moeda escolhida é autêntica, a + b = 1010, donde b = 1010 − a e,
portanto,
±(a − b) = ±(a − 1010 + a) = ±(2a − 1010) = ±2(a − 505)
é par.
Assim, Miriam pode descobrir a autenticidade da moeda escolhida observando a paridade do
número indicado pela balança.

5. Não.
Suponha, por absurdo, que os números estão ordenados de forma a respeitar as condições do
enunciado. Enumeremos as posições, da esquerda para a direita, de 1 a 9.
Como há uma quantidade ı́mpar de números entre 1 e 2, esses dois números estão em posições de
mesma paridade. Da mesma forma, 2 e 3 estão em posições de mesma paridade. Da mesma forma, 3
e 4 estão em posições de mesma paridade. Da mesma forma... Enfim, todos os 9 números estão em
posições de mesma paridade. Absurdo!
Portanto, não é possı́vel arranjar os inteiros de 1 a 9 de modo que a quantidade de números
posicionados entre inteiros consecutivos seja sempre ı́mpar.

6. Denotemos por (i, j) a casa que se encontra na linha i e na coluna j. Além disso, para cada conjunto
X de casas, denotemos por S(X) a quantidade de casas de X que contêm 1.
Observe que
  
casas brancas = (i, j) | i e j ı́mpares ∪ (i, j) | i e j pares ,

que
  
(i, j) | i e j ı́mpares = (i, j) | i ı́mpar \ (i, j) | i ı́mpar e j par ,

4
e que   
(i, j) | i e j pares = (i, j) | j par \ (i, j) | i ı́mpar e j par .
Assim
     
S casas brancas = S (i, j) | i e j ı́mpares + S (i, j) | i e j pares
     
= S (i, j) | i ı́mpar + S (i, j) | j par − 2S (i, j) | i ı́mpar e j par .

Daı́,  
S casas brancas
tem a mesma paridade de
   
S (i, j) | i ı́mpar + S (i, j) | j par .

Por um lado, há 1009 linhas ı́mpares


 no tabuleiro, cada uma das quais com uma quantidade ı́mpar
de casas com o número 1. Logo, S (i, j) | i ı́mpar é ı́mpar.
Por outro lado, há 1011 colunas ı́mpares
 no tabuleiro,  cada uma das quais com uma quantidade
ı́mpar de casas com o número 1. Logo,  S (i,
 j) | j par também
 é ı́mpar.
Desas forma, S (i, j) | i ı́mpar + S (i, j) | j par , uma soma de dois números ı́mpares, é
par.  
Portanto, S casas brancas é par, isto é, a quantidade de casas brancas que possuem o número
1 é par.

7. Vamos construir, em vários passos, um conjunto B que tem as caracterı́sticas desejadas.


Comecemos com B = ∅ e iteremos sobre os elementos de X, do maior ao menor, executando o
seguinte procedimento: se um determinado elemento x não divide a paridade correta de números de
B (isto é, se x ∈ A e x divide uma quantidade par de números de B, ou se x ∈ / A e x divide uma
quantidade ı́mpar de números de B), adicionamos x a B; caso contrário, nada fazemos.
Ao final de todas as iterações (veja que X é finito), teremos construı́do o conjunto B desejado.
Entendamos por que esse conjunto atende, de fato, a todas as exigências.
Veja que, ao final de cada iteração, temos certeza de o número x sob análise divide a paridade
correta de elementos de B (se, antes da iteração, isso não acontecia, adicionamos x a B, o que faz
com que a paridade de elementos de B que são divisı́veis por x mude e, portanto, a condição passe
a ser satisfeita; caso contrário, não fazemos nada, de modo que a condição continue a ser satisfeita).
Além disso, em cada iteração, não influenciamos na paridade de elementos de B que cada um dos
números analisados anteriormente divide. De fato, como estamos analisando os números do maior
para o menor, nenhum dos números já analisados divide o número sob análise no momento. Dessa
forma, ao terminarmos de analisar todos os números de X, teremos ”consertado”as paridades para
cada um deles e garantiremos, assim, que os números que dividem uma quantidade ı́mpar de elementos
de B são exatamente os elementos de A.

Material elaborado por Valentino Amadeus Sichinel

5
Problemas Resolvidos

Nı́vel 2

Jogos

Material elaborado por Hugo Fonseca Araújo


Problemas

Problema 1. Andressa e Bruna jogam um jogo. Elas iniciam com 2008 moedas dispostas em um
cı́rculo e fazem suas jogadas em turnos alternados, Andressa realizando a primeira jogada. Em seu
turno, uma jogadora pode remover uma moeda ou, se duas moedas adjacentes permanecem no cı́rculo,
ela pode remover as duas. Vence quem remover a última moeda. Prove que Bruna tem a estratégia
vencedora, independente de como Andressa jogue.

Problema 2. Arnaldo e Bárbara jogam o seguinte jogo. No primeiro turno, o número 2019 está
escrito numa folha de papel. Arnaldo deve então escolher um dos dı́gitos diferentes de zero deste
número e subtraı́-lo de 2019, repassando o resultado para Bárbara. Em sua vez, Bárbara deve subtrair
do número que recebeu de Arnaldo um dos dı́gitos diferentes de zero deste número e repassar o
resultado para Arnaldo. Eles repetem essa regra alternadamente e quem não puder mais jogar perde,
ou seja, quem receber o número 0 de seu oponente. Quem tem a estratégia vencedora?

Problema 3. (Centro-americana 2001) Dois jogadores, Adriana e Bruno, estão sentados em um


grande cı́rculo com mais 2001 pessoas, de forma que Adriana e Bruno não estão em posições consecu-
tivas. Começando por Adriana, eles jogam o seguinte jogo em turnos alternados. Cada jogada consiste
em tocar no ombro uma das pessoas que é sua vizinha no cı́rculo. Esta pessoa então se levanta e deixa
o cı́rculo. Vence o último jogador que permanecer no cı́rculo. Mostre que um dos dois jogadores tem
uma estratégia vencedora e apresente-a.

Problema 4. Alice lançou o seguinte desafio a Betina. Ela irá desenhar um retângulo em um quadro-
negro. Betina tentará então desenhar uma reta que divida o retângulo em duas partes de mesma área
e, ao mesmo tempo, o quadro-negro em duas partes de mesma área. Se Betina conseguir desenhar a
reta, ela vence; caso contrário, Alice vence. Quem vencerá o desafio?

Problema 5. (Torneio das Cidades 2003) Alessandra e Beto colorem em turnos alternados os lados
de um polı́gono de n lados. Em sua vez, Alessandra pode colorir qualquer lado que tem 0 ou 2 vértices
em comum com lados já coloridos. Beto pode colorir qualquer lado que tem exatamente 1 vértice em
comum com lados já coloridos. Perde o jogador que não puder colorir nenhum lado em sua vez. Para
quais valores de n Beto tem a estratégia vencedora?

Problema 6. (Centro-americana 2004) Em um quadro-branco, os números de 1 a 9 estão escritos.


Artur e Bento jogam o seguinte jogo em turnos, Artur sendo o primeiro. Em sua vez, cada jogador
escolhe um dos números escritos no quadro-branco e apaga, além dele, todos os seus múltiplos que
ainda estão escritos no quadro. Perde quem remover o último número. Determine se algum dos
jogadores tem uma estratégia vencedora e justifique.

Problema 7. (Alemanha 2009) No inı́cio de um jogo há três caixas com 2008, 2009 e 2010 pedras.
Amanda e Bianca jogam in turnos seguindo a seguinte regra:
Quando é o seu turno, escolha duas caixas. Esvazie-as e depois distribua as pedras da terceira
caixa entre as três caixas, de maneira que nenhuma caixa fique vazia. Se você não puder fazer isto,
você perde o jogo.
Quem tem a estratégia vencedora quando Amanda joga primeiro?

2
Problema 8. (OBM 2010 adaptado) Arnaldo e Bernaldo participam do seguinte jogo em um tabuleiro
m×n, m, n ≥ 2. Arnaldo começa escolhendo uma casinha e colocando um cavalo na casinha escolhida;
em seguida, Bernaldo e Arnaldo movem alternadamente o cavalo, começando por Bernaldo, com a
restrição de que o cavalo não pode cair em casinhas que já foram visitadas. Perde quem não poder
mover o cavalo. Determinar qual jogador tem uma estratégia para ganhar o jogo, não importando os
movimentos do outro jogador e mostrar como ele deve jogar para ganhar, quando:

a) m = 2 e n ≥ 2.

b) m = 3 e n = 4.

Observação: Cada movimento de um cavalo consiste em ir duas casas na vertical ou na horizontal


e, em seguida, uma casa na direção perpendicular.

Problema 9. Existem 1001 degraus em uma escadaria que sobe uma colina, com pedras em alguns
desses degraus (não mais que uma pedra por degrau). Sı́sifo pode pegar qualquer pedra e leva-lá
um ou mais degraus acima até o degrau vazio mais próximo. Em seguida, seu rival Hades, rola uma
pedra (com um degrau vazio diretamente abaixo dela) um degrau para baixo. Existem 500 pedras,
originalmente localizadas nos primeiros 500 degraus. Sı́sifo e Hades movem as pedras em turnos,
começando por Sı́sifo. Seu objetivo é mover uma pedra para o degrau mais alto. Hades é capaz de
detê-lo?

Problema 10. (Centro-americana 2003) Dois jogadores, Alfredo e Bernardo, jogam em turnos al-
ternados o seguinte jogo. No inı́cio, há uma pilha com 2003 pedras. No seu primeiro turno, Alfredo
escolhe um divisor de 2003 e remove este número de pedras da pilha. Em sua vez, Bernardo escolhe
um divisor do número de pedras restantes e remove este número de pedras da nova pilha. Daı́ por
diante, eles repetem esta mesma regra, alternando seus turnos. O jogador que remover a última pedra
perde. Mostre que um deles tem uma estratégia vencedora e a descreva.

Problema 11. (Tuymaada 2019) André, Bete, Carlos e Daniel jogam em um tabuleiro 1000 × 1000.
Eles jogam em turnos: André primeiro, depois Bete, depois Carlos, finalmente Daniel, e depois disso
André novamente e assim por diante. Em cada turno, cada jogador deve pintar casas do tabuleiro
formando um retângulo 2 × 1, 1 × 2, 1 × 3, ou 3 × 1. O jogador que não conseguir jogar perde. Prove
que existem três jogadores que podem cooperar de maneira que o outro sempre perca.

Problema 12. (Holanda 2010) Antônio e Benjamin disputam um jogo usando 2010 moedas. Ao longo
do jogo, as moedas são divididas em várias pilhas com ao menos uma moeda em cada uma. Uma
jogada consiste em escolher uma ou mais pilhas e dividir cada uma delas em duas pilhas menores (logo
pilhas de apenas uma moeda não podem ser escolhidas). Inicialmente há apenas uma pilha contendo
todas as 2010 moedas. Antônio e Benjamin jogam em turnos alternados começando por Antônio.
Perde quem não puder fazer mais jogadas. Mostre que Antônio pode vencer o jogo, independente de
como Benjamin joga.

Problema 13. (Torneio das cidades 2003) Uma barra de chocolate no formato de um triângulo
equilátero cujos lados tem comprimento igual a n consiste de pedacinhos triangulares equiláteros
menores, de lados paralelos ao lados da barra e com comprimento igual a 1. Dois jogadores, Ana e
Bruno, revezam-se em turnos comendo a barra de chocolate, começando por Ana. Na sua vez, ela ou
ele parte um pedaço triangular da barra, come-o e passa o restante para o outro jogador. Enquanto
a barra tiver mais de um pedacinho triangular, nenhum jogador pode comê-la inteiramente.

3
Perde quem não conseguir jogar ou deixar apenas um pedacinho para seu oponente. Para cada
valor de n natural, determine quem tem a estratégia vencedora.
OBS: Partir um pedaço triangular da barra significa escolher uma linha respeitando a fronteira
dos pedacinhos e cortar a barra ao longo desta, removendo uma das duas partes e esta tem que ser
triangular. Por exemplo, na figura à esquerda há apenas duas maneiras de partir a barra, já na figura
à direita não é mais possı́vel.

Problema 14. Um plano infinito está dividido em quadrados por dois conjuntos de linhas paralelas.
Alex e Bianca disputam o seguinte jogo: Alex escolhe um quadrado e o pinta de vermelho, depois
Bianca escolhe um quadrado ainda não pintado e o pinta de azul, depois Alex escolhe um quadrado
não pintado e o pinta de vermelho, e assim por diante alternado os turnos. O objetivo de Alex é pintar
de vermelho quatro quadrados cujos centros formam um quadrado com lados paralelos às retas dos
dois conjuntos de retas paralelas. O objetivo de Bianca é impedir isso. Alex pode vencer?

Problema 15. (Espanha 2020) Ana e Bento se enfrentam em um jogo que dura 2020 turnos. Inicial-
mente, existem 2020 cartas em uma mesa, numeradas de 1 a 2020, e Ana possui uma carta extra com
o número 0. No k-ésimo turno, o jogador que não possui a carta k − 1 escolhe entre pegar a carta com
o número k ou entregá-la a seu oponente. O número em cada carta indica seu valor em pontos. No
fim do jogo vence quem tiver mais pontos. Determine se algum jogador tem estratégia vencedora ou
se os jogadores podem forçar um empate e descreva a estratégia.

Problema 16. (OBM 2007) Quadradinhos iguais estão arrumados formando um tabuleiro n × n.
Ludmilson e Ednalva jogam o seguinte estranho jogo. Cada jogada de Ludmilson consiste em retirar
4 quadradinhos que formem um quadrado 2 × 2. Cada jogada de Ednalva consiste em retirar apenas
1 quadradinho. Ludmilson e Ednalva jogam alternadamente, sendo Ludmilson o primeiro a jogar.
Quando Ludmilson não puder fazer sua jogada, então Ednalva fica com todas as peças restantes do
tabuleiro. Ganha o jogo aquele que possuir mais quadradinhos no final. Diga se é possı́vel que Ednalva
ganhe o jogo, não importando como Ludmilson jogue, em cada um dos seguintes casos:

a) n = 10.

b) Caso geral (n qualquer).

Problema 17. São dadas 2000 pedras em uma mesa. Ágata e Berenice jogam um jogo em turnos,
Ágata sendo a primeira. A regra é:

1. A cada turno, a jogadora poder retirar 1, 2, 3, 4 ou 5 pedras da mesa.

2. Elas não podem retirar o mesmo número de pedras que a jogadora anterior retirou.

Quem retirar a última pedra vence. Quem tem a estratégia vencedora? Prove.

4
Problema 18. (Lusófona 2015) No centro de um quadrado há um coelho e, em cada um dos vértices,
um lobo. Os lobos se movem ao longo das arestas do quadrado e o coelho se move livremente no
plano. Sabendo que o coelho tem velocidade igual a 10 Km/h e os lobos tem velocidade máxima de
14 Km/h, determine se existe alguma estratégia que permita ao coelho escapar do quadrado sem ser
capturado por algum dos lobos.

Problema 19. (Torneio das cidades 2007 - adaptado) A audiência faz uma fila com n moedas. A
sequência de caras e coroas é escolhida de forma arbitrária por ela. A audiência também escolhe um
número de 1 a n inclusive. Após isso, a assistente vira uma das moedas e a mágica é trazida de volta
ao palco para examinar a sequência resultante. Por causa de um esquema combinado anteriormente
com a assistente, a mágica tenta determinar qual foi o número escolhido pela audiência. Prove que se
elas tem uma estratégia que funciona para n, elas tem uma estratégia que funciona para 2n.

Problema 20. (Rússia 2007) Dois jogadores alternam turnos desenhando diagonais em um polı́gono
regular de 2n + 1 lados (n > 1). É proibido desenhar uma diagonal que já foi desenhada ou que cruza
um número ı́mpar de diagonais já desenhadas. Quem não puder mais fazer sua jogada perde. Quem
tem a estratégia vencedora?

5
Soluções
Em todas as soluções A indica o jogador cujo nome começa com a letra A e B indica o jogador
cujo nome começa com a letra B. Em um único problema usamos C e D de maneira análoga.
1. Para vencer, basta Bruna jogar sempre de forma simétrica a Andressa em relação ao centro do
cı́rculo. Por simetria, isto sempre é possı́vel (note que após cada jogada de Bruna o arranjo das
moedas é simétrico em relação ao centro). Dessa forma, enquanto A puder jogar, B também pode e
assim A nunca remove a última moeda.

2. A tem a estratégia vencedora. A primeira posição perdedora é 10. Forçosamente o jogador tem que
entregar 9 a seu oponente e então este vence automaticamente. Note que 11, 12, ..., 19 são posições
vencedoras, pois a partir delas é possı́vel entregar 10 a seu oponente. O mesmo argumento utilizado
para 10 mostra que 20 também é uma posição perdedora. Um padrão começa a aparecer.
A estratégia de A é sempre passar para B um número terminado em 0, por fim passando o
próprio número 0 e vencendo assim o jogo. Ele sempre pode fazer isso pois pode fazer inicialmente
2019 − 9 = 2010 e, daı́ em diante, se B recebeu n um número que termina em 0, qualquer movimento
que B fizer devolve para A um número com último dı́gito diferente de 0; A então subtrai este último
dı́gito do número que lhe foi repassado por A, realizando assim sua estratégia.

3. A vence. A estratégia é sempre manter entre ela e B um número ı́mpar de pessoas, em ambas
direções. Isto sempre é possı́vel pois em seu turno sempre há um número ı́mpar de pessoas no cı́rculo.
Logo o número de pessoas entre eles é par em apenas uma das direções. Se este número for igual a 0,
A toca B no ombro e vence. Caso contrário, A toca seu vizinho nesta direção no ombro, repassando
para B uma configuração na qual entre ela e B há um número ı́mpar de pessoas em ambas direções.
Note que B nunca vence o jogo, pois há sempre pelo menos uma pessoa entre A e ele. Assim, como o
jogo termina, já que o número de pessoas no cı́rculo sempre diminui de 1, em algum momento A faz
o último movimento, vencendo o jogo.

4. Betina vence. Basta traçar a linha reta que liga o centro do quadro-negro ao centro do retângulo.
Esta linha divide ambas figuras em duas partes congruentes, por simetria, logo de mesma área.

5. Este é um problema que envolve a ideia de criar um banco de possı́veis jogadas para o futuro. A
vence sempre que n > 4. Sua estratégia é a seguinte.
Depois que ambos realizarem sua primeira jogada, haverá dois lados adjacentes pintados, um por
A e outro por B. A então colore um dos lados que está a um lado de distância desse par de lados
coloridos. Por exemplo, se A1 A2 e A2 A3 são os lados coloridos do polı́gono, A colore A4 A5 ou An−1 An
(note que se n = 5 estes dois lados são na verdade o mesmo lado). Observe que, devido a natureza da
regra imposta à B, ao fazer isto, A cria (ao menos) um lado “banco”, que poderá ser colorido somente
por ela no futuro (caso colora A4 A5 este lado é A3 A4 ). B então faz sua jogada, se puder. Daı́ em
diante, a estratégia de A é colorir um dos lados que está a um lado de distância do conjunto de lados
já coloridos. Observe que cada vez que A segue esta estratégia, ela cria mais um lado “banco”.
Por indução, após o turno de cada um dos jogadores, o conjunto dos lados coloridos e dos lados
“banco” é um arco conexo no polı́gono, ou seja, um conjunto de lados consecutivos. Se, após a
jogada de B, a jogadora A não puder seguir esta estratégia, então este arco satisfaz uma das seguintes
possibilidades
• cobre todos menos um dos lados do polı́gono: neste caso A colore este lado ainda não colorido
e vence.

6
• cobre todos menos dois lados do polı́gono: sejam l1 e l2 estes lados. Neste caso A colore um dos
lados “banco” que ela havia guardado para mais tarde. B então forçosamente colore um dos
dois lados l1 ou l2 , sem perda de generalidade l1 . Em seu último turno A, colore o lado l2 .

OBS: note que o arco não pode cobrir todo o polı́gono, pois neste caso B teria colorido por último
um lado cujos dois vértices pertenceriam também a dois lados já coloridos.
Se n = 3, é fácil ver que A vence automaticamente. Se n = 4 é claro que B vence automaticamente.
Assim, Beto vence apenas quando n = 4.

6. A tem estratégia vencedora. Note que quem tiver que apagar o número 1 perde automaticamente.
Sua primeira jogada é apagar 4 e 8. Daı́ em diante dividimos em casos:

• Se B apaga 2 e 6, os números restantes são 1, 3, 5, 7 e 9. A apaga então 3 e 9, deixando para B


os números 1, 5 e 7. Daı́ em diante eles apagam um número por vez, B sendo forçado a apagar
o 1 no final.

• Se B apaga os números 3, 6 e 9. Os números restantes são 1, 2, 5 e 7, que são apagados um por


vez, até que B forçosamente apaga o número 1 no final.

• Se B apagar 5 ou 7, então A apaga 3, 6 e 9. Restam os números 1, 2, 5 ou 7. Eles são apagados


um por vez (são três números!), B sendo forçado a escolher 1 no final.

• Se B apagar 6 ou 9, então A apaga 9 ou 6 respectivamente. Após isto, ficam no quadro 1, 2, 3,


5 e 7, sendo apagados um de cada vez. Ao fim, B é forçado a apagar o dı́gito 1.

7. Para resolver este problema analisaremos posições vencedoras e perdedoras. Diremos que um
número n é vencedor quando o jogador que receber alguma caixa com n pedras tiver uma estratégia
vencedora escolhendo esta caixa e esvaziando as demais. Diremos que n é perdedor quando o jogador
que escolher uma caixa com n pedras forçosamente entrega a seu oponente uma caixa com um número
vencedor de pedras ou simplesmente não consegue fazer sua jogada.
Os números 1 e 2 são perdedores, pois não se pode fazer uma jogada escolhendo caixas com estes
números de pedras. Por outro lado, 3, 4, 5 e 6 são vencedores, pois uma caixa que tem qualquer um
dentre estes números de pedras pode ser dividida em 3 caixas com 1 ou 2 pedras. Segue que 7 e 8
são números perdedores, pois qualquer divisão entre 3 caixas de uma pilha com 7 ou 8 pedras envolve
ao menos uma caixa com 3, 4, 5 ou 6 pedras. Por outro lado 9 é vencedor, por causa da divisão
1 + 1 + 7 = 9.
Afirmamos que n é vencedor quando n = 6k + 3, 6k + 4, 6k + 5 ou 6k + 6 e perdedor quando
n = 6k + 1 ou 6k + 2 para cada k ≥ 0 inteiro. A prova é por indução forte. O caso base já está feito.
O passo indutivo é o seguinte. Se n = 6(k + 1) + 1 ou 6(k + 1) + 2 pedras estão divididas em três
caixas não-vazias com a, b e c pedras, então um destes valores não deixa resto 1 ou 2 quando dividido
por 6. De fato, se todos deixassem resto 1 ou 2, então a + b + c = n seria um número do tipo 6k 0 + d
com d entre 1 + 1 + 1 = 3 e 2 + 2 + 2 = 6. Mas n = 6k 0 + 1 ou 6k 0 + 2, contradição. Sendo assim,
pela hipótese de indução forte, uma das caixas tem um número vencedor de pedras após a divisão.
Em contrapartida,

6(k + 1) + 3 = 1 + 1 + [6(k + 1) + 1], 6(k + 1) + 4 = 1 + 2 + [6(k + 1) + 1],


6(k + 1) + 5 = 2 + 2 + [6(k + 1) + 1], 6(k + 1) + 6 = 2 + 2 + [6(k + 1) + 2],

e portanto todos estes números são vencedores.


Como 2010 é múltiplo de 6, A vence.

7
8. a) Se n não for múltiplo de 4, A vence; caso contrário, B vence. Se n = 4k + 1, A coloca o
cavalo em qualquer casa da primeira coluna. Se n = 4k + 2 ou 4k + 3, A coloca o cavalo em
qualquer casa da segunda coluna. Note que daı́ em diante cada jogador tem apenas uma jogada
disponı́vel, sempre movendo o cavalo duas casas na horizontal para a direita. No primeiro caso,
após um número ı́mpar de rodadas, A deixa o cavalo na última coluna do tabuleiro e B perde.
O outro caso é semelhante, e após um número ı́mpar de rodadas, A deixa o cavalo ou na última
coluna do tabuleiro ou na penúltima coluna respectivamente e B perde.
Quando n é múltiplo de 4, se A colocar o cavalo na primeira coluna, assim como antes, sempre
há apenas um movimento possı́vel, contudo, após um número par de rodadas, B deixa o cavalo
na penúltima coluna e A perde. Analogamente, se A colocar o cavalo na segunda coluna, os
movimentos são inevitáveis e ao fim B põe o cavalo na última coluna. Por simetria, A também
perde ao colocar o cavalo na última ou penúltima coluna.
Por outro lado, se A põe o cavalo em qualquer outro lugar, a distância dele para a primeira
coluna e a distância para a última coluna são maiores que 2. Uma das duas tem que ser também
do tipo 4k + 2 ou 4k + 3, pois a soma destes dois valores é do tipo 4k + 3. Suponha sem perda de
generalidade que seja para a última coluna. B então move o cavalo para a direita, e daı́ em diante
sempre há apenas um movimento possı́vel. Após um número par de turnos, coloca o cavalo na
última ou na penúltima posição, quando a distância é igual a 4k + 3 e 4k + 2 respectivamente,
vencendo o jogo.

b) Numerando as casas do tabuleiro de 1 a 12, obtemos o seguinte grafo que representa os possı́veis
movimentos do cavalo:

1 10 3 12

7 8 5 6

9 2 11 4

Podemos supor sem perda de generalidade que A põe o cavalo no ciclo da esquerda. Nesse caso
B move o cavalo dentro deste ciclo. Como ele tem 6 casas, se o cavalo nunca sai dele, B faz
o último movimento e vence. Caso A leve o cavalo para o outro ciclo, B passa a movimentar
o cavalo naquele ciclo e vence movendo o cavalo dentro deste cı́rculo. Note que A não pode
retornar ao ciclo anterior, pois um número par de jogadas o separa da outra posição conectada
ao ciclo da esquerda.

9. Hades pode impedir Sı́sifo de completar seu objetivo. Sua estratégia é bastante simples. Sempre
que Sı́sifo move uma pedra existem duas possibilidades: ou ele a move para exatamente um degrau
acima ou ele a move vários degraus para cima. No primeiro caso, Hades simplesmente empurra a
pedra que Sı́sifo moveu um degrau para baixo novamente, fazendo com que ela retorne para a posição
anterior. No segundo caso, obrigatoriamente, havia uma pedra ocupando o degrau imediatamente

8
acima daquele que continha a pedra que Sı́sifo escolheu. Hades move então esta pedra para baixo,
ocupando o degrau vazio que Sı́sifo havia deixado.
Mostremos que esta estratégia permite a Hades vencer. Após cada jogada de Hades, o primeiro
degrau sempre estará ocupado, já que qualquer degrau desocupado por Sı́sifo é preenchido por Hades
logo depois. Além disso, após sua jogada, não existem dois degraus consecutivos vazios abaixo daquele
que contém a pedra mais alta. Isto pode ser provado por indução. No inı́cio, de fato, tal configuração
não existe.
Por outro lado, suponha que Sı́sifo parte de uma configuração em que não existe tal par de degraus.
Se ele move a pedra exatamente um degrau para cima, após a jogada de Hades voltamos para a
configuração anterior, para a qual não existe tal par. Suponha então que ele a move vários degraus
para cima. Considere a maior sequência de degraus consecutivos a partir do escolhido por Sı́sifo tal
que todos continham uma pedra. Denote a por dk , dk+1 , . . . , dl . Após a jogada de Hades, obtemos
uma configuração quase idêntica à anterior, a única diferença é que o bloco de degraus consecutivos
acima do degrau escolhido por Sı́sifo que continham uma pedra, dk+1 , . . . , dl , foi movido um degrau
para cima. Isto cria um degrau vazio acima daquele escolhido por Sı́sifo e diminui em 1 o espaço vazio
acima desta sequência. Em todo caso, o par de degraus consecutivos vazios não é criado, o que conclui
a indução.
Consequentemente, após cada jogada de Hades, a pedra mais alta está no máximo no degrau
número 999. Logo, em sua vez, Sı́sifo pode levar uma pedra no máximo para a posição 1000, mas
nunca para o topo da escada.

10. B tem a estratégia vencedora. Note que todo divisor de um número ı́mpar é também ı́mpar.
Logo, todo jogador que receber uma quantidade ı́mpar de pedras repassa uma quantidade par para
seu oponente. Assim, após a primeira jogada de A, B recebe uma pilha com um número par de pedras.
A estratégia de B é sempre remover exatamente uma pedra da pilha. Por indução , B sempre repassa
para A uma pilha com um número ı́mpar de pedras e logo após recebe de volta de A uma pilha com
um número par de pedras. Dessa maneira, ou B pode jogar ou recebe uma pilha sem pedras, o que
significa que ele venceu.

11. A, B e D podem cooperar para derrotar C. A estratégia é a seguinte. Na primeira jogada de


A e B, eles pintam retângulos 2 × 1 de forma a pintar os quatro quadradinhos centrais do quadrado.
Após cada jogada de C, os jogadores D, A e B pintam o retângulo obtido rodando aquele escolhido
por C a partir do centro em 90°, 180° e 270° respectivamente. Por simetria, sempre que C puder jogar
os outros 3 também podem. Assim, após várias jogadas, ele será o primeiro a não conseguir jogar,
perdendo o jogo.

12. A vence o jogo. Sua estratégia é sempre entregar para B várias pilhas, cada uma delas com um
número ı́mpar de moedas. Como 2010 = 1 + 2019, seu primeiro movimento está garantido. Em geral,
toda pilha com um número par de moedas n pode ser dividida em duas com um número ı́mpar de
moedas, por exemplo 1 e n − 1.
Suponha agora que B recebeu de A várias pilhas, todas elas com um número ı́mpar de moedas.
Se B ainda puder fazer uma jogada, ele tem que pegar ao menos uma das pilhas, que tem um número
ı́mpar maior que 1 de moedas, e dividi-la em duas. Ao fazer isto cria ao menos uma pilha com um
número par, e maior que 1, de moedas. Na vez de A novamente, ele pega todas pilhas com uma
quantidade par de moedas que recebeu de B (note que há ao menos uma) e separa cada uma delas
em duas que tem uma quantidade ı́mpar de moedas, executando assim sua estratégia.
Portanto, enquanto B puder jogar A também poderá jogar. Note que o jogo termina após no
máximo 2009 jogadas, pois o número de pilhas aumenta em 1 após cada jogada. Logo A vence.

9
13. Se n é primo, Bruno tem uma estratégia vencedora; caso contrário, Ana tem.
Seja n um número primo. Em sua primeira jogada, A remove um triângulo de lado k, deixando
a B um trapézio de lados (k, n − k, n − k, n). Sejam a = max (k, n − k) e b = min (k, n − k),
Note mdc(a, b) = mdc(n, n − k) = 1. B então come um triângulo de lado n − k, deixando a A um
paralelogramo de lados a e b. Sempre que A receber um paralelogramos com lados c e d satisfazendo
mdc(c, d) = 1 e d ≤ c, analisemos as possı́veis jogadas.

1. Se A come um triângulo de lado menor que d, B repete sua jogada de forma simétrica (em
relação ao centro do paralelogramo) na outra ponta, e vence o jogo, pois A fica sem jogadas.

2. Se A come um triângulo com lado d e c 6= d, B recebe um trapézio de lados (c − d, d, c, d), tal que
mdc(c − d, d) = mdc(c, d) = 1. B então repete sua estratégia anterior, come um triângulo de
lado d e devolve para A o paralelogramo de lados c − d e d, colocando-a de volta na mesma
situação.

3. Se A come um triângulo com lado d e c = d, então c = d = 1 e B recebeu um pedacinho


triangular de tamanho 1. Isto significa que A perdeu o jogo.

Depois de um certo número de jogadas, o caso 2 acima não acontece e portanto B vence.
Em contrapartida, seja n um número composto e p algum divisor primo de n, de maneira que
n = kp. A inicia comendo um triângulo de tamanho p.
1. Se B comer um triângulo com lado diferente de n − p, A quebra um pedaço de tamanho 1 no
terceiro vértice e vence, pois B fica sem jogadas possı́veis.

2. Se B comer um triângulo de lado n−p, A recebe de volta um paralelogramo de lados p e (k −1)p.


Sendo assim, A come novamente um triângulo de lado p, devolvendo a B um trapézio de lados
(p, (k − 2)p, p, (k − 1)p). Se B comer um triângulo de lado diferente de p, A come um triângulo
de tamanho 1 e deixa B sem jogadas possı́veis. Dessa maneira, A e B alternam seus turnos,
A sempre recebendo de B um paralelogramo de lados p e lp, e repassando a B um trapézio de
lados (p, lp, p, (l − 1)p), até que ao fim repassa a B um triângulo de lados p. Mas neste caso, de
acordo com o provado anteriormente, quem faz a segunda jogada vence, ou seja, A vence.

14. A vence. Para construirmos sua estratégia, analisemos como tem que ser o final do jogo. No
turno anterior a vitória de A, B pintou um quadradinho. Como B joga de forma inteligente, se ela
não conseguiu evitar a vitória de A, então, na sua última vez, haviam dois quadradinhos que poderiam
ser pintados de vermelho para que A vencesse. Este tipo de situação acontece quando os quadradinhos
formam um T, como indicado na figura abaixo.

Note que se B recebe esta posição e não tiver coberto previamente nenhum dos quadradinhos
marcados com um cı́rculo, perde no próximo turno, pois pode cobrir no máximo um deles. Vamos

10
mostrar como A pode criar este tipo de situação. Sejam Ai os quadradinhos marcados por A e Bi
aqueles marcados por B em cada um dos seus turnos i = 1, 2, . . ..
Inicialmente A marca qualquer quadrado A1 e B marca outro B1 . Depois disso, A escolhe outro
quadrado A2 na mesma linha que A1 e à sua direita de maneira que o marcado por B1 esteja ao menos
1000 vezes (exagero) mais próximo de A1 , tanto na direção horizontal, quando na vertical, e que entre
A1 e A2 exista um número ı́mpar de quadradinhos. Temos agora que analisar algumas opções.

1. B pinta seu segundo quadradinho B2 na mesma coluna que um dos quadrados A1 ou A2 , de


maneira que a distância entre B2 e o quadradinho Ai na mesma coluna seja igual a distância
entre A1 e A2 : A faz sua terceira jogada escolhendo A3 como sendo o quadrado médio entre A1
e A2 . Note que a partir daı́, independente da escolha B3 , em seu quarto turno A pode marcar
um dos dois quadradinhos na mesma coluna de A3 , cuja distância a A3 é igual distância de A3
a A2 , de forma a completar a figura T acima ou sua reflexão em relação ao eixo horizontal.

2. Caso contrário e

a. B2 está na mesma coluna que A1 ou à sua direita: A marca A3 na mesma linha que A1 , à
sua esquerda, de maneira que a distância entre A3 e A1 seja a mesma que àquela entre A1
e A2 . Assim como no caso anterior, independente da escolha B3 , em seu quarto turno A
pode marcar um dos dois quadradinhos na mesma coluna de A1 , cuja distância a A1 é igual
distância de A3 a A1 , de forma a completar a figura T acima ou sua reflexão em relação ao
eixo horizontal.
b. B2 está à esquerda A1 : A marca A3 na mesma linha que A2 , à sua direita, de maneira que
a distância entre A3 e A2 seja a mesma que àquela entre A1 e A2 . Daı́ em diante a análise
é análoga ao caso anterior.

15. Eles podem forçar um empate. Note que a escolha de cada jogador nada mais é que o momento
em que decide “passar a vez”, ou seja, o poder de escolha, para o outro jogador. Seja a(k) e b(k) o
número de pontos de A e B respectivamente após k-ésimo turno. Afirmamos que se no k-ésimo turno
A não possuir a carta k − 1, então qualquer sequência de jogadas que ela fizer antes de passar a vez
para B aumenta a diferença a(k) − b(k) em no máximo 1. Com efeito, se ela passar a vez ao pegar a
carta k + l, isto significa que ela deu as cartas k, k + 1, ..., k + l − 1 para B. A diferença
 
l−1
[a(k + l) − b(k + l)] − [a(k) − b(k)] = k + l − [k + (k + 1) + · · · + (k + l − 1)] = 1 − l − (l − 1)k
2

é positiva se, e somente se, l = 1. Além disto, neste caso ela é exatamente 1. Consequentemente, por
simetria, a melhor estratégia para ambos jogadores é passar o número k a seu rival e pegar k + 1 para
si. Contudo, isto implica que cada um joga um número par de vezes, pois 2020 é múltiplo de 4, e ao
fim do jogo as pontuações deles serão iguais (1020605).

16. a) Ednalva não pode impedir Ludmilson de vencer. Divida o tabuleiro em 25 quadrados 2 × 2.
Na sua vez, ele sempre remove um destes quadrados do tabuleiro. Mesmo que Ednalva escolha
em cada turno remover um quadradinho 1 × 1 de um quadrado 2 × 2 ainda intocado, após 12
jogadas de cada um deles, ainda restará um quadrado 2 × 2 para Ludmilson remover. Dessa
maneira, ele sempre remove ao menos 4 × 13 = 52 quadradinhos e vence.

b) Se n é par, assim como na parte anterior, Ednalva não vencerá. Sendo n = 2k, dividimos o
tabuleiro em k 2 quadrados 2 × 2. A estratégia é sempre remover um deles a cada vez. Assim
2
como na parte anterior, ao fim ele remove ao menos 4d k2 e quadradinhos 1 × 1 e Ednalva no
2
máximo 4b k2 c.

11
Se n = 2k + 1 é ı́mpar, Ednalva tem a estratégia vencedora. Considere as colunas e as linhas
enumeradas de 1 a n e pinte os quadradinhos 1 × 1 correspondentes as interseções de uma
coluna par com uma linha par, num total de k 2 quadradinhos. Note que todo quadrado 2 × 2
contém exatamente um quadradinho pintado. A estratégia de Ednalva é sempre remover um
2
quadradinho pintado. Isto garante que Ludmilson remove no máximo d k2 e quadrados 2 × 2. Em
2
contrapartida, ela remove todos os outros. Como 8d k2 e ≤ (2k + 1)2 , Ednalva é a vencedora.

17. Apesar do conjunto de jogadas permitidas a uma jogadora após receber uma pilha com n pedras
depender da jogada anterior de sua oponente, resolveremos este problema através da ideia de análise
de posições perdedoras. Note que isto causa certa complicação, pois, se A recebe uma pilha com 8
pedras e repassa 3 a B, esta vence removendo todas as pedras. Por outro lado, se A recebe uma pilha
com 6 pedras e repassa 3 a B, esta é forçada a remover 1 ou 2 pedras da pilha, perdendo logo em
seguida. Não podemos então dizer que 3 é posição vencedora, nem perdedora, pois tudo depende de
qual jogada foi realizada previamente pela oponente.
Chamaremos então um número n de posição perdedora quando, independentemente da jogada
anterior da sua rival, a jogadora que receber n pedras não for capaz de impedir a vitória de sua
oponente. Uma posição perdedora pequena é 7. De fato, se a jogadora X receber 7 pedras, tem (no
máximo) as seguintes opções:

• remover a pedras, a = 2, 3, 4, 5: como resposta, sua oponente Y remove as outras 7 − a pedras,


já que 7 − a 6= a.

• remover 1 pedra: Sua oponente Y recebe 6 pedras e remove 3 delas. Como X não pode remover
as 3, por causa da regra do jogo, é forçada a remover a pedras, a = 1, 2; e então Y vence
removendo as outras 3 − a 6= a pedras.

Observe que talvez apenas a primeira opção esteja disponı́vel, caso no turno anterior existissem 8
pedras na pilha. Mas isto não faz diferença, para nós basta mostrar que X perde em todos os casos
possı́veis. Analogamente, 13 é outra posição perdedora. Com efeito, se X recebe 13 pedras, tem (no
máximo) as seguintes opções:

• remover a pedras, a = 1, 2, 4, 5: como resposta, sua oponente Y remove as outras 6 − a pedras,


repassando para X um total de 7 pedras, situação que já sabemos ser perdedora.

• remover 3 pedras: Sua oponente Y recebe 10 pedras e remove 5 delas. Como X não pode remover
as 5 que sobram, por causa da regra do jogo, é forçada a remover a pedras, a = 1, 2, 3, 4; e
então Y vence removendo as outras 5 − a 6= a pedras.

Este padrão se repete, e iremos provar por indução que, para todo m ∈ N, os números 13m e
13m + 7 e são posições perdedoras. O caso m = 0 já está feito (0 é posição perdedora por causa da
condição de vitória). O passo indutivo é, basicamente, o mesmo argumento acima.
Com efeito, se X recebe 13m + 13 pedras tem as seguintes opções:

• remover a pedras, a = 1, 2, 4, 5: como resposta, sua oponente Y remove 6−a pedras, repassando
para X um total de 13m + 7 pedras, situação perdedora pela hipótese de indução.

• remover 3 pedras: Sua oponente Y recebe 13m + 10 pedras e remove 5 delas. Como X não
pode remover 5, por causa da regra do jogo, é forçada a remover a pedras, a = 1, 2, 3, 4. Em
seguida Y remove 5 − a 6= a pedras e devolve a X um total de 13m pedras, posição perdedora
pela hipótese de indução.

Ademais, se X recebe 13m + 20 pedras, tem as seguintes opções:

12
• remover a pedras, a = 2, 3, 4, 5: como resposta, sua oponente Y remove 7 − a pedras, já que
7−a 6= a, devolvendo a X um total de 13m+13 pedras, posição perdedora pela hipótese indutiva.
• remover 1 pedra: Sua oponente Y recebe 13m + 19 pedras e remove 3 delas. Como X não pode
remover 3, por causa da regra do jogo, é forçada a remover a pedras, a = 1, 2, 4, 5. Se escolheu
a = 1 ou 2, Y remove 3 − a 6= a pedras e repassa a X um total de 13m + 13 pedras, posição
perdedora pelo provado acima. Caso tenha escolhido a = 4 ou 5, Y remove 9 − a 6= a pedras,
repassando a X um total de 13m + 7 pedras, posição perdedora pela hipótese.
Consequentemente, 1996 é uma posição perdedora, e A vence removendo 4 pedras em sua primeira
jogada.

18. Este é um exemplo de jogo de fuga e nestes tipos de problemas a solução passa por√considerações
geométricas. A condição do enunciado pode ser trocada pela condição vlobo < vcoelho · √2 e o fato de
ser estritamente menor é importante. Tendo isto em mente, seja a = vlobo /vcoelho < 2. Quando o
coelho percorre x, os lobos percorrem no máximo a · x.
No fim o coelho escapa usando a seguinte estratégia. Seja L o comprimento do quadrado ABCD
em questão. Note que
√ !
L a 2
b= 1−
2 2

é positivo, porque a < 2 . Inicialmente, o coelho corre em linha reta na direção de um dos vértices
do quadrado, sem perda de generalidade A. Quando atingir um ponto P cuja distância aos lados
adjacentes AB e AD for igual a c, um valor qualquer menor que b, ele observa a posição do lobo l1 ,
que estava no vértice A no princı́pio. Sejam Q e R as interseções da perpendicular a AP por P com
os lados AB e AD do quadrado respectivamente. Afirmamos que, se l1 não estiver no interior do lado
AB, o coelho corre para o ponto Q e foge; se l1 não estiver no interior do lado AD, o coelho corre
para o ponto R e foge.

A Q B
P
R

D C

Figura mostrando a localização dos pontos P , Q e R.

Com √ efeito, no primeiro caso, indo de P até alcançar o √ponto Q, o coelho percorre uma distância
igual a c 2. Como a distância de A a Q é igual a 2c > ac 2, l1 não alcança o coelho (note que ele
tem que estar no lado AD).
√ Mais ainda, até alcançar o ponto Q, o coelho percorre desde√ o princı́pio
uma distância igual a L 2/2. Isto significa que os outros lobos andaram no máximo a · L 2/2. Como
√ √
2 2
2c + a · L < 2b + a · L = L,
2 2
nenhum dos outros lobos consegue chegar até o ponto Q a tempo de capturar o coelho (pois suas
distâncias originais a A eram de ao menos L e AQ tem comprimento 2c). A análise do outro caso é

13
totalmente análoga. Como l1 não pode estar ao mesmo tempo no interior de AB e no interior de AD,
o coelho foge.

19. Neste problema, não estamos diante de uma ideia de um jogo em turnos, nem de um jogo entre
rivais, mas sim de um jogo cooperativo. O que ele nos pede é mostrar que se o número n é uma
posição vencedora para os jogadores, o número 2n também é. Nosso objetivo então é mostrar que a
partir de uma estratégia vencedora para n, podemos construir uma estratégia vencedora para 2n.
Primeiramente, vamos entender de maneira abstrata o que significa uma estratégia vencedora para
n. O papel da mágica é, a partir de uma sequência de n caras e coroas, determinar o número escolhido
pela audiência. Isto significa que ela e sua assistente tem (em suas cabeças) uma maneira de associar
cada sequência a exatamente um número de 1 a n. Em outras palavras, uma função fn : Xn →
{1, 2, . . . , n}, onde Xn é o conjunto de todas sequências de n caras ou coroas (equivalentemente
Xn = {0, 1}n ). Em contrapartida, dado qualquer j de 1 a n, a assistente tem que ser capaz de, a
partir de qualquer sequência s de caras e coroas, trocar o lado de uma das moedas assim obtendo uma
sequência s0 tal que fn (s0 ) = j. Se estas condições forem satisfeitas, elas conseguem fazer seu truque
(vencem o jogo).
Considere então a situação com 2n moedas. Note que uma fila r de 2n moedas pode ser separada
em 2 filas com n moedas, r1 das n primeiras moedas e r2 das últimas. Sendo assim, quando a mágica
voltar ao palco e olhar para a fila, pode usar a associação em sua cabeça, a função fn , e obter dois
números fn (r1 ) e fn (r2 ) entre 1 e n. Ela poderia até tentar criar uma associação entre pares de
números (a, b) satisfazendo 1 ≤ a, b ≤ n e números de 1 a 2n diretamente, mas ao fazer isto estaria
desperdiçando muita informação que está disponı́vel a ela1 .
De fato, seja I(s) o número de coroas aparecendo na sequência s ∈ Xn . Atente ao fato que o ato
de alterar a face de uma moeda de uma sequência s ∈ Xn troca a paridade de I(s). Podemos usar
a paridade de I(r1 ) para determinar se o número está entre 1 e n ou entre n + 1 e 2n. Além disso,
para cada par (a, b) satisfazendo 1 ≤ a, b ≤ n, seja g(a, b) o número entre 1 e n tal que a + b ≡ g(a, b)
mod n. Dessa forma, a mágica constrói a função
(
g(r1 , r2 ), se I(r1 ) é par,
f2n (r) = f2n (r1 , r2 ) =
g(r1 , r2 ) + n, se I(r1 ) é ı́mpar.

Mostremos que a assistente pode fazer sua jogada de maneira que o truque funcione. Seja r = (r1 , r2 )
a sequência arbitrária sorteada pela audiência e j o número escolhido. Devido à definição da função
f2n , se

• j está entre 1 e n e I(r1 ) é par, a assistente fará sua alteração em alguma moeda de r2 ,

• j está entre 1 e n e I(r1 ) é ı́mpar, a assistente fará sua alteração em alguma moeda de r1 ,

• j está entre n + 1 e 2n e I(r1 ) é par, a assistente fará sua alteração em alguma moeda de r1 ,

• j está entre n + 1 e 2n e I(r1 ) é ı́mpar, a assistente fará sua alteração em alguma moeda de r2 ,

e assim passa para a mágica a informação sobre qual caso ela deve considerar em sua função. Se a e b
são os números que a assistente vê ao usar a função fn , a correspondendo à fila que ela irá fazer sua
alteração, ela altera esta fila de modo a obter uma sequência s0 ∈ Xn tal que fn (s0 ) = c é o número de
1 a n tal que c + b ≡ j mod n. Esta alteração é sempre possı́vel, por hipótese, e implica que a mágica
recebe a informação g(c, b) correta, concluindo a prova.
1
Na verdade, tal tentativa se provaria insuficiente. Tal associação teria que ser capaz de, a partir de qualquer par
(a, b), gerar, através de mudança em uma das coordenadas, todos os números de 1 a 2n, por causa da jogada da assistente.
Entretanto, a partir do par (1, 1), por exemplo, geramos no máximo 2n − 1 pares distintos.

14
20. Este é um problema interessante porque, como veremos adiante, não precisamos inventar uma
estratégia para o vencedor. As jogadas escolhidas pelo vencedor não farão diferença e, em certo sentido,
o resultado final é inevitável. Antes de mais nada, a resposta é: se n é par, o primeiro jogador, que
chamaremos de A, vence; se n é ı́mpar, o segundo jogador, que chamaremos de B, vence.
Começamos a nossa demonstração com a seguinte crucial afirmação:
Afirmação: Toda diagonal do polı́gono é cruzada por um número par de diagonais.
Prova: Com efeito, os vértices não pertencentes a esta diagonal são separados por ela em dois
grupos, um com a vértices e o outro com b vértices. Segue então que ab diagonais cruzam esta
diagonal. Como a + b = 2n − 1, um destes número é par, logo o produto ab é par.
Provada a afirmação, seja S(k) o conjunto de diagonais desenhadas ao final do k-ésimo turno e
T (k) o conjunto das diagonais que ainda não foram desenhadas ao fim do mesmo turno. Note que
S(k)∪T (k) é o conjunto de todas as diagonais do polı́gono, portanto tem (2n+1)(n−1) elementos. Seja
também I(k) o número de cruzamentos entre diagonais desenhadas e diagonais ainda não desenhadas
ao fim do k-ésimo turno, em outras palavras, o número de pares (s, t); s ∈ S(k), T ∈ T (k), tais que s
cruza t. Afirmamos agora que, ao longo do jogo, I(k) é sempre par.
De fato, I(0) = 0 e I(1) é par pela afirmação demonstrada acima. Provemos então por indução.
Para cada t ∈ T (k), seja fk (t) o número de pares (s, t), s ∈ S(k), tais que s cruza t. Note que
X
I(k) = fk (t).
t∈T (k)

Se I(n) é par e um jogador consegue fazer sua jogada no n + 1-ésimo turno desenhando uma diagonal
r, calculemos I(n + 1). Como desenhar r é uma jogada válida, fn (r) é par. Juntando este fato com
a afirmação acima, concluı́mos que r cruza um número par de elementos de T (n), que denotaremos
por 2m. Em contrapartida, S(n + 1) = S(n) ∪ {r} e T (n + 1) = T (n) \ {r}. O valor de fn+1 (t) para
cada t ∈ T (n + 1) é igual a fn (t) para todos os t’s que não cruzam r e aumenta em 1 para todos os
2m outros t’s. Consequentemente,

I(n + 1) = I(n) − fn (r) + 2m

também é par, e a indução está completa.


Em vista disso, todo jogador que, no turno k + 1, receber um conjunto T (k) com um número ı́mpar
de elementos poderá fazer sua jogada. De fato, ao menos um dos valores fk (t), t ∈ T (k), tem que ser
par, pois a soma de todos eles é I(k), que é par. Dessa forma, a diagonal correspondente a tal valor
cruza um número par de diagonais já desenhadas e pode ser escolhida pelo jogador em questão.
Note porém que o número de elementos de T (k) é igual a (2n − 1)(n − 1) − k, e é ı́mpar quando
n é par e k é par - ou seja, na vez de A; ou quando n é ı́mpar e k é ı́mpar - ou seja, na vez de B.
Portanto, se n é par, B fica sem jogadas antes de A; e se n é ı́mpar, A fica sem jogadas antes de B, o
que conclui a prova.

Material elaborado por Hugo Fonseca Araújo

15
Problemas Resolvidos

Nı́vel 2

Princı́pio da Casa dos Pombos

Material elaborado por Susana Frómeta Fernández


Problemas

Problemas
Problema 1. Cinquenta e um pontos são postos no interior de um quadrado de lado 1 metro. Prove
que existe um conjunto de três desses pontos que podem ser cobertos por um quadrado de lado 20
centı́metros.

Problema 2. Prove que em um conjunto de cinco inteiros quaisquer, sempre há três cuja soma é
divisı́vel por 3.

Problema 3. Mostre que em um subconjunto de 51 elementos de {1, 2, 3, . . . , 100} sempre existem


dois cuja a diferença é 10.

Problema 4. Prove que existe uma potência de 3 terminada nos dı́gitos 001 (na base decimal).

Problema 5. Prove que todo número natural possui um múltiplo positivo que se escreve apenas com
os algarismos 0 e 1 na base decimal.

Problema 6 (Longlist IMO 1977 - Romênia). Dados 37 pontos no espaço com coordenadas inteiras,
prove que pelo menos um dos triângulos formado por três destes pontos possui o baricentro com
coordenadas inteiras.

Problema 7. O plano é pintado usando duas cores. Prove que existem dois pontos de mesma cor
distando exatamente um metro.

Problema 8 (Putnam). O plano é pintado usando três cores. Prove que existem dois pontos da
mesma cor distando exatamente um metro.

2
Soluções

1. Divida o quadrado em 25 quadrados menores de lado 20 centı́metros. Observe que se há 25 casas
para 51 pombos, pelo menos uma casa terá no mı́nimo 3 pombos. Aqui estamos usando uma versão
generalizada do Princı́pio da Casa dos Pombos: “Se há n casas para nk + 1 pombos, ao menos uma
casa terá mais que k pombos.”

2. Os possı́veis restos na divisão por 3 são 0, 1 e 2. Vamos dividir em casos:


Caso 1. Os três possı́veis restos aparecem: Nesse caso, basta tomar um número com cada resto e
somá-los, pois 0+1+2=3.
Caso 2. Algum dos restos não aparece: Nesses caso, como há 5 números e só dois possı́veis restos,
algum dos restos deverá aparecer 3 vezes, ao menos. Basta então somar três números com o mesmo
resto.

3. Considere os seguintes 50 subconjuntos de {1, . . . , 100}:


Ak,i = {20k + i, 20k + i + 10}, para k = 0, 1, . . . , 4 e i = 1, . . . , 10.
Tomando quaisquer 51 elementos de {1, . . . , 100}, algum dos conjuntos acima deve conter pelo menos
2 desses elementos.

4. Considere a sequência 31 , 32 , 33 , . . ., das potências de 3. Os possı́veis restos na divisão por 1000


formam um conjunto finito (com 1000 elementos). Portanto, pelo PCP, devem existir j < k tais que
as potências 3j e 3k deixam o mesmo resto na divisão por 1000. Assim, 3k − 3j ≡ 0 ( mod 1000).
Donde segue que 3j (3k−j − 1) ≡ 0 ( mod 1000). Como mdc(3, 1000) = 1, podemos concluir então que
3k−j − 1 ≡ 0 ( mod 1000). E daı́ 3k−j ≡ 1. Portanto, 3k−j termina em 001.

5. Seja n um número natural qualquer. Os possı́veis restos na divisão por n formam um conjunto finito
com n elementos, {0, 1, . . . , n − 1} Considere a sequência 1, 11, 111, 1111, 11111, . . .. Como essa lista é
infinita deve existir um resto na divisão por n que se repete. Isto é, existe algum r ∈ {0, . . . , n − 1} e
inteiros i < j tais que o número A formado por j algarismos 1 e o número B formado por i algarismos
1 deixam o mesmo resto r na divisão por n. A diferença A − B é então um múltiplo de n formado
por j − i algarismos 1 seguidos de i algarismos 0.

6. Sejam Pk = (xk , yk , zk ), k = 1, . . . , 37 os pontos. O baricentro do triângulo Pi Pj Pk é o ponto


 
x i + x j + x k yi + yj + yk zi + zj + zk
, , .
3 3 3

Então, precisamos mostrar que existem 3 pontos Pi , Pj , Pk tais que as somas xi + xj + xk , yi + yj + yk e


zi + zj + zk sejam múltiplos de 3. Vamos aplicar o PCP por etapas. Olhemos primeiro para a primeira
coordenada. Há 3 restos possı́veis na divisão por 3. Como 37 = 3 × 12 + 1, podemos obter um conjunto
com 13 pontos tais que em todos a primeira coordenada deixa o mesmo resto na divisão por 3. Sem
perda de generalidade, renomeando os pontos, podemos supor que esses pontos são P1 , . . . , P13 . Agora
olhemos para a segunda coordenada. Como 13 = 3 × 4 + 1 podemos obter um conjunto de 5 pontos,
tais que em todos a segunda coordenada também deixa um mesmo resto na divisão por 3. Sem perda

3
de generalidade, suponha que os pontos sejam P1 , . . . , P5 . Ao tomarmos quaisquer 3 desses pontos, as
duas primeiras coordenadas do baricentro serão inteiras. Falta analisar a terceira coordenada. Para
isso basta usar o resultado do Problema 2 desta lista: dados 5 inteiros sempre podemos achar 3 cuja
soma é divisı́vel por 3.

7. Desenhe no plano um triângulo equilátero de lado 1 metro. Como só há duas cores, haverá dois
vértices do triângulo com a mesma cor.

8. Suponha, para gerar um absurdo, que seja possı́vel pintar o plano com 3 cores de modo que pontos
a uma distância de 1m sempre tenha cores diferentes. Considere o losango que é formado quando
se unem dois triângulos equiláteros de lado 1m, de modo que eles compartilhem √ um lado. Os lados
desse losango medem 1m, uma das diagonais mede 1m e a outra diagonal mede 3m. É fácil ver que
os dois vértices mais afastados desse losango devem ser pintados com
√ a mesma cor. Esse argumento
mostra que nesse plano quaisquer dois pontos a uma distância √ de 3m deve ter a mesma cor. Fixe
agora um ponto do plano e considere a circunferência de raio 3 em torno desse ponto. Todos os
pontos dessa circunferência devem ter a mesma cor. Como o diâmetro dessa circunferência é maior
que 1 podemos tomar uma corda de comprimento 1. As duas extremidades dessa corda têm a mesma
cor. Contradição.

Relacionado aos problemas anteriores, veja o Problema de Hadwiger-Nelson.

É possı́vel pintar o plano usando 7 cores de modo que quaisquer dois pontos distando 1 metro
entre si tenham cores diferentes. Para isso considere um ladrilhamento do plano usando hexágonos
com diâmetro um pouco menor que 1.

Material elaborado por Susana Frómeta Fernández

4
Problemas Resolvidos

Nı́vel 2

Princı́pio da Casa dos Pombos II

Material elaborado por Valentino Amadeus Sichinel


Problemas

Problema 1. Em uma universidade, um professor de matemática ensina duas matérias por semes-
tre. Sabendo que o professor esteve na universidade por 8 anos e que há 15 matérias distintas de
matemática, prove que há ao menos uma matéria que o professor ensinou mais de uma vez.

Problema 2. Escolhemos 17 pontos dentro de um triângulo equilátero de lado 1. Prove que há dois
deles que estão a uma distância menor que ou igual a 14 .

Problema 3. Um criador de pombos tem 200 pombos vivendo em gaiolas. Ao todo, são 101 gaiolas,
nenhuma das quais está vazia. Prove que é possı́vel escolher um subconjunto de gaiolas de tal modo
que o total de pombos vivendo nelas é exatamente 100.

Problema 4. O plano é pintado usando três cores. Prove que existe um retângulo cujos vértices têm
todos a mesma cor.

Problema 5. A sequência de Fibonacci, (Fn ), é definida da seguinte forma:

F0 = 0, F1 = 1 e, para n ≥ 1, Fn+1 = Fn + Fn−1 .

Prove que existe um elemento de sequência de Fibonacci que é divisı́vel por 1000.

Problema 6 (Putnam). Sejam n um inteiro positivo, e X = {x1 , x2 , . . . , xn } um conjunto qualquer


de números reais. Prove que existem um subconjunto não vazio S ⊂ X e um inteiro m tais que
X 1
m+ s ≤ .
n+1
s∈S

Problema 7 (Putnam). São escolhidos 5 pontos em uma esfera. Mostre que existe um hemisfério
(uma metade da esfera, obtida “cortando” a esfera ao meio em alguma direção) que contém pelo menos
4 dos pontos escolhidos.
Obs.: Para os propósitos deste problema, os pontos que estão na borda dos hemisférios (os pontos
que estão sobre a linha pela qual a esfera foi “cortada”) pertencem a ambos os hemisférios.

Problema 8 (Erdös–Szekeres). Sejam a e b inteiros positivos. Mostre que, dada uma sequência de
ab + 1 números reais, existe uma subsequência monótona não-decrescente de comprimento pelo menos
a + 1 ou uma subsequência monótona não-crescente de comprimento pelo menos b + 1.

2
Soluções

1. Trata-se de uma aplicação imediata do princı́pio da casa dos pombos: se o professor não tivesse
ensinado nenhuma matéria mais de uma vez, ele teria ensinado no máximo 15 matérias, mesmo
contando possı́veis repetições (pois há 15 matérias na universidade). No entanto, ensinando duas
matérias por semestre durante 8 anos, o professor ensinou ao todo 16 matérias, contando possı́veis
repetições. Absurdo.

2. Utilizando os pontos médios, dividimos o triângulo original em quatro triângulos equiláteros. De-
pois, aplicamos o mesmo processo a cada um dos triângulos menores, obtendo uma divisão do triângulo
original em 16 triângulos equiláteros, todos de lado 14 .

Pelo princı́pio da casa dos pombos, ao menos dois dos 17 pontos pertencem ao mesmo triângulo
menor. Como a distância entre dois pontos de um triângulo equilátero de lado 14 é no máximo 14 ,
concluı́mos que há, dentre os pontos que escolhemos, dois deles que estão a uma distância menor que
ou igual a 14 .

3. Sejam g1 , g2 , g3 , . . . , g101 as gaiolas. Para cada k ∈ {1, 2, . . . , 101}, seja Ck o conjunto

{g1 , g2 , g3 , . . . , gk },

e seja nk o total de pombos vivendo nas gaiolas de Ck .


Dos fatos de que nenhuma das gaiolas está vazia e de que o total de pombos é 200, segue que
0 < n1 < n2 < · · · < n101 = 200.
Para cada i ∈ {1, 2, . . . , 100}, definamos pi como sendo o par {i, 100 + i}. Pelo que acabamos de
observar, cada um dos números n1 , n2 , . . . , n101 pertence a um dos pares p1 , p2 , . . . , p100 . Como são
101 números e 100 pares, pelo princı́pio da casa dos pombos existem ni e nj que pertencem ao mesmo
par, digamos, pl . Assim, para alguns i, j e l, as gaiolas

g1 , g2 , g3 , . . . , gi

contêm, juntas, l pombos, enquanto que as gaiolas

g1 , g2 , g3 , . . . , gi , . . . , gj

3
contêm, juntas, 100 + l pombos.
Nessa situação, as gaiolas
gi+1 , gi+2 , . . . , gj
hão de conter, juntas, exatamente 100 pombos.

4. Escolha quatro retas r1 , r2 , r3 , r4 paralelas entre si e 34 + 1 = 82 retas s1 , s2 , . . . , s82 perpendiculares


a elas e paralelas entre si.
Para cada uma das retas s, observe as interseções entre ela e as retas r1 , r2 , r3 , r4 . Cada interseção
é um ponto, que pode ter uma dentre três cores. Assim, a configuração formada pelas cores das
interseções (s ∩ r1 , s ∩ r2 , s ∩ r3 , s ∩ r4 ) é uma dentre 34 = 81 possı́veis configurações. Como há 82
retas s, pelo princı́pio da casa dos pombos, ao menos uma configuração se repete. Em outras palavras,
pelo princı́pio da casa dos pombos, existem i e j tais que si ∩ rk tem a mesma cor que sj ∩ rk , para
cada k = 1, 2, 3, 4.
Como são três cores possı́veis, devem existir, mais uma vez pelo princı́pio da casa dos pombos, k1
e k2 tais que si ∩ rk1 tem a mesma cor que si ∩ rk2 .
Dessa forma, si ∩ rk1 , si ∩ rk2 , sj ∩ rk1 e sj ∩ rk2 têm todos a mesma cor. Como as retas r são
paralelas entre si e as retas s são perpendiculares às retas r (e paralelas entre si), os pontos si ∩ rk1 ,
si ∩ rk2 , sj ∩ rk1 e sj ∩ rk2 formam um retângulo.
Existe, portanto, um retângulo cujos vértices têm todos a mesma cor.

5. Considere os pares (Fi , Fi+1 ), formados por termos consecutivos da sequência de Fibonacci, e os
separe em conjuntos, de acordo com os restos das divisões de cada um dos termos por 1000: dois pares
(Fi , Fi+1 ) e (Fj , Fj+1 ) pertencem ao mesmo conjunto se, e somente se, Fi e Fj deixam o mesmo resto
quando divididos por 1000 e Fi+1 e Fj+1 deixam o mesmo resto quando divididos por 1000.
Como existem infinitos pares e apenas uma quantidade finita de possı́veis combinações (r, s), em
que r e s são restos da divisão de um número inteiro por 1000, segue do princı́pio da casa dos pombos
que existem dois pares, digamos, (Fi , Fi+1 ) e (Fj , Fj+1 ), que pertencem ao mesmo conjunto. Assim,
Fi e Fj deixam o mesmo resto quando divididos por 1000, assim como Fi+1 e Fj+1 . Sem perdas,
suponhamos i < j.
De Fj+1 = Fj + Fj−1 , podemos escrever Fj−1 = Fj+1 − Fj . Da mesma forma, Fi−1 = Fi+1 − Fi .
Assim, Fj−1 e Fi−1 também deixam o mesmo resto quando divididos por 1000.
Repetindo o mesmo argumento, mostra-se por indução que, para qualquer k entre 0 e i, Fj−k e
Fi−k deixam o mesmo resto quando divididos por 1000.
Em particular, Fj−i e F0 deixam o mesmo resto quando divididos por 1000. Como F0 = 0, segue
daı́ que Fj−i é divisı́vel por 1000.

6. Todo número real x pode ser escrito de modo único na forma

x=m+r

com m inteiro e r ∈ [0, 1). Ao número r, chamamos de parte fracionária de x, e o representamos


costumeiramente por {x}.
Definamos, para cada k ∈ {1, 2, . . . , n},

sk := x1 + x2 + · · · + xk .

Consideremos as partes fracionárias, {sk }, das somas sk .

4
h i
1
Se existe k tal que {sk } ∈ 0, n+1 , o problema está resolvido: temos

k
X 1
x1 + x2 + · · · + xk = m + {sk } =⇒ (−m) + xk = {sk } ≤
n+1
i=1

para algum inteiro m. h 


n
Da mesma forma, se existe k tal que {sk } ∈ n+1 , 1 , o problema está resolvido: temos

k
X 1
x1 + x2 + · · · + xk = m + {sk } =⇒ (−m − 1) + xk = | − 1 + {sk }| = 1 − {sk } ≤
n+1
i=1

para algum inteiro m.  


1 n
Suponhamos, então, que para cada k ∈ {1, 2, . . . , n}, {sk } ∈ n+1 , n+1 .
 
1 n
Repartamos o intervalo n+1 , n+1 em n − 1 intervalos menores, escrevendo
       
1 n 1 2 2 3 n−1 n
, = , ∪ , ∪ ··· ∪ , .
n+1 n+1 n+1 n+1 n+1 n+1 n+1 n+1

Como estamos lidando com n somas sk e temos, ao todo, n−1 intervalos, deve haver, pelo princı́pio
da casa dos pombos, duas somas cujas partes fracionárias estão no mesmo intervalo. Assim, existem
i e j, com i < j, tais que {si } e {sj } pertencem ambos a um mesmo intervalo
 
k k+1
, .
n+1 n+1
1
Daı́, |{si } − {sj }| ≤ n+1 . Escrevamos si = m1 + {si } e sj = m2 + {sj } (com m1 e m2 inteiros).
Segue daı́ que
j
X
(−m2 + m1 ) + xt = |(−m2 + m1 ) + (sj − si )|
t=i+1

= |(sj − m2 ) − (si − m1 )|
= |{sj } − {si }|
1
≤ .
n+1
Portanto, o subconjunto {xi+1 , xi+2 , . . . , xj } ⊂ X satisfaz a condição desejada.

7. Dos cinco pontos, escolhemos dois e dividimos a esfera (em dois hemisférios) através de uma
linha que passa por eles1 . Pelo princı́pio da casa dos pombos, ao menos dois dos três demais pontos
pertencem ao mesmo hemisfério. Como os dois pontos que escolhemos inicialmente pertencem a
ambos os hemisférios (pois estão sobre o corte), segue daı́ que há ao menos quatro pontos no mesmo
hemisfério.

1
É sempre possı́vel dividir a esfera em dois hemisférios fazendo o “corte” passar por dois pontos dados. De fato, dados
dois pontos no espaço, existe um plano que contém a ambos e ao centro da esfera. Se cortarmos através desse plano,
dividiremos a esfera exatamente ao meio.

5
8. A cada elemento da sequência, associe um par de inteiros positivos (m, n), sendo m o comprimento
da maior subsequência monótona não-decrescente que termina no elemento (incluindo o próprio), e n
o comprimento da maior subsequência monótona não-crescente que termina no elemento (incluindo o
próprio). Veja que m e n são sempre positivos, pois no pior dos casos podemos tomar a subsequência
formada apenas pelo próprio elemento, que é monótona não-crescente e monótona não-decrescente e
tem comprimento 1.
A observação-chave é que, se e1 e e2 são elementos distintos da sequência, então os pares associados
a eles, digamos, (m1 , n1 ) e (m2 , n2 ), respectivamente, são distintos. De fato, suponhamos, sem perdas,
que e1 aparece antes de e2 . Então, se e1 ≤ e2 , toda subsequência monótona não-decrescente que
termina em e1 pode ser estendida a uma subsequência monótona não-decrescente que termina em e2 ,
donde m2 ≥ m1 + 1. Por outro lado, se e1 ≥ e2 , toda subsequência monótona não-crescente que
termina em e1 pode ser estendida a uma subsequência monótona não-crescente que termina em e2 ,
donde n2 ≥ n1 + 1. Isso mostra que (m1 , n1 ) e (m2 , n2 ) são sempre distintos.
Se não houvesse subsequência monótona não-decrescente de comprimento pelo menos a + 1 e
tampouco houvesse subsequência monótona não-crescente de comprimento pelo menos b + 1, então
cada um dos pares (m, n) que associamos aos elementos da sequência original seria tal que m ≤ a e
n ≤ b, isto é, tal que m ∈ {1, 2, . . . , a} e n ∈ {1, 2, . . . , b}. No entanto, nessas condições, terı́amos no
máximo ab pares distintos. Absurdo! Dessa forma, existe uma subsequência monótona não-decrescente
de comprimento pelo menos a + 1 ou uma subsequência monótona não-crescente de comprimento pelo
menos b + 1.

Material elaborado por Valentino Amadeus Sichinel

6
Problemas Resolvidos

Nı́vel 2

Grafos

Material elaborado por Hugo Fonseca Araújo


Problemas

Problema 1. Mostre que em um grafo qualquer o número de vértices de grau ı́mpar é par.

Problema 2. Existe algum grafo cujos vértices tem grau 4, 4, 4, 4, 2?

Problema 3. Cada um dos 50 cientistas participando em uma conferência conhece pelo menos outros
25 cientistas também nela presentes. Prove que existem 4 cientistas que podem se sentar em uma
mesa circular de forma que cada um deles conheça seus dois vizinhos.

Problema 4. Existe algum poliedro que tenha um número ı́mpar de faces tais que cada uma delas
tem um número ı́mpar de arestas?

Problema 5. Prove que existe um grafo com 2n vértices cujos graus são 1, 1, 2, 2, . . . , n, n.

Problema 6. Em um paı́s todos os pares de cidades são conectados por estradas de mão única. Prove
que existe uma cidade a partir da qual é possı́vel viajar para qualquer outra.

Problema 7. Uma árvore é um grafo conexo sem ciclos.

a) Seja G um grafo no qual todo vértice tem grau maior ou igual a 2. Mostre que G tem um ciclo.

b) Prove que em toda árvore com pelo menos uma aresta existe algum vértice de grau 1.1

c) Prove que se uma árvore tem n vértices então ela tem exatamente n − 1 arestas.

d) Todo grafo de n vértices com n − 1 arestas é uma árvore?

e) Mostre que se um grafo tem n vértices e mais que n − 1 arestas, então ele tem um ciclo.

Problema 8. Em Agrabah existe apenas um tipo de meio de transporte: tapete mágico voador.
Cada linha aérea conecta um par de cidades diferente. Vinte e uma linhas de tapete voadores servem
a capital. Uma única linha serve a cidade de Longı́ssima e cada uma das outras cidades é servida por
exatamente vinte linhas de tapetes voadores. Mostre que é possı́vel viajar de tapete mágico da capital
para Longı́ssima (talvez fazendo conexões entre diferentes linhas de tapetes voadores).

Problema 9. Prove que todo grafo conexo de n vértices contém uma “árvore geradora”, ou seja, uma
árvore com n vértices.

Problema 10. Gugulândia é um paı́s formado por 100 cidades além de sua capital. Alguns pares
de cidades (incluindo a capital) são ligados por estradas de mão única. De cada cidade diferente da
capital, saem exatamente 20 estradas e chegam exatamente 21 estradas. Mostre que é impossı́vel
dirigir de qualquer uma destas para a capital respeitando as regras de trânsito.
1
Vértices de grau 1 costumam ser chamados de folhas.

2
Problema 11. Existem 100 cidades em um paı́s e algumas delas são ligadas por linhas áreas. Sabe-se
que a partir de qualquer cidade pode se viajar a qualquer outra (talvez fazendo várias conexões).
Prove que é possı́vel viajar pelo paı́s e visitar todas as cidades fazendo não mais que 196 voos.

Problema 12. Todos os vértices de um grafo G tem grau menor ou igual a d. Mostre que é possı́vel
colorir os vértices de G utilizando d + 1 cores de forma que nenhum par de vértices adjacentes tenha
a mesma cor.

Problema 13. (EUA 78) Nove matemáticos se encontraram em uma conferência internacional. Dados
quaisquer 3 matemáticos, ao menos dois deles falam uma mesma lı́ngua. Se cada um dos matemáticos
conhece no máximo 3 lı́nguas, prove que existe um trio de matemáticos que pode conversar entre si
em uma mesma lı́ngua.

Problema 14. (Rússia 2004) Um paı́s tem 1001 cidades, e cada par de cidades está conectado por
uma estrada de mão única. De cada cidade saem exatamente 500 estradas e chegam exatamente
500 outras estradas. Este ano, uma república de 668 cidades se declarou independente. Mostre que
é possı́vel ir de qualquer cidade da república a qualquer outra cidade da república através dessas
estradas respeitando sua orientação e sem ter que em algum momento visitar alguma cidade fora da
república.

Problema 15. (EUA 89) Os 20 membros de um clube de tênis disputaram exatamente 14 partidas
entre eles, cada membro havendo disputado ao menos uma partida. Prove que existe um conjunto de
6 partidas que foram disputadas por 12 jogadores distintos.

Problema 16. Seja G um grafo com n vértices. Mostre que existe uma partição do seu conjunto de
vértices V = V1 ∪ V2 tal que todo vértice v em V1 tem pelo menos a mesma quantidade de vizinhos
em V2 que ele tem em V1 e todo vértice v em V2 tem pelo menos a mesma quantidade de vizinhos em
V1 que ele tem em V2 .
Suponha agora que nenhum vértice é isolado. Mostre que existe um subconjunto A de no máximo
n/2 vértices tal que todo vértice de G pertence a A ou está conectado a algum vértice de A.

Problema 17. (IMC 1999) São marcados 2n pontos em um reticulado n × n. Prove que para algum
k > 1, podemos selecionar 2k pontos marcados distintos, digamos a1 , a2 , . . . , a2k , de maneira que a1
e a2 estão na mesma linha, a2 e a3 estão na mesma coluna, ..., a2k−1 e a2k estão na mesma linha e a2k
e a1 estão na mesma coluna.

Problema 18. (Treinamento Cone Sul 2008) Em um torneio de tênis com 14 jogadores, cada um joga
com todos os outros exatamente uma vez e não há empates. Prove que é possı́vel escolher 3 jogadores
para os quais qualquer um dos outros 11 jogadores perdeu para pelo menos um desses 3.

Problema 19. (Cone Sul 2013) Semciclolândia é um paı́s com 500 cidades e 2013 estradas de mão
dupla, cada uma conectando diretamente duas cidades. Duas cidades A e B são chamadas de vizinhas
se existe uma estrada que as conecta e duas cidades A e B são chamadas de quase-vizinhas se existe
uma cidade C tal que A é vizinha de C e C é vizinha de B. Sabemos que em Semciclolândia, não
existem duas cidades conectadas diretamente por mais de uma estrada e não existem quatro cidades
A, B, C e D tais que simultaneamente A é vizinha de B, B é vizinha de C, C é vizinha de D e D é

3
vizinha de A. Demonstrar que existe uma cidade que é quase-vizinha de pelo menos 57 cidades.

Problema 20. (OBM 2007) Em um certo paı́s, há 21 cidades e o governo pretende construir n
estradas (todas de mão dupla), sendo que cada estrada liga exatamente duas das cidades do paı́s.
Qual o menor valor de n para que, independente de como as estradas sejam construı́das, seja possı́vel
viajar entre quaisquer duas cidades (passando, possivelmente, por cidades intermediárias)?

Problema 21. (Bay Area 2005) Existem 1000 cidades no reino de Euleria e alguns pares de cidades
são ligados por uma estrada de terra. E possı́vel viajar de uma cidade para qualquer outra através
das estradas de terra. Prove que o governo de Euleria pode pavimentar algumas estradas de maneira
que de cada cidade saia um número ı́mpar de estradas pavimentadas.

Problema 22. Um grafo G possui n vértices (n > 3). Suponha que quaisquer dois vértices de G
possuam exatamente um vértice vizinho.

a) Se x e y não são adjacentes, prove que eles têm o mesmo grau.

b) Prove que se não existe vértice de grau n − 1, então o grafo é regular, isto é, todos os vértices
possuem o mesmo grau.2

Problema 23. (Teste Cone Sul 2017) Uma empresa de instalação é contratada para colocar cabos
entre 25 computadores. Esses cabos serão usados para passar informações entre computadores de
forma que: cada cabo liga exatamente dois computadores, uma informação pode ir nos dois sentidos
num cabo (isto é, se um cabo liga o computador A ao computador B, então qualquer informação
pode ir do computador A para o computador B ou vice-versa por esse cabo) e, para todo conjunto
de quatro computadores, deve haver um caminho, entre cabos, para a informação sair de qualquer
computador desse grupo e chegar a qualquer outro computador desse mesmo grupo, usando apenas
cabos que conectem computadores de tal grupo. Determine a quantidade mı́nima necessária de cabos
que a empresa deverá colocar.

Problema 24. (Turquia 2011) Em um exame, cada estudante escolhe 1 problema de matemática e
1 problema de fı́sica dentre 20 problemas de matemática e 11 problemas de fı́sica. Nenhum par de
problemas foi escolhido por mais que 1 estudante. Além disso, pelo menos um dos problemas escolhidos
por cada estudante foi resolvido por no máximo 1 um outro estudante. No máximo, quantos estudantes
realizaram este exame?

2
Na verdade, o Teorema da amizade mostra que esta hipótese não acontece em grafo nenhum, porém sua demonstração
está além dos objetivos deste programa.

4
Soluções
1. Como vimos nas aulas e no matéria teórico, o número de arestas é igual a metade da soma dos
graus de cada vértice:
1X
|E| = d(v).
2
v∈G

Note que o número de arestas é inteiro, logo tal soma tem que ser par. Se houvesse um número ı́mpar
de vértices de grau ı́mpar tal soma seria ı́mpar, o que é impossı́vel. Logo tal número é par.

2. Não. Note que se um vértice tem grau 4, então ele se conecta por arestas a todos os outros vértices.
Sendo assim, se os 4 primeiros vértices tem grau 4, cada um deles tem que estar conectado ao quinto,
o que força o grau deste a ser 4 também, e não 2.

3. Se todos os cientistas se conhecerem, então quaisquer 4 podem se sentar na mesa, em quaisquer


posições, e a condição será satisfeita. Caso contrário, basta mostrar que o grafo associado tem um
ciclo de tamanho 4.
Considere dois cientistas que não se conheçam. No grafo, eles são representados por dois vértices,
v1 e v2 , que não estão conectados. Olhe para o conjunto formado pelos outros 48 vértices, V \ {v1 , v2 }.
As arestas que estão conectadas a v1 e v2 são ao menos 50, pois o grau de cada um destes vértices
é ao menos 25. Além disso, a outra extremidade de cada uma delas está no conjunto V \ {v1 , v2 },
pois v1 e v2 não estão conectados. Pelo princı́pio da casa dos pombos, ao menos 50 − 48 = 2 destas
extremidades coincidem, logo existem v3 e v4 tais que v3 está conectado a v1 e v2 , e v4 está conectado
a v1 e v2 . Assim, obtemos o 4-ciclo v1 , v3 , v2 , v4 , concluindo a prova.

4. Não. Suponha que exista tal poliedro. Considere o grafo no qual os vértices representam as faces
do poliedro e cada par de vértices esta conectado por uma aresta (do grafo!) se e somente se as faces
do poliedro a eles correspondentes tem uma aresta (do poliedro!) em comum. Por exemplo, no caso do
tetraedro, este grafo é o K4 , aquele que tem 4 vértices e todo par de vértices ligados por uma aresta.
Voltando ao nosso problema, nosso grafo teria um número ı́mpar de vértices e cada um deles teria
grau ı́mpar, porque cada face tem um número ı́mpar de lados e cada um deles corresponde a uma
aresta (do poliedro) em comum com outra face. Mas sabemos que a soma dos graus dos vértices em
todo grafo é duas vezes o número de arestas. Portanto o resultado deveria ser par, contradição.

5. Este problema fica mais fácil de resolver se pensamos em casos pequenos.


Note que se n = 1, existe apenas um grafo com dois vértices de graus 1:
aquele no qual eles estão conectados por uma aresta. Por outro lado,
quando n = 2, o único grafo que funciona é este ao lado.

Para valores gerais de n usaremos indução, construindo um grafo Gn+1 cujos vértices tem graus
1, 1, 2, 2, . . . , n + 1, n + 1 a partir de um grafo Gn cujos vértices tem graus 1, 1, 2, 2, . . . , n, n.
Analisemos dois casos.
Suponha que n = 2m é par. Adicione a Gn dois vértices a e b. Separe os vértices de grau maior ou
igual a m + 1 em dois conjuntos A e B, de maneira que, para cada i entre m + 1 e n = 2m, cada um
deles contenha exatamente um vértice de grau i. Agora, conecte a por arestas a cada um dos vértices
de A e conecte b por arestas a cada um dos vértices de B. Neste novo grafo, os graus dos vértices são

1, 1, 2, 2, . . . , m, m, m, m, m + 2, m + 2, . . . , n + 1, n + 1,

5
já que os primeiros, até o primeiro par m, m, representam vértices do grafo que não foram conectados
a mais ninguém; o segundo par m, m representa os vértices a e b; e dali em diante temos os vértices
do conjuntos A e B, cujos graus foram aumentados em 1. Para obter o grafo Gn+1 , basta conectar os
vértices a e b, concluindo a construção neste caso.
Por outro lado, se n = 2m + 1 é ı́mpar, a mesma ideia funciona. Basta fazer a primeira parte da
construção acima. Note que neste caso não é necessário conectar os vértices a e b, pois já obtemos na
primeira parte um novo grafo, cujos vértices tem graus

1, 1, 2, 2, . . . , m, m, m + 1, m + 1, m + 2, m + 2, . . . , n + 1, n + 1.

6. Iremos provar por indução no número n de cidades. Para n = 1 o problema é trivial. suponha
então que o resultado seja válido para algum valor de n.
Dado um paı́s com n+1 cidades, isolamos uma das cidades, que iremos chamar de C e consideramos
o paı́s formado pelas outras cidades e as estradas que as conectam entre si. Pela hipótese de indução,
existe uma cidade C 0 a partir da qual podemos chegar em qualquer outra cidade do novo paı́s.
Olhe agora para o paı́s original e, em particular, para a estrada que conecta C a C 0 . Se o sentido
dela vai de C 0 para C, então C 0 é uma cidade a partir da qual podemos viajar para qualquer uma das
outras n cidades do paı́s original. Caso contrário, podemos ir de C até C 0 e daı́, de C 0 até qualquer
outra cidade do paı́s, portanto C é uma cidade com a propriedade buscada. Em ambos os casos
provamos que tal cidade existe, concluindo a demonstração.

7. a) Seja G um grafo tal que todo vértice tem grau pelo menos 2. Comece em v1 e tome v2
conectado a v1 . Como d(v2 ) ≥ 2 existe v3 diferente de v1 tal que v3 está conectado a v2 .
Analogamente, existe v4 conectado a v3 diferente de v2 . Continue este procedimento criando
uma lista de vértices v1 , v2 , ... vk , ... de maneira que cada um está conectado ao seu sucessor
e vi 6= vi+2 para todo i ≥ 1. Como o número de vértices é finito, em algum momento algum
vértice tem que repetir. Considere assim a primeira repetição va . A sequência a partir dele é va ,
va+1 , va+2 , ..., vk = va , formando assim um ciclo.

b) Note que como o grafo é conexo, todo vértice tem grau pelo menos 1 (como existe uma aresta,
ele tem ao menos dois vértices). Se não há vértice de grau 1, então todos tem grau pelo menos
2. Isto implica que há um ciclo, uma contradição.

c) A prova é por indução. Se a árvore tem dois vértices, necessariamente a aresta entre eles também
faz parte do grafo. Já temos o caso base. Tome agora uma árvore com n + 1 vértices. Pela letra
anterior, existe algum vértice v de grau 1 no grafo. Excluindo-o do grafo, assim como a aresta
a ele conectada, obtemos um novo grafo com n vértices que ainda é uma árvore (Note que ele
ainda é conexo, pois qualquer caminho ligando dois pontos neste grafo não precisava passar por
v no grafo original). Pela hipótese de indução, ele tem n − 1 arestas e portanto o original tinha
n arestas, concluindo a prova.

d) Não. Toda árvore de n vértices tem n − 1 arestas. Porém não vale a volta. De fato, o grafo
abaixo tem 4 arestas e 5 vértices, mas não é conexo.

PS: Observe que se um grafo com n vértices e n − 1 arestas não é uma árvore, então ele contém
um ciclo. Veja o próximo item.

6
e) Considere as componentes conexas do grafo G1 , . . . , Gk . Suponha que cada uma delas tem vk
vértices e que v1 + · · · + vk = n. Se nenhuma delas tiver um ciclo, são todas árvores e portanto
tem vk − 1 arestas. Segue que o grafo todo tem no total n − k arestas, o que não é o caso se ele
tem mais que n − 1 arestas.

8. Considere a componente conexa do grafo associado que contém a capital. Se Longı́ssima não estiver
nesta componente conexa, ela é constituı́da pela capital, um vértice de grau 21, e por outras cidades,
cada um dela um vértice de grau 20. Segue que esta componente conexa é um grafo cuja soma dos
graus dos vértices é ı́mpar, um absurdo. Logo Longı́ssima tem que pertencer a esta componente
conexa.

9. A ideia é ir removendo arestas do grafo até restar uma árvore. Esta árvore será uma árvore
geradora. Se o grafo original for uma árvore, então pronto, não precisamos de mais nada. Se não for
uma árvore, então ele tem um ciclo v1 , v2 , , . . . , vk , v1 . Retire então do grafo a aresta vk v1 . O grafo
que resta ainda é conexo. De fato, dados dois vértices v e v 0 se o caminho que os liga no grafo original
não passar pela aresta vk v1 então eles continuam conectados. Por outro lado, se passar, podemos
trocá-la pelo caminho v1 , v2 , , . . . , vk que ainda permanece no grafo e então os dois vértices ainda são
conectados. Note que o número de ciclos do grafo diminui em ao menos 1. Talvez o grafo resultante
ainda tenha algum ciclo. Neste caso repetimos o procedimento até que ele fique sem ciclos. Como o
número de ciclos no grafo original é finito, em algum momento estas repetições param. O grafo que
sobre é conexo e sem ciclos, e portanto é uma árvore.

10. Considere o grafo associado. A condição do enunciado implica que 100 cidades tem que sair da
capital, com mão única apontando para fora desta. Com efeito, em um grafo orientado, a soma dos
graus de chegada dos vértices tem que ser igual a soma do grau de saı́da dos vértices. Segue que

100 × 20 + dsaı́da (capital) = 100 × 21 + dchegada (capital) =⇒ dsaı́da (capital) − dchegada (capital) = 100.

Contudo, os graus de chegada e saı́da da capital são números inteiros entre 0 e 100, logo dsaı́da (capital) =
100 e dchegada (capital) = 0. Como todas as estradas conectadas à capital apontam para fora desta,
não é possı́vel alcançá-la sem viajar na contramão.

11. Considere um reino com n cidades tal que pode se ir de uma a outra por voos. Provaremos que
se n ≥ 3 é possı́vel fazer o trajeto em 2n − 4 voos. Usaremos indução. Para n = 3, o grafo associado
é conexo e portanto tem que conter algum vértice de grau 2. Percorrendo o “V” determinado por ele
de ponta a ponta, percorremos todas as 3 cidades em dois voos, e a base está completa.
Seja agora um reino com n + 1 cidades. Pelo exercı́cio 9, o grafo associado é conexo e tem uma
árvore geradora. Mostremos que todos os vértices podem ser visitados por um caminho que percorre
2n − 2 arestas todas elas dentro desta árvore geradora. Pelo exercı́cio 7, esta árvore tem uma folha,
um vértice v de grau 1. Seja v 0 o vértice adjacente a v. Excluindo v e a aresta vv 0 do grafo obtemos
um grafo com n vértices que também é conexo. Pela hipótese de indução, existe caminho que passa
por todas as n cidades percorrendo 2n − 4 arestas. Para construir o caminho na árvore original, basta
tomar este caminho e, no momento em que ele passasse pela primeira vez por v 0 adicionar o desvio
v 0 , v, v 0 , percorrendo duas vezes a aresta v 0 v.

12. A ideia aqui é usar um algoritmo para construir a coloração. Inicialmente, tome o vértice v de
grau máximo do grafo; se houver mais que um, escolha qualquer um. Se seu grau d(v) for igual a δ,
colora este vértice com a cor C1 e os vértices a ele ligados com as cores C2 , C3 , ..., Cδ+1 . Observe que

7
este subgrafo determinado pelos δ + 1 vértices já pintados e as arestas que os ligam satisfaz a condição
que dois vértices adjacentes não tem a mesma cor.
O procedimento que o algoritmo irá repetir várias funciona da seguinte maneira. Dado um sub-
conjunto V 0 de vértices do grafo G (que não contenha todos os vértices de G) colorido usando as cores
C1 , . . . , Cδ+1 de forma que quaisquer dois vértices v1 e v2 em V 0 adjacentes (por arestas de G) estão
pintados de cores diferentes, escolheremos um vértice v de G fora do subconjunto V 0 e o pintaremos de
uma cor Ci de forma que no conjunto V 0 ∪ {v} quaisquer dois vértices v1 e v2 adjacentes (por arestas
de G) estarão pintados de cores diferentes.
De fato, dado o subconjunto V 0 , tome v qualquer vértice fora de V 0 . Sejam v10 , v20 , . . . , vk0 os vértices
de V 0 conectados a v. Como o grau de v é menor ou igual a δ, k ≤ δ e estes vértices estão pintados em
no máximo δ cores distintas. Sendo assim, existe uma cor Ci , para algum valor de i = 1, 2, . . . , δ + 1
que ainda não foi utilizada e então colorimos v com esta cor. No conjunto V 0 ∪ {v} não existem dois
vértices adjacentes pintados da mesma cor, pois já não existia em V 0 e todo vértice adjacente a v
está pintado de uma cor diferente de Ci . Isto mostra que o procedimento pode ser executado se as
condições forem satisfeitas.
Agora voltemos ao inı́cio de nossa solução. Se o conjunto de δ + 1 vértices do primeiro parágrafo
constituir todos os vértices de G, não precisamos fazer mais nada, pois já colorimos todos os vértices.
Caso contrário, aplique o procedimento acima considerando V 0 como sendo o conjunto dos δ + 1
vértices. Note que ele satisfaz ambas as condições necessárias: não há vizinhos da mesma cor e não
são todos os vértices de G. Após o procedimento, ficamos com um conjunto de δ + 2 vértices sem
que haja dois adjacentes da mesma cor. Caso estes sejam todos os vértices de G, paramos aqui e
teremos concluı́do nossa tarefa. Caso contrário, o novo conjunto satisfaz as condições para aplicarmos
o procedimento e o aplicamos mais uma vez. Repetimos essa rotina até que não seja mais possı́vel
executá-la, ou seja, quando tivermos pintado todos os vértices do grafo. Note que como o número
de vértices é finito e a cada etapa aumentamos em 1 o número de vértices pintados, a repetição em
algum momento deve terminar, e assim teremos conseguido a coloração buscada.

13. Escolha um cientista C qualquer. Suponha que qualquer um dos outros consegue conversar com
C. Então cada um deles tem que falar uma das três lı́nguas faladas por C. Como são oito cientistas,
pelo Princı́pio da casa dos Pombos, uma das três lı́nguas faladas por C é falada por ao menos 3 deles
e portanto existe 4 cientistas (incluindo C) que falam a mesma lı́ngua e podem conversar entre si.
Caso contrário, existe cientista C 0 que não fala nenhuma lı́ngua em comum com C. Considere os
outros 7 cientistas e os enumere C1 , C2 , ..., C7 . Considerando todos os trios do tipo C, C 0 , Ci , com
i = 1, . . . , 7, verificamos que cada Ci tem que conhecer uma lı́ngua em comum com C ou uma lı́ngua
em comum com C 0 . Sem perda de generalidade, pelo PCP. podemos supor que existem 4 valores de
i, C1 , C2 , C3 e C4 que falam alguma lı́ngua em comum com C. Como C só fala 3 lı́nguas, dois deles
tem que falar uma lı́ngua em comum com C, sem perda de generalidade C1 e C2 . Segue que C, C1 e
C2 forma um trio que pode conversar entre si.

14. Note que o grafo associado com 1001 vértices tem cada par de vértices conectado por uma aresta
que vai de um ao outro em algum sentido. Seja G um subgrafo de 668 vértices. Escolha um vértice v
de G e seja A o conjunto dos vértices de G que podem ser alcançados a partir de v por caminhos que
nunca saem de G (incluindo v) e B o seu complementar em G. Observe que dado um vértice a em A
e um vértice b em B, a aresta que liga a e b aponta para a, caso contrário b também seria alcançável
a partir de v.
Seja K o número de vértices em A. Considerando o subgrafo formado apenas pelos vértices em A,
como todas as arestas possı́veis estão desenhadas em algum sentido,
 
X
0
X
0 K
dsaı́da (v ) = dentrada (v ) = ,
0 0
2
v ∈A v ∈A

8
e portanto, pelo PCP, existe algum vértice em a em A cujo grau de entrada é ao menos (K −1)/2. Pela
argumentação do parágrafo anterior e o fato que B tem 668 − K elementos, segue que dentrada (a) ≥
(K − 1)/2 + 668 − K = (1335 − K)/2. Contudo, dentrada (a) ≤ 500 pela condição do enunciado, de
onde segue que K ≥ 335.
Mostrar que é conexo agora é simples. Dados dois vértices v1 e v2 , o conjunto A1 dos vértices
alcançáveis sem sair de G a partir de v1 tem ao menos 335 vértices de G. Analogamente, o conjunto
A2 dos vértices em G a partir dos quais pode se alcançar v2 por caminhos que nunca saem de G tem
ao menos 335 elementos (basta inverter a ordem de todas as flechas do grafo original para ver isto).
Como G tem 668 elementos, A1 e A2 se intersectam em algum v3 , logo é possı́vel ir de v1 a v3 sem
sair de G e depois de v3 a v2 sem sair de G. Segue que existe caminho de v1 a v2 e como estes foram
escolhidos arbitrariamente, a demonstração está completa.

15. Considere o grafo cujos vértices são os membros do clube e cujas arestas são as partidas disputadas.
Se algum duelo foi disputado mais de uma vez, desenhe apenas uma aresta para representá-lo. Este
grafo tem 20 vértices e no máximo 14 arestas. Observe que todo grafo conexo com n vértices tem
ao menos n − 1 arestas (veja o exercı́cio 7). Considere as componentes conexas G1 , G2 , ..., Gk , cada
uma delas com vk vértices. Segue que v1 + v2 + · · · + vk = 20 e o número de arestas é ao menos
v1 − 1 + v2 − 2 + · · · vk − 1 = 20 − k. Como o grafo tem no máximo 14 arestas, segue que k ≥ 6. Como
cada jogador disputou pelo menos uma partida, segue que cada componente conexa tem ao menos 2
vértices. Escolha assim seis componentes conexas e em cada uma delas um par de vértices conectados
por uma aresta. Este doze vértices representam os doze jogadores buscados.

16. Dado um subconjunto U ⊂ V do conjunto de vértices denotamos por dU (v) o número de vizinhos
que v tem em U . Precisamos encontrar uma partição V = A ∪ B tal que dA (v) − dB (v) ≤ 0 para todo
v ∈ A e dB (v) − dA (v) ≤ 0 para todo v ∈ B. Dada uma partição qualquer V = A ∪ B, considere a
função X X
f (A, B) = dA (v) − dB (v) + dB (v) − dA (v).
v∈A v∈B

Não é muito difı́cil acreditar que a partição que minimiza o valor de tal função satisfaz a condição que
queremos. Note em primeiro lugar que tal mı́nimo existe, pois existe apenas um número finito (2|V | )
de partições distintas. Mostremos que ela de fato funciona.
Com efeito, considere (A, B) minimal e suponha que ela não funciona. Dessa forma, podemos supor
sem perda de generalidade que existe a ∈ A tal que dA (a) > dB (a) (devido a simetria). Analisemos
então o que acontece quando movemos a para o conjunto B, ou seja, qual é o valor de f para a partição
A0 ∪ B 0 = V tal que A0 = A \ {a} e B 0 = B ∪ {a}.
Seja v 6= a um elemento de V . Note que
(
dA0 (v) = dA (v) e dB 0 (v) = dB (v), quando v não é conectado a a;
dA0 (v) = dA (v) − 1 e dB 0 (v) = dB (v) + 1, quando v é conectado a a.

Além disso, estes valores não mudam quando consideramos v = a, porém a sai de A para B. Juntando
tudo e denotando por v ∼ a a propriedade de um vértice estar conectado ao vértice a, obtemos,

f (A0 , B 0 ) − f (A, B) =
! !
X X X X
= dA0 (v) − dB 0 (v) + dB 0 (v) − dA0 (v) − dA (v) − dB (v) + dB (v) − dA (v)
v∈A0 v∈B 0 v∈A v∈B
X X
= −2 + 2 + dB (a) − dA (a) − dA (a) + dB (a) = 4(dB (a) − dA (a)) < 0,
v∈A0 ,v∼a v∈B,v∼a

9
contrariando a minimalidade de (A, B). Assim, não pode existir a ∈ A tal que dA (a) > dB (a), e
portanto (A, B) satisfaz a condição exigida.
Para a segunda parte, tome a partição (A, B) acima. Algum dos conjuntos tem que ter cardi-
nalidade menor ou igual a n/2. Suponha sem perda de generalidade que seja A. Como G não tem
vértices isolados, d(v) ≥ 1 para todo v ∈ B. Porém, como dA (v) ≥ dB (v) para todo v ∈ B, segue que
dA (v) ≥ 1 para todo v ∈ B. Concluı́mos que A é dominante.
PS: Poderı́amos tomar também f sendo a função que mede o número de arestas entre A e B e
considerar seu máximo.

17. Considere um grafo bipartido no qual do lado esquerdo temos n vértices, representando as n linhas
e no lado direito temos n vértices, representando as n colunas. Os 2n pontos marcados representam
as 2n arestas deste grafo: se um ponto está na linha i coluna j ele é representado pela aresta que liga
o vértice i do lado esquerdo com o vértice j do lado direito. Como este grafo tem 2n vértices e 2n
arestas, ele tem que possuir um ciclo (veja exercı́cio 7). Como o grafo é bipartido, este ciclo tem que
alternar entre o lado esquerdo e o lado direito, tendo portanto tamanho par. Sejam e1 , e2 , . . . , e2k as
arestas deste ciclo em ordem, supondo sem perda de generalidade que o vértice em comum entre e1 e
e2 está do lado esquerdo. Considere então os pontos a1 , a2 , . . . , a2k marcados no reticulado, sendo ai
o ponto associado a aresta ei para cada i de 1 até 2k. Segue daı́ que a1 e a2 estão na mesma linha.
Por outro lado, a2 e a3 estão na mesma coluna, pois e2 e e3 tem vértice em comum do lado direito.
Este padrão segue alternando até que chegamos em a2k que tem que estar na mesma coluna que a1
pois e1 e e2k tem vértice em comum do lado direito.

18. Cada jogador disputou um total de 13 partidas. Existe portanto algum jogador que venceu ao
menos 7 partidas. Com efeito, num grafo orientado G,
  X
X |V |
dsaı́da (v) = = dentrada (v). (1)
2
v∈G v∈G

e então, se cada jogador venceu no máximo 6 jogos, cada um deles perdeu no mı́nimo 7, e o lado
esquerdo é menor ou igual a 6 × 14 enquanto o lado direito é maior ou igual a 7 × 14, uma contradição.
Separe este jogador J1 e sejam J8 , J9 , . . . , J14 sete jogadores por ele vencidos. Considere os outros
seis, J2 , . . . , J7 . Provaremos que no sub-torneio disputado apenas entre eles, existe algum par de
jogadores Ja e Jb que, juntos, venceram os outros 4. Juntando-os a J1 encontramos os 3 jogadores que
venceram todos os outros.
Note que se alguém tiver vencido 4 vezes neste sub-torneio o problema termina, basta juntar este
cara a J1 e mais algum outro jogador fora do conjunto dos 11 vencidos pelo par formado por este
cara e J1 . Por outro lado, se o número máximo de vitórias de cada jogador é 3, seja k o número
de jogadores que atingiu este número de vitórias. Segue daı́, pela equação (1), que 3k + 2(6 − k) ≥
6
2 =⇒ k ≥ 15 − 12 = 3.
Suponha sem perda de generalidade que os jogadores J2 , J3 e J4 venceram exatamente 3 vezes.
Suponha que algum destes perdeu para os outros dois. Sem perda de generalidade, podemos supor
que J2 perdeu para J3 e J4 , logo venceu J5 , J6 e J7 . Supondo sem perda de generalidade que J3
venceu J4 podemos então escolher J1 , J2 e J3 como nosso trio.
Por outro lado, se cada um deles venceu outro e perdeu para outro, suponha sem perda de genera-
lidade que J2 venceu J3 que venceu J4 que venceu J2 e que J2 venceu J5 e J6 . Se J4 venceu J7 , basta
tomar J2 e J4 . Caso contrário, J4 também venceu J5 e J6 . Se J3 venceu J7 , bastaria escolher J3 e
J4 . Se isto também não acontecer, J7 venceu J3 e J4 , portanto basta escolher J2 e J7 . Isto conclui a
análise de possibilidades e o problema.

19. Iremos contar o número de “chifres” do grafo. Um chifre nada mais é que um par de arestas

10
e1 e e2 com um vértice em comum. Note que cada chifre tem um único vértice em comum e cada
di
vértice i de grau di determina 2 = (d2i − di )/2 chifres. Como o grafo tem 2013 arestas, sabemos que

P500 3
i=1 di = 4026. Por outro lado, pela desigualdade de Cauchy-Schwarz ,

500
! 500 500
!2
X X X
d2i · 1≥ di .
i=1 i=1 i=1

Somando sobre todos os i, descobrimos que o número de chifres do grafo é ao menos 40262 /1000−2013.
Por outro lado, cada chifre tem duas pontas. Além disso, as duas pontas do chifre são quase
vizinhas. Note que dois chifres distintos não podem ter o mesmo par de pontas, pois se isso acontecesse
existiria um ciclo ABCD como no enunciado em Semciclolândia. Sendo assim, basta mostrar então
que existe alguma ponta que pertence a pelo menos 57 chifres.
Em contrapartida, pelo PCP, existe algum vértice que é ponta de pelo menos

40262
   
2 − 2013 /500 = 57
1000
chifres, concluindo a prova.

20. A resposta é 191. Podemos construir 20



2 = 190 estradas sem tornar o paı́s conexo, basta separar
20 cidades e construir todas as estradas possı́veis entre elas. Mostremos que com 191 estradas o paı́s
se torna conexo.
Suponha que o rei construiu várias estradas de modo que o paı́s ainda seja desconexo. Isto significa
que ele está dividido em várias componentes conexas. Podemos agrupá-las em dois conjuntos A e B,
com a ≥ 1 e b ≥ 1 cidades respectivamente, de maneira que não exista estrada entre qualquer par de
cidades a ∈ A e b ∈ B. Dessa maneira, existem no máximo

a2 + b2 − (a + b)
   
a b
+ =
2 2 2

estradas construı́das. Note porém que a + b = 21, logo a2 + b2 = (a + b)2 − 2ab = 212 − 2ab. O
menor valor possı́vel de ab com a + b = 21 acontece quando a e b estão o mais distantes possı́vel, e
no nosso caso é 20, já que a ≥ 1. Concluı́mos que o número de estradas construı́das é no máximo
(212 − 40 − 21)/2 = 190 e portanto com 191 estradas o paı́s tem que ser conexo.

21. Considere uma maneira arbitrária de pavimentar as estradas. Chamemos uma cidade de par
quando o número de estradas pavimentadas saindo delas for par. Considerando o grafo cujos vértices
são as cidades e as estradas pavimentadas as arestas, observamos que o número de cidades com grau
ı́mpar é par (veja exercı́cio 1). Como 1000 é par, o número de cidades pares também tem que ser par.
Nosso objetivo é encontrar uma maneira de pavimentar as estradas de forma que este número se torne
igual a zero.
Para tanto iremos usar um algoritmo. Começamos de uma pavimentação qualquer e enquanto o
número de cidades pares for positivo faremos o seguinte. Escolha uma cidade par c1 qualquer. Note
que ela existe pela hipótese do número de cidades pares ser positivo. Com certeza existe outra cidade
par c2 , pois o número de cidades pares não é ı́mpar. Escolha agora um caminho qualquer envolvendo
estradas de terra e estradas pavimentadas ligando c1 a c2 . Após isso, faça a seguinte operação:
• se uma estrada neste caminho é de terra, pavimente-a;

• se uma estrada neste caminho é pavimentada, torne-a de terra.


3
ou das médias, aplicadas várias vezes.

11
Seja c uma cidade neste caminho que não é c1 nem c2 . Observe que a paridade do número de estradas
pavimentadas a ela conectadas não muda após o procedimento. Para ver isso, basta analisar os casos
possı́veis: Pavimentada - c - Pavimentada; Terra - c - Terra; Pavimentada - c - Terra. Por outro
lado, ambas cidades c1 e c2 deixam de ser pares após o procedimento. Evidentemente, nada muda nas
cidades fora do caminho.
Concluı́mos que o procedimento diminui o número de cidades pares em 2. Como existe um número
finito de cidades, independentemente da pavimentação original, após um número finito de aplicações
do procedimento o número de cidades pares se torna 0, concluindo a solução.

22. Se dois vértices v e u estão ligados por uma aresta iremos denotá-los por v ∼ u.
a) Sejam x1 , . . . , xk os vértices conectados a x e y1 , . . . , yl os vértices conectados a y. Queremos
mostrar que l = k. Note que para cada i = 1, . . . , l, existe exatamente um vértice v tal que
v ∼ x e v ∼ yi . Como v ∼ x, v tem que ser igual a xj para algum valor de j entre 1 e k.
Mais ainda, se para dois valores yj1 e yj2 algum xi fosse tal que xi ∼ yj1 e xi ∼ yj2 , terı́amos
uma contradição, já que y ∼ yj1 e y ∼ yj2 e, como y 6= xi por causa da condição de y e x não
serem conectados, os vértices yj1 e yj2 teriam dois amigos em comum. Segue então que l ≤ k.
Repetindo a mesma argumentação trocando os papéis de x e y obtemos k ≤ l, de onde segue
que k = l.
b) Sejam x e y dois vértices não conectados do grafo (existem pois x tem grau menor que n−1). Pela
letra a) ambos tem grau k ≥ 1. Como eles em apenas um amigo em comum z, todos os outros
vértices do grafo ou não estão conectados a x ou não estão conectados a y e portanto tem grau
igual a k. Resta provar que z também tem grau k. Observe porém que ele não está conectado
a algum dos outros vértices do grafo, pois tem grau menor que n − 1, e assim, novamente pela
letra a), tem grau k.

23. Considere o grafo associado. Queremos que cada conjunto de 4 vértices seja conexo. Note que
nenhum vértice v pode ter grau menor que 22, pois neste caso existiram 3 vértices u, w, z ao qual ele
não estaria conectado e então o conjunto v, u, w, z não seria conexo. Mostremos que existe um grafo
que satisfaz a condição tal que todo vértice tem grau 22.
Considere os 25 vértices como sendo os vértices de um polı́gono regular de 25 lados. Conecte-os
entre si por arestas de forma que nenhum lado do polı́gono seja desenhado, ou seja, ligue cada vértice
a todos os outros excetuando-se os seus vizinhos no polı́gono. Considere agora um conjunto de 4
vértices. Pelo Teorema de Dirac, se cada vértice do subgrafo induzido por estes 4 vértices tem grau ao
menos 2, então ele é conexo. Só terı́amos problemas então quando algum vértice tivesse grau 1 (note
que nenhum pode ter grau 0 pois pelo menos um dos outros 3 vértices não é seu vizinho no polı́gono).
Neste caso, escolhemos um vértice, seus dois vizinhos no polı́gono e mais algum outro vértice. Se
este último não é vizinho no polı́gono de nenhum dos 3 anteriores então ele está conectado aos 3 e o
grafo é forçosamente conexo. Caso contrário, necessariamente escolhemos 4 vértices consecutivos no
grafo. Mas neste caso, o grafo formado é isomorfo ao grafo desenhado na solução do problema 6, que
é conexo.
Segue então que este grafo satisfaz a condição do enunciado. Como cada vértice tem o grau mı́nimo
possı́vel o número mı́nimo de arestas (cabos) é 22 × 25/2 = 275.

24. Considere o grafo bipartido cujos 20 vértices na esquerda são os problemas de matemática e os 11
vértices na direita são os problemas de fı́sica. Cada aresta corresponde a escolha do par de problemas
feita por algum estudante. O enunciado implica que dada uma aresta, um dos dois vértices que a
define tem grau no máximo 2. Queremos encontrar o número máximo de arestas em um grafo com
estas propriedades. Afirmamos que tal número é 54.

12
Com efeito, existe um grafo como acima que tem 54 arestas. Considere os primeiros 18 vértices
do lado esquerdo conectados aos 2 primeiros do lado direito. Os outros 2 vértices do lado esquerdo
serão todos eles conectados aos demais 9 vértices do lado direito. Claramente cada aresta contém um
vértice de grau 2 e o grau total do lado esquerdo, que é igual ao número de arestas, é 54.
Em contrapartida, considere um grafo qualquer com as propriedades acima. Seja A1 o conjunto
dos vértices do lado esquerdo que tem grau no máximo 2 e A2 o conjunto dos vértices do lado esquerdo
que tem grau maior que 2. Seja também B1 o conjunto dos vértices do lado direito que tem grau no
máximo 2 e B2 o conjunto dos vértices do lado direito que tem grau maior que 2.
Pela condição do enunciado, não podem existir arestas entre B1 e B2 . Em particular A1 ou A2 é
não vazio. Mais ainda, qualquer aresta entre A1 e A2 pode ser substituı́da por uma aresta entre A1 e
B2 sem criar nenhum problema, mudando apenas seu fim do lado direito. Logo podemos supor que
não existem arestas entre A1 e A2 . Segue então que o número de arestas no grafo é igual a soma dos
graus dos vértices em A1 ∪ A2 .
Além disso, seja b1 a cardinalidade de B1 e b2 a de B2 . Se b1 ≤ 1, todo vértice em A2 tem grau no
máximo 1 e se b2 ≤ 1 todo vértice em A1 tem grau no máximo 1. Segue que
(
X 2 · |A1 | = 40 − 2b1 , quando b2 ≥ 2
d(v) ≤
v∈A
|A1 | = 20 − b1 , quando b2 ≤ 1.
1

A versão análoga vale para A2 . Analisando casos para os possı́veis valores de b1 e b2 notamos que o
maior valor das estimativas acima ocorre quando b1 = b2 = 2 e neste caso obtemos a quota 40 − 2b1 +
22 − 2b2 = 62 − 8 = 54, concluindo a prova.

Material elaborado por Hugo Fonseca Araújo

13
Problemas Resolvidos

Nı́vel 2

Invariantes I

Material elaborado por Susana Frómeta Fernández


Problemas

Problemas
Problema 1. Os números 1, 2, . . . , 1989 são escritos em um quadro negro. Podemos apagar dois
números e escrever sua diferença no local. Após muitas operações ficamos apenas com um número.
Esse número pode ser o zero?

Problema 2. Os números 1, 2, . . . , 20 são escritos em um quadro negro. Podemos apagar dois deles
a e b e escrever no lugar o número a + b + ab. Após muitas operações ficamos apenas com um número.
Qual deve ser esse número?

Problema 3. (Leningrado 1987) As moedas dos paı́ses Dillia e Dallia são o diller e o daller, respec-
tivamente. Podemos trocar um diller por dez dallers e um daller por dez dillers. Zequinha possui um
diller e deseja obter a mesma quantidade de dillers e dallers usando essas operações. É possı́vel que
isso ocorra?

Problema 4. Seja d(x) a soma dos dı́gitos de x ∈ N. Determine todas as soluções da equação
d(d(n)) + d(n) + n = 1997.

Problema 5. Em um tabuleiro 8 × 8 uma das casas está pintada de preto e as outras casas estão
pintadas de branco. Podemos escolher qualquer linha ou coluna e trocar a cor de todas as suas casas.
Usando essas operações, podemos obter um tabuleiro inteiramente preto?

2
Soluções

1. Note que 1 + 2 + · · · + 1989 = 1989×1990


2 = 1989 × 995 é ı́mpar. Vamos observar o que acontece com
a soma dos número no quadro ao longo do processo. Ao substituir os números m e n pela diferença
m − n, a variação na soma dos números do quadro será de (m − n) − (m + n) = −2n. Ou seja,
sempre teremos uma variação par. Logo a paridade da soma dos número no quadro é um invariante
do processo. Como essa soma é inicialmente ı́mpar, ela então nunca poderá ser igual a zero.

2. Observe primeiramente que a + b + ab = (a + 1)(b + 1) − 1. Suponha que no quadro há n números


a1 , a2 , . . . , an . Com a identidade anterior, podemos verificar que o número (a1 +1)(a2 +1) · · · (an +1)−1
é um invariante da operação autorizada. Com isso, se ao concluir a operação ficamos com o número
a, deve valer a = (a + 1) − 1 = (a1 + 1)(a2 + 1) · · · (an + 1) − 1. Concluı́mos que o número que teremos
ao final do processo é igual a (1 + 1)(2 + 1) · · · (20 + 1) − 1 = 21! − 1.

3. Vamos observar, ao longo das sucessivas trocas de moedas, o que acontece com a diferença entre
a quantidade de dillers e dallers que Zequinha possui. Suponha que, em um um certo momento,
Zequinha possua m dillers e n dallers, de modo que essa diferença é m − n. Ao trocar 1 diller por 10
dallers a diferença passa a ser (m − 1) − (n + 10) = (m − n) − 11. Da mesma forma, se a troca for de 1
daller por 10 dillers a diferença passa a ser (m + 10) − (n − 1) = (m − n) + 11. Assim percebemos que
ao longo do processo, a diferença entre dillers e dallers mantém invariante a congruência módulo 11.
Como inicialmente essa diferença é igual a 1, ela nunca poderá ser 0. Portanto, usando essas trocas,
Zequinha nunca poderá ter a mesma quantidade de dillers e dallers.

4. O resto na divisão por 9 é um invariante na operação que leva n em d(n). O número 1997 deixa
resto 8 na divisão por 9. Portanto devemos ter 3n ≡ 8 mod 9, o que implica 3n ≡ 8 mod 3, mas isso
é impossı́vel. Logo a equação não possui solução.

5. Suponha que uma linha ou coluna do tabuleiro tenha k casas pretas e 8 − k casas brancas, onde
k ∈ {0, . . . , 8}. Ao inverter as cores, passamos a ter 8 − k pretas e k brancas. Logo, a variação no
número de casas pretas causada por essa inversão é de (8 − k) − k = 8 − 2k ∈ {−8, −6, . . . , 0, . . . , 6, 8}
que é par. Disso concluı́mos que a paridade do número de casas pretas é um invariante do processo.
Como temos inicialmente um número ı́mpar de casas pretas, essa quantidade será sempre ı́mpar e
então nunca poderemos ter 64 casas pretas.

Material elaborado por Susana Frómeta Fernández

3
Problemas Resolvidos

Nı́vel 2

Invariantes II

Material elaborado por Hugo Fonseca Araújo


Problemas

Problema 1. (OBM 2012) Zoroastro escreveu os números 1, 2, . . . , 100 em um quadro negro. Ele irá
executar algumas operações que reduzirão a quantidade de números até que reste apenas um único
número no quadro. A primeira operação consiste em escolher dois números quaisquer a e b e trocá-los
por a + b − 1. A segunda operação consiste em novamente escolher dois números quaisquer a e b e
trocá-los por a + b − 2. Em geral, depois de executar k operações, a nova operação será escolher dois
números quaisquer a e b e substituı́-los por a + b − (k + 1). Determine qual o número que restará no
final.

Problema 2. Em uma fábrica de cartões existem três máquinas. A primeira recebe um cartão (a, b)
e retorna um cartão (a + 1, b + 1). A segunda recebe um cartão (2a, 2b) e retorna um cartão (a, b).
A terceira recebe dois cartões (a, b) e (b, c) e retorna o cartão (a, c). Todas as máquinas também
retornam o(s) cartão(ões) dados. É possı́vel fabricar cartão (1, 1988) se temos inicialmente apenas um
cartão (5, 19)?

Problema 3. Uma professora escreveu três números reais no quadro e pediu para Dimas que di-
minuı́sse um deles em 3%, aumentasse o outro em 4% e diminuı́sse o outro em 5%. Dimas anotou os
resultados em seu caderno e notou que os números que havia anotado eram os mesmos números no
quadro, porém em ordem diferente. Prove que Dimas cometeu algum erro.

Problema 4. Todos os inteiros de 1 a 1.000.000 são escritos num quadro. Então, cada um destes
números é substituı́do pela soma de seus algarismos. Estas substituições são realizadas repetidas vezes
até que tenhamos 1.000.000 números com 1 algarismo cada. Dos números que restaram no quadro,
qual aparece mais vezes: o 1 ou o 2?

Problema 5. (Torneio das Cidades 1985) Na ilha de Camelot vivem 13 camaleões roxos, 15 verdes e
17 amarelos. Quando dois camaleões de cores distintas se encontram, mudam simultaneamente para
a terceira cor. Poderia dar-se a situação na qual todos tenham a mesma cor?

Problema 6. (Cone Sul 2014) Em uma lousa, estão escritos os números inteiros de 1 a 2014, inclusive.
A operação válida é escolher dois números a e b, apagá-los e no lugar deles escrever o mı́nimo múltiplo
comum de a e b, e o máximo divisor comum de a e b. Demonstre que, não importando a quantidade √
de operações realizadas, a soma dos números escritos na lousa é sempre maior do que 2014 · 2014 2014!.

Problema 7. Estando algumas pilhas de discos numa mesa, um movimento admissı́vel é escolher
uma pilha, descartar um dos seus discos e dividir o que resta da pilha em duas pilhas não vazias, não
necessariamente iguais.
Inicialmente, há sobre a mesa uma pilha e esta tem 1000 discos. Determine se é possı́vel, depois de
alguma sucessão de movimentos admissı́veis, chegar a uma situação onde cada pilha tenha exatamente
3 discos.

Problema 8. É dado um triângulo no plano. A cada passo, podemos escolher um vértice e trocá-lo
pelo seu simétrico em relação a algum outro vértice do triângulo. Se o triângulo inicial é equilátero,
prove que após realizarmos uma sequência dessas alterações, não é possı́vel encontrar um triângulo de
perı́metro menor.

2
Problema 9. Doze anões vivem em uma floresta e cada um deles tem uma casa que é pintada de
vermelho ou azul. No i-ésimo mês de cada ano, o i-ésimo anão visita todos os seus amigos e se encontra
a maioria deles vivendo em casas de cor diferente da sua própria, ele decide juntar-se a eles e muda
a cor de sua casa. Mostre que, depois de algum tempo, nenhum anão precisará mudar a cor de sua
casa. (As amizades são mútuas e permanentes).

Problema 10. Inicialmente, existem n números 1 escritos em um quadro. Cada passo consiste em
escolher dois números a e b escritos no quadro, apagá-los, e em seu lugar escrever a+b
4 . Mostre que,
1
no fim deste processo, resta um número maior ou igual a n .

Problema 11. (Bulgária 2004) Considere todas as “palavras” formadas por a’s e b’s. Nestas palavras
podemos fazer as seguintes operações:

• Trocar um bloco aba por um bloco b,

• trocar um bloco bba por um bloco a.

Podemos fazer também as operações de maneira reversa. É possı́vel obter a sequência b |aa {z
. . . a} a
2003
partir de aa . . . a} b?
| {z
2003

Problema 12. (China 1986) Em uma lista com 3972 números, cada um dos números de 1 a 1986 foi
escrito exatamente duas vezes. É possı́vel que, para todo k = 1, 2, . . . , 1986, existam exatamente k
números entre as duas aparições de k na lista?

Problema 13. Vários gafanhotos estão no chão em uma fila. A cada minuto, um gafanhoto, que não
seja o primeiro da fila, pula sobre o gafanhoto à sua frente. Ou seja, sendo a o gafanhoto imediatamente
à frente do gafanhoto que pula e b o gafanhoto imediatamente à frente de a, o gafanhoto que pula o
faz de maneira a aterrissar entre a e b. Prove que depois de 2021 minutos os gafanhotos não podem
estar novamente na posição original.

Problema 14. (OBM 2018) Numa lousa estão escritos inicialmente os números 1, 2, . . . , 10. Para
quaisquer dois números a e b na lousa, chamamos S(a, b) a soma dos números na lousa com exceção
de a e b. Uma operação permitida é escolher dois números a e b, apagá-los e escrever o número
ab
a + b + S(a,b) .
Após realizar essa operação algumas vezes restam na lousa apenas dois números, x e y, com x ≥ y.

a) Quantas operações foram realizadas?

b) Determine o maior valor possı́vel para x.

Problema 15. (Torneio das Cidades 2015) Damião irá distribuir doces para a criançada. Ele tem k
exemplares de cada um dos n tipos de doce que possui. Ele os distribui aleatoriamente em k sacolas,
cada uma delas com n doces, e distribui-as para as crianças, uma sacola para cada uma. Para poder
experimentar todos os tipos de doces, as crianças organizaram a seguinte regra para a troca de doces:
se uma delas não possui doce do tipo i mas tem algum do tipo j e outra criança tem algum doce do
tipo i mas não possui doce do tipo j, então elas podem trocar entre elas um doce dos tipo i por um
doce do tipo j. Prove que elas podem organizar uma sequência de trocas de maneira que, ao fim do
processo, cada uma delas possua um doce de cada tipo.

3
Problema 16. (Rússia 1997) Temos uma fileira longa de copos e n pedras no copo central (copo 0).
Os seguintes movimentos são permitidos:

• Movimento tipo A: Se há pelo menos uma pedra no copo i e pelo menos uma no copo i + 1
podemos fazer uma pedra que está no copo i + 1 pular para o copo i − 1 eliminando uma pedra
do copo i.

• Movimento tipo B: Se há pelo menos duas pedras no copo i podemos pular uma pedra para o
copo i + 2 e outra para o copo i − 1.

Demonstre o seguinte fato: fazendo os movimentos tipo A ou B durante um tempo suficientemente


longo sempre chegamos a uma configuração a partir da qual não é possı́vel fazer nenhum desses dois
tipos de movimento. Além disso, essa configuração final não depende da escolha de movimentos
durante o processo.

4
Soluções
1. Observe que, a cada passo i, a soma dos números escritos no quadro diminui de i. No total, serão
realizados 99 passos. A soma dos números no quadro inicial é

1 + 2 + · · · + 100,

e após 99 passos será


(1 + 2 + · · · + 100) − (1 + 2 + · · · + 99) = 100,
e portanto o número que sobra é 100.

2. Note que a terceira máquina só pode ser utilizada quando temos pelo menos duas cartas à disposição,
e a segunda máquina só pode ser usada quando temos uma carta cujos números são pares. Isto significa
que no primeiro passo utilizamos a primeira máquina.
Considere as diferenças entre os números escritos em cada carta. Seja d um inteiro ı́mpar. Dada
uma carta (a, b), se d divide a − b, então divide (a + 1) − (b + 1), ou seja, ainda divide a diferença dos
números escritos na carta que é devolvida pela primeira máquina.
Em contrapartida, dada uma carta (2a, 2b), se d é tal que d | 2a − 2b, então d também divide a − b,
pois é ı́mpar. Além disso, se d divide a − b e d divide b − c, então d divide a − c = (a + b) + (b − c).
Segue, portanto, que se d divide a diferença entre os números escritos em cada uma das cartas que
temos à disposição, após a realização de qualquer uma das possı́veis operações, obtemos uma nova
carta e d ainda divide a diferença dos números nela escritos. Logo, se d divide a diferença entre os
números escritos na carta original, então dividirá as diferenças entre os números escritos em qualquer
carta que as máquinas irão devolver ao longo do processo.
Para o nosso problema, note que d = 7 divide a diferença associada à carta original, mas d não
divide 1988 − 1 = 1987, então não é possı́vel fabricar o cartão pedido.

3. Sejam a, b e c os números escritos pela professora no quadro, nesta ordem. Os números copiados
por Dimas em seu caderno deveriam ser 0, 97 · a, 1, 04 · b e 0, 95 · c. O produto dos números no quadro
é abc, já o produto dos números anotados no caderno deveria ser 0, 97 · 1, 04 · 0, 95abc = 0.95836abc,
um resultado ligeiramente diferente. Consequentemente, os números anotados por Dimas não podem
corresponder aos números no quadro.

4. Inspirados pelo critério de divisibilidade por 9, podemos descobrir o seguinte fato:

Lema. Considere a representação decimal de um número n = ak ak−1 . . . a1 a0 , onde cada ai representa


um dı́gito. Seja m = ak + ak−1 + · · · + a1 + a0 a soma de seus dı́gitos. Então, o resto encontrado
quando dividimos n por 9 é igual ao resto encontrado quando dividimos m por 9.

Demonstração. Note que

n = ak · 10k + ak−1 · 10k−1 + · · · + a1 · 101 + a0 · 100 .

Por outro lado, 10 ≡ 1 (mod 9), e portanto 10l ≡ 1 (mod 9) para todo l natural. Consequentemente,

n ≡ ak · 10k + ak−1 · 10k−1 + · · · + a1 · 101 + a0 · 100 ≡


≡ ak + ak−1 + · · · + a1 + a0 ≡ m (mod 9),

logo os restos de n e m quando dividimos por 9 são iguais. 

5
Voltando ao problema, cada operação que fazemos em números de dois algarismos ou mais diminui
estes números escritos no quadro. Porém, números de um algarismo permanecem iguais.
Mais ainda, o lema acima nos indica que o resto de cada um dos números quando divididos por
9 permanece inalterado a cada passo. Ou seja, ao fim, o número que esta na posição i é o resto da
divisão de i por 9. Mas 1.000.000 é igual a 9 × 111.111 + 1, o que indica que o resto 1 aparece uma
vez a mais que qualquer outro na lista final.

5. Uma boa ideia para se resolver este problema seria realizar alguns passos iniciais e buscar propri-
edades invariantes sobre as quantidades de camaleões de cada cor. Não é difı́cil perceber que sempre
haverá uma quantidade que deixa resto 0 quando dividida por 3, outra que deixa resto 1 e outra que
deixa resto 2. Demonstremos tal fato.
Inicialmente temos 13, 15 e 17, que deixam resto 1, 0 e 2 respectivamente. Suponha que após certa
quantia de passos, as quantidades de camaleões de cada cor de fato deixem restos distintos quando
divididos por 3. Para ficar mais preciso, suponha que elas sejam a ≡ 0 (mod 3), b ≡ 1 (mod 3), e
c ≡ 2 (mod 3). Temos 3 casos a analisar; escrevemos o que acontece com os restos módulo 3 abaixo:

• Se (a, b, c) → (a + 2, b − 1, c − 1), então os restos (0, 1, 2) → (2, 0, 1);

• Se (a, b, c) → (a − 1, b + 2, c − 1), então os restos (0, 1, 2) → (2, 0, 1);

• Se (a, b, c) → (a − 1, b − 1, c + 2), então os restos (0, 1, 2) → (2, 0, 1);

De fato, todos eles resultam na mesma configuração mod 3, porque x + 2 ≡ x − 1 (mod 3) para
todo x natural. O importante é que podemos provar então por indução que os restos de fato tem
a propriedade enunciada acima. Concluı́mos que não podemos obter todos os camaleões tenham a
mesma cor, pois neste caso os restos seriam todos 0.

6. Neste problema precisamos utilizar o seguinte fato, que não será demonstrado (exercı́cio!).

Fato. Sejam a e b dois números inteiros positivos. Então mdc(a, b) · mmc(a, b) = ab.

Este fato nos indica que, a cada passo, o produto dos números na lista não muda. Este produto
sempre será igual a 2014! portanto. Podemos utilizar a desigualdade das médias para concluir que a
soma dos 2014 números no quadro satisfaz
√ √
2014
x1 + x2 + · · · + x2014 ≥ 2014 · 2014 x1 x2 · · · xn = 2014 · 2014!

7. Suponha que seja possı́vel. Seja m a quantidade de passos realizados até chegar em tal situação.
Note que após m passos, a quantidade de pilhas é igual a m + 1. Por outro lado, m discos teriam sido
removidos do total de pilhas. Como cada pilha tem três discos, isto implica que:

3(m + 1) + m = 1000 ⇐⇒ 4m = 1997,

o que é impossı́vel, pois m é inteiro.

8. Lembre que a área de um triângulo pode ser calculada como o produto do comprimento de sua
base pelo comprimento de sua altura. Considerando como base o lado definido pelo vértice refletido e
o vértice que é o centro da reflexão, notamos que no novo triângulo a altura é exatamente a mesma e
a base tem exatamente o mesmo comprimento. Logo, a área do triângulo é sempre preservada.
Para concluir, basta utilizar o seguinte fato:

Fato. Dentre os triângulos de mesma área, o de menor perı́metro é o equilátero.

6
Isto pode ser demonstrado utilizando-se a forma de Heron para a área. Se a, b e c são os compri-
mentos dos lados e p = a+b+c
2 o semiperı́metro:
p
S = p(p − a)(p − b)(p − c).
Como (p − a) + (p − b) + (p − c) = p, segue da desigualdade das médias que
 p 3 p4
p(p − a)(p − b)(p − c) ≤ p · = ,
3 27
com igualdade apenas quando p − a = √ p − b = p − c, o que equivale a a = b = c. Logo, conhecida a
2
área de um triângulo, vale que p ≥ 3 3S, com igualdade apenas quando a = b = c. Como a área dos
triângulos é sempre a mesma, concluı́mos que o perı́metro será mı́nimo exatamente quando o triângulo
é equilátero.

9. Construı́mos um grafo onde cada vértice é a casa de um anão, e cada par de vértices está ligado
por uma aresta se, e somente se, os anões que moram nas casas correspondentes são amigos. Pintamos
também cada vértice do grafo com a cor da casa correspondente. A cada mês, mudamos a cor dos
vértices de acordo com o que faz o anão com sua casa: se o vértice tiver a maioria de seus vizinhos
pintados de uma cor diferente da sua própria, ele muda para essa cor.
Dizemos que uma aresta neste grafo é boa se os dois vértices em suas pontas estão pintados da
mesma cor. Note que a cada mês, se alguma anão muda a cor da sua casa, o número de arestas boas
aumenta. De fato, as arestas que não tem o vértice correspondente à sua casa dentre suas pontas não
mudam de estado, ou continuam sendo boas ou continuam não sendo boas. Analisemos o que ocorre
com as arestas que tem como vértice esta casa do anão, que foi pintada com a outra cor.
Se a cor da casa anteriormente era a cor 1, então haviam a amigos cuja casa era pintada da cor
2 e b amigos cuja casa era pintada da cor 1, com a > b. Ou seja, o número de arestas boas com
esta casa como vértice era b. Logo após o anão pintar sua casa com a cor 2, as casas de seus amigos
permanecem pintadas da mesma cor, e portanto o número de arestas boas com esta casa como vértice
é agora a, ou seja, a quantia aumentou, como querı́amos provar.
Contudo, o número de arestas no grafo é fixo, e portanto o número de arestas boas não pode
aumentar indefinidamente, indicando que o processo terá que parar após algum tempo.

10. A grande dificuldade deste problema é decidir qual semi-invariante tomar. Poderı́amos ser tentados
a considerar a soma dos números no quadro, mas a + b − a+b 4 é uma quantia difı́cil de ser controlada
a longo prazo, de forma a sermos capazes de estimar a soma final dos números no quadro.
Neste tipo de situação temos que buscar expressões que se parecem ou que podem estar relacionadas
com a operação que é feita com os números. Com um pouco de criatividade, lembramos que a + b
também aparece na média harmônica e somos inspirados a considerar como semi-invariante a soma
dos inversos de todos os números escritos no quadro.
Nesta linha, a soma dos inversos
1 1 a+b
+ = ,
a b ab
é uma expressão um pouco parecida com a mudança que fazemos. Entretanto para calcular a variação
deste semi-invariante temos que considerar o inverso de a+b 4 . Sendo assim, estimamos:

1 1 1 a+b 4 (a + b)2 − 4ab


+ − a+b = − = .
a b 4
ab a+b (a + b)ab
Note agora que, como os números originais são todos positivos, não é difı́cil provar por indução que
todos os números no quadro serão sempre positivos. Por outro lado, é sabido que
(a + b)2 − 4ab = a2 − 2ab + b2 = (a − b)2 ≥ 0,

7
indicando que a diferença acima é não negativa. Logo, a soma dos inversos dos números no quadro
nunca cresce. No momento inicial ela é igual a n, e portanto no momento final será menor ou igual a
n. Se o número final for x, temos então x1 ≤ n =⇒ x ≥ n1 , o que conclui a prova.

11. Dada qualquer expressão do tipo, enumere as posições letras, sendo que o 1 corresponde à posição
da primeira letra. O invariante agora é a paridade do número de letras a em posições ı́mpares.
Consideremos a primeira operação. Note que todas as letras a à esquerda do bloco aba não mudam
de posição após ele se tornar b. Já as letras a à direita do bloco tem suas posições decrescidas de 2.
As duas letras a removidas do bloco estavam em posição de mesma paridade, portanto o invariante de
fato não é alterado. Por uma análise análoga, a inversa desta operação também não muda o invariante.
Analisemos agora a segunda operação. As letras a à esquerda do bloco bba não mudam de posição
após ele se tornam a. Já as letras a à direita do bloco bb tem usa posições decrescidas de 2. Por uma
análise análoga, a inversa desta operação também não muda o invariante.
Para concluir, observe que na configuração inicial temos 1001 letras a em posições ı́mpares mas a
configuração pedida tem 1002 letras em posições ı́mpares.

12. Não é bem um exercı́cio sobre uma quantidade invariante após certa modificação, mas diz respeito
a certas quantidades que devem ser respeitadas por qualquer distribuição dos números em uma lista
de 3972 números. Enumere as posições na lista de 1 a 3972. Nosso “invariante” é a quantidade
de números colocados em posições pares subtraı́do da quantidade de números colocados em posições
ı́mpares. Como 3972 é par, esta diferença tem que ser sempre 0.
Suponha que seja possı́vel construir a lista com as propriedades pedidas pelo enunciado. Se k é
par, como temos k números entre as duas aparições de k na lista, estas acontecem em posições de
paridades distintas, e portanto a contribuição total para o invariante é 0. Por outro lado, se k é ı́mpar,
novamente porque temos k números entre as duas aparições de k, estas acontecem em posições de
paridades iguais, e portanto cada uma delas contribui com +2 ou −2. Note porém que 1986 = 2 × 993,
donde segue que a soma de 993 parcelas +2 ou −2 nunca se anula, e tal configuração é impossı́vel.
Isto pode ser observado analisando a soma mod 4; como −2 ≡ 2 (mod 4), a soma das parcelas seria
993 × 2 ≡ 1 × 2 ≡ 2 (mod 4) e logo não é 0.

13. Este tipo de questão pode ser pensada da seguinte maneira. Considere os gafanhotos numerados
de 1 a n. Partindo da lista inicial 1, 2, . . . , n, podemos, a cada passo, mudar dois números de posição.
O problema se resume a mostrar que não é possı́vel retornar à lista inicial após um número ı́mpar de
passos.
O invariante clássico a ser utilizado nesse tipo de problema é o número de inversões na lista de
gafanhotos. Chamamos de inversão cada par de inteiros (a, b) na lista tais que a > b e a aparece antes
de b. Note que inicialmente há 0 inversões. A cada passo, ao trocarmos dois números x e y vizinhos
na lista de posição, analisemos o que ocorre.
Todas as inversões (a, b) que não envolvem ao mesmo tempo x e y são preservadas, pois as posições
relativas de a e b na lista não mudam. Já as posições relativas de x e y mudaram. Se antes elas
representavam uma inversão, agora não representam mais, e vice-versa. Isto significa que o número
de inversões a cada passo muda em +1 ou −1. Dessa maneira, como 1 ≡ −1 (mod 2), após uma
quantidade ı́mpar de passos, o número de inversões será ı́mpar e portanto a configuração final não
pode ser igual à configuração inicial. Observe também que após uma quantidade par de passos, o
número de inversões será sempre par.

14. A cada operação, a quantidade de números na lousa cai de um. Logo são necessárias 9 operações.
Para responder a letra b), precisamos considerar um invariante mais complicado. Uma maneira de
encontrá-lo é brincar com a operação de troca. Seja S a soma dos números no quadro em certo

8
instante. Sendo a e b dois números na lousa, temos

S − a = S(a, b) + b e S − b = S(a, b) + a.

Além disso,

b · (S(a, b) + a) = b · S(a, b) + ab e a · (S(a, b) + b) = a · S(a, b) + ab,

e a soma destas expressões parece-se muito com a troca realizada. Somos inspirados portanto a
considerar o seguinte invariante.
Sejam x1 , x2 , . . . , xn os números no quadro e S a soma deles. Tome

I = x1 (S − x1 ) + x2 (S − x2 ) + · · · + xn (S − xn ).

Provemos que esta quantidade permanece inalterada após uma operação.


Suponha sem perda de generalidade que a operação será feita utilizando-se x1 e x2 , os demais
números permanecendo inalterados. Se a soma anterior era S = x1 + x2 + · · · + xn , a nova soma será
x1 x2 x1 x2
S 0 = x1 + x2 + + x3 + · · · + xn = S + .
S(x1 , x2 ) S(x1 , x2 )

O novo valor de I é
    
0 x1 x2 0 x1 x2
I = x1 + x2 + · S − x1 + x2 + + x3 (S 0 − x3 ) + · · · + xn (S 0 − xn )
S(x1 , x2 ) S(x1 , x2 )
    
x1 x2 x1 x2 x1 x2
= x1 + x2 + · S+ − x1 + x2 + +
S(x1 , x2 ) S(x1 , x2 ) S(x1 , x2 )
   
x1 x2 x1 x2
+ x3 S + − x3 + · · · + xn S + − xn
S(x1 , x2 ) S(x1 , x2 )
 
x1 x2 x1 x2
= x1 + x2 + · S(x1 , x2 ) + x3 (S − x3 ) + · · · + xn (S − xn ) + (x3 + · · · + xn ) ·
S(x1 , x2 ) S(x1 , x2 )
x1 x2
= (x1 + x2 )S(x1 , x2 ) + x1 x2 + x3 (S − x3 ) + · · · + xn (S − xn ) + S(x1 , x2 ) ·
S(x1 , x2 )
= (x1 + x2 )S(x1 , x2 ) + 2x1 x2 + x3 (S − x3 ) + · · · + xn (S − xn )
= x1 (S − x1 ) + x2 (S − x2 ) + x3 (S − x3 ) + · · · + xn (S − xn ) = I,

onde a passagem da penúltima para a última linha pode ser verificada através dos cálculos realizados
para a e b no inı́cio de nossa solução. Concluı́mos que I é mesmo invariante!
Inicialmente temos S = 55, e portanto

I = x1 (S − x1 ) + · · · + x10 (S − x10 ) = (x1 + · · · x10 ) · S − (x21 + · · · x210 )


= S 2 − (x21 + · · · x210 ) = 552 − (12 + · · · + 102 ) = 2640.

Quando temos apenas dois números x e y, o invariante I é igual a xy + yx = 2xy, donde segue que
2xy = 2640. Contudo, podemos fazer todas as 9 operações sem nunca utilizar o número 1, e neste
caso terı́amos y = 1. Observando que os números escritos na lousa são sempre todos maiores ou iguais
a 1, chegamos à conclusão que o valor máximo para x é alcançado exatamente nesta situação e ele
será 2640/2 = 1320.

15. A solução deste problema é esperta. Note que após qualquer quantidade de trocas, cada criança
tem n doces. Mostraremos que, enquanto as crianças não tiverem um doce de cada tipo, sempre será
possı́vel realizar uma troca com uma propriedade adicional simples, e, além disso, a escolha em si

9
dessas trocas especiais não importa: realizando-as de qualquer maneira, inevitavelmente, depois de
certo tempo, cada uma das crianças terá um doce de cada tipo.
Vamos à primeira afirmação. Se o problema ainda não foi resolvido, seja i o tipo de doce possuı́do
pelo menor número de crianças. Em caso de empate, escolha qualquer um destes tipos. Pelo princı́pio
da casa dos pombos, existe então uma criança C com este tipo de doce em múltiplos exemplares, pelo
menos 2, e portanto C não possui qualquer doce de algum outro tipo j. Considere agora as crianças
C1 , C2 , . . . , Cm que não possuem nenhum exemplar do doce tipo i. Alguma delas tem que possuir um
doce do tipo j, caso contrário este tipo de doce não seria possuı́do por C nem por C1 , C2 , . . . , Cm , e o
número de crianças que tem algum exemplar de doce deste tipo j é menor que o número de crianças
que tem algum exemplar do doce tipo i, o que contraria a minimalidade do tipo i. Consequentemente,
alguma das crianças C1 , C2 , . . . , Cm pode trocar seu doce do tipo j por um doce do tipo i com C, e a
afirmação está correta.
Atenção! Cada uma das trocas que iremos realizar envolve uma criança α que possui pelo menos
2 exemplares de doce do tipo i e uma criança β que possui algum exemplar de doce do tipo j.
Para concluir que em algum momento tem que ser impossı́vel realizar mais trocas, e portanto todas
as crianças tem um doce de cada tipo, consideraremos um invariante. Seja q(γ, l) a quantidade de
doces que a criança γ tem do tipo l. Como queremos que estas quantidades se tornem todas iguais a
1, faz sentido considerar o semi-invariante
X
I= q(γ, l)2 .
1≤γ≤n
1≤l≤n

Analisemos o que ocorre com este número após uma troca de doces. Se a criança α troca um doce do
tipo i por um doce do tipo j com a criança β, então quatro valores no somatório acima mudam:

q 0 (α, i) = q(α, i) − 1; q 0 (α, j) = q(α, j) + 1; q 0 (β, i) = q(β, i) + 1; q 0 (β, j) = q(β, j) − 1.

Note porém que q(α, j) = 0 = q(β, i), porque esta condição é necessária para realizarmos a troca.
Assim, o novo valor de I satisfaz:

I 0 = I + [q 0 (α, i)]2 − [q(α, i)]2 + [q 0 (α, j)]2 − [q(α, j)]2 + [q 0 (β, i)]2 − [q(β, i)]2 + [q 0 (β, j)]2 − [q(β, j)]2
= (−2 · q(α, i) + 1) + 1 + 1 + (−2 · q(β, j) + 1) = 4 − 2 · [q(α, i) + q(β, j)].

Mas de acordo com a propriedade adicional de nossas trocas, a soma [q(α, i) + q(β, j)] é pelo menos 3,
pois uma das crianças tinha ao menos 2 exemplares do doce do tipo a ser trocado. Isto significa que

I 0 ≤ I − 2,

e portanto a quantia I é estritamente decrescente. Como ele é sempre positiva, em algum momento
terá que parar de cair, significando que não é mais possı́vel realizar trocas e que cada uma das crianças
tem um doce de cada tipo.

1+ 5
16. Seja φ = 2 a raiz positiva de x2 − x − 1 = 0. Segue que

φ2 = φ + 1. (1)

Se uma pedra está no copo i, diremos que sua energia é φ−i . Seja E a soma total das energias de
todas as pedras nos copos. Ao realizar um movimento A trocamos duas pedras cuja soma das energias
era φ−i−1 + φ−i por uma cuja energia é φ−i+1 . Multiplicando a equação (1) por φ−i−1 notamos que a
energia total foi preservada. De maneira análoga, notamos que

φ−i−2 + φ−i+1 = φ−i−2 + φ−i−1 + φ−i = φ−i + φ−i ,

10
e, por isso, E também é preservada após realizarmos um movimento do tipo B. Como temos n
pedras no copo 0 inicialmente, teremos E = n em todas as configurações que podemos obter no nosso
problema. Por consequência, existe existe um valor m1 (= −blogφ nc − 1) tal que todo copo à esquerda
de m1 jamais será ocupado por uma pedra, pois, se isto ocorresse, o valor de E seria maior que n.
Seja então k1 o menor inteiro tal que o copo k1 chegou a abrigar alguma pedra em alguma realização
deste jogo. Temos k1 ≥ m1 . Note então que a pedra que ocupou o copo k1 em algum momento jamais
será movida novamente, porque qualquer movimento que fizéssemos levaria-la ou outra pedra para um
copo mais à esquerda de k1 , contradizendo a sua minimalidade.
Daı́ em diante podemos então ignorar esta pedra e considerá-la fora do jogo. Repetimos então
o mesmo argumento partindo da configuração que paramos e agora trabalhando com n − 1 pedras,
cuja somas das energias será n − φ−k1 . Novamente, é impossı́vel que alguma pedra ocupe uma casa
à esquerda de m2 = −blogφ n − φ−k1 c − 1 ≥ m1 e portanto podemos encontrar uma outra pedra que
atingirá uma posição máxima k2 à esquerda e não mais se moverá. Repetimos este procedimento
várias vezes até que todas as pedras não possam mais se mover.
Em uma configuração final, não podem haver duas pedras em um mesmo copo nem duas pedras em
copos vizinhos, do contrário, poderı́amos realizar ao menso mais um movimento. Logo, para provar a
unicidade da configuração final, basta mostrar que todo número inteiro positivo n pode ser escrito de
forma única como
Xk
n= φci ,
i=0

onde cada ci é um inteiro, e ci + 1 < ci+1 para todo inteiro i = 0, . . . , k − 1. Suponha que existam duas
escolhas possı́veis para os valores de ci como acima, de maneira que ambas as somas resultem em n,
k
X l
X
ci
n= φ = φc̃j .
i=0 j=0

Podemos supor sem perda de generalidade que c̃l é maior que ck . De fato, caso sejam iguais, podemos
cortar os termos φck e φc̃l da igualdade acima, reduzindo o número de termos, e analisar repetidamente
as novas igualdades que obtemos até concluir que todos são iguais ou encontrar igualdade na qual o
máximo da sequência dos ci ’s seja diferente do máximo da sequência dos c̃j ’s. Podemos então observar
que
Xk k
X
ck −2i
φ ≥ φ ci ,
i=0 i=0

mas a soma da P.G. em questão é

φck +2 − φck −2k φck +2 − φck −2k


= = φck +1 − φck −2k−1 < φc̃l ,
φ2 − 1 φ
o que contradiz a igualdade acima. 

Material elaborado por Hugo Fonseca Araújo

11
POLOS OLÍMPICOS DE
TREINAMENTO INTENSIVO

teoria dos números


nível 2
Polos Olı́mpicos de Treinamento Intensivo
Curso de Teoria dos Números − Nı́vel 2

Professores: Cleber Assis, Samuel Barbosa e Tiago Miranda


Aula 0
POTI 2015
Curso Básico
. . . . . . . . . . . . . . . . . . . . . . . . . . . . . . . . . . . Teoria dos Números . . . . . . . . . . . . . . . . . . . . . . . . . . . . . . . . . . .

Este material compila os arquivos do projeto Portal da Matemática, disponı́vel em

http://matematica.obmep.org.br/

e serve como introdução aos tópicos iniciais de um curso de treinamento olı́mpico. Em geral, os assuntos
são independentes e podem ser estudados em qualquer ordem. Neles, o leitor encontrará muitos exercı́cios
escolares mesclados com problemas elementares de olimpı́adas, todos com respostas e soluções. Além disso,
no endereço do Portal da Matemática, existem vı́deos que podem ser acessados gratuitamente cobrindo todo
o conteúdo abaixo. Bons estudos!
...

Sumário
1 Números Naturais e Problemas de Contagem . . . . . . . . . . . . . . . . . . . . . . . . . . . . . 1
2 Divisibilidade e Teorema da Divisão Euclideana . . . . . . . . . . . . . . . . . . . . . . . . . . . . 4
3 Potenciação . . . . . . . . . . . . . . . . . . . . . . . . . . . . . . . . . . . . . . . . . . . . . . . . . 6
4 Números Racionais . . . . . . . . . . . . . . . . . . . . . . . . . . . . . . . . . . . . . . . . . . . . . 8
5 Números Irracionais . . . . . . . . . . . . . . . . . . . . . . . . . . . . . . . . . . . . . . . . . . . . 10
6 Radiciação e Expressões Algébricas . . . . . . . . . . . . . . . . . . . . . . . . . . . . . . . . . . . 12
7 Introdução aos Polinômios . . . . . . . . . . . . . . . . . . . . . . . . . . . . . . . . . . . . . . . . 14
8 Produtos Notáveis . . . . . . . . . . . . . . . . . . . . . . . . . . . . . . . . . . . . . . . . . . . . . 17
9 Fatoração de Expressões Algébricas . . . . . . . . . . . . . . . . . . . . . . . . . . . . . . . . . . . 20
10 Sentenças Matemáticas e Notação Algébrica . . . . . . . . . . . . . . . . . . . . . . . . . . . . . 23
Números Naturais e Problemas de Contagem − Soluções . . . . . . . . . . . . . . . . . . . . . . 25
Divisibilidade e Teorema da Divisão Euclideana − Soluções . . . . . . . . . . . . . . . . . . . . . 30
Potenciação − Soluções . . . . . . . . . . . . . . . . . . . . . . . . . . . . . . . . . . . . . . . . . . 38
Números Racionais − Soluções . . . . . . . . . . . . . . . . . . . . . . . . . . . . . . . . . . . . . . 40
Números Irracionais − Soluções . . . . . . . . . . . . . . . . . . . . . . . . . . . . . . . . . . . . . 43
Radiciação e Expressões Algébricas − Soluções . . . . . . . . . . . . . . . . . . . . . . . . . . . . 47
Introdução aos Polinômios − Soluções . . . . . . . . . . . . . . . . . . . . . . . . . . . . . . . . . 48
Produtos Notáveis − Soluções . . . . . . . . . . . . . . . . . . . . . . . . . . . . . . . . . . . . . . 53
Fatoração de Expressões. Algébricas − Soluções . . . . . . . . . . . . . . . . . . . . . . . . . . . . 57
Sentenças Matemáticas e Notação Algébrica − Soluções . . . . . . . . . . . . . . . . . . . . . . . 64

Versão: 45 (Data: 27 de abril de 2015.)


POTI 2015 − Teoria dos Números − Nı́vel 2 − Aula 0 − Cleber Assis, Samuel Barbosa e Tiago Miranda

1 Números Naturais e Problemas de Contagem


Problema 1. Qual a quantidade de elementos do conjunto que possui todos os números naturais de 8 até
908?
Problema 2. Quantos elementos há no conjunto {7, 14, 21, . . . , 679, 686}?
Problema 3. Quantos elementos há no conjunto {14, 19, 24, . . . , 1004, 1009}?
Problema 4. Entre n pessoas existem duas com o mesmo signo. Qual o menor valor de n que garante esse
fato?
Problema 5. Quantos números escrevemos ao numerarmos as páginas de um livro de 10 até 20? E quantos
algarismos são usados para isso?
Problema 6. Numa floresta há 1000 jaqueiras. É conhecido que uma jaqueira não tem mais do que 600 frutos.
Prove que existem 2 jaqueiras que têm a mesma quantidade de frutos.
Problema 7. Uma pessoa entrou num quarto escuro, sem enxergar absolutamente nada, e abriu uma gaveta
na qual havia exatamente 20 meias pretas, 15 meias brancas e 10 meias marrons. Todas estavam misturados e
eram indistinguı́veis ao tato. Qual a quantidade mı́nima de meias que essa pessoa deve retirar para que tenha
certeza de ter retirado:

a) um par de meias de mesma cor?

b) um par de meias brancas?

Problema 8. Prove que:

a) a soma de dois números pares é igual a um número par.

b) a soma de dois números ı́mpares resulta em um número par.

c) a soma de um número par com um número ı́mpar resulta em um número ı́mpar.

d) o produto de dois números ı́mpares é igual a um número ı́mpar.

e) o produto de dois números pares é um número par.

f) o produto de um número par com um número ı́mpar resulta em um número par.

Problema 9. Ao escrevermos todos os números naturais de 40 até 1200, quantos algarismos utilizamos?
Problema 10. Qual é a soma de todos os números de três algarismos?
Problema 11. Qual o número mı́nimo necessário de pessoas num grupo para que tenhamos certeza de que:

a) três delas façam aniversário no mesmo mês?

b) quatro tenham nascido no mesmo dia da semana?

Problema 12. Numa gaveta há 10 blusas amarelas, 12 blusas beges e 8 blusas cinzas. Suponha que sejam
retiradas “n” blusas, no escuro, dessa gaveta (não há como perceber as cores). Qual o valor mı́nimo de “n”
para que tenhamos certeza de que saiam 3 de cores distintas?
Problema 13. Se uma urna contém 7 bolas vermelhas, 9 pretas, 10 azuis e 8 verdes. Qual é o número mı́nimo
de bolas que devemos retirar para que possamos ter certeza da retirada de pelo menos 4 da mesma cor?
Problema 14. Considere o número

S = 1 + 2 + 3 + +4 + · · · + 2011 + 2012 + 2013 + 2014.

Esse número é par ou ı́mpar?

1
POTI 2015 − Teoria dos Números − Nı́vel 2 − Aula 0 − Cleber Assis, Samuel Barbosa e Tiago Miranda

Problema 15. Discos dentados geram um tipo de sistema associado que funciona pela propulsão em um
dos discos e esse proporciona o funcionamento dos demais. A figura 1 ilustra um desses sistemas e o disco
“número 1” gira no sentido horário. Analise as proposições e responda o que se pede.

Figura 1

i) O disco 2 gira no sentido anti-horário.

ii) O disco 4 gira no sentido horário.

iii) O disco 7 gira no mesmo sentido do disco 5.

iv) O disco 10 gira no mesmo sentido do disco 3.

v) Seria possı́vel colocar um disco 11 em contato simultâneio com os discos 1 e 10.

Quantas das proposições acima são verdadeiras?


Problema 16. Escrevendo os números naturais de 1 até 10 em fila e mantendo um espaço vazio entre eles
( ~ ) obtemos
1 ~ 2 ~ 3 ~ 4 ~ 5 ~ 6 ~ 7 ~ 8 ~ 9 ~ 10.
É possı́vel ocupar os ~ com sinais de “+” ou “−” de modo que o resultado da expressão que aparecerá após
a colocação dos sinais seja zero?
Problema 17. Em uma urna há 32 bolas brancas, 16 bolas verdes, 7 bolas vermelhas, 3 bolas pretas e 11
bolas cinzas. Qual é o número mı́nimo de bolas que devemos sacar dessa urna para termos certeza de que
obteremos pelo menos 13 bolas da mesma cor?
Problema 18. Se n é um número inteiro qualquer, qual das expressões abaixo resulta num número ı́mpar?

a) n2 − n + 2 c) n2 + n + 5 e) n3 + 5
b) n2 + n + 2 d) n2 + 5

Problema 19. Qual a paridade do algarismo das unidades do número

20102010 + 20112011 + 20122012 + · · · + 20152015 + 20162016 ?

Problema 20. Qual o menor número de pessoas num grupo para garantir que pelo menos 4 nasceram no
mesmo mês?
Problema 21. Uma máquina contém pequenas bolas de borracha de 10 cores distintas, sendo 10 bolas de
cada cor. Ao inserir uma moeda, uma bola é expelida ao acaso. Para garantir a retirada de 4 bolas da mesma
cor, qual o menor número de moedas inseridas na máquina?
Problema 22. Depois de d lançamentos de um dado de 6 faces temos certeza que uma das faces saiu mais
de 5 vezes. Qual o valor de d?

2
POTI 2015 − Teoria dos Números − Nı́vel 2 − Aula 0 − Cleber Assis, Samuel Barbosa e Tiago Miranda

Problema 23. Observe a sequência de algarismos

12345678910121314151617 . . . .

Qual será o 1002◦ algarismo usado nela?


Problema 24. Qual a soma dos múltiplos de 3 entre 1 e 301?
xy
Problema 25. Quais são os pares de números1 inteiros ( x, y) tais que = 144?
x+y
p
Problema 26. Quantos números inteiros e positivos satisfazem a dupla inequação 2000 < n · ( n − 1) <
2005?

a) 1. b) 2. c) 3. d) 4. e) 5.

Problema 27. Observe que


( a + b)3 = a3 + 3a2 b + 3ab2 + b3 ,
daı́ poderı́amos calcular

23 = (1 + 1)3 = 13 + 3 · 12 · 1 + 3 · 1 · 12 + 13
33 = (2 + 1)3 = 23 + 3 · 22 · 1 + 3 · 2 · 12 + 13
43 = (3 + 1)3 = 33 + 3 · 32 · 1 + 3 · 3 · 12 + 13
53 = (4 + 1)3 = 43 + 3 · 42 · 1 + 3 · 4 · 12 + 13

A partir da análise dos exemplos acima, desenvolva uma fórmula para o cálculo de

12 + 22 + 32 + · · · + n2 .

Problema 28. A figura 2 é o composta por 100 quadrados colocados lado a lado, na qual tem-se indicadas as
medidas dos lados de cada quadrado.

···

1 2 3 100
Figura 2

Qual o valor da área total dessa figura?


Problema 29. Uma rede de computadores é formada por seis computadores. Cada computador é conectado
diretamente a pelo menos um dos outros computadores. Mostre que há pelo menos dois computadores na
rede que estão diretamente conectados ao mesmo número de outros computadores.

1 “Pares de números” que dizer que a resposta sempre será por dois números em ordem, um represnetando o de x e outro o valor
de y. Isso não tem necessarimanete relação com números pares.

3
POTI 2015 − Teoria dos Números − Nı́vel 2 − Aula 0 − Cleber Assis, Samuel Barbosa e Tiago Miranda

2 Divisibilidade e Teorema da Divisão Euclideana


Problema 30. Mostre que 21 divide 58 − 28 .
Problema 31. Mostre que 10 divide 116 − 1.
Problema 32. Sejam a e b ∈ {0, 1, 2, ..., 9}. Determine os valores possı́veis de ( a − b)2 para que 23a1992b seja
divisı́vel por 45.
Problema 33. Mostre que 248 − 1 é múltiplo de 65 e de 63.
Problema 34. Mostre que 10x + y é divisı́vel por 7 se e só se x − 2y também for.
Problema 35. Calcule os números naturais que quando divididos por 8 deixam resto igual ao dobro do
quociente.
Problema 36. Calcule o número natural que quando dividido por 7 resulta no quociente 4 e o resto é o
maior possı́vel.
Problema 37. Mostre que 10x + y é divisı́vel por 13 se e só se x + 4y também for.
Problema 38. Determine o menor inteiro positivo que dividido por 9 gera resto 3 e dividido por 11 gera
resto 4.
Problema 39. Um número inteiro positivo k deixa resto 4 quando dividido por 7.
a) Determine o resto da divisão de k2 + k + 1 por 7.

b) Qual é o menor múltiplo positivo de k que devemos somar a k2 para obter um múltiplo de 7.
Problema 40. Um número inteiro n deixa restos respectivamente iguais a 4 e 6 quando dividido por 7 e 8.
Determine o resto da divisão de n por 56.
Problema 41. Quais os inteiros n = 2a · 3b , como a e b inteiros não-negativos, possuem 15 divisores positivos?
Problema 42. Qual é o valor de n para o qual o número 12345n789 é divisı́vel por 91?
Problema 43. Quais os possı́veis restos de um número quadrado perfeito na divisão por 4?
Problema 44. Dados três números naturais a, b e c tais que a + b + c é divisı́vel por 6, prove que a3 + b3 + c3
também é divisı́vel por 6.
Problema 45. Seja x o maior número natural com três algarismos que ao ser dividido por 2, por 3, por 5 e
por 7 deixa resto 1. Qual a soma dos algarismos de x?
Problema 46. Quantos números naturais menores que 400 são divisı́veis por 17 ou 23 ?
Problema 47. Qual o maior inteiro que divide todos os possı́veis números da forma m2 − n2 onde m e n são
números ı́mpares quaisquer e n < m.?
Problema 48. Quantos números podem ser formados com 4 algarismos, de modo que esses números sejam
divisı́veis por 2, 3, 5 e 9 e que o algarismo dos milhares seja 8?
Problema 49. A multiplicação decrescente de inteiros não-negativos em sequência até o 1 é denominada de
fatorial e é simbolizada por n!. Exemplos:
i) 8! = 8 × 7 × 6 × 5 × 4 × 3 × 2 × 1; e

ii) 5! = 5 × 4 × 3 × 2 × 1; e

iii) 2! = 2 × 1.
Definimos 1! = 1 e 0! = 1. Sendo assim, calcule o maior inteiro positivo x tal que 23x divide 2000!.
Problema 50. Dados três números naturais x, y e z tais que x2 + y2 = z2 , mostre que x e y não são ambos
ı́mpares.
Problema 51. Qual o resto da divisão por 9 do número

1111111111 − 22222?

4
POTI 2015 − Teoria dos Números − Nı́vel 2 − Aula 0 − Cleber Assis, Samuel Barbosa e Tiago Miranda

Problema 52. Em um número natural N de 9 algarismos, tem-se que:

- os algarismos das unidades simples, unidades de milhar e unidades de milhão iguais a X;

- os algarismos das dezenas simples, dezenas de milhar e dezenas de milhão iguais a Y; e

- os algarismos das centenas simples, centenas de milhar e centenas de milhão iguais a Z.

Pode-se afirmar que N sempre será divisı́vel por:

a) 333664. c) 333666. e) 333668.


b) 333665. d) 333667.

Problema 53. Se a e b são números naturais e 2a + b é divisı́vel por 13, então qual das alternativas contém
um múltiplo de 13?

a) 91a + b. c) 93a + b. e) 95a + b.


b) 92a + b. d) 94a + b.

Problema 54. Sabendo-se que o resultado de

12 · 11 · 10 · · · · · 3 · 2 · 1 + 14

é divisı́vel por 13. Qual o resto da divisão do número

13 · 12 · 11 · · · · · 3 · 2 · 1

por 169?
Problema 55. Qual é o resto da divisão de

10111213141516..........979899

por 9?

5
POTI 2015 − Teoria dos Números − Nı́vel 2 − Aula 0 − Cleber Assis, Samuel Barbosa e Tiago Miranda

3 Potenciação
Problema 56. Calcule o valor das expressões:

a) 35 . b) 22 + 32 . c) 54 . d) 23 + 33 . 1 4
e) · 2 · 3.
2

Problema 57. Calcule o valor das expressões:

1  3 1
a) (0, 01)3 . b) 100 · . 5 d) · (0, 3)2 . e) 200 · (0, 04)4 .
52 c) 80 · . 3
2

Problema 58. Se a = 2 e b = 3, calcule o valor das expressões:

a3 b b) ab . c) a3 b2 . d) ( ab2 )2 . e) (b + a)2 − a2 .
a) .
b2

Problema 59. Escreva como um única potência:

24 · 26 46 · 82 c) (−32)3 .
2
105 · 10−3 · 10 e) 83 : 2−5 .
a) . b) . d) .
37 · 33 163 10−7 · 104

Problema 60. Determine quais das seguintes sentenças são verdadeiras e quais são falsas. Em cada item
falso, indique um contraexemplo para a afirmação.
 a n m
a) an bn = ( a · b)n . b) a−n = − an . c)
n m nm
= ( a − b)n . d) ( a ) = a . e) ( an )m = a(n ) .
b

Problema 61. Determine quais das seguintes sentenças são verdadeiras e quais são falsas. Em cada item
falso, indique um contraexemplo para a afirmação.
 a −n  b n
b) ( a + b)n = an + bn . d) ( an )−n = a0 .
a) = .
b a
c) an+m = an + am . e) Se a 6= 0 então a0 = 1.

Problema 62. Calcule as potências:

a) (0, 3)2 . b) (0, 3)−2 . c) (−0, 02)3 . d) (−3)−2 . e) (1, 2)3

Problema 63. Escreva cada um dos seguintes números como uma potência de 2:

a) (−0, 5)−4 . b) [(−0, 25)2 ]−6 . c) 162 : (0, 25)−4 . d) 32−2 : (0, 25)−4 . e) 0, 16 · 102 .

Problema 64. Determine, em cada item, qual dos números é o maior.


 1/2  1/3
a) 21/2 ou 21/3 . 1 1 c) 31/5 ou 51/3 .
b) ou .
2 2

2
Problema 65. Dividindo-se o número 44 por 44 obtemos o número:

a) 2 b) 43 c) 44 d) 48 e) 412

6
POTI 2015 − Teoria dos Números − Nı́vel 2 − Aula 0 − Cleber Assis, Samuel Barbosa e Tiago Miranda

Problema 66. Definamos a operação a ⊗ b como sendo ab . Por exemplo, 2 ⊗ 3 = 8. Determine o valor de:

2 ⊗ (2 ⊗ (2 ⊗ 2))
.
((2 ⊗ 2) ⊗ 2) ⊗ 2
1 1 c) 1 d) 4 e) 256
a) b)
256 4
Problema 67. Para os inteiros a e b definimos a ∗ b = ab + b a . Se 2 ∗ x = 100, a soma dos algarismos de (4x )4
é igual a:

a) 20 b) 25 c) 30 d) 35 e) 27

Problema 68. Com quantos zeros termina o número 156 · 285 · 557 ?

a) 10 b) 18 c) 26 d) 13 e) 5

Problema 69. As potências 2n e 5n , onde n é um inteiro positivo, começam com o mesmo algarismo d. Qual
é este algarismo?
Problema 70. Se a = 240 , b = 320 e c = 710 , então:

a) c < b < a b) a < c < b c) b < a < c d) b < c < a e) c < a < b

Problema 71. Quanto vale 1212 ?
√ √
a) 66 c) 212 · 36 d) 612

b) 22 3
e) 12
12

Problema 72. Se 2(22x ) = 4x + 64, então x é igual a:

a) −2 b) −1 c) 1 d) 2 e) 3

7
POTI 2015 − Teoria dos Números − Nı́vel 2 − Aula 0 − Cleber Assis, Samuel Barbosa e Tiago Miranda

4 Números Racionais
Problema 73. Escreva os seguintes números na notação cientı́fica:

a) 45673. b) 0, 0012345. c) −555. d) 0, 09

Problema 74. Escreva o perı́odo dos decimais periódicos:

a) 0, 342342342 . . .. b) 58, 6777 . . .. c) 456, 989898 . . ..

Problema 75. Encontre a fração geratriz de:

a) 0, 333 . . .. b) 0, 121212 . . . c) 6, 5. d) −0, 666 . . ..

Problema 76. Obtenha as geratrizes das seguintes dı́zimas periódicas:

a) 4, 7222 . . .. b) 1, 8999 . . .. c) 1, 2010101 . . ..

Problema 77. Sem efetuar a divisão, determine se a fração corresponde a um decimal exato ou a uma dı́zima
periódica.

321 15 41 3
a) . b) . c) . d) .
320 6 15 40

Problema 78. Dizemos que um inteiro positivo x está escrito na notação cientı́fica se é da forma x = m · 10k
onde k é um inteiro e m satisfaz:

a) m é inteiro. b) 1 ≤ |m| < 10. c) m < 1. d) 1 ≤ m < 10. e) 0 < m < 1.

Problema 79. Assinale qual o maior dentre os números seguintes:

a) 1, 01. b) 1, 012. c) 1, 0102. d) 1, 01125. e) 1, 011.

Problema 80. Considere o número


X = 1, 01001000100001 . . . .
(O padrão se mantém, ou seja, a quantidade de zeros entre números uns consecutivos sempre aumenta
exatamente uma unidade).

a) Qual é a sua 25a casa decimal após a vı́rgula?

b) Qual é a sua 500a casa decimal após a vı́rgula?

c) O número X é racional ou irracional?

1
Problema 81. Qual é o primeiro dı́gito não nulo após a vı́rgula na representação decimal da fração ?
512

a) 1 b) 2 c) 4 d) 5 e) 7

8
POTI 2015 − Teoria dos Números − Nı́vel 2 − Aula 0 − Cleber Assis, Samuel Barbosa e Tiago Miranda

Problema 82. O valor da expressão

s s − 1
 3
1
 0
2 1 2
 · (0, 666 . . .) + − 
6 3 (1, 333 . . .)

é igual a:
√ r r √ √
2 2 5 5 2 3 5
a) b) c) d) e)
5 5 2 2 5

Problema 83. Observe as multiplicações:

142857 · 1 = 142857
142857 · 2 = 285714
142857 · 3 = 428571
142857 · 4 = 571428
142857 · 5 = 714285
142857 · 6 = 857142
142857 · 7 = 999999
1 142857
Da última multiplicação, podemos concluir que = = 0, 142857. Veja que as seis primeiras
7 999999
multiplicações produzem números com os mesmos dı́gitos de 142857 e este é exatamente o perı́odo da
1
representação decimal de . Você consegue descobrir um número primo p maior que 7 tal que o perı́odo da
7
1
dı́zima que representa possui p − 1 casas decimais?
p

Problema 84. Considere um primo p que divide 10n + 1 para algum n inteiro positivo. Por exemplo, p = 7
1
divide 103 + 1. Analisando o perı́odo da representação decimal de , verifique que o número de vezes que o
p
dı́gito i aparece é igual ao número de vezes que o dı́gito 9 − i aparece para cada i ∈ {0, 1, 2, . . . , 9}.

Problema 85. Considere um número primo p que não divide 10 e suponha que o perı́odo da representação
1
decimal de seja 2k. É sempre possı́vel decompormos o perı́odo em dois blocos de dı́gitos consecutivos
p
1
que somam 10k − 1? Por exemplo, o perı́odo de tem tamanho 6 = 2k pois é igual à 142857. Veja que
7
142 + 857 = 999 = 103 − 1 = 10k − 1.

9
POTI 2015 − Teoria dos Números − Nı́vel 2 − Aula 0 − Cleber Assis, Samuel Barbosa e Tiago Miranda

5 Números Irracionais
Problema 86. No quadro abaixo, determine quais números são irracionais.


23 5, 345 2

1
2, 313131 . . . 3 0, 01001000100001 . . .

Problema 87. Quais das seguintes afirmações são verdadeiras?

a) N ⊂ Q. b) Z ⊂ Q. c) 1 ∈ Q − Z. d) r ∈ Q ⇒ −r ∈ 35
e) ∈ Q − Z.
Q. 5

Problema 88. Represente em uma reta orientada os seguintes números:

9 14 30
3, 5 − 0 5, 2 −
4 7 7
Problema 89. Utilizando a calculadora podemos obter que

2 = 1, 4142135623730950488016887242097 . . .
Agora, também utilizando uma calculadora, calcule os valores abaixo, faça os registro e observe como o
resultado se aproxima cada vez mais do número 2.

a) 1, 42 = b) 1, 412 = c) 1, 4142 =
d) 1, 41422 =

Problema 90. Com base no exercı́cio anterior, utilizando a calculadora, calcule 3. Faça o mesmo procedi-
mento do item anterior, ou seja, calcule o o quadrado do número encontrado apenas com uma casa decimal,
depois com duas casas, depois com três
√ e finalmente com quatro casas. Registre os resultados e observe como
eles se aproximam cada vez mais de 3.
Problema 91. Compare as raı́zes abaixo preenchendo os espaços pontilhadas com os sı́mbolos > ou <.
√ √ √ √ q q √ √
a) 2 . . . . . . 3. b) 81 . . . . . . 121. c) 4
. . . . . . 16
. d) 0, 64 . . . . . . 0, 1.
100 25
√ √
e) n . . . . . . n + 1 com n número real não negativo.

Problema 92. Sem utilizar a calculadora, estime, com uma casa decimal, a melhor aproximação para 11.

Problema 93. Sem utilizar a calculadora, estime, com duas casas decimais, uma boa aproximação para 11.
√ √
Problema 94. Quantos números inteiros positivos existem entre 8 e 80?
√ √
Problema 95. Quantos números inteiros positivos existem entre 37 e 1226?
Problema 96. Quantos dos números abaixo são maiores que 10?
√ √ √ √ √
3 11 , 4 7 , 5 5 , 6 3 , 7 2.

a) 1 b) 2 c) 3 d) 4 e) 5

Problema 97. Explique porque entre dois números racionais sempre podemos encontrar um terceiro número
racional.
√ √
Problema 98. Dados dois reais positivos, 3 3 e 4 4, determine o maior.
p √ √ √ √
Problema 99. O número 1 + 3 4 + 3 16 está situado entre n e n + 2, onde n é inteiro positivo. Deter-
mine n.

10
POTI 2015 − Teoria dos Números − Nı́vel 2 − Aula 0 − Cleber Assis, Samuel Barbosa e Tiago Miranda

Problema
√ 100. Prove que não é possı́vel escrever 2 como uma fração de inteiros. Ou seja, prove que
/ Q.
2∈
Problema 101. Prove que não é possı́vel escrever:
√ √
/ Q.
i 3 como uma fração de inteiros. Ou seja, prove que 3 ∈
√ √
/ Q.
ii 5 como uma fração de inteiros. Ou seja, prove que 5 ∈
√ √
/ Q.
iii p como uma fração de inteiros, sendo p um número primo. Ou seja, prove que p ∈

Problema 102. É verdade que existem números irracionais A e B tais que A B é racional?
a
Problema 103. A sequência Fn de Farey é uma sequência de conjuntos formados pelas frações irredutı́veis
b
com 0 ≤ a ≤ b ≤ n arranjados em ordem crescente. Exibimos abaixo os quatro primeiros termos da sequência
de Farey.

F1 = {0/1, 1/1}

F2 = {0/1, 1/2, 1/1}

F3 = {0/1, 1/3, 1/2, 2/3, 1/1}

F4 = {0/1, 1/4, 1/3, 1/2, 2/3, 3/4, 1/1}

Qual deve ser o conjunto F5 ?


a c
Problema 104. É possı́vel mostrar que se duas frações e são vizinhas na sequência de Farey Fn (veja
b d
a
o exercı́cio anterior) então ad − bc = ±1. Sabendo disso, você consegue determinar que fração está
b
5
imediatamente à esquerda de em F7 sem calcular todos os seus elementos?
7
Problema 105. Considere dois tambores de capacidade suficientemente grande.

a) Determine se é possı́vel colocar exatamente um litro do lı́quido de um dos tambores no outro usando dois
baldes, um com capacidade de 5 e o outro com capacidade de 7 litros.

b) Determine se é possı́vel colocar exatamente


√ um litro do lı́quido de um√dos tambores no outro usando dois
baldes, um com capacidade de 2 − 2 e o outro com capacidade de 2 litros.

Problema 106. Achar o menor inteiro positivo n tal que as 73 frações

19 20 21 91
, , ,...,
n + 21 n + 22 n + 23 n + 93
sejam todas irredutı́veis.

11
POTI 2015 − Teoria dos Números − Nı́vel 2 − Aula 0 − Cleber Assis, Samuel Barbosa e Tiago Miranda

6 Radiciação e Expressões Algébricas


Problema 107. Simplifique as expressões envolvendo radicais:

3
√ p √ √
a) x4 . b) ( 3 8)2 . c) 4 81x8 y4 . d) 32 + 162. e) (4a6 b4 )3/2 .

Problema 108. Transforme a expressão dada em outra sem radicais no denominador como indica o exemplo:

5

5
1 1 x3 x3
√5
= √
5
· √
5
= .
x2 x2 x3 x
2 1 2 1
r
a) √ . b) √ . 1 d) √ . e) √ .
4 3 c) 9 2 . 3
x 4
a
3 x a

Problema 109. Elimine os expoentes negativos das expressões abaixo:


 −2
x −3 y 4 6st−4 ba−4 a −3 a −5

a) 5 −3 . b) . c) . d) · .
x y 2s−2 t2 ab−3 b −2 b −3

1 2

1
3 − +
2 4
Problema 110. Simplificando a expressão  , obtemos:
1 2

3
3 − −
3 2

6 7 6 7 5
a) − . b) − . c) . d) . e) − .
7 6 7 6 7

Problema 111. Simplifique a expressão:


2
3x2 y


a3 b3
3
3xy2


2a2 b2

Problema 112. Simplifique as expressões:


p
3
√ p qp

a) 64x24 . b) 4 x4 y8 z2 . c) x.

Problema 113. Determine o valor da expressão abaixo quando a = 2014 e n = 1000.

1 1 1 1 1 1 1
+ − n +1 + . . . + −1 + 0 + n + +...+ 1 .
a−n+1 a +1 a +1 a +1 a +1 1+a − n + 1 a +1
a) 10002013 b) 20131000 c) 2013 2001 e) 1000.
d)
2

Problema 114. Ao efetuar a soma 131 + 132 + 133 + . . . + 132006 + 132007 , obtemos um número inteiro. Qual
o algarismo das unidades desse número?

a) 1 b) 3 c) 5 d) 7 e) 9.

12
POTI 2015 − Teoria dos Números − Nı́vel 2 − Aula 0 − Cleber Assis, Samuel Barbosa e Tiago Miranda

Problema 115. Efetuando as operações indicadas na expressão

22007 + 22005
 
· 2006
22006 + 22004

obtemos um número de quatro algarismos. Qual é a soma dos algarismos desse número?

a) 4 b) 5 c) 6 d) 7 e) 8

Problema 116. Sejam a, b e c inteiros e positivos. Entre as opções abaixo, a expressão que não pode
representar o número 24 é:

a) ab3 b) a2 b3 c) ac bc d) ab2 c3 e) ab bc c a .

Problema 117. Calcule o valor de


1001 · 1002 · 1003 · . . . · 2000
A=
1 · 3 · 5 · . . . · 1999
a) 21000 b) 2999 c) 1000 d) 999 e) 2.

13
POTI 2015 − Teoria dos Números − Nı́vel 2 − Aula 0 − Cleber Assis, Samuel Barbosa e Tiago Miranda

7 Introdução aos Polinômios


Problema 118. Seja n um número natural. Indique por meio de expressões algébricas:

a) o dobro de n. c) o sucessor de n. e) o cubo de n.


b) 20% de n. d) a metade da soma entre n e 3.

Problema 119. Determine a área de um retângulo cujas dimensões (comprimento e largura) são:

a) 2x e x. b) 2x e ( x + 1). c) ( x − 1) e ( x + 2).

Problema 120. Escreva os graus de cada monômio:



a) 2x5 y3 . b) − 45 m2 n. c) 5p5 qr5 . d) am bn cd.

Problema 121. Considere os monômios A = 8x3 y2 e B = 4xy. Determine:

a) A · B. A
b) .
B

Problema 122. Sejam os polinômios P = 3x2 + 4x − 8 e Q = x2 + 1, determine:

a) P + Q. b) P − Q. c) P · Q.

Problema 123. Efetue as multiplicações:

a2
 
a) ( a + 1)( a2 − 6a + 4). b) (3a − b)(3ab + 2a − b). 8
c) 1, 2a2 + 1, 6a + .
4 3

Problema 124. Um taxista cobra, por corrida, R$3, 00 como preço fixo inicial e mais R$2, 50 para cada km
rodado.

a) Determine a expressão que representa quanto será cobrado por uma corrida de x km.

b) Quanto custa uma corrida de 9km?

Problema 125. Os produtos algébricos da forma ( x + a)( x + b), onde x é variárel e a e b são números reais
quaisquer, podem ser calculados usando-se a distributividade, obtemos assim ( x + a)( x + b) = x2 + ( a +
b) x + ab. Veja que o coeficiente de x é a soma de a e b e o coeficiente independete de x é ab. Por exemplo,
( x + 2)( x + 5) = x2 + 7x + 10. Utilize este princı́pio e calcule os produtos:
  
a) ( x + 1)( x + 2) d) ( x − 4)( x + 4). 1 3
g) x + x+ .
2 2
b) ( x + 3)( x + 9). e) ( x + 5)( x + 5).
c) ( x − 2)( x + 3). f) ( x − 4)( x − 4).

Problema 126. O Teorema do Resto diz que o resto da divisão de um polinômio P de uma variável x por
outro polinômio da forma ( x + a) é igual ao valor de P quando substituimos x por − a. Use este teorema para
calcular o resto da divisão de x3 − 4x2 + 5x − 1 por:

a) x − 1. b) x + 1. c) x − 3. d) x + 4. 1
e) x − .
2

14
POTI 2015 − Teoria dos Números − Nı́vel 2 − Aula 0 − Cleber Assis, Samuel Barbosa e Tiago Miranda

Problema 127. Use o Teorema do Resto para verificar se x4 + 2x3 − 3x2 − 8x − 4 é divisı́vel por:

a) x − 1. b) x + 1. c) x + 2. d) x − 2. e) x + 3.

Problema 128. Em um jogo de perguntas e respostas, ganham-se 5 pontos por acerto e perdem-se 3 pontos
por erro.

a) Determine a expressão que representa o número de pontos obtidos por alguém que acertou x perguntas e
errou y perguntas.

b) Qual a pontuação de Maycon, se ele acertou 8 e errou 2 perguntas?

Problema 129. Um tanque de combustı́vel possui a capacidade máxima de 50 litros. Se já existem x litros de
combustı́vel neste tanque, determine:

a) A expressão que da quantidade de litros que faltam para completá-lo.

b) A expressão que determina do quanto será gasto para completá-lo, se o litro de combustı́vel custa R$3, 00.

Problema 130. Simplifique as expressões:

a) a9 · a−5 . b) (3y2 )(4y5 ). (2x3 )2 (3x4 ) d) (2a3 b2 )(3ab4 )3 .


 3  2 4
x y x
c) . e) .
( x 3 )4 y z

Problema 131. Elimine os expoentes negativos das expressões abaixo:


 −2
x −3 y 4 6st−4 ba−4 a −3 a −5

a) . b) . c) . d) · .
x 5 y −3 2s−2 t2 ab−3 b −2 b −3

Problema 132. Simplifique a expressão:


2
3x2 y


a3 b3
3
3xy2


2a2 b2
Problema 133. Determine o grau dos monômios abaixo:

a) 5a2 b7 .
7 n +1 n +2
b) 2a b , onde n é um número natural.

c) ab2 c3 d4 . . . z26 (onde é colocado em cada letra do alfabeto um expoente correspondendo à sua posição).

Problema 134. Determine o valor do inteiro positivo n para que o grau do monômio 5x n+1 y2n−1 seja 9.
Problema 135. Determine o valor de k para que o produto (kx − 1)(2x + 1) seja um polinômio cuja soma
dos coeficientes é 3.
Problema 136. Use a propriedade de distributividade da multiplicação e resolva os produtos:

a) ( x + an )( x − an ). b) ( x + a2n )( x + a2n ). c) ( x − 2a)2 .

Problema 137. Determine o quociente e o resto das divisões:

a) ( x2 − a2 ) ÷ ( x − a). b) ( x2 + 2xa + a2 ) ÷ ( x + a). c) ( x3 + a3 ) ÷ ( x + a).

15
POTI 2015 − Teoria dos Números − Nı́vel 2 − Aula 0 − Cleber Assis, Samuel Barbosa e Tiago Miranda

−b + b2 − 4ac
Problema 138. Determine o valor de x = quando:
2a
a) a = 1; b = 4; c = 4. b) a = 1; b = −2; c = −8. c) a = 1; b = 5; c = 0.

Problema 139. Determine k para que o polinômio P( x ) = x3 − 2x2 + 3x + k seja divisı́vel por:

a) x − 1 b) x + 1 c) x − 2.

Problema 140. De uma cartolina quadrada de 50cm de lado, retira-se através de cortes um quadrado de x
cm de lado de cada um dos quatro cantos da cartolina, sendo 0 ≤ x ≤ 25 . Determine:
a) A expressão que determina a área da cartolina após os cortes.
b) A área da cartolina cortada se x = 5cm.
c) Suponha que a cartolina cortada é usada para formar uma caixa sem tampa dobrando-se ao longo das
retas determinadas pelos cortes. Qual a expressão envolvendo x fornece o volume de tal caixa?
Problema 141. Simplifique a expressão:
( x2n+1 + x )( x2n+1 − x ) − ( x4 )(n+1/2)
,
( x n + x )2 − x2n − 2x n+1
definida para x 6= 0.
Problema 142. Sejam A = x3 + 6x2 − 2x + 4 e B = x2 − 1, polinômios. Determine o quociente e o resto de
A na divisão por B.
Problema 143. Leila foi avisada em dezembro de 2012, que a mensalidade escolar de seus filhos para o ano
de 2013 teria um aumento de 80%. Ela não concordou com o aumento e procurou o PROCON que, após
analisar o caso, determinou que a escola reduzisse este último valor em 30%. A escola acatou a decisão do
PROCON. Além disso, como Leila tem 3 filhos matriculados, a escola decidiu lhe dar 10% de desconto nas
mensalidades de cada um de seus filhos. Determine:
a) A expressão que determina o preço da mensalidade de cada filho de Leila em 2013.
b) Quanto Leila gastará com mensalidades em 2013, se a mensalidade, em 2012, era R$ 800,00.
p
Problema 144. A expressão 3 −( x − 1)6 é um número real. Determine:
a) O valor da expressão para x = 2.
b) O maior valor possı́vel para a expressão.
Problema 145. a) Calcule o valor de:
     
1 1 1 1
1+ 1+ 1+ ... 1+
2 3 4 99

b) Calcule o valor de:     


1 1 1
1+ 1+ ... 1+
x+1 x+2 x + 98
Problema 146. a) Calcule o valor de
1001 · 1002 · 1003 · . . . · 2000
A=
1 · 3 · 5 · . . . · 1999
b) Se x é um inteiro positivo, calcule o valor de:
( x + 1)( x + 2)( x + 3) · . . . · (2x )
B=
1 · 3 · 5 · . . . · (2x − 1)

16
POTI 2015 − Teoria dos Números − Nı́vel 2 − Aula 0 − Cleber Assis, Samuel Barbosa e Tiago Miranda

8 Produtos Notáveis
Problema 147. Siga o modelo e calcule os produtos notáveis:

( x + 5)2 = x2 + 2 · x · 5 + 52
= x2 + 10x + 25

a) ( x + 1)2 . b) (4 + x )2 . c) ( x + 3)2 . d) (3x + 1)2 . e) (4x + 2)2 .

Problema 148. Calcule os produtos notáveis:

a) (2x + 3)2 . b) (2x + 3y)2 . c) ( x2 + 3)2 . d) ( a2 + 3b2 )2 . e) ( x4 + 32 )2 .

Problema 149. Veja o seguinte exemplo para calcular o quadrado de um número:

422 = (40 + 2)2


= 402 + 2 · 40 · 2 + 22
= 1600 + 160 + 4
= 1764

Calcule os quadrados de 13, 41 e 19 sem usar a calculadora.


Problema 150. Calcule o valor das expressões:
√ √ √
a) ( a + b)2 − 2 ab. c) ( a + 1)2 + 2( a + 1) a + a2 + 2(2a + 1) + 1.
b) ( x + 1)2 + ( x − 1)2 .

Problema 151. Calcule as expressões:

a) (− a − b)2 . b) (−2a + b)2 . c) (2ab + 3c)2 . d) (2a − 2b)2 .

Problema 152. Calcule os produtos:

a) ( x − 1)( x + 1). c) ( x2 − 3z)( x2 + 3z).


√ √ √ √
b) (4 − a)(4 + a). d) ( x + y)( x − y)( x + y)

Problema 153. Siga o modelo abaixo e calcule o valor das expressões dadas.

27 · 33 = (30 − 3)(30 + 3)
= 302 − 32
= 891

a) 99 · 101. b) 1998 · 2002. c) 5 · 15 + 25

Problema 154. Ao efetuarmos a multiplicação ( a + b)( a + b) usando a distributividade, quantas operações


de multiplicação faremos?
Problema 155. Repita o exercı́cio anterior com a multiplicação ( a + b)( a + b)( a + b). Em seguida, determine
quantas cópias de a2 b aparecem no resultado. Finalmente, conclua com argumentos de contagem que:

( a + b)3 = a3 + 3a2 b + 3ab2 + b3 .

17
POTI 2015 − Teoria dos Números − Nı́vel 2 − Aula 0 − Cleber Assis, Samuel Barbosa e Tiago Miranda

Problema 156. Encontre uma figura que explique geometricamente, através do uso de áreas, a equação:
6·7
1+2+3+4+5+6 = .
2
Problema 157. A figura abaixo explica geometricamente, usando áreas, o desenvolvimento do produto
notável
( a + b)2 = a2 + 2ab + b2 .

Você conseguiria obter uma figura que explicasse geometricamente, também usando áreas, a equação

( a + b )2 + ( a − b )2 = 2( a2 + b2 ) ?

Problema 158. Encontre uma figura que explique geometricamente, através do uso de áreas, a equação:

1 + 3 + 5 + . . . + 17 = 92

Problema 159. O professor Medialdo acaba de explicar a seus alunos que a média aritmética de dois números
a+b √
a e b é e a média geométrica é ab. Antes de entregar as notas de duas provas aplicadas anteriormente,
2
ele decidiu testar o conhecimento dos seus alunos perguntando se eles prefeririam que cada um recebesse a
média geométrica ou a média aritmética das duas notas. Considerando que os alunos desejam a maior nota
possı́vel no boletim, o que eles devem dizer ao professor Medialdo?
Problema 160. Sejam a e b números reais.
a) Verifique que ( a + b)2 ≥ 4ab.
1 1 4
b) Verifique que + ≥ .
a b a+b
1 1 4 16 64
c) Verifique que + + + ≥ .
a b c d a+b+c+d
Problema 161. João deseja construir um retângulo usando um arame com 2 metros de comprimento. Qual é
a maior área possı́vel de tal retângulo?
Problema 162. Sejam:
r s r s r
q √ q √ q √ q √
A= 2+ 3· 2+ 2+ 3eB= 2+ 2+ 2+ 3× 2− 2+ 2+ 3

Quanto vale A · B?
√ √ √ √
a) 2 b) 3 c) 1 d) 2 + 2 e) 2 + 3.

Problema 163. Calcule o valor do número:

201420132 − 2(20142013)(20142012) + 201420122

18
POTI 2015 − Teoria dos Números − Nı́vel 2 − Aula 0 − Cleber Assis, Samuel Barbosa e Tiago Miranda
√ √ √ √
Problema 164. Se x, y, a e b são reais tais que x−y = a e x+ y = b, determine o valor de xy.

b4 − a4 a2 a2 + b2 1 e) a2 .
a) b) c) d)
4b2 b b b

Problema 165. João está ajudando seu pai com as finanças de sua loja. Como a quantidade de produtos
ofertados estava influenciando a quantidade de produtos vendidos, ele decidiu procurar algum padrão que
pudesse ajudá-lo a descobrir qual a quantidade ideal de produtos que deveriam ser ofertadas para maximizar
a quantidade de produtos vendidos. Depois de um bom tempo “quebrando a cabeça”, ele percebeu que se
“a” produtos eram ofertados, então a loja vendia “a(10 − a)” itens. Em seguida, com a ajuda de um produto
notável semelhante a essa expressão, foi possı́vel achar a quantidade ideal de produtos que deveriam ser
vendidos. Como ele fez isso?
Problema 166. O pai de João (veja o problema anterior), percebendo a astúcia do filho, decidiu desafiá-lo a
fazer o mesmo com uma fórmula bem diferente e supondo agora que a é um número real qualquer. Nesse
novo problema, dado “a” real, ele deve tentar achar o valor máximo de 4a − a4 . Novamente usando produtos
notáveis, João conseguiu descobrir que o máximo de tal expressão é 3. Você consegue descobrir como ele fez
isso?

19
POTI 2015 − Teoria dos Números − Nı́vel 2 − Aula 0 − Cleber Assis, Samuel Barbosa e Tiago Miranda

9 Fatoração de Expressões Algébricas


Problema 167. Siga o modelo e fatore as expressões:

3a + ba = a(3 + b)

a) 5a + ba. b) am + an. c) xa + xb + xc. d) ax + a. e) ab + bc + abc.

Problema 168. Simplifique as frações fatorando o denominador e o numerador.


3a + 5b 3x + 3y 3a2 + 5a d) x4 + x3
a) . b) . c) . e) 2 .
6a + 10b 8x + 8y 6a + 10 a( x + y) + b( x + y) x +x
.
( a − b) x + ( a − b)y
Problema 169. Fatore por agrupamento as seguintes expressões:

a) a2 + ab + ac + bc. b) ax − bx + ay − c) 2ab + 2a + b + 1. d) ax − bx + 2a − 2b e) 10ab − 2b +


by. 15a − 3

Problema 170. Fatore o numerador e o denominador e simplifique cada expressão dada:

m4 + m2 x3 + x2 + x + 1 m4 + 3m3 + 2m + 6
a) . b) . c) .
m2 + 1 x3 + x2 ( m + 3)2
Problema 171. Fatore as expressões abaixo usando a diferença de quadrados:

a) a2 − 25b2 . b) 4x2 − 1. c) 7 − x2 . d) a2 x2 − b2 y2 . e) a4 − b4

Problema 172. Para cada um dos itens abaixo, decida se a expressão dada é o quadrado de um binômio,
isto é, se pode ser escrita na forma:
( a + b)2 = a2 + 2ab + b2 ou como ( a − b)2 = a2 − 2ab + b2 .

a) x2 − 4x + 3. 1 c) y2 + 6y + 18. d) 4z2 − 12zy + 9y2 . e) 3z2 + 6z + 3.


b) x2 + x + .
4
Em caso afirmativo, escreva o binômio.
Problema 173. Fatore completamente as expressões abaixo:

a) x4 − 2x2 + 1. b) 5a2 − 10a + 5. c) a2 − b2 − 2bc − c2 .

Problema 174. Efetue as multiplicações e divisões indicadas como no exemplo:


2ab 5xy 2ab 5x y
· = ·
3ax 7by 3a x 7by
2 5
= ·
3 7
10
=
21
4a 5b x3 + x x2 + 1 yx + x xy + y
a) · . b) ÷ . c) · .
b a 3y y2 ( x + 1) ( y + 1)2
2

Problema 175. Se xy = 6 e x + y = 7, quanto vale x2 y + y2 x?


a
Problema 176. Se, ao adicionarmos x ao numerador e subtrairmos x do denominador da fração , com a e
b
c
b reais, obtemos a fração , com c e d reais e c 6= −d, qual o valor de x?
d

20
POTI 2015 − Teoria dos Números − Nı́vel 2 − Aula 0 − Cleber Assis, Samuel Barbosa e Tiago Miranda

Problema 177. Fatore as expressões:

a) a2 b − b3 . b) x2 − 2xy + y2 − 9. c) a4 − 32a2 + 256.

Problema 178. Verifique que:


x3 − y3 = ( x − y)( x2 + xy + y2 ).
Em seguida, fatore x3 − 8.
Problema 179. No exercı́cio anterior, o que acontece se trocarmos y por −z?
Problema 180. A soma de dois números é 4 e seu produto é 1. Encontre a soma dos cubos desses números.
Problema 181. Se xy = x + y = 3, calcule x3 + y3 .
1 1
Problema 182. Seja x um número real tal que x + = 2, calcule x2 + 2 .
x x
Problema 183. Qual a forma mais simplificada da expressão ( a − b)2 + (− a + b)2 + 2( a − b)(b − a)?
Problema 184. Simplifique a expressão
√ √ √ √ √ √ √ √ √ √ √ √
( 5 + 6 + 7)( 5 + 6 − 7)( 5 − 6 + 7)(− 5 + 6 + 7).

Problema 185. Fatore completamente x4 + 4.


Problema 186. Verifique que

n(n + 1)(n + 2)(n + 3) + 1 = (n(n + 3) + 1)2

Problema 187. Calcule o valor de:


q
(2014)(2015)(2016)(2017) + 1

Problema 188. Fatore p4 − 1.


p √ p √
Problema 189. Se x = 3 − 8 − 3 + 8, mostre que x é um inteiro negativo.
Problema 190. Fatore n5 + n4 + 1.
√ √
Problema 191. Qual é o menor inteior positivo n tal que n− n − 1 < 0, 01
Problema 192. Encontre o quociente da divisão de a32 − b32 por

( a16 + b16 )( a8 + b8 )( a4 + b4 )( a2 + b2 )

Problema 193. Verifique que


(23 − 1)(33 − 1) . . . (1003 − 1) 3367
3 3 3
= .
(2 + 1)(3 + 1) . . . (100 + 1) 5050
Problema 194. A sequência de Fibonacci é definida recursivamente por Fn+2 = Fn+1 + Fn para n ∈ Z e
F1 = F2 = 1. Determine o valor de:

F2 F2 F2
    
1 − 22 1 − 32 · . . . · 1 − 2013
2
F3 F4 F2014
F2016 F2014 2
F2015 F2015 F2015
a) 2 b) c) d) e) .
F2013 F2013 F 2 2 2F2013 F2014
2013

Problema 195. Se x + y + z = 0, verifique que:

x3 + y3 + z3 = 3xyz.

21
POTI 2015 − Teoria dos Números − Nı́vel 2 − Aula 0 − Cleber Assis, Samuel Barbosa e Tiago Miranda

Problema 196. Define-se o conjunto de 100 números {1, 1/2, 1/3, ..., 1/100}. Eliminamos dois elementos
quaisquer a e b deste conjunto e se inclui, no conjunto, o número a + b + ab ficando assim um conjunto com
um elemento a menos. Depois de 99 destas operações, ficamos só com um número. Que valores pode ter esse
número?
Problema 197. Verifique que

( x + y + z)3 = x3 + y3 + z3 + 3( x + y)( x + z)(y + z).

Problema 198. Verifique que:

( x + y + z)2 = x2 + y2 + z2 + 2xy + 2xz + 2yz

Problema 199. Fatore a expressão

( b − c )3 + ( c − a )3 + ( a − b )3 .

Problema 200. Sejam a, b, c, x, y, z reais distintos tais que ax + by + cz = 0. Verifique que

ax2 + by2 + cz2


bc(y − z)2 + ca(z − x )2 + ab( x − y)2

não depende de x, nem de y, nem de z.

22
POTI 2015 − Teoria dos Números − Nı́vel 2 − Aula 0 − Cleber Assis, Samuel Barbosa e Tiago Miranda

10 Sentenças Matemáticas e Notação Algébrica


Problema 201. Nos parênteses dos ı́tens abaixo, marque A, caso a sentença seja aberta, ou F, caso a sentença
seja fechada.

a) ( ) 42 = 15 + 1. e) ( ) 7 ∈ N.

b) ( ) 2x − 1 = x + 4. 1
√ f) ( ) = 2x.
c) ( ) 1 < 2. x+1

d) ( ) 2a − 1 = b. g) ( ) x2 = 5.

Problema 202. Quais das sentenças fechadas abaixo são verdadeiras?



a) 52 = 42 + 32 . 1 1 1 g) 25 ∈ Q.
d) · = .
2 3 6 √
b) 7 − 13 = −6. h) 3
/ Z.
−8 ∈
e) −6 ∈ N.
1 1 2 √ 1 1
c) + = . f) 16 > 4. i) − <− .
2 3 5 2 3

Problema 203. Utilize sı́mbolos matemáticos e letras para representar as grandezas e reescrever as sentenças
abaixo.

a) O perı́metro de um quadrado é o quádruplo da medida do seu lado.

b) A área de um quadrado é o quadrado da medida do seu lado.

c) A soma das idades de Luiz e Luı́sa é dezesseis.

d) A metade da raiz quadrada de um número é menor que o triplo desse número.

e) O salário de Rodrigo é setecentos reais mais vinte por cento do valor de suas vendas.

f) A área de um retângulo cuja altura é o dobro da base é o dobro do quadrado da base.

Problema 204. Seja l a medida da aresta de um cubo. Determine as expressões correspondentes

a) a sua área A.

b) ao seu volume V.

c) à soma S das medidas de todas as arestas.

Problema 205. Diz a lenda que no túmulo de Diofanto (matemático grego da antiguidade) havia o seguinte
problema:
Viajante, aqui estão as cinzas de Diofanto. É milagroso que os números possam medir a extensão de sua vida: 1/6 dela foi
uma bela infância; depois de 1/12 de sua vida, sua barba cresceu; 1/7 de sua vida passou em um casamento sem filhos;
cinco anos após isso nasceu seu primeiro filho, que viveu metade da vida de seu pai; e, em profundo pesar, o pobre velho
terminou seus dias na terra quatro anos após perder seu filho. Quantos anos viveu Diofanto?
Construa uma equação, utilizando os dados do túmulo, na qual seja possı́vel calcular a idade de Diofanto e a
resolva.

23
POTI 2015 − Teoria dos Números − Nı́vel 2 − Aula 0 − Cleber Assis, Samuel Barbosa e Tiago Miranda

Problema 206. A figura abaixo é o desenho de um terreno retangular dividido em três retângulos menores.
Determine:

a) uma expressão que representa o perı́metro P do terreno.

b) uma expressão que representa a quantidade Q de cerca gasta, se todos os retângulos serão cercados e lados
comuns recebem cerca apenas uma vez.

c) uma expressão que representa a área A do terreno.

Figura 3

Problema 207. O retângulo ABCD abaixo representa um terreno. Deve-se passar uma cerca que o divida de
maneira que a área do polı́gono CDEF seja o dobro da área do polı́gono ABFE. Sobre o lado AD essa cerca
começa a 5m do vértice A e sobre o lado BC essa cerca termina a x metros do vértice B.

a) Represente algebricamente a área dos dois polı́gonos separados pela cerca.

b) Determine o valor de x.

Figura 5

24
POTI 2015 − Teoria dos Números − Nı́vel 2 − Aula 0 − Cleber Assis, Samuel Barbosa e Tiago Miranda

Respostas e Soluções.
1. (Extraı́do da Vı́deo Aula)
Observe que se tivéssemos começado a contar pelo número 1, não haveria dúvidas quanto a quantidade de
elementos do conjunto {1, 2, 3, . . . , 908}. Como começamos sete unidades a mais que o 1, a resposta automática
seria 908 − 8 = 900. Este é um excelente ponto para lembrar que subtração não indica quantidade e sim
“distância” entre dois números. Ao calcularmos a distância do 908 (ou de m) até o 8 (ou de n) estamos contando
apenas o espaço entre eles, sendo assim, após a subtração devemos adicionar uma unidade para calcular a
exata quantidade. Por fim, a quantidade será
908 − 8 + 1 = 901 números.
De modo geral, a quantidade de números inteiros de m até n, sendo m > n, é m − n + 1.

Outra solução: Uma outra estratégia é fazermos um ajuste na contagem deslocando cada valor até o ponto
inicial, o 1, e depois simplesmente olhar onde terminou. Como 8 − 7 = 1 e 908 − 7 = 901, a quantidade de
elementos do conjunto {8, 9, 10, . . . , 908} é mesma que a do conjunto {1, 2, 3, . . . , 901}, isto é, 901 elementos.
2. Perceba que poderı́amos dividir todos os elementos do conjunto por 7 para começarmos a contar do 1
ficando com {1, 2, 3, . . . , 97, 98}. Portanto, há 98 elementos no conjunto inicial.
3. Perceba que podemos subtrair 9 de cada elemento do conjunto inicial e ficaremos com o conjunto
{5, 10, 15, . . . , 995, 1000}. Agora, dividindo todos os elementos do novo conjunto por 5 ficamos com {1, 2, 3, . . . , 199, 200}.
Portanto, há 200 elementos no conjunto inicial.
4. Como há 12 signos do zodı́aco, basta n = 13 para que duas pessoas tenham o mesmo signo. A ideia é
pensar nos Signos como as casas e nas pessoas como os pombos.
• ◦
Pombo sem casa, o 13 elemento.

• • • ··· •
Casa 1 Casa 2 Casa 3 Casa 12

Logo, há 12 casas, e para garantir que alguma das casas tenha dois pombos, basta ter n = 12 + 1 pombos.
5. (Extraı́do da Vı́deo Aula)
Observe que os números usados são
{10, 11, 12, 13, 14, 15, 16, 17, 18, 19, 20}.
São 20 − 10 + 1 = 11 números, cada um com dois algarismos, logo foram usados 11 × 2 = 22 algarismos.
6. Vamos pensar na quantidade de frutos como as casas e nas jaqueiras como os pombos (•)
0, 1, 2, · · · , 600

• • • ··· •
0 fruto 1 fruto 2 frutos 600 frutos
Agora coloquemos as jaqueiras (que serão os pombos) nas respectivas casas que representam suas quantidades
de frutos.

&
Caso ocupemos todas as casas, ainda haverá
399 jaqueiras a serem distribuı́das.

• • • ··· •
0 fruto 1 fruto 2 frutos 600 frutos
Como 1000 > 601, o PCP garante que alguma casa terá dois pombos.

25
POTI 2015 − Teoria dos Números − Nı́vel 2 − Aula 0 − Cleber Assis, Samuel Barbosa e Tiago Miranda

7. Considere as três cores como sendo as casas e as meias retiradas como os pombos.
a) Pelo Princı́pio da Casa dos Pombos, se retirarmos 4 meias, pelo menos duas delas terão a mesma cor. Para
ver que esse é o número mı́nimo, note que é possı́vel pegarmos uma meia de cada cor nas três primeiras
retiradas e não formarmos um par.


4a meia.

• • •
Meias Pretas Meias Brancas Meias Marrons
Resposta: 4 meias.
b) Observe que o cenário mais difı́cil para o objetivo é retirar todas as meias de cor preta, todas as meias de
cor marrom e depois o par de cor branca. Assim, deveremos retirar 20 + 10 + 2 = 32 meias para garantir o
par de cor branca.


32a meia

20 meias • 10 meias
Meias Pretas Meias Brancas Meias Marrons
Resposta: 32 meias.
8. Sejam x e y números inteiros pares, então podemos escrevê-los como x = 2a e y = 2b, para a e b inteiros.
Analogamente, se w e z são números inteiros ı́mpares, podemos escrever w = 2c + 1 e z = 2d + 1, com c e d
inteiros.

a) c) e)

x + y = 2a + 2b x + w = 2a + 2c + 1 x · y = 2a · 2b
= 2( a + b ), = 2( a + c) + 1, = 2 · 2ab,

é par. é ı́mpar. é par.

b) d) f)

w + z = 2c + 1 + 2d + 1 w · z = (2c + 1)(2d + 1) x · w = 2a · (2c + 1)


= 2( c + d + 1), = 2(2cd + c + d) + 1, = 2(2ac + a),

é par. é ı́mpar. é par.


9. (Extraı́do da Vı́deo Aula)
Observe que de 40 até 99 há 99 − 40 + 1 = 60 números de dois algarismos cada, logo foram utilizados
60 × 2 = 120 algarismos. Agora, de 100 até 999 há
999 − 100 + 1 = 900 números de três algarismos, o que totaliza 900 × 3 = 2700 algarismos. Seguindo
de 1000 até 1200 são 1200 − 1000 + 1 = 201 números com quatro algarismos, ou seja, 201 × 4 = 804. Por fim,
teremos
120 + 2700 + 804 = 3624 algarismos utilizados.
10. A soma pedida é
S = 100 + 101 + 102 + · · · + 999
900 · (100 + 999)
=
2
= 494550.

26
POTI 2015 − Teoria dos Números − Nı́vel 2 − Aula 0 − Cleber Assis, Samuel Barbosa e Tiago Miranda

11. (Extraı́do da Vı́deo Aula)


a) Como são 12 meses, com 24 pessoas no grupo não é possı́vel garantir que três delas façam aniversário no
mesmo mês, afinal poderı́amos ter exatamente 2 em cada mês. Agora, com 25 pessoas teremos certeza pois,
se cada mês receber no máximo dois aniversariantes, a 25a pessoa ficará sem data de aniversário possı́vel.
Logo, é preciso, no mı́nimo, 25 pessoas.
b) Como são 7 dias na semana, não basta termos 21 pessoas, pois poderı́amos ter 3 pessoas nascidas em cada
dia. Com a 22a pessoa, com certeza, haverá um dia no qual 4 pessoas nasceram. Portanto, no mı́nimo,
deveremos ter 22 pessoas.
12. (Extraı́do da Vı́deo Aula)
Se tirarmos 8 blusas, podem ser todas cinzas; tirando 10 blusas, podem ser todas amarelas; e sendo 12,
podemos ser todas beges. No caso de 18 poderiam ser as cinzas e as amarelos; para 20, as beges e as cinzas; e
para 22 as amarelas e as beges. Mas, com certeza, se forem 23 teremos uma de cada cor.
13. Como são 4 cores, poderemos dar o “azar” de em várias retiradas sempre chegarmos em 3 bolas de cada
cor, sem antes obtermos na 4a bola de cor repetida. Tirar 3 bolas de cada cor pode ser obtido após 4 × 3 = 12
retiradas. Daı́, com certeza, a 13a bola repetirá pela quarta vez alguma cor. Portanto, temos que retirar, no
mı́nimo, 13 bolas.
14. (Extraı́do da Vı́deo Aula)
Observe que se escrevermos a soma pedida no sentido inverso obteremos
S = 2014 + 2013 + 2012 + 2011 + · · · + 4 + 3 + 2 + 1
e, somando a forma original com sua escrita invertida, também obteremos
S = 1 + 2 +···+ 2013 + 2014
S = 2014 + 2013 +···+ 2 + 1
2S = 2015 + 2015 +···+ 2015 + 2015
2S = 2014 × 2015
S = 1007 × 2015,
que é o produto de números ı́mpares. Logo a soma dada é ı́mpar.
Observação: Veja que
2S = 2015
| + 2015 + ·{z
· · + 2015 + 2015}
2014 parcelas iguas a 2015.

pode ser facilmente transformada em uma multiplicação em função da igualdade das parcelas, resultando em
2S = 2014 × 2015.
Essa ideia pode ser aplicada na soma
S = 1 + 2 + 3 + · · · + (n − 2) + (n − 1) + n.
Repetindo o método chegaremos a
S = 1 + 2 +···+ ( n − 1) + n
S = n + ( n − 1) +···+ 2 + 1
2S = ( n + 1) + ( n + 1) +···+ ( n + 1) + ( n + 1)
2S = n × ( n + 1)

que produz a fórmula para a soma S dos naturais de 1 até n:


n · ( n + 1)
S= .
2

27
POTI 2015 − Teoria dos Números − Nı́vel 2 − Aula 0 − Cleber Assis, Samuel Barbosa e Tiago Miranda

15. (Adaptado do livro Cı́rculos Matemáticos)


Observe que cada disco dentado gira no sentido inverso que o dos seus vizinhos. Como o disco 1 gira no
sentido horário, o 2 ficará no anti-horário, o 3 no horário, e assim por diante. O que conclui que os ı́mpares
ficaram no sentido horário e os pares no anti-horário. Portanto, as proposições verdadeiras são as i e iii. Serão
apenas 2 proposições corretas.
16. (Adaptado da Vı́deo Aula)
Observe que se isso for possı́vel, poderemos separar os números de 1 até 10 em dois conjuntos de modo que a
soma S dos elementos do primeiro seja igual a soma dos elementos do segundo. Como esses conjuntos têm
todos os números citados, então
S + S = 1 + 2 + 3 + 4 + · · · + 10
10 · (1 + 10)
2S =
2
2S = 55.
Mas 2S é um número par e 55 é um número ı́mpar, então essa equação não tem solução inteira, daı́, não tem
como cumprir o que o problema perguntou.
17. Primeiro observe que não poderemos ter qualquer cor com 13 bolas, apenas conseguiremos isso com
as brancas e as verdes. Sendo assim, por “azar”, poderı́amos ter tirado todas as cores que não resolvem o
problema, totalizando 7 + 3 + 11 = 21 bolas. Agora restam apenas duas cores e como queremos treze bolas de
cor repetida devemos tirar ao menos mais 12 + 12 + 1 = 25. O que resulta em
21 + 25 = 46.
18. (Adaptado da OBMEP)
Observe que se n é ı́mpar, então n2 é ı́mpar, como está provado no exercı́cio 8. Assim, n2 ± n será par. Como
deseja-se um número ı́mpar, basta somarmos um ı́mpar. A resposta está na letra c.
19. Observe que 20102010 possui unidade par, pois é o produto de números pares, já 20112011 ficará com
unidade ı́mpar, 20122012 terá unidade par e essa alternância continuará. Por fim, a paridade resultante será
par + ı́mpar + par + ı́mpar + par + ı́mpar + par = ı́mpar.
20. (Extraı́do do Vestibular da PUC/RJ)
Tome os 12 meses como as casas e as n pessoas como os pombos. Se houver uma distribuição de 3 pessoas em
cada mês, não se chegará ao objetivo do problema e já terı́amos 12 × 3 = 36 pessoas no grupo. Agora basta
que mais uma pessoa seja colocada em qualquer uma das casas para concluir o problema. Portanto, 37 pessoas
num grupo garantem que ao menos 4 nasceram no mesmo mês.
21. (Extraı́do do Vestibular da UERJ/RJ - 2011)
Se retirarmos 30 bolas, é possı́vel que existam 3 bolas de cada cor e o objetivo não será cumprido. Com 31
bolas, pelo menos uma cor terá 4 representantes.
22. Como há 6 faces, para ter certeza que ao menos um delas saiu:
i) 2 vezes, deveremos ter ao menos 7 = 1 · 6 + 1 lançamentos;
ii) 3 vezes, deveremos ter ao menos 13 = 2 · 6 + 1 lançamentos;
iii) 4 vezes, deveremos ter ao menos 19 = 3 · 6 + 1 lançamentos;
iv) 5 vezes, deveremos ter ao menos 26 = 4 · 6 + 1 lançamentos; e
v) 6 vezes, deveremos ter ao menos 31 = 5 · 6 + 1 lançamentos.
A resposta é d = 31 lançamentos. A ideia é pensar que o número em cada face representa uma casa
(6 números = 6 casas). Queremos alguma casa com mais do que d pombos (lançamentos) então deve-se
distribuir os resultados dos lançamentos nas respectivas casas. Se tivermos 6d + 1 lançamentos, não é possı́vel
que cada número tenha saı́do no máximo d vezes e assim teremos uma casa com pelo menos d + 1 pombos.

28
POTI 2015 − Teoria dos Números − Nı́vel 2 − Aula 0 − Cleber Assis, Samuel Barbosa e Tiago Miranda

23. (Adaptado da Vı́deo Aula)


i) de 1 até 9 são 9 − 1 + 1 = 9 dı́gitos.
ii) de 10 até 99 são (99 − 10 + 1) × 2 = 180 dı́gitos.
iii) de 100 até 999 são (999 − 100 + 1) × 3 = 2700 dı́gitos.
COmo 9 e 180 são divisı́veis por 3 e 9 + 180 < 1002 < 9 + 180 + 2700, o 1002◦ será o último dı́gito de um
número de três dı́gitos. Observe que de 100 até um número de três algarismos n, temos 100 − n + 1 números
de 3 algarismos, logo, são (n − 100 + 1) × 3 dı́gitos nessa sequência. Queremos encontrar n tal que:
9 + 180 + 3 · (n − 99) = 1002
3 · (n − 99) = 1002 − 189
3 · (n − 99) = 813
3n − 297 = 813
3n = 813 + 297
3n = 1110
1110
n= .
3
n = 370.
Então, ao escrevermos o número 370, teremos 1002 termos na sequência, logo o 1002◦ termo será o 0.
24. (Extraı́do da Vı́deo Aula)
Os múltiplos de 3 entre 1 e 301 são
{3, 6, 9, . . . , 297, 300}.
A sua soma S pode ser escrita como
S = 3 + 6 + 9 + · · · + 297 + 300
S = 3(1 + 2 + 3 + · · · + 99 + 100)
100 · (1 + 100)
 
S = 3·
2
S = 15150.
25. (Extraı́do da OBMEP)
Observe que podemos desenvolver a equação pedida da seguinte forma:
xy
= 144
x+y
xy = 144x + 144y
xy − 144x − 144y + 1442 = 1442
x (y − 144) − 144(y − 144) = 124
( x − 144)(y − 144) = (22 · 3)4
( x − 144)(y − 144) = 28 · 34 .
Como estamos trabalhando com os números inteiros, ( x − 144) e (y − 144) dividem 1442 , ou seja, basta
calcularmos o número de divisores de 1442 = 28 · 34 . Esse número possui
(8 + 1) · (4 + 1) = 45
divisores inteiros positivos. Como não há restrição para os valores positivos, teremos
90 pares ordenados
que resolvem o problema.

29
POTI 2015 − Teoria dos Números − Nı́vel 2 − Aula 0 − Cleber Assis, Samuel Barbosa e Tiago Miranda

26. (Extraı́do da OBMEP)


Observe que podemos desenvolver a inequação dupla (ou simultânea) da seguinte maneira:
q
2000 < n · (n − 1) < 2005
q 2
20002 < n · (n − 1) < 20052

2000 · 2000 < n · (n − 1) < 2005 · 2005.

Então, podemos concluir que


n ∈ {2001, 2002, 2003, 2004, 2005},
totalizando 5 números inteiros e positivos. O que está na letra e.

27. Chame a soma pedida de S2 e siga o que foi iniciado nos exemplos do enunciado até o (n + 1)3 .

23 = 13 + 3 · 12 + 3 · 1 + 1
33 = 23 + 3 · 22 + 3 · 2 + 1
43 = 33 + 3 · 32 + 3 · 3 + 1
53 = 43 + 3 · 42 + 3 · 4 + 1
.. ..
.=.
( n − 1 + 1)3 = ( n − 1)3 + 3 · ( n − 1)2 + 3 · ( n − 1) + 1
( n + 1)3 = n3 + 3 · n2 + 3 · n + 1
Agora, some todos os membros dessas equações observando que todos os termos ao cubo do lado esquerdo se
anulam com os do lado direito, exceto o (n + 1)3 e o 13 . Obtemos assim
 
3 3 n ( n + 1)
( n + 1 ) = 1 + 3 · S2 + 3 · + n.
2
Por fim, chegaremos a
n(n + 1)(2n + 1)
12 + 22 + 32 + · · · + n2 = .
6
28. (Adaptado da Vı́deo Aula)
Utilizaremos a fórmula desenvolvida no exercı́cio 27, pois a área total é equivalente a soma 12 + 22 + 32 +
· · · + 1002 , ou seja, é uma soma de quadrados de números inteiros. Sendo assim, obteremos
100 · 101 · 201
12 + 22 + 32 + · · · + 1002 =
6
= 338350 u.a..
29. Cada computador pode estar conectado a 1, 2, 3, 4 ou 5 outras máquinas. Como há 6 computadores e
cinco opções de conexão, então ao menos dois computadores terão o mesmo número de conexões.
30. (Extraı́do da Vı́deo Aula)
Podemos escrever 58 − 28 como
  
58 − 28 = 54 + 24 54 − 24
 
4 4
52 + 22 52 − 22
 
= 5 +2
 
= 54 + 24 52 + 22 · 21.


Portanto, 21 | 58 − 28 . 

30
POTI 2015 − Teoria dos Números − Nı́vel 2 − Aula 0 − Cleber Assis, Samuel Barbosa e Tiago Miranda

31. (Extraı́do da Vı́deo Aula)


Podemos escrever 116 − 1 como

116 − 1 = 116 − 16
= 113 + 13 113 − 13
 

= 113 + 13 112 + 11 + 1 (11 − 1)


 

= 113 + 13 112 + 11 + 1 · 10.


 

Portanto, 10 | 116 − 1. 
31. (Outra solução.)
Podemos perceber que 116 termina em 1 (tem como algarismo das unidades o 1). E observando só as unidades,
ele será subtraı́do de 1, logo 116 − 1 termina em zero. Sendo assim, ele é divisı́vel por 10. 

32. Um número que é divisı́vel por 45 é, ao mesmo tempo, divisı́vel por 5 e por 9. Para ser divisı́vel por 5 deve
ter o algarismo das unidades igual a 0 ou 5, esses são os possı́veis valores de b. Para ser divisı́vel por 9, a soma
dos seus algarismos deve ser um múltiplo de 9, ou seja, 9 deve dividir 2 + 3 + a + 1 + 9 + 9 + 2 + b = 26 + a + b.

i) Para b = 0, temos que 26 + a deve ser múltiplo de 9, daı́ a = 1 é a única solução no conjunto indicado.

ii) Para b = 5, temos que 31 + a deve ser múltiplo de 9, daı́ a = 5 é a única solução no conjunto indicado.

Por fim, para a = 1 e b = 0 temos (1 − 0)2 = 1 e, para a = 5 e b = 5, temos (5 − 5)2 = 0. Os possı́veis valores
de ( a − b)2 são 0 e 1.

33. (Adaptado da Vı́deo Aula)


Podemos observar que 248 − 1 é igual a
 2
248 − 1 = 224 − 12
  
= 224 + 1 224 − 1
   
= 224 + 1 212 + 1 212 − 1
  
= 224 + 1 212 + 1 26 + 1 26 − 1
 
  
= 224 + 1 212 + 1 · 65 · 63.

Portanto, 65 | 248 − 1 e 63 | 248 − 1. 


34. Se 10x + y é divisı́vel por 7, então 10x + y − 7x − 7y também o é. Agora, temos que

10x + y − 7x − 7y = 3x − 6y,

logo 3x − 6y é divisı́vel por 7. Fatorando o 3, temos

3x − 6y = 3 · ( x − 2y).

Como 7 não divide 3, então 7 divide x − 2y. Para provar a volta, basta tomarmos as operações inversas em
cada passagem anterior de trás para frente.

31
POTI 2015 − Teoria dos Números − Nı́vel 2 − Aula 0 − Cleber Assis, Samuel Barbosa e Tiago Miranda

Observação: Vejamos agora alguns exemplos da aplicação do que foi demonstrado no exercı́cio 34.

a) Para demonstrar que o número 294 é divisı́vel por 7, basta tomarmos x = 29 e y = 4:

29 − 2 · 4 = 21.

Como 7 divide 21, então 7 divide 294.

b) Para verificar se 7 divide o número 248738, o método vai ser aplicado várias vezes, observe.

24873 − 2 · 8 = 24857
2485 − 2 · 7 = 2471
247 − 2 · 1 = 245
24 − 2 · 5 = 14.

Como 7 divide 14, então 7 divide 248738.

c) Usando o método para o número 7557, obtemos:

755 − 2 · 7 = 741
74 − 2 · 1 = 72
7 − 2 · 2 = 3.

Assim, como 7 não divide 3, 7 não divide 7557.


35. (Extraı́do da Vı́deo Aula)
Sabendo que os possı́veis restos numa divisão por 8 são {0, 1, 2, 3, 4, 5, 6, 7} e como o resto é o dobro do
quociente, então só trabalharemos com os valores pares, ficando com os possı́veis restos: 0,2,4 e 6. Seja n o
valor procurado. Simbolicamente teremos:
n 8
2q q
com 0 ≤ 2q < 8. Portanto, 0 ≤ q < 4. Além disso, como

n = 8 · q + 2q = 10q,

basta considerarmos os seguintes casos para o valor do resto:

i) se o resto for zero, o quociente será 0 e n = 0;

ii) se o resto for dois, o quociente será 1 e n = 8 · 1 + 2 = 10;

iii) se o resto for quatro, o quociente será 2 e n = 8 · 2 + 4 = 20; e

iv) se o resto for seis, o quociente será 3 e n = 8 · 3 + 6 = 30.

Portanto, os números são 0, 10, 20 e 30.


36. (Extraı́do da Vı́deo Aula)
Os possı́veis restos numa divisão por 7 são os elementos do conjunto {0, 1, 2, 3, 4, 5, 6}. O maior resto possı́vel
é o 6 e assim queremos descobrir n ∈ N tal que

n 7
6 4

A partir do diagrama anterior, n = 7 · 4 + 6 = 34.

32
POTI 2015 − Teoria dos Números − Nı́vel 2 − Aula 0 − Cleber Assis, Samuel Barbosa e Tiago Miranda

37. Se 10x + y é divisı́vel por 13, então 10x + y − 13x − 13y também o é. Agora, temos que
10x + y − 13x − 13y = −3x − 12y = −3( x + 4y),
e como 13 não divide −3, então 13 divide x + 4y. Para provar o caminho de volta, basta tomarmos as operações
inversas em cada passagem anterior de trás para frente. 
Observação: Vejamos alguns exemplos da aplicação do que foi demonstrado no exercı́cio 37.
a) O número 1001 é divisı́vel por 13, para provar isso tome x = 100 e y = 1 e aplique o exercı́cio anterior:
1001 → 100 + 4 · 1 = 104.
Agora, faça x 0 = 10 e y0 = 4, obtendo
104 → 10 + 4 · 4 = 26.
Como 13 divide 26, então 13 divide 1001.
b) Façamos o mesmo para verificar se 13 divide 2464085:
246408 + 4 · 5 = 246428
24642 + 4 · 8 = 24674
2467 + 4 · 4 = 2483
248 + 4 · 3 = 260
26 + 4 · 0 = 26.
Como 13 divide 26, então 13 divide 2464085.
38. Seja n esse número, logo existem a e b inteiros tais que n = 9a + 3 e n = 11b + 4. Ou seja,
9a + 3 = 11b + 4
9a = 11b + 4 − 3
9a = 11b + 1.
Daı́, 9 divide 11b + 1. Substituindo os valores de b do conjunto {0, 1, 2, . . . , 8}, podemos concluir que o menor
b natural que satisfaz essa afirmação é b = 4. Portanto, a = 5 e n = 48.
39. Podemos escrever k = 7q + 4 para algum q inteiro.
a) Sendo assim,
k2 + k + 1 = (7q + 4)2 + (7q + 4) + 1
= 49q2 + 56q + 16 + 7q + 4 + 1
= 49q2 + 63q + 21
= 7(7q2 + 9q + 3).
Portanto, k2 + k + 1 é múltiplo de 7, ou seja, deixa resto 0 em sua divisão por 7.
b) Seja nk um múltiplo de k que, somado a k2 produz um múltiplo de 7, assim temos
k2 + nk = (7q + 4)2 + n(7q + 4)
= 49q2 + 56q + 16 + 7nq + 4n
= 49q2 + 56q + 7nq + 14 + 4n + 2
= 7(7q2 + 8q + nq + 2) + 4n + 2.
Agora, precisamos encontrar o menor inteiro n tal que 4n + 2 seja múltiplo de 7. Substituindo os valores de
n do conjunto {0, 1, 2, . . . 6}, o menor valor será n = 3 e daı́ teremos 4n + 2 = 14. O menor múltiplo será 3k.

33
POTI 2015 − Teoria dos Números − Nı́vel 2 − Aula 0 − Cleber Assis, Samuel Barbosa e Tiago Miranda

40. Observe que


n = 7a + 4 e
n = 8b + 6,
para a e b inteiros. Multiplicando a primeira por 8 e a segunda por 7, obteremos
8n = 56a + 32 e
7n = 56b + 42.
Subtraindo-as, chegamos a
8n − 7n = 56( a − b) − 10
n = 56( a − b) − 10
= 56( a − b) − 10 + 56 − 56
= 56( a − b) − 56 + 46
= 56( a − b − 1) + 46
Ou seja, n deixa resto 46 quando dividido por 56.
41. Sendo n = 2a · 3b , sua quantidade de divisores será ( a + 1)(b + 1) = 15. Daı́ teremos quatro situações, a
saber:
i) a + 1 = 1 e b + 1 = 15, resultando em n = 314 ;
ii) a + 1 = 3 e b + 1 = 5, resultando em n = 22 · 34 = 324;
iii) a + 1 = 5 e b + 1 = 3, resultando em n = 24 · 32 = 144; ou
iv) a + 1 = 15 e b + 1 = 1, resultando em n = 214 .
42. Para um número ser divisı́vel por 91, ele deve ser divisı́vel ao mesmo tempo por 13 e 7. Um método
prático para a divisão por 7 foi demonstrado no exercı́cio 34.

Vamos aplicá-lo agora: 12345078 + 4 · 9 = 12345114


1234511 + 4 · 4 = 1234527
12345n78 − 2 · 9 = 12345n60 123452 + 4 · 7 = 123480
12345n6 − 2 · 0 = 12345n6 12348 + 4 · 0 = 12348
12345n − 2 · 6 = 123450 + n − 12 1234 + 4 · 8 = 1266
= 123438 + n. 126 + 4 · 6 = 150
15 + 4 · 0 = 15.
Seguindo o mesmo método para descobrir qual o resto Como 13 não divide 15, ele não dividirá 123450789.
da divisão de 123438 por 7. Agora, para n = 7, ficaremos com o número 123457789
e obteremos
12343 − 2 · 8 = 12327 12345778 + 4 · 9 = 12345814
1232 − 2 · 7 = 1218 1234581 + 4 · 4 = 1234597
121 − 2 · 8 = 105 123459 + 4 · 7 = 123487
10 − 2 · 5 = 0. 12348 + 4 · 7 = 12376
1237 + 4 · 6 = 1261
Logo, 123438 é divisı́vel por 7 e n pode ser 0 ou 7. 126 + 4 · 1 = 130,
Agora, vamos para o método do 13 (visto no exercı́cio
37) utilizando n = 0 e depois n = 7. Para n = 0, Como 13 divide 130, ele dividirá 123457789. Final-
teremos mente, ficamos com n = 7.

34
POTI 2015 − Teoria dos Números − Nı́vel 2 − Aula 0 − Cleber Assis, Samuel Barbosa e Tiago Miranda

43. Para analisar os restos de x2 por 4, podemos analisar os possı́veis restos de x por 4.
i) Se x = 4k, então x2 = 4(4k2 ), ou seja, x2 deixa resto 0 na divisão por quatro;
ii) se x = 4k + 1, então x2 = 4(4k2 + 2k ) + 1, ou seja, x2 deixa resto 1 na divisão por quatro;
iii) se x = 4k + 2, então x2 = 4(4k2 + 4k + 1), ou seja, x2 deixa resto 0 na divisão por quatro; e
iv) se x = 4k + 3, então x2 = 4(4k2 + 6k + 2) + 1, ou seja, x2 deixa resto 1 na divisão por quatro.
Apenas os restos 0 e 1 são possı́veis.

43. Outra solução: Se x é ı́mpar, x = 2k + 1 e temos que x2 = 4k (k + 1) + 1 deixa resto 1 na divisão por 4. Se
x é par, x = 2k e temos que x2 = 4k2 deixa resto 0 na divisão por. 4
44. (Extraı́do da Vı́deo Aula)
Se 6 divide a + b + c, então existe um inteiro k tal que a + b + c = 6k. Logo a + b = 6k − c e, elevando ambos
os membros ao cubo, chegamos a
a + b = 6k − c
( a + b)3 = (6k − c)3
a3 + 3a2 b + 3ab2 + b3 = 216k3 − 108k2 c + 6kc2 − c3 .
Observe que 216k3 − 108k2 c + 6kc2 é múltiplo de 6, então para algum m inteiro teremos 216k3 − 108k2 c + 6kc2 =
6m. Voltando à equação encontrada e substituindo o valor acima por 6m obtemos
a3 + b3 + c3 = 6m − 3ab( a + b).
Observe agora que se a ou b forem pares, 3ab é divisı́vel por 6 e que se ambos forem ı́mpares então ( a + b) é
par e 3ab( a + b) é divisı́vel por 6. Logo, em qualquer caso, 6 divide a3 + b3 + c3 . 
45. Como x deixa resto 1 nas divisões por 2, 3, 5 e 7, então x − 1 é divisı́vel por esses números e, portanto, é
um múltiplo comum deles. Sendo assim, existe algum k inteiro tal que x − 1 = 2 · 3 · 5 · 7 · k, ou seja,
x = 210k + 1.
Como x tem três algarismos, temos
100 ≤ ≤ 999
x
100 ≤210k + 1 ≤ 999
99 998
≤ k ≤ .
210 210
Portanto o maior k é 4 e, para tal valor, x = 841. A soma dos algarismos procurada é 8 + 4 + 1 = 13.
46. Seja M17 o conjunto dos múltiplos de 17 menores que 400, daı́
M17 = {17, 34, 51, · · · , 374, 391}.
Como 17 = 1 · 17 e 391 = 23 · 17, então
| M17 | = 23.
Seja M23 o conjunto dos múltiplos de 23 menores que 400, daı́
M23 = {23, 46, · · · , 368, 391}.
Como 23 = 1 · 23 e 391 = 17 · 23, então
| M23 | = 17.
Os dois conjuntos anteriores possuem um elemento em comum, a saber, M17 ∩ M23 = {391}. Por fim, há
23 + 17 − 1 = 39
múltiplos de 17 ou 23 e menores que 400.

35
POTI 2015 − Teoria dos Números − Nı́vel 2 − Aula 0 − Cleber Assis, Samuel Barbosa e Tiago Miranda

47. Para m = 3 e n = 1, m2 − n2 = 8. Portanto, o maior inteiro procurado é menor ou igual à 8. Sendo m e n


ı́mpares, existem x e y inteiros tais que m = 2x + 1 e n = 2y + 1. Substituindo e desenvolvendo a expressão
dada, encontraremos que

m2 − n2 = (2x + 1)2 − (2y + 1)2


= 4x2 + 4x + 1 − 4y2 − 4y − 1
= 4( x ( x + 1) − y(y + 1)).

Concluindo assim que m2 − n2 é múltiplo de 4. Agora, como x ( x + 1) e y(y + 1) são números pares, teremos
que sua subtração será par, ou seja, existe t inteiro tal que

x ( x + 1) − y(y + 1) = 2t

e que m2 − n2 = 4 · 2t = 8t. Isso nos permite concluir que o maior inteiro que divide m2 − n2 , quaisquer que
sejam m e n inteiros ı́mpares é o 8.

48. Se esse número é divisı́vel por 2 e 5, ele é divisı́vel por 10. Assim, o algarismo das unidades como 0.
Como ele é divisı́vel por 9, a soma dos seus dı́gitos é um múltiplo de 9. Então, o número será do tipo

M C D U
8 x y 0

com 8 + x + y divisı́vel por 9. Daı́ teremos dois casos:

i) o primeiro será quando x + y = 1, com duas opções, (1, 0) e (0, 1); e

ii) o segundo será quando x + y = 10, com nove opções, (1, 9), (2, 8), (3, 7), (4, 6), (5, 5), (6, 4), (7, 3), (8, 2) e
(9, 1).
Não é possı́vel que x + y seja 19, pois como x e y são dı́gitos, eles valem no máximo 9. Por fim, ficamos com
11 números.
49. (Adaptado da Olimpı́ada de Matemática do Canadá)
Observe inicialmente que 23 é primo. Agora, cada vez que o 23 ou um dos seus múltiplos aparece na expansão
de 2000!, a potência de 23 que o divide aumenta em uma unidade, no caso dos múltiplos de 232 , essa potência
aumenta uma unidade adicional. Não devemos considerar na análise o 233 , pois 233 > 2000. Por fim, no
primeiro caso existem 86 múltiplos de 23 menores que 2000 e no segundo apenas 3. Logo, 2386+3 divide 2000!
e x = 89.
50. (Extraı́do da Vı́deo Aula.)
Suponha, por absurdo, que x e y são ı́mpares. Portanto, existem a e b inteiros tais que x = 2a + 1 e y = 2b + 1.
Daı́, substituindo na equação dada, obtemos

x 2 + y2 = z2
(2a + 1)2 + (2b + 1)2 = z2
4a2 + 4a + 1 + 4b2 + 4b + 1 = z2
4( a2 + a + b2 + b ) + 2 = z2 .

Chegamos a um quadrado perfeito que deixa resto 2 numa divisão por quatro. Isso contradiz o exercı́cio 43.
Logo x e y não podem ser ambos ı́mpares. 

36
POTI 2015 − Teoria dos Números − Nı́vel 2 − Aula 0 − Cleber Assis, Samuel Barbosa e Tiago Miranda

51. Podemos chamar 11111 = n e substituir na expressão do enunciado ficando com


√ √
1111111111 − 22222 = 1111100000 + 11111 − 2 · 11111
p
= n · 105 + n − 2n
q
= n(105 − 1)
q
= n(99999)

= n · 9 · 11111

= n·9·n

= 9n2
= 3n
= 33333.

Esse número, quando dividido por 9, deixa resto 6.

52. (Extraı́do exame de acesso do Colégio Naval.)


O número em questão será
N = ZYXZYXZYX,
com 0 ≤ X, Y, Z ≤ 9 e Z 6= 0. Logo, podemos escrever N como
Z · 108 + Y · 107 + X · 106 + Z · 105 + Y · 104 + X · 103 + Z · 102 + Y · 101 + X.

E agrupando os termos semelhantes teremos


Z · 102 · (106 + 103 + 1) + Y · 10 · (106 + 103 + 1) + X · (106 + 103 + 1).

O que nos permite conluir que 106 + 103 + 1 = 1001001 divide N. Além disso, 1001001 = 3 · 333667 e,
portanto, 333667 divide N. Esse número está na letra d.

53. (Extraı́do exame de acesso do Colégio Naval.)


Se 2a + b é divisı́vel por 13, então podemos escrever, para algum k inteiro, que 2a + b = 13k. Agora, se
somarmos qualquer múltiplo de 13, o resultado continuará dessa forma. Somando 13 · 7a = 91a obteremos

2a + b + 91a = 93a + b

como outro múltiplo de 13, o que está na letra c.

54. (Extraı́do exame de acesso do Colégio Naval.)


Observe que para algum k ∈ Z temos que

12 · 11 · 10 · · · · · 3 · 2 · 1 + 14 = 13k
12 · 11 · 10 · · · · · 3 · 2 · 1 = 13k − 14
13 · 12 · 11 · 10 · · · · · 3 · 2 · 1 = 13 · (13k − 14)
13 · 12 · 11 · 10 · · · · · 3 · 2 · 1 = 169k − 182
= 169k − 169 − 13
= 169(k − 1) − 13
= 169(k − 1) − 13 + 169 − 169
= 169(k − 2) + 156.

Logo, o resto da divisão de 13 · 12 · 11 · · · · · 3 · 2 · 1 por 169 é 156.

37
POTI 2015 − Teoria dos Números − Nı́vel 2 − Aula 0 − Cleber Assis, Samuel Barbosa e Tiago Miranda

55. Um número é divisı́vel por 9 quando a soma dos seus dı́gitos é um múltiplo de nove. Agora, como
10111213141516 . . . 979899 é composto de
• 10 algarismos uns nas “dezenas” e 9 uns nas “unidades”.
• 10 algarismos dois nas “dezenas” e 9 dois nas “unidades”.
.
• ..
• 10 algarismos noves nas “dezenas” e 9 noves nas “unidades”.
A soma dos dı́gitos será
(1 + 9) · 9
 
19 · 1 + 19 · 2 + · · · + 19 · 9 = 19 · = 19 · 45.
2
Portanto, esse número é múltiplo de 9 e sua divisão então resto 0.
56.

a) 243. b) 4 + 9 = 13. c) 625. d) 8 + 27 = 35. e) 24−1 · 3 = 24.


57.

a) 0, 000001. 125 256


c) 80 · = 1250. e) 200 · = 5, 12.
8 10000
1
b) 4. d) · 0, 09 = 0, 03.
3
58.
8 b) 8. c) 72. d) 324. e) 52 − 22 = 21.
a) .
3
59.
 10
2 b) 26 c) −245 . d) 106 . e) 213 .
a) .
3
60.
a) Verdadeiro.
1
b) Falso. Por exemplo, 2−1 = 6= −2.
2
 2
2
c) Falso. Por exemplo, = 4 6= (2 − 1)2 = 1.
1
d) Verdadeiro.
3
e) Falso. Por exemplo, (22 )3 = 64 6= 256 = 2(2 ) .
61.
a) Verdadeiro.
b) Falso. Por exemplo, (1 + 2)3 = 27 6= 9 = 13 + 23 .
c) Falso. Por exemplo, 22+1 = 8 6= 5 = 22 + 21 .
1
d) Falso. Por exemplo, (22 )−2 = 6= 1 = 20 .
16
e) Verdadeiro.

38
POTI 2015 − Teoria dos Números − Nı́vel 2 − Aula 0 − Cleber Assis, Samuel Barbosa e Tiago Miranda

62.

a) 0, 09. 100 c) −0, 000008. 1 e) 1, 728.


b) . d) .
9 9
63.

a) 24 = 16. b) 224 . c) 1. d) 2−18 . e) 24 .


64.
a) Como 23 > 22 , segue que 21/2 = (23 )1/6 > (22 )1/6 = 21/3 .
1 1
b) Pelo item anterior, 21/2 > 21/3 e consequentemente < 1/3 .
21/2 2
c) Como 33 < 55 , segue que 31/5 = (33 )1/15 < (55 )1/15 = 51/3 .
2 2 −4
65. 44 : 44 = 44 = 412 . Resposta E.
66. Fazendo o desenvolvimento segundo a regra definida no enunciado chegaremos a:

2 ⊗ (2 ⊗ (2 ⊗ 2)) 2 ⊗ (2 ⊗ 4)
=
((2 ⊗ 2) ⊗ 2) ⊗ 2 (4 ⊗ 2) ⊗ 2
2 ⊗ 16
=
16 ⊗ 2
216
=
162
= 28 .

Resposta E.

67. Como 2 ∗ x = 2x + x2 e x é inteiro, devemos ter x2 ∈ {12 , 22 , . . . , 102 }. Dentre os elementos listados, o
único possı́vel para o qual 100 − x2 é uma potência de 2 é x2 = 36 pois nesse caso x = 6 e 100 − x2 = 64 = 26 .
Consequentemente (4x )4 = 256x4 = 256 · 1296 = 331776. Resposta E.

68. Utilizando as propriedades de potências teremos que:

156 · 285 · 557 = (36 · 56 ) · (210 · 75 ) · (57 · 117 )


= 36 · 117 · 53 · 1010

Logo, o número termina em 10 zeros. Resposta A.

69. Representemos os dı́gitos desconhecidos de 2n e 5n com asteriscos. Se k e l são as quantidades de


algarismos de cada um deles, temos:

d · 10k < d ∗ ∗ ∗ . . . ∗ = 2n < (d + 1) · 10k


d · 10l < d ∗ ∗ ∗ . . . ∗ = 5n < (d + 1) · 10l

Multiplicando ambas as inequações, obtemos 10k+l · d2 < 10n < 10k+l · (d + 1)2 . Cancelando 10k+l em ambos
os lados, concluı́mos que existe uma potência de 10 entre d2 e (d + 1)2 . Analisando os quadrados dos dı́gitos
de 1 até 9, percebemos que isso ocorre apenas para d = 3( 32 < 10 < 42 ).

70. a = 240 = 1610 , b = 320 = 910 e c = 710 . Como 16 > 9 > 7, temos a > b > c. Resposta A.

71. 1212 = 126 = (22 · 3)6 = 212 · 36 . Resposta C.

39
POTI 2015 − Teoria dos Números − Nı́vel 2 − Aula 0 − Cleber Assis, Samuel Barbosa e Tiago Miranda

72.

64 = 2(22x ) − 4x
= 2 · 22x − 22x
= 22x .

Como 64 = 26 , temos 2x = 6 e x = 3. Resposta E.

73.

a) 4, 5673 · 104 . b) 1, 2345 · 10−3 . c) −5, 55 · 102 . d) 9 · 10−2 .


74.

a) 342. b) 7. c) 98.
75.

a) b) c) d)

x = 0, 333 . . . x = 0, 121212 . . . x = 6, 555 . . . x = −0, 666 . . .


10x = 3, 333 . . . 100x = 12, 121212 . . . 10x = 65, 555 . . . 10x = −6, 666 . . .
9x = 3 99x = 12 9x = 59 9x = −6
3 1 12 4 59 6 2
Logo, x = = . Logo, x = = . Logo, x = . Logo, x = − =− .
9 3 99 33 9 9 3
76.

a) b) c)

x = 4, 7222 . . . x = 1, 8999 . . . x = 1, 2010101 . . .


10x = 47, 222 . . . 10x = 18, 999 . . . 10x = 12, 010101 . . .
100x = 472, 222 . . . 100x = 189, 999 . . . 1000x = 1201, 010101 . . .
90x = 425 90x = 171 990x = 1189
425 85 171 19 1189
Logo, x = = . Logo, x = = . Logo, x = .
90 18 90 10 990
77.

a) Decimal exato. Isso ocorre pois o denominador só possui fatores primos 2 e 5.
15 5
b) Decimal exato. Isso ocorre pois = e o denominador só possui fator 2.
6 2
c) Dı́zima periódica. Trata-se de uma fração irredutı́vel com um fator primo no denominador que não é 2 e
41
nem 5. De fato, = 2, 7333 . . ..
15
d) Decimal exato. Isso ocorre pois o denominador só possui fatores primos 2 e 5.

78. Resposta B.

79. Resposta B.

40
POTI 2015 − Teoria dos Números − Nı́vel 2 − Aula 0 − Cleber Assis, Samuel Barbosa e Tiago Miranda

80.
a) 0.
b) Um grupo de k zeros é separado de um grupo seguinte de k + 1 zeros por exatamente um número 1. Assim,
contando até o dı́gito 1 que sucede um grupo de k zeros, temos:
k ( k + 3)
1| + 2 + 3{z+ . . . + }k + k = .
|{z} 2
algarismos zeros algarismos uns

30(33)
Se k = 30, já teremos = 495. Consequentemente a 500a casa decimal vale zero pois está no grupo
2
com 31 zeros.
c) O número X não é racional porque sua representação decimal não é periódica uma vez que a quantidade
de algarismos zeros entre dois 1’s consecutivos sempre está aumentando.
212
81. Multiplicando a fração inicial por teremos:
212
1 1 212
= ·
512 512 212
212
=
1012
Como 212 = 4096, o primeiro dı́gito não nulo após a vı́rgula é 4. Resposta C.
82. Veja que
s s
 3  3
1 1 6
· (0, 666 . . .) = ·
6 6 9
r
1
=
62 · 9
1
=
18
Além disso,
s
 0 r
2 1 1
− = 1−
3 1, 333 . . . 12/9
r
9
= 1−
12
r
3
=
12
1
=
2
Assim, o valor da expressão procurada é:
−1/2
10 −1/2
  
1 1
+ =
18 2 18
3
= √
5

3 5
=
5
Resposta E

41
POTI 2015 − Teoria dos Números − Nı́vel 2 − Aula 0 − Cleber Assis, Samuel Barbosa e Tiago Miranda

83. Um valor possı́vel para p é 17 pois:


1
= 0, 05882352994117647.
17
Todos os primos menores que 100 que satisfazem essa propriedade são:

7, 17, 19, 23, 29, 47, 59, 61, 97.


Comentário para professores: Seja p um número primo que não divide 10 e seja n um inteiro com 0 < n < p.
Se d é o menor inteiro positivo tal que 10d − 1 é múltiplo de p, é possı́vel mostrar que o perı́odo da representação
n
decimal de é exatamente d. No exemplo anterior, como 7 não divide 101 − 1, 102 − 1, . . . , 105 − 1 e divide
p
6
10 − 1, temos d = 6.
84. Podemos escrever 10n + 1 = p · a onde a é um número com não mais que n dı́gitos na base 10, digamos
a = a1 a2 . . . an . Queremos dizer com isso que cada número ai é um dos dı́gitos de a. Mesmo que ele possua
estritamente menos que n dı́gitos, podemos colocar alguns ai ’s da esquerda como sendo 0. Temos
1 a
=
p a·p
a
= n
10 + 1
a(10n − 1)
=
102n − 1
[10n ( a − 1) + (10n − 1) − ( a − 1)]
=
102n − 1
O número 10n − 1 é constituı́do por n números iguais a 9 e a diferença (10n − 1) − ( a − 1) reduz cada um
desses dı́gitos 9 por um dı́gito de a. Assim, a representação decimal do numerador é:

a1 a2 . . . an−1 ( an − 1)(9 − a1 )(9 − a2 ) . . . (9 − an−1 )(10 − an ).


1
O numero anterior representa o perı́odo da representação de e cada dı́gito i pode ser pareado com um
p
outro dı́gito da forma 9 − i. Assim, as quantidades de aparições de tais dı́gitos são iguais. No exemplo do
enunciado, o perı́odo de 1/7 é 142857 e temos os seguintes pareamentos:

1 → 8

4 → 5

2 → 7

85. Como 102k − 1 = (10k − 1)(10k + 1) e p é primo, um dentre 10k − 1 e 10k + 1 é múltiplo de p. Não
1 (10k − 1)/p
podemos ter 10k − 1 múltiplo de p pois caso contrário poderı́amos escrever = e obterı́amos
p 10k − 1
uma dı́zima periódica com perı́odo menor do que 2k. Sendo assim, p divide 10k + 1 e podemos usar repetir a
solução anterior para concluir que o perı́odo da representação decimal de 1/p é da forma:

a1 a2 . . . ak−1 ( ak − 1)(9 − a1 )(9 − a2 ) . . . (9 − ak−1 )(10 − ak ).

Somando o número formado pelos k primeiros dı́gitos com o número formado pelos k últimos, obtemos

99 . . . 9} = 10k − 1.
| {z
k vezes

42
POTI 2015 − Teoria dos Números − Nı́vel 2 − Aula 0 − Cleber Assis, Samuel Barbosa e Tiago Miranda

86. Números
√ irracionais são aqueles que possuem representação decimal infinita e não periódica. Sendo
assim, 2 ∈ Q e 0, 01001000100001 . . . ∈ Q0 pois possuem representações decimais não periódicas; ao passo
0

que 23 ∈ N ⊂ Q, 5, 345 ∈ Q, 31 ∈ Q, 2, 313131... ∈ Q possuem representações decimais periódicas.

√ professores: Pode ser difı́cil convencer o aluno em um primeiro contato com os números
Comentário para
irracionais que 2 é irracional e consequentemente nos primeiros exercı́cios o aluno deverá assumir tal fato.
Deixamos a demonstração desta afirmação para o final deste bloco de exercı́cios e sugerimos que o professor
faça o mesmo até seus alunos terem mais familiaridade com as distinções entre os conjuntos numéricos.

87. Já sabemos que valem as inclusões N ⊂ Z ⊂ Q ⊂ R. Assim:

a) N ⊂ Q.Verdadeira! d) r ∈ Q ⇒ −r ∈ Q. Verdadeira!

b) Z ⊂ Q.Verdadeira! 35
e) ∈ Q − Z. Falsa, pois Q − Z é o conjunto das
5
c) 1 ∈ Q − Z.Falsa, pois Q − Z é o conjunto das frações não inteiras e 35
5 = 7.
frações não inteiras.
88. Uma representação seria:

89. Resposta com o uso da calculadora.

a) 1, 42 = 1, 96. b) 1, 412 = 1, 9881. c) 1, 4142 = 1, 999396. d) 1, 41422 = 1, 99996164.


90. Resposta com o uso da calculadora.

3 = 1, 7320508075688772935274463415059 . . .

a) 1, 72 = 2, 89. b) 1, 732 = 2, 9929. c) 1, 7322 = 2, 999824. d) 1, 73202 = 2, 999824.


91.
√ √ q q √ √
a) 3.2< c) 4
< 16 d) 0, 64 > 0, 1.
100 25 .
√ √ √ √
b) 81 < 121. e) n < n + 1 com n.
√ √ √
92. Observe que 9 = 3 < 11 < 16 = 4.
Agora tentemos descobrir a primeira casa decimal após a vı́rgula:

i 3, 12 = 9, 61. ii 3, 22 = 10, 24. iii 3, 32 = 10, 89. iv 3, 42 = 11, 56.


Logo, para apenas a descobrirmos a primeira casa decimal, basta observarmos que:

3, 32 < 11 < 3, 42
10, 89 < 11 < 11, 56,

Então a melhor aproximação com uma casa decimal será o 3, 3.



93. Observe que 11 com uma casa decimal foi aproximado para 3, 3. Agora para a casa do centésimo, basta
considerarmos os quadradados:

(3, 30)2 , (3, 31)2 , (3, 32)2 , . . . , (3, 39)2 , (3, 40)2 .

Repetindo o procedimento do exercı́cio anterior, a melhor aproximação será 3, 31.

43
POTI 2015 − Teoria dos Números − Nı́vel 2 − Aula 0 − Cleber Assis, Samuel Barbosa e Tiago Miranda
√ √ √ √ √ √
94. Como 8 < 9 = 3√ e 64 = 8 < 80 < 81 = 9. O primeiro inteiro positivo maior que 8 é 3 e o
último inteiro menor que 80 é 8. Sendo assim, teremos 6 inteiros positivos, a saber {3, 4, 5, 6, 7, 8}.
95. Temos: √ √
6 = 36 35 = 1225
√ √
< 37 < 1226
√ √
< 49 < 1296
= 7; = 36.

Assim,
√ podemos concluir que o primeiro inteiro positivo maior que 37 é 7 e o último inteiro positivo menor
que 1226 é o 35. Logo, teremos: 35 − 7 + 1 = 29 inteiros positivos compreendidos entre os números do
problema, a saber: {7, 8, 9, . . . , 34, 35}.
96. Os quadrados dos números são respectivamente: 99, 112, 125, 108 e 98. Destes, apenas o primeiro e o
último são menores que o quadrado de 10 que é 100. Assim, os três números do meio são maiores que 10.
Resposta C.
97. Dados dois racionais a e b, somando a aos dois lados da desigualdade, temos:
a < b
a+a < b+a
2a < a + b
a+b
a <
2

Repetindo o procedimento, agora com b, temos:


a < b
a+b < b+b
a + b < 2b
a+b
< b
2

a+b a+b
O que resulta em: a < < b Como também é um racional, isso mostra que existe um racional entre
2 2
a e b.
Comentário para professores: É bom enfatizar que se a construção acima for reiterada com os racionais a e
a+b a+b
(ou com e b) o aluno poderá mostrar que existe uma infinidade de racionais entre a e b. Outros
2 2
comentários comentários que poderiam instigar os alunos sobre a distribuição dos racionais e dos irracionais
na reta seria questioná-los se qualquer intervalo contém números racionais e irracionais.
98. (Extraı́do da UNICAMP)
Uma boa estratégia seria eliminar os radicais elevando ambos números a uma potência múltipla de 3 e 4. Veja
que:

( 3)12 = 34
3

= 81
> 64
= 43
√4
= ( 4)12
√ √ √
Portanto, como ( 3 3)12 > ( 4 4)12 , segue que 3 3 é o maior.

44
POTI 2015 − Teoria dos Números − Nı́vel 2 − Aula 0 − Cleber Assis, Samuel Barbosa e Tiago Miranda

99. (Extraı́do do Colégio Naval)


Façamos uma primeira estimativa: Finalmente, somando as duas últimas desiguldades
1<4<8 obtidas, temos:
13 <√4 < 2√
3 √ √

3 3
3< 3√ 4+ 3 √16 < 5
1 < √4 < 3 8 < + 3
+ 3

1< 34<2
√ 4 p1 √4
3
√16 < 6√
4 < 1 + 4 + 3 16 < 6
Segunda estimativa:
Portanto, n = 4.
8 < 16 < 27
2 3 < 16 < 33
√ √
3

3
8< √ 16 < 3 27
2 < 3 16 < 3

100. Vamos supor que é possı́vel termos uma fração irredutı́vel m
n,m ∈ Z, n ∈ Z∗ tal que 2= m
n. Neste caso,
podemos escrever:
√ m
2 =
n
√ 2  m 2
( 2) =
n
m2
2 =
n2
2n = m2
2

Agora temos a seguinte situação, o membro da esquerda é par, portanto o da direita também o será. Contudo,
não podemos ter m2 par, se m também não for par. Sendo assim, m = 2k, para algum k ∈ Z, e

m = 2k
m2 = 4k2

Agora, voltando à equação 2n2 = m2 e substituindo o m2 pelo 4k2 , e ficamos com:

2n2 = m2
2n2 = 4k2
n2 = 2m2 .

Pelo argumento anterior, n é par, isso contradiz nossa suposição inicial pois tı́nhamos √ assumido que a fração
m
n era irredutı́vel. Essa contradição mostra que a suposição inicial é falsa, ou seja, 2 não é racional.

Comentário para professores: Este é um exemplo clássico de prova por absurdo. Quando mencionado em
sala de aula, sugerimos que o professor comente exemplos cotidianos de afirmações que conduzem a absurdos
para que os alunos se sintam mais confortáveis com tal demonstração.

101. Utilize o mesmo argumento da questão anterior.


√ √ √2
102.Tome A = B = 2. Se o número
√ √2 é racional, o enunciado está satisfeito. Caso contrário, faça
√ √2 √ √ 2
A = 2 e B = 2. Assim, ab = ( 2 ) 2 = 2 servirá como exemplo.

45
POTI 2015 − Teoria dos Números − Nı́vel 2 − Aula 0 − Cleber Assis, Samuel Barbosa e Tiago Miranda
√ √2
Comentário para professores: Já existe uma demonstração de que 2 é de fato irracional.
√ Um exemplo
mais construtivo usando fatos que não são estudados no oitavo ano seria escolher A = 10 e B = log10 4. Daı́,
A B = 2 é um racional.
103. F5 = {0/1, 1/5, 1/4, 1/3, 2/5, 1/2, 3/5, 2/3, 3/4, 4/5, 1/1}.
104. Usando a propriedade dada no enunciado, temos 7a − 5b = ±1. Veja que 7a deve deixar resto 1 ou 6 na
divisão por 5. Dentre os valores possı́veis de a no conjunto {0, 1, 2, . . . , 7}, apenas 2 e 3 satisfazem tal condição.
2 5 3 2
Se a = 2, temos b = 3. Se a = 3, teremos b = 4. Entretanto, como < < , a fração procurada é .
3 7 4 3
105.
a) Basta usar três vezes o balde de 5 litros e, em seguida, retirar duas vezes lı́quido do tambor usando o balde
de 7 litros. Dessa forma, transportamos 3 × 5 − 2 × 7 = 1 litro.
√ √
b) A quantidade a que podemos transportar de um tambor para o outro é da forma k (2 − 2) + l ( 2) litros
onde k e l são inteiros indicando quantas vezes tiramos ou colocamos lı́quidos usando cada um dos baldes.
Se l − k 6= 0, podemos escrever:
√ √
a = k (2 − 2) + l 2

a − 2k = 2( l − k )
a − 2k √
= 2
l−k

Assim, o número 2 seria o quociente√de dois inteiros o que resultaria em um número racional. Sabemos
que isso não pode acontecer porque 2 é irracional. Falta analisarmos o que acontece quando l = k. A
equação se transforma em:
√ √
a = k (2 − 2) + l 2
√ √
= k (2 − 2) + k 2
= 2k.

Veja que 2k é par e assim não podemos levar um valor ı́mpar como a = 1. Em qualquer caso, não é possı́vel
colocar exatamente 1 litro usando os baldes com as capacidades dadas neste item.

106. (Extraı́do da prova da Cone Sul publicada na Revista Eureka número 5)


A fração ba é irredutı́vel se e só se b−a a é irredutı́vel ( se a e b tem um fator comum, então a e b − a têm um
fator comum, e reciprocamente). O problema se transforma em achar o menor valor de n tal que as frações
a
sejam todas irredutı́veis. Observe que as frações anteirores possuem a forma e pelo critério anterior
n+a+2
a
bastaria que fosse irredutı́vel. Tendo isso em mente, se n + 2 é um primo maior que 91, todas as frações
n+2
serão irredutı́veis. Assim, um valor possı́vel de n é 95 pois n + 2 = 97 é um número primo. Verifiquemos que
é o menor possı́vel.
20
(a) Se n + 2 < 97 e n + 2 é par, então n é par e há frações redutı́veis como, por exemplo, n +2 .

(b) Se 19 ≤ n + 2 ≤ 91, obviamente há uma fração redutı́vel.

(c) Se n + 2 < 19, então n + 2 tem um múltiplo entre 19 e 91 e, portanto, há uma fração redutı́vel.
31
(d) Se n + 2 = 93 = 3.31, então n +2 é redutı́vel.
19
(e) Se n + 2 = 95 = 5.19, então n +2 é redutı́vel.
Logo, o valor mı́nimo de n + 2 é 97, que corresponde a n = 95.

46
POTI 2015 − Teoria dos Números − Nı́vel 2 − Aula 0 − Cleber Assis, Samuel Barbosa e Tiago Miranda

107.
√ √
a) x 3 x. c) 3x2 y. e) (4a6 b4 )3/2 = 26 a18 b12 = 8a9 b6 .
√ √ √
b) 4. d) 2 2 + 9 2 = 11 2.
108.
√ √
9

4
2 3 a7 a3
a) . c) . e) .
3 a a
√ √ √
3

3
1 1 4
x 4
x 2 2 x2 2 x2
b) √ 4
= √
4
· √ = . d) √ =√ ·√ = .
x 3 x 3 4
x x 3
x 3
x 3 x2 x
109.

y7 3s3 a10 b5
a) . b) . c) . d) .
x8 t6 b8 a8
110.

1 2
 
1 3 1
3 − + +
2 4 4 4
=

1 2

3 3 3
3 − − −
3 2 9 2
1
=
−7/6
6
= − .
7
111.
2
3x2 y

9x4 y2
a3 b3 a6 b6
3 =
3xy2 27x3 y6


2a2 b2 8a6 b6
8x
= .
3y4

112.

a) 22 x4 . b) xy2 z1/2 c) x1/8


113. Veja que:

1 1 1 1
− k
+ = +
1+a 1 + ak 1 + 1/a k 1 + ak
ak 1
= k
+
1+a 1 + ak
= 1.

Assim, se agruparmos a primeira fração com a última, a segunda com a penúltima e assim sucessivamente;
sempre obteremos o número 1. A única fração que não fará parte de nenhum par é a do meio que vale
1 1 1 2001
0
= . Como a quantidade de pares é igual a n, a resposta é 1000 + = . Resposta D.
1+a 2 2 2

47
POTI 2015 − Teoria dos Números − Nı́vel 2 − Aula 0 − Cleber Assis, Samuel Barbosa e Tiago Miranda

114. Indiquemos com uma seta o último dı́gito de um número. Assim,

131 → 3 132 → 9 133 → 7 134 → 1

135 → 3 136 → 9 137 → 7 138 → 1

...

Como 134 termina em 1, sempre que multiplicarmos os números de uma linha por esse valor para obtermos
os números da próxima, o último dı́gito se manterá. Podemos então agrupar os número de 4 em 4 e obtermos
uma soma que termina em 3 + 9 + 7 + 1 → 0. Como 2007 = 501 · 4 + 3, teremos 501 grupos e sobrarão
números com os dı́gitos 3, 9 e 7 cuja soma terminará em 9. Resposta E.

115.
22007 + 22005 22005 (22 + 1)
 
· 2006 = · 2006
22006 + 22004 22004 (22 + 1)
= 2 · 2006
= 4012.

A soma dos dı́gitos de 4012 é 7. Resposta D.

116. O número 24 = 23 · 3 tem somente dois divisores cubos perfeitos: 1 e 8. Assim, se é possı́vel representar
24 na forma a2 b3 , então b = 1 ou b = 2 e, portanto, a2 = 24 ou a2 = 3, o que é impossı́vel. Além disso, na
alternativa a podemos tomar a = 3 e b = 2; na alternativa c, podemos tomar a = 24 e b = c = 1; na alternativa
d, podemos tomar a = 3, b = 1 e c = 2; e na alternativa e, podemos tomar a = 2, b = 3 e c = 1. Resposta B.

117. Seja

21000 · 1 · 2 · 3 · . . . · 1000
B =
21000 · 1 · 2 · 3 · . . . · 1000
21000 · 1 · 2 · 3 · . . . · 1000
=
2 · 4 · 6 · . . . · 2000
Assim
21000 · 2000!
A·B =
2000!
= 21000

Como, B = 1, concluı́mos que A = 21000 . Resposta C.

Observação: Estamos escrevendo 2000! no lugar de 1 · 2 · 3 · . . . · 2000.

118.

a) 2n b) n/5 c) n + 1 d) (n + 3)/2 e) n3
119.

a) 2x · x = 2x2 . b) 2x · ( x + 1) = 2x2 + 2x. c) ( x − 1)( x + 2) = x2 + x − 2.

48
POTI 2015 − Teoria dos Números − Nı́vel 2 − Aula 0 − Cleber Assis, Samuel Barbosa e Tiago Miranda

120.

a) 8. b) 3. c) 11. d) m + n + 2.
121.

a) 32x4 y3 . 8x3 y2 8 x3 
y2
b) = = 2x2 y2
4xy x y
4
122.

a) 4x2 + 4x − 7. b) 2x2 + 4x − 9. c) 3x4 + 4x3 − 5x2 + 4x − 8.


123.

a) a3 − 5a2 − 2a + 4. b) 9a2 b + 6a2 − 5ab − 3ab2 + b2 . a2


c) 0, 3a4 + 0, 4a3 + .
3
124.

a) 3 + 2, 5x. b) Para x = 9, teremos:

3 + 2, 5x = 3 + 22, 5
= R$25, 50.

125.

a) x2 + 3x + 2. d) x2 − 16. 3
g) x2 + 2x + .
4
b) x2 + 12x + 27. e) x2 + 10x + 25.
c) x2 + x − 6. f) x2 − 8x + 16.
126.

a) 1 b) −11 c) 5 d) −149 e) 5/8

Comentário para professores: É interessante instigar os alunos a calcularem em alguns exemplos numéricos
simples o resto da divisão de um polinômio em x por x + a e compararem com o valor obtido de tal polinômio
quando avaliado em − a. Isto poderá ajudá-los a intuir e a compreender melhor a demonstração do Teorema
dos Restos que segue:

Se P( X ) possui quociente Q( X ) e resto R( x ) ao ser dividido por x + a, então:

P( x ) = Q( x )( x + a) + R( x )
P(− a) = Q(− a)(− a + a) + R(− a)
P(− a) = R(− a)

Como R( x ) é um polinômio constante pois x + a possui grau 1, podemos concluir que R( x ) = P(− a).

127. As expressões das letras (b), (c) e (d) são divisı́veis pelos polinômios dados enquanto que as expressões
em (a) e (e) não o são.

49
POTI 2015 − Teoria dos Números − Nı́vel 2 − Aula 0 − Cleber Assis, Samuel Barbosa e Tiago Miranda

128.

a) 5x − 3y. b) 5 · 8 − 3 · 2 = 34.
129.

a) 50 − x, para 0 ≤ x ≤ 50. b) 150 − 3x, para 0 ≤ x ≤ 50.


130.

a) a4 . d)

(2a3 b2 )(3ab4 )3 = (2a3 b2 )(27a3 b12 )


b) 12y7 .
= 54a6 b14 .
c)
e)

(2x3 )2 (3x4 ) (4x6 )(3x4 )  3  2 4


x y x
 3  8 4
x y x
= = ·
( x 3 )4 x12 y z y3 z4
12x10 x 7 y5
= =
x12 z4
= 12x −2 .
131.

a) y7 /x8 b) 3s3 /t6 c) a10 /b8 d) b5 /a8


132.
2
3x2 y

9x4 y2
a3 b3 a6 b6
3 =
3xy2 27x3 y6


2a2 b2 8a6 b6
8x
= .
3y4
133.

a) 9. b) 2n + 3. c) 1 + 2 + 3 + 4 + . . . + 26 = 351.
134. Devemos ter 9 = (n + 1) + (2n − 1) = 3n. Portanto, n = 3.
135. Pela propriedade de distributividade:
(kx − 1)(2x + 1) = 2kx2 + kx − 2x − 1
= 2kx2 + x (k − 2) − 1.
A soma dos coeficientes é 2k + (k − 2) − 1 = 3k − 3. Tal soma vale 3 apenas quando k = 2.
136.

a) x2 − a2n . b) x2 + 2xa2n + a4n . c) x2 − 4xa + 4a2 .


137. Pelo Teorema dos Restos, todas as divisões anteriores são exatas. Além disso, como:
( x2 − a2 ) = ( x + a)( x − a)
( x2 + 2xa + a2 ) = ( x + a)( x + a)
( x3 + a3 ) = ( x + a)( x2 − ax + a2 ).
Os quocientes são: ( x + a), ( x + a) e x2 − ax + a2 , respectivamente.

50
POTI 2015 − Teoria dos Números − Nı́vel 2 − Aula 0 − Cleber Assis, Samuel Barbosa e Tiago Miranda

138.

a) x = −2 b) x = 4 c) x = 0.
139. Pelo Teorema dos Restos, basta que P(1), P(−1) e P(2) sejam nulos.

a) Calculando P(1) teremos b) Calculando P(−1) teremos c) Calculando P(2) teremos

P (2) = 8 − 8 + 6 + k
P (1) = 1 − 2 + 3 + k P(−1) = −1 − 2 − 3 + k
= 6+k
= 2+k = −6 + k = 0;
= 0; = 0;

Os valores de k devem ser −2, 6 e −6, respectivamente.

140.

a) (2500 − 4x2 )cm.

b) (2500 − 4 · 25) = 2400cm2 .

c) Será formado uma caixa sem tampa cujo fundo é um quadrado de 50 − 2xcm de lado e cujas alturas
medem xcm. Portanto, o volume de tal caixa é x (50 − 2x )2 .

141. (Adaptado do exame do EPCAR − 2012)


Temos:

( x2n+1 + x )( x2n+1 − x ) − ( x4 )(n+1/2)


=
( x n + x )2 − x2n − 2x n+1
( x4n+2 − x2n+2 + x2n+2 − x2 ) − x4n+2
=
x2n + 2x n+1 + x2 − x2n − 2x n+1
− x2
=
x2
−1
142. Pelo algoritmo da divisão, temos:

x3 + 6x2 − 2x + 4 x2 − 1

− x3 + x

6x2 − x + 4 x+6

−6x2 + 6

− x + 10

Assim, o quociente vale x + 6 e o resto − x + 10.

51
POTI 2015 − Teoria dos Números − Nı́vel 2 − Aula 0 − Cleber Assis, Samuel Barbosa e Tiago Miranda

143. (Adaptado do exame do EPCAR − 2014)

a) Seja x a mensalidade em 2012. Após o aumento de b) Basta substituirmos o valor de x e multiplicarmos


80%, o valor da mensalidade passou para: pela quantidade de meses do ano obtendo:
100%x + 80%x = 1, 8x.
1, 134 · 800 · 12 = R$10.886, 40.
A redução de 30% transformou a mensalidade em
1, 8x · 0, 7. Finalmente, com o desconto de 10%, esse
valor passou para:
x · 1, 8 · 0, 7 · 0, 9 = 1, 134x
144. (Adaptado do exame de acesso do Colégio Naval − 2011)

p √
3
a) 3
−(2 − 1)6 = −1 = −1. b) Como todo quadrado de um número real é não ne-
gativo,
p temos ( x − 1)6 ≥ 0. Assim, −( x − 1)6 ≤ 0
e −( x − 1)6 ≤ 0.
3

p
Como 3 −(1 − 1)6 = 0, em virtude da última desi-
gualdade, podemos concluir que o valor máximo
da expressão é 0.
145.

a) b)
    

1

1

1
 
1
 1 1 1
1+ 1+ 1+ ... 1+ = 1+ 1+ ... 1+ =
2 3 4 99 x+1 x+2 x + 98
3 4 5 100 x+2 x+3 x+4 x + 99
· · ·...· = · · ·...· =
2 3 4 99 x+1 x+2 x+3 x + 98
3 4 5 100 x+
2  x+
3  x+
 4 x + 99
· · ·...· = ·  ·  · . . . ·  =
2 3 4 99
 x+1  x+2  x+3 x+
 98
100 x + 99
= 50. .
2 x+1

146.

a) Seja b) Como no item anterior, considere um número auxi-


liar:
21000 · 1 · 2 · 3 · . . . · 1000
C = 2x · 1 · 2 · 3 · . . . · x
21000 · 1 · 2 · 3 · . . . · 1000 D =
2x · 1 · 2 · 3 · . . . · x
21000 · 1 · 2 · 3 · . . . · 1000 2x · 1 · 2 · 3 · . . . · x
= =
2 · 4 · 6 · . . . · 2000 2 · 4 · 6 · . . . · 2x
Assim, Assim,
21000 · 2000! 2x · (2x )!
A·C = B·D =
2000! (2x )!
= 21000 . x
= 2 .

Como, C = 1, concluı́mos que A = 21000 . Como, D = 1, concluı́mos que D = 2x .

Observação: Estamos escrevendo n! no lugar de 1 · 2 · 3 · . . . · n.

52
POTI 2015 − Teoria dos Números − Nı́vel 2 − Aula 0 − Cleber Assis, Samuel Barbosa e Tiago Miranda

147.

a) x2 + 2x + 1. b) 16 + 8x + x2 . c) x2 + 2 3x + 3. d) 9x2 + 6x + 1. e) 16x2 + 16x + 4.
148.

a) 4x2 + 12x + 9.

b) 4x2 + 12xy + 9y2 .

c) x4 + 6x2 + 9.

d) a4 + 6a2 b2 + 9b4 .

e) x8 + 18x4 + 81.
149.

a) Cálculo do valor de 132 . b) Cálculo do valor de 412 . c) Cálculo do valor de 192 .

192 = (20 − 1)2


132 = (10 + 3)2 412 = (40 + 1)2
= 400 − 40 + 1
= 100 + 60 + 9 = 1600 + 80 + 1
= 361.
= 169; = 1681;
150.

a) c)
√ √ √
( a + b)2 − 2 ab =
√ √ ( a + 1)2 + 2( a + 1) a + a2 + 2(2a + 1) + 1 =
a + 2 ab + b − 2 ab = (( a + 1) + a)2 + 2(2a + 1) + 12 =
a + b. (2a + 2)2 .
b)
( x + 1)2 + ( x − 1)2 =
( x2 + 2x + 1) + ( x2 − 2x + 1) =
2x2 + 2.
151.

a) a2 + 2ab + b2 . b) 4a2 − 4ab + b2 . c) 4a2 b2 + 12abc + 9c2 . d) 4a2 − 8ab + 4b2 .


152.

a) x2 − 1. d)
√ √ √ √
( x + y)( x − y)( x + y) =
b) 16 − a2 . ( x − y)( x + y) =
( x 2 − y2 )
c) x4 − 9z2 .
153.

a) 1002 − 12 = 9999. b) 20002 − 4 = 3999996. c) 102 − 52 + 52 = 100.

53
POTI 2015 − Teoria dos Números − Nı́vel 2 − Aula 0 − Cleber Assis, Samuel Barbosa e Tiago Miranda

154. Cada termo obtido após usarmos a distributividade teve um de seus membros vindo de alguma
letra entre os primeiros parênteses e o segundo vindo de alguma entre os segundos parênteses. Assim,
como temos duas possibilidade de escolhas em cada um deles, teremos no total 2 × 2 termos possı́veis na
múltiplicação. Isso pode também pode ser facilmente visualizado se momentaneamente colocarmos um ı́ndice
para distinguirmos de qual parêntese veio cada letra. Por exemplo:

( a1 + b1 )( a2 + b2 ) =
a1 a2 + a1 b2 + b1 a2 + b1 b2

155. Como temos três parênteses e em cada um deles temos duas escolhas, o número de termos é 2 × 2 × 2 = 8.
Para formarmos o termo a2 b, dois parênteses irão fornecer a letra “a“ e o outro a letra b. Uma vez escolhido
aquele que irá fornecer a letra ”b”, os demais estão determinados. Podemos fazer tal escoha de 3 formas e
assim existirão três termos a2 b. O mesmo argumento se aplica ao termo ab2 . A única maneira de formarmos
os termos a3 e b3 é escolhendo a mesma letra em todos os parênteses e isso só pode ser feito de uma forma.
Assim,
( a + b)3 = a3 + 3a2 b + 3ab2 + b3 .

156. O retângulo 6 × 7 desenhando abaixo foi dividido em duas figuras na forma de escada. Em cada
coluna, estamos escrevendo quantos quadrados foram pintados. Como as duas figuras são iguais, a soma dos
quadrados pintados - que corresponde ao termo 1 + 2 + 3 + 4 + 5 + 6 da equação -, deve ser igual à metade
da área do retângulo, ou seja,

Construindo um rentângulo n × (n + 1), é possı́vel


6·7 mostrar que:
1+2+3+4+5+6 = .
2
n ( n + 1)
1+2+3+...+n = .
2

157. Um exemplo seria:

Comentário para professores: O exemplo anterior é de Shirley Wakin e foi retirado do livro “Proofs without
words” escrito por Roger Nelsen. O leitor interessado poderá encontrar mais exemplos interessantes em tal fonte.

54
POTI 2015 − Teoria dos Números − Nı́vel 2 − Aula 0 − Cleber Assis, Samuel Barbosa e Tiago Miranda

158. Um exemplo seria:

Veja que área do quadrado maior de lado 9 é a soma das áreas das regiões destacadas e cada uma delas é da
forma 2n + 1 onde n é o lado do quadrado que a região contorna. É possı́vel construirmos quadrados cada
vez maiores e mostrarmos que a soma dos k primeiros inteiros positivos ı́mpares é igual à k2 .
159. Como todo quandrado perfeito é um número não negativo, se a e b representam as notas de um aluno,
temos:
√ √
( a − b )2 ≥ 0

a − 2 ab + b ≥ 0

a + b ≥ 2 ab
a+b √
≥ ab
2
Assim, é preferı́vel escolher a média aritmética porque ela é sempre maior ou igual à média geométrica.

Comentário: Provamos que se a e b são não negativos, então:


a+b √
≥ ab.
2
Isso é um caso particular do resultado mais geral de que a média aritmética de n números reais não negativos
é sempre maior ou igual à média geométrica de tais números.
160. Vamos usar novamente o fato de que todo quadrado é um número não negativo.

a) Sendo assim, teremos que c) Usaremos o item anterior três vezes:

( a − b )2 ≥ 0 
1 1

4 16
a2 − 2ab + b2 ≥ 0 + + + ≥
a b c d
a2 + 2ab + b2 ≥ 4ab
 
4 4 16
+ + ≥
( a + b )2 ≥ 4ab. a+b c d
 
16 16
b) Dividindo a expressão do item anterior por ab( a + + ≥
a+b+c d
b) obtemos: 64
.
1 1 a+b 4 a+b+c+d
+ = ≥ .
a b ab a+b
161. Sejam a e b as dimensões do retângulo, devemos ter que 2a + 2b = 2, ou seja, a + b = 1. A área obtida
será ab. Pelo exercı́cio anterior,
√ a+b 2 1
ab = ( ab)2 ≤ ( ) = .
2 4
1 1
Assim, a área máxima é . Podemos obtê-la construindo um quadrado de lado .
4 2

55
POTI 2015 − Teoria dos Números − Nı́vel 2 − Aula 0 − Cleber Assis, Samuel Barbosa e Tiago Miranda

162. Pela diferença de quadrados, temos:


s r s r
q √ q √
B = 2+ 2+ 2+ 3× 2− 2+ 2+ 3
r q √
= 2− 2+ 3

Apliquemos novamente a diferença de quadrados para obter o número:


r q √
C = 2+ 2+ 3·B
r r
q √ q √
= 2+ 2+ 3· 2− 2+ 3
q √
= 2− 3

Para terminar, veja que:


q √
A·B = 2+ 3·C
q √ q √
= 2+ 3· 2− 3
= 1
Resposta C
163. Se denotarmos por a = 20142012 o valor da expressão anterior pode ser escrito como:

( a + 1)2 − 2( a + 1) a + a2 = [( a + 1) − a]2
= 12 .
164. (Extraı́do da OBM 2014)
Usando a diferença de quadrados, podemos escrever:
√ √ √ √
( x + y)( x − y) = ( x − y).
Assim, obtemos:
√ √ x−y a2
x− y = √ √ =
x+ y b
√ √
x+ y = b
√ b2 + a2 √ b2 − a2
Resolvendo o sistema anterior, encontramos x= e y= . Assim,
2b 2b
√ b4 − a4
xy = .
4b2
165. Note que:

a(10 − a) = 10a − a2
= 25 − 25 + 10a − a2
= 25 − (5 − a)2

Como (5 − a)2 é sempre um número não negativo, a última expressão é no máximo 25. Tal valor é atingido
apenas quando (5 − a)2 = 0, ou seja, quando a = 5.

56
POTI 2015 − Teoria dos Números − Nı́vel 2 − Aula 0 − Cleber Assis, Samuel Barbosa e Tiago Miranda

166.

4a − a4 = 4a − 2a2 + 2a2 − a4
= 4a + 1 − 2a2 − 1 + 2a2 − a4
= 4a + 1 − 2a2 − ( a2 − 1)2
= 3 − 2 + 4a − 2a2 − ( a2 − 1)2
= 3 − 2( a − 1)2 − ( a2 − 1)2 .

Como 2( a − 1)2 + ( a2 − 1)2 é sempre um número não negativo por se tratar da soma de três quadrados, a
expressão anterior é no máximo 3. Veja que tal valor pode ser atingido quando a = 1.

167.

a) a(5 + b). b) a(m + n). c) x ( a + b + c). d) a( x + 1). e) b( a + c + ac).


168.
1 3 a a+b
a) . b) . c) . d) . e) x2 .
2 8 2 a−b
169.

a) ( a + b)( a + c). b) ( a − b)( x + y). c) (2a + 1)(b + 1). d) ( a − b)( x + 2). e) (5a − 1)(2b + 3).
170.

a) m2 . x2 + 1 m3 + 2
b) . c) .
x2 m+3
171.
√ √
a) ( a − 5b)( a + 5b). b) (2x − 1)(2x + 1). c) ( 7 − x )( 7 + d) ( ax − by)( ax + e) ( a − b)( a +
x ). by). b)( a2 + b2 ).
172.

a) A expressão não representa um bin√ômio perfeito. c) A expressão não representa um binômio


√ perfeito.
Se fosse b2 = 3, deverı́amos ter b = 3. Entretanto, Se fosse b2 = 18, deverı́amos ter b = 3 2. Entre-
−4 6= −2bx. tanto, 6 6= 2by.

d) 4z2 − 12zy + 9y2 = (2z − 3y)2 .


1 √ √
b) x2 + x + = ( x + 1/2)2 . e) 3z2 + 6z + 3 = ( 3z + 3)2 .
4
173.

a) ( x − 1)2 ( x + 1)2 . b) 5( a − 1)2 . c)

a2 − b2 − 2bc − c2 =
a2 − ( b + c )2 =
( a − (b + c))( a + (b + c)) =
( a − b − c)( a + b + c).

174.
xy xy
a) 20. b) . c) .
3 ( x + 1)(y + 1)

57
POTI 2015 − Teoria dos Números − Nı́vel 2 − Aula 0 − Cleber Assis, Samuel Barbosa e Tiago Miranda

175.

x2 y + y2 x = xy( x + y)
= 6·7
= 42.
176. (Extraı́do do vestibular da UNIVASF)
Temos
a+x c
=
b−x d

cb − xc = ad + xd.

Isolando os termos com x de um só lado e fatorando-o, obtemos: cb − ad = xc + xd = x (c + d), ou seja,


bc − ad
x= .
c+d
177.

a) b( a − b)( a + b). b) ( x − y − 3)( x − y + 3). c) ( a − 4)2 ( a + 4)2


178. Pela distributividade, temos:

( x − y)( x2 + xy + y2 ) =
( x3 + 
x2
y + 2
xy ) − (
yx2
+ 2
xy + y3 ) =
x 3 − y3

Usando a fatoração fornecida, temos:

x3 − 8 = ( x − 2)( x2 + 2x + 4).

179. Se y = −z, temos:

x 3 + z3 =
x3 + (−y)3 =
x 3 − y3 =
( x − y)( x2 + xy + y2 ) =
( x + z)( x2 − xz + z2 )

Obtemos assim uma fatoração para a soma dos cubos dada por:

x3 + z3 = ( x + z)( x2 − xz + z2 ).

180. Se x e y são esses números, temos:

x 3 + y3 =
( x + y)( x2 − xy + y2 ) =
( x + y)(( x + y)2 − 3xy) =
4 · ( 42 − 3 ) =
52

58
POTI 2015 − Teoria dos Números − Nı́vel 2 − Aula 0 − Cleber Assis, Samuel Barbosa e Tiago Miranda

181.

x 3 + y3 =
( x + y)( x2 − xy + y2 ) =
( x + y)(( x + y)2 − 3xy) =
3 · (32 − 3) = 18

182.
1
x2 + =
x2
 2
1 1
x+ −2·x· =
x x
22 − 2 = 2
183. (Extraı́do da Olimpı́ada Cearense)

( a − b)2 + (− a + b)2 + 2( a − b)(b − a) =


[( a − b) + (− a + b)]2 = 0.

184. (Extraı́do da AIME) Aplicando a diferença de quadrados nos dois primeiros parênteses e nos dois últimos,
temos:
√ √ √ √ √ √
( 5 + 6 + 7)( 5 + 6 − 7) =
√ √
(( 5 + 6)2 − 7) =

(4 + 2 30)
√ √ √ √ √ √
( 7 + 5 − 6)( 7 − 5 + 6) =
√ √
(7 − ( 5 − 6)2 ) =

(−4 + 2 30)

Assim, o produto é igual à:


√ √
(2 30 + 4)(2 30 − 4) =
4 · 30 − 16 = 104.
185.

x4 + 4 = x4 + 4x2 + 4 − 4x2
= ( x2 + 2)2 − 4x2
= ( x2 − 2x + 2)( x2 + 2x + 2).
186.

( n ( n + 3) + 1)2 = n2 (n + 3)2 + 2n(n + 3) + 1


= n(n + 3)[n(n + 3) + 2] + 1
= n(n + 3)[n2 + 3n + 2] + 1
= n(n + 3)[(n + 1)(n + 2)] + 1
= n(n + 1)(n + 2)(n + 3) + 1

187. Usando o exercı́cio anterior para n = 2014, obtemos (2014)(2017) + 1.

59
POTI 2015 − Teoria dos Números − Nı́vel 2 − Aula 0 − Cleber Assis, Samuel Barbosa e Tiago Miranda

188.

p4 − 1 = ( p2 − 1)( p2 + 1)
= ( p − 1)( p + 1)( p2 + 1)
189. (Extraı́da
p do vestibular da UFRJ)
√ p √
Seja y = 3 − 8 + 3 + 8. Claramente y é um inteiro positivo pois cada um dos radicais o é. Assim, o
produto xy possui o mesmo sinal de x. Calculemos tal produto usando diferença de quadrados:
√ √
xy = (3 − 8) − (3 + 8)

= −2 8.

Portanto, como − 8 é negativo, x também o é.
190.

n5 + n4 + 1 =
n5 + n4 + n3 − n3 − n2 − n + n2 + n + 1 =
n3 ( n2 + n + 1) − n ( n2 + n + 1) + ( n2 + n + 1) =
(n2 + n + 1)(n3 − n + 1).
191. (Extraı́do da Olimpı́ada Cearense)
Usando diferença de quadrados, temos:
√ √ n − ( n − 1)
n− n−1 = √ √
n+ n−1
1
= √ √
n+ n−1
Para que o número anterior seja menor que 0, 01, devemos ter:
√ √
n + n − 1 > 100.

Se n ≤ 502 ,
√ √ √
n+ n − 1 < 50 + 2499
< 100.
Se n = 502 + 1,
√ √ √
n+ n−1 = 2501 + 50
> 100.
Logo, o menor inteiro positivo que satisfaz a desigualdade do enunciado é n = 502 + 1.
192. Aplicando a diferença de quadrados sucessivamente, temos:

a32 − b32 =
( a16 + b16 )( a16 − b16 ) =
( a16 + b16 )( a8 + b8 )( a8 − b8 ) =
( a16 + b16 )( a8 + b8 )( a4 + b4 )( a4 − b4 ) =
( a16 + b16 )( a8 + b8 )( a4 + b4 )( a2 + b2 )( a2 − b2 ) =

Assim, o quociente é a2 − b2 .

60
POTI 2015 − Teoria dos Números − Nı́vel 2 − Aula 0 − Cleber Assis, Samuel Barbosa e Tiago Miranda

193. Note que (( x + 1)2 − ( x + 1) + 1) = ( x2 + x + 1) .

Verifiquemos agora uma fração genérica do produto: Primeiramente vejamos o que acontece quando multi-
plicarmos apenas as frações que constituem a primeira
x3 − 1 parte da expressão:
=
x3 + 1
2
( x − 1)( x + x + 1) 1 2 3 4 5 98
  99

· · · · ·...· · =

= 3 4 5 6 7 100 101
( x + 1)( x2 − x + 1)
2
x − 1 ( x + 1)2 − ( x + 1) + 1 .
· 100 · 101
x+1 x2 − x + 1
A segunda parte produz um cancelamento diferente:
A primeira parte da última expressão é uma fração
onde o numerador e o denominador diferem por 2 e a 32
−3

+1 42
−4+1

1012 − 101 + 1
segunda parte é um quociente de termos envonvendo

2
· 
· . . . · ( =
2 −2+1 
(
32 (2(

−3 + 1

100 −(100
(( +1
a expressão n2 − n + 1 quando n é x + 1 e x. Vamos (
10101
analisar a expressão anterior para cada valor de x no .
3
conjunto {2, 3, . . . , 100}.
Assim, o valor da expressão é:

2 10101 3367
· = .
100 · 101 3 5050
194. (Extraı́do da OBM 2014)
Observe que:
Fk2 Fk2+1 − Fk2
1− =
Fk2+1 Fk2+1
( Fk+1 − Fk )( Fk+1 + Fk )
=
Fk2+1
Fk−1 Fk+2
= .
Fk2+1
Assim,
2
F2 F32
    
F2013
1 − 22 1− 2 ·...· 1− 2 =
F3 F4 F2014
F1 F4 F2 F5 F3 F6 F F
2
· 2 · 2 . . . 20122 2015 =
F3 F4 F5 F2014
F1 F4 F2 F5 F3 F6 F2012
F
2015
2
· 2
· 2
. . . 
2 =
F
3 F
4 F
5
F 
2014
  
F1 · F2 · F2015
=
F3 · F2013 · F2014
F2015
.
2F2013 F2014
Resposta E.
195. Na identidade anterior, podemos trocar a soma de quaisquer dois, pelo simétrico do terceiro obtendo:
( x + y + z )3 =
x3 + y3 + z3 + 3( x + y)( x + z)(y + z) =
x3 + y3 + z3 + 3(−z)(−y)(− x ) =
x3 + y3 + z3 − 3xyz.
Como ( x + y + z)3 = 0, segue o resultado.

61
POTI 2015 − Teoria dos Números − Nı́vel 2 − Aula 0 − Cleber Assis, Samuel Barbosa e Tiago Miranda

196. (Extraı́do da Olimpı́ada do Cone Sul)


Comecemos analisando alguma relação entre a, b e a + b + ab. O último termo lembra a fatoração:

( a + 1)(b + 1) = ab + a + b + 1.

Em cada momento após realizarmos as operações, se analisarmos a quantidade que representa o produto de
todos os números do conjunto acrescidos de uma unidade. A equação anterior nos diz que tal produto nunca
se altera. Consequentemente, no final teremos um único número x tal que:

(1 + 1/2)(1 + 1/3) . . . (1 + 1/100) = (1 + x ).

Ou seja, x = 99/2. Para entender melhor que quantidade estamos analisando, façamos um exemplo pequeno.
Suponha que em um dado momento temos os números 2, 3 e 5, devemos analisar o número

(2 + 1)(3 + 1)(5 + 1).

Se trocarmos a = 2 e b = 3 por ab + a + b = 11 e fizermos o novo produto obteremos:

(11 + 1)(5 + 1).

Perceba que o valor continua sendo o mesmo.

197. Para fazer tal expansão, podemos considerar momentaneamente x + y = w e a expressão que já
conhecemos para o binômio:

( x + y + z )3 =
( w + z )3 =
w3 + 3w2 z + 3wz2 + z3

Além disso,

w3 =
( x + y )3 =
x3 + 3x2 y + 3xy2 + y3 =
x3 + y3 + 3xy( x + y) =
x3 + y3 + 3xyw

Voltando para a expressão original, temos:

( x + y + z )3 =
x3 + y3 + z3 + 3xyw + 3w2 z + 3wz2 .

Resta estudarmos o termo:

3xyw + 3w2 z + 3wz2 =


3w( xy + wz + z2 ) =
3( x + y)( xy + xz + yz + z2 ) =
3( x + y)( x + z)(y + z)

Com isso, podemos concluir que:

( x + y + z)3 = x3 + y3 + z3 + 3( x + y)( x + z)(y + z).

62
POTI 2015 − Teoria dos Números − Nı́vel 2 − Aula 0 − Cleber Assis, Samuel Barbosa e Tiago Miranda

198. Para fazer tal expansão, podemos considerar momentaneamente x + y = w e a expressão que já
conhecemos para o binômio:

( x + y + z )2 =
( w + z )2 =
w2 + 2wz + +z2

Além disso,

w2 =
( x + y )2 =
x2 + 2xy + y2 .

Voltando para a expressão original, temos:

( x + y + z )2 =
x2 + y2 + z2 + 2xy + 2wz =
x2 + y2 + z2 + 2xy + 2( x + y)z =
x2 + y2 + z2 + 2xy + 2x + 2yz

Com isso, podemos concluir que:

( x + y + z)2 = x2 + y2 + z2 + 2xy + 2xz + 2yz.


199. Sejam x = b − c, y = c − a e z = a − b. Pelo exercı́cio anterior, como x + y + z = 0, podemos escrever:

( b − c )3 + ( c − a )3 + ( a − b )3 =
x 3 + y3 + z3 =
3xyz =
3(b − c)(c − a)( a − b).

200. Elevando ao quadrado a igualdade dada, temos

a2 x2 + b2 y2 + c2 z2 + 2( abxy + bcyz + cazx ) = 0

E consequentemente:
−2( abxy + bcyz + cazx ) = a2 x2 + b2 y2 + c2 z2
Daı́, a expressão bc(y − z)2 + ca(z − x )2 + ab( x − y)2 é igual a

x2 ( ab + ac) + y2 (ba + bc) + z2 (ca + cb)


−2( abxy + bcyz + cazx )
= x2 ( a2 + ab + ac) + y2 (ba + b2 + bc) +
+z2 (ca + cb + c2 )
= ax2 ( a + b + c) + by2 ( a + b + c) +
+cz2 ( a + b + c)
= ( ax2 + by2 + cz2 )( a + b + c).

Assim,
ax2 + by2 + cz2 1
= ,
bc(y − z)2 + ca(z − x )2 + ab( x − y)2 a+b+c
que independe de x, y e z.

63
POTI 2015 − Teoria dos Números − Nı́vel 2 − Aula 0 − Cleber Assis, Samuel Barbosa e Tiago Miranda

201.

a) F. d) A. g) A.
b) A. e) F.
c) F. f) A.
202. a, b, d, g, i.
203.
a) P = 4l, sendo P o perı́metro e l a medida do lado.
b) A = l 2 , sendo A a área e l a medida do lado.
c) L + l = 16, sendo L a idade de Luiz e l a idade de Luı́sa.

x
d) < 3x, sendo x o referido número.
2
20
e) S = 700 + · V = 700 + 0, 2V, sendo S o salário e V o valor das vendas.
100
f) A = b · 2b = 2b2 , sendo A a área e b a medida da base.
204.

a) A = 6l 2 . b) V = l 3 . c) S = 12l.
205. (Extraı́do da Vı́deo Aula)
x x x x
Supondo que Diofanto tenha vivido x anos, temos x = + + + 5 + + 4. Para resolver esta equação,
6 12 7 2
primeiramente encontraremos um denominador comum a todas as frações, sendo o menor deles (e mais fácil
de se trabalhar) 84. Escrevendo agora as frações equivalentes, com denominador 84, a cada uma das frações
84x 14x 7x 12x 420 42x 336
da equação, temos = + + + + + . Fazendo as devidas simplificações, chegamos
84 84 84 84 84 84 84
a x = 84, que é a quantidade de anos vividos por Diofanto.
206. (Extraı́do da Vı́deo Aula)
a) P = 2a + b + a + b + b + 2a + b + a = 6a + 4b.
b) Basta somar ao perı́metro encontrado no item anterior, as medidas internas de divisão do terreno. Assim,
ficamos com Q = 6a + 4b + a + b + 2a = 9a + 5b.
c) Vamos calcular cada uma das áreas dos retângulos menores e somá-las. Temos então
A = 2a2 + 2ab + ab + b2 = 2a2 + 3ab + b2 .

Figura 4

64
POTI 2015 − Teoria dos Números − Nı́vel 2 − Aula 0 − Cleber Assis, Samuel Barbosa e Tiago Miranda

207.

a) Representaremos a área de um polı́gono ABCD por [ ABCD ]. Os dois polı́gonos formados são trapézios,
que possui uma fórmula para o cálculo direto de sua área, porém não a utilizaremos. Trace duas
paralelas ao lado AB, uma pelo ponto E e outra pelo ponto F. Pronto! Dividimos cada trapézio em um
30 · ( x − 5)
retângulo e um triângulo. Vamos ao cálculo de suas áreas. [ ABFE] = 30 · 5 + = 75 + 15x.
2
30 · ( x − 5)
[CDEF ] = 30 · (40 − x ) + = 1125 − 15x.
2
b) Como a [CDEF ] é o dobro de [ ABFE], temos:

[CDEF ] = 2[ ABFE]
1125 − 15x = 150 + 30x
45x = 975
65
x= .
3

Figura 6

65
Polos Olímpicos de Treinamento
Curso de Teoria dos Números - Nível 2 Aula 1
Samuel Barbosa Feitosa

Divisibilidade I

Teorema 1. (Algoritmo da Divisão) Para quaisquer inteiros positivos a e b, existe um único


par (q, r) de inteiros não negativos tais que b = aq + r e r < a. Os números q e r são
chamados de quociente e resto, respectivamente, da divisão de b por a.
Exemplo 2. Encontre um número natural N que, ao ser dividido por 10, deixa resto 9, ao
ser dividido por 9 deixa resto 8, e ao ser dividido por 8 deixa resto 7.
O que acontece ao somarmos 1 ao nosso número? Ele passa a deixar resto 0 na divisão por
10, 9 e 8. Assim, um possı́vel valor para N é 10 · 9 · 8 − 1.
Exemplo 3. a) Verifique que an − 1 = (a − 1)(an−1 + an−2 + . . . + a + 1)

b) Calcule o resto da divisão de 42012 por 3.

Para o item a), usando a distributividade e efetuando os devidos cancelamentos no lado


direito, podemos escrever:

an + an−1 + . . . + a2 + a − an−1 − an−2 − . . . − a − 1 = an − 1.

Para o item b), veja que 3 = 4−1 e assim é natural substituir os valores dados na expressão
do primeiro item:
42012 − 1 = 3(42011 + . . . + 4 + 1).
Isso significa que q = (42011 + . . . + 4 + 1) e que r = 1.
Observação 4. O teorema anterior admite um enunciado mais geral: Para quaisquer intei-
ros a e b, com a 6= 0, existe um único par de inteiros (q, r) tais que b = aq + r, 0 ≤ r < |a|.
Por exemplo, o resto da divisão de −7 por −3 é 2 e o quociente é 3.
Iremos agora estudar propriedades a respeito das operações com restos.
Teorema 5. (Teorema dos Restos) Se b1 e b2 deixam restos r1 e r2 na divisão por a,respectivamente,
então:
POT 2012 - Teoria dos Números - Nı́vel 2 - Aula 1 - Samuel Feitosa

b1 + b2 deixa o mesmo resto que r1 + r2 na divisão por a


b1 b2 deixa o mesmo resto que r1 r2 na divisão por a.

Demonstração. Por hipótese, existem q1 , q2 e q tais que: b1 = aq1 + r1 , b2 = aq2 + r2 e


r1 + r2 = aq + r, logo:
b1 + b2 = a(q1 + q2 + q) + r.
Como 0 < r < |a|, b1 + b2 deixa resto r quando dividido por a. A demonstração para o
produto é deixada ao cargo do leitor.

Observação 6. Em alguns casos, é preferı́vel que o professor faça uma demonstração do


resultado anterior para a = 3 ou a = 5 apenas com o intuito de deixar os alunos mais
confortáveis a respeito do resultado. É preferı́vel que mais tempo seja gasto resolvendo
exemplos e problemas. Na seção de congruências, os alunos terão um contato mais apro-
priado com o enunciado anterior.

Exemplo 7. Qual o resto que o número 1002 · 1003 · 1004 deixa quando dividido por 7?

Como 1002 deixa resto 1 por 7, o número acima deixa o mesmo resto que 1 · 2 · 3 = 6 por 7.

Exemplo 8. Qual o resto que o número 45000 deixa quando dividido por 3?

Como 4 deixa resto 1 por 3, 45000 deixa o mesmo resto que 1| · 1 ·{z. . . · 1} = 1 por 3.
5000

Exemplo 9. Qual o resto que o número 22k+1 deixa quando dividido por 3?

Note que 20 deixa resto 1 por 3, 21 deixa resto 2 por 3, 22 deixa resto 1 por 3, 23 deixa
resto 2 por 3, 24 deixa resto 1 por 3. Precebeu alguma coisa? Como 100 é par, o resto
deverá ser 1. Como 22 deixa resto 1, então 22k = 2| 2 · 22 {z
· . . . · 2}2 deixa o mesmo resto que
k
1| · 1 ·{z. . . · 1} = 1 e 22k+1 = 22k · 2 deixa o mesmo resto que 1 · 2 = 2 por 3.
k

Exemplo 10. Qual o resto de n3 + 2n na divisão por 3?

Se o resto de n por 3 é r, o resto de n3 + 2n é o mesmo de r3 + 2r. Para r = 0, esse


resto seria 0. Para r = 1, seria o mesmo resto de 3 que é 0. Finalmente, para r = 2, o
resto seria o mesmo de 8 + 4 = 12 que também é 0. Assim, não importa qual o resto de n
por 3, o número n3 + 2n sempre deixará resto 0. Uma ideia importante nessa solução foi
dividı́-la em casos. Também poderı́amos ter resolvido esse exemplo apelando para alguma
fatoração:

n3 + 2n = n3 − n + 3n = n(n2 − 1) + 3n = n(n − 1)(n + 1) + 3n.

Como n − 1, n e n + 1 são consecutivos, um deles é múltiplo de 3. Assim, o último termo da


igualdade anterior é a soma de dois múltiplos de 3 e consequentemente o resto procurado
é 0.

2
POT 2012 - Teoria dos Números - Nı́vel 2 - Aula 1 - Samuel Feitosa

Observação 11. Fatorações podem ser muito úteis para encontrarmos os valores explı́citos
de q e r.

Exemplo 12. Prove que, para cada n natural,

(n + 1)(n + 2) . . . (2n)

é divisı́vel por 2n .

Veja que
1 · 2 · · · 2n
(n + 1)(n + 2) . . . (2n) = .
1 · 2···n
Para cada número natural k no produto escrito no denominador, temos uma aparição de
2k no produto escrito no numerador. Basta efetuarmos os cancelamentos obtendo:

(n + 1)(n + 2) . . . (2n) = 2n · 1 · 3 · · · (2n − 1).

Exemplo 13. (Olimpı́ada de Leningrado 1991) Cada um dos naturais a, b, c e d é divisı́vel


por ab − cd, que também é um número natural. Prove que ab − cd = 1.

Se chamarmos p = ab − cd, teremos a = px, b = py, c = pz e d = pt onde x, y, z e t são


inteiros. Assim, p = p2 (xy − zt). Consequentemente 1 = p(xy − zt) e concluı́mos que p = 1,
pois p é natural.

Exemplo 14. A soma digital D(n) de um inteiro positivo n é definida recursivamente como
segue: 
n se 1 ≤ n ≤ 9,
D(n) =
D(a0 + a1 + . . . + am ) se n > 9,
onde a0 , a1 , . . . , am são todos os dı́gitos da expressão decimal de n na base 10, i.e.,

n = am 10m + am−1 10m−1 + . . . + a1 10 + a0

Por exemplo, D(989) = D(26) = D(8) = 8. Prove que: D((1234)n) = D(n), para n =
1, 2, 3 . . .

Como 10n − 1n = (10 − 1)(10n−1 + 10n−2 + . . . + 1), podemos concluir que 10n sempre deixa
resto 1 na divisão por 9. Assim, n = am 10m + am−1 10m−1 + . . . + a1 10 + a0 , deixa o mesmo
resto que am + am−1 + . . . + a0 na divisão por 9. Desse modo, D(n) nada mais é do que
o resto na divisão por 9 do número n. Como 1234 deixa resto 1 por 9, o número (1234)n
deixa o mesmo resto que 1 · n por 9, ou seja, D((1234)n) = D(n).

Observação 15. O exemplo anterior contém o critério de divisibilidade por 9, i.e., n deixa
o mesmo resto que D(n) na divisão por 9. O critério de divisibilidade por 3 é análogo pois
10n também sempre deixa resto 1 por 3.

Exemplo 16. Encontre todos os pares de inteiros positivos a e b tais que 79 = ab + 2a + 3b.

3
POT 2012 - Teoria dos Números - Nı́vel 2 - Aula 1 - Samuel Feitosa

Fatoremos a expressão anterior. Somando 6 aos dois lados da equação, obtemos:

85 = 6 + ab + 2a + 3b
= (3 + a)(2 + b)

Assim, (3 + a) e (2 + b) são divisores positivos de 85 maiores que 1. Os únicos divisores


positivos de 85 são 1, 5, 19, 85. Logo, os possı́veis pares de valores para (3 + a, 2 + b) são
(5, 19) ou (19, 5) que produzem as soluções (a, b) = (2, 17) e (16, 3).
2n − 2 22 −1 − 2
n

Problema 17. (Olimpı́ada Russa) Prove que se é um inteiro, então


n 2n − 1
também é um inteiro.
2n − 2
Se k = , então
n
22 −1 − 2 2(22 −2 − 1)
n n

=
2n − 1 2n − 1
 nk 
2 −1
= 2
2n − 1
!
(2n − 1)(2n(k−1) + 2n(k−2) + . . . + 2n + 1)
= 2
2n − 1
= 2(2n(k−1) + 2n(k−2) + . . . + 2n + 1),

é um número inteiro.

Problemas Propostos

Problema 18. Encontre os inteiros que, na divisão por 7, deixam um quociente igual ao
resto.

Problema 19. Determinar os números que divididos por 17 dão um resto igual ao quadrado
do quociente correspondente.

Problema 20. (OCM 1985) Encontre o quociente da divisão de a128 − b128 por

(a64 + b64 )(a32 + b32 )(a16 + b16 )(a8 + b8 )(a4 + b4 )(a2 + b2 )(a + b)

Problema 21. (OCM 1994) Seja A = 777 . . . 77 um número onde o dı́gito ”7”aparece 1001
vezes. Determinar o quociente e o resto da divisão de A por 1001.

Problema 22. Encontre um inteiro que deixa resto 4 na divisão por 5 e resto 7 na divisão
por 13

Problema 23. Encontre o menor inteiro que, dividido por 29 deixa resto 5, e dividido por
31 dá resto 28.

4
POT 2012 - Teoria dos Números - Nı́vel 2 - Aula 1 - Samuel Feitosa

Problema 24. Prove que, para todo inteiro positivo n o número n5 − 5n3 + 4n é divisı́vel
por 120.

Problema 25. (Fatorações Importantes)

a) Seja S = 1 + z + z 2 + z 3 + . . . + z n−1 . Veja que S + z n = 1 + zS então S(z − 1) = z n − 1.


Conclua que, para quaisquer x e y vale:

xn − y n = (x − y)(xn−1 + xn−2 y + xn−3 y 2 + . . . + x2 y n−3 + xy n−2 + y n−1 )

b) Mostre que se n é ı́mpar vale:

xn + y n = (x + y)(xn−1 − xn−2 y + xn−3 y 2 − . . . + x2 y n−3 − xy n−2 + y n−1 )

Problema 26. Prove que, o número 199 + 299 + 399 + 499 + 599 é múltiplo de 5.

Problema 27. Mostre que o número 1n + 8n − 3n − 6n é multiplo de 10 para todo natural


n.

Problema 28. Encontre o resto da divisão 3710 − 1 por 11.

Problema 29. Prove que 22225555 + 55552222 é divisı́vel por 7.

Problema 30. Encontre o último dı́gito do número 19891989 .

Problema 31. Mostre que se n divide a então 2n − 1 divide 2a − 1.

Problema 32. (Cone Sul 1996) Provar que o número

1995 · 19971996 − 1996 · 19971995 + 1


19962
é um inteiro.

Problema 33. Mostre que para n ı́mpar, n divide 1n + 2n + . . . + (n − 1)n

Problema 34. Existe um natural n tal que nn + (n + 1)n é divisı́vel por 2011?

Problema 35. Quantos números inteiros positivos n existem tais que n + 3 divide n2 + 7?

Problema 36. Encontre o número de inteiros n tais que

1. 1000 < n < 8000.

2. nn+1 + (n + 1)n é divisı́vel por 3.

Problema 37. Sejam m e n naturais tais que mn + 1 é múltiplo de 24, mostre que m + n
também é múltiplo de 24.

5
POT 2012 - Teoria dos Números - Nı́vel 2 - Aula 1 - Samuel Feitosa

Problema 38. (Irlanda 1997) Encontre todos os pares de inteiros (x, y) tais que 1+1996x+
1998y = xy.

Dicas e Soluções

18. Os números são {0, 8, 16, 24, . . . , 8 · 7}.

18. Escreva n = 17q + q 2 e note que 0 ≤ q 2 < 17. Assim, q = 0, 1, 2, 3, 4.

19. Use a diferença de quadrados sucessivas vezes para obter (a − b) como quociente.
7(101001 −1) 10999 +1 101001 −1
21. O número do problema é igual a 9 . Além disso, 103 +1
é inteiro e 103 +1
=
10999 +1 100
100 · 103 +1
− 103 +1
.

22. Os números que satisfazem essa propriedade são os números da forma 65k + 59.

24. Basta mostrar que n5 − 5n3 + 4n é múltiplo de 3, 8 e 5. Na divisão por 5, temos


quatro restos possı́veis: {0, 1, 2, 3, 4}. Assim, o número n5 − 5n3 + 4n possui o mesmo
resto na divisão por 5 que um dos cinco números: {05 − 5 · 03 + 40, 15 − 5 · 13 + 4, 25 −
5 · 23 + 8, 35 − 5 · 33 + 12, 45 − 5 · 43 + 16}. Como todos esses números são múltiplos
de 5, segue que n5 − 5n3 + 4n é múltiplo de 5 para todo n inteiro. O procedimento
com 3 e 8 é semelhante.

25. Para o item a), troque z por xy . Para o item b), substitua y por −y no item anterior.

26. Pelo problema anterior, como 99 é ı́mpar temos: 199 + 499 = (1 + 4)(198 + 197 · 4 +
. . . + 1 · 497 + 498 ). Daı́, segue que 199 + 499 é múltiplo de 5. Analogamente podemos
mostrar que 299 + 399 é múltiplo de 5.

27. O número em questão é mútiplo de 2 pois é a soma de dois ı́mpares e dois pares.
Para ver que também é múltiplo de 5, basta notar que 5 divide 1n − 6n e 8n − 3n .
Isso pode ser facilmente mostrado usando a fatoração do exercı́cio 25.

31. Se a = nk, temos (2n − 1)(2n(k−1) + 2n(k−2) + . . . + 2n + 1) = 2nk − 1.

32. Veja que 1995·19971996 −1996·19971995 +1 = 1995·(19971996 −1)−1996·(19971995 −1).


Pela fatoração de xn − y n ,

1996 · (19971995 − 1)
= (19971994 + 19971993 + . . . + 1),
19962
é inteiro. Além disso, pela mesma fatoração,
 
1995 · (19971996 − 1) 19971995 − 1 19971994 − 1 1997 − 1 1996
= 1995· + + . . . + + ,
19962 1996 1996 1996 1996

é uma soma de números inteiros.

6
33. Como n é impar,

(n − i)n + in = ((n − i) + i)((n − i)n−1 − (n − i)n−2 i + . . . − (n − i)in−2 + in−1 ).

34. Faça n = 1005 e use a fatoração de xn + y n .

37. Fatore a expressão como:

(x − 1998)(y − 1996) = xy − 1998y − 1996x + 1998 · 1996 = 19972 .

Os divisores de 19972 são {±1, ±1997, ±19972 }. Resolvendo os sistemas correspon-


dentes à essas possibilidades, temos: (x, y) = (1999, 19972 + 1996), (1997, −19972 +
1996), (3995, 3993), (1, −1), (19972 + 1998, 1997), (−19972 + 1998, 1995).

Referências
[1] F. E. Brochero Martinez, C. G. Moreira, N. C. Saldanha, E. Tengan - Teoria dos
Números um passeio com primos e outros números familiares pelo mundo inteiro,
Projeto Euclides, IMPA, 2010.

[2] E. Carneiro, O. Campos and F. Paiva, Olimpı́adas Cearenses de Matemática 1981-2005


(Nı́veis Júnior e Senior), Ed. Realce, 2005.

[3] S. B. Feitosa, B. Holanda, Y. Lima and C. T. Magalhães, Treinamento Cone Sul 2008.
Fortaleza, Ed. Realce, 2010.

[4] D. Fomin, A. Kirichenko, Leningrad Mathematical Olympiads 1987-1991, MathPro


Press, Westford, MA, 1994.

[5] D. Fomin, S. Genkin and I. Itenberg, Mathematical Circles, Mathematical Words, Vol.
7, American Mathematical Society, Boston, MA, 1966.

[6] I. Niven, H. S. Zuckerman, and H. L. Montgomery, An Introduction to the Theory of


Numbers.
Polos Olímpicos de Treinamento
Curso de Teoria dos Números - Nível 2 Aula 2
Prof. Samuel Feitosa

Divisibilidade II

Definição 1. Dados dois inteiros a e b, com a 6= 0, dizemos que a divide b ou que a é


um divisor de b ou ainda que b é um múltiplo de a e escrevemos a | b se o r obtido pelo
algoritmo de divisão aplicado à a e b é 0, ou seja, se b = aq para algum inteiro q.

Lema 2. Sejam a, b, c, d inteiros. Temos

i) (”d divide”) Se d | a e d | b, então d | ax + by para quaisquer x e y inteiros.

ii) (”Limitação”) Se d | a, então a = 0 ou |d| ≤ |a|.

iii) (Transitividade) Se a | b e b | c, então a | c.

Em particular, segue da propriedade i) que d | a + b e d | a − b.

Exemplo 3. (Olimpı́ada de Maio 2006) Encontre todos os naturais a e b tais que a|b + 1 e
b|a + 1.

Pela propriedade da Limitação, temos a ≤ b + 1 e b ≤ a + 1. Daı́, a − 1 ≤ b ≤ a + 1.


Vejamos os casos:

(i) a = b. Como a|b + 1 e a | b(pois b = a) temos que a | [(b + 1) − b] = 1. Assim, a = 1


Nesse caso, só temos a solução (a, b) = (1, 1)

(ii) a = b + 1. Como b|a + 1 e b|a − 1(pois b = a − 1) temos que b|[(a + 1) − (a − 1)] = 2.


Assim, b = 1 ou b = 2 e nesse caso, só temos as soluções (3, 2) e (2, 1).

(iii) a = b − 1. Esse caso é análogo ao anterior e as soluções para (a, b) são (1, 2) e (2, 3).

Exemplo 4. (Critério de Divisibilidade por 7) Existem alguns métodos práticos para deci-
dirmos se um número é múltiplo de outro. Certamente o leitor já deve ter se deparado com
algum critério de divisibilidade. Existe um critério por 7 bastante popular: Para saber se
um inteiro é multiplo de 7, basta apagar seu último dı́gito, multiplicá-lo por 2 e o subtrair
do número que restou. Se o resultado é múltiplo de 7, então o número original também é
múltiplo de 7.
POT 2012 - Teoria dos Números - Nı́vel 2 - Aula 1 - Samuel Feitosa

Podemos aplicar esse algoritmo sucessivas vezes até que o resultado obtido seja facil-
mente verificável como um múltiplo de 7. Por exemplo, para o número 561421 podemos
escrever:

56142 − 2 = 56140
5614 − 0 = 5614
561 − 8 = 553
55 − 6 = 49

Como 49 é múltiplo de 7, nosso número original também é. Por que esse processo funciona?
Se o nosso número original está escrito na forma 10a + b, então o número obtido após a
operação descrita é a − 2b. Basta mostrarmos que se 7 | a − 2b, então 7 | 10a + b. Se
7 | a − 2b, pela propriedade (i) do lema, concluı́mos que 7 | 10a − 20b. Como 7 | 21b,
também temos que 7 | [(10a − 20b) + 21b] = 10a + b.
Exemplo 5. Mostre que se 7 | 3a + 2b então 7 | 4a − 2b.
Veja que 7 | 7a e 7 | 3a + 2b, então 7 | [7a − (3a + 2b)] = 4a − 2b. Na prática, o que
fizemos foi multiplicar o número 3a + 2b por algum inteiro para posteriormente subtraı́mos
um múltiplo de 7 conveniente e obtermos o número 4a − 2b. Existem outras formas de
fazermos isso. Observe os números 3 · 0, 3 · 1, 3 · 2, 3 · 3, 3 · 4, 3 · 5, 3 · 6. O número 3 · 6 deixa
o mesmo resto que 4 por 7, pois 3 · 6 = 7 · 2 + 4. Como 7|3a + 2b podemos concluir que
7|(18a + 12b) e consequentemente 7 | [18a + 12b − 14a)] = 4a + 12b. Mas 7 | 14b, então
7 | [4a + 12b − 14b] = 4a − 2b.
Para o proximo exemplo, o leitor precisará lembrar dos critérios de divisibilidade por 9
e 3 vistos na aula passada.
Exemplo 6. Usando os dı́gitos 1, 2, 3, 4, 5, 6, 7, construı́mos vários números de sete dı́gitos
distintos. Existem dois deles, distintos, tais que um divide o outro?
Não. Suponha, por absurdo, que m < n sejam dois desses números, com m | n. Claramente
m | n − m e 9 | n − m, pois n e m possuem a mesma soma dos dı́gitos e consequentemente
possuem o mesmo resto na divisão por 9. Por outro lado, sabemos a soma dos dı́gitos de
m: 1 + 2 + · · · + 7 = 3 · 9 + 1. Daı́, m não possui fator 9 e podemos garantir que 9m | n − m.
Mas então 9m ≤ n − m ⇒ 10m ≤ n ⇒ n tem pelo menos oito dı́gitos, uma contradição.

Exemplo 7. (Leningrado 1989) Seja A um número natural maior que 1, e seja B √um
número natural que é um divisor de A2 + 1. Prove que se B − A > 0, então B − A > A.
Seja B − A = q. Assim, A + q | A2 + 1. Como (A − q)(A + q) = A2 − q 2 é divisı́vel por
A + q, podemos concluir que A + q | [(A2 + 1) − (A2 − q 2 )] = q 2 + 1. Pela propriedade de
limitação, A + q ≤ q 2 + 1. Nessa desigualdade, não podemos
√ ter q = 1 pois A > 1. Usando
2 2
então que q > 1, temos A ≤ q − q + 1 < q , ou seja, A < q.
Problema 8. (AIME 1986) Qual é o maior inteiro n para o qual n3 + 100 é divisı́vel por
n + 10?

2
POT 2012 - Teoria dos Números - Nı́vel 2 - Aula 1 - Samuel Feitosa

Para achar explicitamente o quociente de n3 + 100 por n + 10 podemos fazer uso de alguma
fatoração. Utilizaremos a soma dos cubos n3 + 103 = (n + 10)(n2 − 10n + 100). Como,

n3 + 100 = (n + 10)(n2 − 10n + 100) − 900,

podemos concluir que o número 900 deve ser múltiplo de n + 10. O maior inteiro n para o
qual n + 10 divide 900 é 890. Veja que se n = 890, o quociente da divisão de n3 + 100 por
n + 10 é n2 − 10n + 100 − 1 = 8902 − 10 · 890 + 99.

Exemplo 9. (Extraı́do de [1]) Encontre todos os inteiros positivos n tais que 2n2 + 1 |
n3 + 9n − 17.

Utilizando o “2n2 + 1 divide” para reduzir o grau de n3 + 9n − 17, temos que


(
2n2 + 1 | n3 + 9n − 17
2n2 + 1 | 2n2 + 1
=⇒ 2n2 + 1 | (n3 + 9n − 17) · 2 + (2n2 + 1) · (−n)
⇐⇒ 2n2 + 1 | 17n − 34

Como o grau de 17n − 34 é menor do que o de 2n2 + 1, podemos utilizar a “limitação”


para obter uma lista finita de candidatos a n. Temos 17n − 34 = 0 ⇐⇒ n = 2 ou
|2n2 + 1| ≤ |17n − 34| ⇐⇒ n = 1, 4 ou 5. Destes candidatos, apenas n = 2 e n = 5 são
soluções.

Exemplo 10. (Leningrado 1990) Sejam a e b números naturais tais que b2 + ba + 1 divide
a2 + ab + 1. Prove que a = b.

Pela propriedade de limitação, b2 +ba+1 ≤ a2 +ab+1 e daı́ b ≤ a. Além disso, b2 +ab+1 >
a − b. A igualdade b(a2 + ab + 1) − a(b2 + ba + 1) = b − a implica que a − b é divisı́vel por
b2 + ba + 1. Se a − b 6= 0, então b2 + ab + 1 ≤ a − b. Mas isso é um absurdo, logo a − b = 0.

Problemas Propostos

Problema 11. Mostre que se 3 | a + 7b então 3 | a + b.

Problema 12. Mostre que se 7 | a + 3b então | 13a + 11b

Problema 13. Mostre que se 19 | 3x + 7y então 19 | 43x + 75y

Problema 14. Mostre que se 17 | 3a + 2b então 17 | 10a + b

Problema 15. Encontre todos os inteiros positivos n tais que n + 2009 divide n2 + 2009 e
n + 2010 divide n2 + 2010.

Problema 16. Seja n > 1 e k um inteiro positivo qualquer. Prove que (n − 1)2 |(nk − 1) se,
e somente se , (n − 1)|k.

3
POT 2012 - Teoria dos Números - Nı́vel 2 - Aula 1 - Samuel Feitosa

Problema 17. (OBM 2005) Prove que a soma 1k + 2k + . . . + nk , onde n é um inteiro e k


é ı́mpar, é divisı́vel por 1 + 2 + . . . + n.

Problema 18. O número de seis dı́gitos X = abcdef satisfaz a propriedade de que abc−def
é divisı́vel por 7. Prove que X também é divisı́vel por 7.

Problema 19. (Bielorússia 1996) Inteiros m e n, satisfazem a igualdade


4mn
(m − n)2 = .
m+n−1
a) Prove que m + n é um quadrado perfeito.

b) Encontre todos os pares (m, n) satisfazendo a equação acima.

Problema 20. (Olimpı́ada de Leningrado) Os números naturais a,b e c têm a propriedade


que a3 é divisı́vel por b, b3 é divisı́vel por c e c3 é divisı́vel por a. Prove que (a + b + c)13 é
divisı́vel por abc.

Problema 21. (OBM 2000) É possı́vel encontrar duas potências de 2, distintas e com o
mesmo número de algarismos, tais que uma possa ser obtida através de uma reordenação
dos dı́gitos da outra? (Dica: Lembre-se do critério de divisibilidade por 9)

Problema 22. (IMO 1998) Determine todos os pares de inteiros positivos (x, y) tais que
xy 2 + y + 7 divide x2 y + x + y.

Dicas e Soluções

11. Como 3 | 6b, segue que 3 | [(a + 7b) − 6b] = a + b.

12. Como 7 | a + 3b, segue que 7 | 13a + 39b = (13a + 11b) + 28b. Mas 7 | 28b, portanto
7 | [(13a + 11b) + 28b − 28b] = 13a + 11b.

13. Como 19 | 3x + 7y, segue que 19 | 27(3x + 7y) = (43x + 75y) + (38x + 114y). Mas
19 | 19(2x + 6y), portanto 19 | [(43x + 75y) + (38x + 114y) − 19(2x + 6y)] = 43x + 75y.

14. Como 17 | 3a + 2b, segue que 17 | 27a + 18b = (10a + b) + 17(a + b).

16. Veja que

nk − 1 nk−1 − 1 nk−2 − 1
 
n−1 k
= + + ... + + .
(n − 1)2 n−1 n−1 n−1 n−1
l
Como os números nn−1 −1
sempre são inteiros, o número do lado esquerdo da equação
k
será inteiro se, e somente se, o número n−1 for inteiro.

4
POT 2012 - Teoria dos Números - Nı́vel 2 - Aula 1 - Samuel Feitosa

17. Comece dividindo o problema quando em dois casos: n é par ou n é ı́mpar. Sabemos
que 1 + 2 + . . . + n = n(n+1)
2 . Para n ı́mpar, basta mostrar que o número em questão
é divisı́vel por n e n+1
2 . O próximo passo é lembrar do problema 33 da aula 1. Pela
fatoração de x + y , temos que i + (n − i)k é divisı́vel por n. Faça outros tipos de
n n k

pares para mostrar a divisibilidade por n2 . O caso quando n é par é análogo.

18. Veja que X = 103 · abc + def = 1001abc − (abc − def ). Como 1001 é multiplo de 7,
concluı́mos que X é a soma de dois múltiplos de 7.

19. Somando 4mn em ambos os lados, obtemos:


4mn
(m + n)2 = + 4mn
m+n−1
4mn(m + n)
= ⇒
m+n−1
4mn
(m + n) =
m+n−1
= (m − n)2 .

Assim, m + n é o quadrado de um inteiro. Se m − n = t, então m + n = t2 e


2 2
(m, n) = ( t 2+t , t 2−t ). É fácil verificar que para qualquer t inteiro esse par é solução
do problema.

20. Analise a expansão pelo binômio de Newton.

21. Não. Suponha, por absurdo, que existam duas potências de 2, 2m < 2n , satisfazendo
o enunciado. Como 2n é um múltiplo de 2m , podemos ter: 2n = 2·2m , 4·2m , 8·2m , . . ..
n
Além disso, como ambos possuem a mesma quantidade de dı́gitos, temos 1 < 22m <
10. Assim, as únicas possibilidade são 2n = 2 · 2m , 4 · 2m , 8 · 2m . Pelo critério de
divisibilidade por 9, como 2m e 2n possuem os mesmos dı́gitos, podemos concluir
que 2n − 2m é um múltiplo de 9. Entretanto, nenhuma das possibilidade anteriores
satisfaz essa condição e chegamos em um absurdo.

22. Começaremos usando a ideia do exemplo 10. A igualdade y(x2 y + x + y) − x(xy 2 +


y + 7) = y 2 − 7x implica que y 2 − 7x é divisı́vel por xy 2 + y + 7. Se y 2 − 7x ≥ 0,
como y 2 − 7x < xy 2 + y + 7, segue que y 2 − 7x = 0. Assim, (x, y) = (7t2 , 7t) para
algum t ∈ N. É fácil checar que esses pares são realmente soluções. Se y 2 − 7x < 0,
então 7x − y 2 > 0 é divisı́vel por xy 2 + y + 7. Daı́, xy 2 + y + 7 ≤ 7x − y 2 < 7x, que
nos permite concluir que y ≤ 2. Para y = 1, temos x + 8 | 7x − 1 e consequentemente
x + 8 | 7(x + 8) − (7x − 1) = 57. Então as únicas possibilidades são x = 11 e x = 49,
cujos pares correspondentes são (11, 1), (49, 1). Para y = 2, temos 4x + 9 | 7x − 4
e consequentemente 7(4x + 9) − 4(7x − 4) = 79 é divisı́vel por 4x + 9. Nesse caso,
não obtemos nenhuma solução nova. Todas as soluções para (x, y) são: (7t2 , 7t)(t ∈
N), (11, 1) e (49, 1).

5
Referências
[1] F. E. Brochero Martinez, C. G. Moreira, N. C. Saldanha, E. Tengan - Teoria dos
Números um passeio com primos e outros números familiares pelo mundo inteiro,
Projeto Euclides, IMPA, 2010.

[2] E. Carneiro, O. Campos and F. Paiva, Olimpı́adas Cearenses de Matemática 1981-2005


(Nı́veis Júnior e Senior), Ed. Realce, 2005.

[3] S. B. Feitosa, B. Holanda, Y. Lima and C. T. Magalhães, Treinamento Cone Sul 2008.
Fortaleza, Ed. Realce, 2010.

[4] D. Fomin, A. Kirichenko, Leningrad Mathematical Olympiads 1987-1991, MathPro


Press, Westford, MA, 1994.

[5] D. Fomin, S. Genkin and I. Itenberg, Mathematical Circles, Mathematical Words, Vol.
7, American Mathematical Society, Boston, MA, 1966.

[6] I. Niven, H. S. Zuckerman, and H. L. Montgomery, An Introduction to the Theory of


Numbers.
Polos Olímpicos de Treinamento
Curso de Teoria dos Números - Nível 2 Aula 3
Prof. Samuel Feitosa

O Algoritmo de Euclides

Exemplo 1. Seja S um conjunto infinito de inteiros não negativos com a seguinte propri-
edade: dados dois quaisquer de seus elementos, o valor absoluto da diferença entre eles
também pertence a S. Se d é o menor elemento positivo de S, prove que S consiste de
todos os múltiplos de d.

Considere um elemento m qualquer de S. Pelo algoritmo da divisão, m = qd + r com


0 ≤ r < d. Como todos os números m − d, m − 2d, m − 3d, . . . , m − qd = r pertencem
a S e d é o menor elemento positivo de tal conjunto, devemos ter obrigatoriamente que
r = 0. Sendo assim, podemos concluir que todos os elementos de S são múltiplos de d.
Resta mostrarmos que todos os múltiplos de d estão em S. Seja kd um múltiplo positivo
qualquer de d. Como S é infinito, existe um inteiro m ∈ S tal que m = qd > kd. Assim
todos os números m − d, m − 2d, . . . , m − (q − k)d = kd estão em S.

Definição 2. Um inteiro a é um divisor comum de b e c se a | b e a | c. Se b e c não são


ambos nulos, denotaremos por mdc(b, c) o máximo divisor comum de b e c.

Como um inteiro não nulo possui apenas um número finito de divisores, se b e c são ambos
não nulos, o número mdc(b, c) sempre existe, isto é, sempre está bem definido.

Lema 3. (Euclides) Se x 6= 0, mdc(x, y) = mdc(x, x + y)

Demonstração. Seja d um divisor comum de x e y. Então d | x + y e consequentemente d


também á um divisor comum de x e x + y. Reciprocamente, se f é um divisor comum de
x + y e x, f também divide (x + y) − y = x e assim f é um divisor comum de x e y. Como
os conjuntos de divisores comuns dos dois pares de números mencionados são os mesmos,
o maior divisor comum também é o mesmo.

Então podemos calcular:

mdc(123, 164) = mdc(123, 41) = mdc(41, 123) = mdc(41, 82) = mdc(41, 41) = 41.
POT 2012 - Teoria dos Números - Nı́vel 2 - Aula 3 - Samuel Feitosa

Exemplo 4. Três máquinas I, R, S imprimem pares de inteiros positivos em tickets. Para


a entrada (x, y), as máquinas I, R, S imprimem respectivamente (x − y, y), (x + y, y), (y, x).
Iniciando com o par (1, 2) podemos alcançar

a) (819, 357)?

b) (19, 79)?

Para o item a), calculemos inicialmente mdc(819, 357):

mdc(819, 357) = mdc(462, 357) = mdc(105, 357) = mdc(105, 252) = . . . = mdc(21, 21) = 21.

Pelo Lema de Euclides, o mdc entre os dois números em um ticket nunca muda. Como
mdc(1, 2) = 1 6= 21 = mdc(819, 357), não podemos alcançar o par do item a).

Para o item b), indiquemos com → uma operação de alguma das máquinas. Veja que:
R S R R R S R R R
(2, 1) → (3, 1) → (1, 3) → (4, 3) → . . . → (19, 3) → (3, 19) → (22, 19) → (41, 19) →
R
(60, 19) → (79, 19).

Observação 5. Procurar invariantes sempre é uma boa estratégia para comparar confi-
gurações diferentes envolvidas no problema. Confira o problema proposto 31.

Definição 6. Dizemos que dois inteiros p e q são primos entre si ou relativamente primos
se mdc(p, q) = 1. Dizemos ainda que a fração pq é irredutı́vel se p e q são relativamente
primos.
21n + 4
Exemplo 7. (IMO 1959) Prove que é irredutı́vel para todo número natural n.
14n + 3
Pelo lema de Euclides, mdc(21n+4, 14n+3) = mdc(7n+4, 14n+3) = mdc(7n+1, 7n+2) =
mdc(7n + 1, 1) = 1.

O seguinte lema será provado na próxima aula.

Lema 8. (Propriedades do MDC) Seja mdc(a, b) = d, então:

i) Se k =
6 0, mdc(ka, kb) = kd.
 
a b
ii) mdc , = 1.
d d
iii) Se mdc(a, c) = 1, então mdc(a, bc) = d.

Exemplo 9. (Olimpı́ada Inglesa) Se x e y são inteiros tais que 2xy divide x2 + y 2 − x, prove
que x é um quadrado perfeito

2
POT 2012 - Teoria dos Números - Nı́vel 2 - Aula 3 - Samuel Feitosa

Se d = mdc(x, y), então x = da e y = db, com mdc(a, b) = 1. Do enunciado, temos:

2abd2 | d2 a2 + d2 b2 − da ⇒
d2 | d2 a2 + d2 b2 − da ⇒
d2 | −da ⇒
d | a.

Logo, a = dc, para algum c. Como x | y 2 , obtemos d2 c | d2 b2 , ou seja, c|b2 e mdc(c, b2 ) = c.


Usando que mdc(a, b) = 1 e que todo divisor comum de b e c também é um divisor comum
de a e b, podemos concluir que mdc(c, b) = 1. Usando o item iii) do lema anterior,
mdc(c, b2 ) = 1. Assim, c = 1 e x = d2 c = d2 .

Exemplo 10. No planeta X, existem apenas dois tipos de notas de dinheiro: $5 e $78. É
possı́vel pagarmos exatamente $7 por alguma mercadoria? E se as notas fossem de $ 3 e $
78?

Veja que 2 × 78 − 31 × 5 = 1 e consequentemente 14 × 78 − 217 × 5 = 7. Basta darmos


14 notas de de $ 78 para recebermos 217 notas de $ 5 como troco na compra de nossa
mercadoria. Usando as notas de $3 e $78 não é possı́vel pois o dinheiro pago e recebido
como troco por algo sempre é múltiplo de 3 e 7 não é múltiplo de 3.

Queremos estudar a versão mais geral desse exemplo. Quais são os valores que podemos
pagar usando notas de $a e $b? Em particular, estaremos interessados em conhecer qual o
menor valor que pode ser pago. Para responder essa pergunta, precisaremos do algoritmo
de Euclides:

Teorema 11. (O Algoritmo de Euclides) Para os inteiros b e c > 0, aplique sucessivamente


o algoritmo da divisão para obter a série de equações:

b = cq1 + r1 , 0 < r1 < c,


c = r1 q 2 + r2 , 0 < r2 < r1 ,
r1 = r2 q 3 + r3 , 0 < r3 < r2 ,
..
.
rj−2 = rj−1 qj + rj , 0 < rj < rj−1 ,
rj−1 = rj qj+1

A sequência de restos não pode diminuir indefinidamente pois 0 ≤ ri < ri−1 e existe apenas
um número finito de naturais menores que c. Assim, para algum j, obteremos rj+1 = 0.
O maior divisor comum de b e c será rj , ou seja, o último resto não nulo da sequência de
divisões acima.

Demonstração. Pelo Lema de Euclides,

mdc(x + qy, y) = mdc(x + (q − 1)y, y) = mdc(x + (q − 2)y, y) = . . . = mdc(x, y).

3
POT 2012 - Teoria dos Números - Nı́vel 2 - Aula 3 - Samuel Feitosa

Então,
mdc(b, c) = mdc(c, r1 ) = mdc(r1 , r2 ) = . . . = mdc(rj−1 , rj ) = rj .

Exemplo 12. Calcule mdc(42823, 6409).


Pelo Algoritmo de Euclides,

42823 = 6 × 6409 + 4369


6409 = 1 × 4369 + 2040
4369 = 2 × 2040 + 289
2040 = 7 × 289 + 17
289 = 17 × 17.

Portanto, mdc(42823, 6409) = 17.

Podemos extrair mais informações do Algoritmo de Euclides. Para isso, iremos organizar
as equações do exemplo acima de outra forma.

Essencialmente, a equação mdc(x+qy, y) = mdc(x, y) nos diz que podemos subtrair q vezes
um número de outro sem alterar o máximo divisor comum do par em questão. Realizando
esse procedimento sucessivas vezes, subtraindo o número menor do maior, podemos obter
pares com números cada vez menores até que chegarmos em um par do tipo (d, d). Como o
máximo divisor comum foi preservado ao longo dessas operações, d será o máximo divisor
comum procurado. Iremos repetir o exemplo anterior registrando em cada operação quantas
vezes um número é subtraido do outro. Isso será feito através de dois pares de números
auxiliares:

(42823, 6409) | (1, 0)(0, 1)


(4369, 6409) | (1, −6)(0, 1)
(4369, 2040) | (1, −6)(−1, 7)
(289, 2040) | (3, −20)(−1, 7)
(289, 17) | (3, −20)(−22, 147)
(17, 17) | (355, −2372)(−22, 147)

Da primeira linha para a segunda, como subtraı́mos 6 vezes o número 6409 de 42823,
subtraı́mos 6 vezes o par (0, 1) de (1, 0), obtendo: (1, 0) − 6(0, 1) = (1, −6). Se em uma
dada linha, temos:
(x, x + qy)) | (a, b)(c, d);
então, a próxima linha deverá ser:

(x, y) | (a, b)(c − aq, d − bq);

4
POT 2012 - Teoria dos Números - Nı́vel 2 - Aula 3 - Samuel Feitosa

porque representará a operação de subtrairmos q vezes o primeiro número do segundo. Veja


que o par (a, b) foi subtraido de (c, d) exatamente q vezes. Os números escritos nos últimos
dois pares representam os coeficientes dos números originais para cada número do primeiro
par. Por exemplo, analisando a linha:

(289, 2040) | (3, −20)(−1, 7);

obtemos que:

289 = 3 × 42823 − 20 × 6409,


2040 = −1 × 42823 + 7 × 6409.

Em cada linha, essa propriedade é mantida pois a mesma subtração que é realizada no
primeiro par também é realizada entre os dois últimos pares. Analisando o último par,
podemos escrever 17 como combinação de 42823 e 6409 de duas formas diferentes:

17 = −22 × 42823 + 147 × 6409,


17 = 355 × 42823 + −2372 × 6409,

Assim, se no planeta X tivéssemos apenas notas de $42823 e $6409, poderı́amos comprar


algo que custasse exatamente $17.

Como conclusão da discussão anterior e do algoritmo de Euclides, podemos concluir que:

Teorema 13. (Bachet-Bèzout) Se d = mdc(a, b), então existem inteiros x e y tais que
ax + by = d.

De fato, a discussão anterior também nos mostra um algoritmo para encontrarmos x e y.


Voltando à discussão sobre o planeta X, podemos concluir em virtude do teorema anterior
que qualquer valor múltiplo de d poderá ser pago usando apenas as notas de $a e $b.
Como todo valor pago, necessariamente é um múltiplo do máximo divisor comum de a e
b, descobrimos que o conjunto que procurávamos consiste precisamente do conjunto dos
múltiplos de d.

Observação 14. (Para professores) A prova mais comum apresentada para o teorema an-
terior baseia-se na análise do conjunto de todas as combinações lineares entre a e b e quase
sempre se preocupa apenas com mostrar a existência de x e y. Acreditamos que o algoritmo
para encontrar x e y facilite o entendimento do teorema para os alunos mais jovens. Entre-
tanto, frequentemente utilizemos apenas a parte da existência descrita no enunciado. Além
disso, preferimos discutir um exemplo numérico ao invés de formalizarmos uma prova e
sugerimos que o professor faça o mesmo com mais exemplos em aula.

Exemplo 15. (Olimı́ada Russa 1995) A sequência a1 , a2 , ... de naturais satisfaz mdc(ai , aj ) =
mdc(i, j) para todo i 6= j Prove que ai = i para todo i.

5
POT 2012 - Teoria dos Números - Nı́vel 2 - Aula 3 - Samuel Feitosa

Para qualquer inteiro n, mdc(a2n , an ) = mdc(2n, n) = n, consequentemente n | an . Seja


d um divisor qualquer de an diferente de n, então d | mdc(ad , an ). De mdc(ad , an ) =
mdc(d, n), podemos concluir que d | n. Sendo assim, todos os divisores de an que são
diferentes de n são divisores de n. Como já sabemos que an = nk, para algum k, não
podemos ter k > 1 pois nk não divide n e assim concluı́mos que an = n.

Exemplo 16. Mostre que mdc(2120 − 1, 2100 − 1) = 220 − 1.

Pelo lema de Euclides,

mdc(2120 − 1, 2100 − 1) = mdc(2120 − 1 − 220 (2100 − 1), 2100 − 1),


= mdc(220 − 1, 2100 − 1),
= mdc(220 − 1, 2100 − 1 − 280 (220 − 1)),
= mdc(220 − 1, 280 − 1),
= mdc(220 − 1, 280 − 1 − 260 (220 − 1)),
= mdc(220 − 1, 260 − 1),
= mdc(220 − 1, 260 − 1 − 240 (220 − 1)),
= mdc(220 − 1, 240 − 1),
= mdc(220 − 1, 240 − 1 − 220 (220 − 1)),
= mdc(220 − 1, 220 − 1) = 220 − 1.

Exemplo 17. (Olimpı́ada Russa 1964) Sejam x, y inteiros para os quais a fração

x2 + y 2
a=
xy
é inteira. Ache todos os possı́veis valores de a.

A primeira estratégia é cancelar os fatores comuns com o objetivo de reduzir o problema


ao caso em que x e y são primos entre si. Seja d = mdc(x, y), com

x = d · x0
, mdc(x0 , y0 ) = 1,
y = d · y0

então
x2 + y 2 x 0 2 + y0 2
a= = ·
xy x 0 y0
Nessa condição, como x0 divide y02 e y0 divide x20 , cada um deles é igual a 1, donde

12 + 12
a= = 2.
1·1

6
POT 2012 - Teoria dos Números - Nı́vel 2 - Aula 3 - Samuel Feitosa

Definição 18. Os inteiros a1 , a2 , . . . , an , todos diferentes de zero, possuem múltiplo comum


b se ai |b para i = 1, 2, . . . , n(note que a1 a2 . . . an é um múltiplo comum). O menor múltiplo
comum positivo para tal conjunto de inteiros é chamado de mı́nimo múltiplo comum e será
denotado por mmc(a1 , a2 , . . . , an ).
Proposição 19. Se a e b são não nulos, então: mmc(a, b) · mdc(a, b) = |ab|.
(A prova desta proposição também será deixada para a próxima seção)
Exemplo 20. (Olimpı́ada Russa 1995) Sejam m e n interios positivos tais que:
mmc(m, n) + mdc(m, n) = m + n.
Prove que um deles é divisı́vel pelo o outro.
Se d = mdc(m, n), então podemos escrever m = da e n = db. Pela proposição anterior,
d2 ab
mmc(m, n) = = dab.
d
Temos:
mmc(m, n) + mdc(m, n) − m − n = 0 ⇒
dab + d − da − db = 0 ⇒
ab + 1 − a − b = 0 ⇒
(a − 1)(b − 1) = 0.
Portanto, ou a = 1 e m | n ou então b = 1 e n | m.
Exemplo 21. (Torneio das Cidades 1998) Prove que, para quaisquer inteiros positivos a e
b, a equação mmc(a, a + 5) = mmc(b, b + 5) implica que a = b.
Para o item a), como (a + 5) − a = 5, temos mdc(a, a + 5) é igual a 1 ou 5. O mesmo vale
para mdc(b, b + 5). Pela proposição anterior, temos:

a(a + 5)
mmc(a, a + 5) = ,
mdc(a, a + 5)
b(b + 5)
mmc(b, b + 5) = .
mdc(b, b + 5)

Suponha que mdc(a, a + 5) = 5 e mdc(b, b + 5) = 1, então a(a + 5) = 5b(b + 5). Consequen-


temente, a é múltiplo de 5 e a(a + 5) é múltiplo de 25. Isso implica que b(b + 5) também é
múltiplo de 5 e que mdc(b, b + 5) > 1. Uma contradição. Analogamente, não podemos ter
mdc(a, a + 5) = 1 e mdc(b, b + 5) = 5. Sendo assim, mdc(a, a + 5) = mdc(b, b + 5) e:
a(a + 5) − b(b + 5) = 0 ⇒
(a − b)(a + b + 5) = 0.
Como a + b + 5 > 0, concluı́mos que a = b.

7
POT 2012 - Teoria dos Números - Nı́vel 2 - Aula 3 - Samuel Feitosa

Exemplo 22. Uma máquina f executa operações sobre o conjunto de todos os pares de
inteiros positivos. Para cada par de inteiros positivos, ela fornece um inteiro dado pelas
regras:
f (x, x) = x, f (x, y) = f (y, x), (x + y)f (x, y) = yf (x, x + y).
Determine f (2012, 2012! + 1).

Claramente mmc(x, x) = x e mmc(x, y) = mmc(y, x). Usando a proposição anterior e o


lema de Euclides temos:
xy x(x + y)
(x + y)mmc(x, y) = (x + y) =y· = y · mmc(x, x + y)
mdc(x, y) mdc(x, x + y)

Temos então uma forte suspeita de que f = mmc. Seja S o conjunto de todos os pa-
res de inteiros positivos (x, y) tais que f (x, y) 6= mmc(x, y), e seja (m, n) o par em S
com a soma m + n minima. Note que todo par da forma (n, n) não está em S pois
f (n, n) = n = mmc(n, n). Assim, devemos ter m 6= n. Suponha sem perda de generalidade
que n > m. Portanto:

nf (m, n − m) = [m + (n − m)]f (m, n − m) ⇒


= (n − m)f (m, m + (n − m)) ⇒
n−m
f (m, n − m) = · f (m, n)
n

Como o par (m, m − n) não está em S, dado que a soma de seus elementos é menor que
m + n, temos:

f (m, n − m) = mmc(m, n − m) ⇒
n−m
· f (m, n) = (n − m)mmc(m, m + (n − m)) ⇒
n
f (m, n) = mmc(m, n)

Uma contradição. Desse modo, S deve ser um conjunto vazio e f (x, y) = mmc(x, y)
para todos os pares de inteiros positivos. Como 2012 | 2012!, mdc(2012, 2012! + 1) = 1 e
consequentemente mmc(2012, 2012! + 1) = 2012(2012! + 1).

Problemas Propostos

Problema 23. Calcule:

a) mdc(n, n2 + n + 1).

b) mdc(3 × 2012, 2 × 2012 + 1).

8
POT 2012 - Teoria dos Números - Nı́vel 2 - Aula 3 - Samuel Feitosa

240 + 1 8
 
c) mdc ,2 + 1 .
28 + 1
Problema 24. Encontre mdc(2n + 13, n + 7)
12n+1
Problema 25. Prove que a fração 30n+2 é irredutı́vel.
a+b
Problema 26. Sejam a, b, c, d inteiros não nulos tais que ad − bc = 1. Prove que c+d é uma
fração irredutı́vel.
Problema 27. Mostre que mdc(am − 1, an − 1) = amdc(m,n) − 1.
Problema 28. Mostre que se mdc(a, b) = 1, então:

mdc(a + b, a2 − ab + b2 ) = 1 ou 3

Problema 29. Dado que mdc(a, 4) = 2, mdc(b, 4) = 2, prove que:

mdc(a + b, 4) = 4.

Problema 30. Prove que, para todo natural n,

mdc(n! + 1, (n + 1)! + 1) = 1.

Problema 31. No exemplo 4, determine todos os pares que podem ser obtidos começando-se
com o par (1, 2).
Problema 32. Qual o máximo divisor comum do conjunto de números:

{16n + 10n − 1, n = 1, 2, 3 . . .}?


a
Problema 33. A sequência Fn de Farey é a sequência de todos as frações irredutı́veis
b
com 0 ≤ a ≤ b ≤ n arranjados em ordem crescente.
F1 = {0/1, 1/1}
F2 = {0/1, 1/2, 1/1}
F3 = {0/1, 1/3, 1/2, 2/3, 1/1}
F4 = {0/1, 1/4, 1/3, 1/2, 2/3, 3/4, 1/1}
F5 = {0/1, 1/5, 1/4, 1/3, 2/5, 1/2, 3/5, 2/3, 3/4, 4/5, 1/1}
F6 = {0/1, 1/6, 1/5, 1/4, 1/3, 2/5, 1/2, 3/5, 2/3, 3/4, 4/5, 5/6, 1/1}

Claramente, toda fração ab < 1 com mdc(a, b) = 1, está em algum Fn . Mostre que se m/n
e m′ /n′ são frações consecutivas em Fn temos |mn′ − nm′ | = 1.
Problema 34. (Resvista Quantum - Jornal Kvant) Todas as frações irredutı́veis cujos de-
nominadores não excedem 99 são escritas em ordem crescente da esquerda para a direita:
1 1 a 5 c
, ,..., , , ,...
99 98 b 8 d
a c 5
Quais são as frações e em cada lado de ?
b d 8

9
POT 2012 - Teoria dos Números - Nı́vel 2 - Aula 3 - Samuel Feitosa

1 1 1
Problema 35. (OBM) Para cada inteiro positivo n > 1, prove que 1 + 2 + 3 +...+ n não
é inteiro.
1 1
Problema 36. Determine todas as soluções em inteiros positivos para a + b = 1c .

Problema 37. Inteiros positivos a e b, relativamente primos, são escolhidos de modo que
a+b
seja também um inteiro positivo. Prove que pelo menos um dos números ab + 1 e
a−b
4ab + 1 é um quadrado perfeito.

Problema 38. (IMO 1979) Sejam p, q números naturais primos entre si tais que:
p 1 1 1 1
= 1 − + − ... − + .
q 2 3 1318 1319
Prove que p é divisı́vel por 1979.

Respostas, Dicas e Soluções

23. (a)

mdc(n, n2 + n + 1) = mdc(n, n2 + n + 1 − n(n + 1)),


= mdc(n, 1),
= 1.

(b)

mdc(3 × 2012, 2 × 2012 + 1) = mdc(3 × 2012 − (2 × 2012 + 1), 2 × 2012 + 1),


= mdc(2012 − 1, 2 × 2012 + 1),
= mdc(2012 − 1, 2 × 2012 + 1 − 2(2012 − 1)),
= mdc(2012 − 1, 3),
= mdc(2012 − 1 − 3 × 670, 3),
= mdc(2, 3) = 1.

Outra opção seria observar que o mdc procurado deve dividir o número 3(2 ×
2012 + 1) − 2(3 × 2012) = 3 e que 2 × 2012 + 1 não é múltiplo de 3.
(c)

240 + 1 8
 
32 24 16 8 8

mdc , 2 + 1 = mdc 2 + 2 + 2 + 2 + 1, 2 + 1 ,
28 + 1
= mdc (232 − 1) + (224 + 1) + (216 − 1) + (28 + 1) + 1, 28 + 1 ,


= mdc(1, 28 + 1) = 1.

10
POT 2012 - Teoria dos Números - Nı́vel 2 - Aula 3 - Samuel Feitosa

24.

mdc(2n + 13, n + 7) = mdc(2n + 13 − 2(n + 7), n + 7),


= mdc(2n + 13 − 2(n + 7), n + 7),
= mdc(−1, n + 7) = 1

25.

mdc(12n + 1, 30n + 2) = mdc(12n + 1, 30n + 2 − 2(12n + 1)),


= mdc(12n + 1, 6n),
= mdc(12n + 1 − 2(6n), 6n),
= mdc(1, 6n) = 1

26. Seja f = mdc(a + b, c + d). Então f | d(a + b) − b(c + d) = 1 e consequentemente


f = 1.
27. Veja que

mdc(am − 1, an − 1) = mdc(am−n − 1 + (an − 1)am−n , an − 1)


= mdc(am−n − 1, an − 1)

O resultado segue aplicando o Algoritmo de Euclides aos expoentes.


28. Seja f = mdc(a + b, a2 − ab + b2 ). Então f | (a + b)2 − (a2 − ab + b2 ) = 3ab. Se
mdc(f, a) > 0, devemos ter mdc(f, b) > 0 pois f | a + b. O mesmo argumento vale
para mdc(f, b) > 0. Assim, mdc(f, a) = mdc(f, b) = 1. Portanto, f | 3.
30. Pelo lema de Euclides,

mdc(n! + 1, (n + 1)! + 1) = mdc(n! + 1, (n + 1)! + 1 − (n + 1)(n! + 1))


= mdc(n! + 1, −n)
= mdc(n! + 1 − n[(n − 1)!], −n) = 1

34. Sejam l = mmc{1, 2, . . . , n} e ai = l/i. A soma considerada é


a1 + a2 + . . . + an
.
l
Queremos analisar o expoente do fator 2 no numerador e no denominador. Seja k tal
que 2k ≤ n < 2k+1 . Então 2k ||l e ai é par para todo i 6= 2k . Como a2k é ı́mpar, segue
que o numerador é ı́mpar enquanto que o denominador é par. Consequentemente a
fração anterior não representa um inteiro.

11
36. Sejam d = mdc(a, b), a = dx, b = dy. Consequentemente mdc(x, y) = 1 e podemos
escrever a equação como:

1 1 1
+ = ⇒
a b c
bc + ac = ab
dyc + dxc = d2 xy
c(x + y) = dxy

Como mdc(xy, x + y) = 1 pois mdc(x, y) = 1, devemos ter xy | c e consequentemente


c = xyk. Assim, d = k(x + y). O conjunto solução é formado pelas triplas (a, b, c)
onde (a, b, c) = (kx(x + y), ky(x + y), xyk) com mdc(x, y) = 1 e x, y e k inteiros
positivos.

38. Use a identidade de Catalão:

1 1 1 1 1 1 1
1− + − + ... − = + + ... +
2 3 4 2n n+1 n+2 2n
1 1
Em seguida, agrupe os termos da forma + e analise o numerador da
n + i 2n − i + 1
fração obtida.

Referências
[1] S. B. Feitosa, B. Holanda, Y. Lima and C. T. Magalhães, Treinamento Cone Sul 2008.
Fortaleza, Ed. Realce, 2010.

[2] D. Fomin, A. Kirichenko, Leningrad Mathematical Olympiads 1987-1991, MathPro


Press, Westford, MA, 1994.

[3] D. Fomin, S. Genkin and I. Itenberg, Mathematical Circles, Mathematical Words, Vol.
7, American Mathematical Society, Boston, MA, 1966.

[4] I. Niven, H. S. Zuckerman, and H. L. Montgomery, An Introduction to the Theory of


Numbers.
Polos Olímpicos de Treinamento
Curso de Teoria dos Números - Nível 2 Aula 4
Prof. Samuel Feitosa

Números Primos, MDC e MMC.

Definição 1. Um inteiro p > 1 é chamado número primo se não possui um divisor d


satisfazendo 1 < d < p. Se um inteiro a > 1 não é primo, ele é chamado de número
composto. Um inteiro m é chamado de composto se |m| não é primo.

O próximo teorema nos diz que os primos são as ”peças”fundamentais dos números inteiros:

Teorema 2. Todo inteiro n, maior que 1, pode ser expresso como o produto de número
primo.

Demonstração. Se o inteiro n é um primo, então ele mesmo é o produto de um único fa-


tor primo. Se o inteiro n não é primo, existe uma decomposição do tipo: n = n1 n2 com
1 < n1 < n e 1 < n2 < n. Repetindo o argumento para n1 e n2 , podemos escrever n como
o produto de primos ou podemos obter parcelas menores escrevendo n como um produto
de naturais. Como não existe uma sucessão infinita de naturais cada vez menores, após um
número finito de operações desse tipo, poderemos escrever n como um produto de números
primos.

Quantos números primos existem?

Teorema 3. (Euclides) Existem infinitos números primos.

Demonstração. Suponha, por absurdo, que exita apenas uma quantidade finita de primos:
p1 , p2 , . . . , pn . Considere o número X = p1 p1 . . . pn + 1. Pelo teorema anterior, esse número
deve ser o produto de alguns elementos do conjunto de todos os números primos. Entre-
tanto, nenhum dos primos pi divide X.
Exemplo 4. Existe um bloco de 1000 inteiros consecutivos não contendo nenhum primo?
Sim. Um exemplo é o conjunto 1001! + 2, 1001! + 3, . . . , 1001! + 1001. Veja i | 1001! + i para
todo i = 2, 3, . . . , 1001.
POT 2012 - Teoria dos Números - Nı́vel 2 - Aula 4 - Samuel Feitosa

Exemplo 5. (Torneio das Cidades) Existe um bloco de 1000 inteiros consecutivos contendo
apenas um primo?
Para cada bloco de 1000 números consecutivos, contemos sua quantidade de números pri-
mos. Por exemplo, no bloco 1, 2, 3, . . . , 1000, temos 168 números primos (mas só usaremos
o fato de que existem mais de dois primos nesse bloco). Comparando os blocos consecuti-
vos k + 1, k + 2, . . . , k + 1000 e k + 2, k + 3, . . . , k + 1001, ou o número de números primos
aumenta em uma unidade, ou fica constante ou diminui em uma unidade. Analisando to-
dos os blocos consecutivos desde 1, 2, . . . , 1000 até 1001! + 2, 1001! + 3, . . . , 1001! + 1001,
o número de números primos deve ser igual à 1 em algum deles. Para ver isso, usare-
mos um argumento de continuidade discreta: Começando com o número 168 e realizando
alterações de no máximo uma unidade na quantidade de primos em cada bloco, para che-
garmos no número 0, necessariamente deveremos passar pelo número 1 em algum momento.

Relembremos um importante resultado da aula passada:


Teorema 6. (Bachet- Bèzout) Se d = mdc(a, b), então existem inteiros x e y tais que
ax + by = d.
Proposição 7. Sejam a, b e c inteiros positivos com a | bc e mdc(a, b) = 1. Então, a | c.

Demonstração. Pelo teorema anterior, existem x e y inteiros tais que ax + by = 1. Assim,


acx + bcy = c. Como a | acx e a | bcy, podemos concluir que a | c.

Em particular, se p é um número primo e p | ab, então p | a ou p | b. Podemos usar esse


fato para garantir a unicidade em nosso primeiro teorema, obtendo o importante:
Teorema 8. (Teorema Fundamental da Aritmética) A fatoração de qualquer inteiro n > 1,
em fatores primos, é única a menos da ordem dos fatores.
Exemplo 9. (Rússia 1995) É possı́vel colocarmos 1995 números naturais ao redor de um
cı́rculo de modo que para quaisquer dois números vizinhos a razão entre o maior e o menor
seja um número primo?
Não, é impossı́vel. Suponha, por absurdo, que isso seja possı́vel e denotemos por
a
a0 , a1 , . . . , a1995 = a0 tais inteiros. Então, para k = 1, . . . , 1995, k−1
ak é primo ou o in-
verso de um primo. Suponha que a primeira situação ocorra m vezes e a segunda ocorra
1995 − m vezes entre esses quocientes. Como o produto de todos os números da forma
ak−1
ak , para k = 1, . . . , 1995 é igual à 1, podemos concluir que o produto de m primos deve
ser igual ao produto de 1995 − m primos. Em virtude da fatoração única, m = 1995 − m.
Um absurdo pois 1995 é ı́mpar.
Proposição 10. Se as fatorações em primos de n e m são:

n = pα1 1 pα2 2 . . . pαk k ,


m = pβ1 1 pβ2 2 . . . pβk k .

2
POT 2012 - Teoria dos Números - Nı́vel 2 - Aula 4 - Samuel Feitosa

Então, mdc(m, n) = pγ11 pγ22 . . . pγkk e mmc(m, n) = pθ11 pθ22 . . . pθkk , onde γi é o menor dentre
{αi , βi } e θi é o maior dentre {αi , βi }.
Proposição 11. Se a e b são inteiros positivos, mostre que mmc(a, b)mdc(a, b) = ab.

Demonstração. Basta usar a proposição anterior e observar que:

max{x, y} + min{x, y} = x + y.

Exemplo 12. (Torneio das Cidades 1998) É possı́vel que mmc(a, b) = mmc(a + c, b + c)
para alguma conjunto {a, b, c} de inteiros positivos?
Não. Suponha que a + c e b + c possuem algum divisor primo p. Como p | mmc(a + c, b + c),
caso existam tais inteiros, devemos ter que p | mmc(a, b). Assim, usando que pelo menos
um dentre a e b é divisı́vel por p podemos concluir que c também é divisı́vel por p. Então,
podemos cancelar o fator p:
   
a b mmc(a, b) mmc(a + c, b + c) a+c b+c
mmc , = = = mmc , .
p p p p p p

Efetuando alguns cancelamentos, podemos supor então que a+c e b+c não possuem fatores
primos em comum. Obtivemos um absurdo pois:

mmc(a + c, b + c) = (a + c)(b + c) > ab ≥ mmc(a, b).

Exemplo 13. (OCM 2005) Determinar os inteiros n > 2 que são divisı́veis por todos os
primos menores que n.
Como mdc(n, n − 1) = 1, se n − 1 possui algum fator primo, ele não dividirá n. Assim,
n − 1 < 2. Consequentemente não existe tal inteiro.
Exemplo 14. Mostre que n4 + n2 + 1 é composto para n >1.
Veja que n4 + n2 + 1 = n4 + 2n2 + 1 − n2 = (n2 + 1)2 − n2 = (n2 + n + 1)(n2 − n + 1).
Para n > 1, n2 − n + 1 = n(n − 1) + 1 > 1 e assim n4 + n2 + 1 é o produto de dois inteiros
maiores que 1.
Exemplo 15. Mostre que n4 + 4n é composto para todo n > 1.
Se n é par, certamente o número em questão é divisı́vel por 4. Para o caso em que n é
impar, iremos usar a fatoração:

a4 + 4b4 = a4 + 4a2 b2 + 4b4 − 4a2 b2 = (a2 + 2b2 ) − 4b2 b2 = (a2 − 2ab + 2b2 )(a2 + 2ab + 2b2 ).

Para n da forma 4k + 1, faça a = n e b = 4k . Para n da forma 4k + 3, faça a = n e


b = 22k+1 .
Exemplo 16. Se 2n + 1 é um primo ı́mpar para algum inteiro positivo n, prove que n é uma
potência de 2.

3
POT 2012 - Teoria dos Números - Nı́vel 2 - Aula 4 - Samuel Feitosa

Já vimos que an − 1 = (a − 1)(an−1 + an−2 + . . . + 1). Se n é impar,

(−a)n − 1 = (−a − 1)((−a)n−1 + (−a)n−2 + . . . + 1) ⇒


an + 1 = (a + 1)(an−1 − an−2 + . . . − a + 1)

Sendo assim, se n possuı́sse algum divisor primo ı́mpar p com n = pb, poderı́amos escrever:
2n + 1 = (a + 1)(an−1 − an−2 + . . . − a + 1), onde a = 2b . Como an−1 − an−2 + . . . − a + 1 > 1,
o número 2n + 1 não seria primo.
Exemplo 17. Dados que p, p + 10 e p + 14 são números primos, encontre p.
Vamos analisar os possı́veis restos na divisão por 3 de p. Se p deixa resto 1, então p + 14
é um múltiplo de 3 maior que 3 e consequentemente não poderá ser um número primo. Se
o resto é 2, então p + 10 é um múltiplo de 3 maior que 3 e também não poderá ser um
número primo. Assim, o resto de p por 3 é 0 e consequentemente p = 3.
n
Exemplo 18. (Áustria-Polônia) Dados naturais n e a > 3 ı́mpar, mostre que a2 − 1 tem
pelo menos n + 1 divisores primos distintos.
Usando a fatoração da diferença de quadrados, temos que:
k k−1 k−2
a2 − 1 = (a2 + 1)(a2 + 1) . . . (a + 1)(a − 1).
m k
Assim, a2 + 1 | a2 − 1 se k > m. Como a é impar, podemos concluir que:
k m k m m
mdc(a2 + 1, a2 + 1) = mdc(a2 − 1 + 2, a2 + 1) = mdc(2, a2 + 1) = 2.

Sendo assim, na fatoração:


n n−1 n−2
a2 − 1 (a2 + 1) (a2 + 1) (a + 1) (a − 1)
n
= ... ,
2 2 2 2 2
temos o produto de pelo menos n inteiros primos entre si e consequentemente seus fatores
2i
primos são distintos. Para cada termo (a 2+1) , temos um fator primo pi+1 diferente de 2.
n
Daı́, a2 − 1 possui pelo menos n + 1 fatores primos distintos, a saber, {2, p1 , p2 , . . . , pn }.
Exemplo 19. (Rioplatense 1999) Sejam p1 , p2 , . . . , pk primos distintos. Considere todos os
inteiros positivos que utilizam apenas esses primos (não necessariamente todos) em sua
fatoração em números primos, formando assim uma seqüência infinita

a1 < a2 < · · · < an < · · · .

Demonstre que, para cada natural c, existe um natural n tal que

an+1 − an > c.

4
POT 2012 - Teoria dos Números - Nı́vel 2 - Aula 4 - Samuel Feitosa

Suponha, por absurdo, que exista c > 0 tal que an+1 − an ≤ c, ∀ n ∈ N. Isso significa que
as diferenças entre os termos consecutivos de (an )n≥1 pertencem ao conjunto {1, 2, . . . , c},
logo são finitas. Sejam d1 , d2 , . . . , dr essas diferenças. Seja αi o maior expoente de pi que
aparece na fatoração de todos os dj .

Considere então o número M = p1α1 +1 pα2 2 +1 · · · pαk k +1 . É claro que M pertence à seqüência,
ou seja, M = an , para algum n. Vejamos quem será an+1 . Por hipótese, existe i tal que
an+1 − an = di . Como an+1 > an , existe um primo pj que divide an+1 com expoente maior
ou igual a αj + 1. Caso contrário,

an < an+1 < pα1 1 +1 pα2 2 +1 · · · pαk k +1 = an ,


α +1 α +1
absurdo. Daı́, pj j |an ⇒ pj j |di , novamente um absurdo, pela maximalidade de αj .

Logo, o conjunto de todas as diferenças não pode ser finito e, portanto, dado qualquer
c > 0, existe um natural n tal que an+1 − an > c.

Problemas Propostos

Problema 20. Dado que p, 2p + 1 e 4p2 + 1 são números primos, encontre p.


Problema 21. Dado o par de primos p e 8p2 + 1, encontre p.
Problema 22. Dado o par de primos p e p2 + 2, prove que p3 + 2 também é um número
primo.
Problema 23. Dado que p, 4p2 + 1 e 6p2 + 1 são números primos, encontre p.
n
Problema 24. Os números de Fermat são os números da forma 22 + 1. Prove que o
conjunto dos divisores primos dos termos da seqüência de Fermat é infinito.
Problema 25. Mostre que todo inteiro n pode ser escrito de maneira única na forma n = ab,
onde a é um inteiro livre de quadrado e b é um quadrado perfeito. Um inteiro é dito livre
de quadrado se não é divisı́vel por nenhum quadrado perfeito maior que 1.
Problema 26. Prove que todo primo maior que 3 é da forma 6k+1 ou 6k+5.
Problema 27. Prove que todo inteiro da forma 3k+2 tem um fator primo da mesma forma.
Problema 28. Prove que existem infinitos primos da forma 4k+3 e 6k +5.

Problema 29. Prove que se n é composto, então possui um fator primo p ≤ n.
Problema 30. (OBM 1998) São dados 15 números naturais maiores que 1 e menores que
1998 tais que dois quaisquer são primos entre si. Mostre que pelo menos um desses 15
números é primo.
Problema 31. Mostre que n|(n-1)! para todo número composto n.

5
POT 2012 - Teoria dos Números - Nı́vel 2 - Aula 4 - Samuel Feitosa

Problema 32. Suponha que n >1. Mostre que a soma dos inteiros dos inteiros positivos
não excedendo n divide o produto dos inteiros positivos não excedendo n se, e somente se,
n é composto.
Exemplo 33. (Rússia 1995) Encontre todos os primos p para os quais p2 + 11 tenha exata-
mente seis divisores distintos, incluindo 1 e p2 + 11.
Problema 34. (Irlanda 2002 ) Encontre todas as soluções inteiras positivas de p(p + 3) +
q(q + 3) = n(n + 3), onde p, q são primos.
Exemplo 35. Prove que qualquer quadrado perfeito positivo tem mais divisores que deixam
resto 1 na divisão por 3 do que divisores que deixam resto 2 na divisão por 3.

Dicas e Soluções

19. Analisemos o resto de p na divisão por 3. Se p deixar resto 1, o número 2p + 1 será


divisı́vel por 3. Se p deixar resto 2, o número 4p + 1 será divisı́vel por 3. Em ambos
os casos, 2p + 1, 4p + 1 > 3 e obtemos assim um absurdo.

20. Analisemos o resto de p na divisão por 3. Se p deixa resto 1 ou 2, p2 deixa resto 1


e consequentemente 8p2 + 1 deixa resto 0 por 3 mas certamente é maior que 3. Um
absurdo, logo p = 3.

21. Analisemos o resto na divisão por 3. Se p não é múltiplo de 3, p2 + 2 é divisı́vel por


3 e maior que 3. Um absurdo, logo p = 3 e p3 + 2 = 29.

22. Analise os restos na divisão por 5.

23. Iremos usar a fatoração do exemplo 17:


n n−1 n−2
22 − 1 = (22 + 1)(22 + 1) . . . (2 + 1)(2 − 1).

Assim, se k > m,
k m k m m
mdc(22 + 1, 22 + 1) = mdc(22 − 1 + 2, 22 + 1) = mdc(2, 22 + 1) = 1,

produzindo que quaisquer dois números de Fermat distintos são primos entre si e isso
necessariamente implica que o conjunto de seus divisores primos é infinito.

24. Analise os restos na divisão por 2 e 3.

27. Tente imitar a prova de Euclides para a existência de infinitos primos.

29. Se n é composto, podemos escrever n = ab com 1 < a ≤ b ≤<. Assim, a2 ≤ n e



a ≤ n. Para terminar, basta considerar qualquer divisor primo de a.

30. Dado 1 < n < 1998, se ele não for primo, usando o exercı́cio anterior, ele tem que
ter um fator primo menor que 1998, ou seja, um fator primo menor que 45. Como só
existem 14 primos menores que 45, e são 15 números, um deles será primo.

6
31. Escreva n = ab e analise as aparições de a e b no produto (n − 1) · (n − 2) . . . 2 · 1.

33. Se p 6= 3, 3 | p2 + 11. Analogamente, se p 6= 2, 4 | p2 + 11. Assim, exceto nes-


ses dois casos, 12 | p2 + 11 e podemos encontrar mais que 6 divisores distintos:
{1, 2, 3, 4, 6, 12, p2 + 11}. Agora, teste p = 2 e p = 3 para verificar que p = 3 é a única
solução.

34. Seja
n = 3γ · pα1 1 · · · pαnn · q1β1 · · · qm
βm

a decomposição de n em fatores primos, onde cada pi deixa resto 1 por 3 e cada qj


deixa resto 2 por 3. Então

n2 = 32γ · p2α 2αn


1 · · · pn
1
· q12β1 · · · qm
2βm
.

Um divisor de n2 deixa resto 1 por 3 se e somente se possuir uma quantidade par de


primos qj , contados com repetição. Mais especificamente, se e somente se a soma dos
expoentes de q1 , . . . , qm for par. Assim, a quantidade de divisores dessa forma é igual
a
 
1
D1 = (2α1 + 1) · · · (2αn + 1) (2β1 + 1)(2β2 + 1) · · · (2βm + 1) + 1 .
2
Enquanto para se obter um divisor que deixe resto 2 por 3, precisamos de uma
quantidade ı́mpar de fatores primos da forma 3k+2. Assim, a quantidade de divisores
dessa forma é:
 
1
D2 := (2α1 + 1)(2α2 + 1) · · · (2αn + 1) (2β1 + 1)(2β2 + 1) · · · (2βm + 1) .
2
Daı́, segue facilmente que D1 > D2 .

Referências
[1] E. Carneiro, O. Campos and F. Paiva, Olimpı́adas Cearenses de Matemática 1981-2005
(Nı́veis Júnior e Senior), Ed. Realce, 2005.

[2] S. B. Feitosa, B. Holanda, Y. Lima and C. T. Magalhães, Treinamento Cone Sul 2008.
Fortaleza, Ed. Realce, 2010.

[3] D. Fomin, A. Kirichenko, Leningrad Mathematical Olympiads 1987-1991, MathPro


Press, Westford, MA, 1994.

[4] D. Fomin, S. Genkin and I. Itenberg, Mathematical Circles, Mathematical Words, Vol.
7, American Mathematical Society, Boston, MA, 1966.

[5] I. Niven, H. S. Zuckerman, and H. L. Montgomery, An Introduction to the Theory of


Numbers.
Polos Olímpicos de Treinamento
Curso de Teoria dos Números - Nível 2 Aula 6
Prof. Samuel Feitosa

Congruências I

Definição 1. Dizemos que os inteiros a e b são congrentes módulo m se eles deixam o


mesmo resto quando divididos por m. Denotaremos isso por a ≡ b (mod m).
Por exemplo, 7 ≡ 2 (mod 5), 9 ≡ 3 (mod 6), 37 ≡ 7 (mod 10) mas 5 6≡ 3 (mod 4). Veja
que a ≡ b (mod m) se, e somente se, m | a − b.
Teorema 2. Se a ≡ b (mod m) e c ≡ d (mod m), então:

i) a + c ≡ b + d (mod m)

ii) a − c ≡ b − d (mod m)

iii) ka ≡ kb (mod m) ∀k ∈ Z

iv) ac ≡ bd (mod m)

v) ak ≡ bk (mod m) ∀k ∈ N

vi) Se mdc(k, m) = d, então ka ≡ kb (mod m) ⇔ a ≡ b (mod m/d)

Demonstração. Sejam q1 e q2 tais que:

a − b = q1 m
c − d = q2 m

Então, (a + c) − (b + d) = (q1 + q2 )m. Logo, a + c e b + d deixam o mesmo resto por m e


consequentemente a + c ≡ b + d (mod m). Usando que a − b (mod a)k − bk e que m | a − b,
concluı́mos que m (mod a)k − bk . Os demais itens serão deixados para o leitor.

Em termos práticos, podemos realizar quase todas as operações elementares envolvendo


igualdade de inteiros. Uma das diferenças cruciais é a operação de divisão como mostra o
último item do teorema anterior.
POT 2012 - Teoria dos Números - Nı́vel 2 - Aula 5 - Samuel Feitosa

Exemplo 3. Calcule o resto de 4100 por 3.


Como 4 ≡ 1 (mod 3), temos 4100 ≡ 1100 = 1 (mod 3).
Exemplo 4. Calcule o resto de 4100 por 5.
Como 4 ≡ −1 (mod 5), temos 4100 ≡ (−1)100 = 1 (mod 5).
Exemplo 5. Calcule o resto de 4100 por 7.
Você deve ter percebido que encontrar relações do tipo a ≡ ±1 (mod m) podem simplificar
bastante o cálculo de ak (mod m). Procuremos alguma relação como essa para 4 e 7. Veja
que:
40 ≡ 1 (mod 7), 41 ≡ 4 (mod 7), 42 ≡ 2 (mod 7), 43 ≡ 1 (mod 7).
Assim,
499 = (43 )33 ≡ 133 = 1 (mod 7).
Como 43 ≡ 1 (mod 7), os restos das potências de 4 na divisão por 7 se repetem periodica-
mente de 3 em 3 pois 43k+r ≡ 43k · 4r ≡ 4r (mod 7).
Exemplo 6. Qual o resto de 3636 + 4141 na divisão por 77?
Inicialmente devemos perceber que existe uma relação entre os números do problema: 36 +
41 = 77. Assim:

−36 ≡ 41 (mod 77),


(−36)41 ≡ 4141 (mod 77),
36 5 36
36 (1 − 36 ) ≡ 36 + 4141 (mod 77).

Nosso próximo passo é encontrar o resto de 365 na divisão por 77. Como 36 ≡ 1 (mod 7),
365 ≡ 1 (mod 7). Além disso, 36 ≡ 3 (mod 1)1 produzindo 365 ≡ 35 ≡ 1 (mod 1)1.
Como mdc(7, 11) = 1 e ambos dividem 365 − 1, podemos concluir que 77 | 365 − 1. Logo,
3636 + 4141 deixa resto 0 na divisão por 77.
Exemplo 7. Prove que p2 − 1 é divisı́vel por 24 se p é um primo maior que 3.
Se p é um primo maior que 3, p ≡ ±1 (mod 3) e p ≡ 1 (mod 2). Daı́, p2 ≡ 1 (mod 3).
Além disso, se p = 2k + 1, segue que p2 = 4k(k + 1) + 1 ≡ 1 (mod 8) pois k(k + 1) é par.
Como mdc(8, 3) = 1 e ambos dividem p2 − 1, segue que 24 | p2 − 1.
Exemplo 8. (OCM-2001) Achar o menor natural n tal que 2001 é a soma dos quadrados
de n inteiros
Podemos concluir da solução do problema anterior que todo todo inteiro ı́mpar ao quadrado
deixa resto 1 por 8. Usemos isso para estimar o valor de n. Sejam x1 , x2 , . . . , xn inteiros
ı́mpares tais que:
x21 + x22 + . . . x2n = 2001.

2
POT 2012 - Teoria dos Números - Nı́vel 2 - Aula 5 - Samuel Feitosa

Analisando a congruência módulo 8, obtemos:

x21 + x22 + . . . x2n = 2001 (mod 8)


1 + 1 + ... + 1 ≡ 1 (mod 8)
n ≡ 1 (mod 8)

Como 2001 não é quadrado perfeito, não podemos ter n = 1. O próximo candidado para
n seria 1 + 8 = 9. Se exibirmos um exemplo para n = 9, teremos achado o valor mı́nimo.
Veja que:
2001 = 432 + 112 + 52 + 12 + 12 + 12 + 12 + 12 + 12 .
Exemplo 9. (IMO) Seja s(n) a soma dos dı́gitos de n. Se N = 44444444 , A = s(N ) e
B = s(A). Quanto vale s(B)?
Pelo critério de divisibilidade por 9, N ≡ A ≡ B (mod 9). Inicialmente calculemos o
resto de N por 9. Como 4444 ≡ 16 ≡ 7 (mod 9), precisamos encontrar 74444 (mod 9).
Seguindo os métodos dos primeiros exemplos, seria interessante encontrarmos um inteiro r
tal que 7r ≡ ±1 (mod 9). O menor inteiro positivo com essa propriedade é r = 3. Como
4444 = 1481 · 3 + 1, temos:

74444 ≡ 71481·3+1 ≡ (73 )1481 · 7 ≡ 7 (mod 9).

Nosso próximo passo é estimar o valor de s(B). Como N = 44444444 < 105·4444 , A =
s(N ) ≤ 5 · 4444 · 9 = 199980. Além disso, B = s(A) ≤ 1 + 9 · 5 = 46 e s(B) ≤ 12. O único
inteiro menor ou igual a 12 com resto 7 por 9 é o próprio 7, daı́ s(B) = 7.
Exemplo 10. Prove que 11n+2 + 122n+1 é divisı́vel por 133 para qualquer natural n.
Duas relações que podemos extrair dos números envolvidos são: 144−11 = 133 e 133−12 =
121. Assim:

144 ≡ 11 (mod 133),


122 ≡ 11 (mod 133),
122n ≡ 11n (mod 133),
2n+1 n
12 ≡ 11 · 12 (mod 133),
122n+1
≡ 11 · (−121) + 133 · 11n
n
(mod 133),
2n+1 n+2
12 ≡ −11 (mod 133).

Exemplo 11. Prove que n5 + 4n é divisı́vel por 5 para todo inteiro n


Inicialmente note que n5 + 4n = n(n4 + 4). Se n ≡ 0 (mod 5), não há o que fazer. Se
n ≡ ±1 (mod 5), n4 +4 ≡ 1+4 = 0 (mod 5). Finalmente, se n ≡ ±2 (mod 5), n2 ≡ 4 ≡ −1
(mod 5) e consequentemente n4 + 4 ≡ 1 + 4 = 0 (mod 5).
Exemplo 12. Seja n > 6 um inteiro positivo tal que n − 1 e n + 1 são primos. Mostre que
n2 (n2 + 16) é divisı́vel por 720. A recı́proca é verdadeira?

3
POT 2012 - Teoria dos Números - Nı́vel 2 - Aula 5 - Samuel Feitosa

Veja que n é da forma 6k, pois n − 1 e n + 1 são primos maiores que 3, portanto da forma
6k − 1 e 6k + 1, respectivamente. Logo,

n2 (n2 + 16) = 144(9k 4 + 4k 2 ).

Resta provar que 9k 4 + 4k 2 é um múltiplo de 5. Vamos analisar a igualdade acima módulo


5.

i) Se k ≡ 0, 2 ou 3 (mod 5), temos 9k 4 + 4k 2 ≡ 0 (mod 5);

ii) Se k ≡ 1 (mod 5) ⇒ n ≡ 1 (mod 5), temos n − 1 ≡ 0 (mod 5), um absurdo;

iii) Se k ≡ 4 (mod 5) ⇒ n ≡ 4 (mod 5), temos n+1 ≡ 0 (mod 5), novamente um absurdo.

Isso conclui a demonstração. A recı́proca não é verdadeira. Basta tomar, por exemplo,
n = 90.

Problemas Propostos

Problema 13. Determine o resto de 220 − 1 na divisão por 41.


Problema 14. Qual o resto de 12000 + 22000 + . . . + 20002000 na divisão por 7?
Problema 15. Qual o resto na divisão de 270 + 370 por 13?
Problema 16. Qual o resto de 3200 por 100?
Problema 17. (Estônia 2000) Determine todos os possı́veis restos da divisão do quadrado
de um número primo com o 120 por 120.
Problema 18. Qual o último dı́gito de 777777 ?
Exemplo 19. Prove que 22225555 + 55552222 é divisı́vel por 7.
Problema 20. Prove que o número n3 + 2n é divisı́vel por 3 para todo natural n.
Problema 21. Prove que n2 + 1 não é divisı́vel por 3 para nenhum n inteiro.
Problema 22. Prove que n3 + 2 não é divisı́vel por 9 para nenhum n inteiro.
Problema 23. Prove que p2 − q 2 é divisı́vel por 24 se p e q são primos maiores que 3.
Problema 24. Prove que se 2n + 1 e 3n + 1 são ambos quadrados perfeitos, então n é
divisı́vel por 40.
Problema 25. Se n é ı́mpar, prove que 7|22n+1 + 3n+2 .
Problema 26. Seja d(n) a soma dos dı́gitos de n. Suponha que n + d(n) + d(d(n)) = 1995.
Quais os possı́veis restos da divisão de n por 9?
Problema 27. Prove que não existem inteiros positivos x1 , x2 , . . . , x14 tais que:

x41 + x42 + . . . + x414 = 1599.

4
POT 2012 - Teoria dos Números - Nı́vel 2 - Aula 5 - Samuel Feitosa

Problema 28. Escreva uma única congruência que é equivalente ao par de congruências
x ≡ 1 (mod 4) e x ≡ 2 (mod 3).
Problema 29. Prove que 2015 − 1 é divisı́vel por 11 · 31 · 61
Problema 30. (Alemanha 1997) Determine todos os primos p para os quais o sistema
p + 1 = 2x2
p2 + 1 = 2y 2
tem uma solução nos inteiros x, y.
Problema 31. Mostre que se n divide um número de Fibonacci então ele dividirá uma
infinidade.
Dicas e Soluções

13. Veja que


25 = 32 ≡ −9 (mod 41) ⇒
210 ≡ 81 ≡ −1 (mod 42) ⇒
220 ≡ 1 (mod 41).
Assim, o resto procurado é zero.
14. Como i2000 ≡ (i + 7k)2000 (mod 7), podemos simplificar o problema calculando pri-
meiramente o valor de:
12000 + 22000 + 32000 + 42000 + 52000 + 62000 + 72000 (mod 7).
Outra observação importante que simplificará o cálculo é perceber que 23 ≡ 1 (mod 7).
Assim,
23k ≡ 1 (mod 7), 23k+1 ≡ 2 (mod 7), e 23k+2 ≡ 4 (mod 7).
Usando isso e o fato de que 2000 é par, temos:
12000 + 22000 + 32000 + 42000 + 52000 + 62000 + 72000 ≡
12000 + 22000 + (−4)2000 + 42000 + (−2)2000 + (−1)2000 + 02000 ≡
≡ 1+4+2+2+4+1+0
≡ 0 (mod 7).
Dentre os primeiros 2000 naturais consecutivos, podemos formar 285 grupos de 7
números consecutivos cuja soma é múltipla de 7, em virtude da soma anterior. Os
cinco números restantes possuem como resto na divisão por 7 o número:
19962000 + 19972000 + 19982000 + 19992000 + 20002000 ≡ 1 + 4 + 2 + 2 + 4
≡ 6 (mod 7).
Assim, o resto da soma na divisão por 7 é 6.

5
POT 2012 - Teoria dos Números - Nı́vel 2 - Aula 5 - Samuel Feitosa

15. Inicialmente é interessante buscarmos alguma relação entre os números envolvidos no


problema. Como 13 = 4 + 9, podemos escrever:
9 ≡ −4 (mod 13) ⇒
35
9 ≡ (−4)35 (mod 13) ⇒
70 70
3 +2 ≡ 0 (mod 13).

17. Use a fatoração 120 = 3 · 5 · 23 e analise a congruência módulo 3, 5 e 8 separadamente.


18. Se n não é múltiplo de 3, sabemos que n2 ≡ 1 (mod 3). Assim n2 + 2 ≡ 0 (mod 3).
Se n é múltiplo de 3, n ≡ 0 (mod 3). Em qualquer caso, n(n2 + 2) ≡ 0 (mod 3).
19. Basta repetir a análise do problema anterior
20. Podemos montar uma tabela de congruências na divisão por 9:
n 0 1 2 3 4 5 6 7 8
n3 0 1 8 0 1 8 0 1 8

Como nenhum cubo perfeito diexa resto 7 na divisão por 9, n3 + 2 6≡ 0 (mod 9).
23. Proceda como no exemplo 7.
25.
22n+1 + 3n+2 ≡ 4n · 2 + 3n · 9
≡ (−3)n · 2 + 3n · 2
≡ 0 (mod 7).

26. Seja r o resto na divisão por 9 de n. Pelo critério de divisibilidade por 9, temos:
n + d(n) + d(d(n)) ≡ 3r ≡ 1995 (mod 9).

Assim, r ≡ 2 (mod 3) (Pela propriedade vi do teorema 2). Além disso,


n ≤ 1995 ⇒
d(n) ≤ 27 = d(1989) ⇒
d(d(n)) ≤ 10 = d(19).

Consequentemente, n ≥ 1995 − d(n) − d(d(n)) ≥ 1958. Basta procurarmos nos


conjunto {1958, 1959, . . . , 1995} os inteiros que deixam resto 2 por 3 e que satisfazem
a equação do problema. Nesse conjunto, apena o inteiro 1967 cumpre essas condições.
27. Estudando a congruência módulo 16, podemos mostrar que x4 ≡ 0 ou 1 (mod 1)6.
Assim, a soma
x41 + x42 + . . . + x414
é congruente a um dos números do conjunto {0, 1, . . . , 14} m odulo 16 enquanto que
1599 ≡ 15 (mod 16). Um absurdo.

6
28. x ≡ 5 (mod 12).

30. Suponha sem perda de generalidade que x, y ≥ 0. Como p + 1 é par, p 6= 2. Além


disso,
2x2 ≡ 1 ≡ 2y 2 (mod p)
e consequentente, usando que p é ı́mpar, x ≡ ±y (mod p). Como x < y < p, temos

p2 + 1 = 2(p − x)2 = 2p2 − 4px + p + 1,

de modo que p = 4x − 1, 2x2 = 4x. Podemos concluir que x é 0 ou 2 e que a única


possibilidade para p é p = 7.

31. Em virtude da fórmula recursiva da sequência de Fibonacci, é possı́vel mostrarmos


que os restos de seus termos na divisão por qualquer número formam uma sequência
periódica.

Referências
[1] E. Carneiro, O. Campos and F. Paiva, Olimpı́adas Cearenses de Matemática 1981-2005
(Nı́veis Júnior e Senior), Ed. Realce, 2005.

[2] S. B. Feitosa, B. Holanda, Y. Lima and C. T. Magalhães, Treinamento Cone Sul 2008.
Fortaleza, Ed. Realce, 2010.

[3] D. Fomin, A. Kirichenko, Leningrad Mathematical Olympiads 1987-1991, MathPro


Press, Westford, MA, 1994.

[4] D. Fomin, S. Genkin and I. Itenberg, Mathematical Circles, Mathematical Words, Vol.
7, American Mathematical Society, Boston, MA, 1966.

[5] I. Niven, H. S. Zuckerman, and H. L. Montgomery, An Introduction to the Theory of


Numbers.
Polos Olímpicos de Treinamento
Curso de Teoria dos Números - Nível 2 Aula 6
Prof. Samuel Feitosa

Congruências II

Na aula de hoje, aprenderemos um dos teoremas mais importantes do curso: o ”pe-


queno”teorema de Fermat. Começaremos relembrando um resultado da aula passada:
Lema 1. Se ka ≡ kb (mod m) e mdc(m, k) = 1, então a ≡ b (mod m).
Demonstração. Como m | k(a − b) e mdc(m, k) = 1, segue que m | a − b.
Teorema 2. (Teorema de Fermat) Seja p um primo. Se p não divide a então
ap−1 ≡ 1 (mod p).
Além disso, para todo inteiro a, ap ≡ a (mod p)
Demonstração. Considere o conjunto de inteiros B = {a, 2a, 3a, . . . , (p − 1)a} onde a é um
inteiro satisfazendo mdc(a, p) = 1. Nenhum deles é divisı́vel por p e quaisquer dois deles
são incongruentes módulo p, em virtude do lema anterior. Assim, o conjunto dos restos
dos elementos de B coincide com o conjunto dos restos não nulos na divisão por p, a saber,
{1, 2, 3, . . . , p − 1}. Portanto,
a · 2a · 3a . . . (p − 1)a ≡ 1 · 2 · 3 · . . . (p − 1) (mod p),
p−1
a (p − 1)! ≡ (p − 1)! (mod p).

Podemos cancelar o termo (p − 1)! em ambos os lados pois mdc((p − 1)!, p) = 1, concluindo
assim a demonstração do teorema.
n5 n3 7n
Exemplo 3. Prove que + + é um inteiro para todo inteiro n.
5 3 15
n5 n3 7n 3n5 + 5n3 + 7n
Primeiramente note que + + = . Como mdc(3, 5) = 1, basta
5 3 15 15
mostrarmos que o numerador é mútiplo de 3 e 5. Pelo teorema de Fermat:
3n5 + 5n3 + 7n ≡ 5n3 + 7n ≡ 5n + 7n = 12n ≡ 0 (mod 3),
3n5 + 5n3 + 7n ≡ 3n5 + 7n ≡ 3n + 7n = 10n ≡ 0 (mod 5).
POT 2012 - Teoria dos Números - Nı́vel 2 - Aula 5 - Samuel Feitosa

Problema 4. Mostre que n7 ≡ n (mod 42), ∀n ∈ N


Pelo teorema de Fermat,
n7 ≡ n (mod 7)
7
n ≡ (n ) · n ≡ n2 · n = n3 ≡ n
3 2
(mod 3)
7
n ≡ (n ) · n ≡ n · n = (n ) ≡ n2 ≡ n
2 3 3 2 2
(mod 2)

Como 2, 3 e 7 são primos entre si, n7 ≡ n (mod 2 · 3 · 7 = 42).


Exemplo 5. (Bulgária 95) Encontre o número de inteiros n > 1 para os quais o número
a25 − a é divisı́vel por n para cada inteiro a.
Se n satisfaz o enunciado, p2 (p primo) não pode dividı́-lo, pois p25 − p não é divisı́vel
por p2 . Assim, n é múltiplo de primos diferentes. Os fatores primos de n são fatores de
225 − 2 = 2 · 32 · 5 · 7 · 13 · 17 · 241. Entretanto, n não é divisı́vel por 17 e 241 pois 325 ≡ −3
(mod 17) e 325 ≡ 32 (mod 241). Seguindo o exemplo anterior, podemos usar o teorema de
Fermat para mostrar que a25 ≡ a (mod p) para p ∈ {2, 3, 5, 7, 13}. Portanto, n deve ser
igual a um dos divisores de 2 · 3 · 5 · 7 · 13 diferente de 1. A quantidade de tais divisores é
25 − 1 = 31.
Exemplo 6. Prove que para cada primo p, a diferença
111 . . . 11222 . . . 22333 . . . 33 . . . 888 . . . 88999 . . . 99 − 123456789
(onde cada digito está escrito exatamente p vezes) é múltiplo de p.
Uma boa maneira de associar os números do problema com o teorema de Fermat é perceber
que:
10p − 1
111 . . . 11 = .
| {z } 9
p uns

Assim, podemos escrever o número S = 111 . . . 11222 . . . 22333 . . . 33 . . . 888 . . . 88999 . . . 99


como:
10p − 1 10p − 1 10p − 1
S = · 108p + 2 · · 107p + . . . 9 ·
9 9 9
9S = (10p − 1) · 108p + 2 · (10p − 1) · 107p + . . . 9 · (10p − 1)

Para p = 2 ou p = 3, o resultado do enunciado segue dos critérios de divisibilidade por 2 e


3. Podemos então nos concentrar no caso p > 3. Nesse caso, é suficiente mostrarmos que
9(S − 123456789) é divisı́vel por p pois mdc(p, 9) = 1. Pelo teorema de Fermat:

9S = (10p − 1) · 108p + 2 · (10p − 1) · 107p + . . . 9 · (10p − 1)


≡ (10 − 1) · 108 + 2 · (10 − 1) · 107 + . . . 9 + ·(10 − 1) (mod p)
≡ 9 · 123456789 (mod p).

2
POT 2012 - Teoria dos Números - Nı́vel 2 - Aula 5 - Samuel Feitosa

Exemplo 7. Dado um primo p, prove que existem infinitos naturais n tais que p divide
2n − n.
Se p = 2, n pode ser qualquer número par. Suponha que p > 2. Considere (p − 1)2k , pelo
teorema de Fermat temos:
2k 2k−1 2k−1
2(p−1) ≡ (2p−1 )(p−1) ≡ 1(p−1) = 1 ≡ (p − 1)2k (mod p).
Assim, para qualquer k, n = (p − 1)2k satisfaz o problema.
Lema 8. Se mdc(a, m) = 1 então existe um inteiro x tal que
ax ≡ 1 (mod m).
Tal x é único módulo m. Se mdc(a, m) > 1 então não existe tal x.

Demonstração. Pelo teorema de Bachet-Bézout, existem inteiros x e y tais que ax+my = 1.


Analisando essa congruência módulo m, obtemos ax ≡ 1 (mod m). Se y é outro inteiro
que satisfaz a congruência, temos ax ≡ ay (mod m). Pelo primeiro lema, x ≡ y (mod m).
Se d = mdc(a, m) > 1, não podemos ter d | m e m | ax − 1 pois d ∤ ax − 1.
Teorema 9. (Teorema de Wilson) Se p é primo, então
(p − 1)! ≡ −1 (mod p)
Demonstração. Em virtude do lema anterior, para cada a ∈ {2, 3, . . . , p − 2}, existe um
resto x ∈ {0, 1, 2, . . . , p − 1} tal que ax ≡ 1 (mod p). Se x = 1 ou x = p − 1, terı́amos a = 1
ou p − 1. Além disso, não podemos ter a = x pois os únicos restos que satisfazem a2 ≡ 1
(mod p) são 1 e p − 1 (Veja o problema 20). Com isso, podemos agrupar os números de
{2, 3, . . . , p − 2} em pares onde o produto deixa resto 1 por p, o que nos permite concluir
que o produto de todos eles também deixa resto 1 por p. Logo,
(p − 1)! ≡ 1 · (p − 1) ≡ −1 (mod p).
Exemplo 10. (Estônia 2000) Prove que não é possı́vel dividir qualquer conjunto de 18
inteiros consecutivos em dois conjuntos disjuntos A e B tais que o produtos dos elementos
de A seja igual ao produto dos elementos de B.
Suponha, por absurdo, que existam tais conjuntos. Considere o primo p = 19. Como o
produtos dos elementos de A é igual ao produtos dos elementos de B, se um dos conjuntos
contém um múltiplo de 19, o outro necessariamente também conterá. Como entre 18
inteiros consecutivos não existem dois múltiplos de 19, nenhum dos conjuntos do problema
contém tais números. Seja x o resto na divisão por 19 dos produtos dos elementos de A.
Calculemos então o resto na divisão por 19 do produto de todos os 18 inteiros consecutivos:
x · x ≡ n(n + 1)(n + 2)(n + 3) . . . (n + 17)
≡ 1 · 2 · 3 . . . · 18
≡ −1 (mod 19)(Pelo teorema de Wilson).
Como x2 ≡ −1 (mod 19), x18 ≡ (−1)9 ≡ 1 (mod 1)9. Isso contraria o teorema de Fermat
e obtemos um absurdo.

3
POT 2012 - Teoria dos Números - Nı́vel 2 - Aula 5 - Samuel Feitosa

Definição 11. Um conjunto S é chamado de sistema completo de resı́duos módulo n, de-


notado abreviadamente por scr, se para cada 0 ≤ i ≤ n − 1, existe um elemento de s ∈ S
tal que i ≡ s (mod n). Para qualquer a, o conjunto {a, a + 1, a + 2, . . . , a + (n − 1)} é um
exemplo de scr.
Exemplo 12. Se mdc(m, s) = 1, mostre que {t, t + s, t + 2s, . . . t + (m − 1)s} é um scr.
Pelo primeiro lema, se t + is ≡ t + js (mod m), temos is ≡ js (mod m) e i ≡ j (mod m).
Como i, j ∈ {0, 1, . . . , m − 1}, i = j. Isso nos diz que temos m inteiros que deixam restos
distintos na divisão por m. Como existem exatamente m restos na divisão por m, o conjunto
é um scr.
Exemplo 13. Seja m um inteiro positivo par. Suponha que {a1 , a1 , . . . , am } e {b1 , b2 , . . . , bm }
são dois sistemas completos de resı́duso módulo m. Prove que

S = {a1 + b1 , a2 + b2 , . . . , am + bm }

não é um sistema completo de resı́duos.


Suponha que S seja um scr, então:

1 + 2 + . . . + m ≡ (a1 + b1 ) + (a2 + b2 ) + . . . + (an + bn ) (mod m)


≡ (a1 + a2 + . . . + an ) + (b1 + b2 + . . . + bn )
≡ 2(1 + 2 + . . . + n)
≡ 2(1 + 2 + . . . + m)

m(m + 1) m+1
Isso implica que m | , ou seja, é inteiro. Um absurdo pois m é par.
2 2
Exemplo 14. (Polônia 1997) Prove que a sequência an definida por a1 = 1 e

an = an−1 + a n 
2
contém infinitos termos divisı́veis por 7.
Uma maneira natural para mostrarmos que existem infinitos inteiros múltiplos de 7 na
sequência é verificar que o aparecimento de um múltiplo de 7 acarreta o aparecimento de
outro múltiplo na sequência com um ı́ndice maior. Suponha que ak é múltiplo de 7. Seja
a2k−1 = s. Então:

a2k−1 = s
a2k = s + ak ≡ s (mod 7)
a2k+1 = a2k + ak ≡ s (mod 7)

4
POT 2012 - Teoria dos Números - Nı́vel 2 - Aula 5 - Samuel Feitosa

Ou seja, o aparecimento de um inteiro múltiplo de 7 implica no aparecimento de 3 inteiros


com o mesmo resto por 7. Exploremos essa ideia mais uma vez.

a4k−3 = t
a4k−2 ≡ t + a2k−1 ≡ t + s (mod 7)
a4k−1 ≡ t + s + a2k−1 ≡ t + 2s (mod 7)
a4k ≡ t + 2s + a2k ≡ t + 3s (mod 7)
a4k+1 ≡ t + 3s + a2k ≡ t + 4s (mod 7)
a4k+2 ≡ t + 4s + a2k+1 ≡ t + 5s (mod 7)
a4k+3 ≡ t + 5s + a2k+2 ≡ t + 6s (mod 7)

Se s é múltiplo de 7, já teremos conseguido outro múltiplo de 7 na sequência. Em caso


contrário, o conjunto {t, t + s, t + 2s, . . . , t + 6s} é um scr e conterá um múltiplo de 7.
Exemplo 15. Sejam x, y inteiros. Prove que 3x2 + 4y 2 e 4x2 + 3y 2 não podem ser ambos
quadrados perfeitos.

Comecemos com um lema bastante útil:


Lema 16. Seja p um número primo da forma 4k + 3. Então

p | m2 + n2 ⇐⇒ p | m e p | n.

Façamos inicialmente a primeira implicação. Se p ∤ m, então mp−1 ≡ 1 (mod p), e daı́


temos as equivalências módulo p

n2 ≡ −m2
⇒ (nmp−2 )2 ≡ −(mp−1 )2
≡ −1
p−1
p−2
⇒ (nm ) p−1 ≡ (−1) 2
≡ (−1)2k+1
≡ −1,

o que contraria o teorema de Fermat. Assim, p | m e p | n.

A recı́proca é óbvia. Voltando ao problema, suponha que existam w, z inteiros positivos


tais que

3x2 + 4y 2 = w2 e
2 2 2
4x + 3y = z .

Então 7x2 + 7y 2 = w2 + z 2 (∗). Afirmamos que a equação (∗) não possui solução. Para isso,
seja S o conjunto formado pelas soluções inteiras (x, y, w, z) de (∗), e tome (a, b, c, d) ∈ S

5
POT 2012 - Teoria dos Números - Nı́vel 2 - Aula 5 - Samuel Feitosa

com c2 + d2 mı́nimo. Pelo lema, temos que 7|c e 7|d, e daı́ c = 7c′ e d = 7d′ . Mas então
a2 + b2 = 7c′2 + 7d′2 ⇒ (c′ , d′ , a, b) ∈ S, com

a2 + b2 < 7(a2 + b2 ) = c2 + d2 ,

o que contraria a minimalidade de (a, b, c, d).

Problemas Propostos

Problema 17. Prove que se p é primo então

a p ≡ bp (mod p) ⇒ ap ≡ bp (mod p2 )

Problema 18. Encontre os restos da divisões de:

a) 3003000 − 1 por 1001

b) 7120 − 1 por 143

Problema 19. Encontre o resto de 111


| {z. . . 11} por p, onde p é um primo maior que 5.
p−1 uns

Problema 20. Prove que se n é ı́mpar, então n5 ≡ n (mod 240).


Problema 21. Sejam p e q primos distintos. Mostre que

i) (a + b)p ≡ ap + bp (mod p)

ii) pq + q p ≡ p + q (mod pq)


 q 
p + pq
iii) é par se p, q 6= 2.
pq

Problema 22. Mostre que se p é primo e a2 ≡ b2 (mod p), então a ≡ ±b (mod p).
Problema 23. Encontre os últimos três dı́gitos de 79999
Problema 24. Prove que 2015 − 1 é divisı́vel por 11 · 31 · 61
Problema 25. Sejam {a1 , a2 , ..., a101 } e {b1 , b2 , ..., b101 } sistemas completos de resı́duos
módulo 101. Pode {a1 b1 , a2 b2 , ..., a101 b101 } ser um sistema completo de resı́duos módulo
101?
Problema 26. (Balcânica 2003) Existe um conjunto B de 4004 inteiros positivos tal que,
para cada subconjunto A de B com 2003 elementos, a soma dos elementos em A não é
divisı́vel por 2003?
Problema 27. Para um inteiro ı́mpar n > 1, seja S o conjunto de inteiros x,1 ≤ x ≤ n,
tal que ambos x e x + 1 são relativamente primos com n. Mostre que o produto de todos
os elementos de S deixa resto 1 na divisão por n.

6
POT 2012 - Teoria dos Números - Nı́vel 2 - Aula 5 - Samuel Feitosa

Problema 28. Sejam n um inteiro positivo maior que 1 e p um primo positivo tal que n
divide p − 1 e p divide n3 − 1. Mostre que 4p − 3 é um quadrado perfeito.

Dicas e Soluções

17. Pelo teorema de Fermat, a ≡ ap ≡ bp ≡ b (mod p). Assim,

ap−1 + ap−2 b + . . . + abp−2 + bp−1 ≡ ap−1 + ap−1 + . . . + ap−1


≡ pap−1
≡ 0 (mod p)

Como a − b ≡ 0 (mod p), temos:

ap − bp = (a − b)(ap−1 + ap−2 b + . . . + abp−2 + bp−1 ) ≡ 0 (mod p2 )

19. Veja que:


999 . . . 99
111 . . . 11} =
| {z 9
p−1 uns
10p−1 − 1
=
9
Pelo teorema de Fermat, o numerador 10p−1 − 1 é divisı́vel por p visto que p 6= 5.
p−1
Além disso, usando que p 6= 2 e 3, segue que 10 9 −1 também é múltiplo de p.

20. Proceda como no exemplo 20.

21. i)Pelo teorema de Fermat:

(a + b)p ≡ a + b
≡ a p + bp (mod p).

ii) Pelo teorema de Fermat,

pq + q p ≡ 0 + q ≡ p + q (mod p)
q p
p +q ≡ p+0≡p+q (mod q)

22. Veja que (a − b)(a + b) ≡ 0 (mod p) e assim a − b ≡ 0 (mod p) ou a + b ≡ 0 (mod p).

25. Suponha, por abusurdo, que seja possı́vel. Sejam ai e bj tais que ai ≡ bj ≡ 0
(mod 101). Se i 6= j, o conjunto {a1 b1 , a2 b2 , ..., a101 b101 } teria dois inteiros com resto

7
POT 2012 - Teoria dos Números - Nı́vel 2 - Aula 5 - Samuel Feitosa

0 na divisão por p e não poderia ser um scr. Suponha sem perda de generalidade que
i = j = 101, então:

100! ≡ (a1 b1 )(a2 b2 ) . . . (a100 b100 )


≡ (a1 a2 . . . a100 )(b1 b2 . . . b100 )
≡ (100!)(100!)
≡ (100!)2 (mod 101)

Assim, 100! ≡ 1 (mod 101). Isso contradiz o teorema de Wilson.

26. Sim. Um exemplo de tal conjunto é a união de um conjunto de 2002 inteiros positivos
que deixem resto 0 com outro conjunto composto por 2002 inteiros que deixem resto
1 por 2003.

8
Polos Olímpicos de Treinamento
Curso de Teoria dos Números - Nível 2 Aula 7
Prof. Samuel Feitosa

Aula de Revisão e Aprofundamento

Observação 1. É recomendável que o professor instigue seus alunos a pensarem nos pro-
blemas abaixo antes de resolvê-los na aula.
N2 + 7
Exemplo 2. (ASHME 1990) Para quantos inteiros N entre 1 e 1990 a fração não
N +4
é irredutı́vel?

Seja d = (N + 4, N 2 + 7). Um boa estratégia é procurar um múltiplo de N + 4 próximo de


N 2 + 7 pois assim conseguiremos estimar d. Usando a diferença de quadrados, d | N 2 − 16 e
consequentemente d | N 2 +7−(N 2 −16) = 23. Como 23 é primo, a fração não será irredutı́vel
apenas quando d = 23. Para isso acontecer, basta que 23 | N +4 pois N 2 +7 = N 2 −16+23.
O maior múltiplo de 23 menor que 1990 é 1978 = 23 · 86 e 1990 + 4 < 23 · 87. Sendo assim,
a quantidade de inteiros procurada é 86.

Exemplo 3. Dados os primos p e q satisfazendo:

q | p2 + 1 e p | q 2 − 1.

Prove que o número p + q + 1 é composto.

Como p | q 2 − 1 = (q + 1)(q − 1), temos que p | q + 1 ou p | q − 1. No primeiro caso, p + q + 1


é um múltiplo de p. No segundo caso, podemos escrever q − 1 = pk para algum natural k.
Usando que q | p2 + 1, concluı́mos que q | p2 + 1 − (pk + 1) = p(p − k). Como p e q não
podem ser primos iguais, q | p − k. Temos três casos a considerar:

1. p > k. Então:

p − k ≥ kp + 1,
(p + 1)(1 − k) ≥ 2
2. p < k. Então:
k − p ≥ kp + 1,
(k + 1)(1 − p) ≥ 2

3. p = k e q = p2 +1. Como o único primo par é 2, segue que p = 2, q = 5 e p+q +1 = 8.

Os dois primeiros casos conduzem a um absurdo. Logo, ou p + q + 1 é múltiplo de p ou é


igual à 8.
Exemplo 4. (AIME 1985) Os números da sequência
101, 104, 109, 116, . . .
são da forma an = 100 + n2 , onde n = 1, 2, 3, . . . Para cada n, seja dn o máximo divisor
comum de an e an+1 . Encontre o valor máximo de dn quando n varia sobre todo o conjunto
dos inteiros positivos.
Uma boa estratégia é buscar alguma fatoração que nos permita identificar fatores comuns
entre os termos da sequência. Um termo genérico da sequência possui a forma an =
k + n2 . Sendo assim, a2k = k(4k + 1),a2k+1 = (k + 1)(4k + 1) e consequentemente
mdc(a2k , a2k+1 ) = 4k + 1. Nosso próximo passo será mostrar que realmente esse é o valor
máximo. Considere dois termos genéricos a = an = k + n2 , b = an+1 = k + (n + 1)2 e seja
d = mdc(a, b). Usando que d | b − a = 2n + 1, obtemos d | (2n + 1)(b − a) = 4n2 − 1. Como
d | 4a = 4n2 + 4k, segue que d | 4n2 + 4k − (4n2 − 1) = 4k + 1. Assim, 4k + 1 é realmente
o maior valor possı́vel entre os termos da sequência dn .
Exemplo 5. Prove que para qualquer inteiro n > 1, o número n5 + n4 + 1 não é um número
primo.
Considere a fatoração:
n5 + n4 + 1 = n5 + n4 + n3 − n3 − n2 − n + n2 + n + 1
= n3 (n2 + n + 1) − n(n2 + n + 1) + (n2 + n + 1)
= (n2 + n + 1)(n3 − n + 1)
Como n > 1, n3 − n + 1 > 1 e obtemos assim o produto de dois inteiros maiores que 1.
Exemplo 6. (Olimpı́ada Grega) Encontre todos os inteiros n para os quais −54 + 55 + 5n é
um quadrado perfeito.
Como −54 + 55 = 2500, queremos encontrar m e n tais que:
5n = m2 − 2500 = (m − 50)(m + 50).
Isto implica que m + 50 = 5j e m − 50 = 5i , com i < j. Assim,
100 = 5i (5j−i − 1).
Usando a fatoração em primos de 100, encontramos que i = 2 e j −i = 1. Portanto, m = 75
e n = 5.

2
Exemplo 7. (Irlanda) Sejam p um número primo, a e n e inteiros positivos. Prove que se

2p + 3p = an ,

então n = 1.

Se p = 2, claramente a = 13 e n = 1. Se p > 2, p é ı́mpar e 5 | 2p + 3p . Consequentemente


5 divide a. Se fosse n > 1, 25 | an e terı́amos:
an
0 ≡
5
2p + 3p

5
= 2p−1 − 2p−2 · 3 + . . . + 2 · −3p−2 + 3p−1
≡ 2p−1 + 2p−1 + . . . + 2p−1
≡ p2p−1 (mod 5)

A única possı́bilidade é termos p = 5. Entretanto, 25 + 35 não é uma potência perfeita não


trivial. Logo, n = 1.

Exemplo 8. Um inteiro n > 1 tem a seguinte propriedade: para todo divisor positivo d de
n, d + 1 é um divisor de n + 1. Prove que n é primo.
n
Seja p o menor fator primo de n e seja d = . Então,
p
np + p p(n + 1) n+1
= =
n+p p(d + 1) d+1
é um número inteiro. Como n + p também divide p(n + p), podemos concluir que n + p |
(np + p2 ) − (np + p) = p2 − p. Em particular,

n + p ≤ p2 − p
n ≤ p2 − 2p
n ≤ p2 − 2p + 1 = (p − 1)2
n < p2
d < p.

Em virtude da minimalidade de p, d não possui fatores primos e consequentemente n = p.

Exemplo 9. (Olimpı́ada Russa) Mostre que qualquer natural pode ser escrito como a dife-
rença de dois números naturais tendo o mesmo número de fatores primos.

Se n é par, podemos escrevê-lo como 2n − n e é fácil verificar que 2n e n possuem o


mesmo número de fatores primos. Seja d o menor primo ı́mpar que não divide n. Escreva
n = dn − (d − 1)n. O termo dn contém exatamente um primo a mais que n. Pela escolha
de d, todos os outros fatores primos diferentes 2 do número d − 1 são divisores de n e assim
(d − 1)n também contém extamente um primo a mais que n, a saber, o primo 2.

3
Exemplo 10. Os números naturais a e b são tais que
a+1 b+1
+
b a

é um número inteiro. Mostre que o máximo divisor comun de a e b não é maior que a + b.

Seja d = mdc(a, b), com a = md e b = nd. Então:

md + 1 nd + 1 m2 d + m + n2 d + n
+ =
nd md mnd
é um inteiro. Em particular, d | m2 d + m + n2 d + n e consequentemente d | m + n. Daı́,

d ≤ m+n
√ √
d ≤ m+n
p
d ≤ d(m + n)

= a+b

Exemplo 11. Encontre todos os conjuntos A ⊆ N de pelo menos dois elementos tais que
x+y
x, y ∈ A =⇒ ∈ A.
mdc(x, y)

Vamos dividir o problema em dois casos: Primeiro caso: Se 1 6∈ A.

Mostremos que nesse caso devemos ter A = {2, 3, . . .}. Façamos isso seguindo as seguintes
afirmações:

1. 2 ∈ A.
Para ver isso, basta tomar dois elementos iguais.

2. Existe elemento ı́mpar em A.

Suponha, por absurdo, que não existe elemento ı́mpar em A. Seja 2k o menor elemento
de par de A maior que 2. Logo,
2k + 2
k+1= ∈ A.
mdc(2k, 2)

Como k + 1 < 2k, k + 1 é ı́mpar. Absurdo! Note que todos os ı́mpares maiores que
k + 1 pertencem a A. Para tal, basta escolhermos 2 e k + 1 para obtermos k + 3 e
aplicar isso sucessivamente.

3. 3 ∈ A.

4
Tome 2l − 1 > 2k + 1. Assim,
(2l − 1)(2k + 1) + (2k + 1)
(2l − 1)(2k + 1), (2k + 1) ∈ A ⇒ = 2l ∈ A;
2k + 1
2l − 1 + 2l + 1
2l − 1, 2l + 1 ∈ A ⇒ = 4l ∈ A;
mdc(2l − 1, 2l + 1)
4l + 2l
2l, 4l ∈ A ⇒ =3∈A
mdc(4l, 2l)
Isso mostra que todos os ı́mpares maiores ou iguais a 3 estão presentes. Para isso,
basta tomar cada ı́mpar e o 2.

4. Todos os números pares estão em A.


Para isso, tome 2k − 1 e (2k − 1)2 :

(2k − 1)2 + (2k − 1)


= 2k ∈ A,
2k − 1
para todo o k ≥ 2.
Segundo caso: Se 1 ∈ A.

Nesse caso, afirmamos que A = N. Veja que:


1+1
1∈A ⇒ =2∈A
1
x+1
x∈A ⇒ =x+1∈A
mdc(1, x)
Sendo assim, repetindo esse processo, seguirá por indução que A = N.
Exemplo 12. (Olimpı́ada Matemática Argentina) Sejam p1 , p2 , . . . , pn números primos.
Bruno deve escolher n + 1 inteiros positivos que utilizem apenas estes primos em sua de-
composição. Bernardo deve escolher alguns desses números de modo que o produto deles
seja um quadrado perfeito. Determine se é possı́vel, para algum n, que Bruno escolha seus
n + 1 números de maneira que Bernardo não consiga cumprir seu objetivo.
Vamos mostrar que Bernardo sempre consegue cumprir seu objetivo. Para decidirmos se
um número natural é quadrado perfeito, basta analisarmos a paridade dos expoentes de seus
fatores primos. Para cada número pm 1 m2 mn
1 p2 . . . pn escolhido por Bruno, associe a n-upla de
zeros e uns, (r1 , r2 , . . . , rn ), onde cada ri é o resto na divisão por 2 de mi . A multiplicação
de dois inteiros se traduz na soma módulo 2 de tais uplas, i.e., a n-upla associada ao
produto pm 1 m2 m n l1 l2 ln
1 p2 . . . pn · p1 p2 . . . pn é igual à (r1 + q1 (mod 2), r2 + q2 (mod 2), . . . , rn + qn
(mod 2)) onde ri e qi são os restos na divisão por 2 de mi e li , respectivamente. Nosso
problema pergunta se é possivel Bernardo encontrar algumas uplas que somadas deem a
upla (0, 0, 0, . . . , 0). Como temos n + 1 uplas, podemos formar 2n+1 − 1 > 2n somas de
subconjuntos não vazios de uplas. Cada soma corresponde a uma nova upla, como existem

5
apenas 2n tipos de uplas distintas, alguma delas se repetirá dentre as somas (pelo princı́pio
da casa dos pombos). Suponha que para dois conjuntos de uplas A e B tenhamos a mesma
upla associada como soma, então a soma dos elementos de A e B que não pertencem à
A ∩ B corresponde à upla (0, 0, . . . , 0).

Problemas Propostos

Problema 13. Sejam p > 2 um primo ı́mpar e n um inteiro positivo. Prove que p divide

1p + 2p + . . . + (p − 1)p .
n n n

Problema 14. (Olimpı́ada Romena) Sejam a, b, c, d inteiros não nulos com a 6= c e tais que

a a2 + b2
= 2 .
c c + b2

Prove que a2 + b2 + c2 não pode ser um número primo.

Problema 15. Encontre todos os n tais que n! é um quadrado perfeito.

Problema 16. (Hungria) O produto de alguns primos é dez vezes maior que a sua soma.
Quais são esses primos?(não necessariamnete distintos).

Problema 17. Qual o máximo divisor comum entre dois números de Fibonacci consecuti-
vos?
Observação: Os números de Fibonacci são os números da sequência definida por F1 = F2 =
1 e Fn+1 = Fn + Fn−1 .

Problema 18. Mostre que a soma de dois primos consecutivos nunca é o dobro de um
primo.

Problema 19. (Israel) Se Sn é a soma dos n primeiros números primos, prove que há ao
menos um quadrado perfeito entre Sn e Sn+1

Problema 20. (Olimpı́ada Balcânica) Prove que, para todo natural dado n, existe um na-
tural m > n tal que a representação decimal de 5m é obtida da representação decimal de
5n pelo acréscimo de algarismos à esquerda de 5n .

Problema 21. (Inglaterra 1995)

a) Encontre o primeiro inteiro positivo cujo quadrado termina em três quatros.

b) Encontre todos os inteiros positivos cujo quadrado termina em três quatros.

c) Mostre que nenhum quadrado perfeito termina em quatro quatros.

6
Exemplo 22. (Olimpı́ada Indiana) Seja n um inteiro positivo tal que n é um divisor da
soma
1 + (1n−1 + 2n−1 + . . . + (n − 1)n−1 ).
Prove que n não é divisı́vel por qualquer quadrado maior que 1.

Problema 23. Seja S um conjunto de primos tal que a, b ∈ S (a e b não precisam ser
distintos) implicam ab + 4 ∈ S. Mostre que S tem que ser vazio

Dicas e Soluções

1. Em breve!

7
Polos Olímpicos de Treinamento
Curso de Teoria dos Números - Nível 2 Aula 8
Prof. Samuel Feitosa

Equações Diofantinas I

Exemplo 1. Em Gugulândia, o jogo de basquete é jogado com regras diferentes. Existem


apenas dois tipo de pontuações para as cestas: 5 e 11 pontos. É possı́vel um time fazer 39
pontos em uma partida?

Sejam x e y os números de cestas de 5 e 11 pontos, respectivamente. O problema se resume


em descobrirmos se existem inteiros não negativos x e y tais que 5x + 11y = 39. Ao invés
de testarmos os valores de x e y, somemos 11 + 5 em ambos os lados da equação:

5(x + 1) + 11(y + 1) = 55.

Como 5 | 55 e 5 | 5(x + 1), segue que 5 | 11(y + 1) e, com mais razão, 5 | y + 1 pois
mdc(5, 11) = 1. Do mesmo modo, 11 | x + 1. Assim,

55 = 5(x + 1) + 11(y + 1) ≥ 5 · 11 + 11 · 5 = 110,

pois x + 1, y + 1 ≥ 1. Obtemos uma contradição.

Exemplo 2. Qual o menor inteiro positivo m para o qual todo número maior que m pode
ser obtido como pontuação no jogo de basquete mencionado anteriormente?

Como já sabemos que 39 não é possı́vel, é natural começarmos procurando os números
maiores que 39 que não podem ser pontuações. Veja que:

40 = 5 · 8 + 11 · 0
41 = 5 · 6 + 11 · 1
42 = 5 · 4 + 11 · 2
43 = 5 · 2 + 11 · 3
44 = 5 · 0 + 11 · 4
Ao somarmos 5 a cada uma dessas representações, obteremos representações para os próximos
5 números. Repetindo esse argumento, poderemos escrever qualquer número maior que 39
na forma 5x + 11y com x e y inteiros não negativos. Concluı́mos assim que m = 39. Po-
derı́amos mostrar que todo número maior que 44 é da forma 5x + 11y com x e y inteiros
não negativos de outro modo. Se n > 44, considere o conjunto:

n − 11 · 0, n − 11 · 1, n − 11 · 2, n − 11 · 3, n − 11 · 4.

Como mdc(11, 5) = 1, o conjunto anterior é um sistema completo de restos módulo 5 e


consequentemente existe y ∈ {0, 1, 2, 3, 4} tal que

n − 11 · y = 5x

Como n > 44, segue que x > 0.

Exemplo 3. Quais e quantos são os inteiros positivos n que não podem ser obtidos como
pontuação nesse jogo de basquete?

Precisaremos relembrar um teorema da aula 03:

Teorema 4. (Bachet-Bèzout) Se d = mdc(a, b), então existem inteiros x e y tais que

ax + by = d.

A primeira observação que fazemos é que uma vez encontrados inteiros x e y, qualquer
múltiplo de d pode ser representado como uma combinação linear de a e b:

a(kx) + b(ky) = kd.

Isso é particularmente interessante quando mdc(a, b) = 1, onde obtemos que qualquer in-
teiro é uma combinação linear de a e b. Veja que isso não entra em conflito com os exemplos
anteriores pois os inteiros x e y mencionados no teorema podem ser negativos.

A próxima propopsição conterá o que procuramos:

Proposição 5. Todo inteiro positivo k pode ser escrito(de modo único) de uma e, somente
uma, das seguintes formas:

11y − 5x, ou 11y + 5x, com 0 ≤ y < 5 e x ≤ 0

Pelo teorema de Bachet-Bèzout, existem m e n tais que 5m + 11n = 1. Sejam q e r o


quociente e resto da divisão de kn por 5, i.e., kn = 5q + r, 0 ≤ r < 5. Assim,

k = 5(km) + 11(kn)
= 5(km) + 11(5q + r)
= 5(km + 11q) + 11r.

2
Basta fazer x = km + 11q e r = y.

Para ver a unicidade, suponha que 11m ± 5n = 11a ± 5b com 0 ≤ m, a < 5. Então
11(m − a) = 5(±b ± n). Usando que mdc(11, 5) = 1, segue que 5 | m − a. A única opção
é termos m = a pois o conjunto {0, 1, 2, 3, 4} é um scr. Consequentemente ±5n = ±5b e
n = b.

Sendo assim, os elementos do conjunto


B(5, 11) = {11y − 5x ∈ Z∗+ ; 0 ≤ y < 5 e x > 0}
constituem o conjunto das pontuações que não podem ser obtidas. Seus elementos são:

y = 1 ⇒ 11y − 5x = 1, 6
y = 2 ⇒ 11y − 5x = 2, 7, 12, 17
y = 3 ⇒ 11y − 5x = 3, 8, 13, 18, 23, 28
y = 4 ⇒ 11y − 5x = 4, 9, 14, 19, 24, 29, 34, 39
A quantidade de tais inteiros é
(5 − 1) (11 − 1)
20 = · .
2 2
Vale o resultado geral:
Proposição 6. Dados os inteiros positivos a e b com mdc(a, b) = 1, existem exatamente
(a − 1) (b − 1)
·
2 2
números inteiros não negativos que não são da forma ax + by com x, y ≥ 0.
Provaremos tal resultado em uma aula futura fazendo o uso da função parte inteira.
Exemplo 7. Suponha agora que as pontuações das cestas do basquete de Gugulândia tenham
mudado para a e b pontos com 0 < a < b. Sabendo que existem exatamente 35 valores
impossı́veis de pontuações e que um desses valores é 58, encontre a e b.
Perceba que devemos ter mdc(a, b) = 1 pois caso contrário qualquer valor que não fosse
múltiplo de mdc(a, b) não seria uma pontuação possı́vel e sabemos que existe apenas um
número finito de tais valores. Em virtude da proposição anterior, (a−1)(b−1) = 2·35 = 70.
Analisemos os possı́veis pares de divisores de 70 tendo em mente que a < b:
(a − 1)(b − 1) = 1 · 70 ⇒ (a, b) = (2, 71)
(a − 1)(b − 1) = 2 · 35 ⇒ (a, b) = (3, 36)
(a − 1)(b − 1) = 5 · 14 ⇒ (a, b) = (6, 15)
(a − 1)(b − 1) = 7 · 10 ⇒ (a, b) = (8, 11)

3
Não podemos ter (a, b) = (2, 71) pois 58 = 2 · 29. Excluindo os outros dois casos em que
mdc(a, b) 6= 1, temos a = 8 e b = 11.

A equação ax + by = c é um exemplo de uma equanção diofantina, i.e., uma equação


em que buscamos valores inteiros para as variáveis. Tais equações recebem esse nome em
homenagem ao matemático grego Diofanto.

Exemplo 8. Determine todas as soluções inteiras da equação 2x + 3y = 5.

Por paridade, 3y é ı́mpar, donde y = 2k + 1 para algum inteiro k. Daı́,

5 − 3(2k + 1)
x= = 1 − 3k,
2
e consequentemente todas as soluções da equação são da forma (x, y) = (1 − 3k, 2k + 1).

Exemplo 9. Determine todas as soluções inteiras da equação 5x + 3y = 7.

Analisando agora módulo 3, 5x ≡ 7 ≡ 1 (mod 3). Essa condição impõe restrições sobre o
resto de x na divisão por 3. Dentre os possı́veis restos na divisão por 3, a saber {0, 1, 2}, o
único que satisfaz tal congruência é o resto 2. Sendo assim, x é da forma 3k + 2 e

7 − 5(3k + 2)
y= = −1 − 5k,
3
consequentemente, todas as soluções da equação são da forma (x, y) = (3k + 2, −1 − 5k).

Notemos que para a solução da congruência x = 2, obtemos a solução (x, y) = (2, 1) da


equação. Baseado nesses exemplos, é natural imaginarmos que conhecendo uma solução da
congruência consigamos descrever todas as outras.

Teorema 10. A equação ax + by = c, onde a, b, c são inteiros, tem uma solução em inteiros
(x, y) se, e somente se, d = mdc(a, b) divide c. Nesse caso, se (x0 , y0 ) é uma solução, então
os pares  
bk ak
(xk , yk ) = x0 + , y0 − , k∈Z
d d
são todas as soluções inteiras da equação.

Dada a discussão anterior, resta apenas encontrarmos a forma das soluções. Se (x, y) é
outra solução, podemos escrever:

ax + by = ax0 + by0
a(x − x0 ) = b(y0 − y)
a b
(x − x0 ) = (y0 − y)
d d
Como mdc(a/d, b/d) = 1, temos b/d | x − x0 e assim podemos escrever x = x0 + bk/d.
Substituindo na equação original obtemos y = y0 − ak/d.

4
Exemplo 11. Encontre todas as soluções inteiras da equação 21x + 48y = 6
O sitema é equivalente à 7x + 16y = 2. Uma solução é (x, y) = (−2, 1). Pelo teorema
anterior, todas as soluções são da forma:

(xk , yk ) = (−2 + 16k, 1 − 7k).

Exemplo 12. Resolva nos inteiros a equação 2x + 3y + 5z = 11


Podemos transformar esse problema isolando qualquer uma das variáveis no problema que
já sabemos resolver. Por exemplo, podemos resolver 2x + 3y = 11 − 5z. Supondo z fixo,
podemos encontrar a solução particular (x, y) = (4 − z, 1 − z). Assim, todas as soluções
são da forma:
(x, y) = (4 − z + 3k, 1 − z − 2k),
ou seja, as soluções da equação original são da forma (x, y, z) = (4 − z + 3k, 1 − z − 2k, z)
com k e z inteiros.

Vamos estudar agora alguns outros exemplos de equações diofantinas não lineares:
Exemplo 13. Prove que a equação 2n + 1 = q 3 não admite soluções (n, q) em inteiros
positivos.
É facil ver que a equação não admite soluções se n = 1, 2, 3. Assim, podemos supor que
n > 3. Fatorando, temos:
(q − 1)(q 2 + q + 1) = 2n ,
e consequentemente q = 2 ou q = 2k + 1, para algum k ∈ N. Claramente, q = 2 não produz
solução. Então q = 2k + 1 e q 3 − 1 = 8k 3 + 12k 2 + 6k é uma potência de 2, maior ou igual
a 16. Entretanto:
8k 3 + 12k 2 + 6k = 2k(4k 2 + 6k + 3),
não é uma potência de 2, pois 4k 2 + 6k + 3 é ı́mpar. Assim, a equação 2n + 1 = q 3 não
admite soluções inteiras positivas.

xz − 2yt = 3
Exemplo 14. (URSS 1991) Encontre todas as soluções inteiras do sistema
xt + yz = 1.
Uma boa estratégia será aplicar alguma manipulação algébrica, como somar as equações,
multiplicá-las, somar um fator de correção, entre outras para obtermos alguma fatoração
envolvendo esses números. Nesse problema, vamos elevar ambas as equações ao quadrado.
 2 2
x z − 4xyzt + 4y 2 t2 = 9
x2 t2 + 2xytz + y 2 z 2 = 1.

Multiplicando a segunda por dois e somando com a primeira, temos:

x2 (z 2 + 2t2 ) + 2y 2 (z 2 + 2t2 ) = 11
(x2 + 2y 2 )(z 2 + 2t2 ) = 11.

5
Como cada uma das parcelas acima é um inteiro não-negativo, temos dois casos:
 2
x + 2y 2 = 11
⇒ (x, y, z, t) = (±3, ±1, ±1, 0).
z 2 + 2t2 = 1
ou
x2 2y 2

+ = 1
⇒ (x, y, z, t) = (±1, 0, ±3, ±1).
z2 + 2t2 = 11
Logo, as únicas soluções possı́veis são as quádruplas (±1, 0, ±3, ±1) e (±3, ±1, ±1, 0).

Problemas Propostos

Problema 15. Encontre todas as soluções de 999x − 49y = 5000.

Problema 16. Encontre todos os inteiros x e y tais que 147x + 258y = 369.

Problema 17. Encontre todas as soluções inteiras de 2x + 3y + 4z = 5.

Problema 18. Encontre todas as soluções inteiras do sistema de equações:

20x + 44y + 50z = 10


17x + 13y + 11z = 19.

Problema 19. (Torneio das Cidades 1997) Sejam a,b inteiros positivos tais que a2 + b2 é
divisı́vel por ab. Mostre que a = b.

Problema 20. Encontre uma condição necessária e suficiente para que

x + b1 y + c 1 z = d 1 e x + b2 y + c 2 z = d 2

tenham pelo menos uma solução simultanea em inteiros x, y, z, assumindo que os coefici-
entes são inteiros com b1 6= b2 .

Problema 21. (AMC 1989) Seja n um inteiro positivo. Se a equação 2x + 2y + n = 28 tem


28 soluções em inteiros positivos x, y e z, determine os possı́veis valores de n.

6
Polos Olímpicos de Treinamento
Curso de Teoria dos Números - Nível 2 Aula 9
Prof. Samuel Feitosa

O Teorema de Euler

Nesta aula, obteremos uma generalização do teorema de Fermat.


Definição 1. Dado n ∈ N, denotaremos o número de naturais menores ou iguais a n e
relativamente primos com n por φ(n).
Segue imediatamente da definição de φ(n) que φ(1) = 1, φ(2) = 1, φ(3) = 2, φ(5) = 4 e
φ(6) = 2. Se p é primo, φ(p) = p − 1.
Lema 2. Se p é um número primo e k um número natural, então:

φ(pk ) = pk−1 (p − 1).

Os únicos números do conjunto {1, 2, . . . , pk } que não são relativamente primos com pk são
pk
aqueles que são divisı́veis por p. A quantidade de tais números é = pk−1 . Sendo assim,
p
φ(pk ) = pk − pk−1 = pk−1 (p − 1).

Nosso próximo objetivo será encontrar uma fórmula para calcular explicitamente φ(m) em
função da fatoração em primos de m. Precisaremos relembrar um exemplo estudado na
aula 6:
Lema 3. Sejam m um número natural, l um número natural relativamente primo com m
e r um inteiro arbitrário. Então, o conjunto:

r, l + r, 2l + r, . . . , (m − 1)l + r;

é um sistema completo de restos módulo m.


Suponha, por absurdo, que existem dois inteiros i e j com 0 ≤ i < j < m e para os quais
tenhamos r + il ≡ r + jl (mod m). Assim, (j − i)l ≡ 0 (mod m). Como l é relativamente
primo com m, devemos ter j − i ≡ 0 (mod m). Obtemos um absurdo pois 0 < j − i < m.
Consequentemente, temos um conjunto de m inteiros todos incongruentes módulo m e,
portanto, tal conjunto é um sistema completo de restos.
POT 2012 - Teoria dos Números - Nı́vel 2 - Aula 9 - Samuel Feitosa

Teorema 4. Se l e m são números naturais primos entre si, então:

φ(ml) = φ(m)φ(l).

Demonstração. Como φ(1) = 1, o teorema anterior é valido quando m = 1 ou n = 1.


Suponha então que m, l > 1. Façamos uma contagem dupla. Primeiramente, usando a
definição, φ(mn) é o número de inteiros da tabela abaixo que são relativamente primos
com ml.
1, 2, ..., r, ..., l,
1 + l, l + 2, ..., l + r, ..., 2l,
21 + l, 2l + 2, ..., 2l + r, ..., 3l,
..., ..., ..., ..., ..., ...,
(m − 1)1 + l, (m − 1)l + 2, ..., (m − 1)l + r, ..., ml,
Seja r ≤ m um número natural qualquer. Considerando a r-ésima coluna da tabela, se
mdc(r, l) > 1, nenhum de seus elementos é relativamente primo com l. Então, se buscamos
os elementos que não possuem nenhum fator em comum com ml, devemos nos ater às
colunas com mdc(r, l) = 1. O número de tais colunas é φ(l). Considerando agora a r-ésima
coluna e supondo que mdc(r, l) = 1, em virtude do lema anterior, sabemos que os restos de
seus elementos na divisão por m formam exatamente o conjunto {0, 1, . . . , m} e dentre eles
existem exatamente φ(m) números relativamente primos com m. Sendo assim, podemos
contar os números relativamente primos com ml atráves do número de colunas ”boas”e do
número de ”bons”elementos em cada uma delas, obtendo: φ(m)φ(l).
Corolário 5. Se n = pα1 1 pα2 2 . . . pαk k é a fatoração em primos de n, então:
    
1 1 1
φ(n) = n 1 − 1− ... 1 −
p1 p2 pk

De fato, pelo teorema anterior,

φ(n) = φ(pα1 1 pα2 2 . . . pαk k )


= φ(pα1 1 )φ(pα2 2 ) . . . φ(pαk k )
= p1α1 −1 (p1 − 1)pα2 2 −1 (p2 − 1) . . . pαk k −1 (pk − 1)
= p1α1 −1 p2α2 −1 . . . pαk k −1 (p1 − 1)(p2 − 1) . . . (pk − 1)
    
1 1 1
= n 1− 1− ... 1 −
p1 p2 pk

Exemplo 6. Mostre que qualquer n ≥ 7 pode ser escrito na forma a + b, com a e b naturais
primos entre si, ambos maiores que 1.
Podemos escrever b = n − a e nosso objetivo é encontrar a com 1 < a < n − 1 tal que
mdc(a, n − a) = 1. Para isso, basta que mdc(a, n) = 1. Pelo corolário anterior,

φ(n) = pα1 1 −1 pα2 2 −1 . . . pαk k −1 (p1 − 1)(p2 − 1) . . . (pk − 1)

2
POT 2012 - Teoria dos Números - Nı́vel 2 - Aula 9 - Samuel Feitosa

Se a expressão anterior é igual à 2, necessariamente devemos ter αi = 1 e pi = 2 ou 3 para


todo i. Sendo assim, n ≤ 6. Logo, φ(n) > 2 e existe pelo menos outro número natural
diferente de 1 e n − 1 que é relativamente primo com n.

Exemplo 7. Prove que existem infinitos inteiros positivos n tais que


n
φ(n) = .
3
Basta tomar n = 2 · 3m , onde m é um inteiro positivo. Então:
n
φ(n) = φ(2 · 3m ) = φ(2)φ(3m ) = 2 · 3m−1 = .
3
Exemplo 8. Se n é um inteiro positivo composto, então

φ(n) ≤ n − n

Se n = pα1 1 pα2 2 . . . pαk k , usando que n é composto, podemos garantir que existe um fator

primo pi tal que pi ≤ n. Assim,
    
1 1 1
φ(n) = n 1 − 1− ... 1 −
p1 p2 pk
 
1
≤ n 1−
pi
 
1
≤ n 1− √
n

= n− n

Teorema 9. (Teorema de Euler) Se mdc(a, m) = 1, então

aφ(m) ≡ 1 (mod m)

Demonstração. A prova deste teorema será muito similar à prova do teorema de Fermat.
Sejam r1 , r2 , . . . , rφ(m) os restos em {0, 1, 2, . . . , m − 1} que são relativamente primos com
m. Considere o conjunto {ar1 , ar2 , . . . , arφ(m) }. Se dois de seus membros deixam o mesmo
resto por m, digamos:
ari ≡ arj (mod m);
temos ri ≡ rj (mod m) pois mdc(a, m) = 1. Claramente isso é uma contradição. Além
disso, mdc(ari , m) = mdc(m, ri ) = 1. Analisando os restos na divisão por m dos membros
desse novo conjunto, podemos concluir que tal conjunto coincide com o conjunto dos restos
iniciais. Assim,

r1 · r2 · . . . · rφ(m) ≡ ar1 · ar2 · . . . · arφ(m)


≡ aφ(m) r1 · r2 . . . · rφ(m)

Como mdc(r1 · r2 · . . . · rφ(m) , m) = 1, podemos cancelar esse termo em ambos os membros


da congruência anterior obtendo assim o teorema de Euler.

3
POT 2012 - Teoria dos Números - Nı́vel 2 - Aula 9 - Samuel Feitosa

Exemplo 10. Encontre os últimos três dı́gitos de 79999


Como φ(1000) = 400, usando o Teorema de Euler, obtemos:

710000 = (7400 )25


≡ 1 (mod 1000)

Note que 7 · 143 = 1001 ≡ 1 (mod 1000). Assim,

79999 ≡ 79999 · 7 · 143


≡ 710000 · 143
≡ 143 (mod 1000)

Logo, 79999 termina em 143.


Exemplo 11. (Putnam 1972) Prove que não existe um inteiro n > 1 tal que n|2n − 1.
Se existem tais inteiros positivos, denotemos por m o menor deles. Claramente m é ı́mpar,
pelo teorema de Euler, podemos garantir que:

m | 2φ(m) − 1.

Seja d = mdc(m, φ(m)). Pelo problema 27 da aula 3, temos 2d −1 = mdc(2m −1, 2φ(m) −1).
Como m | mdc(2m − 1, 2φ(m) − 1), d > 1. Além disso, d ≤ φ(m) < m e d | 2d − 1. Isso é
um absurdo pois m é o menor inteiro maior que 1 com tal propriedade.
Exemplo 12. (Olimpı́ada de Matemática Argentina) Demostre que para cada número na-
tural n, existe uma potência de 2 cuja expansão decimal tem entre seus últimos n dı́gitos
2n
(da direita) mais de dı́gitos que são iguais a 0.
3
Se 2k tiver um resto muito pequeno módulo 10n , poderemos garantir que existirão muitos ze-
ros consecutivos entre seus últimos dı́gitos. Para obtermos a equação 2k ≡ r (mod 10n ) com
r pequeno, é interessante começarmos analisando 2k (mod 5n ) uma vez que mdc(2, 5n ) = 1.
Façamos isso. Pelo teorema de Euler, temos:

2φ(n) ≡ 1 (mod 5n ) ⇒
2φ(n)+n ≡ 2n (mod 10n ).
n
Como 2n = 8n/3 < 10n/3 , podemos concluir que 2n possui menos que dı́gitos e, conse-
3
n 2n
quentemente, entre os últimos n dı́gitos de 2φ(n)+n existem pelo menos n − = dı́gitos
3 3
consecutivos iguais à zero.
Exemplo 13. (IMO 1971) Prove que a sequência 2n − 3(n > 1) contém uma subsequência
de números primos entre si dois a dois.

4
POT 2012 - Teoria dos Números - Nı́vel 2 - Aula 9 - Samuel Feitosa

Uma boa estratégia é construir uma sequência recursivamente. Suponha que já tenhamos
escolhido os termos a1 , a2 , . . . , ak na sequência de modo que mdc(ai , aj ) = 1. Como pode-
remos escolher o próximo termo ak+1 da forma 2n − 3? Claramente mdc(2, ai ) = 1. Desde
que φ(ai ) | n, poderemos usar o teorema de Euler para obter:

2n − 3 ≡ 1 − 3
6≡ 0 (mod ai )

Sendo assim, pelo teorema 4, basta escolhermos:

n = φ(a1 · a2 · . . . · ak ) = φ(a1 )φ(a2 ) . . . φ(ak );

que naturalmente será um múltiplo de cada φ(ai ). Logo, podemos definir

ak+1 = 2φ(a1 ·a2 ·...·ak ) − 3

e assim temos uma sequência de termos infita satisfazendo as condições do enunciado.

Problemas Propostos

Problema 14. Encontre todos os números naturais n para os quais φ(n) não é divisı́vel por
4.
Problema 15. Prove que se p > 2 e 2p + 1 são ambos números primos, então para n = 4p
vale que
φ(n + 2) = φ(n) + 2.
Problema 16. Encontre todas as soluções nos números naturais da equação φ(n) = φ(2n).
Problema 17. Encontre todas as soluções nos números naturais da equação φ(2n) = φ(3n).
Problema 18. Se n possui k fatores primos distintos, prove que 2k | φ(n).
Problema 19. Prove que para qualquer número natural k, existe pelo menos um número
natural n tal que
φ(n + k) = φ(n).
Dica: Considere o menor divisor primo p que não é um divisor de k e estude o número
n = (p − 1)k.
Problema 20. Mostre que se a e b são inteiros primos entre si, então existem inteiros m e
n tais que am + bn ≡ 1 (mod ab).
Problema 21. (Alemanha) Se n é um número natural tal que 4n + 2n + 1 é primo, prove
que n é potência de 3.

5
Problema 22. (USAMO 1991) Mostre que para qualquer inteiro fixo n ≥ 1, a sequência
2 22
2, 22 , 22 , 22 , . . . (mod n);

é eventualmente constante, isto é, a partir de um certo termo da sequência todos os restos
obtidos na divisão por n serão iguais.

Dica: Tente considerar os casos em que n é par ou n é ı́mpar em separado e use indução.
Problema 23. Encontre os últimos 8 dı́gitos da expansão binária de 271986
Problema 24. Mostre que, para qualquer inteiro positivo n com n 6= 2 e n 6= 6 temos:

φ(n) ≥ n.

Referências
[1] F. E. Brochero Martinez, C. G. Moreira, N. C. Saldanha, E. Tengan - Teoria dos
Números ? um passeio com primos e outros números familiares pelo mundo inteiro,
Projeto Euclides, IMPA, 2010.

[2] E. Carneiro, O. Campos and F. Paiva, Olimpı́adas Cearenses de Matemática 1981-2005


(Nı́veis Júnior e Senior), Ed. Realce, 2005.

[3] S. B. Feitosa, B. Holanda, Y. Lima and C. T. Magalhães, Treinamento Cone Sul 2008.
Fortaleza, Ed. Realce, 2010.

[4] D. Fomin, A. Kirichenko, Leningrad Mathematical Olympiads 1987-1991, MathPro


Press, Westford, MA, 1994.

[5] D. Fomin, S. Genkin and I. Itenberg, Mathematical Circles, Mathematical Words, Vol.
7, American Mathematical Society, Boston, MA, 1966.

[6] I. Niven, H. S. Zuckerman, and H. L. Montgomery, An Introduction to the Theory of


Numbers.
Polos Olímpicos de Treinamento
Curso de Teoria dos Números - Nível 2 Aula 10
Prof. Samuel Feitosa

Divisores

Suponha que n = pα1 1 pα2 2 . . . pαk k é a fatoração em primos do inteiro n. Todos os divisores
de n são da forma m = pβ1 1 pβ2 2 . . . pβk k , onde 0 ≤ βi ≤ αi . Cada um desses números, aparece
exatamente uma vez no produto:

(1 + p1 + p21 + . . . + pα1 1 )(1 + p2 + p22 + . . . + pα2 2 ) . . . (1 + pn + p2n + . . . + pαk k ),

quando o mesmo é expandido usando a distributividade. Como existem αi + 1 termos em


cada parênteses, O número de termos dessa expansão é:

(α1 + 1)(α2 + 1) . . . (αk + 1).

Além disso, sabemos que:

pαi i +1 − 1
1 + pi + p2i + . . . + pαi i = .
pi − 1
Sendo assim, podemos concluir que:

Teorema 1. Se n = pα1 1 pα2 2 . . . pαk k é a fatoração em primos de n, então:

a) O número de divisores de n, denotado por d(n), é: (α1 + 1)(α2 + 1) . . . (αn + 1).

b) A soma dos divisores de n, denotada por σ(n), é:

(1 + p1 + p21 + . . . + pα1 1 )(1 + p2 + p22 + . . . + pα2 2 ) . . . (1 + pn + p2n + . . . + pαnn )

ou, de forma mais sucinta,


! !
pα1 1 +1 − 1 pα2 2 +1 − 1 pnαn +1 − 1
 
...
p1 − 1 p2 − 1 pn − 1
POT 2012 - Teoria dos Números - Nı́vel 2 - Aula 10 - Samuel Feitosa

n
Observação 2. (Pareamento de divisores) Se d é um divisor de n, então também é um
d
divisor de n.
n √
Portanto, pelo menos um dentre {d, } é um divisor de n menor ou igual a n.
d
Exemplo 3. Determine o número de divisores positivos de 20088 que são menores que
20084 .
O número de divisores de 20088 = 224 · 2518 é 225. Como n é um quadrado
√ perfeito e em
virtude da observação anterior, 112 desses divisores são menores que 20088 = 20084 e 112
são maiores.
Exemplo 4. Encontre a soma dos inversos dos divisores postivos de 496.
Sejam d1 , d2 , . . . , dn os divisores de 496 e K a soma de seus inversos. Usando a observação
496 496 496
anterior, o conjunto { + +. . .+ } coincide com o conjunto {dn + dn−1 + . . . + d1 }
d1 d2 dn
e daı́:

1 1 1
+ + ... + = K⇒
d1 d2 dn
496 496 496
+ + ... + = 496K ⇒
d1 d2 dn
dn + dn−1 + . . . + d1 = 496K ⇒
25 − 1 312 − 1
· = 496K ⇒
2 − 1 31 − 1
960
= K.
496
60
Portanto, k = .
31
Exemplo 5. Um número natural n possui exatamente dois divisores e n + 1 possui exata-
mente 3 divisores. Encontre o número de divisores de n + 2.
Se n possui exatamente dois divisores, então n = p é um número primo. Se n + 1 possui
um número ı́mpar de divisores, então n + 1 = x2 é um quadrado perfeito, para algum x
inteiro positivo. Logo, x2 − 1 = (x − 1)(x + 1) = p. Como p é primo, a única possibilidade
é x − 1 = 1 e consequentemente n = 3. O número de divisores de n + 2 = 5 é 2.

Exemplo 6. Encontre todos os inteiros n que possuem exatamente n divisores positivos.

Para n ser inteiro, n deve ser um quadrado perfeito e assim podemos escrever:
2αk
n = p2α 1 2α2
1 p 2 . . . pk .

A condição do problema é equivalente à:

pα1 1 pα2 2 . . . pαk k = (2α1 + 1)(2α2 + 1) . . . (2αk + 1).

2
POT 2012 - Teoria dos Números - Nı́vel 2 - Aula 10 - Samuel Feitosa

Analisando o lado direito, podemos concluir que cada pi é ı́mpar e consequentemente

pαi i ≥ 3αi ≥ 2αi + 1.

Como devemos ter a igualdade, p1 = 3 e 3α1 = 2α1 + 1. Se α1 > 1, vale a desigualdade


estrita(veja o problema 13). Logo, a única solução é n = 9.

Exemplo 7. (Suiça 2011) Encontre todos os inteiros positivos n para o qual n3 é o produto
de todos os divores de n

Claramente n = 1 é solução. Suponha que n > 1 e sejam d1 < d2 < . . . < dk os divisores
de n. Pela observação 2, podemos agrupar os divisores em pares cujo produto é n, assim:

n6 = (d1 d2 . . . dk )(d1 d2 . . . dk )
= (d1 dk )(d2 dk−1 ) . . . (dk d1 )
= nd(n)

Consequentemente, 6 = d(n) e n = p5 ou n = pq 2 com p e q primos distintos. Fica a cargo


do leitor verificar que essas soluções satisfazem o enunciado.

Exemplo 8. (Irlanda 1995) Para cada inteiro positivo n tal que n = p1 p2 p3 p4 , onde p1 , p2 ,
p3 e p4 são primos distintos, sejam:

d1 = 1 < d2 < d3 < . . . < d16 = n,

os 16 inteiros positivos que dividem n. Prove que se n < 1995, então d9 − d8 6= 22.

Suponha que n < 1995 e d9 − d8 = 22. Note inicialmente que d8 não pode ser par pois n
seria divisı́vel por 4 contradizendo o fato de que n possui quatro fatores primos distintos.
Consequentemente d8 , d9 e n são ı́mpares. Também temos a fatoração: 35 · 57 = 1995 =
3 · 5 · 7 · 19. Então, usando a observação 2, d8 d9 = n. Se d8 ≥ 35 terı́amos d9 < d8 para
manter n < 1995 e isso seria um absurdo. Logo, d8 < 35. Os divisores d1 , d2 , . . . , d8 são
produtos de primos ı́mpares distintos. Como 3 · 5 · 7 > 35, nenhum dentre d1 , d2 , . . . , d8
é grande o suficiente para possuir três fatores primos distintos. Como n possui somente
quatro fatores primos distintos, quatro desses di ’s devem ser o produto de dois primos
ı́mpares. Os menores números que são o produto de dois primos são:

15, 21, 33, 35, . . .

e consequentemente devemos ter d8 ≥ 35, uma contradição.

Exemplo 9. Prove que não existe inteiro positivo n tal que σ(n) = nk para algum inteiro
positivo k.

Afirmamos que n = 1 é a única solução. Suponha que n > 1 seja solução e sejam

d1 = 1 < d2 < . . . < dk = n,

3
POT 2012 - Teoria dos Números - Nı́vel 2 - Aula 10 - Samuel Feitosa

os divisores de n. Então
n(n + 1)
σ(n) = d1 + d2 + . . . + dk < 1 + 2 + . . . + n = < n2 .
2
Além disso,
n < n + 1 ≤ d1 + d2 + . . . + dk = σ(n).
Daı́,
n < nk < n2 ,
e obtemos um absurdo.
Exemplo 10. (Olimpı́ada de Leningrado 1989) Duas pessoas jogam um jogo. O número 2
está inicialmente escrito no quadro. Cada jogador, na sua vez, muda o número atual N
no quadro negro pelo número N + d, onde d é um divisor de N com d < N . O jogador que
escrever um número maior que 19891988 perde o jogo. Qual deles irá vencer se ambos os
jogadores são perfeitos.
Nesse problema, basta determinarmos apenas aquele que possui a estratégia vencedora.
Note que o inı́cio do jogo é estritamente determinado: 2 → 3 → 4. Suponha que o segundo
jogador vence o jogo. Após o movimento 4 → 5 do primeiro jogador, o segundo só pode
jogar 5 → 6. Isto significa que 6 é uma posição vencedora. Entretanto, o primeiro jogador
pode obter a posição 6 jogando 4 → 6, uma contradição. Logo, o primeiro jogador possui
a estratégia vencendora.
Exemplo 11. (Olimpı́ada de Leningrado) Duas pilhas de palitos sobre uma mesa contém
100 e 252 palitos, respectivamente. Dois jogadores jogam o seguinte jogo: Cada jogador em
sua vez pode remover alguns palitos de uma das pilhas de modo que o número de palitos
retirados seja um divisor do número de palitos da outra pilha. O jogador que fizer o último
movimento vence. Qual dos dois jogadores irá vencer se ambos são perfeitos?
O primeiro jogador perde. Em cada momento do jogo, podemos registrar o expoente
da maior potência de 2 que divide os números de palitos em cada pilha. Por exemplo,
no inı́cio os números são (2, 2). A estratégia do segundo jogador é manter esse números
sempre iguais. Suponha que, em um dado momento, as pilhas possuem 2m · a e 2m · b
palitos com a e b ı́mpares. O par registrado será (m, m). Vejamos o que acontece quando
retiramos um divisor d da segunda pilha do número de palitos da primeira. Se 2m é a
maior potência de 2 que divide d, então 2m+1 dividirá o número de palitos da primeira
pilha e consequentemente o par registrado terá números diferentes. Se 2k , com k < m,
é a maior potência de 2 que divide d, então 2k será a maior potência de 2 que divide o
número de palitos da primeira pilha e novamente o par registrado terá números diferentes.
Assim, sempre que um jogador receber um par registrado com números iguais, ele irá passar
um par registrado com números diferentes para o outro jogador. Suponha agora que, na
sua vez, as pilhas possuem 2m · a e 2n · b palitos, com m < n e a ≡ b ≡ 1 (mod 2).
Basta o jogador retirar 2m palitos da segunda pilha para passar um par registrado com
números iguais a (m, m). Como inicialmente as pilhas possuem números registrados iguais,
o segundo jogador pode sempre manter essa propriedade e consequentemente o único que
pode passar uma pilha com zero palitos pela primeira vez é o primeiro jogador.

4
REFERÊNCIAS
POT 2012 - Teoria dos Números - Nı́vel 2 - Aula 10 - Samuel Feitosa

Problemas Propostos

Problema 12. Mostre que se k é um inteiro positivo então 3k ≥ 2k +1 e vale a desigualdade


estrita quando k > 1.
Problema 13. (Rússia 2001) Encontre todos os n tais que quaisquer divisores primos dis-
tintos a e b de n o número a + b − 1 também é um divisor de n
Problema 14. O número 332 − 1 tem exatamente dois divisores que são maiores que 75 e
menores que 85. Qual o produto desses dois divisores?
Problema 15. (Irã 2012) Sejam a e b inteiros positivos de modo que o número de divisores
positivos de a,b, ab é 3,4 e 8, respectivamente. Encontre o número de divisores positivos
de b2 .
Problema 16. (Olimpı́ada de São Petesburgo) Enconte todos os inteiros positivos n tais
que 3n−1 + 5n−1 divide 3n + 5n .
Problema 17. Sejam 1 = d1 < d2 < .... < dk = n o conjunto de todos os divisores de um
inteiro positivo n. Determine todos os n tais que:
d26 + d27 − 1 = n.
Problema 18. Um divisor d > 0 de um inteiro positivo n é dito ser um divisor unitário
n
se mdc(d, ) = 1. Suponha que n é um inteiro positivo tal que a soma de seus divisores
d
unitários é 2n. Prove que n não pode ser ı́mpar.

Referências
[1] F. E. Brochero Martinez, C. G. Moreira, N. C. Saldanha, E. Tengan - Teoria dos
Números ? um passeio com primos e outros números familiares pelo mundo inteiro,
Projeto Euclides, IMPA, 2010.
[2] E. Carneiro, O. Campos and F. Paiva, Olimpı́adas Cearenses de Matemática 1981-2005
(Nı́veis Júnior e Senior), Ed. Realce, 2005.
[3] S. B. Feitosa, B. Holanda, Y. Lima and C. T. Magalhães, Treinamento Cone Sul 2008.
Fortaleza, Ed. Realce, 2010.
[4] D. Fomin, A. Kirichenko, Leningrad Mathematical Olympiads 1987-1991, MathPro
Press, Westford, MA, 1994.
[5] D. Fomin, S. Genkin and I. Itenberg, Mathematical Circles, Mathematical Words, Vol.
7, American Mathematical Society, Boston, MA, 1966.
[6] I. Niven, H. S. Zuckerman, and H. L. Montgomery, An Introduction to the Theory of
Numbers.

5
Polos Olímpicos de Treinamento
Curso de Teoria dos Números - Nível 2 Aula 11
Prof. Samuel Feitosa

O Teorema Chinês dos Restos

Iremos estudar um antigo teorema descoberto pelos chineses no inı́cio século XIII. Come-
cemos recordando um lema da aula 06:

Lema 1. Se mdc(a, m) = 1, então existe um inteiro x tal que:

ax ≡ 1 (mod m).

Tal inteiro é único módulo m. Se mdc(a, m) > 1, não existe x satisfazendo tal equação.

Demonstração. Pelo teorema de Bachet-Bézout, existem inteiros x e y tais que ax+my = 1.


Analisando essa congruência módulo m, obtemos ax ≡ 1 (mod m). Se y é outro inteiro
que satisfaz a mesma congruência, temos ax ≡ ay (mod m). Pelo primeiro lema, x ≡ y
(mod m). Se d = mdc(a, m) > 1, não podemos ter d | m e m | ax − 1 pois d ∤ ax − 1.
Exemplo 2. Encontre x inteiro tal que:

x ≡ 1 (mod 11);
x ≡ 2 (mod 7).

A primeira congruência nos diz que x = 11k + 1 para algum k ∈ Z. Sejam q e r o quociente
e o resto da divisão de k por 7, respectivamente. Assim, k = 7q + r e x = 77q + 11r + 1.
Para x satisfazer a segunda congruência, devemos encontrar r ∈ {0, 1, 2, 3, 4, 5, 6} tal que
11r + 1 ≡ 2 (mod 7), ou seja, 4r ≡ 1 (mod 7). Como o inverso de 4 (mod 7) é 2, obtemos
r = 2 e x = 77q + 23. Veja que para qualquer q inteiro, tal x é solução do sistema de
congruências.
Exemplo 3. Encontre x inteiro tal que:

x ≡ 1 (mod 11)
x ≡ 2 (mod 7)
x ≡ 4 (mod 5)
POT 2012 - Teoria dos Números - Nı́vel 2 - Aula 11 - Samuel Feitosa

Pelo exemplo anterior, para x satisfazer as duas primeiras equações, devemos ter
x = 77q +23. Dividindo q por 5, obtemos q = 5l+s com 0 ≤ s < 5. Daı́, x = 385l+77s+23.
Para satisfazer a última congruência, devemos ter 77s + 23 ≡ 4 (mod 5), ou seja, 2s ≡ 1
(mod 5). Como 3 é o inverso de 2 (mod 5), s = 3 e consequentemente x = 385l + 254.

Perceba que nos dois exemplos anteriores, o problema foi reduzido à encontrarmos o inverso
de um inteiro. No último exemplo, a solução geral possui a forma: x = 11·7·5l+231+22+1.
Essencialmente, o trabalho de encontrar esses inversos foi possı́vel pois os inteiros 5, 7 e 11
são primos entre si dois a dois.

Veremos agora um mecanismo levemente diferente para resolver tais sistemas equações.
Teorema 4. (Teorema Chinês dos Restos) Sejam m1 , m2 , . . . , mr , inteiros positivos
primos entre si, dois a dois, e sejam a1 , a2 , . . . , ar ; r inteiros quaisquer. Então, o sistema
de conguências:

x ≡ a1 (mod m1 )
x ≡ a2 (mod m2 )
..
.
x ≡ ar (mod mr )

admite uma solução x. Além disso, as soluções são únicas módulo m = m1 m2 . . . mr .


m
Demonstração. Escrevendo m = m1 m2 . . . mr , vemos que é um inteiro e
  m j
m
mdc , mj = 1. Então, pelo lema inicial, para cada j, existe um inteiro bj tal que
 mj  
m m
bj ≡ 1 (mod mj ). Claramente bj ≡ 0 (mod mi ) para i 6= j. Definamos
mj mj
m m m
x0 = b1 a1 + b2 a 2 + . . . + br a r
m1 m2 mr
m
Consideremos x0 módulo mi : x0 ≡ bj aj ≡ ai (mod mi ). Logo, x0 é uma solução do
mi
nosso sistema. Se x0 e x1 também o são, podemos escrever x0 ≡ x1 (mod mi ) para cada i.
Como mdc(mi , mj ) = 1, para i 6= j, pbtemos x0 ≡ x1 (mod m).
Observação 5. Se cada uma das equações do sistema anterior fosse do tipo
bi x ≡ ai (mod m)i , com mdc(bi , m) = 1, ainda poderı́amos usá-lo. Bastaria reescrever
bi x ≡ ai (mod m)i como x ≡ bi ai (mod m)i , onde bi é o inverso de bi (mod m)i .
Exemplo 6. Encontre o menor inteiro positivo x tal que x ≡ 5 (mod 7), x ≡ 7 (mod 11) e
x ≡ 3 (mod 13).
Usando o teorema anterior com m1 = 5, m2 = 7, m3 = 11, a1 = 5, a2 = 7 e a3 = 3 podemos
achar x ≡ 887 (mod 1001) = 7.11.13. Como a solução é única módulo m, isso significa
que, dentre os números 1, 2, · · · , 1001 a menor solução positiva é 887.

2
POT 2012 - Teoria dos Números - Nı́vel 2 - Aula 11 - Samuel Feitosa

Exemplo 7. (OBM 2009) Sejam m e n dois inteiros positivos primos entre si. O Teorema
Chinês dos Restos afirma que, dados inteiros i e j com 0 ≤ i < m e o ≤ j < n, existe
exatamente um inteiro a, com 0 ≤ a < mn, tal que o resto da divisão de a por m é igual
a i e o resto da divisão de a por n é igual a j. Por exemplo, para m = 3 e n = 7, temos
que 19 é o único número que deixa restos 1 e 5 quando dividido por 3 e 7, respectivamente.
Assim, na tabela a seguir, cada número de 0 a 20 aparecerá exatamente uma vez.

0 1 2 3 4 5 6
0 A B
1 C D
2 E F

Qual a soma dos números das casas com as letras A, B, C, D, E e F ?


Usando o teorema chinês dos restos, podemos encontrar A = 15, B = 12, C = 10, D =
13, E = 8 e F = 11. Assim, A + B + C + D + E + F = 69.
Exemplo 8. (Estônia 2000) Determine todos os restos possı́veis da divisão do quadrado de
um número primo com 120 por 120.
Seja n tal que mdc(n, 120) = 1. Como 120 = 3 · 5 · 8, temos que n 6≡ 0 (mod 3), (mod 5)
(mod 2). Daı́, n2 ≡ 1 (mod 3), n2 ≡ 1 (mod 8) e n2 ≡ 1 ou 4 (mod 5). Sendo assim, n2
satisfaz o sistema:

x ≡ 1 (mod 3)
x ≡ 1 (mod 8)
x ≡ ±1 (mod 5)

cujas soluções são x ≡ 1 (mod 120) e x ≡ 49 (mod 120).

Aconselhamos ao leitor a resolução de alguns exemplos numéricos até adquirir prática com o
algoritmo usado para encontrar x0 . Provamos, no teorema passado, que todas as soluções
daquele sistema de congruências são os termos de uma P.A de razão m. Geralmente
usaremos aquele teorema apenas para garantir que um sistema de congruências admite
uma solução. Os próximos exemplos podem deixar isso mais claro.
Exemplo 9. Para cada número natural n, existe uma sequência arbitrariamente longa de
números natu rais consecutivos, cada um deles sendo divisı́vel por uma s-ésima potência
de um número natural maior que 1.

Demonstração. Dado m ∈ N, considere o conjunto {p1 , p2 , . . . , pm } de primos distintos.


Como mdc(psi , psj ) = 1, então pelo teorema 3, existe x tal que x ≡ −i (mod psi ) para
i = 1, 2, . . . m. Cada um dos números do conjunto {x + 1, x + 2, . . . , x + m} é divisı́vel por
um número da forma psi .

3
POT 2012 - Teoria dos Números - Nı́vel 2 - Aula 11 - Samuel Feitosa

Exemplo 10. (USAMO 1986)

(a) Existem 14 inteiros positivos consecutivos tais que, cada um é divisı́vel por um ou
mais primos p do intervalo 2 ≤ p ≤ 11?

(b) Existem 21 inteiros positivos consecutivos tais que, cada um é divisı́vel por um ou
mais primos p do intervalo 2 ≤ p ≤ 13?

Demonstração. (a) Não. Suponha que existam tais inteiros. Da nossa lista de 14 inteiros
consecutivos, 7 são números pares. Vamos observar os ı́mpares: a, a + 2, a + 4, a + 6, a +
8, a + 10 e a + 12. Podemos ter no máximo três deles divisı́veis por 3, dois por 5, um por
7 e um por 11. Veja que 3 + 2 + 1 + 1 = 7. Pelo Princı́pio da Casa dos Pombos, cada um
desses ı́mpares é divisı́vel por exatamente um primo do conjunto {3, 5, 7, 11}. Além disso,
note que os múltiplos de 3 só podem ser {a, a + 6, a + 12}. Dois dos números restantes em
(a + 2, a + 4, a + 8, e a + 10) são divisı́veis por 5. Mas isso é impossı́vel. (b) Sim. Como
os números {210, 11, 13} são primos entre si, dois a dois, pelo teorema 3 existe um inteiro
positivo n > 10 tal que:
n ≡ 0( mod 210 = 2 · 3 · 5 · 7·)
n ≡ 1( mod 11)
n ≡ −1( mod 13)
Veja que o conjunto {n − 10, n − 9, . . . , n + 9, n + 10} satisfaz as condições do item (b).
Exemplo 11. Sejam a e b inteiros positivos tais que, para qualquer n natural, an +n | bn +n.
Prove que a = b.
Seja p um primo maior que a e b. Então mdc(p, a) = mdc(p, b) = 1. Como mdc(p, p−1) = 1,
existe um inteiro positivo n tal que n ≡ 1 (mod p − 1) e n ≡ −a (mod p). Pelo teorema de
Fermat, an + n ≡ 0 (mod ) e bn + n ≡ b − a (mod p). Assim, p | |b − a|. Como |b − a| < p,
segue que |b − a| = 0 e a = b.
Exemplo 12. (Olimpı́ada Nórdica 1998)

(a) Para quais inteiros positivos n existe um sequência x1 , x2 , . . . , xn contendo cada um


dos inteiros 1, 2, . . . , n exatamente uma vez, e tal que k divide x1 + x2 + · · · + xk para
k = 1, 2, · · · , n?

(b) Existe uma sequência infinita x1 , x2 , . . . contendo todo inteiro positivo exatamente
uma vez, e tal que para cada inteiro positivo k, k divide x1 + x2 + · · · + xk ?

a) Suponha que n é um inteiro que satisfaz o enunciado. Naturalmente n divide a soma:

n(n + 1)
x1 + x2 + . . . xn = .
2
n+1 n+1
Daı́, é um inteiro e n deve ser ı́mpar. Seja m = . Usando que
2 2
(n − 1) | x1 + x2 + . . . xn−1 = mn − xn ,

4
POT 2012 - Teoria dos Números - Nı́vel 2 - Aula 11 - Samuel Feitosa

temos xn ≡ m (mod n − 1) se n ≥ 3 e, consequentemente, xn = m. Repetindo a mesma


análise para n − 2 no lugar de n − 1, obtemos xn−1 = m para n ≥ 5. Como não podem
existir dois termos iguais, temos um absurdo. Analisando os casos quando n ≤ 4, encon-
tramos n = 1 e n = 3 como únicas soluções.

b) Iremos construir a sequência indutivamente. Suponha que já tenhamos definido os termos
x1 , x2 , . . . , xn satisfazendo a condição k | x1 + x2 . . . xk para todo k ≤ n. Seja m o menor in-
teiro positivo que ainda não apareceu na sequência. Pelo Teorema Chinês dos Restos, existe
x tal que x ≡ −(x1 +x2 +. . .+xn ) (mod n+1) e x ≡ −(x1 +x2 +. . .+xn )−m (mod n+2).
Escolha l, inteiro positivo, tal que l > x1 , x2 , . . . , xn , m e l ≡ x (mod (n+1)(n+2)). Defina
xn+1 = l e xn+2 = m. Veja que a condição k | x1 + x2 . . . xk agora é verdadeira para todo
k ≤ n + 2. Para o inı́cio, basta definir x1 = 1.
Exemplo 13. (Olimpı́ada de São Petesburgo 1990) Dado um polinômio F (x) com coefi-
cientes inteiros, tal que, para cada inteiro n, o valor de F (n) é divisı́vel por pelo menos
um dos inteiros a1 , a2 , · · · , am . Prove que podemos encontrar um ı́ndice k tal que F (n) é
divisı́vel por ak para cada inteiro positivo n.

Demonstração. Suponha que não exista tal ı́ndice. Para cada ı́ndice k (k = 1, 2, . . . , m),
existe um inteiro xk tal que F (xk ) não é divisı́vel por ak . Assim, existem números
dk = pαk k (onde pk são números primos), tais que dk divide ak mas não divide F (xk ). Se
existem potências do mesmo primo entre esses números, podemos apagar aquelas repeti-
das deixando apenas uma que tem expoente mı́nimo. Caso F (x) não seja divisı́vel por
uma potência apagada, não será pela potência que tem expoente mı́nimo. Essas deleções
garatem que nossa nova coleção d1 , d2 , . . . , dj de potências de primos contenham apenas
inteiros primos entre si, dois a dois. Pelo teorema chinês dos restos, exite um inteiro N
tal que N ≡ xk (mod d)k , para k ∈ {1, 2, . . . , j}. Suponhamos que dk | F (N ). Sabemos
que x − y | F (x) − F (y) e consequentemente N − xk | F (N ) − F (xk ). Como dk | N − xk ,
devemos ter dk | F (xk ). Uma contradição! Logo, F (N ) não é divisı́vel por nenhum dk e
isso contradiz a hipótese sobre os ai .

Problemas Propostos

Problema 14. Encontre o menor inteiro positivo (com a exceção de x = 1) que satisfaça o
seguinte sistema de congruências:

x ≡ 1 (mod 3)
x ≡ 1 (mod 5)
x ≡ 1 (mod 7)

5
POT 2012 - Teoria dos Números - Nı́vel 2 - Aula 11 - Samuel Feitosa

Problema 15. Encontre todas as soluções do sistema:

x ≡ 2 (mod 3)
x ≡ 3 (mod 5)
x ≡ 5 (mod 2)

Problema 16. Encontre todos os inteiros que deixam restos 1, 2 e 3 quando divididos por
3, 4 e 5, respectivamente.
Problema 17. Encontre todas as soluções do sistema:

3x ≡ 1 (mod 4)
2x ≡ 1 (mod 3)
4x ≡ 5 (mod 7)

Problema 18. Encontre todas as soluções das congruências:

a) 20x ≡ 4 (mod 30).

b) 20x ≡ 30 (mod 4).

c) 353x ≡ 254 (mod 400).

Problema 19. Se a é escolhido ao acaso no conjunto {1, 2, 3, . . . , 14} e b é escolhido ao


acaso no conjunto {1, 2, . . . , 15}, qual a probabilidade de que a equação ax ≡ b (mod 15)
possua pelo menos uma solução?
Problema 20. Sejam a e b inteiros tais que mdc(a, b) = 1 e c > 0. Prove que existe um
inteiro x tal que mdc(a + bx, c) = 1.
Problema 21. Existem n inteiros consecutivos tal que cada um contém um fator primo
repetido k vezes?
Problema 22. Seja n um número natural arbitrário. Prove que existe um par de naturais
(a, b) tais que mdc(a + r, b + s) > 1 ∀ r, s = 1, 2, . . . , n.
Problema 23. Um ponto (x, y) ∈ Z 2 é legal se mdc(x, y) = 1. Prove ou disprove: Dado
um inteiro positivo n, existe um ponto (a, b) ∈ Z 2 cuja distância a todo ponto legal e pelo
menos n?
Problema 24. Sejam mo , m1 , ..., mr inteiros positivos que são primos entre si, dois a dois.
Mostre que existem r+1 inteiros consecutivos s, s+1, ..., s+r tal que mi divide s+i para i =
0, 1, ..., r.
Problema 25. (Romênia 1995) Seja f : N−{0, 1} → N definida por f (n) = mmc[1, 2, ..., n].
Prove que para todo n ≥ 2, existem n números consecutivos para os quais f é constante.

6
Problema 26. (OBM 2005) Dados os inteiros positivos a, c e o inteiro b, prove que existe
um inteiro positivo x tal que ax + x ≡ b (mod c).
Problema 27. (Cone Sul 2003) Demonstrar que existe uma sequência de inteiros positivos
x1 , x2 , . . . que satisfaz as duas condições seguintes:

(a) contém exatamente uma vez cada um dos inteiros positivos,

(b) a soma parcial x1 + x2 + . . . xn é divisı́vel por nn .

Problema 28. (República Tcheca e Eslovaca 1997) Mostre que existe uma sequência cres-
cente {an }∞
n=1 de números naturais tais que para k ≥ 0 , a sequência {an + k} contém um
número finito de primos.
Problema 29. Considere o inteiro c ≥ 1 e a sequência definida por a1 = c e ai+1 = cai .
Mostre que esta sequência se torna eventualmente constante quando a reduzimos módulo n
para algum inteiro positivo n (isto significa que am ≡ aj (mod n) se m ≥ j).
Problema 30. (Coréia 1999) Encontre todos os inteiros n tais que 2n − 1 é um múltiplo de
2n − 1
3e é um divisor de 4m2 + 1 para algum inteiro m.
3
Problema 31. (OBM 2006) Prove que, para todo inteiro n ≤ 2, o número de matrizes
quadradas 2 × 2 com entradas inteiras e pertencentes ao conjunto {0, 1, 2, . . . , n − 1} que
têm determinante da forma kn + 1 para algum k inteiro é dado por
 
Y 1
1− 2 .
p
p primo
p|n

Problema 32. Encontre todos os subconjuntos S ⊂ Z+ tais que todas as somas de uma
quantidade finita de elementos de S(com possı́veis repetições de elementos) são números
compostos.
Problema 33. Existe algum natural n para o qual existem n − 1 progressões aritméticas
com razões 2, 3, . . . , n tais que qualquer natural está em pelo menos uma das progressões?
Problema 34. Seja P (X) um polinômio com coeficientes inteiros e k é um inteior qualquer.
Prove que existe um inteiro m tal que P (m) tem pelo menos k fatores primos distintos.

Acompanhe as dicussões dos problemas propostos no fórum do POTI:


www.poti.impa.br/forum/
Referências
[1] F. E. Brochero Martinez, C. G. Moreira, N. C. Saldanha, E. Tengan - Teoria dos
Números ? um passeio com primos e outros números familiares pelo mundo inteiro,
Projeto Euclides, IMPA, 2010.

[2] E. Carneiro, O. Campos and F. Paiva, Olimpı́adas Cearenses de Matemática 1981-2005


(Nı́veis Júnior e Senior), Ed. Realce, 2005.

[3] S. B. Feitosa, B. Holanda, Y. Lima and C. T. Magalhães, Treinamento Cone Sul 2008.
Fortaleza, Ed. Realce, 2010.

[4] D. Fomin, A. Kirichenko, Leningrad Mathematical Olympiads 1987-1991, MathPro


Press, Westford, MA, 1994.

[5] D. Fomin, S. Genkin and I. Itenberg, Mathematical Circles, Mathematical Words, Vol.
7, American Mathematical Society, Boston, MA, 1966.

[6] I. Niven, H. S. Zuckerman, and H. L. Montgomery, An Introduction to the Theory of


Numbers.
Polos Olímpicos de Treinamento
Curso de Teoria dos Números - Nível 2 Aula 12
Prof. Samuel Feitosa

Equações Diofantinas II

Continuaremos nosso estudo das equações diofantinas abordando agora algumas equações
quadráticas. Começaremos peloo clássico problema das ternas pitagóricas.

Desejamos encontrar todas as soluções (x, y, z) da equação:

x2 + y 2 = z 2 ,
x y z 
em inteiros positivos. Seja d = mdc(x, y). Como d2 | z 2 , segue que d | z e que , ,
d d d
também é solução. Além disso, podemos concluir que:

mdc(x/d, y/d) = mdc(x/d, z/d) = mdc(y/d, z/d) = 1.

Uma terna que é solução e possui a propriedade de que quaisquer dois de seus termos
são primos entre si, será chamada de solução primitiva. Assim, toda solução (x, y, z) é da
forma (dx1 , dy1 , dz1 ) onde (x1 , y1 , z1 ) é uma solução primitiva. Para cumprimirmos nosso
objetivo, bastará nos concentrarmos em encontrar todas as soluções primitivas. Analisando
a equação módulo 4 e lembrando que todo quadrado perfeito pode deixar apenas os restos 0
ou 1, concluı́mos que exatamente um dentre x e y é par. Suponha sem perda de generalidade
que y seja par. Fatorando a equação, obtemos:
z+x z−x  y 2
· =
2 2 2

Como mdc((z + x)/2, (z − x)/2) = 1, concluı́mos que (z + x)/2 e (z − x)/2 devem ser ambos
quadrados perfeitos, i.e., existem inteiros positivos r e s, com r > s e mdc(r, s) = 1, tais que
(z + x)/2 = r 2 e (z − x)/2 = s2 (veja o primeiro problema proposto). Consequentemente,
x = r 2 − s2 , y = 2rs e z = r 2 + s2 . Reciprocamente, se (x, y, z) = (r 2 − s2 , 2rs, r 2 + s2 ),
temos:

x2 + y 2 = (r 2 − s2 )2 + (2rs)2 = (r 2 + s2 )2 = z 2 .

O próximo teorema resume nossa discussão original:


POT 2012 - Teoria dos Números - Nı́vel 2 - Aula 12 - Samuel Feitosa

Teorema 1. Todas as soluções primitivas de x2 +y 2 = z 2 com y par são da forma x = r 2 −s2 ,


y = 2rs e z = r 2 + s2 , onde r e s são inteiros de paridade oposta com r > s > 0 e
mdc(r, s) = 1.

Exemplo 2. Encontre todas as ternas pitagóricas (a, b, c) tais que a + b + c = 1000.

Seja k = mdc(a, b, c) e suponha sem perda de generalidade que b/k é par. Pelo teorema
anterior, (a, b, c) = (k(x2 − y 2 ), k(2xy), k(x2 + y 2 )), onde x > y, mdc(x, y) = 1 e pelo menos
um dentre x e y par. Assim, (x2 − y 2 ) + 2xy + (x2 + y 2 ) = 2x(x + y) é um divisor de 1000.
Com mais razão, x(x + y) | 500. Usando que mdc(x, x + y) = 1 e a fatoração em primos de
500, podemos concluir que um deles é uma potência de 5 e o outro uma potência de 2. Veja
que x não pode ser uma potência de 5 pois nesse caso y deveria ser ı́mpar para garantir
que x + y seja uma potência de 2. Assim, x | 500 e x = 2k , produzindo como possibilidades
x = 1, 2 ou 4. Analisando cada um desses casos e levando em conta que y < x, é fácil
encontrar que x = 4 e y = 1 são as únicas opções possı́veis. Nesse caso, x = 15, y = 8 e
z = 17. Consequentemente, (a, b, c) = (20 · 15, 20 · 8, 20 · 17).

Exemplo 3. Mostre que se a, b e c são inteiros positivos tais que a2 + b2 = c2 , então


(ab)4 + (bc)4 + (ca)4 é um quadrado perfeito.

Veja que:

(ab)4 + (bc)4 + (ca)4 = (a2 b2 + b4 )2 + (a2 b2 )2 + (a2 b2 + a4 )2 = (a4 + a2 b2 + b4 )2 .

Exemplo 4. Encontre todas as soluções de x2 + 2y 2 = z 2 em inteiros positivos com


mdc(x, y, z) = 1.

Como 2y 2 ≡ 0 (mod 2), devemos ter x ≡ z (mod 2). Além disso, se fosse x ≡ z ≡ 0
(mod 2) terı́amos 4 | z 2 − x2 = 2y 2 e consequentemente 2 | y, contradizendo a hipóstese
mdc(x, y, z) = 1. Fatorando a expressão, temos:

2y 2 = (z − x)(z + x).

Como mdc(x, z) = 1 e ambos são ı́mpares; mdc(z − x, z + x) = 2 e apenas um deles é


côngruo à 2 (mod 4). Temos dois casos a considerar: 1) z + x ≡ 0 (mod 4) e z − x ≡ 2
(mod 4). Nesse caso, y 2 = (z − x)/2 · (z + x) com mdc((z − x)/2, (z + x)) = 1. Daı́,
existem inteiros positivos r e s tais que (z − x)/2 = r 2 e (z + x)/2 = s2 , produzindo a
solução (x, y, z) = ((s2 − 2r 2 )/2, rs, (2r 2 + s2 )/2) com mdc(r, s) = 1 e s ≡ 0 (mod 2). Um
raciocı́nio análogo para o caso z + x ≡ 2 (mod 4) e z − x ≡ 0 (mod 4) produz (x, y, z) =
((2s2 − r 2 )/2, rs, (r 2 + 2s2 )/2) com mdc(r, s) = 1 e r ≡ 0 (mod 2).

Problema 5. (USAMO 1976) Encontre todas as soluções naturais da equação

a2 + b2 + c2 = a2 b2 .

A equação pode ser reescrita como:

c2 = (a2 − 1)(b2 − 1) − 1.

2
POT 2012 - Teoria dos Números - Nı́vel 2 - Aula 12 - Samuel Feitosa

Se pelo menos um dentre a ou b é ı́mpar, teremos c2 ≡ 3 (mod 4). Como os quadrados


perfeitos só podem deixar resto 0 ou 1 (mod 4), temos um absurdo. Portanto, a, b e
consequentemente c são números pares. Seja k o maior inteiro tal que 2k divida esses três
números. Assim, a = 2x , b = 2k y, c = 2k z onde pelo menos um dentre x, y e z ı́mpar.
Assim,
x2 + y 2 + z 2 = 22r x2 y 2 .
Como r > 0, x2 + y 2 + z 2 ≡ 0 (mod 4). Entretanto, isso não é possı́vel se um dentre os
x, y, z é ı́mpar pois a soma só poderia ser congruente à 1, 2, 3 (mod 4).
Exemplo 6. (Extraı́do de [1]) Determine todas as ternas (a, b, c) de inteiros positivos tais
que a2 = 2b + c4 .
Como a2 = 2b + c4 ⇐⇒ (a − c2 )(a + c2 ) = 2b , pelo Teorema Fundamental da Aritmética
existem dois naturais m > n tais que m + n = b, a − c2 = 2n e a + c2 = 2m . Subtraindo as
duas últimas equações, obtemos que 2c2 = 2m − 2n , assim c2 = 2n−1 (2m−n − 1). Como 2n−1
e 2m−n − 1 são primos entre si e o seu produto é um quadrado perfeito (i.e. os expoentes
das potências de primos distintos são pares), novamente pelo Teorema Fundamental da
Aritmética 2n−1 e 2m−n − 1 devem ser ambos quadrados perfeitos, logo n − 1 é par e
2m−n −1 = (2k−1)2 para algum inteiro positivo k. Como 2m−n = (2k−1)2 +1 = 4k(k−1)+2
é divisı́vel por 2 mas não por 4, temos m − n = 1. Assim, fazendo n − 1 = 2t, temos que
todas as soluções são da forma (a, b, c) = (3 · 22t , 4t + 3, 2t ) com t ∈ N e é fácil verificar que
todos os números desta forma são soluções.
O próximo exemplo ilustrará o método da descida de Fermat que faz uso do princı́pio
da boa ordenação: todo subconjunto não vazio de inteiros positivos possui um elemento
mı́nimo.
Exemplo 7. Determine todas as soluções da equação x4 + y 4 = z 2 em inteiros positivos
com mdc(x, y) = 1.
Como (x2 )2 + (y 2 )2 = z 2 e mdc(x2 , y 2 ) = 1, podemos usar o primeiro teorema para concluir
que existem u e v tais que x2 = u2 − v 2 , y 2 = 2uv, z = u2 + v 2 , u > v > 0 e mdc(u, v) = 1
(Estamos assumindo sem perda de generalidade que x é ı́mpar). Se u é par, então v será
ı́mpar e teremos x2 ≡ 3 (mod 4). Como isso é um absurdo, u deve ser ı́mpar e v deve ser
par. Sendo assim, (y/2)2 = u · v/2 com mdc(u, v/2) = 1. Devemos ter u = r 2 , v/2 = s2 ,
com mdc(r, s) = 1, r, s > 0, r ı́mpar e y = 2rs. Além disso, como x2 + v 2 = u2 , obtemos
x2 + 4s2 = r 4 . Como mdc(r, 2s) = 1, novamente pelo primeiro teorema, existem m e n tais
que x = m2 − n2 , 2s2 = 2mn e r 2 = m2 + n2 com mdc(m, n) = 1 e m > n > 0. Como
mn = s2 , podemos escrever m = f 2 e n = g2 com f, g > 0 e mdc(f, g) = 1. Portanto,
r 2 = f 4 + g4 . Note que dada a solução em inteiros positivos (x, y, z), obtivemos outra
solução (f, g, r), também nos inteiros positivos, com 0 < r < z. Isso nos diz que existe
uma infinidade decrescente de possı́veis valores para o inteiro positivo z e naturalmente
obtemos uma contradição do princı́pio da boa ordenação. Sendo assim, a equação anterior
não possui solução nos inteiros positivos.
Observação 8. Outra maneira de formalizar o argumento anterior é escolher dentre as
ternas nos inteiros positivos que são soluções, aquela com z mı́nimo. A nova terna (f, g, r)
caracterizaria um absurdo.

3
POT 2012 - Teoria dos Números - Nı́vel 2 - Aula 12 - Samuel Feitosa

Exemplo 9. Prove que para todo inteiro n > 2, existem inteiros positivos p e q tais que
n 2 + q 2 = p2 .

Fatorando a expressão, obtemos n2 = (p − q)(p + q). Se n é ı́mpar, podemos encontrar p e


q tais que p + q = n2 e p − q = 1, bastando para isso resolver o sistema originado, obtendo
2 2
(n, q, p) = (n, n 2−1 , n 2+1 ). Se n é par, podemos fazer algo semelhante e encontrar p e q tais
2 2
que p + q = n2 /2 e p − q = 2, cuja solução é (n, q, p) = (n, n4 − 1, n4 + 1).

Exemplo 10. (Extraı́do de [3]) Prove que a equação

x2 + y 2 + z 2 + w2 = 2xyzw (1)

não possui soluções inteiras positivas.

Por contradição, suponha que (1) possua pelo menos uma solução não-trivial, digamos
(x0 , y0 , z0 , w0 ). Se x0 , y0 , z0 , w0 forem todos ı́mpares, o lado esquerdo é um múltiplo de 4
e o lado direito não. Se apenas um ou três deles forem pares, o lado esquerdo é ı́mpar e o
direito é par. Se dois deles forem pares e dois forem ı́mpares, o lado direito é um múltiplo
de quatro e o esquerdo não. Desse modo, x0 , y0 , z0 , w0 são todos pares, ou seja, x0 = 2x1 ,
y0 = 2y1 , z0 = 2z1 e w0 = 2w1 . Substituindo em (1) e dividindo por quatro, concluı́mos
que x1 , y1 , z1 , w1 satisfazem a igualdade

x21 + y12 + z12 + w12 = 8x1 y1 z1 w1 .

Com uma análise de paridades análoga à acima, obtemos x1 = 2x2 , y1 = 2y2 , z1 = 2z2 e
w1 = 2w2 , e daı́
x22 + y22 + z22 + w22 = 32x2 y2 z2 w2 .
Procedendo dessa maneira, x0 , y0 , z0 , w0 devem ser todos múltiplos de 2n , qualquer que seja
n ≥ 1. Então x0 = y0 = z0 = w0 = 0, absurdo.

Exemplo 11. (Extraı́do de [3]) Encontre todas as quadrúplas (x, y, z, k) de números inteiros,
com x, y, z > 0 e k ≥ 0, tais que

x6 + y 6 + z 6 = 4826 · 7k .

Vamos mostrar o seguinte fato:

(x, y, z, k) é solução, com k ≥ 1 ⇐⇒ (x/7, y/7, z/7, k − 6) é solução,


e nesse caso k ≥ 6.

(=⇒) Temos x6 + y 6 + z 6 ≡ 0 (mod 7). Como x6 , y 6 , z 6 ≡ 0 ou 1 (mod 7), devemos ter


x, y, z múltiplos de 7. Daı́, 76 |4826 · 7k ⇒ 76 |7k ⇒ k ≥ 6. Ademais, vale a igualdade
 x 6  y 6  z 6
+ + = 4826 · 7k−6 ,
7 7 7

4
ou seja, (x/7, y/7, z/7, k − 6) também é solução.
(⇐=) Claro.
O fato acima garante que podemos ir subtraindo 6 de k e retirando um fator 7 de x, y,
z enquanto k ≥ 1, até que o expoente de 7 no lado direito da igualdade seja 0. Em outras
palavras, existe n ≥ 0 tal que k = 6n, com x = 7n · x0 , y = 7n · y0 , z = 7n · z0 , e

x0 6 + y0 6 + z0 6 = 4826.

A equação acima só tem a solução (1, 3, 4) e suas permutações. Assim, as soluções da
equação original são (7n , 3 · 7n , 4 · 7n , 6n), n ≥ 0, e suas permutações nas três primeiras
coordenadas.

Problemas Propostos

Problema 12. Mostre que se a · b = x2 e mdc(a, b) = 1 então existem r e s tais que a = r 2


e b = s2 .
1 1 1
Problema 13. Prove que todas as soluções positivas da equação 2 + 2 = 2 com
x y z
mdc(x, y, z) = 1 são dadas por

(x, y, z) = (r 4 − s4 , 2rs(r 2 + s2 ), rs(r 2 − s2 ))

ou
(x, y, z) = (2rs(r 2 + s2 ), r 4 − s4 , rs(r 2 − s2 )),
onde r > s > 0, mdc(r, s) = 1 e r e s de paridades opostas.
Problema 14. Encontre todos os pares de racionais (x, y) tais que x2 + y 2 = 1.
Problema 15. Resolva simultaneamente em inteiros positivos:

a2 + b2 = c2
a2 + c2 = d2

onde a, b, c e d são inteiros positivos relativamente primos entre si dois dois.


Problema 16. (Torneio das Cidades 1997) Prove que a equação

x2 + y 2 − z 2 = 1997

tem infinitas soluções inteiras (x, y, z).


Problema 17. Encontre todas as soluções inteiras de x2 + y 2 + z 2 = t2 .
Problema 18. Encontre todas as soluções de 5m2 + n2 = 52011
Problema 19. Encontre todas as soluções em números naturais m e n da equação:

m2 = 1 + 2 + . . . + n.
Referências
[1] F. E. Brochero Martinez, C. G. Moreira, N. C. Saldanha, E. Tengan - Teoria dos
Números: um passeio com primos e outros números familiares pelo mundo inteiro,
Projeto Euclides, IMPA, 2010.

[2] E. Carneiro, O. Campos and F. Paiva, Olimpı́adas Cearenses de Matemática 1981-2005


(Nı́veis Júnior e Senior), Ed. Realce, 2005.

[3] S. B. Feitosa, B. Holanda, Y. Lima and C. T. Magalhães, Treinamento Cone Sul 2008.
Fortaleza, Ed. Realce, 2010.

[4] D. Fomin, A. Kirichenko, Leningrad Mathematical Olympiads 1987-1991, MathPro


Press, Westford, MA, 1994.

[5] D. Fomin, S. Genkin and I. Itenberg, Mathematical Circles, Mathematical Words, Vol.
7, American Mathematical Society, Boston, MA, 1966.

[6] I. Niven, H. S. Zuckerman, and H. L. Montgomery, An Introduction to the Theory of


Numbers.
Polos Olímpicos de Treinamento
Curso de Teoria dos Números - Nível 2 Aula 13
Prof. Samuel Feitosa

Equações Diofantinas III

Já estudamos as equações diofantinas lineares e equações em que alguma fatoração conveni-
ente poderia facilitar a busca por soluções. Nesta aula, estaremos interessados em encontrar
módulos convenientes para analisar os termos de uma equação.

Exemplo 1. Encontre todas as soluções em inteiros da equação x2 − 7y = 1004.

Analisando os restos na divisão por 7, obtemos x2 ≡ 3 (mod 7). Entretando, os únicos


inteiros que são restos de quadrados perfeitos na divisão por 7 são 0, 1, 2 e 4. Como
3 ≡ 1004 (mod 7) não faz parte dessa lista, não existem soulçoes inteiras para a equação.

Exemplo 2. Encontre todas as soluções em inteiros da equação x3 + 98y 2 + 5 = 0.

Analisemos os possı́veis restos de x3 (mod 7) fazendo uma tabela dos restos correspondentes
de x e x3 :

x 0 1 2 3 4 5 6
x3 0 1 1 6 1 6 6

Como os únicos restos possı́veis são 0, 1, −1 (mod 7), o lado esquerdo da equação só pode
deixar resto 5, 6, 4 (mod 7). Como o resto do lado direito não faz parte dessa lista, não
existem soluções em inteiros.

Exemplo 3. Prove que a equação x2 = 3y 2 + 8 não possui soluções em inteiros x e y.

Analisando o resto na divisão por 3, obtemos x2 ≡ 2 (mod 3). Como os únicos restos de
um quadrado por 3 são 0 e 1, não existem soluções em inteiros.

Nos próximo problema, usaremos congruências para encontrarmos informações sobre as


incógnitas envolvidas nos expoentes e buscaremos alguma fatoração apropriada para reduzir
o problema à resolução de um sistema de equações.

Exemplo 4. Encontre todas as soluções em inteiros positivos da equação 3m + 7 = 2n


POT 2012 - Teoria dos Números - Nı́vel 2 - Aula 13 - Samuel Feitosa

Analisando o resto módulo 3 do lado esquerdo, podemos concluir que 2n ≡ 1 (mod 3).
Como 2n ≡ (−1)n (mod 3), concluı́mos que n é par, ou seja, n = 2k, para algum k ∈ N.
Assim, como o lado direito é múltiplo de 4, podemos concluir que:

3m ≡ −7 (mod 4)
(−1)m ≡ 1 (mod 4)

Logo, m é par, ou seja, m = 2t, para algum t ∈ N. Usando diferença de quadrados,


podemos escrever:
7 = (2k − 3t )(2k + 3t ).
Como 7 é primo, temos as seguintes opções:

7 = 2k + 3t ⇒ 2k − 3t = 1
1 = 2k + 3t ⇒ 2k − 3t = 7

Em ambos os casos, 8 = 2k+1 e daı́ k = 2. Substituindo nas equações, obtemos solução


apenas no primeiro caso com t = 1. Assim, (m, n) = (2, 4).

Exemplo 5. Encontre todas as soluções em inteiros positivos da equação 3 · 2m + 1 = n2 .

Analisandoa equação módulo 3, n2 ≡ 1 (mod 3) e assim, n ≡ ±1 (mod 3). No primeiro


caso, se n = 3k + 2, temos 3 · 2m + 1 = n2 = 9k 2 + 12k + 4 e daı́ 2m = (3k + 1)(k + 1).
Como o lado esquerdo possui apenas um fatores 2, temos 3k + 1 = 2i , k + 1 = 2j ,com
j ≤ i. Daı́, 3 · 2j − 2i = 2. Se j = i, temos 2i+1 = 2 e consequentemente i = 0 produzindo
k = 0 e (m, n) = (0, 2). Se j < i, temos j = 1 pois o lado esquerdo possui um único fator
2 e por conseguinte, i = 2, (m, n) = (3, 5). No segundo caso, quando n = 3k + 1, é tratado
analogamente produzindo apenas a nova solução (m, n) = (4, 7).

Exemplo 6. Encontre todas as soluções da equação x2 − xy + y = 3 em inteiros x, y.

Fixado o valor de y, podemos encontrar os valores de x usando a fórmula de Báskara. Como


x é inteiro, o discriminante y 2 − 4(y − 3) = (y − 2)2 + 8 deve ser um quadrado perfeito,
digamos z 2 . Assim,

z 2 − (y − 2)2 = (z − y + 2)(z + y − 2) = 8.

Como z − y + 2 e z + y − 2 possuem a mesma paridade, o produto anterior dever (±2) · (±4).


Em qualquer caso, somando ambos os termos, obtemos 2z = ±6 e z = ±3. Logo, y−2 = ±1.
Substituindo os valores de y na equação original, obtemos os valores correspondentes para
x. As soluções são: (x, y) = (2, 1), (−1, 1), (0, 3), (3, 3).

Exemplo 7. (Hungria 1969) Seja n um inteiro positivo. Prove que se 2 + 2 28n2 + 1 é um
inteiro, então é um quadrado perfeito.

2
POT 2012 - Teoria dos Números - Nı́vel 2 - Aula 13 - Samuel Feitosa


Necessariamente 28n2 + 1 deve ser racional e para isso 28n2 + 1 deve ser um quadrado
perfeito. Assim,

28n2 + 1 = t2  
2 t−1 t+1
7n =
2 2

t+1 t−1
Como 7 é primo, 7 | ou que 7 | . No primeiro caso,
2 2
  
2 t+1 t−1
n +1 =
14 2

Além disso, como mdc((t − 1)/2, (t + 1)/2) = 1, existem a e b tais que


t+1
a2 =
14
t−1
b2 =
2
Daı́, 7a2 − b2 = 1 e b2 ≡ −1 (mod 7). Como quadrados perfeitos só podem deixar restos
0, 1, 2, 4 (mod 7), esse caso não gera soluções. No segundo caso,
t+1
a2 =
2
t−1
b2 = .
14

Logo, 2 + 2 28n2 + 1 = 2 + 2t = 2 + 2(2a2 − 1) = (2a)2 .

Exemplo 8. (Reino Unido 1996) Encontre todas as soluções em inteiros não negativos
x, y, z da equação:
2x + 3y = z 2 .

Se y = 0, então 2x = z 2 − 1 = (z − 1)(z + 1). Analisando a fatoração em primos, existem


i, j, com i > j, tais que z + 1 = 2i e z − 1 = 2j . A diferença das duas equações produz
2 = 2i − 2j = 2j (2i−j − 1). Como o lado esquerdo possui apenas um fator 2, j = 1 e
i − j = 1. Nossa primeira solução encontrada é (x, y, z) = (3, 0, 3). Se y > 0, 2x ≡ z 2
(mod 3). Como 2x ≡ ±1 (mod 3) e z 2 ≡ 0, 1 (mod 3) temos, 2x ≡ z 2 ≡ 1. Isso implica
que x é par, ou seja, x = 2m. Fatorando, obtemos:

3y = z 2 − 22m
= (z − 2m )(z + 2m )

Novamente, analisando a fatoração em primos, existem l e k, com l < k, tais que z − 2m =


3l , z + 2m = 3k . A diferença das duas equações produz 2m+1 = 3l (3k−l − 1). Novamente

3
POT 2012 - Teoria dos Números - Nı́vel 2 - Aula 13 - Samuel Feitosa

analisando a fatoração em primos, l = 0 e 2m+1 = 3k − 1. Se m = 0, temos k = 1 e


(x, y, z) = (0, 1, 2). Se m > 0,

3k = 1 (mod 4)
(−1)k = 1 (mod 4).

e devemos ter k par, ou seja, existe t tal que k = 2t. Fatorando novamente, 2m+1 =
(3k − 1)(3k + 1). Escrevendo 3k + 1 = 2p e 3k − 1 = 2q , temos 2 = 2q (2p−q − 1). Veja
que já tratamos essa equação no inı́cio e assim podemos concluir que q = 1 e p − q = 1.
Produzindo a solução (x, y, z) = (4, 2, 5).

Nos próximos dois problemas, contruiremos soluções indutivamente.


Exemplo 9. (Bulgária) Prove que para qualquer número natural n ≥ 3, existem números
naturais ı́mpares xn e yn tais que 7x2n + yn2 = 2n .
Para n = 3, basta tomar x1 = y1 = 1. Suponha que tenhamos encontrado xk e yk ı́mpares,
satisfazendo
7x2k + yk2 = 2k .
Um dos números (xk + yk )/2, (xk − yk )/2 é ı́mpar e assim podemos escolher um deles de
modo a satisfazer o enunciado para k + 1:

x k ± yk 2 x k ∓ yk 2
   
7 + = 2(7x2k + yk2 ) = 2k+1 .
2 2
Exemplo 10. Mostre que existe uma sequência infinita de inteiros positivos a1 , a2 , . . . tais
que a21 + a22 + . . . + a2n é um quadrado perfeito para todo n natural.
Definamos a1 = 3. Suponha que a sequência já esteja definida para a1 , a2 , . . . , ak com

a21 + a22 + . . . + a2k = (2t + 1)2 .

Vejamos que podemos definir o próximo termo de modo que a soma de todos os primeiros
k + 1 termos ao quadrado ainda seja um quadrado perfeito de um inteiro ı́mpar. Basta
fazer ak+1 = 2t2 + 2t. Veja que:

a21 + a22 + . . . + a2k + a2k+1 = (2t + 1)2 + (2t2 + 2t)2


= (2t2 + 2t + 1)2 .

que é novamente o quadrado de um ı́mpar.

Problemas Propostos

Problema 11. Encontre todas as soluções em inteiros x, y, z, t da equação:

x2 + y 2 + z 2 = 8t − 1.

4
POT 2012 - Teoria dos Números - Nı́vel 2 - Aula 13 - Samuel Feitosa

Problema 12. Encontre todas as soluções em inteiros positivos da equação


1 1 1
+ + = 1.
a b c
Problema 13. Encontre todas as soluções em inteiros de x2 − y 2 − 1988

Problema 14. Mostre que para todo inteiro z, existem inteiros x e y satisfazendo x2 − y 2 =
z3

Problema 15. Encontre todas as soluções de 1 + x + x2 + x3 = 2y em inteiros positivos x


e y.

Problema 16. Mostre que a equação diofantina 5m2 − 6mn + 7n2 = 1988, não possui
solução nos inteiros.

Problema 17. (Rússia 1996) Sejam x, y, p, n, k números naturais tais que

x n − y n = pk .

Prove que se n > 1 é ı́mpar, e p é um primo ı́mpar, então n é uma potência de p.

Problema 18. (Rússia 1997) Encontre todas as soluções inteiras da equação

(x2 − y 2 )2 = 1 + 16y.

Problema 19. (OBM 2009) Prove que não existem inteiros positivos x e y tais que x3 +y 3 =
22009 .

Apêndice: A conjectura de Catalan


Em alguns dos problemas anteriores, nos deparamos com a questão de encontrarmos duas
potências perfeitas consecutivas não triviais. As únicas soluções que apareceram foram
23 = 8 e 32 = 9. Em 1844, Eugène Catalan conjecturou que essa seria a única solução.
Recentemente, tal conjectura se mostrou verdadeira atráves do:

Teorema 20. (Mihalnescu - 2002)Existe uma única solução nos números naturais de

xa − y b = 1,

com x, a, y, b > 1 que é (x, y, a, b) = (3, 2, 2, 3).

Problema 21. Encontre toda as soluções em inteiros positivos da seguinte equação diofan-
tina:
2y 2 = x4 + x.

5
Polos Olímpicos de Treinamento
Curso de Teoria dos Números - Nível 2 Aula 14
Prof. Samuel Feitosa

A Função Parte Inteira - I

1 O jogo de Wythoff
O objetivo da aula de hoje é resolver o seguinte problema:
Exemplo 1. Dois jogadores jogam alternadamente removendo pedras de duas pilhas sobre
uma mesa. Na sua vez, cada jogador pode remover qualquer quantidade de pedras de uma
pilha ou igual número de pedras de ambas as pilhas. O ganhador é aquele que retirar a
última pedra. Determine todas posições perdedoras.
Uma boa estratégia para identificar as posições perdedoras nesse jogo, é associar o mo-
vimento dos jogadores ao movimento de uma peça em um tabuleiro. Suponha que ini-
cialmente as duas pilhas possuem 5 e 7 pedras. Colocaremos uma peça no canto direito
superior em um tabuleiro 8 × 6. O movimento de retirar x pedras da coluna de 5 será
traduzido como um deslocamento vertical de x casas para baixo, enquanto que a mesma
retirada da outra coluna será traduzido como um movimento horizontal para a esquerda
de mesmo deslocamento. Um movimento de retirada de x pedras de ambas as colunas
será traduzido como um deslocamento diagonal da direita para a esquerda dessa mesma
quantidade de casas. O jogo terminará quando a peça chegar na casa do canto extremo
esquerdo simbolizando que ambas as colunas estão com 0 pedras.

A posição (0, 0) é perdedora porque uma vez que um jogador a receba, ele terá perdido
o jogo. Qualquer posição do tipo (x, 0), (0, x) ou (x, x), com x > 0, será uma posição
vencedora. Marquemos essas posições no tabuleiro:
POT 2012 - Teoria dos Números - Nı́vel 2 - Aula DEFINIÇ
2 14 ÃO EFeitosa
- Samuel PROPRIEDADES

+ +
+ +
+ +
+ +
++
−+++++++

As próximas posições perdedoras que encontramos são (1, 2),(2, 1). A partir dessa nova
posição, podemos preencher o tabuleiro com as novas posições vencedoras.

+ + + + ++
+ + + + + +
+ + + + +
+ − + + + +++
+ + − + + +++
− + + + + +++

As próximas posições perdedoras que encontramos são (3, 5) e (5, 3). Como existe simetria
entre as duas pilhas, basta procurarmos as posições perdedoras (x, y) com x < y. Repetindo
o processo anterior, podemos listar as primeiras posições perdedoras ordenadas (xn , yn ) com
x n < yn .
n 0 1 2 3 4 5 6 7 8 9 10 11 12
xn 0 1 3 4 6 8 9 11 12 14 16 17 19
yn 0 2 5 7 10 13 15 18 20 23 26 28 31
As próximas seções nos ajudarão a estabelecer alguma padrão entre os valores de (xn , yn )
em função de n.

2 Definição e Propriedades
Definição 2. A parte inteira de um número real x é o maior inteiro ⌊x⌋ que não é maior
que x. Definimos a parte fracionária {x} de x por {x} = x − ⌊x⌋. (exemplos: ⌊3⌋ =
3 , ⌊3, 5⌋ = 3 e ⌊−4, 7⌋ = −5)
Teorema 3. Sejam x e y números reais. Então:
1. ⌊x⌋ ≤ x < ⌊x⌋ + 1 e 0 ≤ {x} < 1.

2. ⌊x + m⌋ = ⌊x⌋ + m se m é um inteiro.

3. ⌊x⌋ + ⌊y⌋ ≤ ⌊x + y⌋ ≤ ⌊x⌋ + ⌊y⌋ + 1.


j k  
4. ⌊x⌋
m = m x
se m é um inteiro positivo.
jnk
5. Se n e a são inteiros positivos, é o número de inteiros entre 1, 2, ..., n que são
a
divisı́veis por a.

2
POT 2012 - Teoria dos Números - Nı́vel 2 - Aula DEFINIÇ
2 14 ÃO EFeitosa
- Samuel PROPRIEDADES

Demonstração. Os primeiros dois itens decorrem facilmente da definição e serão deixados


a cargo do leitor.
Para provar (3), veja que:

⌊x⌋ + ⌊y⌋ ≤ ⌊x⌋ + ⌊y⌋ + ⌊{x} + {y}⌋


= ⌊⌊x⌋ + ⌊y⌋ + {x} + {y}⌋
= ⌊x + y⌋.

Como {x} + {y} < 2, ⌊{x} + {y}⌋ ≤ 1 e daı́:

⌊x + y⌋ = ⌊x⌋ + ⌊y⌋ + ⌊{x} + {y}⌋


≤ ⌊x⌋ + ⌊y⌋ + 1.

Para provar (4), seja ⌊x⌋ = qm + r com 0 ≤ r < m − 1, então:


  j
⌊x⌋ rk
= q+ = q.
m m

Como 0 ≤ {x} < 1,


    j k
r + {x} qm + r + {x} x
q=q+ = = .
m m m

Finalmente, para provar (5), sejam a, 2a, ..., ja todos os inteiros positivos ≤ n que são
divisı́veis por a. Então,
n
ja ≤ n < (j + 1)a ⇒ j ≤ <j+1
jan k
⇒ j= .
a
Observação 4. Em alguns dos problemas desta seção, será usada a notação de somatório.
Recomenda-se que o professor escrevar por extenso os primeiros somatórios até que os
alunos se sintam confortáveis com a manipulação dos ı́ndices.
Teorema 5 (Fórmula de Polignac). Seja p um primo. Então o maior expoente p na fa-
toração em primos de n! é:
∞  
X n
tp (n) = .
pi
i=1
 
n
Demonstração. O que significa ? Ele conta o número de inteiros positivos menores ou
pi
iguais a n divisı́veis por pi . Cada múltiplo de p contribui com um expoente 1 para p em n!,
cada múltiplo de p2 contribui com expoente 2 para p em n! e assim sucessivamente. Então,
∞  
X n
é a soma de todas essas contribuições (veja que um múltiplo de pi é contado i
pi
i=1 j k j k j k
vezes em np , pn2 , ..., pni ).

3
POT 2012 - Teoria dos Números - Nı́vel 2 - Aula DEFINIÇ
2 14 ÃO EFeitosa
- Samuel PROPRIEDADES

Observação 6. Se p é primo e pα é a maior potência de p que divide n, usaremos a notação


pα k n.
Exemplo 7. Em quantos zeros termina a representação decimal de 1000!?
Para determinarmos o número de zeros, basta determinarmos a maior potência de 10 que
divide 1000!. Existem mais fatores 2 do que fatores 5 e assim bastará encontrarmos o
expoente de 5 na fatoração de 1000!. Pelo teorema anterior, tal número é:
∞  
X 1000
= 200 + 40 + 8 + 1 = 249.
5i
i=1
k−1  
X i
Exemplo 8. Mostre que x+ = ⌊kx⌋.
k
i=0
j j+1
Sejam a = ⌊x⌋, b = {x} e j ∈ {0, 1, . . . , k − 1} tal que ≤b< . Então ⌊kb⌋ = j.
k k
Além disso, ⌊b + i/k⌋ = 0 se i < k − j e ⌊b + j/k⌋ = 1 se k − j ≤ i ≤ k − 1. Daı́,
k−1
X k−1
X
⌊b + i/k⌋ = ⌊b + i/k⌋
i=0 i=k−j
= j
= ⌊kb⌋

Exemplo 9. Mostre que ⌊x + y⌋ + ⌊x⌋ + ⌊y⌋ ≤ ⌊2x⌋ + ⌊2y⌋.


Sejam a = {x} e b = {y}. A desigualdade é equivalente à

2⌊x⌋ + 2⌊y⌋ + ⌊a⌋ + ⌊b⌋ + ⌊a + b⌋ ≤ 2⌊x⌋ + 2⌊y⌋ + ⌊2a⌋ + ⌊2b⌋.


que pode ser reescrita como:

⌊a⌋ + ⌊b⌋ + ⌊a + b⌋ ≤ ⌊2a⌋ + ⌊2b⌋.

Temos que 0 ≤ ⌊a⌋ + ⌊b⌋ < 2 e ⌊a⌋ = ⌊b⌋ = 0. Se ⌊a + b⌋ = 1 segue que pelo menos um
dentre a, b é maior ou igual à 1/2. Daı́,

⌊a⌋ + ⌊b⌋ + ⌊a + b⌋ = 1
≤ ⌊2a⌋ + ⌊2b⌋.

Caso contrário, ⌊a⌋ + ⌊b⌋ + ⌊a + b⌋ = 0 e a desigualdade segue.


(2m)!(2n)!
Exemplo 10. Mostre que se m e n são inteiros positivos, então é um
(m)!(n)!(m + n)!
inteiro.

4
POT 2012 - Teoria dos Números - Nı́vel 2 - Aula DEFINIÇ
2 14 ÃO EFeitosa
- Samuel PROPRIEDADES

Pelo teorema anterior, basta mostrarmos que:


         
2m 2n m n m+n
+ k ≥ k + k + ,
pk p p p pk

para todo primo p e todo inteiro k. A desigualdade segue do exemplo anterior.


Exemplo 11. Prove que 2n

n divide M M C{1, 2, ..., 2n}.

Seja p um primo. Se pα k 2n β β+1 , pelo teorema anterior, temos:



n e p ≤ 2n < p

∞    
X 2n n
α = −2 j
pj p
j=1
β    
X 2n n
= −2 j
pj p
j=1
≤ β,

pois ⌊2x⌋ − 2⌊x⌋ = 0 ou 1. Como pβ k M M C{1, 2, ..., 2n} e α ≤ β, segue o resultado.


Exemplo 12. Mostre que
∞  
X n 1
+ = n.
2i 2
k=1

O número de inteiros que são múltiplos de 2k , mas não de 2k+1 é ⌊n/2k ⌋ − ⌊n/2k+1 ⌋. Se
n = 2k+1 q + r, esse número é q + ⌊r/2k ⌋. Para x ∈ [0, 1), é verdade que ⌊2x⌋ = ⌊x + 1/2⌋.
Assim,

⌊n/2k ⌋ − ⌊n/2k+1 ⌋ = q + ⌊r/2k ⌋


= ⌊r/2k + 1/2 + q⌋
= ⌊r/2k+1 + 1/2⌋

Somando a quantidade de números que são múltiplos de 2k mas não de 2k+1 entre 1 e n
para k = 0, 1, . . . obtemos a quantidade total de números, ou seja,
∞  
X n 1
+ = n.
2i 2
k=1

Teorema 13. Seja vp (n) a soma dos dı́gitos da representação de n na base p. Mostre que
n − vp n
o expoente de p na fatoração em primos de n! é .
p−1
Demonstração. Seja kp (n!) o maior expoente de p que divide n!. Considere a representação
de n na base p: n = d0 + d1 p + ... + dr pr onde 0 ≤ di < p. Então,

5
POT 2012 - Teoria dos Números - Nı́vel 2 - Aula DEFINIÇ
2 14 ÃO EFeitosa
- Samuel PROPRIEDADES

 
n
= d1 + d2 p + . . . + dr pr−1 ,
p
 
n
= d2 + d3 p + . . . + dr pr−2 ,
p2
  ...
n
= dr .
p

Somando tudo, obtemos:

kp (n!) = d1 + d2 (p + 1) + d3 (p2 + p + 1) + · · · + dr (pr−1 + · · · + 1)


1
d1 (p − 1) + d2 (p2 − 1) + · · · + dr (pr − 1)

=
p−1
n − vp (n)
=
p−1

Exemplo 14. Seja B(m) o conjunto dos inteiros r tais que 2r é um termo na representação
na base 2 de n. Por exemplo, B(100) = {2, 5, 6} pois 100 = 22 + 25 + 26 . Prove que nk é


ı́mpar se, e somente se, B(k) ⊆ B(n).


Aproveitando a notação dos teoremas anteriores, t2 (n) = n − v2 (n). Assim,
 
n
≡ 1 (mod 2) ⇔ [n − v2 (n)] − [(k − v2 (k)) − ((n − k) − v2 (n − k))] = 0
k
⇔ v2 (k) + v2 (n − k) − v2 (n) = 0
⇔ v2 (k) + v2 (n − k) = v2 (n).

A última equação nos diz que na soma das representações na base 2 de n − k e k não ocorre
a operação de ”vai um”, ou seja, B(k) ⊆ B(n).
Observação 15. Esse exemplo também mostra que nk ≡ 0 (mod 2) para todo k ∈ {1, 2, . . . , n−


1} se, e somente se, n é uma potência de 2.


Exemplo 16. (Olimpı́ada Rioplatense) Seja r um real tal que
     
19 20 92
r+ + r+ + ··· + r + = 554.
100 100 100

Calcule ⌊100r⌋ .
Sejam ⌊r⌋ = a e {x} = b. Podemos reescrever a equação como:
73  
X 19 + i
74a + b+ = 554.
100
i=0

6
POT 2012 - Teoria dos Números - Nı́vel 2 - Aula DEFINIÇ
2 14 ÃO EFeitosa
- Samuel PROPRIEDADES

 
19 + i
Como 0 ≤ b + < 2 para todo i ∈ {0, 1, . . . , 73}, segue que 74a ≤ 554 ≤ 74a + 74.
100
Não podemos ter 554 = 74(a + 1) pois 74 ∤ 554. Logo, a < 554/74 < a + 1 e por conseguinte
a = ⌊554/74⌋ = 7. Assim,
73  
X 19 + i
b+ = 554 − 74 · 7 = 36.
100
i=0
Pelo primeiro exemplo,
99  
X i
b+ = j.
100
i=0
 
j j+1 19
onde ≤b< . Não podemos ter r + = 1 pois nesse caso terı́amos
100 100 100
73  
X 19 + i
r+ ≥ 73 6= 36.
100
i=0
   
i i
Logo, r + = 0 para todo i ≤ 19. Como r + = 1 se 100 − j ≤ i ≤ 99, temos:
100 100
⌊100b⌋ = j
99  
X i
= r+
100
i=0
92      
X i 93 99
= r+ + r+ ... r +
100 100 100
i=19
= 36 + 7
= 43.
Finalmente,
⌊100r⌋ = 100a + ⌊100b⌋
= 700 + 43
= 743.
Exemplo 17. Prove que existe um natural n tal que a representação decimal de n2 começa
( da esquerda para a direita ) com o número 201120112011. . . 2011 ( 2011 vezes).
Podemos encontrar n tal que n2 comece com qualquer sequência de dı́gitos (c1 c2 ...cr ) = m.
√ √
Tome k suficientemente grande tal que 2 m < 10k−1 . Seja n = ⌊10k m + 1⌋. Então,
√ √
0 < 10k m < n ≤ 10k m + 1 ⇒

102k m < n2 ≤ 102k m + 2.10k m + 1 ⇒
102k m < n2 ≤ 102k m + 102k−1 + 1 ⇒
102k m < n2 < 102k (m + 1).
Assim, n2 começa com a sequência de dı́gitos m.

7
POT 2012 - Teoria dos Números - Nı́vel 2 - Aula DEFINIÇ
2 14 ÃO EFeitosa
- Samuel PROPRIEDADES

Exemplo 18. (OBM 1999) Prove que há pelo √ menos um algarismo diferente de zero entre
a 1000000a e a 3000000a casa decimal de 2 após a vı́rgula.
6 6√ 6√ 6√
Suponha que não, então 102·10 ⌊1010 2⌋ = ⌊103·10 2⌋. Se k = ⌊1010 2⌋, temos:
6 6√ 6
102·10 k ≤ 103·10 2 < 102·10 k + 1 ⇒
k √ k 1
·106 < 2 < ·106 + 3·106 ⇒
10 10 10
k2 k2 2k 1
6 < 2 < 6 + 6 +
10 2·10 102·10 104·10 10 6
6·10

√ 6
2k 2 21010 1
Como 2·106 < 6 ≤ ,
10 102·10 2
6
k 2 < 2 · 102·10 < k 2 + 1 ⇒
6
0 < 2 · 102·10 − k 2 < 1.
6
Isso é um absurdo pois 2 · 102·10 − k 2 ∈ Z.
1 1
Exemplo 19. ( Sequência de Beatty) Se α e β são irracionais satisfazendo α+β = 1 então,
as sequências
⌊α⌋, ⌊2α⌋, ⌊3α⌋, . . . ;
e
⌊β⌋, ⌊2β⌋, ⌊3β⌋, . . . ;
incluem todos os números naturais exatamente uma vez.
Primeiramente, provemos a unicidade. Suponha que ⌊kα⌋ = ⌊lβ⌋ = n, como α e β são
irracionais, n < kα < n + 1 e n < lβ < n + 1. Assim,
k l 1 1
+ < +
n+1 n+1 α β
k l
< +
n n
k+l k+l
Isso nos diz que <1< . Temos um absurdo pois a desigualdade anterior diz
n+1 n
que o inteiro k + l está entre dois inteiros consecutivos.

Mostremos agora que todo natural aparece nas sequências. Dado n ∈ N, existe k ∈ Z+ tal
que
k−1 1 k
< < .
n α n

8
A SOLUÇ
POT 2012 - Teoria dos Números - Nı́vel 23- Aula 14 -ÃO DO PROBLEMA
Samuel Feitosa INICIAL

k−1 1 k
Dividamos o intervalo [k/(n+1), k/n] em duas partes. Se < < , temos ⌊kα⌋ = n.
n α n
k−1 1 k
Se por outro lado, < < , temos:
n α n+1
k−1 1 k
<1− < ⇒
n β n+1
n+1−k 1 n+1−k
< < ⇒
n+1 β n
⌊(n + 1 − k)β⌋ = n

Em qualquer caso, n faz parte da sequência.

3 A solução do problema inicial


Voltemos ao exemplo inicial. Veja que toda linha ou coluna do tabuleiro deve possuir no
máximo uma posição perdedora. Se a k-ésima coluna não possuir nenhuma posição per-
dedora, para cada uma de suas casinhas, poderemos encontrar um posição perdedora na
mesma linha ou diagonal e isso implicaria na existência de uma infinidade de posições
perdedoras entre as k − 1 primeiras colunas. O mesmo argumeto se aplica para as linhas.
Consequentemente, cada natural aparece exatamente uma vez dentre os termos da sequência
x0 , x1 , . . . ; y0 , y1 , . . .. Além disso, também é fácil concluir que as sequências (xn ) e (yn ) são
crescentes. Induzidos do exemplo anterior, isso nos leva a conjecturar a existência de dois
irracionais α e β que possam de alguma forma gerar os termos das posições perdedoras.

A inserção dos pontos (xn , yn ) em um gráfico sugere que esses pontos estão próximos a
alguma reta. Isso poderia ser
√ traduzido dizendo que o quociente yn /xn é próximo a algum
1+ 5
valor. De fato, se α = , xn ≈ αn e yn ≈ (α + 1)n. Como as posições são números
2
inteiros, podemos conjecturar que:

1+ 5
Conjectura 20. Se α = , então (xn , yn ) = (⌊n · α⌋, ⌊n · (α + 1)⌋)
2
Veja que α é irracional e que:
1 1 2α + 1
+ =
α α+1 α2 + α
2α + 1
=
2α + 1
= 1.

Provemos a afirmação anterior por indução. Ela é facilmente verificável para os casos
iniciais apresentados na primeira tabela. Suponha sua validade para todos os inteiros no
conjunto {0, 1, 2, . . . , k}. Provemos que o mesmo também pe válido para k + 1. Seja t o
menor natural que não está no conjunto {x0 , x1 , . . . , xk , y0 , y1 , . . . , yk }. Como as sequências

9
A SOLUÇ
POT 2012 - Teoria dos Números - Nı́vel 23- Aula 14 -ÃO DO PROBLEMA
Samuel Feitosa INICIAL

xn e yn são crescentes e xn < yn , se xk+1 6= t, o inteiro t não aprecerá entre os termos


das sequências e isso contradiz nossa observação inicial. Em virtude da unicidade de rep-
sentação da sequência de Beatty, o inteiro ⌊α(k + 1)⌋ ainda não apareceu dentre os termos
das k + 1 primeiras posições perdedoras. Se t < ⌊α(k + 1)⌋, como xn é crescente, deve
existir j tal que ⌊(α + 1)j⌋ = t com j > k. Nesse caso,

t = ⌊(α + 1)j⌋
≥ ⌊(α + 1)(k + 1)⌋
= ⌊α(k + 1)⌋ + k + 1
> ⌊α(k + 1)⌋

Contrariando a suposição inicial sobre t. Logo, devemos ter xk+1 = ⌊α(k + 1)⌋. Seja
l = yk+1 − xk+1 . Se l < k + 1, O movimento diagonal

(xk+1 , yk+1 ) → (xl , yk+1 − xk+1 + xl ) = (xl , yl ),

passa uma posição perdedora para contradizendo o fato de que (xk+1 , yk+1 ) era uma posição
perdedora. Se l > k + 1, yk+1 > ⌊(α + 1)(k + 1)⌋ e o jogador naquela posição pode remover
pedras de apenas uma pilha obtendo:

(xk+1 , yk+1 ) → (⌊α(k + 1)⌋, ⌊(α + 1)(k + 1)⌋).

Usando novamente hipótese de indução e lembrando que yi − xi 6= k + 1 para todo i ∈


{0, 1, . . . , k}, qualquer movimento do próximo jogador, conduzirá a uma posição em que exa-
tamente uma das pilhas possui um número de pedras igual a um dos números x0 , y0 , . . . , xk , yk .
Assim, o oponente poderá passar uma posição perdedora. Novamente temos um absurdo
pois (xk+1 , yk+1 ). Logo, yk+1 − xk+1 = k + 1 e consequentemente yk+1 = ⌊(α + 1)(k + 1)⌋
concluindo a prova da conjectura para k + 1.

Problemas Propostos


Problema 21. Mostre que a parte fracionária do número 4n2 + n não é maior que 0, 25.
Problema 22. Sejam {ai }0≤i≤r , inteiros não negativos com n = a1 + a2 + · · · + ar . Mostre
n!
que é um inteiro.
a1 !a2 !...ar !
∞ 
n + 2i−1
X 
Problema 23. Prove que, para qualquer n natural, = n.
2i
i=1

 
2n
Problema 24. Seja p um divisor primo do número , com p ≥ 2n. Então o expoente
  n
2n
de p na fatoração em primos do do número é igual a 1.
n

10
A SOLUÇ
POT 2012 - Teoria dos Números - Nı́vel 23- Aula 14 -ÃO DO PROBLEMA
Samuel Feitosa INICIAL

Pn √ √
Problema 25. (Coréia 1997) Expresse k=1 ⌊ k⌋ em termos de n e ⌊ n⌋.
Problema 26. (Canadá 1998) Determine o número de soluções reais da equação
jak jak jak
+ + = a.
2 3 5
√ √ √
Problema 27. Encontre todos o reais α tais que a igualdade ⌊ n⌋+⌊ n + α⌋ = ⌊ 4n + 1⌋
é verdadeira para todos os naturais n.
Problema 28. Se a, b, c são reais e ⌊na⌋ + ⌊nb⌋ = ⌊nc⌋ para todo n natural, então a ∈ Z
ou b ∈ Z.
Problema 29. Sejam a, b, c e d números reais. Suponha que ⌊an⌋ + ⌊bn⌋ = ⌊cn⌋ + ⌊dn⌋
para todos os inteiros positivos n. Mostre que pelo menos um dentre a + b, a − c, a − d é
inteiro.
Problema 30. Seja n ≥ 3 um inteiro positivo. Mostre que é possı́vel eliminar no máximo
dois elementos do conjunto {1, 2, . . . , n} de modo que a soma dos números restantes seja
um quadrado perfeito.
m−1
X  ax + b 
Problema 31. Sejam a, m, b inteiros dados, com mdc(a, m) = 1. Calcule .
m
x=0

Problema 32. Encontre todos os naturais n tais que 2n−1 | n!.


Problema 33. Determine os pares (a, b) de reais tais que a⌊bn⌋ = b⌊an⌋ para todo inteiro
positivo n.
 k
p
Problema 34. Se p é primo , então ≡ 0 (mod p)( para 1 ≤ i ≤ pk − 1).
i
 √ √
Problema 35. Prove que ( 3 n + 3 n + 2)3 é divisı́vel por 8.

jnk jnk jnk
Problema 36. Prove que, t1 + t2 + . . . + tn = + + ... + , onde tn é o número
1 2 n
de divisores do natural n.
   
n n
Problema 37. Prove que, se p é um número primo, então a diferença − é divisı́vel
p p
por p.

11
Polos Olímpicos de Treinamento
Curso de Teoria dos Números - Nível 2 Aula 15
Prof. Samuel Feitosa

Equações de Pell

Uma Equação Famosa


Vejamos um problema que servirá de motivação ao nosso estudo.

Exemplo 1. Sejam Fn e Ln as seqüências de Fibonacci e Lucas, respectivamente, definidas


por
F1 = 1, F2 = 1 e Fn+1 = Fn + Fn−1 , n ≥ 2,
L1 = 1, L2 = 3 e Ln+1 = Ln + Ln−1 , n ≥ 2.
Mostre que a equação 5x2 − y 2 = 4 admite uma solução (x, y) em inteiros positivos se, e
somente se, (x, y) = (F2n−1 , L2n−1 ) para algum natural n.

Note que (1, 1) é a única solução com y ≤ 2. Podemos supor, portanto, que y ≥ 3. Sendo
α, β as duas raı́zes da equação x2 − x − 1 = 0, é conhecido que
αn − β n
Fn = e Ln = α n + β n .
α−β
É trivial verificar que os pares (F2n−1 , L2n−1 ) satisfazem nossa equação. A parte difı́cil é
mostrar que essas são as únicas soluções. Seja
S = {(F2n−1 , L2n−1 ), n ≥ 1}.
Por absurdo, suponha que exista uma solução (x, y) 6∈ S, e tome aquela que minimiza o
valor de x. Como x e y têm a mesma paridade, as frações (3x − y)/2 e (3y − 5x)/2 são
inteiros positivos, pois
x2 > −1 ⇒ 9x2 > 5x2 − 4 ⇒ 9x2 > y 2 ⇒ 3x > y
e
2
y >5 ⇒ 9y > 5y + 20 ⇒ 9y 2 > 25x2 ⇒ 3y > 5x.
2 2
POT 2012 - Teoria dos Números - Nı́vel 2 - Aula 15 - Samuel Feitosa

Afirmamos que ((3x − y)/2, (3y − 5x)/2) é uma solução da equação que não está em S. De
fato,
   
3x − y 2 3y − 5x 2 20x2 − 4y 2
5 − = = 4,
2 2 4
e, se ((3x − y)/2, (3y − 5x)/2) ∈ S, existiria n para o qual
3x − y 3y − 5x
= F2n−1 e = L2n−1
2 2
⇐⇒ x = F2n+1 e y = L2n+1 ,
o que contraria o fato de (x, y) não estar em S. Para terminar, note que (3x − y)/2 < x,
ou seja, obtemos uma solução cuja primeira coordenada é menor que x. Pelo método da
Descida de Fermat, concluı́mos que todas as soluções estão em S.
Exemplo 2. (Vietnã 1999) A seqüência an é definida por a1 = 1, a2 = 2 e an+2 =
3an+1 − an , n ≥ 1. A seqüência bn é definida por b1 = 1, b2 = 4 e bn+2 = 3bn+1 − bn ,
n ≥ 1. Mostre que os inteiros positivos (a, b) satisfazem 5a2 − b2 = 4 se, e somente se,
mdc(an , bn ) = mdc(a, b).

Veja que encontramos uma famı́lia infinita de soluções para o problema anterior. Curi-

osamente, essa famı́lia satisfaz uma recorrência linear bem simples. Note ainda que 5
apareceu na fórmula que encontramos para F2n+1 e L2n+1 . Será que tudo isso foi coin-
cidência? Nosso próximo objetivo será estudar mais detalhadamente equações como a do
problema anterior.

A equação x2 − dy 2 = N , onde d é natural e N é inteiro, é chamada equação de Pell. Jonh


Pell contribuiu muito pouco para a análise desta equação. Ela recebeu seu nome apenas
por um engano de Euler. Lagrange foi o primeiro a provar que, se d não é um quadrado
perfeito, então x2 − dy 2 = 1 tem infinitas soluções. Estamos interessados em descrever
todas as possı́veis soluções desta equação, caso existam, e tentar obter alguns critérios para
dizer quando ela não tem solução. Trataremos apenas do caso em que d não é um quadrado
perfeito. O outro caso é deixado como exercı́cio para o leitor.

Observação 3. (Para professores) Caso os alunos não estejam preparados para o forma-
lismo nas próximas demonstrações, o professor poderá ater-se apenas aos problemas. Bas-
taria o aluno entender como se caracterizam as soluções de uma equação de Pell. Após
alguma amadurecimento,ficaria mais fácil estudar as provas e entender porque as soluções
só podem ser daquela forma.
A próxima proposição é um conhecido exercı́cio de princı́pio da casa dos pombos.

Proposição 4. Se ξ é um número irracional, existem infinitos números racionais x/y, com


mdc(x, y) = 1, tais que
x 1
ξ− < 2·
y y

2
POT 2012 - Teoria dos Números - Nı́vel 2 - Aula 15 - Samuel Feitosa

Demonstração. Considere a partição


     
1 1 2 n−1
[0, 1) = 0 , ∪ , ∪ ··· ∪ , 1 .
n n n n
Pelo princı́pio da casa dos pombos, dois dentre os números 0, {ξ}, {2ξ}, . . ., {nξ} estão em
um mesmo intervalo da partição, digamos
 
i i+1
{kξ}, {jξ} ∈ , , 0 ≤ k < j ≤ n e 0 ≤ i < n.
n n
Então
1
|{jξ} − {kξ}| <
n
1
|(j − k)ξ − (⌊jξ⌋ − ⌊kξ⌋)| <
n
x′ 1
ξ− < ,
y′ ny ′
onde x′ = ⌊jξ⌋ − ⌊kξ⌋ e y ′ = j − k. Seja d = mdc(x′ , y ′ ), com x′ = dx e y ′ = dy,
mdc(x, y) = 1. Temos
x x′ 1 1 1
ξ− = ξ− ′ < < ′2 < 2 ·
y y ny ′
y y
Para obter infinitas soluções, tome m > 1/ |ξ − x/y| 6= 0. Com o mesmo argumento acima,
particionando agora o intervalo [0, 1) em m intervalos de mesmo tamanho, obtemos um par
de inteiros (x1 , y1 ) primos entre si, 0 < y1 < m, tais que
x1 1 x
ξ− < < ξ−
y1 my1 y
e portanto x1 /y1 é uma aproximação racional de ξ melhor do que x/y.
Dizemos que um inteiro n é livre de quadrados se não existe nenhum natural k > 1 tal que
k 2 |n.

Proposição 5. Seja d um inteiro positivo livre de quadrados. Existe uma constante M tal
que a desigualdade
x2 − dy 2 < M
tem infinitas soluções inteiras positivas (x, y).


Demonstração. É claro que, se d é livre de quadrados, então d é irracional. Pela pro-
posição anterior,
√ existem infinitos pares de inteiros (x, y), com y > 0 e mdc(x, y) = 1, tais
que |x − y d| < 1/y. Aplicando a desigualdade triangular, temos
√ √ √ 1 √
x + y d ≤ x − y d + 2y d < + 2y d.
y

3
POT 2012 - Teoria dos Números - Nı́vel 2 - Aula 15 - Samuel Feitosa

Multiplicando as duas desigualdades encontradas, obtemos


 
2 2 1 1 √ √
x − dy < + 2y d ≤ 2 d + 1,
y y
o que conclui a demonstração. 

Se (x, y) é solução da equação de Pell, é claro que (±x, ±y) também é. Suporemos, a partir
daqui, que x, y > 0.

Teorema 6. Se d é um inteiro positivo livre de quadrados, então a equação x2 −dy 2 = 1 tem


infinitas soluções inteiras. Ademais, existe uma solução (x1 , y1 ), chamada
√ de solução√fun-
damental, tal que toda solução é da forma (xn , yn ), n ≥ 1, onde xn + yn d = (x1 + y1 d)n .

Demonstração. Vamos mostrar a existência de uma solução. Pela proposição anterior,


existem infinitos pares de inteiros (x, y) tais que x2 − dy 2 < M . Existe, portanto, um
inteiro m, 1 ≤ m ≤ M , tal que a equação x2 − dy 2 = m tem infinitas soluções. Módulo
m, cada solução está no conjunto finito
 
Zm × Zm = 0, 1, . . . , m − 1 × 0, 1, . . . , m − 1 ,

e daı́, com um argumento análogo ao anterior, existe um par (a, b) ∈ Zm × Zm tal que
uma quantidade infinita de soluções (x, y) satisfaz (x, y) = (a, b). Em particular, existem
soluções distintas (x1 , y1 ) e (x2 , y2 ) tais que (x1 , y 1 ) = (x2 , y 2 ). Temos

 √  √ 
x 1 + y1 d x 1 + y 1 d x 2 − y 2 d
√ =
x 2 + y2 d m

(x1 x2 − dy1 y2 ) + (x2 y1 − x1 y2 ) d
=
√ m
= u + v d,

onde u, v são inteiros, pois

x1 x2 − dy1 y2 ≡ x1 2 − dy1 2 ≡ 0 (mod m)


x 2 y 1 − x 1 y2 ≡ x 1 y 1 − x 1 y 1 ≡ 0 (mod m) .

Ademais,
 √   √ 
u2 − dv 2 = u+v d · u−v d
√ √
x 1 + y1 d x 1 − y1 d
= √ · √
x 2 + y2 d x 2 − y2 d
x1 2 − dy1 2
=
x2 2 − dy2 2
= 1,

4
POT 2012 - Teoria dos Números - Nı́vel 2 - Aula 15 - Samuel Feitosa

mostrando a existência de solução. Vejamos agora como são as soluções.


√ √
Diremos que uma solução (x1 , y1 ) é maior que uma solução (x2 , y2 ) se x1 +y1 d > x2 +y2 d.
Seja (x1 , y1 ) a menor solução, com x1 , y1 > 0. Chamaremos esta solução √ de fundamental.

Dada outra solução (x, y), vamos mostrar que existe n para o qual x + y d = (x1 + y1 d)n .
De fato, se esse não é o caso, para algum n valem as seguintes desigualdades
 √ n √  √ n+1
x 1 + y1 d < x + y d < x 1 + y1 d ,

e daı́  √  √ −n √
1 < x + y d x 1 + y1 d < x 1 + y1 d
 √  √ n √
1 < x + y d x 1 − y1 d < x 1 + y1 d
√ √
1 < A+B d < x1 + y1 d,
onde (A, B) é uma solução formada por inteiros positivos (prove!). Isso√contraria a minimi-

lidade do par (x1 , y1 ). Assim, toda solução (x, y) deve satisfazer x + y d = (x1 + y1 d)n ,
para algum n ≥ 1, e uma fácil verificação mostra que esses pares são de fato soluções da
equação. 

Exemplo 7. Encontre todos os triângulos cujos lados são inteiros consecutivos e cuja área
é inteira.

Sejam a = n − 1, b = n, c = n + 1 os lados do triângulo. Pela fórmula de Herão, a área do


triângulo é
1p np 2
A= (a + b + c)(b + c − a)(a + c − b)(a + b − c) = 3(n − 4).
4 4
A é inteiro se e somente se n é par e 3(n2 − 4) é um quadrado perfeito. Substituindo n
por 2x e A por ny, obtemos a equação 3x2 − 3 = y 2 . Então y é divisı́vel por 3, digamos
y = 3z, e daı́ a equação anterior equivale à equação de Pell x2 − 3z 2 = 1. A solução
fundamental dessa última é (x1 , z1 ) = (2, 1), donde todas as outras soluções são geradas
pelas recorrências xn+1 = 2xn + 3zn e zn+1 = xn + 2zn , n ≥ 1. Os triângulos procurados
são os que têm lados de medidas 2xn − 1, 2xn e 2xn + 1, cuja área é 3xn zn .

Exemplo 8. Encontre o menor inteiro positivo n para o qual 19n + 1 e 95n + 1 sejam ambos
quadrados perfeitos.

Sejam 19n + 1 = x2 e 95n + 1 = y 2 . Então 5x2 − y 2 = 4, que é a equação analisada no


problema 45. Suas soluções são os pares (F2m−1 , L2m−1 ), m ≥ 1. Para que n seja mı́nimo,

5
POT 2012 - Teoria dos Números - Nı́vel 2 - Aula 15 - Samuel Feitosa

basta calcular o menor número da seqüência de Fibonacci múltiplo de 19, que é F17 = 1597.
Assim,
F17 2 − 1
n= = 134232.
19
Exemplo 9. Dado n > 1, mostre que existe um conjunto S de n pontos no plano tal que:

1. Não existem três pontos de S colineares;

2. A distância entre quaisquer dois pontos de S é um inteiro.

Como a equação de Pell x2 − 2y 2 = −1 admite a solução (1, 1), ela admite infinitas. Sejam
então a1 , a2 , . . . , an inteiros satisfazendo as seguintes condições:

(i) ai 2 + 1 = 2bi 2 , i = 1, 2, . . . , n;

(ii) a1 < a2 < · · · < an .

Tome S = {P1 , P2 , . . . , Pn }, onde P1 , P2 , . . . , Pn são pontos sobre a circunferência unitária


definidos por  
2ai ai 2 − 1
Pi = , , i = 1, 2, . . . , n.
ai 2 + 1 ai 2 + 1
Um cálculo simples mostra que a distância entre quaisquer dois pontos de S é racional.
Multiplicando as coordenadas desses pontos pelo mı́nimo múltiplo comum dos denomina-
dores de todas as frações das distâncias entre dois pontos, obtemos um conjunto de n
pontos sobre uma mesma circunferência, em particular não existindo três colineares, tais
que a distância entre quaisquer dois deles é inteira.
Problema 10. (IMO 1975) Existem 1975 pontos sobre uma circunferência de raio 1 de
modo que a distância entre quaisquer dois desses pontos é racional?

Problema 11. Seja n ≥ 3 um inteiro. Mostre que existe um conjunto S de n pontos no


plano tal que a distância entre quaisquer dois pontos de S é irracional e a área de qualquer
triângulo com vértices em S é racional.

Problema 12. Prove que a equação x2 − dy 2 = −1 não tem solução se d é divisı́vel por um
primo da forma 4k + 3.

Problema 13. Sejam d, k inteiros positivos tais que d não é um quadrado perfeito. Mostre
que existem infinitos pares de inteiros positivos (x, y) tais que k|y e x2 − dy 2 = 1.

6
POT 2012 - Teoria dos Números - Nı́vel 2 - Aula 15 - Samuel Feitosa

Outros Resultados
Para o leitor familiarizado
√ com frações contı́nuas, basta sabermos
√ as n-ésimas convergências
da expansão de d para determinarmos as soluções de x2 − dy 2 = 1, como diz a proposição
abaixo:

Proposição 14. Todas as soluções de x2 − dy 2 = 1 podem ser encontradas em xn = hn
hn
, yn = kn , onde são as n-ésimas convergências da expansão em frações contı́nuas de
√ kn √
d. Se r é o perı́odo da expansão em frações contı́nuas de d temos:

i) Se r √ é par então x2 − dy 2 = −1 não tem solução e todas soluções positivas de
x2 − dy 2 = 1 são dadas por x = hnr−1 , y = knr−1 para n = 1, 2, 3, . . ..

2 − dy 2 = −1
ii) Se r ı́mpar então x = hnr−1 , y = knr−1 produzem todas √ as soluções de x
quando n = 1, 3, 5, . . .., e todas as soluções de x2 − dy 2 = 1 quando n = 2, 4, 6 . . ..

Proposição 15. (Equação de Pell generalizada)Consideremos ax2 − by 2 = c, onde a e b não


são simultaneamente iguais a 1 e eles também não são divisı́veis por nenhum quadrado,
então as soluções são obtidas da seguinte maneira: Se c = 1 , e ambos a e b são diferentes
de 1, determine primeiro a solução fundamental se existir.
√ Todas as outras
√ soluções são
√ √
obtidas da solução fundamental (x0 , y0 ) por xn a + yn b = (x0 a + y0 b)2n+1 . Se c 6= 1
, primeiro determine a solução fundamental da equação ax2 − by 2 = 1. Se a ou b é igual
a 1, então esta equação tem no máximo um solução fundamental (x0 , y0 ). Se (x′0 , √ y0′ ) é
2 2 √
uma solução√fundamental de√ax − by = c todas as outras são obtidas de xn a + yn b =
√ √
(x0 a + y0√ b)n (x′0 a + y0′ √b). Se nem a ou√b for igual a 1, as soluções são geradas por
√ √ √
xn a + yn b = (x0 a + y0 b)2n (x′0 a + y0′ b).
Corolário 16. Suponha que N é um inteiro não nulo e que d seja livre de quadradados. Se
x2 − dy 2 = N tem uma solução, então tem infinitas.
hn
Proposição 17. Seja d um inteiro que não é um quadrado perfeito, sejam as n-ésimas
√ kn
convergências da expansão em fração contı́nua de d. Seja√N um inteiro tal que |N | < d.
Então qualquer solução positiva de x = s, y = t de x2 − dy 2 = N com mdc(s, t) = 1
satisfaz s = hn , t = kn para algum n.

Problemas Propostos

Problema 18. Mostre que existem infinitos inteiros n para os quais n2 + (n + 1)2 é um
quadrado perfeito.

Problema 19. Dado um inteiro positivo k, mostre que não existem inteiros (x, y) tais que
x2 − (k 2 − 1)y 2 = −1.

7
POT 2012 - Teoria dos Números - Nı́vel 2 - Aula 15 - Samuel Feitosa

Problema 20. Mostre que, se d ≡ 1 (mod 4), então a equação x2 − dy 2 = −1 tem solução.

Problema 21. (Banco IMO 2002) Existe um inteiro positivo m para o qual a equação
1 1 1 1 m
+ + + =
a b c abc a+b+c
tem infinitas soluções inteiras positivas a, b, c?

Problema 22. Determine todos os pares (k, n) de inteiros positivos tais que

1 + 2 + · · · + k = (k + 1) + (k + 2) + · · · + n.

Problema 23. Encontre todos os números da forma m(m+1)/3 que são quadrados perfeitos.

Problema 24. Encontre todos os números triangulares que são quadrados perfeitos.

Problema 25. Resolva a equação (x + 1)3 − x3 = y 2 em inteiros positivos.

Problema 26. Encontre todos os inteiros positivos n para os quais 2n + 1 e 3n + 1 sejam


ambos quadrados perfeitos e mostre que todos esses inteiros são divisı́veis por 40.

Problema 27. Seja n um inteiro positivo tal que 3n + 1 e 4n + 1 são ambos quadrados
perfeitos. Mostre que n é divisı́vel por 56.

Problema 28. Prove que existem infinitos inteiros positivos n para os quais n2 +1 divide n!.

Problema 29. Prove que a equação

x2 + y 2 + z 2 + 2xyz = 1

admite infinitas soluções inteiras positivas (x, y, z).

Problema 30. Encontre todos os inteiros positivos n para os quais existem inteiros positivos
distintos a1 , a2 , . . . , an tais que
1 2 n a1 + a2 + · · · + an
+ + ··· + = ·
a1 a2 an n
Problema 31. (Vietnã 1992) Encontre todos os pares de inteiros positivos (x, y) satisfa-
zendo a equação
x2 + y 2 − 5xy + 5 = 0.

8
REFERÊNCIAS
POT 2012 - Teoria dos Números - Nı́vel 2 - Aula 15 - Samuel Feitosa

Problema 32. Encontre todos os naturais n para os quais n + 1 e 3n + 1 são ambos qua-
drados perfeitos.

Problema 33. Encontre todos os pares de naturais (m, n) satisfazendo a igualdade

1 + 2 + 3 + · · · + n = m2 .

Problema 34. (Banco IMO 1967) √ Qual fração p/q, onde p, q são inteiros positivos menores
que 100, é a mais próxima de 2? Encontre todos os dı́gitos após a vı́rgula da representação

decimal dessa fração que coincidem com os dı́gitos da representação decimal de 2.

Problema 35. (Banco IMO 2003) Seja b > 5 um inteiro. Para cada natural n, seja xn a
representação na base b do número

|11 {z
· · · 1} 22 · · · 2} 5.
| {z
n−1 n

Prove que a seguinte condição é verdadeira se e somente se b = 10:

“Existe um inteiro positivo n0 tal que, para cada n > n0 , o número xn é um quadrado
perfeito.”

Problema 36. (Torneio das Cidades 1997) Prove que a equação

x2 + y 2 − z 2 = 1997

tem infinitas soluções inteiras (x, y, z).

Problema 37. (Irlanda 1995) Determine todos os inteiros a para os quais a equação

x2 + axy + y 2 = 1

tem infinitas soluções inteiras positivas (x, y) tais que x 6= y.

Referências
[1] F. E. Brochero Martinez, C. G. Moreira, N. C. Saldanha, E. Tengan - Teoria dos
Números ? um passeio com primos e outros números familiares pelo mundo inteiro,
Projeto Euclides, IMPA, 2010.

[2] E. Carneiro, O. Campos and F. Paiva, Olimpı́adas Cearenses de Matemática 1981-2005


(Nı́veis Júnior e Senior), Ed. Realce, 2005.

[3] S. B. Feitosa, B. Holanda, Y. Lima and C. T. Magalhães, Treinamento Cone Sul 2008.
Fortaleza, Ed. Realce, 2010.

9
REFERÊNCIAS
POT 2012 - Teoria dos Números - Nı́vel 2 - Aula 15 - Samuel Feitosa

[4] D. Fomin, A. Kirichenko, Leningrad Mathematical Olympiads 1987-1991, MathPro


Press, Westford, MA, 1994.

[5] D. Fomin, S. Genkin and I. Itenberg, Mathematical Circles, Mathematical Words, Vol.
7, American Mathematical Society, Boston, MA, 1966.

[6] I. Niven, H. S. Zuckerman, and H. L. Montgomery, An Introduction to the Theory of


Numbers.

10
Polos Olímpicos de Treinamento
Curso de Teoria dos Números - Nível 2 Aula 16
Prof. Samuel Feitosa

Ordem

Definição 1. O menor inteiro positivo k para o qual ak ≡ 1 (mod m), onde mdc(a, m) = 1,
é chamado ”ordem de a módulo m”e será denotado por ordm a.
Teorema 2. Se a é um inteiro relativamente primo com m, então an ≡ 1 (mod m) se, e
somente se, ordm a|n. Ademais, an0 ≡ an1 (mod m) se , e somente se, n0 ≡ n1 (mod ordm a)
Demonstração. Sejam b = ordm a e n = qb + r com 0 ≤ r < b. Como ab ≡ 1 (mod m),

an ≡ 1 (mod m) ⇔ aqb+r ≡ 1 (mod m)


r
⇔ a ≡1 (mod m)

Como 0 ≤ r < b, devemos ter r = 0. Usando que mdc(a, m) = 1 e supondo que n0 > n1 ,

an0 ≡ an1 (mod m) ⇔ an0 −n1 ≡ 1 (mod m)


⇔ n0 − n1 ≡ 0 (mod b)

Teorema 3. Se mdc(a, m) = 1, ordm a|φ(m)

Demonstração. Pelo teorema de Euler, aφ(m) ≡ 1 (mod m). O resultado segue do teorema
anterior.
Problema 4. (Putnam 1972) Prove que não existe inteiro positivo n > 1 tal que n|2n − 1.
Suponha, por absurdo, que existe um inteiro positivo n > 1 com essa propriedade e que k
é o menor dentre eles. Se d = ordk 2, então d | k. Como 2d ≡ 1 (mod k), temos 2d ≡ 1
(mod d). Em virtude da minimalidade de k, temos d = 1 ou d = k. No primeiro caso,
terı́amos k = 1 produzindo uma contradição. No segundo caso, em decorrência do teorema
anterior, k | φ(k). Entretanto, se k > 1, φ(k) ≤ k − 1 e obtemos assim um absurdo.
Problema 5. (Leningrado 1990) Prove que para todos os inteiros a > 1 e n , n|φ(an − 1).
Se k = ordan −1 a, como an ≡ 1 (mod an − 1), temos k | n e consequentemente k ≤ n. Não
podemos ter k < n porque an − 1 | ak − 1 ⇒ an − 1 ≤ ak − 1. Assim, k = n e usando o
teorema anterior podemos concluir que k | φ(an − 1).
Problema 6. Mostre que
a) ord3n 2 = 2 · 3n−1

b) Se 2m ≡ −1 (mod 3n ), então ⇒ 3n−1 | m.


k
Provaremos por indução que 23 + 1 = 3k+1 mk com 3 ∤ mk . Suponha que a afirmação vale
para k. Provemos para k + 1:

k+1
23 = (3k+1 mk − 1)3
= 33k+3 m3k − 32k+3 m2k + 3k+2 mk − 1
= 3k+2 (32k+1 m3k − 3k+1 + mk ) − 1
= 3k+2 mk+1 − 1

Claramente 3 ∤ mk+1 . Voltemos ao problema. Seja b = ord3n 2, então b | φ(3n ) = 2 · 3n−1 .


n−1
Temos duas possibilidades: ou b = 2 · 3j ou b = 3j . Como 23 ≡ −1 (mod 3n ) e 3j | 3n−1
j j
se j ≤ n − 1, devemos ter b = 2 · 3j . Assim, (23 − 1)(23 + 1) ≡ 1 (mod 3)n . Usando que
j j
23 − 1 ≡ 1 (mod 3), temos 23 ≡ −1 (mod 3n ). Novamente pelo lema provado no inı́cio,
3j ≥ 3n−1 e assim b = 2 · 3n−1 . Para o item b), de 2m ≡ −1 (mod 3n ), podemos concluir
que 22m ≡ 1 (mod 3)n . Daı́, 2 · 3n−1 | 2m e o resultado segue.
Problema 7. (Bulgária 1997) Encontre todos os números inteiros m, n ≥ 2 tais que
n n
1 + m3 + m2·3
n
é um inteiro
Claramente n é ı́mpar, mdc(m, n) = 1 e n > 2. Se n = 3, como mdc(m, n) = 1 devemos
n n
ter que m ≡ 1 (mod 3) pois caso contrário 1 + m3 + m2·3 ≡ 1 − 1 + 1 ≡ 1 (mod 3).
É fácil ver que todo par (m, n) = (3k + 1, 3) é solução. Suponha agora n > 3 e seja
n+1
3 n 3n 2·3n m3 −1
k = ordn m. Se n > 3 ⇒ m 6≡ 1 (mod n). Como 1 + m + m = segue
m3n − 1
n+1
que n | m3 − 1 ⇒ k | 3n+1 . Logo, k = 3n+1 . Pelo teorema de Euler, mφ(n) ≡ 1 (mod n)
então k ≤ φ(n) e 3n+1 ≤ φ(n) ≤ n − 1, uma contradição.
Problema 8. Prove que que se p é primo, então pp − 1 tem um fator primo congruente a
1 módulo p
pp − 1
Seja q um primo que divide . Como q | pp − 1 segue que ordq p | p. Se ordq p = 1
p−1
então q | pp − 1 e 0 ≡ pp−1 + pp−2 + . . . p + 1 ≡ 1 + 1 + . . . + 1 + 1 ≡ p (mod q). Mas isso é
um absurdo pois p 6= q. Logo ordq p = p e obtemos p | φ(q) = q − 1. Daı́, todos os divisores
pp − 1
primos de são congruentes a 1 módulo p.
p−1

Problemas Propostos

Problema 9. Se orda m = h, ordm b = k e mdc(h, k) = 1 mostre que ordm ab = hk.

2
Problema 10. Prove que se a, b são números naturais tais que a > b , n > 1, então cada
divisor primo do número an − bn é ou da forma nk + 1, onde k é um inteiro, ou um divisor
de um número an1 − bn1 , onde n1 |n e n1 < n.
Problema 11. Prove que se a, b são números naturais tais que a > b , n > 1, então cada
divisor primo do número an + bn é ou da forma 2nk + 1, onde k é um inteiro, ou um
divisor de um número an1 + bn1 , onde n1 é o quociente obtido por dividir o número n por
um número ı́mpar mairo que 1.
Problema 12. Seja p um primo que não divide 10, e seja n um inteiro, 0 < n < p. Seja d
a ordem de 10 módulo p.

1. Mostre que o comprimento do perı́odo da representação decimal de n/p é d.

2. Prove que que se d é par, então o perı́odo da representação decimal de n/p pode ser
dividido em duas partes cuja soma é 10d/2 − 1. Por exmeplo, 1/7 = 0, 142857, então
d = 6, e 142 + 857 = 999 = 103 − 1.
h
3. Se ordm a = h ⇒ ordm ak =
mdc(h, k)
Problema 13. Se p é um primo maior que 3, então qualquer divisor maior que 1 do número
2p + 1
é da forma 2kp + 1, onde k é um número natural.
3
Teorema 14. Se p é um primo maior que 2, então qualquer número natural que divida o
número 2p − 1 é da forma 2kp + 1, onde k é um inteiro.
Problema 15. (Bulgária 1995) Encontre todos os primos p e q tais que o número 2p + 2q
seja divisı́vel por pq.
Problema 16. Mostre que se k > 1 então 2k−1 6≡ −1 (mod k)
n
Problema 17. Mostre que se 3 ≤ d ≤ 2n−1 , então d ∤ (a2 +1) para qualquer inteiro positivo
a.
Problema 18. (Eureka) Prove que se p é um primo da forma 4k + 3, então 2p + 1 também
é primo se e somente se 2p + 1 divide 2p − 1.
n
Problema 19. Prove que todos os divisores dos números de Fermat 22 + 1, n > 1, são da
forma 2n+2 k + 1.
Problema 20. (IMO 1990) Encontre todos os inteiros positivos n > 1 tais que

2n + 1
n2
é um inteiro.
Problema 21. (Teste Cone Sul 2002) Encontre o perı́odo na representação decimal de
1
.
32002

3
Problema 22. (Teste de Seleção do Irã para a IMO) Seja a um natural fixo. Mostre que o
n
conjunto dos divisores primos de 22 + a, para n ∈ N, é infinito.
Problema 23. (Colômbia 2009) Encontre todas as triplas de inteiros positivos (a, b, n) que
satisfazem a equação:
ab = 1 + b + . . . + b n .
Problema 24. (IMO 2003) Seja p um número primo. Demonstre que existe um número
primo q tal que, para todo inteiro n, o número np − p não é divisı́vel por q.

4
Polos Olímpicos de Treinamento
Curso de Teoria dos Números - Nível 2 Aula 17
Prof. Samuel Feitosa

Aula de Revisão e Aprofundamento

Exemplo 1. O mı́nimo multiplo comum dos inteiros a, b, c, d e d é igual à a + b + c + d.


Prove que abcd é divisı́vel por 3 ou por 5.

Solução: Suponha inicialmente que mdc(a, b, c, d) = 1 e seja L = a + b + c + d. Como L é


o mı́nimo múltiplo comum, existem x, y, z, w tais que aw = bx = cy = dz = L. É fácil ver
que L também é o mı́nimo múltiplo comum de x, y, z, w e que

1 1 1 1 a b c d
+ + + = + + +
x y z w L L L L
= 1.

Suponha sem perda de generalidade que w ≤ x ≤ y ≤ z. Da equação anterior, o maior


valor de w é 4 e ocorrendo igualdade deverı́amos ter a = b = c = d = 1 que não satisfaz
o enunciado. Para w = 3, o leitor poderá facilmente verificar que a = b = 4, c = 3
e d = 1 é a única solução. Para w = 2, temos as seguintes soluções: (a, b, c, d) =
(5, 2, 2, 1), (10, 5, 4, 1), (6, 4, 1, 1) , (9, 6, 2, 1), (12, 8, 3, 1), (21, 14, 6, 1). Em todos os casos, L
é divisı́vel por 3 ou 5.

Exemplo 2. Seja A = {a1 < a2 < a3 < · · · } uma seqüência crescente de inteiros positivos
em que o número de fatores primos de cada termo, contando fatores repetidos, nunca é
maior que 2007. Prove que é sempre possı́vel extrair do conjunto A um subconjunto infinito

B = {b1 < b2 < b3 < · · · }

tal que o máximo divisor comum entre bi e bj é sempre o mesmo para quaisquer naturais
i 6= j.

O número de primos usados como fatores dos ai não pode ser finito, pois se essa quantidade
é n, terı́amos apenas n2007 possı́veis elementos ai . Podem ocorrer duas situações:

a) Nenhum primo divide infinitos ai . Seja b1 = a1 . Como cada fator primo de a1


aparece um número finito de vezes como fator dos ai , existirá um termo al > a1 tal que a1
e al são primos entre si. Seja b2 = al . Esse argumento pode ser repetido para gerar um
subconjunto infinito B = {b1 < b2 < b3 < · · · } de modo que mdc(bi , bj ) = 1, quaisquer que
sejam os naturais i, j.

b) Existe um primo p que divide infinitos ai . Como o expoente de p1 = p em cada


ai que é múltiplo de p1 é um elemento do conjunto {1, . . . , 2007}, pelo menos um deles,
digamos r1 > 0, deverá ocorrer infinitas vezes. Seja A1 o conjunto de todos os termos
ai para os quais o expoente de p1 em ai é r1 . Se nenhum primo p2 6= p1 divide infinitos
elementos de A1 , o caso anterior mostra que A1 possui um subconjunto tal que o máximo
divisor comum de quaisquer dois elementos é p1 r1 . Senão, seja p2 6= p1 um primo que divide
infinitos elementos de A1 . O expoente de p2 em cada elemento de A1 que é múltiplo de p2
pertence ao conjunto {1, . . . , 2007 − r1 }. Sejam r2 > 0 um expoente que ocorre infinitas
vezes e A2 o conjunto dos elementos de A1 para os quais p1 e p2 têm expoentes r1 e r2 ,
respectivamente. Esse processo deve terminar em i passos, i ≤ 2007, e nessa situação Ai
é um subconjunto de A para o qual todo elemento é um múltiplo de P = pr11 pr22 · · · pri i ,
digamos
Ai = {P c1 < P c2 < P c3 < · · · },
onde C={c1 < c2 < c3 < · · · } é um conjunto em que cada termo é o produto de não mais
que 2007 − (r1 + r2 + · · · + ri ) fatores primos, nenhum dos quais ocorrendo infinitas vezes.
Pelo caso a), C possui um subconjunto B1 tal que quaisquer dois elementos são primos
entre si. O conjunto
B = P · B1 = {P x | x ∈ B1 }
satisfaz as condições do problema, pois o máximo divisor comum entre quaisquer dois de
seus elementos é igual a P .

Exemplo 3. (Seletiva Rioplatense 2001) Encontre todos os pares (m, n) de números naturais
com m < n tais que m2 + 1 é um múltiplo de n e n2 + 1 é um múltiplo de m.

Afirmamos que todas as soluções são da forma (F2k−1 , F2k+1 ), k ≥ 0 (Fn é o n-ésimo
termo da sequência de Fibonacci). É fácil ver que F2k−1 F2k+3 = F2k+1 2 + 1 e portanto
os pares anteriores são soluções. Seja P o conjunto das soluções que não são da forma
(F2k−1 , F2k+1 ). O conjunto P contém um par (a, b) tal que a + b é mı́nimo. Suponhamos
a < b(se a = b ⇒ (a, b) = (1, 1) = (F−1 , F1 ) 6∈ P ). Como b | a2 + 1, a2 + 1 = bb′
e b′ < a. É fácil ver que a | b′ 2 + 1 e b′ | a2 + 1. Logo (b′ , a) é uma solução com
b′ + a < a + b Entretanto, (b′ , a) 6∈ P e daı́ (b′ , a) = (F2k−1 , F2k+1 ). COnsequentemente,
F2k−1 b = b′ b = a2 + 1 = F2k+1
2 ⇒ b = F2k+3 ⇒ (a, b) = (F2k+1 , F2k+3 ) 6∈ P . Logo P deve
ser vazio.

Exemplo 4. (URSS 1988) A sequência de inteiros an é dada por a0 = 0, an = P (an−1 ),


onde P (x) é um polinômio cujos coeficientes são inteiros positivos. Mostre que para quais-
quer inteiros positivos m, k com máximo divisor comum d, o máximo divisor comum de am
e ak é ad .

Quando temos um polinômio com coeficientes inteiros é sempre bom lembrar que a − b |
P (a) − P (b). Essa será nossa principal ferramenta nesta solução.

2
1. am | amr . Provaremos por indução. Se am(r−1) ≡ 0 (mod am ) ⇒ am(r−1)+1 ≡ P (0)
(mod am ) ⇒ am(r−1)+2 ≡ P (P (0)) (mod am ) ⇒ amr = am(r−1)+m ≡ P (P (. . . (P (0)) =
| {z }
m vezes
am ≡ 0 (mod am ).
2. Se l | at e l | af ⇒ l | at−f (Supondo t > f ). (Deixaremos a prova dessa afirmação
para o leitor).
Pelo teorema de Bezout, existem inteiros positivos x, y tais que mx − ky = d. Seja n =
mdc(am , ak ). Como n | am | amx e n | ak | aky , pelo item 2, n | amx−ky = ad . Mas ad | am
e ad | ak , então ad | n. Portanto ad = n.
Exemplo 5. Prove que existem infinitos números compostos n para os quais
n | 3n−1 − 2n−1 .
t
Lema: 2t | 32 − 1 ∀t ∈ N
Vamos provar o lema por indução. Para t = 1 é trivial. Suponha que a afirmação
seja válida para t = r, provemos que também é válida para t = r + 1. Fatorando
t+1 t+1 t t t t
22 − 33 = (22 − 33 )(22 + 33 ), como o primeiro parêntesis é múltiplo de 2t e o
segundo é múltiplo de 2, o produto deles é múltiplo de 2t+1 .
t t t t
Seja n = 32 − 22 com t > 1. Como n − 1 ≡ (32 − 1) − 22 ≡ 0 (mod 2t ), pelo lema,
t t
obtemos n = 32 − 22 | 3n−1 − 2n−1 . Aqui estamos usando o fato que xk − y k | xmk − y mk .
Exemplo 6. Consideramos todas as sequências (xn )n≥1 de inteiros positivos satisfazendo
xn+2 = mdc(xn , xn+1 ) + 2008, ∀ n ≥ 1.
Alguma dessas seqüências contém exatamente 102008 números distintos?
A idéia é construir a sequência de trás para frente. Mostraremos que para qualquer inteiro
positivo k > 1, existe uma tal sequência contendo exatamente k números distintos. Basta
encontrarmos uma sequência que satisfaça:
xn+2 = mdc(xn , xn+1 ) + 2, ∀ n ≥ 1. (1)
pois a multiplicação de todos os termos por 1004 produz a sequência do problema. De-
(an−1 − 2)(an−2 − 2)
finamos a1 = 4, a2 = 6, a3 = 8, a4 = 2(a3 − 2) e an = para n ≥ 5.
2
É fácil ver por indução que todos os ai serão pares e que a sequência é crescente. Con-
sequentemente, todos os ai com i ≥ 1 são inteiros e distintos. Além disso, definamos os
termos com ı́ndices não positivos por a3k = 4, a3k−1 = 6, a3k−2 = 4 para k ≤ 0.
. . . a6 , a5 , a4 , a3 = 8, a2 = 6, a1 = 4, a0 = 4, a−1 = 6, a−2 = 4, a−3 = 4, a−4 = 6, . . .
Afirmamos que a sequência anterior satisfaz mdc(an+2 , an+1 )+2 = an ∀n ∈ Z. É imediato
verificar isso para n ≤ 1 Para n = 2, mdc(a4 , a3 ) + 2 = mdc(12, 8) + 2 = 6 = a2 . Para
n > 2,
(an+1 − 2)(an − 2) an − 2
mdc(an+2 , an+1 )+2 = mdc( , an+1 )+2 = mdc((an+1 ) −an +2, an+1 )+2
2 2

3
Portanto,
an−1 − 2
mdc(an+2 , an+1 ) + 2 = mdc(an − 2, (an − 2) ) + 2 = an − 2 + 2 = an .
2
Para encontrarmos uma sequência satisfazendo (1) com exatamente k > 1 termos distintos
basta escolhermos x1 = ak e x2 = ak−1 e definirmos o resto da sequência usando a relação
de recorrência xn+2 = mdc(xn , xn+1 ) + 2

Problemas Propostos

Problema 7. Resolva em inteiros a equação:


xy xz yz
+ + =3
z y x
Exemplo 8. Alguns inteiros positivos estão escritos no quadro. Podemos apagar quaisquer
dois inteiros distintos e substituı́-los pelo máximo divisor comum e o mı́nimo divisor comum
dos dois números. Prove que eventualmente a operação de alterar os números não será mais
executada.

Exemplo 9. Mostre que se k > 1 então 2k−1 6≡ −1 (mod k)

Exemplo 10. (Estônia 2005) Sejam a, b inteiros positivos primos entre si tais que (a +
b)/(a − b) é um inteiro positivo. Prove que ao menos um dentre os números ab + 1 e 4ab + 1
é um quadrado perfeito.

Exemplo 11. (Ibero 1999) Seja n um inteiro maior que 10 tal que cada um de seus dı́gitos
pertence ao conjunto S = {1, 3, 7, 9}. Prove que n tem algum divisor primo maior ou igual
a 11.

4
Polos Olímpicos de Treinamento
Curso de Teoria dos Números - Nível 2 Aula 18
Prof. Samuel Feitosa

Resı́duos Quadráticos

Definição 1. Para todos a tais que mdc(a, m) = 1, a é chamado resı́duo quadrático módulo
m se a congruência x2 ≡ a (mod m) tem solução. Se ela não tem solução, então a é
chamado de resı́duo não quadrático módulo m.

Exemplo 2. Seja n um inteiro. Prove que se 2 + 2 28n2 + 1 é um inteiro, então é um
quadrado perfeito.
√ √
Se 2+2 28n2 + 1 é inteiro, o número 28n2 + 1 é um racional e consequentemente devemos
ter que 28n2 + 1 é o quadrado de um inteiro ı́mpar, digamos:

28n2 + 1 = (2k + 1)2 ⇒


28n2 + 1 = 4k 2 + 4k + 1 ⇒
7n2 = k(k + 1)

Devemos considerar dois casos: 7 | k ou 7 | k+1. Além disso, lembremo-nos do seguinte fato:

Se mdc(a, b) = 1, e a · b = n2 então existem inteiros x e y tais que a = x2 e y = b2 .

Como mdc(k, k + 1) = 1, temos os dois casos para analisar:


Primeiro caso:
= x2

k
(k + 1)/7 = y 2
Assim, 1 = (k + 1) − k = 7y 2 − x2 . Analisando essa equação módulo 7, temos x2 ≡ −1
(mod 7). Entretanto, analisando os quadrados dos restos da divisão por 7, podemos notar
que −1 não é um resı́duo quadrático e consequentemente temos um absurdo.

Segundo caso:

k/7 = x2


k + 1 = y2
POT 2012 - Teoria dos Números - Nı́vel 2 - Aula 18 - Samuel Feitosa


Daı́, 2 + 2 28n2 + 1 = 2 + 2(2k + 1) = 4(k + 1) = (2y)2 e isso conclui o problema.

Em geral, se p é um primo da forma 4k + 3, −1 nunca é resı́duo quadrático. Para ver isso,


suponha que existe x tal que:

x2 ≡ −1 (mod p) ⇒
(x2 )(p−1)/2 ≡ (−1)(p−1)/2 (mod p) ⇒
p−1
x ≡ −1 (mod p).

Isso contradiz o teorema de Fermat.


 
a
Definição 3. Se p denota um primo ı́mpar, então o sı́mbolo de Legendre é definido
p
por 1 se a é um resı́duo quadrático, −1 se a não é um resı́duo quadrático módulo p, e 0 se
p|a.
Teorema 4. Se p é um primo ı́mpar. Então
 
a p−1
a) ≡ a 2 (mod p)
p
    
a b ab
b) =
p p p
   
a b
c) a ≡ b (mod p) implica que =
p p
 2  2   
a a b b
d) Se mdc(a, p) = 1 então =1e =
p p p
   
1 −1 p−1
e) = 1, = (−1) 2
p p
Provemos inicialmente o item a) quando mdc(a, p) = 1. Em virtude do teorema de Fermat,
perceba que se mdc(a, p) = 1 então:

p | ap−1 − 1 = (ap−1/2 + 1)(ap−1/2 − 1).


 
a
Daı́, ap−1/2 ≡ ±1 (mod p). Suponha que = 1, então existe x tal que
p

x2 ≡ a (mod p) ⇒
p−1 p−1
(x2 ) 2 ≡ a 2 ⇒
p−1
xp−1 ≡ a 2

p−1
Pelo teorema
 de Fermat, a última congruência nos diz que a 2 ≡ 1 (mod p). Suponha
a
agora que = −1, ou seja, que não existe x tal que x2 ≡ a (mod p). Assim, podemos
p

2
POT 2012 - Teoria dos Números - Nı́vel 2 - Aula 18 - Samuel Feitosa

separar os números do conjunto {1, 2, . . . , p − 1} em pares (i, j) onde ij ≡ a (mod p) e


i 6= j. Daı́, o produto de todos esses pares é

1 · 2 · 3 · . . . · (p − 1) ≡ a · a · . . . · a (mod p)
p−1
≡ a 2 (mod p)
 
p−1 a
Usando o teorema de Wilson, concluı́mos que a 2 ≡ −1 (mod p). Se p | a, ≡0≡
p
p−1
a 2 (mod p). Os demais itens seguem de a).
Exemplo 5. Suponha que p é um primo ı́mpar. Seja n o menor não-resı́duo quadrático

positivo módulo p. Prove que n < 1 + p.
Seja m o maior inteiro positivo tal que mn > p, ou seja, (m − 1)n < p < mn. Assim,
0 < mn − p < n e consequentemente:
 
mn − p
1 =
p
 
mn
=
p
   
m m
= ·
p p
 
m
= −
p
Daı́, m ≥ n e
(n − 1)2 < n(n − 1)
≤ n(m − 1)
< p.

Portanto, n − 1 < p.
Teorema 6. (Lema de Thue) Sejam m um número natural e a um inteiro primo com m,
então existem inteiros x e y tais que:

1. 0 < |x|, |y| < m;
2. ax ≡ y (mod m).

Demonstração. Considere o conjunto {au − v|u, v ∈ Z, 0 ≤ u, v ≤ ⌊ p⌋}. Como existem

⌈ p⌉2 > p tais pares (u, v), existirão (u1 , v1 ) 6= (u2 , v2 ) tais que
au1 − v1 ≡ au2 − v2 (mod p)
Sejam x = v1 − v2 e y = u1 − u2 . Claramente ii) está satisfeito. Por construção, x, y não
podem ser ambos nulos e, caso um deles seja, o outro também o será. Logo i) também é
verdade.

3
POT 2012 - Teoria dos Números - Nı́vel 2 - Aula 18 - Samuel Feitosa

Proposição 7. Sejam D ∈ Z e m ∈ N inteiros relativamente primos tais que −D é um


resı́duo quadrático módulo m. Então existem inteiros k, x, y ∈ Z com 0 < k ≤ D e

0 < |x|, |y| < p tais que:

x2 + Dy 2 = kp

Demonstração. Seja a tal que a2 ≡ −D (mod p) e x, y como no teorema anterior com


m = p. Então, por um lado:

0 < x2 + Dy 2 < (1 + D)p

e por outro lado,

x2 + Dy 2 ≡ (a2 + D)y 2 ≡ 0 (mod p)


 
−3
Exemplo 8. Seja p > 3 um primo ı́mpar tal que = 1, existem x e y tais que
p
x2 + 3y 2 = p.

Pelo teorema anteiror, Exitem x, y, k tais que x2 + 3y 2 = pk com |x|, |y| ≤ p. Assim,
x2 + 3y 2 < 4p. Temos tres casos a considerar:
Primeiro caso: x2 + 3y 2 = 3p. Devemos ter x2 ≡ 0 (mod 3) e x = 3x0 . Daı́, 3x20 + y 2 = p.

Segundo caso: x2 + 3y 2 = 2p. Como 2p é par, devemos ter x e y ambos ı́mpares ou ambos
pares. Em qualquer caso, x2 + 3y 2 será múltiplo de 4 e consequentemente 2 | p. Isso é um
absurdo.

Terceiro caso: x2 + 3y 2 = p. Não há o que fazer nesse caso.


Teorema 9. (Lema de Gauss ) Seja p um primo ı́mpar e a um inteiro tal que mdc(a, p) = 1,
a(p − 1)
Considere os inteiros a, 2a, . . . , e seus restos módulo p. Se n denota o número
2 
a
desses restos que excedem p2 então = (−1)n
p
Demonstração. Sejam r1 , r2 , . . . rn os resı́duos que excedem p/2 e sejam s1 , s2 , . . . , sk os
resı́duos restantes. Naturalmente todos esses restos são distintos e nenhum deles é nulo.
Considere agora os números da forma p − ri e perceba que 0 < p − ri < p/2. Se tivéssemos
p − ri ≡ sj (mod p) para algum par (i, j), também terı́amos ri + sj ≡ 0 (mod p) e por con-
a(p − 1)
seguinte p dividiria a soma de dois números do conjunto {a, 2a, . . . , }. Entretanto,
2
isso é um absurdo porque a soma de quaisquer dois número desse conjunto é da forma ak
com 0 < k < p e a não é divisı́vel por p. Logo, os números da forma p − rj são todos
distintos dos números da forma si e todos eles pertencem ao conjunto {1, 2, . . . (p − 1)/2}.

4
POT 2012 - Teoria dos Números - Nı́vel 2 - Aula 18 - Samuel Feitosa

Como n + k = (p − 1)/2, podemos concluir que:


p−1
(p − r1 )(p − r2 ) . . . (p − rn )s1 s2 . . . sk = 1 · 2 · . . . · ⇒
2
p−1
(−r1 )(−r2 ) . . . (−rn )s1 s2 . . . sk ≡ 1 · 2 · ... · (mod p) ⇒
2
p−1
(−1)n r1 r2 . . . rn s1 s2 . . . sk ≡ 1 · 2 · ... · (mod p) ⇒
2
p−1 p−1
(−1)n a · 2a · . . . · a ≡ 1 · 2... (mod p) ⇒
2 2
(−1)n a(p−1)/2 ≡ 1 (mod p) ⇒
(−1)n ≡ a(p−1)/2 (mod p).

Pelo critério de Euler, o resultado segue.


 
a
Teorema 10. Se p é um primo ı́mpar e mdc(a, 2p) = 1, então = (−1)t onde t =
p
p−1
2    
X ja 2 p2 −1
e = (−1) 8 .
p p
j=1

a(p − 1)
Demonstração. Consideraremos novamente o conjunto {a, 2a, . . . , } e usaremos a
2
mesma notação do teorema anterior. Quando o inteiro ja é dividido por p, obtemos como
quociente o número ⌊ja/p⌋. Assim, podemos escrever:

(p−1)/2 (p−1)/2 n k
X X X X
ja = p⌊ja/p⌋ + rj + sj
j=1 j=1 j=1 j=1

e
(p−1)/2 n n k
X X X X
j = (p − ri ) + sj + sj
j=1 i=1 j=1 j=1
n
X k
X
= np − rj + sj
j=1 j=1

Substituindo na equação anterior, obtemos:


 
(p−1)/2 (p−1)/2 n
X X X
(a − 1) j = p  ⌊ja/p⌋ − n + 2
 rj
j=1 j=1 j=1

Como
(p−1)/2
X p2 − 1
j= ,
8
j=1

5
POT 2012 - Teoria dos Números - Nı́vel 2 - Aula 18 - Samuel Feitosa

temos:
(p−1)/2
p2 − 1 X
(a − 1) ≡ ⌊ja/p⌋ − n (mod 2)
8
j=1

(p−1)/2
X
Se a é ı́mpar, n ≡ ⌊ja/p⌋ (mod 2). Se a = 2, isto implica que n ≡ (p2 −1)/8 (mod 2)
j=1
pois ⌊2j/p⌋ = 0 para 1 ≤ j ≤ (p − 1)/2. O resultado decorre do teorema anterior.
2n − 1
Exemplo 11. Encontre todos os inteiros positivos n tais que 2n −1 é divisı́vel por 3 e
3
tem um múltiplo da forma 4m2 + 1 para algum natural m.

Teorema 12. (Lei da reciprocidade quadrática) Se p e q são primos ı́mpares distintos, então
  
p q p−1 q−1
= (−1) 2 · 2
q p

Demonstração. Seja S o conjunto de todos os pares de inteiros (x, y) satisfazendo 1 ≤ x ≤


(p − 1)/2, 1 ≤ y ≤ (q − 1)/2. O conjunto S possui (p − 1)(q − 1)/4. Suponha que exista
um par (x, y) tal que qx = py. Como mdc(p, q) = 1, segue que q | y e p | x. Entretanto,
nos internvalos mencionados não existem tais múltilplos. Separemos então esse conjunto
em dois outros mutuamente exclusivos:

S1 = {(x, y)|qx > py}


S2 = {(x, y)|qx < py}

b b
S1 b b b b

b b b b b
S2 b

b b b b b b

b b b b b b

(p−1)/2 (q−1)/2
X X
Os números de pares em S1 e S2 são ⌊qx/p⌋ e ⌊py/q⌋. Fazendo a contagem
x=1 y=1
total de pares, temos:
(p−1)/2 (q−1)/2
X X p−1 q−1
⌊qx/p⌋ + ⌊py/q⌋ = ·
2 2
x=1 y=1

e, em virtude do teorema anterior, obtemos:


  
p q p−1 q−1
= (−1) 2 · 2
q p

6
POT 2012 - Teoria dos Números - Nı́vel 2 - Aula 18 - Samuel Feitosa

x2 − 2
Exemplo 13. Mostre que nunca é um inteiro quando x e y são inteiros.
2y 2 + 3
Exemplo 14. Seja q = 4n + 1 onde n é um inteiro positivo. Prove que q é um primo se, e
q−1
somente se, 3 2 ≡ −1 (mod q)
Se q é um primo, então q ≡ 2 (mod 3) e pela lei da reciprocidade quadrática temos:

1 = (−1)(q−1)/2·1
  
3 q
=
q 3
 
3
= (−1)
q

Em virtude dessa equação e do critério de Euler, temos:


 
3
−1 =
q
q−1
≡ 3 2 (mod q)
q−1
Reciprocamente, se 3 2 ≡ −1 (mod q), então ordq 3 = 4n . Como ordq 3 | ϕ(q), teremos
ϕ(q) = q − 1, ou seja, q é primo.

Problemas Propostos

Problema 15. Prove que se p é um primo maior que 3 então a soma dos resı́duos quadráticos
módulo p é divisı́vel por p.
Problema 16. Mostre que se a é um resı́duo quadrático módulo m, e ab ≡ 1 (mod m),
então b é também um resı́duo quadrático. Prove que o produto dos resı́duos quadráticos
módulo p é congruente a +1 ou −1 módulo p.
Problema 17. Prove que se p é um primo da forma 4k + 3, e se m é o número de resı́duos
quadráticos menores que p2 , então:
1 · 3 · 5 · . . . · (p − 2) ≡ (−1)m+k+1 (mod p)
2 · 4 · 6 · . . . · (p − 1) ≡ (−1)m+k (mod p)
Problema 18. Seja q = 4n + 1 onde n é um inteiro positivo. Prove que q é um primo se,
q−1
e somente se, 3 2 ≡ −1 (mod q)
Problema 19. Os inteiros positivos a e b são tais que os números 15a + 16b e 16a − 15b
são ambos quadrados de inteiros positivos. Qual é o menor valor possı́vel que pode ter o
menor desses números?

7
POT 2012 - Teoria dos Números - Nı́vel 2 - Aula 18 - Samuel Feitosa

Problema 20. (Olimpı́ada Búlgara) Sejam m e n números naturais tais que

(m + 3)n + 1
A=
3m
é um inteiro. Prove que A é ı́mpar.
Problema 21.
2p
− 2 é divisı́vel por p2 .

a) Prove que para qualquer primo p, o número p

b) Mostre que se p é um primo e 0 ≤ m < n < p então


 
np + m
≡ (−1)m+n+1 p (mod p2 )
mp + n

2p−1
− 1 é divisı́vel por p3 .

c) Prove que para qualquer primo p > 3, o número p−1

Problema 22. Caracterize todos os inteiros que podem ser expressos na forma:
a) a2 + ab + b2

b) a2 + 2b2
Problema 23. Se n é um inteiro tal que 7n é da forma a2 + 3b2 , prove que n também é
dessa forma.
Problema 24. Encontre todos os inteiros positivos n para os quais existe um inteiro m tal
que m2 + 9 é um múltiplo de 2n − 1.
Problema 25. Mostre que dado qualquer primo p, existem inteiros x, y, z, w satisfazendo
x2 + y 2 + z 2 − wp = 0 e 0 < w < p
Problema 26. Mostre que p é um divisor de ambos os números da forma m2 + 1, n2 + 2,
se e somente se é um divisor de algum número da forma k 4 + 1.
Problema 27. Seja A o conjunto de todos os inteiros da forma a2 + 2b2 , onde a e b são
inteiros e b 6= 0. Mostre que p é um número primo e p2 ∈ A, então p ∈ A.
Problema 28. Seja p um primo da forma 4k + 1. Mostre que:
p−1 
2k 2 k2
  
X p−1
−2 = .
p p 2
k=1

Problema 29. Mostre que se x não é divisı́vel por 3, então 4x2 + 3 tem pelo menos um
fator primo da forma 12n + 7. Mostre que existem infinitos primos dessa forma.
Problema 30. Suponha que φ(5m − 1) = 5n − 1 com m, n números naturais. Prove que
mdc(m, n) > 1
Problema 31. (Coréia 2001) Seja f : Z → Z. Dado um primo ı́mpar p, encontre todas as
funções f : Z → Z satisfazendo as seguintes condições:

8
REFERÊNCIAS
POT 2012 - Teoria dos Números - Nı́vel 2 - Aula 18 - Samuel Feitosa

1. Se m ≡ n (mod p) com m, n ∈ Z, então f (m) = f (n)

2. f (mn) = f (m)f (n) para quaisquer m, n ∈ Z.

Problema 32. Para a congruência z 2 ≡ D (mod 2a ), onde D é ı́mpar e a é um natural, ser


solúvel, é necesário e suficiente que D seja da forma 2k + 1, 4k + 1 ou 8k + 1 de pendendo
de a = 1, a = 2 ou a > 2.
Problema 33. (OBM 2007) Para quantos numeros inteiros c, −2007 ≤ c ≤ 2007 , existe
um inteiro x tal que x2 + c e múltiplo de 22007 ?
Problema 34. (Teorema de Wolstenhome) Se p ≥ 5 é um primo, mostre que o numerador
da fração
1 1 1
+ + ... +
1 2 p−1
é múltiplo de p2 .


2p
Problema 35. Se p é um primo maior que 3 e q = , prove que
3
     
p p p
+ + ...
1 2 q

é divisı́vel por p2 .
(Dica: Use a identidade de Catalão e o teorema de Wolstenhome)

Referências
[1] E. Carneiro, O. Campos and F. Paiva, Olimpı́adas Cearenses de Matemática 1981-2005
(Nı́veis Júnior e Senior), Ed. Realce, 2005.

[2] S. B. Feitosa, B. Holanda, Y. Lima and C. T. Magalhães, Treinamento Cone Sul 2008.
Fortaleza, Ed. Realce, 2010.

[3] D. Fomin, A. Kirichenko, Leningrad Mathematical Olympiads 1987-1991, MathPro


Press, Westford, MA, 1994.

[4] D. Fomin, S. Genkin and I. Itenberg, Mathematical Circles, Mathematical Words, Vol.
7, American Mathematical Society, Boston, MA, 1966.

[5] I. Niven, H. S. Zuckerman, and H. L. Montgomery, An Introduction to the Theory of


Numbers.

9
Polos Olímpicos de Treinamento
Curso de Teoria dos Números - Nível 2 Aula 19
Prof. Samuel Feitosa

A Função Parte Inteira - II

Exemplo 1. Considere um tabuleiro T , de dimensões m × n, onde m e n são inteiros posi-


tivos. Prove que uma diagonal de T passa por exatamente m + n − mdc(m, n) quadradinhos
1 × 1.

Suponhamos os quadradinhos de lado unitário. Vamos fazer primeiro o caso em que


mdc(m, n) = 1. Esse tabuleiro em Z × Z pode ser reperesentado por um retângulo de
vértices : O = (0, 0), A = (m, 0), B = (m, n), C = (0, n). Queremos porvar que a diago-
nal OB passa por exatamente m + n − 1 quadradinhos. Quando esta diagonal corta um
quadradinho, um segmento de reta dela, fica totalmente contido no quadradinho. Basta
contarmos em quantos segmentos esses quadradinhos dividem OB.

Se um vértice, digamos (a, b), de algum dos quadradinhos do tabuleiro está em OB, usando
semelhança de triângulos podemos concluir que:
m
a = b⇒
n
an = bm

Como m|an e mdc(m, n) = 1, temos m | a resultando que a = 0 ou a ≥ m. No pri-


meiro caso (a, b) = O e no segundo, como a ≤ m pois a está no inteiror do retângulo,
temos (a, b) = B. Assim, OB não contém vértices de quadradinhos diferentes de O e B.
Consequentemente OB corta cada uma das retas x = 1, 2, ..., m − 1 e y = 1, 2, ..., n − 1
em pontos distintos determinando assim m + n − 2 pontos sobre OB. Juntando esses
m + n − 2 pontos marcados sobre a diagonal com O e B teremos m + n pontos e por
conseguinte OB corta m + n − 1 quadradinhos. Agora se mdc(m, n) = d podemos escrever
m = dm1 , n = dn1 com mdc(m1 , n1 ) = 1. Divida agora o tabuleiro em d sub-tabuleiros :
Ti = {Oi = ((i − 1)m1 , (i − 1)n1 ), Ai = (im1 , 0), Bi = (im1 , in1 ), Ci = (0, in1 )} para
1 ≤ i ≤ d. Basta usar o que fizemos para cada um desses sub-tabuleiros.
POT 2012 - Teoria dos Números - Nı́vel 2 - Aula 19 - Samuel Feitosa

Exemplo 2. Suponha que mdc(p, q) = 1. Então


q−1   p−1  
X ip X iq (p − 1)(q − 1)
= = .
q p 2
i=1 i=1

Prova: Considere o retângulo T = O = (0, 0), A = (q, 0), B = (q, p), C = (0, p). Claramente
existem (p − 1)(q − 1) pontos de Z × Z no interior do retângulo T . Pelo exemplo anterior,
não pode existir pontos de Z × Z na diagonal OB. Por simetria, existem (p−1)(q−1)
 2 pontos
ip
no interior do retângulo OAB. Dado 1 ≤ i ≤ (q − 1) existem exatamente pontos da
q
q−1  
X ip (p − 1)(q − 1)
forma (i, j) ∈ Z × Z no interior do triângulo OAB. Assim = .
q 2
i=1

Conjugados

Suponha que α seja um irracional e que estamos interessados em calcular o resto de ⌊αn ⌋
mod m. Nesse caso, tentaremos encontrar β tal que 0 < β < 1 , α + β e αβ ∈ Z. Para
entendermos o propósito disso, considere a equação: x2 − ax − b = 0 onde a = α + β e
b = αβ. Como α e β são raı́zes:

α2 = aα + b ⇒ αn+1 = aαn + bαn−1


β 2 = aβ + b ⇒ β n+1 = aβ n + bβ n−1

Se Kn = αn + β n , temos Kn+1 = aKn + bKn−1 . Como a e b são inteiros e K1 = α + β ∈ Z,


K2 = (α + β)2 − 2αβ ∈ Z, segue que Kn ∈ Z para todo natural n. Além disso,

Kn ∈ Z ⇒
{α } + ⌊α ⌋ + {β } + ⌊β n ⌋ ∈ Z
n n n

Consequentemente {αn }+{β n } ∈ Z. Como 0 < {αn }+{β n } < 2, devemos obrigatoriamente
ter {αn } + {β n } = 1. Usando que 0 < β < 1, também podemos concluir que ⌊β n ⌋ = 0
e por conseguinte Kn = ⌊αn ⌋ + 1. Conhecendo-se a recursão de Kn , podemos facilmente
determinar o perı́odo dos restos dos termos da sequência na divisão por m. Os próximos
exemplos servirão para ilustrar essa heurı́stica. Também podemos modificar um pouco a
idéia anterior para tratar do caso −1 < β < 0.
Exemplo 3. Prove que, para todo natural n temos:
$ √ !n %
7 + 37
3| .
2
√ √
7+ 37 7− 37
Prova: Sejam α = 2 eβ= 2 . Se Kn = αn + β n , então:

Kn+1 = 7Kn − 3Kn−1 .

2
POT 2012 - Teoria dos Números - Nı́vel 2 - Aula 19 - Samuel Feitosa

Como K1 = 7 ≡ 1 (mod 3) e K2 = 43 ≡ 1 (mod 3). Temos que todos os termos da


sequência Kn são inteiros e que todo deixam resto 1 na divisão por 3 pois Kn+1 ≡ 7Kn ≡ Kn
(mod 3). Portanto,
⌊αn ⌋ + 1 = Kn ≡ 1( mod 3) ∀n ∈ N.

Exemplo 4. Encontre a maior potência de 2 que divide ⌊(3 + 11))2n+1 ⌋.
√ √
Sejam α = 3 + 11, β = 3 − 11 e Kn = αn + β n . Então, Kn+1 = 6Kn + 2Kn−1 .

1 Lema: 2n+1 | K2n e 2n+1 k K2n+1

Provaremos o lema por indução. Suponha que 2k+1 | K2k e que 2k+1 k K2k+1 , ou seja,
K2k = 2k+1 a e K2k+1 = 2k+1 b com b ≡ 1 (mod 2). Então:

K2k+2 = 6 · 2k+1 b + 2 · 2k+1 a = 2k+2 (3b + a)


K2k+2 = 6 · 2k+2 (3b + a) + 2 · 2k+1 a = 2k+2 (18b + 7a)

e o resultado segue. Assim,

K2n+1 = ⌊α2n+1 ⌋ + ⌊β 2n+1 ⌋ + {β 2n+1 } + {α2n+1 }


= ⌊α2n+1 ⌋ + (−1) + 1
= ⌊α2n+1 ⌋

pois −1 < β < 0 e consequentemente ⌊β 2n+1 ⌋ = −1. Daı́, em virtude do lema, a maior
potência de 2 é 2n+1 .

Problemas Propostos

Problema 5. (Teste de Seleção do Brasil para√ a Cone Sul)Prove que para todo inteiro
positivo k , a parte interia do número (7 + 4 3)k é ı́mpar.
1 √
Problema 6. (Olimpı́ada Iraniana) Mostre que, k n − ⌊k n ⌋ = 1 − n
onde k = 2 + 3.
k

Problema 7. (Hungria 2000) Se A = (1000 + 10002 + 1)1000 , determine o 2000-ésimo
algarismo após a vı́rgula de sua representação decimal.
Problema 8. Prove que para todo inteiro m > 2 existe um irracional r que depende de m,
tal que ⌊rk ⌋ ≡ −1( mod m).
Problema 9. Considere a sequêcica de reais positivos a1 , a2 , ..., tal que a1 = 1 an = an+1 +
1
an+2 , para todo inteiro n > 0. Prove que o dı́gito das unidades de não pode ser 0,3,5
ai
ou 8 para todo i ∈ N.
1 n
a k b significa que an | b mas an+1 ∤ b

3
REFERÊNCIAS
POT 2012 - Teoria dos Números - Nı́vel 2 - Aula 19 - Samuel Feitosa

Problema 10. (Seletiva do Brasil para√a IMO-2001) Encontre todos os naturais n tais que
1+ 5
αn − n2 α é um inteiro onde α = .
2
Problema 11. (Revista Eureka)Seja α a maior raiz da equação x3 − 3x2 + 1 = 0. Prove
que ⌊α2004 ⌋ é divisı́vel por 17.

Problema 12. (Taiwan 1998) Mostre que, para inteiros positivos m e n , mdc(m, n) =
m−1
X  kn 
2 + m + n − mn.
m
k=0

Problema 13. (Balcânica 2003) Seja ABCD um tabuleiro m × n de quadrados unitários.


Assuma que mdc(m, n) = 1 e m, n são ı́mpares. Os pontos de interseção entre a diagonal
principal AC e os lados dos quadrados unitários são A1 , A2 , ..., Ak , nesta ordem
√ (k ≥ 2) e
m + n2
2
A1 = A, Ak = C. Prove que A1 A2 − A2 A3 + A3 A4 − ... + (−1)k Ak−1 Ak = .
mn
Problema 14. (Geórgia 1998) dado n > 5 , as retas x = n e y = n são desenhadas no
plano cartesiano. considere os pontos com coordenadas inteiras no interior (ou bordo) do
quadrado formado por essas retas e pelos eixos. Quantos desses pontos tem a soma das
coordenadas multiplo de 5?

Problema 15. Um jogador solitário recebe após cada jogada a ou b pontos (a e b são
inteiros positivos com a < b) e estes se acumulam jogada após jogada. Existem 35 valores
impossı́veis para a pontuação acumulada e um desses valores é 58. Encontre a e b.

Referências
[1] E. Carneiro, O. Campos and F. Paiva, Olimpı́adas Cearenses de Matemática 1981-2005
(Nı́veis Júnior e Senior), Ed. Realce, 2005.

[2] S. B. Feitosa, B. Holanda, Y. Lima and C. T. Magalhães, Treinamento Cone Sul 2008.
Fortaleza, Ed. Realce, 2010.

[3] D. Fomin, A. Kirichenko, Leningrad Mathematical Olympiads 1987-1991, MathPro


Press, Westford, MA, 1994.

[4] D. Fomin, S. Genkin and I. Itenberg, Mathematical Circles, Mathematical Words, Vol.
7, American Mathematical Society, Boston, MA, 1966.

[5] I. Niven, H. S. Zuckerman, and H. L. Montgomery, An Introduction to the Theory of


Numbers.

4
Teoria dos Números 01 - Divisibilidade

Problema 1. O dı́gito das unidades de um quadrado de um inteiro é 9 e o dı́gito das dezenas


desse quadrado é 0. Prove que o dı́gito das centenas é par.

Solução. Seja n esse inteiro. Então temos n2 = 100k + 9, onde k ∈ N. Note que o dı́gito das
centenas é par se, e somente se, k é par. Logo, devemos mostrar que k é par.
Suponha que n = 10A + B, onde A é um inteiro qualquer e B é um algarismo na base 10.
Dessa forma, 100k + 9 = 10A2 + 20AB + B 2 e , portanto, B 2 − 9 é múltiplo de 10. Como
0 ≤ B ≤ 9, os possı́veis valores de B são 3 e 7.
Se B = 3, então

100k + 9 = 100A2 + 60A + 9 ⇐⇒ 5k = 5A2 + 3A


⇐⇒ 5k = A(5A + 3).

Veja que, independentemente da paridade de A, o termo A(5A + 3) é sempre par e portanto


k é par.
Agora, se B = 7, então

100k + 9 = 100A2 + 140A + 49 ⇐⇒ 5k = 5A2 + 7A + 2


⇐⇒ 5k = A(5A + 7) + 2.

Analogamente, k é par.

Problema 2. Qual o menor natural que deixa resto 4 na divisão por 7 e resto 10 na divisão por
13?

Solução. Seja n um natural satisfazendo essas condições. Então n + 3 deve ser divisı́vel por 7
e por 10. O menor valor de n é mmc(10, 7) − 3 = 70 − 3 = 67.

Problema 3. Qual o menor multiplo de 5 que deixa resto 2 quando dividido por 3 e por 4?

Solução. Se x é um múltiplo de 5 que satisfaz estas condições então devemos ter x = 3m+2 =
4n+2, para m e n inteiros positivos. De forma que 3m = 4n. Concluı́mos que m = 4t e n = 3t,
para algum t. Com isso, x = 12t + 2. O menor múltiplo de 5 desta forma é 50 = 12 · 4 + 2.

3 +2n
Problema 4. Qual o resto de 2n na divisão por 7?

Solução. Note que n3 + 2n = n(n + 1)(n + 2) − 3n2 . Como n, n + 1 e n + 2 são consecutivos,


um deles é múltiplo de 3. Assim, n3 + 2n será sempre divisı́vel por 3. Sendo assim, escrevamos
n3 + 2n = 3m. Queremos saber o resto de 23m na divisão por 7. Observando que 23m = 8m e
que 8m − 1 = (8 − 1)(8m−1 + . . . + 8 + 1), concluı́mos que o resto procurado é 1.

42015 − 1
Problema 5. Qual o resto de na divisão por 3?
3
Solução. Primeiramente note que 4n deixa mesmo resto que 1n = 1 para todo n natural.
Além disso, sabemos que
xn − 1
= (xn−1 + xn−2 + . . . + x + 1).
x−1
42015 − 1
Portanto, deixa o mesmo resto que
3
42014 + 42013 + . . . + 4 + 1

que por sua vez deixa o mesmo resto que

1 + 1 + . . . + 1 + 1 = 2015

Portanto, o resto é o mesmo que o de 2015 na divisão por 3 e é igual a 2.

Problema 6. Qual é o último dı́gito do número 20152015 ?

Solução. O ultimo dı́gito é igual ao resto que o número deixa na divisão por 10. Portanto,
o último dı́gito de 20152015 é o mesmo que o de 52015 . Toda potência de 5 deixa resto 5 na
divisão por 10, logo o dı́gito que procuramos é 5.

Problema 7. Encontre o algarismo das unidades do número 1 × 3 × 5 × . . . × 97 × 99.

Solução. Queremos o resto na divisão por 10. O resto é igual ao do número

1 × 3 × 5 × 7 × 9 × 1 × . . . × 7 × 9 = (1 × 3 × 5 × 7 × 9)10 .

Temos 1 × 3 × 5 × 7 × 9 = 945 que deixa resto 5. Sendo assim, o resto que queremos é o
mesmo que 510 que é 5.

Problema 8. Quais são os possı́veis restos na divisão de n! + 13 na divisão por 5?

Solução. Vamos analisar os casos:

i. Se n = 1 então n! + 13 = 14 que deixa resto 4;

ii. Se n = 2 temos n! + 13 = 15 que deixa resto 0;

2
iii. Se n = 3, n! + 13 = 19 e o resto é 4;

iv. Para n = 4 temos n! + 13 = 37 com resto 2;

v. Para n ≥ 5, n! + 13 deixa mesmo resto que 0 + 13, isto é, 3.

Finalmente, os possı́veis resto são 0, 2, 3 e 4

Problema 9. Encontra o resto de 62009 na divisão por 37.

Solução. Note que 62 deixa resto −1 na divisão por 37. Logo, 64 deixa resto 1. Dessa forma,
62009 = (64 )502 · 6 deixa o mesmo resto que 1502 · 6 = 6.

Problema 10. Encontre todos os pares de inteiros a e b tais que 22 = ab + 3a − b.

Solução. Perceba que

22 = ab + 3a − b ⇐⇒ 19 = (a − 1)(b + 3).

Como 19 é primo, devemos ter a − 1 = 19 e b + 3 = 1 ou a − 1 = 1 e b + 3 = 19 ou


a − 1 = −1 e b + 3 = −19 ou a − 1 = −19 e b + 3 = −1. Isso nos dá as soluções
(a; b) = (20; −2), (2; 16), (0; −22), (−18; −4).
Teoria dos Números 03 - Algoritmo de Euclides

Problema 1. Calcule mdc(3k + 2, 5k + 3) onde k é um inteiro qualquer.

Solução. Se d = mdc(3k + 2, 5k + 3), então d divide 3k + 2 e divide 5k + 3. Logo d divide


5 · (3k + 2) − 3 · (5k + 3) = 1. Como d deve ser positivo, temos d = 1.

7n + 2a
Problema 2. Encontre todos os valores de a tais que as frações da forma
n+5
sejam todas irredutı́veis.
Solução. Devemos ter, para todo n inteiro,

1 = mdc(7n + 2a, n + 5)
= mdc(7n + 2a − 7(n + 5), n + 5)
= mdc(2a − 35, n + 5).

Suponha que 2a − 35 é diferente de 1 e de −1. Então tomando n = |2a − 35| − 5 teremos mdc
igual a |2a − 35| , isto é, diferente de 1.
Logo, para que mdc(7n + 2a, n + 5) = 1 devemos ter 2a − 35 = ±1. Dessa forma, a = 18 ou
a = 17.

Problema 3. Calcule mdc(3600 , 7)

Solução. Vamos primeiramente descobrir o resto de 3600 na divisão por 7. Veja que 33 = 27
deixa resto 6 por 7. Logo, 36 deixa mesmo resto que 62 = 36, isto é, deixa resto 1 por 7.
Dessa forma, 3600 = (36 )100 deixa mesmo resto que 1100 = 1 na divisão por 7. Concluı́mos que
mdc(3600 , 7) = mdc(1, 7) = 1.

Problema 4. Na sequência de Fibonacci 1, 1, 2, 3, 5, 8, 13, . . ., cada número depois do segundo


é soma dos dois anteriores. Denotando por fn o n-ésimo termo da sequência temos portanto
que
fn = fn−1 + fn−2
e que f1 = f2 = 1. Mostre que o máximo divisor comum de dois termos consecutivos da
sequência de Fibonacci é sempre 1.

Solução. Temos o seguinte:

mdc(fn , fn−1 ) = mdc(fn−1 + fn−2 , fn−1 )


= mdc(fn−1 + fn−2 − fn−1 , fn−1 )
= mdc(fn−2 , fn−1 )
Proseguindo desta maneira, encontramos que o mdc inicial é igual a mdc(fn−2 , fn−3 ),
mdc(fn−3 , fn−4 ), . . .. De forma que mdc(fn , fn−1 ) é constante para todo n natural. Con-
sequentemente, este valor é sempre igual a mdc(f1 , f2 ) = mdc(1, 1) = 1.

Problema 5. Para quais inteiros positivos k ocorre mdc(n! + k, 3n) = 3 para todo n > 3?

(n − 1)!
Solução. Se n > 3 então q = é inteiro. Sendo assim temos que mdc(n! + k, 3n) =
3
mdc(n! + k − 3nq, 3n) = mdc(k, 3n) = 3. Devemos ter, então, k = 3t, t ∈ Z. Logo, deve
ocorrer mdc(t, n) = 1 para todo n. Como k é natural, t = 1 e k = 3. k só pode assumir um
valor natural.

Problema 6. Qual o maior valor possı́vel para mdc(n, 2015 + n)?

Solução. Note que mdc(n, 2015 + n) = mdc(n, 2015 + n − n) = mdc(2015, n). Tomando
n = 2015 temos o mdc máximo e igual a 2015.

Problema 7. Qual é o maior valor possı́vel para mdc(n + 1, n3 + 5)?

Solução. Da fatoração n3 + 1 = (n + 1)(n2 − n + 1) temos que n3 + 5 = (n + 1)k + 4.


Portanto mdc(n + 1, n3 + 5) = mdc(n + 1, (n + 1)k + 4) = mdc(n + 1, 4) que tem máximo 4.

Problema 8. Quantos inteiros n, com 1 ≤ n ≤ 100, podem ser escritos na forma n =


462x + 966y?

Solução. Pelo algoritmo de Euclides encontramos mdc(966, 462) = 42. Pelo teorema de
Bezout, existem x0 e y 0 tais que 42 = 462x0 + 966y 0 . Portanto, se 42 divide n, digamos
n = 42k, então existem x = k · x0 e y = k · y 0 tais que n = 42k = 462x + 966y. Além disso,
se n pode ser escrito dessa forma então, claramente, é múltiplo de 42. Concluı́mos que n pode
ser escrito na forma 462x + 966y se, e só se, for múltiplo de 42. Existem 2 múltiplode 42 entre
1 e 100. Sendo assim, a resposta é 2.

Problema 9. Qual é o maior valor possı́vel para mdc(122 + n2 , 122 + (n + 1)2 )?

Solução. Temos que

mdc(122 + n2 , 122 + (n + 1)2 ) = mdc(122 + n2 , 2n + 1).

2
Como 2n + 1 é ı́mpar, multiplicar 122 + n2 por 2 não altera o mdc. Logo,

mdc(122 + n2 , 122 + (n + 1)2 ) = mdc(244 + 2n2 , 2n + 1)


= mdc(244 + 2n2 − n(2n + 1), 2n + 1)
= mdc(244 − n, 2n + 1).

Novamente, como 2n + 1 é ı́mpar, multiplicar 244 − n por 2 não afetará o mdc. Dai,

mdc(122 + n2 , 122 + (n + 1)2 ) = mdc(488 − 2n, 2n + 1)


= mdc(488 − 2n + (2n + 1), 2n + 1)
= mdc(489, 2n + 1).

Portanto, o maior valor para o mdc será 489, quando 2n + 1 for múltiplo de 489.

Problema 10. Qual é o menor valor positivo de N , tal que existem inteiros x e y satisfazendo

2013x + 3102y = N

.
Solução. Usando o algoritmo de Euclides:

mdc(3102, 2013) = mdc(1089, 2013)


= mdc(1089, 924)
= mdc(924, 165)
= mdc(165, 99)
= mdc(99, 66)
= mdc(66, 33) = 33.

Claramente 33 = mdc(3102, 2013) divide N . Pelo teorema de Bezout existem inteiros x e y


tais que 2013x + 3102y = 33. Nessas condições, o menor valor de N é 33.
Teoria dos Números 04 - MMC, MDC e os Números Primos

Problema 1. Um primo p pode ser expresso como a diferença de quadrados de dois inteiros
positivos. Encontre o resto da divisão de p2 + 138 por 4.

Solução. Temos p = a2 − b2 = (a − b)(a + b). Como p é primo devemos ter a − b = 1. Dai,


p = a + b = 2b + 1. Portanto, p2 + 138 = 4b2 + 4b + 1 + 138 = 4(b2 + b + 34) + 3 deixa resto
3 na divisão por 4.

Problema 2. Encontre todos os primos p tais que p = 3m − 3n , onde m, n são inteiros não
negativos.

Solução. Devemos ter n < m. Sendo assim, p = 3m − 3n = 3n (3m−n − 1). Como p é primo
e 3m−n − 1 6= 1, devemos ter 3m−n − 1 = p e 3n = 1. Concluı́mos que n = 0 e p = 3m − 1 é
par. Dessa forma, p é igual ao único primo par, 2.

Problema 3. Encontre todos os inteiros positivos n para os quais 3n − 4, 4n − 5 e 5n − 3 são


todos primos.

Solução. A soma dos três números é um número par, então plo menos um deles é par. O
único número primo par é 2. Só 3n − 4 e 5n − 3 podem ser pares. Resolvendo as equações
3n − 4 = 2 e 5n − 3 = 2 encontramos n = 2 e n = 1, respectivamente. É trivial conferir que
n = 2 torna todos os três números primos.

Problema 4. Se p e q são primos e x2 − px + q = 0 tem duas raizes inteiras positivas distintas,


encontre p e q.

Solução. Sejam x1 e x2 , com x1 < x2 , as duas raizes. Então x2 − px + q = (x − x1 )(x − x2 ),


implicando que p = x1 + x2 e q = x1 x2 . Já que q é primo, x1 = 1. Daı́, q = x2 e p = x2 + 1
são dois primos consecutivos; são eles, q = 2 e p = 3.

Problema 5. Prove que se p e p2 + 8 são primos, então p3 + 8p + 2 é primo.

Solução. Se p não é múltiplo de 3 então p2 deixa resto 1 na divisão por 3, de forma que p2 + 8
é múltiplo de 3 e não é primo. Logo, p deve ser múltiplo de 3. Como é primo, p = 3. Daı́,
p3 + 8p + 2 = 53 que é primo.

Problema 6. Encontre todos os possı́veis valores de n ≥ 1 para os quais existem n inteiros


positivos consecutivos tais que sua soma é um número primo.
Solução. Claramente, nós temos n = 1 tomando qualquer número primo. Nós também temos
n = 2 já que todo número ı́mpar pode ser escrito como soma de dois números consecutivos.
Suponha p = a + (a + 1) + . . . + (a + k) para algum primo p e inteiros positivos a e k ≥ 2.
Então 2p = (k + 1)(2a + k). Tanto k + 1 quanto 2a + k são maiores que 2. De forma que
(k + 1)(2a + k)
é composto. Isso é uma contradição já que p é um número primo. Sendo
2
assim, n = 1 ou 2.

Problema 7. Seja p um primo ı́mpar. Encontre os pares de inteiros positivos (x, y) tais que
x2 = p + y 2 .

Solução. Temos p = x2 − y 2 = (x − y)(x + y). Como x, y são inteiros positivos x − y < x + y.


p+1
Do fato de p ser primo temos x − y = 1 e x + y = p. Resolvendo encontramos x = e
2
p−1
y= .
2

Problema 8. Para quantos valores de n o número 1! + 2! + . . . + n! é primo?

Solução. Para n = 1 temos 1! = 1 que não é primo. Para n = 2 temos 1! + 2! = 3 que é


primo. Para n = 3 temos 1! + 2! + 3! = 9. Agora, para n ≥ 4 temos

1! + 2! + 3! + 4! + . . . + n! = 9 + 4! + . . . + n!.

Note que todos os termos são divisı́veis por 3, logo para n ≥ 4 não temos solução. Sendo
assim, o único inteiro n que satisfaz essa condição é n = 3.

Problema 9. Mostre que não existe nenhuma progressão aritmética infinita composta somente
de números primos.

Solução. Suponha que existe uma progressão deste tipo. Seja p1 o termo inicial e r a razão da
sequência. Então o termo geral da sequência é pn = p1 + (n − 1)r. Tomando n = p1 + 1 temos
pp1 +1 = p1 + [(p1 + 1) − 1]r = p1 (1 + r), um número composto. Chegamos a um absurdo.
Logo, não existe nenhuma P.A. infinita composta somente de primos.

Problema 10. Encontre todos os valores inteiros positivos de n para os quais n4 + 4 é um


número primo.

Solução. n4 + 4 = n4 + 4n2 + 4 − 4n2 = (n2 + 2n + 2)(n2 − 2n + 2). Para n = 1,


n4 + 4 = 5 · 1, um primo. Para n > 1, n2 + 4 é um número composto uma vez que ele tem
dois fatores n2 + 2n + 2 e n2 − 2n + 2, cada um maior do que 1.
Sendo assim, n4 + 4 é primo somente para n = 1.
Problemas Resolvidos

Nı́vel 2

Divisibilidade I

Material elaborado por Susana Frómeta Fernández


Problemas

Problemas
Problema 1. Encontre os inteiros que, na divisão por 7, deixam um quociente igual ao resto.

Problema 2. Determinar os números que divididos por 17 dão um resto igual ao quadrado do quo-
ciente correspondente.

Problema 3. (OCM 1994, adaptado) Seja A = 777 . . . 77 um número onde o dı́gito 7 aparece 1001
vezes. Determinar o resto da divisão de A por 1001.

Problema 4. Encontre um inteiro que deixa resto 4 na divisão por 5 e resto 7 na divisão por 13.

Problema 5. Encontre o menor inteiro positivo que, dividido por 29 deixa resto 5, e dividido por 31
dá resto 28.

Problema 6. Prove que, para todo inteiro positivo n o número n5 − 5n3 + 4n é divisı́vel por 120.

Problema 7. Prove que o número 199 + 299 + 399 + 499 + 599 é múltiplo de 5.

Problema 8. Mostre que o número 1n + 8n − 3n − 6n é múltiplo de 10 para todo natural n.

Problema 9. Encontre o resto da divisão de 3710 − 1 por 11.

Problemas que usam fatoração


Problema 10. Prove que 22225555 + 55552222 é divisı́vel por 7.

Problema 11. Encontre o último dı́gito de 19891989 .

Problema 12. Mostre que se n divide a, então 2n − 1 divide 2a − 1.

Problema 13. (Cone Sul 1996) Prove que o número

1995 · 19971996 − 1996 · 19971995 + 1


19962
é um inteiro.

Problema 14. Mostre que para n ı́mpar, n divide 1n + 2n + · · · (n − 1)n .

2
Soluções

1. Se um número inteiro n deixa resto igual ao quociente na divisão por 7, então ele pode ser escrito
como n = 7q + q, onde o quociente e o resto são representados por q. Como q é resto na divisão por
7 ele deve satisfazer 0 ≤ q < 7. Note que n = 8q e, considerando todos os possı́veis valores de q,
encontramos que os possı́veis valores de n são 0, 8, 16, 24, 32 e 48.

2. Os números inteiros n que estamos procurando devem satisfazer n = 17q + q 2 , onde 0 ≤ q 2 < 17 ou,
equivalentemente, −4 ≤ q ≤ 4. Note que n pode ser inteiro negativo. Considerando todos os possı́veis
valores de q, obtemos que os possı́veis valores de n são −52, −42, −30, −16, 0, 18, 38, 60 e 84.

3. Note que 106 deixa resto 1 na divisão por 1001, logo, para qualquer inteiro positivo k, temos que
106k também deixa resto 1 na divisão por 1001. O maior múltiplo de 6 que é menor que 1001 é 996.
Vamos escrever A como
10996 − 1
 
996
A = 7 × 11111 × 10 + .
9
Então o resto da divisão de A por 1001 será o mesmo resto da divisão de 7 × 11111 por 1001, que é
igual a 700.

4. Um número inteiro n que satisfaz as condições do problema pode ser escrito como n = 5q1 + 4 e
como n = 13q2 + 7, onde q1 e q2 são os respectivos quocientes da divisão de n por 5 e por 13. Somando
6 nos dois lados das duas igualdades, temos n + 6 = 5(q1 + 2) = 13(q2 + 1). Então n + 6 é divisı́vel
por 5 e por 13. Podemos escolher, por exemplo, n + 6 = 5 · 13, o que daria n = 59.

5. Um número inteiro n que satisfaz as condições do problema pode ser escrito como n = 29q1 + 5
e como n = 31q2 + 28. Então q1 e q2 satisfazem 29q1 = 31q2 + 23, o que que dizer que 29q1 deixa
resto 23 na divisão por 31. Testando, encontramos que o menor inteiro positivo q1 que satisfaz essa
condição é q1 = 4. Então n = 29 · 4 + 5 = 121.

6. Usaremos uma fatoração:


n5 − 5n3 + 4n = n((n4 − n2 ) − (4n2 − 4))
= n(n2 (n2 − 1) − 4(n2 − 1))
= n(n2 − 1)(n2 − 4)
= (n − 2)(n − 1)n(n + 1)(n + 2).
Temos então que n5 − 5n3 + 4n é o produto de 5 números inteiros consecutivos. Pelo menos um
deles será múltiplo de 5, também pelo menos um deles será múltiplo de 3. Veja também que entre 5
números consecutivos sempre haverá pelo menos dois números pares consecutivos, portanto, um deles
será múltiplo de 4. Isso garante que o produto dos 5 números será divisı́vel por 8. Então temos que
n5 − 5n3 + 4n é divisı́vel por 5 · 3 · 8 = 120.

7. Como 599 é divisı́vel por 5, descartamos este termo no nosso análise. Observe que 14 , 24 , 34 e 44
deixam resto 1 na divisão por 5. Vamos escrever 99 = 3 + 4 · 24. Então, para a = 1, 2, 3 e 4, temos
que a99 deixa o mesmo resto que a3 na divisão por 5. Com isso, temos que basta encontrar o resto na
divisão por 5 de 13 + 23 + 33 + 43 , que é o mesmo resto na divisão por 5 de 1 + 3 + 2 + 4, que é igual
a 0.

3
8. O número em questão é múltiplo de 2, pois é a substração dos números ı́mpares 1n + 8n e 3n + 6n .
Basta ver que ele também é múltiplo de 5. Veja que 8 deixa resto 3 e que 6 deixa resto 1 na divisão por
5. Então, na divisão por 5, o número que estamos analisando deixa o mesmo resto que 1n +3n −3n −1n ,
que é igual a 0.

9. Veja que 37 deixa resto 4 na divisão por 11. Basta encontrar o resto de 410 − 1 na divisão por 11.
Observe que 45 = 1024 deixa resto 1, logo 410 = 45·2 também. Isso mostra que 410 − 1 é divisı́vel por
11.

10. Na divisão por 7, 2222 deixa resto 3 e 5555 deixa resto 4. Nos resta mostrar que 35555 + 42222
é divisı́vel por 7. Veja também que 35 deixa resto 5 e que 42 deixa resto 2 na divisão por 7. Basta
mostrar então que 51111 + 21111 é divisı́vel por 7.
Usaremos a seguinte fatoração: se n é ı́mpar, então

xn + y n = (x + y) xn−1 − xn−2 y + xn−3 y 2 − . . . + x2 y n−3 − xy n−2 + y n−1 .



(1)

Fatorando 51111 + 21111 dessa mesma forma, vemos que aparece o fator 5 + 2 = 7, o que conclui a
nossa prova.

994
11. Veja que 19891989 = 1989 · 19892 . O número 19892 termina em 1, logo o último dı́gito de
994
19892 também é 1. Concluı́mos que o último dı́gito de 19891989 será 9.


12. Usaremos a seguinte fatoração:

xn − y n = (x − y) xn−1 + xn−2 y + xn−3 y 2 + . . . + x2 y n−3 + xy n−2 + y n−1 .



(2)

Como n divide a, escrevemos a = nk para um certo k inteiro. Logo

2a − 1 = (2n )k − 1
 
= (2n − 1) 2n(k−1) + 2n(k−2) + 2n(k−3) + . . . + 22n + 2n + 1 ,

o que conclui a prova.

13. Escrevemos a = 1996 e a expressão do exercı́cio fica:


(a − 1)(a + 1)a − a(a + 1)a−1 + 1
.
a2
Faremos algumas manipulações algébricas no numerador:

(a − 1)(a + 1)a − a(a + 1)a−1 + 1 = (a2 − 1)(a + 1)a−1 − a(a + 1)a−1 + 1


= a2 (a + 1)a−1 − (a + 1)a−1 − a(a + 1)a−1 + 1
= a2 (a + 1)a−1 − ((a + 1)a − 1) .

Basta mostrar que (a + 1)a − 1 é divisı́vel por a2 . Usaremos a fatoração (2):

(a + 1)a − 1 = (a + 1 − 1)((a + 1)a−1 + (a + 1)a−2 + (a + 1)a−3 + · · · + (a + 1)2 + (a + 1) + 1).

O primeiro fator no lado direito da igualdade acima é exatamente a, e o segundo fator é a soma de a
termos que deixam resto 1 na divisão por a, logo será também divisı́vel por a. Portanto, o produto
será divisı́vel por a2 , como querı́amos mostrar.

4
14. A expressão 1n + 2n + · · · (n − 1)n é a soma de n − 1 termos. Como n − 1 é par (pois n é ı́mpar),
podemos agrupar em pares os termos da soma que estamos estudando. Faremos isso da seguinte forma:

1n + 2n + · · · (n − 1)n = [1n + (n − 1)n ] + [2n + (n − 2)n ] + · · · + [k n + (n − k)n ],


n−1 n−1
onde k = . Acabamos de escrever a expressão acima como soma de termos da forma
2 2
[an + (n − a)n ], onde a = 1, 2, . . . n−1
2 .
Vamos mostrar que cada um dos termos da forma [an + (n − a)n ] é divisı́vel por n. Para isso, como
n é ı́mpar, podemos usar a fatoração (1):

an +(n−a)n = (a+n−a)(an−1 −an−2 (n−a)+an−3 (n−a)2 −· · ·+a2 (n−a)n−3 −a(n−a)n−2 +(n−a)n−1 ).

O primeiro termo da fatoração acima é exatamente n, o que mostra que an + (n − a)n é divisı́vel por
n. Como isso vale para todo a = 1, 2, . . . n−1
2 , concluı́mos a prova.

Material elaborado por Susana Frómeta Fernández

5
Problemas Resolvidos

Nı́vel 2

Congruências I

Material elaborado por Valentino Amadeus Sichinel


Problemas

Problema 1. Seja n um inteiro ı́mpar. Mostre Problema 12. Mostre que todo número primo
que n2 − 1 é divisı́vel por 8. da forma 3k + 1 é da forma 6n + 1.

Problema 2. Seja n um inteiro ı́mpar. Mostre Problema 13. Mostre que não existem inteiros
que n(n2 − 1) é divisı́vel por 24. x e y tais que x3 − 117y 3 = 5.

Problema 3. Seja n um inteiro ı́mpar que não Problema 14 (IMO). Mostre que a fração
é divisı́vel por 3. Mostre que n2 − 1 é divisı́vel 21n+4
14n+3 é irredutı́vel para todo n natural.
por 24.

Problema 15. Prove que, para todo inteiro po-


Problema 4. Mostre que, se n é um inteiro sitivo n,
ı́mpar, n2 só pode deixar resto 1 ou 9 na divisão
(a) n3 − n é divisı́vel por 3.
por 16.
(b) n5 − n é divisı́vel por 5.
(c) n7 − n é divisı́vel por 7.
Problema 5. Determine todos os restos Obs.: Veja que n9 − n não é necessariamente
possı́veis da divisão de um cubo perfeito por 7. divisı́vel por 9∗ . De fato, 29 − 2 = 510 não é
divisı́vel por 9.
Problema 6. Determine todos os restos
possı́veis da divisão de um cubo perfeito pro 9. Problema 16. Prove que 36n − 26n é divisı́vel
por 35, qualquer que seja o inteiro positivo n.
Problema 7. Sejam a e b números inteiros, e
seja k um inteiro positivo qualquer. Mostre que Problema 17. Seja n > 2 um número natural.
a − b divide ak − bk . Prove que se um dos números 2n − 1 e 2n + 1 é
primo, então o outro é composto.
Problema 8. Sejam a e b números inteiros, e
seja k um inteiro positivo ı́mpar. Mostre que Problema 18. Prove que, se tanto p quanto
a + b divide ak + bk . 8p − 1 são primos, então 8p + 1 é composto.

Problema 9. Seja n um inteiro positivo. Prove Problema 19. Prove que, se tanto p quanto
que 198n − 1 é divisı́vel por 17. 8p2 + 1 são primos, então 8p2 − 1 também é
primo.
Problema 10. Prove que, se n é um inteiro po-
sitivo ı́mpar, então 45 divide 133n + 173n . Problema 20. Prove que n2 + 3n + 5 nunca é
divisı́vel por 121, seja qual for o inteiro n.
Problema 11. Mostre que não existem inteiros
x, y, z e w tais que x2 + y 2 + z 2 = 8w + 7.


Os itens a-c são casos especiais de um teorema famoso. Para entender melhor, veja o material sobre o pequeno
teorema de Fermat.

2
Soluções

1. Como n é ı́mpar, temos quatro possibilidades: podemos ter


n≡1 (mod 8), ou n ≡ 3 (mod 8), ou n ≡ 5 (mod 8), ou n ≡ 7 (mod 8).
Analisemos cada caso separadamente.
• Se n ≡ 1 (mod 8), n2 ≡ 12 ≡ 1 (mod 8), donde n2 − 1 é divisı́vel por 8.
• Se n ≡ 3 (mod 8), n2 ≡ 32 ≡ 9 ≡ 1 (mod 8), donde n2 − 1 é divisı́vel por 8.
• Se n ≡ 5 (mod 8), n2 ≡ 52 ≡ 25 ≡ 1 (mod 8), donde n2 − 1 é divisı́vel por 8.
• Se n ≡ 7 (mod 8), n2 ≡ 72 ≡ 49 ≡ 1 (mod 8), donde n2 − 1 é divisı́vel por 8.
Dessa forma, se o inteiro n for ı́mpar, n2 − 1 será divisı́vel por 8, seja qual for o valor de n.

2. Como mdc(8, 3) = 1, para mostrarmos que n(n2 − 1) é divisı́vel por 8 × 3 = 24, é suficiente que
mostremos que n(n2 − 1) é divisı́vel por 8 e por 3. Já vimos, no problema 1, que n2 − 1 é divisı́vel por
8. Basta mostrarmos, então, que n(n2 − 1) é divisı́vel por 3. Temos três casos a analisar:
• se n ≡ 0 (mod 3), n(n2 − 1) ≡ 0 · (n2 − 1) ≡ 0 (mod 3);
• se n ≡ 1 (mod 3), n(n2 − 1) ≡ 1 · (12 − 1) ≡ 1 · 0 ≡ 0 (mod 3);
• se n ≡ 2 (mod 3), n(n2 − 1) ≡ 2 · (4 − 1) ≡ 2 · 3 ≡ 0 (mod 3).
Em qualquer caso, n(n2 − 1) é divisı́vel por 3. Como n2 − 1 é divisı́vel por 8, concluı́mos que
n(n2 − 1) é divisı́vel por 8 × 3 = 24.

3. Utilizaremos o resultado dos dois problemas anteriores.


Como n é ı́mpar, sabemos pelo problema 1 que n2 − 1 é divisı́vel por 8.
Além disso, vimos no problema 2 que n(n2 − 1) é divisı́vel por 3. Isso quer dizer que ou 3 divide
n, ou 3 divide n2 − 1 (ou ambos). Bem, estamos supondo, neste problema, que n não é divisı́vel por
3. Logo, n2 − 1 há de ser divisı́vel por 3.
Dessa forma, n2 − 1 é divisı́vel por 3 e por 8 e, portanto, por 3 × 8 = 24 (já que mdc(3, 8) = 1).

4. Vimos no problema 1 que, se n é ı́mpar, então n2 deixa resto 1 na divisão por 8. Assim, sendo n
ı́mpar, n2 = 8k + 1, para algum inteiro k.
Se k é par, digamos, igual a 2k 0 , n2 = 8k + 1 = 16k 0 + 1 ≡ 1 (mod 16).
Se k é ı́mpar, digamos, igual a 2k 0 + 1, n2 = 8k + 1 = 16k 0 + 8 + 1 ≡ 9 (mod 16).
Portanto, dado que n é ı́mpar, n2 só pode deixar resto 1 ou 9 quando dividido por 16.

5. Analisemos os casos.
• Se n ≡ 0 (mod 7), n3 ≡ 0 (mod 7).
• Se n ≡ 1 (mod 7), n3 ≡ 1 (mod 7).
• Se n ≡ 2 (mod 7), n3 ≡ 8 ≡ 1 (mod 7).
• Se n ≡ 3 (mod 7), n3 ≡ 27 ≡ −1 (mod 7).
Agora, utilizamos um pequeno truque, para encurtar os cálculos: seja qual for n ∈ Z, n3 ≡ −(−n)3
(mod 7). Bom, se n deixa um resto r ≥ 4 quando dividido por 7, então −n deixa um resto 7 − r ≤ 3
quando divido por 7. Assim, o resto de (−n)3 (quando dividido por 7) é 0, 1 ou −1 e, portanto, o
resto de n3 quando divido por 7, que é o mesmo que o resto de −(−n)3 , só pode ser 0, −1 ou 1.
Dessa forma, se n é um número inteiro, então n3 deixa resto 0, 1 ou −1 quando dividido por 7.

3
6. Analisemos os casos.
• Se n ≡ 0 (mod 9), n3 ≡ 0 (mod 9).
• Se n ≡ 1 (mod 9), n3 ≡ 1 (mod 9).
• Se n ≡ 2 (mod 9), n3 ≡ 8 ≡ −1 (mod 9).
• Se n ≡ 3 (mod 9), n3 ≡ 27 ≡ 0 (mod 9).
• Se n ≡ 4 (mod 9), n3 ≡ 4 · 16 ≡ 4 · 7 ≡ 28 ≡ −1 (mod 9).
Agora, utilizamos um pequeno truque, para encurtar os cálculos: seja qual for n ∈ Z, n3 ≡ −(−n)3
(mod 7). Bom, se n deixa um resto r ≥ 5 quando dividido por 9, então −n deixa um resto 7 − r ≤ 4
quando divido por 9. Assim, o resto de (−n)3 (quando dividido por 9) é 0, 1 ou −1 e, portanto, o
resto de n3 quando divido por 9, que é o mesmo que o resto de −(−n)3 , só pode ser 0, −1 ou 1.
Dessa forma, se n é um número inteiro, então n3 deixa resto 0, 1 ou −1 quando dividido por 9.
Observação: O fato de haver apenas três possibilidades para o resto da divisão de um cubo perfeito
por 9 pode ser útil em algumas situações. É um resultado que vale a pena ter em mente.

7. Observe que a − b ≡ 0 (mod a − b) ⇐⇒ a ≡ b (mod a − b).


Elevando à k-ésima potência, ficamos com ak ≡ bk (mod a − b), isto é, ak − bk ≡ 0 (mod a − b).
Dessa forma, a − b divide ak − bk .

8. Observe que a + b ≡ 0 (mod a + b) ⇐⇒ a ≡ −b (mod a + b).


Elevando à k-ésima potência, ficamos com ak ≡ (−b)k ≡ (−1)k · bk (mod a + b). Como k é ı́mpar,
(−1)k · bk ≡ (−1) · bk ≡ −bk (mod a + b).
Logo, ak ≡ −bk (mod a + b) ⇐⇒ ak + bk ≡ 0 (mod a + b).
Dessa forma, a + b divide ak + bk .

9. Temos 19 ≡ 2 (mod 17). Logo, 198n − 1 ≡ 28n − 1 (mod 17).


Observe agora que 24 ≡ 16 ≡ −1 (mod 17). Dessa forma, 28 ≡ (24 )2 ≡ (−1)2 ≡ 1 (mod 17).
Assim, 28n ≡ (28 )n ≡ 1n ≡ 1 (mod 17).
Logo, 198n − 1 ≡ 28n − 1 ≡ 0 (mod 17) e, portanto, 17 divide 198n − 1.

10. Como mdc(5, 9) = 1 e 45 = 5 × 9, é suficiente analisarmos a divisibilidade por 5 e por 9 separa-


damente.
Em primeiro lugar, 133n + 1713 ≡ 33n + 23n (mod 5).
Como n é ı́mpar, 3n também o é. Pelo problema 8, isso implica que 23n + 33n é divisı́vel por
2 + 3 = 5.
Resta analisarmos, então, a divisibilidade por 9. Temos 133n + 173n ≡ 43n + (−1)3n (mod 9).
Como 3n é ı́mpar, (−1)3n ≡ −1 (mod 9).
Além do mais, 42 ≡ 16 ≡ 7 (mod 9), donde 43 ≡ 4 · 7 ≡ 28 ≡ 1 (mod 9) e, portanto, 43n ≡ (43 )n ≡
n
1 ≡ 1 (mod 9).
Assim, 133n + 173n ≡ 43n + (−1)3n ≡ 1 − 1 ≡ 0 (mod 9).
Dessa forma, 133n + 173n é divisı́vel tanto por 5 quanto por 9 e, portanto, por 5 × 9 = 45.

11. Queremos mostrar que não existem inteiros x, y e z tais que x2 +y 2 +z 2 ≡ 7 (mod 8). Analisemos,
então, os possı́veis restos de um quadrado módulo 8.

4
Se n é divisı́vel por 4 (congruente a 0 ou a 4 módulo 8), n2 é divisı́vel por 42 = 16. Em particular,
n2 é divisı́vel por 8 e, portanto, é congruente a 0 módulo 8.
Se n é divisı́vel por 2, mas não por 4, n é congruente a 2 ou a 6 módulo 8. Se n ≡ 2 (mod 8),
n ≡ 4 (mod 8). Se n ≡ 6 (mod 8), n2 ≡ 36 ≡ 4 (mod 8).
2

Por fim, se n é ı́mpar, n2 ≡ 1 (mod 8) (de fato, pelo problema 1, n2 − 1 é divisı́vel por 8).
Portanto, os possı́veis restos de um quadrado módulo 8 são 0, 1 e 4. Isto é: se n é um número
inteiro, então ou n2 ≡ 0 (mod 8), ou n2 ≡ 1 (mod 8), ou n2 ≡ 4 (mod 8).

Suponhamos, agora, que x, y e z são três inteiros tais que x2 + y 2 + z 2 ≡ 7 (mod 8).
Se x, y e z fossem pares, x2 + y 2 + z 2 também seria e, portanto, não poderia ser congruente a 7
módulo 8. Dessa forma, ao menos um dos inteiros é ı́mpar. Sem perder generalidade, suporemos que
z é ı́mpar. Temos, então, z 2 ≡ 1 (mod 8). Como x2 + y 2 + z 2 ≡ 7 (mod 8), isso implica x2 + y 2 ≡ 6
(mod 8).
Mas x2 e y 2 só podem ser 0, 1 ou 4 módulo 8, de modo que x2 + y 2 só pode ser 0, 1, 2, 4 ou 5
módulo 8. Absurdo!
Tendo nossa suposição nos levado a um absurdo, concluı́mos que não há maneira de ela ser válida.
Em outras palavras: não existem inteiros x, y e z tais que x2 + y 2 + z 2 ≡ 7 (mod 8) (e, consequente-
mente, não existem inteiros x, y, z e w tais que x2 + y 2 + z 2 = 8w + 7).

12. Suponhamos que p = 3k + 1 é um número primo.


Se k ≡ 1 (mod 2), 3k + 1 ≡ 3 + 1 ≡ 4 ≡ 0 (mod 2). Logo, nesse caso, p é par. Como o único
primo par é 2, p = 2. Mas 2 não é da forma 3k + 1. Absurdo!
Dessa forma, devemos ter k ≡ 0 (mod 2). Daı́, existe um inteiro n tal que k = 2n. Temos então
p = 3k + 1 = 6n + 1.

13. Suponhamos, por absurdo, que x e y são dois inteiros tais que x3 − 117y 3 = 5. Essa igualdade
nos indica, em particular, que x3 − 117y 3 ≡ 5 (mod 9). Bem, 117 ≡ 0 (mod 9). Logo, isso é o mesmo
que x3 ≡ 5 (mod 9). No entanto, um cubo perfeito só pode deixar resto 0, 1 ou −1 quando dividido
por 9. Absurdo! Dessa forma, não existem inteiros x e y tais que x3 − 117y 3 = 5.

14. Entendamos, antes de mais nada, qual o nosso objetivo. Queremos mostrar que a fração é
irredutı́vel. Mas o que isso significa? Mostrar que uma fração é irredutı́vel é o mesmo que mostrar que
o numerador e o denominador não têm fator primo em comum. Por exemplo: a fração 35 é irredutı́vel,
porque 5 e 3 não têm nenhum fator primo em comum. Por outro lado, a fração 10 6 não é irredutı́vel,
já que tanto 10 quanto 6 são divisı́veis por 2 (10 e 6 têm o fator primo 2 em comum).
O que devemos fazer, então, é mostrar que, seja qual for o número natural n, 21n + 4 e 14n + 3
não têm fator primo em comum.
Procedamos por contradição. Suponhamos, por absurdo, que n é um número natural tal que
21n + 4 e 14n + 3 têm um fator primo em comum. Chamemos esse fator primo de p. Na prática,
então, estamos supondo que existem um natural n e um primo p tais que

21n + 4 ≡ 0 (mod p) e 14n + 3 ≡ 0 (mod p),

e queremos chegar a uma contradição.


Multiplicando a primeira congruência por 2 e a segunda por 3, ficamos com

42n + 8 ≡ 0 (mod p) e 42n + 9 ≡ 0 (mod p).

A primeira é equivalente a 42n ≡ −8 (mod p). A segunda, a 42n ≡ −9 (mod p). Dessa forma,
temos −8 ≡ −9 (mod p) ⇐⇒ 9 − 8 ≡ 0 (mod p) ⇐⇒ 1 ≡ 0 (mod p). Absurdo, pois não existe
número primo que divida 1!

5
15. (a) Consideremos os possı́veis casos:
• se n ≡ 0 (mod 3), n3 − n ≡ 0 − 0 ≡ 0 (mod 3);
• se n ≡ 1 (mod 3), n3 − n ≡ 1 − 1 ≡ 0 (mod 3);
• se n ≡ 2 (mod 3), n3 − n ≡ 8 − 2 ≡ 6 ≡ 0 (mod 3).
Dessa forma, n3 − n é sempre divisı́vel por 3.

(b) Queremos mostrar que 5 | n(n4 − 1). Quando n ≡ 0 (mod 5), o resultado é evidente. Nos
outros casos, mostraremos que 5 | n4 − 1. Utilizaremos um pequeno truque: o fato de que (−n)4 ≡ n4
(mod 5). Em função disso, é suficiente checar para n ≡ 1 (mod 5) e n ≡ 2 (mod 5):
• se n ≡ 1 (mod 5), n4 − 1 ≡ 1 − 1 ≡ 0 (mod 5);
• se n ≡ 2 (mod 5), n4 − 1 ≡ 16 − 1 ≡ 0 (mod 5).
Dessa forma, n5 − n é sempre divisı́vel por 5.

(c) Queremos mostrar que 7 | n(n6 − 1). Quando n ≡ 0 (mod 7), o resultado é evidente. Nos
outros casos, mostraremos que 7 | n6 − 1. Utilizaremos um pequeno truque: o fato de que (−n)6 ≡ n6
(mod 7). Em função disso, é suficiente checar para n ≡ 1 (mod 7), para n ≡ 2 (mod 7) e para n ≡ 3
(mod 7):
• se n ≡ 1 (mod 7), n6 − 1 ≡ 1 − 1 ≡ 0 (mod 7);
• se n ≡ 2 (mod 7), n6 − 1 ≡ 64 − 1 ≡ 0 (mod 7);
• se n ≡ 3 (mod 7), n6 − 1 ≡ 36 − 1 ≡ 93 − 1 ≡ 23 − 1 ≡ 8 − 1 ≡ 0 (mod 7).
Dessa forma, n7 − n é sempre divisı́vel por 7.

16. Temos 36 ≡ 93 ≡ (−1)3 ≡ −1 (mod 5).


Além disso, 26 ≡ 64 ≡ −1 (mod 5).
Logo, 36n − 26n ≡ (−1)n − (−1)n ≡ 0 (mod 5).
Analisemos agora a divisibilidade por 7.
Temos 36 ≡ 93 ≡ 23 ≡ 1 (mod 7).
Além disso, 26 ≡ 82 ≡ 12 ≡ 1 (mod 7).
Logo, 36n − 26n ≡ 1n − 1n ≡ 0 (mod 7).
Dessa forma, 36n − 26n é divisı́vel tanto por 5 quanto por 7, seja qual for o inteiro positivo n. Como
mdc(5, 7) = 1, segue que 36n − 26n é divisı́vel pro 5 × 7 = 35, seja qual for n ∈ Z.

17. Consideremos os dois casos.


Caso I: 2n − 1 é primo.
Como n > 2, 2n − 1 > 3. Logo, se 2n − 1 é primo, 2n − 1 6≡ 0 (mod 3).
Além disso, é claro que 2n − 1 6≡ −1 (mod 3), já que 2n 6≡ 0 (mod 3).
Assim, 2n − 1 ≡ 1 (mod 3) e, portanto, 2n + 1 ≡ 3 ≡ 0 (mod 3).
Segue daı́ que 2n + 1 não é primo.
Caso II: 2n + 1 é primo.
Como n > 2, 2n + 1 > 3. Logo, se 2n + 1 é primo, 2n + 1 6≡ 0 (mod 3).
Além disso, é claro que 2n + 1 6≡ 1 (mod 3), já que 2n 6≡ 0 (mod 3).
Assim, 2n + 1 ≡ −1 (mod 3) e, portanto, 2n − 1 ≡ −3 ≡ 0 (mod 3).
Como 2n − 1 > 3, segue daı́ que 2n − 1 não é primo.

6
18. Suponhamos que o primo p é tal que 8p − 1 também é primo.
Se p = 3, 8p + 1 = 25 é composto. Consideremos, então, daqui para a frente, que p 6= 3.
Temos p 6≡ 0 (mod 3) e 8p − 1 6≡ 0 (mod 3) ⇐⇒ −p − 1 6≡ 0 (mod 3) ⇐⇒ p 6≡ −1 (mod 3).
Logo, p ≡ 1 (mod 3).
Assim, 8p + 1 ≡ 1 + 2 = 3 ≡ 0 (mod 3) e, portanto, 8p + 1 não pode ser primo.

19. Suponhamos que o primo p é tal que 8p2 + 1 também é primo.


Se p 6≡ 0 (mod 3), p2 ≡ 1 (mod 3) e, assim, 8p2 + 1 ≡ 8 + 1 ≡ 9 ≡ 0 (mod 3). Mas 8p2 + 1 > 3,
donde, sendo primo, 8p2 + 1 não pode ser divisı́vel pro 3. Logo, devemos ter p ≡ 0 (mod 3).
Como p é primo, a única maneira de termos p ≡ 0 (mod 3) é que tenhamos p = 3. Assim, p = 3,
e 8p2 − 1 = 71 também é primo.

20. Observe que n2 + 3n + 5 = n2 + 3n − 28 + 33 = (n + 7)(n − 4) + 33.


Assim, se n2 + 3n + 5 é divisı́vel por 121, n2 + 3n + 5 é, em particular, divisı́vel por 11 e, como 33
é divisı́vel por 11, (n + 7)(n − 4) também o deve ser.
Sendo 11 primo, (n + 7)(n − 4) só pode ser divisı́vel por 11 se n + 7 ou n − 4 o for. Veja agora que
→ se n + 7 ≡ 0 (mod 11), então n − 4 ≡ n + 7 − 11 ≡ 0 (mod 11);
→ se n − 4 ≡ 0 (mod 11), então n + 7 ≡ n − 4 + 11 ≡ 0 (mod 11).
Dessa forma, tanto n + 7 quanto n − 4 devem ser divisı́veis por 11.
Daı́, (n + 7)(n − 4) deve ser divisı́vel por 11 · 11 = 121.
Logo, se n2 + 3n + 5 = (n + 7)(n − 4) + 33 for divisı́vel por 121, 33 também o deve ser. Mas 33
não é divisı́vel por 121. Portanto, não existe natural n para o qual n2 + 3n + 5 é divisı́vel por 121.

Material elaborado por Valentino Amadeus Sichinel

7
Problemas Resolvidos

Nı́vel 2

Congruências II

Material elaborado por Valentino Amadeus Sichinel


Problemas

100
1 Pequeno Teorema de Fermat Problema 10. Se um “googolplex” é 1010 ,
que dia da semana será daqui a um googolplex
Problema 1. Encontre o resto da divisão de: de dias?
(a) 331 por 7. Obs.: Suponha que hoje seja um domingo.
(b) 21000 por 13.
(c) 128129 por 17.
Problema 11. Mostre que não existe inteiro
positivo n > 1 tal que n | 2n − 1.
Problema 2. Sejam p e q dois primos distintos
e a um inteiro positivo tal que aq ≡ a (mod p)
Problema 12 (IMO). Determine todos os in-
e ap ≡ a (mod q). Prove que apq ≡ a (mod pq).
teiros positivos m tais que
mdc(m, 2n + 3n + 6n − 1) = 1 ∀ n ∈ N.
Problema 3. Mostre que existem infinitos 1 1 1
Dica: 2 + 3 + 6 = 1.
números da forma 1000...01 que são divisı́veis
por 101.
Dica: 101 é primo.
2 Teorema de Wilson
Problema 4. Mostre que não existe inteiro po-
Problema 13. Seja n umnúmero natural. Cal-
sitivo x tal que 103 | x3 − 2.
cule mdc n! + 1, (n + 1)! .
Dica: 103 é primo.

Problema 14. Seja p um primo ı́mpar. Mostre


Problema 5. Sejam a e b inteiros positivos. que
Prove que a12 ≡ b12 (mod 91) se, e somente se,
 !2
mdc(a, 91) = mdc(b, 91).

p−1
! ≡ (−1)(p+1)/2 (mod p).
2
Problema 6. Seja p um primo ı́mpar para o
qual existe x ∈ Z tal que x2 ≡ −1 (mod p).
Problema 15. Seja p um número primo tal
Prove que p ≡ 1 (mod 4).
que p ≡ 1 (mod 4). Prove que a congruência
x2 ≡ −1 (mod p) tem solução.
Problema 7. Encontre um inteiro positivo k Obs.: Compare este resultado com o do problema
tal que 5k ≡ 97 (mod 101). 6.
Dica: 101 é primo.
Problema 16 (IMO). Determine todos os in-
Problema 8. Quantos números primos p exis- teiros positivos n para os quais o conjunto
tem tais que 27p + 1 é um múltiplo de p?
{n, n + 1, n + 2, n + 3, n + 4, n + 5}

Problema 9. Um número da forma pn , em que pode ser repartido em dois subconjuntos de tal
p é um número primo e n é um inteiro positivo, maneira que o produto dos elementos de um é
n
é dito legal se pn | 7p + 1. Determine a soma igual ao produto dos elementos do outro.
de todos os números legais.

2
Soluções

1. (a) 7 é primo e não divide 3. Assim, pelo Pequeno Teorema de Fermat, 36 ≡ 1 (mod 7). Logo,
331 ≡ 330 · 3 ≡ (36 )5 · 3 ≡ 15 · 3 ≡ 3 (mod 7).

(b) 13 é primo e não divide 2. Assim, pelo Pequeno Teorema de Fermat, 212 ≡ 1 (mod 13). Logo,
21000 ≡ 2996 · 24 ≡ (212 )83 · 24 ≡ 183 · 24 ≡ 24 ≡ 16 ≡ 3 (mod 13).

(c) 17 é primo e não divide 128. Assim, pelo Pequeno Teorema de Fermat, 12816 ≡ 1 (mod 17).
Logo,
128129 ≡ 128128 · 128 ≡ (12816 )8 · 128 ≡ 18 · 128 ≡ 128 ≡ 9 (mod 17).

2. Pelo Pequeno Teorema de Fermat, ap ≡ a (mod p). Assim,


apq ≡ (ap )q ≡ aq ≡ a (mod p).
Da mesma forma, o Pequeno Teorema de Fermat nos conta que aq ≡ a (mod q), o que implica
apq ≡ (ap )q ≡ aq ≡ a (mod q).
Assim, temos que p | (apq − a) e q | (apq − a). Como p e q são primos distintos, isso implica
pq | (apq − a). Portanto, apq − a ≡ 0 (mod pq), ou seja, apq ≡ a (mod pq).

3. Queremos mostrar que existem infinitos números da forma 10k + 1 que são divisı́veis por 101.
Observe que
10k + 1 ≡ 0 (mod 101) ⇐⇒ 10k ≡ 100 (mod 101) ⇐⇒ 10k−2 ≡ 1 (mod 101).
Como 101 é primo e 101 - 10, sabemos, pelo Pequeno Teorema de Fermat, que 10100 ≡ 1 (mod 101).
Assim, se k − 2 = 100 · t para algum t (inteiro positivo), temos
10k−2 ≡ 10100t ≡ (10100 )t ≡ 1t ≡ 1 (mod 101).
Dessa forma, para que 10k + 1 seja múltiplo de 101, é suficiente que tenhamos k − 2 = 100 · t para
algum t inteiro positivo, isto é, é suficiente que tenhamos k = 100t + 2, para algum t inteiro positivo.
Como existem infinitos números da forma 100t+2, concluı́mos que existem infinitos números da forma
10k + 1 que são divisı́veis por 101, isto é, que existem infinitos números da forma 1000...01 que são
divisı́veis por 101.

4. Suponhamos, por absurdo, que existe x ∈ Z tal que 103 | x3 − 2, isto é, tal que
x3 − 2 ≡ 0 (mod 103) ⇐⇒ x3 ≡ 2 (mod 103).
Como 2 6≡ 0 (mod 103), x3 6≡ 0 (mod 103). Como 103 é primo, isso é o mesmo que x 6≡ 0
(mod 103). Sendo assim, segue do Pequeno Teorema de Fermat que x102 ≡ 1 (mod 103). Temos,
então,
234 ≡ (x3 )34 ≡ x102 ≡ 1 (mod 103).
Mas 234 6≡ 1 (mod 103). De fato,
234 ≡ 227 · 27 ≡ (29 )3 · 27 ≡ (512)3 · 128 ≡ (−3)3 · 25 ≡ −675 ≡ 46 (mod 103).
Dessa forma, nossa hipótese não se verifica, isto é, não há como existir x ∈ Z tal que 103 | x3 − 2.

3
5. Comecemos por fatorar 91: temos 91 = 7 · 13. Assim, para um inteiro x qualquer, temos



 91, se 7 | x e 13 | x

 13, se 7 - x e 13 | x
mdc(x, 91) = . (1)


 7, se 7 | x e 13 - x

 1, se 7 - x e 13 - x

Além disso, segue da fatoração que


(
x≡y (mod 7)
x≡y (mod 91) ⇐⇒ , (2)
x≡y (mod 13)

sejam quais forem os inteiros x e y.


7 é primo. Logo, pelo Pequeno Teorema de Fermat,

7 - x ⇐⇒ x6 ≡ 1 (mod 7) ⇒ x12 ≡ 1 (mod 7).

Como x12 ≡ 1 (mod 7) implica 7 - x, temos, na verdade, a equivalência

7 - x ⇐⇒ x12 ≡ 1 (mod 7).

Daı́, vem que (


12 0 (mod 7), se 7 | x
x ≡ . (3)
1 (mod 7), se 7 - x
Por fim, 13 é primo, donde, pelo Pequeno Teorema de Fermat,

13 - x ⇒ x12 ≡ 1 (mod 13),

seja qual for x ∈ Z. Disso, segue que


(
0 (mod 13), se 13 | x
x12 ≡ , (4)
1 (mod 13), se 13 - x

para todo x ∈ Z.
Agora, é só juntar as informações.
Por (1), (3) e (4), temos



 91, se x12 ≡ 0 (mod 7) e x12 ≡ 0 (mod 13)
 13, se x12 ≡ 1 (mod 7)

e x12 ≡ 0 (mod 13)
mdc(x, 91) = .


 7, se x12 ≡ 0 (mod 7) e x12 ≡ 1 (mod 13)
 1, se x12 ≡ 1 (mod 7)

e x12 ≡ 1 (mod 13)

Isso mostra que, para inteiros x e y,


(
x12 ≡ y 12 (mod 7)
mdc(x, 91) = mdc(y, 91) ⇐⇒ .
x12 ≡ y 12 (mod 13)

Aplicando esta última equivalência, juntamente com a equivalência (2), aos inteiros a e b, con-
cluı́mos que
a12 ≡ b12 (mod 91) ⇐⇒ mdc(a, 91) = mdc(b, 91),
tal como querı́amos.

4
6. Se x2 ≡ −1 (mod p), p - x. Assim, pelo Pequeno Teorema de Fermat, xp−1 ≡ 1 (mod p). Como p
é ı́mpar, p−1
2 é inteiro. Daı́,

x2 ≡ −1 (mod p) ⇒ xp−1 ≡ (−1)(p−1)/2 (mod p)


p−1
⇒ (−1)(p−1)/2 ≡ 1 (mod p) ⇒ ≡ 0 (mod 2)
2
⇒ p ≡ 1 (mod 4).

7. Observe que 97 ≡ −4 (mod 101). Assim, queremos encontrar k tal que 5k ≡ −4 (mod 101). Como
mdc(101, 25) = 1, essa congruência é equivalente a 5k+2 ≡ −100 (mod 101). E −100 ≡ 1 (mod 101).
Dessa forma, o que queremos é encontrar k tal que 5k+2 ≡ 1 (mod 101).
Como 101 é primo e 101 - 5, segue do Pequeno Teorema de Fermat que 5100 ≡ 1 (mod 101).
Portanto, para alcançar nosso objetivo, podemos fazer k = 98.

8. É claro que se 27p + 1 é múltiplo de p, p não divide 27. Assim, se p é um número primo tal que
27p + 1 ≡ 0 (mod p), temos, pelo Pequeno Teorema de Fermat, que 27p ≡ 27 (mod p).
Combinando as duas congruências, ficamos com 28 ≡ 27 + 1 ≡ 27p + 1 ≡ 0 (mod p), isto é, p deve
dividir 28. Os únicos primos que dividem 28 são 2 e 7.
Será que 2 e 7 satisfazem a condição do enunciado? A resposta é sim, e para verificar, basta
percorrer o caminho inverso: se p = 2 ou 7,
27p + 1 ≡ 27 + 1 ≡ 28 ≡ 0 (mod p).
Portanto, há exatamente dois números primos p tais que 27p + 1 é um múltiplo de p.

9. Sejam p um número primo e n um inteiro positivo tais que pn é um número legal.


n
De pn | 7p + 1, segue que p - 7. Daı́, pelo Pequeno Teorema de Fermat, 7p−1 ≡ 1 (mod p). Além
disso, pn ≡ 1 (mod p − 1) (já que p ≡ 1 (mod p − 1)), donde existe k ∈ N tal que pn = (p − 1)k + 1.
Assim,
n
7p + 1 ≡ 7(p−1)k+1 + 1 ≡ (7p−1 )k · 7 + 1 ≡ 1k · 7 + 1 ≡ 8 (mod p).
n n
Mas p | 7p + 1, pois p | pn e pn | 7p + 1. Logo, p | 8, donde p deve ser igual a 2.
n n
Como 7 ≡ −1 (mod 4), 72 ≡ 1 (mod 4), seja qual for o inteiro n ≥ 1. Assim, 72 + 1 ≡ 2
n
(mod 4) ∀ n ≥ 1, donde 4 - 72 + 1.
Dessa forma, para que 2n seja legal, n deve ser no máximo 1. Em outras palavras, 2 é o único
possı́vel número legal. Como 2 | 72 + 1, 2 é, de fato, legal.
Portanto, a soma de todos os números legais é igual a 2.

10. Como os dias da semana se repetem em um ciclo de tamanho 7, queremos saber qual o resto da
divisão de um googolplex por 7.
Pelo Pequeno Teorema de Fermat, 106 ≡ 1 (mod 7). Para utilizar isso a nosso favor na hora de
100
calcular 1010 (mod 7), precisamos decompor 10100 em um múltiplo de 6 e um resto.
Observe que 102 ≡ 100 ≡ 10 (mod 6). Daı́, 10k ≡ 102 · 10k−2 ≡ 10 · 10k−2 ≡ 10k−1 (mod 6) para
todo k ≥ 2 inteiro. Consequentemente,
10100 ≡ 1099 ≡ 1098 ≡ 1097 ≡ . . . ≡ 103 ≡ 102 ≡ 10 ≡ 4 (mod 6).
Isso quer dizer que existe um inteiro k tal que 10100 = 6k + 4.
Dessa forma,
100
1010 ≡ 106k+4 ≡ (106 )k · 104 ≡ 1k · 104 ≡ 104 ≡ 34 ≡ 81 ≡ 4 (mod 7).
Portanto, o dia da semana em que estaremos daqui a um googolplex de dias será 4 dias da semana
depois do dia de hoje. Se hoje é domingo, então, daqui a um googolpex de dias será uma quinta-feira.

5
11. Suponhamos, por absurdo, que n > 1 seja um inteiro tal que n | 2n − 1.
Seja p o menor fator primo de n. Como n | 2n − 1, p | 2n − 1. Daı́, p deve ser distinto de 2.
Pelo Pequeno Teorema de Fermat, então, p | 2p−1 − 1. De que maneira podemos combinar essas duas
informações (a saber, a informação de que p | 2n − 1 e a informação de que p | 2p−1 − 1)?
A resposta está no teorema de Bézout1 . Como p é o menor fator primo de n, os fatores primos
de p − 1 são menores que qualquer fator primo de n e, assim, mdc(p − 1, n) = 1 - eis o motivo pelo
qual escolhemos p como sendo o menor fator primo de n. Dessa forma, existem inteiros a e b tais que
a(p − 1) + bn = 1. Como p − 1 e n são ambos maiores que 1, ou a é positivo e b é negativo, ou a é
negativo e b é positivo.
Se a é positivo e b é negativo, temos a(p − 1) = |b|n + 1. Daı́,

1 ≡ 1a ≡ (2p−1 )a ≡ 2a(p−1) ≡ 2|b|n+1 ≡ (2n )|b| · 2 ≡ 1|b| · 2 ≡ 2 (mod p).

Absurdo!
Por outro lado, se a é negativo e b é positivo, temos bn = |a|(p − 1) + 1. Daı́,

1 ≡ 1b ≡ (2n )b ≡ 2nb ≡ 2|a|(p−1)+1 ≡ (2p−1 )|a| · 2 ≡ 1|a| · 2 ≡ 2 (mod p).

Absurdo também!
Portanto, não é possı́vel que haja n ∈ N, n > 1, tal que n | 2n − 1.

12. Afirmamos que nenhum inteiro positivo maior que 1 é primo com todos os termos da sequência
{2n + 3n + 6n − 1}n≥1 . Para vermos isso, é suficiente que provemos que cada primo divide ao menos
um elemento da sequência.
Antes de mais nada, observemos que todos os termos da sequência são pares. Assim, 2 divide
todos eles. Além disso, o segundo termo da sequência é divisı́vel por 3. De fato, temos

22 + 32 + 62 − 1 ≡ 22 − 1 ≡ 3 ≡ 0 (mod 3).

Dessa forma, basta olharmos para os primos p diferentes de 2 e de 3.


Seja, então, p um primo distinto de 2 e de 3.
Como p - 2, p - 3 e p - 6, o Pequeno Teorema de Fermat nos garante que

2p−1 ≡ 1 (mod p), 3p−1 ≡ 1 (mod p) e 6p−1 ≡ 1 (mod p).

Daı́, segue que

6 2p−2 + 3p−2 + 6p−2 ≡ 3 · 2p−1 + 2 · 3p−1 + 6p−1 ≡ 3 + 2 + 1 ≡ 6



(mod p).

Como mdc(6, p) = 1, isso implica

2p−2 + 3p−2 + 6p−2 ≡ 1 (mod p).

Dessa forma, p divide 2p−2 + 3p−2 + 6p−2 − 1.


Como todos os primos dividem ao menos um termo da sequência {2n + 3n + 6n − 1}n≥1 , não existe
nenhum inteiro m maior que 1 tal que mdc(m, 2n + 3n + 6n − 1) = 1 ∀ n ∈ N. Portanto, o único
inteiro positivo que satisfaz a condição expressa no enunciado é o número 1.

1
O Teorema de Bézout diz que se x e y são inteiros tais que mdc(x, y) = 1, existem inteiros a e b tais que ax + by = 1.

6
13. Se p é um primo tal que p | n!, é claro que p - n!+1. Por outro lado, se p é primo tal que p | (n+1)!,

então p | n! ou p | (n + 1). Dessa forma, para analisarmos os fatores primos de mdc n! + 1, (n + 1)! ,
basta que olhemos para os fatores primos de n + 1.
Se n + 1 é composto, todos os seus fatores primos são estritamente menores que n + 1 e, assim,
dividem n!. Dessa forma, nesse caso, mdc n! + 1, (n + 1)! = 1.
Por outro lado, se n + 1 é primo, o teorema de Wilson  nos diz que n! ≡ −1 (mod n + 1). Assim,
nesse caso, n + 1 | n! + 1 e, então, mdc n! + 1, (n + 1)! = n + 1.
Portanto, (
 1, se n + 1 é composto
mdc n! + 1, (n + 1)! = .
n + 1, se n + 1 é primo

14. Observe que


p + 1 p − 1
(p − 1)! ≡ (p − 1) · (p − 2) · · · · ···2 · 1
2 2
 p − 1   p − 1 
≡ (−1) · (−2) · · · − · ···2 · 1
2 2
 p − 1 2
≡ (−1)(p−1)/2 · 12 · 22 · · ·
2
  2!
(p−1)/2 p−1
≡ (−1) · ! (mod p).
2

Pelo Teorema de Wilson, (p − 1)! ≡ −1 (mod p). Assim,


  !2
(p−1)/2 p−1
(−1) · ! (mod p) ≡ −1 (mod p)
2

e, portanto,
  !2
p−1
! ≡ (−1)(p+1)/2 (mod p).
2

15. Pelo problema anterior, se p é um primo tal que p ≡ 1 (mod 4),


 
p−1
x := !
2

é tal que x2 ≡ (−1)(p+1)/2 ≡ −1 (mod p).

16. Afirmamos que não há nenhum inteiro n que satisfaz a condição exposta no enunciado. Para
vermos pro que, suponhamos por absurdo que n é tal que {n, n + 1, n + 2, n + 3, n + 4, n + 5} pode
ser repartido em dois subconjuntos, A e B, o produto dos elementos de um sendo igual ao produto
dos elementos do outro. Temos, então,

A ∩ B = ∅, A ∪ B = {n, n + 1, n + 2, n + 3, n + 4, n + 5}

e
Y Y
a= b.
a∈A b∈B

7
Nenhum dos elementos de {n, n + 1, n + 2, n + 3, n + 4, n + 5} pode ser divisı́vel por 7. De fato,
como
Q são seis
Q inteiros consecutivos, se um deles fosse múltiplo de 7, nenhum outro seria, e então ou
a∈A a ou b∈B b seria múltiplo de 7, mas não ambos (e isso nos conduziria a um absurdo, pois os
produtos são iguais).
Dessa forma, como estamos lidando com seis inteiros consecutivos, nenhum dos quais divisı́vel por
7,
n(n + 1)(n + 2)(n + 3)(n + 4)(n + 5) ≡ 6! (mod 7).
Pelo teorema de Wilson, 6! ≡ −1 (mod 7). Por outro lado,
 Y  Y   Y 2
n(n + 1)(n + 2)(n + 3)(n + 4)(n + 5) = a b = a .
a∈A b∈B a∈A

Assim,
 Y 2
a ≡ −1 (mod 7).
a∈A

Isso é um absurdo, no entanto, conforme mostra o resultado do problema 6.

Material elaborado por Valentino Amadeus Sichinel

8
Problemas Resolvidos

Nı́vel 2

Congruências III

Material elaborado por Valentino Amadeus Sichinel


Problemas

Problema 1. Determine os dois últimos alga- Problema 9. Mostre que existe um natural n
rismos de: tal que 2n > 102000 e tal que 2n possui, entre
suas últimas 2000 casas decimais, ao menos 1000
(a) 31000
zeros consecutivos.
(b) 13100
(c) 172020
(d) 77
1000 Problema 10. Prove que para cada número na-
tural k existe ao menos um número natural n tal
que
Problema 2. Prove que 504 | n9 − n3 , seja qual φ(n + k) = φ(n).
for n ∈ N.

Dica: Considere o menor primo p que não di-


Problema 3. Seja n um inteiro positivo ı́mpar vide k e olhe para o número n = (p − 1)k.
que não é divisı́vel por 5. Mostre que existe um
número da forma 111...11 que é divisı́vel por n.
Problema 11 (EUA). Seja n um número in-
teiro maior que 1. Mostre que a sequência
Problema 4 (OBM). Mostre que existem in- 2 22
finitos números da forma 1999...91 que são di- 2, 22 , 22 , 22 , . . . (mod n)
visı́veis por 1991.
é eventualmente constante, isto é, a partir de
um certo termo da sequência, todos os termos
Problema 5. Seja n > 1 um inteiro. Prove que deixam o mesmo resto quando divididos por n.

X nφ(n)
a = . Problema 12. Mostre que, para qualquer in-
2
0<a<n
mdc(a,n)=1 teiro positivo n distinto de 2 e de 6,

φ(n) ≥ n.
Problema 6. Sejam m e n inteiros maiores que
1 tais que mdc(m, n) = 1. Mostre que
Problema 13. Para quantos inteiros 1 ≤ i ≤
m φ(n)
+n φ(m)
≡ 1 (mod mn). 1000 existe um inteiro 1 ≤ j ≤ 1000 tal que
i | 2j − 1?

Problema 7. Seja n um inteiro maior que 4 tal


Problema 14. Mostre que existem infinitos
que tanto n − 1 quanto n + 1 são primos. Mostre
números da forma 2000...021 que são divisı́veis
que
n por 2021.
φ(n) ≤ .
3

Problema 8. Seja n um inteiro positivo. Prove


que X
φ(d) = n.
d|n

2
Soluções

1. (a) Para encontrar os dois últimos dı́gitos de 31000 , é necessário e suficiente que encontremos o
resto da divisão de 31000 por 100.
Como mdc(3, 100) = 1, o Teorema de Euler nos diz que 3φ(100) ≡ 1 (mod 100).
Como 100 = 22 · 52 , φ(100) = 1 · 21 · 4 · 51 = 40. Assim, 340 ≡ 1 (mod 100).
Dessa forma,
31000 ≡ 340·25 ≡ (340 )25 ≡ 125 ≡ 1 (mod 100).
Portanto, os dois últimos algarismos de 31000 são 0 e 1, nessa ordem.

(b) Para encontrar os dois últimos dı́gitos de 13100 , é necessário e suficiente que encontremos o
resto da divisão de 13100 por 100.
Como mdc(13, 100) = 1, o Teorema de Euler nos diz que 13φ(100) ≡ 1 (mod 100).
Como 100 = 22 · 52 , φ(100) = 1 · 21 · 4 · 51 = 40. Assim, 1340 ≡ 1 (mod 100).
Dessa forma,

13100 ≡ 1340·2+20 ≡ (1340 )2 · 1320 ≡ 12 · 1320 ≡ 1 · 1320 ≡ 1320 (mod 100).

Uma conta simples mostra que

133 ≡ 169 · 13 ≡ 69 · 13 ≡ 897 ≡ 97 ≡ −3 (mod 100).

Logo,

1320 ≡ 133·6+2 ≡ (133 )6 · 132 ≡ (−3)6 · 132 ≡ 729 · 169 ≡ 29 · 69 ≡ 2001 ≡ 1 (mod 100).

Portanto, os dois últimos algarismos de 13100 são 0 e 1, nessa ordem.

(c) Para encontrar os dois últimos dı́gitos de 172020 , é necessário e suficiente que encontremos o
resto da divisão de 172020 por 100.
Como mdc(17, 100) = 1, o Teorema de Euler nos diz que 17φ(100) ≡ 1 (mod 100).
Como 100 = 22 · 52 , φ(100) = 1 · 21 · 4 · 51 = 40. Assim, 1740 ≡ 1 (mod 100).
Dessa forma,

172020 ≡ 1740·50+20 ≡ (1740 )50 · 1720 ≡ 150 · 1720 ≡ 1720 (mod 100).

E agora? Os restos que as primeiras potências de 17 deixam quando divididas por 100 parecem
grandes demais para que tentemos simplificar 1720 (mod 100)... O truque está em olhar módulo 4 e
módulo 25 separadamente:
Temos mdc(17, 4) = 1 e, portanto,

1720 ≡ 172·10 ≡ (172 )10 ≡ (17φ(4) )10 ≡ 110 ≡ 1 (mod 4),

ou seja, 4 | 1720 − 1.
Por outro lado, mdc(17, 25) = 1 e, portanto,

1720 ≡ 17φ(25) ≡ 1 (mod 25),

ou seja, 25 | 1720 − 1.
Como tanto 4 quanto 25 dividem 1720 − 1, e como mdc(4, 25) = 1, segue que 4 × 25 = 100 divide
17 − 1. Daı́, 1720 ≡ 1 (mod 100).
20

Portanto, os dois últimos algarismos de 172020 são 0 e 1, nessa ordem.

3
1000
(d) Para encontrar os dois últimos dı́gitos de 77 , é necessário e suficiente que encontremos o
1000
resto da divisão de 77 por 100.
Como mdc(7, 100) = 1, o Teorema de Euler nos diz que 7φ(100) ≡ 1 (mod 100).
Como 100 = 22 · 52 , φ(100) = 1 · 21 · 4 · 51 = 40. Assim, 740 ≡ 1 (mod 100).
É útil, então, que decomponhamos 71000 como soma de um múltiplo de 40 e um resto.
Veja que 72 ≡ 49 ≡ 9 (mod 40). Assim, 74 ≡ 92 ≡ 81 ≡ 1 (mod 40). Daı́,

71000 ≡ (74 )250 ≡ 1250 ≡ 1 (mod 40).

Isso implica a existência de um inteiro positivo k tal que 71000 = 40 · k + 1. Dessa forma,
1000
77 ≡ 740·k+1 ≡ (740 )k · 7 ≡ 1k · 7 ≡ 7 (mod 100).
1000
Portanto, os dois últimos algarismos de 77 são 0 e 7, nessa ordem.

2. O primeiro passo é fatorar 504: 504 = 23 ·32 ·7. Como 23 , 32 e 7 são dois a dois primos entre si, para
mostrar que 504 | n9 − n3 , é necessário e suficiente que mostremos que 23 | n9 − n3 , que 32 | n9 − n3 ,
e que 7 | n9 − n3 .
Comecemos pelo 23 .
Se n ≡ 0 (mod 2), n9 − n3 = n3 (n6 − 1) ≡ 0 (mod 23 ).
Por outro lado, se n 6≡ 0 (mod 2), então mdc(n, 23 ) = 1. Daı́, n2 ≡ 1 (mod 23 )1 . Logo, n9 − n3 ≡
(n )4 · n − n2 · n ≡ n − n ≡ 0 (mod 23 ).
2

Dessa forma, em qualquer caso, 23 | n9 − n3 .


Analisemos a divisibilidade por 32 .
Se n ≡ 0 (mod 3), n9 − n3 = n2 (n7 − n) ≡ 0 (mod 32 ).
Por outro lado, se n 6≡ 0 (mod 3), então mdc(n, 32 ) = 1. Daı́, pelo Teorema de Euler, n6 ≡ 1
(mod 32 ) (de fato, φ(32 ) = 6). Logo, n9 − n3 ≡ n3 (n6 − 1) ≡ n3 (1 − 1) ≡ 0 (mod 32 ).
Dessa forma, em qualquer caso, 32 | n9 − n3 .
Por fim, se n ≡ 0 (mod 7), então n9 − n3 = n(n8 − n2 ) ≡ 0 (mod 7) e, se n 6≡ 0 (mod 7), o
Teorema de Euler nos dá n6 ≡ 1 (mod 7), donde n9 − n3 = n3 (n6 − 1) ≡ n3 (1 − 1) ≡ 0 (mod 7).
Dessa forma, em qualquer caso, 7 | n9 − n3 .
Como n9 − n3 é divisı́vel por 23 , por 32 e por 7, independentemente do valor de n, concluı́mos que
504 = 23 · 32 · 7 | n9 − n3 , seja qual for n ∈ N.

3. Antes de mais nada, entendamos o problema: o número 111...11, com k 1’s, nada mais é que

10k − 1
.
9
O que queremos, então, é mostrar que, se n não é divisı́vel nem por 2, nem por 5, existe um inteiro
k
positivo k tal que 10 9−1 é divisı́vel por n.
Ora, se n não é divisı́vel nem por 2, nem por 5, mdc(n, 10) = 1. Mais que isso, na verdade,
mdc(9n, 10) = 1 (9 também não tem nenhum fator em comum com 10). Daı́, o teorema de Euler nos
diz que
10φ(9n) ≡ 1 (mod 9n),
ou seja,
9n | 10φ(9n) − 1.
1
Verifique!

4
Isso quer dizer que existe um inteiro m tal que

10φ(9n) − 1 = 9n · m.

Como essa igualdade é equivalente a

10φ(9n) − 1
= n · m,
9
concluı́mos que
10φ(9n) − 1
n| .
9

4. Queremos mostrar que existem infinitos números da forma 10k +9(10k−1 +10k−2 +· · ·+102 +10)+1
que são divisı́veis por 1991. Observe, antes de mais nada, que

10k − 10
10k−1 + 10k−2 + · · · + 102 + 10 = .
9
Assim, o que queremos é mostrar que existem infinitos k ∈ N para os quais 10k +10k −10+1 = 2·10k −9
é divisı́vel por 1991. Veja agora que

2 · 10k − 9 ≡ 0 (mod 1991) ⇐⇒ 2 · 10k − 9 ≡ 1991 (mod 1991) ⇐⇒ 2 · 10k ≡ 2000 (mod 1991).

Como mdc(1991, 2) = 1, segue que a congruência é equivalente a

10k ≡ 1000 (mod 1991).

Por fim, como mdc(1991, 10) = 1, esta última equivale a

10k−3 ≡ 1 (mod 1991).

Portanto, o problema se resume a demonstrar que existem infinitos k ∈ N tais que

10k−3 ≡ 1 (mod 1991).

Como mdc(10, 1991) = 1, segue do Teorema de Euler que 10φ(1991) ≡ 1 (mod 1991). Daı́, se
k = φ(1991) · m + 3 para algum m ∈ N, temos

10k−3 ≡ 10φ(1991)·m ≡ (10φ(1991) )m ≡ 1m ≡ 1 (mod 1991).

Como existem infinitos números da forma φ(1991) · m + 3 (um para cada valor de m), concluı́mos
que existem infinitos k ∈ N para os quais 10k−3 ≡ 1 (mod 1991).
Dessa forma, existem infinitos números da forma 1999...91 que são divisı́veis por 1991.

5. O truque é utilizar que mdc(a, n) = mdc(n − a, n). Com isso, concluı́mos que mdc(a, n) = 1 ⇐⇒
mdc(n − a, n) = 1. Além disso, é claro também que 0 < a < n ⇐⇒ 0 < (n − a) < n. Daı́,
X X
a = (n − a),
0<a<n 0<a<n
mdc(a,n)=1 mdc(a,n)=1

5
pois em ambos os somatórios somamos cada inteiro entre 0 e n que é primo relativo com n exatamente
uma vez. Dessa forma,
X X X
2· a = a + (n − a)
0<a<n 0<a<n 0<a<n
mdc(a,n)=1 mdc(a,n)=1 mdc(a,n)=1
X
= [a + (n − a)]
0<a<n
mdc(a,n)=1
X
= n = nφ(n).
0<a<n
mdc(a,n)=1

Portanto,
X nφ(n)
a = .
2
0<a<n
mdc(a,n)=1

Esse resultado pode ser interpretado como uma demonstração de que a média dos φ(n) inteiros
entre 0 e n que são primos com n (ou seja, cujo mdc com n é igual a 1) é sempre igual a n2 . E isso
acontece porque, como vimos, a é um tal número se, e somente se, n − a também o é.

6. Como mdc(m, n) = 1, segue do Teorema de Euler que mφ(n) ≡ 1 (mod n). Além disso, nφ(m) ≡ 0
(mod n), já que φ(m) > 0. Assim,
mφ(n) + nφ(m) ≡ 1 (mod n),
ou seja,
n | mφ(n) + nφ(m) − 1.
Reciprocamente, vem do Teorema de Euler que nφ(m) ≡ 1 (mod m). Como φ(n) > 0, também
temos que mφ(n) ≡ 0 (mod m). Daı́,
mφ(n) + nφ(m) ≡ 1 (mod m),
ou seja,
m | mφ(n) + nφ(m) − 1.
Esses dois resultados, juntamente com o fato de que mdc(m, n) = 1, implicam
mn | mφ(n) + nφ(m) − 1,
ou seja,
mφ(n) + nφ(m) ≡ 1 (mod mn).

7. Como n + 1 é primo e n > 1, n deve ser par. Além disso, n deve ser múltiplo de 3. De fato:
• se n ≡ 1 (mod 3), n − 1 ≡ 0 (mod 3), o que é um absurdo, porque n > 4, e n − 1 é primo;
• se n ≡ 2 (mod 3), n + 1 ≡ 0 (mod 3), o que é um absurdo, porque n > 4, e n + 1 é primo.
Dessa forma, n = 2α · 3β · m, para alguns α ≥ 1, β ≥ 1 e m inteiros, com mdc(m, 6) = 1. Daı́,
2α · 3β · φ(m)
φ(n) = 2α−1 · 2 · 3β−1 · φ(m) = 2α · 3β−1 · φ(m) ≤ .
3
Como φ(m) ≤ m (a igualdade ocorre quando m = 1), segue que
2α · 3β · m n
φ(n) ≤ = .
3 3

6
8. Vamos contar a quantidade de números no conjunto {1, 2, . . . , n}. Por um lado, é claro que essa
quantidade é igual a n. Por outro lado...
n
Se a ∈ {1, 2, . . . , n} é um número qualquer, a = mdc(a, n) · k para algum inteiro 1 ≤ k ≤ mdc(a, n)
n

tal que mdc k, mdc(a, n) = 1. Por definição, mdc(a, n) é um divisor de n.
Reciprocamente, se escolhemos um divisor d de n e um inteiro 1 ≤ k ≤ nd tal que mdc(k, nd ) = 1,
o número d · k será um inteiro entre 1 e n cujo mdc com n é igual a d.
Portanto, a quantidade de números no conjunto {1, 2, . . . , n} é exatamente igual à quantidade de
pares (d, k) em que d é um divisor de n e k é um inteiro entre 1 e nd tal que mdc(k, nd ) = 1. Para cada
d, existem exatamente φ nd inteiros k entre 1 e nd tais que mdc(k, nd ) = 1. Dessa forma, a quantidade
de números no conjunto {1, 2, . . . , n} é igual a
X n
φ .
d
d|n

De onde saiu a fórmula do enunciado? Ora, no somatório acima, estamos somando nada mais,
nada menos, que o φ de todos os divisores de n. De fato, quando d varia entre os divisores de n, nd
também varia entre os divisores de n. Então,
X n X
φ = φ(d).
d
d|n d|n

Portanto, X
φ(d) = n.
d|n

2000 )
9. Como mdc(2, 52000 ) = 1, segue do Teorema de Euler que 2φ(5 ≡ 1 (mod 52000 ). Assim, existe
k ∈ N tal que
2000
2φ(5 ) = 52000 · k + 1.
Daı́,
2000 )
22000+φ(5 = 22000 · 52000 · k + 22000 = 102000 · k + 22000 .
Assim,
2000 )
22000+φ(5 ≡ 22000 (mod 102000 ),
2000
ou seja, os últimos 2000 algarismos de 22000+φ(5 ) coincidem com os últimos 2000 algarismos de
22000 . Mas
22000 < (23 )667 = 8667 < 10667 ,
ou seja, 22000 tem menos de 668 algarismos.
2000
Dessa forma, dentre os últimos 2000 algarismos de 22000+φ(5 ) há ao menos 1033 zeros consecu-
tivos.

10. Seja k um número natural qualquer.


Se k = 0, qualquer n serve.
Se k = 1, podemos tomar n = 1.
Suponhamos, então, k > 1.
Seja p o menor primo que não divide k.
Escrevamos
p − 1 = q1α1 · q2α2 · · · qm
αm
,

7
a fatoração em primos de p−1. Pela definição de p, cada um dos primos qi divide k. Podemos escrever,
então,
k = q1β1 · q2β2 · · · qm
βm
· k0 ,
para alguns β1 , β2 , . . . , βm , todos maiores que 0, e algum inteiro k 0 , que não é divisı́vel por nenhum
dos qi .
Seja n := (p − 1)k.
Nesse contexto, temos

φ(n + k) = φ(pk) = φ(p)φ(k) = (p − 1)φ(k),

já que mdc(p, k) = 1 (pois p é primo e não divide k), e

= φ q1α1 +β1 · q2α2 +β2 · · · qm


αm +βm
· k0
 
φ(n) = φ (p − 1)k

= (q1 − 1)q1α1 +β1 −1 · (q2 − 1)q2α2 +β2 −1 · · · (qm − 1)qm αm +βm −1


· φ(k 0 )

= q1α1 · q2α2 · · · qm
αm
(q1 − 1)q1β1 −1 · (q2 − 1)q2β2 −1 · · · (qm − 1)qmβm −1
· φ(k 0 )
 

= (p − 1)φ(k).

Logo, φ(n + k) = φ(n).

11. Para facilitar a escrita, chamemos os i-ésimo termo da sequência de ai .


Procederemos por indução.
Se n = 2, todos os termos da sequência são divisı́veis por n e, portanto, são congruentes ao mesmo
resto 0 módulo n.
Suponhamos que a afirmação seja válida para todo n < n0 , isto é, que para cada n < n0 , existe
um ı́ndice kn tal que ai ≡ aj (mod n) ∀ i, j ≥ kn . Vamos mostrar que existe kn0 tal que ai ≡ aj
(mod n0 ) ∀ i, j ≥ kn0 .
Se n0 é par, n0 = 2α · m com m ı́mpar, podemos tomar kn0 = max{α, km }. De fato, ai ≡ 0
(mod 2α ) ∀ i ≥ α, e ai ≡ aj (mod m) ∀ i, j ≥ km , de modo que ai ≡ aj (mod 2α · m) ∀ i, j ≥
max{α, km } (teorema Chinês dos Restos).
Suponhamos, então, que n0 seja ı́mpar. Pelo teorema de Euler, 2φ(n0 ) ≡ 1 (mod n0 ). Isso implica
que, se a ≡ b (mod φ(n0 )), então 2a ≡ 2b (mod n0 ). De fato, se a ≡ b (mod φ(n0 )), existe t ∈ Z tal
que a = φ(n0 ) · t + b e, então, 2a ≡ (2φ(n0 ) )t · 2b ≡ 2b (mod n0 ).
Pela hipótese de indução, existe k ∈ N tal que ai ≡ aj (mod φ(n0 )) ∀ i, j ≥ k. Daı́, se i, j ≥ k+1,

ai ≡ 2ai−1 ≡ 2aj−1 ≡ aj (mod n0 ).

Portanto, nesse caso, podemos fazer kn0 = k + 1.

12. Se m é um natural ı́mpar,


φ(2m) = φ(m).
Com vista nisso, mostraremos que √
φ(n) ≥ 2n
para todo n ı́mpar distinto de 3.
Se m e n são primos entre si e tais que
√ √
φ(m) ≥ 2m e φ(n) ≥ 2n,

8
então √ √
φ(mn) = φ(m)φ(n) ≥ 4mn > 2mn. (1)
Dessa forma, é suficiente que provemos o resultado para as potências de primos.
Comecemos pelas potências de 3: seja 3α uma potência de 3 com α > 1. Temos
√ √ √
φ(3α ) = 2 · 3α−1 = 6 · 3α−2 > 18 · 3α−2 ≥ 18 · 3α−2 = 2 · 3α .

Agora, seja pα uma potência de um primo ı́mpar distinto de 3 (desta vez, α pode ser qualquer
inteiro positivo, inclusive 1). Temos
p
φ(pα ) = (p − 1) · pα−1 > 2p · pα−1 ≥
p p
2p · pα−1 = 2pα ,

sendo a primeira desigualdade válida porque

p ≥ 5 ⇒ p2 > 4p − 1 ⇒ p2 − 2p + 1 > 2p ⇒ (p − 1)2 > 2p ⇒ (p − 1) > 2p.


p


Assim, φ(n) ≥ 2n sempre que n for distinto de 3 e igual a uma potência de um primo ı́mpar.
Daı́, por (1), √
φ(n) ≥ 2n para todo n ı́mpar maior que 3.
Com isso, já conseguimos provar a desigualdade do enunciado para os inteiros da forma 2α · m,
com m ı́mpar e maior que 3: temos
√ √ √
φ(2α · m) = 2α−1 · φ(m) ≥ 2α−1 2m ≥ 2α−1 · 2m = 2α · m.

Olhemos agora para as potências de 2: se 2α é uma potência de 2 com α > 1,


√ √ √
φ(2α ) = 2α−1 = 2 · 2α−2 = 4 · 2α−1 ≥ 4 · 2α−2 = 2α .

Restam somente os números da forma 2α · 3 com α > 1, e o número 3. Para o 3 é fácil:


√ √
φ(3) = 2 = 4 > 3.

Por fim, se 2α · 3 é tal que α > 1, então


√ √ √
φ(2α · 3) = 2α = 4 · 2α−2 > 12 · 2α−2 ≥ 12 · 2α−2 = 2α · 3.

13. Seja 1 ≤ i ≤ 1000.


• Se i é par, é claro que não existe inteiro j tal que i | 2j − 1.
• Se i é ı́mpar, o teorema de Euler nos garante que i | 2φ(i) −1. Como 1 ≤ i ≤ 1000, 1 ≤ φ(i) ≤ 1000.

Dessa forma, existe 1 ≤ j ≤ 1000 tal que i | 2j − 1 se, e somente se, i é ı́mpar.
Portanto, existem exatamente 500 inteiros 1 ≤ i ≤ 1000 para os quais existe um inteiro 1 ≤ j ≤
1000 tal que i | 2j − 1.

14. Queremos mostrar que existem infinitos números da forma 2 · 10k + 21 que são divisı́veis por 2021.
Em outras palavras, queremos mostrar que existem infinitos inteiros positivos k tais que

2 · 10k + 21 ≡ 0 (mod 2021).

9
Veja que

2 · 10k + 21 ≡ 0 (mod 2021) ⇐⇒ 2 · 10k + 21 ≡ 2021 (mod 2021)

⇐⇒ 2 · 10k ≡ 2000 (mod 2021)

⇐⇒ 10k ≡ 1000 (mod 2021)

⇐⇒ 10k−3 ≡ 1 (mod 2021),

sendo a terceira equivalência válida porque mdc(2, 2021) = 1, e a quarta, porque mdc(10, 2021) = 1.
Como mdc(10, 2021) = 1, o teorema de Euler garante que 10φ(2021) ≡ 1 (mod 2021). Daı́, se
k = φ(2021) · m + 3 para algum m ∈ N,

10k−3 ≡ 10φ(2021)·m ≡ (10φ(2021) )m ≡ 1m ≡ 1 (mod 2021).

Logo, para que tenhamos 10k−3 ≡ 1 (mod 2021), é suficiente que façamos k = φ(2021) · m + 3,
para algum m ∈ N. Como existem infinitos números dessa forma, existem infinitos k que respeitam a
condição que buscamos satisfazer.
Portanto, existem infinitos números da forma 2000...021 que são divisı́veis por 2021.

Material elaborado por Valentino Amadeus Sichinel

10
Problemas Resolvidos

Nı́vel 2

O Teorema Chinês dos Restos

Material elaborado por Valentino Amadeus Sichinel


Problemas

Problema 1. Encontre o menor inteiro posi- Problema 8. Demonstre que, se a, b e c são


tivo, com a exceção de x = 1, que satisfaz o três inteiros diferentes, existem infinitos inteiros
seguinte sistema de congruências: positivos n para os quais a + n, b + n e c + n são
primos relativos.
x ≡ 1 (mod 3)
x ≡ 1 (mod 5)
Problema 9. Demonstre que para todo inteiro
x ≡ 1 (mod 7)
positivo m e todo número par 2k, este último
pode ser escrito como a diferença de dois intei-
Problema 2. Encontre todas as soluções do sis- ros positivos, cada um dos quais é primo relativo
tema: com m.

3x ≡ 1 (mod 4)
Problema 10 (República Tcheca e Es-
2x ≡ 1 (mod 3) lováquia). Mostre que existe uma sequência
4x ≡ 5 (mod 7) crescente {an }∞
n=1 de números naturais tais que,
para cada k ≥ 0, a sequência {an +k}∞
n=1 contém
uma quantidade finita de números primos.
Problema 3. Sejam m0 , m1 , . . . , mr inteiros
positivos dois a dois primos entre si. Mos-
Problema 11. Considere um inteiro c ≥ 1 e
tre que existem r + 1 inteiros consecutivos s,
a sequência {an }∞
n=1 , definida por a1 = c e
s + 1, . . . , s + r tais que mi divide s + i para cada
ai+1 = cai . Mostre que essa sequência se torna
i ∈ {0, 1, . . . , r}.
eventualmente constante quando a reduzimos
módulo n para algum inteiro n > 1 (isso sig-
Problema 4. Sejam a e b inteiros positivos nifica que exite um j tal que am ≡ aj (mod n)
tais que mdc(a, b) = 1, e seja c > 0 um in- para todo m ≥ j).
teiro. Prove que existe um inteiro x tal que
mdc(a + bx, c) = 1.
Problema 12. Mostre que, para todo inteiro
positivo n, existem inteiros positivos a e b tais
Problema 5. É verdade que, para cada n, k ∈ que 4a2 + 9b2 − 1 é divisı́vel por n.
N, existem n inteiros consecutivos tais que cada
um é divisı́vel pela k-ésima potência de algum
primo? Problema 13 (EUA). Seja n um inteiro posi-
tivo qualquer. Prove que existem n inteiros po-
sitivos k1 , k2 , . . . , kn primos entre si, todos mai-
Problema 6. Um ponto (x, y) ∈ Z2 é legal se ores que 1, tais que k1 k2 · · · kn − 1 é o produto
mdc(x, y) = 1. Prove ou disprove: Dado um de dois inteiros consecutivos.
inteiro positivo n, existe um ponto (a, b) ∈ Z2
cuja distância a todo ponto legal é maior que n.
Problema 14. Um inteiro é livre de quadrados
se ele não é divisı́vel pelo quadrado de nenhum
Problema 7. Demonstre que, dados k e n na- número inteiro maior do que 1. Prove que exis-
turais, é possı́vel encontrar n inteiros positivos tem intervalos arbitrariamente grandes de intei-
consecutivos, cada um dos quais tem ao menos ros consecutivos, nenhum dos quais é livre de
k fatores primos distintos. quadrados.

2
Soluções

1. Como mdc(3, 5) = mdc(5, 7) = mdc(7, 3) = 1, o sistema apresentado admite exatamente uma


solução módulo 3 · 5 · 7 = 105. Como 1 é solução, concluı́mos que x é solução para o sistema se, e
somente se, x = 105k + 1 para algum k ∈ Z.
Dessa forma, o menor inteiro positivo, à exceção de 1, que satisfaz as três congruências dadas é
105 · 1 + 1 = 106.

2.
Temos 3 ≡ −1 (mod 4) , donde 32 ≡ (−1)2 ≡ 1 (mod 4).
Assim,
3x ≡ 1 (mod 4) ⇐⇒ x ≡ 3 (mod 4).
Da mesma forma,
2 ≡ −1 (mod 3) ⇒ 22 ≡ (−1)2 ≡ 1 (mod 3),
donde
2x ≡ 1 (mod 3) ⇐⇒ x ≡ 2 (mod 3).
Por fim, temos
4·2≡1 (mod 7)
e, portanto,
4x ≡ 5 (mod 7) ⇐⇒ x ≡ 2 · 5 ≡ 3 (mod 7).
Dessa forma, o que queremos é resolver o sistema

 x ≡ 3 (mod 4)

x ≡ 2 (mod 3) (1)

x ≡ 3 (mod 7)

Como mdc(4, 7) = 1, o sistema (


x ≡ 3 (mod 4)
x ≡ 3 (mod 7)

possui exatamente uma solução módulo 4 · 7 = 28. É claro que essa solução é 3. Desse modo, se x é
solução para o sistema original, então x = 28k + 3, para algum k ∈ Z. Como

28k + 3 ≡ 2 (mod 3) ⇐⇒ k ≡ 2 (mod 3),

x só é solução do sistema original se for da forma 28k + 3, para algum k da forma 3q + 2, isto é, se
x for da forma 28(3q + 2) + 3 = 84q + 59. Por outro lado, se x = 84q + 59 para algum q ∈ Z, uma
simples verificação mostra que x satisfaz as três congruências do sistema (1).
Concluı́mos, assim, que as soluções ao sistema apresentado no problema são exatamente os números
da forma 84q + 59, com q ∈ Z.

3
3. Devemos mostrar que existe um inteiros s tal que
s ≡ 0 (mod m0 )
s+1 ≡ 0 (mod m1 )
..
.
s+r ≡ 0 (mod mr )
Isso é o mesmo que mostrar que existe um inteiro s tal que
s ≡ 0 (mod m0 )
s ≡ −1 (mod m1 )
..
.
s ≡ −r (mod mr )
Como m0 , m1 , . . . , mr são dois a dois primos entre si, o teorema chinês dos restos garante a
existência de um tal inteiro s. Isso conclui, então, a solução.

4. Queremos um inteiro x tal que


a + bx 6≡ 0 (mod p) ∀ p primo tal que p | c.
Para os primos p tais que p | b, qualquer x basta: se p | b, a+bx ≡ a (mod p) e, como mdc(a, b) = 1,
se p | b, então p - a.
Sejam, então, p1 , p2 , . . . , pk os fatores primos de c que não dividem b. Queremos mostrar que existe
x ∈ Z tal que
a + bx 6≡ 0 (mod p1 )
a + bx ≡6 0 (mod p2 )
..
.
a + bx 6≡ 0 (mod pk )

É claro que é suficiente encontrarmos x tal que


a + bx ≡ 1 (mod p1 )
a + bx ≡ 1 (mod p2 )
..
. (1)
a + bx ≡ 1 (mod pk )
E essa tarefa pode ser facilmente resolvida com o teorema Chinês dos restos.
Se qi é o inverso de b módulo pi (isto é, b · qi ≡ 1 (mod pi )) para i = 1, 2, . . . , k, o sistema (1), que
é equivalente a
bx ≡ 1 − a (mod p1 )
bx ≡ 1 − a (mod p2 )
..
.
bx ≡ 1 − a (mod pk )

4
, é equivalente a

x ≡ q1 (1 − a) (mod p1 )
x ≡ q2 (1 − a) (mod p2 )
..
.
x ≡ qk (1 − a) (mod pk )

que, pelo teorema Chinês dos restos, tem solução.


Dessa forma, existe x ∈ Z tal que a + bx não é divisı́vel por nenhum primo que divide c, isto é, tal
que mdc(a + bx, c) = 1.

5. Sim. Vejamos por que.


Dados n, k naturais, o que queremos é encontrar m ∈ Z e primos p1 , p2 , . . . , pn tais que
k
pi | m + i ∀ i ∈ {1, 2, . . . , n}. Em outras palavras, queremos encontrar m e p1 , p2 , . . . , pk tais que

m ≡ −1 (mod pk1 )
m ≡ −2 (mod pk2 )
..
.
m ≡ −n (mod pkn )

Isso é evidentemente possı́vel, segundo o teorema Chinês dos restos, bastando que escolhamos
primos p1 , p2 , . . . , pk distintos entre si. De fato, se pi 6= pj e pi e pj são primos, então mdc(pi , pj ) = 1.

6. Vamos provar que existem, de fato, pontos arbitrariamente isolados dos pontos legais.
Seja, então, n ∈ N. Queremos a, b ∈ Z tais que mdc(p, q) 6= 1, sejam quais forem
p ∈ {a − n, . . . , a + n} e q ∈ {b − n, . . . , b + n}.
Para tanto, faremos o seguinte: para cada par (i, j) ∈ {−n, . . . , n}×{−n, . . . , n}, escolheremos um
primo pi,j , responsável por assegurar que a+i e b+j não serão em primos entre si. Em outras palavras,
escolhemos (2n + 1)2 primos pi,j distintos, um para cada par (i, j) ∈ {−n, . . . , n} × {−n, . . . , n} e,
então, encontramos a, b ∈ Z tais que pi,j | a + i e pi,j | b + j, para todos i, j.
Essa é uma simples tarefa para o teorema Chinês dos restos: uma vez escolhidos os primos pi,j ,
dois a dois distintos, tudo de que precisamos é ter

a ≡ −i (mod pi,j ) ∀ (i, j) ∈ {−n, . . . , n} × {−n, . . . , n}

e
b ≡ −j (mod pi,j ) ∀ (i, j) ∈ {−n, . . . , n} × {−n, . . . , n}.
E isso certamente é possı́vel, de acordo com o teorema.
Dessa forma, existem inteiros a, b tais que, se p ∈ {a − n, . . . , a + n} e q ∈ {b − n, . . . , b + n},
então mdc(p,pq) > 1. Para tais a e b,√temos que (x, y) ser legal implica |x − a| > n e |y − b| > n, o
que implica (x − a)2 + (y − b)2 > n 2 > n e, portanto, (a, b) está a uma distância maior que n de
qualquer ponto legal.

5
7. Sejam k e n números naturais quaisquer.
Queremos mostrar que existem números primos pi,j , um para cada par
(i, j) ∈ {1, . . . , n} × {1, . . . , k}, sendo pi,j1 distinto de pi,j2 para j1 6= j2 , e um número inteiro positivo
a, tais que a + i é divisı́vel por pi,j , para todos 1 ≤ i ≤ n e 1 ≤ j ≤ k.
Se fizermos isso, teremos n números inteiros positivos consecutivos, a + 1, a + 2, . . . , a + n, tais que
cada um dos quais possui ao menos k fatores primos distintos: a + i é divisı́vel por pi,1 , por pi,2 , ..., e
por pi,k .
Mais uma tarefa simples para o teorema Chinês dos restos.
Podemos escolher primos pi,j quaisquer (distintos dois a dois; um para cada par (i, j)), e tudo de
que precisamos é assegurar que
a+1 ≡ 0 (mod p1,j ) ∀ j ∈ {1, . . . , k}
a+2 ≡ 0 (mod p2,j ) ∀ j ∈ {1, . . . , k}
..
.
a+n ≡ 0 (mod pn,j ) ∀ j ∈ {1, . . . , k}
, isto é, que
a ≡ −i (mod pi,j ) ∀ (i, j) ∈ {1, . . . , n} × {1, . . . , k}.
O teorema Chinês dos restos nos garante que um tal número a existe, uma vez que escolhemos os
primos pi,j distintos dois a dois.

8. Sejam a, b e c inteiros positivos distintos. Para qualquer n ∈ N, mdc(a+n, b+n) = mdc(a+n, b−a).
Assim, para que tenhamos mdc(a + n, b + n) = 1, basta que tenhamos a + n 6≡ 0 (mod p) para todos
os primos p que dividem b − a. Em outras palavras, se p1 , p2 , . . . , pk são os fatores primos de b − a,
temos mdc(a + n, b + n) = 1 se, e somente se,
n 6≡ −a (mod p1 )
n ≡ 6 −a (mod p2 )
..
.
n 6≡ −a (mod pk )

De modo análogo, se q1 , q2 , . . . , ql são os fatores primos de c−b, e r1 , r2 , . . . , rt são os fatores primos


de a − c, então mdc(b + n, c + n) = 1 se, e somente se,
n 6≡ −b (mod q1 )
n ≡6 −b (mod q2 )
..
.
n 6≡ −b (mod ql )

, e mdc(c + n, a + n) = 1 se, e somente se,


n 6≡ −c (mod r1 )
n ≡ 6 −c (mod r2 )
..
.
n 6≡ −c (mod rt )

6
Por simplicidade, vamos chamar os primos que dividem b − a ou c − b ou a − c de s1 , s2 , . . . , sm .
Pelo teorema Chinês dos restos, é suficiente encontrarmos, para cada primo si , um resı́duo xi tal que

 −a se si | b − a

xi 6≡ −b se si | c − b (mod si ) (1)

−c se si | a − c

Parece simples, mas temos um empecilho: pode ser que si divida mais de uma das diferenças. E
se si for 2 ou 3, isso causaria um problema, já que só temos dois resı́duos possı́veis módulo 2, e três
módulo 3. Felizmente, se si divide duas das diferenças, então −a ≡ −b ≡ −c (mod si ). De fato, se
si | b − a, por exemplo, então b − a ≡ 0 (mod si ) ⇒ −a ≡ −b (mod si ) e, de modo análogo, uma
diferença envolvendo c é suficiente para concluir que −c ≡ −a (mod si ) ou −c ≡ −b (mod si ). Dessa
forma, podemos sempre escolher xi tal que (1) é satisfeito. Após efetuarmos tais escolhas, tudo de
que precisamos para que a + n, b + n e c + n sejam primos entre si é que
n ≡ x1 (mod s1 )
n ≡ x2 (mod s2 )
..
.
n ≡ xm (mod sm )

De acordo com o teorema Chinês dos restos, existe um resı́duo n0 módulo P = m


Q
i=1 si tal que
P K + n0 satisfaz tais condições, seja qual for K ∈ Z. Portanto, todo número n da forma n = P K + n0 ,
com K inteiro positivo, é um inteiro positivo tal que a + n, b + n e c + n são primos relativos e, dessa
forma, tais inteiros existem em quantidade infinita.

9. Primeiro, vamos mostrar que existem inteiros positivos a e b, primos relativos com m, tais que
a − b ≡ 2k (mod m).
Para tanto, escrevamos m = pα1 1 ·pα2 2 · · · pαl l , a decomposição em fatores primos de m, e consideremos
i ∈ {1, . . . , l} qualquer. Afirmo que existem xi e yi tais que xi 6≡ 0 (mod pαi i ), yi 6≡ 0 (mod pαi i ) e
xi − yi ≡ 2k (mod pαi i ). Vejamos por que.
Se 2k ≡ 0 (mod pαi i ), basta considerarmos xi = yi = 1.
Se 2k 6≡ 0 (mod pαi i ), pαi i > 2 (pois 2k ≡ 0 (mod 2)) e, assim, existem ao menos três resı́duos
distintos módulo pαi i . Em particular, existe um resı́duo que não é congruente a 0 nem a 2k. Se
chamarmos esse resı́duo de xi , podemos tomar yi = 2k − xi , e as condições serão satisfeitas.
Existem, portanto, para cada i ∈ {1, . . . , l}, xi e yi tais que xi 6≡ 0 (mod pαi i ), yi 6≡ 0 (mod pαi i ) e
xi − yi ≡ 2k (mod pαi i ). Pelo teorema chinês dos restos, existem a e b tais que
a ≡ x1 (mod pα1 1 )
a ≡ x2 (mod pα2 2 )
..
.
a ≡ xl (mod pαl l )
e
b ≡ y1 (mod pα1 1 )
b ≡ y2 (mod pα2 2 )
..
.
b ≡ yl (mod pαl l )

7
Em função das condições sob as quais encontramos os xi ’s e os yi ’s, tais inteiros a e b são primos
relativos com pα1 1 · pα2 2 · · · pαl l = m, e são tais que a − b ≡ 2k (mod pα1 1 · pα2 2 · · · pαl l = m).
Temos, então, inteiros a e b tais que mdc(a, m) = mdc(b, m) = 1 e a − b = mq + 2k para algum
q ∈ Z. Se fizermos b0 = b + mq, ficamos com inteiros a e b0 , primos relativos com m, cuja diferença é
igual a 2k.
Ainda não terminamos. Precisamos que os inteiros sejam positivos. Mas isso é simples: certamente,
a + m(|a| + |b|) > 0 e b0 + m(|a| + |b0 |) > 0. Esses dois novos números, a + m(|a| + |b|) e b0 + m(|a| + |b0 |),
são então inteiros positivos. Como cada um deles é um múltiplo de m somado a a ou a b0 , eles são
ainda primos relativos com m. Como o múltiplo que somamos em ambos é o mesmo, a diferença desses
dois novos números é igual a a − b0 , que é igual a 2k. Isso encerra a demonstração.

10. Sejam p0 , p1 , p2 , . . . os números primos (todos eles: 2, 3, 5, . . . ). Sim, começamos a indexação por
0. O motivo ficará claro em breve.
Em um certo sentido, o k-ésimo número primo será responsável por evitar que a sequência {an +
k}∞n=1 tenha infinitos primos.
Definiremos a sequência {an }∞ n=1 indutivamente.
Começamos por escolher a1 ∈ N tal que a1 + 0 ≡ 0 (mod p0 ).
Depois, escolhemos a2 ∈ N tal que a2 + 0 ≡ 0 (mod p0 ), a2 + 1 ≡ 0 (mod p1 ) e a2 > a1 . Veja que
isso pode ser feito: pelo teorema chinês dos restos, existe x tal que todo número da forma p0 p1 q + x
satisfaz as congruências; basta, então, escolher q grande o suficiente para que tenhamos p0 p1 q +x > a1 .
De uma forma geral, quando nos concentrarmos em escolher o n-ésimo termo da sequência, já
teremos escolhido a1 , a2 , . . . , an−1 , e poderemos, então, tomar um natural an tal que

an + 0 ≡ 0 (mod p0 )
an + 1 ≡ 0 (mod p1 )
..
.
an + n − 1 ≡ 0 (mod pn−1 )

e an > an−1 . De fato, pelo teorema chinês dos restos, existe x tal que todo número da forma
p0 p1 · · · pn−1 q + x satisfaz as congruências acima. Tomando q grande o suficiente, teremos
p0 p1 · · · pn−1 q + x > an−1 e, então, podemos fazer an = p0 p1 · · · pn−1 q + x.
Seguindo esse procedimento, definimos uma sequência crescente {an }∞ n=1 de números naturais tais
que, para todo i,

ai + 0 ≡ 0 (mod p0 )
ai + 1 ≡ 0 (mod p1 )
..
.
ai + i − 1 ≡ 0 (mod pi−1 )

Se assim o fizermos, para cada k ≥ 0, a sequência {an +k}∞ n=1 será tal que, a partir do (k +1)-ésimo
(a saber, ak+1 + k), todos os termos são divisı́veis por pk . Como a sequência é crescente, no máximo
um desses termos é igual a pk e, assim, todos os demais são compostos.
Dessa forma, para cada k ≥ 0, a sequência {an + k}∞ n=1 contém no máximo k + 1 números primos.

11. Se c = 1, a sequência toda é constante (igual a 1). Suponhamos, então, que c > 1.
Provaremos por indução que, para cada n ∈ N maior que 1, existe um ı́ndice j a partir do qual a
sequência {ai }∞
i=1 é constante módulo n.
A base (n = 2) é simples: se c é ı́mpar, ai ≡ 1 (mod 2) ∀i e, se c é par, ai ≡ 0 (mod 2) ∀i.

8
Suponhamos, então, que para todo n0 < n existe um ı́ndice j 0 a partir do qual {ai }∞ i=1 é constante
0 0 0
módulo n , isto é, tal que am ≡ aj 0 (mod n ) para todo m ≥ j . Essa é a hipótese de indução.
Vamos, agora, ao passo indutivo, isto é, vamos mostrar que o resultado é válido para n também.
Se n = pα1 1 · pα2 2 · · · pαk k , a fatoração em primos de n, for tal que k ≥ 2, não há muito que fazer.
De fato, nesse caso, pαi i < n para todo i ∈ {1, . . . , k} e assim, pela hipótese de indução, existe, para
cada i ∈ {1, . . . , k}, um ji tal que am ≡ aji (mod pαi i ) ∀m ≥ ji . Se fizermos j := max(j1 , j2 , . . . , jk ),
teremos

am ≡ aj (mod pα1 1 )
am ≡ aj (mod pα2 2 )
..
.
am ≡ aj (mod pαk k )

para todo m ≥ j, e o teorema chinês dos restos nos assegura que, nesse caso,

am ≡ aj (mod n)

(para todo m ≥ j).


Dessa forma, é suficiente que consideremos o caso em que n é uma potência de primo. Suporemos,
então, que n = pα , para algum primo p.
Como c > 1, {ai }∞ i=1 é crescente. Logo, existe j tal que ai > α para todo i ≥ j. Como ai+1 = c ,
ai

segue daı́ que, se p | c, então pα | ai+1 para todo i ≥ j. Assim, nesse caso, ai+1 ≡ 0 (mod pα ) para
todo i ≥ j. Resta considerarmos, então, o caso em que p - c.
Observe que φ(pα ) < pα . Assim, pela hipótese de indução, existe j 0 tal que am ≡ aj 0 (mod φ(pα ))
para todo m ≥ j 0 . Se p - c, mdc(c, pα ) = 1 e, assim, vem daı́ que cam −aj 0 ≡ 1 (mod pα ) para todo
m ≥ j 0 . Em outras palavras, cam ≡ caj 0 (mod pα ) para todo m ≥ j 0 , ou seja, am+1 ≡ aj 0 +1 (mod pα )
para todo m ≥ j 0 . Fazendo j = j 0 + 1, concluı́mos que, nesse caso, am ≡ aj (mod n = pα ) para todo
m ≥ j.
Isso conclui o passo indutivo, e o princı́pio de indução finita conclui nossa demonstração.

12. Escrevamos n = pα1 1 · pα2 2 · · · pαk k , a fatoração em primos de n.


Se, para cada i ∈ {1, . . . , k}, encontrarmos ai e bi tais que 4a2i + 9b2i − 1 é divisı́vel por pαi i , será
suficiente que encontremos a e b tais que

a ≡ a1 (mod pα1 1 )
a ≡ a2 (mod pα2 2 )
..
.
a ≡ ak (mod pαk k )

b ≡ b1 (mod pα1 1 )
b ≡ b2 (mod pα2 2 )
..
.
b ≡ bk (mod pαk k )
α
Mas isso é simples: como mdc(pαi i , pj j ) = 1 para quaisquer i 6= j, o teorema chinês dos restos nos
garante a existência de tais a e b.

9
Vamos, então, considerar uma potência de primo pα qualquer, e mostrar que existem a e b tais que
pα divide 4a2 + 9b2 − 1. Dividiremos a solução em dois casos:
• Se p = 2, basta tomarmos a ≡ 0 (mod 2α ) e b ≡ 3−1 (mod 2α ) (3−1 é o inverso de 3 módulo 2α ).
• Se p 6= 2, basta tomarmos a ≡ 2−1 (mod pα ) e b ≡ 0 (mod pα ) (2−1 é o inverso de 2 módulo pα ).

13. k1 k2 · · · kn − 1 ser o produto de dois inteiros consecutivos equivale à existência de um inteiro a tal
que
k1 k2 · · · kn − 1 = a(a + 1),
isto é, tal que
a(a + 1) + 1 = k1 k2 · · · kn .
Dessa forma, a existência de inteiros positivos k1 , k2 , . . . , kn primos entre si, todos maiores que 1,
tais que k1 k2 · · · kn − 1 é o produto de dois inteiros consecutivos equivale à existência de um inteiro a
tal que a(a + 1) + 1 possui ao menos n fatores primos distintos.
Assim, se encontrarmos n primos distintos p1 , p2 , . . . , pn e n inteiros a1 , a2 , . . . , an tais que
ai (ai + 1) + 1 ≡ 0 (mod pi ) ∀ i ∈ {1, . . . , n},
teremos resolvido o problema. De fato, pelo teorema chinês dos restos, existe, nesse caso, um inteiro
a tal que
a ≡ a1 (mod p1 )
a ≡ a2 (mod p2 )
..
.
a ≡ an (mod pn )
e, portanto, tal que
a(a + 1) + 1 ≡ 0 (mod pi ) ∀ i ∈ {1, . . . , n}.
É suficiente que mostremos, então, que a quantidade de números primos que divide algum número
da forma a(a + 1) + 1 é pelo menos n. Vamos mostrar que, de fato, existem infinitos números primos
que dividem algum número da forma a(a + 1) + 1.
Suponhamos, por absurdo, que existe apenas uma quantidade finita de tais números primos, e
chamemos esses números primos de p1 , p2 , . . . , pk . Se P := p1 p2 · · · pk ,
P (P + 1) + 1 ≡ 1 6≡ 0 (mod pi ) ∀ i ∈ {1, . . . , k}.
Mas P (P + 1) + 1 não é igual a 1, já que P (P + 1) 6= 0 e, portanto, deve ser divisı́vel por algum
número primo. Absurdo!
Dessa forma, há de haver infinitos primos que dividem algum número da forma a(a + 1) + 1. Isso
completa a solução.

14. Devemos mostrar que, dado um inteiro positivo n qualquer, existem n inteiros consecutivos
a + 1, a + 2, . . . , a + n, nenhum dos quais é livre de quadrados.
Para garantirmos que um número não é livre de quadrados, basta que garantamos que ele é
divisı́vel pelo quadrado de algum número primo. Dessa forma, para atingirmos nosso objetivo, basta
que encontremos um número inteiro a e n números primos p1 , p2 , . . . , pn tais que
a+1 ≡ 0 (mod p21 )
a+2 ≡ 0 (mod p22 )
..
.
a+n ≡ 0 (mod p2n )

10
isto é, tais que

a ≡ −1 (mod p21 )
a ≡ −2 (mod p22 )
..
.
a ≡ −n (mod p2n )

Isso é garantidamente possı́vel, de acordo com o teorema chinês dos restos, bastando para tanto
que escolhamos primos p1 , p2 , . . . , pn distintos entre si.
Dessa forma, para cada inteiro n, existe um intervalo de ao menos n inteiros consecutivos, nenhum
dos quais é livre de quadrados.

Material elaborado por Valentino Amadeus Sichinel

11
Problemas Resolvidos

Nı́vel 2

Equações diofantinas I

Material elaborado por Valentino Amadeus Sichinel


Problemas

Problema 1. Encontre todas as soluções inteiras de 999x − 49y = 5000.

Problema 2. Encontre todos os inteiros x e y tais que 147x + 258y = 369.

Problema 3. Encontre todas as soluções inteiras de 2x + 3y + 4z = 5.

Problema 4. Encontre todas as soluções inteiras do sistema de equações:


(
x + 2y + 3z = 4
.
x + 3y + 4z = 5

Problema 5. Encontre uma condição necessária e suficiente sobre os coeficientes b1 , b2 , c1 , c2 , d1 e


d2 para que o sistema de equações (
x + b1 y + c1 z = d1
x + b2 y + c2 z = d2
tenha ao menos uma solução (x, y, z) nos inteiros.

Problema 6. Sejam a e b inteiros positivos tais que a2 + b2 é divisı́vel por ab. Prove que a = b.

Problema 7 (EUA). Encontre todos os inteiros positivos n para os quais a equação 2x + 2y + z = n


tem exatamente 28 soluções (x, y, z), nos inteiros positivos.

Problema 8. Um inteiro positivo n é dito potencial se, para todo primo p que divide n, p2 divide n.
Mostre que todo inteiro positivo potencial pode ser escrito na forma a2 b3 , para alguns inteiros a e b.

2
Soluções

1. Resolver a equação
999x − 49y = 5000 (1)
é o mesmo que resolver a equação
999x + 49y 0 = 5000, (2)
se fizermos y 0 = −y.
Veja que mdc(999, 49) = 1. De fato, 49 = 72 , e 7 - 999. Pelo teorema 10, então, as soluções de (2)
são da forma
(x0 + 49k, y00 − 999k), k ∈ Z,
sendo (x0 , y00 ) uma solução qualquer da equação. Tudo que precisamos, então, é encontrar uma solução
para a equação.
Veja que 999 = 1000 − 1 = 50 × 20 − 1 = 49 × 20 + 19. Assim,

999x + 49y 0 = 5000 ⇐⇒ (49 × 20 + 19)x + 49y 0 = 5000 ⇐⇒ 19x + 49(20x + y 0 ) = 5000.

Fazendo y 00 := 20x + y 0 , segue que é suficiente que encontremos uma solução para a equação

19x + 49y 00 = 5000.

Observe que 49 = 2 × 19 + 11. Dessa forma,

19x + 49y 00 = 5000 ⇐⇒ 19x + (2 × 19 + 11)y 00 = 5000 ⇐⇒ 19(x + 2y 00 ) + 11y 00 = 5000.

Portanto, fazendo x0 := x + 2y 00 , podemos nos ater a encontrar uma solução à equação

19x0 + 11y 00 = 5000.

Agora, os coeficientes estão pequenos o suficiente para que partamos para uma abordagem mais
objetiva. Devemos ter
19x0 ≡ 5000 (mod 11). (3)
Veja que 19 ≡ 8 (mod 11). Como 7 × 8 = 56 ≡ 1 (mod 11), segue que 7 × 19 ≡ 1 (mod 11). Daı́,

19x0 ≡ 5000 (mod 11) ⇐⇒ 7 × 19x0 ≡ 7 × 5000 (mod 11) ⇐⇒ x0 ≡ 35000 (mod 11).

Como 11 | 33000, 3500 ≡ 2000 (mod 11).


Como 11 | 1100, 2000 ≡ 900 (mod 11).
Como 11 | 880, 900 ≡ 20 (mod 11).
Por fim, 20 ≡ 9 (mod 11).
Dessa forma, devemos ter x0 ≡ 9 (mod 11). Podemos tentar x0 = 9 em (3). Ficamos com

19 × 9 + 11y 00 = 5000 ⇐⇒ 11y 00 = 5000 − 19 × 9


⇐⇒ 11y 00 = 5000 − 20 × 9 + 9
⇐⇒ 11y 00 = 5000 − 180 + 9
⇐⇒ 11y 00 = 4829
⇐⇒ y 00 = 439.

Lembremos que x0 = x + 2y 00 . Daı́. x = x0 − 2y 00 = 9 − 878 = −869.

3
Como y 00 = 20x + y 0 , segue que y 0 = y 00 − 20x = 439 + 20 × 869 = 17.819. Portanto, as soluções de
(2) são os pares da forma
(−869 + 49k, 17819 − 999k), k ∈ Z
e, como y 0 = −y, as soluções de (1) são, então, os pares da forma

(−869 + 49k, −17819 + 999k), k ∈ Z.

2. Temos

mdc(147, 258) = mdc(147, 258 − 147) = mdc(147, 111)


= mdc(111, 147 − 111) = mdc(111, 36)
= 3,

já que 36 = 22 · 32 , 2 - 111, 3 | 111, e 32 - 111.


147
Como 3 | 369, 3 = 49, e 2583 = 86, segue do teorema 10 que as soluções da equação

147x + 258y = 369 (1)

são os pares da forma


(x0 + 86k, y0 − 49k), k ∈ Z,
sendo (x0 , y0 ) uma solução da equação. Tudo que precisamos, então, é encontrar uma solução para
(1).
Observe que 258 − 147 = 369 − 258 = 111. Essa simples observação nos permite reescrever a
equação de uma maneira luminosa:

147x + 258y = 369 ⇐⇒ 147x + (147 + 111)y = 147 + 111 + 111


⇐⇒ 147(x + y) + 111y = 147 + 2 × 111.

Podemos fazer, então, x + y = 1 e y = 2, isto é, (x, y) = (−1, 2). Dessa forma, as soluções da
equação dada no enunciado são os pares da forma

(−1 + 86k, 2 − 49k), k ∈ Z.

3. Sabemos encontrar soluções para equações diofantinas com duas variáveis. Utilizemos isso a nosso
favor: para cada valor de z, queremos encontrar x e y tais que

2x + 3y = 5 − 4z. (1)

Como mdc(2, 3) = 1, as soluções dessa equação são da forma

(x0 + 3k, y0 − 2k), k ∈ Z,

e tudo de que precisamos é encontrar uma solução inicial (x0 , y0 ).


Uma estratégia é procurar por pares da forma (ax + bx z, ay + by z). Nesse caso, seria suficiente que
tivéssemos 2ax + 3ay = 5 e 2bx + 3by = −4. Uma solução para a primeira equação é (1, 1) e, para na
segunda, podemos tomar (bx , by ) = (1, −2). Esses pares nos dão a solução (1 + z, 1 − 2z) para (1).
Veja que (1 + z, 1 − 2z, z) é solução da equação do enunciado, seja qual for o valor de z. Portanto, as
soluções da equação do problema são as triplas da forma

(1 + z + 3k, 1 − 2z − 2k, z), k, z ∈ Z.

4
4. Subtraindo a primeira da segunda equação, ficamos com

y + z = 1. (1)

Uma solução para essa equação é (y, z) = (1, 0). Assim, pelo teorema 10, as soluções de (1) são
os pares
(1 + k, −k), k ∈ Z.
Substituindo na primeira equação do sistema dado, ficamos com

x + 2 + 2k − 3k = 4 ⇒ x = 2 + k.

Como y e z foram escolhidos de tal forma que a equação que se obtém quando se toma a diferença
entre as equações seja satisfeita, x = 2 + k satisfaz (juntamente com y = 1 + k e z = −k) também a
segunda equação do sistema (verifique!). Portanto, as soluções do sistema apresentado são as triplas

(2 + k, 1 + k, −k), k ∈ Z.

5. Se (x, y, z) é uma solução do sistema,

(b1 − b2 )y + (c1 − c2 )z = d1 − d2 . (1)

Pelo teorema 10, para que isso aconteça, é necessário que tenhamos mdc(b1 − b2 , c1 − c2 ) | (d1 − d2 ).
Por outro lado, se mdc(b1 − b2 , c1 − c2 ) | (d1 − d2 ), o teorema 10 garante que existem y0 e z0 inteiros
tais que
(b1 − b2 )y + (c1 − c2 )z = d1 − d2 .
Se fizermos x0 := d1 − b1 y0 − c1 z0 , a primeira equação do sistema é automaticamente satisfeita. Como
(y0 , z0 ) é solução de (1), a segunda equação também o é:

x0 + b2 y0 + c2 z0 = x0 + b1 y0 + c1 z0 − (b1 − b2 )y0 + (c1 − c2 )z0 ]
= d1 − (d1 − d2 )
= d2 .

Dessa forma, uma condição necessária e suficiente para que o sistema apresentado tenha solução
nos inteiros positivos é a de que

mdc(b1 − b2 , c1 − c2 ) | (d1 − d2 ).

6. Suponhamos, por absurdo, que a 6= b. Existe, então, um primo p tal que a maior potência de p que
divide um dos dois inteiros é (estritamente) maior que a potência de p que divide o outro. Sem perder
generalidade, suponhamos que a maior potência de p que divide a, α, é maior que a maior potência de
p que divide b, β. Em outras palavras, p é um primo, α e β são inteiros com α > β, e existem inteiros
k1 e k2 , que não são divisı́veis por p, tais que a = pα k1 e b = pβ k2 . Nessas condições,

a2 + b2 = p2α k12 + p2β k22 = p2β p2α−2β k12 + k22 ab = pα+β k1 k2 .



e

Da primeira igualdade, vem que a maior potência de p que divide a2 + b2 é p2β . Da segunda, que a
maior potência de p que divide ab é pα+β . Mas 2β < α + β, e a maior potência de p que divide a2 + b2
deveria ser maior que a maior potência de p que divide ab, já que a2 + b2 é múltiplo de ab. Absurdo!
Dessa forma, nossa hipótese é insustentável e, portanto, havemos de ter a = b.

5
7. Antes de mais nada, observemos o seguinte: se t é um inteiro positivo, a equação

x+y =t

tem exatamente t − 1 soluções nos inteiros positivos. De fato, se (x, y) é uma solução, temos, por um
lado, x ≥ 1, porque x é positivo e, por outro, x = t − y ≤ t − 1, porque y é positivo. Assim, x deve
pertencer ao conjunto {1, 2, . . . , t − 1}. E, para cada valor de x em {1, 2, . . . , t − 1}, há exatamente
um valor inteiro positivo para y tal que x + y = t (a saber, y = t − x).
Nos encaminhemos, agora, à equação do problema. Queremos encontrar triplas (x, y, z) inteiras
e positivas tais que
n−z
x+y = . (1)
2
A primeira observação que fazemos é a de que z deve ter a mesma paridade que n. Além disso,
pelo que vimos acima, para cada valor de z com a mesma paridade de n e tal que n−z 2 é positivo,
existem exatamente n−z 2 − 1 soluções para (1).
Como z é positivo, os valores positivos que n−z 2 pode atingir quando z varia são exatamente
1, 2, . . . , d n2 e − 1. Lembre-se de que d n2 e = n+1
2 , se n é ı́mpar, e d n2 e = n2 , se n é par.
Dessa forma, para um n fixo, o número de soluções da equação apresentada no enunciado é
 n   n 
(1 − 1) + (2 − 1) + (3 − 1) + · · · + d e − 1 − 1 = 0 + 1 + 2 + · · · + d e − 1 − 1
2 2
  
d n2 e − 1 − 1 d n2 e − 1
=
2
k(k + 1)
= ,
2
onde k := d n2 e − 2.
Queremos os valores de n para os quais k(k+1)
2 = 28, isto é, para os quais k = 7.
Se n é ı́mpar, lnm n+1
k = 7 ⇐⇒ − 2 = 7 ⇐⇒ = 9 ⇐⇒ n = 17.
2 2
Se n é par, lnm n
k = 7 ⇐⇒ − 2 = 7 ⇐⇒ = 9 ⇐⇒ n = 18.
2 2
Portanto, a equação 2x + 2y + z = n tem exatamente 28 soluções inteiras positivas (x, y, z) se, e
somente se, n = 17 ou n = 18.

8. Se n = 1, não há nada que fazer: 1 = 12 ·13 . Consideremos, então, os inteiros potenciais estritamente
maiores que 1.
Comecemos por analisar o caso em que o inteiro potencial é uma potência de primo. Seja pα um
inteiro potencial que é potência do primo p. Por pα ser potencial, α ≥ 2. De fato, α > 0, já que
pα > 1, donde p | pα . Daı́, p2 | pα e, portanto, α ≥ 2.
α 2
Se α é par, pα = p 2 · 13 .
α−3 2
Se α é ı́mpar, α ≥ 3, donde α − 3 é par e não-negativo. Podemos, então, escrever pα = p 2 · p3 .
Portanto, toda potência de primo com expoente maior que 1 (isto é, toda potência de primo que
é inteiro potencial) pode ser escrita na forma a2 b3 .
Seja, agora n > 1 um inteiro potencial qualquer. Seja

n = pα1 1 · pα2 2 · · · pαk k

6
a fatoração em primos de n. Para cada i = 1, 2, . . . , k, pi | n, donde p2i | n. Daı́, αi ≥ 2 ∀ i = 1, 2, . . . , k.
Logo, para cada i = 1, 2, . . . , k, existem inteiros positivos ai e bi tais que pαi i = a2i b3i . Dessa forma,
2 3
n = a21 b31 · a22 b32 · · · a2k b3k = a1 a2 · · · ak b1 b2 · · · bk .

Da arbitrariedade de n, segue que todo inteiro positivo potencial pode ser escrito na forma a2 b3 ,
para alguns inteiros positivos a e b.

Observe que mostrar que pα pode ser escrito na forma a2 b3 é o mesmo que mostrar que existem
inteiros não-negativos x e y tais que 2x + 3y = α.

Material elaborado por Valentino Amadeus Sichinel

7
Problemas Resolvidos

Nı́vel 2

Equações diofantinas II

Material elaborado por Valentino Amadeus Sichinel


Problemas

Problema 1. Mostre que se a, b e x são inteiros positivos tais que a · b = x2 e mdc(a, b) = 1, então
existem inteiros r e s tais que a = r2 e b = s2 .

Problema 2. Prove que todas as soluções positivas da equação


1 1 1
2
+ 2 = 2
x y z
com mdc(x, y, z) = 1 são ou da forma
 
(x, y, z) = r4 − s4 , 2rs(r2 + s2 ), rs(r2 − s2 ) ,

ou da forma  
(x, y, z) = 2rs(r2 + s2 ), r4 − s4 , rs(r2 − s2 ) ,

onde r > s > 0, mdc(r, s) = 1 e r e s têm paridades distintas.

Problema 3. Encontre todos os pares de racionais positivos (x, y) tais que x2 + y 2 = 1.

Problema 4. Encontre todas as quádruplas (a, b, c, d) de inteiros positivos tais que


(
a2 + b2 = c2
.
a2 + c2 = d2

Problema 5 (Torneio das Cidades). Prove que a equação

x2 + y 2 − z 2 = 1997

tem infinitas soluções inteiras (x, y, z).

Problema 6. Encontre todas as soluções inteiras de

5m2 + n2 = 52020 .

Problema 7. Encontre todas as triplas (a, b, c) de inteiros positivos tais que

a2 + b2 = 2c2 .

2
Soluções

1. Sejam
a = pα1 1 pα2 2 · · · pαk k , b = pβ1 1 pβ2 2 · · · pβk k e x = pγ11 pγ22 · · · pγkk
as fatorações em primos de a, b e x, respectivamente.
Temos
x2 = p2γ1 2γ2
1 p2 · · · pk
2γk
e a · b = pα1 1 +β1 pα2 2 +β2 · · · pαk k +βk .
Logo,
αi + βi = 2γi ∀ i ∈ {1, 2, . . . , k}. (1)
Observe, no entanto, que se, para algum i, αi > 0 e βi > 0, tanto a quanto b teriam pi como fator
primo. Daı́, nesse caso, mdc(a, b) não seria igual a 1. Assim, para todo i, ou αi = 0, ou βi = 0. Por
(1), isso é o mesmo que

αi = 2γi ou βi = 2γi ∀ i ∈ {1, 2, . . . , k}.

Podemos reescrever esse fenômeno da seguinte maneira: para todo i, se pi | a, então αi = 2γi e
pi - b; por outro lado, se pi | b, então βi = 2γi e pi - a. Dessa maneira, podemos reescrever as fatorações
em primos de a e b da seguinte forma:
Y 2γ Y 2γ
a= pi i e b= pi i .
i; pi |a i; pi |b

Isso nos permite escrever


 Y 2  Y 2
a= pγi i e b= pγi i .
i; pi |a i; pi |b

Portanto, a = r2 e b = s2 , onde

pγi i pγi i .
Y Y
r= e s=
i; pi |a i; pi |b

2. Sejam x, y e z inteiros positivos tais que mdc(x, y, z) = 1 e


1 1 1
+ = 2.
x2 y 2 z
Temos  xy 2
x2 + y 2 = .
z
Sendo um racional cujo quadrado é inteiro, xy 1 xy
z deve ser inteiro . Logo, x, y e z formam uma terna
pitagórica. Daı́, existem r > s > 0 inteiros positivos primos entre si e de paridades distintas e d > 0
um inteiro positivo tais que
xy  
(x, y, ) = d(r2 − s2 ), d · 2rs, d(r2 + s2 ) (1)
z
1
Prove isto: que, se q ∈ Q e q 2 ∈ Z, então q ∈ Z.

3
ou
xy  
) = d · 2rs, d(r2 − s2 ), d(r2 + s2 ) .
(x, y, (2)
z
Consideremos o caso (1). Temos, nesse cenário,
xy
= d(r2 + s2 ) ⇐⇒ xy = d(r2 + s2 )z
z
⇐⇒ d(r2 − s2 ) · d2rs = d(r2 + s2 )z
⇐⇒ d · 2rs(r2 − s2 ) = (r2 + s2 )z.

Afirmo que mdc(2rs(r2 − s2 ), r2 + s2 ) = 1. De fato,


• mdc(2, r2 + s2 ) = 1, pois r e s têm paridades distintas;
• mdc(r, r2 + s2 ) = 1, pois se p é primo, p | r e p | (r2 + s2 ), então p | r2 e p | (r2 + s2 ), donde
p| s2 e, daı́, p | s. Assim, p | r e p | s, o que é um absurdo, pois mdc(r, s) = 1;
• mdc(s, r2 + s2 ) = 1, pois se p é primo, p | s e p | (r2 + s2 ), então p | s2 e p | (r2 + s2 ), donde
p| r2 e, daı́, p | r. Assim, p | s e p | r, o que é um absurdo, pois mdc(r, s) = 1;
•mdc(r2 − s2 , r2 + s2 ) = 1, já que, se p | (r2 − s2 ) e p | (r2 + s2 ), então p | (r2 − s2 ) + (r2 + s2 ) = 2r2
e p | (r2 + s2 ), o que é um absurdo pois, como vimos, mdc(2, r2 + s2 ) = mdc(r, r2 + s2 ) = 1, o que
implica mdc(2r2 , r2 + s2 ) = 1.

Dessa forma, como d·2rs(r2 −s2 ) = (r2 +s2 )z, todos os fatores primos de (r2 +s2 ) devem pertencer
a d. Por outro lado, se d tivesse algum dos fatores primos de z, esse fator primo seria comum a x, y
e z, já que tanto x quanto y são divisı́veis por d. Mas isso é absurdo, pois mdc(x, y, z) = 1. Logo, d
contém todos os fatores primos de (r2 + s2 ), e nada além deles. Em outras palavras, d = r2 + s2 . Daı́,

d · 2rs(r2 − s2 ) = (r2 + s2 )z ⇒ (r2 + s2 ) · 2rs(r2 − s2 ) = (r2 + s2 )z ⇒ 2rs(r2 − s2 ) = z.

Assim, temos
x = d(r2 − s2 ) = (r2 + s2 )(r2 − s2 ) = r4 − s4 ,
y = d · 2rs = 2rs(r2 + s2 ),
z = 2rs(r2 − s2 ),
onde r e s são inteiros positivos primos entre si e de paridades distintas, tais que r > s > 0.
O caso (2) é totalmente análogo; mudando apenas os papeis de x e de y. Portanto, temos ou
 
(x, y, z) = r4 − s4 , 2rs(r2 + s2 ), rs(r2 − s2 ) ,

ou  
(x, y, z) = 2rs(r2 + s2 ), r4 − s4 , rs(r2 − s2 ) ,

onde r > s > 0, mdc(r, s) = 1 e r e s têm paridades distintas.

3. Sejam x e y racionais positivos tais que

x2 + y 2 = 1.

Escrevamos
a p
x= e y= ,
b q

4
com
mdc(a, b) = mdc(p, q) = 1.
Observe que
 a 2  p 2  bp 2
+ =1 ⇒ a2 + = b2 .
b q q
2
Daı́, bpq é inteiro. Sendo racional cujo quadrado é inteiro, bp
q é inteiro1 . Logo, q | bp. Como
mdc(p, q) = 1, segue que q | b.
Por outro lado,
 a 2  p 2  aq 2
+ =1 ⇒ + p2 = q 2
b q b
e, por um raciocı́nio totalmente análogo, concluı́mos que b | q.
Como b e q são inteiros positivos tais que q | b e b | q, b = q. Para não haver confusão, denotaremos,
daqui para frente, b e q por m.
O que queremos, então, é encontrar inteiros positivos a, p e m tais que mdc(a, m) = 1, mdc(p, m) =
1e  a 2  q 2
+ = 1, isto é, a2 + q 2 = m2 .
m m
Em outras palavras, queremos que (a, q, m) seja uma terna pitagórica primitiva. Sabemos que
isso acontece se, e somente se, existem inteiros positivos r > s > 0, primos entre si e de paridades
distintas, tais que ou
(a, q, m) = (r2 − s2 , 2rs, r2 + s2 ),
ou
(a, q, m) = (2rs, r2 − s2 , r2 + s2 ).
Portanto, os racionais positivos x e y são tais que x2 + y 2 = 1 se, e somente se,
 r2 − s2 2rs 
(x, y) = ,
r2 + s2 r2 + s2
ou  2rs r2 − s2 
,
(x, y) = ,
r2 + s2 r2 + s2
onde r e s são inteiros positivos de paridades distintas tais que r > s > 0 e mdc(r, s) = 1.

4. Dentre todas as quádruplas que satisfazem as equações do enunciado, consideremos uma cujo
valor de d é o menor possı́vel. Afirmamos que tanto (a, b, c) quanto (a, c, d) são ternas pitagóricas
primitivas. Para ver isso, como a e c são elementos de ambas as ternas, é suficiente que mostremos
que mdc(a, c) = 1. E isto, de fato, ocorre: se p é um primo talque p | a e p | c, vem da primeira
equação do enunciado que p | b e, da segunda, que p | d. Assim, ap , pb , pc , e dp são todos inteiros.
Dividindo cada uma das equações por p2 , ficamos com
  2  2  2
 ap + pb = pc

 2  2  2 ,
 a + c = d

p p p
   c  d 
a b
o que é um absurdo, pois o último elemento da quádrupla p , p , p , p é menor que d, que
supomos ser mı́nimo. Dessa forma, (a, b, c, d), que é uma quádrupla que estamos supondo satisfazer
1
Prove isto: que, se um número racional é tal que seu quadrado é inteiro, então o número é inteiro.

5
as condições do enunciado e ter o valor de d mı́nimo, é tal que o mdc entre quaisquer dois de seus
elementos é igual a 1.
Isolando c2 nas duas equações do enunciado e somando, ficamos com

(d + b)2 (d − b)2
2c2 = d2 + b2 = + . (1)
2 2
Veja que, como as ternas são primitivas, c e d são ı́mpares. Como c é ı́mpar, vem da segunda
equação que a é par. Daı́, pela primeira equação, b também é ı́mpar. Usaremos mais tarde que a é
par. Por hora, precisamos somente do fato de que b e d têm a mesma paridade. Isso implica que b+d 2
e b−d
2 são ambos inteiros. (1) pode ser reescrita como
 2  2
d+b d−b
c2 = + .
2 2
 
d+b d−b d+b d−b
   
A terna pitagórica 2 , 2 , c é primitiva. De fato, se p divide tanto 2 quanto 2 ,
p divide
d + b d − b
+ =d
2 2
e d + b d − b
− = b,
2 2
e mdc(b, d) = 1. Assim, existem r e s inteiros positivos primos entre si e de paridades distintas tais
que  
d + b d − b
, , c = (r2 − s2 , 2rs, r2 + s2 ) (2)
2 2
ou  
d + b d − b
, , c = (2rs, r2 − s2 , r2 + s2 ). (3)
2 2
Voltemos às equações do enunciado. Isolando a2 nas duas e somando, encontramos

2a2 = d2 − b2 = (d + b)(d − b).

Independentemente de estarmos em (2) ou em (3), isso é o mesmo que

2a2 = 8rs(r2 − s2 ).
a
Como a é par, 2 é inteiro. Podemos reescrever a última equação como
 a 2
= rs(r2 − s2 ) = rs(r − s)(r + s).
2
Observe agora que, como mdc(r, s) = 1 e r e s têm paridades distintas, r, s, (r − s) e (r + s) são
dois a dois primos entre si. De fato:
• se p | r e p | s, p = 1, pois mdc(r, s) = 1;
• se p | r e p | (r − s), então p | r − (r − s) = s; daı́, p = 1, já que mdc(r, s) = 1;
• se p | r e p | (r + s), então p | (r + s) − r = s; daı́, p = 1, já que mdc(r, s) = 1;
• se p | s e p | (r − s), então p | (r − s) + s = r; daı́, p = 1, já que mdc(r, s) = 1;
• se p | s e p | (r + s), então p | (r + s) − s = r; daı́, p = 1, já que mdc(r, s) = 1;

6
• se p | (r − s) e p | (r + s), então p | (r − s) + (r + s) = 2r e p | (r + s) − (r − s) = 2s; daı́, p | 2,
já que mdc(r, s) = 1; como r e s têm paridades distintas, no entanto, (r − s) é ı́mpar; logo, p | 2 e p
divide um número ı́mpar, donde p = 1.

Dessa forma, segue do problema 11 que r, s, (r − s) e (r + s) são quadrados perfeitos. Escrevendo


r = x2 , s = y 2 , r − s = z 2 e r + s = w2 , encontramos as igualdades
(
y 2 + z 2 = x2
.
y 2 + x2 = w 2

Voilà: (y, z, x, w) é uma das quádruplas que satisfazem as condições do enunciado. Mas

d+b
w2 = r + s < 2r ≤ 2rs ≤ < d ≤ d2 ,
2
ou seja, w < d. Absurdo, pois supomos que d era mı́nimo!
Dessa maneira, concluı́mos que não há quádrupla (a, b, c, d) de inteiros positivos que satisfaz
(
a2 + b2 = c2
.
a2 + c2 = d2

5. Queremos encontrar triplas (x, y, z) tais que

y 2 − z 2 = 1997 − x2 ⇐⇒ (y − z)(y + z) = 1997 − x2 .

É suficiente que encontremos triplas tais que

y−z =1 e y + z = 1997 − x2 .

Estas duas igualdades se verificam se, e somente se,

1998 − x2 1996 − x2
y= e z= .
2 2
Precisamos que x seja par. Por outro lado, se x for par, digamos, igual a 2t, podemos definir

1998 − 4t2 1996 − 4t2


y= = 999 − 2t2 e z= = 998 − 2t2 ,
2 2
e a equação será satisfeita.
Dessa forma, as soluções inteiras de

x2 + y 2 − z 2 = 1997

são as triplas da forma


(2t, 999 − 2t2 , 998 − 2t2 ),
onde t é inteiro. Como cada valor de t gera uma tripla distinta, concluı́mos que existem infinitas
triplas que satisfazem a equação do enunciado.
1
Na verdade, não tão diretamente. Mas, dado o resultado do problema Q 1, é fácil de provar por indução que, se os
inteiros positivos a1 , a2 , ..., an são dois a dois primos entre si e tais que n
i=1 ai é um quadrado perfeito, então cada um
dos ai é um quadrado perfeito - fica o exercı́cio!

7
6. Suponhamos, por um instante, que m, n 6= 0.
Seja α o maior inteiro tal que 5α | n e β o maior inteiro tal que 5β | m. A maior potência de 5 que
divide 5m2 é 52β+1 , enquanto que a maior potência de 5 que divide n2 é 52α . Dessa forma, o expoente
da maior potência de 5 que divide 5m2 + n2 é min(2β + 1, 2α). Como, por hipótese, 5m2 + n2 = 52020 ,
esse expoente deve ser igual a 2020. Como 2β + 1 é ı́mpar, só podemos ter, então, 2α = 2020 e
2β + 1 > 2α, isto é, β ≥ α = 1010.
Escrevendo
n = 5α · a e m = 5β · b,
encontramos de
5m2 + n2 = 52020
que
52β+1 a2 + 52α b2 = 52020 ⇒ 5(2β+1)−2α a2 + b2 = 1.
Absurdo, pois
5(2β+1)−2α a2 + b2 ≥ 5a2 + b2 ≥ 5 + 1 = 6.
Dessa forma, a hipótese que fizemos não se verifica, isto é, não é verdade que m, n 6= 0.
Se n = 0, devemos ter 5m2 = 52020 ⇐⇒ m2 = 52019 , o que não pode acontecer, pois 52019 não é
quadrado perfeito.
Se m = 0, devemos ter n2 = 52020 ⇐⇒ n = 51010 .
Portanto, o único par de inteiros (m, n) para o qual 5m2 + n2 = 52020 é (m, n) = (0, 51010 ).

7. Assim como quando lidamos com terna pitagóricas, é suficiente olharmos para as triplas primitivas,
isto é, tais que mdc(a, b, c) = 1. Seja, então, (a, b, c) uma tripla de inteiros positivos primos entre si
tais que
a2 + b2 = 2c2 .
Podemos reescrever a equação acima como

(a + b)2 (a − b)2
+ = 2c2 ,
2 2
ou  a + b 2  a − b 2
+ = c2 .
2 2

 Como a2 + b2= 2c2 , a e b têm a mesma paridade, donde a+b a−b


2 e 2 são ambos inteiros. Assim,
a+b a−b
 
2 , 2 , c é uma terna pitagórica. Além disso, a terna é primitiva: se

a + b a − b
p| e p| ,
2 2
então
a + b a − b a + b a − b
p| + =a e p| − = b.
2 2 2 2
Daı́, se p também divide c, p divide a, b e c e, portanto, p = 1 (pois mdc(a, b, c) = 1).
Dessa forma, existem r > s > 0 inteiros primos entre si e de paridades distintas tais que
 
a + b a − b
, , c = (r2 − s2 , 2rs, r2 + s2 )
2 2
ou  
a + b a − b
, , c = (2rs, r2 − s2 , r2 + s2 ).
2 2

8
O primeiro caso nos dá

(a, b, c) = (r2 + 2rs − s2 , r2 − 2rs − s2 , r2 + s2 ),

e o segundo,
(a, b, c) = (r2 − 2rs − s2 , r2 + 2rs − s2 , r2 + s2 ).
É fácil verificar que em ambas as situações temos, de fato, uma solução à equação do problema.
Portanto, as triplas (a, b, c) de inteiros positivos tais que a2 + b2 = 2c2 são exatamente as triplas
que são ou da forma
d(r2 − 2rs − s2 ), d(r2 + 2rs − s2 ), d(r2 + s2 ) ,


ou da forma
d(r2 + 2rs − s2 ), d(r2 − 2rs − s2 ), d(r2 + s2 ) ,


sendo, em ambos os casos, d um inteiro positivo, e r e s inteiros positivos primos entre si, de paridades
distintas e tais que r2 − 2rs − s2 > 0.

Material elaborado por Valentino Amadeus Sichinel

9
Problemas Resolvidos

Nı́vel 2

Equações diofantinas III

Material elaborado por Valentino Amadeus Sichinel


Problemas

Problema 1. Encontre todas as soluções em inteiros x, y, z, t da equação:

x2 + y 2 + z 2 = 8t − 1.

Problema 2. Encontre todas as soluções em inteiros positivos da equação


1 1 1
+ + = 1.
a b c

Problema 3. Mostre que para todo inteiro z existem inteiros x e y tais que

x2 − y 2 = z 3 .

Problema 4. Encontre todas as soluções de

1 + x + x2 + x3 = 2y

em inteiros positivos x e y.

Problema 5. Mostre que a equação diofantina

5m2 − 6mn + 7n2 = 1988

não possui solução nos inteiros.

Problema 6 (Rússia 1996). Sejam x, y, p, n, k números inteiros positivos tais que

xn + y n = pk .

Prove que, se n > 1 é ı́mpar, e p é um número primo, então n é uma potência de p.

Problema 7 (OBM 2009). Prove que não existem inteiros positivos x e y tais que

x3 + y 3 = 22009 .

2
Soluções

1. Um quadrado perfeito só pode deixar resto 0, 1 ou 4, quando dividido por 8. Dessa forma, se x, y
e z são inteiros, x2 + y 2 + z 2 pode deixar qualquer resto quando dividido por 8, à exceção de 7.
Como 8t − 1 ≡ 7 (mod 8), seja qual for t ∈ Z, concluı́mos que não existem inteiros x, y, z, t
satisfazendo a equação enunciada.

2. Seja (a, b, c) uma solução. Sem perdas, suponhamos que a ≥ b ≥ c. Como


1 1 1
+ + = 1 ⇐⇒ ab + bc + ca = abc ⇐⇒ bc = a(bc − b − c), (1)
a b c
bc é divisı́vel por a. Dessa forma, existe k inteiro positivo tal que bc = ak. Substituindo no lado direito
da última igualdade acima, ficamos com
bc = a(ak − b − c).
Se k ≥ 3, isso nos dá
bc = a(ak − b − c) ≥ a(3a − b − c) ≥ a(3a − a − a) ≥ a2 .
Por outro lado, a ≥ b ≥ c implica a2 ≥ bc, com igualdade se, e somente se, a = b = c. Daı́,
a = b = c. Substituindo na equação original, obtemos
3
= 1,
a
o que implica a = 3.
Dessa forma, encontramos a primeira solução (a, b, c) = (3, 3, 3).
Restam somente os casos k = 2 e k = 1.
Se k = 2, temos bc = 2a, donde
bc = a(bc − b − c) ⇒ 2a = a(bc − b − c) ⇒ 2 = bc − b − c ⇒ 2 + c = bc − b. (2)
Portanto, neste caso, b | 2 + c. Como b ≥ c ≥ 2, segue que ou 2 + c = 2b, caso em que b = c = 2, ou
2 + c = b. Não podemos ter b = c = 2 porque, nesse caso, terı́amos a1 + 1b + 1c = 1 + a1 . Logo, c + 2 = b.
Substituindo em (2), encontramos
b = bc − b ⇒ 1 = c − 1 ⇒ c = 2.
Assim, c = 2, b = c + 2 = 4, e temos a = bc 2 = 4.
Isso nos leva à nossa segunda solução: (a, b, c) = (4, 4, 2).
Resta, agora, apenas o caso k = 1. Neste caso, temos bc = a.
Substituindo em (1), ficamos com
bc = a(bc − b − c) ⇒ a = a(bc − b − c) ⇒ 1 = bc − b − c ⇒ 1 + c = b(c − 1). (3)
b divide c + 1. Como b ≥ c e b ≥ 2, só podemos ter b = c + 1. Substituindo em (3), encontramos
1 + c = b(c − 1) ⇒ b = b(c − 1) ⇒ 1 = c − 1 ⇒ c = 2.
Logo, c = 2, b = c + 1 = 3, a = bc = 6, e temos uma terceira solução: (a, b, c) = (6, 3, 2).
Portanto, as soluções inteiras positivas para a equação do enunciado são, a menos de permutação,1
(3, 3, 3), (4, 4, 2) e (6, 3, 2).
1
Lembre-se de que supomos, no inı́cio da solução, que a ≥ b ≥ c.

3
3. Seja z um inteiro. Temos

x2 − y 2 = z 3 ⇐⇒ (x − y)(x + y) = z 3 .

Se conseguirmos mostrar que existem x e y tais que x − y = z e x + y = z 2 , teremos resolvido o


problema.
Ora, o sistema (
x−y =z
x + y = z2
é fácil de resolver: sua única solução é
 z2 + z z2 − z 
(x, y) = , .
2 2
2 z 2 −z
Como tanto z 2+z quanto 2 são inteiros, isso mostra que somos capazes de obter os inteiros por
que procurávamos.

4. Podemos fatorar o lado esquerdo da equação: temos

x4 − 1 (x2 − 1)(x2 + 1) (x − 1)(x + 1)(x2 + 1)


1 + x + x2 + x3 = = = = (x + 1)(x2 + 1).
x−1 x−1 x−1
Dessa forma, queremos inteiros positivos x e y tais que

(x + 1)(x2 + 1) = 2y .

Para que seu produto seja uma potência de 2, (x + 1) e (x2 + 1) devem ser ambos potências de 2.
Os únicos restos que um quadrado perfeito pode deixar quando divido por 4, no entanto, são 0 e 1.
Dessa forma, (x2 + 1) deixa resto 1 ou 2 quando dividido por 4. Como x deve ser positivo, (x2 + 1)
não pode ser igual a 1 e, portanto (sendo potência de 2), não pode deixar resto 1 quando dividido por
4. Dessa forma, (x2 + 1) deixa resto 2 quando dividido por 4. Como a única potência de 2 que iguala
tal feito é 2, devemos ter x2 + 1 = 2.
Como x é positivo, segue que x = 1. De 1 + x + x2 + x3 = 2y concluı́mos que 4 = 2y , isto é, y = 2.
Portanto, a única solução em inteiros positivos para a equação apresentada é

(x, y) = (1, 2).

5. Seja (m, n) uma hipotética solução inteira para a equação. Veja que

5m2 − 6mn + 7n2 = 1988 ⇐⇒ 100m2 − 120mn + 140n2 = 20 · 1988


⇐⇒ 100m2 − 120mn = 20 · 1988 − 140n2
⇐⇒ 100m2 − 120mn + 36n2 = 20 · 1988 − 140n2 + 36n2
⇐⇒ (10m − 6n)2 = 20 · 1988 − 104n2 ,

donde 20 · 1988 − 104n2 há de ser um quadrado perfeito.


(Obs.: Você poderia ter chegado à mesma conclusão através da fórmula de Bhaskara.)

Dessa forma, 20·1988 é um quadrado perfeito módulo 13 (veja que 13 | 104). Será que isso acontece
de fato?

4
Uma fatoração rápida mostra-nos que 1988 = 4 · 7 · 71. Assim, 20 · 1988 = 16 · 5 · 7 · 71 é quadrado
módulo 13 se, e somente se, 5 · 7 · 71 é quadrado módulo 13. No entanto,1
       
5 · 7 · 71 5 7 71
= · ·
13 13 13 13
     
13 13 6
= · ·
5 7 13
   
2 3
= (−1) · (−1) · ·
13 13
   
2 13
= ·
13 3
= (−1) · 1
= −1.

A contradição mostra que não existe solução inteira (m, n) para a equação 5m2 −6mn+7n2 = 1988,
tal como querı́amos.

6. Antes de mais nada, mostraremos que podemos supor que p - x e p - y.


Escrevamos x = pa x0 e y = pb y 0 , com p - x0 e p - y 0 . Sem perdas, suponhamos que a ≥ b. Temos
então
pna x0n + pnb y 0n = pk ⇒ pn(a−b) x0n + y 0n = pk−nb .
Se a − b 6= 0, o lado esquerdo da igualdade acima não é divisı́vel por p. Logo, nesse caso, temos de
ter pk−nb = 1. Mas
pn(a−b) x0n + y 0n ≥ pa−b + 1.
Absurdo! Logo, a − b = 0, e (1) reescreve-se como

x0n + y 0n = pk−nb ,

com p - x0 e p - y 0 .
Como a conclusão a que queremos chegar diz respeito a n somente, segue daı́ que podemos supor,
sem perdas, que p - x e p - y.
Voltemos então ao cenário original, desta vez com nossa mais nova suposição em mãos (isto é, a
de que p não divide nem x, nem y).
Por absurdo, suponhamos que existe um primo q 6= p tal que q | n, e seja l tal que n = ql. Como l
é ı́mpar (l não pode ser par, porque n é ı́mpar), podemos escrever

xn + y n = (xq )l + (y q )l = xq + y q (xq )l−1 − (xq )l−2 y q + · · · − xq (y q )l−2 + (y q )l−1 .


 

Assim, já que xn + y n é uma potência de p, xq + y q também é uma potência de p.


Uma fatoração similar à que utilizamos acima nos dá

xq + y q = (x + y)(xq−1 − xq−2 y + · · · − xy q−2 + y q−1 ).

Daı́, vem que


x+y ≡ 0 (mod p), isto é, x ≡ −y (mod p)
1 a

A partir daqui, nesta solução, b
é o sı́mbolo de Legendre. Nas igualdades que se seguem, utilizamos várias vezes a
Lei da Reciprocidade Quadrática.

5
e
xq−1 − xq−2 y + · · · − xy q−2 + y q−1 ≡ 0 (mod p).
Substituindo a primeira na segunda, ficamos com

y q−1 + y q−1 + · · · + y q−1 + y q−1 ≡ 0 (mod p),

isto é,
qy p−1 ≡ 0 (mod p).
Mas p - q, já que p e q são primos distintos. Por outro lado, p - y. Absurdo!
A contradição mostra que a hipótese que fizemos é absurda. Portanto, não existe primo q, distinto
de p, que divide n e, assim, n é uma potência de p.

7. Pelo Pequeno Teorema de Fermat, 26 ≡ 1 (mod 7). Daı́,

22009 ≡ 26·334+5 ≡ (26 )334 · 25 ≡ 25 ≡ 32 ≡ 4 (mod 7).

Por outro lado, os possı́veis restos de um cubo perfeito módulo 7 são 0, 1 e 6, donde os possı́veis
resto da soma de dois cubos perfeitos módulo 7 são 0, 1, 2, 5 e 6. Assim, não é possı́vel que existam
inteiros x e y tais que
x3 + y 3 = 22009 .

Material elaborado por Valentino Amadeus Sichinel

6
Problemas Resolvidos

Nı́vel 2

Equações diofantinas IV

Material elaborado por Valentino Amadeus Sichinel


Problemas

Problema 1. Sejam x e y números inteiros tais que x2 − 2y 2 = −1. Prove que x e y são ambos
ı́mpares.

Problema 2. Prove que a equação x2 − dy 2 = −1 não tem solução nos inteiros se d é divisı́vel por
um primo da forma 4k + 3.

Problema 3. Mostre que existem infinitos inteiros n para os quais n2 + (n + 1)2 é um quadrado
perfeito.

Problema 4. Dado um inteiro positivo k, mostre que não existem inteiros (x, y) tais que

x2 − (k 2 − 1)y 2 = −1.

Problema 5. Seja d um inteiro positivo que deixa resto 1 quando dividido por 4. Mostre que, se x e
y são inteiros tais que x2 − dy 2 = 1, então x é ı́mpar.

Problema 6. Seja p um primo da forma 4k + 1. Prove que a equação x2 − py 2 = −1 tem solução


inteira.

Dica: Considere a equação de Pell x2 − py 2 = 1 e utilize o resultado do problema anterior.

Problema 7. Seja n ≥ 3 um inteiro. Mostre que existe um conjunto S de n pontos no plano tal que
a distância entre quaisquer dois pontos de S é irracional e a área de qualquer triângulo com vértices
em S é positiva e racional.
2t t2 −1

Dica: Procure por pontos da forma t2 +1 , t2 +1 ; utilize o resultado do problema anterior.

Problema 8. Encontre todos os pares (x, y) de inteiros positivos tais que

x2 − 2y 2 = −1.

Problema 9. Encontre todos os pares (k, n) de inteiros positivos tais que

1 + 2 + · · · + k = (k + 1) + (k + 2) + · · · + n.

2
Soluções

1. Olhando a equação do enunciado módulo 2, observamos que x2 ≡ −1 (mod 2). Daı́, x há de ser
ı́mpar.
Sendo x ı́mpar, x2 ≡ 1 (mod 4). Daı́,
x2 − 2y 2 = −1 =⇒ 1 − 2y 2 ≡ −1 (mod 4) =⇒ 2y 2 ≡ 2 (mod 4).
Portanto, y também é ı́mpar (se fosse par, 2y 2 seria divisı́vel por 4).

2. Se p divide d, x2 − dy 2 = −1 implica x2 ≡ −1 (mod p). Como sabemos, isso é absurdo quando p


é da forma 4k + 3.
Para refrescar nossa memória, segue a justificativa. Para que se tenha x2 ≡ −1 (mod p), p não
pode dividir x. Daı́, pelo Teorema de Fermat, xp−1 ≡ 1 (mod p). No entanto,
p−1
x2 ≡ −1 (mod p) ⇒ xp−1 ≡ (−1) 2 (mod p) ⇒ xp−1 ≡ −1 (mod p),
onde na última passagem utilizamos que p é da forma 4k + 3. O resultado segue por absurdo.

3. Devemos mostrar que existem infinitos pares (n, m) de inteiros tais que
n2 + (n + 1)2 = m2 .
Observe que
n2 + (n + 1)2 = m2 ⇐⇒ 2n2 + 2n + 1 = m2
⇐⇒ 4n2 + 4n + 2 = 2m2
⇐⇒ (2n + 1)2 + 1 = 2m2
⇐⇒ (2n + 1)2 − 2m2 = −1.
Bem, 2 não é um quadrado perfeito, e a equação x2 − 2y 2 = −1 tem uma solução: (1, 1). Logo,
x2 − 2y 2 = −1 tem infinitas soluções inteiras. É claro que, em todas elas, x é ı́mpar e, portanto, da
forma 2n + 1.
Dessa forma, segue que a equação n2 + (n + 1)2 = m2 tem infinitas soluções inteiras, isto é, que
existem infinitos inteiros n para os quais n2 + (n + 1)2 é um quadrado perfeito.

4. Suponhamos, por absurdo, que k, x e y são inteiros tais que


x2 − (k 2 − 1)y 2 = −1.
Se k é par, k 2 é divisı́vel por 4. Daı́, nesse caso, (k 2 − 1) ≡ −1 (mod 4). Dessa forma, k ser par
implicaria x2 + y 2 ≡ −1 (mod 4). Esta congruência é absurda, dado que um quadrado perfeito só
pode deixar resto 0 ou 1 quando dividido por 4.
Por outro lado, se k é ı́mpar, k 2 deixa resto 1 quando dividido por 4 e, daı́, (k 2 − 1) ≡ 0 (mod 4).
Dessa forma, k ser ı́mpar implicaria x2 ≡ −1 (mod 4). Absurdo também, dado que x2 só pode deixar
resto 0 ou 1 quando dividido por 4.
Segue assim que não existem inteiros k, x e y tais que
x2 − (k 2 − 1)y 2 = −1.

3
5. Se x fosse par, terı́amos x2 ≡ 0 (mod 4), donde
−dy 2 ≡ 1 (mod 4),
isto é,
−y 2 ≡ 1 (mod 4) ⇒ y 2 ≡ −1 (mod 4).
Absurdo, pois nenhum quadrado perfeito deixa resto −1 quando dividido por 4.

6. Como p é primo, p não é um quadrado perfeito e, portanto, a equação


x2 − py 2 = 1
tem infinitas soluções inteiras. Seja (a, b) a solução mı́nima.
Podemos reescrever a2 − pb2 = 1 como a2 − 1 = pb2 , isto é,
(a − 1)(a + 1) = pb2 . (1)
Como vimos no problema anterior, do fato de que p é da forma 4k + 1 segue que a é ı́mpar. É
claro que, para que (1) seja satisfeita, b tem que ser par. Escrevendo a = 2s + 1 e b = 2t, observamos
por (1) que
2s(2s + 2) = p4t2 ⇐⇒ s(s + 1) = pt2 .
Como mdc(s, s + 1) = 1, vem daı́ que ou s = pm2 e s + 1 = n2 para alguns inteiros m e n, ou
s = m2 e s + 1 = pn2 (para alguns inteiros m e n). Se fosse o primeiro caso, terı́amos
n2 − pm2 = (s + 1) − s = 1,
donde (n, m) seria outra solução para a equação x2 − py 2 = 1. Isso seria absurdo, pois n ≤ s + 1 ≤ a,
e (a, b) é a solução mı́nima. Logo, o segundo caso é o que ocorre de fato, isto é, temos s = m2 e
s + 1 = pn2 para alguns inteiros m e n. Essas igualdades nos dão
m2 − pn2 = s − (s + 1) = −1,
o que mostra que a equação do enunciado tem solução.

7. A área de qualquer triângulo ser positiva equivale a não existirem (em S) três pontos colineares.
Formaremos então o conjunto S com pontos da circunferência unitária; assim, garantimos que não
haverá tripla de pontos colineares.
A área do triângulo determinado pelos pontos (a1 , b1 ), (a2 , b2 ) e (a3 , b3 ) é
|a1 b2 + a2 b3 + a3 b1 − a2 b1 − a3 b2 − a1 b3 |
.
2
Dessa forma, para garantirmos que a área de qualquer triângulo seja um número racional, basta que
escolhamos todos os pontos com coordenadas racionais.
Os pontos da circunferência unitária cujas coordenadas são racionais são exatamente os pontos da
forma
t2 − 1
 
2t
, ,
t2 + 1 t2 + 1
com t um número racional (na verdade, há um ponto de coordenadas racionais, pertencente à circun-
ferência unitária, que não é da forma acima: o ponto (0, 1). Mas ele não nos fará falta.).1
Com um pouco de conta, podemos descobrir qual a distância entre dois pontos da forma acima: a
2 −1 2 −1 
distância cartesiana entre os pontos a22a+1 , aa2 +1 e b22b+1 , bb2 +1 é
1
Por favor, não se assuste com o resultado. Por mais que ele seja verdadeiro, não nos interessa que todos os pontos
de coordenadas racionais são da forma citada; tudo de que precisamos para o problema é que todos os pontos da forma
citada têm coordenadas racionais, e isso é muito mais simples de verificar.

4
s 2 2
a2 − 1 b2 − 1

2a 2b
2
− 2 + −
a +1 b +1 a2 + 1 b2 + 1

s 2 2
2ab2 + 2a − 2a2 b − 2b a2 b2 + a2 − b2 − 1 − a2 b2 − b2 + a2 + 1

= +
(a2 + 1)(b2 + 1) (a2 + 1)(b2 + 1)
s 2  2
2(a − b)(1 − ab) 2(a − b)(a + b)
= +
(a2 + 1)(b2 + 1) (a2 + 1)(b2 + 1)

2(a − b) p
= (1 − ab)2 + (a + b)2
(a2 + 1)(b2 + 1)
2(a − b) p
= 1 + a2 b2 + a2 + b2
(a2 + 1)(b2 + 1)
2(a − b) p
= (a2 + 1)(b2 + 1).
(a2 + 1)(b2 + 1)
2 −1  2 −1 
Assim, quando a e b são racionais, a distância entre a22a+1 , aa2 +1 e b22b+1 , bb2 +1 é racional se, e
p
somente se, (a2 + 1)(b2 + 1) é racional.
Agora já estamos aptos a entender a solução do problema. A proposta é escolher pontos
t2i − 1
 
2ti
, ,
t2i + 1 t2i + 1
com cada ti inteiro e tal que t2i + 1 = pi ri2 , onde ri é um inteiro qualquer e pi é um primo. A chave,
aqui, é que os primos pi sejam todos distintos. De fato, observe que, se a e b são tais que a2 + 1 = pm2
e b2 + 1 = qn2 , então p √
(a2 + 1)(b2 + 1 = mn pq,
que jamais será racional se p e q forem primos distintos.
Tudo de que precisamos, portanto, é garantir que existem inteiros (t1 , t2 , t3 , . . . ) e primos
(p1 , p2 , p3 , . . . ), todos distintos, tais que t21 + 1 = p1 r12 , t22 + 1 = p2 r22 , t23 + 1 = p3 r32 , e assim
sucessivamente, para alguns inteiros (r1 , r2 , r3 , . . . ). Ora, esse resultado nos dá o problema anterior:
para todo primo p da forma 4k + 1, existem inteiros t e r tais que t2 − pr2 = −1, isto é, tais que
t2 + 1 = pr2 . Como existem infinitos primos da forma 4k + 1, o problema está resolvido.

8. A solução mı́nima da√equação de Pell x2 −2y 2 = 1 é (3, 2). De fato, se tivéssemos y = 1 na equação,
haverı́amos de ter x = 3; assim, y ≥ 2. Por outro lado, (3, 2) é uma solução, como podemos verificar
facilmente.
Passemos à equação do problema. x2 − 2y 2 = −1 tem uma solução: (1, 1). É claro que esta é a
solução mı́nima. Como
√  √  √
x + 2y 3 + 2 2 = (3x + 4y) + 2(2x + 3y),

segue da teoria estudada que as soluções inteiras e positivas da equação x2 − 2y 2 = −1 são os pares
(xn , yn ), onde (xn ) e (yn ) são sequências tais que
(
xn+1 = 3xn + 4yn
x 0 = y0 = 1 e ∀ n ∈ N. (1)
yn+1 = 2xn + 3yn

5
Para um n ∈ N qualquer, temos

xn+2 = 3xn+1 + 4yn+1 = 3(3xn + 4yn ) + 4(2xn + 3yn ) = 17xn + 24yn .

Daı́,
xn+2 − 6xn+1 = 17xn + 24yn − 6(3xn + 4yn ) = −xn ,
ou seja,
xn+2 = 6xn+1 − xn .
Logo, (xn ) é a sequência que satisfaz x0 = 1, x1 = 7 (o valor de x1 pode ser calculado através de
(1)) e xn+2 = 6xn+1 − xn ∀ n ∈ N. A resolução de recorrências foge do escopo deste material, mas
métodos padrões nos mostram que, nessas condições,
√ √
1+ 2 √ n 1 − 2 √ n
xn = 3+2 2 + 3−2 2 ∀ n ∈ N.
2 2
Podemos repetir o mesmo processo para encontrar a fórmula de yn . Para um n ∈ N qualquer,
temos
yn+2 = 2xn+1 + 3yn+1 = 2(3xn + 4yn ) + 3(2xn + 3yn ) = 12xn + 17yn .
Daı́,
yn+2 − 6yn+1 = 12xn + 17yn − 6(2xn + 3yn ) = −yn ,
ou seja,
yn+2 = 6yn+1 − yn .
Logo, (yn ) é a sequência que satisfaz y0 = 1, y1 = 5 e yn+2 = 6yn+1 − yn ∀ n ∈ N. Resolvendo a
recorrência, encontramos que
√ √
2+ 2 √ n 2 − 2 √ n
yn = 3+2 2 + 3−2 2 ∀ n ∈ N.
4 4
Portanto, as soluções inteiras positivas da equação

x2 − 2y 2 = −1

são os pares da forma (xn , yn ), sendo


√ √
1+ 2 √ n 1 − 2 √ n
xn = 3+2 2 + 3−2 2
2 2
e
√ √
2+ 2 √ n 2 − 2 √ n
yn = 3+2 2 + 3−2 2 .
4 4

6
9. Queremos encontrar todos os pares (k, n) de inteiros positivos tais que

1 + 2 + · · · + k = (k + 1) + (k + 2) + · · · + n ⇐⇒ 2 1 + 2 + · · · + k = 1 + 2 + · · · + n
n(n + 1)
⇐⇒ k(k + 1) =
2
⇐⇒ 8k(k + 1) = 4n(n + 1)
⇐⇒ 8k 2 + 8k = 4n2 + 4n
⇐⇒ 8k 2 + 8k + 1 = 4k 2 + 4k + 1
⇐⇒ 2(4k 2 + 2k + 1) − 1 = 4n2 + 4n + 1
⇐⇒ 2(2k + 1)2 − 1 = (2n + 1)2
⇐⇒ (2n + 1)2 − 2(2k + 1)2 = −1.

Como 2 não é um quadrado perfeito, a equação de Pell x2 −2y 2 = 1 possui infinitas soluções inteiras
positivas. Por outro lado, a equação x2 − 2y 2 = −1 possui uma solução: (1, 1). Logo, x2 − 2y 2 = −1
possui infinitas soluções inteiras positivas.
Pelo problema 1, sabemos que todas as soluções de x2 − 2y 2 = −1 são formadas por um par de
números ı́mpares, isto é, são da forma (2n + 1, 2k + 1). Além disso, a única solução inteira e positiva
que não serve para os nossos preceitos é (1, 1), já que se 2n + 1 = 1, então n = 0. É fácil verificar que,
se uma das variáveis for maior que 1, a outra também será.
Portanto, os pares (k, n) de inteiros positivos que satisfazem

1 + 2 + · · · + k = (k + 1) + (k + 2) + · · · + n

são exatamente os pares da forma b−1 a−1



2 , 2 , onde a e b são inteiros maiores que 1 tais que (a, b) é
solução da equação x2 − 2y 2 = −1.
A solução do problema anterior faz o resto do trabalho: os pares (k, n) de inteiros positivos que
satisfazem
1 + 2 + · · · + k = (k + 1) + (k + 2) + · · · + n
são exatamente os pares bm2−1 , am2−1 , onde


√ √
1+ 2 √ m 1 − 2 √ m
am = 3+2 2 + 3−2 2 ,
2 2
√ √
2+ 2 √ m 2 − 2 √ m
bm = 3+2 2 + 3−2 2
4 4
e
m ≥ 1.

Material elaborado por Valentino Amadeus Sichinel

Você também pode gostar